IPE Ulti

You might also like

Download as pdf or txt
Download as pdf or txt
You are on page 1of 1742

1. Find the work possess for a helium gas at 20°C.

A. 609 kJ/kg B. 168 kJ/kg C. 229 kJ/kg D. 339 kJ/kg


Solution:
( )

( )

2. Two kilogram of gas is confined in a 1 m3 tank at 200 kPa and 88°C. What type of gas is in the
tank?
A. Helium B. Ethane C. Methane D. Ethene
Solution:

( ) ( )( )

( )

3. Find the enthalpy of Helium if its internal energy is 200 kJ/kg.


A. 144 kJ/kg B. 223.42 kJ/kg C. 333.42 kJ/kg D. 168 kJ/kg
Solution:

( )

4. Compute the mass of a 2 m3 propane at 280 kPa and 40°C.


A. 6.47 kg B. 5.1 kg C. 10.20 kg D. 9.47 kg
Solution:
( ) ( )

( ) ( )( )

5. Compute the air flow in ft3/min of mechanical ventilation required to exhaust an


accumulation of refrigerant due to leaks of the system capable of revolving air from the
machinery for a mass of 4 lbs refrigerant.
A. 200 B. 210 C. 220 D. 230
Solution:
1

6. Compute the free aperture cross section in m3 for the ventilation of a machinery room if the
mass of refrigerant is 9 kg.
A. 0.314 B. 0.414 C. 0.514 D. 0.614
Solution:

7. A 29.53” x 39.37” pressure vessel contains ammonia with f = 0.041. Compute the minimum
required discharge capacity of the relief device in kg/hr.
A. 106.71 kg/hr B. 108.71 kg/hr C. 110.71 kg/hr D. 112.71 kg/hr
Solution:
( )
( )( ) ( )

8. Compute the maximum length of the discharge pipe installed on an outlet of a pressure-relief
device in feet for internal pipe diameter of 0.5 inch and rated discharge capacity is 8 lb/min of
air. The rated pressure of relief valve is 10 psig.
A. 0.286 ft B. 0.386 ft C. 0.486 ft D. 0.586 ft
Solution:
( )
( ) ( )
( )

9. A thermal power plant has the heat rate of 11,363 Btu/kW-hr. Find the thermal efficiency of
the plant.
A. 28% B. 30% C. 34% D. 40%
Solution:

10. What is the hydraulic gradient of a 1 mile, 17 inches inside diameter pipe when 3300
gal/min of water flow with f = 0.03?
A. 0.00714 B. 0.00614 C. 0.00234 D. 0.00187
Solution:

( )

2
( )( )
( )( )

11. Find the loss of head in the pipe entrance if speed of flow is 10 m/s.
A. 5.10 m B. 10.2 m C. 17.4 m D. 2.55 m
Solution:
( ) [ ( )
]

12. Wet material, containing 220% moisture (dry basis), is to be dried at a rate of 1.5 kg/s in a
continuous dryer to give a product containing 10% (dry basis). Find the moisture removed in
kg/hr.
A. 3543.75 kg/hr B. 3513.75 kg/hr C. 3563.75 kg/hr D. 3593.75 kg/hr
Solution:

( ) ( )
( )

13. Copra enters a dryer containing 70% moisture and leaves 7% moisture. Find the moisture
removed on each pound of solid in the final product.
A. 6.258 lbs B. 1.258 lbs C. 4.258 lbs D. 2.258 lbs
Solution:

14. A 1 m x 1.5 m cylindrical tank is full of oil with SG = 0.92. Find the force acting at the bottom
of the tank in dynes.
A. 106.33 x 103 dynes C. 106.33 x 105 dynes
B. 106.33 x 104 dynes D. 106.33 x 106 dynes
Solution:

( )

3
15. Find the pressure at the 100 fathom depth of water in kPa.
A. 1,793.96 kPa B. 1,893.96 kPa C. 1,993.96 kPa D. 1,693.96 kPa
Solution:

( )

16. Find the depth in furlong of the ocean (SG = 1.03) if the pressure at the sea bed is 2,032.56
kPa.
A. 1 B. 2 C. 3 D. 4
Solution:

( )( )( )

17. Find the mass of 10 quartz of water.


A. 10.46 kg B. 9.46 kg C. 11.46 kg D. 8.46 kg
Solution:
-
( )( )( )
-
( )

18. Find the mass of carbon dioxide having a pressure of 20 psia at 200°F with 10 ft 3 volume.
A. 1.04 lbs B. 1.14 lbs C. 1.24 lbs D. 1.34 lbs
Solution:

( )( ) ( )( )

19. Find the heat needed to raise the temperature of water from 30°C to 100°C with 60%
quality. Consider an atmospheric pressure of 101.325 kPa. Use the approximate enthalpy
formula of liquid.
A. 293.09 kJ/kg B. 1,772.90 kJ/kg C. 1,547.90 kJ/kg D. 1,647.29 kJ/kg
Solution:

( )

4
( )
( ) ( )

20. Find the enthalpy of water at 212°F and 14.7 psi if the dryness factor is 30%. Use the
approximate enthalpy formula of liquid.
A. 461 Btu/lb B. 471 Btu/lb C. 481 Btu/lb D. 491 Btu/lb
Solution:

( )

21. An air compressor consumed 1200 kW-hr per day of energy. The electric motor driving the
compressor has an efficiency of 80%. If indicated power of compressor is 34 kW, find the
mechanical efficiency of the compressor.
A. 117.65% B. 75% C. 85% D. 90%
Solution:
( )
( )

22. A refrigeration system consumed 28,800 kW-hr per month of energy. There is 20% of
energy lost due to the cooling system of the compressor and the motor efficiency is 90%. If the
COP of the system is 6, find the mechanical efficiency of the compressor.
A. 43.15 TR B. 46.15 TR C. 49.15 TR D. 41.15 TR
Solution:
( )( )
( )
̇ ( )

̇
̇ ( )

23. A 23 ton refrigeration system has a heat rejected of 100 kW. Find the energy efficiency ratio
of the system.
A. 13.42 B. 14.42 C. 15.42 D. 16.42
Solution:
5
̇ ̇
( ) ̇
̇
( )
̇
( )

24. A 200 mm x 250 mm, 8-cylinder, 4-stroke diesel engine has a brake power of 150 kW. The
mechanical efficiency is 80%. If two of the cylinders were accidentally cut off, wha t will be the
new friction power?
A. 31.50 kW B. 33.50 kW C. 35.50 kW D. 37.50 kW
Solution:

25. If the energy efficiency ratio of the refrigeration system is 12.6, what is the COP of the
system?
A. 3.69 B. 4.23 C. 5.92 D. 6.83
Solution:

26. An air compressor has a power of 40 kW at 4% clearance. If the clearance will increase to
7%, what is the new power?
A. 70 kW B. 40 kW C. 53 kW D. 60 kW
Solution:

27. What is the approximate value of temperature of water having enthalpy of 208 Btu/lb?
A. 138.67°C B. 115.55°C C. 258.67°C D. 68.67°C
Solution:

6
28. Convert 750°R to K.
A. 390.33 K B. 395.33 K C. 410.33 K D. 416.33 K
Solution:

29. An Otto cycle has a compression ratio of 8. Find the pressure during compression.
A. 18.38 B. 16.38 C. 14.38 D. 12.28
Solution:

( ) ( )

30. A diesel cycle has a cut off ratio of 2.5 and expansion ratio of 4. Find the clearance of the
cycle.
A. 9.11% B. 5.55% C. 11.11% D. 15.15%
Solution:
( )

31. A dual cycle has an initial temperature of 30°C. The compression ratio is 6 and the heat
addition at constant volume process is 600 kJ/kg. If the cut-off ratio is 2.5, find the maximum
temperature of the cycle.
A. 3638.50°C B. 3365.50°C C. 3565.50°C D. 3965.50°C
Solution:
( ) ( ) ( )( )

( )( )( ) ( )

( ) ( )

32. A 3-stage air compressor compresses air from 100 kPa to 1000 kPa. Find the intercooler
pressure between the 1st and 2nd stage.
A. 505.44 kPa B. 108.44 kPa C. 316.23 kPa D. 215.44 kPa
Solution:
7
( ) ( )

33. A 10-stage air compressor compresses air from 100 kPa to 800 kPa. Find the intercooler
pressure between the 1st and 2nd stage.
A. 282.84 kPa B. 113.21 kPa C. 123.11 kPa D. 333.51 kPa
Solution:
√ √

34. A 3-stage air compressor compresses air from 100 kPa to 700 kPa. Find the intercooler
pressure between the 2nd and 3rd stage.
A. 365.88 kPa B. 375.88 kPa C. 385.88 kPa D. 395.88 kPa
Solution:
( ) ( )

35. Carnot cycle A, B and C are connected in series so that the heat rejected from A will be the
heat added to B. and heat rejected from B will be added to C. Each cycle operates between
30°C and 400°C. If heat added to A is 1000 kW, find the work output of C.
A. 111.44 kW B. 549.78 kW C. 247.53 kW D. 141.89 kW
Solution:

̇ ̇ ̇
̇ ̇
̇

̇ ̇
̇

̇ ̇
̇
̇
̇

36. Air compressed adiabatically from 30°C to 100°C. If the mass of air being compressed is 5
kg, find the change of entropy.
A. 1.039 kJ/K B. 0.746 kJ/K C. 0 D. 1.245 kJ/K
Solution:

8
37. Two kilogram of air in a rigid tank changes its temperature from 32°C to 150°C. Find the
work done during the process.
A. 236 B. 170 C. 195 D. 0
Solution:

38. Determine the atmospheric pressure at a location where barometric reading is 740 mm Hg
and gravitational acceleration is g = 9.7 m/s 2. Assume the temperature of mercury to be 10°C,
at which the density is 13,570 kg/m3.
A. 99.45 kPa B. 97.41 kPa C. 95.44 kPa D. 98.66 kPa
Solution:
( )( )

39. The barometer of a mountain hiker reads 930 mbars at the beginning of a hiking trip and
780 mbars at the end. Neglecting the effect of altitude on local gravitational acceleration,
determine the vertical distance climbed. Assume g = 9.7 m/s 2.
A. 1274.21 m B. 1289.00 m C. 1267.34 m D. 1583.34 m
Solution:

( )( ) ( )( )

40. The lower half of a 10 m high cylindrical container is filled with water and the upper half
with oil that has SG = 0.85. Determine the pressure difference between the top and bottom if
the cylinder.
A. 90.74 kPa B. 92.74 kPa C. 83.38 kPa D. 98.10 kPa
Solution:
( )( ) ( )

41. An ideal gas at 0.80 atmospheres and 87°C occupies 0.450 liter. How many moles are in the
sample? (R = 0.0821 liter-atm/mole-K).
A. 0.0002 mole B. 0.0378 mole C. 0.0122 mole D. 0.0091 mole
Solution:

( ) ( )( )

9
42. A certain gas at 101.325 kPa and 10°C whose volume is 2.83 m3 capacity. Before admission,
the storage vessel contained gas at a pressure and temperature of 137.8 kPa and 26°C; after
admission, the pressure increased to 1171.8 kPa. What should be the final temperature of the
gas in the vessel in Kelvin?
A. 298.0 B. 319.8 C. 180.0 D. 314.2
Solution:

( )
( )

( )
( )

( )

43. A perfect gas has a value of R = 58.8 ft-lb/lb-°R and k = 1.26. If 20 Btu are added to 10 lbs of
this gas at constant volume when initial temperature is 90°F, find the final temperature.
A. 97°F B. 104°F C. 154°F D. 185°F
Solution:

( ) -
( )( )

44. Ammonia weighing 22 kg is confirmed inside a cylinder equipped with a piston has an initial
pressure of 413 kPa at 38°C. If 3200 kJ of heat is added to the ammonia until its final pressure
and temperature is 413 kPa and 100°C, respectively. What is the amount of work done by the
fluid in kJ?
A. 667 B. 304 C. 420 D. 502
Solution:

( )( )

10
( )( )

( )

45. A tank contains 90 ft3 of air at a pressure of 350 psig; if the air is cooled until its pressure
and temperature decreases at 200 psig and 70°F respectively, what is the decrease in internal
energy?
A. 6232.09 Btu B. -5552 Btu C. 5552 Btu D. -6232.09 Btu
Solution:

( )( )
( )

( )
( )( ) - -

46. A large mining company was provided with a 3 m3 of compressed air tank. Air pressure in
the tank drops from 700 kPa to 150 kPa while the temperature remains constant at 28°C. What
percentage has the mass of air in the tank been reduced?
A. 74.09 B. 72.45 C. 76.56 D. 78.57
Solution:

47. A 4 m3/hr pump delivers water to a pressure tank. At the start, the gage read 138 kPa until
it reads 276 kPa and then the pump was shut off. The volume of the tank is 180 liters. At 276
kPa, the water occupied 2/3 of the tank volume. Determine the volume of the water that can
be taken out until the gage reads 138 kPa.
A. 31.20 liters B. 34.59 liters C. 16.87 liters D. 29.50 liters
Solution:

( )

( ) ( )( )

( ) ( )

11
48. A refrigeration plant is rated at 15 tons capacity. How many pounds of air per hour will it
cool from 70°F to 90°F at constant temperature?
A. 50,000 lb/hr B. 37,500 lb/hr C. 52,000 lb/hr D. 45,000 lb/hr
Solution:
̇ ̇

( ) ̇( )( )

49. An air standard engine has a compression ratio of 18 and a cut-off ratio of 4. If the intake air
pressure and temperature are 100 kPa and 27°C, find the work in kJ per kg.
A. 2976 B. 2166 C. 1582 D. 2751
Solution:

( )

( ) ( )

( )

-
( )( )

( )

50. Determine the air-standard efficiency of an engine operating on the diesel cycle with a
clearance of 6% when the suction pressure is 99.7 kPa and the fuel is injected for 7% of the
stroke. Assume k = 1.4.
A. 62.11% B. 51.20% C. 73.58% D. 60.02%
Solution:

12
( )( ( )
)

51. Steam at 2 MPa and 250°C in a rigid cylinder is cooled until the quality is 30%. Find the heat
rejected from the cylinder.
@ 2 Mpa and 250°C: υ = 0.11144 m3/kg u = 2679.6 kJ/kg
3
@ 2 Mpa, (saturated): υf = 0.0011767 m /kg υg = 0.09963 m3/kg
uf = 906.44kJ/kg ug = 1693.8 kJ/kg
A. -423.23 kJ/kg B. -926.26 kJ/kg C. -1265.02 kJ/kg D. 1082.34 kJ/kg
Solution:

( )
-

52. At 1.3 MPa, a mixture of steam and water has an entropy of 3 kJ/kg-K, Find the enthalpy of
the mixture.
@ 1.3MPa: s f = 2.2515 kJ/kg-K s g = 6.4952 kJ/kg-K
hf = 814.93 kJ/kg hfg = 1972.7 kJ/kg
A. 1627.71 kJ/kg B. 1533.33 kJ/kg C. 1234.45 kJ/kg D. 1162.40 kJ/kg
Solution:

( )

( )

53. A mixture with 70% quality at 500 kPa is heated isothermally until its pressure is 300 kPa.
Find the heat added during the process.
@ 600 kPa: s f = 1.8607 kJ/kg-K s fg = 4.9606 kJ/kg-K
@ 300 kPa and 151.86°C: s = 7.0888 kJ/kg-K
A. 745.92 kJ/kg B. 535.16 kJ/kg C. 982.44 kJ/kg D. 765.34 kJ/kg
Solution:
( )

( )( )

13
54. A tank contains exactly one kilogram of water consisting of liquid and vapor in equilibrium
at 1 MPa. If the liquid contains one-third and the remaining is vapor of the volume of the tank,
what is the enthalpy of the contents of the tank?
@ 1MPa: υf = 0.0011273 m3/kg υfg = 0.19444 m3/kg
hf = 762.81 kJ/kg hfg = 2015.3 kJ/kg
A. 644.40 kJ/kg B. 774.40 kJ/kg C. 785.92 kJ/kg D. 435.29 kJ/kg
Solution:

( )

55. Water substance at 70 bar and 65°C enters a boiler tube of constant inside diameter of 25
mm. The water leaves the boiler tube at 50 bar and 700 K at velocity of 150 m/s. Calculate the
inlet velocity (m/s).
From steam tables:
@ 70 bar (7Mpa) and 65°C: υ1 = 0.001017 m3/kg
@ 50 bar (5Mpa) and 700 K (427°C): υ2 = 0.06081 m3/kg
A. 1.56 B. 2.51 C. 1.672 D. 3.230
Solution:
̇ ̇

( )

56. Water substance at 70 bar and 65°C enters a boiler tube of constant inside diameter of 35
mm. The water leaves the boiler tube at 50 bar and 700 K at velocity of 150 m/s. Calculate the
inlet volume flow (liters/s).
From steam tables:
@ 70 bar (7Mpa) and 65°C: υ1 = 0.001017 m3/kg
@ 50 bar (5Mpa) and 700 K (427°C): υ2 = 0.06081 m3/kg
A. 0.821 B. 1.261 C. 0.344 D. 1.609
Solution:
̇ ̇
14
( )

( ) ( ) -

57. Steam leaves an industrial boiler at 827.4 kPa and 171.6°C. A portion of the steam is passed
through a throttling calorimeter and is exhausted to the atmosphere when the calorimeter
pressure is 101.4 kPa. How much moisture is leaving the boiler container if the temperature of
the steam at the calorimeter is 115.6°C?
@ 827.4 kPa and 171.6°C: hf = 727.25 kJ/kg hfg = 2043.2 kJ/kg
@ 101.4 kPa and 115.6°C: h2 = 2707.6 kJ/kg
A. 3.78% B. 3.08% C. 4.56% D. 2.34%
Solution:

( )

58. A throttling calorimeter is connected to a desuperheated steam line supplying steam to the
auxiliary feed pump on a ship. The line pressure measures 2.5 MPa. The calorimeter pressure is
110 kPa at 150°C. Determine the entropy of the steam line.
@ 110 kPa and 150°C: h2 = 2775.6 kJ/kg
@ 2.5 MPa: hf = 962.11 kJ/kg hfg = 1841 kJ/kg
s f = 2.5547 kJ/kg-K s fg = 3.7028 kJ/kg-K
A. 6.8 kJ/kg-K B. 6.2 kJ/kg-K C. 6.6 kJ/kg-K D. 7.5 kJ/kg-K
Solution:

( )

( ) -

59. Atmospheric pressure boils at 212°F. At the vacuum pressure at 24 in Hg, the temperature is
142°F. Find the boiling temperature when the pressure is increased by 40 psia from
atmospheric.
15
A. 449.42°F B. 536.34°F C. 479.13°F D. 263.45°F
Solution:
- ( )

60. A certain coal has the following ultimate analysis:


C = 69% N2 = 5% H = 2.5% S =7%
Determine the amount of oxygen if the heating value of fuel is 26,961.45 kJ/kg.
A. 1.5% B. 2.5% C. 3.5% D. 4.5%
Solution:
( )
( ) ( ) ( )

61. A diesel engine consumed 945 liters of fuel per day at 35°C. If the fuel was purchased at
15.6°C and 30°API at P29.00/li. Determine the cost of fuel to operate the engine per day.
A. P5,677.50 B. P4,677.50 C. P48,088.90 D. P27,127.76
Solution:

[ ( )]

( )

62. A cylindrical tank 4 m long and 3 m diameter is used for oil storage. How many days can the
tank supply the engine having 27°API with fuel consumption of 60 kg/hr?
A. 17.53 B. 5.84 C. 12.84 D. 19.84
Solution:
( ) ( )

16
̇ ̇ ( )
̇

̇
( )

63. A logging firm in Isabella operates a Diesel Electric Plant to supply its electric energy
requirements. During a 2 hour period, the plant consumed 250 gallons of fuel at 80°F and
produced 2900 kW-hrs. Industrial fuel is used at 30°API and was purchased at P30/li at 60°F.
Determine the overall thermal efficiency of the plant.
A. 26.08% B. 34.23% C. 28.00% D. 18.46%
Solution:
( ) ( )

[ ( )]
( )

̇ ( )( ) ( )( )

-
( )

̇ ( )

64. The dry exhaust gas from the oil engine has the following gravimetric analysis:
CO2 = 21.6% O2 = 4.2% N = 74.2%
Specific heats at constant pressure for each component of the exhaust gas in kCal/kg-°C are:
CO2 = 0.203 O2 = 0.219 N = 0.248
Calculate the specific gravity if the molecular weight of air is 28.97 kg/kg-mol.
A. 0.981 B. 1.244 C. 1.055 D. 0.542
Solution:

17
∑ -
-

65. A bituminous coal has the following composition:


C = 71.5% H = 5.0% O = 7.0% N = 1.3% S = 3% Ash = 7.6%
W = 3.4%
Determine the theoretical weight of Nitrogen in lb/lb of coal.
A. 2.870 B. 7.526 C. 2.274 D. 6.233
Solution:
( ) ( )

( ) ( ) ( )

( )

( )

66. A gaseous fuel mixture has a molal analysis:


H2 = 14% CH4 = 3% CO = 27% O2 = 0.6% CO2 = 4.5% N2 = 50.9%
Determine the air-fuel ratio for complete combustion on molal basis.
A. 2.130 B. 3.230 C. 1.233 D. 1.130
Solution:

( )

67. A volumetric analysis of a gas mixture is a follows:


CO2: 12% O2: 4% N2: 80% CO: 4%
What percentage of CO 2 on a mass basis?
18
A. 17.55% B. 15.55% C. 12.73% D. 19.73%
Solution:

68. The following coal has the following ultimate analysis by weight:
C = 70.5% H2 = 4.5% O2 = 6.0% N2 = 1.0% S = 3.0% Ash = 11%
Moisture = 4%
A stocker fired boiler of 195,000 kg/hr steaming capacity uses this coal as fuel. Calculate volume
of air in m3/hr with air at 60°F and 14.7 air pressure if the boiler efficiency is 70% and FE = 1.10.
A. 234,019 m3/hr B. 215,830 m3/hr C. 213,830 m3/hr D. 264,830 m3/hr
Solution:
( ) ( )

( ) ( ) ( )

( ) ( ) ( ) ( )

( )
( ) ( ) ( )

̇ ( ) ̇ ( )
̇ ̇
( )
̇ ( )

̇ ( )
̇ ̇
̇ ( )( )
̇

19
69. 23.5 kg of steam per second at 5 MPa and 400°C is produced by a steam generator. The
feedwater enters the economizer at 145°C and leaves at 205°C. The steam leaves the boiler
drum with a quality of 98%. The unit consumes 3 kg of coal per second as received having value
of 25,102 kJ/kg. What would be the overall efficiency of the unit in percent?
Steam properties:
@ 5 MPa and 400°C: h = 3195.7 kJ/kg
@ 5 MPa: hf = 1154.23 kJ/kg hfg = 1640.1 kJ/kg
@ 205°C: hf = 875.04 kJ/kg
@ At 145°C: hf = 610.63 kJ/kg
A. 65.72 B. 80.67 C. 88.28 D. 78.82
Solution:
̇ ( ) ( )
̇ ( )

70. In a Rankine cycle steam enters the turbine at 2.5 MPa (enthalpies & entropies given) and
condenser of 50 kPa (properties given), what is the thermal efficiency of the cycle?
@ 2.5 MPa: hg = 2803.1 kJ/kg s g = 6.2575 kJ/kg-K
@ 50 kPa: s f = 1.0910 kJ/kg-K s fg = 6.5029 kJ/kg-K
hf = 340.49 kJ/kg hfg = 2305.4 kJ/kg
3
υf = 0.00103 m /kg
A. 25.55% B. 45.23% C. 34.23% D. 12.34%
Solution:

( )

( )

( ) ( )
( ) ( ) ( ) ( )

71. A thermal power plant generates 5 MW and the heat generated by fuel is 13,000 kJ/s. If the
thermal efficiency is 36.15%, find the power needed for the auxiliaries.
A. 310 kW B. 300 kW C. 400 kW D. 350 kW
Solution:

20
72. A superheat steam Rankine cycle has turbine inlet conditions of 17.5 MPa and 530°C
expands in a turbine to 0.007 MPa. The turbine and pump polytropic efficiencies are 0.85 and
0.75 respectively. Pressure losses between the pump and turbine inlet are 1.5 MPa. What
should be the pump work in kJ/kg?
A. 17.34 B. 27.32 C. 25.32 D. 47.33
Solution:

( ) ( )

73. In an open feedwater heater for a steam plant, saturated steam at 7 bar is mixed with
subcooled liquid at 7 bar and 25°C. Just enough steam is supplied to ensure that the mixed
steam leaving the heater will be saturated liquid at 7 bar when heater efficiency is 95%.
Calculate the mass flow rate of the subcooled liquid if steam flow rate is 0.865 kg/s.
Steam properties:
@ 7 bar, saturated vapor: hg = 2763.5 kJ/kg
@ 7 bar and 25°C: hf = 105.5 kJ/kg
@ 7 bar, saturated liquid: hf = 697.22 kJ/kg
A. 2.725 kg/s B. 3.356 kg/s C. 2.869 kg/s D. 3.948 kg/s
Solution:
̇ ( )
̇ ( )
̇ ( )
( )
̇

74. A steam condenser receives 10 kg/s of steam with an enthalpy of 2770 kJ/kg. Steam
condenses and leaves with an enthalpy of 160 kJ/kg. Cooling water passes through the
condenser with temperature increases from 13°C to 24°C. Calculate the water flow rate in kg/s.
A. 583 B. 567 C. 523 D. 528
Solution:

̇ ( ) ̇ ( )
( ) ̇ ( )( )
21
̇

75. Steam expands adiabatically in a turbine from 2000 kPa, 400°C to 400 kPa, 250°C. What is
the effectiveness of the process in percent assuming an atmospheric pressure of 18°C? Neglect
changes in kinetic and potential energy.
Steam properties:
@ 2000 kPa and 400°C: h = 3247.6 kJ/kg s = 7.1271kJ/kg-K
@ 400 kPa and 250°C: h = 2964.2 kJ/kg s = 7.3789 kJ/kg-K
A. 82 B. 84 C. 79.60 D. 79.46
Solution:

( ) ( )( )

76. A heat exchanger was installed purposely to cool 0.50 kg of gas per second. Molecular
weight is 32 and k = 1.32. The gas is cooled from 150°C to 80°C. Water is available at the rate of
0.30 kg/s and at a temperature of 15°C. Calculate the exit temperature of the water in °C.
A. 44.86 B. 42.86 C. 46.45 D. 40.34
Solution:
-
( )
-

̇ ( ) ̇ ( )
( )( ) ( )( )

77. A 350 mm x 450 mm steam engine running at 280 rpm has a clearance steam condition of 2
MPa and 230°C and exits at 0.1 MPa. The steam consumption is 2000 kg/hr and mechanical
efficiency is 85%. If indicated mean effective pressure is 600 kPa, determine brake thermal
efficiency.
@ 2 MPa and 230°C: h1 = 2849.6 kJ/kg s 1 = 6.4423 kJ/kg-K
@ 0.1 MPa: s f = 1.3026 kJ/kg-K hf = 417.46kJ/kg s fg = 6.0568 kJ/kg-K
hfg = 2258 kJ/kg hf2 = 417.46 kJ/kg
A. 23.34% B. 15.25% C. 14.16% D. 27.34%
Solution:

22
( ) [( ) ( ) ]( )( ) ( )( )( )

̇ ( ) ( )( )

78. A steam turbine receives 5,000 kg/hr of steam at 5 MPa and 400°C and velocity of 30 m/s. It
leaves the turbine at 0.006 MPa and 85% quality and velocity of 15 m/s. Radiation loss is 10,000
kJ/hr. Find the kW developed.
@5 MPa and 400°C: h1 = 3195.7 kJ/kg s 1 = 6.6459 kJ/kg-K
@ 0.006 Mpa: hf = 151.53 kJ/kg hfg = 2415.9 kJ/kg
A. 1273.29 B. 2173.29 C. 1373.60 D. 7231.29
Solution:
( )
̇ ̇ ( )( )
̇ ̇ ( )( )

̇ ̇ ̇ ̇ ̇ ̇
̇ ( ̇ ̇ ) ̇( ) ̇
( ) ( )

79. A steam turbine with 85% stage efficiency receives steam at 7 MPa and 550°C and exhausts
at 20 kPa. Determine the turbine work.
@ 7 MPa and 550°C: h1 = 3530.9 kJ/kg s 1 = 6.9486 kJ/kg-K
@ 20 kPa (0.020 MPa): s f = 0.8320 kJ/kg-K hf = 251.40 kJ/kg
s fg = 7.0766 kJ/kg-K hfg = 2358.3 kJ/kg
A. 1,117 kJ/kg B. 1,132 kJ/kg C. 1,123.34 kJ/kg D. 1,054.95 kJ/kg
Solution:

( )

( )

23
80. A steam turbine with 80% stage efficiency receives steam at 7 MPa and 550°C and exhausts
at 20 kPa. Determine the quality at exhaust.
@ 7 MPa and 550°C: h1 = 3530.9 kJ/kg s 1 = 6.9486 kJ/kg-K
@ 20 kPa (0.020 MPa): s f = 0.8320 kJ/kg-K hf = 251.40 kJ/kg
s fg = 7.0766 kJ/kg-K hfg = 2358.3 kJ/kg
A. 96.96% B. 76.34% C. 82.34% D. 91.69%
Solution:

( )

( )

( )

81. An 18,000 kW geothermal plant has a generator efficiency of 90% and 80% respectively. If
the quality after throttling is 20% and each well discharges 400,000 kg/hr, determine the
number of wells required to produce if the change of enthalpy at entrance and exit of turbine is
500 kJ/kg.
A. 4 wells B. 2 wells C. 6 wells D. 8 wells
Solution:
̇ ̇ ( )
( )
̇ ( )
̇
̇
̇

82. A liquid dominated geothermal plant with a single flash separator receives water at 204°C.
The separator pressure is 1.04 MPa. A direct contact condenser operates at 0.034 MPa. The
turbine has a polytropic efficiency of 0.75. For cycle output of 60 MW, what is the mass flow
rate of the well water in kg/s?
@ 204°C: hf = 870.51 kJ/kg
@ 1.04 MPa: hf = 770.38 kJ/kg hfg = 2009.2 kJ/kg hg = 2779.6 kJ/kg

24
s g = 6.5729 kJ/kg-K
@ 0.034 MPa: hf = 301.40 kJ/kg hfg = 2328.8 kJ/kg s f = 0.09793 kJ/kg-K
s fg = 6.7463 kJ/kg-K
A. 2,933 B. 2,100 C. 1,860 D. 2,444
Solution:

( )

( )
̇ ̇ ( )
̇ ( )( )
̇
( )

( )

̇ ̇
̇
̇

83. An engine-generator rated 9000 kVA at 80% power factor, 3 phase, 4160 V has an efficiency
of 90%. If the overall plant efficiency is 28%, what is the heat generated by the fuel?
A. 18,800 kW B. 28,800 kW C. 7,500 kW D. 25,714 kW
Solution:

84. The indicated thermal efficiency of a two stroke diesel engine is 60%. If friction power is
15% of heat generated, determine the brake thermal efficiency of the engine.
A. 43% B. 45% C. 36% D. 37%
Solution:

̇
25
̇
̇ ̇ ̇
̇
̇ ̇

85. A 305 mm x 457 mm four stroke single acting diesel engine is rated at 150 kW at 260 rpm.
Fuel consumption at rated load is 0.56 kg/kW-hr with a heating value of 43,912 kJ/kg. Calculate
the break thermal efficiency.
A. 10.53% B. 27.45% C. 14.64% D. 18.23%
Solution:
̇
-

( )

86. A waste heat recovery boiler produces 4.8 MPa (dry saturated) steam from 104°C
feedwater. The boiler receives energy from 7 kg/s of 954°C dry air. After passing through a
waste heat boiler, the temperature of the air has been reduced to 343°C. How much steam in
kg is produced per second? Note: @ 4.80 MPa dry saturated, h = 2796 kJ/kg.
A. 1.30 B. 0.92 C. 1.81 D. 3.43
Solution:

( )

̇ ( ) ̇ ( )
( )( ) ̇ ( )
̇

87. A diesel electric power plant supplies energy for Meralco. During a 24-hour period, the plant
consumed 240 gallons of fuel at 28°C and produced 3930 kW-hr. Industrial fuel used is 28°API
and was purchased at P30 per liter at 15.6°C. What is the cost of fuel to produce one kW-hr?
A. P6.87 B. P1.10 C. P41.07 D. P5.00
Solution:

( )
[ ( )]
( )

26
( )
-

88. In a gas unit, air enters the combustion chamber at 550 kPa, 227°C and 43 m/s. The
products of combustion leave the combustor at 511 kPa, 1004°C and 180 m/s. Liquid fuel enters
with a heating value of 43,000 kJ/kg. For fuel-air ratio of 0.0229, what is the combustor
efficiency in percent?
A. 70.38% B. 79.38% C. 75.38% D. 82.38%
Solution:
( )

( ) ( ) ( )

89. The specific speed of turbine is 85 rpm and running at 450 rpm. If the head is 20 m and
generator efficiency is 90%, what is the maximum power delivered by the generator?
A. 450.51 kW B. 354.52 kW C. 650.53 kW D. 835.57 kW
Solution:


( )

( )

90. In Francis turbine, the pressure gage leading to the turbine casing reads 380 kPa. The
velocity of water entering the turbine is 8 m/s. If net head of the turbine is 45 m, find the
distance from the center of the spiral casing to the tailrace.
A. 3.0 m B. 3.5 m C. 4.0 m D. 4.5 m
Solution:

( )
27
91. A turbine has a mechanical efficiency of 93%, volumetric efficiency of 95% and total
efficiency of 82%. If the effective head is 40 m, find the total head.
A. 48.72 m B. 40.72 m C. 36.22 m D. 34.72 m
Solution:

( )

( )

92. A Pelton type turbine has 25 m head friction loss of 4.5 m. The coefficient of friction head
loss (Morse) is 0.00093 and penstock length of 80 m. What is the penstock diameter?
A. 1,355.73 mm B. 3476.12 mm C. 6771.23 mm D. 1686.73 mm
Solution:

√ √ ( )( )

( )( )( )

93. In a 9,000 kW hydro-electric plant, the overall efficiency is 88% and the actual power
received by the costumer is 110,000 kW-hrs for that day. What is the secondary power that this
plant could deliver during the entire day?
A. 58,960 kW-hrs B. 80,080 kW-hrs C. 65,960 kW-hrs D. 70,960 kW-hrs
Solution:
( )( ) -
-

94. A Pelton type turbine was installed 30 m below the head gate of the penstock. The head
loss due to friction is 12 percent of the given elevation. The length of the penstock is 100 m and
the coefficient of friction is 0.00093. Determine the power output in kW. (Use Morse equation).
A. 22,273 B. 23,234 C. 32,345 D. 34,452
Solution:
( )

28
√ √ ( )( )

( )( )( )

( ) ( )
( )( )

95. Water flows steadily with a velocity of 3.05 m/s in a horizontal pipe having a diameter of
25.24 cm. At one section of the pipe, the temperature and pressure of the water are 21°C and
689.3 kPa respectively. At a distance of 304.8 m downstream, the pressure is 516.9 kPa. What is
the friction factor?
A. 0.134 B. 0.0050 C. 0.0307 D. 0.641
Solution:

( )( )
( )( )

96. A hydro-electric plant having 30 sq. km reservoir area and 100 m head is used to generate
power. The energy utilized by the consumers whose load is connected to the power plant
during a five-hour period is 13.5x106 kWh. The overall generation efficiency is 75%. Find the fall
in the height of water in the reservoir after the 5-hour period.
A. 5.13 m B. 1.32 m C. 3.21 m D. 2.20 m
Solution:

( ) ( )( )

( )

97. The gas density of chimney is 0.75 kg /m3 and air density of 1.15 kg /m3. Find the driving
pressure if the height of the chimney is 63.71 m.
A. 0.15 kPa B. 0.25 kPa C. 0.35 kPa D. 0.45 kPa
Solution:
29
( ) ( )( )

98. The actual velocity of gas entering in a chimney is 8 m/s. The gas temperature is 25°C with a
gas constant of 0.287 kJ/kg-K. Determine the gas pressure for a mass of a gas is 50,000 kg/hr
and chimney diameter of 1.39 m.
A. 95 kPa B. 98 kPa C. 101 kPa D. 92 kPa
Solution:
̇ ( ) ( )
̇ ̇
( ) ( )( )

99. A steam generator with economizer and air heater has an overall draft loss of 25.78 cm of
water. If the stack gases are at 177°C and if the atmosphere is at 101.3 kPa and 26°C, what
theoretical height of stack in meters is needed when no draft fan is used? Assume that the gas
constant for the flue gases is the same as that for air.
A. 611.10 B. 631.10 C. 651.10 D. 671.10
Solution:

( )

( )
( )
( )
( )

100. A foundation measures 12 ft x 14 ft x 16 ft. Find the number of sacks of cement needed for
a 1:2:4 mixture.
A. 302 B. 404 C. 356 D. 598
Solution:
( )

( )

101. A rectangular foundation cross-section has a bed plate dimension of 8 ft x 10 ft. The
uniform clearance on each side is 1 ft. The height of the foundation is 4.5 ft. If the weight of the

30
steel bar reinforcements needed is ½% of weight of foundation, find the weight of the steel
bars. Use concrete density of 2400 kg/m3.
A. 173.47 kg B. 183.47 kg C. 163.47 kg D. 153.47 kg
Solution:
( )( )
( )
( )
( )

102. A steam pipe having a surface temperature of 250°C passes through a room where the
temperature is 27°C. The outside diameter of the pipe is 100 mm and emissivity factor is 0.8.
Calculate the radiated heat loss for 3 m pipe length.
A. 1434.47 W B. 3746.35 W C. 2851.82 W D. 3546.45 W
Solution:
( )( )

( )
-
( )( )( )
-

103. Brine enters a circulating brine cooler at the rate of 60 m3/hr at -8°C and leaves at -18°C.
Specific heat of brine is 1.072 kJ/kg-K and a specific gravity of 1.12. Determine the tons of
refrigeration.
A. 53.5 TR B. 65.3 TR C. 33.5 TR D. 56.9 TR
Solution:
( )
( )
̇
̇ ̇ ( )(- )

104. A turbo-charged, 16 cylinder, Vee-type diesel engine has an air consumption of 3,000 kg/hr
per cylinder at rated load and speed. This air is drawn in through a filter by a centrifugal
compressor directly connected to the exhaust gas turbine. The temperature of the air from the

31
compressor is 135°C and a counter flow air cooler reduces the air temperature to 45°C before it
goes to the engine suction header. Cooling water enters the air cooler at 30°C and leaves at
40°C. Calculate the log mean temperature difference.
A. 47.23°C B. 87.82°C C. 43.34°C D. 65.24°C
Solution:

105. Water is flowing in a pipe with radius of 30 cm at a velocity of 5 m/s at the temperature in
the pipe. The density and viscosity of the water are as follows: density = 997.9 kg/s viscosity =
1.131 Pa-s. What is the Reynolds number for this situation?
A. 2647 B. 96.2 C. 3100 D. 1140
Solution:

( )

( )

106. Compute the amount of condensate formed during a 10 minute warm-up of 180 meter
pipe conveys the saturated steam with enthalpy of vaporization, h fg = 1947.8 kJ/kg. The
minimum external temperature of pipe is 2°C. The final temperature of pipe is 195°C. The
specific heat of pipe material is 0.6 kJ/kg-°C. The specific weight is 28 kg/m.
A. 240.69 kg B. 982.45 kg C. 299.64 kg D. 423.45 kg
Solution:
( )

( ) ( )
( ) ( )( )

107. The discharge pressure of an air compressor is 5 times the suction pressure. If volume flow
at suction is 0.1 m3/s, what is the suction pressure if compressor work is 19.57 kW (use n =
1.35).

32
A. 97 kPa B. 98 kPa C. 99 kPa D. 100 kPa
Solution:

̇ [( ) ]
( )
[ ]

108. The initial condition of air in an air compressor is 98 kPa and 27 °C and discharges air at
450 kPa. The bore and stroke are 355 mm and 381 mm, respectively with percent clearance of
8% running at 300 rpm. Find the volume of air at suction.
A. 541.62 m3/hr B. 551.62 m3/hr C. 561.62 m3/hr D. 571.62 m3/hr
Solution:

( ) ( )

( ) ( )( )
( )

109. An air compressor has a suction volume of 0.35 m3/s at 97 kPa and discharges at 650 kPa.
How much power is saved by the compressor if there are two stages?
A. 18.27 kW B. 16.54 kW C. 13.86 kW D. 11.58 kW
Solution:
( )( )
̇ [( ) ] [( ) ]

√ √ ( )
( )( )( )
̇ [( ) ] [( ) ]

110. A two stage air intercooler has an intercooler pressure of 4 kg/cm2. What is the discharge
pressure if suction pressure is 1 kg/cm2?
A. 3 kg/cm2 B. 9 kg/cm2 C. 12 kg/cm2 D. 16 kg/cm2
Solution:

33
111. A two-stage air compressor at 100 kPa and 22°C discharges to 750 kPa. If the intercooler
intake is 105°C, determine the value of n.
A. 1.400 B. 1.326 C. 1.345 D. 1.288
Solution:
√ ( )

( )

( )

112. A single acting compressor has a volumetric efficiency of 89%, operates at 500 rpm. It
takes in air at 100 kPa and 30°C and discharges it at 600 kPa. The air handled is 8 m 3/min
measured at discharged condition. If compression is isentropic, find the effective mean
pressure in kPa.
A. 233.34 B. 973.17 C. 198.34 D. 204.82
Solution:

( )

( )( )
̇ [( ) ] [( ) ]

113. A water-jacketed air compressor handles 0.343 m3/s of air entering at 96.5 kPa and 21°C
and leaving at 480 kPa and 132°C; 10.9 kg/hr of cooling water enters the jacket at 15°C and
leaves at 21°C. Determine the compressor brake power.
A. 23.163 kW B. 62.65 kW C. 34.44 kW D. 19.33 kW
Solution:
34
( )

( )

( )( )
̇ [( ) ] [( ) ]

̇ ̇ ( ) ( )( )
̇ ̇

114. A double suction centrifugal pump delivers 20 ft3/s of water at a head of 12 m and running
at 650 rpm. What is the specific speed of the pump?
A. 5014.12 rpm B. 6453.12 rpm C. 2770.73 rpm D. 9968.73 rpm
Solution:

115. Determine the number of stages needed for a centrifugal pump if it is used to deliver 400
gal/min of water and pump power of 15 hp. Each impeller develops a head of 30 ft.
A. 6 B. 4 C. 5 D. 7
Solution:
̇
( )

116. The suction pressure of a pump reads 3 in. of mercury vacuum and discharge pressure
reads 140 psi is used to deliver 120 gpm of water with specific volume of 0.0163 ft 3/lb.
Determine the pump work.
A. 4.6 kW B. 5.7 kW C. 7.4 kW D. 8.4 kW
Solution:
- -

35
-

( )( )

117. A submersible pump delivers 350 gpm of water to a height of 5 ft from the ground. The
pump where installed 150 ft below the ground level and a drawdown of 8 ft during the
operation. If water level is 25 ft above the pump, determine the pump power.
A. 7.13 kW B. 4.86 kW C. 7.24 kW D. 9.27 kW
Solution:
( )

̇ ( )( )

118. A vacuum pump is used to drain a flooded mine shaft of 20°C water. The pump pressure of
water at this temperature is 2.34 kPa. The pump is incapable of lifting the water higher than 16
m. What is the atmospheric pressure?
A. 159.30 B. 132.33 C. 196.22 D. 171.9
Solution:

( )

119. A submersible, multi-stage, centrifugal deep well pump 260 gpm capacity is installed in a
well 27 ft below the static water level and running at 3000 rpm. Drawdown when pumping at
rated capacity is 10 feet. The pump delivers the water into a 25,000 gallons capacity overhead
storage tank. Total discharge head developed by pump, including friction in piping is 243 ft.
Calculate the diameter of the impeller of this pump in inches if each impeller diameter
developed a head of 38 ft.
A. 3.28 B. 5.33 C. 3.71 D. 6.34
36
Solution:


( ) √ ( )( )

120. A fan draws 1.42 m3 per second of air at a static pressure of 2.54 cm of water through a
duct 300 mm diameter and discharges it through a duct of 275 mm diameter. Determine the
static fan efficiency if total fan is 75% and air is measured at 25°C and 760 mm Hg.
A. 50.11% B. 53.69% C. 65.67% D. 45.34%
Solution:

( )
( )

( )

( )

( )

( )( )

( )( )

121. A water cooler uses 50 lb/hr of melting ice to cool running water from 80°F to 42°F. Based
on the inside coil area, Ui = 110 Btu/hr-ft2-°F. Find the gpm of water cooled.
A. 0.10 GPM B. 0.21 GPM C. 0.38 GPM D. 0.45 GPM
Solution:
̇ ̇ ̇ ( )
( ) ̇ ( )( )
̇
( )

37
122. The charge in a Diesel engine consists of 18.34 grams of fuel, with lower heating value of
42,571 kJ/kg, and 409 grams of fuel and products of combustion. At the beginning of
compression, T1 = 60°C. Let rk = 14. For constant c p = 1.11 kJ/kg-°C, what should be the cut-off
ratio in the corresponding ideal cycle?
A. 2.05 B. 2.34 C. 5.34 D. 2.97
Solution:
( )

( )

( )
( )
( )( )

123. The gain of entropy during isothermal nonflow process of 5 lb of air at 60°F is 0.462 Btu/°R.
Find the V1/V2.
A. 3.85 B. 0.259 C. 1.0 D. 0.296
Solution:

( )

124. An auditorium seating 1500 people is to be maintained at 80°F dry bulb and 60°F wet bulb
temperature when outdoor air is at 91°F dry bulb and 75°F wet bulb. Solar heat load is 110,000
Btu/hr and supply air is at 60°F, determine the amount of supply air.
A. 93,229.17 lb/hr B. 83,229.17 lb/hr C. 73,229.17 lb/hr D. 63,229.17 lb/hr
Solution:

̇ ( )
̇ ̇ ( )
̇( )( )
38
̇

125. In a Brayton cycle that operates between temperature limits of 300 K and 1773 K with k =
1.4, determine the temperature at the end of compression (isentropic) for maximum work of
the cycle.
A. 780 K B. 690.5 K C. 730 K D. 350 K
Solution:

( )

126. A 35% solution leaves the absorber and 30% solution enters the absorber. The heat
removed from the absorber by cooling water is 547.6 Btu and ammonia is superheated by 10°.
Find the pound per pound of ammonia gas from the evaporating coils.
A. 11 B. 12 C. 13 D. 14
Solution:

127. A Carnot refrigeration system operates at T max/Tmin = 1.5. Find the kW per ton of
refrigeration.
A. 1.91 B. 2.15 C. 1.76 D. 1.55
Solution:
̇
( ) ( )

( )

128. Assume 8 ft3 of air at 100 psi and 100°F are compressed isothermally to a volume of 2 ft 3.
For each end state of the process, find the bulk modulus.
A. 400 and 100 psi B. 400 and 110 psi C. 400 and 120 psi D. 400 and 130 psi
Solution:
- ( )

( ) ( )

- ( )

39
- ( )

129. Predict the pressure of nitrogen gas at T = 200 K and υ = 0.00385 L/g and b = 0.00141 L/g; a
= 0.178 L2-kPa/g2. Use Van der Waals equation.
A. 15,331 kPa B. 14,331 kPa C. 13,331 kPa D. 12,331 kPa
Solution:
( )
( )

130. A Francis turbine is to be operated at a speed of 600 rpm and with discharge of 4 m3/s. If r1
= 0.6 m, β = 110°, and the blade height B is 10 cm, what should be the guide vane angle α1 for
nonseparating flow condition at the runner entrance?
A. 14.4° B. 15.4° C. 16.4° D. 17.4°
Solution:
( )

( )( )

( )( )

( )

131. The total head of fan is 187 m and has a static pressure of 210 mm of water gage, what is
the velocity of air flowing if the density of air is 1.15 kg/m3?
A. 6.85 m/s B. 3.45 m/s C. 4.39 m/s D. 9.28 m/s
Solution:
( )

( )

132. A fan delivers 5.7 m3/s at a static pressure of 5.08 cm of water when operating at a speed
of 400 rpm. The power input required is 2.963 kW. If 7.05 m3/s are desired in the same fan and
installation, find the pressure in cm of water.
A. 7.77 B. 17.14 C. 11.43 D. 5.08
Solution:

40
( )

( )

133. A rigid container is closed at one end and measures 8 in diameter by 12 in long. The
container is held vertical and is slowly moved downward until the pressure in the container is
17 psia. What will be the depth of the top of the container from the free water surface?
A. 42.36 in B. 59.29 in C. 63.69 in D. 69.82 in
Solution:

( )

134. An empty, open can is 30 cm high with a 15 cm diameter. The can, with the open end
down, is pushed under water with a density of 1000 kg/m3. Find the water level in the can
when the top of the can is 50 cm below the surface.
A. 17.20 cm B. 2.12 cm C. 4.20 cm D. 5.87 cm
Solution:
( )

( ) [ ( )]

41
135. A cylindrical pipe with water flowing downward at 0.03 m3/s having top diameter of 0.08,
bottom diameter of 0.04 m and height of 1.5 m. Find the pressure inside the pipe.
A. 154.63 kPa B. 197.93 kPa C. 252.44 kPa D. 243.92 kPa
Solution:

( )

( )

( )

( )
(- )

136. Determine the size of pipe which will deliver 8 liters of medium oil (ν = 6.10x10-6 m2/s)
assuming laminar flow conditions.
A. 622 mm B. 754 mm C. 950 mm D. 835 mm
Solution:

( -
)

137. The type of flow occupying in a 1 cm diameter pipe which water flows at a velocity of 2.50
m/s. Use ν = 1.13x10-6 m2/s for water.
A. turbulent B. constant C. laminar D. none of these
Solution:
( ) -

138. What force is exerted by a water jet, 60 mm in diameter, if it strikes a wall at the rate of 15
m/s?
42
A. 636.17 N B. 442.62 N C. 764.23 N D. 563.34 N
Solution:

[ ( ) ]( )
( )( )( )

139. A 300 mm diameter pipe discharges water at a rate of 200 li/s. Point 1 on the pipe has a
pressure of 260 kPa and 3.4 m below point 1 is point 2 with a pressure of 300 kPa. Compute the
head loss between points 1 and 2.
A. 4.29 m B. 2.59 m C. 6.32 m D. 1.87 m
Solution:

( )

140. Water flowing at the rate of 10 m/s from an orifice at bottom of a reservoir. Find the
pressure at the bottom of reservoir.
A. 30 kPa B. 40 kPa C. 50 kPa D. 60 kPa
Solution:

( )
( )

141. Steam flows through a nozzle at 400°C and 1 MPa (h = 3263.9 kJ/kg) with velocity of 300
m/s. Find the stagnation enthalpy.
A. 3300 kJ/kg B. 3290 kJ/kg C. 3320 kJ/kg D. 3309 kJ/kg
Solution:

142. Air flows through a nozzle at a speed of 350 m/s. Find the stagnation temperature if the
entrance temperature is 200°C.
A. 241.25°C B. 251.25°C C. 261.25°C D. 271.25°C
Solution:
( ) ( )

43
143. Carbon dioxide flows through a nozzle with a speed of 400 m/s. Compute the dynamic
temperature.
A. 92.56 K B. 94.56 K C. 96.56 K D. 98.56 K
Solution:
-

( )

144. Carbon dioxide flows through a nozzle with speed of 380 m/s. The entrance condition of
nozzle is 250°C and 1200 kPa. Find the stagnation pressure.
A. 2,136.34 kPa B. 2,146.34 kPa C. 2,156.34 kPa D. 2,166.34 kPa
Solution:

( )

( )

145. Air enters a diffuser with a velocity of 200 m/s. Determine the velocity of sound if air
temperature is 30°C.
A. 349 m/s B. 359 m/s C. 369 m/s D. 379 m/s
Solution:
√ √ ( )( )( )

146. Air flows through a nozzle with temperature of entrance of 420 K stagnation temperature
of 468 K. Find the Mach number.
A. 0.744 B. 0.754 C. 0.764 D. 0.774
Solution:

√ √ ( )( )( )
44
147. Air at 300 K and 200 kPa is heated at constant pressure to 600 K. Determine the change of
internal energy.
A. 245.58 kJ/kg B. 235.58 kJ/kg C. 225.58 kJ/kg D. 215.58 kJ/kg
Solution:
( ) ( )( )

148. An insulated rigid tank initially contains 1.5 lb of helium at 80°F and 50 psia. A paddle
wheel with power rating of 0.02 hp is operated within the tank for 30 min. Determine the final
temperature.
A. 159.22°F B. 169.22°F C. 179.22°F D. 189.22°F
Solution:
( )

( )( ) ( )( )

149. A 4 m2 asphalt pavement with emissivity of 0.85 has a surface temperature of 50°C. Find
the maximum rate of radiation that can be emitted from the surface.
A. 2,068 .32 watts B. 2,078.32 watts C. 2,088.32 watts D. 2,098.32 watts
Solution:
̇ ( -
) ( )( )

150. Air at 10°C and 80 kPa enters a diffuser of a jet engine steadily with a velocity of 200 m/s.
The inlet area of diffuser is 0.40 m2. Determine the mass flow rate of air.
A. 72.79 kg/s B. 74.79 kg/s C. 76.79 kg/s D. 78.79 kg/s
Solution:

( )
̇ ( )( )

151. Consider a refrigerator whose 40 watts light bulb remains on continuously as a result of a
malfunction of the switch. If the refrigerator has a COP of 1.3 and the cost of electricity is 8
cents per kW-hr, determine the increase in the energy consumption of the refrigerator and its
cost per year if the switch is not fixed.
A. P 49.59 B. P 47.59 C. P 45.59 D. P 43.59
45
Solution:

̇
̇
̇ ̇ ̇
̇
( )( )

152. A 75 hp motor that has an efficiency of 91% is worn out and is replaced by a high-efficiency
motor that has an efficiency of 95.4%. Determine the reduction in heat gain of the room due to
higher efficiency under full-load conditions.
A. 2.24 kW B. 2.44 kW C. 2.64 kW D. 2.84 kW
Solution:
( )
( )
̇

153. A household refrigerator that has a power input of 450 watts and a COP of 2.5 is to cool
five large watermelons, 10 kg each, to 8°C. If the watermelons are initially at 20°C, determine
how long it will take for the refrigerator to cool them. The watermelons can be treated as water
whose specific heat is 4.2 kJ/kg-K.
A. 2220 seconds B. 2230 seconds C. 2240 seconds D. 2250 seconds
Solution:

( )
( )( )

154. When a man returns to his wall-sealed house on a summer day, he finds that the house is
at 32°C. He returns on the air conditioner which cools the entire house to 20°C in 15 minutes. If
COP is 2.5, determine the power drawn by the air conditioner. Assume the entire mass within
the house is 800 kg of air for which c v = 0.72 kJ/kg-K, cp = 1.0 kJ/kg-K.
A. 1.072 kW B. 2.072 kW C. 3.072 kW D. 4.072 kW
Solution:
46
̇ ( ) ( )( )( )

155. A heat source at 800 K losses 2000 kJ of heat to a sink at 500 K. Determine the entropy
generated during this process.
A. 1.5 kJ/K B. 2.5 kJ/K C. -2.5 kJ/K D. 4 kJ/K
Solution:
- -
-

156. Helium gas is compressed in an adiabatic compressor from an initial state of 14 psia and
50°F to a final temperature of 320°F in a reversible manner. Determine the exit pressure of
Helium.
A. 38.5 psia B. 40.5 psia C. 42.5 psia D. 44.5 psia
Solution:

( )

( )

157. Air passes thru a nozzle with efficiency of 90%. The velocity of air at the exit is 600 m/s.
Find the actual velocity at the exit.
A. 382 m/s B. 540 m/s C. 458 m/s D. 568 m/s
Solution:
( )

( )

158. A 50 kg block of iron casting at 500 K is thrown into a large lake that is at a temperature of
285 K. The iron block eventually reaches thermal equilibrium with the lake water. Assuming
average specific heat of 0.45 kJ/kg-K for the iron, determine the entropy generated during this
process.

47
A. -12.65 kJ/K B. 16.97 kJ/K C. 4.32 kJ/K D. 6.32 kJ/K
Solution:
( ) -
( )( )

159. A windmill with a 12 m diameter rotor is to be installed at a location where the wind is
blowing at an average velocity of 10 m/s. Using standard condition of air (1 atm, 25°C),
determine the maximum power that can be generated by the windmill.
A. 68 kW B. 70 kW C. 72 kW D. 74 kW
Solution:

( )

̇ ( )( )

̇ ( )

160. Consider a large furnace that can supply heat at a temperature of 2000°R at a steady rate
of 3000 Btu/s. Determine the exergy of this energy. Assume an environment temperature of
77°F.
A. 2305.19 kW B. 2315.19 kW C. 2325.19 kW D. 2335.19 kW
Solution:
( )

̇ ̇ ( )

161. A heat engine receives heat from a source at 1200 K at a rate of 500 kJ/s and rejects the
waste heat to a medium at 300 K. The power output of the engine is 180 kW. Determine the
irreversibility rate for this process.
A. 190 kW B. 195 kW C. 200 kW D. 205 kW
Solution:

̇ ̇
̇

48
162. A dealer advertises that he has just received a shipment of electric resistance heaters for
residential buildings that have an efficiency of 100 percent. Assuming an indoor temperature of
21°C and outdoor temperature of 10°C, determine the second law efficiency of these heaters.
A. 8.74% B. 6.74% C. 3.74% D. 4.74%
Solution:

163. A thermal power plant has a heat rate of 11,363 Btu/kW-hr. Find the thermal efficiency of
the plant.
A. 34% B. 24% C. 26% D. 30%
Solution:

164. A rigid tank contains 2 kmol of N2 and 6 kmol of CO 2 gases at 300 K and 115 MPa. Find the
tank volume using the ideal gas equation.
A. 7.33 m3 B. 5.33 m3 C. 3.33 m3 D. 1.33 m3
Solution:

( )( )( )

165. A spherical balloon with a diameter of 6 m is filled with helium at 20°C and 200 kPa.
Determine the mole number.
A. 9.28 kmol B. 10.28 kmol C. 11.28 kmol D. 13.28 kmol
Solution:

[ ( ) ] ( )( )

166. The air in an automobile tire with a volume of 0.53 ft3 is at 90°F and 20 psig. Determine the
amount of air that must be added to raise the pressure to the recommended value of 30 psig.
Assume the atmospheric pressure to be 14.7 psia and the temperature and the volume to
remain constant.
A. 0.026 lb B. 0.046 lb C. 0.066 lb D. 0.086 lb
49
Solution:

( )( )( ) ( )( )

( )( )( ) ( )( )

167. A rigid tank contains 20 lbm of air at 20 psia and 70°F. More air is added to the tank until
the pressure and temperature rise to 35 psia and 90°F, respectively. Determine the amount of
air added to the tank.
A. 11.73 lb B. 13.73 lb C. 15.73 lb D. 17.73 lb
Solution:

( )( )

( ) ( )( )

168. A rigid tank contains 5 kg of an ideal gas at 4 atm and 40°C. Now a valve is opened, and
half of mass of the gas is allowed to escape. If the final pressure in the tank is 1.5 atm, the final
temperature in the tank is?
A. -38°C B. -30°C C. 40°C D. 53°C
Solution:

( )( )

( ) ( )
- -

169. The pressure of an automobile tire is measured to be 200 kPa (gage) before the trip end
220 kPa (gage) after the trip at a location where the atmospheric pressure is 90 kPa. If the
temperature of the air in the tire before the trip is 25°C, the air temperature after the trip is?

50
A. 45.6°C B. 54.8°C C. 27.5°C D. 26.7°C
Solution:

170. Water is boiling at 1 atm pressure in a stainless steel pan on an electric range. It is
observed that 2 kg of liquid water evaporates in 30 min. The rate of heat transfer to the water
is?
A. 2.07 kW B. 0.47 kW C. 2.51 kW D. 3.12 kW
Solution:
̇ ̇ ( )
( )

171. Consider a person standing in a breezy room at 20°C. Determine the total rate of heat
transfer from this person if the exposed surface area and the average outer surface
temperature of the person are 1.6 m2 and 29°C, respectively, and the convection heat transfer
coefficient is 6 W/m2 with emissivity factor of 0.95.
A. 86.40 watts B. 81.70 watts C. 198.1 watts D. 168.1 watts
Solution:
̇ ( ) ( )( )
̇ ( -
)( )[( ) ( ) ]
̇ ̇ ̇

172. Water is boiler in a pan on a stove at sea level. During 10 minutes of boiling, it is observed
that 200 grams of water has evaporated. Then the rate of heat transfer to the water is:
A. 0.84 kJ/min B. 45.1 kJ/min C. 41.8 kJ/min D. 53.5 kJ/min
Solution:
̇ ̇ ( )

173. An aluminum pan whose thermal conductivity is 237 W/m-°C has a flat bottom whose
diameter is 20 cm and thickness of 0.4 cm. Heat is transferred steadily to boiling water in the
pan through its bottom at a rate of 500 watts. If the inner surface of the bottom of the pan is
105°C, determine the temperature of the surface of the bottom of the pan.
A. 95.27°C B. 105.27°C C. 115.27°C D. 125.27°C
Solution:

51
( )
( )
̇
( )( )

174. For heat transfer purposes, a standing man can be modeled as a 30 cm diameter, 170 cm
long vertical cylinder with both the top and bottom surfaces insulated and with the side surface
at an average temperature of 34°C. For a convection heat transfer coefficient of 15 W/m 2-°C,
determine the rate of heat loss from this man by convection in an environment at 20°C.
A. 316.46 watts B. 326.46 watts C. 336.46 watts D. 346.46 watts
Solution:
̇ ( ) ( )( )

175. A 5 cm diameter spherical ball whose surface is maintained at a temperature of 70°C is


suspended in the middle of a room at 20°C. If the convection heat transfer coefficient is 15
W/m2-°C and the emissivity of the surface is 0.8, determine the total heat transfer from the
ball.
A. 23.56 watts B. 32.77 watts C. 9.22 watts D. 43.45 watts
Solution:
( )
̇ ( ) ( )( )
̇ ( -
)( )[( ) ( ) ]
̇ ̇ ̇

176. A frictionless piston-cylinder device and a rigid tank contain 1.2 kmol of an ideal gas at the
same temperature, pressure and volume. Now heat is transferred, and the temperature of both
systems is raised by 15°C. The amount of extra heat that must be supplied to the gas in the
cylinder that is maintained at constant pressure is?
A. 0 B. 50 kJ C. 100 kJ D. 150 kJ
Solution:
( ) ( )( )

177. A supply of 50 kg of chicken at 6°C contained in a box is to be frozen to -18°C in a freezer.


Determine the amount of heat that needs to be removed. The latent heat of the chicken is 247
kJ/kg, and its specific heat is 3.32 kJ/kg-°C above freezing and 1.77 kJ/kg-°C below freezing. The

52
container box is 1.5 kg, and the specific heat of the box material is 1.4 kJ/kg -°C. Also the
freezing temperature of chicken is -2.8°C.
A. 15,206.4 kJ B. 50.4 kJ C. 15,156 kJ D. 1,863 kJ
Solution:
[ ( ) (- )]
( )( )

178. Water is being heated in a closed pan on top of a range while being stirred by a paddle
wheel. During the process, 30 kJ of heat is transferred to the water, and 5 kJ of heat is lost to
the surrounding air. The paddle wheel work amounts to 500 N-m. Determine the final energy of
the system if its initial energy is 10 kJ.
A. 35.5 kJ B. 40.5 kJ C. 25.5 kJ D. 14.5 kJ
Solution:

179. A classroom that normally contains 40 people is to be air-conditioned with window air-
conditioning units of 5 kW cooling capacity. A person at rest may be assumed to dissipate heat
at a rate about 360 kJ/hr. There are 10 light bulbs in the room, each with a rating of 100 watts.
The rate of heat transfer to the classroom through the walls and the windows is estimated to
be 15,000 kJ/hr. If the room is to be maintained at a constant temperature of 21°C, determine
the number of window air-conditioning units required.
A. 1 units B. 2 units C. 3 units D. 4 units
Solution:
̇ ( )( ) ( )

180. A 4 m x 5 m x 6 m room is to be heated by a baseboard resistance heater. It is desired that


the resistance heater be able to raise the air temperature in the room from 7 to 23°C within 15
minutes. Assuming no heat losses from the room and an atmospheric pressure of 100 kPa,
determine the required power of the resistance heater. Assume constant specific heats at room
temperature.
A. 2.34 kW B. 1.91 kW C. 4.56 kW D. 6.34 kW
Solution:

( )
( )
53
( ) ( )( )
( )

181. A student living in a 4 m x 6 m x 6 m dormitory room turns on her 150 watts fan before she
leaves the room on a summer day, hoping that the room will be cooler when she comes back in
the evening. Assuming all the doors and windows are tightly closed and disregarding any heat
transfer through the walls and the windows, determine the temperature in the room when she
comes back 10 hours later. Use specific heat values at room temperature, and assume the room
to be at 100 kPa and 15°C in the morning when she leaves.
A. 28.13°C B. 38.13°C C. 48.13°C D. 58.13°C
Solution:

( )
( )
( )
( ) ( )( )

182. A piston-cylinder device whose piston is resting on top of a set of stops initially contains
0.50 kg of helium gas at 100 kPa and 25°C. The mass of the piston is such that 500 kPa of
pressure is required to raise it. How much heat must be transferred to the helium before the
piston starts rising?
A. 1557.13 kJ B. 1657.13 kJ C. 1757.13 kJ D. 1857.13 kJ
Solution:

-
( )( )

( ) ( )( )

183. In order to cool 1 ton (1000 kg) of water at 20°C in an insulated tank, a person pours 80 kg
of ice at -5°C into the water. Determine the final equilibrium temperature in the tank. The
melting temperature and the heat of fusion of ice at atmospheric pressure are 0°C and 333.7
kJ/kg, respectively.
A. 12.43°C B. 14.43°C C. 16.43°C D. 18.43°C
Solution:

54
( )( ) ( )( ) ( ) ( )( )

184. A fan is powered by 0.5 hp motor and delivers air at a rate of 85 m3/min. Determine the
highest value for the average velocity of air mobilized by the fan. Take the density of air to be
1.18 kg/m3.
A. 18.23 m/s B. 21.12 m/s C. 25.34 m/s D. 32.23 m/s
Solution:

( ) ( )

√ ( )( )

185. An Ocean-Thermal Energy Conversion power plant generates 10,000 kW using a warm
surface water inlet temperature of 26°C and cold deep-water temperature of 15°C. On the basis
of a 3°C drop in the temperature of the warm water and a 3°C rise in the temperature of the
cold water due to removal and addition of heat, calculate the power required in kW to pump
the cold-deep water to the surface and through the system heat exchanger if the required
pumping pressure increase is 12 kPa. Assume a Carnot cycle efficiency and density of cold water
to be 100 kg/m3.
A. 108 B. 250 C. 146 D. 160
Solution:

̇
̇

̇
̇
̇ ̇ ̇
̇ ̇
̇( )( )
̇

( )( )

186. A plate-type solar energy collector with an absorbing surface covered by a glass plate is to
receive an incident radiation of 800 W/m2. The glass plate has a reflectivity of 0.12 and a
55
transmissivity of 0.85. The absorbing surface has an absorptivity of 0.90. The area of the
collector is 5 m2. How much solar energy in watts is absorbed by the collector?
A. 2500 B. 2880 C. 3510 D. 3060
Solution:
̇ ( )( )( )

187. A tank contains liquid nitrogen at -190°C is suspended in a vacuum shell by three stainless
steel rods of 0.80 cm in diameter and 3 meters long with a thermal conductivity of 16.3 W/m 2-
°C. If the ambient air outside the vacuum shell is 15°C, calculate the magnitude of the
conductive heat flow in watts along the support rods.
A. 0.168 B. 0.0587 C. 0.182 D. 0.176
Solution:
̇ ( )[ (- )]

188. An elastic sphere containing gas at 120 kPa has a diameter of 1.0 m. Heating the sphere
causes it to expand to a diameter of 1.3 m. During the process the pressure is proportional to
the sphere diameter. Calculate the work done by the gas in kJ.
A. 41.8 B. 50.6 C. 87.5 D. 35.4
Solution:

( )

( )

∫ ∫ ( ) ∫

189. An ideal gas with a molecular weight of 7.1 kg/kg mol is compressed from 600 kPa and 280
K to a final specific volume of 0.5 m3/kg. During the process the pressure varies according to P =
620 + 150υ + 95 υ2 where p is in kPa and υ in m3/kg. Calculate the work of compression in kJ/kg.
A. 32.8 B. 28.7 C. 35.6 D. 33.3
Solution:
( )

56
∫ ∫ ( )

190. A one cubic meter container contains a mixture of gases composed of 0.02 kg-mol of
oxygen and 0.04 kg-mol of helium at a pressure of 220 kPa. What is the temperature of this
ideal gas mixture in Kelvin?
A. 441 B. 350 C. 400 D. 450
Solution:
( ) ( )

191. Methyl alcohol (CH3OH) is burned with 25% excess air. How much unburned oxygen in kg-
moloxygen/kg-molfuel will there be in the products if the combustion is complete?
A. 0.35 B. 0.45 C. 0.37 D. 0.65
Solution:

( ) ( )

( ) ( )( )
( )( ) ( )
( )

192. A 12 DC electrical motor draws a current of 15 amps. How much work in kJ does this motor
produce over a 10-minute period of operation?
A. 108.0 B. 129.6 C. 216.0 D. 318.2
Solution:
( )( )

193. A 4 liter (2-liter per revolution at standard pressure and temperature) spark ignition engine
has a compression ratio of 8 and 2200 kJ/kg heat addition by the fluid combustion. Considering
a cold air-standard Otto cycle model, how much power will the engine produce when operating
at 2500 rpm?
A. 166.53 hp B. 73.12 hp C. 97.4 hp D. 148 hp
Solution:
( )
̇

57
( )
̇ ( )

194. A simple Rankine cycle produces 40 MW of power, 50 MW of process heated and rejects
50 MW of heat to the surroundings. What is the utilization factor of this cogeneration cycle
neglecting the pump work?
A. 50% B. 60% C. 64% D. 80%
Solution:
̇ ̇ ̇ ̇
̇ ̇
̇

195. The rate of heat transfer to the surroundings from a person at rest is 400 kJ/hr. Suppose
that the ventilation system fails in an auditorium containing 120 people and assuming that the
energy goes into the air of volume 1500 m3 initially at 300 K and 101 kPa, calculate the rate in
°C /min of air temperature change.
A. 0.81 B. 0.53 C. 0.63 D. 1.0
Solution:
̇ ̇
̇ ̇
( ) ̇( )( )
̇
( ) ( )

196. An insulated box containing helium gas falls from a balloon 4.5 km above the earth’s
surface. Calculate the temperature rise in °C of the helium when box hits the ground.
A. 15.2 B. 12.6 C. 25.3 D. 14.1
Solution:
-

( )( ) ( )

197. Consider two Carnot heat engines operating in series. The first engine receives heat from
the reservoir at 2400 K and rejects the waste heat to another reservoir at temperature T. The
second engine receives heat by the first one, convert some of it to work, and rejects the rest to
58
a reservoir at 300 K. If thermal efficiencies of both engines are the same, determine the
temperature T.
A. 849 K B. 578 K C. 763 K D. 978 K
Solution:

198. An ideal gas mixture consists of 2 kmol of N2 and 6 kmol of CO 2. The mass fraction of CO 2
is?
A. 0.175 B. 0.250 C. 0.825 D. 0.750
Solution:
( )
( ) ( )

199. An ideal gas mixture consists of 2 kmol of N2 and 6 kmol of CO 2. The apparent gas constant
of mixture is?
A. 0.208 B. 0.231 C. 0.531 D. 0.825
Solution:
( ) ( )
-

200. A Carnot cycle operates between the temperature limits of 300 K and 1500 K, and
produces 600 kW of net power. The rate of entropy change of the working fluid during heat
addition process is?
A. 0 B. 0.4 C. 0.5 D. 2.0
Solution:
̇ ( )
( )

201. Air in an ideal Diesel cycle is compressed from 3 L to 0.15 L and then it expands during the
constant pressure heat addition process to 0.3 L. Under cold air standard conditions, the
thermal effieciency of this cycle is?
A. 35% B. 44% C. 65% D. 70%
Solution:

59
[ ( )
] [ ( )
]

202. Helium gas in an ideal Otto cycle is compressed from 20°C and 2 L to 0.25 L and its
temperature increases by an additional 800°C during the heat addition process. The
temperature of Helium before the expansion process is?
A. 1700°C B. 1440°C C. 1240°C D. 880°C
Solution:

( )
( )( )

203. In an ideal Otto cycle, air is compressed from 1.20 kg/m3 and 2.2 L to 0.26 L and the net
work output of the cycle is 440 kJ/kg. The mean effective pressure for the cycle is?
A. 612 kPa B. 599 kPa C. 528 kPa D. 416 kPa
Solution:
- - -

-
( )

204. An ideal Brayton cycle has a net work output of 150 kJ/kg and backwork ratio of 0.4. if
both the turbine and the compressor had an isentropic efficiency of 80%, the net work output
of the cycle would be?
A. 60 kJ/kg B. 75 kJ/kg C. 98 kJ/kg D. 120 kJ/kg
Solution:

60
( )
( )

205. Air enters a turbojet engine at 200 m/s at a rate of 20 kg/s and exits at 800 m/s relative to
the aircraft. The thrust developed by the engine is?
A. 6 kN B. 12 kN C. 16 kN D. 20 kN
Solution:
( ) ( )

206. A thermal power has a net power of 10 MW. The backwork ratio of the plant is 0.005.
Determine the compressor work.
A. 50.15 kW B. 50.35 kW C. 50.25 kW D. 50.45 kW
Solution:
̇ ̇ ̇
̇
̇
̇
̇
̇ ̇
̇ ̇ ̇
̇ ̇
̇
̇ ( )

207. A heat engine receives heat from a source at 1200 K at a rate of 500 kJ/s and rejects the
wasted heat to a sink at 300 K. If the power output of the engine is 200 kW, the second law
efficiency of this heat engine is?
A. 35% B. 40% C. 53% D. 75%
Solution:

61
208. A water reservoir contains 100,000 kg of water at an average elevation of 60 m. The
maximum amount of electric power that can be generated from this water is ?
A. 8 kWh B. 16 kWh C. 1630 kWh D. 58,800 kWh
Solution:
( )

209. A house is maintained at 22°C in winter by electric resistance heaters. If the outdoor
temperature is 5°C, the second law efficiency of the resistance heaters is ?
A. 0% B. 5.8% C. 34% D. 77%
Solution:

210. A thermoelectric refrigerator that resembles a small ice chest is powered by a car battery,
and has a COP of 0.10. If the refrigerator cools a 0.350 L canned drink from 20°C to 4°C in 30
min, determine the average electric power consumed by the thermoelectric refrigerator.
A. 130 watts B. 110 watts C. 120 watts D. 140 watts
Solution:
̇ ̇ ( ) ( )( )

211. A Carnot refrigerator operates in a room in which the temperature is 25°C and consumes 2
kW of power when operating. If the food compartment of the refrigerator is to be maintained
at 3°C, determine the rate of heat removal from the food compartment.
A. 1504.8 kJ/min B. 12.86 kJ/min C. 1625 kJ/min D. 9.57 kJ/min
Solution:

( ) ( )
̇ ̇ ( )

212. A household refrigerator with EER 8.0 removes heat from the refrigerated space at a rate
of 90 kJ/min. Determine the rate of heat transfer to the kitchen air.
A. 101.25 kJ/min B. 63.05 kJ/min C. 128.46 kJ/min D. 80 kJ/min
Solution:

62
̇
̇ ̇
̇

213. An air-conditioning system is used to maintain a house at 75°F when the temperature
outside is 95°F. The house is gaining the heat through the walls and windows at a rate of 1250
Btu/min, and the heat generation rate within the house from people, lights, and appliances
amounts to 350 Btu/min. Determine the minimum power input required for this air-
conditioning system.
A. 10.06 hp B. 1.36 hp C. 1.41 hp D. 7.94 hp
Solution:
̇

( ) ( )

̇
̇

214. A refrigeration system is to cool bread loaves with an average mass of 450 g from 22°C to -
10°C at a rate of 500 loaves per hour by refrigerated air. Taking the average specific and latent
heats of bread to be 2.93 kJ/kg-°C and 109.3 kJ/kg, respectively, determine the product load.
A. 541.7 kJ/min B. 351.6 kJ/min C. 761.5 kJ/min D. 409.9 kJ/min
Solution:
̇ ( )
̇ ̇ ̇ ( ̇ ) ( ̇ )
( )[ (- )] ( )

215. A house that was heated by electric resistance heaters consumed 1200 kWh of electric
energy in a winter month. If this house were heated instead by a heat pump that has an
average performance factor, PF of 2.4, determine how much money the homeowner would be
saved that month. Assume a price of 0.085 $/kWh for electricity.
A. $42.5 B. $59.50 C. $102 D. $97.75
Solution:
̇
̇
( )( )

63
216. An ammonia simple saturation cycle operates with a suction pressure of 291.6 kPa and a
condenser pressure of 1204 kPa develops 15 tons of refrigeration. Determine the theoretical
horsepower of the compressor. The following enthalpies have been found: condenser entrance
= 1653 kJ/kg, exit = 346.6 kJ/kg; compressor entrance = 1450.2 kJ/kg, exit = 1653 kJ/kg.
A. 7.23 hp B. 13 hp C. 15 hp D. 8.23 hp
Solution:
̇
̇
( )
̇ ̇( )

217. An ammonia ice plant operates between a condenser temperature of 35°C and evaporator
of -15°C. It produces 10 metric tons of ice per day from water at 30°C to ice at -5°C. Assuming
simple saturation cycle, determine the horsepower of the motor if the adiabatic efficiency of
the compressor nc = 0.85 and mechanical efficiency nm = 0.95. The specific heat of ice is 2.094
kJ/kg-°C and the latent heat is 335 kJ/kg. From the table for ammonia the following enthalpies
are: condenser entrance = 1703 kJ/kg, exit = 366.1 kJ/kg; compressor entrance = 1443.9 kJ/kg,
exit = 1703 kJ/kg.
A. 17.68 hp B. 18.61 hp C. 15.5 hp D. 21.9 hp
Solution:
( ) ( )
( ) [ (- )]
( )
̇
̇
̇
( )
̇ ̇( )
̇
( )

218. A Freon 22 air conditioning under standard operating conditions of 35°C condensing and
5°C evaporating temperatures. The volume flow rate entering the compressor is 23.72 L/s.
Determine the refrigerating capacityif the refrigerating effect is 164 kJ/kg. From the table for
R22 the specific volume at the compressor entrance is 40.46 L/kg.
A. 339.3 TR B. 79.3 TR C. 96.4 TR D. 27.4 TR
Solution:

( )

64
219. The refrigerant volume flow rate at the entrance of compressor were obtained from a test
on a twin cylinder, single acting 15 cm x 20 cm, 320 rpm compressor ammonia refrigerating
plant to be 33 L/s. Determine the volumetric efficiency of the compressor.
A. 77.65% B. 87.6% C. 97.6% D. 65.65%
Solution:
( ) ( ) ( )( )( )

( )

220. A twin cylinder ammonia compressor with volume displacement of 14,726 cm3 operates at
300 rpm. Condenser and evaporator pressure are 1200 kPa and 227 kPa respectively. Specific
volume of refrigerant at the entrance of compressor is 528.26 L/kg. Compression process is
polytropic with n = 1.20 and clearance factor of compressor is 2 percent. Determine the
horsepower required.
A. 60 hp B. 70 hp C. 80 hp D. 90 hp
Solution:

( ) ( )

( ) ( )( )
( )
( )( )
̇ [( ) ] [( ) ]

( )

221. A reversed Carnot has a refrigerating COP of 2.5. Determine the ratio TH/TL.
A. 1.4 B. 1.5 C. 1.25 D. 1.2
Solution:

222. Three thousand cubic feet per minute of air are circulated over an air-cooled condenser. If
the load on the condenser is 64,800 Btu/hr, compute the temperature rise of the air passing
over the condenser. Specific volume of standard air (13.34 ft3/lb).
65
A. 10°F B. 15°F C. 20°F D. 25°F
Solution:

̇
( ) ( )
̇

223. Saturated vapor ammonia at -16°C (hT = 1442.60 kJ/kg) leaves the evaporator and enters
the compressor at -6°C (h1 = 1465 kJ/kg). The refrigerant leaves the condenser as saturated
liquid at 40°C (h4 = 390.6 kJ/kg) and enters the expansion valve at 35°C (h5 = 366.1 kJ/kg). Heat
rejected from the condenser amount to 50 kW. The work to compressor is 208 kJ/kg, while the
heat loss from compressor is 33 kJ/kg. If 95 kJ/kg of heat are lost in the piping between the
compressor discharge and condenser inlet, determine the refrigerating capacity of the system.
A. 49.5 TR B. 46.61 TR C. 12.88 TR D. 13.24 TR
Solution:

̇ ̇ ( )

224. In an actual refrigeration cycle using R12 as working fluid, the refrigerant flow rate is 0.05
kg/s. Vapor enters the expansion valve at 1.15 MPa, 40°C (h = 238.5 kJ/kg) and leaves the
evaporator at 175 kPa, -15°C (h = 345 kJ/kg). The electric iput to motor driving the compressor
is measured and found 3.0 kW. Motor efficiency at this load 92% and mechanical efficiency
82%. Determine the actual coefficient of performance for this cycle.
A. 1.58 B. 2.36 C. 1.78 D. 1.34
66
Solution:
̇ ( )
̇ ( )( )

225. In an ammonia refrigeration system, the temperature in the evaporator is -12°C and the
ammonia at the evaporator entry 0.1511 dry while at exit is 0.95 dry. If the rate of ammonia
circulation is 5.64 kg/min, determine the refrigerating capacity of the system. Enthalpy of
saturated liquid and vapor at -12°C is 144.929 kJ/kg and 1447.74 kJ/kg respectively.
A. 17.82 B. 34.82 C. 27.82 D. 4.82
Solution:
( ) ( )

( ) ( )

( )
̇ ̇( )

226. A two stage cascade vapor compression refrigeration system uses ammonia in the low-
temperature loop and R-12 in the high-temperature loop. The ammonia provides 15 tons of
cooling. If the high-loop temperature requires 10.12 kW compressor power and low loop 15.93
kW, determine the COP of the system.
A. 2.027 TR B. 5.22 TR C. 3.314 TR D. 9.1 TR
Solution:
̇
( )

227. When a man returns to his well-sealed house on a summer day, he finds that the house is
at 32°C. He turns on the air conditioner, which cools the entire house to 20°C in 15 min. If COP
of the air conditioner system is 2.5, determine the power drawn by the air conditioners.
Assume the entire mass within the house is equivalent to 800 kg of air for which c = 0.72 kJ/kg -
°C.
A. 7.08 kW B. 3.07 kW C. 19.2 kW D. 12.08 kW
Solution:
( ) ( )( )
̇
( )
̇
̇

67
228. It is desired to double the COP of a reversed Carnot engine for cooling from 5.0 by raising
the temperature of heat addition while keeping the temperature of heat rejection constant. By
what percentage must the temperature of heat addition be raised?
A. 10.1% B. 9.1% C. 8.1% D. 7.1%
Solution:

229. An ammonia water-cooled compressor receives the refrigerant at specific volume 62 L/kg.
It has a piston displacement rate of 5 m3/min. If a squirrel cage motor running at 1200 rpm
drives the compressor and average piston speed is 490 m/min, calculate size of cylinder bore.
A. 20.4 cm B. 26.0 cm C. 16.13 cm D. 13.6 cm
Solution:

( )

( )( )

230. If the initial volume of an ideal gas is compressed the one-half its original volume and to
twice its temperature, the pressure:
A. Doubles B. Quadruples C. Remains constant D. Halves
Solution:

68
( )

231. If the gage pressure of a medium is 30 kPa (vacuum) and the atmospheric pressure is 101.3
kPa, the absolute pressure will be?
A. 131.3 kPa B. -71.3 kPa C. 71.3 kPa D. -131.3 kPa
Solution:

232. If a particle has a velocity of 4 meters per second and a kinetic energy of 144 Joules, then
the mass, in kilograms of this particle must be?
A. 44 B. 16 C. 18 D. 24
Solution:

( )

233. A condenser vacuum gauge reads 715 mm Hg when the barometer stands at 757 mm Hg.
State the absolute pressure in the condenser in kN/m2 or kPa.
A. 5.6 kPa B. 5.9 kPa C. 6.5 kPa D. 5.2 kPa
Solution:
( )

234. Determine the force in newton in a piston of 465 mm2 area with a pressure of 0.172 MPa.
A. 65 N B. 72 N C. 80 N D. 111 N
Solution:
( )( )( )

235. One piston of a hydraulic press has an area of 1 cm2. The other piston has an area of 25
cm2. If a force of 150 N is applied on the smaller piston, what will be the total force on the
larger piston if both piston surfaces are on the same level?
A. 6 N B. 175 N C. 3750 N D. 4250 N
Solution:

69
236. The work done on air is 10.86 kJ/kg, determine the compressor power if it is receiving 272
kg/min of air.
A. 36.72 hp B. 49.23 hp C. 2954 hp D. 66 hp
Solution:
( ) ( )

237. A water tank 18 ft wide, 14 ft long, and 4 ft high, calculate the pressure at the bottom of
the tank.
A. 1.733 psi B. 1.999 psi C. 2.337 psi D. 3.773 psi
Solution:
( )( )

238. What is the pressure of 750 mm Hg in kN/m2?


A. 90 B. 100 C. 103 D. 110
Solution:
( )

239. A double purpose tank 18 ft wide, 24 ft long and 4 ft depth is filled with water. What is the
weight of water in the tank in long tons?
A. 49 tons B. 48 tons C. 54 tons D. 50 tons
Solution:
[ ( )( )] ( )

240. Oil flows through a 16 tube single cooler with a velocity of 2 m/s. The internal diameter of
the tube is 30 mm and oil density is 0.85 g m/ml. Find the volume flow in liters per sec.
A. 22.62 B. 32.22 C. 62.22 D. 42.62
Solution:
( ) ( )( )

241. A substance temperature was 620°R. What is the temperature in °C?


A. 50.7 B. 45.54 C. 71.11 D. 91.44
Solution:
70
[( ) ]( )

242. An unknown volume of container gas of 1 atmosphere is allowed to expand to another


container of 10 m3 volume at 500 mmHg at constant temperature. Find the unknown volume.
A. 6.58 m3 B. 6.75 m3 C. 5.67 m3 D. 7.65 m3
Solution:

( )

243. An iron block weighs 5 Newton and has a volume of 200 cm3. What is the density of the
block?
A. 2458 kg/m3 B. 2485 kg/m3 C. 2584 kg/m3 D. 2549 kg/m3
Solution:
( )
( )( )

244. If air is at a pressure of 22.22 psia and at a temperature of 800°R, what is the specific
volume?
A. 11.3 ft3/lbm B. 33.1 ft3/lbm C. 13.3 ft3/lbm D. 31.3 ft3/lbm
Solution:

( )
( )

245. The Specific gravity of mercury is 13.55. What is the specific weight of mercury?
A. 123.9 kN/m3 B. 139.2 kN/m3 C. 132.9 kN/m3 D. 193.2 kN/m3
Solution:
( )

246. The equivalent weight of mass 10 kg at a location where the acceleration of gravity is 9.77
m/s 2?
A. 97.7 N B. 79.7 N C. 77.9 N D. 977 N
Solution:
( )

71
247. A transportation company specializes in the shipment of pressurized gaseous materials. An
order is received from 100 liters of a particular gas at STP (32°F and 1 atm). What minimum
volume tank is necessary to transport the gas at 80°F and maximum pressure of 8 atm?
A. 16 liters B. 14 liters C. 10 liters D. 12 liters
Solution:

( ) ( )
( ) ( )

248. 100 g of water are mixed with 150 g of alcohol (density = 790 kg/m3). What is the specific
volume of the resulting mixtures, assuming that the fluids mixed completely?
A. 0.82 x 10-3 m3/kg B. 0.88 x 10-3 m3/kg C. 0.63 x 10-3 m3/kg D. 1.16 x 10-3 m3/kg
Solution:

249. How much does 30 lbm weigh on the moon? (g moon = 5.47 ft/s 2).
A. 2.0 lbf B. 3.2 lbf C. 3.4 lbf D. 5.096 lbf
Solution:
( )

250. A 10 kg block is raised vertically 3 meters. What is the change in potential energy?
A. 320 J B. 350 km-m2/s 2 C. 294 J D. 350 N-m
Solution:
( )( )

251. How many cubic meters is 100 gallons of liquid?


A. 3.7850 m3 B. 0.1638 m3 C. 0.3785 m3 D. 1.638 m3
Solution:
( )( )

252. Steam turbine is receiving 1014 lbm/hr of steam, determine the horsepower output of the
turbine if the work done by steam is 251 Btu/lbm.

72
A. 100 hp B. 462.7 hp C. 200 hp D. 600 hp
Solution:

̇ ( )( )

253. What is the resulting pressure when one pound of air at 15 psia and 200°F is heated at
constant volume to 800°F?
A. 32.1 psia B. 15 psia C. 28.6 psia D. 36.4 psia
Solution:

( )( )

254. A bicycle tire has a volume of 600 cm3. It is inflated with carbon dioxide to pressure of
551.43 kPa, 20°C. How many grams of CO 2 are contained in the tire? R CO2 = 0.18896 kJ/kg.K.
A. 5.98 g B. 6.43 g C. 4.63 g D. 3.83 g
Solution:
( )

( )

255. The absolute pressure at the bottom of a vertical column of water is 15.5 psia. What is the
height of this column?
A. 22 in B. 9.2 in C. 12 in D. 9.8 in
Solution:
( )( )

256. A water temperature rise of 18°F in the water cooled condenser is equivalent to what in
°C?
A. 7.78°C B. 10°C C. 263.56 K D. -9.44°C
Solution:
( )

257. An oil storage tank contains oil with specific gravity of 0.88 and depth of 20 meters. What
is the hydrostatic pressure at the bottom of the tank in kg/cm2?
A. 1.67 B. 1.76 C. 1.56 D. 1.87
Solution:

73
( )( ) ( )( )( )

258. A vertical column of water will be supported to what height by standard atmospheric
pressure?
A. 34 ft B. 36 ft C. 24 ft D. 26 ft
Solution:
( )

259. The specific weight of liquid is 60 lb/ft3. What is the equivalent to kN/m3?
A. 9.334 B. 9.249 C. 9.643 D. 9.420
Solution:
( )
( )( )[ ( )
]

260. A cylinder weighs 150 lbf. Its cross-sectional area is 40 square inches. When the cylinder
stands vertically on one end, what pressure does the cylinder exert on the floor?
A. 14.1 kPa B. 58.2 kPa C. 0.258 bar D. 0.141 bar
Solution:
( )( )

261. What is the absolute pressure exerted on the surface of a submarine cruising 300 ft below
the free surface of the sea? Assume specific gravity of sea water is 1.03.
A. 133.9 psia B. 148.6 psia C. 100.7 psia D. 103.7 psia
Solution:
( )( )

262. Air enters a nozzle steadily at 2.21 kg/m3 and 30 m/s. What is the mass flow rate through
the nozzle if the inlet area of the nozzle is 80 cm2?
A. 0.35 kg/s B. 3.5 kg/s C. 5.3 kg/s D. 0.53 kg/s
Solution:
( )( )( )

263. What is the work required to accelerate an 800-kg car from rest to 100 km/h on a level
road?
A. 308.6 kJ B. 806.3 kJ C. 608.3 kJ D. 386 kJ

74
Solution:
( )[ ( )( ] [ ( )]
)

264. Assuming that there are no heat effects and no friction effects, find the speed of a 3220-
lbm body after it falls 778 ft from rest.
A. 422 ft/s B. 424 ft/s C. 224 ft/s D. 424 ft/s
Solution:
[ ( )( )]

265. What is the flow rate through a pipe 4 inches in diameter carrying water at a velocity of 11
ft/s?
A. 430.84 gpm B. 7.18 gpm C. 340.28 gpm D. 39.16 gpm
Solution:
( ) ( ) ( )( )

266. If the specific weight of a liquid is 58.5 lbf per cubic foot, what is the specific volume of the
liquid in cm3/g?
A. 0.5321 cm3/g B. 0.6748 cm3/g C. 0.9504 cm3/g D. 1.0675 cm3/g
Solution:

( )
( ) ( ) [( ) ]

267. What is the resulting pressure when one pound of air at 0.3 psig and 200°F is heated at
constant volume to 800°F?
A. 0.572 psig B. 28.6 psia C. 7.857 psia D. 1.2 psig
Solution:

268. A small pump serving as model, when tested in laboratory using water at 3600 rpm,
delivered 30 cfs at a head of 125 ft. If the efficiency of this model pump is 84%, predict the
75
horsepower input to the prototype pump if it is to develop the same head as model pump and
the model pump has a scale ratio of 1:10. Assume the efficiency of the prototype pump is 90%.
A. 50.6 hp B. 4730 hp C. 3740 hp D. 60.5 hp
Solution:
( )( )
( )

( ) ( ) ( ) ( )

( ) ( )( )

( )( )
( )

269. Pump at is best efficiency point (BEP) has a capacity of 10,500 gpm while developing a
head of 60 ft at a rotative speed of 1450 rpm. What is the specific speed of the pump?
A. 2760 B. 1450 C. 2476 D. 6892
Solution:
( )( ) ( )
( )

270. A pump will be installed below the reservoir water surface with a required net positive
suction head (NPSHR) of 50 ft. The barometric pressure is 14.3 psia, and the vapor pressure is
0.5 psia. Assume friction losses in the intake piping are 5 ft. Find the maximum allowable
elevation of the pump relative to the water surface intake to avoid cavitation.
A. 45 ft B. 55 ft C. 18.2 ft D. 23.2 ft
Solution:
( )

( )

( )
( )
( )
( ) ( )
( ) ( ) ( )

271. A centrifugal pump is at best efficiency point (BEP). Assume the pump characteristic are
head, h = 7 m, flow rate, Q = 19 liters/s, and rotative speed n = 1170 rpm. Find the specific
speed in SI units.
76
A. 0.4 B. 0.71 C. 10.41 D. 3.94
Solution:
( )( )

( ) ( )
( )

( )
( ) ( )

272. The pressure of a confined gas at a constant temperature is tripled, what will happen to
the volume?
A. The volume will be tripled
B. The volume will be reduced to one-third of its original value
C. The volume will remain unchanged
D. The volume is constant
Solution:

273. A 15 in. diameter fan operates at 1600 rpm and develops a head of 6 in. of water and
delivers 120 cfm. What volumetric capacity for geometrically similar fan will develop 6 in of
water at 1300 rpm?
A. 147.7 cfm B. 181.8 cfm C. 97.5 cfm D. 79.2cfm
Solution:

( )

( ) ( )

( )

274. A radial-flow pump operating at maximum efficiency at a specific speed of 2500 is to


deliver 260 gpm against a head of 129 ft at a rotative speed of 2100 rpm. Find the required
number of stages (i.e., impellers).
A. 2 stages B. 3 stages C. 4 stages D. 5 stages
77
Solution:
( )

275. How many identical turbines, operating at 139.0 rpm and 91% efficiency (specific speed =
5.4), are needed to exploit a head of 1200 ft and a flow of 1660 ft 3/s?
A. 2 turbines B. 3 turbines C. 4 turbines D. 5 turbines
Solution:
( )

( )( )( )

( )

276. How many poles should a 60-Hz generator have, if it is connected to a turbine operating
under a design head of 3000 ft with a flow of 82 cfs? Assume turbine specific speed and
efficiency 3 and 84 percent respectively.
A.10-pole B. 12-pole C. 14-pole D. 16-pole
Solution:
( )

( )( )( )

( )

( )
-

277. It is proposed to build a dam in a river where the flow rate is 10 m3/s and a 32-m drop in
elevation can be achieved for flow through a turbine. If a turbine is 82 percent efficient, what is
the maximum power that can be achieved? Specific gravity of river is 0.998.
A. 2570 kW B. 3133 kW C. 3820 kW D. 262 kW
Solution:
( )[( )( )]( )

278. What type of turbine delivers 25,000 bhp at 500 rpm under a net head of 5350 ft?
A. Impulse turbine B. Francis turbine C. Kaplan turbine D. Propeller turbine

78
Solution:
( ) ( )

( )

279. A 26-hp pump delivers 475 gpm of gasoline (γ = 42.5 lb/ ft3) at 20 C with 78% efficiency.
What pressure rise result across the pump?
A. 30.2 psi B. 32.7 psi C. 120.3 psi D. 73.2 psi
Solution:
( )
( )

280. A model pump delivering water at 180°F (γ = 60.6 lb/ ft3; pvapor = 7.54 psia) at 900 gpm and
2500 rpm begins to cavitate when the inlet pressure and velocity are 13 psia and 22 fps. Find
the required NPSH of a prototype which is 4 times larger and runs at 1100 rpm.
A. 63.5 ft B. 20.49 ft C. 6.61 ft D. 36 ft
Solution:

[ ( ) ( )]
( )

( ) ( ) ( ) ( )

281. The diameter of the discharge pipe is 8 in. and that of the intake pipe is 10 in. The pressure
gage at discharge reads 32 psi, and vacuum gage at the intake reads 12 in. Hg. If the discharge
flow rate = 4.0 ft3/s of water and the brake horsepower is 49.0, find the efficiency. The intake
and the discharge are at the same elevation.
A. 82.2% B. 80.9% C. 55.8% D. 58.46%
Solution:

( ) ( )
( ) ( )

( ) ( ) (- )( )( )
( ) ( ) ( )

( )( )
( )

79
282. A piston positive-displacement pump (PDP) has a 6-in diameter and a 2.5-in stroke. Its
crankshaft rotates at 300 rpm. Calculate its output at 94 percent volumetric efficiency.
A. 12.27 cfm B. 13.5 cfm C. 10 cfm D. 11.53 cfm
Solution:
( )
( )( )( )

283. A centrifugal pump (efficiency 88%) lifts water through a total height of 40 m from a
reservoir to discharge. Pumping is through 300 m of 75 mm diameter pipe at the rate of 20
liter/s. If pipe friction, f = 0.025, what horsepower is required?
A. 28.4 kW B. 32.2 kW C. 25kW D. 9kW
Solution:
( )
( )

( )[ ( )
]
( )( )

284. In order to predict the behaviour of a small oil pump, tests are to be made on a model
using air. The pump is to be driven by a 1/20-hp motor at 1800 rpm and a ¼-hp motor is
available to drive the air at 600 rpm. Using specific gravity of oil at 0.912 and density of air
constant at 0.076 lb/ ft3, what size model should be built?
A. The model should be 2 times as large as the oil pump.
B. The model should be 5 times as large as the oil pump.
C. The model should be 8 times as large as the oil pump.
D. The model should be 10 times as large as the oil pump.
Solution:
( ) ( )

( )( )( ) ( )( )

285. A double-overhung impulse turbine installation is to develop 20,000 hp at 275 rpm under a
net head of 1100 ft. Determine the specific speed.
A. 4.34 B. 6.14 C. 203.61 D. 144
Solution:
80
( )

286. An impulse wheel at best produces 125 hp under a head of 210 ft. By what percent should
the speed be increased for a 290-ft head?
A. 82.25% B. 17.5% C. 72.41% D. 27.59%
Solution:
( )

( ) ( )

287. What is the power ratio of a pump and its 1/5 scale model if the ratio of heads is 4 to 1?
A. 20 B. 200 C. 12.5 D. 125
Solution:

( ) ( ) ( ) ( )

( ) ( ) ( ) ( )

288. The speed of a centrifugal pump is doubled. By what factor does the pump head change?
A. 0.125 B. 0.25 C. 4 D. 8
Solution:
( )

( ) ( )

289. Compute the specific volume of an air-vapor mixture in cubic meter per kg of dry air when
the following conditions prevail: t = 40°C, w = 0.015 kg/kg, and Pt = 90 kPa.
A. 0.99 m3/kg B. 0.89 m3/kg C. 0.79 m3/kg D. 0.69 m3/kg
Solution:

81
( )

290. A coil has an inlet temperature of 70°F and outlet of 80°F. If the mean temperature of the
coil is 130°F, find the bypass factor of the coil.
A. 0.28 B. 1.2 C. 0.82 D. 0.83
Solution:

291. Compute the pressure drop of 35°C air flowing with a mean velocity of 5m/s in a circular
sheet-metal duct 400mm in diameter and 25 m long. Use friction factor, f = 0.04, and ρair =
1.3799 kg/m3.
A. 431.22 Pa B. 221.34 Pa C. 312.24 Pa D. 422.31 Pa
Solution:
( )( ) ( )
( )

292. Pressure difference of 400 Pa is available to force 20°C air through a circular sheet-metal
duct 450 mm in diameter and 25 m long. At 20°C, ρ = 1.204 kg/m3 and take friction factor, f =
0.016. Determine the velocity.
A. 27.34 ft/s B. 43.72ft/s C. 89.68 ft/s D. 86.98 ft/s
Solution:

( ) ( )
( )

293. A rectangular duct has a dimension of 0.25 m by 2 m. Determine the equivalent diameter
of the duct.
A. 0.50 m B. 0.60 m C. 0.70 m D. 0.40 m
Solution:
( )( )

82
294. To what height will a barometer column rise if the atmospheric conditions are 13.9 psia
and 68°F and barometer fluid is mercury?
A. 3.56 ft B. 5.36 ft C. 2.36 ft D. 3.26 ft
Solution:

295. To what height will a barometer column rise if the atmospheric conditions are 13.9 psia
and 68°F and barometer fluid is ethyl alcohol? Note: @ 68°F; Pv =138.5 lbf/ft3 and specific
gravity of 0.79 for ethyl alcohol:
A. 79.37 in B. 37.79 in C. 353.54 in D. 453.53 in
Solution:
( )
( )

296. What is the pressure 7000 ft below the water surface of the ocean? Neglect
compressibility.
A. 512,000 psf B. 324,500 psf C. 447,000 psf D. 213,000 psf
Solution:
( ) ( )( ) ( )

297. If atmospheric air 14.7 psia and 60°F at sea level, what is the pressure at 14212 ft altitude
if air is incompressible? Note: @ 60°F, the density of air is 0.0763 lb m/ft3; P1 = 14.7 psia.
A. 5.4674 psia B. 7.5304 psia C. 7.1696 psia D. 7.1966 psia
Solution:

( )( )

298. Water (ρ = 62.4 lbm/ft3) is flowing through a pipe. A pitot-static gage registers 3.0 inches of
mercury. What is the velocity of the water in the pipe? Note: ρ Hg = 848.6 lbm/ft3.
A. 14.7 ft/s B. 41.7 ft/s C. 71.4 ft/s D. 74.1 ft/s
Solution:
( ) ( )( )( )
√ √

83
299. The mass of an outside air at 50°C in an air conditioning unit is 60 kg. Find the temperature
after mixing if the outside air mixed with 40 kg with recirculated air at 35°C.
A. 44°C B. 39°C C. 52°C D. 47°C
Solution:

( ) ( ) ( )

300. A creamery must cool 20,000 liters of milk received each day from an initial temperature
of 29°C to a final temperature of 2°C in 5 hours. If refrigeration losses amount to 10 percent of
the cooling load, what must be the capacity of their refrigerating machine? Note: specific heat
of milk is 3.9 kJ/kg-K and SG = 1.05.
A. 38.5 TOR B. 36.5 TOR C. 37.5 TOR D. 39.5 TOR
Solution:

̇ ̇
̇

̇ ( )
( )

̇ [ ( )( )]

301. How many tons of refrigeration is required to produce 10 metric tons of ice per day at -
10°C from raw water at 22°C if miscellaneous losses are 15% of the chilling and freezing load?
A. 17 TOR B. 20 TOR C. 15 TOR D. 24 TOR
Solution:

̇ ̇ ̇ ̇ ̇

̇ ̇
̇ ( )( )( )
( ) -

̇ ( )( )
( )

̇ ( )( )( )
( ) -

̇ ( )
̇

84
302. Five hundred kilograms of poultry enter a chiller at 8°C and are frozen and chilled to a final
temperature of 18°C for storage in 15 hours. The specific heat above and below freezing are
3.18 kJ/kg-°C and 1.55 kJ/kg-°C respectively. The latent heat is 246 kJ/kg and the freezing
temperature is -5°C. Compute the product load.
A. 2.75 kW B. 2.85 kW C. 2.95 kW D. 3.15 kW
Solution:
̇ ̇ ̇ ̇
̇ ( )( )( )
( ) -

̇ ( )( )
( )

̇ ( )( ) (- )
( ) -

303. Fish weighing 11,000 kg with a temperature of 20°C is brought to a cold storage and which
shall be cooled to -10°C in 11 hours. Find the required plant refrigerating capacity in tons of
refrigeration if the specific heat of fish is 0.7 kCal/kg-°C above freezing point and 0.30 kCal/kg-
°C below freezing point. The freezing point is -3°C. The latent heat of freezing is 55.5 kCal/kg.
A. 25.26 TOR B. 15.26 TOR C. 14.38 TOR D. 24.38 TOR
Solution:
̇ ̇ ̇ ̇
̇ ( )( )( )
( ) -

̇ ( )( )
( )

̇ ( )( ) (- )
( ) -

304. The power requirement of a Carnot refrigerator in maintaining a low temperature region
at 300 K is 1.5 kW per ton. Find the heat rejected.
A. 4.02 kW B. 7.02 kW C. 5.02 kW D. 6.02 kW
Solution:
̇

85
̇
̇

̇ ( )

305. A vapor compression refrigeration system is designed to have a capacity of 150 tons of
refrigeration. It produces chilled water from 22°C to 2°C. Its actual coefficient of performance is
5.86 and 35% of the power supplied to the compressor is lost in the form of friction and
cylinder cooling losses. Determine the condenser cooling water required for a temperature rise
of 10°C.
A. 14.75 kg/s B. 15.65 kg/s C. 18.65 kg/s D. 13.75 kg/s
Solution:

̇ ̇ ̇

( )

̇
̇ ̇ ( )( )

306. Determine the heat extracted from 2000 kg of water from 25°C to ice at -10°C.
A. 621,150 kJ B. 721,150 kJ C. 821,150 kJ D. 921,150 kJ
Solution:

( )( )
( )

86
( )( )

307. A single acting, twin cylinder, Ammonia compressor with bore equal to stroke is driven by
an engine at 250 rpm. The machine is installed in a chilling plant to produce 700 kW of
refrigeration at -18°C evaporating temperature. At this temperature the cooling effect per kg
mass is 1160 kJ. The specific volume of vapor entering the compressor is 0.592 m3 per kilogram.
Assume 85% volumetric efficiency, determine the bore in mm.
A. 400 mm B. 300 mm C. 450 mm D. 500 mm
Solution:
( )( )

( )

( ) ( )

308. An iron block weighs 7 Newtons and has a volume of 200 cubic centimeters. What is the
density the block?
A. 3465 kg/m3 B. 3565 kg/m3 C. 1255 kg/m3 D. 2550 kg/m3
Solution:
- ( )
( )[ ( )
]

309. If the density of the gas is 0.003 slugs per cubic foot, what is the specific weight of the gas?
A. 9.04 N/m3 B. 15.2 N/m3 C. 76.3 N/m3 D. 98.2 N/m3
Solution:
-
( )
( ) ( )( ) [( )
]

87
310. The specific gravity of mercury relative to water is 13.55. What is the specific weight of
mercury? (The specific weight of water is 62.4 lbf per cubic foot.)
A. 82.2 kN/m3 B. 102.3 kN/m3 C. 132.9 kN/m3 D. 150.9 kN/m3
Solution:
( )
( )( )[( )
]

311. If the specific weight of a liquid is 58.5 lbf per cubic foot, what is the specific volume of the
liquid in cm3/g?
A. 0.5321 cm3/g B. 0.6748 cm3/g C. 0.9504 cm3/g D. 1.0675 cm3/g
Solution:

( ) ( )
[ ( )
][ ( )
]( )( )

312. Which of the following is not a unit of pressure?


A. Pa B. bars C. kg/m-s 2 D. kg/m2
Solution:
-

313. A cylinder weighs 150 lbf. Its cross-sectional area is 40 square inches. When the cylinder
stands vertically on one end, what pressure does the cylinder exert on the floor?
A. 14.1 kPa B. 25.8 kPa C. 63.2 kPa D. 89.7 kPa
Solution:
( )

314. What pressure is a column of water 100 centimeters high equivalent to?
A. 9810 dyne/cm2 B. 9810 N/m2 C. 0.1 bars D. 0.1 atm
Solution:
( )( )( )( )( )

88
315. Water is flowing in a pipe with a radius of 10” at a velocity of 5 m/s. At the temperature in
the pipe, the density and viscosity of the water are as follows:
ρ = 997.9 kg/m3 and μ = 1.131 Pa-s.
What is the Reynold's number for this situation?
A. 44.1 B. 88.2 C. 1140 D. 2241
Solution:
( ) ( )[ ( )]( ) ( )( )

-
- ( )( )

316. How long must a current of 5.0 amperes pass through a 10 ohm resistor until a charge of
1200 coulombs passes through?
A. 1 min B. 2 min C. 3 min D. 4 min
Solution:
-
( )

317. A car moving at 70 km/hr has a mass of 1700 kg. What force is necessary to decelerate it
at a rate of 40 cm/s 2?
A. 0.680 N B. 42.5 N C. 680 N D. 4250 N
Solution:
( )

318. One hundred milliliters of water in a plastic bag of negligible mass is to be catapulted
upwards with an initial acceleration of 20.0 m/s 2. What force is necessary to do this? Assume
gravity is 9.81 m/s 2 and the density of the water is 1 g/cm3.
A. 2.00 N B. 2.98 N C. 15.0 N D. 2.00 kN
Solution:
( )

[ ( ) ] [( ) ] ( )( )( )
-

319. A boy pulls a sled with a mass of 20 kg horizontally over a surface with a coefficient of
friction of 0.20. It takes him 10 minutes to pull the sled 100 yards. What is his average power
output over these 10 minutes?
A. 4 W B. 6 W C. 8 W D. 10 W
Solution:

89
[ ( )( )]( )
( )( )

320. A force of 200 lbf acts on a block at an angle of 28° with respect to horizontal. The block is
pushed 2 feet horizontally. What is the work done by this force?
A. 215 J B. 320 J C. 480 J D. 540 J
Solution:
( ) ( ) ( ) ( )( )

321. Two particles collide, stick together and continue their motion together. Each particle has
a mass of 10 g and their respective velocities before the collision were 10 m/s and 100 m/s.
What is the energy of the system after the collision?
A. 21.8 J B. 30.2 J C. 42.8 J D. 77.9 J
Solution:
( )
( ) ( ) [( ) ]

( ) [( ) ]( ) ( )

322. A copper bar is 90 centimeters long at 86°F. What is the increase in its length when the bar
is heated to 95°F? The linear expansion coefficient for copper, α, is 1.7 x 10-5/°C.
A. 2.12 x 10 -5 m B. 3.22 x 10 -5 m C. 5.25 x 10 -5 m D. 7.65 x 10 -5 m
Solution:
( )[( ) ] ( )( )
-

323. Calculate the energy transfer rate across a 6" wall of firebrick with a temperature
difference across the wall of 50°C. The thermal conductivity of firebrick is 0.65 BTU/hr-ft-°F at
the temperature of interest.
A. 112 W/m2 B. 285 W/m2 C. 369 W/m2 D. 429 W/m2
Solution:
( )
( )( )( )( ){ ( }
- -
)
[ ( )
]

90
324. A house has brick walls 15 millimeters thick. On a cold winter day, the temperature of the
inner and outer layers of the walls are measured and found to be 20°C and -12°C, respectively.
If there is 120 m2 of exterior wall of race, and the thermal conductivity of bricks is 0.711 J/m-s-
°C, how much heat is lost through the walls per hour?
A. 182 J B. 12.5 kJ C. 655 kJ D. 655 MJ
Solution:
( ){ [ (- )] }
( )( )
- -

325. If a horsepower pump runs for 20 minutes, what is the energy used?
A. 0.06 ergs B. 0.25 kW C. 0.30 MJ D. 0.11 kW-h
Solution:

( ) ( )( )( )

326. A power of 6 kW is supplied to the motor of a crane. The motor has an efficiency of 90%.
With what constant speed does the crane lift the 800 lbf weight?
A. 0.09 m/s B. 0.32 m/s C. 0.98 m/s D. 1.52 m/s
Solution:

( )
( ) ( )

327. An engine has an efficiency of 26%. It uses 2 gallons of gasoline per hour. Gasoline has a
heating value of 20,500 BTU/lbm and a specific gravity of 0.8. What is the power output of the
engine?
A. 0.33 kW B. 20.8 kW C. 26.0 kW D. 41.7 kW
Solution:

̇ ̇ ( )( ) ( )( )( )

( )( )
( )

91
328. Two liters of an ideal gas at a temperature of T 1 = 25°C and a pressure of P1 = 0.101 MPa,
are in a 10 cm diameter cylinder with a piston at one end. The piston is depressed so that the
cylinder is shortened by 10 centimeters. The temperature increases by 2°C. What is the change
in pressure?
A. 0.156 MPa B. 0.167 MPa C. 0.251 MPa D. 0.327 MPa
Solution:

( )( ) { [ ( ) ( )] }
( ) ( )

329. The average power output of a cylinder in a combustion engine is given by:
̅
where:
p = average pressure on the piston during the stroke
L = length of the piston stroke
A = area of the piston head
N = number of strokes per second
An 8-ylinder engine has the following specifications
p = 283 kPa
L = 14 cm
d = diameter of piston head = 12 cm
N = 1500 strokes/min
What is the average power output of this engine?
A. 89.5 N/s B. 89.5 kW C. 89.5 x 103 J-m/s D. 89.5 kJ
Solution:
̅
( )
( )( )[ ( ) ]( ) ( )
( )

330. What is the power required to transfer 97,000 coulombs of charge through a potential rise
of 50 volts in one hour?
A. 0.5 kW B. 0.9 kW C. 1.3 kW D. 2.8 kW
Solution:

92
( - )
( )

331. A current of 7 amperes passes through a 12 ohm resistor. What is the power dissipated in
the resistor?
A. 84 w B. 0.59 hp C. 0.79 hp D. 7 hp
Solution:
( )( )

332. What is the pressure at point A in the tank if h = 2 feet? (g = 32.2 ft/s 2 and ρ = 1.94 slug/ft3)
A. 75 lbf/ft2 B. 85 lbf/ft2 C. 100 lbf/ft2 D. 125 lbf/ft2
Solution:
-

( )( ) ( )

333. Determine the average velocity through a circular section in which the velocity distribution
is given by [ ( ) ]. The distribution is symmetric with respect to the longitudinal
axis, r = 0, ro is the outer radius, vmax is the velocity along the longitudinal axis. Assume flow is
laminar.
A. B. C. D.
Solution:

334. A pipe has a diameter 4” at section AA, and a diameter of 2” at section BB. For an ideal
fluid flow, the velocity given is 1 ft/s at section AA. What is the flow velocity at section BB?
A. 4 ft/s B. 0.5 ft/s C. 1.0 ft/s D. 2.0 ft/s
Solution:

( ) ( ) ( )

335. A mixing tank mixes two inlet streams containing salt. The salt concentration in stream 1 is
5% by weight, at stream 2 it is 15% by weight. Stream 1 flows at 25 kg/s, and stream 2 flows at
10 kg/s. There is only one exit stream. Find the salt concentration in the exit stream.

93
A. 5% B. 8% C. 11% D. 13%
Solution:

( ) ( ) ( )

336. Water is pumped at 1 m3/s to an elevation 5 meters through a flexible hose using a 100%
efficient pump rated at 100 kilowatts. Using the same length of hose, what size motor is needed
to pump 1 m3/s of water to the tank, with no elevation gain? In both cases , both ends of the
hose are at the same temperature and pressure. Neglect kinetic energy effects.
A. 51 kW B. 22 kW C. 36 kW D. 43 kW
Solution:

( )

( )( )

337. A fluid with kinetic viscosity of 2.5 x 10 -5 ft2/s is flowing at 0.1 ft/s from an orifice 3” in
diameter. How can the fluid be described?
A. The fluid is completely turbulent.
B. The fluid is in transition zone.
C. The fluid is laminar.
D. Turbulent cannot be calculated, it must be measured.
Solution:
( )
-
( )

338. The Reynold’s number of a sphere falling in air is 1x106. If the sphere’s radius is 1 ft, what
is its velocity? (ρ = 0.00234 slug/ft3, μair = 3.8x10-7 lbf-s/ft2).
94
A. 2.5 ft/s B. 5.1 ft/s C. 40.6 ft/s D. 81.2 ft/s
Solution:

[ ( ) ]( )

- -

339. The flow rate of water through a cast iron is 5000 gallons per minute. The diameter of the
pipe is 1 foot, and the coefficient of friction is f = 0.0173. What is the pressure drop over a 100
foot length of pipe?
A. 21.078 lbf/ft2 B. 23.78 lbf/ft2 C. 337.26 lbf/in2 D. 337.26 lbf/ft2
Solution:

( )
( )( ) [( ]( ) ( )
)

( )
( )[ ]
( )( )

340. A cylindrical flash tank mounted with its axis horizontal is used to separate liquid ammonia
from ammonia vapor. The ammonia bubbles through the liquid with 70 m3/min leaving the
disengaging surface. The disengaging rate is limited to 60 m/min and the liquid level is to
operate with the liquid level one-third to the diameter from the top. Determine the diameter if
the tank is 1.5 m long.
A. 830 mm B. 730 mm C. 860 mm D. 760 mm
Solution:

95
( ) ( )

(√ )

(√ )( )

341. A 150 Hp motor is used to drive the compressor. If the heat loss from the compressor is 25
kW and the mass flow rate of the refrigerant entering the compressor is 0.50 kg/s, determine
the difference of the enthalpies between the inlet and outlet of the compressor.
A. 143.80 kJ/kg B. 153.80 kJ/kg C. 173.80 kJ/kg D. 183.80 kJ/kg
Solution:

( )

( )

342. To cool farm products, 300 kg of ice at -4.4°C are placed in bunker. Twenty four hours later
the ice has melted into water at 7.2°C. What is the average rate of cooling provided by the ice
in kJ/hr?
A. 2679.28 kJ/hr B. 5679.28 kJ/hr C. 3679.28 kJ/hr D. 4679.28 kJ/hr
Solution:
̇ ̇ ̇ ̇
̇ [ ( ) ( )]
̇

343. Determine the estimated condenser load for an open-type compressor having a cooling
capacity of 16,500 Btu/hr and a heat rejection factor of 1.32.
A. 22,280 Btu/hr B. 20,780 Btu/hr C. 21,780 Btu/hr D. 19,780 Btu/hr
Solution:

( )
96
344. If the load on water-cooled condenser is 150,000 Btu/hr and the temperature rise of the
water in the condenser is 10°F, what is the quantity of water circulated in gpm?
A. 30 GPM B. 40 GPM C. 20 GPM D. 50 GPM
Solution:
̇ ̇
̇ ( )( )
̇

345. The load on a water-cooled condenser is 90,000 Btu/hr. if the quantity of water circulated
through the condenser is 15 gpm, determine the temperature rise of the water in the
condenser.
A. 12°F B. 14°F C. 16°F D. 18°F
Solution:
̇ ̇
̇ ( )( )
-

̇
( )( )

( )

346. The weight of ammonia circulated in a machine is found to be 21.8 lb/hr. if the vapor
enters the compressor with a specific volume of 9.6 ft3/lb, calculate the piston displacement,
assuming 80% percent volume efficiency.
A. 261.6 ft3/hr B. 271.6 ft3/hr C. 281.8 ft3/hr D. 291.6 ft3/hr
Solution:

( )

97
347. A single-stage ammonia compressor is producing 10 tons of refrigeration and the power
consumed is 15 Hp. Suction pressure is 25 psi, condensing pressure is 180 psi. Brine
temperature is 20°F off brine cooler. Determine the actual coefficient of performance.
A. 10.14 B. 11.14 C. 12.14 D. 13.14
Solution:
( )
( )

348. In an ammonia condensing machine (compressor plus condenser) the water used for
condensing is 55°F and the evaporator is at 15°F. Calculate the ideal COP.
A. 11.875 B. 12.875 C. 10.875 D. 13.875
Solution:

349. How much refrigeration capacity is required to cool 2000 cfm of air from 85°F to 70°F?
A. 2.7 TOR B. 3.7 TOR C. 1.7 TOR D. 4.7 TOR
Solution:
̇ ̇

-
̇ ( )( )

̇ ( )( ) ( )( )

350. Determine the coil face area required to maintain a face velocity 400 ft/min if the air flow
rate over the coil is 2100 ft3/min.
A. 3.25 ft2 B. 4.25 ft2 C. 5.25 ft2 D. 6.25 ft2
Solution:

98
351. Calculate the heat transfer per hour through a solid brick wall 6 m long, 2.9 m high, and
225 mm thick, when the outer surface is at 5°C and the inner surface 17°C, the coefficient of
thermal conductivity of the brick being 0.6 W/m-K.
A. 2,004.48 kJ B. 3,004.48 kJ C. 2,400.48 kJ D. 3,400.48 kJ
Solution:
[( )( )]( )

( )( )

352. A vertical furnace wall is made up of an inner wall of firebrick 20 cm thick followed by
insulating brick 15 cm thick and an outer wall of steel 1 cm thick. The surface temperature of
the wall adjacent to the combustion chamber is 1200°C while that of the outer surface of steel
is 50°C. The thermal conductivities of the wall material in W/m-K are: firebrick, 10; insulating
brick, 0.26; and steel, 45. Neglecting the film resistances and contact resistance of joints,
determine the heat loss per sq.m. of wall area.
A. 1.93 W/m2 B. 2.93 W/m2 C. 1.55 W/m2 D. 2.55 W/m2
Solution:
̇

353. A composite wall is made up of an external thickness of brickwork 110 mm thick inside
which is a layer of fiberglass 75 mm thick. The fiberglass is faced internally by an insulating
board 25 mm thick. The coefficients of thermal conductivity are as follow:
Brickwork 1.5 W/m-K
Fiberglass 0.04 W/m-K
Insulating board 0.06 W/m-K
The surface transfer coefficients of the inside wall is 3.1 W/m2-K while that of the outside wall is
2.5 W/m2-K. Take the internal ambient temperature as 10°C and the external temperature is
27°C. Determine the heat loss through such wall 6m high and 10 m long.
A. 330.10 W B. 230.10 W C. 430.10 W D. 530.10 W
Solution:
̇

99
-

[ ( ) ]( )
̇

354. One insulated wall of a cold-storage compartment is 8 m long by 2.5 m high and consists of
an outer steel plate 18 mm thick. An inner wood wall is 22.5 m thick. The steel and wood are 90
mm apart to form a cavity which is filled with cork. If the temperature drop across the extreme
faces of the composite wall is 15°C. Calculate the heat transfer per hour through the wall and
the temperature drop across the thickness of the cork. Take the coefficients of thermal
conductivity for steel, cork and wood as 45, 0.045, and 0.18 W/m-K respectively.
A. 408.24 kJ, 12.12°C C. 608.24 kJ, 13.12°C
B. 708.24 kJ, 11.12°C D. 508.24 kJ, 14.12°C
Solution:
̇

[ ( ) ]( )
̇

( )
̇

355. A cubical tank of 2 m sides is constructed of metal plate 12 mm and contains water at
75°C. The surrounding air temperature is 16°C. Calculate the overall heat transfer coefficient
from water to air. Take the coefficient of thermal conductivity of the metal as 48 W/m-K, the
coefficient of heat transfer of water is 2.5 kW/m2-K and the coefficient of heat transfer of the
air is 16 W/m2-K.
A. 15.84 W/m2-°C B.14.84 W/m2-°C C. 16.84 W/m2-°C D. 13.84 W/m2-°C
Solution:

100
-

356. A cold storage compartment is 4.5 m long by 4 m wide by 2.5 m high. The four walls,
ceiling and floor are covered to a thickness of 150 mm with insulating material which has a
coefficient of thermal conductivity of 5.8x10 -2W/m-K. Calculate the quantity of heat leaking
through the insulation per hour when the outside and inside face temperature of the material is
15°C and -5°C respectively.
A. 2185.44 kJ B. 2285.44 kJ C. 3185.44 kJ D. 4185.44 kJ
Solution:
̇

[ ( ) ( ) ( )]

-
( )( )( )
̇

357. A furnace wall consists of 35 cm firebrick (k = 1.557 W/m-K), 12 cm insulating refractory (k


= 0.346) and 20 cm common brick (k = 0.692) covered with 7 cm steel plate (k = 45). The
temperature at the inner surface of the firebrick is 1,230°C and at the outer face of the steel
plate is 60°C. Atmosphere is 27°C. What is the value of the combined coefficient for convection
and radiation from the outside wall?
A. 31.13 W/m2-K B. 30.13 W/m2-K C. 41.3 W/m2-K D. 40.13 W/m2-K
Solution:
̇

̇ ( )

̇ ̇

101
358. Hot gases at 280°C flow on one side of the metal plate 10 mm thickness and air at 35°C
flows on the other side. The heat transfer coefficient of the gases is 31.5 W/m2-K and that of
the air is 32 W/m2-K. Calculate the over-all heat transfer coefficient.
A. 15.82 W/m2-K B. 16.82 W/m2-K C. 14.82 W/m2-K D. 17.82 W/m2-K
Solution:

359. The surface temperature of the hot side of the furnace wall is 1200°C. It is desired to
maintain the outside of the wall at 38°C. A 152 mm of refractory silica is used adjacent to the
combustion chamber and 10 mm of steel covers the outside. What thickness of insulating bricks
is necessary between refractory and steel, if the heat loss should be kept at 788 W/m 2? Use k =
13.84 W/m-K for refractory silica; 0.15 for insulating brick, and 45 for steel.
A. 220 mm B. 240 mm C. 260 mm D. 280 mm
Solution:

360. An insulated steam pipe located where the ambient temperature is 32°C, has an inside
diameter of 50 mm with 10 mm thick wall. The outside diameter of corrugated asbestos
insulation is 125 mm and the surface coefficient of still air, h o = 12 W/m2-K. Inside the pipe is
steam having a temperature of 150°C with film coefficient h i = 6000 W/m2-K. Thermal
conductivity of pipe and asbestos insulation are 45 and 0.12 W/m-K respectively. Determine the
heat loss per unit length of pipe.
A. 110 W B. 120 W C. 130 W D. 140 W
Solution:
̇

102
( ) ( )

( ) ( )
[ ( ) ]( ) ( ) ( )

[ ( ) ]( )

̇
̇

361. How many watts will be radiated from a spherical black body 15 cm in diameter at a
temperature of 800°C?
A. 5.34 kW B. 4.34 kW C. 6.34 kW D. 3.34 kW
Solution:

-
-
( )

( -
)( )( )

362. A wall with an area of 10 m2 is made of a 2 cm thickness of white pine (k = 0.133 W/m-°C)
followed by a 10 cm of brick (k = 0.649 W/m-°C). The pine is on the inside where the
temperature is 30°C while the outside temperature is 10°C. Assuming equilibrium conditions
exist, what is the temperature at the interface between the two metals?
A. 15.65 °C B. 17.64 °C C. 18.21 °C D. 19.31 °C
Solution:
̇

( )
̇

103
( ) ( )( )
̇ ̇

363. A counter-flow heat exchanger is designed to heat a fuel oil from 45°C to 100°C while the
heating fluid enters at 150°C and leaves at 115°C. Calculate the arithmetic mean temperature
difference.
A. 40°C B. 50°C C. 60°C D. 70°C
Solution:

364. With three different quantities x, y, and z of the same kind of liquid of temperatures 9, 21
and 38°C respectively, it is found that when x and y are mixed together the resultant
temperature is 17°C and when y and z are mixed together the resultant temperature is 28°C.
Find the resultant temperature if x and z were mixed.
A. 25.87°C B. 25.92°C C. 20.85°C D. 24.86°C
Solution:

( ) ( )

( ) ( )

( ) ( )
( ) ( )

104
365. The journals of a shaft are 380 mm diameter, it runs at 105 rpm and the coefficient of
friction between journals and bearings is 0.02. If the average load on the bearings is 200 kN,
find the heat generated per minute at the bearings.
A. 501.375 kJ B. 505.575 kJ C. 401.375 kJ D. 501.575 kJ
Solution:

( )
( )

[ ( )]( ) -
( )
( )( )( )
( )
̇ ( )( )( )

366. A reverse Carnot cycle requires 3 Hp and extracts energy from a lake to heat a house. If the
house is kept at 70°F and requires 2000 Btu per minute, what is the temperature of the lake?
A. 35.29°F B. 36.29°F C. 39.29°F D. 40.29°F
Solution:
̇ ( )

̇ ̇ ̇

̇
̇

̇ ( ) ( )
( )( )

̇ ( )
( )

105
367. An oxygen cylinder of volume 2.3 ft3 has a pressure of 2200 psig and is at 70°F. Determine
the mass of oxygen in the cylinder.
A. 25.66 lbs B. 26.66 lbs C. 27.66 lbs D. 28.66 lbs
Solution:

( )

( )( ) ( )( )

368. A group of 50 persons attend a secret meeting in a room which is 12 m wide by 10 m long
and a ceiling height of 3 m. The room is completely sealed off and insulated. Each person gives
off 150 kCal per hour of heat and occupies a volume of 0.20 m3. The room has an initial
pressure of 101.3 kPa and temperature of 16°C. Calculate the room temperature after 10
minutes. Use R = 0.287 kJ/kg-K and C v = 0.171 kCal/kg-K.
A. 33.1°C B. 37.7°C C. 38.7°C D. 31.7°C
Solution:

( )

( )

[ ( )( ) ( )]

( ) ( )( )

( )( )

106
369. One kilogram of wet steam at a pressure of 8 bar (υ g = 0.2404 m3/kg, υf = 0.0011148
m3/kg) and dryness of 0.94 is expanded until the pressure is 4 bar (υ g = 0.4625 m3/kg, υf =
0.0010836 m3/kg). If expansion follows the law PV n = C, where n = 1.12, find the dryness
fraction of the steam at the lower pressure.
A. 0.9072 B. 0.4197 C. 0.2260 D. 0.2404
Solution:

( ) ( )( )

( ) ( )

( ) ( )

( ) ( )( )

370. 2.5 liters of superheated steam at 25 bar and 400 C (υ = 0.1252 m3/kg) is expanded in an
engine to a pressure of 0.1 bar (υ g = 14.674 m3/kg, υf = 0.0010102 m3/kg) when its dryness
fraction is 0.9. Find the final volume of the steam.
A. 163.74 liters B. 263.74 liters C. 363.74 liters D. 463.74 liters
Solution:

( ) ( )
( )( )

371. A 1.5 kg of wet steam at a pressure of 5 bar (hf = 640 kJ/kg, hfg = 2109 kJ/kg) dryness 0.95
is blown into 70 liters of water 12°C (h = 50.4 kJ/kg). Find the final enthalpy of the mixture.
A. 74.80 kJ/kg B. 84.80 kJ/kg C. 94.80 kJ/kg D. 104.80 kJ/kg
Solution:

( ) ( )
107
[ ( )] ( )

( )

372. A 650 BHP diesel engine uses fuel oil of 28°API gravity, fuel consumption is 0.65 lb/BHP-hr.
Cost of fuel is P7.95 per liter. For continuous operation determine the minimum volume of
cubical day tank in cm3, ambient temperature is 45° C.
A. 4,372,890 cm3 B. 5,987,909 cm3 C. 5,291,880 cm3 D. 7,352,789 cm3
Solution:

( )

[ ( )]
[ ( )]
( )

373. A typical industrial fuel oil, C 16H32 with 20% excess air by weight. Assuming complete
oxidation of the fuel, calculate the actual air-fuel ratio by weight.
A. 17.56 kg air/kg fuel C. 16.75 kg air/kg fuel
B. 15.76 kg air/kg fuel D. 17.65 kg air/kg fuel
Solution:

( )

108
( ) ( )

( )( ) ( )

( ) ( )
( ) ( ) ( )

374. Fuel oil in a day tank for use of an industrial boiler is tested with hydrometer. The
hydrometer reading indicates a SG = 0.924 when the temperature of the oil in the tank is 35°C.
Calculate the higher heating value of the fuel.
A. 43,852.13 kJ/kg B. 53,852.13 kJ/kg C. 58,352.13 kJ/kg D. 48,352.13 kJ/kg
Solution:
( )

[ ( )]

( )

375. A diesel electric plant supplies energy for Meralco. During a 24 hr Period, the plant
consumed 200 gallons of fuel at 28°C and produced 3930 Kw-hr. Industrial fuel used is 28°API
and was purchased at P5.50 per liter at 15.6°C. What should the cost of fuel be produce one
kw-hr.?
A. P 1.05 B. P 1.10 C. P 1.069 D. P 1.00
Solution:

( )

[ ( )]
109
[ ( )]
( )

-
( )( )( )( ) -
-

376. A certain coal has the following ultimate analysis:


C = 70.5% H = 4.5% O2 = 6% N2 = 1.0%
S = 3.0% Ash = 11% Moisture = 4%
A stoker fired boiler of 175,000 kg/hr steaming capacity uses this coal as fuel. Calculate the
volume of air in m3/hr with air at 60°F (15.6°F) and 14.7 psia (101.325 kPa) the coal is burned
with 30% excess air. Boiler efficiency is 70% and factor of evaporation of 1.10.
A. 212,861.04 m3/hr B. 221,861.04 m3/hr C. 218,261.04 m3/hr D. 281,261.04 m3/hr
Solution:
̇( )( )
̇

( ) ( ) ( ) ( )

( )
( ) ( ) ( )

̇ ( ) ̇ ( ) ( )( )
̇ ̇ ̇ ( )
( )
̇ ( )

̇
̇ ̇ ( ) ( ) ( )( )

( )( )
̇

377. A diesel power plant consumed 1 m3 of fuel with 30°API at 27°C in 24 hrs. Calculate the
fuel rate in kg/hr.
A. 36.21 B. 26.25 C. 29.34 D. 39.42
Solution:
110
̇ ̇
( )

[ ( )]
( )

̇ ( )

378. A diesel power plant uses fuel with heating value of 43,000 kJ/kg. What is the density of
the fuel at 25°C?
A. 840 kg/m3 B. 873 kg/m3 C. 970 kg/m3 D. 940 kg/m3
Solution:

[ ( )]

( )

379. A water tube boiler has a capacity of 1000 kg/hr of steam. The factor of evaporation is 1.3,
boiler rating is 200%, boiler efficiency is 65% and heating surface area is 0.91 m 2 per bo.Hp. And
the heating value of fuel is 18,400 kCal/kg. The total coal available in the bunker is 50,000 kg.
Determine the no. of hrs. to consume the available fuel.
A. 853.36 hrs B. 706.57 hrs C. 979.46 hrs D. 100.75 hrs
Solution:

( )
111
( ) [ ( )]
[ ( )]

380. Two boilers are operating steadily on 91,000 kg of coal contained in a bunker. One boiler is
producing 1591 kg of steam per hour at 1.2 factor of evaporation and an efficiency of 65% and
another boiler produced 1364 kg of steam per hour at 1.15 factor of evaporation and an
efficiency of 60%. How many hrs will the coal in the bunker run the boilers if the heating value
of coal is 7,590 kCal/kg?
A. 230.80 hrs B. 280.54 hrs C. 350.35 hrs D. 300.54 hrs
Solution:

̇ ( )
̇
̇ [ ( )]
̇
[ ( )]
̇ [ ( )]

̇ [ ( )]
̇
[ ( )]
̇ [ ( )]

̇ ̇ ̇

381. The heating value of fuel supplied in a boiler is 40,000 kJ/kg. If the factor of evaporation is
1.10 and the actual specific evaporation is 10, what is the efficiency of the boiler?
A. 62.07% B. 53.08% C. 78.05% D. 54.97%

112
Solution:
̇ ( ) [ ( )] [ ( )]
̇

382. What is the rate of evaporation of a water tube boiler if the factor of evaporation is 1.10,
percent rating of 200% and the heating surface area is 250 m2?
A. 7,817.16 kg/hr B. 7,898.67 kg/hr C. 6,789.45 kg/hr D. 5,768.54 kg/hr
Solution:
̇ ( ) ̇ [ ( )]

̇ [ ( )]

383. Steam is admitted to the cylinder of an engine in such a manner the average pressure is
120 psi. The diameter of the piston is 10 in. and the length of a stroke is 12 in. What is the Hp of
the engine when it is making 300 rpm?
A. 171.4 Hp B. 175 Hp C. 173.2 Hp D. 174.4 Hp
Solution:

( ) ( )( )
( ) [ ]

( )( )

384. Steam enters a turbine stage with an enthalpy of 3628 kJ/kg at 70 m/s and leaves the same
stage with an enthalpy of 2846 kJ/kg and a velocity of 124 m/s. Calculate the power if there are
5 kg/s steam admitted at the turbine throttle?
A. 4597.45 kW B. 3976.55 kW C. 3883.81 kW D. 1675.42 kW
Solution:

113
̇ ̇ ( ) ̇ ( ) ( ) ( )( )
̇

385. Steam with an enthalpy of 800 kCal/kg enters a nozzle at a velocity of 80 m/s. Find the
velocity of the steam at the exit of the nozzle if its enthalpy is reduced to 750 kCal/kg, assuming
the nozzle is horizontal and disregarding heat losses. Take g = 9.81 m/s 2 and J constant = 427 kg
m/kCal.
A. 452.37 m/s B. 245.45 m/s C. 651.92 m/s D. 427.54 m/s
Solution:

( )

386. Steam is expanded through a nozzle and the enthalpy drop per kg of steam from the initial
pressure to the final pressure of 60 kJ. Neglecting friction, find the velocity of discharge and the
exit area of the nozzle to pass 0.20 kg/s if the specific volume of the steam at exit is 1.5 m 3/kg.
A. 346.4 m/s, 879 mm2 C. 765.6 m/s, 467 mm2
B. 356.7 m/s, 278 mm2 D. 346.4 m/s, 866 mm2
Solution:

√ √ ( ) √ ( )

( ) ( )

387. A 6 MW steam turbine generator power plant has a full-load steam rate of 8 kg/kW-hr.
Assuming that no-load steam consumption as 15% of full-load steam consumption, compute for
the hourly steam consumption at 75% load, in kg/hr.
A. 37,800 kg/hr B. 38,700 kg/hr C. 30,780 kg/hr D. 30,870 kg/hr
Solution:
114
-
̇ ( )
-

- ( )
̇ ( )
-

( ) ( )
( ) ( )
( ) ( )

̇
̇

( )

̇ ( )

388. A 4 kg of air enters a turbine with enthalpy of 600 kJ and velocity of 250 m/s. The enthalpy
at exit is 486 kJ and velocity of 170 m/s. What is the work developed if there is a heat loss of 10
kJ?
A. 122.83 kJ B. 171.2 kJ C. 80.2 kJ D. 28.3 kJ
Solution:
̇ ̇
[ ( )( ) ] [ ( )( )]

389. Calculate the drive horsepower for pumping 1703 L/min cold water to a tank suction at
127 mmHg vacuum, delivery at 5.3 kg/cm2 ga., both measured close to pump, e p = 0.65.
A. 31.42 Hp B. 20.42 Hp C. 35.42 Hp D. 23.02 Hp
Solution:

115
( )

- -
( ) ( )

( )( )( )
-
( -
)

390. Find the length of a suspension bunker to contain 181 tons of coal without surcharge;
width = 4.6 m, depth = 4.3 m. The level capacity of a suspension bunker is 5/8 wdL where: w =
width, d = depth and L = length. Density of coal is 800 kg/m3.
A. 18.30 m B. 15.80 m C. 17.61 m D. 12.61 m
Solution:

( )

( )( )

391. A 305 mm x 457 mm four stroke single acting diesel engine is rated at 150 kW at 260 rpm.
Fuel consumption at rated load is 0.26 kg/kW-hr with a heating value of 43,912 kJ/kg. Calculate
the brake thermal efficiency.
A. 31.63% B. 41.63% C. 21.63% D. 35.63%
Solution:

116
̇ ( )
-

( )

392. The brake thermal efficiency of a 1 MW diesel electric plant is 36%. Find the heat
generated by fuel in kW if the generator efficiency is 89%.
A. 3,121.10 kW B. 3,528.64 kW C. 4,121.10 kW D. 4,528.64 kW
Solution:

393. In an air-standard Brayton cycle, the compressor receives air at 101.325kPa, 21°C and it
leaves at 600 kPa at the rate of 4 kg/s. Determine the turbine work if the temperature of the air
entering the turbine is 1000°C.
A. 3000 kW B. 2701 kW C. 2028 kW D. 3500 kW

Solution:
̇ ̇ ( )

( )

( )

̇ ( )( )

117
394. Kerosene is the fuel of a gas turbine plant: fuel-air ratio, mf = 0.012, T3 = 972 K, pressure
ratio, rp = 4.5, exhaust to atmosphere. Find the available energy in kJ per kg air flow. Assume k =
1.34 and C p = 1.13.
A. 352.64 kJ/kg B. 452.64 kJ/kg C. 252.64 kJ/kg D. 552.64 kJ/kg

Solution:

( ) ( )

( )

( )

( )( )( )

395. An ideal gas turbine operates with a pressure ratio of 10 and the energy input in the high
temperature heat exchanger is 300 kW. Calculate the air flow for temperature limits of 30°C
and 1200°C.
A. 0.25 kg/s B. 0.34 kg/s C. 0.41 kg/s D. 0.51 kg/s
Solution:
̇ ̇ ( )

( )

̇ ( )( )
̇

396. In an air-standard Brayton cycle the inlet temperature and pressure are 20°C and 101.325
kPa. The turbine inlet conditions are 1200 kPa and 900°C. Determine the air flow if the turbine
produces 12 MW.
A. 21.41 kg/s B. 20.20 kg/s C. 19.25 kg/s D. 18.10 kg/s
Solution:

118
̇ ( )

( )

( )

̇ ( )( )
̇

397. A gas turbine power plant operating on the Brayton cycle delivers 15 MW to a standby
electric generator. What are the mass flow rate and the volume flow rate of air if the minimum
and maximum pressures are 100 kPa and 500 kPa respectively and temperature of 20°C and
1000°C?
A. 31.97 kg/s, 26.88 m3/s C. 41.97 kg/s, 26.88 m3/s
B. 36.98 kg/s, 28.99 m3/s D. 46.98kg/s, 28.99 m3/s
Solution:
̇ ̇

( )

( )

̇ ̇( )
̇ ( )( )
̇

̇ ( )( )
̇

398. In a hydraulic plant the difference in elevation between the surface of the water at intake
and the tailrace is 650 ft when the flow is 90 cfs, the friction loss in the penstock is 65 ft and the
head utilized by the turbine is 500 ft. The mechanical friction in the turbine is 110 Hp, and the
leakage loss is 4 cfs. Find the hydraulic efficiency.
A. 87.45% B. 84.57% C. 85.47% D. 78.54%
Solution:
119
399. A hydro-electric power plant consumes 60,000,000 kW-hr per year. What is the net head if
the expected flow is 1500 m3/min and over-all efficiency is 63%?
A. 34.34 m B. 43.43 m C. 44.33 m D. 33.44 m
Solution:

( )

400. A pelton type turbine has a gross head of 40 m and a friction head loss 6 m. What is the
penstock diameter if the penstock length is 90 m and the coefficient of friction head loss is
0.001 (Morse)?
A. 2040 mm B. 3120 mm C. 2440 mm D. 2320 mm
Solution:

√ √ ( )( )

( )( )( )

401. The water velocity of a 5 m x 1 m channel is 6 m/s. What is the annual energy produced if
the net head is 120 m and the over-all efficiency is 80%?
A. 494,247,258 kW-hrs C. 247,494,528 kW-hrs
B. 247,497,582 kW-hrs D. 472,497,582 kW-hrs
Solution:
( )
( )
120
( )( )
( )( )

( )( )
-

402. A hydro-electric impulse turbine is directly coupled to a 24 pole, 60 Hz alternator. It has a


specific speed of 60 rpm and develops 3000 Hp. What is the required diameter assuming a
peripheral speed ratio of 0.45?
A. 0.661 m B. 0.552 m C. 0.443 m D. 0.773 m
Solution:

( )

( )

√ ( )( )

403. In a hydroelectric power plant the tailwater elevation is at 500 m. What is the head water
elevation if the net head is 30 m and the head loss is 5% of the gross head?
A. 785.25 m B. 582.57 m C. 528.57 m D. 758.25 m
Solution:

121
404. The tailwater and headwater of a hydro-electric plant are 150 m and 200 m respectively.
What is the water power if the flow is 15 m3/s and a head loss of 10% of the gross head?
A. 6,621.75 kW B. 7,621.65 kW C. 5,621.76 kW D. 4,621.56 kW
Solution:

( )
( )( )

405. In a hydro-electric plant, water flows at 10 m/s in a penstock of 1 m3 cross-sectional area.


If the net head of the plant is 30 m and the turbine efficiency is 85%, what is the turbine
output?
A. 2,501.55 kW B. 2,100.21 kW C. 3,626.34 kW D. 3,124.65 kW
Solution:

( )
( )( )

406. A 75 MW power plant has an average load of 35,000 kW and a load factor of 65%. Find the
reserve over peak.
A. 21.15 MW B. 23.41 MW C. 25.38 MW D. 18.75 MW
Solution:

407. A power plant is said to have/had a use factor of 48.5% and a capacity factor of 42.4%.
How many hrs. did it operate during the year?
A. 6,600.32 hrs B.7,658.23 hrs C. 8,600.32 hrs D. 5,658.23 hrs
122
Solution:
-

( ) ( )

408. A 50,000 kW steam plant delivers an annual output of 238,000,000 kW-hr with a peak load
of 42,860 kW. What is the annual load factor and capacity factor?
A. 0.634, 0.534 B. 0.643, 0.534 C. 0.634, 0.543 D. 0.643, 0.534
Solution:

- -

409. Calculate the use factor of a power plant if the capacity factor is 35% and it operates 8000
hrs during the year?
A. 38.325% B. 33.825% C. 35.823% D. 32.538%
Solution:
( )
( )

410. If the air required for combustion is 20 kg per kg of coal and the boiler uses 3000 kg of coal
per hr, determine the mass of gas entering the chimney. Assume an ash loss of 15%.
A. 40,664 kg/hr B. 70,200 kg/hr C. 62,550 kg/hr D. 50,500 kg/hr
Solution:
̇ ̇ ̇ ̇
123
̇
̇
̇ ̇
̇ ̇ ̇ ̇
̇ ̇ ( )

411. A 15 kg gas enters a chimney at 10 m/s. If the temperature and pressure of a gas are 26°C
and 100 kPa respectively, what is the diameter of the chimney? Use R = 0.287 kJ/kg -K.
A. 1.57 m B. 2.65 m C. 2.22 m D. 1.28 m
Solution:

̇ ( )( )
̇

( )

412. A two-stage air compressor at 90 kPa and 20°C discharges at 700 kPa. Find the polytropic
exponent n if the intercooler intake temperature is 100°C.
A. 1.29 B. 1.33 C. 1.4 D. 1.25
Solution:

( )

√ √ ( )

( )

413. A two-stage compressor receives 0.35 kg/s of air at 100 kPa and 269 K and delivers it at
5000 kPa. Find the heat transferred in the intercooler.
A. 70.49 kW B. 80.49 kW C. 90.49 kW D. 100.49 kW
Solution:
̇ ̇ ( )

124
√ ( )

( )

( )

̇ ( )( )

414. A centrifugal pump discharged 20 L/s against a head of 17 m when the speed is 1500 rpm.
The diameter of the impeller was 30 cm and the brake horsepower was 6.0. A geometrically
similar pump 40 cm in diameter is to run at 1750 rpm. Assuming equal efficiencies, what brake
horsepower is required?
A. 51.55 HP B. 50.15 HP C. 40.14 HP D. 45.15 HP
Solution:

( ) ( )

415. A pump delivers 20 cfm of water having a density of 62 lb/ft 3. The suction and discharge
gage reads 5 in. Hg vacuum and 30 psi respectively. The discharge gage is 5 ft above the suction
gage. If pump efficiency is 70%, what is the motor power?
A. 5.31 HP B. 3.31 HP C. 4.31 HP D. 6.31 HP
Solution:

[ (- ) ( )]
( )

-
( )( ) ( -
)

416. Calculate the air power of a fan that delivers 1200 m3/min of air through a 1 m x 1.5 m
outlet. Static pressure is 120 mm WG and density of air is 1.18.

125
A. 20.45 kW B. 25.64 kW C. 30.45 kW D. 35.64 kW
Solution:

( )

[ ( )]

( )

[ ( )] ( )( )

417. Determine the temperature for which a thermometer with degrees Fahrenheit is
numerically twice the reading of the temperature in degrees Celsius.
A. -24.6 B. 320 C. 160 D. -12.3
Solution:
-
( ) -

( )

[ ( )]

418. During takeoff in a spaceship, an astronaut is subjected to acceleration equal to 5 times


the pull of the earth’s standard gravity. If the astronaut is 180 lb m and the takeoff is vertical,
what force does he exert on the seat?
A. 4810.9 N B. 4414.5 N C. 8829 N D. 9620 N
Solution:

( )
126
( ) ( )( ) ( ) ( )

419. A pressure cooker operates by cooking food at a higher pressure and temperature possible
at atmospheric conditions. Steam is contained in the sealed pot, with vent hole in the middle of
the cover, allowing steam to escape. The pressure is regulated by covering the vent hole with a
small weight, which is displaced slightly by escaping steam. Atmospheric pressure is 100 kPa,
the vent hole area is 7-mm2, and the pressure inside should be 250 kPa. What is the mass of the
weight?
A. 0.107 kg B. 1.05 kg C. 1.75 kg D. 0.1783 kg
Solution:

( )
-
( ) ( )

420. A barometer can be used to measure an airplane’s altitude by comparing the barometric
pressure at a given flying altitude to that on the ground. Determine an airplane’s altitude if the
pilot measures the barometric pressure at 700 mm-Hg, the ground reports it at 758 mm-Hg,
and the average air density is 1.19 kg/m3, g = 9.81 m/s 2.
A. 636 m B. 633 m C. 366 m D. 663 m
Solution:
( ) - ( )
-

( )
( )( )

421. A mixture of 0.4 lbm of helium and 0.2 lbm of oxygen is compressed polytropically from
14.7 psia and 60°F to 60 psia according to n = 1.4. Determine the final temperature, T 2.
A. 727.7°R B. 777.2°R C. 722.7°R D. 277.7°R
Solution:

( )

127
( ) ( )

422. A mixture of 0.4 lbm of helium and 0.2 lbm of oxygen is compressed polytropically from
14.7 psia and 60oF t0 60 psia according to n = 1.4. Determine the polytropic work.
A. 139 Btu B. 239 Btu C. 339 Btu D. 539 Btu
Solution:
- -

( )( - - )
( )( - - )
-
( ) -
- -

( ) ( - )( )

423. A pump in a municipality’s water-supply system receives water from the filtration beds
and pumps it up to the top of a water tower. The tower’s height is 35 m, and the inlet piping to
the pump is 2 m below the pump’s intake. The water temperature is 20 C, measured at both
the inlet and the discharge from the pump. The mass flow rate through the pump is 100 kg/s,
the diameter of the inlet piping is 25 cm, and the diameter of the discharge piping is 15 cm.
Determine the power required by the pump.
A. 77.3 kW B. 33.77 kW C. 34.42 kW D. 42.34 kW
Solution:
̇
̇ ( )

( ) ( )

( )
128
( )

( ) ( )
( )

̇ ( )( )

424. An adiabatic tank containing air is used to power an air turbine during times of peak power
demand. The tank has a volume of 500 m3 and contains air at 1000 kPa and 500 K. Determine
the mass remaining when the pressure reaches 100 kPa.
A. 273.37 kg B. 672.73 kg C. 772.73 k D. 227.73 kg
Solution:

( )

( )

( ) ( - )( )

425. Determine the air-fuel ratio on a molar basis for the complete combustion of octane, with
theoretical amount of air.
A. 95.5 kg air/kg fuel B. 59.9 kg air/kg fuel C.59.5 kg air/kg fuel D. 55.9 kgair/kg fuel
Solution:

( )

( )
[ ( )]
( )

426. During a steady state operation, a gearbox receives 60 kW throughout the input shaft and
delivers power through the output shaft. For the gearbox as the system, the rate of energy
transfer is by convection, h = 0.171 kW/m2-K is the heat transfer coefficient, A = 1.0 m2 is the
outer surface area of the gearbox, Tb = 300 K (27°C) is the temperature at the outer surface, T f =
293 K (20°C) is the temperature of the surroundings away from the immediate vicinity of the

129
gearbox. Determine the power delivered to the output shaft in kW if the heat transfer rate is -
1.2 kW.
A. 98.8 kW B. 78.8 kW C. 68.8 kW D. 58.8 kW
Solution:
̇ ̇ ̇ ̇
̇ ̇ ̇ ̇
̇ -
̇ -
̇ - (- )

427. A single acting air compressor with a clearance of 6% takes in air at atmospheric pressure
and temperature of 85°F and discharges it at a pressure of 85 psia. The air handled is 0.25
ft3/cycle measured at discharge pressure. If the compression is isentropic, find the piston
displacement per cycle, if the compressor is running at 750 rpm.
A. 0.0750 ft3/cycle B. 0.025 ft3/cycle C. 1.030 ft3/cycle D. 1.090 ft3/cycle
Solution:

( )

( ) ( )
( )( )
( )
( )( )
( )

( ) ( )

428. A single acting air compressor with a clearance of 6% takes in air at atmospheric pressure
and temperature of 85°F and discharges it at a pressure of 85 psia. The air handled is 0.25
ft3/cycle measured at discharge pressure. If the compression is isentropic, find the air hp of the
compressor if rpm is 750.
A. 16 hp B. 96 hp C. 69 hp D. 61 hp
Solution:
̇ ( )( )

130
̇ ( )( )( )
̇ [( ) ] [( ) ]
( )

429. A nozzle receives 0.5 kg/s of air at a pressure of 2700 kPa and a velocity of 30 m/s and with
an enthalpy of 923 kJ/kg, and the air leaves at a pressure of 700 kPa and with an enthalpy of
660 kJ/kg. Determine the exit velocity from the nozzle.
A. 923 m/s B. 726 m/s C. 700 m/s D. 660 m/s
Solution:

( ) ( )

( )

430. A two-stage, double acting compressor is to deliver 90 lb/min of air from 14.3 psia to 90°F
to a final pressure of 185 psia. The normal barometer is 29.8 in-Hg and the temperature is 80°F.
The pressure drop in the intercooler is 3 psi and the speed is 210 rpm and pV 1.34 = C during
compression and expansion. The clearance is 5% for both cylinders. Find the volume of free air
if the temperature of the cooling water increased by 18°F.
A. 1282 CFM B. 1230 CFM C. 1320 CFM D. 1822 CFM
Solution:
̇ ( )( )
̇
( )
̇
̇ ̇

̇ ( )[( ) ]
̇
( ) [ - ( )]
-

431. Consider 4800 lb of steam per hour flowing through a pipe at 100 psia pressure. Assume a
velocity of 5280 ft/min. What size of pipe is required? Specific volume of steam at 100 psia υ =
4.432 ft3/lb.
A. 3 in B. 5 in C. 4 in D. 6 in
Solution:

131
( ) ( )
( )
√ ( )

( )

432. A boiler plant generates 225,000 lb of steam and burns 13.9 tons of coal per hour. The coal
has a heating value of 11,400 Btu/lb. A test of the particulates leaving the boiler shows that
3804 lb of particulate is being discharged per hour. What is the particulate discharged per
million Btu heat input to the furnace?
A. 12 lb/106 Btu B. 14 lb/106 Btu C. 15 lb/106 Btu D. 16 lb/106 Btu
Solution:

433. A turbine receives 150 lbm/s of air at 63 psia and 2450°R and expands it polytropically to
14.7 psia. The exponent n is equal to 1.45 for the process. Determine the power.
A. 52,343.16 BTU/s C. 53.343.16 HP
B. 52,343.16 kW D. 53,343.16 ft-lb/s
Solution:

( )

( )

-
( )[( ) ]
( )
̇ -

-
( )
-

434. Find the thrust and efficiency of two 2-m diameter propellers through which flows a total
of 600 m3/s of air to 11.3 N/m3. The propellers are attached to an airplane moving at 250 kph
through still air. Neglect eddy losses.
A. 36,077 N, 73% B. 77,630 N, 37% C. 66,033 N, 33% D. 77,330 N, 77%
Solution:

132
( )
( )

( )

( )

( )( )

435. A liquid with a specific gravity of 1.26 is being pumped in a pipeline from A to B. At A, the
pipe diameter is 60 cm and the pressure is 300 kN/m2. At B, the pipe diameter is 30 cm and the
pressure is 330 kN/m2. Point B is 1.0 m lower than A. Find the flow rate if the pump puts 16 kW
into the flow. Neglect head loss.
A. 4.2 m3/s B. 0.42 m3/s C. 2.4 m3/s D. 0.24 m3/s
Solution:
( )

[ ( )
] [ ( )
]

( ) ( )
- ( ) ( )

436. A reciprocating compressor handles 1,400 cfm of air measured at intake where P1 = 18 psia
and T1 = 90°F. The discharge pressure is 92 psia. Calculate the work if the process of the
compression is isothermal.
A. -180.5 hp B. -179.5 hp C. -227.6 hp D. -228.6 hp
Solution:
( )( )
̇ ( ) -
-
( )
-

437. The fuel oil has the ff. analysis:


C = 89% N2 = 2% H2 = 8% S = 1%
With 3% excess air, what is the actual amount of air needed to burn the fuel oil?

133
A. 13.17 kg air/kg fuel C. 14.47 kg air/kg fuel
B. 13.47 kg air/kg fuel D. 14.17 kg air/kg fuel
Solution:
( )
( ) ( ) ( )
( )( )

438. A pump discharges 550 gpm of water to a height of 35 ft. With an efficiency of 80%, what
is the power input?
A. 6.09 hp B. 6.32 hp C. 4.74 hp D. 4.94 hp
Solution:
( )( )( )
-
( )
-

439. A room contains air at 20°C and 96 kPa at a relative humidity of 75%. Determine the
enthalpy of moist air.
where:
(Psat @ 20°C = 2.339 kPa)
(hg @ 20°C = 2538.1 kJ/kg da )
Given:
Unsaturated air (0% < Ø <100%)
td = 20°C
Pt = 96 kPa
Ø = 75%
A. 45.919 kJ/kg da B. 45.515 kJ/kg da C. 49.515 kJ/kg da D. 41.815 kJ/kg da
Solution:

( )

( ) ( )
( ) ( ) ( )

134
440. A piston moves inside a cylinder at a velocity of 6.0 m/s. The 160 mm diameter piston is
centrally located within the 160.2 mm inside diameter cylinder. The film of oil is separating the
piston from the cylinder has an absolute viscosity of 0.4 N-s/m2. Assuming a linear velocity
profile, find the shear stress in the oil. (T = μ (v/H))
Given:
μ = 0.40 N-s/m2
v = 5 m/s
H = thickness of oil film = 160.2 – 160/2 = 0.1 mm
A. 50,000 N/m2 B. 40,000 N/m2 C. 24,000 N/m2 D. 34,000 N/m2
Solution:
-
( )

441. A centrifugal pump with a 3 ft impeller diameter operates at 800 rpm. If the speed is to be
increased to 1200 rpm, determine the impeller diameter that should be used so that the same
shaft input power would be required.
A. 5.32 ft B. 2.35 ft C. 5.23 ft D. 2.93 ft
Solution:
( )
( )
( ) ( )

( ) ( )

442. Determine the mass of water vapor contained in a 150 m3 room at 100 kPa, 23°C and 40%
relative humidity. From Steam Tables: Psat @ 23°C = 2.810 kPa.
A. 1.6342 kg B. 1.9342 kg C. 1.2342 kg D. 2.2342 kg
Solution:

( )
( )

443. What is the power of the pump, HP, if it delivers 925 gal/min of water against a head of 15
m?
A. 15.38 HP B. 16.38 HP C. 10.5 HP D. 11.5 HP
Solution:

135
[ ( )]( )( ) ( )( )( )
-
-

444. Kerosene is pumped into an aircraft fuel tank through a hose that has an inside diameter
of 4 cm. If the velocity of the kerosene is 8 m/s through the hose, determine the mass flow rate.
Assume that the kerosene has a density of 800 kg/m³.
A. 7.06 kg/s B. 7.56 kg/s C. 8.06 kg/s D. 8.56 kg/s
Solution:
( )
̇ ̇ ( )( )

445. During the working stroke of an engine the heat transferred out of the system was 150
kJ/kg of working substance. The internal energy also decreased by 400 kJ/kg of working
substance. Determine the work done.
A. 250 kJ/kg B. 550 kJ/kg C. 600 kJ/kg D. 350 kJ/kg
Solution:
( )
- (- ) -

446. During the experiment on Charles Law, the volume of the gas trapped in the apparatus is
10000 mm³ when the temperature is 18°C. The temperature of the gas was then raised to
85°C. Determine the new volume of the gas trapped in the apparatus if the pressure exerted on
the gas remained constant.
A. 12302.41 mm³ B. 8128.49 mm³ C. 70833.33 mm³ D. 2117.64 mm³
Solution:

447. Find the density of oil with a specific gravity of 1.6 in-g/cm3.
A. 15.68 g/cm3 B. 99.84 g/cm3 C. 0.8 g/cm3 D. 1.6 g/cm3
Solution:

136
448. What is the absolute pressure if the gauge pressure reading is 9 bar and the atmospheric
pressure is 0.9 bar?
A. 6.3 bar B. 7.8 bar C. 9.9 bar D. 8.1 bar
Solution:

449. The tank of an air compressor has a volume of 3 ft3 and is filled with air at a temperature of
40°F. If a gage on the tank reads 150 psig, what is the mass of the air in the tank?
A. 1.78 lbs B. 2.00 lbs C. 2.67 lbs D. 1.98 lbs
Solution:
- -

( ) ( )( )
-
( )
-

450. What is the mass of acetylene gas, V = 0.94 cu. ft., R = 59.35 ft-lb/lb-°R, T = 90°F, P = 200
psia?
A. 0.816 lb B. 0.841 lb C. 0.829 lb D. 0.852 lb
Solution:
( )( )
( )

451. Specific volume is the number of cubic meters of mixture per kilogram of dry air. If dry air
has these following properties: R a = 287 J/kg-K, T = 303 K, Pa = 99.604 kPa. Solve for the specific
volume.
A. 0.873 m3/kg B. 0.853 m3/kg C. 0.953 m3/kg D. 0.783 m3/kg
Solution:
( )

452. A refrigerating system operates on the reversed Carnot Cycle. The higher temperature of
the refrigerant in the system is 120°F and the lower is 10°F. The capacity is 20 tons. Neglect
losses. Determine the coefficient of performance.
A. 2.732 B. 5.373 C. 7.372 D. 4.273
Solution:

137
( ) ( )

453. At a pressure of 60°F, a motorbike Ire is inflated to 33 psig. As it is driven along the C -5
road, the temperature rose to 76°F. Assuming the volume remains constant, determine the
final gauge pressure.
Given:
P1 = 33 psig + 14.7 psig = 47.7 psig
T1 = 60°F + 460 = 520°R
T2 = 76°F + 460 = 536°R
A. 34.47 psig B. 49.17 psig C. 35.00 psig D. 34.30 psig
Solution:

454. Steam enters a turbine stage with an enthalpy of 3700 kJ/kg and a velocity of 80 m/s and
leaves with an enthalpy of 2864 kJ/kg with a velocity of 12.8 m/s. If the rate of a steam flow
through the turbine is 0.44 kg/s, what is the work done in kW?
A. 365 kW B. 365.64 kW C. 366.0 kW D. 366.50 kW
Solution:

( )
-
( )
-

( )
-
( )
-

455. Aluminum has a specific heat of 0.902 J/g-°C. How much heat is lost when a piece of
aluminum with a mass of 23.984 g cools from a temperature of 415.0°C to a temperature of
22.0°C?
A. 8500 J B. 6000 J C. 80000 J D. 7500 J
Solution:
138
( ) ( )( )

456. If the temperature of an air parcel is -20.5°C, and its density is 0.690 kg/m3, what is the
pressure of the air parcel?
A. 40 kPa B. 50 kPa C. 60 kPa D. 70 kPa
Solution:

457. A 35.0 mL sample of gas is enclosed in a flask at 22 degrees Celsius. If the flask was placed
in an ice bath at 0 degrees Celsius, what would the new gas volume be if the pressure is held
constant?
A. 34.1 mL B. 32.1 mL C. 32.39 mL D. 33.1 mL
Solution:

458. A car engine with a power output of 65 hp has a thermal efficiency of 24%. Determine the
fuel consumption rate of this car if the fuel has a heating value of 19,000 Btu/lb m.
A. 36.28 lbm/hr B. 37.28 lbm/hr C. 37.28 lbm/hr D. 35.30 lbm/hr
Solution:
̇
̇

̇
̇ ( )

̇ ̇ ( )

139
459. The thermal efficiency of a Carnot cycle operating between 170°C and 620°C is closest to:
A. 44% B. 50% C. 63% D. 73%
Solution:

460. Compute the humidity ratio of air at 70% relative humidity and 25°C when the barometric
pressure is 101.325 kPa. From the steam tables: Psat @ 34 °C = 3.169 kPa.
A. 0.014 kg water vapor/kg dry air C. 1.4 kg water vapor/kg dry air
B. 0.14 kg water vapor/kgdry air D. 0.0014 kg water vapor/kg dry air
Solution:

( )
( )

461. A pressure gauge registers 50 psig in a region where the barometer reads 14.8 psia. Find
the absolute pressure in kPa.
A. 666.66 kPa B. 556.66 kPa C. 446.66 kPa D. 336.66 kPa
Solution:

462. Consider 1 kg of air at 32°C that expanded by a reversible polytropic process with n = 1.25
until the pressure is halved. Determine the heat transfer. Specific heat at constant volume for
air is 0.1786 kJ/kg-K.
A. 17.02 kJ heat rejected C. 17.02 kJ heat added
B. 7.05 kJ heat rejected D. 7.05 kJ heat added
Solution:

( ) ( )

( ) ( ) - -
( ) (- )( )
-

140
463. A Carnot cycle uses nitrogen (k = 1.399) as the working substance. The heat supplied is 54
kJ and the adiabatic expansion ratio is 10. Determine the heat rejected.
A. 10 kJ B. 32.4 kJ C. 21.6 kJ D. 54 kJ
Solution:

( )
( )

464. A tank contains 20 kg of air at 200 kPa (gage) and 23°C. During the heating process the
temperature of air rises. For safety purposes a technician installed a relief-type valve so that
pressure of air inside the tank never exceeds 260 kPa (gage). At what air temperature the relief
valve will start releasing air?
A. 112°C B. 92°C C. 82°C D. 102°C
Solution:

( )
( )
( ) ( )

465. An air compressor takes in 9 kg/min of air at 98 kPa; υ 1 = 0.125 m3/kg and discharges it at
680 kPa; υ2 = 0.03 m3/kg. The increase of internal energy is 93 kJ/kg and the work done on air is
163 kJ/kg; the change in potential and kinetic energy are neglected. How much heat is
transferred per kg of air?
A. 264.15 kJ/kg B. 61.85 kJ/kg C. 288.65 kJ/kg D. 78.15 kJ/kg
Solution:
( )
( )
( ) (- )
- ( )

466. During a reversible process, there are abstracted 317 kJ/s from 1.134 kg/s of a certain gas
while the temperature remains constant at 26.7°C. For this gas c p = 2.232 and cv = 1.713 kJ/kg-K.
The initial pressure is 586 kPa. Determine the final volume flow rate.
A. 0.301 m3/s B. 0.03 m3/s C. 0.5 m3/s D. 0.05 m3/s
Solution:

141
-

̇ ̇ - ( )
̇

467. Flow of water in a pipe has a velocity at 10 meters per second. Determine the velocity
head of the water.
A. 50.1 meters B. 5.10 meters C. 8.20 meters D. 100 meters
Solution:

( )

468. A Diesel cycle has a cut-off ratio of 2.20 and a compression ratio of 10. Find the cycle
efficiency.
A. 55.10% B. 59.735% C. 52.23% D. 62.37%
Solution:
[ ( )
] [ ( )
]

469. A diesel cycle has an initial temperature of 27°C. If the cut-off ratio is 2.50 and
compression ratio is 12, find the maximum cycle temperature.
A. 1634.4°C B. 1753.44°C C. 2010.3°C D. 1983.4°C
Solution:
( )

( )

470. A diesel cycle, which takes in air at 1 bar and 26°C, has a compression ratio of 19. Calculate
the operating clearance in percent.
A. 8.08 B. 8.56 C. 7.52 D. 5.55
Solution:

142
471. An Otto cycle has an initial pressure of 100 kPa and has a pressure of 400 kPa after
adiabatic compression. Find the cycle efficiency.
A. 32.70% B. 34.70% C. 36.70% D. 38.70%
Solution:

( ) ( )

472. An Otto cycle has a clearance of 8% and heat added of 1000 kJ. Find the heat rejected.
A. 564 kJ B. 353 kJ C. 709 kJ D. 867 kJ
Solution:

( )

473. An Otto cycle has a heat rejected of 300 kJ and work of 700 kJ. Find the cycle efficiency.
A. 56% B. 60% C. 70% D. 50%
Solution:

474. An Otto cycle has a pressure ratio of 7. What is the cycle compression ratio?
A. 5.18 B. 6.34 C. 7.34 D. 4.01
Solution:

( )

475. Find the power of a rotating shaft which develops a torque of 188 N-m at 1350 rpm.
A. 101.54 hp B. 53.63 hp C. 63.35 hp D. 35.63 hp
Solution:
( )( ) ( )

143
476. Determine the pressure exerted on a diver at 30 m below the free surface of the sea.
Assume a barometric pressure of 101 kPa and the specific gravity of sea water is 1.03.
A. 404 kPa B. 410 kPa C. 420 kPa D. 430 kPa
Solution:
( )( )

477. An air compressor has an inlet enthalpy of 35 Btu/lb and an exit enthalpy of 70 Btu/lb. The
mass flow rate of air is 3 lb/s. If the heat loss is 466.62 Btu/min, find the work input to the
compressor.
A. 139.59 hp B. 149.59 hp C. 159.59 hp D. 169.59 hp
Solution:
̇ ̇
- ( )( ) ̇

̇ - ( ) -

478. An automobile tire is inflated to 35 psig at 54°F. After being driven, the temperature rises
to 80°F. Determine the final gage pressure assuming that the tire is inflexible.
A. 36.51 psig B. 37.51 psig C. 38.51 psig D. 39.51 psig
Solution:

( ) ( )( )

479. A condenser vacuum gauge reads 600 mm Hg when the barometer reads 760 mm Hg.
What is the absolute condenser pressure in bar?
A. 0.0213 B. 0.061 C. 0.213 D. 0.610
Solution:
( ) ( )

480. Water flows in a pipe at the rate of 10 kg/s. If the velocity of flow is 10 m/s, find the pipe
diameter.
A. 30.23 mm B. 35.68 mm C. 38.39 mm D. 42.39 mm
Solution:
̇
144
( )( )

481. What is the resulting pressure when one kilogram of air at 104 kPa and 98°C is heated at
constant volume to 450°C?
A. 202.67 kPa B. 194.67 kPa C. 186.53 kPa D. 198.65 kPa
Solution:

( )

482. Determine the degrees of superheat of steam at 101.325 kPa and 170°C.
A. 50°C B. 70°C C. 60°C D. 80°C
Solution:

483. Calculate the approximate enthalpy of water at 90°C.


A. 366.83 kJ/kg B. 376.83 kJ/kg C. 386.83 kJ/kg D. 396.83 kJ/kg
Solution:
( ) ( )

484. A Carnot cycle operates between 30°C and 350°C. Find the cycle efficiency.
A. 51.36% B. 63.45% C. 45.37% D. 76.45%
Solution:

485. A Carnot cycle has a maximum temperature of 550°F and minimum temperature of 100°F.
If the heat added is 4200 Btu/min, find the horsepower output of the engine.
A. 34.53 B. 40.56 C. 44.13 D. 65.40
Solution:

̇ ̇ ( ) ( )

486. A Carnot cycle has a sink temperature of 100°F and a cycle efficiency of 70%. Find the
temperature of the heat souce.

145
A. 1306.70°F B. 1406.70°F C. 1506.70°F D. 1606.70°F
Solution:

487. The quality of steam is 20%. This means that:


A. mass of liquid is 20%, mass of vapor is 80%
B. mass of liquid is 20%, mass of vapor is 0%
C. mass of liquid is 80%, mass of vapor is 20%
D. none of the above
Solution:

488. Fifty kilograms of cooling water per second enter the condenser at 25°C and leaves at
50°C. Find the heat carried away by water.
A. 1234.45 kW B. 5233.75 kW C. 2340.53 kW D. 3140.25 kW
Solution:
̇ ̇ ( )( )

489. Ten kilograms per second of steam enter the turbine with an enthalpy of 3200 kJ/kg and
enter the condenser with an enthalpy of 2500 kJ/kg in a Rankine cycle. If the turbine efficiency
is 80% and the generator efficiency is 90%, determine the power plant output.
A. 4320 kW B. 3213 kW C. 4056 kW D. 5040 kW
Solution:
̇ ( )( )( )

490. Determine the quality of steam in a vessel containing 2 kg of saturated vapor and 8 kg of
saturated liquid.
A. 100% B. 20% C. 80% D. 60%
Solution:

491. The condenser of a reheat power plant rejects heat at the rate of 600 kW. The mass flow
rate of cooling water is 5 kg/s and the inlet cooling water temperature is 35°C. Calculate the
condenser cooling water exit temperature.
146
A. 43.45°C B. 53.45°C C. 63.66°C D. 74.34°C
Solution:
( )( )

492. A heat engine has a thermal efficiency of 50%. How much power does the engine produce
when heat is transferred at a rate of 10 9 kJ/hr?
A. 50 MW B. 75 MW C. 139 MW D. 147 MW
Solution:
̇ ( )
̇ ( )

493. One kilogram of air is compressed adiabatically and in a steady-flow manner. The
compression efficiency is 80% and the work done on the air is 265 kJ/kg. Compute the heat.
A. 212 kJ/kg B. 100 kJ/kg C. 0 kJ/kg D. 331.25 kJ/kg
Solution:
( )

494. Three-hundred kilojoules of heat flow by conduction from the outside to the inside of a
cold storage in one hour. If the temperature and all other conditions are the same, what is the
heat flowing through the cold storage room in two hours?
A. 600 kJ B. 900 kJ C. 300 kJ D. 1,200 kJ
Solution:
( )

495. Determine the density of air at 760 mmHg absolute and 22°C?
A. 1.014 kg/m3 B. 1.316 kg/m3 C. 1.197 kg/m3 D. 1.266 kg/m3
Solution:

( )( )
-

496. A refrigerating machine that is classified as a one-ton machine has the capacity to produce
a cooling effect of?
A. 3.516 kW B. 12,000 Btu/hr C. 211 kJ/min D. All of the above
Solution:

147
• Question 1
0 out of 3.3 points
Which of the following is considered as comfort condition in air conditioning?
Selected Answer: [None Given]
Answers: a.
30 C DBT, 80% RH
b.
40 C DBT, 80% RH
c.
30 C DBT, 60% RH
d.
20 C DBT, 60% RH
• Question 2
0 out of 3.3 points
Combustion is the rapid burning of fuel and oxygen resulting in:
Selected Answer: [None Given]
Answers: a.
Release of steam
b.
Nitrogen and oxygen
c.
Oxidation
d.
Release of heat
• Question 3
0 out of 3.3 points
During combustion process, only limited air is supplied and not enough to
supply two molecule of oxygen per one molecule of carbon, then the product is:
Selected Answer: [None Given]
Answers: a.
carbon monoxide
b.
carbon dioxide
c.
hydration
d.
carbonic acid
• Question 4
0 out of 3.3 points
Which of the following is not a characteristics of Freon- 12?
Selected Answer: [None Given]
Answers: a.
Corrosive
b.
Non-inflammable
c.
Odorless
d.
Boiling point -21 F
• Question 5
0 out of 3.3 points
An instrument used in measuring air velocity by velocity-pressure method.
Selected Answer: [None Given]
Answers: a.
Flowmeter
b.
Aneroid barometer
c.
Pitot tube
d.
Anemometer
• Question 6
0 out of 3.3 points
When the air is saturated the wet-bulb depression is:
Selected Answer: [None Given]
Answers: a.
indefinite
b.
100%
c.
zero
d.
unity
• Question 7
0 out of 3.3 points
The purpose of the expansion valve by-pass is
to
Selected [None Given]
Answer:
Answers: a.
Increase the capacity of the evaporator
b.
By-pass the compressor
c.
Increase the efficiency of the plant
d.
Controls the refrigerant to the evaporator in case the automatic
valves fail
• Question 8
0 out of 3.3 points
If a gas posses internal energy, then it is due to its:
Selected Answer: [None Given]
Answers: a.
pressure and volume values
b.
height from a certain datum level
c.
velocity
d.
molecular motion
• Question 9
0 out of 3.3 points
A type of polymer used for detergents, milk bottles, oil containers and toys
Selected Answer: [None Given]
Answers: a.
Polyvinyl chloride(PVC)
b.
Polypropylene(PP)
c.
Polystyrene(PS)
d.
High density polyethelene (HDPE)
• Question 10
0 out of 3.3 points
Which of the following materials is suitable for tubing in refrigeration application
where refrigerant ammonia is employed
Selected Answer: [None Given]
Answers: a.
Plastic
b.
Steel
c.
Brass
d.
Copper
• Question 11
0 out of 3.3 points
Which of the following is a characteristic of an impulse turbine?
Selected Answer: [None Given]
Answers: a.
Steam reversing direction
b.
Steam striking blades at zero angle
c.
Steam striking blades on angle
d.
No steam reaction to velocity
• Question 12
0 out of 3.3 points
R-22 is
Selected Answer: [None Given]
Answers: a.
Trichlorodifluoromethane
b.
Monochlorodifluoromethane
c.
Dichlorodifluoromethane
d.
Methyl chloride
• Question 13
0 out of 3.3 points
A pump with capacity Q1 and head H1 is connected in series with another pump
with capacity Q2 and head H2. What is the head developed if Q2 is less than
Q1?
Selected Answer: [None Given]
Answers: a.
H2 + H1
b.
H2H1
c.
H2/H1
d.
2 H2H1
• Question 14
0 out of 3.3 points
Which of the following dehumidifier is often used to reheat the air after moisture
is removed?
Selected Answer: [None Given]
Answers: a.
Condenser
b.
Compressor
c.
Evaporator
d.
Chiller
• Question 15
0 out of 3.3 points
Zinc rods are found in the:
Selected Answer: [None Given]
Answers: a.
Refrigerant side of condenser
b.
Compressor crankcase
c.
Salt-water side of condenser
d.
Evaporator coils
• Question 16
0 out of 3.3 points
Modern stacks are seldom built higher than _______.
Selected Answer: [None Given]
Answers: a.
60 m
b.
40 m
c.
50 m
d.
30 m
• Question 17
0 out of 3.3 points
What is the heat that is removed from the space to be cooled, which is the same
as the heat absorbed by the cooling coils?
Selected Answer: [None Given]
Answers: a.
heating capacity
b.
enthalpy
c.
work compression
d.
refrigerating effect
• Question 18
0 out of 3.3 points
The discharge pressure of the compressor should be:
Selected [None Given]
Answer:
Answers: a.
The pressure which corresponds to a temperature from 5F to 15F
higher than the condenser discharge
b.
The pressure which corresponds to a temperature from 5F to 15F
below that of the condenser discharge
c.
The pressure which corresponds to a temperature equal to that of
the condenser discharge
d.
None of the other choices
• Question 19
0 out of 3.3 points
A rule which states that any solid waste mixed with harazdous waste becomes
hazardous
Selected Answer: [None Given]
Answers: a.
The mixture rule
b.
The environmental rule
c.
The hazard rule
d.
The derived from rule
• Question 20
0 out of 3.3 points
In a window air conditioning unit which of the following is usually done by the
owner?
Selected [None Given]
Answer:
Answers: a.
All of the other choices
b.
Semi-annual cleaning or replacement of filters
c.
Inspection of fan motors and lubricates them
d.
Annual cleaning of the evaporator, condenser, fan blades, fan motor,
compressor and casing
• Question 21
0 out of 3.3 points
The reduction of nuclear radiation intensity (called attenuation) is accomplished
by:
Selected Answer: [None Given]
Answers: a.
Shielding
b.
Antimatter
c.
Matter
d.
Neurons
• Question 22
0 out of 3.3 points
When vapor compression takes place on one side of the piston and one during
one revolution of the crankshaft, then the compressor is:
Selected Answer: [None Given]
Answers: a.
single-acting
b.
four-cycle
c.
double-acting
d.
two-revolution
• Question 23
0 out of 3.3 points
If there were a 15 F to 20 F temperature differential between the temperature
corresponding to the pressure at the compressor discharge and the temperature
at the condenser outlet, it would probably indicate the need for __________.
Selected Answer: [None Given]
Answers: a.
Less circulating
b.
More refrigerant
c.
More circulating water
d.
Purging the system
• Question 24
0 out of 3.3 points
How is the CO2 system is purged?
Selected [None Given]
Answer:
Answers: a.
The CO2 will come out of the purge valve in liquid form
b.
Through the king valve
c.
When CO2 comes out of the purge valve, frost will form on a
piece of metal held near the outlet
d.
Pumped out with a suction pump
• Question 25
0 out of 3.3 points
The disadvantage of a co2 system over an ammonia system is the fact that
Selected [None Given]
Answer:
Answers: a.
The CO2 system operates at a much higher pressure
b.
The pipes and fittings of a co2 system must be of the high-
pressure type
c.
All of the other choices
d.
The CO2 system requires a larger prim e mover
• Question 26
0 out of 3.3 points
Excess fuel oil in the fuel oil system returns to the:
Selected Answer: [None Given]
Answers: a.
Suction line
b.
Combustion chamber
c.
Burner
d.
Fuel oil tank
• Question 27
0 out of 3.3 points
What is the least number of compressors a multistage system that will use?
Selected Answer: [None Given]
Answers: a.
one
b.
four
c.
three
d.
two
• Question 28
0 out of 3.3 points
Which of the following is a type of evaporator?
Selected Answer: [None Given]
Answers: a.
sell-and-tube-water cooler
b.
oil lantern rings
c.
polyphase motors
d.
shell-and-tube
• Question 29
0 out of 3.3 points
It is a form of oxygen photochemically produced in nature.
Selected Answer: [None Given]
Answers: a.
DO2
b.
Ozone
c.
Oxidizing Agent
d.
Oxidation
• Question 30
0 out of 3.3 points
Which of the following statements is correct? The relative humidity of an air
water vapor mixture.
Selected [None Given]
Answer:
Answers: a.
is the ratio of partial pressure of water vapor to the saturation
pressure at the mixture temperature.
b.
indicated the mass of water vapor in the mixture.
c.
is the ratio of the partial pressure of water vapor to the partial
pressure of the air.
d.
is equal to the mole fraction of water vapor in the mixture at the
mixture temperature.
IPE 3Q1920 (SOLVING)
• Question 1
0 out of 1 points
Water ( Specific heat, C = 4.205 kJ/kg-K ) is being heated by 1,878-W heater. What is the rate of change in
temperature of 1 kg of water in K/s.

Selected Answer: [None Given]


Correct Answer: 0.4466 ± 1%
• Question 2
0 out of 1 points
A centrifugal pump has a constant speed of 1,644 rpm and has a head of 89 m. What is the effect on the
head of the pump if the impeller diameter is reduced from 260 mm to 234 mm

Selected Answer: [None Given]


Correct Answer: 72.09 ± 0.1
• Question 3
1 out of 1 points
A rectangular duct has dimensions 0.3 m by 1.1 m. Determine the equivalent diameter of the duct.

Selected Answer: 0.47


Correct Answer: 0.471 ± 1%
• Question 4
0 out of 1 points
Find the heat transfer required to convert 6 kg of water at a pressure of 20 bar and a temperature of 21°C
into steam dryness of fraction 92 at the same pressure.

Properties of steam:

P = 20 bar

hf = 909 kJ/kg

hfg = 1890 kJ/kg

Note: for water at 21°C, h = 88 kJ/kg

Selected Answer: [None Given]


Correct Answer: 15,358.80 ± 0.1
• Question 5
0 out of 1 points
The density of air at 35 ºC and 101 kPa is 1.07 kg/m3. The humidity ratio is:
Selected Answer: 0.042
Correct Answer: 0.0422
• Question 6
0 out of 1 points
A vapor compression refrigeration system is designed to have a capacity of 110 tons. Its actual COP is 5.85
and 32% of the power supplied to the compressor is lost in the form of friction and cylinder cooling losses.
What is the motor power in kW?

Selected Answer: [None Given]


Correct Answer: 97.22 ± 0.1
• Question 7
1 out of 1 points
What horsepower is supplied to air moving at 20 ft per min through a 1 by 4 ft duct under a pressure of 4
in water gage?

Selected Answer: 0.05


Correct Answer: 0.0504 ± 1%
• Question 8
1 out of 1 points
If the latent and sensible heat loads are 34 kW and 76 kW respectively, what is the sensible heat ratio?

Selected Answer: 0.69


Correct Answer: 0.69 ± 0.01
• Question 9
0 out of 1 points
The evaporator and condenser pressures are 152 kPa and 9 atm respectively. The intercooler pressure is
approximately

Selected Answer: [None Given]


Correct Answer: 372.31 ± 0.1
• Question 10
0 out of 1 points
Calculate the change in enthalpy as 1 kg of nitrogen is heated from 1,076 K to 1,495 K, assuming the
nitrogen is an ideal gas a constant pressure. The temperature dependent specific heat of nitrogen is C =
36.36 - 515.1 T^1.5 + 1,070.25 T^-2 - 818.17 T^-3 where C is in kJ/kg-mol, and T is in K.

Selected Answer: [None Given]


Correct Answer: 543.94 ± 0.1
• Question 11
1 out of 1 points
Calculate the specific volume of an air-vapor mixture in cubic meters per kilogram of dry air when the
following conditions were given: t=28ºC, w=0.017 kg/kg, P=84 kPa

Selected Answer: 1.06


Correct Answer: 1.06 ± 1%
• Question 12
1 out of 1 points
Water enters the condenser at 28°C and leaves at 60°C. What is the temperature difference in °F ?

Selected Answer: 57.6


Correct Answer: 57.60 ± 0.1
• Question 13
0 out of 1 points
A cylinder has a bore diameter of 15 cm, stroke of 46 cm, and volumetric compression = 2,040 cm3, find
the volumetric efficiency (in %) of the engine.

Selected Answer: [None Given]


Correct Answer: 25.10 ± 0.1
• Question 14
0 out of 1 points
An ice plant produces 24 tons of ice per day at -16 ºC from the water at 24 ºC. If the losses are 11% of the
freezing and chilling load, calculate the refrigeration capacity of the plant in ton of refrigeration.

Selected Answer: 33.59


Correct Answer: 37.30 ± 0.1
• Question 15
0 out of 1 points
If 14 lbs of water evaporated at atmospheric pressure until a volume of 291.1 ft is occupied, how much
work is done ?

Selected Answer: [None Given]


Correct Answer: 616,200.48 ± 1%
• Question 16
0 out of 1 points
One foot of water is equivalent to how many feet of air
Selected Answer: c.
733
Answers: a.
833
b.
211
c.
733
d.
633
• Question 17
1 out of 1 points
4000 li/hr of distillates are to be cooled from 21C to -12C and 12% of wax by weight
is separated out at 15C. The sp heat of oil is 2 kJ/kg-C and SG is 0.87. The specific
heat of the wax is 2.5 and latent heat is 290 kJ/kg. Allow 10% for the losses, find the
refrigerating capacity of the system.
Selected Answer: c.
31.08 TOR
Answers: a.
20 TOR
b.
40 TOR
c.
31.08 TOR
d.
51.08 TOR
• Question 18
1 out of 1 points

A volume of 450 cm of air is measured at a pressure of 740 mm Hg absolute and a


temperature of 20°C. What is the volume in cm at 760 mm Hg absolute and 0°C ?
Selected Answer: c.
408.25
Answers: a.
620.76
b.
375.85
c.
408.25
d.
516.12
• Question 19
1 out of 1 points
A condenser vacuum gauge reads 715 mm Hg when the barometer stands at 757
mm Hg. What is the absolute pressure in the condenser in kPa.
Selected Answer: b.
5.60 kPa
Answers: a.
95.33 kPa
b.
5.60 kPa
c.
196.25 kPa
d.
100.92 kPa
• Question 20
1 out of 1 points
An electric heater is to heat 11 kg of oil per min. from 4.5°C to 65.5°C. Specific heat
of the oil is 2.1 J/gm-°C. How many watts should this heater consume ?
Selected Answer: a.
23 442 W
Answers: a.
23 442 W
b.
33 448 W
c.
20 448 W
d.
20 400 W
• Question 21
1 out of 1 points

Steam at a pressure of 9 bar ( h = 743 kJ/kg, h = 2031 kJ/kg ) is generated in an


exhaust gas boiler from feedwater at 80°C ( h = 334.9 kJ/kg ). If the dryness fraction
of the steam is 0.96, determine the heat transfer per kilogram of steam.
Selected Answer: d.
2357.86
Answers: a.
3357.86
b.
5357.86
c.
1357.86
d.
2357.86
• Question 22
0 out of 1 points
A vapor compression refrigeration system is designed to have a capacity of 100 tons
of refrigeration. It produces chilled water from 23 C to 2 C. Its actual coefficient of
performance is 5.96 and 35 % of the power supplied to the compressor is lost in the
form of friction and cylinder cooling losses. Determine the condenser cooling water
required in kg/s for temperature rise of 10 C.
Selected Answer: a.
10.33 kg/s
Answers: a.
10.33 kg/s
b.
8.33 kg/s
c.
7.33 kg/s
d.
9.33 kg/s
• Question 23
1 out of 1 points

Wet saturated steam at 17 bar ( h = 872 kJ/kg, h = 1293 kJ/kg ) dryness 0.97 is
produced from feedwater at 85°C ( h = 335.9 kJ/kg ). Find the heat energy supplied
per kg.
Selected Answer: a.
2381.41 kJ/kg
Answers: a.
2381.41 kJ/kg
b.
1381.41 kJ/kg
c.
3381.41 kJ/kg
d.
4381.41 kJ/kg
• Question 24
1 out of 1 points

Three pounds mass of air are contained at 25 psia and 100°F. Given that R =
53.35 ft-lb /lb . What is the volume of the container ?
Selected Answer: a.
24.9 ft
Answers: a.
24.9 ft
b.
15 ft
c.
20.6 ft
d.
10.7 ft
• Question 25
0 out of 1 points
Copra enter dryer containing 40% water and 60% solid and leaves with 5% water and
95% solid. Find the weight of water removed per pound of final product
Selected Answer: a.
1.015 lb/lb
Answers: a.
1.015 lb/lb
b.
0.583 lb/lb
c.
1.3753 lb/lb
d.
0.475 lb/lb
• Question 26
1 out of 1 points
The thermal efficiency of a particular engine operating on an ideal cycle is 35%.
Calculate the heat supplied per 1200 watt-hr of work developed in kJ.
Selected Answer: a.
12 343
Answers: a.
12 343
b.
11 108
c.
10 216
d.
14 218
• Question 27
1 out of 1 points
An auditorium is to be maintained at a temperature of 26ºC dry bulb and 50% RH. Air
is to be supplied at a temperature not lower than 15ºC dry bulb. The sensible heat
gain is 110 kW and the latent gain is 37.5 kW. Take ventilating air as 25% by weight
of the air from the room, and is at 35ºC dry bulb and 60% RH. Determine refrigerating
capacity in tons.
Selected Answer: c.
63.28
Answers: a.
54.23
b.
76.34
c.
63.28
d.
43.45
• Question 28
1 out of 1 points
A duct 0.40m high and 0.80 m wide suspended from the ceiling in a corridor, makes a
right angle turn in the horizontal plane. The inner radius is 0.2m and the outer radius
is 1.0 m measured from the same center. The velocity of air in the duct is 10 m/s.
Compute the pressure frop in this elbow. Assume f=0.3, density = 1.204 kg/m3 and L
= 10m
Selected Answer: c.
341 Pa
Answers: a.
144 Pa
b.
441 Pa
c.
341 Pa
d.
143 Pa
• Question 29
1 out of 1 points
A stream of outdoor air is mixed with a steam of return air in an air conditioning
system that operates at 101 kPa pressure. The flow rate of outdoor system air is 2
kg/s and its condition is 35ºC dry-bulb temperature and 25ºC wet-bulb temperature.
The flow rate of return is 3 kg/s and its condition is 24ºC and 50% relative humidity.
Determine the enthalpy of the mixture.
Selected Answer: c.
59.16 kJ/kg
Answers: a.
91.56 kJ/kg
b.
51.69 kJ/kg
c.
59.16 kJ/kg
d.
69.15 kJ/kg
• Question 30
1 out of 1 points

Helium ( R = 0.4968 Btu/lb °R ) is compressed isothermally from 14.7 psia and 68°F.
The compression ratio is 4. Calculate the work done by the gas.
Selected Answer: b.

- 364 Btu/lb
Answers: a.
- 145 Btu/lb
b.

- 364 Btu/lb
c.

- 46.7 Btu/lb
d.

- 187 Btu/lb
• Question 31
1 out of 1 points
Steam enters the superheats of a boiler at a pressure of 25 bar and dryness of 0.98
and leaves at the same pressure at a temperature of 370°C. Calculate the heat
energy supplied in the superheaters.
Properties of steam: At 25 bar and 370°C ( h = 3171.8 kJ/kg ) and at 25 bar and 0.98 dryness ( h f = 962.11
kJ/kg, h fg = 1841.01 kJ/kg ).
Selected Answer: b.
405.51
Answers: a.
407.46
b.
405.51
c.
408.57
d.
406.54
• Question 32
1 out of 1 points
A belt driven compressor is used in the refrigeration system that will cool 10li/s of
water from 13C to 1 C. The belt efficiency is 98%, motor efficiency is 85% and the
input of the compressor is 0.7 kW per TOR. Find the actual COP if over-all efficiency
is 65%
Selected Answer: a.
6.44
Answers: a.
6.44
b.
4.44
c.
3.44
d.
5.44
• Question 33
0 out of 1 points
An air compressor is to compress 8.5m3/min from 98.56 kPa to 985.6 kPa. Assuming
ideal conditions, and with n = 1.3, what will be the saving in work due to two staging?
Selected Answer: [None Given]
Correct Answer:
5.6kW

• Question 34
1 out of 1 points
Steam at the rate of 500 kg/hr is produced by a steady flow system boiler from
feedwater entering at 40°C. Find the rate at which heat is transformed in kCal/hr if the
enthalpy of steam is 600 kCal/kg and of steam 50 kCal/kg.
Selected Answer: b.
275,000 kCal/hr
Answers: a.
175,000 kCal/hr
b.
275,000 kCal/hr
c.
375,000 kCal/hr
d.
475,000 kCal/hr
• Question 35
1 out of 1 points

Twenty grams of of Oxygen gas (O ) are compressed at a constant temperature of


30°C to 5% of their original volume. What work is done on the system? Use R of air,
0.0619 Cal/gm-K.
Selected Answer: d.
1124 Cal
Answers: a.
924 Cal
b.
944 Cal
c.
824 Cal
d.
1124 Cal
• Question 36
1 out of 1 points
A certain sample of air has a temperature of 70 F (partial pressure of 0.36 psia) and a
dew point temperature of 50 F. The partial pressure of the water is vapor
corresponding to a 50 F dew point temperature is 0.178 psia. Determine the relative
humidity RH.
Selected Answer: a.
49.44 %
Answers: a.
49.44 %
b.
69.44 %
c.
39.44 %
d.
59.44 %
• Question 37
1 out of 1 points
Oil flows full bore at a velocity of 2 m/s through a nest of 16 tubes in a single pass
cooler. The internal diameter of the tubes is 30 mm and the density of the oil is 0.85
g/li. Find the volume flow in liters per second.
Selected Answer: a.
23
Answers: a.
23
b.
20
c.
21
d.
22
• Question 38
1 out of 1 points
A refrigerating machine is driven by a motor of output power 2.25 kW and 2.5 tons of
ice at -7C per day from water at 18C. Calculate the COP of the machine and express
its capacity in terms of tons of ice per 24 hrs from and at 0C, taking the latent heat of
ice = 335 kJ/kg-K
Selected Answer: d.
5.476, 3.17 tons/day
Answers: a.
5.476, 4.17 tons/day
b.
4.476, 3.17 tons/ day
c.
4.476, 4.17 tons/day
d.
5.476, 3.17 tons/day
• Question 39
1 out of 1 points
Wet saturated steam at 17 bar ( h = 872 kJ/kg, h = 1293 kJ/kg ) dryness 0.97 is produced from feedwater
at 85°C ( h = 335.9 kJ/kg ). Find the heat energy supplied per kg.

Selected Answer: a.
2381.41 kJ/kg
Answers: a.
2381.41 kJ/kg
b.
4381.41 kJ/kg
c.
1381.41 kJ/kg
d.
3381.41 kJ/kg
• Question 40
1 out of 1 points
Find the percent slip of a reciprocating pump if the volumetric efficiency is 96 percent.
Selected Answer: b.
5%
Answers: a.
95%
b.
5%
c.
75%
d.
90%
IPE 3Q1920 (TERMS)
• Question 1
1 out of 1 points
A compressor used for large refrigeration system it is not positive displacement but it is similar
to a blower:
Selected Answer: c.

Centrifugal compressor

Answers: a.

Semi-hermetic compressor

b.

Hermetic compressor

c.

Centrifugal compressor

d.

Cylinder compressor

• Question 2
1 out of 1 points
Heat transfer processes which include a change of phase of a fluid are considered _____.

Selected Answer: d.

Convection

Answers: a.

Conduction

b.

Thermal radiation

c.

Radiation

d.

Convection

• Question 3
1 out of 1 points
In heat exchanger design, one transfer unit implies:

Selected b.
Answer:
Condition when the change in temperature of one steam is numerically equal
to the average driving force

Answers: a.

The section of heat exchanger where heat transfer surface area has been one
square meter

b.

Condition when the change in temperature of one steam is numerically equal


to the average driving force

c.

One fluid which is exchanging with another fluid of the same chemical
composition

d.

The section of heat exchanger which will cause temperature drop of one
degree centigrade

• Question 4
1 out of 1 points
Which of the following is another name for the liquid valve?

Selected Answer: d.

King valve

Answers: a.

Master valve

b.

Shutoff valve

c.

Freon valve

d.

King valve

• Question 5
0 out of 1 points
Is a type of refrigerant which does not contain carbon.

Selected Answer: c.

Ammonia-type

Answers: a.

Inorganics

b.

Carbon Dioxide type-

c.

Ammonia-type

d.

None of the choices

• Question 6
1 out of 1 points
An azeotropic mixture of R-22 and R-115, a popular refrigerant for low temperature
refrigeration systems:
Selected Answer: c.

R-502

Answers: a.

R-114

b.

R-14

c.

R-502

d.

R-12

• Question 7
1 out of 1 points
Monochlorodifluoromethane is a refrigerant known as:
Selected Answer: c.

R-22

Answers: a.

R-12

b.

R-13

c.

R-22

d.

None of the choices

• Question 8
1 out of 1 points
The intake pipe to a hydraulic turbine from a dam is:

Selected Answer: d.

Penstock

Answers: a.

Surge tank

b.

Spiral casing

c.

Tailrace

d.

Penstock

• Question 9
1 out of 1 points
When the air is saturated the wet-bulb depression is:

Selected Answer: d.

zero
Answers: a.

indefinite

b.

unity

c.

100%

d.

zero

• Question 10
1 out of 1 points
Which of the following measures the density of salt in water?

Selected Answer: a.

Salimeter

Answers: a.

Salimeter

b.

Pitot tube

c.

Calorimeter

d.

Hydrometer

• Question 11
1 out of 1 points
Ammonia leaks in the condenser can be detected by:

Selected Answer: c.

Applying litmus paper to the circulating water discharge

Answers: a.

Applying a soapy solution to the condenser heads and looking for bubbles
b.

Smelling the discharge water

c.

Applying litmus paper to the circulating water discharge

d.

Adding oil of peppermint to the system and tracing the smell

• Question 12
1 out of 1 points
Which of the following is not a characteristics of Freon- 12?

Selected Answer: a.

Corrosive

Answers: a.

Corrosive

b.

Boiling point -21 F

c.

Non-inflammable

d.

Odorless

• Question 13
1 out of 1 points
Transfer of heat from one fluid to another through dividing wall:
Selected Answer: d.

Conduction

Answers: a.

Radiation

b.

Heat transfer
c.

Convection

d.

Conduction

• Question 14
1 out of 1 points
Palladium Chloride may be used to measure the presence of:

Selected Answer: a.

CO

Answers: a.

CO

b.

ammonia

c.

refrigerant

d.

vapor

• Question 15
1 out of 1 points
The temperature bulb of the solenoid valve is attached to the ___________.

Selected Answer: b.

Wall of the icebox

Answers: a.

Evaporator coil outlet

b.

Wall of the icebox

c.

Evaporator coil inlet


d.

Icebox coil

• Question 16
1 out of 1 points
The ff are all examples of indirect (secondary) miscellaneous methods to measure flow except:

Selected Answer: a.

positive displacement meters


Answers: a.

positive displacement meters


b.

hot-wire anemometers
c.

turbine and proppeler meters


d.

magnetic flow meters


• Question 17
1 out of 1 points
How do you classify a body that has an emissivity factor of 0.7?

Selected Answer: a.

Gray body

Answers: a.

Gray body

b.

Theoretical body

c.

Black body

d.

White body

• Question 18
1 out of 1 points
The amount of heat removed from one ton of water at freezing temperature to ice at the
same temperature in one day is

Selected Answer: c.

one ton of refrigeration

Answers: a.

one tonne of refrigeration

b.

all of the other choices

c.

one ton of refrigeration

d.

one ton of refrigerant

• Question 19
1 out of 1 points
In an air conditioning unit, the thermostat fails. The unit did not start. How do you test the
operation of the thermostat?

Selected a.
Answer:
Cover the air outlet and air inlet with a cloth. The air will recirculate into the unit
and the temperature will quickly drop to the cut out temperature.

Answers: a.

Cover the air outlet and air inlet with a cloth. The air will recirculate into the unit
and the temperature will quickly drop to the cut out temperature.

b.

Cover both the inlet and outlet so that the recirculating air has the same
temperature.

c.

Never cover any air passage so that air can freely move and thermostat
functions well.

d.
Cover the outlet and the air will not circulate and the thermostat functions
because no air movement.

• Question 20
1 out of 1 points
The purpose of the oil trap is:

Selected Answer: a.

To remove oil from the refrigerating gas

Answers: a.

To remove oil from the refrigerating gas

b.

To add oil to the compressor

c.

To remove oil from the charging tank

d.

None of above

• Question 21
1 out of 1 points
Zinc rods are found in the:

Selected Answer: d.

Salt water side of the condenser

Answers: a.

Compressor crankcase

b.

Gas side of the condenser

c.

Evaporator area

d.

Salt water side of the condenser

• Question 22
1 out of 1 points
A 1-2 heat exchanger refers to which of the following?

Selected Answer: a.

Single pass on shell side and double pass on the tube side

Answers: a.

Single pass on shell side and double pass on the tube side

b.

Single pass on tube side and double pass on shell side

c.

Single liquid cools two liquids at different temperature

d.

Two tubes of cold fluid pass through one tube of hot fluid.

• Question 23
1 out of 1 points
A “CARRENE” is a type of :

Selected Answer: c.

Refrigerant

Answers: a.

Refrigerant oil

b.

Scale cleaner

c.

Refrigerant

d.

None of the other choices

• Question 24
0 out of 1 points
A substance that is able to absorb liquids or gasses and is used for removing them from a
given region or medium.
Selected Answer: c.

Adsorbent

Answers: a.

Absorbent

b.

Adhesive

c.

Adsorbent

d.

Cohesive

• Question 25
1 out of 1 points
A leaky suction valve can usually detected by:

Selected Answer: a.

Any of the other choices

Answers: a.

Any of the other choices

b.

A higher suction pressure

c.

Closing in on the suction valve having no effect on the suction pressure

d.

A fluctuating suction pressure gauge

• Question 26
0 out of 1 points
Which among the following laws of thermodynamics is the basis for the principle of
refrigeration.

Selected Answer: c.
All of the other choices

Answers: a.

Clausius Statement

b.

Second law

c.

All of the other choices

d.

Kelvin-Plank Statement

• Question 27
1 out of 1 points
What do you call the liquid reaching the compressor through the suction?

Selected Answer: c.

Flooding back

Answers: a.

Recycling

b.

Superheating

c.

Flooding back

d.

Overflowing

• Question 28
1 out of 1 points
If W is the work output of a heat engine and H is the total heat input of an engine, then
determine its thermal efficiency, e.

Selected Answer: d.

e=W/H
Answers: a.

e=WxH

b.

e = I – (W – H)

c.

e=H/W

d.

e=W/H

• Question 29
1 out of 1 points
Based on the PSME Code, what should be the humidity of air to be used for comfort cooling?

Selected Answer: b.

50% - 60% relative humidity

Answers: a.

70% - 75% relative humidity

b.

50% - 60% relative humidity

c.

75% - 80% relative humidity

d.

60% - 70% relative humidity

• Question 30
1 out of 1 points
Compare in refrigerating effect per unit mass of refrigerant circulated for a superheated cycle
that produces useful cooling and a saturated cycle, for the same vaporizing and condensing
temperature.

Selected Answer: d.

greater for a superheated cycle

Answers: a.
greater for a saturated cycle

b.

the same

c.

lower for a superheated cycle

d.

greater for a superheated cycle

• Question 31
1 out of 1 points
The solenoid valve is:

Selected Answer: c.

A temperature-controlled stop valve

Answers: a.

A Freon-controlled check valve

b.

None of the other choices

c.

A temperature-controlled stop valve

d.

A pressure-controlled stop valve

• Question 32
1 out of 1 points
A centrifugal pump in which the pressure is developed by the propelling or lifting action of the
vanes of the impeller on the liquid.
Selected Answer: a.

Axial Flow

Answers: a.

Axial Flow
b.

Mixed Flow

c.

Cyclone

d.

none of the above

• Question 33
1 out of 1 points
A rule which states that any waste derived from the treatment of a hazardous waste remains a
hazardous waste

Selected Answer: b.

The derived from rule

Answers: a.

The hazard rule

b.

The derived from rule

c.

The treatment rule

d.

The mixture rule

• Question 34
1 out of 1 points
If R is the delivery rate of a pump, H is the total pumping head and e is the efficiency of the
pump, then compute the power required to drive the pump.

Selected Answer: c.

(R x H) / e

Answers: a.

1 – (R + H) / e

b.
R/Hxe

c.

(R x H) / e

d.

(h – R) / e

• Question 35
1 out of 1 points
All of the ff express units of concentration except

Selected Answer: b.

Isotropy

Answers: a.

Molarity

b.

Isotropy

c.

Formality

d.

Normality

• Question 36
1 out of 1 points
What is another name of discharge pressure?

Selected Answer: a.

Head pressure

Answers: a.

Head pressure

b.

Absolute pressure
c.

Suction pressure

d.

Condenser pressure

• Question 37
1 out of 1 points
The greatest decrease in the temperature of the refrigerant is at the ______.

Selected Answer: a.

Expansion valve

Answers: a.

Expansion valve

b.

Evaporator

c.

Condenser

d.

Receiver

• Question 38
1 out of 1 points
Which of the following components of the window air conditioning system must be cleaned
annually?
a.

Selected Answer: d.

All of these

Answers: a.

Evaporator and Condenser

b.

Motor and Compressor

c.
Fan Blades and Fan Motor

d.

All of these

• Question 39
1 out of 1 points
It raises pressure of CO2 gas to 250 Psig so that it can be liquefied at –100°F.

Selected Answer: c.

CO2 compressor

Answers: a.

Deodorizer

b.

Trim Cooler

c.

CO2 compressor

d.

Foam Trap

• Question 40
1 out of 1 points
The process of lowering the temperature of a given space and maintaining it for the purpose
of preserving and chilling certain substances

Selected Answer: a.

refrigeration

Answers: a.

refrigeration

b.

chilling

c.

air-conditioning
d.

cooling

• Question 41
1 out of 1 points
An instrument used in measuring air velocity by velocity-pressure method.

Selected Answer: a.

Pitot tube

Answers: a.

Pitot tube

b.

Anemometer

c.

Flowmeter

d.

Aneroid barometer

• Question 42
1 out of 1 points
Which of the following would cause expansion valve failure?

Selected Answer: a.

Any of the other choices

Answers: a.

Any of the other choices

b.

Moisture in the system

c.

Bulb in icebox ruptured

d.

Dirt in the valve


• Question 43
1 out of 1 points
The temperature in the meat and fishbox should be approximately

Selected Answer: b.

10 to 20 deg. F

Answers: a.

0 to -10 deg. F

b.

10 to 20 deg. F

c.

-10 to 10 deg. F

d.

-10 to 0 deg. F

• Question 44
1 out of 1 points
What will happen when the pressure at any point inside a centrifugal pump goes below the
vapor pressure corresponding to the temperature of the liquid?

Selected Answer: d.

cavitation

Answers: a.

turbulent flow

b.

priming

c.

laminar flow

d.

cavitation

• Question 45
1 out of 1 points
Which do you think is the cause of a hot suction line of a refrigerating compressor?

Selected Answer: b.

Lack of refrigerant

Answers: a.

Insufficient condensing cooling water or excess refrigeration

b.

Lack of refrigerant

c.

Insufficient condensing cooling water

d.

Excess refrigeration

• Question 46
1 out of 1 points
A device whose primary function is to meter the flow of refrigerant to the evaporator

Selected Answer: d.

Thermostatic expansion valve

Answers: a.

Sniffer valve

b.

Equalizer

c.

Crossover valve

d.

Thermostatic expansion valve

• Question 47
1 out of 1 points
When excess solute in a solution settles to the bottom of the container, the process is called:

Selected Answer: a.
Precipitation

Answers: a.

Precipitation

b.

Salvation

c.

Deemulsification

d.

Equation

• Question 48
1 out of 1 points
Which of the following heat exchange device used to provide heat transfer between the
exhaust gases and air prior to the entrance of a combustor?

Selected Answer: c.

regenerator

Answers: a.

reheater

b.

economizer

c.

regenerator

d.

condenser

• Question 49
1 out of 1 points
The amount of heat necessary to bring up temperature of a unit mass of a substance through
unit degree is called:

Selected Answer: a.
total heat

Answers: a.

total heat

b.

specific volume

c.

thermal heat

d.

specific heat

• Question 50
0 out of 1 points
Which of the following statements is true for industrial drying.

Selected Answer: d.

Both air heating and product drying are constant wet bulb process

Answers: a.

Air heating is a constant wet bulb process

b.

Drying is a constant specific humidity process

c.

Air heating is a constant specific humidity process

d.

Both air heating and product drying are constant wet bulb process

• Question 51
1 out of 1 points
In convection heat transfer, what mechanism heat transfer where the fluid moves due to the
decrease in its density caused by increase in temperature?

Selected Answer: a.

Natural convection
Answers: a.

Natural convection

b.

Density convection

c.

Forced convection

d.

Radial convection

• Question 52
1 out of 1 points
What do bodies at a temperature above absolute zero?

Selected Answer: d.

It emits Thermal Radiation

Answers: a.

It emits Heat for Convection

b.

It emits Energy

c.

It emits Heat for Conduction

d.

It emits Thermal Radiation

• Question 53
0 out of 1 points
At what particular condition that no more heat can that be removed from a substance and the
temperature can no longer be lowered?

Selected Answer: c.

Critical point

Answers: a.
Absolute zero

b.

Ground zero

c.

Critical point

d.

Freezing point

• Question 54
1 out of 1 points
Peak load for a period of time divided by installed capacity is

Selected Answer: d.

Utilization factor

Answers: a.

Capacity factor

b.

Load factor

c.

Demand factor

d.

Utilization factor

• Question 55
1 out of 1 points
Which of the following statements is correct? The relative humidity of an air water vapor
mixture.

Selected c.
Answer:
is the ratio of partial pressure of water vapor to the saturation pressure at the
mixture temperature.

Answers: a.

indicated the mass of water vapor in the mixture.


b.

is the ratio of the partial pressure of water vapor to the partial pressure of the
air.

c.

is the ratio of partial pressure of water vapor to the saturation pressure at the
mixture temperature.

d.

is equal to the mole fraction of water vapor in the mixture at the mixture
temperature.

• Question 56
1 out of 1 points
Air can be prevented from getting into the system by:

Selected Answer: c.

Keeping all glands and stuffing boxes on the low-pressure side tight

Answers: a.

Keeping all glands and stuffing boxes on the high- pressure side tight

b.

Running the refrigerant through an aerator

c.

Keeping all glands and stuffing boxes on the low-pressure side tight

d.

Keeping the dehydrator clean at all times

• Question 57
1 out of 1 points
How do you call the water vapor content of air?

Selected Answer: c.

Humidity

Answers: a.

Moisture
b.

Vapor

c.

Humidity

d.

Dew

• Question 58
1 out of 1 points
All of the following process can be found on a psychrometric chart except:

Selected Answer: b.

Natural convection

Answers: a.

Cooling and dehumidification

b.

Natural convection

c.

Sensible cooling

d.

humidification

• Question 59
1 out of 1 points
A valve the released steam from the boiler by opening at a pre-determined pressure in order
to keep the steam pressure from rising more than the pressure from which the valve is set is
called a

Selected Answer: a.

safety valve

Answers: a.

safety valve
b.

check valve

c.

ammonia valve

d.

stop valve

• Question 60
1 out of 1 points
At ordinary temperature what substance will behave as inert gas and will not react in the
combustion process?

Selected Answer: b.

Nitrogen

Answers: a.

Sulfur

b.

Nitrogen

c.

Hydrogen

d.

Carbon
IPE
C 1. Monochlorofluoromethane is a refrigerant B 13.A vessel permanently connected to a system
known as: by inlet and outlet pipes of storage of liquid
a. R-12 c. R-22 refrigerant
b. R-13 d. None of the above a. refrigerant container c. flash tank
b. liquid receiver d. surge tank
D 2. Which of the ff. is a proper way of leak
detection for R-12 and halocarbon B 14.Air flows through coil without making
refrigeration systems? contact with the coil surface is known as
a. stains c. toxic odor a. drawn air c. contact air
b. corrosion d. loss of cooling capacity b. bypass air d. air load

C 3. It refers to the final constant weight reached A 15.the process of lowering the temperature of
by a hygroscopic substance after being dried given space and maintaining it for the
out purpose of preserving and chilling certain
a. Bone Dry Weight c. Both a&b substances
b. Dry Weight d. NOTA a. refrigeration c. chilling
b. cooling d. air-conditioning
B 4. Is a type of dryer in w/c material to be dried
is supported on floor through w/c hot gases D 16.it will either cool or maintain a body at
pass temp below that of its surrounding is known
a. Tray dryer c. Rotary dryer as the
b. Hearth dryer d. Tower dryer a. heat engine c. heat pump
b. heat sink d. refrigerating machine
B 5. It is the temp. at w/c liquid starts to boil or
the temp. at w/c vapor begins to condense B 17.a reciprocating compressor has a piston
a. Dry blub temp c. Dew point Temp speed of 200 fpm at 400 rpm. Its stroke is
b. Sat. Temp d. Wet bulb temp approx.
a. 2.56 in c. 5.32 in
D 6. The tendency of pump to cavitate will be b. 3 in. d. 20 in
increased if:
a. impeller has smooth finish C 18.a refrigeration system that combines two
b. inlet edges are rounded refrigeration units
c. suction line velocities are lowered a. multi-evaporation system
d. fluid temp is raised b. multi-stage compression system
c. cascade system
B 7. For a single component system, the number d. air-refrigeration cycle
of properties required to define a phase
uniquely is: C 19.fire involving energized electrical equipment
a. 1 c. 3 a. class A fire c. class C fire
b. 2 d. 4 b. class B fire d. class D fire

A 8. It draws the flue gas from the system and D 20.dew point temp is temp where water starts
series them up the stack is: a. to condense
a. induced draft fan c. forced draft fan b. to boil
b. cyclone d. wind tunnel c. to condense and boil
d. to condense and boil at given pressure
D 9. The study of properties of air and its water
vapor contents A 21.glass prod process is broken down into 2
a. humidification c. ventilation gen. categories: container prod (glass
b. refrigeration d. psychrometry bottle/bottle) and sheet prod (float glass)
a. container & sheet prod
B 10.It exist when the total suction is above b. box & sheet prod
atmospheric pressure c. container & flattening prod
a. suction lift c. suction pressure d. NOTA
b. suction head d. suction line
C 22.modern detergents are comprised of three
C 11.It is the wastewater that will flow into each main components
unit of operation of the wastewater treatment a. builders, blend, extender
plant that is partially or completely treated b. binders, bleach, enzymes
a. liquid waste c. influent c. builders, bleach, enzymes
b. solid waste d. effluent d. No answer

A 12.It raises pressure of CO2 gas to 250 psig so


that it can be liquefied at -100F
a. CO2 compressor c. deodorizer
b. trim cooler d. foam trap
A 23.is a broad term applied to large number of C 30.water substance at 70 bar and 65C enters a
carbohydrates composed of carbon, boiler tube of constant inside diameter of 25
hydrogen, and oxygen present in many mm. the water leaves the boiler tube at 50
plants and characterized by a more or less bar and 700K at velocity of 100 m/s.
sweet taste Calculate the inlet volume flow in L/sec.
a. sugar c. iron At 70 bar & 65C:
b. salt d. sweet v1 = 0.001017 m3/kg
At 50 bar & 700K:
B 24.a closed vessel contains air at a pressure of v2 = 0.06081 m3/kg
160 kN/m2 gauge and temp of 30C. The air a. 0.75 c. 0.82
is heated at const. vol. to 60C w/ atm b. 0.64 d. 0.96
pressure of 759mmHg. What is the final
gauge pressure? D 31.twenty grams of oxygen gas (O2) are
a. 174 c. 167 compressed at const temp of 30C to 5% of
b. 186 d. 172 their original vol. What work is done on the
system?
C 25.what is the total reqd heating energy in `Use R of air = 0.0619 cal/gm-K
raising the temp of given amt of water when a. 824 cal c. 944 cal
the energy supplied is 1000kWh w/ heat b. 894 cal d. 1124 cal
losses of 25% ( )( )
a. 1000 c. 1333
b. 1500 d. 1250 C 32.gas is enclosed in a cylinder w/ weighted
̅̅̅̅ piston as top boundary. The gas is heated
and expands from a volume of 0.04 m3 to
0.10 m3 at const pressure of 200 kPa.
B 26.a certain gas at 101.325 kPa and 16C whose Calculate the work done by system.
vol is 2.83m3 are compressed into a storage a. 8 kJ c. 12 kJ
vessel of 0.31 m3 capacity. Before b. 10 kJ d. 14 kJ
admission, the storage vessel contained the ( )
gas at pressure and temp of 137.8kPa and
24C. After admission, the pressure has B 33.if F scale is twice C scale, what is the
increased to 1171.8 kPa. What should be the reading in the Fahrenheit scale?
final temp of gas in vessel (in Kelvin)? a. 160 c. 140
a. 298.0 c. 180 b. 320 d. 280
b. 319.0 d. 420
B 34.a cylinder & piston arrangement contains
A 27.a tank contains 80 ft3 of air at pressure of sat water vapor at 110C. The vapor is
350 psi. If the air is cooled until its pressure compressed in a reversible adiabatic process
and temp decrease to 200 psi and 70F until pressure is 1.6 MPa. Determine work
respectively, what is the decrease in internal done by the system per kg of water
energy? At 110C:
a. 4575 c. 5552 S1 = 7.2387 kJ/kg-K
b. -5507 d. 0 U1 = 2518.1 kJ/kg
At 1.6 MPa:
D 28.an electric heater is to heat 11 kg of oil per S2 = 7.2374 kJ/kg-K
min, from 4.5C to 65.5C. Specific heat of oil U2 = 2950.1 kJ/kg
is 2.1 J/gm-C. How many watts should this T2 = 400C
heater consume? a. -500 kJ/kg c. -632 kJ/kg
a. 20 448 W c. 20 400 W b. -432 kJ/kg d. -700 kJ/kg
b. 33 448 W d. 23 442 W ( )

A 35.During polytrophic process of ideal gas


C 29.find the heat transfer reqd to convert 5 kg of
state changes from 138 kPa and 5C to 827
water at pressure if 20 bar and temp of 21C
kPa and 171C. Find value of n.
into steam dryness fraction of 0.90 at same
a. 1.354 c. 1.345
pressure.
b. 1.253 d. 1.234
Properties of steam:
P = 20 bar ( ) ( )
hf = 909 kJ/kg
hfg = 1899 kJ/kg
Note: for water at 21C: C 36.calculate specific vol of air-vapor mixture
h = 88 kJ/kg in cu. m per kg of dry air when the ff.
a. 10 160 kJ c. 12 610 kJ conditions are given:
b. 11 610 kJ d. 15 610 kJ t = 30C
{[ ( )] } w = 0.015kg/kg
P = 90kPa
a. 2.12 m3/kg c. 0.99 m3/kg
3
b. 0.622 m /kg d. 1.515 m3/kg
A 37.copra enters dryer containing 40% H2O & C 44.a refrigerator having 30kW capacity
60% solid, and leaves w/ 5% H2O & 95% requires 10HP compressor. Find COP of
solid. Find wt. of H2O removed per lb. of system.
final prod. a. 2.78 c. 4.02
a. 0.583 lb/lb c. 0.475 lb/lb b. 3.78 d. 5.02
b. 1.3753 lb/lb d. 1.015 lb/lb
A 45.A VCRS is designed to have a capacity of
A 38.an air conditioning plant w/ capacity of 100 TOR. It produces chilled water from
400kW has an evaporating & condensing 22C to 2C. Its actual COP is 5.86 and 35%
temp of 3C and 37C respectively. If it uses of power supplied to compressor is lost in
R-12, what is the volume flow rate under form of friction and cylinder cooling losses.
suction conditions? Determine the size of electric motor reqd to
At 3C drive compressor in KW and vol flow rate of
hg = 352.75 kJ/kg chilled water in L/s.
v = 50.47 L/kg a. 92.31 kW, 4.199 L/s
hf = 202.78 kJ/kg b. 90.71 kW, 5.277 L/s
`Enthalpy leaving cond = 235.3503kJ/kg c. 93.75 kW, 5.724 L/s
a. 0.172 m3/s c. 3.411 m3/s d. 91.75 kW, 7.575 L/s
b. 0.281 m3/s d. 3.5 m3/s
A 46.a single acting twin cylinder ammonia
D 39.fish weighing 11,000 kg w/ temp of 20C is compressor w/ bore equal to stroke is driven
brought to cold storage & w/c shall be by engine at 250 rpm. The machine is
cooled to -10 C in 11hrs. Find the reqd plant installed in chilling plant to produce 700 kW
capacity in tons if Cp of fish above and of refrigeration at -18C evaporator temp. At
below freezing is 0.7 kCal/kg-C and 0.3 this temp, cooling effect per kg mass = 1160
kCal/kg-C respectively, w/ freezing pt at - kJ. Specific vol of vapor entering
3C. compressor = 0.592 m3/kg. Assume 85%
`The latent heat of freezing = 55.5 kCal/kg volumetric efficiency, determine the bore in
a. 25.26 c. NOTA mm.
b. 15.26 d. 24.38 a. 400 mm c. 500 mm
b.300 mm d. 450 mm
C 40.an ice plant produces 20 tons of ice per day
at -15C from water at 25C. If losses are 12% A 47.A refrigeration compressor having 10 in.
of freezing and chilling load, calculate flywheel is driven by a 4-pole AC motor. If
refrigeration capacity of plant in TOR. diameter of motor is 4 in, determine speed
a. 28.13 c. 31.5 of compressor.
b. 28.13 d. 41.5 a. 70 rpm c. 600 rpm
b. 500 rpm d. 800 rpm
A 41.calculate tons of refrigeration reqd to freeze
in 4 hrs. 3mm of 10C water on a skating risk A 48.calculate piston displacement of 2-cylinder
61m x 30m if the ground temp is 10C and compressor rotating at 1450 rpm if diameter
the air temp is 10C and the air temp above of cylinder is 2.5 in. & length of stroke is 2
rink is 16C. in.
Heat transfer coefficient a. 16.48 ft3/min c. 14.48 ft3/min
air to 0C ice = 6.8 W/m2-C b. 15.48 ft3/min d. 17.48 ft3/min
grnd to -6C = 0.3402 W/m2-C
a. 101.69 c.140.62 A 49.a duct 0.40 m high and 0.80 m wide
b. 151.68 d. 201.96 suspended from the ceiling in a corridor
makes a right angle turn in the horizontal
D 42.The power requirement of Carnot refrigerator plane. The inner radius is 0.2m and outer
in maintaining a low temp region at 300K is radius is 1.0 m measured from the same
1.5 kW per ton. Find heat rejected. center. The velocity of air in duct is 10 m/s.
a. 4.02 kW c. 6.02 kW Compute the pressure drop in this elbow.
b. 7.02 kW d. 5.02 kW Assume:
f = 0.3
D 43.4000 L/hr of distillates are to be cooled from density = 1.204 kg/m3
21C to -12C and 12% of wax by wt. is L = 10m
separated out at 15C. The cp heat of oil is 2 a. 341 Pa c. 143 Pa
kJ/kg-C and SG is 0.87. The specific heat of b. 441 Pa d. 144 Pa
wax is 2.5 and latent heat is 290 kJ/kg.
Allow 10% for losses, find the refrigerating C 50.determine the sensible heat of 5 lb of air
capacity of system. having dry bulb temp of 70F and humidity
a. 20 TOR c. 40 TOR ratio of 0.0092 lb/lb, the latter corresponding
b. 51.08 TOR d. 31.08 TOR to dew pt temp of 55F.
a. 94 BTU c. 84 BTU
b. 48 BTU d. 49 BTU
C 51.a 0.70 m vane axial fan is running at 2000 A 55.a mixture of dry air and water vapor is at
rpm delivers 7.5 m3/s against 0.08m static temp of 21C under pressure of 101kPa. The
pressure thereby consuming 12 kW. If fan dew pt temp is 15C. Calculate the relative
wheel diameter is increased from 0.70 m to humidity.
0.76 m, so the diameter ration is 1.10:1, a. 68.56% c.56.68%
what is the new static pressure? b. 65.68% d. 58.66%
a. 1.01m c. 0.097m
b. 0.314m d.31.01m A 56.a dryer is to deliver 1000 kg/hr of palay
with a final moisture content of 10%. The
B 52. Centrifugal pump has const speed of 1500 initial moisture content in the feed is 15% at
rpm and has head of 75m. What is the effect atm cond w/ 32C dry-bulb & 21C wet-bulb.
on the head of the pump if the impeller The dryer is maintained at 45C while RH of
diameter is reduced from 280mm to 230 hot humid air from dryer is 80%. If steam
mm? pressure supplied to heater is 2 MPa,
a. 25.6 mm c. 65.6 mm determine air supplied to dryer in m3/hr.
b. 50.6 mm d. 72.6 mm a. 1332.25 m3/hr c. 1223.25 m3/hr
3
b. 1233.25 m /hr d. 1523.13 m3/hr
A 53.in an air conditioning unit, 3.5 m3/s of air at
27C dry-bulb temp, 50%RH & std. atm A 57.An assembly hall was to have an air
pressure enters unit. The leaving condition conditioning unit installed w/c would be
of air is 13C dry-bulb temp and 90% RH. maintained at 26C dry-bulb & at 50%RH.
Using properties from psychro chart, The unit delivers air at 15C dry-bulb temp &
calculate refrigerating capacity in kW. the calculated sensible heat load is 150kW
a. 87.57 kW c. 57.87 kW & latent heat is 51.3kW. 20% by wt of
b. 77.57 kW d. 58.77 kW extracted air is made up of outside air at 34C
dry-bulb & 60% RH, while 80% is extracted
D 54.a stream of outdoor air is mixed with steam by air conditioner from assembly hall.
of return air in an air conditioning system Determine the air conditioner’s refrigerating
that operates at 101kPa pressure. The flow capacity in TOR & its ventilation load in
rate of outdoor system air is 2 kg/s and its kW.
condition is 35C dry-bulb temp & 25C wet- a. 83.22 TOR, 37.47 kW
bulb temp. he flow rate of return is 3 kg/s & b. 76.43 TOR, 57.34 kW
its cond is = 24C & 50%RH. Determine c. 89.56 TOR, 45.77 kW
enthalpy of mixture. d. 56.78 TOR, 47.68 kW
a. 91.56 kJ/kg c.69.15 kJ/kg
b. 51.69 kJ/kg d. 59.16 kJ/kg
71.81% 58. Sol’n:

( ) ( )

( )( ) ( ) ( )
( )

( )( )

59.13% 59. Sol’n:

HEAT BALANCE:
( ) ( )
( )( )( ) ( )

5.6 kW 60. Sol’n:


( )( )( )
[( ) ] [( ) ]

FOR TWO STAGE:


√ √( )( )

( )( )( )
[( ) ] [( ) ]
IPE Exit Exam Reviewer

1. What tonnage of refrigerating machine is required if the refrigerating system extracted


48,000 btu per hour?
Answer: 4 tons

2. An insulator with superior tensile strength but low heat resistance


Answer: Fiberglass

3. What is the combination of a wet and dry bulb thermometer is called a


Answer: Hygrometer or psychrometer

4. The weight of steel bar reinforcement show be how many time the weight of the foundation?
Answer: ½% to 1%

5. Is a high temperature process that turns incinerator ash into a safe, glass – like material
Answer: Vitrification

6. Oil is added to a freon compressor by


Answer: Pumping in with a hand pump

7. The difference between the theoretical draft and the draft loss
Answer: Available draft

8. The purpose of expansion valve by-pass is to


Answer: Controls the refrigerant to the evaporator in case the automatic valves fail

9. Load curve refers to the plot of:


Answer: Load versus time

10. What does a negative Joule-Thompson coefficient means during a throttling process?
Answer: fluid temperature rises

11. The maximum unit pressure of turbine and generator on the reinforced concrete should not
exceed
Answer: 17.62 kg/cm2

12. Water in vapor form remains a vapor as long as temperature is what relations to the dew
point temperature
Answer: below
13. The “refrigerating effect” of a refrigerant is a always:
Answer: Less than its latent heat

14. A burner should always start up in ___ fire and shut down in ___ fire
Answer: Low; low

15. A ___ boiler has water in the tubes and heat gases of combustion passing through the tubes
Answer: Water tube

16. If the compressor had been running satisfactorily for a long period of time but suddenly the
compartment temperature started to rise, the trouble might be:
Answer: Any of the other choices
(A refrigerant leak has developed, The solenoid valve has jammed shut, The expansion valve
may contain frozen water)

17. A type of polymer used for Styrofoam cups and clam shell food containers
Answer: Polystyrene (PS)

18. Mycotoxins are poisonous chemicals produced by:


Answer: Molds

19. The magnitude of the drag coefficient of a sphere in water is dependent upon all of the ff
except
Answer: Units of measure (SI or English engineering system)

20. A refrigeration system in which only part of the refrigerant passes over the heat transfer
surface is evaporated and the balance is separated from the vapor and recirculated
Answer: Flooded system

21. The boiling point of freon-12 at atmospheric pressure is


Answer: -22F

22. Which of the following vital components of the refrigeration system where both temperature
and pressure are increased?
Answer: Compressor

23. Which of the following must be done to eliminate frost on the discharge pipe of the
compressor?
Answer: None of the other choices
(Open the expansion valve, crack bypass valve, Regulates water to the condenser)

24. Orifice coefficients are used to determine:


Answer: Energy losses
25. Which of the following is used as high head turbine?
Answer: Impulse

26. A water-tube condenser has a total of 60 tubes. If these two passes, then compute the number
of tubes per pass
Answer: 15

27. Absolute zero is


Answer: 460 degrees below zero on the Fahrenheit scale

28. Which of the following can be the cause of low head pressure
Answer: Any of the other choices
(Leaky discharge valves, Too much or too cold condensing water, insufficient charge of
refrigerant)

29. What is the most common freon gas used in centrifugal compressors?
Answer: F-11

30. When purging an ammonia condenser into a bucket of water, one can tell when the air is out
and ammonia starts to come through the:
Answer: Change of bubbling sound of air to the cracking sound of ammonia

31. In the deep well installation or operation, the difference between static water level and
operating water level is known as:
Answer: Drawdown

32. White ice is


Answer: Due to dissolved air, gasses and impurities

33. Which of the following is a characteristic of an impulse turbine


Answer: Steam striking blades at zero angle

34. Heavy water is:


Answer: D2O

35. When excess solute in a solution settles to the bottom of the container, the process is called:
Answer: Precipitation

36. Calcium chloride is sometimes used in refrigeration system as a:


Answer: Secondary Coolant

37. If a compressor runs continuously, the cause might be a:


Answer: Stuck low-pressure switch
38. The thermal expansion valve is located between the:
Answer: Solenoid valve and the evaporator coils

39. The cooling water regulator is automatically actuated by which of the following?
Answer: The discharge pressure of the refrigerant

40. Based on the PSME code, what should be provided in each steam outlets if two or more
boilers will be connected in parallel?
Answer: Non-return and shut off valves

41. The amount of sensible heat for a sensible heat ratio of 0.8 and a total cooling load of 100 is
Answer: 80

42. Which refrigerant is used for the air conditioning of passenger aircraft cabin
Answer: Air

43. When a falling object reaches a speed at which the drag force equals its weight, it has
achieved:
Answer: A turbulent boundary layer

44. The minimum vertical distance from the floor or soil level to the top edge of the foundation
must be around
Answer: 120 mm

45. How many moisture be removed from air?


Answer: condensation and absorption

46. The charging valve is located between the:


Answer: King valve and the expansion valve

47. A “cold diffuser” is a:


Answer: Type of evaporator

48. Which of the following stops the compressor before the relief valve opens?
Answer: High pressure control

49. What is the excess refrigerant removed?


Answer: Charging side of the system

50. Which of the following is used in measuring the density of a brine solution?
Answer: A hydrometer
51. All of the following are common types of chemical reaction except
Answer: Fission

52. A ___ after each radiator allows ___ to pass through to the return line
Answer: Steam trap; condensate

53. A leaky solution valve can usually detected by:


Answer: Any of the other choices
(A fluctuating suction pressure gauge, a higher suction pressure, closing in on the suction
valve having no effect on the suction pressure)

54. If the cooling water to the condenser suddenly fails:


Answer: The compressor will shutdown

55. If Et is the total net energy generated by a plant in a certain period of time and Er is the rated
net energy capacity of the plant during the same period of time, then what is the plant
operating factor?
Answer: Et/Er

56. A thermometer sense which of the following?


Answer: Sensible heat

57. Is the subjective method in which the smoke density is visually compared to five
standardized white-black grids
Answer: Ringelman scale

58. Excessive head pressure is caused by


Answer: Flooded condenser tubes or cooling water temperature too high

59. What do you call a plate or vane used to direct or control movement of fluid or air within the
confined area?
Answer: Baffle

60. The total pressure supplied by the fan at maximum operating conditions
Answer: Net rating or fan boost

61. The system should be purged:


Answer: After the system has been shut down for few hours

62. What is likely to occur when sections of the impeller of a centrifugal pump are handling
vapor and other sections are handling liquid.
Answer: complete failure to operate
63. What is the chemical formula of Ozone?
Answer: O3

64. An excessively high head pressure would be caused by:


Answer: Insufficient cooling water to the condenser

65. When an electric motor nameplate indicate a “100-kW electric motor” then what does a 100-
kW rating refers to?
Answer: Mechanical power output

66. During a cooling and dehumidifying process, sensible and latent heats are removed in the
cooling coil. If Hs is the sensible heat and Ht is the total heat transferred, then determine the
coil sensible factor.
Answer: Hs/Ht

67. A single trunk piston-type compressor is undesirable for a Freon unit because the:
Answer: Lubricant mixes with the refrigerant

68. What is the first step that must be done when securing a system?
Answer: Close receiver (King) valve

69. The amount of CO can be determined by the color of Palladium Chloride. An amount of 30
ppm to 70 ppm will cause:
Answer: Slight Darkening

70. The water hammer phenomenon is primarily what kind of fluid mechanism?
Answer: Dynamic (a time-dependent phenomena)

71. The other name for Swamp Cooling is:


Answer: Wet Roof Cooling

72. The design of an air supply duct of an air condition system


Answer: affects the distribution

73. A Freon – 12 gage shows pressure and ____.


Answer: Saturation temperature

74. High superheat of the vapor in the system would cause


Answer: A decrease in capacity

75. When there is no work between the thermodynamic system and its surrounding, the quantity
of net heat transfer is equal to:
Answer: total energy of a closed system
76. The relation between the Fahrenheit absolute scale and the Celsius absolute scale is:
Answer: R = 1.8 Kelvin

77. External frost on inlet of expansion valve indicates:


Answer: Expansion valve plugged or dirty

78. The total cross section of a target atom is made up of


Answer: The absorption and scattering cross sections

79. The motor condition of an air conditioning unit can be checked with:
Answer: The continuity of light or with an ohmmeter

80. A number used to compare energy usage for different areas. It is calculated by dividing the
energy consumption by the footage of the conditioned area.
Answer: Energy utilization Index

81. What is the effect if the refrigerant is removed from the system too fast?
Answer: It may “freeze – up” the condenser

82. After the total force of the steam has lifted the safety valve off its seat, the steam enters the
____.
Answer: Huddling chamber

83. By which of the fooling processes heat mainly dissipates in cooling towers?
Answer: Evaporation

84. All of the following fluid phenomena are based on the force momentum principle of a
flowing fluid except:
Answer: Diesel automobile engines

85. All of the following are forms of drag on a body moving through a fluid except
Answer: D’alembert’s paradox drag

86. What is the use of the suction pressure regulating valve?


Answer: Maintains the back pressure in the evaporator coils

87. There shall be no foundation bolts less than _____.


Answer: 12 mm in diameter

88. The ratio of the rated cooling capacity divided by the amount of electrical power used:
Answer: Energy efficiency ratio (EER)
89. The scale trap is located between the:
Answer: King (liquid) valve and expansion valve

90. The temperature in the vegetable box should be approximately


Answer: 35 to 45 deg. F

91. A thermostat is a
Answer: Temperature-operated switch

92. The bulb for the thermal expansion valve is located:


Answer: Near the evaporator coil outlet

93. The coefficient of velocity is the ratio of the:


Answer: Actual velocity to the theoretical velocity

94. What is the specific humidity of dry air?


Answer: 0

95. What is the reason why a thermometer in vapor compression system is installed close to the
compressor?
Answer: Because it helps the operator in adjusting compressor for greatest efficiency

96. As the steam temperature increases the steam temperature:


Answer: Increases

97. Substances that have the ability to absorb moisture from the air are called:
Answer: Desiccants

98. Evaporative condenser is used to cool


Answer: condenser vapor

99. If the head pressure is too high:


Answer: The high pressure cutout switch should operate before the relief valve opens

100. A material or substance that is accidentally or intentionally introduce to the environment


in a quantity that exceeds what occurs naturally
Answer: Pollutant

101. The greatest decrease in refrigerant temperature occurs in the ____


Answer: Evaporator

102. The diesel cycle is the ideal cycle for a :


Answer: compression-ignition engine
103. The moisture in a refrigeration system can be removed with help of which of the
following?
Answer: Driers

104. What process is employed when the turbine steam power plants experience excessive
moisture?
Answer: reheating

105. The path that directs the flow of refrigerant through the compressor
Answer: Valve

106. Which of the following is another name for the liquid valve?
Answer: King valve

107. Expansion factors take into account the:


Answer: Effects of compressibility

108. The term used to express the amount moisture in a given sample of air. It is compared
with the amount of moisture in a given sample of air.
Answer: Relative Humidity

109. What are the four basic methods of determining whether the proper amount of refrigerant
being added to the system?
Answer: Bull’s eye, weight, pressure and frost line

110. Which of the following components of the window air conditioning system must be
cleaned annually?
Answer: All of these
(Evaporator and Condenser, Motor and Compressor, Fan Blades and Fan Motor)

111. At present, the number of true elementary particles, which include leptons and quarks is
thought to be
Answer: 12

112. All of the following statements about conjugate acids and bases are true except
Answer: A conjugate base results when a base accepts a proton

113. A double-seated valve allows the valve to be


Answer: Packed in the wide open or closed position

114. In order to keep combustion products inside the combustion chamber and stack system,
balanced draft system may actually operate with a slight
Answer: negative pressure
115. What is the cause of pressure drop in the boiler, condenser and the pipings between
different components? Because of this drop, steam leaving the boiler at a lower pressure.
Answer: Fluid friction

116. It is a form of oxygen photochemically produced in nature.


Answer: Ozone

117. Absolute zero on the Fahrenheit scale is equal to


Answer: -460

118. Low suction pressure is caused by:


Answer: Solenoid valve not functioning properly

119. What do you call the storage tank for liquid refrigerant?
Answer: Receiver tank

120. Why should you avoid bending or twisting of fan blades in an air conditioning unit?
Answer: It will wear out the motor bearings and cause noise

121. All of the following statements are characteristics of bases except


Answer: They have a pH between 0 and 7

122. Concrete foundation should have steel bar reinforcement placed vertically and
horizontally to avoid ____.
Answer: Thermal cracking

123. The pump will not cavitate if the available Net Positive Suction Head (NPSH) is:
Answer: equal or greater than the required NPHS

124. What will happen to the capacity if the superheat is increased on the suction side?
Answer: Decreases

125. When heavy electrical currents are involved, the thermostat will be operated by a
Answer: Relay

126. Valves in piping in an ammonia system are made of


Answer: Iron

127. Which of the following is not essential to a compression refrigeration system?


Answer: A receiver

128. Nitrogen occupied almost how much of the Earth’s atmosphere?


Answer: 3/4
129. If frost forms on the cylinders, the cause would be:
Answer: Expansion valve open too wide

130. Which of the following gasket materials should be on Freon system?


Answer: Asbestos or Metallic

131. Which of the following would cause the crankcase and head to get hot with low suction
pressure?
Answer: Insufficient refrigeration

132. Moisture in a system will cause a:


Answer: Faulty expansion valve

133. The steam turbine foundation should be designed to support the machine load plus how
many percent for impact, condenser load, floor loads and dead loads?
Answer: 25

134. When air is heated, what happened to its relative humidity?


Answer: Remain constant

135. Sweating of crankcase is caused by which of the following?


Answer: Too much oil in the system

136. What do you call a material in a dryer?


Answer: Desiccant

137. The matching model and full-scale prototype results a fluid dynamic phenomena
involving a fully submerged body requires equality of:
Answer: Reynolds number

138. Which of the following uses a secondary refrigerant?


Answer: Ice plant

139. When the outlet temperature at the evaporator exceeds the inlet temperature, the
condition is known as:
Answer: Superheating

140. In series pipe systems, all of the following parameters vary from section to section except
Answer: Mass flow

141. Which of the following is used to control refrigeration compressor capacity?


Answer: Unloader
142. TOR is a unit equivalent to:
Answer: 12.66 kN-m/hr

143. The agent used in an indirect reefer system is:


Answer: Calcium chloride or sodium chloride

144. If the W is the work output of a heat engine and H is the total heat input of an engine,
then determine its thermal efficiency, e.
Answer: e = W/H

145. Which of the following items is not important when using a halide torch?
Answer: Adjust to a clear white flame

146. A dehumidifier is usually a small hermitic refrigerating system. It has both a condenser
and an evaporator. Many older systems use R – 12 or R – 500. The newer units are:
Answer: R – 134a

147. What is the method used to evaluate all welds performed on pressure parts of boiler tube
materials?
Answer: Radiographic test

148. Faulty F – 12 compressor valves will be indicated by:


Answer: All of the other choices
(Gradual or sudden decrease in capacity, compressor running continuously, Low head
pressure – high suction pressure)

149. Heat exchanger in which water flows by gravity over the outside of tubes or plates.
Answer: Baudelot cooler

150. Which of the following is not a sound absorptive material?


Answer: Mirrors

151. The desirable temperature inside an air conditioned auditorium is:


Answer: 20 C

152. An odorless refrigerant whose boiling point varies over a wide range of temperatures.
Answer: Freon 12

153. The cooling water regulator is actuated by


Answer: Pressure of the refrigerant

154. The psychrometric chart in air conditioning determines the


Answer: Moist air conditions
155. Blowback of a safety valve is to prevent:
Answer: Chattering

156. When starting a refrigerant unit, be sure the water side of the condenser is ____.
Answer: Vented

157. A temperature measurement in an ordinary temperature which has constant specific


humidity.
Answer: Dew point temperature

158. What adding oil to a freon system, one must be sure that
Answer: All air is removed from the pump and fitting

159. The primary purpose of a turbine in a fluid is to:


Answer: Extract energy from the flow

160. Which of the following is to be checked as regular monthly maintenance schedule of a


console air conditioning units?
Answer: Duct dampers, registers and diffusers

161. A company is interested to produce a water turbine wherein only little energy is required
or necessary because the guide vanes are to be controlled. The turbine must be a:
Answer: Kaplan turbine

162. To check the lack of refrigerant or clogged refrigerant lines:


Answer: Installing service valves must be necessary

163. Condensate from the non return steam trap is pumped from the condensate return tank of
the:
Answer: Boiler

164. The locus of elevation is:


Answer: Hydraulic Gradient

165. Which of the following cannot be used to describe neutron kinetic energy levels
Answer: Freezing

166. It is important to take some moisture from the air dehumidify it if the relative humidity
reaches high levels. To do this, it requires cooling the air:
Answer: Below its dew point temperature

167. The vapor cycle in thermal efficiency as the refrigeration cycle to the
Answer: COP for a refrigerator
168. A scale trap in a Freon system will be found on the
Answer: Suction side

169. Which of the following is not used as method to measure air velocities?
Answer: Open type Barometer

170. Which of the following is considered as comfort condition in air conditioning?


Answer: 20 C DBT, 60% RH

171. Which of the following acts as ignition accelerator for internal combustion engine fuels?
Answer: Acetone peroxide

172. The purpose of the evaporator is to


Answer: Absorb latent heat of vaporization

173. The speed at which a reaction proceeds to equilibrium is the purview of


Answer: Reaction kinetics

174. What do you call the liquid reaching the compressor through the suction?
Answer: Flooding back

175. Which of the following draft rely on the stack effect to draw off combustion gases?
Answer: Natural draft

176. The relative humidity becomes 100% and where the water vapor starts to condense.
Answer: Dew point

177. Which of the following statements is correct? The relative humidity of an air water vapor
mixture.
Answer: is the ratio of partial pressure of water vapor to the saturation pressure at the
mixture temperature

178. The following are standard characteristics of Freon – 11 except:


Answer: Boiling point over 200 F

179. The bhp/ton refrigeration requirement with increase in condenser temperature. In a vapor
compression system that uses reciprocating compressor receiving refrigerant gas at constant
suction temperature, will follow:
Answer: Linearly increasing characteristic
180. In an air conditioning unit, the thermostat fails. The unit did not start. How do you test
the operation of the thermostat?
Answer: Cover the air outlet and air inlet with a cloth. The air will recirculate into the
unit and the temperature will quickly drop to the cut out temperature.

181. All of the following temperature have meaning in psychrometrics except:


Answer: Adiabatic wall temperature

182. The maximum continuous power available from a hydro – electric plant under the most
adverse hydraulic conditions is called
Answer: Firm power

183. Which of the following dehumidifier is often used to reheat the air after moisture is
removed?
Answer: Condenser

184. Which of the following is a type of evaporator?


Answer: shell-and-tube

185. All of the following factors affect rates of reaction except


Answer: Pressure

186. Which of the following would cause a high suction pressure?


Answer: Expansion valve open too wide

187. Zinc rods are found in the:


Answer: Salt water side of the condenser

188. The sum of the internal energy and the product of pressure and specific volume is known
as the:
Answer: enthalpy

189. Any foreign matter in the coal feed mechanism of the screw-feed stoker is best removed
by:
Answer: Using the cutoff gate at the bottom of the hopper

190. Based on the PSME code, what should be the humidity of air to be used for comfort
cooling?
Answer: 50% - 60% relative humidity
191. Which of the following must be checked up if an automatic Freon system will not start
up?
Answer: All of the other choices
(High pressure cutout, reset mechanism, Low-pressure cutout)

192. Refers to the use of composting soil beds


Answer: Biofiltration

193. What is the usual cop of a domestic refrigerator?


Answer: More than 1

194. In a hydro-electric plant using a Francis turbine with medium head, the speed can be
regulated using the
Answer: Wicket gate

195. The temperature bulb of the thermo-expansion valve is attached to which of the
following?
Answer: Evaporator coil outlet

196. Which of the following refrigerants would give the most trouble when operating with
warm circulating water?
Answer: CO2

197. A rule which states that any waste derived from the treatment of a hazardous waste
remains a hazardous waste
Answer: The derived from rule

198. A device for holding open the suction valve and drawing gas from the suction manifold
and returning it to the suction line without compressing it is called
Answer: Cylinder unloader

199. The intake pipe to a hydraulic turbine from a dam


Answer: Penstock

200. What consists of weak solutions of sulfuric, hydrochloric, and to a lesser extent, nitric
acids?
Answer: Acid rain

201. The expansion valve does not seem to be operating properly. There is high superheat.
Test by listening to the sound of Freon flooding through the tubes and ____.
Answer: Warm bulb with hand
202. Swimming pool water needs
Answer: de-chlorination

203. Oxygen is approximately what percent by weight in the atmosphere?


Answer: 23%

204. Which of the following could be used to check a leak in an ammonia system?
Answer: Litmus paper & Sulfur stick

205. The determination of properties and behavior of atmospheric air usually the purview of:
Answer: Psychrometrics

206. A valve the released steam from the boiler by operating at a pre-determined pressure in
order to keep the steam pressure from rising more than the pressure from which the valve is
set is called a
Answer: Safety valve

207. If the critical temperature of a refrigerant is too close to the desired condensing
temperature, the equipment must necessarily be of
Answer: Extra heavy construction

208. If a freon – 12 compressor trip out on “cut-out”, the solenoid valve closes by which of the
following?
Answer: An electrical release

209. Which of the following would cause the suction switch to remain open?
Answer: Bellows broken or jammed

210. In the upper atmosphere, ozone is made by ultraviolet light reacting with:
Answer: Oxygen

211. How is ammonia system purged so that operator will not be overcome by the fumes?
Answer: Into the bucket of water

212. A ton of refrigeration is equal to the removal of


Answer: 288,000 btu per 24 hrs.

213. As a good practical rule, the foundation depth may be taken as how many times of the
engine stroke?
Answer: 2:5 to 3:5

214. Most stacks are built of height less than _____.


Answer: 30 m
215. The phenomenon that warm air rise and cold air settle is called:
Answer: Stratification

216. Type of refrigerant control which maintains pressure difference between high side and
low side pressure in refrigerating mechanism:
Answer: Expansion valve

217. The crossover connection in an ammonia system can be used to ____.


Answer: Hot-gas defrost

218. Which of the following refrigerants has lowest freezing point temperature?
Answer: Freon 22

219. The operation that produces highest noise level is:


Answer: Riveting

220. Select the one in which secondary refrigerant is used


Answer: Ice plant

221. Which of the following is not essential to a centrifugal type compressor system?
Answer: Distiller

222. Two compressor should not be run in parallel because


Answer: There is a possibility of losing oil

223. Which of the following is also known as refrigerant no. R-717?


Answer: Ammonia

224. During sensible heating, the humidity remains constant but the relative humidity.
Answer: decreases

225. The term suction units is used with


Answer: Induced draft

226. If the compressor had been running satisfactorily for a long period of time but the oil
level was rising slowly, one should:
Answer: Shut down the compressor and check the oil level with the machine stopped

227. What amount of air is required in a low bypass factor?


Answer: Lesser

228. The reduction of nuclear radiation intensity (called attenuation) is accomplished by:
Answer: Shielding
229. The ozone concentration of 0.10 parts per million (ppm) is generally considered the
maximum permissible for how many hours exposure?
Answer: 8 Hours

230. Which do you think is very important in adjusting compressor “v” belts?
Answer: Allow ½” slack

231. An engineer inspected an air-conditioning unit. He found out that the unit does not
produce any cooling effect, however, the air-conditioning unit is running. He checked the
temperature of the condenser and evaporator and hat the unit run. He found out that there was
no change in temperature. What should he do?
Answer: Charge with new refrigerant

232. The temperature at which water vapor in the atmosphere begins to condense is known as:
Answer: dew point

233. The ice making capacity is always


Answer: Directly proportional to the refrigerating effect

234. If the compressor short-cycles on the high-pressure cutout, which of the following would
you check?
Answer: All of the other choices
(If plenty of cooling water is running through but it is not picking up heat, the condenser
tubes need cleaning, Be sure system is getting cooling water, Check for too much refrigerant
in the system)

235. What is the instrument used to register relative humidity?


Answer: Hygrometer

236. What has an octane rating of more than 100?


Answer: Benzol

237. A device used to keep moisture from passing through the system is called:
Answer: Dehydrator

238. If the freezing point of water is 0 C, which of the following is its melting point following
is its melting point
Answer: 0 C

239. What is the pressure present inside the casing of an impulse turbine?
Answer: Atmospheric pressure
240. A vena contracta in a fluid jet issuing through a hole in a plate is located approximately:
Answer: At jet’s minimum diameter

241. If the compressor short-cycles on the low-pressure cutout, the trouble might be:
Answer: Any of the other choices
(Too much frost on the evaporator coils, dirty traps and strainers, lack of refrigerant)

242. The amount of heat necessary to bring up temperature of a unit mass of a substance
through unit degree is called
Answer: Total heat

243. If the expansion valve capillary tube is pinched, which of the following be replaced?
Answer: All of the other choices (called the power element unit)
(Bulb, tube, diaphragm)

244. The coefficient of discharge is the ratio of the.


Answer: Actual discharge to the theoretical discharge

245. Pitot tube is used to measure the:


Answer: Velocity of flow

246. The principle of the centrifugal system is based on which of the following?
Answer: Kinetic energy

247. The disadvantage of a CO2 system over an ammonia system is the fact that
Answer: All of the other choices
(The pipes and fittings of a co2 system must be of the high-pressure type, The CO2 system
requires a larger prime remover, The CO2 system operates at a much higher pressure)

248. If ice will form in a solution of water and salt, then it is at temperature called:
Answer: Freezing point depression

249. The suction pressure in a Freon system should be:


Answer: The pressure which corresponds with a temperature about 20F below the
temperature of the icebox.

250. Wb is shaft work of an engine and Wi is indicated work of an engine. If mechanical is


present in the engine mechanism, then.
Answer: Wb is less than Wi

251. What is the relationship of the capacity of a centrifugal pump, Q, to impeller diameter, D,
when there are two impeller diameters in the same pump?
Answer: Q is directly proportional to the ratio of D
252. The combination of enthalpy and kinetic energy of fluid is termed as:
Answer: Stagnation enthalpy

253. Before any repair work is attempted on any gas-fired boiler the:
Answer: Main gas cock must be secured

254. An air-conditioning system in which water is chilled or cooled and which passes the
evaporator coils
Answer: Chilled water system

255. Refers to organic waste produced from biological water waste treatment processes
Answer: Biosolids

256. As far as combustion chamber design is considered, the maximum power output of a
given engine can be increased by:
Answer: Decreasing combustion chamber volume

257. A radioactive gas produce from the decay of radium within the rocks beneath a building
Answer: Radon

258. Ozone filters out what type of radiation that damages crops and causes skin cancer?
Answer: Ultraviolet radiation

259. The ratio of the sum of individual maximum demands of the system to the overall
maximum demand of the whole system
Answer: Diversity factor

260. A bell coleman cycle is also known as


Answer: Reversed Joule cycle

261. It is used deliver concentrated airstreams into a room. Many have one-way or two-way
adjustable air stream deflectors.
Answer: Register

262. The sensible heat ratio is 0.8. that is


Answer: 80% sensible heat and 20% latent heat

263. Closing the solenoid valve will stop the compressor through the ____.
Answer: Low-pressure cutout switch

264. A refrigerating unit of one (1) ton capacity can remove?


Answer: 200 btu’s per min
265. Which of the following is the usual case of slugging?
Answer: Expansion valve not operating properly

266. All of the following occur during reduction of a substance except


Answer: Loss of electrons

267. The secondary refrigerant used in milk chilling plants is generally


Answer: Glycol

268. Where is the solenoid coil installed?


Answer: Vertically over the valve

269. The thermal expansion valve responds to the


Answer: Amount of superheat in the vapor

270. To eliminate transmission of vibration, the foundation should be isolated from the floor
slabs of building footings at least how may mm around its perimeter?
Answer: 25

271. Which one is commonly used liquid absorbent?


Answer: Ethylene glycol

272. The low-pressure control switch:


Answer: Cuts out the compressor to maintain proper flow

273. If the outlet of the thermostatic valve is warmer than the inlet, it indicates
Answer: Thermostatic valve not working properly

274. Compute the wall gain load of a load storage room when A is the outside surface area of
the wall, D is the temperature differential across the wall, and U is the overall coefficient of
heat transmission.
Answer: AUD

275. A leak on the fuel oil suction line between the tank and the suction side of the fuel oil
pump would result in:
Answer: Air entering the suction line

276. How is the CO2 system is purged?


Answer: When CO2 comes out of the purge valve, frost will form on a piece of metal
held near the outlet

277. The relief valve on a CO2 machine is located:


Answer: On the discharge pipe between the compressor and discharge valve
278. The high-pressure side of the system is sometimes referred to as the:
Answer: Hot side

279. Combined pressure of cooling and humidifying is also known as:


Answer: Evaporative cooling process

280. The schedule of a pipe, N, indicates the thickness of the pipe wall. If the allowable stress
of the pipe is S, then what is the internal pressure equal to?
Answer: N x S

281. Which of the following area of work requires lowest noise level?
Answer: Library

282. Which of the following contribute to the deterioration of the earth’s ozone layer?
Answer: Chlorofluorocarbons

283. What is the least number of compressors a multistage system that will use?
Answer: Two

284. Is a soluble compound that reduces a liquid’s surface tension or reduces the interfacial
tension between a liquid and a solid
Answer: Surface – acting agent or surfactant

285. To secure the belts embedded within the foundation, the distance of the edges of the
foundation from the bedplate must be ____.
Answer: 120 mm to 300 mm

286. In Stirling and Ericsson Cycles, the efficiency can be increased by:
Answer: Regeneration

287. The thermal expansion valve


Answer: Controls the quantity of liquid refrigerant going to the evaporator coils

288. Too low suction pressure could be caused by:


Answer: Any of the other choices
(Dirty scale traps, Shortage of refrigerant gas, Too much oil in the system)

289. The static pressure drop due to friction through the boiler and stack.
Answer: Draft loss

290. In a freon-11 system there is no


Answer: Distiller
291. The coefficient of contraction is the ratio of the:
Answer: Area of vena contracta to the orifice area

292. Which of the following should not be used to clean scale traps:
Answer: Cotton waste

293. Most observe properties of light and other radiant energy are consistent with waves in
nature, but in interaction with matter, electromagnetic energy behaves as though it consists of
discrete pieces or
Answer: Quanta

294. Boilers using soft coal must have ____ furnace volume to reduce the danger of ____
Answer: Large; smoking

295. Which of the following would you apply if a person got Freon in his eyes?
Answer: Sterile mineral oil

296. The compressor will run continuously if there is:


Answer: Any of the other choices
(Too heavy a load, Insufficient refrigerant, Air in the system)

297. A hot crankcase and cylinder head accompanied by a low suction pressure would be
caused by
Answer: Insufficient refrigerant

298. One foot water is equal to:


Answer: 62.43 lb/in2

299. The purpose of the oil trap is:


Answer: To remove oil from the refrigerating gas

300. Many pressure gauges on a Freon system have two dials or graduations on one gauge.
The two dial represents:
Answer: Pressure and temperature

301. The faster way to remove frost from a direct expansion finned tube evaporator is to:
Answer: Send hot gas through the coil

302. When ordering an expansion valve which of the following information is necessary?
Answer: Size, tonnage, temperature, and pressure

303. The elements of a thermostat switch are usually of the ____.


Answer: Bimetal type
304. Which of the following characteristics that is not desirable in a refrigerant?
Answer: Low latent heat of vaporization

305. What do bodies at a temperature above absolute zero?


Answer: It emits Thermal Radiation

306. The Horizontal Scale (Abscissa) in the psychrometric Chart represents:


Answer: Dry bulb temperature

307. The thermostatic expansion valve is designed to maintain constant ____.


Answer: Superheat

308. The charging connection in a refrigerating system is located:


Answer: Between the king valve and the solenoid valve

309. The latent heat of fusion of ice is


Answer: 144 Btu

310. The ratio absorbed by the transfer fluid to the original incident energy striking the
collector
Answer: Collector efficiency

311. If m is the mass of dry air and H is the specific enthalpy of the water vapor in air and r is
the humidity ratio, then determine the latent heat of any dry air.
Answer: m(r x H)

312. The dehydrator is located between the:


Answer: Receiver and expansion valve

313. In a vapor compression cycle the lowest temperature is found in


Answer: Evaporator

314. The volume flow passes through a venturimeter is:


Answer: Constant

315. Anchor bolts in a machine foundation should be embedded in concrete of at least how
many time of the bolt diameter?
Answer: 30

316. Which do you think is the effect of “subcooling”?


Answer: It reduces the horsepower per ton of refrigeration
317. When air is saturated, the wet bulb depression is:
Answer: Zero

318. A precooler, if used is located between the:


Answer: Condenser and expansion valve

319. A Kaplan turbine is:


Answer: Low head axial flow turbine

320. The refrigerant temperature is at its maximum just before it enters the ____
Answer: Condenser

321. The machine foundation must have a factor of safety of:


Answer: 5

322. In a refrigerating system, the heat absorbed in the evaporator per kg mass of refrigerant
passing through
Answer: Equals the increase in enthalpy

323. Which of the following is the common classification of ducts?


Answer: All of these
(Conditioned Air – Duct, Recirculating, Fresh – air ducts)

324. All of the following process can be found in a psychrometric chart except:
Answer: Natural convection

325. If an electrically-operated compressor failed to start the cause might be:


Answer: Any of the other choices
(Burned out holding coils in solenoid valve, An open switch, A blown fuse)

326. Five pounds of water heated to raise the temperature 2F requires how many BTU?
Answer: 10 Btu

327. Air circulation in the icebox is accomplished by the use of which of the following?
Answer: Diffuser fans

328. In sensible heating cooling following parameter remains unchanged


Answer: Humidity ratio

329. Venturi meters, pitot static gauges, orifice meters, flow nozzles, and differential
manometers all depend upon the relationship between:
Answer: Flow velocity and pressure
330. It is used to deliver widespread, fan-shaped flows of air into the room
Answer: Diffuser

331. What turbidimeter that gives direct reading in ppm?


Answer: Jackson turbidimeter

332. How much will be removed by one-ton refrigeration unit?


Answer: 200 Btu per min

333. A double-trunk piston is used to:


Answer: All of other choices
(Prevent oil from mixing with the refrigerant, Prevent gas from getting to crankcase, Prevent
oil from mixing with the refrigerant)

334. Purpose of the receiver is to


Answer: Store the refrigerant

335. Latent heat


Answer: Cannot be measured with a thermometer

336. A type of water turbine where a jet of water is made to fall on the blades or buckets and
due to the impulse of water, the turbine starts to move.
Answer: Pelton wheel

337. When does the refrigerant gives-up heat?


Answer: When it condenses

338. A term used to mean the corrective steps taken to return the environment to its original
condition
Answer: Remediation

339. The back – pressure regulating valve:


Answer: Maintains a fixed pressure in the evaporator coils

340. What is the relationship of the horse power of a centrifugal pump, Hp, to the impeller
speed, S, if the pump is at two different rotative speeds?
Answer: Hp is directly proportional to the cube of the ratio of S

341. What must be the value of the available Net Positive Suction Head (NPHS) of a
centrifugal pump compared to its required NPHS to avoid losing priming?
Answer: Available NPHS greater than required NPHS
342. The diesel engine foundation safe soil bearing pressure is:
Answer: 4,890 kg/cm2

343. Aside from maintain appropriate temperature for food cold storage, how is desiccation
minimized or decreased?
Answer: Maintain humidity ratio

344. If C is the capacity of the compressor in a refrigeration system and F is the heat rejection
factor, then calculate the condenser load, L.
Answer: L = C x F

345. The following examples of indirect (secondary) measurements to measure flow rates
using obstruction meters except:
Answer: Weight and mass scales

346. When required, a regulator water valve in refrigerating system should be


Answer: In the water outlet

347. What is the relative humidity when the dew point and dry bulb temperature are equal?
Answer: 100%

348. Compare in refrigerating effect per unit mass of refrigerant circulated for a superheated
cycle that produces useful cooling and a saturated cycle, for the same vaporizing and
condensing temperature
Answer: Greater for a superheated cycle

349. If PV is the power required for a vapor compression refrigeration system, then what is the
power required for an air refrigeration system, assuming that they have the same capacity?
Answer: 5PV

350. What do you call the mixture if the solute particles of a solid suspended in a liquid are
larger than molecules?
Answer: Suspension

351. Lithium bromide is used in refrigeration system in:


Answer: Absorbers

352. The removal of dissolved gas or other volatile component from liquid by exposing the
liquid to air or steam is known as:
Answer: Stripping

353. Which of the following measures the density of salt in water?


Answer: Salimeter
354. In a power driven pump, each piston stroke is displaced by 360 divided the ____.
Answer: Number of cylinders

355. What is the boiling temperature of F 22?


Answer: -40 C

356. What is another name of discharge pressure?


Answer: Head pressure

357. If the compressor were to run out continuously without lowering the temperature, the
trouble would probably be:
Answer: Any of the other choices
(Insufficient refrigerant in the system, Leaky discharge valve, Leaks in the system)

358. The ratio of sum of individual maximum demands of the system to the maximum demand
of the whole system is known as
Answer: Diversity factor

359. The temperature bulb of solenoid valve is attached to the ____.


Answer: Wall of the icebox

360. What must be done to change the direction of rotation of a 440-volt, 3-phase induction
motor?
Answer: Interchange any two power leads

361. In order to remove the fly ashes from the flue gas, which of the following must a power
plant be equipped with?
Answer: Electrostatic precipitator

362. Which of the following is a type of water turbine?


Answer: Pelton

363. In sensible heating the absolute humidity remains constant but the relative humidity:
Answer: Decrease

364. All of the following process can be found on a psychrometric chart except?
Answer: Black body radiation

365. When checking zinc plates in a condenser, one should:


Answer: Clean the plates and renew worn out ones

366. Turbidity in water is due to:


Answer: Finally divided particles of clay, silt and organic matter
367. The force when applied to a mass of one kilogram will give mass an acceleration of one
meter per second for every second called:
Answer: Newton

368. A precooler is sometimes installed between the


Answer: Condenser and expansion valve

369. A type of polymer used for detergents, milk bottles, oil containers and toys
Answer: High density polyethelene (HDPE)

370. The fact that the amount of slightly soluble gas absorbed in a liquid is proportional to the
partial pressure of the gas is known as:
Answer: Henry’s Law

371. What is the pressure at the exit of a draft tube in a turbine?


Answer: Atmospheric

372. What do you call the device that is used as low-pressure control and high-pressure cutout
on a compressor?
Answer: Pressure Controller

373. Redox reactions can often be type of:


Answer: Double replacement

374. Excessive head pressure is caused by


Answer: Any of the other choices
(Dirty condenser tubes, Insufficient cooling water to condenser, Air or non condensable gas
in the system)

375. In which part of the vapor compression cycle there is abrupt change in pressure and
temperature?
Answer: Expansion valve

376. The kinetic energy of a moving fluid is used to isentropically compressed the fluid to
state of zero velocity. The temperature of a moving fluid at the state zero velocity is called:
Answer: Critical temperature

377. Which is not commonly used to cool and dehumidify equipment?


Answer: Calcium chloride

378. What is the primary cause of smog formation?


Answer: Nitrogen oxides
379. The dehydrating agent in a Freon system is usually
Answer: Activated alumina

380. The solenoid valve is actuated by which of the following?


Answer: A magnet

381. Which of the following does not use ambient air for propulsion?
Answer: Turbo-prop

382. Which of the following would not cause high suction pressure?
Answer: Insufficient refrigeration

383. What is the effect of superheating the refrigerant?


Answer: It increase the coefficient of performance

384. The color of the flame of halide torch, in case of leakage of Freon refrigerant, will change
to
Answer: Green

385. If there is too much lube oil in the system what must be done?
Answer: Remove same at once

386. If the thermal bulb becomes loose on the evaporator coils, it will cause ____.
Answer: Improper operation of expansion valve

387. The effect of superheating the refrigerant is to:


Answer: Decrease the COP

388. In power plant electrostatic precipitators is installed between:


Answer: Furnace and chimney

389. Equipment leaks from plant equipment are known as


Answer: Fugitive emissions

390. Per capita consumption of water is generally taken as:


Answer: 150-300 liters

391. What is the heat that is removed from the space to be cooled, which is the same as the
heat absorbed by the cooling coils?
Answer: Refrigerating effect

392. Horsepower per ton of refrigeration is expressed as:


Answer: Cop/4.75
393. What must be done first when opening a single packed stop valve?
Answer: Loosen the packing before opening

394. The amount of CO2 or Freon in a cylinder is measured by


Answer: Weight

395. The coefficient of velocity is approximately


Answer: 0.30 to 0.50

396. Carries needed to deliver air to the conditioned space it is made of sheet of metals like
aluminum, galvanized sheet steel and some structural materials that will not burn
Answer: Duct

397. Which of the following best described a Freon?


Answer: All of the other choices
(Colorless, Odorless, Non-poisonous)

398. Before securing a compressor to do maintenance work on it, be sure to:


Answer: Have spare parts ready and pump down the system

399. A bull’s eye in a full liquid line will appear ____.


Answer: Clear

400. What operates low-pressure cutout switch?


Answer: Bellows

401. A liquid mixture having constant maximum and minimum boiling points. Refrigerants
comprising this mixture do not combine chemically, yet the mixture provides constant
characteristics.
Answer: Azeotropic Mixture

402. Are organic compounds manufactured in oily liquid and solid forms through the late
1970s and subsequently prohibited.
Answer: Polychlorinated biphenyls (PCBs)

403. What is the temperature range of an air-conditioning application where dry air can be
considered ideal gas?
Answer: -10 to 50 C

404. The coefficient of the velocity, Cv accounts for the:


Answer: Small effect on the flow area of contraction, friction, and turbulence
405. During the re-expansion portion of the refrigeration compressor cycle
Answer: The suction valve is closed and the discharge valve is closed

406. Refers to the high-temperature removal of tarry substances from the interior of the carbon
granule, leaving a highly porous structure
Answer: Activated

407. Are by products of reaction between combustion products


Answer: Oxidants

408. Are any solid particulate matter that becomes airborne, with the exception of particulate
matter emitted from the exhaust stack of a combustion process
Answer: Dusts or fugitive dusts

409. The purpose of the dehydrator is to


Answer: Remove moisture from the refrigerant

410. The angle between the stack and guy wire is usually
Answer: 60 degrees

411. A refrigeration that deals with producing temperature of -157 C or lower.


Answer: Cryogenics

412. The purge valve is located:


Answer: In the highest part of the system

413. Thermal expansion valves are usually made of the:


Answer: Diaphragm and Bellows type

414. The purpose of the expansion valve by-pass is to


Answer: Controls the refrigerant to the evaporator in case the automatic valves fail

415. The water regulating valve is operated by the


Answer: None of the other choices
(Compressor discharge pressure, Compressor suction pressure, Compression discharge
temperature)

416. Superheat is heat added ____.


Answer: After all liquid has been changed to vapor

417. The ratio of maximum load to the rated plant capacity is called
Answer: Utilization factor
418. The highest temperature in vapor compression cycle is produced during:
Answer: Compressor discharge

419. Dry analysis is a fractional analysis of the products of combustion which does not
include:
Answer: Water vapor

420. Valves and piping in an ammonia system are made of:


Answer: Iron

421. The locus of elevation to which water will rise in the piezometer tube is termed:
Answer: Hydraulic gradient

422. A substance with surface area per unit of weight, and intricate pore structure, and a
hydrophobic surface
Answer: Adsorbent substance

423. The purpose of the expansion valve is to control the flow of the refrigerant to the
evaporator. The other function is to:
Answer: Reduce the pressure of the liquid refrigerant

424. An ammonia leak will turn litmus paper ____.


Answer: Blue

425. What is the most common freon gas used in reciprocating compressors?
Answer: F-12

426. The empirical coefficient e in machine foundation if not given is assumed


Answer: 0.11

427. The capacity of a centrifugal type compressor is controlled by which of the following?
Answer: Regulating the speed or regulating the suction pressure

428. Antifreeze chemicals are:


Answer: Those that lower down the freezing points of liquids

429. A type of refrigerant that will not damage the ozone layer
Answer: Hydrofluorocarbons (HCF’s)

430. At 0 psig, how may BTU necessary to change 1 lb of water at 212 F of steam?
Answer: 970
431. Modern stacks are seldom built higher than ____.
Answer: 60 m

432. If Pi is the indicated horsepower and Pb is the indicated horsepower of a compressor,


then what is mechanical efficiency, Em, equal to:
Answer: Em = Pi/Pb

433. A refrigeration control that guards the compressor forms overloads brought about by
abruptly increases loads resulting from defrosting, warm products, and others is called:
Answer: Suction hold-back valve

434. When a solvent has dissolve as much as it can, the mixture is called:
Answer: Saturated solution

435. Fan motors in air conditioning usually have:


Answer: 2 or 3 speeds

436. In the upper atmosphere, ozone is made by ultraviolet light reacting with:
Answer: Oxygen

437. Which of the following is the oil used in a refrigeration system?


Answer: Straight mineral oil

438. The following are all examples of indirect (secondary) miscellaneous methods to measure
flow except:
Answer: Positive displacement meters

439. Which of the following is the function of a suction pressure regulating valve?
Answer: Maintains proper back pressure

440. What is the use of back pressure regulating valve?


Answer: Maintains a fixed pressure in the evaporator coils

441. If Wt is the turbine shaft work of a gas turbine unit, Wc in duel, then determine its
thermal efficiency.
Answer: (Wt-Wc) / Q

442. The solenoid valve is controlled by


Answer: The temperature in the icebox

443. Where is the excess refrigerant removed?


Answer: Charging side of the system
444. All of the following units of energy except
Answer: Pascals

445. What usually happened if brine has a high specific gravity?


Answer: It will crystallize

446. There are three basic boiler types, namely;


Answer: Cast-iron, fire-tube and water tube boilers

447. Measurements of a device’s ability to remove atmosphere air from test air.
Answer: Atmospheric Dust Spot Efficiency

448. The normal cut-out setting of a window unit thermostat between:


Answer: 13 C to 16 C

449. The use of water to carry heat occupied spaces


Answer: Hydronic Heating System

450. Reheating process is normally employed in steam plant when:


Answer: Turbine undergoes excessive moisture

451. In the discharge line between the compressor and the condenser one would find:
Answer: High pressure, high temperature gas

452. A good refrigerant should be


Answer: All of the other choices
(Non-poisonous, non-inflammable, non-explosive)

453. What is the relationship of the horse power of a centrifugal pump, Hp, to the impeller
speed, S, if the pump is at two different rotative speeds?
Answer: Hp is directly proportional to the cube of the ratio of S

454. Which of the following is the reason when the crankcase is cooler than the suction line?
Answer: Too much refrigerant

455. Humidity is a measure of which of the following?


Answer: Water vapor content

456. High head turbine is a/an:


Answer: Impulse

457. Critical temperature is that temperature above which


Answer: A gas will never liquefy
458. The fact that a fluid’s velocity increases as the cross-sectional area of the pipe through
which it flow decrease is due to:
Answer: The continuity equation

459. Which of the following is important for evaporator coils?


Answer: It should have air completely surrounding them

460. Which of the following types of air dryers works by absorbing moisture on a solid
desiccant or drying material such as activated alumina, silicone gel, or molecular sieve?
Answer: Deliquescent dryer

461. Which of the following would cause the compressor to run continuously?
Answer: Low pressure switch jammed

462. In a window air conditioning unit which of the following is usually done by the owner?
Answer: Semi-annual cleaning or replacement of filters

463. Which of the following is not similarity between submerged culvert and a siphon?
Answer: Torricelli’s equation holds

464. Where does the final removal of water vapor in an absorption refrigeration system occur?
Answer: Rectifier

465. The constant spending of certain percentage of circulated water in a cooling tower in
order to prevent accumulation of dissolved mineral solids and other impurities in the
condenser water is called
Answer: Bleed-off

466. Which of the following refrigerant is added sometimes to other refrigerant to improve oil
circulation?
Answer: R – 170 (Ethane)

467. The suction pressure switch is operated by which of the following?


Answer: Pressure on a bellow

468. When the air is saturated the wet-bulb depression is:


Answer: Zero

469. Heating and dehumidification can be obtained simultaneously if air is passed through:
Answer: Either a solid absorbent surface or a liquid absorbent spray
470. In cooling cycle, the dry bulb temperature (db) of the air is lowered. When this happens
the relative humidity
Answer: Increase

471. What will happen when the pressure at any point inside a centrifugal pump goes below
the vapor pressure corresponding to the temperature of the liquid?
Answer: Cavitation

472. The ratio between the actual power and the apparent power in any circuit known as the
____ of that circuit
Answer: Power factor

473. Refrigerant leakage from the compressor crankcase is prevented by


Answer: Using shaft seals

474. The matching of scale and full-scale results for a fluid dynamic phenomena involving
compressible fluids requires quality of:
Answer: Mach number

475. Ammonia leaks in the condenser can be detected by:


Answer: Applying litmus paper to the circulating water discharge

476. A swinging support constructed as part of the vessel and that supports the manway cover
when it is unbolted and moved aside.
Answer: Davit

477. Which of the following type valves are not found on a Freon – 12 system?
Answer: Duplex

478. What is the main function of a receiver?


Answer: Store the refrigerant

479. The dividing point between the high pressure and low pressure sides of the refrigeration
cycle occurs at the
Answer: Expansion valve

480. If the solute particles of a solid suspended in a liquid are larger than molecules the
mixture is known as?
Answer: Suspension

481. A mechanism that removes moisture


Answer: Dehumidifiers
482. Is a water soluble organic compound prepared from ammonia. It has significant
biological and industrial usefulness
Answer: Urea or carbamide urea

483. Which of the following fans in air conditioning systems which can be classified as
centrifugal flow
Answer: Propeller fan

484. What is the use of the low – water cutout switch?


Answer: Stops the compressor when there is insufficient cooling water

485. In parallel pipe system originating and terminating in common junctions,


Answer: Pressure drops through each branch are equal

486. When the dry bulb and the wet bulb temperature are identical, the air is said to be:
Answer: Saturates

487. The temperature in which water vapour in the gas begins to condensate in a constant
pressure process.
Answer: Dew point

488. Combination gas/fuel oil burners permit the operator to switch from one fuel to ____
Answer: All of the above
(For economy, Because of a shortage of fuel being used, Because of a failure in the fuel
system being used)

489. Most nuclear particles can react with atoms in several different ways including
Answer: Absorption and scattering

490. What is the effect of superheating the refrigerant?


Answer: It increase the coefficient of performance

491. Boiling temperature of Freon 12 is


Answer: -29.8 C

492. Which of the following would cause low head pressure?


Answer: Too much cooling water and/ or insufficient refrigerant gas

493. At what temperature will water normally turns to steam?


Answer: 212 F

494. What will happen if the expansion valve is opened too wide?
Answer: Liquid will flood back to the compressor
495. All of the following are words used to describe neutron kinetic energy levels except:
Answer: Supersonic

496. If the thermal expansion valve becomes inoperative, the ice boxes will have to be
controlled by the ____.
Answer: Manual expansion valve

497. If the superheat on the suction side of the compressor is increased, what will happen to
the tonnage capacity of the unit?
Answer: Decreases

498. The solenoid valve can be typed as a ____.


Answer: Magnetic soap valve

499. The ratio of fugacity of actual conditions to the fugacity at some reference state is known
as:
Answer: Activity

500. The process that takes place in the evaporator called


Answer: Absorption of the latent heat of vaporization

501. When charging freon system, all the valves should be


Answer: King (liquid) valve

502. If the pressure exerted on a liquid is higher than the saturation corresponding to it
temperature, the liquid is a:
Answer: Sub cooled liquid

503. The most likely cause of high superheat would be:


Answer: Expansion valve closed too much

504. A good refrigerant should have a


Answer: High latent heat

505. What is the device used to protect the compressor from overloading due to high head
pressure
Answer: Overload relay

506. Which of the following it to be checked as part of weekly maintenance schedule of a


console air conditioners?
Answer: Cooling towers
507. Which of the following would not be cause for a refrigerating system to short cycle on hp
cutout?
Answer: Discharge valve leaking

508. A ____ boiler has heat and gases of combustion that pass through tubes surrounded by
water
Answer: Fire tube

509. A type of polymer used for clear bottles.


Answer: Polyvinyl Chloride (PVC)

510. In an ammonia system, the oil gauge must be kept:


Answer: Closed except when checking level oil

511. If a compartment requires the removal of 36,000 BTU per hour, how much is necessary
compressor capacity?
Answer: 3 tons

512. The oil level in the compressor should be checked:


Answer: After a long period of operation

513. Which of the following industries have the highest consumption of water for processing?
Answer: Paper mill

514. What is the lowest temperature to which water could possibly be cooled in a cooling
tower?
Answer: The temperature of adiabatic compression

515. The method of cooling which primarily used where ambient air temperatures are high and
relative humidity is used:
Answer: Swamp cooling
IPE Exit Exam Reviewer

1. What tonnage of refrigerating machine is required if the refrigerating system extracted


48,000 btu per hour?
Answer: 4 tons

2. An insulator with superior tensile strength but low heat resistance


Answer: Fiberglass

3. What is the combination of a wet and dry bulb thermometer is called a


Answer: Hygrometer or psychrometer

4. The weight of steel bar reinforcement show be how many time the weight of the foundation?
Answer: ½% to 1%

5. Is a high temperature process that turns incinerator ash into a safe, glass – like material
Answer: Vitrification

6. Oil is added to a freon compressor by


Answer: Pumping in with a hand pump

7. The difference between the theoretical draft and the draft loss
Answer: Available draft

8. The purpose of expansion valve by-pass is to


Answer: Controls the refrigerant to the evaporator in case the automatic valves fail

9. Load curve refers to the plot of:


Answer: Load versus time

10. What does a negative Joule-Thompson coefficient means during a throttling process?
Answer: fluid temperature rises

11. The maximum unit pressure of turbine and generator on the reinforced concrete should not
exceed
Answer: 17.62 kg/cm2

12. Water in vapor form remains a vapor as long as temperature is what relations to the dew
point temperature
Answer: below
13. The “refrigerating effect” of a refrigerant is a always:
Answer: Less than its latent heat

14. A burner should always start up in ___ fire and shut down in ___ fire
Answer: Low; low

15. A ___ boiler has water in the tubes and heat gases of combustion passing through the tubes
Answer: Water tube

16. If the compressor had been running satisfactorily for a long period of time but suddenly the
compartment temperature started to rise, the trouble might be:
Answer: Any of the other choices
(A refrigerant leak has developed, The solenoid valve has jammed shut, The expansion valve
may contain frozen water)

17. A type of polymer used for Styrofoam cups and clam shell food containers
Answer: Polystyrene (PS)

18. Mycotoxins are poisonous chemicals produced by:


Answer: Molds

19. The magnitude of the drag coefficient of a sphere in water is dependent upon all of the ff
except
Answer: Units of measure (SI or English engineering system)

20. A refrigeration system in which only part of the refrigerant passes over the heat transfer
surface is evaporated and the balance is separated from the vapor and recirculated
Answer: Flooded system

21. The boiling point of freon-12 at atmospheric pressure is


Answer: -22F

22. Which of the following vital components of the refrigeration system where both temperature
and pressure are increased?
Answer: Compressor

23. Which of the following must be done to eliminate frost on the discharge pipe of the
compressor?
Answer: None of the other choices
(Open the expansion valve, crack bypass valve, Regulates water to the condenser)

24. Orifice coefficients are used to determine:


Answer: Energy losses
25. Which of the following is used as high head turbine?
Answer: Impulse

26. A water-tube condenser has a total of 60 tubes. If these two passes, then compute the number
of tubes per pass
Answer: 15

27. Absolute zero is


Answer: 460 degrees below zero on the Fahrenheit scale

28. Which of the following can be the cause of low head pressure
Answer: Any of the other choices
(Leaky discharge valves, Too much or too cold condensing water, insufficient charge of
refrigerant)

29. What is the most common freon gas used in centrifugal compressors?
Answer: F-11

30. When purging an ammonia condenser into a bucket of water, one can tell when the air is out
and ammonia starts to come through the:
Answer: Change of bubbling sound of air to the cracking sound of ammonia

31. In the deep well installation or operation, the difference between static water level and
operating water level is known as:
Answer: Drawdown

32. White ice is


Answer: Due to dissolved air, gasses and impurities

33. Which of the following is a characteristic of an impulse turbine


Answer: Steam striking blades at zero angle

34. Heavy water is:


Answer: D2O

35. When excess solute in a solution settles to the bottom of the container, the process is called:
Answer: Precipitation

36. Calcium chloride is sometimes used in refrigeration system as a:


Answer: Secondary Coolant

37. If a compressor runs continuously, the cause might be a:


Answer: Stuck low-pressure switch
38. The thermal expansion valve is located between the:
Answer: Solenoid valve and the evaporator coils

39. The cooling water regulator is automatically actuated by which of the following?
Answer: The discharge pressure of the refrigerant

40. Based on the PSME code, what should be provided in each steam outlets if two or more
boilers will be connected in parallel?
Answer: Non-return and shut off valves

41. The amount of sensible heat for a sensible heat ratio of 0.8 and a total cooling load of 100 is
Answer: 80

42. Which refrigerant is used for the air conditioning of passenger aircraft cabin
Answer: Air

43. When a falling object reaches a speed at which the drag force equals its weight, it has
achieved:
Answer: A turbulent boundary layer

44. The minimum vertical distance from the floor or soil level to the top edge of the foundation
must be around
Answer: 120 mm

45. How many moisture be removed from air?


Answer: condensation and absorption

46. The charging valve is located between the:


Answer: King valve and the expansion valve

47. A “cold diffuser” is a:


Answer: Type of evaporator

48. Which of the following stops the compressor before the relief valve opens?
Answer: High pressure control

49. What is the excess refrigerant removed?


Answer: Charging side of the system

50. Which of the following is used in measuring the density of a brine solution?
Answer: A hydrometer
51. All of the following are common types of chemical reaction except
Answer: Fission

52. A ___ after each radiator allows ___ to pass through to the return line
Answer: Steam trap; condensate

53. A leaky solution valve can usually detected by:


Answer: Any of the other choices
(A fluctuating suction pressure gauge, a higher suction pressure, closing in on the suction
valve having no effect on the suction pressure)

54. If the cooling water to the condenser suddenly fails:


Answer: The compressor will shutdown

55. If Et is the total net energy generated by a plant in a certain period of time and Er is the rated
net energy capacity of the plant during the same period of time, then what is the plant
operating factor?
Answer: Et/Er

56. A thermometer sense which of the following?


Answer: Sensible heat

57. Is the subjective method in which the smoke density is visually compared to five
standardized white-black grids
Answer: Ringelman scale

58. Excessive head pressure is caused by


Answer: Flooded condenser tubes or cooling water temperature too high

59. What do you call a plate or vane used to direct or control movement of fluid or air within the
confined area?
Answer: Baffle

60. The total pressure supplied by the fan at maximum operating conditions
Answer: Net rating or fan boost

61. The system should be purged:


Answer: After the system has been shut down for few hours

62. What is likely to occur when sections of the impeller of a centrifugal pump are handling
vapor and other sections are handling liquid.
Answer: complete failure to operate
63. What is the chemical formula of Ozone?
Answer: O3

64. An excessively high head pressure would be caused by:


Answer: Insufficient cooling water to the condenser

65. When an electric motor nameplate indicate a “100-kW electric motor” then what does a 100-
kW rating refers to?
Answer: Mechanical power output

66. During a cooling and dehumidifying process, sensible and latent heats are removed in the
cooling coil. If Hs is the sensible heat and Ht is the total heat transferred, then determine the
coil sensible factor.
Answer: Hs/Ht

67. A single trunk piston-type compressor is undesirable for a Freon unit because the:
Answer: Lubricant mixes with the refrigerant

68. What is the first step that must be done when securing a system?
Answer: Close receiver (King) valve

69. The amount of CO can be determined by the color of Palladium Chloride. An amount of 30
ppm to 70 ppm will cause:
Answer: Slight Darkening

70. The water hammer phenomenon is primarily what kind of fluid mechanism?
Answer: Dynamic (a time-dependent phenomena)

71. The other name for Swamp Cooling is:


Answer: Wet Roof Cooling

72. The design of an air supply duct of an air condition system


Answer: affects the distribution

73. A Freon – 12 gage shows pressure and ____.


Answer: Saturation temperature

74. High superheat of the vapor in the system would cause


Answer: A decrease in capacity

75. When there is no work between the thermodynamic system and its surrounding, the quantity
of net heat transfer is equal to:
Answer: total energy of a closed system
76. The relation between the Fahrenheit absolute scale and the Celsius absolute scale is:
Answer: R = 1.8 Kelvin

77. External frost on inlet of expansion valve indicates:


Answer: Expansion valve plugged or dirty

78. The total cross section of a target atom is made up of


Answer: The absorption and scattering cross sections

79. The motor condition of an air conditioning unit can be checked with:
Answer: The continuity of light or with an ohmmeter

80. A number used to compare energy usage for different areas. It is calculated by dividing the
energy consumption by the footage of the conditioned area.
Answer: Energy utilization Index

81. What is the effect if the refrigerant is removed from the system too fast?
Answer: It may “freeze – up” the condenser

82. After the total force of the steam has lifted the safety valve off its seat, the steam enters the
____.
Answer: Huddling chamber

83. By which of the fooling processes heat mainly dissipates in cooling towers?
Answer: Evaporation

84. All of the following fluid phenomena are based on the force momentum principle of a
flowing fluid except:
Answer: Diesel automobile engines

85. All of the following are forms of drag on a body moving through a fluid except
Answer: D’alembert’s paradox drag

86. What is the use of the suction pressure regulating valve?


Answer: Maintains the back pressure in the evaporator coils

87. There shall be no foundation bolts less than _____.


Answer: 12 mm in diameter

88. The ratio of the rated cooling capacity divided by the amount of electrical power used:
Answer: Energy efficiency ratio (EER)
89. The scale trap is located between the:
Answer: King (liquid) valve and expansion valve

90. The temperature in the vegetable box should be approximately


Answer: 35 to 45 deg. F

91. A thermostat is a
Answer: Temperature-operated switch

92. The bulb for the thermal expansion valve is located:


Answer: Near the evaporator coil outlet

93. The coefficient of velocity is the ratio of the:


Answer: Actual velocity to the theoretical velocity

94. What is the specific humidity of dry air?


Answer: 0

95. What is the reason why a thermometer in vapor compression system is installed close to the
compressor?
Answer: Because it helps the operator in adjusting compressor for greatest efficiency

96. As the steam temperature increases the steam temperature:


Answer: Increases

97. Substances that have the ability to absorb moisture from the air are called:
Answer: Desiccants

98. Evaporative condenser is used to cool


Answer: condenser vapor

99. If the head pressure is too high:


Answer: The high pressure cutout switch should operate before the relief valve opens

100. A material or substance that is accidentally or intentionally introduce to the environment


in a quantity that exceeds what occurs naturally
Answer: Pollutant

101. The greatest decrease in refrigerant temperature occurs in the ____


Answer: Evaporator

102. The diesel cycle is the ideal cycle for a :


Answer: compression-ignition engine
103. The moisture in a refrigeration system can be removed with help of which of the
following?
Answer: Driers

104. What process is employed when the turbine steam power plants experience excessive
moisture?
Answer: reheating

105. The path that directs the flow of refrigerant through the compressor
Answer: Valve

106. Which of the following is another name for the liquid valve?
Answer: King valve

107. Expansion factors take into account the:


Answer: Effects of compressibility

108. The term used to express the amount moisture in a given sample of air. It is compared
with the amount of moisture in a given sample of air.
Answer: Relative Humidity

109. What are the four basic methods of determining whether the proper amount of refrigerant
being added to the system?
Answer: Bull’s eye, weight, pressure and frost line

110. Which of the following components of the window air conditioning system must be
cleaned annually?
Answer: All of these
(Evaporator and Condenser, Motor and Compressor, Fan Blades and Fan Motor)

111. At present, the number of true elementary particles, which include leptons and quarks is
thought to be
Answer: 12

112. All of the following statements about conjugate acids and bases are true except
Answer: A conjugate base results when a base accepts a proton

113. A double-seated valve allows the valve to be


Answer: Packed in the wide open or closed position

114. In order to keep combustion products inside the combustion chamber and stack system,
balanced draft system may actually operate with a slight
Answer: negative pressure
115. What is the cause of pressure drop in the boiler, condenser and the pipings between
different components? Because of this drop, steam leaving the boiler at a lower pressure.
Answer: Fluid friction

116. It is a form of oxygen photochemically produced in nature.


Answer: Ozone

117. Absolute zero on the Fahrenheit scale is equal to


Answer: -460

118. Low suction pressure is caused by:


Answer: Solenoid valve not functioning properly

119. What do you call the storage tank for liquid refrigerant?
Answer: Receiver tank

120. Why should you avoid bending or twisting of fan blades in an air conditioning unit?
Answer: It will wear out the motor bearings and cause noise

121. All of the following statements are characteristics of bases except


Answer: They have a pH between 0 and 7

122. Concrete foundation should have steel bar reinforcement placed vertically and
horizontally to avoid ____.
Answer: Thermal cracking

123. The pump will not cavitate if the available Net Positive Suction Head (NPSH) is:
Answer: equal or greater than the required NPHS

124. What will happen to the capacity if the superheat is increased on the suction side?
Answer: Decreases

125. When heavy electrical currents are involved, the thermostat will be operated by a
Answer: Relay

126. Valves in piping in an ammonia system are made of


Answer: Iron

127. Which of the following is not essential to a compression refrigeration system?


Answer: A receiver

128. Nitrogen occupied almost how much of the Earth’s atmosphere?


Answer: 3/4
129. If frost forms on the cylinders, the cause would be:
Answer: Expansion valve open too wide

130. Which of the following gasket materials should be on Freon system?


Answer: Asbestos or Metallic

131. Which of the following would cause the crankcase and head to get hot with low suction
pressure?
Answer: Insufficient refrigeration

132. Moisture in a system will cause a:


Answer: Faulty expansion valve

133. The steam turbine foundation should be designed to support the machine load plus how
many percent for impact, condenser load, floor loads and dead loads?
Answer: 25

134. When air is heated, what happened to its relative humidity?


Answer: Remain constant

135. Sweating of crankcase is caused by which of the following?


Answer: Too much oil in the system

136. What do you call a material in a dryer?


Answer: Desiccant

137. The matching model and full-scale prototype results a fluid dynamic phenomena
involving a fully submerged body requires equality of:
Answer: Reynolds number

138. Which of the following uses a secondary refrigerant?


Answer: Ice plant

139. When the outlet temperature at the evaporator exceeds the inlet temperature, the
condition is known as:
Answer: Superheating

140. In series pipe systems, all of the following parameters vary from section to section except
Answer: Mass flow

141. Which of the following is used to control refrigeration compressor capacity?


Answer: Unloader
142. TOR is a unit equivalent to:
Answer: 12.66 kN-m/hr

143. The agent used in an indirect reefer system is:


Answer: Calcium chloride or sodium chloride

144. If the W is the work output of a heat engine and H is the total heat input of an engine,
then determine its thermal efficiency, e.
Answer: e = W/H

145. Which of the following items is not important when using a halide torch?
Answer: Adjust to a clear white flame

146. A dehumidifier is usually a small hermitic refrigerating system. It has both a condenser
and an evaporator. Many older systems use R – 12 or R – 500. The newer units are:
Answer: R – 134a

147. What is the method used to evaluate all welds performed on pressure parts of boiler tube
materials?
Answer: Radiographic test

148. Faulty F – 12 compressor valves will be indicated by:


Answer: All of the other choices
(Gradual or sudden decrease in capacity, compressor running continuously, Low head
pressure – high suction pressure)

149. Heat exchanger in which water flows by gravity over the outside of tubes or plates.
Answer: Baudelot cooler

150. Which of the following is not a sound absorptive material?


Answer: Mirrors

151. The desirable temperature inside an air conditioned auditorium is:


Answer: 20 C

152. An odorless refrigerant whose boiling point varies over a wide range of temperatures.
Answer: Freon 12

153. The cooling water regulator is actuated by


Answer: Pressure of the refrigerant

154. The psychrometric chart in air conditioning determines the


Answer: Moist air conditions
155. Blowback of a safety valve is to prevent:
Answer: Chattering

156. When starting a refrigerant unit, be sure the water side of the condenser is ____.
Answer: Vented

157. A temperature measurement in an ordinary temperature which has constant specific


humidity.
Answer: Dew point temperature

158. What adding oil to a freon system, one must be sure that
Answer: All air is removed from the pump and fitting

159. The primary purpose of a turbine in a fluid is to:


Answer: Extract energy from the flow

160. Which of the following is to be checked as regular monthly maintenance schedule of a


console air conditioning units?
Answer: Duct dampers, registers and diffusers

161. A company is interested to produce a water turbine wherein only little energy is required
or necessary because the guide vanes are to be controlled. The turbine must be a:
Answer: Kaplan turbine

162. To check the lack of refrigerant or clogged refrigerant lines:


Answer: Installing service valves must be necessary

163. Condensate from the non return steam trap is pumped from the condensate return tank of
the:
Answer: Boiler

164. The locus of elevation is:


Answer: Hydraulic Gradient

165. Which of the following cannot be used to describe neutron kinetic energy levels
Answer: Freezing

166. It is important to take some moisture from the air dehumidify it if the relative humidity
reaches high levels. To do this, it requires cooling the air:
Answer: Below its dew point temperature

167. The vapor cycle in thermal efficiency as the refrigeration cycle to the
Answer: COP for a refrigerator
168. A scale trap in a Freon system will be found on the
Answer: Suction side

169. Which of the following is not used as method to measure air velocities?
Answer: Open type Barometer

170. Which of the following is considered as comfort condition in air conditioning?


Answer: 20 C DBT, 60% RH

171. Which of the following acts as ignition accelerator for internal combustion engine fuels?
Answer: Acetone peroxide

172. The purpose of the evaporator is to


Answer: Absorb latent heat of vaporization

173. The speed at which a reaction proceeds to equilibrium is the purview of


Answer: Reaction kinetics

174. What do you call the liquid reaching the compressor through the suction?
Answer: Flooding back

175. Which of the following draft rely on the stack effect to draw off combustion gases?
Answer: Natural draft

176. The relative humidity becomes 100% and where the water vapor starts to condense.
Answer: Dew point

177. Which of the following statements is correct? The relative humidity of an air water vapor
mixture.
Answer: is the ratio of partial pressure of water vapor to the saturation pressure at the
mixture temperature

178. The following are standard characteristics of Freon – 11 except:


Answer: Boiling point over 200 F

179. The bhp/ton refrigeration requirement with increase in condenser temperature. In a vapor
compression system that uses reciprocating compressor receiving refrigerant gas at constant
suction temperature, will follow:
Answer: Linearly increasing characteristic
180. In an air conditioning unit, the thermostat fails. The unit did not start. How do you test
the operation of the thermostat?
Answer: Cover the air outlet and air inlet with a cloth. The air will recirculate into the
unit and the temperature will quickly drop to the cut out temperature.

181. All of the following temperature have meaning in psychrometrics except:


Answer: Adiabatic wall temperature

182. The maximum continuous power available from a hydro – electric plant under the most
adverse hydraulic conditions is called
Answer: Firm power

183. Which of the following dehumidifier is often used to reheat the air after moisture is
removed?
Answer: Condenser

184. Which of the following is a type of evaporator?


Answer: shell-and-tube

185. All of the following factors affect rates of reaction except


Answer: Pressure

186. Which of the following would cause a high suction pressure?


Answer: Expansion valve open too wide

187. Zinc rods are found in the:


Answer: Salt water side of the condenser

188. The sum of the internal energy and the product of pressure and specific volume is known
as the:
Answer: enthalpy

189. Any foreign matter in the coal feed mechanism of the screw-feed stoker is best removed
by:
Answer: Using the cutoff gate at the bottom of the hopper

190. Based on the PSME code, what should be the humidity of air to be used for comfort
cooling?
Answer: 50% - 60% relative humidity
191. Which of the following must be checked up if an automatic Freon system will not start
up?
Answer: All of the other choices
(High pressure cutout, reset mechanism, Low-pressure cutout)

192. Refers to the use of composting soil beds


Answer: Biofiltration

193. What is the usual cop of a domestic refrigerator?


Answer: More than 1

194. In a hydro-electric plant using a Francis turbine with medium head, the speed can be
regulated using the
Answer: Wicket gate

195. The temperature bulb of the thermo-expansion valve is attached to which of the
following?
Answer: Evaporator coil outlet

196. Which of the following refrigerants would give the most trouble when operating with
warm circulating water?
Answer: CO2

197. A rule which states that any waste derived from the treatment of a hazardous waste
remains a hazardous waste
Answer: The derived from rule

198. A device for holding open the suction valve and drawing gas from the suction manifold
and returning it to the suction line without compressing it is called
Answer: Cylinder unloader

199. The intake pipe to a hydraulic turbine from a dam


Answer: Penstock

200. What consists of weak solutions of sulfuric, hydrochloric, and to a lesser extent, nitric
acids?
Answer: Acid rain

201. The expansion valve does not seem to be operating properly. There is high superheat.
Test by listening to the sound of Freon flooding through the tubes and ____.
Answer: Warm bulb with hand
202. Swimming pool water needs
Answer: de-chlorination

203. Oxygen is approximately what percent by weight in the atmosphere?


Answer: 23%

204. Which of the following could be used to check a leak in an ammonia system?
Answer: Litmus paper & Sulfur stick

205. The determination of properties and behavior of atmospheric air usually the purview of:
Answer: Psychrometrics

206. A valve the released steam from the boiler by operating at a pre-determined pressure in
order to keep the steam pressure from rising more than the pressure from which the valve is
set is called a
Answer: Safety valve

207. If the critical temperature of a refrigerant is too close to the desired condensing
temperature, the equipment must necessarily be of
Answer: Extra heavy construction

208. If a freon – 12 compressor trip out on “cut-out”, the solenoid valve closes by which of the
following?
Answer: An electrical release

209. Which of the following would cause the suction switch to remain open?
Answer: Bellows broken or jammed

210. In the upper atmosphere, ozone is made by ultraviolet light reacting with:
Answer: Oxygen

211. How is ammonia system purged so that operator will not be overcome by the fumes?
Answer: Into the bucket of water

212. A ton of refrigeration is equal to the removal of


Answer: 288,000 btu per 24 hrs.

213. As a good practical rule, the foundation depth may be taken as how many times of the
engine stroke?
Answer: 2:5 to 3:5

214. Most stacks are built of height less than _____.


Answer: 30 m
215. The phenomenon that warm air rise and cold air settle is called:
Answer: Stratification

216. Type of refrigerant control which maintains pressure difference between high side and
low side pressure in refrigerating mechanism:
Answer: Expansion valve

217. The crossover connection in an ammonia system can be used to ____.


Answer: Hot-gas defrost

218. Which of the following refrigerants has lowest freezing point temperature?
Answer: Freon 22

219. The operation that produces highest noise level is:


Answer: Riveting

220. Select the one in which secondary refrigerant is used


Answer: Ice plant

221. Which of the following is not essential to a centrifugal type compressor system?
Answer: Distiller

222. Two compressor should not be run in parallel because


Answer: There is a possibility of losing oil

223. Which of the following is also known as refrigerant no. R-717?


Answer: Ammonia

224. During sensible heating, the humidity remains constant but the relative humidity.
Answer: decreases

225. The term suction units is used with


Answer: Induced draft

226. If the compressor had been running satisfactorily for a long period of time but the oil
level was rising slowly, one should:
Answer: Shut down the compressor and check the oil level with the machine stopped

227. What amount of air is required in a low bypass factor?


Answer: Lesser

228. The reduction of nuclear radiation intensity (called attenuation) is accomplished by:
Answer: Shielding
229. The ozone concentration of 0.10 parts per million (ppm) is generally considered the
maximum permissible for how many hours exposure?
Answer: 8 Hours

230. Which do you think is very important in adjusting compressor “v” belts?
Answer: Allow ½” slack

231. An engineer inspected an air-conditioning unit. He found out that the unit does not
produce any cooling effect, however, the air-conditioning unit is running. He checked the
temperature of the condenser and evaporator and hat the unit run. He found out that there was
no change in temperature. What should he do?
Answer: Charge with new refrigerant

232. The temperature at which water vapor in the atmosphere begins to condense is known as:
Answer: dew point

233. The ice making capacity is always


Answer: Directly proportional to the refrigerating effect

234. If the compressor short-cycles on the high-pressure cutout, which of the following would
you check?
Answer: All of the other choices
(If plenty of cooling water is running through but it is not picking up heat, the condenser
tubes need cleaning, Be sure system is getting cooling water, Check for too much refrigerant
in the system)

235. What is the instrument used to register relative humidity?


Answer: Hygrometer

236. What has an octane rating of more than 100?


Answer: Benzol

237. A device used to keep moisture from passing through the system is called:
Answer: Dehydrator

238. If the freezing point of water is 0 C, which of the following is its melting point following
is its melting point
Answer: 0 C

239. What is the pressure present inside the casing of an impulse turbine?
Answer: Atmospheric pressure
240. A vena contracta in a fluid jet issuing through a hole in a plate is located approximately:
Answer: At jet’s minimum diameter

241. If the compressor short-cycles on the low-pressure cutout, the trouble might be:
Answer: Any of the other choices
(Too much frost on the evaporator coils, dirty traps and strainers, lack of refrigerant)

242. The amount of heat necessary to bring up temperature of a unit mass of a substance
through unit degree is called
Answer: Total heat

243. If the expansion valve capillary tube is pinched, which of the following be replaced?
Answer: All of the other choices (called the power element unit)
(Bulb, tube, diaphragm)

244. The coefficient of discharge is the ratio of the.


Answer: Actual discharge to the theoretical discharge

245. Pitot tube is used to measure the:


Answer: Velocity of flow

246. The principle of the centrifugal system is based on which of the following?
Answer: Kinetic energy

247. The disadvantage of a CO2 system over an ammonia system is the fact that
Answer: All of the other choices
(The pipes and fittings of a co2 system must be of the high-pressure type, The CO2 system
requires a larger prime remover, The CO2 system operates at a much higher pressure)

248. If ice will form in a solution of water and salt, then it is at temperature called:
Answer: Freezing point depression

249. The suction pressure in a Freon system should be:


Answer: The pressure which corresponds with a temperature about 20F below the
temperature of the icebox.

250. Wb is shaft work of an engine and Wi is indicated work of an engine. If mechanical is


present in the engine mechanism, then.
Answer: Wb is less than Wi

251. What is the relationship of the capacity of a centrifugal pump, Q, to impeller diameter, D,
when there are two impeller diameters in the same pump?
Answer: Q is directly proportional to the ratio of D
252. The combination of enthalpy and kinetic energy of fluid is termed as:
Answer: Stagnation enthalpy

253. Before any repair work is attempted on any gas-fired boiler the:
Answer: Main gas cock must be secured

254. An air-conditioning system in which water is chilled or cooled and which passes the
evaporator coils
Answer: Chilled water system

255. Refers to organic waste produced from biological water waste treatment processes
Answer: Biosolids

256. As far as combustion chamber design is considered, the maximum power output of a
given engine can be increased by:
Answer: Decreasing combustion chamber volume

257. A radioactive gas produce from the decay of radium within the rocks beneath a building
Answer: Radon

258. Ozone filters out what type of radiation that damages crops and causes skin cancer?
Answer: Ultraviolet radiation

259. The ratio of the sum of individual maximum demands of the system to the overall
maximum demand of the whole system
Answer: Diversity factor

260. A bell coleman cycle is also known as


Answer: Reversed Joule cycle

261. It is used deliver concentrated airstreams into a room. Many have one-way or two-way
adjustable air stream deflectors.
Answer: Register

262. The sensible heat ratio is 0.8. that is


Answer: 80% sensible heat and 20% latent heat

263. Closing the solenoid valve will stop the compressor through the ____.
Answer: Low-pressure cutout switch

264. A refrigerating unit of one (1) ton capacity can remove?


Answer: 200 btu’s per min
265. Which of the following is the usual case of slugging?
Answer: Expansion valve not operating properly

266. All of the following occur during reduction of a substance except


Answer: Loss of electrons

267. The secondary refrigerant used in milk chilling plants is generally


Answer: Glycol

268. Where is the solenoid coil installed?


Answer: Vertically over the valve

269. The thermal expansion valve responds to the


Answer: Amount of superheat in the vapor

270. To eliminate transmission of vibration, the foundation should be isolated from the floor
slabs of building footings at least how may mm around its perimeter?
Answer: 25

271. Which one is commonly used liquid absorbent?


Answer: Ethylene glycol

272. The low-pressure control switch:


Answer: Cuts out the compressor to maintain proper flow

273. If the outlet of the thermostatic valve is warmer than the inlet, it indicates
Answer: Thermostatic valve not working properly

274. Compute the wall gain load of a load storage room when A is the outside surface area of
the wall, D is the temperature differential across the wall, and U is the overall coefficient of
heat transmission.
Answer: AUD

275. A leak on the fuel oil suction line between the tank and the suction side of the fuel oil
pump would result in:
Answer: Air entering the suction line

276. How is the CO2 system is purged?


Answer: When CO2 comes out of the purge valve, frost will form on a piece of metal
held near the outlet

277. The relief valve on a CO2 machine is located:


Answer: On the discharge pipe between the compressor and discharge valve
278. The high-pressure side of the system is sometimes referred to as the:
Answer: Hot side

279. Combined pressure of cooling and humidifying is also known as:


Answer: Evaporative cooling process

280. The schedule of a pipe, N, indicates the thickness of the pipe wall. If the allowable stress
of the pipe is S, then what is the internal pressure equal to?
Answer: N x S

281. Which of the following area of work requires lowest noise level?
Answer: Library

282. Which of the following contribute to the deterioration of the earth’s ozone layer?
Answer: Chlorofluorocarbons

283. What is the least number of compressors a multistage system that will use?
Answer: Two

284. Is a soluble compound that reduces a liquid’s surface tension or reduces the interfacial
tension between a liquid and a solid
Answer: Surface – acting agent or surfactant

285. To secure the belts embedded within the foundation, the distance of the edges of the
foundation from the bedplate must be ____.
Answer: 120 mm to 300 mm

286. In Stirling and Ericsson Cycles, the efficiency can be increased by:
Answer: Regeneration

287. The thermal expansion valve


Answer: Controls the quantity of liquid refrigerant going to the evaporator coils

288. Too low suction pressure could be caused by:


Answer: Any of the other choices
(Dirty scale traps, Shortage of refrigerant gas, Too much oil in the system)

289. The static pressure drop due to friction through the boiler and stack.
Answer: Draft loss

290. In a freon-11 system there is no


Answer: Distiller
291. The coefficient of contraction is the ratio of the:
Answer: Area of vena contracta to the orifice area

292. Which of the following should not be used to clean scale traps:
Answer: Cotton waste

293. Most observe properties of light and other radiant energy are consistent with waves in
nature, but in interaction with matter, electromagnetic energy behaves as though it consists of
discrete pieces or
Answer: Quanta

294. Boilers using soft coal must have ____ furnace volume to reduce the danger of ____
Answer: Large; smoking

295. Which of the following would you apply if a person got Freon in his eyes?
Answer: Sterile mineral oil

296. The compressor will run continuously if there is:


Answer: Any of the other choices
(Too heavy a load, Insufficient refrigerant, Air in the system)

297. A hot crankcase and cylinder head accompanied by a low suction pressure would be
caused by
Answer: Insufficient refrigerant

298. One foot water is equal to:


Answer: 62.43 lb/in2

299. The purpose of the oil trap is:


Answer: To remove oil from the refrigerating gas

300. Many pressure gauges on a Freon system have two dials or graduations on one gauge.
The two dial represents:
Answer: Pressure and temperature

301. The faster way to remove frost from a direct expansion finned tube evaporator is to:
Answer: Send hot gas through the coil

302. When ordering an expansion valve which of the following information is necessary?
Answer: Size, tonnage, temperature, and pressure

303. The elements of a thermostat switch are usually of the ____.


Answer: Bimetal type
304. Which of the following characteristics that is not desirable in a refrigerant?
Answer: Low latent heat of vaporization

305. What do bodies at a temperature above absolute zero?


Answer: It emits Thermal Radiation

306. The Horizontal Scale (Abscissa) in the psychrometric Chart represents:


Answer: Dry bulb temperature

307. The thermostatic expansion valve is designed to maintain constant ____.


Answer: Superheat

308. The charging connection in a refrigerating system is located:


Answer: Between the king valve and the solenoid valve

309. The latent heat of fusion of ice is


Answer: 144 Btu

310. The ratio absorbed by the transfer fluid to the original incident energy striking the
collector
Answer: Collector efficiency

311. If m is the mass of dry air and H is the specific enthalpy of the water vapor in air and r is
the humidity ratio, then determine the latent heat of any dry air.
Answer: m(r x H)

312. The dehydrator is located between the:


Answer: Receiver and expansion valve

313. In a vapor compression cycle the lowest temperature is found in


Answer: Evaporator

314. The volume flow passes through a venturimeter is:


Answer: Constant

315. Anchor bolts in a machine foundation should be embedded in concrete of at least how
many time of the bolt diameter?
Answer: 30

316. Which do you think is the effect of “subcooling”?


Answer: It reduces the horsepower per ton of refrigeration
317. When air is saturated, the wet bulb depression is:
Answer: Zero

318. A precooler, if used is located between the:


Answer: Condenser and expansion valve

319. A Kaplan turbine is:


Answer: Low head axial flow turbine

320. The refrigerant temperature is at its maximum just before it enters the ____
Answer: Condenser

321. The machine foundation must have a factor of safety of:


Answer: 5

322. In a refrigerating system, the heat absorbed in the evaporator per kg mass of refrigerant
passing through
Answer: Equals the increase in enthalpy

323. Which of the following is the common classification of ducts?


Answer: All of these
(Conditioned Air – Duct, Recirculating, Fresh – air ducts)

324. All of the following process can be found in a psychrometric chart except:
Answer: Natural convection

325. If an electrically-operated compressor failed to start the cause might be:


Answer: Any of the other choices
(Burned out holding coils in solenoid valve, An open switch, A blown fuse)

326. Five pounds of water heated to raise the temperature 2F requires how many BTU?
Answer: 10 Btu

327. Air circulation in the icebox is accomplished by the use of which of the following?
Answer: Diffuser fans

328. In sensible heating cooling following parameter remains unchanged


Answer: Humidity ratio

329. Venturi meters, pitot static gauges, orifice meters, flow nozzles, and differential
manometers all depend upon the relationship between:
Answer: Flow velocity and pressure
330. It is used to deliver widespread, fan-shaped flows of air into the room
Answer: Diffuser

331. What turbidimeter that gives direct reading in ppm?


Answer: Jackson turbidimeter

332. How much will be removed by one-ton refrigeration unit?


Answer: 200 Btu per min

333. A double-trunk piston is used to:


Answer: All of other choices
(Prevent oil from mixing with the refrigerant, Prevent gas from getting to crankcase, Prevent
oil from mixing with the refrigerant)

334. Purpose of the receiver is to


Answer: Store the refrigerant

335. Latent heat


Answer: Cannot be measured with a thermometer

336. A type of water turbine where a jet of water is made to fall on the blades or buckets and
due to the impulse of water, the turbine starts to move.
Answer: Pelton wheel

337. When does the refrigerant gives-up heat?


Answer: When it condenses

338. A term used to mean the corrective steps taken to return the environment to its original
condition
Answer: Remediation

339. The back – pressure regulating valve:


Answer: Maintains a fixed pressure in the evaporator coils

340. What is the relationship of the horse power of a centrifugal pump, Hp, to the impeller
speed, S, if the pump is at two different rotative speeds?
Answer: Hp is directly proportional to the cube of the ratio of S

341. What must be the value of the available Net Positive Suction Head (NPHS) of a
centrifugal pump compared to its required NPHS to avoid losing priming?
Answer: Available NPHS greater than required NPHS
342. The diesel engine foundation safe soil bearing pressure is:
Answer: 4,890 kg/cm2

343. Aside from maintain appropriate temperature for food cold storage, how is desiccation
minimized or decreased?
Answer: Maintain humidity ratio

344. If C is the capacity of the compressor in a refrigeration system and F is the heat rejection
factor, then calculate the condenser load, L.
Answer: L = C x F

345. The following examples of indirect (secondary) measurements to measure flow rates
using obstruction meters except:
Answer: Weight and mass scales

346. When required, a regulator water valve in refrigerating system should be


Answer: In the water outlet

347. What is the relative humidity when the dew point and dry bulb temperature are equal?
Answer: 100%

348. Compare in refrigerating effect per unit mass of refrigerant circulated for a superheated
cycle that produces useful cooling and a saturated cycle, for the same vaporizing and
condensing temperature
Answer: Greater for a superheated cycle

349. If PV is the power required for a vapor compression refrigeration system, then what is the
power required for an air refrigeration system, assuming that they have the same capacity?
Answer: 5PV

350. What do you call the mixture if the solute particles of a solid suspended in a liquid are
larger than molecules?
Answer: Suspension

351. Lithium bromide is used in refrigeration system in:


Answer: Absorbers

352. The removal of dissolved gas or other volatile component from liquid by exposing the
liquid to air or steam is known as:
Answer: Stripping

353. Which of the following measures the density of salt in water?


Answer: Salimeter
354. In a power driven pump, each piston stroke is displaced by 360 divided the ____.
Answer: Number of cylinders

355. What is the boiling temperature of F 22?


Answer: -40 C

356. What is another name of discharge pressure?


Answer: Head pressure

357. If the compressor were to run out continuously without lowering the temperature, the
trouble would probably be:
Answer: Any of the other choices
(Insufficient refrigerant in the system, Leaky discharge valve, Leaks in the system)

358. The ratio of sum of individual maximum demands of the system to the maximum demand
of the whole system is known as
Answer: Diversity factor

359. The temperature bulb of solenoid valve is attached to the ____.


Answer: Wall of the icebox

360. What must be done to change the direction of rotation of a 440-volt, 3-phase induction
motor?
Answer: Interchange any two power leads

361. In order to remove the fly ashes from the flue gas, which of the following must a power
plant be equipped with?
Answer: Electrostatic precipitator

362. Which of the following is a type of water turbine?


Answer: Pelton

363. In sensible heating the absolute humidity remains constant but the relative humidity:
Answer: Decrease

364. All of the following process can be found on a psychrometric chart except?
Answer: Black body radiation

365. When checking zinc plates in a condenser, one should:


Answer: Clean the plates and renew worn out ones

366. Turbidity in water is due to:


Answer: Finally divided particles of clay, silt and organic matter
367. The force when applied to a mass of one kilogram will give mass an acceleration of one
meter per second for every second called:
Answer: Newton

368. A precooler is sometimes installed between the


Answer: Condenser and expansion valve

369. A type of polymer used for detergents, milk bottles, oil containers and toys
Answer: High density polyethelene (HDPE)

370. The fact that the amount of slightly soluble gas absorbed in a liquid is proportional to the
partial pressure of the gas is known as:
Answer: Henry’s Law

371. What is the pressure at the exit of a draft tube in a turbine?


Answer: Atmospheric

372. What do you call the device that is used as low-pressure control and high-pressure cutout
on a compressor?
Answer: Pressure Controller

373. Redox reactions can often be type of:


Answer: Double replacement

374. Excessive head pressure is caused by


Answer: Any of the other choices
(Dirty condenser tubes, Insufficient cooling water to condenser, Air or non condensable gas
in the system)

375. In which part of the vapor compression cycle there is abrupt change in pressure and
temperature?
Answer: Expansion valve

376. The kinetic energy of a moving fluid is used to isentropically compressed the fluid to
state of zero velocity. The temperature of a moving fluid at the state zero velocity is called:
Answer: Critical temperature

377. Which is not commonly used to cool and dehumidify equipment?


Answer: Calcium chloride

378. What is the primary cause of smog formation?


Answer: Nitrogen oxides
379. The dehydrating agent in a Freon system is usually
Answer: Activated alumina

380. The solenoid valve is actuated by which of the following?


Answer: A magnet

381. Which of the following does not use ambient air for propulsion?
Answer: Turbo-prop

382. Which of the following would not cause high suction pressure?
Answer: Insufficient refrigeration

383. What is the effect of superheating the refrigerant?


Answer: It increase the coefficient of performance

384. The color of the flame of halide torch, in case of leakage of Freon refrigerant, will change
to
Answer: Green

385. If there is too much lube oil in the system what must be done?
Answer: Remove same at once

386. If the thermal bulb becomes loose on the evaporator coils, it will cause ____.
Answer: Improper operation of expansion valve

387. The effect of superheating the refrigerant is to:


Answer: Decrease the COP

388. In power plant electrostatic precipitators is installed between:


Answer: Furnace and chimney

389. Equipment leaks from plant equipment are known as


Answer: Fugitive emissions

390. Per capita consumption of water is generally taken as:


Answer: 150-300 liters

391. What is the heat that is removed from the space to be cooled, which is the same as the
heat absorbed by the cooling coils?
Answer: Refrigerating effect

392. Horsepower per ton of refrigeration is expressed as:


Answer: Cop/4.75
393. What must be done first when opening a single packed stop valve?
Answer: Loosen the packing before opening

394. The amount of CO2 or Freon in a cylinder is measured by


Answer: Weight

395. The coefficient of velocity is approximately


Answer: 0.30 to 0.50

396. Carries needed to deliver air to the conditioned space it is made of sheet of metals like
aluminum, galvanized sheet steel and some structural materials that will not burn
Answer: Duct

397. Which of the following best described a Freon?


Answer: All of the other choices
(Colorless, Odorless, Non-poisonous)

398. Before securing a compressor to do maintenance work on it, be sure to:


Answer: Have spare parts ready and pump down the system

399. A bull’s eye in a full liquid line will appear ____.


Answer: Clear

400. What operates low-pressure cutout switch?


Answer: Bellows

401. A liquid mixture having constant maximum and minimum boiling points. Refrigerants
comprising this mixture do not combine chemically, yet the mixture provides constant
characteristics.
Answer: Azeotropic Mixture

402. Are organic compounds manufactured in oily liquid and solid forms through the late
1970s and subsequently prohibited.
Answer: Polychlorinated biphenyls (PCBs)

403. What is the temperature range of an air-conditioning application where dry air can be
considered ideal gas?
Answer: -10 to 50 C

404. The coefficient of the velocity, Cv accounts for the:


Answer: Small effect on the flow area of contraction, friction, and turbulence
405. During the re-expansion portion of the refrigeration compressor cycle
Answer: The suction valve is closed and the discharge valve is closed

406. Refers to the high-temperature removal of tarry substances from the interior of the carbon
granule, leaving a highly porous structure
Answer: Activated

407. Are by products of reaction between combustion products


Answer: Oxidants

408. Are any solid particulate matter that becomes airborne, with the exception of particulate
matter emitted from the exhaust stack of a combustion process
Answer: Dusts or fugitive dusts

409. The purpose of the dehydrator is to


Answer: Remove moisture from the refrigerant

410. The angle between the stack and guy wire is usually
Answer: 60 degrees

411. A refrigeration that deals with producing temperature of -157 C or lower.


Answer: Cryogenics

412. The purge valve is located:


Answer: In the highest part of the system

413. Thermal expansion valves are usually made of the:


Answer: Diaphragm and Bellows type

414. The purpose of the expansion valve by-pass is to


Answer: Controls the refrigerant to the evaporator in case the automatic valves fail

415. The water regulating valve is operated by the


Answer: None of the other choices
(Compressor discharge pressure, Compressor suction pressure, Compression discharge
temperature)

416. Superheat is heat added ____.


Answer: After all liquid has been changed to vapor

417. The ratio of maximum load to the rated plant capacity is called
Answer: Utilization factor
418. The highest temperature in vapor compression cycle is produced during:
Answer: Compressor discharge

419. Dry analysis is a fractional analysis of the products of combustion which does not
include:
Answer: Water vapor

420. Valves and piping in an ammonia system are made of:


Answer: Iron

421. The locus of elevation to which water will rise in the piezometer tube is termed:
Answer: Hydraulic gradient

422. A substance with surface area per unit of weight, and intricate pore structure, and a
hydrophobic surface
Answer: Adsorbent substance

423. The purpose of the expansion valve is to control the flow of the refrigerant to the
evaporator. The other function is to:
Answer: Reduce the pressure of the liquid refrigerant

424. An ammonia leak will turn litmus paper ____.


Answer: Blue

425. What is the most common freon gas used in reciprocating compressors?
Answer: F-12

426. The empirical coefficient e in machine foundation if not given is assumed


Answer: 0.11

427. The capacity of a centrifugal type compressor is controlled by which of the following?
Answer: Regulating the speed or regulating the suction pressure

428. Antifreeze chemicals are:


Answer: Those that lower down the freezing points of liquids

429. A type of refrigerant that will not damage the ozone layer
Answer: Hydrofluorocarbons (HCF’s)

430. At 0 psig, how may BTU necessary to change 1 lb of water at 212 F of steam?
Answer: 970
431. Modern stacks are seldom built higher than ____.
Answer: 60 m

432. If Pi is the indicated horsepower and Pb is the indicated horsepower of a compressor,


then what is mechanical efficiency, Em, equal to:
Answer: Em = Pi/Pb

433. A refrigeration control that guards the compressor forms overloads brought about by
abruptly increases loads resulting from defrosting, warm products, and others is called:
Answer: Suction hold-back valve

434. When a solvent has dissolve as much as it can, the mixture is called:
Answer: Saturated solution

435. Fan motors in air conditioning usually have:


Answer: 2 or 3 speeds

436. In the upper atmosphere, ozone is made by ultraviolet light reacting with:
Answer: Oxygen

437. Which of the following is the oil used in a refrigeration system?


Answer: Straight mineral oil

438. The following are all examples of indirect (secondary) miscellaneous methods to measure
flow except:
Answer: Positive displacement meters

439. Which of the following is the function of a suction pressure regulating valve?
Answer: Maintains proper back pressure

440. What is the use of back pressure regulating valve?


Answer: Maintains a fixed pressure in the evaporator coils

441. If Wt is the turbine shaft work of a gas turbine unit, Wc in duel, then determine its
thermal efficiency.
Answer: (Wt-Wc) / Q

442. The solenoid valve is controlled by


Answer: The temperature in the icebox

443. Where is the excess refrigerant removed?


Answer: Charging side of the system
444. All of the following units of energy except
Answer: Pascals

445. What usually happened if brine has a high specific gravity?


Answer: It will crystallize

446. There are three basic boiler types, namely;


Answer: Cast-iron, fire-tube and water tube boilers

447. Measurements of a device’s ability to remove atmosphere air from test air.
Answer: Atmospheric Dust Spot Efficiency

448. The normal cut-out setting of a window unit thermostat between:


Answer: 13 C to 16 C

449. The use of water to carry heat occupied spaces


Answer: Hydronic Heating System

450. Reheating process is normally employed in steam plant when:


Answer: Turbine undergoes excessive moisture

451. In the discharge line between the compressor and the condenser one would find:
Answer: High pressure, high temperature gas

452. A good refrigerant should be


Answer: All of the other choices
(Non-poisonous, non-inflammable, non-explosive)

453. What is the relationship of the horse power of a centrifugal pump, Hp, to the impeller
speed, S, if the pump is at two different rotative speeds?
Answer: Hp is directly proportional to the cube of the ratio of S

454. Which of the following is the reason when the crankcase is cooler than the suction line?
Answer: Too much refrigerant

455. Humidity is a measure of which of the following?


Answer: Water vapor content

456. High head turbine is a/an:


Answer: Impulse

457. Critical temperature is that temperature above which


Answer: A gas will never liquefy
458. The fact that a fluid’s velocity increases as the cross-sectional area of the pipe through
which it flow decrease is due to:
Answer: The continuity equation

459. Which of the following is important for evaporator coils?


Answer: It should have air completely surrounding them

460. Which of the following types of air dryers works by absorbing moisture on a solid
desiccant or drying material such as activated alumina, silicone gel, or molecular sieve?
Answer: Deliquescent dryer

461. Which of the following would cause the compressor to run continuously?
Answer: Low pressure switch jammed

462. In a window air conditioning unit which of the following is usually done by the owner?
Answer: Semi-annual cleaning or replacement of filters

463. Which of the following is not similarity between submerged culvert and a siphon?
Answer: Torricelli’s equation holds

464. Where does the final removal of water vapor in an absorption refrigeration system occur?
Answer: Rectifier

465. The constant spending of certain percentage of circulated water in a cooling tower in
order to prevent accumulation of dissolved mineral solids and other impurities in the
condenser water is called
Answer: Bleed-off

466. Which of the following refrigerant is added sometimes to other refrigerant to improve oil
circulation?
Answer: R – 170 (Ethane)

467. The suction pressure switch is operated by which of the following?


Answer: Pressure on a bellow

468. When the air is saturated the wet-bulb depression is:


Answer: Zero

469. Heating and dehumidification can be obtained simultaneously if air is passed through:
Answer: Either a solid absorbent surface or a liquid absorbent spray
470. In cooling cycle, the dry bulb temperature (db) of the air is lowered. When this happens
the relative humidity
Answer: Increase

471. What will happen when the pressure at any point inside a centrifugal pump goes below
the vapor pressure corresponding to the temperature of the liquid?
Answer: Cavitation

472. The ratio between the actual power and the apparent power in any circuit known as the
____ of that circuit
Answer: Power factor

473. Refrigerant leakage from the compressor crankcase is prevented by


Answer: Using shaft seals

474. The matching of scale and full-scale results for a fluid dynamic phenomena involving
compressible fluids requires quality of:
Answer: Mach number

475. Ammonia leaks in the condenser can be detected by:


Answer: Applying litmus paper to the circulating water discharge

476. A swinging support constructed as part of the vessel and that supports the manway cover
when it is unbolted and moved aside.
Answer: Davit

477. Which of the following type valves are not found on a Freon – 12 system?
Answer: Duplex

478. What is the main function of a receiver?


Answer: Store the refrigerant

479. The dividing point between the high pressure and low pressure sides of the refrigeration
cycle occurs at the
Answer: Expansion valve

480. If the solute particles of a solid suspended in a liquid are larger than molecules the
mixture is known as?
Answer: Suspension

481. A mechanism that removes moisture


Answer: Dehumidifiers
482. Is a water soluble organic compound prepared from ammonia. It has significant
biological and industrial usefulness
Answer: Urea or carbamide urea

483. Which of the following fans in air conditioning systems which can be classified as
centrifugal flow
Answer: Propeller fan

484. What is the use of the low – water cutout switch?


Answer: Stops the compressor when there is insufficient cooling water

485. In parallel pipe system originating and terminating in common junctions,


Answer: Pressure drops through each branch are equal

486. When the dry bulb and the wet bulb temperature are identical, the air is said to be:
Answer: Saturates

487. The temperature in which water vapour in the gas begins to condensate in a constant
pressure process.
Answer: Dew point

488. Combination gas/fuel oil burners permit the operator to switch from one fuel to ____
Answer: All of the above
(For economy, Because of a shortage of fuel being used, Because of a failure in the fuel
system being used)

489. Most nuclear particles can react with atoms in several different ways including
Answer: Absorption and scattering

490. What is the effect of superheating the refrigerant?


Answer: It increase the coefficient of performance

491. Boiling temperature of Freon 12 is


Answer: -29.8 C

492. Which of the following would cause low head pressure?


Answer: Too much cooling water and/ or insufficient refrigerant gas

493. At what temperature will water normally turns to steam?


Answer: 212 F

494. What will happen if the expansion valve is opened too wide?
Answer: Liquid will flood back to the compressor
495. All of the following are words used to describe neutron kinetic energy levels except:
Answer: Supersonic

496. If the thermal expansion valve becomes inoperative, the ice boxes will have to be
controlled by the ____.
Answer: Manual expansion valve

497. If the superheat on the suction side of the compressor is increased, what will happen to
the tonnage capacity of the unit?
Answer: Decreases

498. The solenoid valve can be typed as a ____.


Answer: Magnetic soap valve

499. The ratio of fugacity of actual conditions to the fugacity at some reference state is known
as:
Answer: Activity

500. The process that takes place in the evaporator called


Answer: Absorption of the latent heat of vaporization

501. When charging freon system, all the valves should be


Answer: King (liquid) valve

502. If the pressure exerted on a liquid is higher than the saturation corresponding to it
temperature, the liquid is a:
Answer: Sub cooled liquid

503. The most likely cause of high superheat would be:


Answer: Expansion valve closed too much

504. A good refrigerant should have a


Answer: High latent heat

505. What is the device used to protect the compressor from overloading due to high head
pressure
Answer: Overload relay

506. Which of the following it to be checked as part of weekly maintenance schedule of a


console air conditioners?
Answer: Cooling towers
507. Which of the following would not be cause for a refrigerating system to short cycle on hp
cutout?
Answer: Discharge valve leaking

508. A ____ boiler has heat and gases of combustion that pass through tubes surrounded by
water
Answer: Fire tube

509. A type of polymer used for clear bottles.


Answer: Polyvinyl Chloride (PVC)

510. In an ammonia system, the oil gauge must be kept:


Answer: Closed except when checking level oil

511. If a compartment requires the removal of 36,000 BTU per hour, how much is necessary
compressor capacity?
Answer: 3 tons

512. The oil level in the compressor should be checked:


Answer: After a long period of operation

513. Which of the following industries have the highest consumption of water for processing?
Answer: Paper mill

514. What is the lowest temperature to which water could possibly be cooled in a cooling
tower?
Answer: The temperature of adiabatic compression

515. The method of cooling which primarily used where ambient air temperatures are high and
relative humidity is used:
Answer: Swamp cooling
1. What tonnage of refrigerating machine is required if the refrigerating system extracted
48,000 btu per hour?
Answer: 4 tons

2. An insulator with superior tensile strength but low heat resistance


Answer: Fiberglass

3. What is the combination of a wet and dry bulb thermometer is called


a Answer: Hygrometer or psychrometer

4. The weight of steel bar reinforcement show be how many time the weight of the foundation?
Answer: ½% to 1%

5. Is a high temperature process that turns incinerator ash into a safe, glass – like
material Answer: Vitrification

6. Oil is added to a freon compressor by


Answer: Pumping in with a hand pump

7. The difference between the theoretical draft and the draft


loss Answer: Available draft

8. The purpose of expansion valve by-pass is to


Answer: Controls the refrigerant to the evaporator in case the automatic valves fail

9. Load curve refers to the plot of:


Answer: Load versus time

10. What does a negative Joule-Thompson coefficient means during a throttling


process? Answer: fluid temperature rises

11. The maximum unit pressure of turbine and generator on the reinforced concrete should
not exceed
Answer: 17.62 kg/cm2

12. Water in vapor form remains a vapor as long as temperature is what relations to the
dew point temperature
Answer: below
13. The “refrigerating effect” of a refrigerant is a always:
Answer: Less than its latent heat

14. A burner should always start up in ___ fire and shut down in ___ fire
Answer: Low; low

15. A ___ boiler has water in the tubes and heat gases of combustion passing through the
tubes Answer: Water tube

16. If the compressor had been running satisfactorily for a long period of time but suddenly
the compartment temperature started to rise, the trouble might be:
Answer: Any of the other choices
(A refrigerant leak has developed, The solenoid valve has jammed shut, The expansion valve
may contain frozen water)

17. A type of polymer used for Styrofoam cups and clam shell food
containers Answer: Polystyrene (PS)

18. Mycotoxins are poisonous chemicals produced by:


Answer: Molds

19. The magnitude of the drag coefficient of a sphere in water is dependent upon all of the
ff except
Answer: Units of measure (SI or English engineering system)

20. A refrigeration system in which only part of the refrigerant passes over the heat
transfer surface is evaporated and the balance is separated from the vapor and
recirculated Answer: Flooded system

21. The boiling point of freon-12 at atmospheric pressure is


Answer: -22F

22. Which of the following vital components of the refrigeration system where both
temperature and pressure are increased?
Answer: Compressor

23. Which of the following must be done to eliminate frost on the discharge pipe of
the compressor?
Answer: None of the other choices
(Open the expansion valve, crack bypass valve, Regulates water to the condenser)

24. Orifice coefficients are used to


determine: Answer: Energy losses
25. Which of the following is used as high head turbine?
Answer: Impulse

26. A water-tube condenser has a total of 60 tubes. If these two passes, then compute the
number of tubes per pass
Answer: 15

27. Absolute zero is


Answer: 460 degrees below zero on the Fahrenheit scale

28. Which of the following can be the cause of low head


pressure Answer: Any of the other choices
(Leaky discharge valves, Too much or too cold condensing water, insufficient charge of
refrigerant)

29. What is the most common freon gas used in centrifugal compressors?
Answer: F-11

30. When purging an ammonia condenser into a bucket of water, one can tell when the air is
out and ammonia starts to come through the:
Answer: Change of bubbling sound of air to the cracking sound of ammonia

31. In the deep well installation or operation, the difference between static water level
and operating water level is known as:
Answer: Drawdown

32. White ice is


Answer: Due to dissolved air, gasses and impurities

33. Which of the following is a characteristic of an impulse turbine


Answer: Steam striking blades at zero angle

34. Heavy water is:


Answer: D2O

35. When excess solute in a solution settles to the bottom of the container, the process is called:
Answer: Precipitation

36. Calcium chloride is sometimes used in refrigeration system as


a: Answer: Secondary Coolant

37. If a compressor runs continuously, the cause might be


a: Answer: Stuck low-pressure switch
38. The thermal expansion valve is located between the:
Answer: Solenoid valve and the evaporator coils

39. The cooling water regulator is automatically actuated by which of the following?
Answer: The discharge pressure of the refrigerant

40. Based on the PSME code, what should be provided in each steam outlets if two or more
boilers will be connected in parallel?
Answer: Non-return and shut off valves

41. The amount of sensible heat for a sensible heat ratio of 0.8 and a total cooling load of 100 is
Answer: 80

42. Which refrigerant is used for the air conditioning of passenger aircraft cabin
Answer: Air

43. When a falling object reaches a speed at which the drag force equals its weight, it
has achieved:
Answer: A turbulent boundary layer

44. The minimum vertical distance from the floor or soil level to the top edge of the
foundation must be around
Answer: 120 mm

45. How many moisture be removed from air?


Answer: condensation and absorption

46. The charging valve is located between the:


Answer: King valve and the expansion valve

47. A “cold diffuser” is a:


Answer: Type of evaporator

48. Which of the following stops the compressor before the relief valve opens?
Answer: High pressure control

49. What is the excess refrigerant removed?


Answer: Charging side of the system

50. Which of the following is used in measuring the density of a brine solution?
Answer: A hydrometer
51. All of the following are common types of chemical reaction except
Answer: Fission

52. A ___ after each radiator allows ___ to pass through to the return
line Answer: Steam trap; condensate

53. A leaky solution valve can usually detected by:


Answer: Any of the other choices
(A fluctuating suction pressure gauge, a higher suction pressure, closing in on the suction
valve having no effect on the suction pressure)

54. If the cooling water to the condenser suddenly


fails: Answer: The compressor will shutdown

55. If Et is the total net energy generated by a plant in a certain period of time and Er is the
rated net energy capacity of the plant during the same period of time, then what is the plant
operating factor?
Answer: Et/Er

56. A thermometer sense which of the following?


Answer: Sensible heat

57. Is the subjective method in which the smoke density is visually compared to five
standardized white-black grids
Answer: Ringelman scale

58. Excessive head pressure is caused by


Answer: Flooded condenser tubes or cooling water temperature too high

59. What do you call a plate or vane used to direct or control movement of fluid or air within
the confined area?
Answer: Baffle

60. The total pressure supplied by the fan at maximum operating


conditions Answer: Net rating or fan boost

61. The system should be purged:


Answer: After the system has been shut down for few hours

62. What is likely to occur when sections of the impeller of a centrifugal pump are handling
vapor and other sections are handling liquid.
Answer: complete failure to operate
63. What is the chemical formula of Ozone?
Answer: O3

64. An excessively high head pressure would be caused by:


Answer: Insufficient cooling water to the condenser

65. When an electric motor nameplate indicate a “100-kW electric motor” then what does a 100-
kW rating refers to?
Answer: Mechanical power output

66. During a cooling and dehumidifying process, sensible and latent heats are removed in the
cooling coil. If Hs is the sensible heat and Ht is the total heat transferred, then determine
the coil sensible factor.
Answer: Hs/Ht

67. A single trunk piston-type compressor is undesirable for a Freon unit because
the: Answer: Lubricant mixes with the refrigerant

68. What is the first step that must be done when securing a system?
Answer: Close receiver (King) valve

69. The amount of CO can be determined by the color of Palladium Chloride. An amount of 30
ppm to 70 ppm will cause:
Answer: Slight Darkening

70. The water hammer phenomenon is primarily what kind of fluid


mechanism? Answer: Dynamic (a time-dependent phenomena)

71. The other name for Swamp Cooling


is: Answer: Wet Roof Cooling

72. The design of an air supply duct of an air condition


system Answer: affects the distribution

73. A Freon – 12 gage shows pressure and


____. Answer: Saturation temperature

74. High superheat of the vapor in the system would cause


Answer: A decrease in capacity

75. When there is no work between the thermodynamic system and its surrounding, the
quantity of net heat transfer is equal to:
Answer: total energy of a closed system
76. The relation between the Fahrenheit absolute scale and the Celsius absolute scale is:
Answer: R = 1.8 Kelvin

77. External frost on inlet of expansion valve indicates:


Answer: Expansion valve plugged or dirty

78. The total cross section of a target atom is made up of


Answer: The absorption and scattering cross sections

79. The motor condition of an air conditioning unit can be checked with:
Answer: The continuity of light or with an ohmmeter

80. A number used to compare energy usage for different areas. It is calculated by dividing
the energy consumption by the footage of the conditioned area.
Answer: Energy utilization Index

81. What is the effect if the refrigerant is removed from the system too fast?
Answer: It may “freeze – up” the condenser

82. After the total force of the steam has lifted the safety valve off its seat, the steam enters the
____.
Answer: Huddling chamber

83. By which of the fooling processes heat mainly dissipates in cooling


towers? Answer: Evaporation

84. All of the following fluid phenomena are based on the force momentum principle of
a flowing fluid except:
Answer: Diesel automobile engines

85. All of the following are forms of drag on a body moving through a fluid
except Answer: D’alembert’s paradox drag

86. What is the use of the suction pressure regulating valve?


Answer: Maintains the back pressure in the evaporator coils

87. There shall be no foundation bolts less than _____.


Answer: 12 mm in diameter

88. The ratio of the rated cooling capacity divided by the amount of electrical power
used: Answer: Energy efficiency ratio (EER)
89. The scale trap is located between the:
Answer: King (liquid) valve and expansion valve

90. The temperature in the vegetable box should be approximately


Answer: 35 to 45 deg. F

91. A thermostat is a
Answer: Temperature-operated switch

92. The bulb for the thermal expansion valve is located:


Answer: Near the evaporator coil outlet

93. The coefficient of velocity is the ratio of the:


Answer: Actual velocity to the theoretical velocity

94. What is the specific humidity of dry


air? Answer: 0

95. What is the reason why a thermometer in vapor compression system is installed close to
the compressor?
Answer: Because it helps the operator in adjusting compressor for greatest efficiency

96. As the steam temperature increases the steam


temperature: Answer: Increases

97. Substances that have the ability to absorb moisture from the air are called:
Answer: Desiccants

98. Evaporative condenser is used to cool


Answer: condenser vapor

99. If the head pressure is too high:


Answer: The high pressure cutout switch should operate before the relief valve opens

100. A material or substance that is accidentally or intentionally introduce to the


environment in a quantity that exceeds what occurs naturally
Answer: Pollutant

101. The greatest decrease in refrigerant temperature occurs in the ____


Answer: Evaporator

102. The diesel cycle is the ideal cycle for a :


Answer: compression-ignition engine
103. The moisture in a refrigeration system can be removed with help of which of
the following?
Answer: Driers

104. What process is employed when the turbine steam power plants experience excessive
moisture?
Answer: reheating

105. The path that directs the flow of refrigerant through the compressor
Answer: Valve

106. Which of the following is another name for the liquid


valve? Answer: King valve

107. Expansion factors take into account the:


Answer: Effects of compressibility

108. The term used to express the amount moisture in a given sample of air. It is
compared with the amount of moisture in a given sample of air.
Answer: Relative Humidity

109. What are the four basic methods of determining whether the proper amount of refrigerant
being added to the system?
Answer: Bull’s eye, weight, pressure and frost line

110. Which of the following components of the window air conditioning system must be
cleaned annually?
Answer: All of these
(Evaporator and Condenser, Motor and Compressor, Fan Blades and Fan Motor)

111. At present, the number of true elementary particles, which include leptons and quarks
is thought to be
Answer: 12

112. All of the following statements about conjugate acids and bases are true except
Answer: A conjugate base results when a base accepts a proton

113. A double-seated valve allows the valve to be


Answer: Packed in the wide open or closed position

114. In order to keep combustion products inside the combustion chamber and stack system,
balanced draft system may actually operate with a slight
Answer: negative pressure
115. What is the cause of pressure drop in the boiler, condenser and the pipings between
different components? Because of this drop, steam leaving the boiler at a lower pressure.
Answer: Fluid friction

116. It is a form of oxygen photochemically produced in nature.


Answer: Ozone

117. Absolute zero on the Fahrenheit scale is equal


to Answer: -460

118. Low suction pressure is caused by:


Answer: Solenoid valve not functioning properly

119. What do you call the storage tank for liquid


refrigerant? Answer: Receiver tank

120. Why should you avoid bending or twisting of fan blades in an air conditioning
unit? Answer: It will wear out the motor bearings and cause noise

121. All of the following statements are characteristics of bases except


Answer: They have a pH between 0 and 7

122. Concrete foundation should have steel bar reinforcement placed vertically
and horizontally to avoid ____.
Answer: Thermal cracking

123. The pump will not cavitate if the available Net Positive Suction Head (NPSH)
is: Answer: equal or greater than the required NPHS

124. What will happen to the capacity if the superheat is increased on the suction
side? Answer: Decreases

125. When heavy electrical currents are involved, the thermostat will be operated by
a Answer: Relay

126. Valves in piping in an ammonia system are made of


Answer: Iron

127. Which of the following is not essential to a compression refrigeration system?


Answer: A receiver

128. Nitrogen occupied almost how much of the Earth’s atmosphere?


Answer: 3/4
129. If frost forms on the cylinders, the cause would
be: Answer: Expansion valve open too wide

130. Which of the following gasket materials should be on Freon system?


Answer: Asbestos or Metallic

131. Which of the following would cause the crankcase and head to get hot with low
suction pressure?
Answer: Insufficient refrigeration

132. Moisture in a system will cause a:


Answer: Faulty expansion valve

133. The steam turbine foundation should be designed to support the machine load plus how
many percent for impact, condenser load, floor loads and dead loads?
Answer: 25

134. When air is heated, what happened to its relative humidity?


Answer: Remain constant

135. Sweating of crankcase is caused by which of the


following? Answer: Too much oil in the system

136. What do you call a material in a dryer?


Answer: Desiccant

137. The matching model and full-scale prototype results a fluid dynamic
phenomena involving a fully submerged body requires equality of:
Answer: Reynolds number

138. Which of the following uses a secondary refrigerant?


Answer: Ice plant

139. When the outlet temperature at the evaporator exceeds the inlet temperature, the
condition is known as:
Answer: Superheating

140. In series pipe systems, all of the following parameters vary from section to section
except Answer: Mass flow

141. Which of the following is used to control refrigeration compressor capacity?


Answer: Unloader
142. TOR is a unit equivalent to:
Answer: 12.66 kN-m/hr

143. The agent used in an indirect reefer system is:


Answer: Calcium chloride or sodium chloride

144. If the W is the work output of a heat engine and H is the total heat input of an
engine, then determine its thermal efficiency, e.
Answer: e = W/H

145. Which of the following items is not important when using a halide torch?
Answer: Adjust to a clear white flame

146. A dehumidifier is usually a small hermitic refrigerating system. It has both a


condenser and an evaporator. Many older systems use R – 12 or R – 500. The newer units
are: Answer: R – 134a

147. What is the method used to evaluate all welds performed on pressure parts of boiler tube
materials?
Answer: Radiographic test

148. Faulty F – 12 compressor valves will be indicated by:


Answer: All of the other choices
(Gradual or sudden decrease in capacity, compressor running continuously, Low
head pressure – high suction pressure)

149. Heat exchanger in which water flows by gravity over the outside of tubes or plates.
Answer: Baudelot cooler

150. Which of the following is not a sound absorptive material?


Answer: Mirrors

151. The desirable temperature inside an air conditioned auditorium is:


Answer: 20 C

152. An odorless refrigerant whose boiling point varies over a wide range of temperatures.
Answer: Freon 12

153. The cooling water regulator is actuated by


Answer: Pressure of the refrigerant

154. The psychrometric chart in air conditioning determines


the Answer: Moist air conditions
155. Blowback of a safety valve is to
prevent: Answer: Chattering

156. When starting a refrigerant unit, be sure the water side of the condenser is ____.
Answer: Vented

157. A temperature measurement in an ordinary temperature which has constant


specific humidity.
Answer: Dew point temperature

158. What adding oil to a freon system, one must be sure that
Answer: All air is removed from the pump and fitting

159. The primary purpose of a turbine in a fluid is to:


Answer: Extract energy from the flow

160. Which of the following is to be checked as regular monthly maintenance schedule of


a console air conditioning units?
Answer: Duct dampers, registers and diffusers

161. A company is interested to produce a water turbine wherein only little energy is required
or necessary because the guide vanes are to be controlled. The turbine must be a:
Answer: Kaplan turbine

162. To check the lack of refrigerant or clogged refrigerant lines:


Answer: Installing service valves must be necessary

163. Condensate from the non return steam trap is pumped from the condensate return tank of
the:
Answer: Boiler

164. The locus of elevation is:


Answer: Hydraulic Gradient

165. Which of the following cannot be used to describe neutron kinetic energy levels
Answer: Freezing

166. It is important to take some moisture from the air dehumidify it if the relative humidity
reaches high levels. To do this, it requires cooling the air:
Answer: Below its dew point temperature

167. The vapor cycle in thermal efficiency as the refrigeration cycle to the
Answer: COP for a refrigerator
168. A scale trap in a Freon system will be found on the
Answer: Suction side

169. Which of the following is not used as method to measure air velocities?
Answer: Open type Barometer

170. Which of the following is considered as comfort condition in air


conditioning? Answer: 20 C DBT, 60% RH

171. Which of the following acts as ignition accelerator for internal combustion engine
fuels? Answer: Acetone peroxide

172. The purpose of the evaporator is to


Answer: Absorb latent heat of vaporization

173. The speed at which a reaction proceeds to equilibrium is the purview of


Answer: Reaction kinetics

174. What do you call the liquid reaching the compressor through the
suction? Answer: Flooding back

175. Which of the following draft rely on the stack effect to draw off combustion gases?
Answer: Natural draft

176. The relative humidity becomes 100% and where the water vapor starts to
condense. Answer: Dew point

177. Which of the following statements is correct? The relative humidity of an air water
vapor mixture.
Answer: is the ratio of partial pressure of water vapor to the saturation pressure at the
mixture temperature

178. The following are standard characteristics of Freon – 11 except:


Answer: Boiling point over 200 F

179. The bhp/ton refrigeration requirement with increase in condenser temperature. In a vapor
compression system that uses reciprocating compressor receiving refrigerant gas at constant
suction temperature, will follow:
Answer: Linearly increasing characteristic
180. In an air conditioning unit, the thermostat fails. The unit did not start. How do you test
the operation of the thermostat?
Answer: Cover the air outlet and air inlet with a cloth. The air will recirculate into
the unit and the temperature will quickly drop to the cut out temperature.

181. All of the following temperature have meaning in psychrometrics except:


Answer: Adiabatic wall temperature

182. The maximum continuous power available from a hydro – electric plant under the most
adverse hydraulic conditions is called
Answer: Firm power

183. Which of the following dehumidifier is often used to reheat the air after moisture is
removed?
Answer: Condenser

184. Which of the following is a type of evaporator?


Answer: shell-and-tube

185. All of the following factors affect rates of reaction


except Answer: Pressure

186. Which of the following would cause a high suction


pressure? Answer: Expansion valve open too wide

187. Zinc rods are found in the:


Answer: Salt water side of the condenser

188. The sum of the internal energy and the product of pressure and specific volume is
known as the:
Answer: enthalpy

189. Any foreign matter in the coal feed mechanism of the screw-feed stoker is best removed
by:
Answer: Using the cutoff gate at the bottom of the hopper

190. Based on the PSME code, what should be the humidity of air to be used for comfort
cooling?
Answer: 50% - 60% relative humidity
191. Which of the following must be checked up if an automatic Freon system will not start
up?
Answer: All of the other choices
(High pressure cutout, reset mechanism, Low-pressure cutout)

192. Refers to the use of composting soil beds


Answer: Biofiltration

193. What is the usual cop of a domestic refrigerator?


Answer: More than 1

194. In a hydro-electric plant using a Francis turbine with medium head, the speed can be
regulated using the
Answer: Wicket gate

195. The temperature bulb of the thermo-expansion valve is attached to which of the
following?
Answer: Evaporator coil outlet

196. Which of the following refrigerants would give the most trouble when operating with
warm circulating water?
Answer: CO2

197. A rule which states that any waste derived from the treatment of a hazardous
waste remains a hazardous waste
Answer: The derived from rule

198. A device for holding open the suction valve and drawing gas from the suction manifold
and returning it to the suction line without compressing it is called
Answer: Cylinder unloader

199. The intake pipe to a hydraulic turbine from a dam


Answer: Penstock

200. What consists of weak solutions of sulfuric, hydrochloric, and to a lesser extent, nitric
acids?
Answer: Acid rain

201. The expansion valve does not seem to be operating properly. There is high
superheat. Test by listening to the sound of Freon flooding through the tubes and ____.
Answer: Warm bulb with hand
202. Swimming pool water needs
Answer: de-chlorination

203. Oxygen is approximately what percent by weight in the


atmosphere? Answer: 23%

204. Which of the following could be used to check a leak in an ammonia system?
Answer: Litmus paper & Sulfur stick

205. The determination of properties and behavior of atmospheric air usually the purview of:
Answer: Psychrometrics

206. A valve the released steam from the boiler by operating at a pre-determined pressure in
order to keep the steam pressure from rising more than the pressure from which the valve is
set is called a
Answer: Safety valve

207. If the critical temperature of a refrigerant is too close to the desired


condensing temperature, the equipment must necessarily be of
Answer: Extra heavy construction

208. If a freon – 12 compressor trip out on “cut-out”, the solenoid valve closes by which of
the following?
Answer: An electrical release

209. Which of the following would cause the suction switch to remain open?
Answer: Bellows broken or jammed

210. In the upper atmosphere, ozone is made by ultraviolet light reacting


with: Answer: Oxygen

211. How is ammonia system purged so that operator will not be overcome by the
fumes? Answer: Into the bucket of water

212. A ton of refrigeration is equal to the removal of


Answer: 288,000 btu per 24 hrs.

213. As a good practical rule, the foundation depth may be taken as how many times of the
engine stroke?
Answer: 2:5 to 3:5

214. Most stacks are built of height less than _____.


Answer: 30 m
215. The phenomenon that warm air rise and cold air settle is called:
Answer: Stratification

216. Type of refrigerant control which maintains pressure difference between high side and
low side pressure in refrigerating mechanism:
Answer: Expansion valve

217. The crossover connection in an ammonia system can be used to ____.


Answer: Hot-gas defrost

218. Which of the following refrigerants has lowest freezing point


temperature? Answer: Freon 22

219. The operation that produces highest noise level is:


Answer: Riveting

220. Select the one in which secondary refrigerant is


used Answer: Ice plant

221. Which of the following is not essential to a centrifugal type compressor


system? Answer: Distiller

222. Two compressor should not be run in parallel because


Answer: There is a possibility of losing oil

223. Which of the following is also known as refrigerant no. R-


717? Answer: Ammonia

224. During sensible heating, the humidity remains constant but the relative
humidity. Answer: decreases

225. The term suction units is used with


Answer: Induced draft

226. If the compressor had been running satisfactorily for a long period of time but the oil
level was rising slowly, one should:
Answer: Shut down the compressor and check the oil level with the machine stopped

227. What amount of air is required in a low bypass factor?


Answer: Lesser

228. The reduction of nuclear radiation intensity (called attenuation) is accomplished


by: Answer: Shielding
229. The ozone concentration of 0.10 parts per million (ppm) is generally considered
the maximum permissible for how many hours exposure?
Answer: 8 Hours

230. Which do you think is very important in adjusting compressor “v” belts?
Answer: Allow ½” slack

231. An engineer inspected an air-conditioning unit. He found out that the unit does not
produce any cooling effect, however, the air-conditioning unit is running. He checked the
temperature of the condenser and evaporator and hat the unit run. He found out that there
was no change in temperature. What should he do?
Answer: Charge with new refrigerant

232. The temperature at which water vapor in the atmosphere begins to condense is known as:
Answer: dew point

233. The ice making capacity is always


Answer: Directly proportional to the refrigerating effect

234. If the compressor short-cycles on the high-pressure cutout, which of the following would
you check?
Answer: All of the other choices
(If plenty of cooling water is running through but it is not picking up heat, the condenser
tubes need cleaning, Be sure system is getting cooling water, Check for too much
refrigerant in the system)

235. What is the instrument used to register relative humidity?


Answer: Hygrometer

236. What has an octane rating of more than


100? Answer: Benzol

237. A device used to keep moisture from passing through the system is
called: Answer: Dehydrator

238. If the freezing point of water is 0 C, which of the following is its melting point
following is its melting point
Answer: 0 C

239. What is the pressure present inside the casing of an impulse turbine?
Answer: Atmospheric pressure
240. A vena contracta in a fluid jet issuing through a hole in a plate is located
approximately: Answer: At jet’s minimum diameter

241. If the compressor short-cycles on the low-pressure cutout, the trouble might be:
Answer: Any of the other choices
(Too much frost on the evaporator coils, dirty traps and strainers, lack of refrigerant)

242. The amount of heat necessary to bring up temperature of a unit mass of a


substance through unit degree is called
Answer: Total heat

243. If the expansion valve capillary tube is pinched, which of the following be replaced?
Answer: All of the other choices (called the power element unit)
(Bulb, tube, diaphragm)

244. The coefficient of discharge is the ratio of the.


Answer: Actual discharge to the theoretical discharge

245. Pitot tube is used to measure


the: Answer: Velocity of flow

246. The principle of the centrifugal system is based on which of the


following? Answer: Kinetic energy

247. The disadvantage of a CO2 system over an ammonia system is the fact that
Answer: All of the other choices
(The pipes and fittings of a co2 system must be of the high-pressure type, The CO2 system
requires a larger prime remover, The CO2 system operates at a much higher pressure)

248. If ice will form in a solution of water and salt, then it is at temperature
called: Answer: Freezing point depression

249. The suction pressure in a Freon system should be:


Answer: The pressure which corresponds with a temperature about 20F below the
temperature of the icebox.

250. Wb is shaft work of an engine and Wi is indicated work of an engine. If mechanical


is present in the engine mechanism, then.
Answer: Wb is less than Wi

251. What is the relationship of the capacity of a centrifugal pump, Q, to impeller diameter,
D, when there are two impeller diameters in the same pump?
Answer: Q is directly proportional to the ratio of D
252. The combination of enthalpy and kinetic energy of fluid is termed as:
Answer: Stagnation enthalpy

253. Before any repair work is attempted on any gas-fired boiler the:
Answer: Main gas cock must be secured

254. An air-conditioning system in which water is chilled or cooled and which passes
the evaporator coils
Answer: Chilled water system

255. Refers to organic waste produced from biological water waste treatment
processes Answer: Biosolids

256. As far as combustion chamber design is considered, the maximum power output of a
given engine can be increased by:
Answer: Decreasing combustion chamber volume

257. A radioactive gas produce from the decay of radium within the rocks beneath a
building Answer: Radon

258. Ozone filters out what type of radiation that damages crops and causes skin
cancer? Answer: Ultraviolet radiation

259. The ratio of the sum of individual maximum demands of the system to the overall
maximum demand of the whole system
Answer: Diversity factor

260. A bell coleman cycle is also known as


Answer: Reversed Joule cycle

261. It is used deliver concentrated airstreams into a room. Many have one-way or two-way
adjustable air stream deflectors.
Answer: Register

262. The sensible heat ratio is 0.8. that is


Answer: 80% sensible heat and 20% latent heat

263. Closing the solenoid valve will stop the compressor through the
____. Answer: Low-pressure cutout switch

264. A refrigerating unit of one (1) ton capacity can remove?


Answer: 200 btu’s per min
265. Which of the following is the usual case of slugging?
Answer: Expansion valve not operating properly

266. All of the following occur during reduction of a substance


except Answer: Loss of electrons

267. The secondary refrigerant used in milk chilling plants is


generally Answer: Glycol

268. Where is the solenoid coil installed?


Answer: Vertically over the valve

269. The thermal expansion valve responds to the


Answer: Amount of superheat in the vapor

270. To eliminate transmission of vibration, the foundation should be isolated from the floor
slabs of building footings at least how may mm around its perimeter?
Answer: 25

271. Which one is commonly used liquid absorbent?


Answer: Ethylene glycol

272. The low-pressure control switch:


Answer: Cuts out the compressor to maintain proper flow

273. If the outlet of the thermostatic valve is warmer than the inlet, it indicates
Answer: Thermostatic valve not working properly

274. Compute the wall gain load of a load storage room when A is the outside surface area
of the wall, D is the temperature differential across the wall, and U is the overall coefficient
of heat transmission.
Answer: AUD

275. A leak on the fuel oil suction line between the tank and the suction side of the fuel
oil pump would result in:
Answer: Air entering the suction line

276. How is the CO2 system is purged?


Answer: When CO2 comes out of the purge valve, frost will form on a piece of
metal held near the outlet

277. The relief valve on a CO2 machine is located:


Answer: On the discharge pipe between the compressor and discharge valve
278. The high-pressure side of the system is sometimes referred to as the:
Answer: Hot side

279. Combined pressure of cooling and humidifying is also known as:


Answer: Evaporative cooling process

280. The schedule of a pipe, N, indicates the thickness of the pipe wall. If the allowable
stress of the pipe is S, then what is the internal pressure equal to?
Answer: N x S

281. Which of the following area of work requires lowest noise level?
Answer: Library

282. Which of the following contribute to the deterioration of the earth’s ozone layer?
Answer: Chlorofluorocarbons

283. What is the least number of compressors a multistage system that will use?
Answer: Two

284. Is a soluble compound that reduces a liquid’s surface tension or reduces the
interfacial tension between a liquid and a solid
Answer: Surface – acting agent or surfactant

285. To secure the belts embedded within the foundation, the distance of the edges of
the foundation from the bedplate must be ____.
Answer: 120 mm to 300 mm

286. In Stirling and Ericsson Cycles, the efficiency can be increased by:
Answer: Regeneration

287. The thermal expansion valve


Answer: Controls the quantity of liquid refrigerant going to the evaporator coils

288. Too low suction pressure could be caused by:


Answer: Any of the other choices
(Dirty scale traps, Shortage of refrigerant gas, Too much oil in the system)

289. The static pressure drop due to friction through the boiler and stack.
Answer: Draft loss

290. In a freon-11 system there is no


Answer: Distiller
291. The coefficient of contraction is the ratio of the:
Answer: Area of vena contracta to the orifice area

292. Which of the following should not be used to clean scale


traps: Answer: Cotton waste

293. Most observe properties of light and other radiant energy are consistent with waves in
nature, but in interaction with matter, electromagnetic energy behaves as though it consists of
discrete pieces or
Answer: Quanta

294. Boilers using soft coal must have ____ furnace volume to reduce the danger of ____
Answer: Large; smoking

295. Which of the following would you apply if a person got Freon in his eyes?
Answer: Sterile mineral oil

296. The compressor will run continuously if there


is: Answer: Any of the other choices
(Too heavy a load, Insufficient refrigerant, Air in the system)

297. A hot crankcase and cylinder head accompanied by a low suction pressure would
be caused by
Answer: Insufficient refrigerant

298. One foot water is equal to:


Answer: 62.43 lb/in2

299. The purpose of the oil trap is:


Answer: To remove oil from the refrigerating gas

300. Many pressure gauges on a Freon system have two dials or graduations on one
gauge. The two dial represents:
Answer: Pressure and temperature

301. The faster way to remove frost from a direct expansion finned tube evaporator is to:
Answer: Send hot gas through the coil

302. When ordering an expansion valve which of the following information is


necessary? Answer: Size, tonnage, temperature, and pressure

303. The elements of a thermostat switch are usually of the ____.


Answer: Bimetal type
304. Which of the following characteristics that is not desirable in a
refrigerant? Answer: Low latent heat of vaporization

305. What do bodies at a temperature above absolute zero?


Answer: It emits Thermal Radiation

306. The Horizontal Scale (Abscissa) in the psychrometric Chart represents:


Answer: Dry bulb temperature

307. The thermostatic expansion valve is designed to maintain constant


____. Answer: Superheat

308. The charging connection in a refrigerating system is located:


Answer: Between the king valve and the solenoid valve

309. The latent heat of fusion of ice


is Answer: 144 Btu

310. The ratio absorbed by the transfer fluid to the original incident energy striking the
collector
Answer: Collector efficiency

311. If m is the mass of dry air and H is the specific enthalpy of the water vapor in air and r is
the humidity ratio, then determine the latent heat of any dry air.
Answer: m(r x H)

312. The dehydrator is located between the:


Answer: Receiver and expansion valve

313. In a vapor compression cycle the lowest temperature is found


in Answer: Evaporator

314. The volume flow passes through a venturimeter is:


Answer: Constant

315. Anchor bolts in a machine foundation should be embedded in concrete of at least


how many time of the bolt diameter?
Answer: 30

316. Which do you think is the effect of “subcooling”?


Answer: It reduces the horsepower per ton of refrigeration
317. When air is saturated, the wet bulb depression
is: Answer: Zero

318. A precooler, if used is located between the:


Answer: Condenser and expansion valve

319. A Kaplan turbine is:


Answer: Low head axial flow turbine

320. The refrigerant temperature is at its maximum just before it enters the ____
Answer: Condenser

321. The machine foundation must have a factor of safety of:


Answer: 5

322. In a refrigerating system, the heat absorbed in the evaporator per kg mass of refrigerant
passing through
Answer: Equals the increase in enthalpy

323. Which of the following is the common classification of ducts?


Answer: All of these
(Conditioned Air – Duct, Recirculating, Fresh – air ducts)

324. All of the following process can be found in a psychrometric chart


except: Answer: Natural convection

325. If an electrically-operated compressor failed to start the cause might


be: Answer: Any of the other choices
(Burned out holding coils in solenoid valve, An open switch, A blown fuse)

326. Five pounds of water heated to raise the temperature 2F requires how many BTU?
Answer: 10 Btu

327. Air circulation in the icebox is accomplished by the use of which of the following?
Answer: Diffuser fans

328. In sensible heating cooling following parameter remains unchanged


Answer: Humidity ratio

329. Venturi meters, pitot static gauges, orifice meters, flow nozzles, and
differential manometers all depend upon the relationship between:
Answer: Flow velocity and pressure
330. It is used to deliver widespread, fan-shaped flows of air into the
room Answer: Diffuser

331. What turbidimeter that gives direct reading in ppm?


Answer: Jackson turbidimeter

332. How much will be removed by one-ton refrigeration unit?


Answer: 200 Btu per min

333. A double-trunk piston is used to:


Answer: All of other choices
(Prevent oil from mixing with the refrigerant, Prevent gas from getting to crankcase, Prevent
oil from mixing with the refrigerant)

334. Purpose of the receiver is to


Answer: Store the refrigerant

335. Latent heat


Answer: Cannot be measured with a thermometer

336. A type of water turbine where a jet of water is made to fall on the blades or buckets and
due to the impulse of water, the turbine starts to move.
Answer: Pelton wheel

337. When does the refrigerant gives-up heat?


Answer: When it condenses

338. A term used to mean the corrective steps taken to return the environment to its original
condition
Answer: Remediation

339. The back – pressure regulating valve:


Answer: Maintains a fixed pressure in the evaporator coils

340. What is the relationship of the horse power of a centrifugal pump, Hp, to the
impeller speed, S, if the pump is at two different rotative speeds?
Answer: Hp is directly proportional to the cube of the ratio of S

341. What must be the value of the available Net Positive Suction Head (NPHS) of
a centrifugal pump compared to its required NPHS to avoid losing priming?
Answer: Available NPHS greater than required NPHS
342. The diesel engine foundation safe soil bearing pressure is:
Answer: 4,890 kg/cm2

343. Aside from maintain appropriate temperature for food cold storage, how is
desiccation minimized or decreased?
Answer: Maintain humidity ratio

344. If C is the capacity of the compressor in a refrigeration system and F is the heat
rejection factor, then calculate the condenser load, L.
Answer: L = C x F

345. The following examples of indirect (secondary) measurements to measure flow


rates using obstruction meters except:
Answer: Weight and mass scales

346. When required, a regulator water valve in refrigerating system should be


Answer: In the water outlet

347. What is the relative humidity when the dew point and dry bulb temperature are
equal? Answer: 100%

348. Compare in refrigerating effect per unit mass of refrigerant circulated for a
superheated cycle that produces useful cooling and a saturated cycle, for the same
vaporizing and condensing temperature
Answer: Greater for a superheated cycle

349. If PV is the power required for a vapor compression refrigeration system, then what is
the power required for an air refrigeration system, assuming that they have the same
capacity? Answer: 5PV

350. What do you call the mixture if the solute particles of a solid suspended in a liquid are
larger than molecules?
Answer: Suspension

351. Lithium bromide is used in refrigeration system in:


Answer: Absorbers

352. The removal of dissolved gas or other volatile component from liquid by exposing
the liquid to air or steam is known as:
Answer: Stripping

353. Which of the following measures the density of salt in water?


Answer: Salimeter
354. In a power driven pump, each piston stroke is displaced by 360 divided the ____.
Answer: Number of cylinders

355. What is the boiling temperature of F 22?


Answer: -40 C

356. What is another name of discharge


pressure? Answer: Head pressure

357. If the compressor were to run out continuously without lowering the temperature, the
trouble would probably be:
Answer: Any of the other choices
(Insufficient refrigerant in the system, Leaky discharge valve, Leaks in the system)

358. The ratio of sum of individual maximum demands of the system to the maximum
demand of the whole system is known as
Answer: Diversity factor

359. The temperature bulb of solenoid valve is attached to the ____.


Answer: Wall of the icebox

360. What must be done to change the direction of rotation of a 440-volt, 3-phase induction
motor?
Answer: Interchange any two power leads

361. In order to remove the fly ashes from the flue gas, which of the following must a
power plant be equipped with?
Answer: Electrostatic precipitator

362. Which of the following is a type of water turbine?


Answer: Pelton

363. In sensible heating the absolute humidity remains constant but the relative humidity:
Answer: Decrease

364. All of the following process can be found on a psychrometric chart


except? Answer: Black body radiation

365. When checking zinc plates in a condenser, one should:


Answer: Clean the plates and renew worn out ones

366. Turbidity in water is due to:


Answer: Finally divided particles of clay, silt and organic matter
367. The force when applied to a mass of one kilogram will give mass an acceleration of one
meter per second for every second called:
Answer: Newton

368. A precooler is sometimes installed between the


Answer: Condenser and expansion valve

369. A type of polymer used for detergents, milk bottles, oil containers and toys
Answer: High density polyethelene (HDPE)

370. The fact that the amount of slightly soluble gas absorbed in a liquid is proportional to
the partial pressure of the gas is known as:
Answer: Henry’s Law

371. What is the pressure at the exit of a draft tube in a turbine?


Answer: Atmospheric

372. What do you call the device that is used as low-pressure control and high-pressure cutout
on a compressor?
Answer: Pressure Controller

373. Redox reactions can often be type of:


Answer: Double replacement

374. Excessive head pressure is caused


by Answer: Any of the other choices
(Dirty condenser tubes, Insufficient cooling water to condenser, Air or non condensable
gas in the system)

375. In which part of the vapor compression cycle there is abrupt change in pressure
and temperature?
Answer: Expansion valve

376. The kinetic energy of a moving fluid is used to isentropically compressed the fluid to
state of zero velocity. The temperature of a moving fluid at the state zero velocity is called:
Answer: Critical temperature

377. Which is not commonly used to cool and dehumidify equipment?


Answer: Calcium chloride

378. What is the primary cause of smog formation?


Answer: Nitrogen oxides
379. The dehydrating agent in a Freon system is usually
Answer: Activated alumina

380. The solenoid valve is actuated by which of the


following? Answer: A magnet

381. Which of the following does not use ambient air for
propulsion? Answer: Turbo-prop

382. Which of the following would not cause high suction


pressure? Answer: Insufficient refrigeration

383. What is the effect of superheating the refrigerant?


Answer: It increase the coefficient of performance

384. The color of the flame of halide torch, in case of leakage of Freon refrigerant, will change
to
Answer: Green

385. If there is too much lube oil in the system what must be done?
Answer: Remove same at once

386. If the thermal bulb becomes loose on the evaporator coils, it will cause ____.
Answer: Improper operation of expansion valve

387. The effect of superheating the refrigerant is to:


Answer: Decrease the COP

388. In power plant electrostatic precipitators is installed between:


Answer: Furnace and chimney

389. Equipment leaks from plant equipment are known as


Answer: Fugitive emissions

390. Per capita consumption of water is generally taken


as: Answer: 150-300 liters

391. What is the heat that is removed from the space to be cooled, which is the same as
the heat absorbed by the cooling coils?
Answer: Refrigerating effect

392. Horsepower per ton of refrigeration is expressed as:


Answer: Cop/4.75
393. What must be done first when opening a single packed stop
valve? Answer: Loosen the packing before opening

394. The amount of CO2 or Freon in a cylinder is measured


by Answer: Weight

395. The coefficient of velocity is approximately


Answer: 0.30 to 0.50

396. Carries needed to deliver air to the conditioned space it is made of sheet of metals like
aluminum, galvanized sheet steel and some structural materials that will not burn
Answer: Duct

397. Which of the following best described a Freon?


Answer: All of the other choices
(Colorless, Odorless, Non-poisonous)

398. Before securing a compressor to do maintenance work on it, be sure


to: Answer: Have spare parts ready and pump down the system

399. A bull’s eye in a full liquid line will appear ____.


Answer: Clear

400. What operates low-pressure cutout switch?


Answer: Bellows

401. A liquid mixture having constant maximum and minimum boiling points. Refrigerants
comprising this mixture do not combine chemically, yet the mixture provides constant
characteristics.
Answer: Azeotropic Mixture

402. Are organic compounds manufactured in oily liquid and solid forms through the
late 1970s and subsequently prohibited.
Answer: Polychlorinated biphenyls (PCBs)

403. What is the temperature range of an air-conditioning application where dry air can
be considered ideal gas?
Answer: -10 to 50 C

404. The coefficient of the velocity, Cv accounts for the:


Answer: Small effect on the flow area of contraction, friction, and turbulence
405. During the re-expansion portion of the refrigeration compressor cycle
Answer: The suction valve is closed and the discharge valve is closed

406. Refers to the high-temperature removal of tarry substances from the interior of the carbon
granule, leaving a highly porous structure
Answer: Activated

407. Are by products of reaction between combustion products


Answer: Oxidants

408. Are any solid particulate matter that becomes airborne, with the exception of
particulate matter emitted from the exhaust stack of a combustion process
Answer: Dusts or fugitive dusts

409. The purpose of the dehydrator is to


Answer: Remove moisture from the refrigerant

410. The angle between the stack and guy wire is


usually Answer: 60 degrees

411. A refrigeration that deals with producing temperature of -157 C or lower.


Answer: Cryogenics

412. The purge valve is located:


Answer: In the highest part of the system

413. Thermal expansion valves are usually made of


the: Answer: Diaphragm and Bellows type

414. The purpose of the expansion valve by-pass is to


Answer: Controls the refrigerant to the evaporator in case the automatic valves fail

415. The water regulating valve is operated by the


Answer: None of the other choices
(Compressor discharge pressure, Compressor suction pressure, Compression discharge
temperature)

416. Superheat is heat added ____.


Answer: After all liquid has been changed to vapor

417. The ratio of maximum load to the rated plant capacity is


called Answer: Utilization factor
418. The highest temperature in vapor compression cycle is produced
during: Answer: Compressor discharge

419. Dry analysis is a fractional analysis of the products of combustion which does not
include:
Answer: Water vapor

420. Valves and piping in an ammonia system are made of:


Answer: Iron

421. The locus of elevation to which water will rise in the piezometer tube is termed:
Answer: Hydraulic gradient

422. A substance with surface area per unit of weight, and intricate pore structure, and a
hydrophobic surface
Answer: Adsorbent substance

423. The purpose of the expansion valve is to control the flow of the refrigerant to
the evaporator. The other function is to:
Answer: Reduce the pressure of the liquid refrigerant

424. An ammonia leak will turn litmus paper ____.


Answer: Blue

425. What is the most common freon gas used in reciprocating


compressors? Answer: F-12

426. The empirical coefficient e in machine foundation if not given is


assumed Answer: 0.11

427. The capacity of a centrifugal type compressor is controlled by which of the


following? Answer: Regulating the speed or regulating the suction pressure

428. Antifreeze chemicals are:


Answer: Those that lower down the freezing points of liquids

429. A type of refrigerant that will not damage the ozone layer
Answer: Hydrofluorocarbons (HCF’s)

430. At 0 psig, how may BTU necessary to change 1 lb of water at 212 F of steam?
Answer: 970
431. Modern stacks are seldom built higher than ____.
Answer: 60 m

432. If Pi is the indicated horsepower and Pb is the indicated horsepower of a compressor,


then what is mechanical efficiency, Em, equal to:
Answer: Em = Pi/Pb

433. A refrigeration control that guards the compressor forms overloads brought about
by abruptly increases loads resulting from defrosting, warm products, and others is
called: Answer: Suction hold-back valve

434. When a solvent has dissolve as much as it can, the mixture is called:
Answer: Saturated solution

435. Fan motors in air conditioning usually


have: Answer: 2 or 3 speeds

436. In the upper atmosphere, ozone is made by ultraviolet light reacting with:
Answer: Oxygen

437. Which of the following is the oil used in a refrigeration system?


Answer: Straight mineral oil

438. The following are all examples of indirect (secondary) miscellaneous methods to
measure flow except:
Answer: Positive displacement meters

439. Which of the following is the function of a suction pressure regulating valve?
Answer: Maintains proper back pressure

440. What is the use of back pressure regulating valve? Answer:


Maintains a fixed pressure in the evaporator coils

441. If Wt is the turbine shaft work of a gas turbine unit, Wc in duel, then determine its
thermal efficiency.
Answer: (Wt-Wc) / Q

442. The solenoid valve is controlled by


Answer: The temperature in the icebox

443. Where is the excess refrigerant removed?


Answer: Charging side of the system
444. All of the following units of energy except
Answer: Pascals

445. What usually happened if brine has a high specific gravity?


Answer: It will crystallize

446. There are three basic boiler types, namely;


Answer: Cast-iron, fire-tube and water tube boilers

447. Measurements of a device’s ability to remove atmosphere air from test air.
Answer: Atmospheric Dust Spot Efficiency

448. The normal cut-out setting of a window unit thermostat


between: Answer: 13 C to 16 C

449. The use of water to carry heat occupied spaces


Answer: Hydronic Heating System

450. Reheating process is normally employed in steam plant when:


Answer: Turbine undergoes excessive moisture

451. In the discharge line between the compressor and the condenser one would
find: Answer: High pressure, high temperature gas

452. A good refrigerant should be


Answer: All of the other choices (Non-
poisonous, non-inflammable, non-explosive)

453. What is the relationship of the horse power of a centrifugal pump, Hp, to the
impeller speed, S, if the pump is at two different rotative speeds?
Answer: Hp is directly proportional to the cube of the ratio of S

454. Which of the following is the reason when the crankcase is cooler than the suction
line? Answer: Too much refrigerant

455. Humidity is a measure of which of the following?


Answer: Water vapor content

456. High head turbine is a/an:


Answer: Impulse

457. Critical temperature is that temperature above which


Answer: A gas will never liquefy
458. The fact that a fluid’s velocity increases as the cross-sectional area of the pipe through
which it flow decrease is due to:
Answer: The continuity equation

459. Which of the following is important for evaporator coils?


Answer: It should have air completely surrounding them

460. Which of the following types of air dryers works by absorbing moisture on a solid
desiccant or drying material such as activated alumina, silicone gel, or molecular
sieve? Answer: Deliquescent dryer

461. Which of the following would cause the compressor to run continuously?
Answer: Low pressure switch jammed

462. In a window air conditioning unit which of the following is usually done by the owner?
Answer: Semi-annual cleaning or replacement of filters

463. Which of the following is not similarity between submerged culvert and a siphon?
Answer: Torricelli’s equation holds

464. Where does the final removal of water vapor in an absorption refrigeration system occur?
Answer: Rectifier

465. The constant spending of certain percentage of circulated water in a cooling tower in
order to prevent accumulation of dissolved mineral solids and other impurities in the
condenser water is called
Answer: Bleed-off

466. Which of the following refrigerant is added sometimes to other refrigerant to improve oil
circulation?
Answer: R – 170 (Ethane)

467. The suction pressure switch is operated by which of the following?


Answer: Pressure on a bellow

468. When the air is saturated the wet-bulb depression


is: Answer: Zero

469. Heating and dehumidification can be obtained simultaneously if air is passed through:
Answer: Either a solid absorbent surface or a liquid absorbent spray
470. In cooling cycle, the dry bulb temperature (db) of the air is lowered. When this happens
the relative humidity
Answer: Increase

471. What will happen when the pressure at any point inside a centrifugal pump goes
below the vapor pressure corresponding to the temperature of the liquid?
Answer: Cavitation

472. The ratio between the actual power and the apparent power in any circuit known as the
____ of that circuit
Answer: Power factor

473. Refrigerant leakage from the compressor crankcase is prevented by


Answer: Using shaft seals

474. The matching of scale and full-scale results for a fluid dynamic phenomena
involving compressible fluids requires quality of:
Answer: Mach number

475. Ammonia leaks in the condenser can be detected by:


Answer: Applying litmus paper to the circulating water discharge

476. A swinging support constructed as part of the vessel and that supports the manway cover
when it is unbolted and moved aside.
Answer: Davit

477. Which of the following type valves are not found on a Freon – 12 system?
Answer: Duplex

478. What is the main function of a receiver?


Answer: Store the refrigerant

479. The dividing point between the high pressure and low pressure sides of the
refrigeration cycle occurs at the
Answer: Expansion valve

480. If the solute particles of a solid suspended in a liquid are larger than molecules the
mixture is known as?
Answer: Suspension

481. A mechanism that removes moisture


Answer: Dehumidifiers
482. Is a water soluble organic compound prepared from ammonia. It has significant
biological and industrial usefulness
Answer: Urea or carbamide urea

483. Which of the following fans in air conditioning systems which can be classified
as centrifugal flow
Answer: Propeller fan

484. What is the use of the low – water cutout switch?


Answer: Stops the compressor when there is insufficient cooling water

485. In parallel pipe system originating and terminating in common junctions,


Answer: Pressure drops through each branch are equal

486. When the dry bulb and the wet bulb temperature are identical, the air is said to
be: Answer: Saturates

487. The temperature in which water vapour in the gas begins to condensate in a
constant pressure process.
Answer: Dew point

488. Combination gas/fuel oil burners permit the operator to switch from one fuel to ____
Answer: All of the above
(For economy, Because of a shortage of fuel being used, Because of a failure in the
fuel system being used)

489. Most nuclear particles can react with atoms in several different ways
including Answer: Absorption and scattering

490. What is the effect of superheating the refrigerant?


Answer: It increase the coefficient of performance

491. Boiling temperature of Freon 12


is Answer: -29.8 C

492. Which of the following would cause low head pressure?


Answer: Too much cooling water and/ or insufficient refrigerant gas

493. At what temperature will water normally turns to steam?


Answer: 212 F

494. What will happen if the expansion valve is opened too wide?
Answer: Liquid will flood back to the compressor
495. All of the following are words used to describe neutron kinetic energy levels except:
Answer: Supersonic

496. If the thermal expansion valve becomes inoperative, the ice boxes will have to be
controlled by the ____.
Answer: Manual expansion valve

497. If the superheat on the suction side of the compressor is increased, what will happen to
the tonnage capacity of the unit?
Answer: Decreases

498. The solenoid valve can be typed as a ____.


Answer: Magnetic soap valve

499. The ratio of fugacity of actual conditions to the fugacity at some reference state is known
as:
Answer: Activity

500. The process that takes place in the evaporator called


Answer: Absorption of the latent heat of vaporization

501. When charging freon system, all the valves should be


Answer: King (liquid) valve

502. If the pressure exerted on a liquid is higher than the saturation corresponding to
it temperature, the liquid is a:
Answer: Sub cooled liquid

503. The most likely cause of high superheat would be:


Answer: Expansion valve closed too much

504. A good refrigerant should have


a Answer: High latent heat

505. What is the device used to protect the compressor from overloading due to high head
pressure
Answer: Overload relay

506. Which of the following it to be checked as part of weekly maintenance schedule of


a console air conditioners?
Answer: Cooling towers
507. Which of the following would not be cause for a refrigerating system to short cycle on hp
cutout?
Answer: Discharge valve leaking

508. A ____ boiler has heat and gases of combustion that pass through tubes surrounded by
water
Answer: Fire tube

509. A type of polymer used for clear bottles.


Answer: Polyvinyl Chloride (PVC)

510. In an ammonia system, the oil gauge must be kept:


Answer: Closed except when checking level oil

511. If a compartment requires the removal of 36,000 BTU per hour, how much is necessary
compressor capacity?
Answer: 3 tons

512. The oil level in the compressor should be checked:


Answer: After a long period of operation

513. Which of the following industries have the highest consumption of water for
processing? Answer: Paper mill

514. What is the lowest temperature to which water could possibly be cooled in a
cooling tower?
Answer: The temperature of adiabatic compression

515. The method of cooling which primarily used where ambient air temperatures are high and
relative humidity is used:
Answer: Swamp cooling

13. Pre-cooler – cools CO2 gas with glucose as cooling medium and removes a considerable amount
of moisture by condensation.
14. Psychrometry – study of property of air and its water vapor contents.
15. Methane Reactor – anaerobic digester where the decomposition or breakdown of organic
matter is taking place
16. Blower – raises fermented CO2 pressure from 2 H2O to 5-7 psig and necessary to overcome pipe
friction
17. DO meter – an instrument used to measure the amount of dissolved oxygen in the aeration tank
18. Isothermal compression – Solely from standpoint of thermodynamic efficiency in which
air compression should approach
19. Regeneration – Heat exchanger used to provide heat transfer between the exhaust gasses
and the air prior to combustion
20. Convection – Heat transfer due to density differential
21. Wastewater – effluent of unwanted surplus substance arising from the application of
any process.
22. Gate Valve – the type of valve which should be used only wide open or fully closed
23. Deodorizer – removes non-water soluble substance from CO2 gas one for regeneration while the
other one is in service
24. Brass – one type of pipe which is not recommended for use in ammonia refrigerating system
25. The CO2 percentage decrease with increase in excess air in the complete combustion of a
fuel with excess air.
26. The air is saturated if the dry and wet bulb reading in the air are identical or the same
27. Expansion valve – type of refrigerant control which maintains pressure difference between
high side and low side pressure refrigerating mechanism
28. The efficiency will have its lowest value when a centerline pump for water is operated with the
valve closed
29. In refrigeration system, the heat absorbed in the evaporator per kg of refrigerant passing
through equals the increase in enthalpy
30. Air passing through a spray chamber in which the spray water is recirculated but not heated or
cooled will be humidified at approximately constant wet bulb temperature
31. A receiver in an air compression system is used to avoid cooling air before using
32. If the operational of the usual rotary gear pump handling a liquid. The liquid is trapped between
the gear teeth and case and is carried around the discharge
33. The absolute viscosity of a liquid. In centipoise is 100 times the number of poise
34. The compressing air in a water jacketed air compressor, the power required to deliver it
depends on the temperature change of air for a given pressure range.
35. In the usual commercial ammonia refrigerating system, the ammonia that has just passed
through the expansion valve is partially vaporized
36. Check Valve - valve designed to allow a fluid to pass through in one direction only
37. Liquid receiver – a vessel permanently connected to a system by inlet and outlet pipes of
storage of liquid refrigerant.
38. Cooling of air below its dew point temperature is described as cooling and dehumidifying
25. Absolute Zero – refers to the temperature at which all molecular motion ceases according to the
kinetic theory of heat
26. Specific Roughness - It is a parameter that measures the average size of imperfections inside
the pipe
27. Saturation Temperature – is a temperature at which liquid starts to boil or the temperature at
which vapor begins to condense
28. Conduction – Heat transfer by contact
29. Coefficient of performance the ratio of refrigerant effect to work of compression
30. Thermometer – Instrument used to measure hotness or coldness of a body
31. Density – reciprocal of specific volume
32. Absolute pressure – considered as the actual pressure of the system
33. Flow work – the amount of energy needed by the fluid to leave or enter a system
34. Conduction – heat transfer from one fluid to another through a dividing wall
35. Evaporator – part of a refrigeration system in which the absorption of heat takes place
36. Internal energy – energy of gas due to its molecular motion
37. Back pressure – pressure of the low pressure side of refrigeration (suction pressure)
38. Automatic expansion valve – a refrigerant control valve that maintains a constant pressure in
a evaporator
39. Dry bulb temperature – surrounding air temperature
40. R-22 – monochlorodiflouromethane, a popular refrigerant for air conditioning system
41. Accumulator – used to prevent or minimize entry of suction line liquid refrigerant into
the compressor
42. Water flow meter – measure condensate, feed water, pump discharge, etc.
43. Density – non-intensive property
44. Cavitation – phenomenon in which liquid flashes into vapor in the suction section of the pump
and then collapse at high pressure section
45. 80 – heat of fusion of ice in calories per gram
46. R-502 – an azeotropic mixture of R-22 and R-115, a popular refrigerant for low temperature
refrigeration systems.
47. Specific humidity – mass of water vapor per unit mass of dry air
48. The tendency of a pump to cavitate will be increased if the fluid temperature is raised
49. Weight – the force acting on a body due to the gravitational pull of 9.81 meter per second
squared
50. Fluid – is a substance that exists as continuum characterized by low resistance to flow and
the tendency to assume the shape of its container
51. Subcooled Liquid – State of a substance whose temperature is below the
saturation temperature corresponding to its pressure
52. Compound pressure gauge is used to measure positive and negative pressure
53. Strainer – select one that is a safety device for a compressor
54. Centrifugal blowers can supply large volumes of air at low pressure
55. Isothermal work / Indicated work – The performance of a reciprocating compressor can be
expressed by
38. An adiabatic process – no heat is transferred
39. Charles’ law – if the pressure of the confined gas is constant, the volume is directly proportional to the
temperature
40. Rotary pump – used for low discharge, low head and pumping viscous liquids
41. Reciprocating pump – used for low discharge, high head, low speed and self-priming
42. Centrifugal pump – used for high discharge, low head, high speed, and not self-priming
43. Influent – wastewater that flows into each unit of operation of the wastewater treatment plant that is
partially or completely treated
44. Compressor – raises the pressure of CO2 gas to 250 psig so that it can be liquefied at -100oF
45. Dew point – the temperature at which the water vapor in the air condensate when the air is cooled at
constant temperature
46. Pumps – a machine that is used to add energy to a liquid in order to transfer the liquid from one point to
another point of higher elevation
47. Toxicity – It is the irreversible inhabitation of both acidogenic and methanogenic bacteria
48. Foam Trap – the equipment that is made to have CO2 gas is passed against a fine water spray
49. Refrigeration – maintaining a space cooler than the surrounding
50. pH – measure of alkalinity and acidity of the wastewater
51. Drift loss in cooling tower is about 10 to 20 %
52. Induced Draft Fan – draws the flue gas from the system and sends them up the stack
53. Packing - Soft material that can be shaped or compressed to provide a seal
54. R-12 – dichlorodifluoromethane
55. Tubing – pipe with a thin wall used to carry fluids
56. Pump down the system and have spare parts before scouring a compressor to do maintenance
57. Hydro-pneumatic tank – the tank is unfired vessel, initially full of air, into which water from mains is
pumped

39. 1. Monochlorofluoromethane is a refrigerant


known as:
a. R-12 c. R-22
b. R-13 d. None of the above

56. 2. Which of the ff. is a proper way of leak detection for R-12 and halocarbon refrigeration systems?
a. stains c. toxic odor
b. corrosion d. loss of cooling capacity

58. 3. It refers to the final constant weight reached by a hygroscopic substance after being dried
out
a. Bone Dry Weight c. Both a&b
b. Dry Weight d. NOTA

51. 4. Is a type of dryer in w/c material to be dried is supported on floor through w/c hot gases
pass
a. Tray dryer c. Rotary dryer
b. Hearth dryer d. Tower dryer

63. 5. It is the temp. at w/c liquid starts to boil or the temp. at w/c vapor begins to condense
a. Dry blub temp c. Dew point Temp
b. Sat. Temp d. Wet bulb temp

76. 6. The tendency of pump to cavitate will be increased if:


impeller has smooth finish
inlet edges are rounded
suction line velocities are lowered
fluid temp is raised

89. 7. For a single component system, the number of properties required to define a phase uniquely is:
a. 1 c. 3
b. 2 d. 4

102. 8. It draws the flue gas from the system and series them up the stack is:
a. induced draft fan c. forced draft fan
b. cyclone d. wind tunnel

103. 9. The study of properties of air and its water vapor contents
a. humidification c. ventilation
b. refrigeration d. psychrometry
106. 10.It exist when the total suction is above atmospheric pressure
a. suction lift c. suction pressure
b. suction head d. suction line

108. 11.It is the wastewater that will flow into each unit of operation of the wastewater treatment plant
that is partially or completely treated
a. liquid waste c. influent
b. solid waste d. effluent

113. 12.It raises pressure of CO2 gas to 250 psig so that it can be liquefied at -100F
a. CO2 compressor c. deodorizer
b. trim cooler d. foam trap
115. 13.A vessel permanently connected to a system
by inlet and outlet pipes of storage of liquid refrigerant
a. refrigerant container c. flash tank
b. liquid receiver d. surge tank

117. 14.Air flows through coil without making contact with the coil surface is known as
a. drawn air c. contact air
b. bypass air d. air load

119. 15.the process of lowering the temperature of given space and maintaining it for the
purpose of preserving and chilling certain substances
a. refrigeration c. chilling
b. cooling d. air-conditioning

122. 16.it will either cool or maintain a body at temp below that of its surrounding is known as the
heat engine c. heat pump
heat sinkd. refrigerating machine

127. 17.a reciprocating compressor has a piston


speed of 200 fpm at 400 rpm. Its stroke is approx.
a. 2.56 in c. 5.32 in
b. 3 in. d. 20 in

129. 18.a refrigeration system that combines two refrigeration units


multi-evaporation system
multi-stage compression system
cascade system
air-refrigeration cycle

133. 19.fire involving energized electrical equipment


a. class A fire c. class C fire
b. class B fire d. class D fire

135. 20.dew point temp is temp where water starts


to condense
to boil
to condense and boil
to condense and boil at given pressure

137. 21.glass prod process is broken down into 2 gen. categories: container prod (glass bottle/bottle) and sheet
prod (float glass)
container & sheet prod
box & sheet prod
container & flattening prod
NOTA

139. 22.modern detergents are comprised of three main components


builders, blend, extender
binders, bleach, enzymes
builders, bleach, enzymes
No answer
142. 23.is a broad term applied to large number 163. 30.water substance at 70 bar and 65C enters
of carbohydrates composed of carbon, a boiler tube of constant inside diameter of
hydrogen, and oxygen present in many 25 mm. the water leaves the boiler tube at 50
plants and characterized by a more or less bar and 700K at velocity of 100 m/s.
sweet taste Calculate the inlet volume flow in L/sec.
a. sugar c. iron At 70 bar & 65C:
b. salt d. sweet v1 = 0.001017 m3/kg
At 50 bar & 700K:
146. 24.a closed vessel contains air at a pressure v2 = 0.06081 m3/kg
of a. 0.75 c. 0.82
160 kN/m2 gauge and temp of 30C. The air b. 0.64 d. 0.96
is heated at const. vol. to 60C w/ atm
pressure of 759mmHg. What is the final 165. 31.twenty grams of oxygen gas (O2) are
gauge pressure? compressed at const temp of 30C to 5% of
a. 174 c. 167 their original vol. What work is done on the
b. 186 d. 172 system?
`Use R of air = 0.0619 cal/gm-K
149. 25.what is the total reqd heating energy in a. 824 cal c. 944 cal
raising the temp of given amt of water when b. 894 cal d. 1124 cal
the energy supplied is 1000kWh w/ heat ( )( )

losses of 25%
a. 1000 c. 1333 168. 32.gas is enclosed in a cylinder w/ weighted
b. 1500 d. 1250 piston as top boundary. The gas is heated
and expands from a volume of 0.04 m3 to
̅̅̅̅

0.10 m3 at const pressure of 200 kPa.


151. 26.a certain gas at 101.325 kPa and 16C Calculate the work done by system.
whose vol is 2.83m3 are compressed into a a. 8 kJ c. 12 kJ
storage vessel of 0.31 m3 capacity. b. 10 kJ d. 14 kJ
Before admission, the storage vessel ( )

contained the
gas at pressure and temp of 137.8kPa and 173. 33.if F scale is twice C scale, what is the
24C. After admission, the pressure has reading in the Fahrenheit scale?
increased to 1171.8 kPa. What should be the
a. 160 c. 140
final temp of gas in vessel (in Kelvin)? b. 320 d. 280
a. 298.0 c. 180
b. 319.0 d. 420
179. 34.a cylinder & piston arrangement contains
sat water vapor at 110C. The vapor is
154. 27.a tank contains 80 ft3 of air at pressure of compressed in a reversible adiabatic process
350 psi. If the air is cooled until its pressure until pressure is 1.6 MPa. Determine work
and temp decrease to 200 psi and 70F done by the system per kg of water
respectively, what is the decrease in internal
At 110C:
energy? S1 = 7.2387 kJ/kg-K
a. 4575 c. 5552
U1 = 2518.1 kJ/kg
b. -5507 d. 0 At 1.6 MPa:
S2 = 7.2374 kJ/kg-K
155. 28.an electric heater is to heat 11 kg of oil U2 = 2950.1 kJ/kg
per min, from 4.5C to 65.5C. Specific heat
T2 = 400C
of oil is 2.1 J/gm-C. How many watts should
a. -500 kJ/kg c. -632 kJ/kg
this heater consume?
b. -432 kJ/kg d. -700 kJ/kg
a. 20 448 W c. 20 400 W ( )

b. 33 448 W d. 23 442 W
180. 35.During polytrophic process of ideal gas
159. 29.find the heat transfer reqd to convert 5 kg state changes from 138 kPa and 5C to 827
of water at pressure if 20 bar and temp of kPa and 171C. Find value of n.
21C into steam dryness fraction of 0.90 at a. 1.354 c. 1.345
same pressure. b. 1.253 d. 1.234
Properties of steam:
( ) ( )
P = 20 bar hf
= 909 kJ/kg
hfg = 1899 kJ/kg 182. 36.calculate specific vol of air-vapor mixture
Note: for water at 21C: in cu. m per kg of dry air when the ff.
h = 88 kJ/kg conditions are given:
a. 10 160 kJ c. 12 610 kJ t = 30C
b. 11 610 kJ d. 15 610 kJ w = 0.015kg/kg
{[ ( )] } P = 90kPa
3
a. 2.12 m3/kg c. 0.99 m /kg
b. 0.622 m3/kg d. 1.515 m3/kg
A 37.copra enters dryer containing 40% H2O & C 44.a refrigerator having 30kW capacity
60% solid, and leaves w/ 5% H2O & 95% requires 10HP compressor. Find COP of
solid. Find wt. of H2O removed per lb. of system.
final prod. a. 2.78 c. 4.02
a. 0.583 lb/lb c. 0.475 lb/lb b. 3.78 d. 5.02
b. 1.3753 lb/lb d. 1.015 lb/lb
A 45.A VCRS is designed to have a capacity of
A 38.an air conditioning plant w/ capacity of 100 TOR. It produces chilled water from
400kW has an evaporating & condensing 22C to 2C. Its actual COP is 5.86 and 35%
temp of 3C and 37C respectively. If it uses of power supplied to compressor is lost in
R-12, what is the volume flow rate under form of friction and cylinder cooling losses.
suction conditions? Determine the size of electric motor reqd to
At 3C drive compressor in KW and vol flow rate of
hg = 352.75 kJ/kg chilled water in L/s.
v = 50.47 L/kg a. 92.31 kW, 4.199 L/s
hf = 202.78 kJ/kg b. 90.71 kW, 5.277 L/s
`Enthalpy leaving cond = 235.3503kJ/kg c. 93.75 kW, 5.724 L/s
a. 0.172 m3/s c. 3.411 m3/s d. 91.75 kW, 7.575 L/s
b. 0.281 m3/s d. 3.5 m3/s
A 46.a single acting twin cylinder ammonia
D 39.fish weighing 11,000 kg w/ temp of 20C is compressor w/ bore equal to stroke is driven
brought to cold storage & w/c shall be by engine at 250 rpm. The machine is
cooled to -10 C in 11hrs. Find the reqd plant installed in chilling plant to produce 700 kW
capacity in tons if Cp of fish above and of refrigeration at -18C evaporator temp. At
below freezing is 0.7 kCal/kg-C and 0.3 this temp, cooling effect per kg mass = 1160
kCal/kg-C respectively, w/ freezing pt at - kJ. Specific vol of vapor entering
3C. compressor = 0.592 m3/kg. Assume 85%
`The latent heat of freezing = 55.5 kCal/kg volumetric efficiency, determine the bore in
a. 25.26 c. NOTA mm.
b. 15.26 d. 24.38 a. 400 mm c. 500 mm
b.300 mm d. 450 mm
185. 40.an ice plant produces 20 tons of ice per day
at -15C from water at 25C. If losses are 12% A 47.A refrigeration compressor having 10 in.
of freezing and chilling load, calculate flywheel is driven by a 4-pole AC motor. If
refrigeration capacity of plant in TOR. diameter of motor is 4 in, determine speed
a. 28.13 c. 31.5 of compressor.
b. 28.13 d. 41.5 a. 70 rpm c. 600 rpm
b. 500 rpm d. 800 rpm
188. 41.calculate tons of refrigeration reqd to freeze
in 4 hrs. 3mm of 10C water on a skating risk A 48.calculate piston displacement of 2-cylinder
61m x 30m if the ground temp is 10C and compressor rotating at 1450 rpm if diameter
the air temp is 10C and the air temp above of cylinder is 2.5 in. & length of stroke is 2
rink is 16C. in.
Heat transfer coefficient a. 16.48 ft3/min c. 14.48 ft3/min
air to 0C ice = 6.8 W/m2-C b. 15.48 ft3/min d. 17.48 ft3/min
grnd to -6C = 0.3402 W/m2-C
a. 101.69 c.140.62 A 49.a duct 0.40 m high and 0.80 m wide
b. 151.68 d. 201.96 suspended from the ceiling in a corridor
makes a right angle turn in the horizontal
D 42.The power requirement of Carnot refrigerator plane. The inner radius is 0.2m and outer
in maintaining a low temp region at 300K is radius is 1.0 m measured from the same
1.5 kW per ton. Find heat rejected. center. The velocity of air in duct is 10 m/s.
a. 4.02 kW c. 6.02 kW Compute the pressure drop in this elbow.
b. 7.02 kW d. 5.02 kW Assume:
f = 0.3
D 43.4000 L/hr of distillates are to be cooled from density = 1.204 kg/m3
21C to -12C and 12% of wax by wt. is L = 10m
separated out at 15C. The cp heat of oil is 2 a. 341 Pa c. 143 Pa
kJ/kg-C and SG is 0.87. The specific heat of b. 441 Pa d. 144 Pa
wax is 2.5 and latent heat is 290 kJ/kg.
Allow 10% for losses, find the refrigerating C 50.determine the sensible heat of 5 lb of air
capacity of system. having dry bulb temp of 70F and humidity
a. 20 TOR c. 40 TOR ratio of 0.0092 lb/lb, the latter corresponding
b. 51.08 TOR d. 31.08 TOR to dew pt temp of 55F.
a. 94 BTU c. 84 BTU
b. 48 BTU d. 49 BTU
C 51.a 0.70 m vane axial fan is running at 2000 A 55.a mixture of dry air and water vapor is at
rpm delivers 7.5 m3/s against 0.08m static temp of 21C under pressure of 101kPa. The
pressure thereby consuming 12 kW. If fan dew pt temp is 15C. Calculate the relative
wheel diameter is increased from 0.70 m to humidity.
0.76 m, so the diameter ration is 1.10:1, a. 68.56% c.56.68%
what is the new static pressure? b. 65.68% d. 58.66%
a. 1.01m c. 0.097m
b. 0.314m d.31.01m A 56.a dryer is to deliver 1000 kg/hr of palay
with a final moisture content of 10%. The
B 52. Centrifugal pump has const speed of 1500 initial moisture content in the feed is 15% at
rpm and has head of 75m. What is the effect atm cond w/ 32C dry-bulb & 21C wet-bulb.
on the head of the pump if the impeller The dryer is maintained at 45C while RH of
diameter is reduced from 280mm to 230 hot humid air from dryer is 80%. If steam
mm? pressure supplied to heater is 2 MPa,
a. 25.6 mm c. 65.6 mm determine air supplied to dryer in m3/hr.
b. 50.6 mm d. 72.6 mm a. 1332.25 m3/hr c. 1223.25 m3/hr
b. 1233.25 m3/hr d. 1523.13 m3/hr
191. 53.in an air conditioning unit, 3.5 m3/s of air at
27C dry-bulb temp, 50%RH & std. atm A 57.An assembly hall was to have an air
pressure enters unit. The leaving condition conditioning unit installed w/c would be
of air is 13C dry-bulb temp and 90% RH. maintained at 26C dry-bulb & at 50%RH.
Using properties from psychro chart, The unit delivers air at 15C dry-bulb temp &
calculate refrigerating capacity in kW. the calculated sensible heat load is 150kW
a. 87.57 kW c. 57.87 kW & latent heat is 51.3kW. 20% by wt of
b. 77.57 kW d. 58.77 kW extracted air is made up of outside air at 34C
dry-bulb & 60% RH, while 80% is extracted
D 54.a stream of outdoor air is mixed with steam by air conditioner from assembly hall.
of return air in an air conditioning system Determine the air conditioner’s refrigerating
that operates at 101kPa pressure. The flow capacity in TOR & its ventilation load in
rate of outdoor system air is 2 kg/s and its kW.
condition is 35C dry-bulb temp & 25C wet- a. 83.22 TOR, 37.47 kW
bulb temp. he flow rate of return is 3 kg/s & b. 76.43 TOR, 57.34 kW
its cond is = 24C & 50%RH. Determine c. 89.56 TOR, 45.77 kW
enthalpy of mixture. d. 56.78 TOR, 47.68 kW
a. 91.56 kJ/kg c.69.15 kJ/kg
b. 51.69 kJ/kg d. 59.16 kJ/kg
A double suction centrifugal pump delivers 70 ft3/sec of water at =505.32 kw
a head of 12 m and running at 1250 rpm. What is the specific What power can a boiler feed pump can deliver a mass of 35 kg/s water at a
speed of the pump? head of 500 m?
=9968.34 rpm =171.675 kw
The pump centerline of a centrifugal pump is located 2.5 m above A pump running at 100 rpm delivers water against a head at 30 m. If pump
from the high tide level. The sea water varies two meters from high speed will increased to 120 rpm, what is the increase in head?
tide to low tide level. If friction loss at the suction is 0.8 m, determine =13.2 m
A pump is used to deliver 50 li/sec of sea water at a speed of 120 rpm. If
the total suction head.
speed will increased to 135 rpm, determine the increased in pump capacity.
=5.3 m =6.25 li/s
A centrifugal pump requires 40 ft head to deliver water from low level to A 15 kW motor running at 350 rpm is used to drive a pump. If speed will
higher level. If pump speed is 1600 rpm, determine the impeller diameter of changed to 370 rpm, what is the increased in power?
the centrifugal pump =2.72 kw
=184.61 mm
The suction pressure of a pump reads 2 in of mercury vacuum and discharge A pump delivers 500 gpm of water against a total head of 200 ft and
pressure reads 130psi is use to deliver 100 gpm of water with specific operating at 1770 rpm. Changes have increased the total head to 375 ft. at
volume of 0.0163 ft3 /lb. Determine the pump work The suction pressure of what rpm should the pump be operated to achieve the new head at the
a pump reads 2 in of mercury vacuum and discharge pressure reads 130psi same efficiency?
is use to deliver 100 gpm of water with specific volume of 0.0163 ft3 /lb. =2424 rpm
Determine the pump work A centrifugal pump delivers 80 liters per second of water on test. Suction
=5.69KW gage reads 10 mm Hg vacuum and 1.2 meters below pump centerline.
A pump is to deliver 150gpm of water at a head of 120 m. If pump efficiency Power input is 70 kW. Find the total dynamic head in meters
is 70%, what is the horsepower rating of motor required to drive the pump? =66m
=21.33 HP A vacuum pump is used to drain a flooded mine shaft of 20°C water. The
A motor is used to drive a pump having an efficiency of 85% and 70% pump pressure of water at this temperature is 2.34 kPa. The pump is
respectively. What is the combined efficiency of pump and motor? incapable of lifting the water higher than 10.16 m. What is the atmospheric
=59.5% pressure?
In a boiler feed pump, the enthalpy at the entrance is 765 kJ/kg. If pump has =101.9
a head of 900 m, what is the exit enthalpy of the pump? It is desired to deliver 5 gpm at a head of 640 m in a single pump having
=773.83 kj/kg specific speed not to exceed 40. If the speed is not to exceed 1352 rpm, how
A submersible pump delivers 350 gpm of water to a height of 50 ft from the many stages are required
ground. The pump were installed 120 ft below the ground level and a draw =2 stages
down of 8 ft during the operation. If water level is 25 ft above the pump, A pump driven by an electric motor moves 25 gpm of water from reservoir A
determine the pump power to reservoir B, lifting the water to a total of 245 feet. The efficiency of the
=7.25 kw pump and motor are 64% and 84% respectively. What size of motor (HP) is
Determine the number of stages needed for a centrifugal pump if it is used required?
to deliver 400 gpm of water and pump power of 15 Hp. Each impeller = 2.42 hp=3 hp
develops a head of 38 ft. A fuel pump is delivering 10 gpm of oil with a specific gravity of 0.83. The
=4 stages total head is 9.14 m, find how much energy does the pump consumes in kJ
A boiler feed pump receives 50 li/sec of water with specific volume of per hour.
0.00112 m3 /kg at a head of 750 m. What is the power output of driving =169 kj/kg
motor if pump efficiency is 65%?
PIPE PROBLEMS F = 0.138 (9)0.5 = 0.414 m2

Find the work posses for a Helium gas at 200C A 29.53” x 39.37” pressure vessel contains ammonia with f = 0.041. Compute the
minimum required discharge capacity of the relief device in kg/hr.
*A. 609 KJ/kg B. 168 KJ/kg C. 229 KJ/kg D. 339 KJ/kg A. 106.71 kg/hr B. 108.71 kg/hr *C. 110.71 kg/hr D. 112.71 kg/hr
Solution:
Solution: C = f D L, kg/s
W = m R T = m (8.314 / M) T C = 0.041(29.53/39.37)(39.37/39.37) = 0.03075 kg/s (3600) = 110.71 kg/hr
For helium, M = 4
W/m = (8.314/4)(20 + 273) = 609 KJ/kg Compute the maximum length of the discharge pipe installed on the outlet of a pressure-relief
device in feet for internal pipe diameter of 0.5 inch and rated discharge capacity is 8 lb/min
Two kilogram of gas is confined in a 1 m3 tank at 200 kpa and 880C. What type of gas is of air. The rated pressure of relief valve is 16 psig.
in the tank? *A. 0.286 ft B. 0.386 ft C. 0.486 ft D. 0.586 ft
A. Helium *B. Ethane C. Methane D. Ethene Solution:
Solution: P = Pg + Patm = 16 x 1.1 + 14.7 = 32.3 psia
L = 9P2d5/16Cr2 = 9(32.3)2(0.5)5/16(8)2 = 0.286 ft
PV=mRT
200 (1) = 2 (8.314/M)(88+273) A thermal power plant has a heat rate of 11,363 Btu/kw-hr. Find the thermal efficiency of
M=30 the plant.
Therefore: the gas is Ethane (C2 H6) A. 28% *B. 30% C. 34% D. 40%
Solution:
Find the enthalpy of Helium if its internal energy is 200 KJ/kg eth = 3412/Heat rate = 3412/11,363 = 30%
A.144 KJ/kg B. 223.42 KJ/kg *C. 333.42 KJ/kg D. 168 KJ/kg
Solution: What is the hydraulic gradient of a 1 mile, 17 inches inside diameter pipe when 3300
gal/min of water flow with f = 0.03.
R = 8.314/4 = 2.0785 *A. 0.00714 B. 0.00614 C. 0.00234 D. 0.0018
K = 1.667 for helium Solution:
Cp = k R/(k - 1) = 1.667(2.0785)/(1.667 – 1) = 5.195 KJ/kg-K v = (3300/7.481)/(π/4)(17/12)2(60) = 4.66 ft/s
Cv = R/(k – 1) = 2.0785/(1.667 – 1) = 3.116 KJ/kg – K L = 1 mile = 5280 ft
∆h/∆U = Cp/Cv hL = fLv2/2_D = 0.03(5280)(4.66)2/2(32.2)(17/12) = 37.7 ft
∆h/200 = 5.195/3.116 Hydraulic gradient = 37.7/5280 = 0.007.14
∆h = 333.42 KJ/kg
Find the loss of head in the pipe entrance if speed of flow is 10 m/s.
Compute the mass of a 2 m3 propane at 280 kpa and 40˚C. A. 5.10 m B. 10.2 m C. 17.4 m *D. 2.55 m
A. 6.47 kg B. 5.1 kg C. 10.20 kg *D. 9.47 kg Solution:
Solution: Loss at entrance = 0.5 (v2/2g) = 0.5 [102 / 2(9.81)] = 2.55 m
Propane is C3 H3-------------- M = 12(3) + 8(1) = 44
PV = m R T Wet material, containing 220% moisture (dry basis) is to be dried at the rate of 1.5 kg/s in
280(2) = m (8.314/44)(40 + 273) a continuous dryer to give a product containing 10% (dry basis) . Find the moisture
m = 9.47 kg removed, kg/hr
*A. 3543.75 kg/hr B. 3513.75 kg/hr C. 3563.75 kg/hr D. 3593.75 kg/hr
Compute the air flow in ft3/min of mechanical ventilation required to exhaust an Solution:
accumulation of refrigerant due to leaks of the system capable of revolving air from the Solid in wet feed = solid in dried product
machinery room for a mass of 4 lbs refrigerant. [1/(1 + 2.2)](1.5) = [1/(1 + 0.1)](x)
*A. 200 B. 210 C. 220 D. 230 x = 0.5156 kg/s (total dried product)
Solution: Moisture removed = 1.5 – 0.5156 = 0.984 kg/s = 3543.75 kg/hr
Q = 100 x G0.5 ft3/min
Q = 100 x (4)0.5 = 200 ft3/min Copra enters a dryer containing 70% moisture and leaves at 7% moisture. Find the
moisture removed on each pound on solid in final product.
Compute the free-aperture cross section in m2 for the ventilation of a machinery room if A. 6.258 lb B. 1.258 lb C. 4.258 lb *D. 2.258 lb
the mass of refrigerant is 9 kg. Solution:
A. 0.314 *B. 0.414 C. 0.514 D. 0.614 Solid in wet feed = solid in dried product
Solution:
F = 0.138 G0.5 m2
0.3x = 1 At 100˚C
x = 3.333 lbs ht = cp t = 4.187 (100) = 418.7 KJ/kg
1 = 0.93y htg = 2257 KJ/kg
y = 1.07527 lb h2 = h1 + x htg = 418.7 + 0.60(2257) = 1,772.9 KJ/kg
Moisture removed = x – y = 3.333 – 1.07527 = 2.258 lb Q = 1(4.187)(100 – 30) + 1(1772.9 – 418.7) = 1,647.29 KJ/kg

Find the enthalpy of water at 212˚F and 14.7 psi if the dryness factor is 30%. Use
A 1 m x 1.5 m cylindrical tank is full of oil with SG = 0.92. Find the force acting at the the approximate enthalpy formula of liquid.
bottom of the tank in dynes. A. 461 Btu/lb *B. 471 Btu/lb C. 481 Btu/lb D. 491 Btu/lb
A. 106.33 x 103 dynes B. 106.33 x 104 dynes C. 106.33 x 105 dynes *D. 106.33 x Solution:
40. dynes ht = (˚F – 32) = (212 – 32) = 180 Btu/lb
Solution: htg = 970 Btu/lb
P = w h = (0.92 x 9.81) (1.5) = 13.5378 kpa h = ht + x htg
F = PA = 13.5378(π/4 x 12) = 10.632 KN = 10,632.56 N x 10,000 h = 180 + 0.3(970) = 471 Btu/lb
dynes/N F = 106.33 x 106 dynes
An air compressor consumed 1200 kw-hr per day of energy. The electric motor driving the
Find the pressure at the 100 fathom depth of water in kpag. compressor has an efficiency of 80%. If indicated power of the compressor is 34 kw, find
*A. 1,793.96 kpag B. 1,893.96 kpag C. 1,993.96 kpag D. 1,693.96 the mechanical efficiency of the compressor.
kpag A. 117.65 % B. 75 % *C. 85 % D. 90 %
Solution: Solution:
H = 100 fathom x 6 = 600 ft P/m = 1200kw-hr/24 hrs = 50 kw
P = w h = (600/3.281)(9.81) = 1,793.96 kpag BP = 50(0.80) = 40 kw
em = 34/40 = 85 %
Find the depth in furlong of the ocean (SG = 1.03) if the pressure at the sea bed is
2,032.56 kpag. A refrigeration system consumed 28,000 kw-hr per month of energy. There are 20 % of
*A. 1 B. 2 C. 3 D. 4 energy is lost due to cooling system of compressor and motor efficiency is 90 %. If COP of
Solution: the system is 6, find the tons of refrigeration of the system.
P=wh A. 43.15 TR B. 46.15 TR *C. 49.15 TR D. 41.15 TR Solution:
2,032.56 = (1.03 x 9.81) h
h = 201.158 m x 3.281 ft/m x 1 yd/3ft x 1 furlong/220yd = 1 furlong P/m = 28,800/(24 x 30) = 40 kw
BP = 40(0.90) = 36 kw
Find the mass of 10 quartz of water. Wc = 36(1 – 0.20) = 28.80 kw
A. 10.46 kg *B. 9.46 kg C. 11.46 kg D. 8.46 kg COP = RE/Wc
Solution: 6 = RE/28.80
V = 10 quartz x 1gal/4quartz x 3.785li/1gal x 1m3/1000li RE = 172.8/3.516 = 49.15 TR
V = 0.0094625 x 10-3m3
w = m/V A 23 tons refrigeration system has a heat rejected of 100 kw. Find the energy efficiency
1000 = m/0.0094625 x 10-3 ratio of the system.
m = 9.46 kg A. 13.42 *B. 14.42 C. 15.42 D. 16.42
Solution:
Find the mass of carbon dioxide having a pressure of 20 psia at 200˚F with 10 ft3 volume. QR = RE + Wc
A. 1.04 lbs B. 1.14 lbs *C. 1.24 lbs D. 1.34 lbs 100 = 23(3.516) + Wc
Solution: Wc = 19.132 kw
PV = m R T COP = RE/Wc = (23 x 3.516) / 19.132 = 4.32
(20 x 144)(10) = m (1545/44)(200 + 460) EER = 3.412 COP = 3.412(4.23) = 14.42
m = 1.24 lbs
A 200 mm x 250 mm, 8-cylinder, 4-stroke diesel engine has a brake power of 150 kw. The
Find the heat needed to raise the temperature of water from 30˚C to 100˚C with 60% mechanical efficiency is 80%. If two of the cylinders were accidentally cut off, what will
quality. Consider an atmospheric pressure of 101.325 kpa. Use the approximate enthalpy be the new friction power?
formula of liquid. A. 31.50 kw B. 33.50 kw C. 35.50 kw *D. 37.50 kw
A. 293.09 KJ/kg B. 1,772.90 KJ/kg C. 1,547.90 KJ/kg *D. 1,647.29 KJ/kg Solution:
Solution: em = BP/IP
0.8 = 150/IP
IP = 187.5 kw Compute the mass of a 2 m3 propane at 280 kpa and 40˚C.
FP1 = IP – BP = 187.5 – 150 = 37.50 kw A. 6.47 kg B. 5.1 kg C. 10.20 kg *D. 9.47 kg
FP1 = FP2 = 37.50 kw Solution:
Propane is C3H6 ------------------------M = 12(3) + 8(1) = 44
If the energy efficiency ratio of the refrigeration system is 12.6, what is the COP of PV = m R T
the system? 280(2) = m (8.314/44)(40 + 273)
*A. 3.69 B. 4.23 C. 5.92 D. 6.83 m = 9.47 kg
Solution:
EER = 3.412 COP Compute the air flow in ft3/min of mechanical ventilation required to exhaust an
12.6 = 3.412 COP accumulation of refrigerant due to leaks of the system capable of revolving air from the
COP = 3.69 machinery room for a mass of 4 lbs refrigerant.
*A. 200 B. 210 C. 220 D. 230
An air compressor has a power of 40 kw at 4 % clearance. If clearance will increase to 70 Solution:
%, what is the new power? Q=100 x G0.5 ft3/min
A. 70 kw *B. 40 kw C. 53kw D. 60 kw Q=100 x (4)0.5 = 200 ft3/min
Solution:
The power of compressor will not be affected with the changes in clearance. Compute the free-aperture cross section in m2 for the ventilation of a machinery room if
Therefore the power will still be 40 kw. the mass of refrigerant is 9 kg.
A. 0.314 *B. 0.414 C. 0.514 D. 0.613
What is the approximate value of temperature of water having enthalpy of 208 Btu/lb? Solution:
A. 138.67 ˚C *B. 115.55 ˚C C. 258.67 ˚C D. 68.67 ˚C F = 0.138 G0.5 , m2
Solution: F = 0.138(9)0.5 = 0.414 m2
h = ˚F – 32
A 29.53” x 39.37” pressure vessel contains ammonia with f = 0.041. Compute the minimum
Find the work posses for a Helium gas at 20°C. required discharge capacity of the relief device in kg/hr.
*A. 609 KJ/Kg B. 168 KJ/Kg C. 229 KJ/Kg D. 339 KJ/Kg A. 106.71 kg/hr B. 108.71 kg/hr *C. 110.71 kg/hr D. 112.71 kg/hr
Solution:
W = m R T = m (8.314/M) T Solution:
For Helium, M = 4 C = f D L, kg/s
W/m = (8.314/4)(20 + 273) = 609 KJ/Kg C = 0.041(29.53/39.37)(39.37/39.37) = 0.03075 kg/s (3600) = 110.71 kg/hr

Two kilogram of gas is confined in a 1 m3 tank at 200 kpa and 88˚C. What type of gas is in Compute the maximum length of the discharge pipe installed on the outlet of a pressure-
the tank? relief device in feet for internal pipe diameter of 0.5 inch and rated discharge capacity is
A. Helium *B. Ethane C. Methane D. Ethene 8lb/min of air. The rated pressure of relief valve is 16 psig.
Solution: *A. 0.286 ft B. 0.386 ft C. 0.486 ft D. 0.586 ft
PV=mRT Solution:
200 (1) = 2 (8.314/m)(88 + 273) P = Pg + Patm = 16 x 1.1 + 14.7 = 32.3 psia
M=30 L = 9P2d5/16Cr2 = 9(32.3)2(0.5)5/16(8)2 = 0.286 ft
Therefore: the gas is Ethane (C2H6)
A thermal power plant has a heat rate of 11,363 Btu/kw-hr. Find the thermal efficiency of
Find the enthalpy of Helium if its internal energy is 200 KJ/kg. the plant.
A. 144 KJ/kg B. 223.42 KJ/kg *C. 333.42 KJ/kg D. 168 KJ/kg A. 28% *B.30% C.34% D.40%
Solution:
Solution: eth = 3412/heat rate = 3412/11,363 = 30%
R = 8.314/4 = 2.0785
K = 1.667 for helium What is the hydraulic gradient of a 1 mile, 17 inches inside diameter pipe when 3300 gal/min
cp = k R/(k – 1) = 1.667(2.0785)/(1.667 – 1) = 5.195 KJ/kg-K of water flow with f…. 0.03
cv = R/(k – 1) = 2.0785/(1.667 – 1) = 3.116 KJ/kg-K *A. 0.00714 B. 0.00614 C. 0.00234 D. 0.00187
∆h/∆U = cp/cv Solution:
∆h/200 = 5.195/3.116 v = (3300/7.481)/(π/4)(17/12)2(60) = 4.66 ft/s
∆h = 333.42 KJ/kg L = 1 mile = 5280 ft
hL =fLv2/2gD = 0.03(5280)(4.66)2/2(32.2)(17/12) = 37.7
ft Hydraulic gradient = 37.7/5280 = 0.00714 Find the mass of 10 quartz of water.
A. 10.46 kg *B. 9.46 kg C. 11.46 kg D. 8.46 kg
Find the loss of head in the pipe entrance if speed of flow is 10 m/s.
A. 5.10 m B. 10.2 m C. 17.4 m *D. 2.55 m Solution:
Solution: V = 10 quartz x 1gal/4quartz x 3,785 li/1gal x 1m3/1000 li
Loss at entrance = 0.5 (v2/2g) = 0.5[102/ 2(9.81)] = 2.55 m V = 0.0094625 x 10-3m3
w = m/V
Wet material, containing 220 % moisture (dry basis) is to be dried at the rate of 1.5 kg/s in 1000 = m/0.0094625 x 10-3
a continuous dryer to give a product containing 10% (dry basis). Find the moisture m = 9.46 kg
removed, kg/hr.
*A. 3543.75 kg/hr B. 3513.75 kg/hr C. 3563.75 kg/hr D. 3593.75 kg/hr Find the mass of carbon dioxide having a pressure of 20 psia at 200°F with 10 ft3 volume.
Solution: A. 1.04 lbs B. 1.14 lbs *C. 1.24 lbs D. 1.34 lbs
Solid in wet feed = solid in dried product Solution:
[1/(1 + 2.2)](1.5) = [1/(1 + 0.1)](x) PV = m R T
x = 0.5156 kg/s (total dried product) (20 x 144)(10) = m (1545/44)(200 + 460)
Moisture removed = 1.5 – 0.5156 = 0.984 kg/s = 3543.75 kg/hr m = 1.24 lbs

Copra enters a dryer containing 70% moisture and leaves at 7% moisture. Find the
moisture removed on each pound of solid in final product. Find the heat needed to raise the temperature of water from 30°C to 100°C with 60%
A. 6.258 lb B. 1.258 lb C. 4.258 lb *D. 2.258 lb quality. Consider and atmospheric pressure of 101.325 kpa. Use the approximate enthalpy
Solution: formula of liquid.
Solid in wet feed = solid in dried product A. 293.09 KJ/kg B. 1,772.90 KJ/kg C. 1,547.90 KJ/kg
0.3x = 1 *D. 1,647.29 KJ/kg
x = 3.333 lbs Solution:
1 = 0.93y At 100°C
y = 1.07527 lb hf = Cp t = 4.187(100) 418.7 KJ/kg
Moisture removed = x – y = 3.333 – 1.07527 = 2.258 lbs hfg = 2257 KJ.kg
h2 = hf + xhfg = 418.7 + 0.60(2257) = 1,772.9 KJ/kg
A 1 m x 1.5 m cylindrical tank is full of oil with SG = 0.92. Find the force acting at the Q = 1(4.187)(100-30) + 1(1772.9 – 418.7) = 1,647.20 KJ/kg
bottom of the tank in dynes.
A. 106. 33 x 103 dynes B. 106.33 x 104 dynes C. 106.33 x 105 dynes *D. Find the enthalpy of water at 212˚F and 14.7 psi if the dryness factor is 30%. Use the
106.33 x 106 dynes approximate enthalpy formula of liquid.
Solution: A. 461 Btu/lb *B. 471 Btu/lb C. 481 Btu/lb D. 491
P = w h = (0.92 x 9.81)(1.5) = 13.5378 kpa Btu/lb
F = PA = 13.5378(π/4 x 12) = 10.632 KN = 10,632.56 N x 10,000 dynes/N Solution:
F = 106.33 x 106 dynes hf = (˚F – 32) = (212 – 32) = 180 Btu/lb
hfg = 970 Btu/lb
Find the pressure at the 100 fathom depth of water in kpag. h = hf + x hfg
*A. 1,793.96 kpag B. 1,893.96 kpag C. 1,993.96 kpag h = 180 + 0.3(970) = 471 Btu/lb
D. 1,693.96 kpag
Solution: An air compressor consumed 1200 kw-hr per day of energy. The electric motor driving the
H = 100 fathom x 6 = 600 ft compressor has an efficiency of 80 %. If indicated power of the compressor is 34 kw, find
P = w h = (600/3.281)(9.81) = 1,793.96 kpag the mechanical efficiency of the compressor.
A. 117.65 % B. 75% *C. 85% D. 90%
Find the depth in furlong of the ocean (SG = 1.03) if the pressure at the sea bed is Solution:
2,032.56 kpag. Pim = 1200kw-hr/24 hrs = 50 kw
*A. 1 B. 2 C. 3 D. 4 BP = 50(0.80) = 40 kw
Solution: em = 34/40 = 85%
P=wh
2,032.56 = (1.03 x 9.81) h
h = 201.158 m x 3.281 ft/m x 1 yd/3ft x 1 furlong/220yd = 1 furlong
A refrigeration system consumed 28,800 kw-hr per month of energy. There are 20 % of 208=F–32
energy is lost due to cooling system of compressor and motor efficiency is 90%. If COP of the ˚F = 240 ˚F = 115.55 ˚C
system is 6, find the tons of refrigeration of the system.
A. 43.15 TR B. 46.15 TR *C. 49.15 TR D. 41.15 TR Solution: Convert 750˚R to ˚K
A. 390.33 ˚K B. 395.33 ˚K C. 410.33 ˚K *D. 416.33 ˚K
Pim = 28,800/(24 x 30) = 40 kw Solution:
BP = 40 (0.90) = 36 kw ˚R = 1.8 ˚K
Wc = 36(1 – 0.20) = 28.80 kw 750 = 1.8 ˚K
COP = RE/Wc ˚K = 416.667
6 = RE/28.80
RE = 172.8/3.516 = 49.15 TR An otto cycle has a compression ratio of 8. Find the pressure ratio during compression.
*A. 18.38 B. 16.38 C. 14.38 D. 12.38
A 23 tons refrigeration system has a heat rejected of 100 kw. Find the energy efficiency Solution:
ratio of the system. P1V1k = P2V22
A. 13.42 *B. 14.42 C. 15.42 D.16.42 Solution: (V1/V2)k = (P2/P1)
rkk = rp
QR = RE + Wc rp = (8)1.4 = 18.38
100 = 23(3.516) + Wc
Wc = 19.132 kw A diesel cycle has a cut off ratio of 2.5 and expansion ratio of 4. Find the clearance of
COP = RE/Wc = (23 x 3.516) / 19.132 = 4.23 the cycle.
EER = 3.412 COP = 3.412(4.23) = 14.42 A. 9.11 % B. 5.55 % *C. 11.11 % D.
15.15 %
A 200 mm x 250 mm, 8-cylinder, 4-stroke diesel engine has a brake power of 150 kw. The Solution:
mechanical efficiency is 80 %. If two of the cylinders were accidentally cut off, what will rk = rc re
be the new friction power? rk = 2.5(4) = 10
A. 31.50 kw B. 33.50 kw C. 35.50 kw *D. 37.50 kw rk = (1 + c)/c
Solution: 10=(1 + c)/c
em = BP/IP c = 11.11 %
0.8 = 150/IP
IP = 187.5 kw A dual cycle has an initial temperature of 30 ˚C. The compression ratio is 6 and the heat
FP1 = IP – BP = 187.5 – 150 = 37.50 kw addition at constant volume process is 600 KJ/kg. If cut-off ratio is 2.5, find the
FP1 = FP2 = 37.50 kw maximum temperature of the cycle.
A. 3638.50 ˚C *B. 3365.50 ˚C C. 3565.50 ˚C D. 3965.50 ˚C
If the energy efficiency ratio of the refrigeration system is 12.6, what is the COP of Solution:
the system? T2 = T1 rkk-1 = (30 +273)(6)1.4-1 = 620.44 ˚K
*A. 3.69 B. 4.23 C. 5.92 D. 60 kw QAV = m cv (T3 – T2)
Solution: 600 = 1(0.7186)(T3 – 620.44)
EER = 3.412 COP T3 = 1455.396 ˚K
12.6 = 3.412 COP rc = T4/T3
COP = 3.69 2.5 = T4/1455.396
T4 = 3638.49 ˚K = 3365.50 ˚C
An air compressor has a power of 40 kw at 4% clearance. If clearance will increase to
7%, what is the new power? A three stages air compressor compresses air from 100 kpa to 1000 kpa. Find the
A. 70 kw *B. 40 kw C. 53 kw D. 60 kw intercooler pressure between the first and second stage.
Solution: A. 505.44 kpa B. 108.44 kpa C. 316.23 kpa *D. 215.44 kpa
The power of compressor will not be affected with the changes in clearance. Solution:
Therefore power will still be 40 kw. Px = (P12P2)1/3
Px = [(100)2(1000)]1/3 = 215.44 kpa
What is the approximate value of temperature of water having enthalpy of 208 Btu/lb?
A. 138.67˚C *B. 115.67˚C C. 258.67˚C D. 68.67˚C A 10-stages air compressor compresses air from 100 kpa to 800 kpa. Find the
Solution: intercooler pressure between 1st and 2nd stage.
h = ˚F – 32 A. 282.84 kpa B. 113.21 kpa *C. 123.11 kpa D. 333.51 kpa
Solution: A. 1273.29 B. 2173.29 *C. 1373.60 D. 7231.29
Px = (P1s-1P2)1/s SOLUTION:
Px = [(100)10-1(1000)]1/10 = 123.11 kpa h2 = hf + xhfg = 151.53 + 0.85(2415.9) = 2205.045 KJ/ kg
KE1 = ½ m v2 = ½ (5,000/3600)(30)2 = 625 W = 0.625 KW
A 3-stages air compressor compresses air from 100 kpa to 700 kpa. Find the KE2 = ½ m v2 = ½ (5,000/3600)(15)2 = 156.25 W = 0.15625 KW
intercooler pressure between the 2nd and 3rd stage. By energy balance:
*A. 365.88 kpa B. 375.88 kpa C. 385.88 kpa D. KE1 + mh1 = KE2 + mh2 + Q + W
395.88 kpa W = (KE1 – KE2) + m(h1-h2) – Q
Solution: 5000 10,000
Px = (P12P2)1/3 W = (0.625 – 0.156) + (3600)(3195.7 – 2205.045) – 3600 = 1373.60 KW
Px = [(100)2(700)]1/3 = 191.28 kpa
Px/P1 = Py/Px A steam turbine with 85% stage efficiency receives steam at 7 Mpa and 550oC and exhausts
Py = Px2/P1 = (191.28)2/100 = 365.88 kpa as 20 Kpa. Determine the turbine work.
At 7 Mpa and 550oC: h1 = 3530.9 Kj/kg s1 = 6.9486
Carnot cycle A, B and C are connected in series so that the heat rejected from A will be the At 20 Kpa (0.020 Mpa): sf = 0.8320 hf = 251.4 sfg = 7.0766 hfg =
heat added to B and heat rejected from B will be added to C, each cycle operates between 2358.3
30 ˚C and 400 ˚C. If heat added to A is 1000 kw, find the work output of C. A. 1,117 KJ/kg B. 1,132 KJ/ kg C. 1,123.34 KJ/ kg *D. 1,054.95
*A. 111.44 kw B. 549.78 kw C. 247.53 kw D. KJ/kg
141.89 kw SOLUTION:
Solution: s1 = s2 = sf + xsfg
e1 = e2 = e3 = (400 – 30)/(400 + 273) = 54.98 % 6.9486 = 0.8320 + x(7.0766)
e1 = W1/QA1 = (QA1 – QR1)/QA1 x = 0.8643
0.5498 = (1000 – QR1)/1000 h2 = 251.40 + 0.8643(2358.3) = 2289.78 KJ/kg
QR1 = 450.22 = QA2 h1 - h2a
0.5498 = (450.22 – QR2)/450.22 eST = h1 – h2
By heat balance: 0.85 =3530.9 – h2a
Qgain = Qloss 3530.9 – 2289.78
mw cp (tb- ta) = mg cpg (t2-t1) h2a = 2475.95 KJ/kg
(0.30)(4.187)( tb – 15) = (0.5)(1.0717)(150 – 80) WT = h1 = h2a = 3530.9 – 2475.95 = 1,054.95 KJ/kg
tb = 44.86 oC
A steam turbine with 80% stage efficiency receives steam at 7 Mpa and 550oC and exhaust
A 350 mm X 450 mm steam engine running at 280 rpm has an entrance steam condition of 2 as 20 Kpa. Determine the quality at exhaust.
Mpa and 230 oC and exit at 0.1 Mpa. The steam consumption is 2,000 kg/hr and mechanical At 7 Mpa and 550oC: h1 = 3530.9 Kj/kg s1 = 6.9486
efficiency is 85%. If indicated mean effective pressure is 600 Kpa, determine brake thermal At 20 Kpa (0.020 Mpa): sf = 0.8320 hf = 251.4
efficiency. *A. 96.96% B. 76.34% C. 82.34% D. 91.69%
At 2 Mpa and 230 oC (Table 3): h1 = 2849.6 s1 = 6. 4423 SOLUTION:
At 0.1 Mpa: sf = 1.3026 hf = 417.46 sfg = 6.0568 hfg = 2258 hf2 = 417.46 sfg = 7.0766 hfg = 2358.3
KJ/kg s1 = s2 = sf + sfg
A. 23.34% *B. 15.25% C. 14.16% D. 27.34% 6.9486 = 0.8320 + x(7.0766)
x = 0.8643
SOLUTION: h2 = 251.40 + 0.8643(2358.3) = 2289.78 KJ/kg
VD = 2[3.1416/4 (0.35) 2 (0.45)(280/60)]= 0.4041 m3/sec nST = h1 – h2a
Indicated Power = Pmi x VD = 600 x 0.4041 =242.45 KW h1 – h2
Brake Power = IP (em) = 242.45 (0.85) = 206.08KW 0.80 = 3530.9 – h2a
Brake Power 206.08 3530.9 – 2289.78
etb = BP/ ms (h1-hf2) =206.08/ (2000/3600)(2849.6 – 417.46)=15.25% h2a = 2538.004 KJ/kg
h2a = hf + x hfg
A steam turbine receives 5,000 kg/hr of steam at 5 Mpa and 4000oc and velocity of 30 2538.004 = 251.40 + x (2358.3)
m/sec. It leaves the turbine at 0.06 Mpa and 85% quality and velocity of 15 m/sec. Radiation x = 96.96%
loss is 10,000 KJ/hr. Find he KW developed.
At 5 Mpa and 400oC: h1 = 3195.7 KJ/kg s1 =6.6459 A 16,000KW geothermal plant has a generator efficiency and turbine efficiency of 90% and
At 0.006 Mpa: hf = 151.53 hfg = 2415.9 80%., respectively if the quality after throttling is 20% and each well discharges 400, 000
kg/hr, determine the number of wells are required to produce if the charge of enthalpy if The indicated thermal efficiency of a two stroke diesel engine is 60%. If friction power is
the change of enthalpy at entrance and exit of turbine is 500KJ/kg. 15% of heat generated, determine the brake thermal efficiency of the engine.
A. 4 wells *B. 2 wells C. 6 wells D. 8 wells A. 43% *B. 45 % C. 36% D. 37%
SOLUTION: SOLUTION:
WT = ms(h3 – h4) ne = IP/ Qg
16,000 = ms (500) 0.60 = IP/Qg
0.9(0.8) IP = 0.60 Qg
ms = 44.44 kg/sec BP = IP- FP = 0.60Qg – 0.15Qg = 0.45Qg
ms = 160,000 kg/hr etb = BP/Qg = 0.45Qg/Qg = 45%
160,000 = 0.20 mg
mg = 800,000 kg/hr A 305 mm x 457 mm four stroke single acting diesel engine is rated at 150 KW at 260
No. of wells = 800,000/400,000 = 2 wells rpm. Fuel consumption at rated load is 0.56 kg/KW-hr with a heating value of 43,912
KJ/kg. Calculate brake thermal efficiency
A liquid dominated geothermal plant with a single flash separator receives water at 204oC. The A. 10.53% B. 27.45% *C. 14.64% D. 18.23%
separator pressure is 1.04 Mpa. A direct contact condenser operates at 0.034 Mpa. The turbine SOLUTION:
has a polytropic efficiency of 0.75. For a cycle output of 60 MW, what is the mass mf = 0.56 kg/KW-hr x 150 KW = 84 kg/hr = 0.0233 kg/sec
flow rate of the well-water in kg/s?
At 204oC: hf = 870.51 KJ/kg Brake thermal efficiency =
At 1.04 Mpa: hf = 770.38 hfg = 2009.2 hg = 2779.6 sg = 6.5729
At 0.034 MPa: hf = 301.40 hfg = 2328.8 sf = 0.9793 sfg = 6.7463 A waste heat recovery boiler produces 4.8 Mpa(dry saturated) steam from 104°C feedwater.
*A. 2,933 B. 2,100 C. 1,860 D. 2,444 The boiler receives energy from 7 kg/sec of 954°C dry air. After passing through a waste
SOLUTION: heat boiler, the temperature of the air is has been reduce to 343°C. How much steam in kg is
h3 = hg at 1.04 MPa = 2779.6 KJ/kg produced per second? Note: At 4.80 Mpa dry saturated, h = 2796.
Solving for h4: A. 1.30 B. 0.92 *C. 1.81 D. 3.43
s3 = s4 = sf + xsfg SOLUTION:
6.5729 = 0.9793 + x4(6.7463) hf = approximate enthalpy of feedwater
x4 = 0.829 hf = Cpt
h4 = 301.4 + 0.829(2328.8) = 2232.3 KJ/kg hf = 4.187(104)
WT = ms (h3 – h4) hf = 435.45 KJ/kg
60,000 = ms (2779.6 – 2232.3) 0.75 Heat loss = Heat gain
ms = 146.17 kg/sec m gc p(t 1 - t 2) = m s(h - h f)
Solving for x2: (h1 = h2) 7(1.0)(954 – 343) = ms(2796.0 – 436.45)
h1 = h2 = hf + xhfg m s = 1.81 kg/sec
870.51 = 770.38 + x2(2009.2)
x2 = 0..049836 A diesel electric plant supplies energy for Meralco. During a 24-hour period, the plant
ms = x mg consumed 240 gallons of fuel at 28°C and produced 3930 KW-hr. Industrial fuel used is
146.17 = 0.049836 mg 28°API and was purchased at P30 per liter at 15.6°C. What is the cost of the fuel be to
mg = 2,933.06 kg/sec produce one KW-hr?
*A. P6.87 B. P1.10 C. P41.07 D. P5.00
An engine-generator rated 9000 KVA at 80% power factor, 3 phase, 4160 V has an SOLUTION:
efficiency of 90%. If overall plant efficiency is 28%, what is the heat generated by the fuel. SG 15.6C = 141.5/(131.5 + 28) = 0.887
A. 18,800 KW B. 28,800 KW C. 7500 KW *D. 25,714 Density at 15.6°C = 0.887(1kg/li) = 0.887 kg/li
KW SG 28C = 0.887[1-.0007(1 – 15.6)] = .879
SOLUTION: Density at 28°C = 0.879(1 kg/li) = 0.879 kg/li
Gen. Output = pf x KVA = 0.8 x 9000 = 7200 KW V28C / V15.6C = SG15.6C / SG28C
eoverall= Gen. Output 240 / V15.6C = 0.887 / 0.879
Qg V15.6C = 237.835 gallons x 3.785 li/gal = 900.21 li
Cost = [(30)(900.21)] / 3930 = P6.87/KW-hr
0.28 = 7200/Qg
Qg = 25,714.28 KW In a gas turbine unit, air enters the combustion chamber at 550 kpa, 277°C and 43 m/s. The
products of combustion leave the combustor at 511 kpa, 1004°C and 180 m/s. Liquid fuel
enters with a heating value of 43,000 KJ/kg. For fuel-air ratio of 0.0229, what is the combustor In an 9,000 KW hydro-electric plant the over-all efficiency is 88% and the actual power
efficiency of the unit in percent? received by the customer is 110,000 KW-hrs for that day. What is the secondary power
A. 70.38% B. 79.385% *C. 75.38% D. 82.38% could this plant deliver during the entire day?
SOLUTION: A. 58,960 KW-hrs *B. 80,080 KW-hrs C. 65,960 KW-hrs D. 70,960 KW-hrs
Heat supplied by fuel = mfQh = 0.0229(43,000) = 984.7 KJ/kg
air Q = heat absorbed by fuel SOLUTION:
Q/m = Cp(T2 – T1) + ½(V22 – V12) Plant Capacity = 9,000(0.88)(24) = 190,080 KW-hrs
Q/m = (1.0)(1004 – 277) + ½[(180) 2 –(43) 2]/1000 =742.28 KJ/kg air Secondary Power = 190,080 – 110,000 = 80,080 KW-hrs

Combustor Efficiency = = 75.38% A Pelton type turbine was installed 30 m below the gate of the penstock. The head loss due to
friction is 12 percent of the given elevation. The length of penstock is 100 m and coefficient
The specific speed of turbine is 85 rpm and running at 450 rpm. If the head is 20 m and of friction is 0.00093. Determine the power output in KW. ( Use Moorse equation)
generator efficiency is 90%, what is the maximum power delivered by the generator. A. 22,273 B. 23,234 C. 32,345 *D. 34,452
A. 450.51 KW B. 354.52 KW C. 650.53 KW*D. 835.57 KW SOLUTION:
SOLUTION: hL = 0.12(30) = 3.6 m
NS = (N√HP)/h5/4 h = 30 – 3.6 = 26.40 m
85 = (450√HP)/(20 x 3.281) 5/4 v = (2gh)1/2 = [(2)(9.81)(26.4)]1/2 = 22.759 m/sec
Hp = 1244.52 hL= (2fLv2)/gD
Generator Output = (1244.52 x 0.746)(0.9) = 835.57 KW 3.6 = (2 x .00093 x 100 x 22.759) / (9.81D)
D = 2.728 m
In Francis turbine, the pressure gage leading to the turbine casing reads 380 Kpa. The
velocity of water entering the turbine is 8 m/sec, if net head of the turbine is 45 m, find the Q=Axv=[ 2](22.759) = 133.03 m3/sec
distance from center of spiral casing to the tailrace. Power = w Q h = 9.81(133.03)(26.4) = 34,452 KW
*A. 3.0 m B. 3.5 m C. 4.0 m D. 4.5m
SOLUTION : Water flows steadily with a velocity of 3.05 m/s in a horizontal pipe having a diameter of
25.4 cm. At one section of the pipe, the temperature and pressure of the water are 21C and
689.3 Kpa, respectively. At a distance of 304.8 m downstream
h= V2/2g
45 = (380/9.81) + z + [82/(2 x 9.81)] A hydro electric plant having 30 sq. km reservoir area and 100 m head is used to generate
z=3m power. The energy utilized by the consumers whose load is connected to the power plant
during a five- hour period is 13.5 x 106 kwh. The overall generation efficiency is 75%.
A turbine has a mechanical efficiency of 93%, volumetric efficiency of 95% and Find the fall in the height of water in the reservoir after the 5-hour period.
total efficiency of 82%. If effective head is 40 m, find the total head. A. 5.13 m B. 1.32 m C. 3.21 *D. 2.20 m
A. 48.72 m B. 40.72 m *C. 36.22 m D. 34.72 m SOLUTION
SOLUTION: Energy Output = Power x time = (w Q h) x
eT = emehev time 13.5 x 106 = 9.81(Q)(100)(0.75)(5) Q = 3669.725
0.8 = 0.93(eh)(.95) m3/s
ηh = 0.9055 Volume after 5 hrs = 3669.725(5 x 3600) = 66,055,050 m3
Total head = h eh = (40)(0.9055) = 36.22 m Volume = A x height
66,055,050 = (30 x 106) h
A Pelton type turbine has 25 m head friction loss of 4.5 m. The coefficient of friction head H =2.202 m
loss (from Moorse) is 0.00093 and penstock length of 80 m. What is the penstock diameter?
*A. 1,355.73 mm B. 3,476.12 mm C. 6771.23 mm D. 1686.73 mm The gas density of chimney is 0.75 kg/m3 and air density of 1.15 kg/m3. Find the driving
pressure if the height of chimney is 63.71 m.
SOLUTION: A. 0.15 kpa *B. 0.25 kpa C. 0.35 kpa D. 0.45 kpa
h =25- 4.5 = 20.5 SOLUTION:
v = √(2gh) = [(2 x 9.81 x 20.5)1/2] = 20.55 m/sec hw = H(da – dg) = 63.71(1.15 – 0.75) (0.00981) = 0.25 kpa
hL = (2fLv2)/gD The actual velocity of gas entering in a chimney is 8 m/sec. The gas temperature is 25C with
4.5 = (2)(0.00093)(80)(20.055)2 / 9.81D a gas constant of 0.287 KJ/kg-K. Determine the gas pressure for a mass of gas is 50,000 kg/hr
D = 1,355,730 m = 1,355.73 mm and chimney diameter of 1.39m.
A. 95 kpa *B. 98 kpa C. 101 kpa D. 92 kpa
SOLUTION:
Brine enters a circulating brine cooler at the rate of 60 m3/hr at -*C and leaves at -18C.
Vg = A x v = / 4 (1.39)2(8) = 12.139 m3/s
Specific heat of brine is 1.072 KJ/kg-K and specific gravity of 1.12. Determine the tons of
PgVg = mgRgTg
refrigeration.
P(12.139) = (50,000/3600)(.287)(25 +273)
A. 53.5 TR B. 65.3 TR C.33.5 TR *D. 56.9 TR
P = 97.85 kpa
A steam generator with economizer and air heater has an overall draft loss of 25.78 cm of SOLUTION:
water. If the stack gases are at 177C and if the atmosphere is at 101.3 Kpa and 26C, what Density of brine = 1.12(1000 kg/m3) = 1120 kg/m3
theoretical height of stack in meters is needed when no draft fan are used? Assume that the m = (1120)(60)/3600 = 18.67 kg/sec
gas constant for the flue gases is the same as that for air. Q = mcp = 18.67(1.072)(-8 + 18) = 200.11 KW
A 611.10 B. 631.10 *C.651.10 D.671.10 TR = 200.11/3.516 = 56.91 Tons of refrigeration
SOLUTION: A turbo-charged, 16 cylinder, Vee-type diesel engine has an air consumption of 3,000 kg/hr
w = P/RT per cylinder at rate load and speed. This air is drawn in through a filter by a centrifugal
da = (101.325)/[(.287)(26 + 273)] = 1.180 kg/m3 compressor directly connected to the exhaust gas turine. The temperature of the air from the
dg = (101.3)/[(0.287)(177 +273)] = 0.784 kg/m3 compressor is 135C and a counter flow air cooler reduces the air temperature to 45C before it
Draft = (0.2578)(1000) = 257.80 kg/m3 goes to the engine suction heater. Cooling water enters air cooler at 30C and leaves at 40C.
Draft = H(da – dw) Calculate the log mean temperature difference.
257.80 = H(1.18 – 0.784) A. 47.23C B. 87.82C *C. 43.34C D. 65.24C
H = 651.10 m SOLUTION:
A foundation measures 12 ft x 14 ft x16 ft. Find the number of sacks of cement needed a = 45-30 = 15C
for 1:2:4 mixture.
b = 135 – 40 = 95C
A. 302 B. 404 C. 356 *D. 598
SOLUTION: mean = [ a - b] / [ln( a b)] = [95-15] / ln(95/15) = 43.34C
V = 12 X 14 X 16 = 2,688 ft3 (1 yd3 / 33 ft3) = 99.55 yd3 of Water is flowing in a pipe with radius of 30 cm at a velocity of 5 m/s at the temperature in
concrete For every 1 yd3 concrete, it needs 6 sacks of cement Therefore: pipe. The density and viscosity of the water are as follows: density 997.9 kg/sec viscosity
= 1.131 Pa-s. What is the Reynolds Number for this situation?
No. of sacks = 6(99.55) = 597.33 sacks or 598 sacks *A. 2647 B. 96.2 C. 3100 D. 1140
A rectangular foundation cross-section has a bed plate dimension of 8 ft x 10 ft. The SOLUTION:
uniform clearance on each side is 1 ft. The height of foundation is 4.5 ft. If the weight of the n = Dvg / v
steel bar reinforcements needed is 1/2% of weight of foundation, find the weight of steel Where:
bars. Use concrete density of 2400 kg/m3 . D = 2(0.30) = 0.60 m
A. 173.47 kg *B. 183.47 kg C. 163.47 kg D. 153.47 kg vg = 5 M/SEC
SOLUTION: v = 1.131/997.9 = 0.0011334 m2 / sec
A = (8 + 2) (10 + 2) = 120 m2 Nm = 0.60(5)/0.0011334 = 2,647
V = Ah = 120(4.5) = 540 ft3 = 15.29 m3 Compute the amount of condensate form during 10 minutes warm-up of 180 meter pipe
W = wV = (2400)(15.29) = 36,693.25 kg conveys the saturated steam with enthalpy vaporization hfg = 1,947.8 LJ/kg. The minimum
Weight of steel bars = (1/2%) Wf = 0.005(36,693.25) = 183.47 kg external temperature of pipe is 2C. The final temperature of pipe is 195C. The specific heat
of pipe material is 0.6 KJ/kg-C. The specific weight is 28 kg/m.
A steam pipe having a surface temperature of 250C passes through a room where the A. 249.69 kg B. 982.45 kg *C. 299.64 kg D.
temperature is 27 C. The outside diameter of pipe is 100 mm and emissivity factor is 0.8. 423.45 kg
Calculate the radial heat loss for 3 m pipe length. SOLUTION:
A. 1434.7 W B. 37.46 W *C. 2651.82 W D. 3545.45 W mp = mass of pipe = 28(180) = 5,040 kg
SOLUTION: Heat Loss by steam = Heat loss from pipe
m(hg - hf) = mpcp (t2 – t1)
A = DL = = 0.425m2 m(1947.8) = (5040)(0.6)(195-2)
Solving for heat due to radiation: m = 299.64 kg
Tg = 250 +273 = 523K
T2 = 27 +273 = 300K
Qa = 20,408.4 x 104 AF(T14 – T24), J/hr = 20,408.4 x 104(0.8)(0.7539)[(523)4 – The discharge pressure of an air compressor is 5 times the suction pressure. If volume flow
(300)4] at suction is 0.1 m3/sec, what is the suction pressure if compressor work is 19.57 kw? (use
Qr = 10,266,539.06 j/hr x 1hr/3600sec = 2851.82 W n=1.35)
A. 97 kpa *B.98 kpa C. 99 kpa D.100 kpa
SOLUTION: *A. 233.34 B. 973.17 C. 198.34 D. 204.82
SOLUTION:
W= [(P2/P1)n-1/n – 1] P1V1K = P2V2K
19.57 = 1.35(P1)(0.1)/(1.35-1)[(5)1.35-1/1.35 – 1] 100(V11.4) = 600(6)1.4
P1 = 98 KPa V1 = 28.768 m3/min
VD = 28.768/0.89 = 32.32 m3/min
The initial condition of air in an air compressor is 98 KPa and 27C and discharge air at 450 W = n P1V1/n-1 x [(P2 / P1)n-1/n – 1] = [(1.4 x 100 x 32.32)/(1.4 – 1)] x
KPa. The bpre and stroke are 355 mm and 381 mm, respectively with percent cleared of [(600/100)1.4-1/1.4 – 1]
8% running at 300 rpm. Find the volume of air at suction. W = 7562.19 KJ/min
A. 541.62 m3/hr B. 551.62 m3/hr C. 561.62 m3/hr *D. W = Pm x Vd
571.62 m3/hr 7562.19 = Pm x 32.32
SOLUTION: Pm = 233.34 KPa
A water-jacketed air compressed handles 0.343 m3/s of air entering at 96.5 KPa and 21C and
ev = 1 + c – c(P2/P1)1/n = 1 + 0.08 - 0.08(450/98)1/1.4 = 0.842
leaving at 460 KPa and 132C; 10.9 kg/h of cooling water enters the jacket at 15C and leaves
VD = D2 LN = (0.355)2 (0.381)(300/60) = 0.1885 m3/sec at 21C. Determine the compressor brake power.
V1 = 0.1885(0.842) = 0.15878 m3/sec = 571.62 m3/hr A. 26.163 KW *B. 62.650 KW C. 34.44 KW D.
An air compressor has a suction volume of 0.35 m3/sec t 97 KPa and discharges to 650 KPa. 19.33 KW
How much power saved by the compressor of there are two stages? SOLUTION:
A. 18.27 KW B. 16.54 KW C. 13.86 KW *D. T2/T1 = (P2/P1) n-1/n
11.58 KW (132+273) / (21+273) = (480/96.5)n-1/n
SOLUTION: n = 1.249
W = (1.249 x 96.5 x 0.343) / (1.249-1) [(480 / 96.5)1.249-1/1.249 –
W= [(P2/P1)n-1/n – 1] = (1.4 x 97 x 0.35)/(1.4 -1) [(650/97)1.4-1/1.4 – 1] = 1] W = 62.57 KW
85.79 KW Q = heat loss = mcp(t2 – t1) = (10.9/3600)(4.187)(21 – 15) 0.075
For two stages : KW Brake power = W + Q = 62.57 + 0.076 = 62.65 KW
Px = (P1P2)1/2 = (97 x 650)1/2 = 251.097 KPa
A double suction centrifugal pumps delivers 20 ft3/sec of water at a head of 12 m and
W= [(Px/P1)n-1/n – 1] = 2(1.4)(97)(0.35)/(1.4 – 1) [(251.0.97/97)1.4-1/1.4 running at 650 rpm. What is the specific speed of the pump?
– 1] = 74.208 KW A. 5014.12 rpm B. 6453.12 rpm *C. 2770.73 rpm D. 9966.73 rpm
POWER SAVED = 85.79 – 74.208 = 11.582 KW SOLUTION:
N = N(Q)1/2 / h3/4
A twop stage air compressor has an intercooler pressure of 4 kg/cm2. What is the Q = 20/2 ft3/sec x 7.481 gal/ft3 x 60 sec/1min = 4,488.6
discharge pressure if suction pressure is 1 kg/cm2? gal/min h = 12 x 3.281 = 39.37 ft
A. 3 kg/cm2 B. 9 kg/cm2 C. 12 kg/cm2 *D. 16 kg/cm2 N = (650 x (4,488.6)1/2)/(39.37)3/4
SOLUTION: N = 2,770.73 rpm
Px = (P1P2)1/2 Determine the number of stages needed for a centrifugal pump if it is used to deliver 400
Px2 = P1(P2) gal/min of water and pump power of 15 Hp. Each impeller develops a head of 30 ft.
42 = 16 kg/cm2 A. 6 B.4 *C.5 D.7
SOLUTION:
A two stage air compressor compresses air at 100 KPa and 22C discharges to 750 KPa. Wp = w Q h
If intercooler intake is 105C. Determine the value of n. 15 x 0.746 = 9.81(400 gal/min x 0.00785m3/gal x
A. 1.400 *B. 1.325 C. 1.345 D. 1.288 1/60)h h = 45.20 m x 3.281 ft/m = 148.317 ft
SOLUTION: Number of stages = 148.317/40 = 4.94 stages = 5 stages
Px = (100 x 750)1/2 =273.86 KPa The suction pressure of a pump reads 3 in. of mercury vacuum and discharge pressure
Tx/T1 = (Px/P1)n-1/n reads 140 psi is use to deliver 120 gpm of water with specific volume of 0.0163 ft3/lb.
(105 + 273)/(22 + 273) = (273.86/100)n-1/n Determine the pump work.
1.281 = (2.6268)n-1/n A. 4.6 KW B. 5.7 KW *C. 7.4 KW D. 8.4 KW
n = 1.326 SOLUTION:
A single acting compressor has a volumetric efficiency of 89%, operates at 500 rpm. It takes in air P1 = -3 in Hg x 101.325/29.92 = -10.16 KPa
at 900 KPa and 30C and discharges it at 600 KPa. The air handled is 8 m3/min measured at P2 = 140 psi x 101.325/14.7 = 965 KPa
discharge condition. If compression is isentropic, find mean effective pressure in KPa. w = 1/v = 1/0.163 = 61.35 lb/ft3 x 9.81/62.3 = 9.645
KN/m3 h = (P2 – P1)/w = (965 +10.16)/9.645 = 101.105 m
Q = 120 gal/min x 3.785/1gal x 1m3/1000li x 1/60 = 0.00757 ep = wa Q hs/BP = (1.18 x 0.00981)(1.42)(21.52) / 0.6588 = 53.69%
m3/sec P = w Q h = 9.645(0.00757)(101.105) = 7.38 KW
A submersible pump delivers 350 gpm of water to a height of 5 ft from the ground. The A water cooler uses 50 lb/hr of melting ice to cool running water from 80 to 42 . Based
pump were installed 150 ft below the ground level and draw down of 8 ft during the on te inside coil area, U1 = 110 Btu/hr-ft2- . Find the gpm of water cooled.
operation. If water level is 25 ft above the pump, determine the pump power. A. 0.10 GPM B. 0.21 GPM *C. 0.38 GPM D. 0.45 GPM
A. 7.13 KW B. 4.86 KW C. 7.24 KW *D. SOLUTION:
9.27 KW Q = mf L = mwcpw(t1 – t2)
SOLUTION: 57. (144) =
h = 5 + 150 – (25 – 8) = 138/3.281 = 42.06 m mW(1)(80-42) mw =
Q = 350 gal/min x 0.003785 m3/gal x 1 min/60sec = 0.02246 189.474 lb/hr
m3/sec Wp = w Q h = 9.81(0.02246)(42.06) = 9.27 KW V = (189.474/62.4) (7.48/60) = 0.38 GPM
The charge in a Diesel engine consists of 18.34 grams of fuel, with lower heating value
A vacuum pump is used to drain a flooded mine shaft of 20 water. The pump pressure of
of 42,571 KJ/kg, and 409 grams of fuel and products of combustion. At the beginning of
water at this temperature is 2.34 KPa. The pump is incapable of lifting the water higher
than 16 m. What is the atmospheric pressure? compression, t1 = 60 . Let rk = 14. For constant cP = 1.11 KJ/kg-C, what should be the
*A. 159.30 B. 32.33 C. 196.22 D. 171.9 cut-off ratio in the corresponding ideal cycle?
SOLUTION: A. 2.05 B. 2.34 C. 5.34 *D. 2.97
Using Bernoulli’s Theorem: SOLUTION:
P1/w + V12/2g + z1 = P2/w + V2/2g + z2 QA = mfQh = 0.01283(42,571) = 780,752 KJ
P1/w = P2/w + (V22 - V12)/2g + (z2 - z1) T2/T1 = rkk-1
P1/9.81 = 2.34/9.81 + 0 + 16 T2 = (60 + 273)1.4-1 = 956.964K
P1 = 159.30 KPa mt + mg = 409
A submersible, multi-stage, centrifugal deep well pump 260 gpm capacity is installed in a mt + ma + mf = 409
well 27 feet below the static water level and running at 3000 rpm. Drawdown when pumping ma = 409 – 2(18.34) = 372.32 grams
at rated capacity is 10 feet. The pump delivers the water into a 25,000 gallons capacity QA = macp(t3 – t2)
overhead storage tank. Total discharge head developed by pump, including friction in piping 780.752 = 0.37232(1.11)(T3 – 956.964)
is 243 feet. Calculate the diameter of the impeller of this pump in inches if each impeller
diameter developed a head of 38 ft. T3 = 2846,146
A. 3.28 B. 5.33 *C. 3.71 D. 6.34 rC = T3/T2 = 2846.146/956.964 = 2.97
SOLUTION:
V= DN
V= The gain of entropy during isothermal nonflow process of 5 lb of air at 60 is 0.462 Btu/R.
Find the V1/V2.
D (3000/60) = (2(32.2)(38))1/2 A. 3.85 *B. 0.259 C. 1.0 D. 0.296
D = 0.315 ft = 3.708 inches SOLUTION:
A fan pressure of 2.54 cm of water t 1.42 m3 per second of air at static pressure of 2.54 cm
of water through a duct 300 mm diameter and discharges it through a duct 275 mm diameter. s = m R T ln(V2/V1)
0.462 = 5 (53.33/778) ln (V2/V1)
Determine the static fan efficiency if total fan mechanical is 75% and air measured at 25 V2/V1 = 3.85
and 60 mm Hg. V1/V2 = 1/3.85 = 0.259
A. 50.11% *B. 53.69% C. 65.67% D. 45.34%
SOLUTION:
wA = P/RT = 101.325/(0.287)(25 + 273) = 1.18 kg/m3 An auditorium seating 1500 people is to be maintained at 80 dry bulb and 85 wet bulb
hA = hwww/wA = (0.0254)(1000)/1.18 = 21.52 m temperature when outdoor air is at 91 dry bulb and 75 wet bulb. Solar heat load is
vA = 1.42/( /4)(0.3)2 = 20.09 m/s 110,000 Btu/hr and supply air at 60 determine the amount of supply air.
Vd = 1.42/( /4)(0.275)2 = 23.9 m/s *A. 93,229.17 lb/hr B. 83,229.17 lb/hr C. 73,229.17 D. 63,229.17 lb/hr
hv = (23.9)2 – (20.09)2 / 2(9.81) = 8.54 m SOLUTION:
h = ha + hv = 21.52 + 8.54 = 30.06 m Sensible heat per person = 225 Btu/hr
eT = wa Q h/BP Qa = 225(1500) + 110,000 = 447,500 Btu/hr
0.75 = (1.18 x 0.00981)(1.42)(30.06) / BP Qa = m cp(t1 – t2)
BP = 0.6588 KW 447,500 = ma(0.24)(80 – 60)
ma = 93,229.17 lb/hr
A cylindrical pope with water flowing downward at 0.03 m3/s having top diameter of
In a Brayton cycle that operates between temperature limits of 300K and 1773K wit k = 1.4, 0.08, bottom diameter of 0.04 m and a height of 1.5m. Find the pressure between the pipe
determine the temperature at the end of the compression (isentropic) for maximum work of A. 154.63 Kpa B. 197.93 Kpa *C. 252.44 Kpa D. 243.92 Kpa
the cycle. SOLUTION:
A. 700K B. 590.5K *C. 730K D. 350K
SOLUTION: +Z1= +Z2
For maximum work: T2 = (T1T3)1/2
T2 = (300 x 1773)1/2 = 730K

At 35% solution leaves the absorber and 30% solution enters the absorber. The heat removed
Z1 – Z2 = 1.5 m
from the absorber by cooling water is 547.6 Btu and ammonia is superheated by 10 . Find the Z2 – Z1 = -1.5 m
pound per pound of ammonia gas from the evaporating coils.
A. 11 B. 12 *C. 13 D. 14
SOLUTION:
V1 = = 5.968 m/s
n = lb/lb of ammonia gas from the coils
n = (1 - x2) / (x1 - x2) – 1
n = (1- 0.3) / (0.35 – 0.3) – 1 = 13
A Carnot refrigeration system operates at Tmax / Tmin = 1.5. Find the KW per tons V2 = = 23.87 m/s
of refrigeration.
A. 1.91 B. 2.15 *C. 1.76 D. 1.55
SOLUTION:
P1 – P2 = 252.44 Kpa
Wo / TR = 3.516 / COP = 3.516 / (Tmin / (Tmax – Tmin)
Wo / TR = [3.516(Tmax – Tmin)] / Tmin
Wo / TR = 3.516[(Tmax / Tmin) – 1] = 3.516(1.5-1) = 1.758 KW/TR
Determine the size of pipe which will deliver 8 liters of medium oil (v= 6.10 x 10-6 m2/s)
Assume 8 ft3 of air at 100 psi, 100 are compressed isothermally to a volume of 2 ft3. For assuming laminar flow conditions:
each of end states of the process, find the bulk modulus. A. 622 mm B. 754 mm C. 950 mm *D. 835 mm
*A. 400 and 100 psi B. 400 and 110 psi C. 400 and 120 psi D. 400 and SOLUTION:
130

An empty open can is 30 cm high with a 15 cm diameter. The can, with the open end and V=
down, is pushed under water with a density of 1000kg/m3. Find the water level in the can
when the top of the can is 50 cm below the surface.
A. 17.20 cm *B. 2.12 cm C. 4.20 cm D. 5.87cm Re =
SOLUTION: For laminar flow, Re = 2000
Consider the water pressure
Pw = w h + 1010.325 + (0.8-x)(9.81) + 101.325 = 109.173 –
9.81x Consider the air pressure 2000 =
P1V1 = P2V2 d = 0.835 m = 835 mm
101.325(Ax0.3) = P2[A(0.3-x)]
The type of flow occupying in a 1 cm diameter pipe which water flows at a velocity of
P2 = 2.50 m/s. Use v = 1.13 x 10-6 m2/s for water.
Pw = P2 *A. turbulent B. constant C. laminar D. none of the above

SOLUTION:
109.173 – 9.81x =
9.81x2 – 112.116x + 2.3705 = 0
Re =
By quadratic formula:
X = 0.02118 m = 2.12 cm Re =

Re = 22,124
Since it is greater than 2000 then it is turbulent flow SOLUTION:
T1 = 250 + 273 = 523oK
What is the force is exerted by water jet 60 mm diameter if it strikes a wall at the rate of To = T1 + v2/2000 = 523 = 3802/2000 = 595.2oK
15 m/s? P1 = 1200 kpa
*A. 636.17 N B.442.62 N C. 764.23 N D. 563.34 N SOLUTION: T1/To = (P1/Po)k-1/k
For CO2: k = 1.289
F=wQv 523/595.2 = (1200/Po)1.289-1/1.289
P0 = 2,136.34 kpa
Q=Av= = 0.0424 m3/s
F = (1000)(0.0424)(15) = 636.17 N Air enters a diffuser with a velocity of 200 m/s. Determine the velocity of sound if
air temperature is 30oC.
A 300 mm diameter pipe discharges water at the rate of 200 li/s. Point 1 on the pipe has a *A. 349 m/s B. 359 m/s C. 369 m/s D. 379 m/s
pressure of 260 kpa and 3.4 m below point 1 is point 2 with a pressure of 300 kpa. SOLUTION:
Compute the head loss between points 1 and 2. C=
A. 4.29 m B. 2.59 m C. 6.32 m *D. 1.87 m
SOLUTION:

hL
Air flows through a nozzle with temperature of entrance of 420oK stagnation temperature
hL = of 468oK. Find the mach number.
A. 0.744 *B. 0.754 C. 0.764 D. 0.774
Water flowing at the rate of 10 m/s from an orifice at the bottom of a reservoir. Find SOLUTION:
the pressure at the bottom of the reservoir. To = T1 + v2/2000Cp
A. 30 kpag B. 40 kpag *C. 50 kpag D. 60 kpag 468 = 420 + v2/2000
SOLUTION: v = 309.838 m/s
h = V2/ 2g = 102/ 2(9.81) = 5.0968 m
P = w h = 9.81(5.0968) = 50 kpag C=
M = v/C = 309.838/410.8 = 0.754
Steam flows through a nozzle at 400oC and 1 Mpa (h = 3263.9 KJ/kg) with velocity of
300 m/s. Find the stagnation enthalpy. Air at 300oK and 200 kpa is heated at constant pressure to 600oK. Determine the change
A. 3300 KJ/kg B. 3290 KJ/kg *C. 3320 KJ/kg *D. 3309 KJ/kg of internal energy.
SOLUTION: A. 245.58 KJ/kg B. 235.58 KJ/kg C. 225.58 KJ/kg *D. 215.58
ho = h + v2/2000 = 3263.9 + 3002/2000 = 3309 KJ/kg KJ/kg
SOLUTION:
Air flows through a nozzle at a speed of 350 m/s. Find the stagnation temperature if ΔU = mCv (T2 – T1) = 1(0.7186)(600 -300) = 215.58 KJ/kg
entrance temperature is 200oC.
A. 241.25oC B. 251.25oC *C. 261.25oC D. 271.25oC An insulated rigid tank initially contains 1.5 lb of helium at 80oF and 50 psia. A paddle
SOLUTION: wheel with power rating of 0.02 hp is operated within the tank for 30 min. Determine the
To = T1 + v2/2000Cp = (2000 + 273) + 3502/2000(1) final temperature.
To = 534.25oK = 261.25 A. 159.22oF B. 169.22oF *C. 179.22oF D. 189.22 oF
SOLUTION:
Carbon dioxide flows through a nozzle with a speed of 400 m/s. Compute the W = ΔU = m Cv (T2 – T1)
dynamic temperature. 0.02 hp (0.50hr)(2545Btu/hr/hp) = 1.5(0.171)(t2 – 80)
A. 92.56oK *B. 94.56oK C. 96.56oK D. 98.56oK t2 = 179.22oF
SOLUTION:
For CO2: Cp = 0.846 KJ/kg-K A 4m2 asphalt pavement with emissivity of 0.85 has a surface temperature of 50oC. Find
Dynamic temperature = v2/2000Cp = 4002/2000(0.846) = 94.56oK the maximum rate of radiation that can be emitted from the surface.
A. 2,068.32 watts B. 2,078.32 watts C. 2,088.32 watts *D. 2.098.32 watts
Carbon dioxide flows through a nozzle with a speed of 380 m/s. The entrance condition of
nozzle is 250oC and 1200 kpa. Find the stagnation pressure. SOLUTION:
*A. 2,136.34 kpa B. 2,146.34 kpa C. 2,156.34 kpa D. 2,166.34 Qr = e kev A Ts4
Kev = 5.67 x 10-8 ( Stefan Boltzman constant)
kpa
Qr = 0.85(5.67 z 10-8)(4)*50 +273)4 = 2,098.32 watts
Wc = 7.68/2.5 = 3.072 KW
Air at 10oC and 90 kpa enters a diffuser of a jet engine steadily with a velocity of 200 m/s.
The inlet area diffuser is 0.40 m2. Determine the mass flow rate of air. A heat source at 8000K losses 2000 KJ of heat to a sink at 500oK. Determine the
A. 72.79 kg/s B. 74.79 kg/s C. 76.79 kg/s *D. 78.79 kg/s entropy generated during this process.
SOLUTION: *A. 1.5 KJ/K B. 2.5 KJ/K C. -2.5 KJ/K D. 4
W = P/RT = 80/0.287(10 + 273) = 0.985 kg/m3 KJ/K
m = w v A = 0.985(200)(0.40) = 78.79 kg/s SOLUTION:
ΔSsource = -2000/800 = -2.5
Consider a refrigeration whose 40 watts light bulb remains on continuously as a result of a ΔSsink = 2000/500 = 4
malfunction of the switch. If the refrigerator has a COP of 1.3 and the cost of electricity is 8 ΔSgen. = -2.5 + 4 = 1.5 KJ/K
cents per kw-hr., determine the increase in the energy consumption of the refrigerator and
its cost per year if the switch is not fixed. Helium gas is compressed in an adiabatic compressor from an initial state of 14 psia and 50oF to a
*A. P49.59 B. P47.59 C. P45.59 D. P43.59 final temperature of 320oF in a reversible manner. Determine the exit pressure of Helium.
SOLUTION: A. 38.5 psia *B. 40.5 psia C. 42.5 psia D.
COP = RE/Wref 44.5 psia
1.3 = 40/Wref SOLUTION:
Wref = 30.769 watts T2/T1 = (P2/P1)n-1/n
W = Wb + Wref = 40 + 30.769 = 70.77 watts (320 + 460)/(50 +460) = (P2/14)1.587-1/1.587
W = 0.07077 Kw P2 = 40.5 psia
Cost = 0.07077(8760)(P0.08) = P49.59
Air pass thru a nozzle with efficiency of 90%. The velocity of air at the exit is 600 m/s. Find
A 75 hp motor that has an efficiency of 91% is worn out and is replaced by a high-efficiency the actual velocity at the exit.
motor that has an efficiency of 95.4%. Determine the reduction in heat gain of the room due A. 382 m/s B. 540 m/s C. 458 m/s *D. 568 m/s
to higher efficiency under full-load conditions. SOLUTION:
A. 2.24 KW *B. 2.44 KW C. 2.64 KW D. 2.84 KW SOLUTION: e = (v2/v3)2
0.9 = (v2/600)2
P01 = (75 x 0.746)(0.91) = 50.91 KW v2 = 568.21 m/s
P02 = (75 x 0.746)(0.954) = 53.376 KW
Qreduced = 53.376 – 50.91 = 2.44 KW A 50 kg block of iron casting at 500K is thrown into a large lake that is at a temperature of
258oK. The iron block eventually reaches thermal equilibrium with the lake water. Assuming
A household refrigerator that has a power input of 450 watts and a COP of 2.5 is to cool average specific hear of 0.45 KJ/kg-K for the iron, determine the entropy generated during
five large watermelons, 10 kg each, to 8oC. If the watermelons are initially at 20oC, this process.
determine how long will take for the refrigerator cool them. The watermelons can be treated *A. -12.65 KJ/k B. 16.97KJ/K C. 4.32 KJ/K D. 6.32 KJ/K
as a water whose specific heat is 4.2 KJ/kg-oK. SOLUTION:
A. 2220 seconds B. 2230 seconds *C.2240 seconds D. 2250 ΔSiron = m c ln (T2/T1) = 50(0.45)ln(285/500) = -12.65
seconds KJ/K ΔSlake = Q/T = [50(0.45)(500-285)]/285 = 16.97 KJ/K
SOLUTION: ΔSgen. = -12.65 + 16.97 = 4.32 KJ/K
COP = RE/Wc
2.5 = RE/450 A windmill with a 12 m diameter rotor is to be installed at a location where the wind is
RE = 1,125 watts blowing at an average velocity of 10 /s. Using standard conditions of air (1 atm,
RE = m cp (t2 – t1) 25oC), determine the maximum that can be generated by the windmill.
450 t = (10 x 5)(4.2)(20-8) A. 68 KW *B. 70 KW C. 72 KW D. 74
t = 2240 seconds KW
SOLUTION:
When a man returns to his wall-sealed house on a summer day, he finds that the house is at w = P/RT = 101.325/(0.28)(25+ 273) = 1.1847 kg/m3
32oC. He returns on the air conditioner which cools the entire house to 20oC in 15 minutes, m = w A v = 1.1847(π/4 x 122)(10) = 1,1339.895
if COP is 2.5, determine the power drawn by the airconditioner. Assume the entire mass kg/s KE = v2/2000 = 102/2000 = 0.05 KJ/kg Power =
within the house is 800 kg of air for which cv = 0.72 KJ/kg-K, cp = 1.0KJ/kg-K. m KE = 1,1339.895(0.05) = 70 KW
A. 1.072 KW B. 2.072 KW *C. 3.072 KW D. 4.072 KW SOLUTION:
Consider a large furnace that can supply heat at a temperature of 2000oR at a steady rate of
RE = m cv (T2 –T1) = (800/15x60)(0.72)(32-20) 3000Btu/s. Determine the energy of this energy. Assume an environment temperature of 77oF.
RE = 7.66 KW
A. 2305.19 KW *B. 2315.19 KW C. 2325.19 KW D. Assume the atmospheric to be 14.7 psia and the temperature and the volume to
2335.19 KW remain constant.
SOLUTION: *A. 0.026 lb B. 0.046 lb C. 0.066 lb D. 0.086 lb
SOLUTION:
e= = 0.7315 PV=mRT
W = e Q = 0.7315(3000) = 2194.5 Btu/s = 2315.19 KW (20 + 14.7)(144)(0.53) = m1 (53.3)(90 + 460)
m1 = 0.09034 lb
(30 + 14.7)(144)(0.53) = m2(53.3)(90 + 460)
m2 = 0.11634 lb
A heat engine receives hat from a source at 1200oK at a rate of 5000KJ/s and rejects the madded = m2 – m1 = 0.11634 – 0.09034 = 0.026 lb
waste heat to a medium at 3000oK. The power output of the heat engine is 180 KW.
Determine the irreversible rate for this process. A rigid tank contains 20 lbm of air at 20 psia and 70oF. More air is added to the tank until
A. 190 KW *B. 195 KW C. 200 KW D. 205 KW SOLUTION: the pressure and temperature rise to 35 psia and 90 oF, respectively. Determine the amount
of air added to the tank.
e = (1200 – 300) / 1200 = 0.75 A. 11.73 lb *B. 13.73 lb C. 15.73 lb D. 17.73 lb
W = 0.75(500) = 375 KW SOLUTION:
Irreversibilities = 375 – 180 195 KW P1V1 = m1 R1T1
(20 x 144)(V1) = 20 (53.3)(70 + 460)
A dealer advertises that he has just received a shipment of electric resistance heaters for V = 196.17 ft3
residential buildings that have an efficiency of 100 percent. Assuming an indoor P2V2 = m2R2T2
temperature of 21oC and outdoor temperature of 10oC, determine the second law efficiency (35 x 144)(196.17) = m2 (53.3)(90 + 460)
of these heaters. m2 = 33.73 lbs
A. 8.74% B. 6.74% *C. 3.74% D. 4.74% madded = m2 –m1 = 33.73 – 20 = 13.73 lb
SOLUTION:
COP1 = 100% efficient = 1 A rigid tank contains 5 kg of an ideal gas at 4 atm and 40oC. Now a valve is opened, and
COP2 = (21 + 273) / (21 – 10) = 26.72 half of mass of the gas is allowed to escape. If the final pressure in the tank is 1,5 atm, the
e = COP1 /COP2 = 1/ 26.72 = 3.74 % final temperature in the tank is:
*A. -38oC B. -30oC C. 40oC D. 53oC
A thermal power plant has a heat rate of 11,363 Btu/KW-hr. Find the thermal efficiency of SOLUTION:
the plant. PV = m R T
A. 34% B. 24% C. 26% *D. 30% (4 x 9.81)(V) = 5(0.287)(40 + 273)
SOLUTION: V = 11.446 m3
e = 3412 / Heat rate = 3412 / 11363 = 30 % PV = mRT
(1.5 x 9.81)(11.446) = (5/2)(0.287)(T)
A rigid tank contains 2 kmol of N2 and 6 kmol of CO2 gasses at 300oK and 115 Mpa. Find T = 234.74oK = -38.26oC
the tank volume us ideal gas equation.
A. 7.33 m3 B. 5.33 m3 C. 3.33 m3 *D. 1.33 m3 The pressure of an automobile tire is measured to be 200 kpa(gage) before the trip and 220
SOLUTION: kpa(gage) after the trip at a location where the atmospheric pressure is 90 kpa. If the
PmVm = Nm R Tm temperature of the air in the tire before the trip is 25oC, the air temperature after the trip is:
15,000 Vm = (6 + 2)(8.314)(300) *A. 45.6oC B. 54.6oC C. 27.5oC D. 26.7oC
Vm = 1.33 m3 SOLUTION:
T2/T1=P2/P1
A spherical balloon with a diameter of 6 m is filled with helium at 20oC and 200 kpa. T2 / (25+ 273) = (220 +90) / (200 + 90)
Determine the mole number. T2 = 318.55 K
*A. 9.28 Kmol B. 10.28 Kmol C. 11.28 Kmol D. 13.28 Kmol t2 = 45.55oC
SOLUTION:
PV=NRT Water is boiling at 1 atm pressure in a stainless steel pan on an electric range. It is observed that
(200)[(4/3)(π)(6/2)3] = N (8.314)(20 + 273) 2 kg of liquid ater evaporates in 30 mins. The rate of heat transfer to the water is:
N = 9.28 Kmol A. 2.97 KW B. 0.47 KW *C. 2.51 KW D. 3.12 KW SOLUTION:
The air in an automobile tire with a volume of 0.53 ft3 is at 90oF and 20 psig. Determine the
amount of air that must be added to raise the pressure to the recommended value of 30 psig. Q = mL = (2257) = 2.51 KW
Consider a person standing in a breezy room at 20oC. Determine the total rate of heat system is raised by 15oC. The amount of extra heat that must be supplied to the gas in
transfer from this person if the exposed surface area and the average outer surface the cylinder that is maintained at constant pressure.
temperature of the person are 1.6 m2 and 29oC, respectively, and the convection heat
transfer coefficient is 6 W/m2 with emissivity factor of 0.95. SOLUTION:
A. 86.40 watts B. 61.70 watts C. 198.1 watts *D. 168.1 watts A. 0 B. 50 KJ C. 100 KJ *D. 150 KJ Q = m cp (t2 – t1) = (1.2 x
SOLUTION: 8.314)(1)(15) = 150 KJ
Qc = h A (t2 – t1) = (6)(1.6)(29.20) = 86.40 watts
Qr = (0.95)(5.67 x 10-6)[(1.6)(29 + 273)4 – (20 + 273)4] = 81.7 watts A supply of 50 kg of chicken needs at 6oC contained in a box is to be frozen to -18oC in a
Q = Qc + Qr = 86.40 + 81.7 = 168.1 watts freezer. Determine the amount of heat that needs to be removed. The latent heat of chicken is
247 KJ/kg, and its specific heat is 3.32 KJ/kg-oC above freezing and 1.77 KJ/kg-oC below
Water is boiler in a pan on a stove at sea level. During 10 minutes of boiling, it is observed that freezing. The container box is 1.5 kg, and the specific heat of the box material is 1.4 Kj/kg-
200 grams of water has evaporated. Then the rate of heat transfer to the water is: oC. Also the freezing temperature of chicken is -2.8oC.
A. 0.84 KJ/min *B. 45.1 KJ/min C. 41.8 KJ/min D. 53.5 KJ/min *A. 15,206.4 KJ B. 50.4 KJ C. 15,156 KJ D. 1,863 KJ SOLUTION:
SOLUTION:
Q = m L = (0.2 / 10) (2257) = 45.1 KJ/min Qchicken = 50 [3.32(6 + 2.8) = 247 1.77(-2.8 + 18)] = 15,156 KJ
Qbox = 1.5(1.4)(6 + 8) = 50.4 KJ Q = 15,156 + 50.4 = 15, 206.4
An aluminum pan whose thermal conductivity is 237 W/m-C has a flat bottom whose KJ
diameter is 20 cm and thickness 0.4 cm. Heat is transferred steadily to boiling water in the pan
through its bottom at a rate of 500 watts. If the inner surface of the bottom of the pan is Water is being heated in a closed pan on top of a range while being stirred by a paddle
105oC, determine the temperature of the surface of the bottom of the pan. wheel. During the process, 30 KJ of heat is transferred to the water, and 5 KJ of heat is lost
A. 95.27 oC *B. 105.27oC C. 115.27oC D. 125.27oC to the surrounding air. The paddle wheel work amounts to 500 N-m. Determine the final
SOLUTION: energy of the system if its initial energy is 10 KJ.
A = π / 4 ( 0.20)2 = 0.0314 m2 *A. 35.5 KJ B. 45.5 KJ C. 25.5 KJ D. 14.5 KJ
SOLUTION:
Q= Final energy = Qa + ΔU – Qloss + W = 30 + 10 – 5 + 0.50 = 35.5 KJ

A classroom that normally contains 40 people is to be air- conditioned with window air-
500 =
conditioning units of 5 KW cooling capacity. A person at rest may be assumed to dissipate
heat at a rate of about 360 KJ/hr. There are 10 light bulbs in the room, each with a rating of
T2 = 105.27oC
100 watts. The rate of heat transfer to the classroom through the walls and the windows is
estimated to be 15,00 KJ/hr. If the room air is to be maintained at a constant temperature of
For heat transfer purposes, a standing man can be modeled as a 30 cm diameter, 170 cm long
21oC, determine the number of window air- conditioning units required.
vertical cylinder with both the top and bottom surfaces insulated and with the side surface at an
A. 1 unit *B. 2 units C. 3 units D. 4 units
average temperature of 34oC. For a convection heat transfer coefficient of 15 W/m2- oC,
determine the rate of heat loss from this man by convection in an environment at 20oC.
SOLUTION:
Q = total head load = 40(360/3600) + 10(0.100) + 15,000/3600 = 9.167
A. 316.46 watts B. 326.46 watts *C. 336.46 watts D. 346.46 watts
SOLUTION: KW No. of air-conditioning = 9.167/5 = 1.833 = 2 units
Qc = k A (t2 – t1) = 15 (π x 0.30 x 1.7) (34 – 20) = 336.46 watts
A 4m x 5m x 6m room is to be heated by a baseboard resistance heater. It is desired that the
resistance heater be able to raise the air temperature in the room from 7 to 23oC within 15
A 5cm diameter spherical ball whose surface is maintained at a temperature of 70oC is
minutes. Assuming no heat losses from the room and an atmospheric pressure of 100 kpa,
suspended in the middle of a room at 20oC. If the convection heat transfer coefficient is 15
determine the required power of the resistance heater. Assume constant specific heats at
W/m2 – C and the emissivity of the surface is 0.8, determine the total heat transfer from
room temperature.
the ball.
A. 2.34 KW *B. 1.91 KW C. 4.56 KW D.
A. 23.56 watts *B. 32.77 watts C. 9.22 watts D. 43.45 watts
6.34 KW
SOLUTION:
SOLUTION:
A = 4 π r2 = 4 π (0.05)2 = 0.0314 m2
w = P / R T = 100 / (0.287)(7 +273) = 1.244 kg / m3
Qc = h A (t2 – t1) = 15 (0.0314)(70 – 20) = 23.56 watts
m = 1.244 (4 x 5 x 6) = 149.28 kg
Qr = (0.80)(5.67 x 10-6)(0.0314)[(70 + 273)4 – (50 + 273)4] = 9.22
Q = m cv (t2 – t1 ) = 149.28 (0.7186)(23 – 7) = 1,716.36
watts Q = Qr + Qc = 23.56 + 9.22 = 32.77 watts
KJ Power = 1,716.36 / (15 x 60) = 1.91 KW
A frictionless piston-cylinder device and rigid tank contain 1.2 kmol of ideal gas at the same
A student in a 4m x 6m x 6m dormitory room turns on her 150 watts fan before she leaves
temperature, pressure, and volume. Now heat is transferred, and the temperature of both
the room on a summer day, hoping that the room will be cooler when she comes back in the
evening. Assuming all the doors and windows are tightly closed and disregarding any heat pump the cold- deep water to the surface and through the system heat.Assume a Carnot
transfer through the walls and the windows, determine the temperature in the room when she cycle efficiency and density of cold water to be 1000 kg/m3.
comes back 10 hours later. Use specific heat values at room temperature, and assume the A. 108 *B. 250 C. 146 D. 160
room to be at 100 kpa and 15oC in the morning when she leaves SOLUTION:
A. 28.13oC B. 38.13oC C. 48.13oC *D. 58.13oC e = (Th – TL)/ TH = [(26 + 273) – (15 + 273)] / (26 + 273) =
SOLUTION: 0.0679 e =W / Qa
w = P / R T = 100 / (0.287)(15 + 273) = 1.2098 kg / 0.03676 = 10, 000 / QA
m3 m = 1.2098(4 x 6 x 6) = 174.216 kg Q = m cv (t2 – QA = 271, 612. 99 KW
t1) QR = Qa – W = 271,812.99 – 10, 000 = 261,813
0.15(10 x 3600) = 174.216 (0.7186)(t2 – 15) KW QR = m cp (Δt)
t2 = 58.13oC 261, 813 = m (4.187)(3)
M = 20, 643.32 kg/s
A piston cylinder device whose piston is resting on top of a set stops initially contains 0.50 Q = 20,843.32 kg/s or 20,843.32 li/s = 20.843 m3/s
kg of helium gas at 100 kpa and 25oC. The mass of the piston is such that 500 kpa of H = P / w = 12 / 9.81 = 1.223 m
pressure is required to raise it. How much heat must be transferred to the helium before the Wp = w Q h = 9.81(20.843)(1.223) = 250.12 KW
piston starts rising?
A. 1557.13 KJ B. 1657.13 KJ C. 1757.13 KJ *D. 1857.13 A plate – type solar energy collector with an absorbing surface covered by a glass plate is
KJ to receive an incident radiation of 800 W/m2. The glass plate has a reflective of 0.12 and a
SOLUTION: transmissivity of 0.85. The absorbing surface has an absorptivity of 0.90. The area of the
For helium: cv = R / (k-1) = (8.314 / 4) (1.667 – 1) = 3,116 KJ/ kg- collector is 5m2. How much solar energy in watts is absorbing by the collector?
K T2 = (25 + 273)(500 / 100) = 1,490oK T1 = 25 + 273 = 298oK A. 2500 B. 2880 C. 3510 *D. 3060
SOLUTION:
Q = m cv (T2 – T1) = 0.50(3.116)(1490 – 298) = 1857.13 KJ Q = heat absorbed from sun
Q = (800 W/m2)(5 m2)(0.85)(0.9) = 3,060 watts
In order to cool 1 ton (100kg) of water at 20oC in an insulated tank, a person pours 80 kg
of ice at -5oC into the water. Determine the final equilibrium temperature in the tank. The A tank contains liquid nitrogen at -190oC is suspended in a vacuum shell by three stainless
melting temperature and the hat of fusion of ice at atmospheric pressure are 0oC and 333.7 steel rods 0.80 cm in diameter and 3 meters long with a thermal conductivity of 16.3 W/m2-
kJ/kg, respectively. C. If the ambient outside the vacuum shell is 15oC, calculate the magnitude of the conductive
*A. 12.43oC B. 14.43oC C. 16.43oC D. 18.43oC heat flow in watts along the support rods.
SOLUTION: *A. 0.168 B. 0.0587 C. 0.182 D. 0.176
Qwater =Qice SOLUTION:
1000(4.187)(20 –te) = 80(2.09)(0 + 5) + 80(333.7) + 80(4.187)(te – 0) Q = h A (Δt) = 16.3 (π/4 x 0.0082)(15 – (-190)) = 0.168 watts
te = 14.43oC An elastic sphere containing gas at 120 kpa has a diameter of 1.0 m. Heating the sphere
causes it to expand to a diameter of 1.3 m. During the process the pressure is proportional to
A fan is powered by a 0.5 hp motor and delivers air at a rate of 85 m3/min. Determine the the sphere diameter. Calculate the work done by the gas in KJ.
highest value for the average velocity of air mobilized by the fan. The density of air to A. 41.8 B. 50.6 *C. 87.5 D. 35.4
1.18 kg/m3. SOLUTION:
PαD
A. 18.23 m/s *B. 21.12 m/s C. 25.34 m/s D. P=kD
32.23 m/s 120 = k(1)
SOLUTION: K=120
P=wQh P=120D
0.50(0.746) = ( 1.18 x 0.00981)( 85 / 60) (h) V = 4/3 π(D/d)2 = 4/24 π D3
h = 22.74 m dV = (12/24) π D2 dD

v= = 21.12 m/s W=
D3dD W = 87.47 KJ
An Ocean – Thermal Energy Conversion power plant generates 10,000 KW using a warm
surface water inlet temperature of 26oC and a cold deep- water temperature of 15oC. ON the An ideal gas with a molecular weight of 7.1 kg/kg mol is compressed from 600 kpa and
basis of a 3oC drop in the temperature of the warm water and a 3oC rise in the temperature 280oK to a final specific volume of 0.5 m3/kg. During the process the pressure varies
of the cold water due to removal and addition of heat, calculate the power required in KW to according to p = 620 + 150v + 95v2 where p is in kpa and v in m3/kg. Calculate the work
of compression in KJ/kg?
A. 32.8 B. 28.7 C. 35.6 *D. 33.3
SOLUTION: A simple Rankine cycle produces 40 MW of power, 50 MW of process heated and rejects
V1 = R T / P = (8.314 / 7.1)(280) / (600) = 0.546 m3/kg 50 MW of heat to the surroundings. What is the utilization factor of this cogeneration cycle
neglecting the pump work?
W= A. 50% B. 60% *C. 64% D. 80%
SOLUTION:
QA = WT + WprocessWp = 40 + 50 + 50 = 140 KW

UF= =64%
One cubic meter container contains a mixture of gases composed of 0.02 kg- mol of oxygen
and 0.04 kg-mol of helium at a pressure of 220 kpa. What is the temperature of this ideal The rate of heat transfer to the surroundings’ from a person at a rest is about 400 KJ/hr.
gas mixture in degrees Kelvin? Suppose that the ventilation system fails in an automobile in an auditorium containing 120
*A. 441 B. 350 C. 400 D. 450 people and assuming that the energy goes into the air of volume 1500 m3 initially at 300oK
SOLUTION: and 101 kpa, calculate the rate in oC/min of air temperature change.
V=V1+V2 A. 0.81 B. 0.53 *C. 0.63 D. 1.0
SOLUTION:
VT = Q = m cv Δt
PV = m RT
1= 101(1500) = m (0.287)(300)
m = 1,759.58 kg
T = 441 oK Q = m cv Δt
120(400 / 60) = 1759.58 (0.7186)(Δt)
Methyl alcohol (CH3OH) is burned with 25% excess air. How much unburned oxygen in Δt = 0.633 oC / min
kg-mol-oxygen / kg-mol fuel will there be in the products if the combustion is complete?
A. 0.35 B. 0.45 *C. 0.37 D. 0.65 An insulated box containing helium gas falls from a balloon 4.5 km above the earth’s surface.
SOLUTION: Calculate the temperature rise in oC of the helium when box hits the ground.
CH3OH + O2 + (3.76)N2 = CO2 + H2O + (3.76)N2 A. 15.2 B. 12.6 C. 25.3 *D. 14.1
CH3OH + 1.5O2 + 1.5(3.76)N2 = 1 CO2 + 2H2O + 1.5(3.76)N2 SOLUTION:
Consider 25% excess air: Cv of helium = 3118.9 j/kg-C
CH3OH + 1.25(1.5)O2 + 1.25(1.5)(3.76)N2 = 1CO2 + 2H2O + 1.25(1.5)(3.76)N2 m g h = m cv Δt
+ 0.25(1.5)O2 m (9.81)(4500) = m (3118.9) Δt
Unburned O2 = 0.25(1.5) = 0.375 Δt = 14.15 oC

A 12 DC electrical motor draws a current of 15 amps. How much work in KJ does this Consider two Carnot heat engines operating in series. The first engine receives heat from the
motor produce over a 10 minute period of operation? reservoir at 2500oK and rejects the waste heat to another reservoir at temperature T. The
*A. 108.0 B. 129.6 C. 216.0 D. 318.2 second engine receives heat by the first one, convert some of it to work, and rejects the rest
SOLUTION: to a reservoir at 300oK. If thermal efficiencies of both engines are the same, determine the
W=E=QV temperature T.
W = (15 x 10 x 60)(12) = 108,000 J = 108 KJ *A. 849oK B. 578 oK C. 763 oK D. 976 oK
SOLUTION:
A 4 liter (2-liter per revolution at standard pressure and temperature) spark ignition engine et = e2
has a compression ratio of 8 and 2200 KJ/kg heat addition by the fluid combustion.
Considering a cold air-standard Otto cycle model, how much power will the engine produce
when operating at 2500 rpm? T = 849oK
*A. 166.53 hp B. 73.12 hp C. 97.4 hp D. 148 hp
SOLUTION: An ideal gas mixture consists of 2 kmol of N2 and 6 kmol of CO2. The mass fraction of CO2
W = 1.2 kg/m3 (standard density of air) is:
M = 2 li/rev x 2500 rev/min x 1.2 kg/m3 x 1 m3/ 1000 li x 1 min/ 60 sec = 0.10 A. 0.175 B. 0.250 *C. 0.825 D. 0.750
kg/s e = W / QA SOLUTION:
e = 1 -1 / 8 1.4 -1 = 0.5647
0.5647 = W / 2200 6(44)
W = 1,242.34 KJ/kg (0.10 kg/s) = 124.23 KW = 166. 53 hp Mass fraction of CO2 = = 0.825
6(44) + 2(28) Air entered a turbojet engine at 200 m/s at a rate of 20 kg/s, and exists at 800 m/s
An ideal gas mixture consists of 2 kmol of N2 and 6 kmol of CO2. The relative to the aircraft. the thrust developed by the engine is:
apparent gas constant of mixture is: A. 6KN *B. 12KN C.16KN D. 20KN
*A. 0.208 B. 0.231 C. 0.531 D. 0.825 Solution:
Solution: Thrust developed = m(v2 – v1) = 20(800 – 200) = 12,000N = 12KN A thermal
M= (2/8)(28) + (6/8)(44) = 40 power has a net power 10MW. The backwork ratio of the plant is 0.005. Determine
R= 8.314/M = 8.314/40 = 0.208 KJ/kg-K the compressor wor.
A Carnot cycle operates between the temperature limits of 300OK and 1500OK, A. 50.15KW B. 50.35KW *C.50.25KW D. 50.45KW
and produces 600 KW of net power. Solution:
The rate of entropy changes of the working fluid during the heat addition Wnet= WT + WP
process is: BW= WP / WT
A. 0 B. 0.4KW/K *C. 0.5KW/K D.2.0KW/K 0.005 =WP / WT
Solution: WP= 0.005WT
W= ( s) (TH – TL) Wnet= WT - WP
600=( s) (1500 – 300) 10,000 = WT – 0.005WT
59. s)= 0.50 KW/K WT= 10,050.25 KW
Air in an ideal Diesel cycle is compressed from 3L to 0.15L and then it expands WC= 0.005(10,050.25) = 50.25KW
during the constant pressure heat addition process to 0.3L. Under cold air standard A heat engine receives heat from a source at 1200oK at a rate of 500KJ/s and
conditions, the thermal efficiency of this cycle is: rejects the waste heat to a sink at 300oK. If the power output of the engine is
rk = 3/0.15= 20 200KW, the second law efficiency of the heat engine is:
rc= 0.3/0.15= 2 A.35% B.40% *C.53% D.75% Solution:

1 rck- 1 1 21.2 - 1 ea= 200/500 = 0.40


e=1- = 1- et= (TH – TL)/TH = (1200 – 300)/1200 = 0.75 es=
= 0.6467 0.40/0.75= 53.33%
rkk-1 k(rc – 1) 201.4-1 1.4 (2- A water reservoir contains 100,000 kg of water at an average elevation of 60 m. The
1) maximum amount of electric power that can be generated from this water is:
=64.67%
Helium gas is an ideal Otto cycle is compressed from 20oC and 2L to 0.25L A.8KWh *B.16KWh C.1630KWh D.58, 800KWh
and its temperature increases by an additional 800oC during the addition
process. The temperature of helium before the expansion process is: Solution:
*A. 1700oC B. 1440oC C. 1240oC D.880oC P= m h = (100,000 x 0.00981)(60)= 58,860 KJ P=
Solution: 58,860 KJ x KWh/3600 KJ = 16.35KWh
rk =2/0 25 = 8 A house is maintained at 22oC in winter by electric resistance heaters. If the
T2 = (20+273) (8)1.667-1 =1,172K outdoor temperature is 5oC, the second law of efficiency of the resistance heaters
T3 = T2 + 800 = 1172 + 800 = 1972oK is:
t3 = 1699oC = 1700oK A.0% *B.5.8% C.34% D.77% Solution:
In an ideal Brayton cycle has a net work output of 150KJ/kg and backwork ratio of
0.4. If both the turbine and the compressor had an isentropic efficiency of 80%, the ea= 100% of resistance heaters
net work output of the cycle would be. et= (22 – 5)/(22 + 273) = 5.8%
A. 50KJ/kg *B. 75KJ/kg C. 98KJ/kg D.120KJ/kg es= 5.8/100 = 5.8%
Solution: A thermoelectric refrigerator that resembles a small ice chest is powered by a car
Backwork ratio = WO/WT battery, and has a COP of 0.10. If the refrigerator cools at 0.350L canned drink from
0.40 = WO/WT 20OC to 4OC in 30 min. determine the average electric power consumed by the
WO = 0.40 WT thermoelectric refrigerator.
Wnet = WT - WO *A.130 watts B.110 watts C.120 watts
150 = WT – 0.4 WT D.140 watts
WT =250 KJ/kg Solution:
WT ‘=250(0.8) = 200KJ/kg (1 x 0.35)
WP = 0.40(200) =100KJ/kg Q= m cp(t2 – t1) = (4.187)(20 – 4) = 13 watts 30 x 60
WP’ = 100/0.80 =125 KJ/kg
COP= RE/Wc
Wnet= WT’ – WC’= 200 – 125 = 75 KJ/kg
0.10= 13/0.10= 130 watts $ Savings per month = (1200 – 500) (0.085) = $59.50
A Carnot refrigerator operates in a room in which the temperature is 25OC and An ammonia simple saturation cycle operates with a suction pressure of 291.6
consumes 2 kW of power when operating. If the food compartment of the kPa and a condenser pressure of 1204 kPa develops 15 tons of refrigeration.
refrigerator is to be maintained at 3OC, determine the rate of heat removal from the Determine the theoretical horsepower of the compressor.
food compartment. The following enthalpies have been found: condenser entrance = 1653
*A.1504.8 kJ/min B.12.86 kJ/min C.1625 kJ/min D.9.57 kJ/min kJ/kg, exit =346.6 kJ/kg, compressor entrance = 1450.2 kJ/kg, exit= 1653kJ/kg.
Solution: A.7.23 hp *B.13 hp C. 15 hp
COP= TL /TH– TL = (3 + 273)/ (25 + 273) – (3+273) =12.54 QL= D.8.23 hp
COP x W = 12.54 x 2(60) = 1504.8 kJ/min Solution:
A household refrigerator with EER 8.0 removes heat from the refrigerated space at m= Qe/ (h1 – h4) = (15 x 3.52) / (1450.2 – 346.6) = 0.0478 kg/s W= m
a rate of 90 kJ/min. Determine the rate of heat transfer to the kitchen air. (h2 – h1) = (0.0478)(1653 – 1450.2) / 0.746 = 13 hp
An ammonia ice plant operates between a condenser temperature of 35OC and
A.101.25 kJ/min B.63.05 kJ/min *C.128.46 kJ/min evaporator of -15OC. It produces 10 metric tons of ice per day from water at 30OC to
D.80 kJ/min ice at -5OC. Assuming simple saturation cycle, determine the horsepower of the
Solution: motor if the adiabatic efficiency of the compressor ηc=0.85 and mechanical efficiency
COP= EER /3.412 = 8/ 3.412 = 2.34 ηm=0.95. The specific heat of ice is 2,094kJ/kg. OC and the latent heat is 335kJ/kg.
COP= QL /QH – QL = 2.34 = 90 / QH – 90
QH=128.46Kj/min From the table for ammonia the following enthalpies are: condenser entrance
An air-conditioning system is used to maintain a house at 75OF when the = 1703 kJ/kg, exit= 366.1 kJ/kg; compressor entrance= 1443.9kJ/kg, exit =
temperature outside is 95OF. The house is gaining heat through the walls and 1703kJ/kg
windows at a rate of 1250 Btu/min, and the heat generation rate within the house A.17.68 hp B.18.61 hp C.15.5 hp *D.21.9 hp
from people, lights and appliances amounts to 350 Btu/min. Determine the Solution:
minimum power input required for this air-conditioning system. qe=cpa (te – tf) lhlatentlcph(tf – ts)
64. (4.187) (30 – 0) + 335 + (2.094) [(0- (-5)] = 471.08 kJ/kg Qe=
A.10.06 hp B.1.36 hp *C.1.41 hp D.7.94 hp (10 x 1000) (471.08) / 24 = 196,283.33 kJ/hr= 54.523kJ/s m= Qe / (h2 –
h4) = (54.523)/ (1443.9 – 366.1) = 0.05059kg/s
W= m (h2 – h1) = (0.0509) (1703 – 1443.9)/ 0.746 = 17.68 hp
Solution: Wmotor= 17.68 / (0.85) (0.95) =21.9 hp
QL= 1250 + 350 = 1600 Btu/min A Freon 22 air conditioning under standard operating conditions of 35OC is
COP= TL / TH – TL= (75 + 460)/(95 + 460) - (75+460) = 26.75 W= QL condensing and 5OC evaporating temperatures. The volume flow rate entering the
/COP = (1600 / 26.75) / 42.4 = 1.41 hp compressor is 23.72 L/s. determine the refrigerating capacity if the refrigerating
A refrigeration system is to cool bread loaves with an average mass of 450 g from effect is 164 kJ/kg. From the table for R22 the specific volume at the compressor
22OC to -10OC at a rate of 500 loaves per hour by refrigerated air. Taking the entrance is 40.36L/kg.
average specific and latent heats of bread to be 2.93 kJ/kg, OC and 109.3 kJ/kg, A.393.3 TR B.79.3 TR C.96.4 TR *D.27.4 TR Solution:
respectively, determine the product load.
A.541.7 kJ/min B.351.6 Kj/min *C.761.5 kJ/min D.409.9 kJ/min m= V1/v1= 23.72 / 40.36 = 0.5877 kg/s Qe= m
Solution: (qe) = 0.5877 (164)/ 3.52 = 27.4 TR
Mbread= (500 breads/h) (0.45 kg / bread) = 225 kg/h Qtotal=
Qbread + Qfreezing
52. (mcp∆T)breadl (mhlatent)bread = (225)(2.93)[22-(-10)] l (225)
(109.3)
Qtotal= 45,688.5 kJ/h = 761.5 kJ/min The refrigerant volume flow rate at the entrance of compressor were obtained
A house that was heated by electric resistance heaters consumed 1200 kWh of from a test on a twin cylinder single acting 15 cm x 20 cm, 320 rpm compressor
electric energy in a winter month. If this house were heated instead by a heat pump ammonia refrigerating plant 33 L/s. Determine the volumetric efficiency of the
that has an average performance factor, PF of 2.4, determine how much money the compressor.
homeowner would be saved that month. Assume a price of 0.085$/kWh for A.77.65% *B.87.6% C.97.6 TR D.65.65% Solution:
electricity.
A. $42.5 *B. $59.50 C.$109 D.$97.75 VD = (π D2 L/4) N= (π /4) (0.15)2(0.2) (320) (2) = 2.26 m3/min nv
Solution: =V1/VD = 33/2.26 (1000/60) = 0.876 or 87.6 %
W= QH/PF = 1200kWh / 2.4 = 500 kWh
A twin cylinder ammonia compressor with volume displacement of 14,726 cm3 m = Qc/qc = 50/1154.4 = 0.0433kg/s
operates at 300 rpm. Condenser and evaporator pressure are 1200 kPa and 227 Solving for refrigerating effect using energy balance about the
kPa respectively. Specific volume of refrigerant at the entrance of compressor is evaporator h5 – h4 = 366.1kJ/kg
528.26 L/kg. Compression process is polytrophic with a 1.20 and clearance factor of
compressor is 2%. Determine horsepower required. qe= h1 – h5 = 1442.6 – 366.1 = 1076.5kJ/kg
Therefore, the refrigerating capacity
A.60hp B.70hp C.80hp D.90hp Qe= m qe = [(0.0433) (1076.5)] / 3.52 = 13.24 TR
Solution:
nv= 1+c-c(p2/p1)1/n= 1 + 0.02 – 0.02(1200/227)1.20= 0.8725 VD= (π An actual refrigerating cycle using R12 as working fluid, the refrigerant flow rate is
D2 L/4) N= (0.014726) (320) (2) = 9.424 m3/min V1= (nv) VD- (0.8725) 0.05 kg/s. Vapor enters the expansion valve at 1.15 MPa, 40OC (h = 238.5kJ/kg)
(9.424) - 8.2224 m3/min W= (n p1 V1 /1-n) [(p2/p1) n-1/n-1] and leaves the evaporator at 175 kPa, -15OC (h = 345kJ/kg). The electric input to
motor driving the compressor is measured and found 3.0 kW. Motor efficiency at this
= [(1.20 x 2.27 x 8.2224)/ (1-1.20)] [(1200/2.27)1.2-1/1.2 -1] = 3582 load is 92% and mechanical efficiency 82%. Determine the actual coefficient of
kJ/min performance for this cycle.
W= 3582/ (60) (0.746) = 80 hp
A.1.58 B.2.36 C.1.78 D.1.34 Solution:
A reversed Carnot cycle has a refrigerating COP of 2.5. Determine the ratio TH/TL?
Qe= (0.05) (345 - 238.5) = 5.325kW
A.1.4 B.1.5 C.1.25 D.1.2 Solution: W = (3) (0.92) (0.82) = 2.26kW
COPactual= 5.325 / 2.26 = 2.36
COP= TH/ (TH-TL)
1/COP = (TH-TL)/TL= TH/TL-1 An ammonia refrigeration system the temperature in the evaporator is
TH/TL =1 + 1/COP =1 +1/2.5 =1.4 -12OC and the ammonia at the evaporator entry is 0.1511 dry while at exit is 0.95
dry. If the rate of ammonia circulation is 5.64 kg/min, determine the refrigerating
Three thousand cubic feet per minute of air are circulated over an air-cooled capacity of the system. Enthalpy of saturated liquid and vapor at -12OC is
condenser. If the load on the condenser is 64,800 Btu/hr, compute the temperature 144.929kJ/kg and 1447.74kJ/kg respectively.
rise of the air passing over the condenser. Specific volume of standard air A.17.82 B.34.82TR C.27.82TR D.4.82TR
(13.34ft3/lb)
A.10OF B.15OF C.20OF D.25OF Solution:
Solution: h4= hf4 + x4 (hg4 – hf4) = 144.929 + (0.1511) (1447.74 – 144.929) = 341.78
Qc= m c ∆t
∆t = Qc/ m c = 64,800/ [3000 (60) / (13.34)] (0.24) = 20OF h1= hf1 + x1 (hg1 – hf1) = 144.929 + (0.95) (1447.74 – 144.929) =
1382.6kJ/kg
Saturated vapor ammonia at -16OC (h1 = 1442.60 kJ/kg) leaves the evaporator and Qe = m (h1 – h4) = (5.64) (1382.6 – 341.78) / 211 = 27.82 TR
enters the compressor at -6OC (h1 = 1465kJ/kg. The refrigerant leaves the
condenser as saturated liquid at 40OC (h4= 390.6 kJ/kg) and enter the expansion A two stage cascade vapor compression refrigeration system uses ammonia in the
valve at 35OC (h5 =366.1kJ/kg). Heat rejected from the compressor amount to low -temperature loop and R-12 in the high-temperature loop. The ammonia
50kW. The work to compressor is 208kJ/kg, while the heat loss from compressor is provides 15 tons cooling. If the high-loop temperature requires 10.12 kW
33kJ/kg. If 95kJ/kg of heat are lost in the piping between the compressor discharge compressor power and low-loop 15.93 kW, determine the COP of the system.
and condenser inlet, determine the refrigerating capacity of the system.
A.2.027 B.5.22TR C.3.314TR D.9.1TR Solution:
A.49.5 TR B.46.61 TR C.12.88 TR D.13.24 TR Solution:
Wtotal= 10.12 + 15.93 = 26.05 kW
Solving for the enthalpy at the exit of compressor using energy balance COP = 15 (3.52) /26.05 = 2.027
about the compressor
h2 = h1 + w - qwc = 1465 + 208 – 33= 1640 kJ/kg When a man returns to his well-sealed house on a summer day, he finds that the
Solving for the enthalpy at the entrance of condenser using energy balance house is at 32OC. He turns on the air conditioner, which cools the entire house to
about piping from compressor exit to condenser entrance 20OC in 15min. If the COP of the air-conditioner system is 2.5, determine the power
h3= h2 – q2-3= 1640 – 95 = 1545kJ/kg drawn by the air conditioners. Assume the entire mass within the house is equivalent
Solving for heat rejected in condenser using energy balance about the to 800 kg of air for which c= 0.72kJ/kg OC
condenser
qc = h3 – h4 = 1545 – 390.6 = 1154.4kJ/kg A.7.68Kw B.3.07kW C.19.2kW D.12.03kW
Solution: m = 18kg
Qe = m c (t1 – t2) / ∆time = (800) (0.72) (32 – 20) / (15 x 60) = 7.68kW A condenser vacuum gauge reads 715 mm Hg when the barometer stands at 757
mm Hg. State the absolute pressure in the condenser in kN/m or kPa.
W= Qe / COP = 7.68 / 2.5 = 3.07kW
A.5.6kPa B.5.9kPa C.6.5kPa D.5.2kPa Solution:
It is desired to double the COP of a reversed Carnot engine for cooling from 5.0 by
raising the temperature of the heat addition while keeping the temperature of the heat Pabs = Patm – Pvac = 757 – 715 = 42 mm Hg (101.325kPa/760 mm Hg) =
rejection constant. By what percentage must the temperature of heat addition be 5.60kPa
raised? Determine the force in Newton in a piston of 465 mm2 area with a pressure of
A.10.1% B.9.1% C.8.1% D.7.1% Solution: 0.172MPa.
A.65N B.72N C.80N D.111 Solution:
Percent increase of the temperature = (TL – TL) TL = TL / TL – 1 of heat
addition F= PA – (0.172MPa) (105 Pa/MPa) (465 mm2) (m2 / 105 mm2)
Original COP: 5 = TL / (TH – TL); (TH – TL) TL = 1/5; TH / TL -1 = 1/5 F = 79.98N
77. 02; TH / TL One piston of a hydraulic press has an area of 1cm2. The other piston has an area
= 1.2 eq.1 of 25cm2. If a force of 130N is applied on the smaller piston, what will be the total
Doubling the COP: force on the larger piston is both piston surfaces are the same level?
10 = TL / (TH – TL); (TH – TL) TL = 1/10; TH / TL – 1 =1/10 = 0.1;
TH/TL=1.1 A.6N B.175N C.3750N D.4250N
Dividing eq.1 and eq.2: Solution:
(TH/TL) (TH/TL) = 1.2 / 1.1 = 1.091 F1/A1 = F2/A2
Percent Increase of heat addition = 1.091 – 1 = 0.091 or 9.1% An ammonia (150/1) = (F2/ 25)
water-cooled compressor receives the refrigerant at specific volume 62 L/kg. It has F2 = 3750N
a piston displacement rate of 3m3/min. If a squirrel cage motor is running at 1200 If the pressure of a confined gas at a constant temperature is tripled, what will
rpm drives the compressor and average piston speed is 490m/min, calculate size of happen to the volume?
cylinder bore. A. The volume will be tripled B. The
A.20.4 cm B.26.0 cm C.16.13 cm D.13.6 cm volume will remain
Solution: C. The volume will be reduced to one-third of its original value D.
Piston speed = 2LN = 490 = 2(L) (1200); L= 0.204 m = 20.4 cm The volume is constant
Vp = (π D2 L/4) N = (π/4) D2 (0.204) (1200) =5 Solution:
D = 0.1613m = 16.13cm P1V1 = P2V2
If the initial volume of an ideal gas is compressed to one-half its original P1V1 = (3P1) V2
volume and to twice its temperature, the pressure: V2 = (1/3) V1
A. Doubles B. Quadruples C. Remains constant
90. Halves The work done on air is 10.86kJ/kg, determine the compressor power if it is receiving
Solution: 272 kg/min if air.
P1V1/T1 = P2V2/T2 P1V1/T1= A.36.72 hp B.49.23 hp C. 29.54 hp D.66 hp
P2 (1/2 V1) / (2T1) P2=4P1 Solution:
W= (10.86) (272.60) = 49.232 kJ/s or kW
If the gage pressure of a medium is 30kPa (vacuum) and the atmospheric pressure = 49.232 kW (1 hp/ 0.746kW) = 65.99 hp
is 101.3kPa, the absolute pressure will be: A water tank of 18 ft long and 4 ft high, calculate the pressure at the
A.131.3kPa B.-71.3kPa C.71.3kPa D.- bottom of the tank.
131.3kPa A.1.733psi B.1.999psi C.2.337psi D.3.773psi
Solution: Solution:
Pabs = Patm - Pvac P= (62.4 lbf/ft3) (4ft) (1ft2 / 144 in2) = 1.733psi The
= 101.3kPa - 30kPa = 71.3kPa pressure of 750 mm Hg in kN/m2
If a particle has a velocity of 4 meters per second and a kinetic energy of A.90 B.100 C.103 D110 Solution:
144 Joules, then the mass, in kilograms of this particle must be:
A.44 B.16 C.18 D.24 Solution: P= 750 mm Hg (101.325 kPa/ 760 mm Hg)
= 99.99 kPa
KE = (1/2) (m) (V2) / 2k A double purpose tank 18 ft wide, 24 ft long and 4 ft depth is filled with water.
144 – (1/2) (m) (4)2 / [(2) (1)] What is the weight of water in the tank in long tons?
A.49tons B.48tons C.54tons D.50tons The equivalent weight of mass 10 kg at a location where the acceleration of gravity
Solution: is 9.77 m/sec2
W = (62.4 lb/ft2) [(18) (24)(4)] ft3 = 107,827.2 lbf = A. 97.7N B.79.7N C.77.9N
107,827.2 lb (1ton/ 2200 lb) = 49tons D.977N
Solution:
Oil flow through a 16 tubes on a single cooler with a velocity of 2 m/s, the internal Weight = mg/k = [(10) (9.77)] /1 =97.7N
diameter of the tube is 30mm and oil density is 0.85gm/ml. Find the volume flow in A transportation company specializes in the shipment of pressurized gaseous
liters per sec. materials. An order is received from 100 liters of a particular gas at STP (32OF and
A.22.62 B.32.22 C. 62.22 D.42.62 Solution: 1 atm). What minimum volume tank is necessary to
transport the gas at 80OF and maximum pressure of 8 atm?
Volume flow rate = (3.1416) (0.015)2 (2) (16) = 0.02262 m3/s or 22.62 A.16 liters B.14 liters C.10 liters D.12 liters
liters/s Solution:
P1V1/ T1 = P2V2/ T2
A substance temperature was 620OR. What is the temperature in OC? [(1) (100)/ (32 + 460)] / [(8) (V2)/ (80 + 460)
A.50.7 B.45.54 C.71.11 D.94.44 V2 = 13.72 liters
100 g of water are mixed with 150 g of alcohol (density = 790 kg/m3). What is the
Solution: specific volume of the resulting mixtures, assuming that the
T, OC = [(620 – 460) – 32] (5/9) = 71.11 fluids mixed completely?
A.0.82x10-3 cu.m/kg B.0.88x10-3 cu.m/kg C.0.63x10-3 cu.m/kg
Unknown volume of container gas of gas of 1 atmosphere is allowed to expand to D.1.16x10-3 cu.m/kg
another container of 10 m3 volume at 500 mm Hg at constant temperature. Find Solution:
the unknown volume. Mass of mixtures = 100 + 150 =250 g
A.6.58 m3 B.6.75 m3 C.5.67 m3 D.7.65 m3 Volume of mixture = [(0.100) / (1000)] + [(0.150)/ (790)] = 0.00029
Solution: m3
P1V1 – P2V2 Specific volume of mixture = (0.00029)/ (0.250) = 1.16x10-3 cu.m/kg
103. V1 = (500) (10)
V1= 6.58 m3 How much does 30 lbm weigh on the moon? (gmoon = 5.47 ft/s2
A.20 lbr B.3.2 lbr C.3.4 lbr D.5.096 lbr
An iron block weighs 5 Newton and has volume of 200 cm3. What is the density
of the block? Solution:
A. 2458kg/m3 B. 2485 kg/m3 C. 2584 kg/m3 Weight = mg/k = {[(30) (5.42)]/32.174} = 5.1 lbf
D. 2549 kg/m3 A 10 kg block is raised vertically 3 meters. What is the change in potential energy?
Solution:
Density = specific weight [at sea level or near the surface of the A.320 J B.350 kg-m2/s2 C.294 J
earth] D.350 N-m
= (5N/200 cm) (10 cm3/m3) (1kg/9.8066N) = 2549.30 kg/m3 Solution:
PE = mgs/k = {[(10) (9.8066) (3)]/1} = 294.2 J How
If air is at a pressure of 22.22 psia and at temperature of 800 OR, what is the many cubic meters is 100 gallons of liquid?
specific volume? A.3.7850 cu.m B.0.1638 cu.m C.0.3785 cu.m D.1.638 cu.m
A.11.3 ft3/lbm B.33.1 ft3/lbm C.13.3 ft3/lbm Solution:
D.31.3 ft3/lbm 100gal (785 liters/gal) (1m3/1000 liters) = 0.3785 m3 Steam turbine is
Solution: receiving 1014 lbm/hr of steam, determine the horsepower output of the turbine if the
Pv = RT work done by steam is 251 Btu/lbm
v = (53.34) (800) / [(22.22) (144)] = 13.33 ft3/lbm A.100 hp B.462.7 hp C.200 hp D.6002.7
hp
The specific gravity of mercury is 13.55. What is the specific weight of Solution:
mercury? W= (251 Btu/lbm) (1014 lbm/hr) 1hp/2545 Btu/hr) = 100 hp
A.123.9 kN/m3 B.139.2 kN/m3 C.139.9 kN/m3 What is the resulting pressure when one pound of air at 15 psia and 200OF
D.193.2 kN/m3 is heated at constant volume to 800OF?
Solution: A.52.1 psia B.15 psia C.28.6 psia D.36.4 psia
? = (13.55) (9.8066) = 132.88 kN/m3
Solution:
P1/T1 = P2/T2 ANSWER: D
P2 = [(800+460) / (200+460)] (15) = 28.64 psia As a rule of thumb, for a specified amount of compressed air, the power
A bicycle tire has a volume of 600 cm3. It is inflated with carbon dioxide to pressure consumption of the compressor decreased by ______ for each 3OC drop in
of 551.43kPa at 20OC. How many grams of CO2 are contained in the tire? Rcoz = the temperature inlet air to the compressor.
0.18896 kJ/kg.K A. 1% B. 1.5% C. 2% D. 2.5%
A.5.98 g B.6.43 g C.4.63 g ANSWER: A
D.3.83 g Modern way of detecting air compressor leak is by using
Solution: A. Soup and water B. Air leak detector C. Acoustic leak detector
m = rv/RT = [(551.43) (600)/ (102)] / [(0.18896) (20+273)] = 0.00598 D. Ammonia leak detector
kg or 5.98 g ANSWER: C
For foundation of stacks, the maximum pressure on the soil is equal to the
Is the most common dryer used which consist of rotating cylinder inside pressure due to the weight and the .
which the materials flow while getting in contact with hot gas. A. Soil movement B. Wind movement C. Ground movement
A. Tower dryer B. Centrifugal dryer C. Tray D. Engine movement
dryer D. Rotary dryer ANSWER: B
ANSWER: D Foundation bolts of specified size should be used and surrounded by a pipe
Is the ratio of the mass water-vapor in air and the mass of air if it is sleeve with an inside diameter of at least
saturated is called: A. 3 times the diameter of engine bolt B. 3 times the diameter
A. Humidity ratio B. Mass ratio C. Vapor ratio D. of anchor bolt
Relative humidity C. 2 times the diameter of engine bolt D. 2 times the diameter
ANSWER: D of anchor bolt
The hands feel painfully cold when the skin temperature reaches: ANSWER: B
A.8OC B.10OC C.12OC D.14OC For multi stage compression of an ideal Brayton cycle, the backwork ratio
ANSWER: B will
The refrigerant used in steam jet cooling is: A. Increase B. Decrease C. Remain the same D.
A. Steam B.R-11 C. Ammonia D. Water none of these
ANSWER: D ANSWER: B
The total heat of the air is a function of Type of turbine that has a specific speed below 5
A. WB temperature B.DP temperature C.DB temperature D.WB depression A. Impulse turbine B. Propeller turbine C. Francis turbine D.
Deriaz turbine
ANSWER: A ANSWER: A
Boiling point of Freon-12 at atmospheric pressure is: A high discharge type of turbine
A. 21OF B.15OF C.5OF D.28OF A. Impulse turbine B. Francis turbine C. Propeller turbine D.
ANSWER: A Deriaz turbine
Which of the following is NOT a type of water cooled condenser in ANSWER: C
refrigeration? Use to minimize the speed rise due to a sudden load rejection
A. Double pipe B. Double shell C. Shell and coil D. Shell A. needle valve B. wicket gate C. Shut-off valve
and tube D. jet deflector
ANSWER: B ANSWER: D
Component of absorption refrigeration system in which the solution is Is the speed of a turbine when the head on the turbine is one meter?
cooled by cooling water: A. specific speed B. rated speed C. utilized speed D. unit speed
A. Rectifier B. Generator C. Evaporator
104. Absorber ANSWER: D
ANSWER: D
Cascade refrigeration cycle is often used in industrial process where objects Is a fluid property which refers to the intermolecular attraction by which the
must be cooled to temperature below: separate particles of the fluid arc held together?
A. -46OC B. -56OC C. -66OC D. -76OC A. Cohesion B. Adhesion C. Surface tension D.
ANSWER: A hypertension
Type of refrigerant control designed to maintain a pressure difference while the ANSWER: A
compressor is operating.
A. Thermostatic expansion valve B. Automatic expansion valve C. Using low side Which of the following is NOT the cause of black smoke in diesel engine?

float flooded system D. Capillary tube


A. fuel valve open too long B. high compression pressure C. carbon in exhaust B.5 20°C
pipe D. overload on engine C.6 20°C
D. 020°C
Which of the following is not a method of starting a diesel engine? ANSWER C
A. Manual rope, crank and kick B.Electric (battery) C. Compressed air D. Using The most common gases employed in Stirling and Ericsson cycles are:
another generator A. Air and helium B. Oxygen and helium C. Hydrogen and helium D. Nitrogen and
ANSWER D helium
Two-stroke engine performs to complete one cycle
ANSWER C
A. Suction and discharge stroke B. Power and exhaust stroke C. Compression and power In most common designs of Gas turbine, the pressure ratio ranges from
stroke A. 10 to 12
D. Suction and exhaust stroke B. 11 to 16
ANSWER C C. 12 to 18
A type of geothermal plant used when there is a presence of brine extracted from underground A.
D. 15 to 20
Dry geothermal plant B. Double-flesh geothermal plant C.Single-flash geothermal
ANSWER B
plant In Brayton cycle, the heat is transformed during what process?
D. Binary geothermal plant
ANSWER D A. constant temperature
Is the most important safety device on the power boiler B. sentropic process
107. Check valve C .isobaric process
108. Gate valve D.isochoric process
109. Safety valve ANSWER C
110. Globe valve The fuel injection process in diesel engine starts when piston
ANSWER C 118. Is at the TDC
During hydrostatic test, the safety valves should be 119. Leaving TDC
109. Removed 120. Approaches TDC
110. Open 121. Halfway of the stroke
111. Closed ANSWER C
112. Partially closed If the cut-off ratio of diesel cycle increases, the cycle efficiency will
ANSWER A 120. Decrease
Where deaerating heaters are not employed, it is recommended that the temperature of 121. Increase
the feed less than 122. Remains the same
114. 197°C 123. None of these
115. 102°C ANSWER A
116. 104°C The fuel used in a power plant that is used during peak periods
117. 106°C 123. Gas
ANSWER A 124. Solid
Is a reaction during which chemical energy is released in the form of heat? 125. Liquid
A. Cosmic reaction. B.Ethnic reaction C.Endothermic reaction D.Exothermic 126. None of these
reaction ANSWER C
ANSWER D Typical compression ratio of Otto cycle is
By reheating the steam in an ideal Rankine cycle the heat rejected will: 128. 6
A. Increase B. Decrease C. Remains the same D. None of these above 129. 8
ANSWER A 130. 10
By increasing the boiler pressure in Rankine cycle the moisture content at boiler exit will: 131. 12
116. Increase ANSWER B
117. Decrease If Joule Thompson coefficient is equal to zero, then the process will become
118. Remains the same 130. Isentropic
119. None of these above 131. Isenthalpic
ANSWER A 132. Isobaric
Presently the highest steam temperature allowed at the turbine inlet is 133. Isothermal
about A. 40°C ANSWER D
If the fluid passed through a nozzle its entropy will: 155. Hallway of aggress
134. Increase 156. Pathway of aggress
135. Decrease ANSWER B
136. Remains the same Any device or portion of the equipment used to increase refrigerant pressure
137. None of these 156. Pressure relief device
ANSWER C 157. Pressure imposing element
Refrigerants consisting of mixtures of two or more different chemical compounds, often 158. Pressure lift device
used individually as refrigerants for other applications 159. Pressure limiting device
136. Suspension ANSWER B
137. Compound reaction The quantity of refrigerant stored at some point is the refrigeration system for
138. Blends operational, service, or standby purposes
139. Mixing of refrigerants 160. Pressure vessel
ANSWER C 161. Pumpdown charge
Pairs of mating stop valves that allow sections of a system to be joined before opening these 162. Liquid receiver
valves or separated after closing them 163. Accumulator
138. Check valve ANSWER B
139. Gate valve Secondary refrigerant is a liquid used for the transmission of heat, without a change of state,
140. Safety valve and having no flash point or a fl ash poi nt above as determined from ASTM
141. Companion valve A. 150°F B. 160°F C. 180°F D. 200°F
ANSWER D ANSWER A
An enclosed passageway that limits travel to a single path A service valve for dual pressure-relief devices that allows using one device while
140. Corridor isolating the other from the system maintaining one valve in operation at all times
141. Hallway 164. Three-way valve
142. Lobby 165. Two-way valve
143. Tunnel 166. One-way valve
ANSWER A 167. Four-way valve
For Immediate Dangerous to Life or Health (IDHL) the maximum concentration from which ANSWER A
unprotected persons have time to escape within without escape-impairing symptoms or Tubing that is unenclosed and therefore exposed to crhsing, abrasion, puncture, or
irreversible health similar damage after installation
143. 16 minutes 166. Protected tubing
144. 1 minute 167. Bare tubing
145. 20 minutes 168. Open tubing
146. 30 minutes 169. Unprotected tubing
ANSWER D ANSWER D
The volume as determined from internal dimensions of the container with no allowance for Refers to blends compromising multiple components of different volatile that, when used
the volume of internal parts in refrigeration cycles, change volumetric composition and saturation temperature as they
147. Internal allowance volume evaporate (boil) or condense at constant pressure
148. Internal gross volume 169. Zeoline
149. Internal interference volume 170. Blending
150. Internal fots volume 171. Composition
ANSWER B 172. Zertropic
A waiting room or large hallway serving as a waiting room ANSWER D
150. Terrace Is a premises or that portion of a premise from which, because they are disabled,
151. Test room debilitated, or confined, cooperate cannot readily leave without the assistance of others
152. Compound room 174. Institutional occupancy
153. Lobby 175. Public assembly occupancy
ANSWER D 176. Residential occupancy
A continuous and unobstructed path of travel from any in a building or structure to a 177. Commercial occupancy
public way ANSWER A
152. Average of aggress Is one in which a secondary coolant is in direct contact with the air or other substance to
153. Mean o aggress be cooled or heated
180. Double indirect open spray system 202. Liquid suction heat exchanger
181. Indirect closed system ANSWER C
182. Indirect open spray system A type of valve connected from discharge of compressore directly to suction that is
183. Indirect vented closed system normally closed and will open automatically only if there is high discharge pressure
ANSWER B 210. Check valve
Refrigerant number R-744 is: 211. Solenoid valve
181. Butane 212. King valve
182. Carbon monoxide 213. Relief valve
183. Propane ANSWER B
184. Carbon dioxide Use to increase the capacity of condenser
ANSWER D 213. Water regulating valve
Refrigerant number R-1150 is: 214. Desuperheating coils
183. Propylene 215. Liquid-suction heat exchanger
184. Ethane 216. Condenser heating coils
185. Ethane ANSWER B
186. Methyl formale Is use to subcooled the refrigerant from the condenser
ANSWER B 215. Liquid subcooler
Refrigerant number R-40 is: 216. Condenser subcooler
186. Chlorodifluoromethane 217. Desuperheating coils
187. Difluoromethane 218. Liquid receiver
188. Ammonia ANSWER A
189. Chloromethane Which of the following is NOT a part of low pressure side in refrigeration system?
ANSWER D 218. Liquid line
When the air duct system serves several enclosed spaces, the permissible quantity of refrigerant in 219. Refrigerant flow control
the system shall not exceed the amount determined by using the total volume of 220. Evaporator
those spaces in which the airflow cannot be reduced to less than of its maximum when 221. Suction line
the fan is operating ANSWER A
189. One-quarter Which of the following is NOT a part of high pressure side in refrigeration system?
190. One half-quarter 220. Compressor
191. Three-quarter 221. Condenser
192. One fourth-quarter 222. Liquid line
ANSWER A 223. Suction line
The space above a suspended ceiling shall not be include 1 in calculating the ANSWER D
permissible quantity of refrigerant in the system unless such space is continuous and is Which of the following is NOT a part of condensing unit?
part of the air return system 223. Compressor
192. Partition 224. Discharge line
193. Plenum 225. Condenser
194. Separator 226. Liquid line
195. Plate divider ANSWER D
ANSWER B By subcooling the refrigerant in refrigerant system, the compressor power per unit mass will
Which of the following is NOT a possible location of service valve? 226. Increase
193. Suction of compressor 227. Decrease
194. Discharge of compressor 228. Remains the same
195. Outlet of liquid receiver 229. None of these
196. Outlet of condenser ANSWER C
ANSWER D Superheating the refrigerant in refrigeration system without useful cooling, the
A cool to serve with evaporator that is use to prevent the liquid refrigerant entering refrigeration effect per unit mass will
the compressore 228. Increase
200. Accumulator 229. Decrease
201. Liquid superheater 230. Remains the same
202. Drier loop 231. None of these
ANSWER C 261. Discharge line
Which of the following is NOT a type of air-cooled condenser? 262. Liquid line
229. Shell and tube 263. Between condenser and liquid receiver
230. Natural draft 264. Suction line
231. Forced draft ANSWER B
232. Induced draft Use to detects a vibration in current caused by the ionization of decomposed
ANSWER A refrigerant between two opposite-charged platinum electrodes
A type of refrigerant control typically used in household refrigeration 263. Electronic detector
231. Thermostatic expansion valve 264. Halide torch
232. Automatic expansion valve 265. Bubble method
233. Capillary tube 266. Pressurizing
234. High side float ANSWER B
ANSWER C Joints and all refrigerants containing parts of a refrigerating system located in an air duct
Type of condenser that operates like cooling tower carrying conditioned air to and from an occupied space shall be constructed to withstand a
234. Air-cooled condenser temperature of without leakage into the airstream
235. Evaporative condenser 265. 550°F
236. Shell and tube condenser 266. 600°F
237. Water-cooled condenser 267. 650°F
ANSWER B 268. 700°F
The major problem of heat pump is ANSWER D
236. Refrigerant used Refrigerant piping crossing an open space that affords passageway in any building shall be
237. Outside air less than_________ above the floor unless the piping is located against the ceiling of such
238. Supply air space and is permitted by the authority having jurisdiction
239. Frosting 269. 2.2 m
ANSWER D 270. 3.2 m
Dominant refrigerant used in commercial refrigeration system 271. 4.2 m
240. R-11 272. 5.2 m
241. R-22 ANSWER A
242. R-12 Methyl chloride shall not be in contact with
243. R-502 272. Aluminum
ANSWER D 273. Zinc
Cascade refrigeration system are connected in 274. Magnesium
245. Series 275. All of these
246. Parallel ANSWER D
247. Series-parallel Shall not be in contact with any halogenated refrigerants
248. Parallel-series 273. Aluminum
ANSWER A 274. Zinc
Is use to heat up the solution partially before entering the generator in absorption 275. Magnesium
refrigeration system 276. All of these
250. Rectifier ANSWER C
251. Absorber Are suitable for use in ammonia system
252. Regenerator 277. Copper
253. Pump 278. Aluminum and its alloy
ANSWER C 279. Plastic
The COP of actual absorption refrigeration system is usually 280. Cast iron
252. Less than 1 ANSWER B
253. Less than 2 If a pressure-relief device is used to protect a pressure vessel having an inside dimension of 6
254. Less than 3 in or less, the ultimate strength of the pressure vessel so protected shall be sufficient to
255. Less than 4 withstand a pressure at least the design pressure
ANSWER A 278. 2 times
Sight glass is often located at: 279. 3 times
282. 4 times 304. Inside the tube
283. 5 times 305. Outside the tube
ANSWER B 306. Inside the shell
Seats and discs shall be limited in distortion, by pressure or other cause, to a set pressure 307. Outside the shell
change of not more than in a span of five years ANSWER A
287. 1%
288. 5% In use if extracted steam upon condensation gets subcooled
289. 10% 306. Trap
290. 50% 307. Deaerator
ANSWER B 308. Filter
Liquid receivers, if used, or parts of a system designed to receive the refrigerant charge during 309. Drain cooler
pumpdown charge. The liquid shall not occupy more than of the volume when
temperature of the refrigerant is 90°F ANSWER D
288. 80% Needs only single pump regardless of number of heaters
289. 85%
290. 90% 309. Open heater
291. 95% 310. Closed heater
311. Mono heater
ANSWER C 312. Regenerative heater
The discharge line (B4) shall be vented to the atmosphere through a fitted to its upper ANSWER B
extremity Is also known as deaerator
291. Nozzle 313. Open heater
292. Convergent-divergent nozzle 314. Closed heater
293. Pipe 315. Reheat heater
294. Diffuser 316. Regenerative heater
ANSWER D ANSWER A
Convert fossil fuels into shaft work Dissolve gases like makes water corrosive react with metal to form iron oxide
296. Nuclear power plant 315. O2 and N2
297. Gas turbine power plant 316. O2 and CO
298. Dendrothermal power plant 317. O2 and CO2
299. Thermal power plant 318. N2 and SO2
ANSWER D ANSWER C
Ultimate strength drops by 30% as steam temperature raises from for unalloyed steel A cycle typically used in paper mills, textile mills, chemical factories, sugar factories and
299. 300 to 400°C rice mills
300. 400 to 500°C
301. 600 to 700°C 317. Cogeneration cycle
302. 700 to 800 °C 318. Combined cycle
319. By-product cycle
ANSWER B 320. Cascading cycle
Recent practice limits steam temperature to ANSWER A
300. 438°C When process steam is basic need and power is byproduct, this cycle is known as
301. 538°C
302. 638°C 320. Cogeneration cycle
303. 738°C 321. Combined cycle
322. By-product cycle
ANSWER A 323. Cascading cycle
In a closed feed water heater, the feed water pass through ANSWER C
A type of turbine employed where steam continuously extracted for process heating ANSWER D
A. Back pressure turbine Which of the following is lowest grade of coal?
B. Gas turbine
A. Peat
C. Steam turbine
B. Lignite
D. Passout turbine C. Sub-bituminous
ANSWER D D. Bituminous
Which of the following is used for Binary cycle power generation for high temperature ANSWER B
application Which of the following helps in the ignition of coal?
A. Mercury A. Moisture
B. Sodium B. Ash
C. Potassium C. Fixed carbon
D. All of these D. Volatile matter
ANSWER D ANSWER D
critical temperature of mercury is is the ratio of fixed carbon and volatile matter
A. 1160°C A. Air-fuel ratio
B. 1260°C B. Fuel ratio
C. 1360°C C. Combustion ratio
D. 1460°C D. Carbon-volatile ratio
ANSWER D ANSWER B
Critical pressure of mercury is A suspension of a finely divide fluid in another
A. 100 Mpa A. Filtration
B. 108 Mpa B. Floatation
C. 128 Mpa C. Emulsion
D. 158 Mpa D. Separation
ANSWER B ANSWER C
Method used in converting heat directly to electricity by magnetism Contains 90% gasoline and 10% ethanol
A. Electronic induction A. Gasohol
B. Magnetodynamic B. Gasonol
C. Magnetohyrdodynamic C. Gasothanol
D. Thermoelectric D. Gasethanol
ANSWER C ANSWER A
Which of the following is NOT a material used for thermoelectric elements Process used commercially in coal liquefaction
A. Bismuth telluride A. Tropsch process
B. Lead telluride B. Fisher process
C. Zinc telluride C. Fisher-Tropsch process
D. Germanium D. Mitch-Tropsch process
ANSWER C ANSWER C
A type of coal formed after anthracite Is an organic matter produced by plants in both land and water
A. Lignite A. Bio-ethanol
B. Bituminous B. Biomass
C. pear C. Petroleum
D. graphite D. Biodegradable
ANSWER B ANSWER A
In thermal power plant, induced draft fans are located at the PH of water varies with
322. Exit of furnace 338. Pressure
323. Foot if the stack 339. Temperature
324. Above the stock 340. Density
325. Top of the stock 341. Volume
ANSWER B ANSWER B
In thermal power plant, forced draft fans are installed at the Ph value of is usually maintained for boiler water to minimized corrosion
324. Foot of the stack 340. 8.5
325. Top of the stack 341. 9.5
326. Exit of the preheater 342. 10.5
327. Inlet of the preheater 343. 11.5
ANSWER D ANSWER C
Known as drum less boiler
What type of turbine that has a degree of reaction of ?
329. La Mont boilers
330. Fire tube boiler 342. Impulsive turbine
343. Reaction turbine
331. Forced circulation boiler
332. Once-through boiler 344. Rarsons turbine
345. Deriaz turbine
ANSWER D
ANSWER C
Reduces the steam temperature by spraying low temperature water from boiler drum
The cooling water is made to fall in series of baffles to expose large surface area for steam
330. Reheater
fed from below to come in direct contact
331. Preheater
332. Desuperheater 347. Spray condenser
333. Superheater 348. Surface condenser
349. Jet condenser
ANSWER C
350. Barometric condenser
Carbon dioxide can be removed by:
ANSWER D
332. Deaeration
Show the variation of river flow (discharge) with time
333. Aeration
334. Evaporation 352. Hydrograph
335. Vaporization 353. Hyetograph
ANSWER B 354. Mass curve
355. Flow duration curve
Is often used to absorb silica from water
ANSWER A
334. Sorbent
Is an open channel erected on surface above the ground
335. Rectifier
336. Silica gel 354. Canal
337. Magnesium hydroxide 355. Tunnel
ANSWER D 356. Penstock
357. Flume
Presence of excess hydrogen ions makes the water
ANSWER D
336. Acidic
Type of turbine used up to 300 m head
337. Alkalinity
338. Base 356. Impulsive turbine
339. Hydroxicity 357. Francis turbine
360. Propeller turbine 378. End of combustion
361. Deriaz turbine 379. Middle of combustion
380. Beginning of combustion
ANSWER D
381. Beginning of interaction
A turbine that has a diagonal flow ANSWER A
363. Impulsive turbine In a compression ignition engine, the detonation occurs near the
364. Francis turbine 379. End of combustion
365. Propeller turbine 380. Middle of combustion
366. Deriaz turbine 381. Beginning of combustion
382. Beginning of interation
ANSWER D ANSWER C
Oil is optimized either by air blast or pressure jet at about Morse test is use to m easure the of multi-cylinder engine
A. Brake power
366. 60 bar B. Indicated power
367. 70 bar C. Friction power
368. 80 bar
D. Motor power
369. 90 bar
ANSWER B
ANSWER B Ignition delay can be minimized by adding to decrease engine knocking
A. Ethel ether
Type of solid injection that use single pump supplies fuel under high pressure to a fuel header
B. Ethyl chloride
367. Common rail injection C. Ethyl nitrate
368. Individual pump injection system D. Ethyl oxide
369. Distributor system ANSWER C
370. Single rail injection For the submerged plane surface, the point on the surface where the resultant force acts
ANSWER A is called the
384. Center of buoyancy
Water flow in diesel engine that is caused by density differential 385. Center of gravity
369. Thermosiphon cooling 386. Center of pressure
370. Thermostat cooling 387. Center of attraction
371. Pressurized water cooling ANSWER C
372. Evaporating cooling At any point in fluid at rest, the pressure is the same in all direction. This principle is
known as:
ANSWER A 386. Bernoulli Principle
Type of lubrication system in diesel engine in which oil from pump is carried to a 387. Archimedes Principle
388. Pascal’s Law
separate storage tank outside the engine cylinder and used for high capacity engine. 389. Torricelli’s Law
372. Mist lubrication system ANSWER C
373. Wet pump lubrication system The hot-wire manometer is used to measure
374. Splash system 388. Pressure in gases
375. Dry sump lubrication system 389. Pressure in liquids
390. Wind velocities at airports
ANSWER D 391. Gas velocities
Produces extreme pressure differentials and violent gas vibration ANSWER D
The pitot static tube measures
374. Vibration
393. The static pressure
375. Detonation
394. The gage pressure
376. Explosion
395. The total pressure
377. Knocking
396. The dynamic pressure
ANSWER B ANSWER A
In a spark ignition engine, the detonation occurs near the The terminal velocity of a small sphere setting in a viscous fluid varies as the
396. First power of its diameter
398. Inverse of fluid viscosity 421. Cross-compound turbine
399. Inverse square of the diameter 422. Back pressure turbine
400. Inverse of the diameter ANSWER D
ANSWER B A governor with 0% regulation is termed as:
Pressure drag results from 425. Isochronous governor
400. Skin friction 426. Synchronous governor
401. Deformation drag 427. Isenchronous governor
402. Breakdown of potential flow near the forward stagnation point 428. Isobarnous governor
403. Occurrence of wake ANSWER A
ANSWER A The speed of regulation for most turbine-generators is adjustable from
The pressure coefficient is the ratio of pressure forces to 429. 2 to 6%
405. Viscous forces 430. 4 to 8%
406. Inertia forces 431. 6 to 10 %
407. Gravity forces 432. 8 to 12%
408. Surface tension force ANSWER A
ANSWER B Poppet valves of steam turbine are used for extraction pressures of
Tranquil flow must always occur 431. 20 to 120 psig
408. Above the normal depth 432. 20 to 150 psig
409. Above the critical depth 433. 20 to 130 psig
410. Below the normal depth 434. 20 to 140 psig
411. Below the critical depth ANSWER B
ANSWER B When both bearings of steam engine are on one side of the connecting rod, the engine
Which of the following head loss coefficient among the following types of entrance? is referred to as
410. Bell mouth 435. Center-crank engine
411. Square edge 436. Side crank engine
412. Reentrant 437. Under crank engine
413. It depends 438. Standard crank engine
ANSWER C ANSWER B
What waste water treatment method involves of algae from stabilization pond effluents? When the valve in steam engine is in mid-position of its travel, it will cover the steam port
413. Sedimentation by an amount known as
414. Floatation 440. Steam lap
415. Filtration 441. Partial lap
416. Microscreening 442. Full lap
ANSWER D 443. Angular lap
The number of nozzles will depend on the quantity of steam required by the turbine. If ANSWER A
nozzle occupy the entire arc of the ring, the turbine is said to have A type of governor in steam engine that do not control the actual admission of steam to
415. Partially full peripheral admission the cylinder but controls the pressure of the steam
416. One-half full peripheral admission 443. Flyball governor
417. Maximum peripheral admission 444. Variable cut-off governor
418. Full peripheral admission 445. Throttling governor
ANSWER D 446. Shaft governor
Tandem compound units may also have two low pressure castings that produces ANSWER C
417. Single flow By inter-cooling using two stage compressor of brayton cycle, the backward ratio will
418. Double flow 444. Increase
419. Triple flow 445. Decrease
420. Quadruple flow 446. Remains the same
ANSWER C 447. None of these
A type of turbine used for driving pumps, fans, and other auxiliaries in power plant ANSWER B
commonly operate at exhaust pressure approximating atmospheric On dynamic similitude, the relation which represents the ratio of inertia force to pressure
418. Tandem compound turbine force is
419. Passout turbine 447. Froude number
450. Cauchy number 476. Thermo-hydrograph
451. Euler number ANSWER C
452. Raynolds number Entrance losses between tanks and pipe or losses through elbows, fittings and valves
ANSWER C are generally expresses as a function of
What is the maximum velocity in a sewer flowing full? 478. Kinetic energy
453. 0.6 m/sec 479. Pipe diameter
454. 0.9 m/sec 480. Friction factor
455. 1.2 m/sec 481. Volume flow rate
456. 1.8 m/sec ANSWER A
ANSWER A The air that contains no water vapor is called
A temporary structure constructed to exclude water from the site of the foundation during 482. Zero air
its excavation and construction is called: 483. Saturated air
456. Caisson 484. Dry air
457. Retaining wall 485. Humid air
458. Coffer dam ANSWER C
459. Earth dam In psychometric chart, the constant enthalpy lines coincide with constant temperature lines
ANSWER C at temperature
Which is not a physical characteristics of water? 485. Above 50°C
458. Total suspended and dissolved solids 486. Below 40°C
459. Turbidity 487. Below 50°C
460. Color 488. Above 10°C
461. Hardness ANSWER C
ANSWER D The amount of moisture in air depends on its
Which dam is best for weak foundation? 491. Pressure
460. Gravity 492. Volume
461. Arch 493. Temperature
462. Buttress 494. Humidity
463. Earth ANSWER C
ANSWER C The deep body temperature of healthy person is maintained constant at
What is the volume of water which will drain freely the aquifer? 493. 27°C
462. Specific yield 494. 37°C
463. Reservoir yield 495. 47°C
464. Safe yield 496. 48°C
465. Secondary yield ANSWER B
ANSWER A Air motion also plays important role in
What is the line defined by water level in a group of artesian walls? A. Surroundings
468. Water table B. Cooling
469. Peizometric surface C. Human comfort
470. Specific yield D. None of these
471. All of the above ANSWER C
ANSWER B During simple heating and cooling process has a humidity ratio
Select the one that is positive indication of pollution of a river A. Increasing
470. Acidity B. Decreasing
471. Oxygen content C. Constant
472. Chloride content D. None of these
473. Nitrite content ANSWER C
ANSWER C The follows a line of constant wet bulb temperature on the psychometric chart
Which instruments is used to measure humidity of the atmosphere continuously? 495. Evaporative cooling process
474. Barograph 496. Condensing process
475. Thermograph 497. Direct cooling process
476. Hydrograph 498. None of these
ANSWER A In the absence of any work interactions between a system and its surroundings, the amount of
A vapor which is not about to condense is called a net heat transfer is equal
499. Mixture of vapor and liquid To the change in total energy of a closed system
500. Critical vapor To heat and work
501. Superheated vapor Energy interactions
502. None of these None of these
ANSWER C ANSWER A
Passing from the solid phase directly into vapor phase is called The constant volume and constant pressure specific heats are identical for
501. Condensation Compressible substance
502. Fusion Incompressible substance
503. Sublimation Compressible gas
504. None of these None of these
ANSWER C ANSWER B
Robert Boyle observed during his experiments with a vacuum chamber that the pressure The velocity of fluid is zero at the wall and maximum at the center because of the
of gases is inversely proportional to their Velocity effect
504. Temperature Viscous effect
505. Pressure Temperature effect
506. Volume None of these
507. None of these ANSWER B
ANSWER C For steady flow devices, the volume of the control volume is
Is energy in transition Increase
507. Heat Decrease
508. Work Constant
509. Power None of these
510. None of these ANSWER C
ANSWER A The work done in a turbine is since it is done by the fluid
Is the mode of energy transfer between solid surface and the adjacent liquid or gas which is A. Positive
in motion, and it involves combine effects of conduction and fluid motion B. Negative
510. Conduction C. Zero
511. Convection D. None of these
512. Radiation ANSWER A
513. None of these Reheating process in Brayton cycle, the turbine work will
ANSWER B Increase
Radiation is usually considered as Decrease
513. Surface phenomenon Remains the same
514. Surface interaction None of these
515. Surface corrosion ANSWER A
516. None of these Which of the following is the chemical formula of ethanol
ANSWER A C7H16
Work is between the system and the surroundings C2H6O
516. Work interaction C7H8
517. Energy interaction C6H12
518. Heat interaction ANSWER B
519. None of these Which of the following is the chemical formula of Heptane?
ANSWER B C7H16
Is a process during which the system remains in equilibrium at all times C2H6O
Quasi-equilibrium C7H8
Static equilibrium C6H12
Dynamic equilibrium ANSWER D
None of these Which of the following is the chemical formula of Toluene?
ANSWER A C7H16
C2H6O Work
C7H8 Heat added
C6H12 Heat rejected
ANSWER C ANSWER D
As the air passes through a nozzle, which of the following will increase? By decreasing the temperature source of Carnot cycle, which of the following will not
Temperature be affected?
Enthalpy Efficiency
Internal energy Work
Mach number Heat added
ANSWER D Heat rejected
As the air passes through a diffuser, which of the following will decrease? ANSWER C
Temperature By superheating the refrigerant in vapor compression cycle with useful cooling, which of
Enthalpy the following will increase? (Use per unit mass analysis)
Internal energy Condenser pressure
Mach number Evaporator pressure
ANSWER D Quality after expansion
As the air passes through a nozzle, which of the following will decrease? Heat rejected from condenser
Entropy ANSWER D
Velocity By superheating the refrigerant in vapor compression cycle with useful cooling, which of the
Internal energy following will decrease? (Use per unit mass analysis)
Mach number A. Refrigerant effect B. COP C. Compressor power D. Mass flow rate
ANSWER C ANSWER D
As the air passes through a diffuser, which of the following will increase? By superheating the refrigerant in vapor compression cycle without useful cooling, which of
Density the following will decrease? (Use per unit mass analysis)
Entropy A. Heat rejected B. COP C. Compressor power D. Specific volume at
Mach number suction
Velocity ANSWER B
ANSWER A By superheating the refrigerant in vapor compression cycle without useful cooling, which
As the air passes through a diffuser, which of the following will NOT be affected? of the following will increase? (Use per unit mass analysis)
ANSWER B Refrigerant effect
After passing through a convergent-divergent nozzle, the temperature of air will: COP
Increase Compressor power
Decrease Mass flow rate
Remains the same ANSWER C
None of these By superheating the refrigerant in vapor compression cycle without useful cooling, which
ANSWER B of the following will not be affected? (Use per unit mass analysis)
After passing though a convergent-divergent nozzle, the density of air will: Refrigerant effect
Increase COP
Decrease Compressor power
Remains the same Mass flow rate
None of these ANSWER C
ANSWER B By sub-cooling the refrigerant in vapor compression cycle at condenser exit, which of
After passing through a convergent-divergent nozzle, the match number of air will: the following will increase? (Use per unit mass analysis)
Increase Refrigerating effect
Decrease Specific volume at suction
Remains the same Compressor power
None of these Mass flow rate
ANSWER B ANSWER A
By increasing the temperature source of Carnot cycle, which of the following will not By sub-cooling the refrigerant in vapor compression cycle at condenser exit, which of
be affected? the following will decrease (Use per unit mass analysis)
Efficiency
A. Coefficient of performance B. Heat rejected from condenser ANSWER: C
C. Refrigerating effect D. Mass flow rate By increasing the boiler pressure in Rankine cycle, which of the following will decrease?
ANSWER: D (Use per unit analysis)
By increasing the vaporizing temperature in vapor compression cycle, which the A. heat rejected B. pump work C. cycle efficiency D. moisture
following will increase? (Use per unit mass analysis) content
A. mass flow rate B. COP C. specific volume D. compressor ANSWER: A
work By superheating the steam to a higher temperature in Rankine cycle, which of the
ANSWER: B following will decrease? (Use per unit analysis)
By increasing the vaporizing temperature in vapor compression cycle, which the A. moisture content at the turbine exhaust B. turbine work C. heat added D. heat
following will decrease? (Use per unit mass analysis) rejected
A. Refrigerating effect B. COP C. evaporator temperature D. temperature ANSWER: A
difference between evaporator and compressor ANSWER: D By superheating the steam to a higher temperature in Rankine cycle, which of the
following will increase? (Use per unit analysis)
By increasing the condenser pressure in vapor compression cycle, which of the following A. moisture content at the turbine exhaust B. pump work C. condenser
will increase? (Use per unit mass analysis) pressure D. cycle efficiency
A. Refrigerating effect B. COP C. Specific volume at suction ANSWER: D, B
D. Compressor power By reheating the steam before entering the second stage in Rankine cycle, which of the
ANSWER: D following will decrease?
By increasing the condenser pressure in vapor compression cycle, which of the following A. Turbine work B. moisture content after expansion C. heat
will decrease? (Use per unit mass analysis) added D. heat rejected
A. moisture content after expansion B. compressor power C. heat ANSWER: B
rejected in the condenser D. mass flow rate What Rankine cycle is modified with regeneration, which of the following will increase?
ANSWER: A A. turbine work B. heat added C. heat rejected D. cycle
If the pressure drop in the condenser increases in a vapor compression cycle, which of efficiency
the following will increase? (Use per unit mass analysis) ANSWER:D
A. mass flow rate B. compressor power C. heat rejected in the condenser D. Is the combination of base load and peaking load?
specific volume at suction A. rated load B. intermediate load C. combine load D.
ANSWER: C over-all load
If the pressure drop in the condenser increases in a vapor compression cycle, which of ANSWER: B
the following will decrease? (Use per unit mass analysis) Sum of the maximum demand over the simultaneous maximum demand?
A. Refrigerating effect B. mass flow rate C. heat rejected in the condenser D. A. use factor B. capacity factor C. demand factor D. diversity
Compressor power factor
ANSWER: B ANSWER: D
If the pressure drop in the condenser increases in a vapor compression cycle, which of Regenerative with feed heating cycle with infinite number of feed water heaters thus
the following will not be affected? (Use per unit mass analysis) efficiency is equal to?
A. compressor power B. mass flow rate C. heat rejected in the A. otto cycle B. stirling cycle C. erricson cycle D. carnot
condenser D. COP cycle
ANSWER: A ANSWER: D
If the pressure drop in the evaporator increases in a vapor compression cycle, which of A type of turbine used in desalination of sea water.
the following will increase? (Use per unit mass analysis) A. back pressure turbine B. passout turbine C. peaking turbine D.
A. Refrigerating effect B. vaporizing temperature reaction turbine
C. heat rejected in the condenser D. COP ANSWER: A
ANSWER: C State that when conductor and magnetic field move relatively to each other, an electric
If the pressure drop in the evaporator increases in a vapor compression cycle, which of the voltage is induced in the conductor.
following will decrease? (Use per unit mass analysis) A. Maxwell’s law B. Kirchhoff’s law C. Faraday’s law D.
A. specific volume at suction B. compressor power C. heat rejected in Newton’s law
the condenser D. COP ANSWER: C
ANSWER: D Transfers heat directly to electrical energy by utilizing thermionic emissions.
By lowering the condenser pressure in Rankine cycle, which of the following will decrease? A. thermionic motor B. thermionic generator C. thermionic converter D. thermionic
(Use per unit analysis) cell
A. Pump work B. turbine work C. heat rejected D. cycle ANSWER: B
efficiency
Is the largest group of coal containing 46-89% of fixed carbon and 20% to 40% A. radiant superheater B. desuperheater C. convective superheater
volatile matter. D. pendant superheater
A. anthracite B. sub-anthracite C. bituminous D. sub- ANSWER: D
bituminous Regenerative superheater is a storage type of heat exchangers have an energy storage
ANSWER: C medium called:
When 1 gram of coal is subjected to a temperature of about 105C for a period of 1 hour, the A. matrix B. regenerator C. boiler D. recuperator
loss in weight of the sample gives the: ANSWER: A
A. Volatile matter B. ash C. fixed carbon D. moisture content Stirling cycle uses a ____ as working fluids.
ANSWER: D A. incompressible gas B. incompressible fluids C. compressible
When 1 gram of sample of coal is placed in a crucible and heated 950C and maintain at refrigerant D. compressible fluids
the temperature for 7 minutes there is a loss in weight due to elimination of: ANSWER: D
A. volatile matter and moisture B. ash C. fixed carbon D. moisture content In stirling process the heat added is added during?
A. isobaric process B. isentropic process C. isothermal process
ANSWER: A D. heat process
Consist of hydrogen and certain hydrogen carbon compounds which can be removed ANSWER: C
from coal by heating. Brayton cycle is also known as:
A. moisture content B. product of combustion C. ash D. volatile A. carnot cycle B. joule cycle C. diesel cycle D. rankine
matter cycle
ANSWER: D ANSWER: B
By heating 1 gram of coal in an uncovered crucible until the coal is completely burned, the Is applied to propulsion of vehicles because of certain practical characteristics.
____ will formed. A. diesel cycle B. otto cycle C. carnot cycle D. brayton
A. volatile matter and moisture B. ash C. fixed carbon D. moisture cycle
content ANSWER: D
ANSWER: B
Caking coal are use to produce coke by heating in a coke oven in the absence of ____ with Heat exchangers typically involve
volatile matter driven off. A. no work interactions B. no heat interactions C. no energy
A. air B. oil C. oxygen D. nitrogen interactions D. none of these
ANSWER: A ANSWER: A
Gindability of standard coal is: A device that is used to convert the heat to work is called
A. 80 B. 90 C. 100 D. 110 A. adiabatier B. regenerator C. heat engines D. none of
ANSWER: C these
ANSWER: C
Major constituent of all natural gases is: The objective of a heat pump is to maintain a heated space at
A. ethane B. methane C. propane D. cethane A. Low temperature B. high temperature C. medium
ANSWER: B temperature D. none of these
Two types of fans are: ANSWER: B
A. centrifugal and axial B. reciprocating and axial C. centrifugal and A device that violates the second law of thermodynamics is called
rotary D. tangential and rotary A. perpetual motion machine of second kind B. perpetual motion machine of third
ANSWER: A kind C. perpetual motion machine of first kind D. none of these
Enthalpy of substance at specified state due to chemical composition. ANSWER: A
A. Enthalpy of reaction B. enthalpy of combustion C. enthalpy of formation A process is called ____ if no irreversibility’s occur outside the system boundaries during the
D. enthalpy of product process.
ANSWER: C A. externally reversible B. internally reversible C. reversible D.
A type of boiler used for super critical pressure operation. none of these
A. La Mont boiler B. Once- through-circulation boiler C. Force ANSWER: A
circulation boiler D. Natural circulation boiler An energy interaction which is not accompanied by entropy transfer is
ANSWER: B A. energy B. heat C. work D. none of these
Economizer in a water tube boiler is heated by: ANSWER: C
A. electric furnace B. electric current C. incoming flue gas D. outgoing A ____ is used in aircraft engines and some automotive engine. In this method, a turbine
flue gas driven by the exhaust gases is used to provide power to compressor or blower at the inlet.
ANSWER: D A. discharging B. turbo charging C. supercharging D. scavenging
Receives heat partly by convection and partly by radiation. ANSWER: B
The only devices where the changes in kinetic energy are significant are the Superheating the steam to higher temperatures decreases the moisture content of the steam at
A. compressor B. pumps C. nozzles and diffusers D. none of the ____.
these A. turbine inlet B. compressor inlet C. compressor exit
ANSWER: C D. turbine exit
The distance between TDC and BDC in which the piston can travel is the ANSWER: D
A. Right extreme position B. displacement stroke C. stroke of Regeneration also provides a convenient of dearating the feedwater to prevent?
the engine D. swept stroke A. boiler explosion B. boiler scale production C. boiler corrosion D. compressor
ANSWER: C damage
In compression-engine, the combustion of air-fuel mixture is self-fuel ignited as a result of ANSWER: C
compressing the mixture above its Can be apply steam turbine cycle(rankine). Gas turbine cycle(brayton) and combined cycle.
A. self developed temperature B. mixing temperature C. self feed A. hydroelectric plant B. nuclear power plant C. cogeneration plant D. tidal power
temperature D. self ignition temperature plant
ANSWER: D
ANSWER: C
The thermal efficiency of an ideal Otto cycle depends ____ of the working fluid. In a rankine cycle with fixed turbine inlet conditions. What is the effect of lowering
the pressure ratio of the engine and the specific heat ratio the condenser pressure, the heat rejected will:
the temperature ratio of the engine and the specific heat ratio A. increase B. decrease C. remains the same D. none of
the moles ratio of the engine and the specific heat ratio these
the compression ratio of the engine and the specific heat ratio ANSWER: B
ANSWER: D In an ideal rankine cycle with fixed boiler and condenser pressure. What id the effect of
As the number of stages is increased, the expansion process becomes superheating the steam to a higher temperature, the pump work input will.
A. isentropic B. isothermal C. isometric D. polytropic A. increase B. decrease C. remain the same D. none of
ANSWER: B these
ANSWER: C
Aircraft gas turbines operate at higher pressure ratio typically between The fact that total energy in any one energy system remains constant is called the principle of
A. 6 to 8 B 12 to 24 C. 10 to 18 D. 10 to 25 ____?
ANSWER: D A. conversion of energy B. second law of thermodynamics C. conservation of
The first commercial high-pass ratio engines has a bypass ratio of mass D. zeroth law of thermodynamics
A. 1 B. 3 C. 5 D. 7 ANSWER: A
ANSWER: C A process for which the inlet and outlet enthalphies are the same
The single-stage expansion process of an ideal Brayton cycle without regeneration is replaced A. isntropic B. enthalphy conservation C. throttling D. steady state
by a multistage expansion process with reheating the samepressure limits. As a result of ANSWER: C
modification, thermal efficiency will:
A. increase B. decrease C. remain constant D. none of The sum of energies of all the molecules in system, energies that in several complex forms.
these A. kinetic energy B. internal energy C. external energy D. flow work
ANSWER: B ANSWER: B
Which of the following is/are the application of Brayton cycle. A system that is completely impervious to its surrounding. Neither mass nor energy cross
A. propulsion system B. automotive turbine engine C. aircraft turbine its boundaries.
engineD. all of these A. Open system B. closed system C. adiabatic system D.
ANSWER: D isolated system
It is used as working fluid in high-temperature applications of vapor cycles. ANSWER: D
A. helium B. deuterium C. mercury D. water A device used to measure small and moderate pressure difference.
ANSWER: C A. manometer B. bourdon gage C. barometer D. piezometer
The superheated vapor enters the turbine and expands isentropically and produces work ANSWER: A
by the rotating shaft. The ____ may drop during the process. A vapor having a temperature higher that the saturation temperature corresponding to its
A. density B. viscosity of fuel C. temperature and pressure pressure.
D. none of these A. superheated pressure B. saturated vapor C. super saturated vapor
ANSWER: C D. subcooled vapor
Only ____ of the turbine work output is required to operate the pump. ANSWER: A
A. 0.01% B. 0.02% C. 0.03% D. 0.04% The energy or stored capacity for performing work possessed by a moving body, by virtue of
ANSWER: D its momentum.
A. internal energy B. work C. gravitational potential energy
D. kinetic energy
ANSWER: D
A thermodynamic process wherein temperature is constant and the change in internal energy PIPE ELEMENTS
is zero.
A. isobaric process B. isometric process C. isothermal process D. polytropic Which of the following compressors are so widely used for today’s refrigeration
process system? Ans. Centrifugal
The science terminology concerned with precisely measuring energy and enthalpy A valve sometimes known as the magnetic valve.
A. thermodynamics B. chemistry C. calorimetry D. none of these Ans. Solenoid valve
ANSWER: C Which of the following valve use to regulate the flow of refrigerant to the
The rate of doing work per unit time evaporator? Ans. Expansion valve
A. torque B. power C. force D. moment Which of the following use control the flow of refrigerant gas from the evaporator coil.
ANSWER: B This type of control valve is also known as back pressure regulator of an evaporator.
A vapor having a temperature higher than the saturation temperature corresponding to the Ans. Suction line regulator
existing pressure. Which of the following refrigerant control is used to limit the flow of gas to the compressor
A. superheated vapor B. saturated vapor C. wet vapor D. none of the above to prevent surge of excessive load from overloading the compressor.
ANSWER: A Ans. Hold-back valve
It is the work done in pushing a fluid across a boundary, usually or out of a system. Which of the following refrigerant control is used in the liquid suction or discharge to
A. mechanical work B. nonflow work C. flow work D. electrical work interrupt the flow on demand from any one of the several types of temperature or pressure
ANSWER: C sensing devices.
A liquid that has a temperature lower that the saturation temperature corresponding to the Ans. Solenoid valve
existing pressure. What is the pressure drop from the receiver to the expansion valve?
A. subcooled liquid B. saturated liquid C. unsaturated liquid D. water Ans. 5 psi
ANSWER: A If any of the electricity controlled device in a Freon system malfunction, which
In this type of boiler, the water passes through the tubes while the flue gases burn outside the following valves will also automatically shut off?
tubes. Ans. Solenoid valve
A. water-tube boiler B. fire-tube boiler C. steam generator D. electric The relief valve on a CO2 machine is located:
boiler Ans. On the discharge pipe between the compressor and the discharge valve.
ANSWER: A When checking zinc plates in a condenser, one should:
It shows the water level in the boiler drum. Ans. Clean the plate and renew worn-out ones.
A. water column B. try cocks C. gauge glass D. all of the above A refrigerant should have a
ANSWER: C Ans. High latent heat
It prevents damage to the boiler by giving warning of low water. The mechanical energy of a device is the ratio of:
A. safety valve B. fusible plug C. relief valve D. try cocks Ans. Mechanical energy input to the mechanical energy output
ANSWER: B When the winding or circuit is open, what is the resistance reading?
it is heat exchanger which utilizes the heat of the flue gases to preheat air needed Ans. Infinity
for combustion. If the meter scale of the ohmmeter reads 0 ohms and the range adjustment is R 10, what is
A. economizer B. feedwater heater C. reheater D. air preheater the resistance reading?
ANSWER: D Ans. 100
It is the subject that deals with the behavior of moist air. What is the resistance reading of an electric motor in good condition?
A. psychrometer B. psychometry C. refrigeration D. pneumatics Ans. With resistance
ANSWER: B The basic unit of electrical pressure is:
Air whose condition is such that any decreases in temperature will result in condensation of Ans. Volt
water vapor into liquid. What is the most commonly used conductor?
A. saturated air B. unsaturated air C. saturated vapor D. moist air Ans. Copper
ANSWER: A Which of the following is the effect of superheating the refrigerant?
It is the warm water temperature minus the cold water temperature leaving the cooling tower Ans. Increase is COP
A. approach B. terminal difference C. cooling range D. Which of the following statement is true in ice making capacity?
LMTD Ans. Ice making capacity is always proportional to the refrigerating
ANSWER: C effect Which of the following has the largest heat load in cold storage room?
The surrounding air ____ temperature is the lowest temperature to which water could possibly Ans. Product heat load
cooled in a cooling tower. Which of the following is a halocarbon refrigerant?
A. dry-bulb B. wet-bulb C. dew-point D. saturated temperature Ans. Methyl chloride
ANSWER: B A refrigerant system in which pressure- imposing element is mechanically operated?
Ans. Compressor Ans. Blue
Which of the following compressors have the compressing element and drive seals in a Where is the compound gauge installed in a refrigerant system?
single housing? Ans. Suction line
Ans. hermitically sealed compressor A gauge used to measure pressure both below and above atmospheric
Which of the following consists of two mating helically grooved male and female pressure Ans. Compound gauge
grooves? Ans. Helical rotary-screw What is the valve position if the valve stem is turned all the way in?
What is the another name of helical-rotary screw compressors? Ans. Front-seated
Ans. Lysholm type What is the valve position if the valve stem is turned all the way out?
At what head is the helically-rotary compressor are designed to operate? Ans. Back-seated
Ans. High head pressure Ohm’s law states that resistance is inversely proportional to:
The bulk quantity of oil separated from the refrigerant in the helically-rotary compressor Ans. Current
falls by? To prevent corrosion within the flue gas, what must be prevented from condensing?
Ans. Gravity to a sump Ans. Vapor
If the temperature in the icebox is too high, the trouble could be: Ans. Evaporative cooling
Ans. Automatic control not functioning properly An adiabatic saturation process. This process can be produced with _____, which is
If any of the electrically controlled devices in a Freon system malfunction, which of essentially a regular thermometer with its bulb wrapped in wet cotton or gauze.....
the following valves also automatically shut-off? Ans. Sling psychrometer
Ans. Solenoid valve A high velocity gas is defined as a gas moving with a velocity in excess off
Natural ice in an ice box was used in the early approximately Ans. 300ft/s/ 100m/s
Ans. 19th century The theoretical maximum velocity is achieved when.....
The amount of matter present in a quantity of any substance is called: Ans. All internal and pressure energies are converted to kinetic energies
Ans. Mass If the gas flow is adiabatic and frictionless (i.e.
What pressure is indicated by a barometer? reversible) Ans. Both A and B
Ans. Atmospheric pressure A property by which the refrigerant remains at its original chemical form/original condition
The basic unit measurement for heat is: is imposed by the operation.....
Ans. BTU Ans. Stability of refrigerant
The force that acts upon a unit of area is termed: The maximum temperature at which a gas is condense into liquid. Above temperature, a
Ans. Pressure vapor imposed by the operation.....
What is the physical state of a refrigerant entering the flow control? Ans. Stability of refrigerant
Ans. Sub-cooled Which of the following is the name given for halogenated hydrocarbon?
What method of heat transfer does not depend on molecular motion? Ans. Halocarbon
Ans. Radiation Which of the following refrigerants are produced by carrier
Where s the filter drier is located? corporation? Ans. Carrene
Ans. After the condenser Which of the following refrigerants is produced by E.L.DU
A motor drive device which removes the heat-laden vapor refrigerant from the evaporator FONT? Ans. Freon
is the: #Which of the following is the other name of R 718?
Ans. Compressor Ans. Water
The function of the compressor is to: Energy per unit volume stored in a deformed material is
Ans. Squeeze out refrigerant from the evaporator called: Ans. Strain energy
In which part of the compressor is the piston attached to the crankshaft? The ratio of ultimate failure strain to yielding strain is known
Ans. Connecting rod as: Ductility
This is a compressor which has an off center-rotor. One ton of refrigeration is equal to:
Ans. Rotary Ans. 3.5KW
Which of the following compressor is commonly used in household refrigerators? With what cycle does air refrigeration
Ans. Hermetic works? Ans. Bell-coleman cycle
What is the length of the tubing extended from the part to be What cycle does air refrigeration
swaged Ans. ½ inch works? Ans. Coal
The best method of making leak proof connection is: What is produced by fission process?
Ans. Silver brazing Ans. Radiation
The refrigerant that should not be used copper or brass tubing: # Loss of power is due to:
Ans. R-717 Ans. Low injection pressure
The color of the hose connected to suction line or process be # A branch system of pipes to carry waste emissions away from the combustion chamber.
Ans. Exhaust manifold Ans. Acoustic leak detector
The type of filter where the filtering elements is For foundation of stacks, the maximum pressure on the soil is equal to the pressure due to
replaceable. Ans. Metal-edge filter the weight and the _______.
When four events takes place in one revolution of a crankshaft of an engine, the engine is Ans. Wind movement
called Ans. Two stroke engine Foundation bolts of specified size should be used and surrounded by a pipe sleeve with an
Which of the following does not belong to the group. inside diameter of at least
Ans. Time injection system Ans. 3 times the diameter of anchor bolt
# The function of a super heater is to: For multi stage compression of an ideal Brayton cycle, the back work ratio will
Ans. Exchange heat to increase energy to the flow during an adiabatic, internally Ans. decrease
reversible process Type of turbine that has a specific speed below
What is true about change in 5. Ans. Impulse turbine
entropy? Ans. it is always zero A high discharge type of turbine
How does an adiabatic process compare to an isentropic Ans. Propeller turbine
process? Ans. Both heat transfer = Q, isentropic reversible Use to minimize the speed rise due to a sudden load
Adiabatic heat transfer within a vapour cycle refers to : rejection Ans. jet deflector
Ans. The transfer of energy from one stream to another in a heat exchanger that the energy of Is the speed of a turbine when the head on the turbine is one
the input streams equal to the energy of output. meter. Ans. Unit speed
All of the following process are irreversible except: Is a fluid property which refers to the intermolecular attraction by which the separate particles of
Ans. An isentropic compression of a perfect fluid the fluid arc held together.
Refers to the system where the evaporator coils are placed in the ice Ans. Cohesion
box Ans. Direct system Which of the following is NOT the cause of black smoke in diesel engine?
What is the usual dehydrating agent in Freon system? Ans. high compression pressure
Ans. Activated alumina Which of the following is not a method of starting a diesel
Refers to the system where the evaporator coils are indicated in a brine solution and brine is engine? Ans. Using another generator
pumped thru the ice box Two-stroke engine performs _____ to complete one cycle.
Ans. Indirect system Ans. compression and power stroke
Is the most common dryer which consist of rotating cylinder inside which the materials flow A type of geothermal plant used when there is a presence of brine extracted from
while getting in contact with hot gas. undergroungd Ans. Binary geothermal plant
Ans. Rotary dryer Is the most important safety device on the power boiler.
Is the ratio of the mass of water-vapor in air and mass of air if it is saturated is Ans. Safety valve
called: Ans. Relative humidity During hydrostatic test, the safety valves should be
The hands feel painfully cold when the skin temperature reaches Ans. removed
Ans. 10°C Where deaerating heaters are not employed, it is recommended that the temperature of
The total heat of air is a function the feed water be not less than _______
of Ans. WB temperature Ans. 197°C
Boiling point of Freon-12 at atmospheric pressure What is the possible cause of too much suction pressure?
is: Ans. 21°F Ans. Shortage refrigerant
Which of the following is NOT a type of water cooled condenser in Where is the oil separator located?
refrigeration? Ans. Double shell Ans. Between the compressor and condenser
Component of absorption refrigeration system in which the solution is cooled by cooling What is the purpose of the low pressure cutout switch? Ans.
water. Ans. Absorber To cut compressor in and out at present pressure
Cascade refrigeration cycle is often used in industrial process where objects must be cooled Freon unit will tend to short cycle when operating
to temperature below: under Ans. Light loads
Ans. -46°C What cause hot suction line?
Type of refrigerant control designed to maintain a pressure difference while the compressor Ans. Insufficient refrigerant
is operating. When changing a Freon system which valve is not
Ans. Capilliary tube used? Ans. King valve
As a rule of thumb, for a specified amount of compressed air, the power consumption of Where is the scale trap located?
the compressor decreased by ____________for each 3°C drop in the temperature inlet air to Ans. Between the king valve and expansion valve
the compressor. # Where is the solenoid valve located?
Ans. 1 percent Ans. Between the scale trap and thermal expansion valve
Modern way of detecting air compressor leak is by using # Water tube boilers have how many fusible plugs?
Ans. 4 The water vapour mass is often reported
What is the main cause of air pollution as a result of burning in Ans. Grains of water
fuel? Ans. Nitrogen dioxide Approximately how many grains of water are there in 1
Refers to the increase of enthalpy of a substance when it undergoes some phase change pound? Ans. 7000
at constant pressure and temperature. The ratio of actual humidity ratio to the saturated humidity ratio at the inlet temperature and p
Ans. Heat of vaporization
Which of the following keeps moisture from passing thru the system? Ressure is known as
Ans. Dehydrator Ans. Degree of saturation
What is the lowest temperature to which water could possibly be cooled in a cooling What is another term for “degree of
tower? Ans. Temperature of adiabatic saturation saturation”? Ans. Percentage of humidity
#What do you call the intake pipe to a hydraulic turbine from a dam? The partial pressure of the water vapour divided by the saturation
Ans. Penstock pressure? Ans. Relative humidity
# An ideal fluid is one that The point where sonic velocity has been achieved (M=1) is known
Ans. is frictionless and incompressible as: Ans. Critical point
What principal was used by McLeod gauge which is used for low pressure The ratio of the thrust in equal mass flow rate is known as...
measurement/ Ans. Boyles Law Ans. Effective exhaust velocity
#The difference between the brake and hydraulic power Is the velocity of gas dropped from supersonic to subsonic, gas will
is Ans. Friction power experience Ans. Shock wave.
Product of motor pump efficiency is known An adiabatic flow with friction, which would be approximately as flow through an insulated duct...
as Ans. Over-all efficiency Ans. Fan no flow
The most efficient cross-section of an open channel is the one that What is an inevitable by product of the combustion of hydro carbon fuel?
Ans. All of the above Ans. Water vapour
#Refers to the structure across an open channel over which water flows. A compound pressure gauge is used to measure:
Ans. Weir Ans. Positive and negative pressure
When can we say that a trapezoidal weir is a “cipoletti” Which of the following best describes a Curtis turbine?
weir? Ans. When side slope 4 vertical and 1 horizontal Ans. It is a velocity pressure compound turbine
A weir is broad-crested if: What causes low head pressure?
Ans. The weir length is greater than half of the head Ans. Too much cooling water and in sufficient refrigeration gas
A feature of a dam over where water is discharged is What causes high head pressure?
called: Ans. Spillway Ans. All of the above
Laminar flow occurs when the Reynolds number is approximately less A commercial pipe which has the characteristics of being immune to electrolysis and corrosion,
than Ans. 2100 light in weight weak structurally.
Turbulent flow occurs when the Reynolds number exceeds: Ans. Asbestos cement pipe
Ans. 4000 # A commercial pipe which is durable, water tight, low maintenance and smooth interior.
Laminar flow is typical when Ans. Concrete pipe
Ans. All of the above A commercial pipe which has the characteristics of being corrosion, and scour and
When the Reynolds number is between 2100 to 4000, the flow is said to be erosion Resistance.
in Ans. Critical zone Ans. Vitrified clay pipe
Which of the following characterizes a laminar flow? A commercial pipe which that is ductile, high strength, shock resistant, very smooth
Ans. All of the above internally. Ans. Steel pipe
Without a pump and turbine, which of the following rules is true in a frictionless environment a
about energy and hydraulic gradient? A commercial pipe that is chemically insert, resistant to corrosion, very smooth, light weigth and
Ans. All of the above very cheap
What is the latent heat of vaporization of water? Ans. Plastic (PVC and ABS)
Ans. 970 Btu Which commercial pipe that is primarily used for water condensate, and refrigerant lines easier
A dry bulb temperature at which water starts to condense but when moist appears in a to bent by hand, good thermal conductivity
constant pressure process Ans. Brass and copper pipe
Ans. Dew point temperature A fluid pathway that expose parts of the fluid to the atmosphere is the:
A mixture of dry air and saturated vapour is known as: Ans. Open channel
Ans. Saturated air 41. The ratio of the area in flow to the width of the channel at fluid surface in an open
What is another term for “humidity channel is known as:
ratio”? Ans. Specific humidity Ans. Hydraulic depth
42. The most efficient cross-section for an open channel is known Ans. Hot-wire anemometer
as: Ans. Semi-circular section 60. Which of the following is used for R-
43. The most efficient section rectangular section has: 12? Ans. All of the above
Ans. A width twice the depth 61. Which of the following tubing (pipes) usually used in ammonia
44. What is the maximum power that the motor can refrigerant? Ans. Ferrous pipe
provide Ans. Rated horsepower 62. Which of the following tubing (pipes) usually used in halo-carbon
Which of the following is a possible cause of refrigerant? Ans. K and L copper tubing
cavitation Ans. All of the above 63. Properties of superheated vapour can be found in?
45. Is a reaction during which chemical energy is released in the form of heat Ans. Superheated table
46. When forebay is not part of the generating plant design, it will be desirable to produce a # Properties of non-reacting gas mixture are given by:
________ in order to relieve the effect of rapid changes in flow rate. Ans. Volumetric weight for molecular weight and density and geometric weighing for all
Ans. Surge chamber the
#What is used to keep the turbine in a hydroelectric generating plant up to 15 feet above the properties except entropy.
tail water surface? 64. The relation between the total volume of a mixture of a non-reacting gases for initial
Ans. Draft tube volume is given by:
47. If a draft tube is not employed, water may be return to the tail water by way of a chamber Ans. Amagat’s law
known as the: 65. Which of the following is the first definition of enthalpy?
Ans. Tail race Ans. The amount of useful energy in a system.
48. An impulse turbine consists of a rotating shaft on which buckets of blades are mounted. What is 66. A consists press thermodynamics process
the term used for rotating shaft? obeys: Ans. Charles law
Ans. Turbine runner 67. The volume of an ideal is halved, while it’s temperature is double, what happens to the
49. Which of the following refrigerant that belong to group B refrigerants and harmful or pressure? Ans. Pressure is multiplied by 4
toxic refrigerants? 68. A liquid boils when its pressure equal.
Ans. All of the above Ans. Ambient pressure
50. Which of the following refrigerants are combustible? 69. A system composed of ice and water at zero °C is said to
Ans. All of the above be: Ans. All of the above
51. Which of the following refrigerants is used for frozen foods and ice cream display cases 70. A heat of fusion for a pure substance is ________.
Warehouses and food freezing plants, medium temperature display cases truck refrigeration and Ans. The energy required to melt the substance.
heat pumps? #The heat of vaporization involves the change of enthalpy due to:
Ans. R-502 Ans. The change in phase from liquid to gas.
52. The refrigerant that was formerly the most widely used for air conditioning and 71. The heat of sublimation involves the change of enthalpy due
refrigeration. It was the principal refrigerant for automotive air conditioning. to: Ans. The change in phase from solid to gas.
Ans. R-12 72. The unit for absolute viscosity which is dyne-sec/cm3 is also known
53. How do we know that there is shortage of refrigerant considering that we are in the liquid as: Ans. Poise
line? Ans. Vapor bubbles in the sight glass appear 73. What is the unit of absolute viscosity?
54. The amount of power actually entering a fluid is known as: Ans. Pa-second
Ans. All of the above 74. The ratio of absolute viscosity to mass density is also known
#The input power to the pump will be: as: Ans. Kinetic viscosity
Ans. Equal to the output power of the electric motor driving the pump 75. What is the kinetic viscosity?
55. A bent or curved carries fluid from a container at a high elevation to another container at a Ans. m2/s
lower elevation. 76. Stroke is a unit of kinetic viscosity which is equivalent
Ans. Siphon to: Ans. cm2/s
56. A water path, usually a large diameter pipe used to channel water around or through an 77. What are volatile liquids?
obstruction feature. Ans. The liquid that vaporizes immediately
Ans. Culvert # When does boiling occur?
57. Which of the following principles governs the distributions of flow between the two branches Ans. When the liquid temperature is increased to the point that the vapour present is equal to
in a pipe system. the local ambient pressure.
Ans. All of the above 78. The liquid boiling pressure is dependent
58. A method used to determine the network flow in a multi-hoop piping system. on: Ans. Both A and B
Ans. Hardy-cross method 79. Referred as the mass flow the rate divided by the generator output in
59. A device used to measure velocity by determining the cooling effect of fluid flowing kilowatts Ans. Water rate of stream rate
over an electrically heated tungsten. 80. The power after the auxiliary loads has been removed is known as:
Ans. Net electrical output 103. The hot-wire manometer is used to
81. _________ of a turbine is the ratio of actual to ideal energy measure Ans. gas velocities
extraction? Ans. Both A and B 104. The pitot static tube measures
82. What is the approximate maximum practical metallurgical limit on Ans. the static pressure
superheat? Ans. 1150°C and 625°C 105. The terminal velocity of a small sphere setting in a viscous fluid varies as
83. What is used to increased the mean effective temperature at which heat is added without the Ans. inverse of fluid viscosity
producing significant expansion in the liquid-vapor region. 106. Pressure drag results from
Ans. Superheat Ans. skin friction
84. If some of the heat energy from these waste product is recovered and used for space 107. The pressure coefficient is the ratio of pressure
heating or cooling the process is called forces to: Ans. inertia forces
Ans. Cogeneration 108. Tranquil flow must always occur
85. In cogeneration, the recovered heat Ans. above the critical depth
Ans. Is used as heat 109. Which of the following head loss coefficient among the following types of
86. The ratio of useful energy to the energy input is entrance? Ans. re-entrant
called? Ans. Fuel utilization 110. What waste water treatment method involves of algae from stabilization pond
87. The ratio of the energy of a turbine to the recovered heat is known effluents? Ans. microscreening
as: Ans. Power to heat ratio 111. The number of nozzles will depend on the quantity of steam required by the turbine. If
88. If the recovered heat is used to vaporized water in a vapour cycle this is the nozzles occupy the entire arc of the ring, the turbine is said to have:
called: Ans. Combined cycle Ans. full peripheral admission
89. What is the best coolant for high temperature gas reactor? 112. Tandem compound units may also have two low-pressure casting that produces:
Ans. Helium (He) Ans. triple flow
90. What is the liquid metal most frequently used as coolant, in liquid metal 113. By inter-cooling using two stage compressor of Brayton cycle, the backwork ratio
reactor? Ans. Sodium (Na) will: Ans. decrease
91. Combustion power cycles differ from vapour power in that: 114. On dynamic similitude, the relation which represents the ratio of inertia force to pressure
Ans. The combustion product cannot be returned into their initial condition for reuse. force is: Ans. Euler number
92. A closed system using a fixed amount of ideal air as the working 115. What is the maximum velocity in a sewer flowing full?
fluid? Ans. Air standard cycle Ans. 0.6 m/sec
93. Which of the following is not a specific compound but is a mixture of octane and 116. A temporary structures constructed to exclude water from the site of the foundation
lighter hydrocarbon? during its excavation and construction is called:
Ans. Gasoline Ans. coffer dam
94. A series of process that eventually brings the system back to its original condition is known 117. Which is not a physical characteristics of water?
as? Ans. Cycle Ans. hardness
95. The _____ of a power cycle is defined as the ratio of usual work output to the supplied 118. Which dam is best for weak
input energy? foundation? Ans. buttress
Ans. Thermal efficiency 119. What is the volume of water which will drain freely from the
96. Is use to heat up the solution partially before entering the generator in absorption aquifer? Ans. specific yield
refrigeration system. 120. What is the line defined by the water level in a group of artesian
Ans. regenerator wells? Ans. peizometric surface
97. The COP of actual absorption refrigeration system is usually 121. Select the one that is a positive indication of pollution of a river.
Ans. less than 1 Ans. chloride content
98. Sight glass is often 122. Which instruments is used to measure humidity of the atmosphere
located at: Ans. liquid continuously? Ans. hydrograph
line 123. Entrance losses between tank and pipe or losses through elbows, fittings and valves are
99. Use to detects a vibration in current caused by the ionization of decomposed refrigerant generally expresses as a function of:
between two opposite-charged platinum electrodes. Ans. kinetic energy
Ans. Electronic detector 124. The air that contains no water vapour is called:
100. The ability of oil to mix with refrigerants Ans. dry air
Ans. miscibility
101. For the submerged plane surface, the point on the surface where the resultant force acts is
called the 125. What effectively states that it is impossible to built a cylindrical engine that will have a
Ans. center of pressure thermal efficiency of 100%?
102. At any point in fluid at rest, the pressure is the same in all directions. This principle is Ans. Kelvin-Planc statement of second law
known as: Ans. Pascal’s Law
126. Refers to the maximum possible work that can be obtained from a Ans. Q positive in, W positive out and ∆U negative for decreased internal energy
cycle. Ans. Availability 152. The heat transfer term in the first law of thermodynamics may be due to any of the ff.
127. The difference between the maximum and the actual work output is known except: Ans. Internal heat generation
as? Ans. Process inversibility 153. A system that experiences no mass crossing the system boundaries called:
128. The study of the property of atmospheric air? Ans. Closed system
Ans. Psychrometry 154. A substance in which a substance is allowed to enter and have is most properly
129. The temperature of the air that has gone through an adiabatic saturation process is known called: Ans. Open system
as? Ans. Wet-bulb temperature 155. First and second law of thermodynamics are:
130. How often should compressor oil be changed? Ans. Energy equations
Ans. At least semi-annually 156. A constant temperature thermodynamic process
131. Which of the following is the possible effect of the weak solution of brine in a refrigeration obeys: Ans. Boyles Law
system? Ans. Corrosion 157. A series of process that eventually bring the system back to the original condition is
132. In Freon refrigeration system, where is the scale trap located? called a: Ans. Cycle
Ans. On the suction side of the compressor 158. If the refrigerant leaves the evaporator with a quality of < 1, the cycle is known as:
133. At what pressure is the high pressure cut-off in the Freon (R- Ans. Wet vapor compression cycle
12) Ans. 125-150 psi 159. For even lower temperature, are required as in commercial freezing equip. What is
134. What is the other name for brine refrigeration system? used as refrigerant?
Ans. Indirect refrigeration system Ans. Ammonia
135. What is the indication that there is air in the refrigeration 160. What is another name for Air-Refrigeration cycle?
system? Ans. Unusual high head pressure Ans. Brayton cycle
136. A Freon-12 leaking can be detected by halide torch. What color will it turn to in the presence 161. What is the major disadvantage of Air-Refrigeration
of a refrigerant if this torch has a normal blue flame? cycle? Ans. High power consumption
Ans. Green 162. What refrigeration is practically used when a large quantity of waste-inexpensive heat is
137. All of the following process are irreversible except avail? Ans. Heat-driven refrigeration cycle
Ans. Inelastic tension and release of steel bar 163. For an absorption cycle, how many working fluids are required?
138. Which of the following is a point function Ans. Two (2)
except Ans. Work 164. In absorption cycle, what is needed in NH3 water system to remove any remaining
139. All heat transfer process require medium of energy exchange traces of absorbents from the refrigerant and is placed between generator and condenser.
except Ans. Radiation Ans. Rectifier
140. Thermal conduction is described by: 165. Which is a “TEMA” stands for?
Ans. Fourier Law Ans. All of the above
141. Convection is described # The acronym “ TEMA” stands for?
by: Ans. Newton’s Ans. Tubular Exchangers Manufacturing Association
Law # The acronym “API” stands for?
142. Radiant heat transfer is described Ans. American Petroleum Institute
by: Ans. The Stefan-Boltzman 166. What occurs when the exit temperature of the cold fluid is above the exit
Law temperature of the hot fluid?
143. The equivalence of ratios of emissive power to absorptivity equilibrium is Ans. Temperature cross
described by Ans. Kirchoff’s Law 167. Are heat exchangers whose purpose is to heat with condensing stream.
144. The temperature potential between temperature difference. Ans. Closed feedwater heater
Ans. The logarithmic temperature difference 168. Refers to the corrosion, precipitation of compound in solution, setting of particular
145. A thermodynamic process whose deviation from equilibrium is infinitesimal at all solid and biological activity that adhere to a heat transfer
times. Ans. In quasi-equilibrium Ans. Fouling
146. A thermodynamic property best describes the molecular activity of a substance? 169. Fouling in a heat exchanger industries is commonly known as:
Ans. Internal energy Ans. Silent thief
147. The combination of conditions that best describes thermodynamic process is 170. An operation with 1 fluid flow rate substantially known
given: Ans. Has successive states thru which the system passes as: Ans. Down turn
148. A substance whose properties are uniform throughout is called a: 171. Indicate the false statement?
Ans. Pure substance Ans. Duo to viscosity, liquid cannot resist instantaneously change velocity
149. All of the following are thermodynamic properties 172. Ideal liquid are assumed to
except: Ans. Modulus be? Ans. Newtonian
150. A process that is adiabatic and reversible is also fluids
called: Ans. Isentropic 173. Which of the following is a Newtonian fluids?
151. The first law of thermodynamics for a closed system is Q= ∆U + W, the sign convention is:
Ans. All of the above Ans. energy
174. Thermal radiation heating body can be absorb, reflected or transmitted, this is 196. Is a process during which the system remains in equilibrium at all
known as Ans. Radiation Conservation law times Ans. Quasi-equilibrium
175. The rate of thermal emitted per unit area of a body is known as: 197. In the absence of any work interactions between a system and its surrounding, the
Ans. Emissive power amount of net heat transfer is equal
176. States that for any two bodies in thermal equilibrium, the ratio of emissive Ans. to the change in the total energy of a closed system
power to absorptivity are 198. The constant volume and constant pressure specific heats are identical for
Equal. Ans. incompressible substance
Ans. Kirchoff’s Radiation law 199. The velocity of fluid is zero at the wall and maximum at the center because of
177. Bodies that radiate at absorptivity of 1 are known as: the Ans. viscous effect
Ans. Black bodies or ideal radiators 200. For steady flow devices, the volume of the control volume is
178. Which of the following does not radiate at the ideal Ans. constant
level? Ans. Real body 201. The work done in a turbine is _____ since it is done by the
179. The ratio of actual to ideal emissive power is known fluid. Ans. positive
as: Ans. Emissivity 202. Reheating process in Brayton cycle, the turbine work will
180. What is the refrigerator’s main function? Ans. increase
Ans. To cool this low temperature area 203. As the air passes through a nozzle, which of the following will
181. The rate of energy removal from the low temperature area is known as: increase? Ans. mach number
Ans. Either of the refrigeration capacity or refrigerating effect 204. As the air passes through a diffuser, which of the following will
182. The refrigeration capacity is measured in refrigeration ton where one ton is _____ decrease? Ans. mach number
heat remove 205. As the air passes through a nozzle, which of the following will
Ans. All of the above decrease? Ans. internal energy
183. The refrigeration ton is derived from the heat flow required to melt the ton of ICE 206. As the air passes through a diffuser, which of the following will
in: Ans. 24 hours increase? Ans. density
184. In psychrometric chart, the constant- enthalpy lines coincide with constant- 207. As the air passes through a diffuser, which of the following will NOT be
temperature lines at temperature affected? Ans. entropy
Ans. below 50°C 208. After passing through a convergent-divergent nozzle, the temperature of air
185. The amount of moisture in air depends on its will: Ans. decrease
Ans. temperature 209. After passing through a convergent-divergent nozzle, the density of air will:
186. The deep body temperature of healthy person is maintained Ans. decrease
constant at Ans. 37°C 210. After passing through a convergent-divergent nozzle, the mach number of air
187. Air motion also plays important role in will: Ans. decrease
Ans. human comfort 211. By increasing the temperature source of Carnot cycle, which of the following will
188. During simple heating and cooling process has a_____ humidity not be affected?
ratio. Ans. constant Ans. heat rejected
189. The ________ follows a line of constant wet-bulb temperature on the psychrometric 212. By decreasing the temperature sink of Carnot cycle, which of the following will not be
chart. Ans. evaporative cooling process affected? Ans. heat added
190. A vapor which is not about to condense is called a Ans. Exothermic reaction
Ans. superheated vapor 213. By reheating the steam in an ideal Rankine cycle the heat rejected will:
191. Passing from the solid phase directly into vapor phase is Ans. increase
called Ans. sublimation 214. By increasing the boiler pressure in Rankine cycle the moisture content at boiler exit
192. Robert Boyle observed during his experiments with a vacuum chamber that the will: Ans. increase
pressure of gases is inversely proportional to their 215. Presently the highest steam temperature allowed at the turbine inlet is about
Ans. volume ______. Ans. 620°C
193. _____ is energy in transition. 216. Two most common gases employed in Stirling and Ericson cycles are:
Ans. Heat Ans. Hydrogen and helium
194. Is the mode of energy transfer between a solid surface and the adjacent liquid or gas which 217. In most common designs of Gas turbine, the pressure ratio ranges
is in motion, and it involves combine effects of conduction and fluid motion. from: Ans. 11 to 16
Ans. convection 218. In Brayton cycle, the heat is transformed during what process?
195. Radiation is usually considered as Ans. isobaric process
Ans. surface phenomenon 219. The fuel injection process in diesel engine starts when the
# Work is _____ between the system and the surroundings. piston_______. Ans. approaches TDC
220. If the cut-off ratio of diesel cycle increases, the cycle efficiency Ans. Rankine vapor cycle
will: Ans. decrease 243. What can be considered to be theoretically exact and can be used to determine local thermal
221. The fuel used in a power plant that is used during peak film coefficient at a distance X from the leading edge of an isothermal plate in laminar flow?
periods. Ans. liquid Ans. Pohlhausen Solution
222. Typical compression ratio of Otto cycle is 244. Which one is exact and can be derived from the Pohlhausen correction by setting cube
Ans. 8 root of Prandtl number equal to 108 and is useful in gases and water?
223. If Joule Thompson coefficient is equal to zero, then the process will Ans. Blausius Solution
become Ans. isothermal 245. What another term for skin friction coefficient?
224. If the fluid passed through a nozzle its entropy will: Ans. Fanning friction factor
Ans. remains the same 246. The Darcy friction factor is
225. Which of the following is NOT a possible location of service ________. Ans. 4 times
valve? Ans. outlet of condenser 247. The ______ for submerged bodies in the sum of the skin friction coefficient
226. A coil in series with evaporator that is use to prevent the liquid refrigerant entering is? Ans. Drag coefficient
the compressor. 248. What can predict the average film coefficient along the entire length of laminar
Ans. Drier loop flow? Ans. Labarsky-Kauffman Correlation
227. A type of valve connected from discharge of compressor directly to suction that is 249. When can we say that the laminar flow is “Fully Developed”?
normally closed Ans. When the difference between the surface (wall) and the mean fluid temperature is
And will open automatically only if there is high discharge Constant
pressure. Ans. solenoid valve 250. Where is the accumulator usually
228. Use to increase the capacity of condenser. located? Ans. Before the suction line
Ans. desuperheating coils 251. A device used to remove moisture and foreign particles inside the refrigeration
229. Is use to subcooled the refrigerant from the system. Ans. Filter drier
condenser. Ans. liquid subcooler 252. What is the standard value of the atmospheric pressure at the atmospheric sea
230. Which of the following is NOT a part of low pressure side in refrigeration level? Ans. 14.7 psi
system? Ans. liquid line 253. Requirements are classified according to:
231. Which of the following is NOT a part of high pressure side in refrigeration Ans. Manner of absorbing heat
system? Ans. suction line 254. A push or pull on any thing is
232. Which of the following is NOT a part of condensing unit? called: Ans. Force
Ans. liquid line 255. Anything that takes up space has
233. By subcooling the refrigerant in refrigeration system, the compressor power per unit mass mass Ans. Matter
will Ans. remains the same 256. Factor use in calculating the overall heat transfer through the tube walls of the condenser
234. Superheating the refrigerant in refrigeration system without useful cooling, the tube of Another heat transfer surface.
refrigeration effect Ans. Fouling factor
Per unit mass will 257. What could be the possible cause of the abnormal discharge temperature?
Ans. remains the same Ans. All of the above
235. By subcooling the refrigerant in refrigeration system, the specific volume at compressor # Where is the relief valve located?
suction will Ans. At the discharge side of the compressor
Ans. remains the same 258. What will you do before starting a refrigeration
236. Pressure loss due to friction at the condenser, the compressor power per unit mass will system? Ans. Vent the condenser
Ans. remains the same 259. What is the purpose of the expansion valve bypass?
237. Which of the following is NOT a type of air-cooled Ans. To control the refrigerant to the evaporator in case the automatic valve fails.
condenser? Ans. shell and tube 260. How many feed water lines are connected to the
238. A type of refrigerant control typically used in household boiler? Ans. 2
refrigeration. Ans. Capilliary tube 261. Exhaust gases from the engine possesses what type of
239. Type of condenser that operates like a cooling tower. energy? Ans. Kinetic energy
Ans. evaporative condenser 262. What is the instrument used to measure density?
240. The major problem of heat Ans. Hydrometer
pump is Ans. frosting 263. Engine overall efficiency is
241. Dominant refrigerant used in commercial refrigeration system ________. Ans. Brake thermal
242. The Carnot refrigeration cycle is: efficiency
Ans. All of the above 264. What is the ratio of the fuel usage rate to the power
# The vapour compression cycle is essentially a reverse of: generated? Ans. Specific fuel consumption
265. What is the ratio of the air mass that enters the engine to each mass of fuel?
Ans. Air-fuel ratio Ans. Loss of boiler efficiency
266. The ohmmeter is an instrument used to 291. How are tubes secured in a fired tube boiler will result
measure Ans. Resistance in: Ans. Rolled and beaded over
267. The ammeter is an electrical instrument used to 292. A tube in a fire tube boiler is surrounded by:
measure Ans. Current Ans. Water
268. The electrical pressure that pushes the electrical current or 293. What is the formula for the factor of
objections. Ans. Voltage evaporation? Ans. H-b/970.3
269. The voltmeter is an instrument used to measure: 294. What can cause a super heater tube to
Ans. Voltage overheat? Ans. Dirty tube
270. When the winding is shorted, what is the resistance 295. How many square feet of heating surface does a 3-in fire tube 20 ft long with a 3/16 in .
reading? Ans. Zero ohm wall have?
271. Which tools are used to enlarge the end of the tubing to connect a Ans. 13.7
fitting? Ans. Flaring tools 296. In a high temperature hot water boiler; what could cause a tube to over heat?
272. The tubing is never used when the fluid temperature goes beyond 100 to 175 Ans. Poor circulation
mm? Ans. Plastic # A down corner is:
273. The tubing to be swaged is damped in a Ans. Found in a water tube boiler
Ans. Flaring block 297. Extreme firebox temperature changes
274. The hand tools used to lightened and loosen the fitting cause: Ans. Spalling
are Ans. Wrenches 298. Low water cu. off:
275. In soldering process, What substance is used to prevent Ans. Shuts the burner when the boiler is low on water
oxidation? Ans. Flux 299. Steam coming from the bottom by cock would
276. The process of applying molten metal to heated metals is indicate: Ans. Low water level
called: Ans. Soldering 300. Why is the method to cool water called condenser not
277. What is the physical state of the refrigerant? practical? Ans. Water is expensive
Ans. Gas 301. A device use to collect liquid refrigerant entering the compressor to prevent
278. Which of the following appliances uses a finned-coil trouble in compressor is
evaporator? Ans. Window-type airconditioner Ans. Accumulator
279. The operation of condenser is the opposite of that of 302. A disiccant is a material that has high affinity to water. Which is the common used
Ans. Evaporator dissicant Ans. Silica gel
280. Which of the following devices maintains the pressure difference between the evaporator 303. The only means of preserving food in its original fresh state is _____________.
and the condenser? Ans. Refrigeration
Ans. Flow control # PVC means
281. What device is ideal for measuring the flow of liquid metal? Ans. Poly Vinyl Chloride
Ans. Magnetic flow meter # What does ABS means?
282. The frictional force that acts parallel but opposite to the direction of Ans. Acrylonitrite Butadiene Styrene
motion. Ans. Drag 304. What measures the average roughness of imperfection inside the
283. A pump that transform kinetic energy into fluid static pressure. pipe? Ans. Specific roughness
Ans. Kinetic pump 305. Darcy factor, friction factor is not constant but decreases as the Reynolds number
284. Due to friction losses between the fluid and the pump and mechanical losses in the pump increases up to a certain point known as:
itself. Ans. The brake pump power will be greater than hydraulic power Ans. Full turbulent flow
285. The net energy actually transferred to fluid per unit time is: 306. What presents the friction factor graphically as 6 functions of Reynolds number
Ans. Hydraulic power and relative roughness.
286. The input power delivered by motor to Ans. Moody diagram
pump. Ans. Brake pump power 307. If the flow in truly laminar, and fluid flowing in a circular pipe then which of the
287. Boiler tubes are specified by equation is appropriate to use?
Ans. Outside diameter Ans. Hagen Poisenille Equation
288. The maximum size of the boiler down lines valves and fittings shall not 308. Which is true about Hagen Williams equation?
exceed Ans. 2.5 inches Ans. It is primarily use for water
289. The purpose of an expansion tank in a hot water heating system is to provide for the 309. For highly turbulent flow, what causes shear
expansion of stress? Ans. Momentum Effect
Ans. Water 310. In a circular pipe laminar flow momentum
290. Heavy accumulation of a soot in the boiler will result in: flux is: Ans. Maximum at the pipe wall
# If the head pressure is too high 330. By sub-cooling the refrigerant in 50apour compression cycle at condenser exit, which of
Ans. The high pressure cut-out switch should operate the following will increase? (Use per unit mass analysis)
311. In a combustion engine cycle, what is the ratio of the net output power to the Ans. Refrigerating effect
input? Ans. Thermal efficiency 331. By sub-cooling the refrigerant in 50apour compression cycle at condenser exit, which of
312. In a reciprocating engines, what is the ratio of actual to ideal volumes of entering the following will decrease? (Use per unit mass analysis)
gases? Ans. Volumetric efficiency Ans. Mass flow rate
313. In a combustion engine, what is the ratio of the actual power developed to the ideal 332. By increasing the vaporizing temperature in vapor compression cycle, which of the
power developed? following will increase? (Use per unit mass analysis)
Ans. Mechanical efficiency Ans. COP
314. The equivalent of ratio of emissive power to absorptivity for both thermal 333. By increasing the vaporizing temperature in 50apour compression cycle, which of
equilibrium is described the following will decrease? (Use per unit mass analysis)
by: Ans. temperature difference between evaporator and compressor
Ans. Kirchoffs law 334. By increasing the condenser pressure in 50apour compression cycle, which of the
315. The ideal cycle based on the concept that the combination process in both diesel following will increase? (Use per unit mass analysis)
and gasoline in Ans. Compressor power
the combination of heat transfer process that is constant pressure and constant 335. By increasing the vaporizing temperature in 50apour compression cycle, which of
volume. Ans. Dual cycle the following will decrease? (Use per unit mass analysis)
316. The ratio of the volume at the end of heat addition to the volume at the start of Ans. moisture content after expansion
heat addition is called: 336. If the pressure drop in the condenser increases in a vapor compression cycle, which
Ans. Cut-off ratio of the following will increase? (Use per unit mass analysis)
317. A theoretical body which when heated to incandescence would emit continuous light Ans. heat rejected in the condenser
ray spectrum. 337. If the pressure drop in the condenser increases in a 50apour compression
Ans. Blackbody cycle, which of the following will decrease? (Use per unit mass analysis)
318. Which of the following is the reason for insulating the Ans. mass flow rate
pipes? Ans. Heat loss from the surface is minimize 338. If the pressure drop in the condenser increases in a 50apour compression
319. Heat transfer due to density differential cycle, which of the following will not be affected? (Use per unit mass analysis)
Ans. Convection Ans. compressor power
320. A process involving a gas that cannot, even in principle be reserved with being 339. If the pressure drop in the evaporator increases in a 50apour compression
done. Ans. An isobaric compression cycle, which of the following will increase? (Use per unit mass analysis)
321. The maximum amount of heat that can be converted into mechanical energy Ans. heat rejected in the condenser
Ans. Depends on the intake and exhaust temperature 340. If the pressure drop in the condenser increases in a 50apour compression
322. The maximum amount of mechanical energy converted into cycle, which of the following will decrease? (Use per unit mass analysis)
heat Ans. 100% Ans. COP
323. The work output of energy heat engine. 341. By lowering the condenser pressure in Rankine cycle, which of the following
Ans. Equals the difference between its heat intake and heat exhaust will decrease? (Use per unit mass analysis)
324. To increase the output of a centrifugal pump, you Ans. heat rejected
must Ans. Speed of rotation 342. By increasing the boiler pressure in Rankine cycle, which of the following
325. By superheating the refrigerant in vapor compression cycle with useful cooling, will decrease? (Use per unit mass analysis)
which of the following will increase? (Use per unit mass analysis) Ans. heat rejected
Ans. Heat rejected from condenser 343. By superheating the steam to a higher temperature in Rankine cycle, which of
326. By superheating the refrigerant in vapor compression cycle with useful cooling, the following will decrease? (Use per unit mass analysis)
which of the following will decrease? (Use per unit mass analysis) Ans. moisture content at the turbine exhaust
Ans. Mass flow rate 344. By superheating the steam to a higher temperature in Rankine cycle, which of
327. By superheating the refrigerant in vapor compression cycle with useful cooling, the following will increase? (Use per unit mass analysis)
which of the following will decrease? (Use per unit mass analysis) Ans. cycle efficiency
Ans. COP 345. By superheating the steam to a higher temperature in Rankine cycle, which of
328. By superheating the refrigerant in vapor compression cycle with useful cooling, the following will decrease? (Use per unit mass analysis)
which of the following will increase? (Use per unit mass analysis) Ans. pump work
Ans. Compressor power 346. What type of boiler incorporates furnace water cooling in the circulatory system?
329. By superheating the refrigerant in vapor compression cycle with useful cooling, Ans. Integral-furnace boiler
which of the following will not be affected? (Use per unit mass analysis) #The main components of a combined cycle power plant are:
Ans. Compressor power Ans. Gas turbine and waste heat boiler
347. Which of the following indicators is used to determine the anti-knocking Ans. Down in the direction of the flow
characteristics of gasoline? #The presence of minor loss in the energy grade line will cause the line to
Ans. Octane number slope Ans. Down in the direction of the flow
348. Indicate the false statement 369. If the Mach number is greater than 1 but less
Ans. The heat transfer cannot exceed the work done. than 5 Ans. Supersonic
# In fluid flow, linear momentum is: 370. The flow is called sonic when mach
Ans. A vector quantity equal to the product of mass and velocity number is Ans. Equal to 1
349. A fact that a fluid’s velocity increases as the cross-sectional area of the pipe through 371. The flow is sub-sonic when mach number is
which it flow decreases due to: Ans. Less than 1
Ans. The continuity equation 372. To check water level in the gauge glass of a steam
350. The coefficient of contraction is the ratio of: boiler Ans. Use the
Ans. Area of vena contracta to the orifice area 373. The flame failure control is tested by?
# The coefficient of discharge is the ratio of: Ans. Shutting off the fuel supply to the burner
Ans. Actual discharge to the theoretical discharge 374. During purge cycle what is the minimum amount air volume changes
# The coefficient of velocity is the ratio of: required? Ans. Four (4)
Ans. Actual velocity divided the theoretical velocity 375. The purge cycle time for gas compared to oil is:
351. Flow measuring devices include all of the following Ans . The same
except: Ans. Magnetic dynamometers #A high (CO) Carbon monoxide reading, indicates:
352. In the series pipe systems, all of the following parameters vary from section to section Ans. None of these
except Ans. Mass flow 376. The induced draft fan is
353. The coefficient of velocity accounts for the: located: Ans. In the
Ans. Small effect of friction and turbulence of the orifice breaching
354. Expansion factors take into account 377. A balanced draft boiler required:
the Ans. Effects of Ans. Both forced and induced draft
compressibility 378. What percent would expect in a well maintain boiler burning number 6
355. The water hammer phenomenon is primarily what kind of fluid oil? Ans. 15%
mechanics? Ans. Dynamic (a time-dependent phenomena) 379. Laminar friction factor of fluid is flowing through a pipe is a function of all the
356. All of the following are forms of drug on a body moving through a fluid following except:
except: Ans. D’Alembert’s paradox drug Ans. Pipe roughness
357. The function of a turbine is to: 380. The stream function is a useful parameter in describing:
Ans. Extra energy from the flow Ans. Conservation of mass
358. The fact that there is no bodies moving through an ideal fluids is known 381. The study of the practical law of fluid and the resistance of open pipes and channels is
as: Ans. D’Alembert’s paradox the
359. Liquids and gases takes the following characteristics of their _____
contents. Ans. Shapes Ans. Hydraulics
360. All of the following dimensionless parameters are applicable to fluid flow problem 382. The most common methods of calculating frictional energy loss for laminar flow
excepts Ans. Bolt number fluids in non circular pipes:
361. All of the following can be characteristics of fluids except: Ans. Equation
Ans. Hysteresis 383. The parameter in the expression for head loss is:
362. The most common method for calculating frictional energy loss for laminar flowing Ans. Darcy friction factor
fluid is non- circular pipes is: 384. The characteristic length of Reynolds number use to calculate the friction in non-circular
Ans. The Darcy equation full running pipes is based on the:
363. For computation convenience, fluids are usually classed as Ans. Hydraulic diameter
Ans. Real and ideal 385. The hydraulic radius of a non-circular pipe is:
364. Which of the following is not a characteristics of real fluids? Ans. The ratio of flow area to wetted parameter:
Ans. Experience of eddy currents and turbulence 386. An electromagnetic radiation wavelength is in the 0.1 to 10.000 mm
365. Property of a fluid whereby its own molecules are attracted is known range: Ans. Thermal radiation
as: Ans. Cohesion 387. What characteristics makes the difference between a Newtonian and non-newtonian
366. The term subsonic flow refers to a flowing gas with a speed. fluids? Ans. Their viscous behaviour
Ans. Less than the local speed of sound 388. What instrument use to measure salt solution:
367. The difference between stagnation pressure and total pressure Ans. Salimeter
Ans. None of the terms are interchangeable 389. A measure of fluid resistance to
368. The presence of friction in the hydraulic grade line will always cause the line to slope. flow? Ans. Viscosity
390. What is another for absolute viscosity?
Ans. Dynamic 411. The work done in an adiabatic (isentropic, process is _____ the work done by the
391. What is the reciprocal of isothermal process;
viscosity? Ans. Fluidity Ans. Less than
392. The density of fresh water is a ship will float. 412. The work done in an isentropic compressor is ____ the work done by the
Ans. Lower in fresh water than in sea warer isothermal compressor;
393. Bernoulli’s equation is based on? Ans. Greater than
Ans. Conservation of energy 413. The term used to describe a partially compressed gases withdrawn cooled and
394. An express train goes past a station flatform at high speed a person standing in the edge compressed further;
of the flatforms tend to be: Ans. Intercooling
Ans. Attracted to the train 414. A perfect intercooling refers to the case where the gases cooled to:
395. The volume of fluid flowing per second out of an orifice at the bottom. A tank does Ans. The original inlet temperature
not depend on: 415. Multi stage blowers can reach pressure up to ____ of
Ans. Density of fluid water; Ans. 100 inches
396. Addition of detergent to water? 416. What represents the ratio of the coefficient of the friction at the expanded in
Ans. Decreases its surface tension pumping a fluid;
397. Water neither rise or falls in silver capillary. This suggest that the contact angle Ans. Fried heat transfer efficiency factor
between the water and silver is: 417. What is the proper oil storage temp for No. 6 oil?
Ans. 90 degrees Ans. 20 ºF above pour point
398. Most pressure in the Freon system have two dials or graduation on 1 gage what does the # To prime a pump
two dial represent: Ans. Fill casing with water
Ans. Temperature and pressure # Balance draft means;
399. The flame detector is directed: Ans. Almost atmosphere pressure in the furnace
Ans. Into the furnace area 418. A gallon of No. 6 oil contains:
#Which of the following statement is true? Ans. 152,000 Btu
Ans. All of the above 419. What is the efficiency of a steam plant with a heat rate of 12,000
#Which of the following is a pseudo plastic fluid? Btu/KW? Ans. 28.4%
Ans. All of the choices 420. What type of fuel must be preheated to burn properly?
400. A type of fluids which are capable of indefinitely resisting a small shear stress but Ans. Heavy oil
move easily when the stress becomes large: 421. The fuel oil for mechanical atomizing burners is usually
Ans. Bingham fluids heated Ans. 150 ºF
401. Which of the following is an example of Bingham fluids? 422. Fuel suppliers specify a minimum flash point for their oil because a low flash point
Ans. All of the above oil is; Ans. Dangerous
402. The Bingham fluids will become pseudo plastic 423. How many cubic feet of furnace volumes are needed to burn one gallon of No. 6
when: Ans. The stress increases oil? Ans. 10
403. A type of fluids which exhibits viscosities that increases with an increasing velocity 424. A fuel oil is heated its viscosity
gradient. Ans. Dilatant fluid Ans. Decreases
404. The viscosity decreases with time the fluids is said to be: 425. Only ____ of the turbine work output is required to operate the
Ans. Thixotropic fluid pump. Ans. 0.04%
405. Viscosity increases with time the fluids is said to 426. Superheating the steam to higher temperature decreases the moisture content of the
be: Ans. Rneopectic fluids steam at the __
406. What is the dominating cause of viscosity in Ans. Turbine exit
fluids: Ans. Molecular cohesion 427. Regeneration also provides a convenient means of dearating the feedwater to prevent.
407. In a compressor, the piston is said to be at _____ when it is at its most petracted Ans. Boiler corrosion
position in the stroke, 428. Can be apply in Steam turbine cycle (Rankine), Gas turbine cycle (Brayton) and
Ans. Bottom dead center Combined cycle.
408. Reciprocating compressors are characterized by their Ans. Cogeneration plant
Ans. Clearance volume 429. In a Rankine cycle with fixed turbine inlet conditions. What is the effect of lowering
409. The gases remaining in the clearance volume after the discharge in the dead center the condenser pressure, the heat rejected will;
are known as the: Ans. Decrease
Ans. Residual gases 430. In an ideal Rankine cycle with fixed boiler and condenser pressures. What is the
410. The ratio of the clearance volume to the swept volume is known as: effect of superheating the steam to a higher temperature, the pump work input will;
Ans. Percent clearance or just clearance Ans. Remains the same
431. How do the following quantities change when the simple ideal Rankine cycle is 454. The ratio of total dissolve solids in the recirculating water to the total solids in the
modified with regeneration? The heat rejected will; make-up water.
Ans. Decreases Ans. Cycles of concentration
432. During a combustion process, the components which exist before the reaction are 455. What is the another term for Bryton cycle:
called Ans. Reactants Ans. Joule cycle
433. Is an obvious reason for incomplete combustion. # The process of transferring heat from low temperature area to high temperature
Ans. Insufficient oxygen area; Ans. Refrigeration
434. Higher heating value when H2O in the product of combustion #The device to remove heat from water:
is in Ans. Liquid form Ans. Chiller
435. Device which transfer heat from low temperature medium to a high temperature one 456. What device which transfer heat from low temperature area to operate on
is a Ans. Heat pump refrigeration system:
436. A rule of thumb is that the COP improves by _____ for each °C the evaporating Ans. Heat pump
temperature is raised or the condensing temperature is lowered. 457. Refers to the process of one substance mixing with another substance:
Ans. 2 to 4% Ans. Diffusion
437. Are generally more expensive to purchase and install than other heating systems, but 458. Which of the following statement about Newtonian fluid is most
they save money in the long run, accurate: Ans. Shea stress is proportional to the rate of strain
Ans. Heat pumps 459. Which is not characteristic of fluids?
438. The most widely used absorption system is the ammonia-water system, where Ans. Hysteresis
ammonia serves as refrigerant and H2O as the 460. The relationship between pressure and altitude in the
Ans. Transport medium atmosphere: Ans. Barometric height relationship
439. Known as drum less boiler. 461. Flash point means:
Ans. Once-through boiler Ans. Ignition temperature
440. Reduces the steam temperature by spraying low temperature water from boiler # Fire point means:
drum. Ans. Desuperheater Ans. None of the above
441. Carbon dioxide can be removed by; # Bernoulli’s equation is a
Ans. Aeration Ans. Conservation of energy equation
442. Is often used to absorb silica from 462. The pressure at given depth due to several immiscible
water. Ans. Magnesium hydroxide liquid is: Ans. The sum of individual pressure
443. Presence of excess hydrogen ions makes the 463. The relationship between pressure and altitude in the atmosphere is given by
water Ans. Acidic the Ans. Barometric height relationship
444. PH of water varies with 464. The fact that the buoyant force on a floating object is equal to the weight of the
Ans. Temperature water displaced
445. PH value of ____ is usually maintained for boiler water to minimize Ans. Archimedes principle
corrosion. Ans. 10.5 465. Which of the following terms does not appear in the steady flow energy equation in
446. What type of turbine that has a degree of reaction of 1/2? the extended Bernoulli’s equation
Ans. Rarsons turbine Ans. Hysteresis losses
447. The cooling water is made to fall in series of baffles to expose large surface area for 466. The pitot tube can be use to measure fluid velocity as described by the Bernouilli
steam fed from below to come in direct contact. equation and the relationship between;
Ans. Barometric condenser Ans. Kinetic energy and static pressure
448. Show the variatiom of river flow (discharge) with time. 467. The difference between stagnation pressure and total pressure is;
Ans. Hydrograph Ans. None. The terms are interchangeable
449. The pressure at the bottom of a vessel filled with liquids does not depend on 468. Fully turbulent flow in a pipe is characterized by all the following
the; Ans. Area of the liquid surface except: Ans. Parabolic velocity profile
450. A person stands on a very sensitive scale, inhales deeply the reading on the scale: 469. What is the bulk modulus of the water is;
Ans. Depends on the expansion of the person’s chest, relative to the volume inhaled. Ans. 300,000 psi
451. Buoyancy occurs because, with increasing depth in a 470. Atmospheric pressure does not correspond to
fluid: Ans. The pressure increases approximately; Ans. 98 N/m2
452. In order for an object to sink when placed in water its average specific gravity must 471. Hydraulic press is able to produce a mechanical advantage because;
be; Ans. More than 1 Ans. An external pressure extended on a fluid is transmitted uniformly through its
453. The average solar energy heating the outer edge atmosphere is approximately 442 Btu volume
per hour-foot squared of 1.41KW per meter squared is known as: 472. In the operation of a hydraulic press, it is impossible for the output piston to exceed the
Ans. Solar constant input piston
Ans. Work 496. The following are all examples of indirect (secondary) measurements to measure flow
473. If one of the management legs is inclined it is known rates using flow meters except;
as: Ans. Draft gauge Ans. Positive displacement meters
474. Why are manometer tube generally large in 497. In series pipe systems which of the following parameters vary from section to
diameter? Ans. To avoid significant capillary section except;
effect Ans. Mass flow
475. Bernoulli’s equation is an energy conservation based on several reasonable 498. Venturimeter, pitot static gauges, flow nozzles, manometer vary with;
assumptions: Ans. All of the above Ans. Flow velocity and pressure
476. Body that emits a constant emissivity, regardless of the wavelength; 499. Other name for R
Ans. Gray body 729? Ans. Air
477. What gives the total emissive power from a black 500. In an indirect refrigeration system, which of the following is used to measure the
body Ans. Stefan-Boltzmann law of the fourth density of brine?
power Ans. Hydrometer
478. What accounts for the spatial arrangement of the body and their 501. Which of the following is the result of opening a hand expansion valve too much
emissivity; Ans. Emissivity factor in a refrigeration system?
479. The gray body shape factor is the product of the Ans. Evaporator will “freeze back” to compressor
Ans. Black body shape factor and emissivity factor If one of the manometer legs is inclined, it is known as:
480. The product of the area and the shape factor is known Ans. Draft gauge
as; Ans. Geometric flux Why are monometer tubes generally large in diameter?
481. Surfaces that re-radiates absorb thermal radiation are known Ans. To avoid significant capillary effect
as; Ans. Refractor materials of refractories Bernoulli’s equation is an energy conservation based on several reasonable
482. Pitot tube used to measure assumptions:
Ans. Velocity Ans. All of the above
483. Liquid gases take the following characteristics of the Body that emits a constant emissivity, regardless of the wavelength:
containers: Ans. Shape Ans. Gray body
484. For computational convenience usually classed as: What gives the total emissive from a black body:
Ans. Real and ideal Ans. Stefan-boltzman law of the fourth power
485. Which of the following statement about Newtonian fluid is more What accounts for the spatial arrangements of the body and their
accurate? Ans. Shear stress is proportional to the rate of strain emissivity?
486. Which of the following is not a characteristic of a real of fluid? Ans. Emissivity factor
Ans. Experience of eddy current and turbulence The gray body shape factor is the product of the
487. One could expect the possibility of Reynold’s number similarity in all of the following Ans. Black body shape factor and emissivity factor
cases except The product of the area and the shape factor is known as:
Ans. Closed pipe turbulent flow Ans. Geometric flux
488. One could expect the possibility of Froude number similarity in all of the following Surfaces that re-radiates absorb thermal radiation are known as:
cases except Ans. Refractory materials of refractories
Ans. Subsonic airflows Pito- tube used to measure
489. The absolute viscosity of a fluid varies in pressure and temperature as a function of; Ans. Velocity
Ans. Shear and angular deformation rate Liquid gases take the following characteristic of the containers:
490. Turbulent flow of a fluid in a pipe. All of the following Ans. Shape
except; Ans. Reynold’s number will be less than 2300 For computation convenience usually classed as:
491. Flow measuring devices include all of the following Ans. Real and ideal
except; Ans. Magnetic dynamometer Which of the following statement about Newtonian fluid is more accurate? Ans.
492. Flow measuring devices include all of the following Shear stress is proportional to the rate of strain
except; Ans. Mercury barometer Which of the following is not a characteristic of a real fluid?
493. Flow measuring devices include all of the following Ans. Experience of eddy current and turbulence
except; Ans. Precision tachometer One could expect the possibility of Reynold’s number similarity in all of the following
494. The following are all examples of indirect (secondary) measurements to measure flow cases except
rates using obstruction meters except; Ans. Closed pipe turbulent flow
Ans. Volume tanks One could expect the possibility of Froude number similarity in all of the following
495. The following are all examples of indirect (secondary) measurements to measure flow cases except
rates using velocity meters except; Ans. Subsonic airflows
Ans. Weight and mass scales
Difference between energy grade line (friction) and the energy line friction loss is: Ans. End of combustion
In a compression ignition engine, the detonation occurs near the
Ans. Friction and minor losses _____________.
The absolute viscosity of a fluid varies in pressure and temperature as a function Ans. Beginning of combustion
of: Morse test is use to measure the _________ of multi-cylinder engine.
Ans. Shear and angular deformation rate Ans. Indicated power
Turbulent flow of a fluid in a pipe. All of the following are true except: Ignition delay can be minimized by adding ______ to decrease engine
Ans. Reynold’s number will be less than 2300 knocking.
Flow measuring devices include all of the following except: Ans. Ethyl nitrate
Ans. Mercury barometer The work done in throttling valve is
Flow measuring devices include all of the following except: Ans. Zero
Ans. Precision tachometer Stagnation enthalpy represents the enthalpy of a fluid when it is brought to rest
The following are all examples of indirect (secondary) measurements to measure ______.
flow rates using obstruction meters except: Ans. Adiabatically
Ans. Volume tank Represents the temperature an ideal gas attains when it is brought to rest
The following are all examples of indirect (secondary) measurements to measure adiabatically.
flow rates using velocity meters except: Ans. Stagnation temperature
Ans. Weight and mass scales After passing through a nozzle the density of fluid decreases as the fluid velocity.
The following are all examples of indirect (secondary) measurements to measure
flow rates using flow meters except: Ans. Increases
Ans. Positive displacement meter The volume flow passes through a venture meter will Ans.
In series pipe systems which of the following parameters vary from section to section Constant
except: 502. converging-diverging nozzle is the standard equipment in Ans.
Ans. Mass flow Supersonic aircraft
Venturimeter, pitot static gauges, flow nozzles, manometers vary with: Nozzles efficiencies range from
Ans. Flow velocity and pressure Ans. 90%-99%
Other name for R 729? By reheating the steam before entering the second stage in Rankine cycle, which of
Ans. Air the following will increase?
In an indirect refrigeration system, which of the following is used to measure Ans. Heat rejected
the density of the brine? Contains 90% gasoline and 10% ethanol
Ans. Hydrometer Ans. Gasohol
Which of the following is the result of opening a hand expansion valve too much in In an evaporator section in refrigerating unit, which of the following is the function of
a refrigeration system? evaporator?
Ans. Evaporator will “freeze back” to compressor Types Ans. Absorption of latent heat of vaporization
of turbine used up to 300m head. Which of the following may not be the type of bulb in R-12 refrigeration system?
Ans. Deriaz turbine
A turbine that has a diagonal flow. Ans. Duplex
Ans. Deriaz turbine Which of the following shuts down the compressor motor when discharge pressure
Oil is atomized either by air blast or pressure jet at about Ans. rises to a predetermined point, it cuts in motor when pressure drops to predetermined
70 bar pressure:
Type of solid injection that use single pump supplies fuel under high Ans. High pressure cut-off switch
pressure to a fuel header. Which of the following shuts down the compressor motor when discharge pressure
Ans. Common rail injection rises to a predetermined pressure and shuts down the compressor motor when there
Water flow in diesel engine that is caused by density differential. is decrease in pressure in the suction line.
Ans. Thermosiphon cooling Ans. Low pressure cut-off switch
Type of lubrication system in diesel engine in which oil from pump is carried to a The P.O. service pump must have:
separate storage tank outside the engine cylinder and used for high capacity engine. Ans. A means of shutting down from outside the machinery space The
relief valve on the P.O. transfer pump discharge
Ans. Dry sump lubricating system Ans. Back to the transfer pump discharge
Produces extreme pressure differentials and violent gas vibration Ans. When heating hot water, feed pump should at least be how many mm to
Detonation prevent vapor lock? Ans.1220 mm
In a spark ignition engine, the detonation occurs near the______________. The water column should be provided with a valve drain at least:
Ans. 15 mm pipe size A dry cooling tower where stream travels through large diameter trunks to a cross-flow
Lubricating oils have flash points from heat exchanger where it is condensed and cooled by air;
Ans. 375-800 degree Fahrenheit Ans. Direct condensing tower
A device receives information in one form of instrument signal and transmits an output An economizer is used to:
in another form; Ans. Heat the feedwater before it enters the boiler
Ans. converter A manometer measures:
Refinement petroleum Naphtha which by its composition is suitable for use in internal Ans. Air pressure
combustion engine; A pyrometer measures:
Ans. Gasoline Ans. Stack pressure
It is possible for water to get into the fuel oil by: An attemperator is another name for a
Ans. All of the above Ans. Desuperheater
Air chambers are attached to the: Waterwalls are used to:
Ans. Discharge side of the reciprocating pumps Ans. Carry of excess heat from the furnace walls
On automatic combustion control: The studs on waterwalls tubes are to:
Ans. Reduce the number of burners when the oil pressure approaches Ans. Give added surface area to tubes The
minimum specified heats remove from the cold area
The volumetric efficiency of the compressor increases as the suction Ans. Use for energy transfer
pressure: On the upstroke of the piston, the low pressure vapor is first compressed and then
Ans. Increase discharged as high pressure vapor through the discharged valves into the:
As the discharge pressure increases, the volumetric efficiency:
Ans. Increases Ans. Head of the compressor
When the suction And discharge pressure are varied in such a direction that the The vapor that remains the clearance space at the end discharge stroke is called
compression ratio is increased, the volumetric efficiency of the compressor:
Ans. Clearance vapor
Ans. Decrease Which of the following steps would you take if you found an accumulation of oil on the
For a compressor of any given clearance, the volumetric efficiency Ans. furnace floor?
Varies with compression ratio Ans. Open all registers
The useful energy transfer in Btu/hr divided by input power in Watts; Ans. Which of the following method can be used to clean up the inside of the boiler?
Energy efficiency ratio
If air is humidified by injecting steam of by pressuring the air through a hot water Ans. All of the above
spray, the dry bulb temperature and enthalpy of air Another term for the oil discharge strainer is the:
Ans. Increase Ans. Hot strainer
Air passing through a solid or absorbed bed, such as silica gel or activated alumina In a controlled feed water system the power water level maintained by:
will: Ans. The fluctuation water level in the boiler
Ans. Decrease in humidity The difference between the enthalpy of the product at a specific state and the
A device that passes air through dense spray of recirculating water; Ans. Air enthalpy of reactants at the same state for a complete reaction:
washer Ans. Enthalpy of reaction
The mass of water sprayed to the mass of air passing through the washer per unit time. The amount of heat released during a steady flow combustion process when I kg of fuel
burned completely at a specific temperature and pressure.
Ans. Spray ratio Ans. Enthalpy of combustion
When the cooling tower is used to provide cold water for the condense of a The enthalpy of a substance at a specific state due to its chemical
refrigerator system the water circulation system will be approximately: composition
Ans. 3 gal/min-ton Ans. Enthalpy of formation
Water lost in small droplets and carried away by the air flow: The inlet side of condenser tube are rolled and flared to:
Ans. Drift Ans. Allows a smoother entrance flow of circulating water
The ratio of total dissolved solids in the circulating water to the dissolved solids in The purpose of inspection plates on the main condenser is to:
the make-up water Ans. Check tubes without removing condenser heads
Ans. Ratio of concentration A pump that has one steam cylinder and two liquid cylinders is called a:
Through windage removes some solids, most must be removed by bleeding some of Ans. Tandem pump
the water. A mixture of dry air and superheated water vapor Ans.
Ans. Bleed-off Unsaturated air
When the air is unsaturated, what is the relation between the dry bulb Soot blowers should be used in proper sequence so that Ans.
temperature and wet bulb temperature? The soot will be upright toward the uptake
Ans. Less than What is the first thing you would check on taking over a watch?
The difference between the dry bulb temperature and wet bulb temperature Ans. Wet Ans. The water level
bulb depression A boiler with a fan/blower located in the uptake is operating on Ans.
The relative ratio of water vapor to dry air is: Induced draft
Ans. Humidity ratio A fire room that is isolated(closed) operates on:
Also known as humidity ratio: Ans. Forced draft
Ans. Specific humidity How many feed water lines are connected to the boiler?
The ratio of the actual humidity ratio to the saturated humidity ratio: Ans. Two
Ans. Degree of saturation If the water in the gauge glass has not been blown for a period of time, the level of
Also known as saturation ration: water in the glass will be:
Ans. Percentage humidity Ans. less than that in the steam and water drum The
The partial pressure of the water vapor divided by the saturation pressure: enthalpy of fluid when it is brought to rest adiabatically
Ans. Relative humidity Ans. Stagnation enthalpy
Rapidly increasing the temperature through the air of a chord will cause the water in In the absence of any heat and work and any changes in potential energy, the
the gauge to stagnation enthalpy of a fluid during a steady flow process:
Ans. Evaporate Ans. remains constant
Since the increase in the water vapor a latent heat contents equals the decrease During the stagnation process, which of the following is converted to
in the air’s sensible heat, the total enthalpies before and after adiabatic saturation enthalpy?
is: Ans. Kinetic energy
Ans. Equal The cross section of a nozzle at the smallest flow area is called:
An adiabatic saturation process follows a line of constant Ans. Ans. Throat
Dew point temperature The properties of a fluid at a location where the Mach number is unity are called:
Which of the following used with adiabatic saturation process?
Ans. Saturation efficiency Ans. Critical properties
The saturation efficiency of a large commercial air washer is typically Ans. What is the average fuel oil temperature range of the oil in the discharge line?
80%-90%
The presence of oil or scale on the boiler tubes causes: Ans. 180-200 degree F
Ans. Overheating Which of the following is a common type of oil burner?
What would you do before giving a boiler bottom blow off? Ans. All of the above
Ans. Raise the water level The boiler gauge glass should be blown down.
When installing a new gauge glass in water gauge, you should secure the bolts Ans. At the beginning of every watch
from? Gauge pressure of 200 pounds is equivalent to what absolute pressure? Ans.
Ans. Center, alternately, toward each end 215
One of the main purpose of refractories in the boiler furnace is to: In a fuel oil service system, the quick closing fuel oil shut-off valve is located between
Ans. Prevent excessive furnace heat losses the:
A fuel oil service pump steam valves are fitted with: Ans. Master fuel oil shutoff valve and the boiler
Ans. Each rod leading to a location outside the fire room Water A fuel oil meter placed between the fuel oil service pumps and the fuel oil heaters.
tube boilers have
Ans. No fusible plugs Ans. Cold-type meter
The valve that prevents water from backing out of the boiler in the feed water The presence of oil or scale on boiler tubes causes:
line: Ans. Overheating
Ans. Feed check valve What would you do before giving a boiler bottom blow off:
The fuel oil heater is located Ans. raises the water level
Ans. On the discharge side of the service pump When installing a new gauge glass in a water gauge you should secure the bolts
In the forced draft system: from:
Ans. One fan supplies air to all furnaces In Ans. Center, alternately towards each ends
the closed fire room system One of the main purpose of refractories in the boiler furnace is to:
Ans. The fire room is supplied with air from one fan Ans. Prevent excessive furnace heat losses
The air cock on a boiler is located at the: A fuel oil service pump steam valves are fitted with:
Ans. Highest point of the steam and water drum Ans. Each rod leading to a location outside the fire room
Water tube boilers have Ans. Cold-type meter
Ans. No fusible plugs When securing a scotch boiler
The valve that prevents water from backing out of the boiler in the feed water Ans. The belly plug must be removed
line: If the cooling coils temperature is between the airs dew point, the moisture will
Ans. Feed check valve
The fuel oil heater is located Ans. Condense
Ans. On the discharge side of the service pump What occurs when the air passes through a water spray in an air washer? Ans.
In the forced draft system: Evaporative cooling process
Ans. One fan supplies air to all furnaces In To prevent ice buildup, the cooled air temperature should be kept from dropping
the closed fire room system
Ans. The fire room is supplied with air from one fan Ans. Below the freezing point of water
The air cock on a boiler is located at the: What precaution must be taken before using steam soot blowers?
Ans. Highest point of the steam and water drum Soot Ans. Drain thoroughly
blowers should be used in proper sequence so that Which of the following would you do before blowing the tubes with steam soot
Ans. The soot will be upright toward the uptake blower?
What is the first thing you would check on taking over a watch? Ans. All of the above
Ans. The water level Which of the following would cause a flareback?
A boiler with a fan blower located in the uptake is operating on Ans. Ans. Trying to relight from hot
Induced draft Sputtering oil burners might indicate
A fire room that is isolated (closed) operates on: Ans. Water in the fuel oil
Ans. Forced draft The boiler feed water in the feed water heater is heated by:
How many feed water lines are connected to the boiler? Ans. Auxiliary exhaust steam
Ans. Two
If the water in the gauge glass has not been blown down for a period, the level of If the boiler is smoking white smoke, a possible cause would be:
water in the glass will be: Ans. Too much air
Ans. Less than that in the steam and water The Edwards-type air pump has:
The enthalpy of fluid when it is brought to rest adiabatically. Ans. One set of valve
Ans. Stagnation enthalpy A scoop condenser is a:
In the absence of any heat and work and any changes in potential energy, the Ans. Single pass condenser
stagnation enthalpy of a fluid during a steady flow process; The purpose of the steam to baffle in a condenser
Ans. Remains constant Ans. Prevent the steam from hitting directly on the cooler tubes
During the stagnation process, which of the following is converted to The condenser vacuum feed valve is open and the water in the reserve tank is dry.
enthalpy? The result is:
Ans. Kinetic energy Ans. A loss of vacuum
The cross section of a nozzle at the smallest flow area is called: The division plate in a two-pass-condenser
Ans. Throat Ans. Prevents the circulating water from passing directly to overload
The properties of a fluid at a location where a Mach number is unity are called: discharge
The holes in Kinghorn-valve disks
Ans. Critical properties Ans. B
What is the average fuel oil temperature range of the oil in the discharge line? The equilibrium temperature that a regular thermometer measure if exposed to
atmospheric air:
Ans. 180-200 degree Fahrenheit Ans. Dry bulb temperature
Which of the following is a common type of burner? The temperature of air that has gone through an adiabatic saturation process Ans. Wet
Ans. All of the above bulb temperature
The boiler gauge glass should be blown down. If the vapor pressure equals the saturation process, the air is said to be:
Ans. At the beginning of every watch Ans. Saturated
Gauge pressure of 200 pounds is equivalent to what absolute pressure? Ans. When the air is saturated, the dry bulb, wet bulb and the dew point
215 temperature are
In a fuel oil service system, the quick closing fuel oil shut-off valve is located between Ans. Equal
the: On taking over a watch, one should check
Ans. Master fuel oil shutoff Ans. All of the above
A fuel oil meter placed between the fuel oil service pump and fuel oil heater When you are cleaning fuel oil burner tips, use a
Ans. Brass knife Ans. Dry bulb temperature
The amount of steam generated by a boiler is dependent upon Ans. All -The temperature of air that has gone through an adiabatic saturation
of the above process Ans. Wet bulb temperature
A boiler operated at a pressure not exceeding 1.055 kg/cm2 gauge steam of -if the vapor pressure equals the saturation process the air is said to
temperature not exceeding 120oC. be Ans. Saturated
Ans. Low pressure heating boiler -When the air is saturated, the dry bulb, wet bulb and dew point temperature are
No part of the steam generator should be closer than how many from the wall? Ans. Equal
-On taking over a watch, one should check
Ans. 1 m Ans. All of the above
Steam generator should be mounted over a suitable foundation or concrete not less -When you are cleaning fuel oil burner tips, use a
than 305 mm thick and with sufficient area at base to be supported the bearing Ans. Brass knife
capacity of the soil with a factor of safety of -The amount of steam generated by a boiler is dependent upon
Ans. Not less than 4 Ans. All of the above
No smoke stacks should be closer than how many millimeters from the exposed
woodwork or framing -A boiler operated at pressure not exceeding 1.055 gauge steam of steam temperature
Ans. 305 mm not exceeding 120oC.
The air on an extinguisher fire should be Ans. Low pressure heating boiler
Ans. Kept closed -No part of the steam generator should be closer than how many from the wall?
Too low F.O. temperature will cause Ans. 1m
Ans. Poor combustion and smoky fires -Steam generator should be mounted over a suitable foundation or concrete of not less than
-when securing a scotch boiler 305 mm thick and with sufficient area at base to be supported the bearing capacity of the soil a
Ans. The belly plug must be remove factor of safety of
-if the cooling coils temperature is between the airs dew point, the moisture will Ans. Not less than 4
Ans. Condense -No smoke stacks should be closer than how many millimeters from exposed wood work or
-what occurs when the air passes through a water spray in an air washer? framing?
Ans. Evaporative cooling process Ans. 305 mm
-to prevent ice build up, the cooled air temperature should be kept from dropping -The air register on an extinguisher fire should be
Ans. Below freezing point of water Ans. Kept closed
-What precaution must be taken before using steam soot blowers? -Too low F.O. temperature will cause
Ans. All of the above Ans. Poor combustion and smoky fires
-Which of the following would cause a flareback? -When the fuel-oil temperature is too high it causes
Ans. Trying to relight from hot brickwork Ans. Carbon deposits on the fuel-oil heater coils
-Sputtering oil burners might indicate -A dry cooling tower where steam is condensed by cold water
Ans. Water in the fuel oil jets Ans. Indirect condensing dry cooling tower
-The boiler feed water in the feed water heater is heated by: -The hot condensate is pumped to cross heat exchangers whose it is cooled by.
Ans. Auxiliary exhaust steam Ans. Air
-If a boiler is smoking white smoke, a possible cause could be: -which of the following is the refrigerant “of choice” in entering air conditioning
Ans. Too much air Ans. R-22
-The Edwards-type air pump has; -In new equipment, which of the following replaces R-
Ans. One set of valve 11 Ans. R-12
-A scoop condenser is a: -The super heater is used to
Ans. Single pass condenser Ans. Remove moisture from steam
- The purpose of the steam baffle in a condenser is to: -Water tube boiler are
Ans. Prevent the steam from hitting directly on the cooler tubes Ans. Rolled in
-The condenser vacuum feed valve is open and the water in the reserve tank is dry. -The recirculating valve is used to
Ans. A loss of vacuum Ans. Recirculate and heat the fuel oil prior to lighting
-The division plate in a two-pass-condenser off -The fusible plug in a scotch boiler are found in the
Ans. Prevents the circulating water from passing directly to overload discharge Ans. Fire tubes
-The holes in kinghorn-valve discs -Which of the following valves are arrange in a Y-branch fitting?
Ans. B Ans. Boiler-drum safety valves
-The equilibrium temperature that a regular thermometer measure if exposed to -The coils in the feedwater heater are secured by.
atmospheric air; Ans. Union fitting
-Natural draft towers can cool the water to within. Ans. Equal to the mass of the suction vapor that the compressor takes in the entire suction inlet
Ans. 10 to 12oF unit time
Force draft towers can cool the water to within -In order to increase back pressure, one must
Ans. 5 to 8oF Ans. Close in on the back pressure valve
-When you light a fire in the burner, you must -The actual steam that condenses in the feedwater
always Ans. Use a torch heater Ans. Discharge to the hotwell
-The recirculating line on the burner valves returns the oil to -The excess steam pressure in the back pressure system exhausts to
the Ans. Suction side of service pump the Ans. Atmosphere through a relief valve
-The valve which shuts off liquid line with the least amount of resistance to flow. -The air pressure supplied to the boiler is measure in
Ans. Gate valve Ans. Inches of water
-The process applied to iron pipe which retards corrosion is called -The condensate from the coils in the fuel oil heater to the fuel oil heaters goes to
Ans. Galvanizing the Ans. Observation tank
-Listed in sequence, iron pipe series are; The valve between the fuel oil heaters and the burner valve is called
Ans. ¼ ,3/8, ½, ¾ the Ans. Root valve
-The two main purpose of the main condenser are; -The safety device located in the crown sheet of a scotch boiler is called a
Ans. Convert exhaust steam to water Ans. Fusible plug
-Pyrometer is a -Tube retarders in scotch boilers are used to
Ans. High temperature thermometer Ans. Slow down the combustion gases
-The temperature and humidity of the air to be used for comfort cooling shall be maintained -In Order for oil to burn properly it must
at effective temperature at air movement within the range from Ans. 0.0762 to 0.127m/s Ans. Al of the above
-The steam that is discharge from safety valves goes to the
-As the air enters the living zone the air motion in such occupied space and which the Ans. Atmospheric line
only source of contaminants shall have a velocity of not more than Ans. 0.294m/s -The polytrophic exponent approaches one with the change in the process____approaches
zero
-In air conditioning and ventilation standards, as the air enters into living zone, the distance Ans. Temperature
above the floor is -The change in kinetic energy of a process ___ as the change in temperature of the process
Ans. 1.603mm path increases
-Carbon dioxide concentration in air when measured 910 mm above the floor shall be Ans. Increases
estimated -The final temperature of an isobaric process if the system work in a _____Quantity and the
Ans. 1000ppm Surrounding heat transfer to the system
-The useful refrigerating effect equals to 211kj/mm Ans. Positive
is Ans. Tons of refrigeration -As the volume of an isothermal expansion process approaches its first value the rate of
-Another name for the fuel oil suction strainer; change in the system pressure.
Ans. Cold strainer Ans. Decreases
-In a manually controlled feed water heater system the proper water level is maintained -As a fluid flow through a pipe , their potential
by Ans. Operating the feed check valve energy Ans. Decreases
-The actual refrigerating capacity of the compressor -The heat of compression___ as suction vapor becomes super
Ans. Always less than the theoretical capacity heated Ans. Increases
-The ratio of the actual displacement of the compressor to its piston displacement -The heat content of the refrigeration cycle____ when a liquid- suction line heat exchanger is
Ans. Total or real volumetric efficiency installed
-Cyclone super heater are located in the Ans. Remains the same
Ans. Steam and water drum -The COP of refrigeration cycle___ with subcooling
-The purpose of the blow valve is Ans. Increases
Ans. Removes oil and scum from surface of water
-The purpose of the salinometer cock is to -Which defrost method is commonly used small commercial application where the forms on
Ans. Obtain a sample of boiler water for testing the evaporator surfaces?
-Salt in boiler water usually contains from Ans. Hot gas
Ans. Leaky condenser tubes -Which of the following liquid-chilling evaporator types incorporates overflow heat
-Before blowing down a boiler always exchangers?
Ans. Open skin valves Ans. Baudelot
-The mass flow rate produced by the compressor is -Which of the following is not a unit of energy?
Ans. Watt
-The unit of special heat are: Ans. None of the above
Ans. Btu/lbm.oF -Which of the following is not category of refrigeration application?
-Phase change process are constant________process. Ans. Transportation air conditioning
Ans. None of the above -Which of the following is not an industrial application
-Which of the following sequences accurately indicates the responses that occur when heat Ans. Meat display cases
is transferred from a gas? -Which of the following variable is not most often maintain by a commercial air
Ans. Sensible heat of vapors, latent heat of vaporization, sensible heat of liquids, latent heat of conditioning unit?
fusion, sensible heat of fluids Ans. Temperature
-Btu/lbm are units of -Providing clean, filtered air for trouble free operation of equipment and instrumentation is
Ans. Specific enthalpy a function of?
-As a liquid changes phase to vapor, its enthalpy Ans. Industrial air conditioning process
Ans. Increases -______ is one of the most common application of mechanical refrigeration.
503. As a liquid changes phase to a vapor , its Ans. Preservation of perishable commodities.
entropy Ans. Decreases -The air conditioning systems for cabins on a luxury ocean liner belongs to the ____ category.
-The vaporization process that occurs at temperatures below the triple point of a Ans. Commercial air conditioning
substance is_____ Mechanical refrigeration system makes it possible
Ans. Sublimation to; Ans. All of the above
-The Vaporization process that occurs when the vapor pressure of a substance is equal to the -As the liquid changes to a vapor. The enthalpy
atmospheric pressure is: Ans. Increases
Ans. Boiling -Which of the following systems has the most complicated oil return system?
-As the pressure of a vapor increases, the amount of work increases and its Ans. Direct staged
enthalpy__ Ans. Increases -Which of the following process does not after the kinetic energy level of a
-The entropy of R-134a in a saturated liquid phase at 40 psi is substance? Ans. Fusion
approximately Ans.21 BTU/lb-oF -200 degrees F is equal to
-The latent heat of vaporization of R-134a in its saturated vapor phase at 0 degree C Ans. None of the above
is approximately. -An evaporator in a refrigeration unit makes use of which heat transfer mode?
Ans.196.7Kj/Kg Ans. All of the above
-The condenser of a commercial display cause that is located within the unit is called -When a service technician places his/her hand on a suction line to check the operation of
Ans. B and C a system, he/she is using which heat transfer modes? Ans. Conduction
-Which of the following characteristics of early refrigeration systems which applies to
today’s units. -Energy added to a vapor is known as Ans. B and C
Ans. Relative expensive -The rate of changes velocity is called:
-Mechanical refrigeration system make it possible Ans. Acceleration
in. Ans. All of the above -Which of the following devices should be used to measure a pressure of 90kpa?
-_______is method used to reduce the dehydration of sensitive fruits and vegetable during Ans. All of the above
the chilling process -The height of the mercury column in a barometer placed on a mountain with a local pressure
Ans. Water chilling of 12.5 psi will be?
-Which of the following is not a step in preparing vegetables for frozen Ans. All of the above.
storage? Ans. Coating with syrup -A compound gauge measuring a pressure of 22.44 inches of mercury is equivalent to an
-Loss of food juices by osmosis is a consequence of the ______ freezing absolute pressure of
process. Ans. Immersion Ans. 11.0 Psia
-Heat transfer that occurs primarily by conduction is used for ____ freezing -The electric meter on a home or building measure the amount of that was consumed over the
process. Ans. All of the above billing period.
-A disadvantage of sharp freezing is Ans. Energy
the Ans. All of the above -Which of the following is not a method of food preservation?
-Frozen storage chamber temperature set points are Ans. Grilling
usually. Ans. -20.5 degree C -Which of the following is not plant, animal or fungi?
-Which of the following is not a unit of Ans. Enzyme
density? Ans. Lb/ft3 -Which of the following refrigerant group denote a more toxic vapor?
-Wrapping and refrigerating food products extends the storage life Ans. B3
of Ans. Meat -What is the molecular weight of helium in 3600 Btu lb K?
-The approximate life of strawberries is. Ans. 4.0 lb mol
-In an isobaric process changes in pressure can be caused by changes in. Ans. Increase
Ans. None of the above -As the water temperature entering a water-cooled condenser decrease, the power drawn by
-Boyles law states that pressure and volume changes in gas process is ___related. the compressor.
Ans. Inversely Ans. Decrease
-As a gas is heated in an isothermal gases the volume -Which of the does not increase the volumetric efficiency of a compressor?
Ans. Increases Ans. Decreasing the discharge pressure
-As a gas is heated in an isobaric gases the volume -What is the compressor operating with a 30 psig suction pressure and a discharge
Ans. Decreases pressure? Ans. 3.13
-As a gas is heated in an isometric gases the volume -As the length of the tubes in a chiller barrel increases, the pressure drop across the inlet and
Ans. Remains constant outlet of the barrel:
-As the volume of a gas decreases, its specific gas Ans. Increases
constant Ans. Remains constant -_____ decreases the volumetric efficiency of a compressor
-Vapor do not behave as ideal gases because they Ans. Decreasing the suction pressure
experience Ans. Friction -The mechanical efficiency of a compressor_____ as the compressor load increases.
-The unit of latent heat of vaporization are: Ans. Remain the same
Ans. None of the above -The saturated suction temperature used in the selection of a compressor corresponds to the
-The unit of latent heat of fusion are: pressure of the vapor at the
Ans. Btu-lbm Ans. Inlet of the compressor
-A smaller temperature difference between the refrigerant in the liquid line and that in -A smaller temperature difference between the refrigerant in the liquid line and that in the
the evaporator___ the mass flow rate of the refrigerant needed per ton of refrigeration evaporator ____ the mass flow rate of the refrigerant needed per ton of refrigerating
effect. Ans. Decreases effect. Ans. Decreases
-Oil separator are used in system where: -The difference between the saturation pressure in the evaporator and that in the condenser___
Ans. All of the above as the suction temperature decreases.
-Which evaporator design incorporates secondary surface heat transfer? Ans. Increases
Ans. Finned-tube -The COP and the efficiency of a refrigeration cycle____ as the vaporizing temperature
-Eutectic filled plate evaporator have a higher_____ than evacuated plate evaporators: increases
Ans. All of the above Ans. Increases
-Which evaporator circuit configuration is less effective when used in expansion load -The volume of vapor that the compressor must handle per minute per ton____ as the
applications. vaporizing temperature increases.
Ans. Cross-flow Ans. Decreases
-Which of the following evaporators feed methods produces the greatest surface without the -The quantity of heat rejected at the condenser per unit capacity per minute____ as the
used of a liquid pump. vaporizing temperature decreases.
Ans. Flooded Ans. Decreases
-What type of force convection unit cooler is used in vegetable storage application that require -As the amount of scale on the refrigerant tubes of an evaporative condenser increases, the
a velocity of approximately 120m/min? scale formation of hot surfaces
Ans. Medium velocity Ans. Decreases
-Which of the following is not a hydrocarbon refrigerants -As the cycles of concentration in a tower or evaporative increases, the scale formation of hot
Ans. inane surfaces:
-Which of the following refrigerants is most hazardous? Ans. Increases
Ans. R-717 -The____ can only be effectively employed in industrial process that has constant refrigeration
-A mixture of two or more refrigerants is called load.
a___ Ans. Zoetrope Ans. Hand expansion valve
-Dessicants are used to remove___ from refrigerants -The device used to transfer forces across a sealed boundary is called:
Ans. Moisture Ans. Diaphragm
-As an oil-miscible refrigerant mixes with oil, the viscosity of the oil___ -The device used to measure the effects of a large pressure drop across the evaporator is
Ans. Decreases called a
-The society that sponsors research on refrigerants is called: Ans. External equalizer
Ans. ASHRAE -The mass flow rate of refrigerant per unit capacity____ as the condensing pressure
-The scale factor for hard water used in a condenser is: decreases.
Ans. 0.002 Ans. Decreases
-As the altitude of a forced air-condenser increases, the surface are of the coil or volume -It consists of a tapered glass tube set vertically in the fluid or gaseous piping system with its
flow rateof the fan must: large end at the top and a metering float free to move vertically in the tube
Ans. Rotameter
-Also termed as monitor light, it indicates which among a number of normal conditions of a Branch of which is larger than the run. Ans. Bull head tee
system or device exist.
Ans. Pilot light A larger pipe or drum into which each group of boiler is connected: Ans. Header
-Which of the following is not a caliper?
Ans. Feeler gage In pipe identification, the color fro pipe used for electricity: Ans. Light orange
-An underground formation that contains sufficient saturated permeable material to yield
significant quantities of water. Has the same equipment as the refrigerating circuits each with a condenser, evaporator, and
Ans. Aquifer a pressure imposing elements, where the evaporator of one circuit cools the condenser of the
-Chemical used to speed up sewage sedimentation: other circuit. Ans. Cascade
Ans. Lime
-Power expenditure when a current of the one ampere flow across two points having a voltage The temperature of air to be used for comfort cooling shall be in the range of: Ans. 20 deg
drop of one volt. to 23.3 deg
Ans. Watt
-Prandtl number for air is generally in the order of A valve held closed by a spring or other means and designed automatically relieve pressure
Ans. 0.70 in excess of its settling. Ans. Pressure relieve device.
Relative humidity of air to be used in standard air cooling: Ans. 55%-60%
A refrigerating system in which the pressure-imposing element is mechanically operated: A refrigerants which should not be used in public assembly occupancies. Ans. Group
Ans. Compression 3 refrigerants
A cushioning device all the end of a trolley, bridge or other moving parts of a crane
operating on rails to minimize shock Fire involving flammable liquids and gases. Ans. Class B fires
in the event of collision: Ans. Buffer
Mechanics of water or other liquids whether at rest or motion. Ans. Hydraulics
A boom type mobile crane mounted on endless track or treated belts: Ans. Crawler crane
Measure the pressure of water discharging from a nozzle by having its open end on the
An apparatus for raising or lowering a load by the application of a pulling force but does water and the end connected to a manometer. Ans. Pitot Tube
not include a car or platform riding in guides: Ans. Hoist
A vertical turbine pump with the pump and the motor closed coupled and designed to
Most widely used industrial pressure gage applied to both pressure and vacuum: Ans. be installed underground. Ans. Submersible pump
Bourdon tube gage
Which of the following is a type of deep well pump. Ans. All of the above.
Hydrocarbons found in liquefied petroleum gas: Ans. All of the above
Steel pipe coated with zinc to resist corrosion. Ans. Galvanized pipe
Which of the following is not a solid fuel: Ans. Tar
A fitting with a number of branches in the connecting the smaller pipes. Ans. Manifold
Amount of cooling produced by 2000 lbs of ice in melting over a period of 24 hrs.: Ans. B
and C The ratio of peak load to the Average load is termed as _____ in variable load nomenclature.
Ans. Load factor
It smoothens the flow due to the nature of flow of the liquid from a reciprocating pump:
Ans. Air Chamber A heat exchange device used to provide heat transfer between exhaust gases and the air
prior to its entrance to the combustor. Ans. Regenerator
Science of force exerted by water in motion: Ans. Hydrodynamics
In a sensible heating process, the moisture content: Ans. Remains constant
Lowest permissible water level of a boiler without internal furnace: Ans. 1/3 height of shell
Flow on both sides on a normal shockwave: Ans. Supersonic on one side, Subsonic on
In pumps, it transmits power from the driver to the impeller: Ans. Shaft the other

Cavitation occurs when the pressure at any point inside a pump drops below the vapor pressure A change of phase directly from vapor to solid, without passing through the liquid state:
corresponding to the temperature of liquid. Its effect include: Ans. All of the above Ans. Deposition

Positive displacement pump consisting of a fixed casing containing gears, cams, screws, vanes, What reaction occurs when the enthalpy of the product is less than the enthalpy of
plungers or similar elements actuated by rotation of the drive shaft. Ans. Rotary pumps the reactants? Ans. Exothermic
Refrigerant used in passenger aircraft bins. Ans. Air
A well designed engine has a volumetric efficiency within the ranged: Ans. 75%-90%
It refers to atoms of the same atomic number but differ in atomic masses and
It is referred to as the maximum continuous power available from a hydroelectric power molecular weights. Ans. Isotopes
plant even under the most adverse hydraulic condition: Ans. Firm power
Piston rings are normally made of: Ans. Cast Iron
There are how many feed water lines connected to the boiler? Ans. Two
Two isothermal and two reversible adiabatic process comprise a: Ans. Carnot cycle
It is the difference in pressure as measured above or below the atmospheric pressure.
Ans. Draft In terms of viscosity, density can be expressed as: Ans. Dynamic viscosity/ kinematic viscosity

The compression ratio of a gas turbine is in the range of 5-8. Ans. 5 to 8 Compressor often used in supercharging Diesel engine: Ans. Root blower type

The average pH of a normal rainfall is generally: Ans. Slightly less than 7 In treating a person whose eyes accidentally got in contact with Freon use: Ans.
Sterile mineral oil
Another term used for liquid valve. Ans. King valve
The coefficient of performance of a domestic (local) refrigerator is always: Ans. Greater A device in vapor compression refrigeration system whose primary function is to meter
than one the flow of
Refrigerant to the evaporator: Ans. Thermostatic expansion valve
A belt-Coleman cycle is a reversed : Ans. Joule cycle
From the mathematical perspective, a thermodynamic property is: Ans. A point function
In an air compressor system, the function of a receiver is to: Ans. Collects water and
grease suspended in air In the production of beer, a raw materials called yeast is added in the: Ans. Starting tubs

General layout plan for each floor drawn to scale should not be less than scale of: Ans. 1:200 Purpose is to keep the moisture from entering the system. Ans. Dehydrator

Highest vapor drop in a vapor compression refrigeration cycle occurs in the: Ans. Medium pressure as applied to valves and fittings implies that a working pressure of _____ is
Expansion valve suitable enough. Ans. 862-1200psi

Which of the following pairs represent the two broad classification of lubricating oils? A liquid-vapor mixture with a dryness factor is allowed to absorb heat. Which of the
Ans. Straight and Additives following is likely to occur? Ans. Increases

A form of misalignment between the pump and the driver shaft wherein the shafts Work rooms referring to the maintenance shop and machine rooms shall be _____ in the
are concentric but not parallel. Ans. Angular Misalignment height from floor to the ceiling? Ans. 3,000mm

In the field of metal corrosion, it is the process wherein it exhibits in the quality Defined as a passageway made of sheet metal or other suitable material not necessarily
deterioration of metals: Ans. Passivation leak tight, used for conveying air or other gases at low temperature. Ans. Duct

It is the ratio of the density of liquid substance to the density of water at standard conditions: As a good practical rule, the foundation depth may be taken as _____ times the engine stroke,
Ans. Specific gravity the lower factor for well-balanced multi-cylinder engine and the higher factor for engines
with fewer cylinder s or on less firm soil: Ans. 3.2 to 4.2
Term associated with an increase on pressure on a pipe caused by sudden velocity
decrease. Ans. Water hammer What is the suggested maximum permissible dos (MDP) of a gamma ray exposed
for individuals now working in a nuclear environment in rem year? Ans. ½
A process which takes place without change in volume. Ans. Isochoric
Most commercially petroleum lubricating oil deteriorates starting from temperature of:
Weir refers to an opening: Ans. Having partial full flow Ans. 200 F

The Westphal balance is a laboratory instrument used to: Ans. Specific gravity States that the external pressure applied to a confined liquid increases the pressure of
every point in the fluid by an amount equal to the external pressure: Ans. Pascal’s Law
Generally, permissible velocity of water flowing through concrete tunnel is: Ans. 4-5 m/s
The average fuel-oil temperature range of the oil in the discharge line to the boilers. Ans. The chemical formula of R-12 or dichlorofluoromethane. Ans. CCL₂F₂
180-200 F
Which of the following is not desirable property of a refrigerant? Ans. Low
Boiler gage glasses should be blown down: Ans. At the start of every watch thermal conductivity

The lowest portion of the storage basin where the water is not drawn: Ans. Dead Refrigerant widely used in room air conditions. Ans. R-22

storage Which among the following do not measure relative humidity? Ans. Piezometer What occurs when pumps are connected in parallel? Ans. Increase discharge, same head

Water behind the dam of a hydro electric power plant? Ans. Pondage Converts energy Which of the following is not a cause of cavitation? Ans. Low velocity

of water to mechanical energy: Ans. Turbine Pipe attached to the penstock to be able to let the water be at atmospheric pressure. Ans.
Surge chamber
Ratio of average load to the rating of the equipment supplying the load. Ans. Plant factor
Which of the following is not considered as gaseous fuel? Ans. Bunker
Ratio of actual maximum demand load to the connected load. Ans. Demand factor
Ratio of the density of a liquid to the density of water. Ans. Specific
A device which preheats feed water by utilizing the heat of the flue gases. Ans. Gravity Measurement of randomness or disorder. Ans. Entropy
Economizer Study of the proportion of water vapor content of air. Ans. Air conditioning
Energy cannot be created nor destroyed but can only be transformed from one form to
Actual temperature of air. Ans. Dry bulb temperature another. Ans. First law of thermodynamics

For a machine foundation with class A mixture what is the proportion of cement, sand In standard Otto cycle when the compression ratio is increased then the thermal
and gravel? Ans. 1:2:4 efficiency will: Ans. Increase

Component included in the proximate analysis of fuel. Ans. Ash, moisture, Volatile Heat energy produced by the movement of molecule within a substance caused by
matter, fixed carbon its temperature. Ans. Internal energy

The prime mover must have its main steam line: Ans. In loops The law which states that one cannot operate a 100% efficient machine. Ans. 2nd law
of thermodynamics
Intercooler are primarily used with: Ans. Gas compressors
In relation to brake power the instrument used to measure torque: Ans. Dynamometer
In the psychrometric chart, the diagonal lines represents: Ans. Wet bulb
Ratio of the radiation of an actual body to the radiation of the black body. Ans.
temperature Moderator in certain types of nuclear reactors. Ans. Heavy water
Emittance Refrigerant used in ice plant. Ans. Ammonia
The main advantage of turbulous boilers. Ans. Steam pressure can be raised in short time
Measures the pressure of water discharging from the nozzle by having its open end in
The law which states that the entropy of all perfect crystalline solids is zero at absolute the water and the other end connected to a manometer. Ans. Pitot tube
zero temperature. Ans. 3rd law of thermodynamics
Defined as a wall designed to prevent the spread of fire having a fir resistance for four
Operates between two constant temperature reservoir. Ans. Carnot engine hours. Ans. Fire wall

Specific measurements of moisture content of air. Ans. Degree of saturation The work done per unit charge when the charge is moved from one point to another?
Ans. Potential at a point
Simultaneous on site generation of electric energy and steam from the same plant output. Ans.
Co-generation In the relation of PVⁿ is constant, what value makes the process isobaric? Ans.

Instrument used for pressure readings. Ans. Manometer Zero In actual gas behavior, molecular collisions are: Ans. Inelastic An ideal

Instrument used to analyze gases. Ans. Orsat Apparatus refrigeration should have: Ans. Low freezing point
The immediate undesirable products from the petroleum based lubricating oil subjected Difference between the indicated power and the brake power. Ans. Friction power
to high pressure and temperature is referred as: Ans. Carbon Dioxide
In Diesel power plant, its purpose is to reduce the weight-to-power ratio: Ans. Supercharging
A rotary dynamic pump, fluid is at no time confined by moving boundaries in its
passage through the pump. Ans. Non-positive displacement pump Change of phase from solid to gas. Ans. Sublimation

Also known as tangential or Pelton wheel. Ans. Impulse turbine The memory lost when operating power is removed. Ans. Volatile memory

Term given to a chimney made of steel. Ans. Stack A refrigeration system using direct method in which refrigerant is delivered to two or
more evaporators in space room or in refrigerators. Ans. Multi-pressure
An integral part of reaction turbine used to recover energy head. Ans. Draft tube
The carbon dioxide concentration of air in all rooms when measured 910mm above the floor
Based on good practice, the vertical distance from the floor soil level to the top edge of the space not exceed: Ans. 100ppm
foundation must be around _____ as minimum distance in machiner foundation. Ans. 6 inches
Consists of helmets shall be removed immediately after having been used or the seal
The geometric view factor for a black body is always. Ans. One broken, must be removed atleast every: Ans. 2 years
For every 1000ft ascent, there is a corresponding pressure decrease approximately. Ans. 1 in
of Hg. Pump whose purpose is to increase the effective water pressure by sucking water from
public service main or private use water system: Ans. Booster pump
Used for impounding water storage and for creating head for the power plant. Ans. Dam
A small fitting with a double offset, or shaped like the letter C with the ends turned out.
The temperature at which the phase change takes place at a given pressure. Ans. Ans. Cross-cover
Saturation pressure
Material interposed between two relatively moving machine elements to reduce wear and tear:
Distinguishes the system from its surroundings may be at rest or in motion. Ans. Ans. Lubricant

Boundary Quality is a measure of: Ans. Dryness Measure temperature by electromotive force: Ans. Thermocouple

The highest pressure under which distinguished liquid vapor phase can exist in equilibrium: According to SAE which of the following is a type of lubricating oil? Ans. All of the above
Ans. Critical pressure
A device that information in one form of an instrument signal and transmits an output
In a thermodynamic gaseous substance, the relationship between P-V-T given by the: signal on another form: Ans. Digital
Ans. Equation of state
A device whose function to pass on information in an unchanged form or in some
A correction factor for gas approaching real gas behavior: Ans. Compressibility factor modified term. Ans. Switch

Furl production process wherein heavy oil is changed into gasoline by means of a high A device that receives information on one form of one or more physical quantities modifies the
pressure, high temperature and longer exposure time. Ans. Thermal cracking information and/or its form and produces a resultant output signal. Ans. Transducer

A shut off valve for controlling the flow of refrigerant. Ans. Stop valve Which of the following is not a requirement in designing pipe installations? Ans.
Galvanized pipe should be used for steam
Measured by the amount of its pressure below the prevailing atmospheric pressure.
Ans. Partial vaccum Part of a valve used to guide and support valve stem: Ans. Bonnet

A relief valve that can positively lift the disc from its seat at least 1.5 mm when there is In piping installations, the color of pipe tube used for water is: Ans. Green
no pressure in the boiler. Ans. Water relief valve
The color code for pipes used in communication. Ans. White
Vapor produced in the vaporization of a small amount of liquid refrigerant downstream of
the expansion valve. Ans. Flash gas Which of the method is used for classifying coal. Ans. All of the above

Product of the rate of evaporation and the factor of evaporation. Ans. Equivalent evaporation Alcohol frequently considered as fuel for internal combustion engine. Ans. Ethyl alcohol
Measure of resistance to flow. Ans. Viscosity PV = mRT
280(2) = m(8.314/44)(40+273)
Converts Solar energy to electrical energy. Ans. Photovoltaic cell m =9.46 ANS

Which of the following is not an instrument used to measure flow rates? Ans. Velometer 507. A thermal power plant has a heat rate of a 11,363 Btu/kw-hr. Find the thermal
efficiency of the plant.
Refrigeration follows what cycle? Ans. Reversed Carnot cycle Solutionj:
eth = 3412/Heat rate = 3412/11,363 = 0.3_ANS.
An evaporator constructed on pipe tubing. Ans. Expansion coil
508. What is the hydraulic gradient of a 1 mile, 17 inches inside diameter pipe when
Reinstalled or second hand boilers shall have a minimum factor of safety of? Ans. 6 3300 gal/min of water flow with f = 0.03.
A line that shows the rotation of the consumption and the load a steam turbine generator. Solution:
Ans. Willans Line V = (3300/7.481)/(π/4)(17/12)2(60) = 4.66
ft/s L = 1 mile = 5280 ft
Instrument that indicate the percentage of carbon dioxide in flue gases in a power plant. hL = fLv2/2gD = 0.03(5280)(4.66)2/2(32.2)(17/12) = 37.7
Ans. Ranarex indicator ft Hydraulic gradient = 37.7/5280 =6.51x10^-3ANS.

For real process, the net entropy change in the universe is: Ans. Positive 509. Find the loss of head in the pipe entrance if speed of flow is 10 m/s.
Solution:
The sum of the internal energy and the product of the pressure and specific volume. Loss at entrance = 0.5(v2/2g) = 0.5[102 / 2(9.81)] = _2.599_ANS.
Ans. Enthalpy
Wet material, containing 220% moisture (dry basis) is to be dried at the rate of 1.5 kg/s in
Ratio of the maximum demand of the system to the rated capacity of the system. a continuous dryer to give a product containing 10% (dry basis). Find the moisture
Ans. Utilization factor removed, kg/hr.
Solution:
Source of thermal energy. Ans. All of the above Solid in wet feed = solid in dried product
[1/(1 + 2.2)](1.5) = [1/(1 + 0.1)](x)
Cycle that has two isentropic and two isometric process. Ans. Otto cycle x = 0,5156 kg/s (total dried product)
Moisture removed = 1.5 – 0.5156 = 0.984 kg/s ANS.
Cycle that has two isentropic and two isobaric process. Ans. Brayton cycle
510. Copra enters a dryer contacting 70% moisture and leaves at 7% moisture. Find the
504. Two kilogram of gas is confined in a 1 m3 tank at 200 kpa and 88°C. What type of gas moisture removed on each pound of solid in final product.
is in the tank? Solution:
Solution: Solid in wet feed = solid in dried
PV=mRT product 0.3x = 1
200(1) = 2(8.314/M) (88+273) x = 3.333 lbs
M=30 1= 0.93y
Therefore: gas is Ethane Y = 1.07527 lb
Moisture removed = x – y = 3.333 – 1.07527 = _2.2577 ANS.
505. Find the enthalpy of helium if its internal energy is 200 KJ/kg.
Solution: 511. A 1 m x 1.5 m cylindrical tank is full of oil with SG = 0.92. Find the force acting at
R = 8.314/4 = 2.0785 the bottom of the tank in dynes.
K = 1.667 for helium Solution:
Cp = k R/(k-1) = 2.0785/(1.667 – 1) = 3.116 KJ/kg- P = w h = (0.92 x 9.81) (1.5) = 13.5378 kpa
F = PA = 13.5378(π/4 x 12) = 10,632 KN = 10,632.56 N x 10,000
K h/ = Cp/Cv
dynes/N F =106325600ANS.
/200 = 5.195/3.116
512. Find the pressure at the 100 fathom depth of water in kpag
= 1.667 ANS Solution:
H = 100 fathom x 6 = 600 ft
506. Compute the mass of a 2 m3 propane at 280 kpa and P = w h = (600/3,281)(9.81) =1793.9652 ANS.
40°C. Solution:
Propane is C3 H8 --------- M = 12(3) + 8(1) = 44
513. Find the depth in furlong of the ocean (SG = 1.03) if the pressure at the sea bed is BP = 40 (0.90) = 36 kw
2,032.56 kpag. Wc = 36(1-0.20) = 28.80 kw
Solution: COP = RE/Wc
P=wh 6 = RE/28.80
2,032.56 = (1.03 x 9.81) h RE = 172.8/3.516 = _49.15_ANS.
H = 201.158 m x 3.281ft/m x 1 yd/3ft x 1 furlong/220 yd = 1 furlong
520. A 23 tons refrigeration system has a heat rejected of 100 kw. Find the energy efficiency
514. Find the mass of 10 quartz of water. ratio of the system.
Solution: Solution:
V = 10 quartz x 1gal/4quartz x 3.785li/gal x 1m3/1000li QR = RE + Wc
V= 0.0094625 x 10-3 m3 100 = 23(3.516) +
W = m/V Wc Wc = 19.132 kw
1000 = m/0/0094625 x 10-3 COP = RE/Wc = (23 x 3.516) / 19.132 = 4.23
m = _9.4625x10-3 ANS. EER = 3.412 COP = 3.412(4.23) = 14.43 ANS.

515. Find the mass of carbon dioxide having a pressure of 20 psia at 200°F with 10 ft3 521. If the energy efficiency ratio of the refrigeration system is 12.6, what is the COP of
volume. Solution: the system?
PV = m R T Solution:
(20 x 144)(10) = m (1545/44)(200 + EER = 3.412 COP
12.6 = 3.412 COP
460) m = 1.24 ANS.
COP = 3.69 ANS.
516. Find the heat needed to raise the temperature of water from 30°C to 100°C with 60%
quality. Consider and atmospheric pressure of 101.325 kpa. Use the approximate enthalpy 522. An air compressor has a power of 40 kw at 4% clearance. If clearance will increase to
formula of liquid. 7%, what is the new power?
Solution: Solution:
At 100°C The power of compressor will not be affected with the changes in
hf = Cp t = 4.187(100) 418.7 KJ/kg clearance. Therefore the power will still be 40 kw.
hfg = 2257 KJ.kg
h2 = hf + xhfg = 418.7 + 0.60(2257) = 1,772.9 KJ/kg 523. What is the approximate value of temperature of water having enthalpy of 208
Q = 1(4.187)(100-30) + 1(1772.9 – 418.7) = 1647.29 ANS. Btu/lb? Solution:
h = °F – 32
517. Find the enthalpy of water at 212°F and 14.7 psi if the dryness factor is 30%. Use 208 = °F -32
the approximate enthalpy formula of liquid. °F = 240 ANS.
Solution:
hf = (°F – 32) = (212 – 32) = 180 524. An Otto cycle has a compression ratio of 8. Find the pressure ration during
Btu/lb hfg = 970 Btu/lb compression. Solution:
h = hf + xhfg P1V1k = P2V2k
h = 180 + 0.3(970) = 471ANS. (V1/V2)k =
(P2/P1) rK = rp
518. An air compressor consumed 1200 kw-hr per day of energy. The electric motor driving rp = (8)1.4 = 11.2 ANS.
the compressor has an efficiency of 80%. If indicated power of the compressor is 34 kw,
find the mechanical efficiency of the compressor.
Solution:
Pim = 1200kw-hr/24 hrs = 50
kw BP = 50(0.80) = 40 kw
em = 34/40 = _0.85_ANS.

519. A refrigeration system consumed 28,800 kw-hr per month of energy. There are 20% of
energy is lost due to cooling system of compressor and motor efficiency is 90%. If COP of
the sustem is 6, find the tons of refrigeration of the system.
Solution:
Pim = 28,800/(24 x 30) = 40 kw
525. A diesel cycle has a cut off ratio of 2.5 and expansion ratio of 4. Find the clearance of 531. A perfect gas has a value of R = 58.8 ft-lb/lb-R and k = 1.23. if 20 Btu are added to 10
the cycle. lbs of this gas at constant volume when initial temperature is 90°F, find the final temperature.
Solution
526. Determine the atmospheric pressure at a location where barometric reading is 740 mm Q = m cv (t2 – t1)
Hg and gravitational acceleration is g = 9.7 m/s2. Assume the temperature of mercury to be
10°C, at which the density is 13,570 kg/m3. cv = R / (k -1) = = 0.29086 Btu/lb-F
Solution: 20 = 10(0.29086)(t2 – 90)
P = (wg)h = (13,570 x 9.7)(9.74) (1Kpa/1000 N/m2) = 1242,41 ANS. t2 = _96.88 ANS.
527. The barometer of a mountain hiker reads 930 mbars at the beginning of a hiking trip and 532. Ammonia weighing 22kgs is confirmed inside a cylinder equipped with a piston has
780 mbars at the end. Neglecting the effect of altitude on local gravitational acceleration,
determine the vertical distance climb. Assume g = 9.7 m/s2. an initial pressure of 413KPa at 38 C. If 3200KJ of heat is added to the ammonia until its final
Solution: pressure and temperature are 413KPa and 100oC respectively, what is the amount of work
P1 – P2 = w h done by the fluid in KJ?
(0.93 – 0.78)(100 kpa/bar) = (1.2 x 0.00981) SOLUTION
h h = 1274.21 m Since the molecular weight of ammonia is 17. Then
h = 1274.21 (9.81/9.7) = _1288.66 ANS. R = 8.3143/M = 8.3143/17 = 0.489 KJ/kgoK
P1V1 = mRT1
528. The lower half of a 10 m high cylindrical container is filled with water and the upper 413V1 = 22(0.489)(38 + 273)
half with oil that has SG = 8.85. Determine the pressure difference between the top and V1 = 8.101 m3
bottom of the cylinder. P2V2 = mRT2
Solution: 413V2 = 22(0.489)(100 + 273)
V2 = 9.716 m3
P = Pw + Po = 9.81(5) + (0.85 x 9.81)(15) = 174.13 ANS. W = P(V1-V2) = 413(9.716 – 8.101) = 666.995ANS.
533. A tank contains 90ft3 of air at a pressure of 350 psig; if the air is cooled until its pressure
529. An ideal gas at 0.80 atmospheres and 87°C occupies 0.450 liter. How many moles are in and temperature decreases to 200 psig and 70oF respectively. What is the decrease in internal
the sample? (R = 0.0821 liter-atm/mole – K) energy?
Solution: SOLUTION
PV = nRT m = PV/RT = (200 + 14.7)(90)(144)/(53.342)(70 + 460) = 98.50
lbs for constant volume process:
(0.80 atm)(0.450 li) = n (0.0821 )(87+273)K P1/T1 = P2/T2
n = ___0.012___ANS. T2 = 70 + 460 = 530oR

530. A certain gas at 101.325 Kpa and 10°C whose volume is 2.83 m3 are compressed into a =
storage vessel of 0.31 m3 capacity. Before admission, the storage vessel contained the gas at a T1 = 900oR
pressure and temperature of 173.8 Kpa and 26°C, after admission the pressure has increased to
ΔU = mcv(T2 – T1) = 98.50(0.171)(530 – 900) =-6042.29ANS.
1171.8 Kpa. What should be the final temperature of the gas in the vessel in Kelvin?
534. A large mining company was provided with a 3m3 of compressed air tank. Air pressure
Solution:
in the tank drops from 700KPa while the temperature remains constant at 28oC. What
Solving for the mass of gas which is to be compressed:
percentage has the mass of air in the tank been reduced?
PV = mRT
SOLUTION
101.325(2.83) = m1R(10 + 273)
m1 = 1.01325/R Percent mass reduced = (700 – 150)/700 =11/14ANS.
Solving for the mass of gas initially contained in the 535. A 4m3/hr pump delivers water to a pressure tank. At the start, the gauge reads 138KPa
vessel: PV = mRT until it reads 276KPa and then the pump was shut off. The volume of the tank is 180liters. At
137.8(0.31) = m2R(26 + 273) 276KPa the water occupied 2/3 of the tank volume. Determine the volume of the water that
m2 = 0.14286/R can be taken out until the gauge reads 138KPa.
Solving for the final temperature: SOLUTION
m3 = m1 + m2 Consider the air
m3 = 1.01325/R + 0.14286/R = 1.156/R pressure: V2 = 1/3 (180)
1171.8(0.31) = (1.156/R)RT3 = 60liters P1V1 = P2V2
T3 = __314.24 ANS. (138 + 101.325)(V1) = (276 + 101.325)(60)
V1 = 94.59 liters
Amount of water to be removed = 2/3 (180) – (180 – 94.59) =34.59ANS.
536. A refrigeration plant is rated at 15tons capacity. How many pounds of air per hour will it h = hf + xhfg = 814.93 + 0.17614(1972.7) =1162.4 ANS.
cool from 70 to 90oF at constant pressure. 541. Mixture with 70% quality at 500KPa is heated isothermally until its pressure is 300KPa.
SOLUTION Find the heat added during the process.
Tons of refrigeration = mcp(t2 – t1)/12000 At 500KPa: sf = 1.8607, sfg = 4.9606
At 300KPa and 151.86oC s = 7.088 KJ/kg
SOLUTION
s1 = sf + xsfg = 1.8607 + 0.70(4.9606) = 5.333 KJ/kg
m =37500ANS. s2 = 7.088 KJ/kg
537. An air standard engine has a compression ratio of 18 and a cut – off ratio of 4. If the Q = T(s2 - s1) = (151.86 + 273)(7.088 – 5.333) =745.63ANS.
intake pressure and temperature are 100KPa and 27oC, find the work in KJ per kg. 542. A tank contains exactly one kilogram of water consisting of liquid and vapor equilibrium
SOLUTION
at 1MPa. If the liquid contains one – third and the remaining is vapor of the volume of the
tank, what is the enthalpy of the contents of the tank?
At 1MPa: vf = 0.0011273 vfg = 0.19444
hf = 762.81 hfg = 2015.3
T1 = 27 + 273 = 300oK SOLUTION
Process 1 to 2 is isentropic procees: Let V = total volume of the tank
T2 = T1(V1/V2)k-1 = 300(18)1.4-1 = 953.301oK T2 = T1(V1/V2)k-1 = 300(18)1.4 – 1 = 953.301oK
Process 2 to 3 is constant pressure process: Process 2 to 3 is constant process:
T3/T2 = V2/V3 = rc
T3 = 953.301 (4) = 3813.205oK
QA = mcp(T3 – T2) = 1(3813.205 -953.301) = 2860 KJ/kg
W = (.5531)(2860) =1581.87ANS. T3 = 953.301(4) = 3813.205oK
538. Determine the air – standard efficiency of an engine operating on the diesel cycle with QA = mcp(T3 – T2) = 1(3813.205 - 953.301) = 2850 KJ/kg
clearance of 6% when the suction pressure is 99.97KPa and the fuel injected for 7% of the W = (0.5531)( 2850) =1576.34ANS.
stroke. Assume k = 1.4. 543. Determine the air standard efficiency of an engine operating on the diesel cycle with
SOLUTION clearance of 6% when the suction pressure is 99.97KPa and the fuel is injected for 7% of the
V3 – V2 = 0.07VD stroke. Assume k = 1.4.
V2 = 0.06VD SOLUTION
V3 - 0.06VD = 0.07VD V3 - V2 = 0.07VD
V3 = 0.13VD V2 = 0.06VD
rc = V3/ V2 = 0.13VD/0.06VD = V3 = 0.07VD + 0.06VD
2.167 rk = (1 + 0.06)/0.06 = 17.667 V3 = 0.13VD
rc = V3/ V2 = 0.13VD /0.06VD = 2.167
=0.621ANS. rk = (1 + 0.06)/(0.060) = 17.667
539. Steam at 2MPa and 250oC in a rigid cylinder is cooled until the quality is 30%. Find the
heat rejected from the cylinder. e= =0.621 ANS.
At 2MPa and 250oC: v = 0.11144m3/kg u = 2679.6 KJ/kg 544. A steam at 2MPa and 250oC in a right rigid cylinder is cooled until the quality is 30%.
At 2MPa, (saturated): vf = 0.0011767m3/kg, vg = 0.09963m3/kg uf = 906.44 Find the heat rejected from the cylinder.
KJ/kg, At 2MPa and 250oC: v = 0.11144 m3/kg u = 2679.6 KJ/kg
ufg = 1693.8KJ/kg At 2MPa, (saturated): vf = 0.0011767 m3/kg, vg = 0.09963 m3/kg
SOLUTION uf = 906.44 KJ/kg ufg = 1693.8 KJ/kg
Q=U2–U1 SOLUTION
U1 = 2679.6 KJ/kg Q = (u2 – u1)
U2 = Uf + xUfg = 906.44 + 0.30(1693.8) = 1414.58 KJ/kg u1 = 2679.6KJ/kg
Q = 1414.58 – 2679.6 =-1265.02ANS. u2 = uf + x ufg = 906.44 + 0.3(1693.8) = 1414.58KJ/kg
540. At 1.3MPa, mixture steam and water has an entropy of 3KJ/kg - oK. Find the enthalpy of Q = (1414.58 – 2679.6) = 1265.02 ANS
the mixture. 545. At 1.3MPa, mixture steam and water has an entropy of 3KJ/kg-oK. Find the enthalpy of
At 1.3MPa: sf = 2.2515, sg = 6.4953, hf = 814.93, hfg = 1972.7 the mixture.
SOLUTION At 1.3MPa: sf = 2.2515, sg = 6.4953, hf = 814.93, hfg = 1972.7
s = sf + xsfg SOLUTION
3 = 2.2515 + x(6.4953) s = sf + x(sg – sf)
x = 0.17614 3 = 2.2515 + x(6.4953 – 2.2515)
x = 0.17637 550. A throttling calorimeter is connected to the desuperheated steam line supplying steam to
h = hf + xhfg = 814.93 + 0.17637(1972.7) =1162.855ANS. the auxiliary feed pump on a ship. The line pressure measures 2.5MPa. The calorimeter
546. Mixture with 70% quality at 500KPa is heated isothermally until its pressure is 300KPa. pressure is 110KPa and 150oC. Determine the entropy of the steam line.
Find the heat added during the process. At 110KPa and 150.C: h2 = 2775.6KJ.kg
At 500KPa: sf = 1.8607, sfg = 4.9606 At 2.5MPa: hf = 962.11KJ/kg, hfg = 1841 KJ/kg, sf = 2.5547, sfg = 3.7028
At 300KPa and 151.86oC: s = 7.0888 SOLUTION
SOLUTION For throttling process: (h1 = h2)
For isothermal process, t1 = t2 h1 = h2 = hf + x hfg
s1 = sf + xsfg = 1.8607 + 0.7(4.9606) = 5.333 2775.6 = 962.11 + x(1841)
s2 = 7.0888 x = 98.5%
Q = T(s2 – s1) = (151.86 + 273)(7.0888 – 5.333) =745.96 ANS. s1 = sf + x sfg = 2.5447 + 0.985(3.7028) =6.19ANS.
547. A tank contains exactly one kilogram of water consisting of liquid and vapor in 551. Atmospheric pressure boils at 212oF. At the vacuum pressure at 24 in Hg, the
equilibrium at 1MPa. If the liquid contains one third and the remaining is vapor of the volume temperature is 142oF. Find the boiling temperature when the pressure is increased by 40psia
of the tank, what is the enthalpy of the contents of the tank? from the atmospheric.
At 1MPa: vf = 0.0011273 vfg = 0.19444 SOLUTION
hf = 762.81 hfg = 2015.3 P2 = 14.7 + 40 = 54.7 psia
SOLUTION P1 = -24(14.7/29.92) + 14.7 = 2.908 psia
Let V = total volume of tank By interpolation:
mL = VL/vL = (V/3) / 0.0011273 = 295.69 V
mv = Vv / vv = (2V/3) / 0.1944 = 3.429V

t2 =449.45ANS.
x= = ANS.
552. A certain coal has the following ultimate analysis:
548. Water substance at 70 bar and 65oC enters a boiler tube of constant inside diameter of
C=69% N2=5% H2=2.5% S=7% Determine the amount of
35mm. The water leaves the boiler tube at 50 bar and 700oK at velocity of 100m/s. Calculate
oxygen of the heating value of fuel is 26 961.45 KJ/kg.
the inlet volume flow(li/sec).
SOLUTION
SOLUTION From
steam tables:
At 70bar(7MPa) and 65oC Qh = 33 820C + 144 212 (H - ) + 9304S
v1 = 0.001017m3/kg 26961.45 = 33820(.69) + 144212 (0.025 – O/8) + 9304(0.07)
at 50bar(5MPa) and 700oK(427oC) O =0.035 ANS.
v2 = 0.06081m3/kg 553. A diesel engine consumed 945 liters of fuel per day at 35oC. If the fuel was purchased at
m1 = m2 15.5oC and 30oAPI at P29.00/li, determine the cost of fuel to operate the engine per day.
Q1/ v1 = Q2/ v2 SOLUTION

16

V1 = 1.672m/sec
Q1 = A x v = (π/4)(0.035)2(1.672) = ANS.
549. Steam leaves an industrial boiler at 827.4KPa and 171.6oC. A portion of the steam is
passed through a throttling calorimeter and is exhausted to the atmosphere when the
calorimeter pressure is 101.4KPa. How much moisture does the steam leaving the boiler
contain if the temperature of the steam at the calorimeter is 115.6oC?
At 827KPa(171.6oC): hf = 727.25 KJ/kg, hfg = 2043.2 KJ/kg
From table 3: At 101.4KPa and 115.6oC: h2 = 2707.6 KJ/kg
SOLUTION Cost = P29.00/li(935.44li) =27127.76ANS.
Let x = quality of steam entering the throttling calorimeter 554. A cylindrical tank 4m long and 3m diameter is used for oil storage. How many days can
h1 = h2 the tank supply the engine having 27oAPI with fuel consumption of 60kg.hr?
hfg + xhfg = h2 SOLUTION
727.25 + x(2043.2) = 2707.6 V = π/4 D2 h = π/4 (3)2(4) = 28.274 m3
x = 0.9692; y = 1 - 0.9692 = ANS.

Density of fuel = 0.89274(1000kg/m3) = 892.74kg/m3


w = m/V 0.14H2 + 0.070O2 = 0.14H2O
V = 60/892.74 = 0.0672 m3/hr 0.03CH4 + 0.060O2 = 0.03CO2 + 0.06H2O
Number of days = 28.274/0.0672 = 420.75hrs =17.5ANS. 0.27Co + 0.135O2 = 0.27CO2
555. A logging firm in Isabella operates a Diesel Electric Plant to supply its electric energy 0.265O2
requirements. During a 24 period, the plant consumed 250 gallons of fuel at 80oF and Actual O2 in product = 0.265 O2 – 0.006 O2 = 0.259 O2
produced 2900KW-hrs. Industrial fuel used is 30oAPI and was purchased at P30.00/li at 60oF. Molal A/F = 0.259 + 0.259(3.76) = ANS.
Determine the overall thermal efficiency of the plant. 561. A volumetric analysis of a gas mixture is as follows:
SOLUTION CO2: 12% N2: 80%
Qh = 41130 + 139.6xoAPI = 41130 + 139.6(30) = 45318 O2: 4% CO: 4%
KJ/kg 60oF = 15.6oC What is the percentage of CO2 on a mass basis?
80oF = 26.6oC SOLUTION
Converting to mass basis:
CO2 = 0.12 x 44 = 5.28
O2 = 0.014x32 = 1.28
N2 = 0.82x28 = 22.96
At 26.6oC CO = 0.02x28 = 0.56
mf = 250gal/24hrs x 3.785li/gal x 0.869kg/li x Total mass of product = 5.28 + 1.28 + 22.96 + 0.56 = 30.08 kg
1hr/3600sec mf = 0.00952 kg/sec % mass of CO2 = 5.28/30.08 =0.176ANS.
load = 2900/24 = 120.833KW 562. The following coal has the following ultimate analysis by weight:
C = 70.5% H2 = 4.5% O2 = 6.0% N2 = 1.0%
overall efficiency = = 0.28ANS. S = 3.0% ash = 11% moisture = 4%
556. The dry exhaust gas from oil engine has the following gravimetric analysis: A stocker fired boiler of 195000kg/hr steaming capacity uses this coal as fuel. Calculate
CO2 = 21.6% O2 = 4.2% N2 = 74.2% volume of air in m3/kg with air at 60oF and 14.7 psia pressure of boiler efficiency is 70% and
Specific heats at constant pressure for each component of the exhaust gas in Kcal/kgoC are: FE = 1.10.
CO2 = 0.203 O2 = 0.219 N2 = 0.248 SOLUTION
Calculate the specific gravity if the molecular weight of air is 28.97 kg/kg-mol.
SOULTION Theo. A/F = 11.5C + 34.5(H – O/8) + 4.3S
Converting the gravimetric analysis to volumetric:
CO2 = 0.219/44 = 0.004909 563. 11.5(0.705) + 34.5(0.045 – 0.06/8) + 4.3(0.03)
O2 of nitrogen in lb/= 0.042/32 = 0.001312
N2 = 0.742/28 = 0.026500 564. 9.53
0.032721 mols/kg-mol
Molecular weight = 1/0.032721 = 30.56kg/kg-mol Actual A/F = 9.53(1.3) = 12.389 kg air/kg fuel
SG = 30.56/28.97 =1.055 ANS.
557. A bituminous coal has the following composition:
C = 71.5% H = 5.0% O = 7.0% N = 1.3% S = 3% Ash =
7.6%
W = 3.4%
Determine the theoretical weight of nitrogen in lb/lb of coal.
SOLUTION
Theo. A/F = 11.5C + 34.5(H – O/8) + 4.3S
558. 11.5(0.715) + 34.5 (0.05 – 0.07/8) + 4.3(0.03)
559. 9.8 lb air / lb coal
N2 in air by weight = 76.8%
therefore:
theoretical weight of N2 = 0.768(9.8) =7.53ANS.
560. A gaseous fuel mixture has a molal analysis:
H2 = 14% CH4 = 3% CO = 27%
O2 = 0.6% CO2 = 4.5% N2 = 50.9%
Determine the air fuel ratio for complete combustion of molal basis.
SOLUTION
Chemical reaction with oxygen:
h3 = 340.49 KJ/kg
; mf = 23107.56 (12.389) = 286279.57 kg/hr
h4 = hf + vf (P2 – P1) = 340.49 + 0.00103(2500 - 50) = 342.98 KJ/kg

101.325(V) = 286279.57(0.287)(15.6 + 273)

V =234019.46 ANS. Efficiency =0.255 ANS.

565. 23.5 kg of steam per second at 5MPa and 400oC is produced by a steam generator. The 567. A two-stage air compressor air at 100 Kpa and 22⁰C discharges to 750Kpa. If
feedwater enters the economizer at 145oC and leaves at 205oC. The steam leaves the boiler intercooler intake is 105⁰C, determine the value of n.
drum with a quality of 98%. The unit consumes 3kg of coal per second as received having a Solution:
heating value of 25102 KJ/kg. What would be the overall efficiency of the unit in percent?
Px = = 273.86 Kpa
Steam properties:

At 5MPa and 400oC: h = 3195.7KJ/kg At 5MPa: hf = 1154.23, hg =1640.1

At 205oC: hf = 875.04 At 145oC: hf = 610.63

SOLUTION
1.281 =
n = 1.34_ ANS.
568. A single acting air compressor has a volumetric efficiency of 89%, operates at 500 rpm.
=0.807ANS.
It takes in air at 100kpa and 30⁰C and discharges it at 600 Kpa. The air handled is 8 m3/min
measured at discharge condition. If compression is isentropic, find mean effective pressure in
566. In a Rankine cycle steam enters the turbine at 2.5MPa (enthalpies and entropies given) Kpa
and condenser of 50KPa (properties given), what is the thermal efficiency of the cycle? SOLUTION: P1V1k =
P2V2k 100(V11.4) =
At 2.5MPa: hg = 2803.1 sg = 6.2575 600(8)1.4 V1 =
28.37687 m3/min
At 50KPa: sf = 1.0910 sfg = 6.5029 hf = 340.49 hfg = 2305.4 vf = VD = 28.768/0.89 = 32.32 m3/min
0.0010300

SOLUTION
W= =
h1 = 2803.1 KJ/kg

solving for h2: W = Pm x VD


7562.19 = Pm x 32.32
s = sf + xsfg Pm = _234.85 ANS.
569. A water-jacketed air compressor handles 0.343 m3/s of air at 96.5 kpa and 21⁰C and
leaving at 480 kpa and 132⁰C; 10.9 kg/h of cooling water enters the jacket at 15⁰C and leaves
6.2575 = 1.0910 + x(6.5029)
at 21⁰C. Determine the compressor break power.
SOLUTION:
x = 0.7945

h2 = hf + xhfg = 340.49 + 0.7945(2305.4) = 2172.13 KJ/kg


Q = heat loss = mCp(t2 – t1) = (10.9/3600)(4.187)(21 – 15) = 0.076 KW
Brake power = W + Q = 62.57 + 0.076 = _65.65 ANS.
570. A double suction centrifugal pump delivers 20 ft3/sec of water at a head of 12 m P1 = 159.3 ANS.
and running at 650 rpm. What is the specific speed of the pump? 575. A submersible, multi-stage, centrifugal deep well pump 260 gpm capacity is installed in
SOLUTION: a well 27 feet below the static water level and running at 3000 rpm. Drawdown when
pumping at rated capacity is 10 ft. the pump delivers the water into a 25,000 gallons capacity
Ns = overhead storage tank. Total discharge head developed by pump, including friction in piping
is 243 ft. Calculate the diameter of the impeller of this pump in inches if each impeller
diameter developed a head of 38 ft.
Q= SOLUTION:
h = 12 x 3.281 = 39.37 ft V=πDN

Ns = __2770.73__ANS. V=
571. Determine the number of stages needed for a centrifugal pump if it is used to deliver 576. D (3000/60) =
400 gal/min of water and pump power of 15 Hp. Each impeller develops a head of 30 ft.
D = 0.315 ft
SOLUTION:
ANS.
Wp = w Q h
# A fan draws 1.42 m3/sec of air at a static pressure of 2.54 cm of water through a duct 300
15 x 0.746 = 9.81(400 gal/min x 0.003785 m3/gal x 1/60) h
mm diameter and discharges it through a duct of 275 mm diameter. Determine the static
h =45.20 m x 3.281 ft/m = 148.317 ft
fan efficiency if total fan mechanical is 75% and air is measured at 25⁰C and 760 mmHg.
Number of stages = 148.317/30 = 4.94 stages
SOLUTION:
572. The suction pressure of a pump reads 3 in. of mercury vacuum and discharge pressure
reads 140psi is use to deliver 120 gpm of water with specific volume of 0.0163 ft3/lb.
Determine the pump work. wa =
SOLUTION: 577. A total head of fan is 187 m and has a static pressure of 210 mm of water gage, what is
P1 = -3 in Hg x 101.325/29.92 = -10.15 the velocity of air flowing if density of air is 1.15 kg/m3?
Kpa P2 = 140 psi x 101.325/14.7 = 965 Kpa SOLUTION:
W = 1/v = 1/.0163 = 61.35 lb/ft3 x 9.81/62.4 = 9.645 KN/m3 hs = 0.21(1000-1.15) = 182.61 m
h = hs + hv
h= = = 101.105 m 187 = 182.61 +
Q = 120 gal/min x 3.785 li/1gal x 1m3/1000li x 1/60 = 0.00757 m3/sec hv hv = 4.39 m
P = w Q h = 9.645(0.00757)(101.105) = __7.38_ ANS. 4.39 = v2 /2(9.81)
573. A submersible pump delivers 350 gpm of water to a height of 5 ft from the ground. v = _9.28_ANS.
The pump was installed 150 ft below the ground level and a draw down of 8 ft during the 578. A fan delivers 5.7 m3/sec at a static pressure of 5.08 cm of water when operating at a
operation. If water level is 25 ft above the pump, determine the pump power. speed of 400 rpm. The power input required is 2.963 KW. If 7.05 m3/sec are desired in the
SOLUTION: same fan and installation, find the pressure in cm of water.
h = 5+ 150 – (25 – 8) = 138/3.281 = 42.06 m SOLUTION:
Q = 350 gal/min x 0.003785 m3/gal x 1min/60sec = 0.02246
m3/sec Wp = w Q h = 9.819.02246)(42.06) = _9.28_ ANS.
574. A vacuum pump is used to drain a flooded mine shaft of 20⁰C water. The pump
pressure of water at this temperature is 2.34 Kpa. The pump is incapable of lifting the water
higher than 16 m. What is the atmospheric pressure?
SOLUTION: N2 = 494.74 rpm

h2 =7.77 ANS.
579. A rigid container is closed at one end and measures 8 in diameter by 12 in long. The d = 0.835 m ANS.
container is held vertically and is slowly moved downward until the pressure in the container 582. The type of flow occupying in a 1 cm diameter pipe which water flows at a velocity of
is 17 psia. What will be the depth of the top of the container from the free water surface? 2.50 m/s. Use v=1.13x10-6 m2/s for water.
SOLUTION: SOLUTION:
Patm = Pgage + Re =dV/ v
Patm 17 = Pgage +
14.7 Pgage = 2.30 Re =
psi Pgage = w h Since it is greater than 2000, then it is _________ANS
2.30(144) = 62.4 (h) 583. An insulated rigid tank initially contains 1.5 lb of helium at 80⁰F and 50 psia. Apaddle
h = 5.3077ft x 12 =_63.69_ans. wheel with power rating of 0.02 hp is operated within the tank for 30 min. Determine the
An empty, open can is 30 cm high with a 15 cm diameter. The can, with the open end final temperature.
down, is pushed under water with a density of 1000 kg/m3. Find the water level in the SOLUTION:
can when the top of the can is 50 cm below the surface. W= ΔU = mcv(T2 – T1)
SOLUTION: 0.02hp (0.50 hr)(2545 Btu/hr/hp) = 1.5 (0.171)(T2 – 80)
Consider the water pressure T2 = _179.22_ANS.
Pw = wh + 101.325 = (0.8 - x)(9.81) + 101.325 = 109.173 – 584. A 4 m2 asphalt pavement with emissivity of 0.85 has a surface temperature of 50⁰C.
9.81x Consider the air pressure: Find the maximum rate of radiation that can be emmited from the surface.
P1V1 = P2V2 SOLUTION:
101.325(A x 0.3) = P2[A(0.3 – x)]
Qr = e ksvA
Ksv = 5.67 x 10-8 (Stefan Boltzman constant)
P2 = Qr = 0.85(5.67 x 108)(4)(50 +273)4 = 672493.752 ANS.
Pw = P2 585. Air at 10⁰C and 80 kpa enters a diffuser of a jet engine steadily with a velocity of 200
m/s. The inlet area of diffuser is 0.40 m2. Determine the mass flow rate of air.
109.173 – 9.81x = SOLUTION:
9.81x2 – 112.116x + 2.3705 = 0 w = P/RT = 80/0.287(10+273) = 0.985 kg/m3
By quadratic formula: m = wvA = 0.985(200)(0.40) =78.8 ANS.
X = 0.02118 m ANS 586. Consider a refrigeration whose 40watts light bulb remains on continuously as a result
580. A cylindrical pipe with water flowing downward at 0.03 m3/s having top diameter of of a malfunction of the switch. If the refrigerator has a COP of 1.3 and the cost of electricity
0.08, bottom diameter of 0.04 m and height of 1.5 m. Find the pressure between the pipe. is 8 cents per kw-hr, determine the increase in the energy consumption of the refrigeration
SOLUTION: and its cost per year if the switch is not fixed.
SOLUTION:
COP = Re / Wref
Z1 – Z2 = 1.5 m 1.3 = 40 / Wref
Z2 – Z1 = -1.5 m Wref = Wb + Wref = 40 + 30.769 = 70.77
watts W = 0.07077 KW
V1 = Cost = 0.07077(8760)(P0.08) = _49.595_ANS.
V2 = 587. A 75 hp motor that has an efficiency of 91% is worn out and is replaced by a high-
efficiency motor that has an efficiency motor that has an efficiency of 95.4%. Determine the
reduction in heat gain of the room due to higher efficiency under full-load conditions.
+ (-1.5) SOLUTION:
P1 – P2 = _-28.41_ANS. P01 = (75x0.746)(0.91) = 50.91 KW
581. Determine the size of pipe which will deliver 8 liters of medium oil (v = 6.10 x 10-6 P02 = (75x0.746)(0.954) = 53.376 KW
m2/s) assuming laminar flow conditions. Qreduced = 53.376 – 50.91 = 2.47 ANS.
588. A household refrigerator that has a power input of 450 watts and a COP of 2.5 is to cool
five large watermelons, 10 kg each, to 8⁰C. If the watermelons are initially at 20⁰C, determine
V=
how long will it take for the refrigerator to cool them. The watermelons can be treated as
water whose specific heat is 4.2 KJ/kg-⁰K.
Re =
SOLUTION:
For laminar flow, Re = 2000
COP = Re/Wc
2.5 = Re/450 Re
2000 = = 1,125 watts
Re = mcp(t2 – t1) Solution:
450t = (10 x 5)(4.2)(20 – 8)
t = __5.6_ANS.
589. When a man returns to his wall-sealed house on a summer day, he finds that the house is W = e Q = 0.7315 (3000) = 2194.5 Btu/s
at A heat engine receives hat from a source at 1200°K at a rate of 500KJ/s and rejects the
32⁰C. He returns on the air conditioner which cools the entire house to 20⁰C in 15 minutes. If waste heat to a medium at 300°K. The power output of the heat engine is 180 KW.
COP is 2.5, determine the power drawn by the air conditioner. Assume the entire mass within Determine the irreversibility rate for this process.
the house is 800 kg of ait which cv= 0.72 KJ/kg-K, cp=1.0 KJ/kg-K. Solution:
SOLUTION: e = (1200-300) / 1200 = 0.75
Re = mcv(t2 – t1) = (800/15x60)(0.72)(32 – 20) W= 0.75(500) = 375 KW
Re = 7.68 KW Irreversibilities = 375-180 =195 ANS
Wc = 7.68 / 2.5 = _3.072_ANS. A dealer advertises that he has just received a shipment of electric resistance heaters for
residential buildings that have an efficiency of 100 percent. Assuming an indoor
A heat source at 800°K losses 2000 KJ of heat to a sink at 500°K. Determine the temperature of 21°C and outdoor temperature of 10°C, determine the second law efficiency
entropy generated during the process. of these heaters.
Solution: Solution:
∆Ssource = -2000/800 = -2.5 COP1 = 100 efficient = 1
∆Ssink = 2000/500 = 4 COP2 = (21+273) / (21-10) = 26.72
∆Sgen = -2.5/4 = -0.625_ANS esl = COP1 / COP2 = 1 / 26.72 = __0.037__ANS
Helium gas in compressed in an adiabatic compressor from an initial state of 14 psia and 50°F to a A thermal power plant has a heat rate of 11,363 Btu/KW-hr. Find the thermal efficiency of
final temperature of 320°F in a reversible manner. Determine the exit pressure of Helium. the plant.
Solution: Solution:
T2/T1 = (P2/P1)n-1/n e = 3412 / Heat rate = 3412 / 11363 = _0.300_ANS
(320+460)(50+460) = (P2/14)1.667-1/1.667 A rigid tank contains 2 kmol of N2 and 6 kmol of CO2 gases at 300°K and 115 Mpa. Find
P2 = _3340856__ANS the tank volume using ideal gas equation
Air pass thru a nozzle with efficiency of 90%. The velocity of air at the exit is 600 m/s. Solution:
Find the actual Velocity at the exit. Pm Vm = Nm Ru Tm
Solution: 15,000 Vm = (6+2)(8.314)(300)
e = (Va/V3)2 Vm = _1.33__ANS
0.9 = (Va/600)2 A spherical balloon with a diameter of 6 m is filled with helium at 20°C and 200 kpa.
Va = _270_ANS Determine the mole number.
A 50 kg block of iron casting at 500°K is thrown into a large lake that is at a temperature of Solution:
285°K. The iron block eventually reaches the thermal equilibrium with the lake water. PV=NRT
Assuming average specific heat of 0.45 KJ/kg-K for the iron, determine the entropy
generated during the process.
(200) = N (8.314) (20+273)
Solution:
N = _9.285_ANS
∆Siron = m c ln(T2/T1) = 50(0.45)ln (285/500) = -12.65 KJ/K
The air is an automobile tire with a volume of 0.53 ft3 is at 90°Fand 20 psig. Determine the
∆Slake = Q/T = [50(0.45)(500-285)] / 285 = 16.97 KJ/K
amount of air that must be added to raise the pressure to the recommended value of 30
∆Sgen = -12.65 + 16.97 = __4.32_ANS
psig. Assume the atmospheric pressure to be 14.7 psia and the temperature and the volume
A windmill with a 12 m diameter rotor is to be installed at a location where the wind is
to remain constant.
blowing at an average velocity of 10 m/s. Using standard condition of air (1 atm,
Solution:
25°C), determine the maximum power that can be generated by the windmill.
PV=mRT
Solution:
(20+14.7)(144)(0.53) = m1 (53.3)(90+460)
w = P/RT = 101.325 / (0.287)(25+273) = 1.1847
m1 = 0.09034 lb
kg/m3 m = w A v = 1.1847(π/4 x 122)(10) = 1,339.895
(30+14.7)(144)(0.53) = m2 (53.3)(90+460)
kg/s KE = v2/2000 = 102/2000= 0.05 KJ/kg
m2 = 0.11634 lb
Power = m KE = 1,339.895(0.05) = 66.99_ANS
madded = m2 – m1 = 0.11634 - 0.09034 = _0.026_ANS
Consider a large furnace that can supply heat at a temperature of 2000°R at a steady rate
A rigid tank contains 20 lbm of air at 20 psia and 70°F. More air is added to the tank until
of 3000 Btu/s. Determine the exergy of this energy. Assume an environment temperature the pressure and temperature rise to 35 psia and 90°F, respectively. Determine the amount of
of 77°F. air added to the tank.
Solution:
P1 V1 = m1 R1 T1
(20 x 144) (V1) = 20(53.3)(70+460)
V = 196.17 ft3
P2 V2 = m2 R2 T2 t2 = __105.23_ANS
(35 x 144)(196.17) = m2(53.3)(90+460) For a heat transfer purposes, a standing man can be modeled as a 30 cm diameter, 170 cm long
m2 = 33.73 lbs vertical cylinder with both the top and bottom surfaces insulated and with the side surface at an
madded = m2 – m1 = 33.73 – 20 = _13.73__ANS average temperature of 34°C. For a convection heat transfer coefficient of 15 W/m2-°C, determine
A rigid tank contains 5 kg of an ideal gas at 4 atm and 40°C. Now a valve is opened and the rate of heat loss from this man by convection in an environment at 20°C.
half of mass of the gas is allowed to escape. If the final pressure in the tank is 1.5 atm, the Solution:
final temperature in the tank is: Qc = k A (t2-t1) = 15(π x 0.30 x 1.7) (34-20) = _336.46_ANS
Solution: A 5 cm diameter spherical ball whose surface is maintained at a temperature of 70°C is
PV=mRT suspended in the middle of a room at 20°C. If the convection heat transfer coefficient is 15
(4 x 9.81)(V) = 5(0.287)(40+273) W/m2-°C and the emissivity of the surface is 0.8, determine the total heat transfer from
V = 11.446 m3 the ball.
PV=mRT Solution:
(1.5 x 9.81)(11.446) = (5/2)(0.287)(T) A = 4 π r2 = 4 π (0.05)2 = 0.0314 m2
T = _234.74_ANS Qc = h A (t2-t1) = 15(0.0314) (70-20) = 23.56 watts
The pressure of an automobile tire is measured to be 200 kpa(gage) before the trip and 220 Qr = (0.80) (5.67 x 10-8) (0.0314) [(70+273)4-(50+273)4] = 9.22
kpa(gage) after the tip at a location where the atmospheric pressure is 90 kpa. If the watts Q = Qc + Qr = 23.56 + 9.22 = _32.78_ANS
temperature of the air in the tire before the trip is 25°C, the air temperature after the trip is: A frictionless piston-cylinder device and a rigid tank contain 1.2 kmol of an ideal gas at the
Solution: same temperature, pressure, and volume. Now heat is transferred, and the temperature of
T2/T1 = P2/P1 both system is raised by 15°C. The amount of extra heat that must be supplied to the gas in
T2 / (25+273) = (220+90) / (200+90) the cylinder that is maintained at constant pressure.
T2 = 318.55°K Solution:
Water is boiling at 1 atm pressure in a stainless steel pan on an electric range. It is observed that Q = m Cp (t2-t1) = (1.2 x 8.314)(1)(15) = _149.65ANS
2 kg of liquid water evaporates in 30 min. The rate of heat transfer to the water is: A supply of 50 kg of chicken at 6°C contained in a box to be frozen to -18°C in a freezer.
Solution: Determine the amount of heat that needs to be removed. The latent heat of the chicken is 247
KJ/kg, and its specific heat is 3.32 KJ/kg-C above freezing and 1.77 KJ/kg-C below freezing.
Q = mL = = _25.08_ANS The container box is 1.5 kg, and the specific heat of the box material is 1.4 KJ/kg-C. Also
Consider a person standing in a breezy room at 20°C. Determine the total rate of heat the freezing temperature of chicken is -2.8°C.
transfer from this person if the exposed surface area and the average outer surface Solution:
temperature of the person are 1.6 m2 and 29°C, respectively, and the convection heat Qchicken = 50[3.32(6+2.8) + 247 + 1.77(-2.8+18)] = 15,156 KJ
transfer coefficient is 6W/ m2 with emissivity factor of 0.95. Qbox = 1.5(1.4)(6+18) = 50.4 KJ
Solution: Q = 15,156 + 50.4 = _15206.4_ANS
Qc = h A (t2-t1) = (6)(1.6)(29.20) = 86.40 watts Water is being heated in a closed pan on top of a range while being stirred by a paddle
Qf = (0.95)(5.67 x 10-8)(1.6)[(29+273)4-(20+273)4] = 81.7 wheel. During the process, 30 KJ of heat is transferred to the water and 5 KJ of heat is lost to
watts Q = Qc + Qf = 86.40 + 81.7 = _168.1_ANS the surrounding air. The paddle-wheel work amounts to 500 N-m. Determine the final
Water is boiled in a pan on a stove at sea level. During 10 minutes of boiling, it is observed that energy of the system if its initial energy is 10 KJ.
200 grams of water has evaporated. Then the rate of heat transfer to the water is: Solution:
Solution: Final energy = QA + ∆U – Qlose + W = 30 + 10 – 5 + 0.50 = _35.5_ANS
Q = mL = (0.2/10) (2257) = _45.14_ANS A classroom that normally contains 40 people is to be air-conditioned with window air-
An aluminum pan whose thermal conductivity is 237 W/m-C has a flat bottom whose conditioning units of 5 KW cooling capacity. A person at rest may be assumed to dissipate
diameter is 20 cm and thickness 0.4 cm. Heat is transferred steadily to boiling water in the pan heat at rate of about 360 KJ/hr. There are 10 light bulbs in the room, each with a rating of 100
through its bottom at a rate of 500 watts. If the inner surface of the bottom of the pan is watts. The rate of heat transfer to the classroom through the walls and the windows is
105°C, determine the temperature of the surface of the bottom of the pan. estimated to be 15,000 KJ/hr. If the room to be maintained at a constant temperature of 21°C,
Solution: determine the number of window air-conditioning units required.
A = π/4 (0.20)2 = 0.0314m2 Solution:
Q = total heat load = 40(360/3600) + 10(0.100) +15,000/3600 = 9.167
KW No. of air conditioning = 9.167/5 = 1.833_ANS
A 4m x 5m x 6m room is to be heated by a baseboard resistance heater. It is desired that the
resistance heater be able to raise the air temperature in the room from 7 to 23°C within 15
minutes. Assuming no heat losses from the room and an atmospheric pressure of 100 kpa,
determine the required power of the resistance heater. Assume constant specific heats at Solution:
room temperature. e = (TH-TL)/TH = [(28+273)-(15+273)(28+273)] =
Solution: 0.03679 e = W/QA
w = P/RT = 100 / (0.287)(7+273) = 1.244 kg/m3 0.03679 = 10,000/QA
m = 1.244(4 x 5 x 6) = 149.28 kg QA = 271,812.99 KW
Q= m Cv (t2-t1) = 149.28(0.7186)(23-7) = 1,716.36 KJ QR = QA – W = 271,812.99 -10,000 = 261,813 KW
Power = 1,716.36 / (15 x 60) = _1.9_ANS QR = m Cp (∆t)
A student living in a 4m x 6m x 6m dormitory room turns on her 150 watts fan before she 261,813 = m (3)(4.187)
leaves the room on a summer day, hoping that the room will be cooler when she comes back m = 20,843.32 kg/s
in the evening. Assuming all the doors and windows are tightly closed and disregarding any 20,843.32 kg/s or 20,843.32 lit/s = 20.843 m3/s
heat transfer through walls and the windows, determine the temperature in the room when she h = P/w = 12/9.81 = 1.223 m
comes back 10 hours later. Use specific heat values at room temperature, and assume the Wc = w Q h = 9.81(20.843)(1.223) = _250.07_ANS
room to be at 100 kpa and 15°C in the morning when she leaves. A plane-type of solar energy collector with an absorbing surface covered by a glass plate is
Solution: to receive an incident radiation of 800 W/m2. The glass plate has a reflectivity of 0.12 and a
w = P/RT = 100 / (0.287)(15+273) = 1.2098 kg/m3 transmissivity of 0.85. The absorbing surface has an absorptivity of 0.90. The area of the
m = 1.2098(4 x 6 x 6) = 174.216 kg collector is 5 m2. How much solar energy in watts is absorbed by the collector?
Q= m Cv (t2-t1) Solution:
0.15(10 x 3600) = 174.216(0.7186)(t2 - 15) Q = heat absorbed from the sun
t2 = _53.13_ANS Q = 16.3(π/4 x 0.008)[15-(-190)] = __20.99_ANS
A piston-cylinder device whose piston is resting on top of a set of stops initially contains 0.50
kg of helium gas at 100 kpa and 25°C. The mass of the piston is such that 500 kpa of pressure 590. A one cubic meter container contains a mixture of gases composed of 0.02 kg-mol of
is required to raise it. How much heat must be transferred to the helium before the piston starts oxygen and 0.04 kg-mol of helium gas at a pressure of 220 kPa. What is the temperature of
rising? this ideal gas mixture in degrees Kelvin?
Solution: Solution:
For helium: Cv = R/(k-1) = (8.314/4)(1.667-1) = 3.116 KJ/kg- V=V1+V2
K T2 = (25+273)(500/100) = 1,490°K T1 = 25+273 = 298°K VT = m1R1T1/P1 + m2R2T2/P2
1 = (0.02x32)(8.314/32)(T)/220 +
Q= m Cv (T2-T1) = 0.50(3.116)(1490-298) = _____________ANS (0.04x32)(8.314/4)(T)/220 T = -2196.93_ANS.
In order to cool 1 ton (1000 kg) of water at 20°C in an insulated tank, a person pours 80 kg
of ice at -5°C into the water. Determine the final equilibrium temperature in the tank. The
melting temperature and the heat of fusion of ice at atmospheric pressure are 0°C and 333.7
KJ/kg, respectively.
Solution:
Qwater = Qice
1000(4.187)(20-tB) = 80(2.09)(0+5) + 80(333.7) + 80(4.187)(tB-0)
tB = _12.43_ANS
A fan is powerd by a 0.5 hp motor delivers air at a rate of of 85 m3/min. Determine the
highest value for the average velocity of air mobilized by the fan. Take the density of air to be
1.18 kg/m3.
Solution:
P=wQh
0.50(0.746) = (1.18 x 0.00981)(85/80)(h)

= _21.12_ANS
An Ocean-Thermal Energy Conversion power plant generates 10,000 KW using a warm
surface water inlet temperature of 28°C and a cold deep-water temperature of 15°C. On the
basis of a 3°C drop in the temperature of the warm water and a 3°C rise in the temperature of
the cold water due to removal and addition of heat, calculate the power required in KW to
pump the cold-deep water to the surface and through the system heat exchanger if the
required pumping pressure increase is 12 kpa.Assume a Carnot cycle efficiency and density of
cold water to be 1000 kg/m3.
Solution:
591. Methyl alcohol (CH3OH) is burned with 25% excess air. How much unburned oxygen in Cv of helium = 3118.9 J/kg-C
kg-mol-oxygen/kg-mol-fuel will there be in the products if the combustion is complete?
m g h = m Cv
Solution:
CH3OH + O2 +3.76N2 = CO2 + H2O + 3.76N2 m (9.81) (4500) = m (3118.9)
CH3OH + 1.5O2 + (1.5) (3.76) N2 = 1CO2 + H2O + 1.5(3.76) N2
Consider 25% excess air: = __14.15_ANS.
CH3OH + 1.5O2 + 1.25(1.5) (3.76) N2 = 1CO2 + H2O + 1.25(1.5) (3.76) N2 + 0.25(1.5)
597. Consider two Carnot heat engines operating in series. The first engine receives heat from
O2 Unburned O2 = 0.25(1.5) = _0.375_ANS.
the reservoir at 2400 K and rejects the waste heat to another reservoir at temperature T. The
592. A 12 DC electrical motor draws a current of 15 amps. How much work in KJ does this second engine receives heat by the first one, convert some of it to work, and rejects the rest to
motor produce over a 10-minute period of operation? a reservoir at 300K. If thermal efficiencies of both engines are the same, determine the
Solution: temperature T.
W=E=QV Solution:
W = (15 x 10 x 60)(12) = 108,000 J ANS. e1 = e2

593. A 4liter (2-liter per revolution at standard pressure and temperature) spark ignition
engine has a compression ratio of 8 and 2200 KJ/kg heat addition by the fluid combustion. T = -848.53ANS.
Considering a cold air-standard Otto cycle model, how much power will the engine
produce when operating at 2500 rpm? 598. An ideal gas mixtrure consists of 2 kmol of N2 and 6 mol of CO2. The mass fraction of
Solution: CO2 is:
w= 1.2 kg/m3 (standard density of air) Solution:
m = 2 li/rev x 2500 rev/min / 1.2 kg/m3 x 1 m3/1000 li x 1 min/60 sec = 0.10 M = (2/8)(28) + (6/8)(44) = 40
kg/s e = W/QA R = 8.314/M = 8.314/40 = _0.207_ANS.
e = 1 – 1/81.4-1 = 0.5647
0.5647 = W/2200 599. A Carnot cycle operates between the temperature limits of 300K and 1500K, and
W = 1,242.34 KJ/kg (0.10 kg/s) = 124.23 KW ANS. produces 600 KW of net power. The rate of entropy change of the working fluid during the
head addition process is:
594. A simple Rankine cycle produces a 40 MW of power, 50 MW of process heated and Solution:
rejects 50MW of heat to the surroundings. What is the utilization factor of this cogeneration W=
cycle neglecting the pump work?
Solution: 600 =
QA = WT + Wprocess WP = 40 + 50 + 50 = 140 KW
= _1/2_ANS.
UF = (Qprocess + WT)/QA = (50 +40)/140 = __50.2857_ANS.
600. Air in an ideal Diesel cycle is compressed from 3 L to 0.15 L and then it expands during
595. The rate of heat transfer to the surroundings from a person at rest is about 400 KJ/hr.
the constant pressure heat addition process to 0.3 L. Under cold air standard conditions, the
Suppose that the ventilation system fails in an auditorium containing 120 people and
thermal efficiency of this cycle is:
assuming that the energy goes into the air of volume 1500 m3 initially at 300K and 101 kPa,
Solution:
calculate the rate in °C/min of air temperature change.
rK = 3/0.15 = 20
Solution:
rC = 0.3 / 0.15 = 2
Q = m CV e = 1 – (1/rKk-1)[(rCK – 1)/k(rC – 1)] = 0.6467 ANS.
PV = mRT
101(1500) = m(0.287)(300) 601. Helium gas in an ideal Otto cycle is compressed from 20°C and 2 L to 0.25 L
m = 1,759.58 kg and its temperature increases by an additional 800°C during the head addition
process. The temperature of helium before the expansion process is:
Q = m CV Solution:
120(400/60) = 1759.58(0.7186) rk = 2/0.25 = 8
T2 = (20 + 273)(8)1.667-1 = 1,172 K
= __0.632_ANS. T3 = T2 + 800 = 1172 + 800 = 1972K ANS.
596. An insulated box containing helium gas falls from a balloon 4.5 km above the earth’s 602. In an ideal Otto cycle, air is compressed from 1.20 kg/m3 and 2.2 L to 0.26 L and the
surface. Calculate the temperature rise in °C of the helium when box hits the ground. net work output of the cycle is 440 KJ/kg. The mean effective pressure for the cycle is:
Solution:
VD = V2 – V1 = 2.2 x 10-3 m3 – 0.26 x 10-3 m3 = 1.94 x 10-3 m3 W = 440 KJ/kg (1.2 kg/m3 x 2.2
x 10-3 m3) = 1.1616 KJ
Pm = W/VD = 1.1616 / 1.94 x 10-3 = _598.763_ANS.

603. An ideal Brayton cycle has a net work output of 150 KJ/kg and backwork ratio of 0.4. Of both the turbine and the
compressor had an isentropic efficiency of 80%, the net work output of the cycle would be?
Solution:
Backwork Ratio = WC / WT WC = 0.40 WT
Wnet = WT - WC Wnet = 250 KJ/kg
Wnet’ = 250(0.80) – 0.40(200)/0.80 = 100_ANS.

604. Air enters a turbojet engine at 200 m/s at a rate of 20 kg/s, and exits at 800 m/s relative to the aircraft. The thrust developed
by the engine is:
Solution:
Thrust developed = m (v2 – v1) = 20(800-200) = _12000_ANS.

605. A thermal power plant has a net power 10 MW. The backwork ratio of the plant is 0.005. Determine the compressor
work.
Solution:
Wnet = WT + WP BW=WP/WT WP = 0.005WT 10,000 =
WT – 0.005WT WT = 10,050.25 KW
WC = 0.005(10,050.25) = _50.25ANS.
606. A heat engine receives heat from a source at 1200K at a rate of 500 KJ/s and rejects the waste heat to a sink at 300K. If
the power output of the engine is 200 KW, the second law efficiency of this heat engine is?
Solution:
ea = 200/500 = 0.40
et = (TH - TL)/TH = (1200-300)/1200 = 0.75 es = 0.40/0.75 = _0.53_ANS.

607. A water reservoir contains 100,000 kg of water at an average elevation of 60m. The maximum amount of electric
power that can be generated from this water is:
Solution:
P = mh = (100,000x0.00981)(60) = 58,860 KJ
P = 58,860 KJ x KWh/3600KJ = 16.35ANS.

608. A house is maintained at 22°C in winter by electric resistance heaters. If the outdoor temperature is 5°C, the second
law efficiency of the resistance heaters is:
Solution:
ea = 100% resistance heaters et = (22-15)/(22+273) = 5.8% es =
5.8/100 =0.058 ANS.

609. A thermoelectric refrigerator that resembles a small ice chest is powered by a car battery, and has a COP of 0.10. If the
refrigerator cools a 0.350 L canned drink from 20°C to 4°C in 30 min, determine the average electric power consumed by the
thermoelectric refrigerator.
Solution:
Q = m cp (t2 – t1) =[(1 x 0.35)/(30 x 60)] (4.187) (20-4) = 13 watts COP=RE/WC
0.10 = 13/0.10 = _130_ANS.
610. What is the pressure 8,000 ft. (2000m) below the surface of the ocean? Neglect the
compressibility factor, SI units. Answer: B
21.4
20.1
21.0
22.3

611. What temperature at which water freezes using the Kelvin scale? Answer: B
373
273
278
406

612. The SI units of temperature is: Answer: B


oF
oK
BTU
oR

613. The pressure reading of 35 psi in kpa is: Answer: A


342.72
724.00
273.40
427.30

614. 1 torr is equivalent to pressure Answer: D


1 atm
2mm Hg
14.7
1/760 atm

615. What is the standard temperature in the US? Answer: A


Fahrenheit
Rankine
Celsius
Kelvin

616. Given steam pressure of 900 lb/ft2, temperature of 300oF, specific volume of 5.8 ft3/lb. If
the specific enthalpy is 9500 ft-lb/lb, what is the internal energy per lb of the system? Answer: D
4400
3900
3600
4280
58. The barometer reads 29.0 inches (737 mm) of mercury. What is the absolute pressure if a vaccum
gage reads 9.5 psi (66 kpa) in SI? Answer: A
32.02 kpa
33.04 kpa
31.36 kpa
31.86 kpa

59. A fluid with a vapor pressure of 0.2 Pa and a specific gravity of 12 is used in a barometer. If the
fluid column height is 1m, what is the atmospheric pressure? Answer: D
150.6 kpa
115.5 kpa
144.4 kpa
117.7 kpa

60. What is the atmospheric pressure on a planet if the pressure is 100 kpa and the gage pressure is 10
kpa? Answer: D
10 kpa
100 kpa
80 kpa
90 kpa

61. A column of water 200 cm high will give a pressure equivalent to: Answer: D
9810 dyne/cm2
0.1 atm
0.10 bar
19,620 N/m2

62. What is the equivalent oR of 400ok? Answer: A


720.60oR
851.15oR
670.27oR
344.25oR

63. If the temperature inside the furnaces is 700oK, what is the corresponding reading in oF? Answer:
D
700.60
750.60
860.60
800.60

64. If the oF scale is twice the oC scale, what will be the corresponding reading in each scale?
Answer: B
22oC and 44oF
160oC and 320oF
40oC and 80oF
100oC and 200oF
60. Water enters the condenser at 25oC and leaves at 40oC. What temperature difference in oF?
Answer: C
25
26
27
28

61. Water enters the heater with 28oC and leaves at 75oC. What is the temperature change in oF?
Answer: C
74.80
38.29
84.80
57.36

62. The specific gravity of mercury relative to water is 13.55. What is the specific weight of
mercury? The specific weight of water is 62.4 lb per cubic foot. Answer: B
102.3 KN/m3
132.9 KN/m3
150 KN/m3
82.2 KN/m3
63. An iron block weighs 5 N and has a volume of 200 cm3. What is the density of the block? Answer:
C
988 kg/m3
1255 kg/m3
2550 kg/m3
800 kg/m3

64. The suction pressure of a pump reads 540 mmHg vaccum. What is the absolute pressure in kpa?
Answer: D
40
3.3
60
29.3

65. A boiler installed where the atmospheric pressure is 755 mmHg. Has a pressure 12 Kg/cm2 . What
is the absolute pressure in Mpa? Answer: B
1.350
1.277
1.200
1,700

66. A storage tank contains oil with specific gravity of 0.88 and depth of 20 m. What is the
hydrostatic pressure at the bottom of th tank in kg/cm2? Answer: A
1.76
2.0
6.0
3.0

67. A batch of concrete consisted of 200 lbs fine aggregate, 350 lbs coarse aggregate, 94 lbs, cement,
and 5 gallons water. The specific gravity of the sand and gravel may be taken as 2.65 and that of
the cement as 3/10/ What was the weight of concrete in place per cubic foot? Answer: D
172 lbs
236 lbs
162 lbs
153 lbs

65. A batch of concrete consisted of 200 lbs fine aggregate, 350 lbs coarse aggregate, 94 lbs, cement,
and 5 gallons water. The specific gravity of the sand and gravel may be taken as 2.65 and that of
the cement as 3.10. How much by weight of cement is required to produce one cubic yard?
Answer: D
765
657
675
567

66. A cylindrical tank 2 m diameter, 3 m high is full of oil. If specific gravity of oil is 0.9, what is the
mass of oil in the tank? Answer: A
8482 kg
4500 kg
1800 kg
7000 kg

67. 100 g of water are mixed with 150 g of alcohol (w = 790 kg/m3). What is the specific volume of
the resulting mixtures, assuming the fluids mixed completely? Answer: D
0.82 cm3/kg
0.88 cm3/kg
0.63 cm3/kg
1.20 cm3/kg
68. 100 g of water are mixed with 150 g of alcohol (w = 790 kg/m3). What is the specific gravity of
the resulting mixtures, assuming the fluids mixed completely? Answer: D
0.96
0.63
0.82
0.86

69. A spherical tank is full of water that has a total mass of 10,000 kg. If the outside diameter of the
tank is 2722 mm, how thick is the wall of the tank? Answer: B
50 mm
25 mm
30 mm
35 mm

70. A cylindrical tank is filled with water at the rate of 5000 gal/min. The height if the water in the
tank after minutes is 20.42 ft. What is the diameter of the tank? Answer: B
30 ft
25 ft
20 ft
9m

71. Water is flowing through a 1 foot diameter pipe at the rate of 10 m/s. What is the volume flow of
water flowing? Answer: C
7.50 ft3/sec
7.95 ft3/sec
7.85 ft3/sec
0.22 ft3/sec

91. A certain fluid is flowing in a 0.5 m x 0.3 m channel at the rate of 3 m/sec and has a specific
volume of 0.0012 m3/kg. Determine the mass of water flowing in kg/sec. Answer: C
380 kg/sec
390 kg/sec
375 kg/sec
370 kg/sec

92. One useful equation used is the change of enthalpy of compressible liquid with constant specific
heat is:
hsub2 - hsub1 = c(Tsub2 – Tsub1) + v(Psu2 – Psub1)
where: Tsubn = Temperature at state
n Psubn = Pressure at state n
V = specific volume of liquid
Water with enthalpy with csubp = 4.187 KJ/kg-oK and v = 1.00 x 10 -3 m3/kg has the following
states:
State I: Tsub1= 19oC Psub1 = 1. 013 x 105
State II: Tsub2 = 30oC Psub2 = 0.113 Mpa

46.0 kpa/kg
56.0 KJ/kg
46.0 KJ/kg
46.0 KN/kg

105. What is the potential energy of a 500 kg body if it is dropped to a height of 100 m?
Answer: A
490.50 KJ
765.50 KJ
560.50 KJ
645.48 KJ

106. The flow energy of 124 li/min of a fluid passing a boundary to a system is 2 KJ/sec.
What is the pressure at this point. Answer: B
100 kpa
140.30 psi
1000 kpa
871 KJ

107. Steam at 1000 lb/ft3 pressure and 330oR has a specific volume of 6.5 ft3/lb and a specific
enthalpy of 9800 ft-lb/lb. Find the internal energy per pound mass of steam. Answer: B
5400
3300
6400
2500
111. Air and fuel enter a furnace used for home heating. The air has an enthalpy of 302 KJ//kg
and the fuel has an enthalpy of 43,207 KJ/kg. The gases leaving the furnace have an enthalpy of
616 KJ/kg. of fuel. The house requires 17.6 KW of heat. What is the fuel consumption per day?
Answer: B
85 kg
41 kg
45 kg
68 kg

112. The power plant furnace burns coal at the rate of 108,200 kg/hr. Air at 100.8 kpa, 28oC is
supplied at the rate of 13.8 kg/kg coal. Determine the volume flow rate of air flow in m3/min.
Answer: A
21,327 m3/min
19,414.63 m3/min
20,435.26 m3/min
24,535.54 m3/min

113. The enthalpy of air is increased by 139.586 KJ/kg in a compressor. The rate of air flow is
16.42 kg/min. The power output is 48.2 KW. Which of the following values most nearly equals
the heat loss from the compressor in KW? Answer: A
-10
+10.2
-9.95
+9.95

114. A steam turbine receives 70 pounds of steam per minute with an enthalpy of 1600 Btu
per pound and velocity of 100 ft/sec. It leaves the turbine at 900 ft/sec and 1320 Btu/lb enthalpy.
The radiation loss is 84,000 Btu/hr. Find the horsepower output. Answer: A
403 hp
400hp
380 hp
410 hp

115. Steam enters a turbine stage with enthalpy of 3628 KJ/kg 70 m/sec and leaves the same
stage with an enthalpy of 2846 KJ/kg and a velocity of 124 m/sec. Calculate the work done by
the steam.
Answer: A
776.76 KJ/kg
873 KJ/kg
567.23 KJ/kg
923.34 KJ/kg

116. Steam with an enthalpy of 800 kcal/kg enters a nozzle at a velocity of 80 m/sec. Find the
velocity of the steam at the exit of the nozzle if its enthalpy is reduced tp 750 kcal/kg. assuming
the nozzle is horizontal and disregarding heat losses. Take g(9.81) m/sec and J = 427 kg-m/kcal.
Answer: D
561.24 m/s
142.5 m/s
52.41 m/s
652.14 m/s

118. A volume of 450 cc of air is measured at a pressure of 740 mm HG absolute and a


temperature of 20oC. What is the volume in cc at 760 mm HG abs. and 0 oC? Answer:B
516.12
408.25
620.76
375.85

119. Assuming compression is according to the law PV = constant. Calculate the initial
volume of gas at a pressure of 2 bar which will occupy a volume of 6 cubic meters when it is
compressed to a pressure of 42 bar. Answer: A
126 m3
120 m3
130 m3
136 m3

120. How much heat, KJ must be transferred to 20kg of air to increase the temperature from
20 degrees C to 280 degrees C if the pressure is maintained constant. Answer: C
2500
2050
5200
5500

121. If air is at pressure, p, of 3200 lb/ft2. And at a temperature, T, of 800oR, what is the
specific volume , v? Answer: B
14.2 ft3/lb
13.3 ft3/lb
11.2 ft3/lb
9.8 ft3/lb

122. The mass of air in the room 3m x 5m x 20m is known to be 350 kg. Find its density.
Answer: C
1.617 kg/m3
1.716 kg/m3
1.167 kg/m3
1.176 kg/m3

123. A transportation company specializes in the shipment of pressurized gaseous materials.


An order is received for 100 liters of a particular gas at STP (32oF and 1 atm). What minimum
volume tank is necessary to transport the gas at 80oF and a maximum pressure of 8 atm? Answer:
B
16 liters
14 liters
10 liters
12 liters

124. A bicycle has a volume of 600 cm3. It is inflated with carbon dioxide to pressure of 80 psi
at 20 oC. How many grams of carbon dioxide are contained in the fire? Answer: A
5.98 g
6.43 g
4.63 g
3.83 g

122. An ideal gas at 0.60 atm. And 87oC occupies 0.450 liters. How many moles are in the
sample? (R
0.0821 atm/mole K) Answer: D
0.0002 mole
0.0378 mole
0.0198 mole
0.0091 mole

123. Air compressed in a diesel engine from an initial pressure of 13 psia and a temperature of
120oF to one twelfth of its original volume. Calculate the final temperature assuming compression
to be adiabatic. Answer: D
987
980
981
1107

124. The compression ratio of an Otto cycle is 6:1, Psub1 is 14.7 psia, Tsub1 is 68oF. Find the
pressure and temperature at state 2. Answer: D
180.6 psia, 1081oF
180.6 psia, 139 oF
180.6 psia, 139 oF
180.6 psig, 1081oR

125. An automobile tire is inflated to 30 psig pressure at 50oF. After being driven, the
temperature rise to 75oF. Determine the final gage pressure assuming the volume remains
constant. Answer:A
32.19 psig
55 psig
0 psig
38.9 psig
126. If 10 m3 of atmospheric air at zero degrees centigrade temperature are compressed to a
volume of 1 m3 at 100oC, what will be the pressure of air in kpa? Answer: B
1500
1384
2000
1000

127. If 8 lbs of a substance receives 240 Btu of heat at constant colume and undergo a
tempearure change of 150oF/ What is the average specific heat of the substance during the
process?
Answer: D
0.30 Btu/lb-oF
0.25 Btu/lb-oF
0.50 Btu/lb-oF
0.20 Btu/lb-oF
128. A certain gas at 101.325 kpa and 16oC whose volume is 2.83 m3 are compressed into a
storage vessel of 0.31 m3 capacity. Before admission, the storage vessel contained the gas at a
pressure
and temperature at 137.8 kpa and 24oC; after temperature of the gas in the vessel in Kelvin?
Answer: B
298.0
319.8
180.0
420.0

127. The temperature of an ideal gas remains constant while the absolute pressure changes
from 103.4 kpa to 827.2 kpa. If the initial volume is 80 liters, what is the final volume?
Answer:B
100 li
10 li
8 li
1000 li

128. What is the density of air under standard condition: Answer: C


1.1 kg/m3
1 kg/m3
1.2 kg/m3
1 KN/m3

129. What is the specific volume of an air at 30oC and 150 kpa? Answer: D
1 m3/kg
0.5 m3/kg
1.2 m3/kg
0.579 m3/kg

130. The compression ratio of an Otto cycle is 5. If the initial pressure is 100 kpa, determine
the final pressure. Answer: B
1000 kpa
952 kpa
300 kpa
100 kpa

131. How much work is done when 20 ft3 of an air is initially at a pressure of 15 psia and a
temperature of 40oF experience an increase of pressure to 80 psi while volume remains constant?
Answer: C
1000 Btu
3000 Btu
Zero
2000 Btu

132. A perfect gas has a value of R = 58.8 ft-lb/lb-oR and k = 1.26. Btu are added to 5 lbs of
this gas at constant volume when initial temperature is 90oF, find the initial temperature. Answer:
B
100oF
104 oF
154 oF
185 oF

133. While the pressure remains constant and 689.5 kpa, the volume of air changes from 0.567
m 3 to 0.283 m3. What is the work done?Answer: A
-196 KJ
204 KJ
206 KJ
-406 KJ

134. During the polytropic process of an ideal gas, the state changes from 20 psia and 40oF to
120 psia and 340oF. Determine the value of n. Answer: C
1.233
1.255
1.355
1.400

135. A group of 50 persons attend a secret meeting in a room which is 12 meters wide by 10
meters long and a ceiling height of 3 m. The room is completely sealed off and insulated. Each
person gives off 120 kcal per hour of heat and occupies a volume of 0.2 m3. The room has an
initial pressure of 101.3 kpa and temperature of 16oC. Calculate the room temperature after 10
minutes.
Answer: B
36.35
33.10
23.24
76.32

136. The compression ratio of an ideal Otto cycle is 6:1. Initial conditions are 101.3 kpa and
20oC. Find the pressure and temperature at the end of adiabatic compression. Answer: A
1244.5 kpa, 599.96oK
1244.5 kpa, gage, 60oC
1244.5 kpa, 60oC
1244.5 kpa, 599.96oC

137. Ammonia weighing 22 kgs is confirmed inside a cylinder equipped with a piston has an
initial pressure of 413 kpa and 38oC. If 2900 KJ if heat is added to the ammonia until its final
pressure and temperature are 413 kpa and 100oC, respectively, what is the amount of work done
bt rhe fluid in KJ? Answer: A
667
304
420
502

138. Determine the average Cp value in KJ/kg-K of a gas of 522 KJ/kg of heat is necessary to
raise the temperature from 300oK to 800oK making the pressure constant: Answer: B
1.440
1.044
1.038
1.026

139. A tank contains 80 ft3 of air at a pressure of 350 psi; if the air is cooled until its pressure
and temperature decreases to 200 psi and 70 oF respectively, what is the decrease in internal
energy?
Answer: B
+4575
-5544
5552
0

140. If 10 lbs of water are evaporated at atmospheric pressure until a volume of 288.5 ft3 is
computed , how much work is done? Answer: C
1680 Btu
No work
610000 ft-lb
550000 ft-lb

141. A large mining company was provided with a 3 m3 of compressed air tank. Air pressure
in the tank drops from 700 kpa to 180 kpa while the temperature remains constant at 28oC. What
percentage has the mass of air in the tank been reduced? Answer: A
74
72
76
78

142. In a diesel cycle, the air compressed to one-tenth of its original volume. If the initial
temperature of the air is 27oC, what is the final temperature? Answer: D
420oC
440 oC
460 oC
480 oC

143. Determine the vacuum efficiency of a surface condenser which operates at a vacuum of
635 mm Hg and exhaust steam enters the condenser at 45.81o. The barometric is 760 mm Hg.
Answer: C
80.4 %
85.2%
92.2%
98.3%

144. A 4 m3/hr pump delivers water to a pressure tank. At the start, the gage reads 138 kpa
until in reads 276 kpa and then the pump was shut off. The volume of the tank is 150 liters. At
276 kpa the water occupied 2/3 of the tank volume. Determine the volume of water that can be
taken out unitl the gage reads 138 kpa. Answer: D
31.20 liters
65.32 liters
16.87 liters
29.50 liters

145. How much work is necessary to compress air in an insulated cylinder from 0.20 m3 to
0.01 m3. Use T = 20oC and P1 = 100 kpa. Answer: D
113.4 KJ
121.4 KJ
110.1 KJ
115.6 KJ

144. A gas initially 103.4 kpa and 0.0566 m3 undergoes a process to 620.36 kpa and 0.017 m3
during which the enthalpy increases by 16.35 KJ, Cv = 10.217 KJ/kgoK. Determine the Cp of gas.
Answer: A
14.33 KJ/kg-oK
2.34 Btu/lb-oR
13.23 KJ/kg-oK
10.76 Btu/lb-oK

145. A perfect gas has a value of R = 5808 ft-lb/lb-oR and k = 1.26. If 20 Btu are added to 5 lb
of this gas at constant volume when initial temperature is 90oF, find the final temperature.
Answer: C
100.76oC
167.76 oC
103.76 oC
145.76 oC

146. While the pressure remains constant at 689.5 kpa the volume of a system of air changes
from 0.567 m3 to 0.283 m3. Find the change of internal energy. Answer: B
389.68 KJ
493.63 KJ
678.68 KJ
245.68 KJ

147. If initial volume of an ideal gas is compressed to one half its original volume and to twice
its original temperature, the pressure. Answer: B
Doubles
Quadruples
Remains constant
Halves

148. What horsepower is required to isothermally compress 800 ft3 of air per minute from 14.7
psia to 120 psia? Answer: D
13900 Hp
28 Hp
256 Hp
108 Hp

149. An ideal gas at 45 psig and 80oF is heated in a closed container to 130oF. What is the final
pressure?Answer
150. A Carnot engine requires 35 KJ/sec from the hot source. The engine produces 15 kw of
power ant the temperature of the sink 26oC. What is the temperature of the hot source? Answer C
245.57
210.10
250.18
260.68

151. A Carnot engine receives 130 Btu of heat from a hot reservoir at 700oF and rejects 49 Btu
of heat. Calculate the temperature of the cold reservoir. Answer: D
-21.9oF
-24.2 oF
-20.8 oF
-22.7 oF

151. The maximum thermal efficiency possible for a power cycle operating between 1200 oF
and
225 oF is:
58%
58.73%
57.54%
57.40%

152. A heat engine is operated between temperature limits of 1370oC and 260oC. Engine
supplies with 14,142 KJ per KWH. Find the Carnot cycle efficiency in percent.Answer: C
70.10
65.05
67.56
69.32

153. An Otto cycle has clearance volume of 7%. It produces 300 kw power. What is the
amount of heat rejected in KW? Answer: C
170
160
152
145

154. A closed vessel contains air at a pressure of 160 KN/m2 gauge and temperature of 30oC.
The air is heated at constant volume of 60oC with the atmospheric pressure as 759 mm Hg. What
is the final gauge pressure? Answer: C
174
169
185.8
172
155. An air standard engine has a compression ratio of 20 and cut-off ratio of 5. If the intake
air pressure and temperature are 100 kpa and 27oC, find the work in KJ per kg. Answer: B
2976
2166
2437
2751

156. The thermal efficiency of a particular engine operating on an ideal cycle is 35%.
Calculate the heat supplied per 1200 watt-hr of work developed in KJ. Answer:A
12343
10216
14218
11108

157. In an air standard Otto cycle, the clearance volume is 18% of the displacement volume.
Find the compression ratio and or thermal efficiency. Answer: C
0.52
0.53
0.55
0.60

154. The clearance of a diesel cycle is 1.0%. If initial temperature is 27oC, determine the final
temperature. Answer: A
510oC
610 oC
540 oC
1000 oC

155. For an ideal cycle with overall value of k = 1.33, compression ratio is 15 and cur-off ratio
of 2.1, determine the cycle efficiency.Answer: B
50.62%
52.90%
46%
49%

156. An ideal Otto cycle with 15% clearance operates on 0.227 kg/sec of air with k = 1.32.
Determine the efficiency of the cycle. Answer: D
50.65%
43.84%
41.32%
47.89%

157. An ideal Otto cycle, operating in hot air with k = 1.34 has compression ratio of 5.
Determine the efficiency of the cycle. Answer: A
42.14%
62.27%
64.27%
36.46%

158. An engine operates on the air standard Otto cycle. The cycle work is 900 KJ/kg, the
maximum cycle temperature at the end of isentropic compression is 600oC. Determine the engines
compression ratio. Answer: A
6.388
10.45
8.67
7.87

159. In an air standard diesel cycle, compression start at 100 kpa and 300oK. The compression
ratio is 16 to 1. The maximum cycle temperature is 2031oK. Determine the cycle efficiency.
Answer: C
60.34%
56.23%
65.98%
45.45%
160. A diesel engine is operating on a 4-stroke cycle, has a heat rate of 11315.6 KJ/KW-hr brake.
The compression ratio is 13. The cut-off ratio is 2. Using k = 1.32, what is the brake engine
efficiency.
Answer: A
63.5%
51.2%
73.5%
45.3%

160. Determine the air-standard efficiency of an engine operating on the diesel cycle with
clearance of 8% when the suction pressure is 99.97 kpa and the fuel is injected for 6% of the
stroke. Assume k
1.4. Answer: D
63.5%
51.2%
73.5%
60.02%

161. A heat engine ( Carnot cycle ) has its intake and exhaust temperature if 157oC and 100oC,
respectively. What is its efficiency? Answer: D
12.65%
14.75%
15.35%
13.25%

162. Find the enthalpy of 100 psi and 97% quality, hf = 298.55 Btu/lb; hfg = 889.119 Btu/lb.
Answer: B
1170 Btu/lb
1161 Btu/lb
1734 Btu/lb
1803 Btu/lb

168. 180 grams of saturated water of temperature 95oC undergoes evaporation process until all
vapor completely vaporized. Determine the changed in volume. At 95oC, vf= 0.0010397 m3/kg.
Answer:
D
0.1656 m3
0.4235 m3
0.2565 m3
0.3656 m3

169. Five kilograms of saturated liquid at kpa is heated until its moisture content is 5%. Find the work
done for this process. Answer A
813.59 KJ/kg
643.23 KJ/kg
542.34 KJ/kg
753.12 KJ/kg

170. Twenty kilograms of water at 40oC is confined in a rigid vessel. The heat is supplied until all the
water is completely vaporized. Find the heat added in KJ. Answer: C
45,422 KJ
43,122 KJ
45,252 KJ
65,233 KJ

171. Steam at 2 Mpa and 250oC in a rigid cylinder is cooled until the quality is 50%. Find the heat
rejected from the cylinder.
At 2 Mpa and 250oC
V = 0.11144 m3/kg u = 2679.6 KJ/kg
At 2 Mpa, (saturated)
Vf = 0.0011767 m3/kg vg = 0.09963 m3/kg uf = 906.44 1693.8 Answer: B
170. -432.23 KJ/kg
171. -926.26 KJ/kg
172. -834.55 KJ/kg
173. 1082.34 KJ/kg

173. At 1.3 Find the entropy in KJ/kg-K at 90% moisture of a 1 Mpa steam-water mixture?

4.87
6.34
2.583
4.36

184. At 1.3 Mpa, mixture steam and water has an entropy of 4 KJ/kg-K. Find the enthalpy of the
mixture.
At 1.3 Mpa: sf = 2.2515, sg = 6.4953, hf = 514.93, hfg = 1972.7 Answer: A
1627.71 KJ/kg
1533.33 KJ/kg
1234.45 KJ/kg
1734.45 KJ/kg

187. Mixture with 80% quality at 500 kpa is heated isothermally until its pressure is 300 kpa. Find
the heat added during the process.
At 500 kpa: sf = 1.8607, sfg = 4.9606
At 300 kpa and 151.86oC: s = 7.0888 KJ/kg Answer: B
652.34 KJ/kg
535.16 KJ/kg
983.44 KJ/kg
765.34 KJ/kg

193. A tank contains exactly one kilogram of water consisting of liquid and vapor in equilibrium at 1
Mpa. If the liquid and vapor each occupy one-half the volume of the tank, what is the enthalpy of
the content of the tank? Answer: B
644.40 KJ/kg
774.40 KJ/kg
833.40 KJ/kg
435.29 KJ/kg

194. A vessel with a volume of 1 m3 contains liquid water and water vapor in equilibrium at 600 Kpa.
The liquid water has a mass of 1 kg. Using steam tables, calculate the mass of water vapor.
Answer: A
3.16 kg
0.99 kg
1.57 kg
1.89 kg

195. Water substance at 70 bar and 65oC enters a boiler tube of constant inside diameter of 25 mm.
The water leaves the boiler tube at 50 bar and 700oK at velocity of 100 m/s. Calculate the inlet
velocity(m/sec) Answer: C
1.56
2.12
1.672
3.230

197. Water substance at 70 bar and 65oC enters a boiler tube of constant inside diameter of 25 mm.
The water leaves the boiler tube at 50 bar and 700oK at velocity of 100 m/s. Calculate the inlet
volume of 100 m/s. Calculate the inlet volume flow (li/sec) Answer: A
0.821
1.561
0.344
2.133

198. One lb (0.455kg) of a mixture of steam and water at 160 psia(1103.2 kpa) is in rigid vessel. Heat
is added to the vessel until the content are at 560 psia (3861.2 kpa) and 600 oF (315.55oC).
Determine the quantityof heat in KJ added to the water and steam in the tank. Answer: A
1423.70
1392.34
1562.34
1294.45

199. Steam at the rate of 600 kg/hr is produced by a steady flow system boiler from feedwater
entering at 40oC. Find the rate at which heat is transformed in Kcal per hour if enthalpy of steam
is 660 Kcal/kg and of the feedwater at 40 Kcal/kg. Answer: A
372000
387000
345200
312444

200. Steam leaves and industrial boiler at 827.4 kpa and 171.6oC. A portion of the steam is passed
through a throttling calorimeter and is exhausted to the atmosphere when the calorimeter pressure
is 101.4 kpa. How much moisture does the steam leaving the boiler contain if the temperature is
115.6oC? Answer: B
3.78%
3.08%
4.56%
2.34%

203. Steam enters a throttling calorimeter at a pressure of 1.03 Mpa. The calorimeter downstream
pressure and temperature are respectively 0.100 Mpa and 125oC. What is the percentage
moisture if the supply steam?
Properties of steam:
P, Mpa hf hfg hg
1.03 2010.7 2779.25
Note: @ 0.1.. Mpa and 125oC: h = 2726.6 KJ/kg. Answer: A
2.62
5.21
3.15
1.98
214. A vessel of 0.058 m3 capacity is well insulated and is divided equally by a rigid conducting
diaphragm. Initially both halves contain air at pressure of 137.8 kpa and 413.4 kpa and
temperature 27oC and 177oC respectively. What is the increase of entropy of the system in KJ/oC?
Answer: C
1.002
0.5080
0.00173
0.1080

215. Using steam table. Find the enthalpy of steam at kpa if its specific volume is 0.3598 m3/kg.
Answer: A
1625.86 KJ/kg
1785.34 KJ/kg
1543.45 KJ/kg
1687.55 KJ/kg

216. A throttling calorimeter is connected to the desuoerheated steam line supplying steam to the
auxiliary feed pump on a ship. The line pressure measures 2.5 Mpa. The calorimeter pressure of
137.8 kpa and 413.4 kpa and temperature of 27oC and 177oC respectively. What is the increase of
entropy of the system in KJ/oC? Answer: C
6.8 KJ/kg-oK
7.2 KJ/kg-oK
6.6 KJ/kg-oK
7.5 KJ/kg-oK

217. Steam enters an isothermal compressor at 400oC and 100 kpa. The exit pressure is 10 Mpa,
determine the change of enthalpy. Answer: D
198 KJ/kg
178 KJ/kg
187 KJ/kg
182 KJ/kg

218. Steam enters an adiabatic at 300oC and 400 kpa. It exits as a saturated vapor at 30 kpa.
Determine the work done. Answer: B
476.34 KJ/kg
441.50 KJ/kg
436.33 KJ/kg
524.34 KJ/kg

219. A 0.5 m3 tank contains saturated steam at 300 kpa. Heat is transferred until pressure reaches 100
kpa. Find the final temperature.Answer: C
94.45oC
83.23oC
99.63oC
103.2oC
217. A 500 li contains a saturated mixture of steam and water at 300oC. Determine the mass of vapor
if their volumes are equal. Answer: A
11.54 kg
10.34 kg
13.45 kg
16.34 kg

218. A 1 kg steam-water mixture at 1.0 Mpa is contained in an inflexible tank. Heat is added until the
pressure rises to 3.5 Mpa and 400oC. Determine the heat added. Answer: A
1378.64 KJ
1532.56 KJ
1456.78 KJ
1635.45 KJ
219. Atmospheric pressure boils at 212oF. At the vacuum pressure at 24 in Hg, the temperature is
142oF. Find the boiling temperature when the pressure is increased by 45 psia. Answer: C
342.34oF
526.34oF
479.13oF
263.45oF

220. What is the temperature in degree C of 2 liters of water at 30oC after 500 calories of heat have
been added to? Answer: B
35.70
30.25
38.00
39.75

221. A mass of 0.36 kg metal having a temperature of 100oC is plunged into 0.04 kg if water at 20oC.
The temperature of the water and metal becomes 48oC. The latent heat of ice at 0oC is 335 KJ.kg-
oK, Assuming no heat loss to the surroundings, determine the specific heat capacity of the metal in

KJ/kg-oK. Answer: D
0.234
0.564
0.754
0.451

222. What is the total energy required heating in raising the temperature of a given amount of water
when the energy applied is 1000 KWH with heat losses of 25%? Answer: C
1000
1500
1333
1250

223. A 30 kg iron was put in a container with 14.33 kg water. The water is at 10oC and the iron has an
initial temperature of 439oK until the iron was in thermal equilibrium with water. Find the change
in entropy. (Cp = 0.4 KJ/kgoK) Answer: D
-12.56 KJ/oK
-43.58 KJ/oK
-25.78 KJ/oK
-6.6 KJ/oK
FUELS & COMBUSTION

219. A diesel power plant consumes 650 liters of fuel at 26oC in 24 hours with 28oAPI. Find
the fuel rate in kg/hr. Answer: A
23.83
24.85
22.85
26.85

220. A boiler burns fuel oil with 15% excess air. The fuel oil may be represented by C14H34.
Calculate the molal air fuel ratio. Answer: B
14
117.69
102.34
17.14

221. A diesel power plant uses fuel that has a density of 892.74 kg/m3 at 15.67oC. Find the
heating value of fuel. Answer: A
44,900 KJ/kg
19,301 Btu/lb
43,000 KJ/kg
4356.2 KJ/kg

222. A certain coal has the following ultimate analysis:


C=69% N2=5%
H2 = 2.5% S = 7% Moisture = 8%
O2 = 3.5% Ash = 5%

Determine the heating value of fuel used.Answer: A


26,961.45 KJ/kg
45,256 KJ/kg
25,002.4 KJ/kg
26,000 KJ/kg

224. A diesel power plant uses fuel with heating value of 45,038.8 KJ/kg. What is the density
of fuel used at 30°C?Answer: D
0.900 kg/li
0.887 kg/li
0.850 kg/li
0.878 kg/li

225. A diesel engine consumed 945 liter of fuel per day at 30°C. If the fuel was purchased at
15.5°C and 30° API at P5.00/li, determined the cost of fuel to operate the engine per day.
Answer: B
P5677.50
P4677.50
P4888.90
P5000.00
227. A cylindrical tank 3 m long and 2 m diameter is used for oil storage. How many days can
the tank supply the engine having 27° API with fuel consumption of 60 kg/hr. Answer: B
4.84
5.84
7.84
8.84

228. Determine the minimum volume of day tank in m3 of 28° API fuel having a fuel
consumption of 200 kg/hr. Answer: B
10.43 m3
5.41 m3
6.87 m3
7.56 m3

229. Given the following ultimate analysis:


C=70% O2=4%
N2 = 5% S2=6% Moisture = 8%
H2 = 3% Ash = 5%

11.46
24.85
23.85
26.85

232. A 650 Bhp diesel engine uses fuel oil of 28° API gravity, fuel consumption is 0.65
lb/Bhp-hr. Cost of fuel is P7.95 per liter. For continuous operation, determine the minimum
volume of cubical day tank in cm3, ambient temperature is 45°C. Answer: A
5,291,880 cm3
5,391,880 cm3
5,491,880 cm3
5,591,880 cm3

233. A 650 Bhp diesel engine uses fuel oil of 28° API gravity, fuel consumption is 0.65lb/Bhp-hr.
Cost of fuel is P7.95 per liter. For continuous operation, determine the cost of fuel per day at 45°C.
Answer: B
P42,870.45
P42,070.45
P42,570.45
P42,170.45

234. A logging firm in Isabela operates a Diesel Electric Plant to supply its electric energy
requirements. During a 24 hour period, the plant consumed 250 gallons of fuel at 80°F and
produced 2700 KW-Hrs. Industrial fuel used is 30° API and was purchased at P3.00li/at 60°F.
Determine cost of fuel to produce one Kwh. Answer: C
P3,043/KW-h
P4,043/KW-h
P1,043/KW-h
P2,043/KW-h
233. A logging firm in Isabela operates a Diesel Electric Plant to supply its electric energy
requirements. During a 24 hour period, the plant consumed 250 gallons of fuel at 80°F and
produced 2700 KW-hrs. Industrial fuel used is 30° API and was purchased at P3.00/li at 60°F.
Determine the overall thermal efficiency of the plant. Answer: A
26.08%
43.23%
43.12%
18.46%

234. A diesel electric plant in one of the remote provinces in the south utilized diesel fuel with
an °API of 28 at 15.6°C. The plant consumes 680 liters of diesel fuel at 26.6°C in 24 hrs, while
the power generated for the same period amounts to 1,980KW-hrs. Determine overall thermal
efficiency of the plant. Answer: A
26.47%
12.34%
23.45%
34.34%

235. A circular fuel tank 45 feet long and 5.5 feet diameter is used for oil storage. Calculate
the number of days the supply tank can hold for continuous operation the following conditions:
Steam flow = 2000 lbs/hr
Steam dry and saturated at 200 psia
Feedwater temperature = 230°F

12.34
17.58
13.45
23.44

238. A steam generator burns fuel oil with 20% excess air. The fuel oil may be represented by
C14 H30. The fuel gas leaves the preheater at 0.31 Mpa. Determine the actual air-fuel ratio in kg air
per kg fuel. Answer: A
17.89
15.67
12.34
19.45

239. A steam generation burns fuel oil with 20% excess air. The fuel oil may be represented
by C14H30. The fuel gases leave the preheater at 0.31 Mpa. Determine partial pressure of H2O.
Answer: C
23.34 kpa
29.34 kpa
35.7 kpa
32.34 kpa
240. In the boiler design, it is desired to have the flue gas exit temperature above the dew
point. Estimate the dew point temperature of the flue gas produced by combustion having the
gravimetric analysis of: Answer: D
N2 = 71.84% CO2 = 20.35% O2 = 3.61% H2O = 4.20%
244. 32°C
245. 23°C
246. 45°C
247. 39°C

19. A flue gas has following volumetric analysis:


CH4 = 68% C2H6 = 32%
Assume a complete combustion with 15% excess air at 101.325 Kpa, what is the partial pressure
of water vapor in Kpa. Answer: A
249. 15.95
250. 23.12
251. 12.45
252. 27.34

20. The dry exhaust gas from oil engine has the following gravimetric analysis:
CO2 = 21.6% O2 = 4.2% N2 = 74.2%
Specific heat at constant pressure for each components of the exhaust gas in Kcal/kg °C are:
CO2 = 0.203 O2 = 0.219 N2 = 0.248
Calculate the gas constant in J/kg°K.Answer: A
A. 272 C. 274
B. 276 D. 278

21. The dry exhaust gas from oil engine has the following gravimetric analysis:
CO2 = 21.6% O2 = 4.2% N2 = 74.2%
Specific heat at constant pressure for each component of the exhaust gas in Kcal/kg °C are:
CO2 = 0.203 O2 = 0.219 N2 = 0.248
Calculate the specific gravity if the molecular weight of air is 28.97 kg/kg-mol. Answer: C
254. 0.981
255. 1.244
256. 1.055
257. 0.542

22. The dry gas from oil engine has the following gravimetric analysis:
CO2 = 21.6% O2 = 4.2% N2 = 74.2%
Specific heat at constant pressure for each component of the exhaust gas in Kcal/kg °C are:
CO2 = 0.203 O2 = 0.219 N2 = 0.248

256. 0.872
257. 0.992
258. 0.452
0.673

267. A bituminous coal has the following composition:


C = 71.5% H = 5.0% O = 7.0% N=1.3
S = 3.6% Ash = 8.2% W = 3.4%

2.870
3.120
2.274
6.233

273. There are 20 kg of flue gases formed per kg of fuel oil burned in the combustion of a fuel
oil C12H26. What is the excess air in percent?Answer: A
26.67%
18.34%
12.34%
20.45%

274. A gaseous fuel mixture has a molal analysis:


H2 = 14% CH4 = 3% CO = 27%
O2 = 0.6% CO2 = 4.5% N2 = 50.9%
Determine the air-fuel ratio for complete combustion on molal basis.Answer: C
276. 2.130
277. 3.230
278. 1.233
279. 1.130

277. A steam generation burns fuel oil that has the following chemical analysis by mass in
percent:
Answer: C
C = 85.3 H2 = 14.1 S=0.5 N2 = 0.1
A. 19.85 C. 17.93
B. 11.14 D. 14.20

281. Calculate the theoretical air needed for the complete combustion of ethane C2H6 for 20 kg
fuel.
Answer: D
432.23 kg
287.45 kg
234.45 kg
320.32 kg

282. A fuel oil burned with 50 percent excess air, and the combustion characteristics of the
fuel oil are similar to C12H26. Determine the volumetric (molal) analysis of CO2 in the production
of combustion. Answer: B
9.34%
8.66%
6.34%
7.45%

283. Fuel oil, C12H26 is burned in air at atmospheric pressure. The Orsat analysis of the
products of combustion yields.
CO2 : 12.8%
O2 : 3.5%
CO : 0.2%
N2 : 83.5%

12.34%
8.34%
10.52%
18.45%

284. An unknown fuel has the following Orsat


analysis: CO2 : 12.5
CO: 0.3%
O2: 3.1%

17.13
12.34
19.23
23.23

292. What mass of liquid oxygen is required to completely burned 1000kg of liquid butane, C
4 H10 on a rocket ship? Answer: C
4568
2746
3586
6345

293. A volumetric analysis of a gas mixture is as follows:


CO2 : 12.5%
O2 :4%
N2 : 82%
CO : 2%

What is the percentage of CO on a mass basis? Answer: D


1.0%
1.5%
1.2%
1.9%

300. What is the percent theoretical air for a combustion process to which the fuel and
combustion gas analysis are known as follows:Answer: A
Fuel: % by volume CO2
: 12.4% CO :
27%
H2: 2.2%
N2 : 58.4%
Combustion gas: % by volume
CO2 : 24.6%
O2 : 1.0%
H2 : 74.4%
303. 111
304. C.116
305. 121
306. D.126

VARIABLE LOAD

304. A 50 MW power plant has an average load of 31,500 KW and a factor of 70%. Find the
reserve over peak. Answer: C
4 MW
3 MW
5 MW
6 MW

305. The daily energy produced in a certain power is 280,000 KW – hrs. What is the daily average
load?
Answer: D
10 MW
15 MW
25 MW
20 MW

306. The annual energy produced in a 100 MW power plant is 438,000,000 KW – hrs. What is
the annual capacity factor of the plant? Answer: B
40%
50%
35%
60%

307. A power plant has a use factor of 50% and factor of 44%. How many hours did it operate
during the year? Answer: C
7700 hrs.
7800 hrs.
7709 hrs.
7805 hrs.

STEAM CYCLE

1. In a Rankine cycle steam enters the turbine at 2.5 Mpa (enthalpies & entropies given) and
condenser of 50 Kpa (properties given), what is the thermal efficiency of the cycle. Answer A
At 2.5 Mpa: hg = 2803.1 KJ/kg sg = 6.2575
At 50 kpa: sf = 1.0910 sfg = 6.5029
hf = 340.49 hfg = 2305.4
vf = 0.0010300
308. 25.55%
309. 45.23%
34.23%
12.34%

313. In an ideal Rankine cycle, the steam throttle condition is 4.10 Mpa and 4400C. If turbine
exhaust is 0.105 Mpa, determine the pump work in Kj per kg. Answer C
6.34
5.34
4.17
2.12

314. A thermal power plant generates 5 MW has also 300 OW power needed for auxiliaries. If
the heat generated by fuel is 13,000 KJ/sec, determine the net thermal efficiency. Answer B
35.78%
36.15%
30.56%
33.67

315. In an ideal Rankine cycle, the steam throttle condition is 4.10 Mpa and 4400C. If turbine
exhaust is 0.105 Mpa, determine the thermal efficiency of the cycle. Answer B
20.34%
27.55%
34.44%
43.12%

316. In a Rankine cycle, saturated liquid water at 1 bar is compressed isentropically to 150
bar. First by heating in a boiler, and then by superheating at constant pressure of 150 bar, the
water substance is brought to 7500K. After adiabatic reversible expansion in a turbine to 1 bar, it
is then cooled in a condenser to saturated liquid. What is the thermal efficiency of the
cycle(%)?Answer
C
23.45%
16.23%
34.24%
18.23%

317. A steam generating plant has two 20 MW turbo-generators. Steam is supplied at 1.7 Mpa
and 3200C. Exhaust is 0.006 MPa. Daily average load factor is 80%. The steam generating units
operate at 70% efficiency when using bunker fuel having a heating value of 31,150 KJ/kg and an
average steam rate of 5 kg steam/KW-hr. Calculate the Mtons of fuel oil/bunker fuel required per
24 hours. Answer A
515
432
6.17
762
317. A back pressure steam turbine of 100,000 KW serves as a prime mover in a cogeneration
system. The boiler admits the return water at a temperature of 660C and produces the steam at 6.5
Mpa and 4550C. Steam then enters a back pressure turbine and expands to the pressure of the
process, which is 0.52 Mpa. Assuming a boiler efficiency of 80% and neglecting the effect of
pumping and the pressure drops at various location, what is the incremental heat rate of electric?
The following enthalpies have been found; turbine entrance = 3306.8 KJ/kg, exit = 2700.8; boiler
entrance = 276.23 KJ/kg, exit =3306.8. Answer B
21,504 KJ/KW-hr
22,504 KJ/KW-hr
23,504 KJ/KW-hr
24,504 KJ/KW-hr

318. A coal-fired power plant has a turbine-generator rated at 1000 MW gross. The plant
required about 9% of this power for its internal operations. It uses 9800 tons of coal per day. The
coal has a heating value of 6,388.9 Kcal/kg and the steam generator efficiency is 86%. What is
the net station efficiency of the plant in percent? Answer C
30%
25%
33%
38%

319. A superheat steam Rankine cycle has a turbine inlet conditions of 17.5 Mpa and 5300C
expands in a turbine to 0.007 Mpa. The turbine and pump polytropic efficiencies are 0.9 and 0.7
respectively, pressure between pump and turbine inlet are 1.5 Mpa. What shoul be the pump work
in KJ/kg.
Answer B
17.3
27.3
37.3
47.3

319. A steam plant operates with initial pressure of 1.70 Mpa and 3700C temperature and
exhaust to a heating syste at 0.17 Mpa. The Condensate from the heating system is returned to the
boiler at 65.50C and the heating system utilizes from its intended purpose 90% of the energy
transferred from the steam it receives. The nT is 70%. If boiler efficiency is 80%, what is the
cogeneration
efficiency of the system in percent. Neglect pump work. Answer
D Steam Properties:
At 1.70 Mpa and 3700C: h = 3187.1 KJ/kg s = 7.1081
At 0.17 Mpa: hf = 483.20 sf=1.4752
hg = 2216.0 sg=5.7062
At 65.50C: hf = 274.14
A. 78
102.10
91.24
69

324. In a cogeneration plant, steam enters the turbine at 4 Mpa and 4000C. One fourth of the
steam is extracted from the turbine at 600 Kpa pressure for process heating. The remaining steam
continues to expand to 10 Kpa. The extracted steam is then condensed and mixed with feedwater
are constant pressure and the mixture is pumped to the boiler pressure of 4 Mpa. The mass flow
rate of steam through the boiler is 30 kg/sec. Disregarding any pressure drops and heat losses in
the piping, and assuming the turbine and pump to be isentropic, how much process heat is
required in KW? Answer A
Steam properties:

At 600 Kpa: hf = 670.56 sf = 1.9312


hfg = 2086.3 sfg
= 4.8288
15,646.8
2,458.2
1.9312
1,027.9

328. In an ideal Reheat cycle, the steam throttled condition is 8 Mpa and 4800C. The steam is
then reheated to 2 Mpa and 4600C, determine cycle efficiency. Answer A
38.3%
24.3%
34.3%
45.2%

329. A reheat steam cycle has 13,850 Kpa throttle pressure at the turbine inlet and a 2800 Kpa
reheat pressure, the throttle and reheat temperature of the steam is 540 0C, condenser pressure is
3.4 Kpa, engine efficiency of high and low pressure is 75% find the cycle thermal efficiency.
Answer A
34.46%
38.40%
36.49%
43.45%

330. Steam is delivered to turbine at 5.4 Mpa and 6000C. Before condensation at 310C, steam
is extracted for feedwater heating at 0.6 Mpa. For an ideal regenerative cycle, find the thermal
efficiency. Answer D
23.45%
34.34%
28.34%
44.14%

331. A steam condenser receives 10 kg/s of steam with an enthalpy of 2570 KJ/kg. Steam
condenses into a liquid and leaves with an enthalpy of 160 KJ/kg. Cooling water passes through
the condenser with temperature increases from 13 degrees C to 240C/ Calculate the water flow
rate in kg/s. Answer C
533
518
523
528

334. In an open feedwater heater for a steam plant, saturated steam at 7 bar is mixed with
subcooled liquid at 7 bar and 250C. Just enough steam is supplied to ensure that the mixed steam
leaving the heater will be saturated liquid at 7 bar when heater efficiency is 90%. Calculate the
mass flow rate of subcooled liquid if steam flow rate is 0.865 kg/s.
Steam Properties are: Answer A
At 7 bar, saturated vapor: hg = 2763.5 KJ/kg
At 7 bar, and 250C: hf = 105.5 KJ/kg
At 7 bar, saturated liquid: hf = 697.22 KJ/kg
2.725
3.356
2.286
3.948
`
336. Steam expands adiabatically in a turbine from 2000 Kpa, 4000C to 400 Kpa, 2500C. What
is the effectiveness of the process in percent assuming an atmospheric pressure of 150C. Neglect
changes in kinetic and potential energy. Answer C
Steam Properties are:
At 2000 Kpa and 4000C.
h = 3247.6 KJ/kg
s = 7.1270 KJ/kg-K At
400 Kpa and 2500C.

82
84
80
86

340. Steam enters the turbine of a cogeneration plant at 0.7 Mpa and 5000C. Steam at a flow
rate of 7.6 kg/s is extracted from the turbine at 600 Kpa pressure for process heating. The
remaining steam continues to expand to 10 Kpa. The recovered condensates are pumped back to
the boiler. The mass flow rate of steam that enters the turbine is 30 kg/s. Calculate the
cogeneration efficiency in percent. Answer No exact answer in the choices
Steam Properties
At 7.0 Mpa and 5000C:
h = 3410.3
s = 6.7976
At 600 Kpa:
hf = 670.56
hfg = 2086.3
sfg = 4.8288
At 10 Kpa:
hf = 191.83
hfg = 2392.8
sf = 0.6493
sfg = 7.5009
60
50
65
55

344. A heat exchanger was installed purposely to cool 0.50 kg of gas per second. Molecular
weight is 28 and k = 1.32. The gas is cooled from 1500C to 800C. Water is available at the rate of
0.30 kg/s and at a temperature of 120C. Calculate the exit temperature of the water in 0C. Answer
A
48
42
46
44

345. Pump work of Rankine cycle is 15 KJ/kg. Density of water entering the pump is 958
kg/m3. If condenser pressure is 100 Kpa, what is the pressure at the entrance of the turbine?
Answer A
14.47 Mpa
20.48 Mpa
15.67 Mpa
17.77 Mpa

346. In a Rankine cycle the turbine work is 1,000 KJ/kg and pump work of 13 KJ/kg. If heat
generated by generator is 2800 KJ/kg, what is the efficiency of the cycle? Answer A
35.25%
4.75%
38.65%
30.25%

347. In a Reheat power plant the difference in enthalpy at the entrance and exit is 550 KJ/kg
for first stage and second stage is 750 KJ/kg. If both stages has an efficiency of 92% and heat
added to boiler is 3,000 KJ/kg. Determine the plant cycle efficiency neglecting the pump work.
Answer C
30%
35%
40%
45%

348. An adiabatic feed pump in a steam cycle delivers water to a steam generator at a
temperature of 2000C and a pressure of 10 Mpa. The water eneters the pump as a saturated liquid
at 1800C. If the power supplied to the pump is 75 kW, determine the mass flow rate. Answer C
6.23 kg/s
8.34 kg/s
7.39 kg/s
9.12 kg/s
346. A condenser receives steam from a turbine at 15 kPa, 90% quality, and with a velocity of
240 m/s. Determine the increase in circulating water required because of high pressure exit
velocity if the steam flow rate is 38 kg/s and the cooling water temperature rise is12 0C with an
inlet condition of 300C. Answer D
11.23 kg/s
17.23 kg/s
13.23 kg/s
21.78 kg/s

347. A steam generator has an exit enthalpy of 3195.7 KJ/kg at the rate of 10 kg/s. The
enthalpy available at the turbine inlet is 3000 KJ/kg. Determine the heat lost between boiler outlet
and turbine inlet. Answer B
1957 kW
-1957 kW
1873 kW
-1873 kW

348. A Rankine cycle has a turbine with available enthalpy of 800 KJ/kg. The pump has also
10 KJ/kg energy available. Find the net cycle output of the plant if mass flow rate is 5 kg/s.
Answer B
2619 kW
3950 kW
8745 kW
4234 kW

349. A Rankine cycle, the steam is extracted from the turbine at 2 Mpa and 250 0C for
feedwater heating and it is mixed with condenser exit at 30 Kpa after pumping. Find the fraction
of vapor extracted from the turbine. Answer A
0.23464
0.19338
0.5632
0.3855

350. An adiabatic turbine in a steam generating plant receives steam at a pressure of 7.0 Mpa
and 5500C and exhausts at 20 kPa. The turbine inlet is 3 m higher than the turbine exit, the inlet
steam velocity is 15 m/s and the exit is 300 m/s. Calculate the turbine work in KJ/kg. Answer B
1297.45
1197.10
1093.45
1823.45

351. A steam power plant operates on the Rankine cycle. The steam enters the turbine at 7
Mpa and 5500C with a velocity of 30 m/s. It discharge to the condenser at 20 kpa with a velocity
of 90 m/s. Calculate the net work in kW for a flow of 37.8 kg/s. Answer D
23.23 MW
53.34 MW
34.22 MW
46.54 MW
351. A Carnot cycle uses steam as the working substance and operates between pressure of 7
Mpa and 7 kpa. Determine the cycle thermal efficiency. Answer A
44.17%
54.23%
34.23%
59.4%

352. A supercritical power plant generates steam at 25 Mpa and 5600C. The condenser
pressure is 7.0 kpa. Determine the exit quality of steam if it expands through a turbine in this
power plant.
Answer D
45.66%
68.45%
56.56%
74.26%

353. Steam enters a turbine at 1.4 Mpa and 3200C. The turbine internal efficiency is 70%, and
the total requirement is 800 kW. The exhaust is to the back pressure system, maintained at 175
kpa. Find the steam flow rate. Answer A
2.62 kg/s
4.23 kg/s
3.23 kg/s
5.34 kg/s

BOILERS

356. The heating surface are of water tube boiler is 200 m2, what is the equivalent rated boiler
horsepower? Answer D
217 Hp
2365.93 Hp
200 Hp
219.78 Hp

357. The rated boiler horsepower of a fire tube boiler is 500 Hp. What is the heating surface
area of the boiler? Answer D
500 m2
300 m2
400 m2
550 m2

358. A water tube boiler has a heating surface area of 500 m2. For a developed boiler hp of
825. Determine the percent rating of the boiler. Answer C
120.15%
160.15%
150.15%
300.15%
358. The factor of evaporation of boiler is 1.1 and a steam rate of 0.79 kg/sec. What is the
developed boiler horsepower? Answer B
300
200
869
250

359. The percent rating of water tube boiler is 200%, factor of evaporation of 1.08 and heating
surface area is 256 m2. Find the rate of evaporation. Answer A
8,153.02 kg/hr
5,153.02 kg/hr
7,200 kg/hr
8,500.46 kg/hr

360. The actual specific evaporation of a certain boiler is 10. Factor of evaporation is 1.05. If
the heating value of fuel is 30,000 KJ/kg, find the boiler efficiency. Answer A
60%
65%
70%
79%

361. The ASME evaporation units of a boiler is 24,827,000 KJ/hr. The boiler auxiliaries
consume 1.5 MW. What is the net boiler efficiency if the heat generated by the fuel is 30,000
KJ/kg?Answer A
64.75%
68.94%
62.76%
68.54%

362. A 100,000 kg of coal supplied two boilers. One has a capacity of 150 kg/hr. How many
days to consume the available fuel if the other boiler consumes 200 kg/hr? Answer D
5 days
7 days
15 days
12 days

363. The following coal has the following ultimate analysis by weight:
C = 70.5% H2 = 4.5% O2 = 6.0% N2 = 1.0%
S = 3.0% Ash = 11% Moisture = 4%
A stocker fired boiler of 175,000 kg/hr steaming capacity uses this coal as fuel. Calculate volume
of air in m3/hr with air at 600F and 14.7 psia pressure if boiler efficiency is 70% and FE = 1.10.
Answer A
358. 212,830 m3/hr
359. 215,830 m3/hr
360. 213,830 m3/hr
361. 214,830 m3/hr

10. The following coal has the following ultimate analysis by weight:
C = 70.5% H2 = 4.5% O2 = 6.0% N2 = 1.0%
S = 3.0% Ash = 11% Moisture = 4%
A stocker fired boiler of 175,000 kg/hr steaming capacity uses this coal as fuel. Volume of air in
m3/hr with air at 600F and 14.7 psia pressure. Weight in metric tons of cal needed for 24 hours
operation at rated capacity if boiler efficiency is 70% and FE = 1.10. Answer D
503.443 Mtons
508.443 Mtons
502.443 Mtons
504.443 Mtons

367. Two boilers are operating steadily on 91,000 kg of coal contained in a bunker. One boiler
is producing 1591 kg of steam per hour at 1.2 factor of evaporation and an efficiency of 65% and
another boiler produced 1364 kg of steam per hour at 1.15 factor of evaporation and an efficiency
of 60%. How many hours will the coal in the bunker run the boilers if the heating value of coal is
7590 Kcal/kg.Answer C
220.8 hrs
256.2 hrs
230.8 hrs.
453.3 hrs.

368. A steam generating plant consisting of a boiler, an economizer and superheater generates
superheated steam at the rate of 50 tons/hr. Feed water enters the boiler at 5 Mpa and 120 0C.
Steam leaves the superheater at 4.5 Mpa and 3200C. If the coal used has a heating value of 30,000
KJ/kg, calculate the number of tons of coal fired per hour for a gross efficiency of 85%. Answer
A
4.89
6.34
5.34
45.5

369. A water tube boiler has a capacity of 1000 kg/hr of steam. The factor of evaporation is
1.3, boiler rating is 200%, boiler efficiency is 5%, heating surface area is 0.91 m2/boiler Hp, and
the heating value of fuel is 18,400 Kcal/kg. The total coal available in the bunker is 50,000 kg.
Determine total number of hours to consume the available fuel. Answer D
533.45
743.12
634.34
853.26

370. A boiler operating at 11 kg/cm2 is required to generate a minimum of 50,000 kj/hr of saturated
steam. Feed water enters the boiler at 800C. Compute the fore coal at an average rate 4,800 kg/hr
and boiler efficiency is 85%. Compute the minimum heating value of local coal that could be
used in order to ensure the generation of required steam. Answer B
28,464
29,977
23,556
30,976

371. A boiler operating at 11 kg/cm2 is required to generate a minimum of 50,000 kg/hr of


saturated steam. Feed water enters the boiler at 800C. The furnace is designed to fire coal at an
average rate 4,800 kg/hr and boiler efficiency is 85%. Compute the developed boiler horsepower.
Answer A
3462.56 hp
1234.56 hp
2345.67 hp
4233.34 hp

371. A steam boiler generating 7.1 kg/s of 4.137 Mpa, 426.70-C steam is continuously blown
at the rate of 0.31 kg/sec. Feed water enters the economizer at 149.90C. The furnace burns 0.75 kg
coal/sec of 30,470.6 KJ/kg higher heating value. Calculate the overall thermal efficiency of steam
boiler. Answer C
76.34%
84.23%
82.78%
88.34%

STEAM ENGINE

373. A steam engine has 10% brake thermal efficiency and delivers 750 kg/hr steam. The
enthalpy of engine entrance is 2800 KJ/kg and condenser exit is 450 KJ/kg. Determine the brake
power of the engine. Answer C
46 KW
47 KW
49 KW
48 KW

374. The indicated efficiency of a steam engine is 60%. The engine entrance is 2700 KJ/kg
and exit is 2000 KJ/kg. If steam consumption is 800 kg/hr and mechanical efficiency is 90%,
what is the brake power of the engine? Answer B
55 KW
84 KW
65 KW
70 KW

375. A steam engine has bore and stoke of 300 mm x 420 mm running at 250 rpm has mean
indicated pressure of the engine is 400 Kpa. Determine the indicate power. Answer D
100 KW
50 KW
65 KW
99 KW

376. The crank shaft of a double acting steam engine rotates at 220 rpm. The bore and stroke
of the steam engine is 300 mm x 450 mm, and the mean effective pressure acting upon the piston
is 4 kg/cm2. Find the indicated horsepower developed in the cylinder. Answer A
122.7 hp
110.3 hp
143.2 hp
176.3 hp
376. A steam engine develops 60 Bhp with dry saturated steam at 1034.25 Kpa absolute and
exhaust at 124.11 Kpa. Steam consumption is 736.36 kg/hr. Calculate the indicated engine
efficiency based on 90% mechanical efficiency. Answer D
34.23%
54.23%
45.23%
66.74%

377. A steam engine develops 60 Bhp with dry saturated steam at 1034.25 Kpa absolute and
exhaust at 124.1 Kpa. Steam consumption is 736.36 kg/hr. calculate the thermal efficiency of
equivalent Rankine engine. Answer A
15.59%
34.23%
12.45%
21.34%

378. A 350 mm x 450 mm engine running at 260 rpm has an entrance steam condition of 2
Mpa and 2300C and exit at 0.1 Mpa. The steam consumption is 2,000 kg/hr and mechanical
efficiency is 88%. If indicated mean effective pressure is 600 Kpa, determine the brake thermal
efficiency.
Answer C
23.34%
18.34%
14.66%
27.34%

379. A 350 x 450 mm engine running at 260 rpm has an entrance steam condition of 2 Mpa
and 2300C and exit at 0.1 Mpa. The steam consumption is 2,000 kg/hr and mechanical efficiency
is 88%. If indicated mean effective pressure is 600 Kpa, determine indicated thermal efficiency.
Answer A
16.66%
34.23%
12.34%
21.23%

380. Steam is admitted to the cylinder of an engine in such a manner that the average pressure
is 120 psi. The diameter of the piston is 10” and the length of stroke is 12”. Answer D
171.5
175
173.2
174.4

STEAM TURBINE

378. A steam turbine receives 5,000 kg/hr of steam at 5 Mpa and 4000C and velocity of 25
m/sec. It leaves the turbine at 0.006 Mpa and 85% quality and velocity of 20 m/sec. Radiation is
10,000 KJ/hr. Find the KW developed. Answer C
1273.29
2173.29
1373.29
7231.29
382. An industrial power plant requires 1.5 kg of dry saturated steam per second at 165 0C for
heating purposes. This steam may be supplied from an extraction turbine which receives steam at
4 Mpa and 3800C and is exhaust to condenser at the rate of 0.8 kg steam per second at 0.0034
Mpa while rejecting 1400 KW to the cooling water. If mechanical-electrical efficiency is 95%
and that the heat loss in the turbine casing is 10 KW, calculate the power generated by the plant.
Answer B
2,126.44 KW
1,556.50 KW
3,123.34 KW
4,344.33 KW

383. A steam turbine with 90% stage efficiency receives steam at 7 Mpa and 5500C and
exhausts as 20 Kpa. Determine the turbine work. Answer A
117 KJ/kg
132 KJ/kg
123 KJ/kg
143 KJ/kg

384. A steam turbine with 90% stage efficiency receives steam at 7 Mpa and 5500C and
exhausts as 20 Kpa. Determine the quality at exhaust. Answer D
87.45%
76.34%
82.34%
91.69%

385. A small steam turbine power plant of 5,000 KW capacity has a full load steam rate of 6
kg steam pwer KW-hr. No load steam consumption may be taken as 10% of the full load steam
consumption. Write the equations of WILLANS LINE of this turbine and at 60% of rated load,
calculate the hourly steam consumption of this unit. Answer C
19,100 kg/hr
19,300 kg/hr
19,200 kg/hr
19,400 kg/hr

386. Steam flows into a turbine at the rate of 10 kg/sec and 10 KW of heat are lost from the
turbine. Ignoring elevation and kinetic energy effects, calculate the power output from the
turbine. Inlet enthalpy is 2739 KJ/kg and exhaust enthalpy is 2300.5 KJ/kg. Answer C
4605 KW
4973 KW
4375 KW
4000 KW

387. Steam entering the turbine has a rate of 10 kg/sec with enthalpy of 3400 KJ/kg and 2600
KJ.kg at the exhaust. If 100 KW of heat is rejected from turbine casing, what is the turbine work?
Answer
A
7900 kW
7700 KW
5600 KW
5400 KW
383. A steam turbine of 6 MW capacity has a Willian’s line equation of ms = 5.5L + 3,000
kg/hr. Determine the steam consumption at 70% load. Answer C
3564 kg/hr
3546 kg/hr
26,100 kg/hr
58,000 kg/hr

384. A steam turbine has an entrance enthalpy of 3050 KJ/kg. The exit has 2500 KJ/kg.
Determine the actual enthalpy after isentropic expansion if stage efficiency is 90%. Answer B
1255 KJ/kg
2555 KJ/kg
2500 KJ/kg
2000 KJ/kg

385. Steam enters the turbine at the rate of 2.5 kg/sec with enthalpy of 3200 KJ/kg and
exhaust enthalpy of 1100 KJ/kg. Steam is extracted from the turbine at the rate of 1 kg/sec for
heating purposes with enthalpy of 2750 KJ/kg. What is the turbine work? Answer D
2000 KW
2500 KW
3000 KW
3600 KW

386. A steam turbine has an entrance enthalpy of 3400 KJ/kg and 2500 KJ/kg at exit. If
generator generates 2430 KW and has 90% efficiency, what is the mass of steam entering the
turbine?
Answer D
10,400 kg/hr
10,600 kg/hr
10,700 kg/hr
10,800 kg/hr

387. Steam turbine in Rankine cycle has an exhaust enthalpy of 2650 KJ/kg and delivers 0.8
kg/sec of steam. Determine the heat rejected from the condenser if enthalpy at exit is 200 KJ/kg.
Answer A
1960 KW
1940 KW
1995 KW
1909 KW

GEOTHERMAL POWER PLANT

388. Mass low rate of ground water in a geothermal power plant is 1,500,000 kg/hr and the
quality after throttling is 30%. Determine the brake power of turbine if the change of enthalpy of
steam at inlet and outlet is 700 KJ/kg. Answer B
68.5 MW
87.5 MW
64.5 MW
89.5 MW
390. Ground water of geothermal power plant has an enthalpy of 700 KJ/kg and at turbine
inlet is 2,750 KJ/kg and enthalpy of hot water in flash tank is 500 KJ/kg. What is the mass of
steam flow entering the turbine if mass flow of ground water is 45 kg/sec? Answer C
3.27 kg/sec
2.27 kg/sec
4.27 kg/sec
9.27 kg/sec

391. The enthalpy entering the turbine of a geothermal power plant is 2750 KJ.kg and mass
rate of 1 kg/sec. The turbine brake power is 1000 KW condenser outlet has enthalpy of 210
KJ/kg. If temperature rise of cooling water in condenser is 80C, what is the mass of cooling water
requirement? Answer C
44 kg/sec
45 kg/sec
46 kg/sec
47 kg/sec

392. In a 12 MW geothermal power plant, the mass flow of steam entering the turbine is 26
kg/sec. The quality after throttling is 25% and enthalpy of ground water is 650 KJ/kg. Determine
the overall efficiency of the plant. Answer D
7.4%
9.6%
5.4%
15.4%

393. A 16,000 KW geothermal plant has a generator efficiency and turbine efficiency of 90%
and 80%, respectively. If the quality after throttling is 20% and each well discharges 200,000
kg/hr, determine the number of wells are required to produce if the change in enthalpy at the
entrance and exit of turbine is 500 KJ/kg. Answer A
4 wells
5 wells
6 wells
8 wells

394. A geothermal power plant draws pressurized water from a well at 20 Mpa and 300 0C. To
produce a steam water mixture in the separator, where the unflashed water is removed, this water
is throttled to a pressure of 1.5 Mpa. The flashed steam which is dry and saturated passes through
the steam collector and enters the turbine at 1.5 Mpa and expands to 1 atm. The turbine efficiency
is 85% at a rated power output of 10 MW. Calculate overall plant efficiency. Answer A
7.29%
12.34%
9.34%
19.45%

395. A flashed steam geothermal power plant is located where underground hot water is
available as saturated liquid at 700 Kpa. The well head pressure is 600 Kpa. The flashed steam
enters a turbine at 500 Kpa and expands to 15 Kpa, when it is condensed. The flow rate from the
well is 29.6 kg/sec. Determine the power produced in KW. Answer A
430.13 kg/s
540.23 kg/s
370.93 kg/s
210.34 kg/s

397. A flashed steam geothermal power plant is located where underground hot water is
available as saturated liquid at 700 Kpa. The well head pressure is 600 Kpa. The flashed steam
enters a turbine at 500 Kpa and expands to 15 Kpa, when it is condensed. The flow rate from the
well is 29.6 kg/sec. Determine the cooling water flow in kg/sec if water is available at 300 C and a
100C rise is allowed through the condenser. Answer

DIESEL POWER PLANT

397. Determine the indicated mean effective pressure of an engine in psi having a brake mean
effective pressure of 750 Kpa and 80% mechanical efficiency. Answer A
136 psi
137 psi
138 psi
140 psi

398. Determine the friction power of an engine if the frictional torque developed is 0.3 KN.m
running at 1200 rpm. Answer B
40.6 KW
37.7 KW
36.5 KW
50.3 KW

399. What is the power developed in the cylinder if indicated thermal efficiency is 44%, the
engine uses 0.05 kg/sec fuel with heating value of 44,000 KJ/kg? Answer C
1000 KW
775 KW
968 KW
588 KW

400. A 750 KW diesel electric plant has a brake thermal efficiency of 34%. If the heat
generated by fuel is 9,000,000 KJ/hr, what is the generator efficiency? Answer D
85.33%
65.88%
75.55%
88.23%

401. A 16-cylinder V-type diesel engine is directly coupled to a 5000 KW AC generator. If


generator efficiency is 90%, calculate the brake horsepower of the engine. Answer A
7447 Hp
6468 Hp
8542 Hp
7665 Hp

402. Determine the brake power of the engine having a brake thermal efficiency of 35% and
uses 250API fuel consumption of 40 kg/hr. Answer B or C
165.84 KW
173.52 KW
173.52 KW
160.67 KW

404. Determine the specific gravity of fuel oil having a heating value of 44,899.2 KJ/kg.
Answer D
0.90
0.80
0.877
0.893

405. A 500 KW diesel engine operates at 101.3 Kpa and 270C in Manila. If the engine will
operate in Baguio having 93 Kpa and 230C, what new brake power will developed if mechanical
efficiency is 85%? Answer C
600 KW
754 KW
459 KW
971 KW

406. What is the displacement volume of 300 mm x 4400 mm, 4 – stroke, 1200 rpm, 8
cylinder diesel engine? Answer B
0.243 m3/sec
2.262 m3/sec
5.75 kg/sec
1.25 m3/sec

407. What is the friction horsepower of a 300 KW diesel engine having a mechanical
efficiency of 86%? Answer D
86.5 Hp
87.5 Hp
90.5 Hp
65.5 Hp

408. Determine the output power of a diesel power plant if the engine and generator efficiency
is 83% and 95%, respectively. The engine uses 250API fuel and has a fuel consumption of 0.08
kg/sec.
Answer D
2795 KW
8642 KW
9753 KW
2815 KW

409. Determine the piston speed of a 250 mm x 300 mm diesel engine running at 1200 rpm.
Answer B
6 m/sec
12 m/sec
18 m/sec
5 m/sec

410. Determine the speed of a 6-pole generator of 60 hz. Answer C


600 rpm
1000 rpm
1200 rpm
3600 rpm
409. A 50 Bhp blast furnace engine uses fuel with 10 ft3/Bhp-hrs. The heating value of gas is
33,500 KJ/m3. Determine the brake thermal efficiency. Answer D
80%
50%
25.76%
28.31%

410. The heat generated by fuel is 2500 KW. If the jacket water loss is 30%. Determine the
mass of water circulated in the engine if the temperature rise is 80C. Answer B
20.5 kg/sec
22.4 kg/sec
58.5 kg/sec
12.3 kg/sec

411. An engine-generator rated 9000 KVA at 80% power factor, 3 phase, 4160 V has an
efficiency of 90%. If overall plant efficiency is 25%, what is the heat generated by the fuel.
Answer B
18,800 KW
28,800 KW
7500 KW
20,000 KW

412. A 4-strok, 8 cylinder diesel engine with bore and stroke of 9 in. x 12 in, respectively and
speed of 1000 rpm has a break mean effective pressure of 165 psi. Determine the engine brake
horsepower. Answer D
1753.34 Hp
1850.34 Hp
1950.34 Hp
1272.34 Hp

413. What torque is developed by the 300 KW engine running at 900 rpm? Answer C
2.65 KN.m
6.85 KN.m
3.18 KN.m
5.65 KN.m

414. What is the mechanical efficiency of a diesel engine if friction power is 30 KW and
brake power of 150 KW? Answer B
85.55%
83.33%
65.44%
75.88%

415. A 3 MW diesel engine consumes 240 liters of 250API fuel generates 900 KW-hr.
Determine the rate of fuel consumed by the engine. Answer A
0.2 kg/sec
0.4 kg/sec
0.6 kg/sec
0.8 kg/sec
412. The indicated thermal efficiency of a two stroke diesel engine is 50%. If friction power is
3% of heat generated, determine the brake thermal efficiency of the engine. Answer D
33%
34%
36%
37%

413. During the dynamometer test of an engine for 1 hr steady load, the engine consumes 40
kg fuel having 43,000 KJ/kg heating value. If the torque developed is 2.5 KN-m during the test at
600 rpm, what is the brake thermal efficiency of the engine? Answer B
31.22%
32.88%
55.77%
25.99%

414. A waste heat recovery boiler generates 250 kg/hr with hs – hf = 2300 KJ/kg. What is the
exhaust loss from the engine if the actual heat developed is 35%? Answer C
492.6 KW
365.7 KW
456.4 KW
845.4 KW

415. What maximum power that can be delivered by the 2000 KW engine at 3000 ft elevation
considering the pressure effect alone? Answer D
1600 KW
1700 KW
1900 KW
1800 KW

416. A 1000 KW diesel engine operates at an altitude of 1500 m elevation. Considering the
pressure effect alone, find the power developed by the engine at higher elevation? Answer D
753 KW
823 KW
345 KW
983 KW

417. The piston speed of an engine running at 1200 rpm is 12 m/s. Find the stroke of the
engine in inches. Answer C
15.74 mm
16 in
300 mm
15.75 in

418. The density of air entering the engine is 1.176 kg/m3 has a volume flow rate of 0.375
m3/sec. If the air fuel ratio is 21, find the mass flow rate of fuel. Answer B
0.10 kg/sec
0.02 kg/sec
0.30 kg/sec
0.40 kg/sec
414. A diesel engine develops a torque of 5 KN-m at 1800 rpm. If the brake thermal efficiency
is 31%, find the heat generated by the fuel. Answer B
3050.25 KW
3040.25 KW
3000.25 KW
5000 KW

415. In a diesel engine the fuel is injected at 6.5% of the stroke, the clearance volume is 10% of the
stroke. Find the cut-off ratio. Answer B
1.80
1.65
1.70
1.40

416. A 373 KW (500 Hp) internal combustion engine has a break mean effective pressure of
551.5 Kpa at full load. What is the friction power if mechanical efficiency is 85%? Answer A
88.23 hp
97.33 hp
56.34 hp
76.23 hp

417. A 4-stroke 394 mm bore and 534 mm stroke single acting diesel engine with four
cylinders is guaranteed to deliver 350 Bhp at 300 rpm. The engine consumed 66.8 kg/hr of fuel
with a heating value of 44,251 KJ/kg. Calculate indicated mean effective pressure in Kpa if
mechanical efficiency is 89%. Answer A
450.8 kpa
234.5 kpa
345.6 kpa
643.2 kpa

418. A 305 mm x 457 mm four stroke single acting diesel engine is rated at 150 KW at 260
rpm. Fuel consumption at rated load is 0.26 kg/KW-hr with a heating value of 43,912 KJ.kg.
Calculate brake thermal efficiency. Answer A
31.53%
27.45%
45.34%
54.23%

419. A four-stroke, 8 cylinder Diesel engine with bore and stroke of 9 inches and 12 inches,
respectively and speed of 950 rpm has a brake effective pressure of 164 psi. The specific fuel
consumption is 0.39 lb/bhp-hr and fuel heating value is 18,500 Btu/lb. Determine thermal
efficiency. Answer A
35.27%
45.23%
38.23%
54.23%

420. A dynamometer test was done for one hour at steady load on a 6 cylinder diesel engine. It
was found to use 42 kg of fuel having Qh = 42,000 J/g. Cylinder is 22.8 cm x 27.2 cm, 4-cycle
type.
Speed, 550 rpm and dynamometer torque at 27000 kg-cm. Determine the brake thermal
efficiency. Answer A
31.13%
43.22%
35.34%
45.32%

419. A certain diesel engine with the following classification, 8 cylinder, 400 mm x 600 mm,
four stroke cycle has a fuel consumption of 0.6 lbs/hp-hr based on 19,100 Btu/lb. Engine speed is
280 rpm with an indicated mean effective pressure 130 psi. If the jacket water carries away an
estimated 25% of the heat supplied, find its capacity (gpm) required if the allowable rise is 400F.
Answer C
234
873
242
292

420. When the pressure is 101.325 Kpa and temperature is 270C, a diesel engine has the full-
throttle characteristics listed:
Brake power = 275 KW
Brake specific fuel consumption = 0.25 kg/KW-
hr Air fuel ratio = 22
Mechanical efficiency = 88%
If the engine is operated at 84.5 Kpa and temperature of 15.50C, find the brake specific fuel
consumption kg/kwh.
0.294
0.862
0.423
1.08

421. Find the power which a 2.5 MW natural gas engine can develop at an altitude of 1981.2
meters taking into consideration the pressure and temperature change. Answer: B
2.34 MW
1.912 MW
2.56 MW
1.67 MW

422. A 119 KW, six cylinder, 589 mm x 711 mm, 225 rp, four stroke diesel engine has a fuel
consumption of 0.23 kg/KWh based on 44,099 KJ/kg fuel (heating value). A waste heat recovery
boiler recovers 35% of the exhaust loss. Jacket water loss are estimated at 30%. Assume losses
due to friction, etc. at 8%. Calculate the quantity of 136 Kpa steam that can be produced in kg/hr,
if jacket water at 70oC is used for boiler feed. Answer: A
439.4 kg/hr
623.4 kg/hr
543.32 kg/hr
984.45 jg/hr

423. A 2000 KW diesel engine unit uses 1bbl oil per 525 KWH produced. Oil is 25 oAPI.
Efficiency of generator 93%, mechanical efficiency of engine 80%. What is the thermal of engine
on indicated power(%)?Answer:C
31.69
421. 29.47
422. 39.60
423. 35.60

40. A waste heat recovery boiler produces 4.8 Mpa (dry saturated) steam from 104 °C feedwater.
The boiler receives energy from 5 kg/sec of 954°C dry air. After passing through a waste
heat boiler, the temperature if the air has been reduce to 343°C. How much steam in kg
produced per second? Note: Ar 4.80 Mpa dry saturated, h = 2796.

1.3
0.92
2.1
3.4

423. A diesel electric plant supplies energy for Meralco. During a 24 hour period, the plant
consumed 200 gallons of fuel at 28°C and produced 3930 KW-hr. Industrial fuel used is 28°API
and was purchased at P5.50 per liter at 15.6°C. What is the cost of fuel be to produce one KW-hr?
Answer: A
P1.05
P1.10
P1.069
P.00

429. A single-acting, four cylinder, 4 stroke cycle diesel engine with a bore x stroke of 21.59 x
27.94 cm, operating at 275 rpm, consumes 8.189 kg/hr of fuel whose heating value is 43,961.4
KJ/kg. The indicated mean effective pressure is 475.7 kpa. The load on the brake arm, which is
93.98 cm is 113.4 kg. What is the brake mean effective pressure in Kpa? Answer: A
415.20
124.17
319.95
645.33

430. A supercharged six-cylinder four stroke cycle diesel engine of 10.48 cm bore and 12.7
cm stroke has a compression ratio of 15. When it is tested on a dynamometer with a 53.34 cm
arm at 2500 rpm, the scale reads 81.65 kg 2.86 kg of fuel of 45,822.20 KJ/kg heating value are
burned during a 6 min test, and air metered to the cylinder at the rate of 0.182 kg/sec. Find the
brake thermal efficiency.Answer: 0.327
0.327
0.67
0.307
0.357
433. In a double acting, 2 stroke compression ignition engine, 8-cylinder, the diameter of the
cylinder is 700 mm, stroke is 1350 mm and the piston rod diameter is 250 mm. When running at
108 rpm, the indicated mean effective pressure above and below the piston are 5.80 abr and 4.90
bar respectively. Calculate the brake power of the engine with a mechanical efficiency of 80% in
kilowatts.Answer: B
6050
6030
6010
6070

434. In a test laboratory, it was found out that of the 80 Bhp developed by an engine on the
test, 45 Ho are absorbed by the cooling water that is pumped through the water jacket and the
radiator. The water enters the top of the radiator at 200°F. At that temperature, enthalpy of the
water is 168.7 Btu/lb. Water leaves the bottom of the radiator at 190°F and with an enthalpy of
158.03 Btu/lb. What is the water flow rate fir a steady-state oeration? Answer: C
25 gpm
20 gpm
23 gpm
24 gpm

435. A sic cylinder, four stroke diesel engine with 76 mm bore x 89 mm stroke was run in the
laboratory at 2000 rpm, when it was found that the engine torque was out. The engine consumed
12.2 kg of fuel per hour with heating value of 54,120 KJ/kg and 252.2 kg of air at 15.6°C per
hour. Determine the indicated power.Answer: B
32.1 kw
38.4 kw
23.3 kw
48.3 kw

436. A waste heat recovery boiler receives energy from 10 kg/s of 950°C hot gases from a
diesel engine. The exit temperature of hot gases (Cp = 1.0) has been reduced tp 250°C. Steam is
produced at 5 Mpa (dry saturated) from 95°C feedwater. How much steam in kg is produced per
hour? At 5 Mpa, hg = 2794.3 KJ/kgAnswer: A
10,515
10,155
11,055
11,515

437. A six cylinder, four stroke cycle diesel engine has an engine thermal efficiency of 34%.
The engine was tested on a dynamometer with a 23 in arm, running at 1800 rpm, the scale reads
210 lbs. During the 15 min test, the engine uses fuel with 28°API and air metered to the cylinder
at the rate of 0.201 kg/s. Find the fuel consumption during the 15 min test.Answer: C
6.33 liters
5.97 liters
6.83 liters
8.97 liters
435. A3 MW diesel electric power plant uses 3700 gallons in a 24 hour period. Oil is 25°API.
What is the thermal efficiency if the engine based on indicated power if the generator is 90%
efficiency and 95% mechanical efficiency is assumed?Answer: B
55%
60%
65%
70%

436. A single acting, 8 cylinder, 4 stroke cycle diesel engine with a bore to stroke of 15.24 cm
x 22.86 cm, operates at 1200 rpm. The load on the brake arm which is 101.60cm is 120 kg. What
is the brake mean effective pressure in Kpa?Answer: A
450.5 kpa
455.5 kpa
445.5 kpa
495.5 kpa

437. A six cylinder four stroke diesel with 76 mm bore and 89 mm stroke was run in the
laboratory at 2000 rpm. The engine consumed 137.4 kg of air at 15.5°C per hour. Determine the
volumetric efficiency of air only.Answer: D
69.84%
88.32%
92.54%
77.23%

GAS TURBINE POWER PLANT

439. An air-standard Braytin cycle has a pressure ratio of 12. Find the thermal efficiency of
the cycleAnswer: B
34.23%
50.83%
56.32%
65.23%

440. An air standard Braytin cycle has an air leaving the high-temperature heat exvhanger at
850°C and leaving the turbine at 310°C. What is the thermal efficiency?Answer: C
42.21%
23.34%
48.08%
56.34%

441. An air standard Brayton cycle has a pressure ratio of 8. The air properties at the start of
compression are 100 kpa and 25°C. The maximum allowable temperature is 1100°C. Determine
the net work.Answer: A
373.24 KJ/kg
283.45 KJ/kg
321.34 KJ/kg
398.23 KJ/kg

442. The air standard Brayton cycle has a net power output of 100 kw. The working substance
is air, entering the compressor at 30°C, leaving the high-temperature heat exchanger at 750°C and
leaving the turbine 300°C. Determine the mass flow rate of air.Answer: A
1698 kg/hr
1756 kg/hr
1543 kg/hr
2344 kg/hr

447. The compressor for an actual gas turbine requires 300 KJ/kg of work to quadruple the
inlet pressure. The inlet air temperature is 100°C. Determine the compressor air exit
temperature.Answer: D
234°K
542°K
653°K
673°K

448. The compressor for an actual gas turbine requires 300 KJ/kg of work to quadruple the
inlet pressure. The inlet air temperature is 100°C. Determine the compressor efficiency.Answer:
C
34.56%
53.23%
60.42%
76.34%

449. An ideal gas turbine operates with a pressure ratio of 8:1 and temperature limits of 20°C
and 1000°C. The energy input I the high temperature heat exchanger is 200 Kw. Determine the
air flow rate.Answer: B
560 kr/hr
970 kg/hr
873 kg/hr
453 kg/hr

450. In an air standard Brayton cycle the compressor inlet conditions are 100 kpa and 280°K.
The turbine inlet conditions are 1000 Kpa and 1167°K. The turbine produces 11,190Kw.
Determine the air flow rate.Answer: A
20 kg/s
25 kg/s
30 kg/s
35 kg/s

451. An air standard Brayton cycle has air enters the compressor at 27°C and 100 kpa. The
pressure ratio is 10 and the maximum allowable temperature in the cycle is 1350°K. Determine
the cycle efficiency per kilogram of air.Answer: A
48.20 %
51.34 %
45.23 %
65.23 %
448. An air standard Brayton cycle has air enters the compressor at 27°C and 100 Kpa. The
pressure ratio is 10 and maximum allowable temperature is 1350°K. The compressor discharges
and the turbine inlet. Determine thermal efficiency of the cycle per kilogram of air.Answer: C
34.23%
43.23%
30.50%
27.23%

449. A gas turbine working on an air standard Brayton cycle has air enter into the compressor
at atmospheric condition and 22°C. The pressure ratio is 9 and the maximum temperature in the
cycle is 1077°C. Compute for the cycle efficiency per kg of air in percent.Answer:D
44.85%
43.92%
41.65%
46.67%

450. In a gas turbine unit, air enters the combustion chamber at 550 kpa, 277°C and 43 m/s.
The products of combustion leave the combustor at 511 kpa, 1004°C and 140 m/s. Liquid
fuelenters with a heating value of 43,000 KJ.kg. For fuel-air ratio of 0.0229, what is the
combustir efficiency of the unit in percent?Answer: A
79.8%
76.5%
85.2%
69.8%

451. Air compressor of a gas turbine at 100 kpa and 300°K with a volume flow rate of 5
m³/sec. The compressor pressure is 10 and its isentropic efficiency is 85%. At the inlet to the
turbine, the pressure is 950 kpa and the temperature is 1400°K. The turbine has an isentropic
efficiency of 88% and the exit pressure is 100 Kpa. On the basis of air standard analysis, what is
the thermal efficiency of the cycle in percent?Answer: C
42.06
60.20
31.89
25.15

HYDRO-ELECTRIC PLANT

448. A hydro-electric power plant consumes 52,650 KW-hrs per annum. Expected flow is
1665 m³/min and overall efficiency is 65%. What is the net head?Answer: D
30 m
31 m
32 m
34 m

449. Ina a hydro-electric power the tail water level fixes at 480 m. The net head is 27 m and
head loss is 4% of the gross head. What is the head water elevation?Answer: B
500.34 m
508.12 m
456.34 m
567.34 m
453. The available flow of water is 25 m³/ sec at 30 m elevation. If a hydro-electric plant is to
be installed with turbine efficiency of 85% and generator efficiency of 90%, what maximum
power that the plant could generate?Answer: C
4658.5 KW
3478.5 KW
5628.5 KW
4756.5 KW

454. For a proposed hydro-electric plant, the tail water and head elevation is 160 m and 195
m, respectively. If available flow is 10 m³/sec and head loss of 5% of water available head. What
is the water power?Answer: A
3261.8 KW
4254.6 KW
5874.5 KW
2456.5 KW

455. The flow of a river is 20 m³/sec and produces a total brake of 6,000 KW. If it is proposed
to install two turbines each has 85% efficiency, what is the available head?Answer: A
35 m
37 m
39 m
36 m

456. Two turbines generates a total power of 5000 KW. If one unit is thrice the capacity of the
other, find the capacity of smaller unit.Answer: A
1250 KW
3450 KW
2456 KW
5763 KW

457. In a hydro-electric plant the brake power is 1800 KW running at 450 rpm and net head of
30 m. Determine the specific speed of the turbine.Answer: D
60.29 rpm
65.29 rpm
75.29 rpm
71.29 rpm

458. The specific speed of turbine is 75 rpm and running at 450 rpm. If the head is 20 m and
generator efficiency is the maximum power delivered by the generator.Answer: C
450.5 KW
345.5 KW
650.5 KW
780.5 KW

459. For a generator running at 300 rpm and 60Hz, find the number of generator
poles.Answer: A
24 poles
8 poles
18 poles
20 poles

460. The penstock if hydro-electric plant is 0.5 x 0.5 m with velocity of 5.5 m/sec has a head
of 20 m. What is the output of the turbine if the turbine efficiency is 87%?Answer: D
845.32 KW
789.34 KW
654.56 KW
234.56 KW

461. A 4m x 0.5m channel has water velocity of 5 m/sec. If the head is 100 m, what is the
annual energy produced if overall efficiency is 70%?Answer: C
40,456,000 KW-hrs
34,456,345 KW-hrs
60,154,920 KW-hrs
54,867,234 KW-hrs

462. In Francis turbine, the pressure gage leadig to the turbine casing reads 380 Kpa and
center of spiral casing is 3 m above the tailrace. If velocity of water entering the turbine is 8
m/sec, what is the net head of the turbine?Answer: D
30 m
35 m
40 m
45 m

463. A turbine has a mechanical efficiency of 95%, volumetric efficiency if 97% and total
efficiency of 80%, If effective head is 40 m, find the total head.Answer: D
48.72 m
40.72 m
46.72 m
34.72 m

464. A Pelton type turbine has 30 m head friction loss of 4.5 m. The coefficienct of friction
head loss(from Moorse) is 0.00093 and penstock length of 80 m. What is the penstock diameter?
Answer: D
1234 mm
3476 mm
677 mm
1686 mm

465. From a height of 65 m water flows at the rate of 0.85 m³/s and is driving a turbine
connected to 160rpm generator. If frictional torque is 540 N-m, calculate the turbine brake
power.Answer: A
533 KW
677 KW
455 KW
488 KW

466. What is the turbine output and delivering 1 m³/sec in a hydro-electric power plant if
turbine efficiency is 88%%?Answer: A
863.28 KW
734.28 KW
167.34 KW
492.34 KW

464. In an 8,000 KW hydro-electric plant the over-all efficiency is 88% and the actual power
receive by the customer is 110,000 KW-hrs for that day. What is the secondary power could this
plant deliver during the entire day?Answer: A
58,960 KW-hrs
60,960 KW-hrs
65,960 KW-hrs
70,960 KW-hrs

465. At a proposed hydroelectric plant site, the average elevation of headwater is 600 m, the
tailwater elevation is 480 m. The average annual water flow is determined to be equal to the
volume flowing through a rectangular channel 4 m wide and 0.5 m deep and average velocity of
5.5 m/sec. Assuming that the plant site can develop if the hydraulic turbine that will be used has
an efficiency of 80% and generator efficiency of 92%. Consider a headwork loss of 4% of the
available head.Answer: A
76,854,851
65,234,556
75,234,455
82,456,677

466. In Maria Christina Hydroelectric Project, the available head is 140 m; the water flow is
one cubic meter per second; efficiency of the turbine is 95%; efficiency of the generator is 95%,
three phase, 60cycle, voltage delivered is 4160 V. Determine yearly income of the plant if the
cost of the generated electric energy is P0.60 per Kwh.Answer: C
P5,234,567
P7,385,678
P6,514,778
P9,354,677

467. A Pelton type was installed 30 m below the head gate of the penstock. The head loss due
to friction is 15% of the given elevation. The length of penstock is 80 m and coefficient of
friction is 0.00093. Determine the power output in KW.Answer: D
12,273
13,234
12,345
12,493
472. From a height if 65 m, water flows at the rate of 0.85 m³/sec and is driving a water
turbine connected to an electric generator revolving at 160 rpm. Calculate the power developed
by the turbine in KW if the total resisting torque due to friction is 540 N-m and the velocity of the
water leaving the turbine blades is 4.75 m/sec.Answer: B
753
523
635
833

473. A proposed hydro-electric power plant has the following data:

6
10
8
12

477. A Francis turbine is installed with a vertical draft tube. The top of the draft is 1 m below
the centerline of the spiral casing while the tail race water level is 3 m from the top of the draft
tube. There is no velocity of whirl at the top or bottom of the draft tube and leakage are
negligible. The elevation of water from the reservoir to the centerline of the turbine casing is 50
m, and water velocity at the inlet is 5 m/sec. Discharge is 2.5 m³/sec, hydraulic efficiency is 86%.
Overall efficiency if 83%. Determine the reading of a pressure gage(in psi) if one is installed at
the penstock just before the water inlet to the turbine.Answer: A
69.35
74.33
56.34
92.45

478. A Francis turbine is installed with a vertical draft tube. The top of the draft is 1 m below
the centerline of the spiral casing while the tail race water level is 3 m from the top of the draft
tube. There is no velocity of whirl at the top or bottom of the draft tube and leakage are
negligible. The
elevation of water from the reservoir to the centerline of the turbine casing is 50 m, and water
velocity at the inlet is 5 m/sec. Discharge is 2.5 m³/sec, hydraulic efficiency is 86%. Overall
efficiency if 83%. Determine the mechanical efficiency of the turbine. Answer: C
76.23%
83.45%
95.51%
64.34%

482. A remote community in Mountain Province plans to put up a small hydro-electric plant
to service six closely located barangays estimated to consume 52,650,000 KW-hrs per annum.
Expected flow if water is 1665 m³/min. The most favorable location for the plant fixes the tail
water level at 480 m. The manufacturer of turbine generator set have indicated the following
performance data: turbine efficiency 87%; generator efficiency is 92%; loss in head work not
exceed 3.8% of
available head. In order to pinpoint the most suitable area for the dam, determine the head water
elevation.Answer: D
234 m
842 m
345 m
509 m

489. A hydro-electric plant has a 20 MW generator with an efficiency of 96%. The generator
is directly coupled to a vertical Francis type hydraulic turbine having an efficiency of 80%. The
total gross head on the turbine is 150 m while the loss of head due to friction is 4% of the gross
head. The runaway speed is not to exceed 750 rpm. Determine the flow of water through the
turbine in cfs.Answer: A
651
534
763
827

490. A hydro-electric plant has a 20 MW generator with an efficiency if 96%. The generator is
directly coupled to a vertical Francis type hydraulic turbine having an efficiency of 80%. The total
gross head on the turbine is 150 m while loss head due to friction is 4% of the gross head. The
runaways speed is not to exceed 750 rpm. Determine the rated speed of the turbine.Answer: C
300
500
400
600

491. Water flows steadily with a velocity of 3.05 m/s in a horizontal pipe having a diameter of
15.24 cm. At one section of the pipe, the temperature and pressure of the water are 21°C and
689.3 Kpa, respectively. At a distance of 304.8 m downstream. The pressure is 516.9 kpa. What
is the friction factor?Answer: C
0.134
0.0050
0.0189
0.641
492. A hydroelectric generating unit station is supplied from a reservoir of capacity 6,000,000
m³ at a head of 170 m. Assume hydraulic efficiency of 80% and electrical efficiency 90%. The
fall in the reservoir level after a load is 15 MW has been supplied for 3 hours, if the area of the
reservoir is 2.5 sq. Km is closet to:Answer: A
5.39 cm
4.32 cm
5.98 cm
4.83 cm

493. A pelton wheel is to be designed to rin at 300 rpm under an effectice head of 150 m. The
ratio of the nozzle diameter of the pitch circle is 1/12. Assuming efficiency of 84%, what is the
size of the wheel in meters. Assume a speed ratio of 0.45.Answer: C
1.05
2.00
1.55
2.86
495. A Francis turbine is installed with a vertical draft tube. The total head to the center of the
spiral casing at the inlet is 38 m and velocity of water at the inlet is 5 m/sec. The discharge is 2.1
m³/sec. The hydraulic efficiency is 0.87 and overall efficiency is 0.84. The top of draft tube is
1m(water) below the centerline of the spiral casing while the tailrace (water) level is 3m from the
top of the draft tube. Neglected velocities of whirl and leakage losses. What is the total head on
the turbine in meters?Answer: B
34.72
43.27
55.20
48.12

496. A hydro-electric plant discharges water at the rate of 0.75 m³/sec and enters the turbine at
0.35 mps with a pressure of 275 Kpa. Runner inside diameter is 550 mm, speed is 520 rpm and
the turbine efficiency is 88%. Find the turbine speed factor.Answer: A
0.638
0.386
0.368
0.836

497. A hydraulic turbine receives water from a reservoir at an elevation of 100 meters above
it. What is the minimum water kg/s to produce a steady turbine output of 50 Mw?Answer: B
50,247
50,968
50,672
50,465

498. A hydro-electric plant having 50 sq. km reservoir area and 100 m head is used to
generate power. The energy utilized by the consumers whose load is connected to the power plant
during a five-hour period is 13.5 x 106kwh. The overall generation efficiency is 75%. Find the fall
in the height of water in the reservoir after the 5-hour period.Answer: B
2.13 m
1.32 m
3.21 m
0.53 m
499. A reaction turbine develops 500 bhp. Flow through the turbine is 50cfs. Water enters at
20fps with a 100ft pressure head. The elevation of the turbine above the tail water level is 10 ft.
Find the effective head.Answer: D
130 ft
120 ft
110 ft
116.2 ft

CHIMNEY

497. A boiler uses 2500 kg of coal per hour and air required for the combustion in 16 kg per
kg coal. If ash loss is 10%, determine the mass of the gas entering the chimney. Answer: A
42,250 kg/hr
78,300 kg/hr
85,452kg/hr
33,800 kg/hr
499. The gas density of chimney is 0.75 kg/m3 and air density of 1.15 kg/m3. If the driving
pressure is 0.25 Kpa, determine the height of chimney. Answer: B
54.6 m
63.7 m
74.6 m
68.5 m

500. The actual velocity of gas entering in a chimney is 8m/sec. The gas temperature is 25°C
and pressure of 98 Kpa with a gas constant of 0.287 KJ/kg-°K. Determine the chimney diameter
if mass of gas is 50,000 kg/hr. Answer: D
1.57 m
1.81 m
3.56 m
1.39 m

501. A coal fired steam boiler uses 3000 kg of coal per hour. Air required for combustion is
15.5 kg per kg of coal at barometric pressure of 98.2 Kpa. The flue has temperature of 285°C and
an average molecular weight of 30. Assuming an ash loss of 11% and allowable gas velocity of
7.5 m/sec, find the diameter of chimney. Answer: C
2.0 m
1.53 m
1.91 m
2.5 m

502. A power plant situated at an altitude having an ambient air of 96.53 Kpa and 23.88°C.
Flue gases at a rate of 5.0 kg/sec enter the stack at 200°C and leaves at 160°C. The flue gases
gravimetric analyses are 18% Co2, 7% O2 and 75% N2. Calculate the height of stack necessary
for a driving pressure of 0.20 Kpa. Answer: D
59.56 m
60. 15 m
55. 83 m
56.95 m

503. A power plant situated at an altitude having air of 96.53 Kpa and 23.88°C. Flue gases at a
rate of 5.0 kg/sec enter the stack at 200°C and leaves at 160°C. The flue gases gravimetric
analysis 18% Co2, 7% O2 and 75% N2. Calculate the diameter of stack in meters for a driving
pressure of 0.20 Kpa.Answer: B
0.50 m
0.95 m
O.60 m
1.15 m

504. A steam generator with economizer and air heater has an overall draft loss of 21.78 cm of
water. If the stack gases are at 177°C and if the atmosphere is at 101.3 Kpa and 26°C, what
theoretical height of stack in meters is needed when no draft fan are used? Assume that the gas
constant for the flue gases is the same as that for air. Answer: D
565
535
545
550
503. If the actual draft required for a furnace is 6.239 cm of water and the frictional losses in
the stack are 15% of the theoretical draft, calculate the required stack height in meters. Assume
that the flue gas have an average temperature of 149°C and molecular weight of 30. Assume air
temperature of 21°C.Answer: C
215
230
220
210

504. A steam boiler plant consumes 9,000 kg of coal per hour and produces 20 kg of dry flue
gases per kg of coal fired. Outside air temperature is 32°C, average temperature of the flue gas
entering the chimney is 343°C and average temperature of dry flue gas in the chimney is 260°C.
The gage fluid density of 994.78 per m3 and the theoretical draft of 2.286 cm of H2O at the
chimney base is needed when the barometric pressure is 760 mm Hg. Determine the height of the
chimney.Answer: A
46
50
40
56

MACHINE FOUNDATION

505. All heavy machinery should be supported on ________ of sufficient mass and base area
to prevent or minimize the transmission of vibrations.Answer: C
Steel foundation
Wooden foundations
Solid foundations
Soil foundations

506. Foundations mass should be from _______ times the weight of the machinery it is
supposed to support.Answer: A
3 to 5
5 to 7
4 to 6
1 to 2

507. If the unbalanced inertia forces produced by the machine can be calculated, a mass
weight equal to ________ times the forces should be used to dampen vibration.Answer: C
2 to 4
5 to 6
10 to 20
10 to 15

508. For stability, the total combined engine, driven equipment, and foundation center of
gravity must be kept ________ the foundation’s top.Answer: B
Above
Below
1m
None of these

511. The machine foundation must have a factor safety of________Answer: C


3
4
5
6

512. Foundations should be isolated from floor slabs or building footings at least ________
mm around its perimeter to eliminate transmission vibration.Answer: B
20
23
30
35

513. Foundations are preferably built of concrete in the proportion of ________Answer: D


1:2:3
1:3:3
1:2:5
1:2:4

514. The machine should not be placed on the foundation until 7 days have elapsed or
operated until another ________ days have passed.Answer: B
6
7
8
10

515. Concrete foundations should have steel bar reinforcements placed both vertically and
horizontally, to avoid ________Answer: B
Damaging
Thermal cracking
Superheating
Superposing

516. The weight of reinforced steel in a foundation should be from ________ of the weight of
the foundation.Answer: A
½% to 1%
1% to 2%
2% to 3%
3% to 4%

517. Foundation bolts of specified size should be used and surrounded by a pipe sleeve with
an inside diameter of at least ________ times the diameter of the anchor bolt.Answer: C
1
2
3
4
514. Foundation bolts length should be at least ________ times the diameter of the anchor
bolt.Answer: D
12
14
16
18

515. No foundation bolts shall be less than ________ mm diameterAnswer: B


10
12
14
16

516. Machine should be leveled by driving wedges between the machine’s base and concrete
foundation and with the aid of a ________.Answer: C
Hose and water
Try square
Spirit level
Level hose

517. Grouting all spaces under the machine bed with a thin mixture of one part cement and
________
part sand.Answer: A
1
2
3
4

518. For foundation of stacks, the maximum pressure on the soil is ________ to the pressure
due to weight and wind movement.Answer: C
More than
Less than
Equal
None of the above

519. Guyed stacks seldom exceed 1.83 m diameter and ________ m high.Answer: A
32.45
36.34
43.23
30.48

520. The angle between the stacks and the guy wire usually ________ degrees. Answer: B
45
60
75
90

521. The angle between wires in a set is ________ degrees for a set of three.Answer: D
90
100
110
120

520. The maximum unit pressure of turbine and generator on the reinforced concrete should
not exceed ________ kg/cm2.Answer: A
17.62
15.34
19.34
21.34

521. For steam turbine foundations the concrete mixture should beAnswer: B
1:2:3
1:2:4
1:3:5
1:2:2

522. The steam turbine foundations should be designed to support the machine load plus
________ for impact, condenser load, floor loads and dead loads.Answer: C
10%
15%
25%
50%

523. For diesel engines foundation the concrete mixture must be 1:2:4, and the broken stone or
gravel must have a size of ________ mm maximum.Answer: C
20
40
50
60

524. In pouring a concrete mixture for foundation of diesel engine, it should be poured
________
times.Answer: A
1
2
3
4

525. After pouring the foundation of diesel engine, the tops should be covered and wet down
twice daily until the forms are removed at the end of the ________ day.Answer: A
Third of fourth
Second or third
Fourth or fifth
None of these

526. The diesel engine should not be placed on the foundation until 10 days have elapsed, nor
operated until after another ________ days.Answer: D
7
8
9
10
The safe soul bearing pressure of diesel engine foundations isAnswer: B
2,600 kg/cm2
4,890 kg/cm2
5,633 kg/cm2
2000 kg/cm2

The foundation depth may be taken as good practical rule, to be ________ times the engine
stroke.Answer: B
2.3 to 4.5
3.2 to 4.2
3.2 to 5.2
5.2 to 6.2

Anchor bolts should be embedded in concrete of at least ________ times the bolt
diameter.Answer: C
10
20
30
40

What is the required base area of the foundation to support an engine with specified speed of
1200 rpm and weight of 9,000 kg? Assume bearing capacity of soil as 47.867 kpa. Use e =
0.11Answer: A
5.57 m2
8.87 m2
7.75 m2
10.5 m2

A foundation measures 3m x 4m x 5m. Find the yd3 of stone needed for a concrete mixture of
1:2:4.Answer: D
65.34
56.45
78.48
69

A foundation measures 10ft x 12 ft 15 ft. Find the number of sacks of cement needed for 1:2:4.
MixtureAnswer: B
300
400
350
450

A machine foundation has a trapezoidal cross-sectional with bases of 2m and 3m. The height is
1.5 m and foundation length of 4m. Find the required yd3 of sand to be used for 1:2:4
mixture.Answer: D
7.33
12.4
9.34
8.63
A foundation has a weight equal to 4 times the weight of the engine. The safe bearing pressure of
soil is 60 kpa and the foundation has a base dimension of 2m x 4m. Find the maximum weight of
the engine to be mounted.Answer: C
8,234 kg
6,455 kg
9,736 kg
12,344 kg

A rectangular foundation cross-section has a bed plate dimension of 4 ft x 8 ft. The uniform
clearance on each side is 1 ft. The height of foundation is 2.5 ft. If the weight of the steel bar
reinforcements needed is ½% of weight of foundation, find the weight of steel bars. Use concrete
density of 2400 kg/m3.Answer: A
51 kg
45 kg
65 kg
76 kg

HEAT TRANSFER

A 15 cm thick wall has a thermal conductivity of 5W /m-°K. If inside and outside surface
temperature of the wall are 200°C and 20°C, respectively. Determine the heat
transmitted.Answer: A
5.67 KW/m2
4.68 KW/m2
8.87 KW/m2
6.87 KW/m2

Two walls of cold storage plant are composed of an insulating material (l = 0.25 KJ/hr-m-°C),
100 mm thick at the outer layer and material (l = 3.5 KJ/hr-m-°C), 15 cm thick at inner layer. If
the surface temperature at cold side is 30°C and hot side is 250°C, find the heat transmitted per
square meter.Answer: A
0.138 KW/m2
0.450 KW/m2
0.025 KW/m2
0.065 KW/m2

Sea water for cooling enters a condenser at 27°C and leaves at 37°C. The condenser temperature
is 45°C, what is the log mean temperature difference?Answer: B
50.65°C
12.33°C
37.82°C
80.54°C

Determine the thermal conductivity of a material that uses a 2 m2 test panel, 25 mm thick with a
temperature difference of 10.8°F between the surfaces. During the 5 hours test period, the heat
transmitted is 200 KJ.Answer: C
0.045 W/m-°K
0.560 W/m-°K
0.023 W/m-°K
0.370 W/m-°K
A steam pipe having a surface temperature of 200 °C passes through a room where the
temperature is 27°C. The outside diameter of pipe is 80 mm and emissivity factor is 0.8 Calculate
the radiated heat loss for 3 m pipe length.Answer: A
1434.47 W
2746.35 W
2756.57 W
3546.45 W

A counter flow heat exchanger is designed to heat fuel oil from 30°C to 90°C while the heating
fluid enters at 140°C at 105°C. Determine the arithmetic mean temperature difference.Answer: B
72.5°C
62.5°C
45.5°C
67.5°C

A heat exchanger has an overall coefficient of heat transfer of 0.50 KW/m2-°C. Heat loss is 11
KW and the mean temperature difference is 15°C. What is the heat transfer are in ft2?Answer: D
51.80 ft2
37.30 ft2
56.80 ft2
15.80 ft2

Brine enters a circulating brine cooler at the rate of 60 m3/hr at -10°C and leaves at -16°C.
Specific heat of brine is 1.072 KJ/kg-°K and specific gravity of 1.1. Determine the tons of
refrigeration.Answer: C
53.5 TR
65.3TR
33.5 TR
44.5 TR

A heat exchanger has a hot gas temperature of 300°C and surface conductance on hot side is 200
W/m2-°K. If heat transmitted is 1000 W/m2, what is the surface temperature on the wall at hot
side?Answer: A
295°C
465°C
234°C
354°C

A heat exchanger is to be designed for the following specifications:


Hot gas temperature, 1145°C
Cold gas temperature is 45°C
Unit surface conductance on the hot side, 230 W/m2-°K
Unit surface conductance on the cold side, 290 W/m2-°K
Thermal conductivity of the metal wall, 115 W/m-°K
Find the maximum thickness of metal wall between the hot gas and cold gas, so that the
maximum temperature of the wall does not exceed 545°C.Answer: C
A. 10 mm
30mm
20mm
40mm

An uninsulated steam pipe passes through a room in which the air and walls are at 25°C. The
outside diameter of the pipe is 70 mm, and its surface temperature and emissivity are 200°C and
0.80, respectively. If the coefficient associated with free convection heat transfer from the surface
to the air is 15 W/m2-°K, what is the rate of heat loss from the surface per unit length of pipe?
Answer: A
998 w/m2
872 w/m2
762 w/m2
422 w/m2

A turbo-charged, 16 cylinder, Vee-type diesel engine has an air consumption of 3,000 kg/hr per
cylinder at rated load and speed. This air is drawn in through a filter by a centrifugal compressor
directly connected to the exhaust gas turbine. The temperature of the air from the compressor is
145°C and a counter flow air cooler reduces the air temperature to 45°C before it goes to the
engine suction header. Cooling water enters air cooler at 30°C and leaves at 38°C. Calculate the
log mean temperature difference.Answer: A
47°C
87°C
34°C
65°C

A turbo-charged, 16 cylinder, Vee type diesel engine has an air consumption of 3,000 kg/her per
cylinder at rated load and speed. This air is drawn in through a filter by a centrifugal compressor
directly connected to the exhaust gas turbine. The temperature of the air from the compressor is
145°C and a counter flow air cooler reduces the air temperature to 45°C before it goes to the
engine suction header. Cooling water enters air cooler at 30°C and leaves at 38°C. Calculate the
quantity of cooling water in m3/hr required to cool the total requirements of the engine at rated
load and speed.Answer: A
144
136
123
165

An oil heater heats 100 kg per minute of oil from 35°C to 100°C in a counter flow heat
exchanger. The average specific heat of the oil is 2.5KJ/kg-°C. Exhaust gases used for heating
enter the heater with an average specific heat of 1 KJ/kg-°C, a mass flow rate of 250 kg/min and
an initial temperature of 200°C. The overall heat transfer coefficient is 75 W/m2-°C. Determine
heating surface in square meters.Answer: D
30
63
40
36
A surface condenser serving a 50,000 KW steam turbo-generator unit receives exhaust steam at
the rate of 196,000 kg/hr. Vacuum in condenser is 702 mm Hg. Sea water for cooling enters at
29.5°C and leaves at 37.5°C. For steam turbine condensed as heat given up to cooling water.
Calculate logarithmic mean temperature difference in °F.Answer: C
10
14
12
16

A surface condenser serving a 50,000 KW steam turbo-generator unit receives exhaust steam at
the rate of 196,000 kg/hr. Vacuum in condenser is 702 mm Hg. Sea water for cooling enters at
29.5°C and leaves at 37.5°C. For steam turbine condenser design, manufacturers consider 950
Btu/lb of steam turbine condensed as heat given up to cooling water. Calculate the required
quantity of cooling water in cubic meters per hour.Answer: D
10,374
12,445
11,345
13,509

Calculate the energy transfer rate across 6” wall of firebrick with a temperature differences across
the wall of 50°C. The thermal conductivity of the firebrick is 0.95 Btu/hr-ft-°F at the temperature
interest.Answer: B
285 W/m2
369 W/m2
112 W/m2
429 W/m2

Water is flowing in a pipe with radius of 25.4 cm at a velocity of 5m/sec at the temperature in the
pipe. The density and viscosity of the water are as follows: density = 997.9 kg/sec, viscosity =
1.131 Pa-s. What is the Reynolds Number for this situation?Answer: A
2241
96.2
3100
1140

The hot combustion gases of a furnace are separated from the ambient air and its surrounding,
which are at 25°C, by a brick wall 0.15 m thick. The brick has a thermal conductivity of 1.2
W/m-°K and surface emissivity of 0.8. Under steady state conditions and outer surface
temperature of 100°C is measured. Free convection heat transfer to the air adjoining this surface
is characterized by a convection coefficient of 20W/m2-°K. What is the brick inner surface
temperature in °C?Answer: B
623.7
352.5
461.4
256.3

Steam initially saturated at 2.05 Mpa, passes through a 10.10 cm standard steel pipe for a total
distance of 152 m. The steam line is insulated with a 5.08 cm thickness of 85% magnesia. For an
ambient temperature of 22°C, what is the quality of the steam which arises at its destination if the
mass flow rate is 0.125 kg steam per second?
Properties of steam:
Pressure = 2.05 Temperature = 213.67
Enthalpy: hf = 914.52 hfg = 1885.5
hg = 28000.0
k for 85% magnesia = 0.069 W/m-°K
fo for still air = 9.36 W/m-°KAnswer: A
93%
98%
84%
76%

Compute the amount of condensate formed during 10 minutes warm-up of 150 meter pipe
conveys the saturated steam with enthalpy vaporization hfg = 1,947.8 LJ/kg.The minimum external
temperature of pipe is 2°C. The final temperature of pipe is 195°C. The specific heat of pipe
material is 0.6 KJ/kg-°C. The specific weight is 28 kg/m.Answer: C
249.69 kg
982.45 kg
294.54 kg
423.45 kg

What is the heat transfer in the glass surface area of 0.7 m2 having an inside temperature (room)
of 23°C and 13°C outside temperature (surrounding). The thickness of glass surface is 0.0007 m.
The thermal conductivity is 1.8 W/m-°K.Answer: D
5.8 kw
3.6 kw
6.2 kw
2.34 kw

AIR COMPRESSOR

An air compressor takes air at 100 Kpa and discharges to 600 Kpa. If the volume flow of
discharge is 1.2 m3/sec, determine the capacity of air compressor.Answer: A
4.32 m3/sec
3.33 m3/sec
6.85 m3/sec
7.42m3/sec

The discharge pressure of an air compressor is 5 times the suction pressure. If volume flow of
suction is 0.1 m3/sec, what is the compressor power assuming n = 1.35 and suction pressure is 98
Kpa?Answer: B
21.67 KW
19.57 KW
25.87 KW
10.45 KW

A 10 Hp motor is used to drive an air compressor. The compressor efficiency is 75%. Determine
the compressor work.Answer: A
6.0 KW
6.5 KW
7.6 KW
5.6 KW
The initial condition of air in an air compressor is 98 Kpa and 27°C and discharges air at 400
Kpa. The bore and stroke are 355 mm and 381 mm, respectively with percent clearance of 5%
running at 300 rpm. Find the volume of air at suction.Answer: C
600 m3/hr
610 m3/hr
620 m3/hr
630 m3/hr

An air compressor has a suction volume of 0.25 m3/sec at 97 Kpa and discharges to 650 Kpa.
How much power saved by the compressor if there are two stages?Answer: A
8.27 KW
6.54 KW
3.86 KW
10.0 KW

The suction condition of an air is 98 Kpa, 27°C and 0.2 m3/sec. If the surrounding air is 100 Kpa
and 20°C, determine the free air capacity in m3/sec.Answer: B
0.15
0.19
0.25
0.23

A 355 mm x 381 mm air compressor has a piston displacement of 0.1885 m3/sec. Determine the
operating speed of the compressor.Answer: B
250 rpm
300 rpm
350 rpm
600 rpm

Determine the percent clearance of an air compressor having 87% volumetric efficiency and
compressor air pressure to be thrice the suction pressure.Answer: D
5%
7%
15%
11%
The compressor work of an air compressor is 100 KW. If the piston speed is 15 m3/min,
determine the mean effective pressure.Answer: C
200 Kpa
300 Kpa
400 Kpa
600 Kpa

A double acting air compressor has 16 in x 7 in, 600 rpm has what volume displacement?
Answer: D
688 ft3/min
755 ft3/min
488 ft3/min
977 ft3/min
A two-stage air compressor has a suction pressure of 14 psi and discharge pressure of 130 psig.
What is the intercooler pressure in Kpag.Answer: A
209 Kpag
600 Kpag
477 Kpag
300 Kpag

A two stage air compressor has an intercooler pressure of 3 kg/cm2. What is the discharge
pressure if suction pressure is 1kg/cm2?Answer: B
3 kg/cm2
9kg/cm2
23 kg/cm2
15 kg/cm2

The piston speed of an air compressor is 140 m/min and has a volume displacement of 0.2m3/sec.
Determine the diameter of compressor cylinder.Answer: C
500 mm
358 mm
467 mm
246 mm

An air compressor piston displacement is 5000 cm3 when operates at 900 rpm and volumetric
efficiency of 75%/ Determine the mass flow of air standard density.Answer: B
365.3 kg/hr
243.5 kg/hr
386 kg/hr
465.2 kg/hr

A two-stage compressor air at 100 Kpa and 22°C discharges to 690 Kpa. If intercooler intake is
105°C, determine the value of n.Answer: C
1.400
1.358
1.345
1.288

The piston displacement of a double acting compressor running at 300 rpm is 0.4 m3/sec. If bore
and stroke are unity, determine the length of stroke.Answer: C
350 mm
380 mm
371 mm
400 mm

An air compressor takes air at 97 Kpa at the rate of 0.5 m3/sec and discharge 500 Kpa. If power
input to the compressor is 120 KW, determine the heat loss in the compressor.Answer: B
26.5 KW
18.55 KW
30.45 KW
22.36 KW

A single acting air compressor has a volumetric efficiency of 87%, operates at 500 rpm. It takes
in air at 100 Kpa and 30°C and discharges it at 600 Kpa. The air handled is 6m3/min measured at
discharge condition. If compression is isentropic, find mean effective pressure in Kpa.Answer: D
182
983
198
204

A single acting reciprocating air compressor has a clearance volume of 10%. Air is received at 90
Kpa and 90.3°C and is discharged at 600 Kpa. The compression and expansion are polytropic
with n – 1.28. The pressure drop is 5 Kpa at suction port and 10 Kpa at discharge port. The
compressor piston displacement is 500 cm3 when operating 900 rpm. Determine the mass of
compressed air in kg/hr.Answer: A
16.76
20.45
98.33
28.23

A single acting air compressor operates at 150 rpm with an initial condition of air at 97.9 Kpa
and 27°C and discharges the air at 379 Kpa to a cylindrical tank. The bore and stroke are 355 mm
and 381 mm respectively, with 5% clearance. If the surrounding air is at 100 Kpa and 20°C while
the compression and expansion process are PV1.3 = C, determine air capacity, m3/secAnswer: A
0.0919
2.13
1.23
4.23

The piston displacement of a double acting compressor is 0.358 m3/sec, delivers gas from 101.
325 Kpa and 300 °K to 675 Kpa at the rate of 0.166 m3/sec at 150 rpm. Value of n for
compression and expansion is 1.33. Find the compressor percent clearance.Answer: A
16.96
14.23
12.34
18.44

The piston displacement of a double acting compressor is 0.358 m3/sec, delivers gas from 101.
325 Kpa and 300°K to 675 Kpa at the rate of 0.166 m3/sec at 150 rpm. Value of n for
compression and expansion is 1.33. Find the bore and stroke assuming bore = stroke.Answer: D
300 mm
400mm
350 mm
450 mm

A 2-stage, double acting, L-type air compressor 16” x 10” x 7”, 600 rpm, has a free air unloader
at each end for capacity control. It is driven through V-belts by a 150 Hp electric motor, 460 V, 3
phase, 60 Hz, 1200 rpm. Barometric pressure is 125 psi gage. Calculate piston displacement in
m3/hrAnswer: C
562 m3/hr
762 m3/hr
649 m3/hr, 1661 m3/hr
833 m3/hr, 724 m3/hr

A two-cylinder single-acting air compressor is directly coupled to an electric motor running at


1000 rpm. Other data are as follows:
Size of each cylinder = 150 mm x 200 mm
Clearance volume = 10% of displacement
Exponent (n) for both compression and re-expansion process =
1.6 Air constant = 1.4
Air molecular mass M = 29
Calculate the volume rate of air delivery in terms of standard air for a delivery pressure 8 times
ambient pressure under ambient air conditions of 300 °K and 1 bar.Answer: D
2 m3/hr
4m3/hr
3m3/hr
5m3/hr

A two stage compressor with first stage piston displacement of 94390 cm3/sec is driven by a
motor. Motor output is 35 Hp, suction temperature is 22°C, volumetric efficiency is 85%.
Mechanical efficiency is 95%, the intercooler pressure is 30 psi gage. Air temperature in and out
of the intercooler are 105°C and 44°C. Final discharge pressure is 100 psi gage, suction estimated
14.5 psi. Find the compression efficiency.Answer: A
77.60%
63.34%
87.34%
98.23%

An air compressor is to compress 8.5 m3/min from 98.56 Kpa to 985.6 Kpa. Assuming conditions
are ideal, and with n = 1.3, what will be the saving in work due to two staging?Answer: C
Zero
4.6 KW
5.6 KW
3.5 KW

A single stage air compressor handles 0.454 m3/sec of atmospheric pressure, 27°C air, and
delivers it to a receiver at 652.75 Kpa. Its volumetric efficiency on an isothermal basis is 0.85 and
its mechanical efficiency is 0.90. If it operates at 350 rpm, what power in KW is required to drive
it?Answer: B
95
112
120
100

A two stage compressor receives 0.50 kg/s of air at 120 kpa and 300°K and delivers it at 7 Mpa.
Find the heat transferred in the intercooler.Answer: A
118.14 kw
124.55 kw
233.34 kw
187.34 kw

An air compressor is tested and it is found that the electric motor used 37.3 kw when the
compressor handled 0.189 m3/s of air at 101.4 kpa and 300 °K and discharged it at 377.1 kpa.
Determine the adiabatic efficiency.Answer: C
67.34%
76.34%
81.89%
92.34%

An air compressor is tested and it is found that the electric motor used 37.6 kw when the
compressor handled 0.189 m3/s of air at 101.4 kpa and 300°K and discharged it at 377.1 kpa.
Determine the isothermal efficiency.Answer: A
67.34%
76.34%
81.89%
67.48%

Calculate the volumetric efficiency of a single-cylinder, double acting compressor with a bore
and stroke of 0.45 x 0.45 m. The compressor is tested at 15- rpm and found to deliver gas from
101.3 kpa and 300°K to 675 kpa at a rate of 0.166 m3/s when n- 1.33 for expansion and
compression process.Answer: A
46.39%
56.23%
48.34%
74.23%

A reciprocating compressor with a 3% clearance receives air at 100 kpa and 300°K and
discharges it at 1.0 Mpa. The expansion and compression are polytropic with n = 1.25. There is a
5% pressure drop through the inlet and outlet valves. Find the volumetric efficiency.Answer: B
76.23%
82.50%
98.33%
65.33%

A reciprocating compressor has a 5% clearance with a bore and stroke of 25 x 30 cm. The
compressor operates at 50 rpm. The air enters the cylinder at 27°C and 96 kpa and discharges at
2000 kpa. If n = 1.3, determine the compressor power.Answer: C
2.34 kw
18.45 kw
27.25 kw
34.32 kw

A water-jacketed air compressor handles 0.143 m3/s air entering at 96.5 kpa and 21°C and
leaving at 480 kpa and 132°C; 10.9 kg/h of cooling water enters the jacket at 15°C and leaves at
21°C. Determine the compressor power.Answer: A
26.163 kw
17.23 kw
34.44 kw
19.33 kw

A 2-stage compressor operates between constant pressure limits of 98.6 kpa and 1.103 Mpa. The
swept volume of the low pressure piston is 0.142 m3. Due to failure of the cooling water supply to
the intercooler, air is passed to the high pressure cylinder without reduction in temperature. Using
PV1.2 = C, determine the percentage increase in power.Answer: C
26
21
11
16

An ideal-single stage air compressor without clearance takes in air at a100 kpa with a temperature
of 16°C and delivered at 413 Kpa after isentropic compression. What is the discharge work done
b the compressor in KJ/kg?
-59.22
-52.43
-54.75
-56.13
Answer: -145 KJ/kg (no answer in the given choices)

PUMPS

A double suction centrifugal pumps delivers 70 ft3/sec of water at a head of 12 m and running
at 1250 rpm. What is the specific speed of the pump?Answer: D
5014 rpm
5453 rpm
2345 rpm
9968 rpm

The pump centerline of a centrifugal pump is located 2.5 m above from the high tide level.
The sea water varies two meters from high tide to low tide level. If friction loss at the suction
is 0.8 m, determine the total suction head.Answer: A
5.30 m
2.30 m
6.30 m
8.23 m

A centrifugal pump requires 40 ft head to deliver water from low level to higher level. If
pump speed is 1600 rpm, determine the impeller diameter of the centrifugal pump.
Answer: A
185 mm
160 mm
154 mm
176 mm

The suction pressure of a pump reads 2 in. of mercury vacuum and discharge pressure reads
130 psi is used to deliver 100 gpm of water with specific volume of 0.0163 ft 3/lb. Determine
the pump work.Answer: B
4.6 KW
5.7 KW
7.4 KW
9.4 KW

A pump is to deliver 150 gpm of water at ahead of 120 m. If pump efficiency is 70%, what is
the horsepower rating of motor required to drive the pump?Answer: D
40.44 Hp
25.66 Hp
38.44 Hp
21.33 Hp

A motor is used to d rive a pump having an efficiency of 85% and 70 % respectively. What is
the combined efficiency of pump and motor?Answer: A
59.50%
61.50%
62.50%
65.50%

In a boiler feed pump, the enthalpy at the entrance is 765 KJ/kg. If pump has a head of 900
m, what is the exit enthalpy of the pump.Answer: C
897 KJ/kg
465 KJ/kg
774 KJ/kg
864 KJ/kg

A submersible pump delivers 350 gpm of water to a height of 5 ft from the ground. The
pump was installed 120 ft below the ground level and a draw down of 8 ft during the
operation. If water level is 25 ft above the pump, determine the pump power.Answer: C
7.13 KW
4.86 KW
7.24 KW
864 KW

Determine the number of stages needed for a centrifugal pump if it is used to deliver 400
gal/min of water and pump power of 15 Hp. Each impeller develops a head of 38 ft.
Answer: B
6
4
8
7

A boiler feed pump receives 50 li/sec of water with specific volume of 0.00112 m 3/kg at
ahead of 750 m. What is the power output of driving motor if pump efficiency is 65%?
Answer: A
505.32 KW
643.54 KW
785.56 KW
356.45 KW

What power can a boiler feed pump can deliver a mass of 35 kg/s water at a head of 500 m?
Answer: D
356.56 KW
354.54 KW
456.64 KW
171.67 KW

A pump running at 100 rpm delivers water against a head of 30 m. IF pump speed will
increase to 120 rpm, what is the increase in head?Answer: B
43.2 m
13.2 m
34.6 m
56.3 m

A pump is used to deliver 50 li/sec of sea water at a speed of 120 rpm. If speed will increase
to 135 rpm, determine the increase in pump capacity.Answer: D
56.25 li/sec
34.56 li/sec
87.54li/sec
6.260 li/sec

A 15 KW motor running at 350 rpm is used to drive a pump. If speed will be changed to 370
rpm, what is the increase in power?Answer: A
2.72 KW
17.72 KW
56.45 KW
5.67 KW

A certain pump is used to deliver 150 gpm of water having a density of 61.2 lb/ft3. The
suction and discharge gage reads 4 in Hg vacuum and 25 psi, repectively, The discharge gage
is 2 ft above the suction gage. What is the brake power of the motor if pump efficiency is
75%?Answer: A
3.24 Hp
2.67 Hp
5.45 Hp
6.89 Hp

The discharge pipe of a pump is 400 mm in diameter delivers 0.5 m3/sec of water to a
building which maintains a pressure of 100 Kpa at a height of 30 m above the reservoir. If
equivalent head is 2m, what power must be furnished by the pump?Answer: A
211 KW
480 KW
340 KW
240 KW

A centrifugal pump is designed for 1800 rpm and head of 61m. Determine the speed of
impeller if diameter is reduces from 305 mm to 254 mm.Answer: C
1000 rpm
1250 rpm
1500 rpm
1600 rpm

Water from a reservoir A 10 m elevation is drawn by a motor driven pump to an upper


reservoir B at 72m elevation. Suction and discharge head loss are 0.15m, respectively. For
discharge rate of 15li/sec, find the power input to the motor if overall efficiency is
65%.Answer: C
12.65 KW
23.54 KW
14.17 KW
45.35 KW

The elevation of suction reservoir is 5m above the pump centerline and delivers to 85m
elevation tank which maintain 150 Kpa. If 1.5m3/sec of water is used to deliver a total head
of 3m, determine the power needed by the pump.Answer: A
1446 KW
2546 KW
4675 KW
3456 KW

Water from a reservoir is pumped over a hill through a pipe 900 mm in diameter and a
pressure of one kg/cm2 is maintained at the pipe discharge where the pipe is 85m from the
pump centerline. The pump has a positive suction head of 5m. Pumping rate of the pump at
1000 rpm is 1.5 m3/sec. Friction losses is equivalent to 3m of head loss. What amount of
energy must be furnished by the pump in KW?Answer: A
1372 kw
1523 kw
1234 kw
1723 kw

Water from a reservoir is pumped over a hill through a pipe 450 mm in diameter and a
pressure of 1 kg/cm2 is maintained at the summit. Water discharge is 30m above the reservoir.
The quantity pumped is 0.5 m3/sec. Frictional losses in the discharge and suction pipe and
pump is equivalent to 1.5m head loss. The speed of the pump is 800 rpm what amount of
energy must be furnished by the pump in KW?Answer: B
202
206
204
208

Water from an open reservoir A at 8m elevation is drawn by a motor-driven pump to an open


reservoir B at 70m elevation. The inside diameter of the suction pipe is 200mm and 150mm
for the discharge pipe. The suction line has a loss of head three times that of the velocity head
in the 200 mm pipe. The discharge line has a loss of head twenty times that of the velocity
head in the discharge pipeline. The pump centerline is at 4m. Overall efficiency of the system
is 78%. For a discharge rate of 10li/sec, find the power input to the motor and the pressure
gage reading installed just at the outlet and inlet of the pump in Kpag.Answer: D
2.34 kw
5.45 kw
6.59 kw
7.84 kw

A pump is to deliver 80 gpm of water at 140°F with a discharge pressure of 150 psig. Suction
pressure indicates 2 inches of mercury vacuum. The diameter of suction and discharge pipes
are 5 inches, respectively. The pump has an efficiency of 70%, while the motor efficiency is
80%. Determine power input to the drive motor.Answer: D
7.23 kw
2.34 kw
8.34 kw
9.39 kw

Determine the water horsepower and the mechanical efficiency of a centrifugal water pump
which has an input of 3.5 Hp if the pump has an 8 inches nominal size suction and 6 inches
nominal size discharge if it handles 150 gpm of water at 150°F. The suction line gage shows
4” Hg vacuum and the discharge gage shows 26 psi. The discharge gage is located 2 feet
above the center of the discharge pipe line and pump inlet and discharge lines are at the same
elevation.Answer: A
2.52 hp
6.33 hp
4.23 hp
8.34 hp

A boiler feed pump receives 40 liters per second at 4 Mpa and 180°C. It operates against a
total head of 900 m with an efficiency of 60%. Determine the enthalpy leaving the pump in
KJ/kgAnswer: B
783.45
773.57
756.23
765.23

A boiler feed pump receives 40 liters per second at 180°C. It


operates against a total head of 900 m with an efficiency of 60%. Determine power output of
the driving motor in KW.Answer: D
453.23
623.34
983.45
523.27

A boiler feed pump receives 40 liters per second at 180°C. It operates against a total head of
900m with an efficiency of 60%. Determine discharge pressure in Kpa for a suction pressure
of 4 Mpa.Answer: A
11,850 kpa
13,455 kpa
12,566 kpa
14,233 kpa

A boiler feed pump receives 45 li/sec of water at 190°C and enthalpy of 839.33 KJ/kg. It
operates against a head of 952m with efficiency of 70%. Estimate the water leaving
temperature assuming that the temperature rise as due to the inefficiency of the input
energyAnswer: A
191°C
143°C
123°C
165°C

A submersible, multi-stage, centrifugal deep well pump 260 gpm capacity is installed in a
well 27 feet below the static water level. Drawdown when pumping at rated capacity is 10
feet. The pump delivers the water into a 25,000 gallons capacity overhead storage tank. Total
discharge head developed by pump, including friction in piping is 243 feet. Calculate brake
horsepower required to drive the pump if pump efficiency is 70%.Answer: C
31.31
41.41
21.21
51.51

A submersible, multi-stage, centrifugal deep well pump 260 gpm capacity is installed in a
well 27 feet below the static water level and running at 3450 rpm. Drawdown when pumping
at rated capacity overhead storage tank. Total discharge head developed by pump, including
friction in piping is 243 feet. Calculate the diameter of the impeller of this pump in inches if
each impeller diameter developed a head of 38 ft.Answer: A
3.28
5.33
4.23
6.34

A double suction, single stage, centrifugal pump delivers 900 m3/hr of sea water(SG = 1.03)
from a source where the water level varies two meters from high tide to low tide level. The
pump centerline is located 2.6 meters above the surface of the water at high tide level. The
pump discharges into a surface condenser, 3m above pump centerline. Loss of head due to
friction in suction pipe is 0.8m and that in the discharge side is 3m. Pump is directly coupled
to a 1750 rpm, 460 V, 3 phase, 60 Hz motor. Calculate the specific speed of pump in
rpm.Answer: B
3131 rpm
5151 rpm
4141 rpm
6161 rpm

A pump driven by an electric motor moves 25 gal/min of water from reservoir A to reservoir
B, lifting the water to a total of 245 feet. The efficiency of the pump and motor are 64% and
84% respectively. What size of motor(HP) is required?Answer: B
Hp
3 Hp
4 Hp
7.5 Hp
Water is pumped at 1m3/sec to an elevation of 5m through a flexible hose using a 100%
efficiency pump rated at 100 KW. Using the same length of hose, what size of motor is
needed to pump 1 m3/sec of water to a tank with no elevation gain? In both cases both ends of
hose are at atmospheric pressure. Neglect kinetic energy.Answer: A
51 KW
18 KW
43 KW
22 KW

A vacuum pump is used to drain a flooded mine shaft of 20°C water. The pump pressure of
water at this temperature is 2.34 Kpa. The pump is incapable of lifting the water higher than
10.16m. What is the atmospheric pressure?Answer: D
109
112
98
101.9

It is desired to deliver 5 gpm at a head of 640m in a single stage pump having specific speed
not to exceed 40. If the speed is not to exceed 1352 rpm, how many stages are required?
Answer: D
3
4
5
2

The rate of flow of water in a pump installation is 60.6 kg/sec. The intake static gage is
located 1.22m below the pump center line and reads 68.95 Kpa gage; the discharge static
gage is 0.61m below the pump centerline and reads 344.76 Kpa gage. The gages are located
close to the pump as much as possible. The areas of the intake and discharge pipes are 0.093
m2 and 0.069 m2, respectively. The pump efficiency is 70%. Take density of water equals
1000 kg/m3. What is the hydraulic power in KW?Answer: A
17.0
24.5
31.9
15.2

Water in rural areas is often extracted from underground water source whose free surface is
60m below ground level. The water is to be raised 5m above the ground by a pump. The
diameter of the pipe is 10 cm at the inlet and 15 cm at the exit. Neglecting any heat
interaction with the surroundings and frictional heating effects, what is the necessary power
input to the pump for a steady flow of water at the rate of 15 li/sec in KW if pump efficiency
is 85%?Answer: D
9.54
5.343
7.82
11.23
A pump lifts water at a rate of 283 lps from a lake and force it into a tank 8m above the level
of the water at a pressure of 137 kpa. What is the power required in kilowatts.Answer: C
71
41
61
51

A pump receives 8kg/s of water at 220 Kpa and 110°C and discharges it at 1100 kpa.
Compute for the power required in kilowatts.Answer: C
8.126
5.082
7.041
6.104

A fuel pump is delivering 10 gallons per minute of oil with a specific gravity of 0.83. The
total head is 9.14m, find how much energy does the pump consumes in KJ per hour.
Answer: A
169
19
189
179

A pump discharges 150 liters per second of water to a height of 75 meters. If the efficiency is
75% and the speed of the pump is 1800 rpm, what is the torque in N-m to which the drive
shaft is subjected?Answer: B
771
781
791
681

A pump with a 400 mm diameter suction and a 350 mm diameter discharge pipe is to deliver
20,000 liters per minute of 15.6°C water. Calculate the pump head in meters if suction gage
is 7.5cm below pump centerline and reads 127mm Hg vacuum and discharge gage is 45cm
above the pump centerline and reads 75 kpa.Answer: D
15 m
5m
20 m
10 m

A centrifugal pump delivers 300,000 liters per hour of water to a pressurized tank whose
pressure is 280 kpa. The source of water is 5m below the pump. The diameter of the suction
pipe is 300mm and the discharge pipe is 250mm. Calculate the kw rating of the driving motor
assuming the pump efficiency to be 72%.Answer: D
41.75 kw
35.75 kw
43.26 kw
38.16kw
A centrifugal pump delivers 80 liters per second of water on test. Suction gage reads 10 MM
Hg vacuum and 1.2 meters below pump centerline. Power input is 70 kw. Find the total
dynamic head in meters. Answer: A
66
60
62
64

A pump delivers 500 gpm of water against a total head of 200 ft and operating at 1770 rpm.
Changes have increased the total head to 375 ft. At what rpm should the pump be operated to
achieve the new head at the same efficiency?Answer: D
2800 rpm
3600 rpm
3434 rpm
2424 rpm

FANS AND BLOWERS

A fan whose static efficiency is 40% has a capacity of 60,000 ft 3/hr at 60°F and barometer of
30 in Hg and gives a static pressure of 2 in of water column on full delivery. What size of
electric motor should be used to drive the fan?Answer: B
½ Hp
1 Hp
2 Hp
1 ½ Hp

A fan draws 1.42 m3 per second of air at a static pressure of 2/54 cm water through a duct
300mm diameter and discharges it through a duct of 275mm diameter. Determine the static
fan efficiency if total fan mechanical is 70% and air is measured at 25°C and 760 mm
Hg.Answer: A
50.11%
54.34%
65.37%
45.34%

Find the air horsepower of an industrial fan that delivers 25 m3/s of air through a 900mm by
1200mm outlet. Static pressure is 127mm of water gage and air density is 1.18
kg/m3.Answer: B
65.35 Hp
52.35 Hp
60.35 Hp
70.35 Hp
A boiler requires 90,000 m3/hr of standard air. The mechanical efficiency of fan to be
installed is 65%. Determine the size of driving motor assuming fan can deliver a total
pressure of 150mm of water gage.Answer: A
56.6 KW
78.5 KW
45.5 KW
23.5 KW

At 1.2 kg/m3 air density a fan develops a brake power of 100 KW. It operates at 98 Kpa and
32°C with the same speed, what is the new brake power of the fan?Answer: D
68.4 KW
36.7 KW
67.5 KW
93.3 KW

A fan has a suction pressure of 30mm water vacuum with air velocity of 3m/sec. The
discharge has 150 mm of water gage and discharge velocity of 7m/sec. Determine the total
head of fan if air density is 1.2 kg/m3.Answer: B
150 m
152 m
154 m
156 m

A 50 KW motor is used to drive a fan that has a total head of 110 m. If fan efficiency is 70%,
what is the maximum capacity of the fan using standard density of air?Answer: A
27 m3/sec
29 m3/sec
31 m3/sec
33 m3/sec

A fan using standard air condition can develop a static pressure head of 310 mm water gage.
If fan will operate 93°C and 735 mm of Hg, find the new static pressure required.Answer: C
150 mm water gage
180 mm water gage
241 mm water gage
456 mm water gage

The total head of fan is 185m and has a static pressure of 210 mm of water gage, what is the
velocity of air flowing if density of air is 1.15 kg/m3?Answer: A
6.85 m/sec
3.45 m/sec
4.75 m/sec
8.54 m/sec

The volume flow of air delivered by fan is 20 m3/sec and 180 mm water gage. The density of
air is 1.185 kg/m3 and motor power needed to drive the fan is 44 KW. What is the fan
efficiency?Answer: B
70.26%
80.26%
75.26%
90.26%
A fan is listed as having the following performances with standard
air: Volume discharged = 120 m3/sec
Speed = 7.0 rps
Static pressure = 310 mm water gage
Brake power required = 620 KW
The system duct will remain the same and the fan will discharge the same volume of 120
m3/sec of air at 93°C and a barometric pressure of 735 mm Hg when its speed is 7.0 rps. Find
the brake power input and the static pressure required.Answer: C
231 mm water
251 mm water
241 mm water
261 mm water

Air enters a fan through a duct at velocity of 6.3 m/s and an inlet static pressure of 2.5 cm of
water less than atmospheric pressure. The air leaves the fan through a duct a t a velocity of
11.24 m/s and a discharge static pressure of 7.62 cm of water above the atmospheric pressure.
If the specific weight of the air is 1.20 kg/m3 and the fan delivers 9.45 m3/sec, what is the fan
efficiency when the power input to the fan is 12.75 KW at the coupling?Answer: A
71.81%
91.23%
81.34%
61.34%

A fan delivers 4.7 m3/sec at a static pressure of 5.08 cm of water when operating at a speed of
400 rpm. The power input required is 2.863 KW. If 7.05 m3/sec are desired in the same fan
and installation, find the pressure in cm of water.Answer: C
7.62
17.14
11.43
5.08

A fan described in a manufacturer’s table is rated to deliver 500 m3/min at a static pressure
(gage) of 254 cm of water when running at 250 rpm and requiring 3.6 KW. If the fan speed is
changed to 305 rpm and air handled were at 65% instead of standard 21°C, find the power in
KW.Answer: D
3.82
5.08
4.66
5.68

Air is flowing in a duct with velocity of 7.62 m/s and static pressure of 2.16 cm water gauge.
The duct diameter is 1.22m, the barometric pressure 99.4 Kpa and the gage fluid temperature
and air temperature are 30°C. What is the total pressure against which the fan will operate in
cm of water?Answer: B
3.25
2.5
3.75
1.25

FLUID
A perfectventure with throat diameter of 1.8 inches is placed horizontally in a pipe with a 5 in
inside diameter. Eighty (80) lb of water flow through the pipe each second. What is the
difference between the pipe and venturi throat static pressure?Answer: B
29.9 lb/in2
34.8 lb/in2
5020 lb/in2
72.3 lb/in2

What is the expected head loss per mile of a closed circular pipe (17 in inside diameter),
friction factor of 0.03 when 3300 of water flow under pressure?Answer: A
38 ft
3.580 ft
0.007 ft
0.64 ft

A rigid container is closed at one end and measure 8 in diameter by 12 in long. The container
is held vertically and is slowly moved downward until the pressure in the container is 15.5
psia. What will be the depth of the water surface measure from the free water surface?
Answer: A
22 in
9.2 in
12 in
9.8 in

The pressure drop across a turbine is 25 psi. The flow rate is 55 gallons per minute. Calculate
the power output of the turbine.Answer: A
0.802 hp
0.41 hp
1.05 hp
2.54 hp

Two tubes are mounted to the roof of a car. One tube points to the front of the car while the
other point to the rear. The tubes are connected to a manometer filled with fluid of specific
gravity. 0.60. When the height difference is 2 inches, what is the car’s speed?Answer: C
46 mph
43.8 mph
50 mph
62 mph

What is the density of a stone that weighs 19.9 lb (88 N) in air and 12.4 lb (55 N) in water?
Answer: A
2,651.2 kg/m3
2,612.5 kg/m3
2,578.2 kg/m3
26,700 kg/m3

The flow rate of water through a cast iron pipe is 5000 GPM. The pipe is 1 ft and the
coefficient of friction f = 0.0173. What is the pressure drop over a 100 ft length of pipe?
Answer: 5.404 ft of water
337.26 lb/in2
21.078 lb/in2
337.26 lb/in2
23.780 lb/in2

Pressure drop across a turbine is 30 psi, the flow rate is 60 gpm. Calculate the power output
of the turbine.Answer: B
0.41 hp
1.05 hp
6.30 hp
2.54 hp

The fluid in a manometer tube is 60% water and 40% alcohol (SG = 0.8). What is the
monometer fluid height difference if a 6.2 psi pressure is applied across the tow ends of a
manometer?Answer: D
15.5 in
186 in
36 in
215 in

An air bubble rises from the bottom of a well where the temperature is 25°C, to the surface
where the temperature is 27°C. Find the percent increase in volume of the bubble if the depth
of the well is 5m. Atmospheric pressure is 101.528 Kpa.Answer:A
49.3
31.3
56.7
38.6

An empty, open can is 30 cm high with a 10-cm diameter. The can, with the open end down,
is pushed under water with a density of 1000 kg/m3. Find the water level in the can when the
top of the can is 50 cm below the surface.Answer: B
17.20 cm
2.12 cm
4.20 cm
5.87 cm

A 24 inches long rod floats vertically in water. It has a 1 sq. in. cross section and has a
specific gravity of 0.6. What length L is submerged?Answer: A
14.4 in
2.4 in
9.6 in
18.0 in
Flow of water taking over in a pipe having a velocity of 10 m/s. Determine the velocity head
of the water.Answer: B
50.1 m
5.1 m
8.2 m
100m

The length of pipe is 168 meters. If the pressure drop is 50 Kpa for every 30 meters, what is
the total pressure drop?Answer: C
260 kpa
300 kpa
280 kpa
100 kpa

Hydraulic efficiency 85%, find Q in li/s. Power developed 10,500 kw under head of 320
m.Answer: A
3935
3452
3271
3623

A cylindrical pipe with water flowing downward at 0.02 m3/s having top diameter of 0.08,
bottom diameter of 0.04 and height of 1.5 m. Find the pressure between the pipes. Answer: A
104 kpa
87 kpa
120 kpa
143 kpa

A fluid that has a velocity of 18 m/s will have an equivalent head of?Answer: A
16.51 m
12.44 m
18.34 m
10.34 m

Determine the size of pipe which will deliver 5 liters of medium oil (v = 6.10 x 10-6 m2/s)
assuming laminar flow conditions.Answer: A
522 mm
454 mm
550 mm
650 mm

The power available in a jet having a cross-sectional area of 0.005 m2 with a velocity of 25.80
m/s.Answer: C
34 kw
49 kw
43 kw
23 kw

The type of flow occupying in a 30 cm diameter pipe which water flows at a velocity of 2.10
m/s. Use v = 1.13 x 10-6 m2/s for water.Answer: A
Turbulent
Constant
Laminar
None of these

A man weighing 64 kg causes a flat area 30 cm thick to be just fully submerged in a sea
water (SG = 1.03). Neglecting weight the area must be:Answer:D
0.085 m2
0.756 m2
0.172 m2
0.062 m2

What force is exerted by water jet 50 mm diameter if it strikes a wall at the rate of 15
m/sAnswer: B
342 N
442 N
764 N
563 N

A 300 mm x 75 mm venture meter is inserted in a 300 mm diameter pipeline where water


flows at 55 liters/s. Neglecting friction loss, compute the drop in pressure head from the inlet
to the throat.Answer: A
8m
10 m
6m
12 m

A jet of water 50 mm diameter with a velocity of 35 m/s is being discharged in a horizontal


direction from a nozzle mounted on a fire truck. The force required to hold the nozzle
stationary is:Answer: B
1.34 KN
2.40 KN
4.23 KN
3.23 KN

An open storage vessel has 3 m of oil (SG = 0.82) and 6 m of water. The pressure at the
bottom is:Answer: C
45 kpa
65 kpa
83 kpa
92 kpa

A 200 mm diameter pipe gradually reduces to a 100 mm diameter. The 200 mm diameter
pipe is connected to another pipe having a pressure of 600 kpa horizontally with a flow of
0.04 m3/s. Find the pressure at the 100 mm diameter.Answer: A
588 kpa
642 kpa
566 kpa
598 kpa
A 200 mm diameter pipe discharges water at the rate of 200 li/s. Point 1 on the pipe has a
pressure of 280 kpa and 3.4 m below point 1 is point 2 with a pressure of 300 kpa. Compute
the head loss between points 1 and 2.Answer: D
4.2 m
2.5 m
6.3 m
1.4 m

An object weighs 90 N in air and when immersed in water weighs 50 N. Compute the
specific gravity of the object.
1.25
3.25
2.25
4.25

REFRIGERATION

The enthalpy at the entrance of the condenser is 1660 KJ/kg and exit is 315 KJ/kg. The
compressor has an enthalpy of 1450 KJ/kg at entrance. Determine the COP.Answer: C
4.2
2.0
5.4
6.0

An evaporator has a temperature of 3°C with entrance enthalpy of 352.75 KJ/kg. At 3, hf =


319.56 KJ/kg and hg = 642.45 KJ/kg. Find the quality after expansion.Answer: D
16.27%
21.48%
15.67%
10.27%

The refrigerating effect of 100 tons refrigeration is 117 KJ/kg. Determine the mass flow of
refrigerant.Answer: C
1kg/sec
2kg/sec
3 kg/sec
4 kg/sec

The density of R-12 refrigerant at compressor suction is 19.81 kg/m3. For mass flow rate of 2
kg/sec, determine the volume flow at suction.Answer: A
0.10 m3/sec
0.25 m3/sec
0.30 m3/sec
0.40 m3/sec

A 100 ton refrigeration system has a COP of 5. Determine the compressor horsepower.
Answer: A
94.26 Hp
86.76 Hp
86.76 Hp
65.65 Hp

A 90 ton refrigeration system has a compressor input of 0.97 KW per ton refrigeration and
COP of 5. What is the efficiency of the compressor?Answer: A
72.49%
90.42%
56.34%
83.33%

A 90 ton refrigeration system has a compressor input of 0.97 KW per ton refrigeration. If
compressor efficiency is 75%, determine the heat rejected from the condenser.Answer: D
350 KW
260 KW
500 KW
382 KW

The mass flow rate of refrigerant entering the compressor is 0.25 kg/s and the change of
enthalpy between the inlet and outlet is 320 KJ/kg. If 134 Hp motor is used to drive the
compressor, determine the heat loss from the compressor.Answer: B
15 KW
20 KW
25 KW
30 KW

A 100 ton refrigeration system is used to produce chilled water from 22°C to 2°C. Determine
the volume flow of water in li/sec.Answer: C
3.0
3.5
4.2
5.5

An industrial plant requires cooling 120 gal/min of water from 20°C to 5°C. Determine the
tons of refrigeration required. Answer: D
100 TR
130 TR
145 TR
135 TR

The difference in temperature between the water leaving the evaporator and evaporator
temperature is 5°C. If the temperature of water leaving is 32°F, what is the evaporator
temperature?Answer: D
0°C
-2°C
-4°C
-5°C
The heat rejected from condenser is 300 KW. The water required to cool the refrigerant is 5
li/sec. Determine the temperature of water leaving the condenser if water enters at
25°C.Answer: D
30°C
35°C
45.33°C
39.33°C

The mass flow of refrigerant entering the compressor is 0.1 kg/sec with change of enthalpy of
400 KJ/kg. For compressor efficiency of 70% and motor efficiency of 80%, find the electrical
energy needed for one day.Answer: D
1524 KW-hr
1685 KW-hr
3456 KW-hr
1714 KW-hr

An 80 ton refrigeration system requires 105 KW from VECO. If motor compressor efficiency
is 60%, find the COP.Answer: C
5.84
3.56
4.46
3.78

The change of enthalpy between the inlet and outlet of evaporator is 1000 KJ/kg and mass
flow of refrigerant is 12 kg/min. What is the capacity of plant?Answer: C
15 TR
25 TR
57 TR
60 TR

The mass flow of water entering the condenser is 10 kg/sec. If the temperature difference
between inlet and outlet temperature is 18°F, determine the heat rejected from the
condenser.Answer: B
400 KW
419 KW
500 KW
324 KW

The change of enthalpy in the condenser is 1500 KJ/kg. The temperature change of water is
8°C and the refrigerant flow is 0.13 kg/sec. Determine the gpm of water required for
cooling.Answer: A
92 gpm
88 gpm
45 gpm
67 gpm
The heat rejected from the condenser is 200 KW. The mass flow of water entering is 5 kg/sec
at 23°C. If the temperature between the condenser cooling water outlet and condenser
temperature is 5°C, what is the condenser temp?Answer: D
30.56°C
39.94°C
35.78°C
37.67°C

The heat from the condenser of Carnot refrigeration is 6000 KJ/min. The minimum and
maximum temperature is -25°C and 72°C respectively. Determine the horsepower required to
drive the compressor. Answer: C
30 Hp
35 Hp
38 Hp
42 Hp

The temperature difference between the minimum and maximum temperature of Carnot cycle
is 50°C. What is the minimum temperature if COP is 5.5Answer: A
0.31°C
2°C
3°C
5°C

The power required to drive the compressor in Carnot refrigeration is 50 Hp. It operates
between -5°C and 40°C. Determine the tons of refrigeration.Answer: D
57 TR
34 TR
55TR
63 TR

Determine the heat to be removed from one ton of water at 26°C to an ice at -6°C.Answer: B
215,765 KJ
413,956 KJ
345,654 KJ
834,582 KJ

The heat required to remove from beef 110 kg is 36,437 which will be cooled from 20°C to -
18°C. The specific heat above freezing is 3.23 KJ/kg-°K and latent heat of fusion is 233
KJ/kg. If specific heat below freezing is 1.68 KJ/kg-°K, find the freezing
temperature.Answer: B
-5.65°C
-2.19°C
-1.01°C
-10°C
A refrigeration compressor has a specific volume of 0.0482 m3/kg entrance and 0.017 m3/kg
at exit. If volumetric efficiency is 90%, determine the percent clearance of the
compressor.Answer: A
5.45%
6.35%
10%
12%

Milk must be received and cooled from 80°F to 38°F in 5 hrs. If 4000 gallons of fresh milk
received having SG of 1.03 and cp = 0.935 Btu per lb-°F, find the refrigeration
capacity.Answer: C
20 TR
30TR
22.5TR
34.6 TR

An air-conditioning plant with a capacity of 400 KW of refrigeration has an evaporating and


condensing temperature of 3°C and 37°C respectively. If it uses Refrigerant 12, what will be
the mass of flash gas per kg of refrigerant circulated?Answer: A
21.80%
12.45%
14.56%
18.34%

An air-conditioning plant with a capacity of 400 KW of refrigeration has an evaporating and


condensing temperature of 3°C and 37°C respectively. If it uses Refrigerant 12, what is the
volumetric rate of flow under suction condition?Answer: C
0.272 m3/s
0.453 m3/s
0.172 m3/s
0.243 m3/s

An air-conditioning system of a high rise building has a capacity of 350 KW of refrigeration;


uses R-12. The evaporator and condenser temperature are 0°C and 35°C, respectively.
Determine work of compression in KW.Answer: B
34 KW
52 KW
43 KW
65 KW

A vapor compression refrigeration system has a 30 KW motor driving the compressor. The
compressor inlet pressure and temperature are 64.17 Kpa and 20°C respectively and
discharge pressure of 960 Kpa. Saturated liquid enters the expansion valve. Using Freon 12
as refrigerant, determine the capacity of the unit in tons of refrigeration.Answer: D
16.15 tons ref.
20 tons ref.
25 tons ref.
17.14 tons ref

A simple vapor compression cycle develops 15 tons of refrigeration using ammonia as


refrigerant and operating at condensing temperature of 24°C and evaporating temperature of -
18°C and assuming compression are isentropic and that the gas leaving the condenser is
saturated, find the power per ton.Answer: D
0.333 kw/ton
0.533 kw/ton
0.452 kw/ton
0.702 kw/ton

A vapor compression refrigeration system is designed to have a capacity of 100 tons of


refrigeration. It produces chilled water from 22°C to 2°C. Its actual coefficient of
performance is 5.96 and 35% of the power supplied to the compressor is lost in the form of
friction and cylinder cooling losses. Determine size of the electric motor required to drive the
compressor in kilowatts.Answer: A
92.31 kw
76.24 kw
87.23 kw
98.23 kw

A vapor compression refrigeration system is designed to have a capacity of 100 tons of


refrigeration. It produces chilled water from 22°C to 2°C. Its actual coefficient of
performance is 5.86 and 35% of the power supplied to the compressor is lost in the form of
friction and cylinder cooling losses. Determine the condenser cooling water required in
kg/sec for a temperature rise of 10°C.Answer: A
9.83
7.45
12.23
4.34

A belt driven compressor is used in a refrigeration system that will cool 10 li/sec of water
from 13°C to 1°C. The belt efficiency is 98%, motor efficiency is 85%, and the input of the
compressor is 0.7 KW/ton of refrigeration. Find the actual coefficient of performance if
overall efficiency is 65%.Answer: D
2.34
4.43
3.45
6.44
A belt driven compressor is used in a refrigeration system that will cool 10 li/sec of water
from 13°C to 1°C. The belt efficiency is 98%, motor efficiency is 85%, and the input of
compressor is 0.7 KW/ton of refrigeration. Find the mass flow rate of condenser cooling
water warmed from 21°C to 32°C if overall efficiency is 65%.Answer: B
10.34 kg/s
12.60 kg/s
23.23 kg/s
15.34 kg/s

Calculate the bore in mm of a single-acting, 6 cylinder ammonia compressor running at 1200


rpm to compress 700 kg/hr refrigerant which vaporized at -15°C, given the following
Bore and stroke = Unity
Volumetric Efficiency = 70%
Specific volume of NH3 = 8.15 ft3/lb at 5°FAnswer: C
110 mm
106 mm
114 mm
124 mm

An ammonia compressor operates at an evaporator pressure of 316 Kpa and a condenser


pressure of 1514.2 Kpa. The refrigerant is subcooled 5 degrees and is superheated 8 degrees.
A twin cylinder compressor with a bore to stroke ratio of 0.85 is to be used at 1200 rpm. The
mechanical efficiency is 78%. For a load of 87.5 kw, determine the bore and stroke for 5%
clearance.Answer: C
123.44 mm
109.23 mm
117.40 mm, 99.79 mm
234.23 mm, 86.79 mm

In a certain refrigeration system for a low temperature application, a two stage operation is
desirable which employs ammonia system that serves a 30 ton evaporator at -30°C. The
system uses direct contact cascade condenser, and the condenser temperature is 40°C. Find
the total power required in kw.Answer: B
23.43 kw
35.37 kw
28.34 kw
45.23 kw

A refrigeration system operates on the reversed Carnot cycle. The minimum and maximum
temperatures are -25°C and 72°C, respectively. If the heat rejected to the condenser is 6000
KJ/min, draw the T-S diagram and find power input required.Answer: D
20.34 kw
54.33 kw
13.45 kw
28.12 kw

A refrigerating system operates on the reversed Carnot cycle. The highest temperature of the
refrigerant in the system is 120°F and the lower temperature is 10°F. The capacity is to be 20
tons. Determine the heat rejected from the system in Btu/minAnswer: A
4936
5634
3432
7421

An ice plant produces 20 tons of ice per day at -15°C from water at 25°C. If miscellaneous
losses are 12% of the freezing and chilling load, calculate the refrigeration capacity of the
plant in tons of refrigeration. Answer: C
21.35
43.12
31.5
36.3

Fish weighing 11,000 kg with a temperature of 20°C is brought to a cold storage and which
shall be cooled to -10°C in 11 hours. Find the required plant refrigerating capacity in tons of
refrigeration if the specific heat of fish is 0.7 above freezing and 0.3 below freezing point
which is -3°C. The latent heat of freezing is 55.5 Kcal/kg.Answer: B
21.3
24.38
28.34
32.12

Compute the heat to be removed from 110 kg of lean beef if it were to be cooled form 20°C
to 4°C, after which it is to be frozen and cooled to -18°C. Specific heat above freezing is
given as 3.23 KJ/kg°C and below freezing as 1.68 KG/kg-°C. Freezing point is -2.2°C and
latent heat of fusion is 233 KJ/kg.Answer: C
32,455 KJ
23,455 KJ
36,437.5 KJ
54,233.2 KJ

Magnolia Dairy products plant must cool 4000 gallons of fresh milk received from the farm
per day from an initial temperature of 80°F to a temperature of 38°C in 5 hours. If the density
of milk is 8.6 lbs/gallon, specific gravity is 1.03, and specific heat is 0.935 how much brine
must be circulated if the change in temperature is 15°F, specific gravity is 1.182, and specific
heat is 0.729?Answer: A
41.77 gpm
54.22 gpm
34.11 gpm
65.23 gpm
A mass of ice at 25°F is needed to cool 250 pound of vegetables in a bunker, for 24 hours.
The initial temperature of the vegetables is assumed to be 85°F. It is also assumed that the
average temperature inside the bunker is 45°F, within the 24-hour period. If the heat gained
per hour in the bunker is 30% of the heat removed to cool the vegetables from 85°F to 45°F,
what would be the required mass of ice?Answer: D
Specific heat of ice = 0.463
Specific heat of vegetables = 0.80
Specific heat of water = 1.01 Heat
of fusion of ice = 144 Btu/lb
34.56 lbs
74.23 lbs
43.23 lbs
64.8 lbs

A 10 ton ice plant using ammonia refrigerant operates between evaporator and condenser
temperature of -20°C and 35°C respectively. The ice plant is to produce ice at -12°C from
water at 30°C in 24 hours. Assuming losses to be 18 of the heat absorbed from water,
determine the power required by the compressor.Answer: B
21.23 kw
16.79 kw
56.32 kw
43.32 kw

An ideal vapor compression refrigeration cycle requires 2.5 KW to power the compressor.
You have found the following data for the cycle: the enthalpy at condenser entrance = 203
KJ/kg, exit = 55; evaporator entrance = 55 KJ/kg, exit = 178. If mass flow rate of the
refrigerant is 0.10 kg/sec, then the coefficient of performance of the refrigeration cycle is
most nearly:Answer: D
592
59.2
5.92
4.92

A Freon – 12 waste water system operating at a 5°C suction temperature and a 40°C
condensing temperature has an evaporator load of 5 tons. If the condenser is selected for a
7°C water temperature rise, how many gpm must be circulated through the condenser?
The following enthalpies have been found: condenser entrance = 377 KJ/kg, exit = 238.5;
evaporator entrance = 238.5 KJ/kg, exit = 353.6.Answer: A
11.46 gpm
15.23 gpm
13.45 gpm
23.22 gpm

A refrigeration system operates on an ideal vapor-compression using Refrigerant-12 with an


evaporator temperature of -30°C and a condenser exit temperature of 49.3°C and requires a
74.6 KW motor to drive the compressor. What is the capacity of the refrigerator in tons of
refrigeration? Enthalpy of condenser entrance = 362 KJ/kg, exit = 248.15; evaporator
entrance = 248.15, exit = 338.14.Answer: A
43.5
34.5
21.3
18.2

Liquid ammonia at a temperature of 26°C is available at the expansion valve. The


temperature of the vaporizing ammonia in the evaporator is 2°C. Find the percentage of
liquid vaporized while flowing through the expansion valveAnswer: A
TemperaturePressure
(°C) (Kpa) hf hfg hg
2°C 462.49 190.4 1255.2 1445.6
26°C 1033.97 303.6 1162.0 1465.6
A. 9.02
B. 90.98
91.08
8.92

A 140 mm x 140 mm single effect, twin-cylinder single acting Freon-12 compressor with a
refrigeration capacity of 40 kw operates between a discharge pressure of 1300 kpa and a
suction pressure of 350 kpa. The speed of the compressor is 600 rpm. If the discharge
pressure shall be raised to 1400 kpa, at what speed (rpm) should the compressor be run to
produce the same refrigeration capacity and assuming the volumetric efficiency to remain the
same?Answer: C
610
614
622
630

A refrigeration system using R-22 has a capacity of 320 kw of refrigeration. The evaporating
temperature is minus 10 degrees C and the condensing temperature is 40°C. Calculate the
fraction of vapor in the mixture before the evaporator.Answer: A
Properties of R-22 are:
At -10°C
hg = 401.60 KJ/kg
ht = 188.426 KJ/kg
At 40°C
hf = 249.686 KJ/kg
0.287
0.315
0.245
0.227

What is the coefficient of a vapor compression refrigeration system having the following
data: Enthalpy entering the compression is 181.79; enthalpy after compression work is 207.3
KJ/kg. After condensation the enthalpy is 58.2 and the throttled from 0.19 Mpa to 0.18
Mpa.Answer: C
5.8
3.2
4.84
5.6

A refrigeration system having a 22 kw capacity needs 7.8 compressor. Find the COP of the
systemAnswer: C
3.12
4.62
3.78
6.34

AIR CONDITIONING
When 100 kg/min of outside air at 32°C dry bulb and 200 kg/min recirculated air at 22°C dry
bulb are mixed with an air conditioning system, the resulting dry bulb temperature will
be:Answer: A
25.33°C
35.44°C
46.33°C
26.88°C

The amount of water carried by air in a cooling tower is 15 lb/min. The change in humidity
ratio in outlet and inlet is 0.025 lb/lb. Determine the volume flow of air needed if specific
volume is 15 ft3/lb.Answer: B
6000 ft3/min
7800 ft3/min
7500 ft3/min
5000 ft3/min

The change of enthalpy of air in a cooling tower is 35 Btu/lb and the mass flow of air is
453.17 lb/min. Water enters the tower at the rate of 50 gpm and 115 °F. Determine the exit
temperature. Answer: D
45°F
55°F
65°F
77°F

The change of enthalpy in air conditioning unit is 10 Btu/lb. The mass of supply air is
150,000 lb/hr. What is the conditioner capacity?Answer: C
125 TR
100 TR
150 TR
200 TR

Recirculated air of 8 kg/sec with 53 KJ/kg enthalpy and outside air of 2 kg/sec with 90 KJ/kg
enthalpy enters the conditioning unit. Determine the air conditioning capacity if supply
enthalpy to conditioned space is 42 KJ/kg.Answer: D
154 KW
164 KW
174 KW
184 KW
Outside air of an air conditioning system is 25% of recirculated air. Determine the mass of
outside air if mass of supply air is 15 kg/sec.Answer: C
1 kg/sec
2 kg/sec
3 kg/sec
4 kg/sec

The sensible heat load and latent heat load in an air conditioning system is 120 KW and 47
KW, respectively. What is the sensible heat factor?Answer: D
65.34%
29.45%
76.54%
71.86%

The total heat load and latent heat load of theater is 150 KW and 60 KW, respectively.
Supply air is at 15°C and has a mass of 9 kg/sec. Determine the temperature to be maintained
in the theater.Answer: B
20°C
25°C
30°C
35°C

A cooling tower has an efficiency of 65%. Water enters the tower at 55°C. The wet bulb
temperature of surrounding air is 27°C. What is the temperature of water leaving the tower?
Answer: A
36.8°C
44.5°C
46.9°C
30.4°C

The change of temperature entering the cooling tower and wet bulb temperature of
surrounding air is 25°C and efficiency is 65%. If mass of water leaving the tower is 10
kg/sec, determine the heat carried by air.Answer: C
720 KW
540 KW
680 KW
700 KW

The approach and efficiency of cooling tower is 10°C and 60%, respectively. If temperature
of water leaving the tower is 37°C, what is the temperature of water entering?Answer: B
46°C
52°C
68°C
48°C

A cooling tower is used to cool a jacket water loss from the engine. The heat generated by
fuel is 2500 KW and cooling loss is 30%. If temperature range of the tower is 15°C.
Determine the mass flow of water entering the tower.Answer: A
12 kg/sec
14 kg/sec
16 kg/sec
18 kg/sec

A dryer is to deliver 1000 kg/hr of cassava with 2% moisture and 20% moisture in the feed.
Determine the mass of air required if change in humidity ratio is 0.0165.Answer: C
3.57 kg/sec
4.67 kg/sec
3.79 kg/sec
5.36 kg/sec
The moisture removed from a material is 250 lb/hr and change of humidity ratio in dryer is
0.0175. Determine the fan capacity if specific volume of air entering is 35 ft3/lb.Answer: C
400,000 ft3/hr
450,000 ft3/hr
500,000 ft3/hr
550,000 ft3/hr

The change of enthalpy in a heating chamber of dryer is 25 Btu and the mass of air supplied
is 30,000 lb/hr. What is the heat supplied by heater?Answer: D
560 KW
450 KW
350 KW
220 KW

Water at 55°C is cooled in a cooling tower which has an efficiency of 65%. The temperature
of the surrounding air is 32°C dry bulb and 70% relative humidity. The heat dissipated from
the condenser is 2,300,000 KJ/hr. Find the capacity in liters per second of the pump used in
the cooling water. Answer: A
8.55
7.34
6.34
9.23

Fifty gallons per minute of water enters a cooling tower at 115°F. Atmospheric air at 60°F
and 55% relative humidity enters the tower at 6,000 cfm and leaves at 90°F saturated.
Determine the volume of water that leaves the tower in gpm.Answer: B
32.34
48.62
34.23
65.33

Fifty gallons per minute of water enters a cooling tower at 115°F. Atmospheric air at 60°F
and 55% relative humidity enters the tower at 6,000 cfm and leaves at 90°F saturated.
Determine the exit temperature of water, °FAnswer: C
56.33
65.33
76.61
45.34

An atmospheric cooling tower is to provide for the jacket water of a four stroke, 800 KW
diesel generators with useful output of 34% and cooling loss of 30%. The cooling tower
efficiency is 60% at a temperature of approach of 10°C. If ambient air has a relative humidity
of 70% and dry bulb temperature of 32°C, determine the cooling water supplied to the diesel
engine in li/hr. Generator efficiency is 97%.Answer: A
41,713
45,232
43,345
47,234

250,000 kg/hr of water at 35°C enters a cooling tower where it is to be cooled to 17.5°C. The
energy is to be exchanged with atmospheric air entering the unit at 15°C and leaving the unit
at 30°C. The air enters at 30% RH and leaves at 85% RH. If all process are assumed to occur
at atmospheric pressure, determine the percentage of total water flow that is make up
water.Answer: D
2.44%
8.34%
3.94%
2.22%

An auditorium is to be maintained at a temperature of 26°C dry bulb and 50% RH. Air is to
be supplied at a temperature not lower than 15°C dry bulb. The sensible heat gain is 110 Kw
and the latent heat gain is 37.5 KW. Take ventilating air as 25% by weight of the air from the
room, and is at 35°C dry bulb and 60% RH. Determine the refrigeration capacity in tons?
Answer: C
43.45
54.23
63.28
76.34

An assembly hall was to have an air conditioning unit installed which would be maintained at
26°C dry bulb and at 50% RH. The unit delivers air at 15°C dry bulb temperature and the
calculated sensible heat load is 150 kw and latent heat is 51.3 KW. Twenty percent by weight
of extracted air is made up of outside air at 34°C dry bulb and 60% RH while 80% is
extracted by the air conditioner from the assembly hall. Determine the air conditioner’s
refrigeration capacity in tons refrigeration and its ventilation load in KW.Answer: A
83.22, 132
75.34, 412
56.23, 45.32
54.23, 83.23

An air conditioned heater is to be maintained at 80°F dry bulb temperature and 50% RH. The
calculated total sensible heat load in the theater is 620,000 Btu/hr, and the latent heat load is
210,000 Btu/hr. The air mixture is 84°F dry bulb and 72°F wet bulb temperature is cooled to
63°F dry bulb and 59°F wet bulb temperature by chilled water cooling coil and delivered as
supply air to the theater. Calculate the tons of refrigeration required.Answer: B
123
125
124
128

A room being air conditioned is being held at 25°C dry bulb and 50% relative humidity. A
flow rate of 5 m3/s of supply air at 15°C dry bulb and 80% RH is being delivered to the room
to maintain that steady condition at 100 Kpa. What is the sensible heat absorbed from the
room air in KW?Answer: B
50.8
60.8
40.5
70.9

Copra enters a dryer containing 60% water and 40% solids and leaves with 5% water and
95% solids. Find the weight of water removed based one each pound of original
product.Answer: D
0.34 lb
0.63 lb
0.86 lb
0.58 lb

Copra enters a dryer containing 60% water and 40% solids and leaves with 5% water and
95% solids. Find the weight of water removed based one each pound of final
product.Answer: A
1.375 lb
1.19 lb
1.872 lb
2.345 lb

The temperature of the air in a dryer is maintained constant by the use of steam coils within
the dryer. The product enters the dryer at the rate of 1 metric ton per hour. The initial
moisture content is 3 kilograms moisture per kilograms dry solid and will be dried to a
moisture content of 0.10 kg moisture per kg solid. Air enters the dryer with a humidity ratio
of 0.016 kg moisture per kg dry air and leaves with a relative humidity of 100% while the
temperature remains constant at 60°C. If the total pressure of air is 101.325 Kpa, determine
capacity of forced draft fan to handle this air in m3/min.Answer: D
80
84
82
86

Wet material, containing 215% moisture (dry basis) is to be dried at the rate of 1.5 kg/sec in a
continuous dryer to give a product containing 5% moisture (wet basis). The drying medium
consists of air heated to 373°K and containing water vapor equivalent to a partial pressure of
1.40 Kpa. The air leaves the dryer at 310°CK and 70% saturated. Calculate how much air
will be required to remove the moisture.Answer: C
213,233 kg/hr
177,142 kg/hr
177,242 kg/hr
198,234 kg/hr

A Dryer is to deliver 1000 kg/hr of palay with a final moisture content of 10%. The initial
moisture content in the feed is 15% at atmospheric condition with 32°C dry bulb and 21
degrees centigrade wet bulb. The dryer is maintained 45°C while the relative humidity of the
hot humid air from the dryer is 80%. If the steam pressure supplied to the heater is 2 Mpa,
determine the heat supplied by heater in kw.Answer: B
3.23
5.46
4.23
6.23

A Dryer is to deliver 1000 kg/hr of palay with a final moisture content of 10%. The initial
moisture content in the feed is 15% at atmospheric condition with 32°C dry bulb and 21
degrees centigrade wet bulb. The dryer is maintained 45°C while the relative humidity of the
hot humid air from the dryer is 80%. If the steam pressure supplied to the heater is 2 Mpa,
determine the air supplied to dryer in m3/hr.Answer: A
1332.25
1532.34
1234.23
1982.34

At 30°C, air-vapor mixture has a relative humidity of 70%. Find the humidity ratio if
barometric pressure is 100°C. At 30°C, Psat = 4.246 kpaAnswer: D
0.123
0.986
0.054
0.019

The humidity ratio of air is 0.045. If barometric pressure is 101 kpa, find the partial pressure
of water vapor.Answer: C
4.23 kpa
7.34 kpa
6.81 kpa
5.23 kpa

Air at 36°C and pressure of 101.2 kpa has a density of 1.08 kg/m3. Find the humidity ratio of
air.Answer: A
0.0352
0.6350
0.0635
0.0173

What is the enthalpy of the air-vapor mixture at 60% RH and 35°C when the barometric
pressure is 102 kpa? At 35°C: Psat = 5.628, hg = 2565.3Answer: A
89.63 KJ/kg
74.23 KJ/kg
67.34 KJ/kg
53.34 KJ/kg

Air-vapor mixture has an enthalpy of 75 KJ/kg at 30°C. Find the partial pressure of water
vapor. At 30°C: hg = 2556.3 KJ/kgAnswer: D
6.34 kpa
1.34 kpa
4.231 kpa
2.791 kpa
Air in an air conditioner enters at 60% RH with w = 0.021 and leaves at 25°C dry bulb and
16°C wet bulb. If mass of air is 10 kg/s, find the refrigeration capacity in tons of
refrigeration.Answer: B
At 60% RH, 2 = 0.021: h = 87KJ/kg, v = 0.903 m3/kg
At 25°C db, 16°C wb: h = 45 KJ/kg;
134.34 TR
119.45 TR
102.45 TR
98.34 TR

A 160 kw air conditioner has an initial condition of 40°C and 60% RH and leaves at 40°C db
and 60% RH: h = 112 KJ/kg, w = 0.93m3/kg; At 20°C db and 20% RH: h = 26
KJ/kg.Answer: D
3.22 m3/s
2.62 m3/s
2.12 m3/s
1.73 m3/s

Outside air in an air conditioning unit has a mass of 50 kg at 45°C is mixed with recirculated
air at 30 kg at 26°C. Find the temperature after mixing.Answer: D
35.23°C
43.23°C
37.87°C
40.40°C

Twenty kilograms air enters the dryer at 25°C db and 30% RH. Fifteen kg of recirculated air
from the dryer has a partial pressure of water vapor of 10 kpa. Find the humidity ratio after
mixing.Answer: A
At 25°C db, 30% RH: w = 0.0059
0.0325
0.1930
0.0653
0.0842

Sixty kilogram air with enthalpy of 80 KJ/kg is mixed with another x air until the mixture is
100 kg and 70 KJ/kg enthalpy. Find the enthalpy of x air before mixing.Answer: D
23.23 KJ/kg
17.45 KJ/kg
43.75 KJ/kg
55.01 KJ/kg

An 80% efficient cooling tower has water entering at 48% and wet bulb temperature of air
entering at 25°C. Find the water exit temperature of water. Answer: B
16.23°C
29.60°C
32.34°C
23.44°C

Water enters the cooling water at 50°C and leaves at 28°C while the heat rejected to air is
100 KW. Find the capacity of the pump needed to pump the water
1.08 li/s
3.22 li/s
2.34 li/s
4.23 li/s

The evaporative condenser of an ammonia refrigeration plant has a water flow rate of 130
kg/s and enters at natural draft cooling tower at 43°C. The water is cooled to 30°C by air
entering at 27°C db 24°C wet bulb. The air leaves the tower as 90% RH and 32°C db.
Calculate the percent make-up water needed the tower.Answer: B
At 27°C db and 24°C wb: h = 72.5 KJ/kg, w = 0.0178
At 90% RH and 32°C db: h = 102 KJ/kg, w = 0.0275
2.34%
1.79%
3.12%
2.84%

A cooling tower cools water from 39°C to 27°C at the rate of 20 kg/s. Air enters he tower at
22°C at the rate of 36 kg/s. Find the temperature of air leaving the tower.Answer: B
50.71°C
28.86°C
45.23°C
34.45°C

The sensible heat rate is 0.8, which of the following is correct?Answer: A


20 latent and 80 sensible heat
80 latent heat and 20 sensible heat
80 sensible heat and 20 sensible heat and latent heat
20 latent heat and 80 sensible and latent heat

The amount of sensible heating, the sensible heat ratio of 0.80 and a total cooling load of
100:Answer: A
80
20
100
60

A coil has an inlet temperature of 60°F and outlet of 90°F, if the mean temperature of coil is
110°F, find the bypass factor of the coil.Answer: A
0.4
2
1
3
A 4m x 4m x 4m room has a relative humidity of 80%. The pressure in the room is 120 kpa
and temperature is 36°C (Psat = 5.628). What is the mass of vapor in the room (Rv= 0.4615
Kpa-m3/kg-°K).Answer: A
2.03
1.50
0.80
4.80

The evaporative condenser of an ammonia refrigeration plant has a water flowrate of 126 kg
per second and enters a natural draft cooling tower at 40°C. The water is cooled to 29°C by
air entering at 38°C and 24°C wb. The air leaves the tower as saturated 40°C db. Calculate
the make-up water required in kg per hour. Answer: C
Water properties:
At 40°C, hf = 167.48 KJ/kg
At 29°C, hg = 121.43 KJ/kg
Air properties:
At 38°C db and 24°C wb:
h = 72.5 KJ/kg w
= 0.013 KJ/kg
At 40°C db saturated:
h = 166 KJ/kg
w = 0.0488 KJ/kg
8977
8699
8003
8388

PAST BOARD EXAM PROBLEMS

Given: Q1 = 160 li/s, N = 1800 rpm (constant) D1 = 254 mm, D2 = 203.2 mm.Answer: C
130 li/s
180 li/s
128 li/s
145 li/s

Reversed Carnot operates between -8°C and 18°C. Find the COP.Answer: D
4.56
8.34
6.34
10.19

150 grams at 76°C vaporized completely. Find the head added during isothermal process.Answer:
A
348 KJ
643 KJ
456 KJ
123 KJ

A 120 m3/s of air has a dry bulb temperature of 18°C. What is the standard air volume?Answer: A
121.28 m3/s
128.34 kg/s
124.34 kg/s
132.45 kg/s

Given CH4 and 15% excess air. Find the mass of air in kg per mass of kg fuel.Answer: C
17.16 kg/kg
21.23 kg/kg
18.73 kg/kg
34.23 kg/kg

Available enthalpy of turbine is H = 1140.9 KJ/kg, turbine efficiency is 70%, full load capacity is
4,500 Kw. Find the full load steam consumption in kg/Kw-hr.Answer: B
2.34
4.51
3.45
7.45

150 grams of water at 75°C is heated at constant pressure. The water is completely vaporized,
what is heat added?Answer: D
2321.37 KJ
4231.34 KJ
3214.23 KJ
348.21 KJ

An engine is required to produce 4,500 kg of dry steam per hour. The quality of steam in it is
90%. What must be the weight of the wet steam?Answer: D
477.8 kg/hr
528 kg/hr
400 kg/hr
500 kg/hr

The refrigeration system has refrigerating cycle per kilogram of 220 KJ. The heat required to
remove is 630 KJ/hr. Calculate the mass circulated per hour.Answer: D
3.186
8 kg/hr
10 kg/hr
2.86 kg/hr

The pressure and temperature entering the turbine is 18000 Kpa and 380°C. The temperature
leaving the turbine is 20 kpa. The quality of steam is entering the condenser is 90%. What is the
turbine work?Answer: A
833.33 KJ/kg
753 KJ/kg
654.32 KJ/kg
945.67 KJ/kg

Air in a rigid vessel has an initial temperature of 30°C and a final temperature of 80°C. The work
done is:Answer: D
3 KJ
30 KJ
20 KJ
0 KJ

A fuel is represented with C3H8 burned with 50% excess air. If mass of fuel is 0.07 kg, find the
mass of air needed.Answer: C
1.23 kg
1.638 kg
23.4 kg
5.34 kg

A 30 cm x 40 cm reciprocating pump running at 250 rpm discharges 210 li/s of water. Find the
percent slip of the pump.Answer: D
4.23%
8.34%
5.76%
10.87%

A Rankine cycle has an initial mass of liquid flowing at 10 kg at 76%. If final enthalpy is 379.6
KJ, find the change of enthalpy of the system.Answer: A
655.75 KJ
834.34 KJ
763.45 KJ
934.56 KJ

A venturi meter has an initial inlet pressure of 500 Kpa and velocity of 2 m/s. The top has a
diameter of 18 cm and 8 cm diameter. The top is 6 m high and bottom has 4.5 m high. Find the
exit pressure.Answer: D
324.45 kpa
534.23 kpa
452.34 kpa
546.46 kpa

A venturi meter has a pressure head difference of 8 m. The volume of water flowing is 6900
li/min. The inlet diameter is 20 cm. Find the outlet diameter.Answer: A
10.60 cm
14.23 cm
12.34 cm
16.34

An Otto cycle has an inlet condition of 101 kpa and 30°C. The compression ratio is 8 maximum
temperature of the cycle 1500°C. Find the mean effective pressure of the system.Answer: A
578 kpa
735 kpa
653 kpa
823 kpa

An Otto cycle has a compression ratio of 6 and inlet condition of 98 kpa and 15°C. The maximum
temperature of the system is 3000°C. Find the maximum pressure of the system.Answer: B
4334.32 kpa
6682.23 kpa
5634.45 kpa
7049.53 kpa

A turbine has an available enthalpy of 3000 KJ/kg in a Rankine cycle. The pump work has also
20 KJ/kg. For a flow of 2 kg/s, find the system output.Answer: A
5960 kw
6080 kw
643 kw
9834.45 kw
The initial pressure of the system during isometric process is 100 kpa. If the heat during the
process is 10 kw, find the enthalpy change.Answer: C
5 kw
15 kw
10 kw
20 kw

In a Rankine cycle, the steam turbine inlet has a condition of 3126 KJ/kg enthalpy and entropy of
6.68 KJ/kg-K. The quality after expansion is 80% and enthalpy of 2104.3 KJ/kg with hf = 209.33
KJ/kg. Find the efficiency of the cycle.Answer: B
23.45%
35.04%
30.34%
38.44%

Find the degree superheat of steam at 0.89 Mpa and 180°C. At 0.89 Mpa: tsat = 174.90°CAnswer:
C
4.34°C
6.44°C
5.10°C
7.45°C

The available enthalpy of steam of the exit of boiler in a Rankine cycle is 3000 KJ.kg and
enthalpy at the entrance of turbine in 2950 KJ/kg. If mass of steam is 1 kg/s, find the heat loss
between the boiler exit and turbine entrance.Answer: D
10 kw
30 kw
20 kw
50 kw

Find the pressure at the bottom of an 8 m column of water.Answer: D


45.23 kpa
65.23 kpa
53.45 kpa
78.48 kpa
Air is flowing through a 20 cm diameter pipe has a velocity of 5 m/s. If temperature of air is 25
°C kpa pressure, find the mass flow of air in the pipe.Answer: C
0.11 kg/s
0.33 kg/s
0.22 kg/s
0.44 kg/s

An ammonia refrigeration operates between -6°C evaporator temperature and 30°C condensing
temperature. Find the quality after expansion.Answer: C

At -6°C: hf = 172.371 KJ/kg hg = 1455.00 KJ/kg At


30°C hf = 341.769 KJ/kg hg = 1486.14 KJ/kg
11.23%
15.34%
13.21%
16.34%

Air with 20°C wb and 30°C had a relative humidity of:Answer: B


20%
40%
30%
50%

A pump delivers 100 li/s of water at a head of 50 m. Find the water power.Answer: B
29 kw
49 kw
39 kw
59 kw

A steam turbine has an inlet enthalpy of 2800 KJ/kg with velocity of 40 m/s. The exit steam
condition 2650 KJ/kg. Find the exit velocity.Answer: A
549.18 m/s
323.55 m/s
456.34 m/s
658.23 m/s

Flow of water taking over in a pipe having a velocity of 10 m/s. Determine the velocity head of
the water. Answer: B
50.1 m
5.1 m
8.2 m
100 m

The length of pipe is 168 meters. If the pressure drop is 50 Kpa for every 30 meters, what is the
total pressure drop?Answer: C
260 kpa
300 kpa
280 kpa
100 kpa
After checking on the properties and phase descriptions of the quality is found to be x = 0.70.
How many % of the mass is in the liquid phase?Answer: D
70%
100%
0.70%
30%

The diameter of a pipe at the larger end is 0.5 cm and at the smaller end is 0.2 m. The larger and
smaller ends are located 7 meters and 3 meters, respectively from datum line. If at the larger end,
the velocity is 1.2 m/s and the pressure is 5200 KN/m2, compute the pressure at the smaller
end.Answer: C
6501.84 kpa
7815.99 kpa
5211.82 kpa
4800.92 kpa

Convert a vacuum pressure of 110 mm of mercury into absolute pressure if atmospheric pressure
is 10.5 of H2O.Answer: B
3.7 m of H2O
9.1 m of H2O
45.6 m of H2O
99.5 H2O

A tank contains H2O. What is the intensity of pressure at a depth of 6 meters.Answer: C


68 kpa
78 kpa
58.8 kpa
48.7 kpa

Water is flow through a pipe at 15 li/sec. The inlet diameter of pipe is 12 cm and the exit
diameter is 15 cm. If H2O is to be pumped from H2O source and heat loss is 2 kw, determine the
power input to the pump.Answer: A
2.2 kw
1.2 kw
3.2 kw
4.2 kw

Three tons of fish is to be stored at a temperature of -10°C for 24 hours. The product enters the
chiller at a temperature of 8°C. The latent heat above freezing is 0.41 Kcal/kg-C and 0.76
Kcal/kg-C respectively and its latent heat of fusions is 51 kcal/kg. If the freezing temperature of
the product is -2.2°C, determine the product load in Kcal/24 hours.Answer: C
153,000
158,019
168,566
10,000

A boiler has a bursting pressure, Bp of 600 kpa and a factor of safety; FS, of 8 is employed in
design. An engineer, would you advice to have a working pressure, WP, of 500 kpa?
Answer: C
No, WP must be higher than 500 kpa
Yes, since BP is 600 Pa.
No, WP is only 75 kpa at a FS of 8
Yes, to attain better efficiency

The analysis of a product of combustion on dry basis, when C8H18 was burned with atmospheric
air, is as follow: CO2 = 12%, CO = 75%, O2 = 3.10%, N2 = 84.24%. Compute the % theoretical
used for combustion.Answer: C
100%
121%
112%
21%

What is the specific humidity of dry air?Answer: D


150
50
100
0

Water enters the cooling tower at 45°C. The approximate leaving water temperature is 30°C. If
the atmospheric condition is 25 degrees C wet bulb, determine the cooling effectiveness of the
cooling tower?Answer: B
95%
75%
25%
50%

The temperature of a solution is 31°C. Convert the equivalent Fahrenheit reading to absolute
Fahrenheit temperature.Answer: D
560.8°R
520.2°R
575.5°R
547.8°R

One kilogram of air is compressed adiabatically and in steady flow. The compression efficiency
is 80% and work done is 265 KJ/kg. Compute heat.Answer: B
212 KJ/kg
0
1 KJ/kg
331.25 KJ/kg

What is the ton of refrigeration required to cool 15,000 lb of fresh pork from a temperature of
89°F to 32°F in 24 hours? Specific heat above freezing of fresh pork is 0.68 Btu/lb-°F.Answer: C
593.78 tons
410.6 tons
2.02 tons
426.12 tons
A room is 10 ffet long, 12 feet wide and 8 feet high. The inside and outside temperature are 8°F
and 65°F, respectively. Compute the heat transmitted through the walls if the coefficient of heat
transmission is 0.12 Btu/square-°F-hr.Answer: B
4049.28 Btu/hr
2407.78 Btu/hr
996.81 Btu/hr
1208.84 Btu/hr

A refrigerator is maintained at 5°C. Heat is removed from the stored food at a rate of 330 KJ/min.
What is the refrigerator’s coefficient of performance if the necessary power input to the
refrigerator is 3.5 kw?Answer: C
0.45
8.95 kw
1.57 kw
94.28 kw

A building has to be maintained at 18°C at all times. A heat pump is required for this. When
temperature outside the building drops to -6°C, the building losses heat at a rate of 12,000 KJ/hr.
Compute the least power necessary to drive the heat pump.Answer: D
48.15 kw
25.06 kw
0.16 kw
0.275 kw

A refrigerating machine that is classified as a one-ton machine has the capacity to produce a
cooling effect of:Answer: B
500 Kcal/hr
3000Kcal/hr
300 Kcal/hr
50Kcal/hr

Using a psychometric chart, what is the wet bulb temperature of air contained in a room at a
temperature of 308°K, relative humidity of 40% and a pressure of 1 atmosphere?Answer: A
350°K
297°K
294°K
290°K

300 KJ of heat flow by conduction from the outside to the inside cold storage in one hour. If the
temperature and all other conditions are the same, what is the heat flowing through the cold
storage in two hours?
600 KJ
300 KJ
900 KJ
1,200 KJ

Boiling temperature of water is:Answer: C


273°K
32°F
100°C
212°K
A 25 cm x 38 cm, 4-stroke, single acting, running at 600 rpm diesel engine has a brake power of
Find the volumetric rate, m3/kw-hr.Answer: C
4.23
1.23
2.24
3.45

At 1.11°C evaporator temperature the inlet enthalpy is 410.4 KJ/kg and exit enthalpy = 1471.6
KJ/kg. If mass flow rate is 3 kg/min, find the refrigerating effect.Answer: D
23
34
45
53

When dry bulb and wet bulb temperatures are the same, its relative humidity is:Answer: C
0%
50%
100%
75%

Bypass factor of coils is 0.7 for one row. What is the bypass factor if there are 10 coils in a row?
Answer: B
0.029
0.70
700
0.49

A heat pump is used to heat a house during the winter. The house is to be maintained at 21°C at
all times. The house is estimated to be losing heat at the rate of 35,000 KJ/hr when the outside
temperature drops to -5°C. Determine the power required to drive this heat pump unit.Answer: D
3.32 kw
4.34 kw
5.34 kw
0.86 kw

An engine has a power output of 4.5 hp with 74% efficiency. What will be the KW/hp?
Registered to the meter?Answer: A
1
2
3
4

At 30°C and saturated, Uf = 125.79 KJ/kg. If mass is 2 kg, find the internal energy, KJ?Answer:
C
125.79
134.56
251.58
345.67
A rigid vessel has a temperature of 80°C (vf = 0.0010291 m3/kg). If mass is 10 kg, find the
volume of liquid.Answer: B
8.34 li
10.29 li
12.33 li
14.33 li

A diesel engine is 447 kw at sea level, it will operate at 2700 m elevation with correction factor
of 0.71, temperature of 40.1°C correction factor of 0.90. Find the brake power if it will operate at
higher elevation.Answer: A
285.63 kw
342.34 kw
387.23 kw
452.34 kw

A refrigeration system using refrigerant 22 has a compressor of 60 kw. The evaporator entrance =
254.4 KJ/kg, exit = 401.6 KJ/kg. If mass flow of refrigerant is 1.2 kg/s, find the COP of the
system.Answer: B
2
3
4
5

A turbine has a suction pressure of 200 kpa. If head loss is 4 m volume flow is 5 m3/s, find the
water power.Answer: C
456.77 kw
1196.20 kw
803.80 kw
1863.45 kw

A boiler operates at 82.5% efficiency while the mass of steam generated is 885,000lb in 4 hrs.
The enthalpy of steam is 1362 Btu/lb and feed is 249.10 Btu/lb while the fuel used for boiler has
a heating value of 13,850 Btu/lb. Find the mass of fuel needed per day in short tons.Answer: D
10.78
8.34
3.45
258.59

In a 150 m pipe, for every 20 m length there’s a head loss of 40 kpa. What is the total head loss?
Answer: A
300 kpa
200 kpa
150 kpa
400 kpa

A 20 cm x 35 cm diesel engine with 4 cylinders and operating on a four stroke, has a rated power
of 160 kw and is running at 250 rpm. Find the volume displacement per brake power
developed.Answer: D
0.9 m3/min-kw
0.06 m3/min-kw
0.1 m3/min-kw
0.0344 m3/min-kw

What is the BTU equivalent for 1 horsepower?Answer: B


778
2545
746
3.41

A high temperature source at 950°K provides 580 KJ heat to a heat engine. The heat engine
converts 200 KJ net work and rejects the balance to a temperature sink at 298°K. Compare the
thermal efficiency Et of this engine to the thermal efficiency Erev of the Carnot cycle reversible
heat engine.Answer: B
Et = Erev
Et< Erev
Erev< Et
None of these

A refrigerator plant stored 8 metric tons of eggs at a temperature of 15°C. To preserve the eggs by
not spoiling them, they have to be cooled at -8°C. What is the refrigerating capacity of the plant
in tons if it is cooled in 12 hours? The specific heat of the eggs above the below freezing is 0.95
Kcal/kg-°C and 0.4 Kcal/kg-°C respectively. The latent heat of fusion is 68.50 Kcal/kg. The
freezing temperature is -3°C.Answer: B
12.5 tons of ref
19.3 tons of ref
23.1 tons of ref
34.2 tons of ref

A refrigerator plant stored 8 metric tons of eggs at a temperature of 15°C. To preserve the eggs by
not spoiling them, they have to be cooled at -8°C. What is the refrigerating capacity in Kcal? The
specific heat of the eggs above the below freezing is 0.95 Kcal/kg-°C and 0.4 Kcal/kg-°C
respectively. The latent heat of fusion is 68.50 Kcal/kg. The freezing temperature is -
3°C.Answer: B
180,000.6
700,800
14,000.2
142,000

A 90 kg air at 20°C is mixed with 45 kg air at 45°C. Find the temperature after mixing.Answer: C
23.34°C
26.34°C
28.33°C
32.34°C

Air is compressed isothermally at 20°C from 95 kpa to 750 kpa. Find the work done during the
process.Answer: A
173.75 KJ/kg
187.45 KJ/kg
123.44 KJ/kg
109.34 KJ/kg
Rigid container contains 20 grams of saturated vapor hf = hfg = hg = 2729 KJ/kg. Heater passes 0.3
A and 120 volts in 8 min, find the final enthalpy of steam assuming that Qloss = 4 KJ.Answer: C
2890 KJ/kg
2065 KJ/kg
3393 KJ/kg
3084 KJ/kg

A Rankine cycle has a turbine work of 590 KJ/kg and efficiency of 90%, pump work is 2.5 KJ/kg
and efficiency of 76%. If cycle efficiency is 28%, find the heat added to the cycle. Answer: D
2577.60 KJ/kg
2345.19 KJ/kg
2685.67 KJ/kg
1852.40 KJ/kg

A 200 mm x 300 mm single stage air compressor running at 450 rpm has a clearance of 2% of the
stroke and pressure ratio of 8. What is the volumetric flow?Answer: A
237 m3/hr
123 m3/hr
198 m3/hr
383 m3/hr

1 Bo. Hp boils 34.5 lb of steam at 212°F in 24 hours. If you have a 600 Bo. Hp, find the pound of
steam.Answer: B
12,344 lbs
20,700 lbs
16,390 lbs
25,300 lbs

In piping system a bare pipe has a heat loss of 6 kw/m. By insulating the pipe, the heat loss will
reduce to 0.90 kw/m. If total length of pipe is 40 m and operation per month is 600 hrs, how
much energy saved per month because of insulation?Answer: B
2.345 x 108 KJ
1.102 x 106 KJ
4.41 x 108 KJ
1.453 x 106 KJ

A fluid at 120 kpa and has 25°C temperature has a velocity of 180 m/s. Find the stagnation
pressure.Answer: C
125.54 kpa
243.45 kpa
144.34 kpa
110.23 kpa

In a Rankine cycle the quality of steam leaving the turbine is 90% with mass flow of 4800 kg/hr
wet steam. Find the mass of vapor.Answer: D
1295 kg/hr
2841 kg/hr
3089 kg/hr
4320 kg/hr
A turbine has an efficiency of 75% and a kilowatt output of 5500 kw at full load. If the available
energy of the turbine is 1200 KJ/kg, determine the steam flow at the turbine per hour.Answer: B
900 kg/hr
22,000 kg/hr
7,333.33 kg/hr
6.11 kg/hr

In an air compressor, the compression takes place at a constant internal energy and 50 KJ of heat
are rejected to the cooling water every kg air. Find the work required for compression.Answer: B
5 KJ/kg
50 KJ/kg
15 KJ/kg
30 KJ/kg

A certain location has barometric reading of 720 mm Hg and the gravitational acceleration is 9.2
m/s2. If the temperature of mercury is 9°C at which density is 12,850 kg/m3, compute the
atmospheric pressure.Answer: D
94.62 kpa
830 kpa
58.51 kpa
85.12 kpa

One Kmol of C8H18 is burned with 100% air containing 25 Kmol of O2. Determine the air-fuel
ratio for this combustion process.Answer: A
30.38 kg air/kg fuel
25.73 kg air/kg fuel
35.72 kg air/kg fuel
45.07 kg air/kg fuel

In a certain steam plant, the turbine develops 1000 Kw. The heat supplied to the steam boiler is
2800 KJ/kg, the heat rejected by the system to the cooling water in the condenser is 2100 KJ/kg
and the feed pump work is 5 kw. Calculate the steam flow round the cycle in kg/s.Answer: A
1.42
1.24
4.21
2.41

The heat transferred to the engine from a furnace is at a rate of 60 MW, compute the thermal
efficiency of the heat when the rate of waste heat rejection from a nearby reservoir is 40
MW.Answer: C
66.67%
50.0%
33.33%
100%

A steam turbine receives a steam flow of 1.35 kg/s and delivers 500 KW. The heat loss in the
casing is negligible. Find the heat drop across the turbine, the entrance velocity is 60 m/s. the exit
velocity is 36 m/s and the inlet pipe is 3 m above the exhaust pipe.Answer: A
372 KJ/kg
307 KJ/kg
703 KJ/kg
320 KJ/kg

A nozzle which is well insulated has a fluid enthalpy of 3025 KJ/kg and a velocity of 60 m/s at
the inlet. The fluid leaves the nozzle with an enthalpy of 2790 KJ/kg. Determine the fluid rate of
flow in kg/s if the inlet area is 0.10 m2 and specific volume at inlet is 0.19 m3/kg.Answer: C
6.13
3.16
31.6
63.1

What is the volume of a rigid tank that contains 15 kilograms of water at a temperature of 95°C
(P = 84.55 kpa, vf= 0,001040 m3/kg, vg = 1.982 m3/kg), if 10 kilograms of this water liquid and
the remaining is in vapor form?Answer: B
12.892 cubic meters
9.9204 cubic meters
0.0156 cubic meters
5.6549 cubic meters

A single acting reciprocating pump with a diameter of 0.18 m and a stroke of 0.30 m, delivers
water at a rate of 0.18 cubic meter per minute. What is the percentage slip it delivers at 25
revolutions per minute?Answer: A
5.7%
8.4%
10.5%
1.6%

At P2.50 per kw-hr, how much will it cost to maintain a temperature of 96 degrees F for 24 hour
in a box 2 ft square on each side if the outside temperature is 72 degrees F and the overall heat
transfer coefficient for the box is 0.1 Btu/hr-sq-ft degree difference in temperature?Answer: A
P1.01
P2.50
P1.50
P2.01

An air bubbles rises from the bottom of a water storage tank. The temperature at the surface of
the bottom of the tank is 77 degree F rises to the surface where the temperature is 83 degree F.
what is the percent increase in volume of the bubble if the height of water in the tank is 20 feet?
The atmospheric pressure is 14.7 psia. If the height of the water in the tank is 20 feet? The
atmospheric pressure is 14.7 psia.Answer: C
80%
75%
61%
52%

If the available water power of a hydrostatic station is 3,500 MW and head of water is 280 m,
determine the water rate of low.Answer: D
1420 cubic meters
12,500 cubic meters
875 cubic meters
1275.10 cubic meters

Determine the heat transfer to the cooling fluid in a condenser operating under steady flow
condition with steam entering with an enthalpy of 2300 KJ/kg and a velocity of 350 m/s. The
condensate leaves with an enthalpy of 160 KJ/kg and a velocity of 70 m/s.Answer: D
-1992 KJ/kg
-2910 KJ/kg
2910 KJ/kg
-2199 KJ/kg

A single acting reciprocating pump has a diameter of 0.12 meter and a stroke of 0.25 meter.
Compute the theoretical discharge if the pump delivers water at 60 rpm.Answer: C
0.5965 cubic meter/min
0.05965 cubic meter/min
0.1698 cubic meter/min
0.01698 cubic meter/min

A gas turbine unit has a pressure ratio of 6/1 and a maximum cycle temperature of 600°C. The
isentropic efficiencies of the compressor and turbine are 0.82 and 0.85 respectively. Calculate the
power output in KW of an electric generator geared to the turbine when air enters the compressor
at 15°C and a rate of 15 kg/s. For the compression process c p = 1.0005 KJ/kg-K and k = 1.4, for
the expansion process cp = 1.11 KJ/kg-K and k = 1.333.Answer: D
902 KW
920 KW
209 KW
935 KW

A diesel generating unit with a generator efficiency of 85% has an output of 900 kw-hr. If the
mass of fuel is 200 kg, compute the fuel rate of the engine.Answer: B
0.80 kg/kw-hr
0.19 kg/kw-hr
0.40 kg/kw-hr
0.95 kg/kw-hr

A water-tube condenser has a total of 60 tubes. If these two are passes, then compute the number
of tube per pass.Answer: B
60
30
15
2

A tank contains saturated vapor of 35 kilogram at a temperature of 90 degrees C (h g = 2660.15


KJ/kg). Compute enthalpy.Answer: B
61920
93105
82677
47632

A steam ower plant operates on a simple ideal Rankine cycle. The steam enters the turbine at 3
Mpa and 350°C ( h = 3115.3 KJ/kg, s = 6.7428 KJ/kg-K) and is condensed at a pressure of 75 kpa
(tsat = 91.78°C). What is the back work ratio of the plant?Answer: C
Steam data: 75 kpa: sf = 1.213; sfg =6.2434
vf = 1.037 li/kg
hf = 384.39; hfg = 2278.6
0.8%
0.6%
0.43%
0.1%

What is the power produced by a steam power plant which has a pump work of 5 KJ/kg and a
turbine work of 550 KJ/kg? The mass flow rate is 15 kg/sec?Answer: D
7250 KW
9000 KW
9500 KW
8175 KW

A prony brake test was done for an electric motor. The motors turn 1,800 revolutions per
minute with a torque of 3 Newton-meter. Calculate the motor power output in watts.Answer: B
166.66 watts
565.44 watts
5,400 watts
600 watts

What is the specific gravity of an oil which has a Baume reading of 28?Answer: B
0.215
0.886
0.562
0.774

Heat flows inside a cold storage by conduction. The amount of heat flowing is 250 KJ in
one hour. Compute the amount of heat flowing to the storage in 3 hours, assuming all conditions
are the same.Answer: C
83.3 KJ
250 KJ
750 KJ
500 KJ

An electric motor draws a current of 25 amperes from a power source of 120 volts.
Compute the power used.Answer: B
6 kw
2 kw
4.8 kw
0.21 kw
An ideal gas at 50 psig and 110°F is heated in a closed cylindrical vessel. What is the
temperature if the pressure gauge reading is 100 psig?Answer: B
700 degree F
550.5 degree F
532.76 degree F
700 degree F

The compression ratio of an ideal Otto cycle is 6. Under the cold air assumption,
compute the thermal efficiency of the cycle.Answer: B
48.84%
51.16%
58.29%
60.40%

The primary winding of a transformer is connected to a 240 V power source. What is the
secondary voltage if the transformer has a rated ratio of 1 to 2?Answer: B
240
480
12100
400

One foot of H2O is equal to Answer: A


62.4 psf
43 psf
79.3 psf
84.5 psf

The relation between the °R scale and °K scale isAnswer: A


R=1.8K
R=0.55K
R=32K
R=6K

A vapor compression engine operates between temperatures -10°F and -190°F. Find the
coefficient of performance.Answer: D
4.5
3.5
6
2.5

An Otto cycle has an initial temperature of 30°C and pressure of 100 kpa. The maximum
temperature of the cycle is 1500°C with compression ratio of 6. Find the maximum pressure of
the cycle.Answer: D
567 kpa
2938 kpa
3304 kpa
3511 kpa
In an R-12 refrigeration system, the mass flow of refrigerant is 3.5 kg/s and refrigerating
effect of 250 KJ/kg. If COP is 5, gind the heat rejected.Answer: C
1000 kw
1025 kw
1050 kw
1200 kw

112What is the efficiency of the pump having a head of 500 m and mass of water of 50 kg/s if
motor driving has a power of 300 kw.Answer: C
73.45%
78.49%
81.75%
87.35%

The pressure of water is 30 kpa. What is the absolute pressure in meter of water?Answer:
B
10.23 m of H2O
13.38 m of H2O
15.34 m H2O
17.34 m of H2O

A gas turbine has a turbine work of 1000 KJ/kg and compressor work of 500 KJ/kg. If
the mass of circulating is 2 kg/s, find the net work output of the turbine?Answer: C
500 kw
750 kw
1000 kw
1250 kw
A “direct” refrigeration system is one in which
= The refrigerant passes through coils in the evaporator
The capacity of a centrifugal type compressor is controlled by which of the following?
= Regulating the speed or regulating the suction pressur
The coefficient of velocity is equal to the
= Actual velocity divided by the theoretical velocity
In sensible cooling, moisture content.
=increases
The matching of scale model and full-scale results for a fluid dynamic phenomena with a free surface
requires equality of
=Froude number
If the Hs is the total suction head of a pump, Hp is its suction surface pressure, and Hf is its suction
friction head, then calculate the total suction head, Ht.
= Ht = Hs + Ht + Hf
The boiling gas is preferred in refrigerant system:
=F 12
If no gaskets are used in the piping joints of a Freon system, the joints must be
= Ground joints
• Question 1
0 out of 3.3 points
A bell coleman cycle is also known as
Selected Answer: [None Given]
Answers: a.
Reversed Otto cycle
b.
Reversed Joule cycle
c.
Reversed Rankine cycle
d.
Reversed Carnot cycle
• Question 2
0 out of 3.3 points
A device for holding open the suction valve and drawing gas from the
suction manifold and returning it to the suction line without compressing it
is called _____.
Selected Answer: [None Given]
Answers: a.
Cylinder unloader
b.
Relief valve
c.
Discharge line by-pass
d.
Suction line by-pass
• Question 3
0 out of 3.3 points
How much will be removed by one-ton refrigeration unit?
Selected Answer: [None Given]
Answers: a.
144 Btu per min
b.
400 Btu per min
c.
2000 Btu per min
d.
200 Btu per min
• Question 4
0 out of 3.3 points
What is the chemical formula of Ozone?
Selected Answer: [None Given]
Answers: a.
D2O
b.
O2
c.
O1
d.
O3
• Question 5
0 out of 3.3 points
The heat used to change a liquid to a gas or vapor is latent heat of
Selected Answer: [None Given]
Answers: a.
fusion
b.
liquid
c.
vaporization
d.
absorption
• Question 6
0 out of 3.3 points
When air contains all of the water vapor it can hold, it is said to be
Selected Answer: [None Given]
Answers: a.
simulated
b.
moisture
c.
saturated
d.
loaded
• Question 7
0 out of 3.3 points
As far as combustion chamber design is considered, the maximum power
output of a given engine can be increased by:
Selected Answer: [None Given]
Answers: a.
providing small values
b.
increasing combustion chamber volume
c.
increasing combustion time
d.
decreasing combustion chamber volume
• Question 8
0 out of 3.3 points
Absolute zero on the Fahrenheit Scale is equal to:
Selected Answer: [None Given]
Answers: a.
0
b.
-460
c.
-273
d.
-100
• Question 9
0 out of 3.3 points
Which of the following should not be used to clean scale traps?
Selected Answer: [None Given]
Answers: a.
Cotton waste
b.
Diesel oil
c.
Compressed air
d.
Kerosene
• Question 10
0 out of 3.3 points
Which of the following industries have the highest consumption of water for
processing?
Selected Answer: [None Given]
Answers: a.
Automobile industry
b.
Foundry
c.
Aluminum industry
d.
Paper mill
• Question 11
0 out of 3.3 points
The agent used in an indirect reefer system is:
Selected Answer: [None Given]
Answers: a.
Calcium chloride or sodium chloride
b.
Calcium chloride
c.
Sodium chloride
d.
Potassium chloride
• Question 12
0 out of 3.3 points
According to ASME code, safety valves on low pressure boilers should be
tested by hand at least
Selected Answer: [None Given]
Answers: a.
Once a shift
b.
Once a year
c.
Twice a year
d.
Once a month
• Question 13
0 out of 3.3 points
Venturi meters, pitot static gauges, orifice meters, flow nozzles, and
differential manometers all depend upon the relationship between:
Selected Answer: [None Given]
Answers: a.
Flow velocity and friction
b.
Pressure and mass flow
c.
Friction and pressure
d.
Flow velocity and pressure
• Question 14
0 out of 3.3 points
Flows through multi-loop systems may be computed by:
Selected Answer: [None Given]
Answers: a.
Any closed- form solution of simultaneous equations
b.
The hardy-cross method
c.
Trial and error
d.
All of the other choices
• Question 15
0 out of 3.3 points
The oil separator is located between the:
Selected Answer: [None Given]
Answers: a.
Condenser and dehydrator
b.
Solenoid valve and thermal expansion valve
c.
Evaporator and compressor
d.
Compressor and condenser
• Question 16
0 out of 3.3 points
A bull’s eye in a full liquid line will appear _________.
Selected Answer: [None Given]
Answers: a.
Latent
b.
Clear
c.
Cloudy
d.
Cloudy or latent
• Question 17
0 out of 3.3 points
The reactor plate is essential to the operation of a _______.
Selected Answer: [None Given]
Answers: a.
Thermostatic valve
b.
Solenoid bulb
c.
None of the other choices
d.
Halide torch
• Question 18
0 out of 3.3 points
A single trunk piston-type compressor is undesirable for a Freon unit
because the:
Selected Answer: [None Given]
Answers: a.
Lubricant mixes with the refrigerant
b.
Lubricant temperature becomes excessive
c.
Refrigerant becomes superheated
d.
Refrigerant reduces the crankcase pressure
• Question 19
0 out of 3.3 points
There shall be no foundation bolts less than ___________.
Selected Answer: [None Given]
Answers: a.
20 mm in diameter
b.
16 mm in diameter
c.
18 mm in diameter
d.
12 mm in diameter
• Question 20
0 out of 3.3 points
Which of the ff area of work requires lowest noise level?
Selected Answer: [None Given]
Answers: a.
Gymnasiums
b.
Kitchens
c.
Lecture halls
d.
Library
• Question 21
0 out of 3.3 points
A water conservation device which employs both air and water is condensed
a condenser and a cooling tower combined into one is called:
Selected Answer: [None Given]
Answers: a.
shell and tube condenser
b.
water-cooled condenser
c.
air-cooled condenser
d.
evaporative condenser
• Question 22
0 out of 3.3 points
A double-pipe condenser has __________.
Selected [None Given]
Answer:
Answers: a.
None of the other choices
b.
Two piping system side by side, one with cooling water and one
with refrigerant
c.
A small pipe inside a larger pipe, and the cooling water passing
through the small pipe and the refrigerant through large pipe
d.
Two pipes for cooling water and one for refrigerant
• Question 23
0 out of 3.3 points
The ff are examples of indirect (secondary) measurements to measure flow
rates using obstruction meters except:
Selected Answer: [None Given]
Answers: a.
Direction-sensing probes
b.
Pitot static meters
c.
Weight and mass scales
d.
Static pressure probes
• Question 24
0 out of 3.3 points
Which of the following refrigerant is added sometimes to other refrigerant to
improve oil circulation?
Selected Answer: [None Given]
Answers: a.
R – 117
b.
R – 777
c.
R – 270
d.
R – 170 (Ethane)
• Question 25
0 out of 3.3 points
The scale trap is located between the:
Selected Answer: [None Given]
Answers: a.
Evaporator coils and compressor
b.
Compressor and oil separator
c.
Expansion valve and evaporator coils
d.
King (liquid) valve and expansion valve
• Question 26
0 out of 3.3 points
The suction pressure in a Freon system should be:
Selected [None Given]
Answer:
Answers: a.
The pressure which corresponds with a temperature about 20F
below the temperature of the icebox.
b.
None of the other choices
c.
The pressure which corresponds with a temperature about 20F
above the temperature of the icebox.
d.
The pressure which corresponds with a temperature equal to
the temperature of the icebox.
• Question 27
0 out of 3.3 points
Which of the following draft rely on the stack effect to draw off combustion
gases?
Selected Answer: [None Given]
Answers: a.
Balanced draft
b.
Induced draft
c.
Natural draft
d.
Forced draft
• Question 28
0 out of 3.3 points
Faulty F – 12 compressor valves will be indicated by :
Selected Answer: [None Given]
Answers: a.
Compressor running continuously
b.
Gradual or sudden decrease in capacity
c.
All of the other choices
d.
Low head pressure – high suction pressure
• Question 29
0 out of 3.3 points
If the compressor short-cycles on the low-pressure cutout, the trouble might
be:
Selected Answer: [None Given]
Answers: a.
Dirty traps and strainers
b.
Too much frost on the evaporator coils
c.
Any of the other choices
d.
Lack of refrigerant
• Question 30
0 out of 3.3 points
The cooling-water side of the condenser should be opened for inspection
every:
Selected Answer: [None Given]
Answers: a.
Two years
b.
Three months
c.
Six months
d.
Year
Tuesday, March 17, 2020 3:40:20 PM PST
• Question 1
0 out of 3.3 points
Theoretically, what is the effect of the compressor clearance on horsepower?
Selected Answer: [None Given]
Answers: a.
varies in direct proportion
b.
there is no effect
c.
decrease power
d.
increase power
• Question 2
0 out of 3.3 points
Large leaks in a Freon system cannot always be detected with halide torch
because it changes color with slightest amount of Freon present. A large leak
can be detected easier by applying ____________.
Selected [None Given]
Answer:
Answers: a.
A thin layer of mineral oil to all joints and watch for bubbles
b.
Sheets of litmus paper to all joints and watch for color change
c.
A lighted candle at the joints and watch for leaky spots blowing
candle flame
d.
A soapsuds solution, mixed with a little glycerin to hold the
solution together, and watch for bubbles
• Question 3
0 out of 3.3 points
Antifreeze chemicals are:
Selected Answer: [None Given]
Answers: a.
Those that lower down the freezing points of liquids
b.
Those that are added to refrigerants for better performance
c.
Same as refrigerants
d.
Those that do not freeze at all
• Question 4
0 out of 3.3 points
Which of the following devices used to measure the discharge of outflow
from a vessel?
Selected Answer: [None Given]
Answers: a.
Pump head
b.
Obstruction meter
c.
Orifice
d.
Pitot tube
• Question 5
0 out of 3.3 points
The moisture in a refrigeration system can be removed with help of which of
the following?
Selected Answer: [None Given]
Answers: a.
Driers
b.
Coolers
c.
Dehumidifies
d.
Evaporators
• Question 6
0 out of 3.3 points
When the dry bulb and the wet bulb temperature are identical, the air is said
to be:
Selected Answer: [None Given]
Answers: a.
Dehumidified
b.
Saturates
c.
Compressed
d.
Humidified
• Question 7
0 out of 3.3 points
What is the chemical formula of methylene chloride?
Selected Answer: [None Given]
Answers: a.
CBRF
b.
CHCLF
c.
CCLF
d.
CH2CL3
• Question 8
0 out of 3.3 points
Which of the following vital components of the refrigeration system where
both temperature and pressure are increased?
Selected Answer: [None Given]
Answers: a.
Compressor
b.
Condenser
c.
Evaporator
d.
Compressor and evaporator
• Question 9
0 out of 3.3 points
The static pressure drop due to friction through the boiler and stack.
Selected Answer: [None Given]
Answers: a.
Draft loss
b.
Stack effect
c.
Fan boost
d.
Available draft
• Question 10
0 out of 3.3 points
The secondary refrigerant used in milk chilling plants is generally
Selected Answer: [None Given]
Answers: a.
Brine
b.
Ammonia
c.
Sodium silicate
d.
Glycol
• Question 11
0 out of 3.3 points
Too low suction pressure could be caused by:
Selected Answer: [None Given]
Answers: a.
Shortage of refrigerant gas
b.
Too much oil in the system
c.
Dirty scale traps
d.
Any of the other choices
• Question 12
0 out of 3.3 points
In a cooling tower, the water is cooled mainly by:
Selected Answer: [None Given]
Answers: a.
conduction
b.
Evaporation
c.
convection
d.
Condensation
• Question 13
0 out of 3.3 points
The drop in a solvent’s vapor pressure and the increase in mole fraction as solute is
added is describe by:
Selected Answer: [None Given]
Answers: a.
Dalton’s law
b.
Raoult’s law
c.
Henry’s law
d.
Boyle’s law
• Question 14
0 out of 3.3 points
The main purpose of a subcooler in a refrigerating system especially a two
stage system is to:
Selected [None Given]
Answer:
Answers: a.
Increase the heat rejection per ton and avoid system shutdown
b.
Reduce the total power requirements and return oil to the
compressor
c.
Improve the flow of evaporator gas per ton and increase the
temperature
d.
Reduce the total power requirements and heat rejection to the
second stage
• Question 15
0 out of 3.3 points
In series pipe systems, all of the ff parameters vary from section to section
except
Selected Answer: [None Given]
Answers: a.
Mass flow
b.
Pressure drop
c.
Head loss
d.
Friction loss
• Question 16
0 out of 3.3 points
The reactor plate is essential to the operation of a _______.
Selected Answer: [None Given]
Answers: a.
None of the other choices
b.
Solenoid bulb
c.
Halide torch
d.
Thermostatic valve
• Question 17
0 out of 3.3 points
When vapor compression takes place on one side of the piston and one
during one revolution of the crankshaft, then the compressor is:
Selected Answer: [None Given]
Answers: a.
two-revolution
b.
double-acting
c.
single-acting
d.
four-cycle
• Question 18
0 out of 3.3 points
Which of the following is used in measuring the density of a brine solution?
Selected Answer: [None Given]
Answers: a.
A hydrometer
b.
Litmus paper or a chemical test
c.
A chemical test
d.
Litmus paper
• Question 19
0 out of 3.3 points
A bull’s eye in a full liquid line will appear _________.
Selected Answer: [None Given]
Answers: a.
Cloudy
b.
Clear
c.
Cloudy or latent
d.
Latent
• Question 20
0 out of 3.3 points
The purpose of the oil trap is:
Selected Answer: [None Given]
Answers: a.
To remove oil from the refrigerating gas
b.
None of above
c.
To remove oil from the charging tank
d.
To add oil to the compressor
• Question 21
0 out of 3.3 points
The amount of CO can be determined by the color of Palladium Chloride. An
amount of 30 ppm to 70 ppm will cause:
Selected Answer: [None Given]
Answers: a.
Black Color
b.
Red Color
c.
Grey Color
d.
Slight Darkening
• Question 22
0 out of 3.3 points
A method of cooling which uses water as refrigerant. Pressure on the water
surface is reduced to lower its boiling temperature.
Selected Answer: [None Given]
Answers: a.
Evaporative Cooling
b.
Pressurized Cooling
c.
Steam Jet Cooling
d.
Vortex Tube Cooling
• Question 23
0 out of 3.3 points
Closing the solenoid valve stops the compressor through the :
Selected Answer: [None Given]
Answers: a.
High water – pressure cutout switch
b.
Low – pressure cutout switch
c.
Lower water – pressure cutout switch
d.
High – pressure cutout switch
• Question 24
0 out of 3.3 points
The greatest decrease in the temperature of the refrigerant is at the ______.
Selected Answer: [None Given]
Answers: a.
Receiver
b.
Condenser
c.
Expansion valve
d.
Evaporator
• Question 25
0 out of 3.3 points
Compute the wall gain load of a load storage room when A is the outside
surface area of the wall , D is the temperature differential across the wall,
and U is the overall coefficient of heat transmission.
Selected Answer: [None Given]
Answers: a.
AUD
b.
AU / D
c.
A / UD
d.
D / AU
• Question 26
0 out of 3.3 points
An insulator with superior tensile strength but low heat resistance
Selected Answer: [None Given]
Answers: a.
Fiberglass
b.
Plastic
c.
Rubber
d.
Asbestos
• Question 27
0 out of 3.3 points
When a hot soup was served in a cup during dinner, an Engineer was o eager
to drink it. Since it was hot, he added ice cubes of ice to cool the soup stirred
it. He noticed that dew starts to form on the outermost surface of the cup.
He wanted to check the temperature of the outermost surface of the cup.
What is this temperature equal to?
Selected Answer: [None Given]
Answers: a.
equal to air’s dew point temperature
b.
superheated temperature
c.
equal to zero
d.
standard temperature
• Question 28
0 out of 3.3 points
The highest temperature in vapor compression cycle is producing during:
Selected Answer: [None Given]
Answers: a.
Expansion valve
b.
Compressor discharge
c.
Evaporator
d.
Condenser discharge
• Question 29
0 out of 3.3 points
The weight of steel bar reinforcement show be how many time the weight of
the foundation?
Selected Answer: [None Given]
Answers: a.
½% to 1%
b.
3% to 5%
c.
1% to 1.5%
d.
3.2% to 4.2%
• Question 30
0 out of 3.3 points
Very large pipe diameter means a thicker wall and high cost. It also means
Selected Answer: [None Given]
Answers: a.
constant temperature
b.
low pressure drop
c.
zero pressure drop
d.
high pressure drop
Tuesday, March 17, 2020 3:44:51 PM PST
• Question 1
0 out of 3.3 points
Which of the following type valves are not found on a Freon – 12 system?
Selected Answer: [None Given]
Answers: a.
Diaphragm
b.
Single packed
c.
Bellows
d.
Duplex
• Question 2
0 out of 3.3 points
Which of the following is considered as comfort condition in air
conditioning?
Selected Answer: [None Given]
Answers: a.
30 C DBT, 80% RH
b.
40 C DBT, 80% RH
c.
20 C DBT, 60% RH
d.
30 C DBT, 60% RH
• Question 3
0 out of 3.3 points
All of the ff are common types of chemical reaction except
Selected Answer: [None Given]
Answers: a.
Fission
b.
Double replacement
c.
Decomposition
d.
Direct combination
• Question 4
0 out of 3.3 points
The boiling gas is preferred in refrigerant system:
Selected Answer: [None Given]
Answers: a.
F 11
b.
NH3
c.
CO2
d.
F 12
• Question 5
0 out of 3.3 points
When coil surface is below the dew point temperature of the air.
Selected Answer: [None Given]
Answers: a.
air is dehumidified
b.
moisture will condense out of the air
c.
vapor will evaporate through the air
d.
air is humidified and saturated
• Question 6
0 out of 3.3 points
Absolute zero is
Selected Answer: [None Given]
Answers: a.
144 degrees below zero on the Fahrenheit scale
b.
The same as zero on the Fahrenheit scale
c.
460 degrees below zero on the Fahrenheit scale
d.
970 degrees below zero on the Fahrenheit scale
• Question 7
0 out of 3.3 points
Which of the following troubles commonly occurs inside an air conditioning
unit?
Selected Answer: [None Given]
Answers: a.
Stuck Compressor
b.
Clogged Refrigerant Circuit
c.
Short Circuit, open circuit, grounded motor winding
d.
All of these
• Question 8
0 out of 3.3 points
As far as combustion chamber design is considered, the maximum power
output of a given engine can be increased by:
Selected Answer: [None Given]
Answers: a.
increasing combustion chamber volume
b.
providing small values
c.
increasing combustion time
d.
decreasing combustion chamber volume
• Question 9
0 out of 3.3 points
The solenoid valve controls which of the following?
Selected Answer: [None Given]
Answers: a.
Amount of refrigerant entering the evaporator
b.
Amount of circulating water to the condenser
c.
Pressure in the evaporator coils
d.
Flow of refrigerant to the expansion valve
• Question 10
0 out of 3.3 points
Concrete foundation should have steel bar reinforcement placed vertically
and horizontally to avoid ___________.
Selected Answer: [None Given]
Answers: a.
Thermal cracking
b.
Melting
c.
Breaking
d.
Vibration
• Question 11
0 out of 3.3 points
The term used when the static pressure is equal to the atmospheric
pressure.
Selected Answer: [None Given]
Answers: a.
Natural draft
b.
Balanced draft
c.
Induced draft
d.
Forced draft
• Question 12
0 out of 3.3 points
A type of refrigerant that will not damage the ozone layer.
Selected Answer: [None Given]
Answers: a.
R-12
b.
R-22
c.
Hydrochlorofluorocarbons (HCHF’s)
d.
Hydrofluorocarbons (HCF’s)
• Question 13
0 out of 3.3 points
The maximum continuous power available from a hydro -electric plant under the
most adverse hydraulic conditions is called
Selected Answer: [None Given]
Answers: a.
Base power
b.
Secondary power
c.
Firm power
d.
Primary power
• Question 14
0 out of 3.3 points
The size of a reciprocating pump is stamped on the builder’s plate 3” x 4’ x
6”. The diameter of the liquid cylinder is:
Selected Answer: [None Given]
Answers: a.
7”
b.
6”
c.
5”
d.
4”
• Question 15
0 out of 3.3 points
The normal cut-out setting of a window unit thermostat is between:
Selected Answer: [None Given]
Answers: a.
16 C to 21 C
b.
13C to 16 C
c.
17C to 22C
d.
10C to 15C
• Question 16
0 out of 3.3 points
The coefficient of contraction is the ratio of the :
Selected Answer: [None Given]
Answers: a.
Effective head to the actual head
b.
Area of vena contracta to the orifice area
c.
Actual velocity to the theoretical velocity
d.
Actual discharge to the theoretical discharge
• Question 17
0 out of 3.3 points
The main purpose of a subcooler in a refrigerating system especially a two
stage system is to:
Selected [None Given]
Answer:
Answers: a.
Reduce the total power requirements and return oil to the
compressor
b.
Reduce the total power requirements and heat rejection to the
second stage
c.
Increase the heat rejection per ton and avoid system shutdown
d.
Improve the flow of evaporator gas per ton and increase the
temperature
• Question 18
0 out of 3.3 points
If a compressor runs continuously, the cause might be a:
Selected Answer: [None Given]
Answers: a.
Defective thermal bulb
b.
Stuck low-pressure switch
c.
Clogged scale trap
d.
Stuck high-pressure switch
• Question 19
0 out of 3.3 points
During sensible heating, the humidity remains constant but the relative
humidity.
Selected Answer: [None Given]
Answers: a.
remains constant
b.
zero
c.
decreases
d.
increases
• Question 20
0 out of 3.3 points
A fibrous silicate mineral material that is inert, strong and incombustible
Selected Answer: [None Given]
Answers: a.
Fiberglass
b.
Asbestos
c.
Plastic
d.
Rubber
• Question 21
0 out of 3.3 points
To help a person who had been overexposed to ammonia gas, one would
Selected Answer: [None Given]
Answers: a.
Wrap in warm blankets
b.
Apply artificial respiration
c.
Apply cold compresses
d.
Douse with cold water
• Question 22
0 out of 3.3 points
The operation that produces highest noise level is:
Selected Answer: [None Given]
Answers: a.
Wielding
b.
Machining
c.
Pressing
d.
Riveting
• Question 23
0 out of 3.3 points
A good refrigerant should be
Selected Answer: [None Given]
Answers: a.
Non-poisonous
b.
All of the other choices
c.
Non-inflammable
d.
Non-explosive
• Question 24
0 out of 3.3 points
The coefficient of velocity is approximately
Selected Answer: [None Given]
Answers: a.
0.30 to 0.50
b.
0.40 to 0.60
c.
0.20 to 0.40
d.
0.50 to 0.70
• Question 25
0 out of 3.3 points
Flows through multi-loop systems may be computed by:
Selected Answer: [None Given]
Answers: a.
Trial and error
b.
All of the other choices
c.
The hardy-cross method
d.
Any closed- form solution of simultaneous equations
• Question 26
0 out of 3.3 points
The reduction of nuclear radiation intensity (called attenuation) is accomplished
by:
Selected Answer: [None Given]
Answers: a.
Shielding
b.
Antimatter
c.
Matter
d.
Neurons
• Question 27
0 out of 3.3 points
What do you call the device that is used as a low- pressure control and high-
pressure cutout on a compressor?
Selected Answer: [None Given]
Answers: a.
Cutout
b.
Cutout switch
c.
Controller switch
d.
Pressure controller
• Question 28
0 out of 3.3 points
The temperature below which water vapor in the air will start to condense:
Selected Answer: [None Given]
Answers: a.
Dew point Temperature
b.
Dry bulb Temperature
c.
Wet bulb Temperature
d.
Condensing Temperature
• Question 29
0 out of 3.3 points
A double-pipe condenser has __________.
Selected [None Given]
Answer:
Answers: a.
Two pipes for cooling water and one for refrigerant
b.
Two piping system side by side, one with cooling water and one
with refrigerant
c.
A small pipe inside a larger pipe, and the cooling water passing
through the small pipe and the refrigerant through large pipe
d.
None of the other choices
• Question 30
0 out of 3.3 points
Which of the ff is not pollutant from a sulfuric acid plants?
Selected Answer: [None Given]
Answers: a.
Hydrogen sulfide
b.
Acid moist
c.
Sulfur dioxide
d.
Sulfur trioxide
Tuesday, March 17, 2020 3:46:51 PM PST
• Question 1
0 out of 3.3 points
How should the window type air conditioning unit be placed?
Selected Answer: [None Given]
Answers: a.
Slant toward the outside of the home
b.
Slant at approximately 15 from the horizontal
c.
Level
d.
Slant toward the inside of the home
• Question 2
0 out of 3.3 points
Absolute zero on the Fahrenheit Scale is equal to:
Selected Answer: [None Given]
Answers: a.
-100
b.
-460
c.
-273
d.
0
• Question 3
0 out of 3.3 points
In turbine installation, the air is removed from condenser by
Selected Answer: [None Given]
Answers: a.
Air ejector
b.
Air injector
c.
Air pump
d.
Air jet
• Question 4
0 out of 3.3 points
Fans that are used to draw combustion products through the furnace bed,
stack, and pollution control system by injecting air into the stack after
combustion
Selected Answer: [None Given]
Answers: a.
Natural draft
b.
Induced draft
c.
Balanced draft
d.
Forced draft
• Question 5
0 out of 3.3 points
Swimming pool water needs
Selected Answer: [None Given]
Answers: a.
super-chlorination
b.
pre-chlorination
c.
dual-chlorination
d.
de-chlorination
• Question 6
0 out of 3.3 points
Which of the following is the most appropriate definition of latent heat?
Selected Answer: [None Given]
Answers: a.
Heat added to change temperature of a substance
b.
Heat removed to change temperature of substance
c.
Heat removed to melt ice
d.
Heat added to change the state of a substance
• Question 7
0 out of 3.3 points
If PV is the power required for a vapor compression refrigeration system ,
then what is the power required for an air refrigeration system, assuming
that they have the same capacity?
Selected Answer: [None Given]
Answers: a.
2V
b.
PV/10
c.
1/PV
d.
5PV
• Question 8
0 out of 3.3 points
If the expansion valve capillary tube is pinched, which of the following must
be replaced?
Selected Answer: [None Given]
Answers: a.
Tube
b.
Diaphragm
c.
Bulb
d.
All of the other choices (called the power element unit)
• Question 9
0 out of 3.3 points
What is the relative humidity when the dew point and dry bulb temperature
are equal?
Selected Answer: [None Given]
Answers: a.
100%
b.
50%
c.
110%
d.
0%
• Question 10
0 out of 3.3 points
Which of the following troubles commonly occurs inside an air conditioning
unit?
Selected Answer: [None Given]
Answers: a.
Stuck Compressor
b.
Clogged Refrigerant Circuit
c.
Short Circuit, open circuit, grounded motor winding
d.
All of these
• Question 11
0 out of 3.3 points
If there is a temperature drop between the temperature corresponding to
the pressure at the compressor discharge and the condenser outlet, one
should :
Selected Answer: [None Given]
Answers: a.
None of the other choices
b.
Purge the system
c.
Increase the amount of circulating water
d.
Decrease the amount of circulating water
• Question 12
0 out of 3.3 points
Very large pipe diameter means a thicker wall and high cost. It also means
Selected Answer: [None Given]
Answers: a.
low pressure drop
b.
constant temperature
c.
high pressure drop
d.
zero pressure drop
• Question 13
0 out of 3.3 points
What adding oil to a freon system, one must be sure that
Selected Answer: [None Given]
Answers: a.
The condenser is shut down
b.
All air is removed from the pump and fitting
c.
The discharge pressure is not too high
d.
There is not too high a suction pressure
• Question 14
0 out of 3.3 points
A precooler is sometimes installed between the ________.
Selected Answer: [None Given]
Answers: a.
Evaporator and compressor
b.
Condenser and expansion valve
c.
Compressor and condenser
d.
Expansion valve and evaporator
• Question 15
0 out of 3.3 points
The ratio of the sum individual maximum demands of the system to the maximum
demand of the whole system is:
Selected Answer: [None Given]
Answers: a.
Utilization factor
b.
Diversity factor
c.
Demand factor
d.
Power factor
• Question 16
0 out of 3.3 points
The principle of mechanical refrigeration is
Selected [None Given]
Answer:
Answers: a.
The absorption of temperature under heat, pressure,
compression and expansion
b.
The conversion of a liquid to a gas
c.
The absorption of heat under temperature, compression,
pressure and expansion
d.
The compression of a liquid under temperature and expansion
• Question 17
0 out of 3.3 points
The high-pressure cutout switch :
Selected [None Given]
Answer:
Answers: a.
Recirculates the refrigerant through the compressor under
emergency conditions
b.
Stops compressor when head pressure is too high
c.
Stops flow of water to condenser when there is no refrigerant
passing through
d.
Stops the flow of refrigerant when condenser pressure is too
high
• Question 18
0 out of 3.3 points
Before securing a compressor to do maintenance work on it, be sure to:
Selected Answer: [None Given]
Answers: a.
Pump down the system
b.
Have spare parts ready
c.
Have spare parts ready and pump down the system
d.
Purge the system
• Question 19
0 out of 3.3 points
The relief valve on a CO2 machine is located:
Selected [None Given]
Answer:
Answers: a.
On the discharge pipe between the compressor and discharge
valve
b.
On the discharge pipe from the condenser
c.
In the compressor head
d.
Next to the king valve
• Question 20
0 out of 3.3 points
Excess fuel oil in the fuel oil system returns to the:
Selected Answer: [None Given]
Answers: a.
Burner
b.
Suction line
c.
Combustion chamber
d.
Fuel oil tank
• Question 21
0 out of 3.3 points
What does fossil-fuel fired power plants release, which in turn produces the
key ingredient in acid rain?
Selected Answer: [None Given]
Answers: a.
Nitrogen
b.
Sulfur Emission
c.
Carbon Dioxide
d.
Carbon Monoxide
• Question 22
0 out of 3.3 points
Water in vapor form remains a vapor as long as temperature is what
relations to the dew point temperature.
Selected Answer: [None Given]
Answers: a.
below
b.
almost
c.
equals
d.
above
• Question 23
0 out of 3.3 points
Which do you think is the effect of “subcooling”?
Selected Answer: [None Given]
Answers: a.
It increase the horsepower per ton of refrigeration
b.
It increases the compression ratio
c.
It reduces the horsepower per ton of refrigeration
d.
It causes flooding back to the compressor
• Question 24
0 out of 3.3 points
Which of the following devices used to measure the discharge of outflow
from a vessel?
Selected Answer: [None Given]
Answers: a.
Orifice
b.
Pitot tube
c.
Pump head
d.
Obstruction meter
• Question 25
0 out of 3.3 points
The expansion valve does not seem to be operating properly. There is high
superheat. Test by listening to the sound of Freon flooding through the
tubes and _______.
Selected Answer: [None Given]
Answers: a.
Any of the other choices
b.
Place bulb in hot water
c.
Warm bulb with hand
d.
Place bulb in cold water
• Question 26
0 out of 3.3 points
Closing the solenoid valve will stop the compressor through the ___________.
Selected Answer: [None Given]
Answers: a.
Low water-pressure cutout switch
b.
Bypass relief valve
c.
Low-pressure cutout switch
d.
High-pressure cutout switch
• Question 27
0 out of 3.3 points
How is one-ton plant described?
Selected Answer: [None Given]
Answers: a.
Melt one ton of ice in 24 hours
b.
Remove one ton heat from the reefer box
c.
Make one ton of ice in 24 hours
d.
Remove the heat required to melt one ton of ice in 24 hours
• Question 28
0 out of 3.3 points
An impulse turbine are used for:
Selected Answer: [None Given]
Answers: a.
Very low head
b.
High head
c.
Low head
d.
Medium head
• Question 29
0 out of 3.3 points
What must be the value of the available Net Positive Suction Head (NPHS) of
a centrifugal pump compared to its require NPHS to avoid losing priming?
Selected Answer: [None Given]
Answers: a.
available NPHS lesser than required NPHS
b.
available NPHS greater than required NPHS
c.
available NPHS constant at all times
d.
available NPHS equal then required NPHS
• Question 30
0 out of 3.3 points
The design of an air supply duct of an air conditioning system
Selected Answer: [None Given]
Answers: a.
adds moisture to the air
b.
affects the distribution
c.
lowers the temperature of the air
d.
does not affect the moisture of the air
Tuesday, March 17, 2020 3:48:50 PM PST
• Question 1
0 out of 3.3 points
All of the ff factors affect rates of reaction except
Selected Answer: [None Given]
Answers: a.
Temperature
b.
Concentration
c.
Pressure
d.
Exposed surface area
• Question 2
0 out of 3.3 points
The total pressure supplied by the fan at maximum operating conditions
Selected Answer: [None Given]
Answers: a.
Stack effect
b.
Draft loss
c.
Available draft
d.
Net rating or fan boost
• Question 3
0 out of 3.3 points
If there were a 15 F to 20 F temperature differential between the
temperature corresponding to the pressure at the compressor discharge
and the temperature at the condenser outlet, it would probably indicate the
need for __________.
Selected Answer: [None Given]
Answers: a.
More refrigerant
b.
Less circulating
c.
More circulating water
d.
Purging the system
• Question 4
0 out of 3.3 points
The locus of elevation to which water will rise in the piezometer tube is
termed:
Selected Answer: [None Given]
Answers: a.
Energy gradient
b.
Hydraulic gradient
c.
Critical path
d.
Friction head
• Question 5
0 out of 3.3 points
It is used deliver concentrated airstreams into a room. Many have one-way
or two-way adjustable air stream deflectors.
Selected Answer: [None Given]
Answers: a.
Diffuser
b.
Grille
c.
Damper
d.
Register
• Question 6
0 out of 3.3 points
The purge valve is located:
Selected Answer: [None Given]
Answers: a.
In the highest part of the system
b.
On the receiver discharge
c.
On the evaporator coils
d.
In the lowest part of the system
• Question 7
0 out of 3.3 points
A good refrigerant should be
Selected Answer: [None Given]
Answers: a.
Non-poisonous
b.
All of the other choices
c.
Non-explosive
d.
Non-inflammable
• Question 8
0 out of 3.3 points
The motor condition of an air conditioning unit can be checked with:
Selected Answer: [None Given]
Answers: a.
The continuity of light or with an ohmmeter
b.
The flow of refrigerant in the receiver
c.
The discharge pressure in the compressor
d.
All of these
• Question 9
0 out of 3.3 points
Which of the following secondary refrigeration generally used in milk chilling
plants?
Selected Answer: [None Given]
Answers: a.
Glycol
b.
Brine
c.
Ammonia solution
d.
Sodium silicate
• Question 10
0 out of 3.3 points
The absorption of water by a dessicant often demonstrates:
Selected Answer: [None Given]
Answers: a.
Heat of fusion
b.
Endothermic heat of solution
c.
Exothermic heat of solution
d.
Heat of vaporization
• Question 11
0 out of 3.3 points
Which is not commonly used to cool and dehumidify equipment?
Selected Answer: [None Given]
Answers: a.
activated alumina
b.
silica gel
c.
sodium zeolite
d.
calcium chloride
• Question 12
0 out of 3.3 points
The relationship of water vapor in the air at dew point temperature to the
amount that would be in the air if the air were saturated at the dry bulb
temperature is:
Selected Answer: [None Given]
Answers: a.
Percentage humidity
b.
partial pressure actual at dew point
c.
Relative humidity
d.
Partial pressure of water
• Question 13
0 out of 3.3 points
An odorless refrigerant whose boiling point varies over a wide range of
temperatures.
Selected Answer: [None Given]
Answers: a.
Ammonia
b.
Freon 12
c.
Freon11
d.
Freon 22
• Question 14
0 out of 3.3 points
The ratio of the sum individual maximum demands of the system to the maximum
demand of the whole system is:
Selected Answer: [None Given]
Answers: a.
Utilization factor
b.
Diversity factor
c.
Demand factor
d.
Power factor
• Question 15
0 out of 3.3 points
Dry ice is
Selected Answer: [None Given]
Answers: a.
Solidified form of carbon dioxide
b.
Free from dissolved air or gases
c.
Does not contain impurities
d.
Free from water
• Question 16
0 out of 3.3 points
R-22 is
Selected Answer: [None Given]
Answers: a.
Trichlorodifluoromethane
b.
Dichlorodifluoromethane
c.
Monochlorodifluoromethane
d.
Methyl chloride
• Question 17
0 out of 3.3 points
Brazing is used for joining two
Selected Answer: [None Given]
Answers: a.
Two ferrous material
b.
Two non metals
c.
One ferrous and non ferrous material
d.
Two non ferrous material
• Question 18
0 out of 3.3 points
A single trunk piston-type compressor is undesirable for a Freon unit
because the:
Selected Answer: [None Given]
Answers: a.
Lubricant mixes with the refrigerant
b.
Refrigerant reduces the crankcase pressure
c.
Refrigerant becomes superheated
d.
Lubricant temperature becomes excessive
• Question 19
0 out of 3.3 points
A term used to mean the corrective steps taken to return the environment to its
original condition
Selected Answer: [None Given]
Answers: a.
Opacity
b.
Greenhouse effect
c.
Remediation
d.
Stabilization
• Question 20
0 out of 3.3 points
When an electric motor nameplate indicate a “100-kw electric motor” then
what does a 100-kw rating refers to?
Selected Answer: [None Given]
Answers: a.
thermal energy output
b.
electrical power output
c.
mechanical power output
d.
thermal energy input
• Question 21
0 out of 3.3 points
The purpose of the expansion valve by-pass is
to
Selected [None Given]
Answer:
Answers: a.
By-pass the compressor
b.
Increase the efficiency of the plant
c.
Increase the capacity of the evaporator
d.
Controls the refrigerant to the evaporator in case the
automatic valves fail
• Question 22
0 out of 3.3 points
As Freon leaves the expansion valve:
Selected Answer: [None Given]
Answers: a.
Pressure decrease - Volume increase
b.
Pressure increase - Volume increase
c.
Pressure decrease - Volume decrease
d.
Pressure increase - Volume decrease
• Question 23
0 out of 3.3 points
Is a water soluble organic compound prepared from ammonia. It has significant
biological and industrial usefulness
Selected Answer: [None Given]
Answers: a.
Oxidants
b.
Urea or carbamide urea
c.
Nitrite
d.
Chlorinated organics
• Question 24
0 out of 3.3 points
Air delivered to the room from the supply duct, moving at a velocity of 150
ft/min or more is called:
Selected Answer: [None Given]
Answers: a.
Primary Air
b.
Air Turbulence
c.
Secondary Air
d.
Saturated Air
• Question 25
0 out of 3.3 points
The temperature bulb of the thermo- expansion valve is attached to which of
the following?
Selected Answer: [None Given]
Answers: a.
Evaporator coil inlet
b.
Evaporator coil outlet
c.
Icebox coil
d.
Wall of the icebox
• Question 26
0 out of 3.3 points
The solenoid valve is actuated by which of the following?
Selected Answer: [None Given]
Answers: a.
A bellows
b.
Spring
c.
All of the other choices
d.
A magnet
• Question 27
0 out of 3.3 points
What is the main function of a receiver?
Selected Answer: [None Given]
Answers: a.
Store the refrigerant
b.
Separate the oil from the refrigerant
c.
Condense the refrigerant
d.
Cool the hot gases
• Question 28
0 out of 3.3 points
Thermal expansion valves are usually made of the:
Selected Answer: [None Given]
Answers: a.
Bellows type
b.
Diaphragm
c.
Magnetic type
d.
Diaphragm and Bellows type
• Question 29
0 out of 3.3 points
When the air is saturated the wet-bulb depression is:
Selected Answer: [None Given]
Answers: a.
unity
b.
100%
c.
zero
d.
indefinite
• Question 30
0 out of 3.3 points
The method of cooling which primarily used where ambient air
temperatures are high and relative humidity is used:
Selected Answer: [None Given]
Answers: a.
Hydroionic cooling
b.
Swamp cooling
c.
Condensate cooling
d.
Evaporative cooling
Tuesday, March 17, 2020 3:51:01 PM PST
• Question 1
0 out of 3.3 points
Which of the following best described a Freon?
Selected Answer: [None Given]
Answers: a.
Odorless
b.
Colorless
c.
Non-poisonous
d.
All of the other choices
• Question 2
0 out of 3.3 points
Which of the ff cannot be phenomenon of neutrons interactions?
Selected Answer: [None Given]
Answers: a.
Fission
b.
Fusion
c.
Elastic scattering
d.
Inelastic scattering
• Question 3
0 out of 3.3 points
When heavy electrical currents are involved, the thermostat will be operated
by a
Selected Answer: [None Given]
Answers: a.
Fusetron
b.
Relay
c.
Pressure pipe
d.
Small circuit breaker
• Question 4
0 out of 3.3 points
The solenoid valve is controlled by
Selected Answer: [None Given]
Answers: a.
The amount of liquid in the system
b.
The temperature in the icebox
c.
The temperature in the condenser
d.
The amount of gas in the system
• Question 5
0 out of 3.3 points
Which of the following refrigerants is popular in the industrial refrigerating
system; also popularly absorption system of refrigerant.
Selected Answer: [None Given]
Answers: a.
R – 717 (Ammonia)
b.
R – 611 (Methyl Formate)
c.
R – 504
d.
R – 600 (Butane)
• Question 6
0 out of 3.3 points
In balance system, the available draft is
Selected Answer: [None Given]
Answers: a.
100
b.
Zero
c.
Infinite
d.
Unity
• Question 7
0 out of 3.3 points
The cooling water regulator is actuated by
Selected Answer: [None Given]
Answers: a.
Pressure of the refrigerant
b.
Temperature of the refrigerant
c.
An electric relay
d.
None of the other choices
• Question 8
0 out of 3.3 points
The coefficient of the velocity, Cv accounts for the:
Selected [None Given]
Answer:
Answers: a.
Effects of compressibility
b.
Effects on the flow area of contraction, friction, and
turbulence
c.
Small effect of friction and turbulence of the orifice
d.
Changes in diameters of converging pipe
• Question 9
0 out of 3.3 points
Which of the following refrigerant is added sometimes to other refrigerant to
improve oil circulation?
Selected Answer: [None Given]
Answers: a.
R – 170 (Ethane)
b.
R – 270
c.
R – 117
d.
R – 777
• Question 10
0 out of 3.3 points
A leaky suction valve can usually detected by:
Selected [None Given]
Answer:
Answers: a.
A higher suction pressure
b.
Closing in on the suction valve having no effect on the suction
pressure
c.
A fluctuating suction pressure gauge
d.
Any of the other choices
• Question 11
0 out of 3.3 points
Per capita consumption of water is generally taken as:
Selected Answer: [None Given]
Answers: a.
400-500 liters
b.
150-300 liters
c.
750-1000 liters
d.
50-100 liters
• Question 12
0 out of 3.3 points
Which of the following uses a secondary refrigerant?
Selected Answer: [None Given]
Answers: a.
Ice plant
b.
Domestic refrigerator
c.
Room air conditioner
d.
Deep freezer
• Question 13
0 out of 3.3 points
The purpose of the receiver is to
Selected Answer: [None Given]
Answers: a.
Cool the refrigerant gas
b.
Condense the refrigerant
c.
Store the refrigerant
d.
Separate the oil from the refrigerant
• Question 14
0 out of 3.3 points
When does the refrigerant gives-up heat?
Selected Answer: [None Given]
Answers: a.
When it boils
b.
When it condenses
c.
When it evaporates
d.
When it vaporizes
• Question 15
0 out of 3.3 points
What usually happened if brine has a high specific gravity ?
Selected Answer: [None Given]
Answers: a.
It will solidify
b.
Nothing will happen
c.
It will freeze
d.
It will crystallize
• Question 16
0 out of 3.3 points
In power plant electrostatic precipitators is installed between:
Selected Answer: [None Given]
Answers: a.
Primary air and secondary air
b.
Furnace and chimney
c.
Forced draft fan and induced draft fan
d.
Furnace and forced draft fan
• Question 17
0 out of 3.3 points
The capacity in ice making is always
Selected Answer: [None Given]
Answers: a.
Greater than the refrigerating effect
b.
Directly proportional to the refrigerating effect
c.
Less than the refrigerating effect
d.
Equal to the refrigerating effect
• Question 18
0 out of 3.3 points
When coil surface is below the dew point temperature of the air.
Selected Answer: [None Given]
Answers: a.
air is dehumidified
b.
moisture will condense out of the air
c.
air is humidified and saturated
d.
vapor will evaporate through the air
• Question 19
0 out of 3.3 points
A type of polymer used for grocery bags and food wraps
Selected Answer: [None Given]
Answers: a.
Polyvinyl chloride(PVC)
b.
Polypropylene(PP)
c.
Polystyrene(PS)
d.
Low density polyethelene(LDPE)
• Question 20
0 out of 3.3 points
When securing a Freon – 12 system for repairs
Selected Answer: [None Given]
Answers: a.
Pump down to a slight vacuum
b.
Pump down to 10” vacuum
c.
Open the line at 5 to 10 pounds pressure
d.
Open the line at 1 to 2 pounds pressure
• Question 21
0 out of 3.3 points
A steam trap that failed to open would cause the heating unit to become:
Selected Answer: [None Given]
Answers: a.
Steambound
b.
Very hot
c.
Waterlogged
d.
All of the above
• Question 22
0 out of 3.3 points
A thermometer sense which of the following?
Selected Answer: [None Given]
Answers: a.
Latent heat
b.
Sensible heat
c.
Specific heat
d.
Heat of Fusion
• Question 23
0 out of 3.3 points
The purpose of the evaporator is to
Selected Answer: [None Given]
Answers: a.
Absorb latent heat of fusion
b.
Transfer latent heat of fusion
c.
Absorb latent heat of vaporization
d.
Transfer latent heat of vaporization
• Question 24
0 out of 3.3 points
Which of the following is the most appropriate definition of latent heat?
Selected Answer: [None Given]
Answers: a.
Heat added to change temperature of a substance
b.
Heat removed to melt ice
c.
Heat removed to change temperature of substance
d.
Heat added to change the state of a substance
• Question 25
0 out of 3.3 points
The main purpose of a subcooler in a refrigerating system especially a two
stage system is to:
Selected [None Given]
Answer:
Answers: a.
Reduce the total power requirements and return oil to the
compressor
b.
Increase the heat rejection per ton and avoid system shutdown
c.
Reduce the total power requirements and heat rejection to the
second stage
d.
Improve the flow of evaporator gas per ton and increase the
temperature
• Question 26
0 out of 3.3 points
Which of the following could be used to check a leak in an ammonia system?
Selected Answer: [None Given]
Answers: a.
Sulfur stick
b.
Halide torch
c.
Litmus paper & Sulfur stick
d.
Litmus paper
• Question 27
0 out of 3.3 points
What is the usual cop of a domestic refrigerator?
Selected Answer: [None Given]
Answers: a.
Equal to 1
b.
Less than 1
c.
Depends upon the make of it
d.
More than 1
• Question 28
0 out of 3.3 points
A term used to mean the corrective steps taken to return the environment to its
original condition
Selected Answer: [None Given]
Answers: a.
Opacity
b.
Greenhouse effect
c.
Stabilization
d.
Remediation
• Question 29
0 out of 3.3 points
What is the effect on saturated temperature if pressure of the fluid is
decreased:
Selected Answer: [None Given]
Answers: a.
there is no effect
b.
saturation temperature remains constant
c.
saturation temperature increases
d.
saturation temperature decreases
• Question 30
0 out of 3.3 points
Which of the following is used as high head turbine?
Selected Answer: [None Given]
Answers: a.
Propeller
b.
Francis
c.
Impulse
d.
Reaction
Tuesday, March 17, 2020 3:52:33 PM PST
• Question 1
0 out of 3.3 points
In an ammonia system, the oil gauge must be kept:
Selected Answer: [None Given]
Answers: a.
Closed except when checking level oil
b.
Close when machine is shut down
c.
Open when machine is shut down
d.
Open at all times
• Question 2
0 out of 3.3 points
When a hot soup was served in a cup during dinner, an Engineer was o eager
to drink it. Since it was hot, he added ice cubes of ice to cool the soup stirred
it. He noticed that dew starts to form on the outermost surface of the cup.
He wanted to check the temperature of the outermost surface of the cup.
What is this temperature equal to?
Selected Answer: [None Given]
Answers: a.
standard temperature
b.
equal to air’s dew point temperature
c.
equal to zero
d.
superheated temperature
• Question 3
0 out of 3.3 points
The average flue gas temperature is
Selected Answer: [None Given]
Answers: a.
The temperature halfway up the stack
b.
The sum of the temperatures
c.
The temperature leaving the stack
d.
The temperature entering the stack
• Question 4
0 out of 3.3 points
When ordering an expansion valve which of the following information is
necessary?
Selected Answer: [None Given]
Answers: a.
Pressure and temperature
b.
Size and tonnage
c.
Size, and pressure
d.
Size ,tonnage , temperature and pressure
• Question 5
0 out of 3.3 points
What amount of air is required in a low bypass factor?
Selected Answer: [None Given]
Answers: a.
Increases
b.
indeterminate
c.
greater
d.
lesser
• Question 6
0 out of 3.3 points
The refrigerant temperature is at its maximum just before it enters the
_______
Selected Answer: [None Given]
Answers: a.
Condenser
b.
Evaporator
c.
Expansion valve
d.
Compressor
• Question 7
0 out of 3.3 points
Ice cubes added to a glass of water and stirred. Moisture starts to from on
the outer surface of the glass. At this point, what is the temperature at the
outer surface called?
Selected Answer: [None Given]
Answers: a.
surface temperature
b.
dew point temperature
c.
critical temperature
d.
saturation temperature
• Question 8
0 out of 3.3 points
The amount of heat necessary to bring up temperature of a unit mass of a
substance through unit degree is called:
Selected Answer: [None Given]
Answers: a.
specific volume
b.
specific heat
c.
thermal heat
d.
total heat
• Question 9
0 out of 3.3 points
What is the reason why a thermometer in vapor compression system is
installed close to the compressor?
Selected [None Given]
Answer:
Answers: a.
Because temperature indicates whether liquid or vapor
refrigerant is going to compressor
b.
Because temperature helps in calculating the cop
c.
Because the performance of other units of the cycle can be
controlled
d.
Because it helps the operator in adjusting compressor for
greatest efficiency
• Question 10
0 out of 3.3 points
When coil surface is below the dew point temperature of the air.
Selected Answer: [None Given]
Answers: a.
vapor will evaporate through the air
b.
air is dehumidified
c.
moisture will condense out of the air
d.
air is humidified and saturated
• Question 11
0 out of 3.3 points
A mechanism that removes moisture
Selected Answer: [None Given]
Answers: a.
cooling towers
b.
dehumidifiers
c.
moisturizers
d.
humidifiers
• Question 12
0 out of 3.3 points
Wb is shaft work of an engine and Wi is indicated work of an engine. If
mechanical is present in the engine mechanism, then.
Selected Answer: [None Given]
Answers: a.
Wb is less than Wi
b.
Wb is proportional to Wi
c.
Wb is equal to Wi
d.
Wb is greater then Wi
• Question 13
0 out of 3.3 points
Excess frost on the evaporator coils:
Selected Answer: [None Given]
Answers: a.
Reduces efficiency of the plant
b.
Takes load off compressor
c.
Keeps the icebox cooler
d.
Does not affect the system
• Question 14
0 out of 3.3 points
Refers to the high – temperature removal of tarry substances from the interior of
the carbon granule, leaving a highly porous structure
Selected Answer: [None Given]
Answers: a.
Breakthrough
b.
Absorbent
c.
Adsorbent
d.
Activated
• Question 15
0 out of 3.3 points
What process is employed when the turbine steam power plants experience
excessive moisture?
Selected Answer: [None Given]
Answers: a.
supercooling
b.
reheating
c.
freezing
d.
subcooling
• Question 16
0 out of 3.3 points
Most stacks are built of height less than __________.
Selected Answer: [None Given]
Answers: a.
50 m
b.
40 m
c.
60 m
d.
30 m
• Question 17
0 out of 3.3 points
Which of the following is not essential to a centrifugal type compressor
system?
Selected Answer: [None Given]
Answers: a.
Evaporator
b.
Distiller
c.
Expansion valve
d.
Condenser
• Question 18
0 out of 3.3 points
In power plant electrostatic precipitators is installed between:
Selected Answer: [None Given]
Answers: a.
Primary air and secondary air
b.
Forced draft fan and induced draft fan
c.
Furnace and forced draft fan
d.
Furnace and chimney
• Question 19
0 out of 3.3 points
Before securing a compressor to do maintenance work on it, be sure to
Selected [None Given]
Answer:
Answers: a.
Have gas mask handy
b.
Have gas mask handy and make arrangements to have
perishables taken care of
c.
Notify the engineer
d.
Make arrangements to have perishables taken care of
• Question 20
0 out of 3.3 points
The diesel cycle is the ideal cycle for a:
Selected Answer: [None Given]
Answers: a.
steam-jet
b.
gas turbine
c.
compression-ignition engine
d.
absorption
• Question 21
0 out of 3.3 points
What is the device used to protect the compressor from overloading due to
high head pressure
Selected Answer: [None Given]
Answers: a.
Overload relay
b.
Hold back suction valve
c.
Expansion valve
d.
Thermostatic expansion valve
• Question 22
0 out of 3.3 points
What is the effect if the refrigerant is removed from the system too fast?
Selected Answer: [None Given]
Answers: a.
None of the other choices
b.
It will lower the icebox temperature
c.
It may “freeze – up” the condenser
d.
It may flood the evaporator
• Question 23
0 out of 3.3 points
The secondary refrigerant used in milk chilling plants is generally
Selected Answer: [None Given]
Answers: a.
Ammonia
b.
Brine
c.
Sodium silicate
d.
Glycol
• Question 24
0 out of 3.3 points
If C is the capacity of the compressor in a refrigeration system and F is the
heat rejection factor, then calculate the condenser load, L.
Selected Answer: [None Given]
Answers: a.
L=F/C
b.
L=C/F
c.
L=CxF
d.
L=F–C
• Question 25
0 out of 3.3 points
Carries needed to deliver air to the conditioned space it is made of sheet of
metals like aluminum, galvanized sheet steel and some structural materials
that will not burn.
Selected Answer: [None Given]
Answers: a.
Diffuser
b.
Duct
c.
Air Inlet
d.
Air Outlet
• Question 26
0 out of 3.3 points
Which of the following items is not important when using a halide torch?
Selected Answer: [None Given]
Answers: a.
Move flame slowly around the joints
b.
Adjust to a clear white flame
c.
Hold flame close to the joints
d.
Adjust to a clear blue flame
• Question 27
0 out of 3.3 points
The relief valve on an ammonia machine is located:
Selected Answer: [None Given]
Answers: a.
On the discharge pipe of the condenser
b.
In the compressor head
c.
On the discharge pipe of the compressor
d.
On the discharge pipe of the condenser and compressor
• Question 28
0 out of 3.3 points
A method of cooling which uses water as refrigerant. Pressure on the water
surface is reduced to lower its boiling temperature.
Selected Answer: [None Given]
Answers: a.
Steam Jet Cooling
b.
Vortex Tube Cooling
c.
Pressurized Cooling
d.
Evaporative Cooling
• Question 29
0 out of 3.3 points
All of the following are forms of drag on a body moving through a fluid
except:
Selected Answer: [None Given]
Answers: a.
Profile drag
b.
Skin friction
c.
Wake drag
d.
D’alembert’s paradox drag
• Question 30
0 out of 3.3 points
In the upper atmosphere, ozone is made by ultraviolet light reacting with:
Selected Answer: [None Given]
Answers: a.
Nitrogen
b.
Oxygen
c.
Water Vapor
d.
Hydrogen
Tuesday, March 17, 2020 3:55:50 PM PST
• Question 1
0 out of 3.3 points
To secure the belts embedded within the foundation, the distance of the
edges of the foundation from the bedplate must be __________.
Selected Answer: [None Given]
Answers: a.
120 mm to 300 mm
b.
200 mm to 380 mm
c.
100 mm to 280 mm
d.
150 mm to 330 mm
• Question 2
0 out of 3.3 points
An impulse turbine are used for:
Selected Answer: [None Given]
Answers: a.
Low head
b.
Very low head
c.
High head
d.
Medium head
• Question 3
0 out of 3.3 points
The ratio absorbed by the transfer fluid to the original incident energy striking the
collector
Selected Answer: [None Given]
Answers: a.
Collector efficiency
b.
Transmittance
c.
Shading factor
d.
Betz coefficient
• Question 4
0 out of 3.3 points
The capacity of a centrifugal type compressor is controlled by which of the
following?
Selected Answer: [None Given]
Answers: a.
Regulating the speed
b.
Regulating the speed or regulating the suction pressure
c.
Regulating the suction pressure
d.
Regulating the discharge pressure
• Question 5
0 out of 3.3 points
What must be the value of the available Net Positive Suction Head (NPHS) of
a centrifugal pump compared to its require NPHS to avoid losing priming?
Selected Answer: [None Given]
Answers: a.
available NPHS lesser than required NPHS
b.
available NPHS greater than required NPHS
c.
available NPHS constant at all times
d.
available NPHS equal then required NPHS
• Question 6
0 out of 3.3 points
The temperature below which water vapor in the air will start to condense:
Selected Answer: [None Given]
Answers: a.
Wet bulb Temperature
b.
Dry bulb Temperature
c.
Dew point Temperature
d.
Condensing Temperature
• Question 7
0 out of 3.3 points
Most people are comfortable with the relative humidity of:
Selected Answer: [None Given]
Answers: a.
20 % to 30%
b.
40 % to 80 %
c.
30 % to 70 %
d.
10 % to 40 %
• Question 8
0 out of 3.3 points
Which of the following refrigerants is popular in the industrial refrigerating
system; also popularly absorption system of refrigerant.
Selected Answer: [None Given]
Answers: a.
R – 600 (Butane)
b.
R – 504
c.
R – 611 (Methyl Formate)
d.
R – 717 (Ammonia)
• Question 9
0 out of 3.3 points
A Kaplan turbine is:
Selected Answer: [None Given]
Answers: a.
An outward flow reaction turbine
b.
An inward flow impulse turbine
c.
Low head axial flow turbine
d.
A high head mixed flow turbine
• Question 10
0 out of 3.3 points
Which of the following is another name for the liquid valve?
Selected Answer: [None Given]
Answers: a.
Freon valve
b.
King valve
c.
Master valve
d.
Shutoff valve
• Question 11
0 out of 3.3 points
How is a thermostatic valve tested?
Selected Answer: [None Given]
Answers: a.
Shorting out the cutout switch
b.
Immersing its bulb in warm water
c.
Immersing its bulb in ice water
d.
Holding its bulb in one’s hand
• Question 12
0 out of 3.3 points
The thermostatic expansion valve is designed to maintain a constant _______.
Selected Answer: [None Given]
Answers: a.
Pressure
b.
Flow
c.
Temperature
d.
Superheat
• Question 13
0 out of 3.3 points
When vapor compression takes place on one side of the piston and one
during one revolution of the crankshaft, then the compressor is:
Selected Answer: [None Given]
Answers: a.
two-revolution
b.
single-acting
c.
double-acting
d.
four-cycle
• Question 14
0 out of 3.3 points
According to ASME code, safety valves on low pressure boilers should be
tested by hand at least
Selected Answer: [None Given]
Answers: a.
Once a shift
b.
Once a year
c.
Twice a year
d.
Once a month
• Question 15
0 out of 3.3 points
The dehydrator is located between the:
Selected Answer: [None Given]
Answers: a.
Receiver and the king valve
b.
Condenser and receiver
c.
Receiver and the expansion valve
d.
Condenser and the king valve
• Question 16
0 out of 3.3 points
The cooling component of a refrigeration cycle is called ________.
Selected Answer: [None Given]
Answers: a.
A desiccant
b.
A condenser
c.
An evaporator
d.
A receiver
• Question 17
0 out of 3.3 points
Refers to organic waste produced from biological water waste treatment
processes
Selected Answer: [None Given]
Answers: a.
Toxic waste
b.
Biosolids
c.
Process waste
d.
Extrinsic waste
• Question 18
0 out of 3.3 points
What tonnage of refrigerating machine is required if the refrigerating system
extracted 48,000 btu per hour?
Selected Answer: [None Given]
Answers: a.
4 tons
b.
2 tons
c.
5 tons
d.
3 tons
• Question 19
0 out of 3.3 points
During combustion process, only limited air is supplied and not enough to
supply two molecule of oxygen per one molecule of carbon, then the
product is:
Selected Answer: [None Given]
Answers: a.
carbon monoxide
b.
carbon dioxide
c.
hydration
d.
carbonic acid
• Question 20
0 out of 3.3 points
A _____ after each radiator allows _____to pass through to the return line
Selected Answer: [None Given]
Answers: a.
Water trap; condensate
b.
Steam trap; steam
c.
Water trap; steam
d.
Steam trap; condensate
• Question 21
0 out of 3.3 points
A precooler is sometimes installed between the ________.
Selected Answer: [None Given]
Answers: a.
Expansion valve and evaporator
b.
Compressor and condenser
c.
Evaporator and compressor
d.
Condenser and expansion valve
• Question 22
0 out of 3.3 points
The angle between the stack and the guy wire is usually
Selected Answer: [None Given]
Answers: a.
30 degrees
b.
75 degrees
c.
45 degrees
d.
60 degrees
• Question 23
0 out of 3.3 points
An airconditioning system which is a combinaton of heating and cooling
system. It is designed to serve an individual room or zone.
Selected Answer: [None Given]
Answers: a.
multizone ductless split system
b.
window type air conditioner
c.
console air conditioner
d.
packaged terminal air-conditioner
• Question 24
0 out of 3.3 points
Which of the following is the probable cause of hot suction line?
Selected Answer: [None Given]
Answers: a.
Expansion valve closed too much
b.
Too much refrigerant
c.
Insufficient refrigerant
d.
Insufficient calibration
• Question 25
0 out of 3.3 points
In a window air conditioning unit which of the following is usually done by the
owner?
Selected [None Given]
Answer:
Answers: a.
Annual cleaning of the evaporator, condenser, fan blades, fan motor,
compressor and casing
b.
Inspection of fan motors and lubricates them
c.
Semi-annual cleaning or replacement of filters
d.
All of the other choices
• Question 26
0 out of 3.3 points
What is the compression ratio of a refrigerator compressor?
Selected [None Given]
Answer:
Answers: a.
The ratio of the excessive liquid refrigerant in the receiver to
that in the system
b.
The ratio of the absolute suction pressure to the absolute
discharge pressure
c.
The ratio of the absolute discharge pressure to the absolute
suction pressure
d.
None of the other choices
• Question 27
0 out of 3.3 points
When required, a regulator water valve in refrigerating system should be
Selected Answer: [None Given]
Answers: a.
in the water inlet
b.
anywhere in the system
c.
on the suction line on compressor line
d.
in the water outlet
• Question 28
0 out of 3.3 points
What is likely to occur when sections of the impeller of a centrifugal pump
are handling vapor and other sections are handling liquid.
Selected Answer: [None Given]
Answers: a.
excessive vibration
b.
high head and low capacity
c.
erosion of the pump
d.
complete failure to operate
• Question 29
0 out of 3.3 points
A “CARRENE” is a type of :
Selected Answer: [None Given]
Answers: a.
Refrigerant
b.
Scale cleaner
c.
Refrigerant oil
d.
None of the other choices
• Question 30
0 out of 3.3 points
If the compressor short-cycles on the low-pressure cutout, the trouble might
be:
Selected Answer: [None Given]
Answers: a.
Too much frost on the evaporator coils
b.
Lack of refrigerant
c.
Dirty traps and strainers
d.
Any of the other choices
Tuesday, March 17, 2020 3:57:53 PM PST
• Question 1
0 out of 3.3 points
In series pipe systems, all of the ff parameters vary from section to section
except
Selected Answer: [None Given]
Answers: a.
Friction loss
b.
Pressure drop
c.
Head loss
d.
Mass flow
• Question 2
0 out of 3.3 points
Compare in refrigerating effect per unit mass of refrigerant circulated for a
superheated cycle that produces useful cooling and a saturated cycle, for the
same vaporizing and condensing temperature.
Selected Answer: [None Given]
Answers: a.
greater for a saturated cycle
b.
lower for a superheated cycle
c.
greater for a superheated cycle
d.
the same
• Question 3
0 out of 3.3 points
Which of the ff cannot be phenomenon of neutrons interactions?
Selected Answer: [None Given]
Answers: a.
Inelastic scattering
b.
Elastic scattering
c.
Fission
d.
Fusion
• Question 4
0 out of 3.3 points
Which of the following is the type of refrigerant that damage Ozone layer?
Selected Answer: [None Given]
Answers: a.
Hydrochlorofluorocarbons (HCHF’s)
b.
R-12
c.
R-22
d.
All of these
• Question 5
0 out of 3.3 points
Too high suction pressure could be caused by:
Selected Answer: [None Given]
Answers: a.
Leaky suction valves
b.
Expansion valve open too wide
c.
Expansion valve bulb not working properly
d.
Any of the other choices
• Question 6
0 out of 3.3 points
What is the specific humidity of dry air?
Selected Answer: [None Given]
Answers: a.
150
b.
50
c.
0
d.
100
• Question 7
0 out of 3.3 points
The maximum continuous power available from a hydro -electric plant under the
most adverse hydraulic conditions is called
Selected Answer: [None Given]
Answers: a.
Firm power
b.
Primary power
c.
Base power
d.
Secondary power
• Question 8
0 out of 3.3 points
Refrigerant leakage from the compressor crankcase is prevented by
Selected Answer: [None Given]
Answers: a.
using shaft seals
b.
maintaining a vacuum in the crankcase
c.
using skirt type pistons
d.
using lantern rings
• Question 9
0 out of 3.3 points
Concrete foundation should have steel bar reinforcement placed vertically
and horizontally to avoid ___________.
Selected Answer: [None Given]
Answers: a.
Thermal cracking
b.
Vibration
c.
Breaking
d.
Melting
• Question 10
0 out of 3.3 points
Lithium bromide is used in refrigeration system in:
Selected Answer: [None Given]
Answers: a.
Absorbers
b.
Condensate return lines
c.
Ion exchangers
d.
Centrifugal compressors
• Question 11
0 out of 3.3 points
If the compressor had been running satisfactorily for a long period of time
but suddenly the compartment temperature started to rise, the trouble
might be:
Selected Answer: [None Given]
Answers: a.
Any of the other choices
b.
The expansion valve may contain frozen water
c.
The solenoid valve has jammed shut
d.
A refrigerant leak has developed
• Question 12
0 out of 3.3 points
The high pressure of refrigeration system consist of the line to the
expansion valve, the receiver, the uppermost half of the compressor and the
Selected Answer: [None Given]
Answers: a.
Evaporator
b.
Lower most half of compressor
c.
Condenser
d.
Line after the expansion valve
• Question 13
0 out of 3.3 points
Which of the following components of the window air conditioning system
must be cleaned annually?
a.
Selected Answer: [None Given]
Answers: a.
Evaporator and Condenser
b.
Motor and Compressor
c.
Fan Blades and Fan Motor
d.
All of these
• Question 14
0 out of 3.3 points
Latent heat
Selected Answer: [None Given]
Answers: a.
Can be measured with a thermometer
b.
Changes as the refrigerant cools
c.
Cannot be measured with a thermometer
d.
Can be measured with a pyrometer
• Question 15
0 out of 3.3 points
Mycotoxins are poisonous chemicals produced by:
Selected Answer: [None Given]
Answers: a.
Molds
b.
Bacteria
c.
Virus
d.
Algae
• Question 16
0 out of 3.3 points
Which of the following is not essential to a compression refrigeration
system?
Selected Answer: [None Given]
Answers: a.
A dehydrator
b.
An evaporator
c.
A receiver
d.
A condenser
• Question 17
0 out of 3.3 points
The fact that there is no resistance bodies moving through an ideal(non-
viscous) fluid is known as:
Selected Answer: [None Given]
Answers: a.
D’alembert’s paradox
b.
Reynold’s analogy
c.
Newton’s 2nd law
d.
The 2nd law of thermodynamics
• Question 18
0 out of 3.3 points
Air conditioning is the process of:
Selected [None Given]
Answer:
Answers: a.
removing heat from a specific area
b.
maintaining the air at a required temperature and humidity
c.
keeping a place cool and maintaining the air at a required
temperature and humidity
d.
keeping a place cool
• Question 19
0 out of 3.3 points
The coefficient of contraction is the ratio of the :
Selected Answer: [None Given]
Answers: a.
Actual velocity to the theoretical velocity
b.
Area of vena contracta to the orifice area
c.
Actual discharge to the theoretical discharge
d.
Effective head to the actual head
• Question 20
0 out of 3.3 points
The method of cooling which primarily used where ambient air
temperatures are high and relative humidity is used:
Selected Answer: [None Given]
Answers: a.
Evaporative cooling
b.
Hydroionic cooling
c.
Swamp cooling
d.
Condensate cooling
• Question 21
0 out of 3.3 points
Which of the following must be considered when adding or removing oil
from a refrigerator unit?
Selected Answer: [None Given]
Answers: a.
Do not overcharge
b.
All of the other choices
c.
Watch crankcase pressure
d.
Use new oil
• Question 22
0 out of 3.3 points
Excess fuel oil in the fuel oil system returns to the:
Selected Answer: [None Given]
Answers: a.
Burner
b.
Combustion chamber
c.
Fuel oil tank
d.
Suction line
• Question 23
0 out of 3.3 points
Most nuclear particles can react with atoms in several different ways including
Selected Answer: [None Given]
Answers: a.
Reflection and absorption
b.
Scattering
c.
Absorption
d.
Absorption and scattering
• Question 24
0 out of 3.3 points
Heating and dehumidification can be obtained simultaneously if air is passed
through:
Selected [None Given]
Answer:
Answers: a.
either a solid absorbent surface or a liquid absorbent spray
b.
sprays of water kept at a temperature higher than the dew point
temperature of the entering air
c.
a liquid absorbent spray
d.
a solid absorbent surface
• Question 25
0 out of 3.3 points
The relief valve on a CO2 machine is located:
Selected [None Given]
Answer:
Answers: a.
On the discharge pipe between the compressor and discharge
valve
b.
On the discharge pipe from the condenser
c.
Next to the king valve
d.
In the compressor head
• Question 26
0 out of 3.3 points
What is the probable cause if a compressor runs continuously?
Selected Answer: [None Given]
Answers: a.
faulty cooling water valve
b.
insufficient refrigerant
c.
any of the other choices
d.
a clogged condenser
• Question 27
0 out of 3.3 points
Refers to organic waste produced from biological water waste treatment
processes
Selected Answer: [None Given]
Answers: a.
Biosolids
b.
Extrinsic waste
c.
Process waste
d.
Toxic waste
• Question 28
0 out of 3.3 points
Reheating process is normally employed in steam plane when:
Selected Answer: [None Given]
Answers: a.
there is excess steam
b.
turbine undergoes excessive moisture
c.
dehumidifying is required
d.
subcooling is required
• Question 29
0 out of 3.3 points
The most common reason for the small usage of CO2 systems is :
Selected Answer: [None Given]
Answers: a.
It is too costly
b.
The machinery is too heavy
c.
It is too dangerous
d.
Its upkeep is too high
• Question 30
0 out of 3.3 points
The compressor will run continuously if there is
Selected Answer: [None Given]
Answers: a.
Air in the system
b.
Any of the other choices
c.
Insufficient refrigerant
d.
Too heavy a load
Tuesday, March 17, 2020 4:00:43 PM PST
• Question 1
0 out of 3.3 points
The oil level in the compressor should be checked:
Selected Answer: [None Given]
Answers: a.
After a long period of operation
b.
Just before starting the compressor
c.
While the compressor is in compressor
d.
After an extended lay-up period
• Question 2
0 out of 3.3 points
All of the following temperature have meaning in psychometrics except:
Selected Answer: [None Given]
Answers: a.
dew point
b.
wet-bulb temperature
c.
adiabatic wall temperature
d.
dry-bulb temperature
• Question 3
0 out of 3.3 points
Boilers using soft coal must have ____ furnace volume to reduce the danger
of ____
Selected Answer: [None Given]
Answers: a.
Small; smoking
b.
Large; smoking
c.
Small; overfiring
d.
Large; overfiring
• Question 4
0 out of 3.3 points
Too high suction pressure could be caused by:
Selected Answer: [None Given]
Answers: a.
Expansion valve bulb not working properly
b.
Any of the other choices
c.
Expansion valve open too wide
d.
Leaky suction valves
• Question 5
0 out of 3.3 points
A sulfur stick burning in the presence of ammonia will give off a ______.
Selected Answer: [None Given]
Answers: a.
Dense white smoke
b.
Dense red smoke
c.
Dense green smoke
d.
Dense yellow smoke
• Question 6
0 out of 3.3 points
If Wt is the turbine shaft work of a gas turbine unit, Wc in duel, then
determine its thermal efficiency.
Selected Answer: [None Given]
Answers: a.
Q / (Wt + Wc)
b.
(Wt –Wc) /Q
c.
(Wt + Wc) / Q
d.
Q / (Wt – Wc)
• Question 7
0 out of 3.3 points
Which of the following is used as high head turbine?
Selected Answer: [None Given]
Answers: a.
Impulse
b.
Reaction
c.
Francis
d.
Propeller
• Question 8
0 out of 3.3 points
A thermostat is a
Selected Answer: [None Given]
Answers: a.
Superheat-operated switch
b.
Temperature-operated switch
c.
Pressure-operated switch
d.
Back-pressure-operated switch
• Question 9
0 out of 3.3 points
Which of the ff is a cold cathode lamp?
Selected Answer: [None Given]
Answers: a.
Sodium vapor lamp
b.
Low pressure mercury vapor lamp
c.
High pressure mercury vapor lamp
d.
Neon lamp
• Question 10
0 out of 3.3 points
If the thermal expansion valve becomes inoperative, the iceboxes will have
to be controlled by the _____________.
Selected Answer: [None Given]
Answers: a.
Manual expansion valve
b.
Solenoid valve
c.
Manual solenoid valve
d.
King valve
• Question 11
0 out of 3.3 points
Substances that have the ability to absorb moisture from the air are called:
Selected Answer: [None Given]
Answers: a.
Desiccants
b.
Moisturizer
c.
Dehygroscopic substances
d.
Moisture absorber
• Question 12
0 out of 3.3 points
The bhp/ton refrigeration requirement with increase in condenser
temperature. In a vapor compression system that uses reciprocating
compressor receiving refrigerant gas at constant suction temperature, will
follow:
Selected Answer: [None Given]
Answers: a.
First increase slowly and then rapidly
b.
Linearly decreasing characteristic
c.
First increase rapidly and then decrease slowly
d.
Linearly increasing characteristic
• Question 13
0 out of 3.3 points
The fraction of the radiation energy incident on a surface which is absorbed
by the surface is called:
Selected Answer: [None Given]
Answers: a.
Convection
b.
Radiation
c.
Absorptivity
d.
Emission
• Question 14
0 out of 3.3 points
The pressure of ammonia was detected in brine solution. How can ammonia
be removed?
Selected Answer: [None Given]
Answers: a.
heat the brine to a high temperature enough to free ammonia
b.
throw the brine solution
c.
freeze the brine solution
d.
lower the temperature of the brine
• Question 15
0 out of 3.3 points
Is a high- temperature process that turns incinerator ash into a safe, glass –like
material
Selected Answer: [None Given]
Answers: a.
Biofitration
b.
Vitrification
c.
Bioventing
d.
Advance oxidation
• Question 16
0 out of 3.3 points
The fact that a fluid’s velocity increases as the cross-sectional area of the pipe
through which it flow decrease is due to:
Selected Answer: [None Given]
Answers: a.
Bernoull’s equation
b.
The continuity equation
c.
The perfect gas law
d.
The momentum equation
• Question 17
0 out of 3.3 points
What is the boiling temperature of F 12
Selected Answer: [None Given]
Answers: a.
-40C
b.
-78.5 C
c.
-29.8C
d.
-33.33 C
• Question 18
0 out of 3.3 points
A double-pipe condenser has __________.
Selected [None Given]
Answer:
Answers: a.
None of the other choices
b.
Two piping system side by side, one with cooling water and one
with refrigerant
c.
A small pipe inside a larger pipe, and the cooling water passing
through the small pipe and the refrigerant through large pipe
d.
Two pipes for cooling water and one for refrigerant
• Question 19
0 out of 3.3 points
How is a Freon system purged?
Selected Answer: [None Given]
Answers: a.
The same as an ammonia system
b.
With a reefer pump
c.
Back to the extra supply tank
d.
The same as CO2 system
• Question 20
0 out of 3.3 points
Very large pipe diameter means a thicker wall and high cost. It also means
Selected Answer: [None Given]
Answers: a.
high pressure drop
b.
zero pressure drop
c.
constant temperature
d.
low pressure drop
• Question 21
0 out of 3.3 points
Cooling towers are used for cooling water:
Selected Answer: [None Given]
Answers: a.
to be used for filtration of air
b.
to be used for humidification
c.
to be injected in circulating air
d.
to be used for cooling the compressor
• Question 22
0 out of 3.3 points
The steam turbine foundation should be designed to support the machine
load plus how many percent for impact, condenser load, floor loads and
dead loads?
Selected Answer: [None Given]
Answers: a.
30
b.
25
c.
20
d.
15
• Question 23
0 out of 3.3 points
The refrigerant temperature is at its maximum just before it enters the
_______
Selected Answer: [None Given]
Answers: a.
Expansion valve
b.
Compressor
c.
Evaporator
d.
Condenser
• Question 24
0 out of 3.3 points
Which of the following fans in air conditioning systems which can be
classified as centrifugal flow?
Selected Answer: [None Given]
Answers: a.
Axial Fan
b.
Propeller fan
c.
Bi-axial fan
d.
None of these
• Question 25
0 out of 3.3 points
Which of the following is the cause if the outlet of a thermostatic valve is
warmer than the inlet side?
Selected Answer: [None Given]
Answers: a.
Valve is working properly
b.
Valve is not working properly
c.
Solenoid valve is not working properly
d.
None of the other choices
• Question 26
0 out of 3.3 points
Latent heat
Selected Answer: [None Given]
Answers: a.
Cannot be measured with a thermometer
b.
Can be measured with a pyrometer
c.
Can be measured with a thermometer
d.
Changes as the refrigerant cools
• Question 27
0 out of 3.3 points
The boiling point of co2 at atmospheric pressure is
Selected Answer: [None Given]
Answers: a.
110 F
b.
-110 C
c.
110 C
d.
-110 F
• Question 28
0 out of 3.3 points
What is another name of discharge pressure?
Selected Answer: [None Given]
Answers: a.
Condenser pressure
b.
Absolute pressure
c.
Head pressure
d.
Suction pressure
• Question 29
0 out of 3.3 points
Why should you avoid bending or twisting of fan blades in an air
conditioning unit?
Selected Answer: [None Given]
Answers: a.
It will wear out the motor bearings and cause noise
b.
It will wear out slice suction line
c.
It decrease the volume flow of refrigerant
d.
It will cause ice build-up
• Question 30
0 out of 3.3 points
Which of the following acts as ignition accelerator for internal combustion engine
fuels?
Selected Answer: [None Given]
Answers: a.
Acetone peroxide
b.
N-heptane
c.
Hydrogen peroxide
d.
Aromatic compounds
Tuesday, March 17, 2020 4:02:31 PM PST
• Question 1
0 out of 3.3 points
When checking zinc plates in a condenser, one should:
Selected Answer: [None Given]
Answers: a.
Ground each plate to the shell
b.
Paint the plates with red lead
c.
Clean the plates and renew worn out ones
d.
Install all new plates
• Question 2
0 out of 3.3 points
Which of the following industries have the highest consumption of water for
processing?
Selected Answer: [None Given]
Answers: a.
Paper mill
b.
Foundry
c.
Aluminum industry
d.
Automobile industry
• Question 3
0 out of 3.3 points
Le Chatelier’s principle predicts the direction of a state of chemical equilibrium
bases on all of the ff factors except:
Selected Answer: [None Given]
Answers: a.
Temperature
b.
Specific volume
c.
Pressure
d.
Concentration
• Question 4
0 out of 3.3 points
The phenomenon that warm air rise and cold air settle is called:
Selected Answer: [None Given]
Answers: a.
Stratification
b.
Ventilation
c.
Sedimentation
d.
Setting Due
• Question 5
0 out of 3.3 points
When the dry bulb and the wet bulb temperature are identical, the air is said
to be:
Selected Answer: [None Given]
Answers: a.
Saturates
b.
Compressed
c.
Humidified
d.
Dehumidified
• Question 6
0 out of 3.3 points
Some causes of a noisy compressor are:
Selected Answer: [None Given]
Answers: a.
Any of the other choices
b.
Slugging due to flooding back of refrigerant
c.
Worn bearings, pins, etc
d.
Too much oil in crankcase
• Question 7
0 out of 3.3 points
Obstruction of the expansion valve is usually caused by:
Selected Answer: [None Given]
Answers: a.
Water in the system
b.
Any of the other choices
c.
Congealed oil in the system
d.
Scale
• Question 8
0 out of 3.3 points
Which of the following is not essential to a compression refrigeration
system?
Selected Answer: [None Given]
Answers: a.
A dehydrator
b.
An evaporator
c.
A condenser
d.
A receiver
• Question 9
0 out of 3.3 points
The system should be purged:
Selected Answer: [None Given]
Answers: a.
After the system has been shut down for few hours
b.
While starting up
c.
Once a week
d.
While system is operating
• Question 10
0 out of 3.3 points
What is the cycle where a refrigeration system generally operates?
Selected Answer: [None Given]
Answers: a.
Closed cycle
b.
Mixed cycle
c.
Open cycle
d.
Hybrid cycle
• Question 11
0 out of 3.3 points
Which is not commonly used to cool and dehumidify equipment?
Selected Answer: [None Given]
Answers: a.
calcium chloride
b.
sodium zeolite
c.
silica gel
d.
activated alumina
• Question 12
0 out of 3.3 points
To eliminate transmission of the vibration, the foundation should be isolated
from the floor slabs of building footings at least how many mm around its
perimeter?
Selected Answer: [None Given]
Answers: a.
30
b.
15
c.
20
d.
25
• Question 13
0 out of 3.3 points
If the outlet of the thermostatic valve is warmer than the inlet, it indicates
Selected Answer: [None Given]
Answers: a.
Thermostatic valve not working properly
b.
Flooding back
c.
Solenoid valve not working properly
d.
Thermostatic valve working properly
• Question 14
0 out of 3.3 points
Which do you think is the cause of a hot suction line of a refrigerating
compressor?
Selected Answer: [None Given]
Answers: a.
Insufficient condensing cooling water
b.
Lack of refrigerant
c.
Excess refrigeration
d.
Insufficient condensing cooling water or excess refrigeration
• Question 15
0 out of 3.3 points
Is the subjective method in which the smoke density is visually compared to five
standardized white-black grids.
Selected Answer: [None Given]
Answers: a.
Dalton scale
b.
Ringelman scale
c.
Dew point scale
d.
Smoke spot scale
• Question 16
0 out of 3.3 points
What usually happened if brine has a high specific gravity ?
Selected Answer: [None Given]
Answers: a.
It will crystallize
b.
It will freeze
c.
It will solidify
d.
Nothing will happen
• Question 17
0 out of 3.3 points
Evaporative condenser is used to cool
Selected Answer: [None Given]
Answers: a.
condenser vapor
b.
condenser surface
c.
condenser liquid
d.
all of these
• Question 18
0 out of 3.3 points
If ice will form in a solution of water and salt, then it is at temperature called:
Selected Answer: [None Given]
Answers: a.
critical point
b.
freezing point depression
c.
boiling point depression
d.
dew point
• Question 19
0 out of 3.3 points
Two compressor should not be run in parallel because :
Selected Answer: [None Given]
Answers: a.
There is a possibility of losing oil
b.
There is a possibility of losing oil or it will give over – capacity
c.
It will give over – capacity
d.
It is not efficient
• Question 20
0 out of 3.3 points
If the solenoid valve closed by accident, the compressor would be stopped
by which of the following?
Selected Answer: [None Given]
Answers: a.
Automatic trip
b.
Low-pressure cutout switch
c.
Low-water cutout switch
d.
High-pressure cutout switch
• Question 21
0 out of 3.3 points
The most likely cause of high superheat would be:
Selected Answer: [None Given]
Answers: a.
Too much refrigerant
b.
Expansion valve open too wide
c.
Expansion valve closed too much
d.
Back-pressure valve set too high
• Question 22
0 out of 3.3 points
The volume flow passes through a venturimeter is:
Selected Answer: [None Given]
Answers: a.
Varying
b.
Constant
c.
Decreasing
d.
Increasing
• Question 23
0 out of 3.3 points
Peak load for a period of time divided by installed capacity is
Selected Answer: [None Given]
Answers: a.
Utilization factor
b.
Load factor
c.
Demand factor
d.
Capacity factor
• Question 24
0 out of 3.3 points
Compare the temperature of discharge vapor refrigerant leaving the
compressor for a superheated cycle and the saturated cycle, for the same
condensing temperature and pressure.
Selected Answer: [None Given]
Answers: a.
lower for the superheated cycle
b.
higher for the saturated cycle
c.
there is no difference
d.
higher for the superheated cycle
• Question 25
0 out of 3.3 points
A precooler, if used is located between the :
Selected Answer: [None Given]
Answers: a.
Compressor and condenser
b.
Condenser and expansion valve
c.
Expansion valve and evaporator
d.
Evaporator and compressor
• Question 26
0 out of 3.3 points
A sulfur stick burning in the presence of ammonia will give off a ______.
Selected Answer: [None Given]
Answers: a.
Dense red smoke
b.
Dense yellow smoke
c.
Dense green smoke
d.
Dense white smoke
• Question 27
0 out of 3.3 points
Which of the following is converted to mechanical energy by a water
Selected Answer: [None Given]
Answers: a.
hydraulic energy
b.
potential energy
c.
internal energy
d.
kinetic energy
• Question 28
0 out of 3.3 points
Which of the following must be considered when adding or removing oil
from a refrigerator unit?
Selected Answer: [None Given]
Answers: a.
All of the other choices
b.
Watch crankcase pressure
c.
Use new oil
d.
Do not overcharge
• Question 29
0 out of 3.3 points
What is the use of back pressure regulating valve?
Selected [None Given]
Answer:
Answers: a.
Controls the evaporator temperature by throttling the flow of
liquid refrigerant
b.
Maintains a fixed pressure in the evaporator coils
c.
Controls the temperature in the evaporator coils
d.
Controls the flow of circulating water in the condenser tubes
• Question 30
0 out of 3.3 points
How many moisture be removed from air?
Selected Answer: [None Given]
Answers: a.
Condensation
b.
Absorption
c.
Condensation and absorption
d.
None of these
Tuesday, March 17, 2020 4:04:30 PM PST
• Question 1
0 out of 3.3 points
The following are standard characteristics of Freon -11 except:
Selected Answer: [None Given]
Answers: a.
Very volatile
b.
Non-toxic
c.
Boiling point over 200 F
d.
Separates from water
• Question 2
0 out of 3.3 points
If C is the capacity of the compressor in a refrigeration system and F is the
heat rejection factor, then calculate the condenser load, L.
Selected Answer: [None Given]
Answers: a.
L=C/F
b.
L=CxF
c.
L=F–C
d.
L=F/C
• Question 3
0 out of 3.3 points
Theoretically, what is the effect of the compressor clearance on horsepower?
Selected Answer: [None Given]
Answers: a.
decrease power
b.
there is no effect
c.
varies in direct proportion
d.
increase power
• Question 4
0 out of 3.3 points
How is one-ton plant described?
Selected Answer: [None Given]
Answers: a.
Make one ton of ice in 24 hours
b.
Remove the heat required to melt one ton of ice in 24 hours
c.
Remove one ton heat from the reefer box
d.
Melt one ton of ice in 24 hours
• Question 5
0 out of 3.3 points
A ____ boiler has water in the tubes and heat gases of combustion passing
through the tubes.
Selected Answer: [None Given]
Answers: a.
Fire box
b.
Water tube
c.
Cast iron sectional
d.
Fire tube
• Question 6
0 out of 3.3 points
When the dry bulb and the wet bulb temperature are identical, the air is said
to be:
Selected Answer: [None Given]
Answers: a.
Humidified
b.
Dehumidified
c.
Saturates
d.
Compressed
• Question 7
0 out of 3.3 points
A hot crankcase and cylinder head accompanied by a low suction pressure
would be caused by
Selected Answer: [None Given]
Answers: a.
Excess refrigerant
b.
Insufficient refrigerant
c.
Air in the system
d.
Stuck discharge valve
• Question 8
0 out of 3.3 points
The sum of the internal energy and the product of pressure and specific
volume is known as the:
Selected Answer: [None Given]
Answers: a.
entropy
b.
total internal energy
c.
enthalpy
d.
total work
• Question 9
0 out of 3.3 points
Air delivered to the room from the supply duct, moving at a velocity of 150
ft/min or more is called:
Selected Answer: [None Given]
Answers: a.
Secondary Air
b.
Primary Air
c.
Saturated Air
d.
Air Turbulence
• Question 10
0 out of 3.3 points
All of the following process can be found on a psychometric chart except:
Selected Answer: [None Given]
Answers: a.
black body radiation
b.
cooling and dehumidification
c.
heating and humidifying
d.
evaporative cooling
• Question 11
0 out of 3.3 points
Which of the following is the usual case of slugging?
Selected Answer: [None Given]
Answers: a.
Too much refrigerant in the system
b.
Too much oil in the system
c.
Expansion valve not operating properly
d.
Too much cooling water to condenser
• Question 12
0 out of 3.3 points
When the air is saturated the wet-bulb depression is:
Selected Answer: [None Given]
Answers: a.
100%
b.
unity
c.
zero
d.
indefinite
• Question 13
0 out of 3.3 points
The purpose of the low-pressure cutout switch is to:
Selected Answer: [None Given]
Answers: a.
Maintain a preset suction pressure to the compressor
b.
Cut out the compressor at a set pressure
c.
Cut compressor in and out at a preset pressure
d.
Maintain liquid refrigerant at the suction of the compressor
• Question 14
0 out of 3.3 points
Which of the following refrigerants has lowest freezing point temperature?
Selected Answer: [None Given]
Answers: a.
Freon 22
b.
Freon 12
c.
Ammonia
d.
Freon 11
• Question 15
0 out of 3.3 points
When coil surface is below the dew point temperature of the air.
Selected Answer: [None Given]
Answers: a.
vapor will evaporate through the air
b.
air is dehumidified
c.
moisture will condense out of the air
d.
air is humidified and saturated
• Question 16
0 out of 3.3 points
A burner should always start up in ____ fire and shut down in ____ fire
Selected Answer: [None Given]
Answers: a.
Low;low
b.
High; high
c.
Low;high
d.
High; low
• Question 17
0 out of 3.3 points
A scale trap in a Freon system will be found on the
Selected Answer: [None Given]
Answers: a.
Receiver
b.
Condenser
c.
Suction side
d.
Discharge side
• Question 18
0 out of 3.3 points
The thermal expansion valve responds to the
Selected Answer: [None Given]
Answers: a.
Pressure in the evaporator coils
b.
Amount of superheat in the vapor
c.
Amount of superheat in the liquid
d.
Temperature in the evaporator coils
• Question 19
0 out of 3.3 points
The solenoid valve is controlled by
Selected Answer: [None Given]
Answers: a.
The amount of liquid in the system
b.
The amount of gas in the system
c.
The temperature in the condenser
d.
The temperature in the icebox
• Question 20
0 out of 3.3 points
The faster way to remove frost from a direct expansion finned tube
evaporator is to:
Selected Answer: [None Given]
Answers: a.
Wash with warm water
b.
Shut down warm let frost melt
c.
Scrape off frost
d.
Send hot gas through the coil
• Question 21
0 out of 3.3 points
The sensible heat ratio is 0.8. that is
Selected Answer: [None Given]
Answers: a.
20% latent heat and 60% sensible heat
b.
80% sensible heat and 20% latent heat
c.
80% sensible heat and 20% sensible heat and latent heat
d.
20% latent het and 80% sensible heat and latent heat
• Question 22
0 out of 3.3 points
The back - pressure regulating valve:
Selected [None Given]
Answer:
Answers: a.
None of the other choices
b.
Controls the water flow through the condenser
c.
Stops the flow of liquid refrigerant when temperature drops in
evaporator
d.
Maintains a fixed pressure in the evaporator coils
• Question 23
0 out of 3.3 points
The force when applied to a mass of one kilogram will give mass an
acceleration of one meter per second for every second called:
Selected Answer: [None Given]
Answers: a.
Watt
b.
Joule
c.
Pascal
d.
Newton
• Question 24
0 out of 3.3 points
Is the subjective method in which the smoke density is visually compared to five
standardized white-black grids.
Selected Answer: [None Given]
Answers: a.
Dew point scale
b.
Smoke spot scale
c.
Ringelman scale
d.
Dalton scale
• Question 25
0 out of 3.3 points
In the discharge line between the compressor and the condenser one would
find:
Selected Answer: [None Given]
Answers: a.
High pressure, high temperature gas
b.
High pressure, low temperature gas
c.
High pressure, low temperature liquid
d.
High temperature, high pressure liquid
• Question 26
0 out of 3.3 points
A device for holding open the suction valve and drawing gas from the
suction manifold and returning it to the suction line without compressing it
is called _____.
Selected Answer: [None Given]
Answers: a.
Suction line by-pass
b.
Cylinder unloader
c.
Relief valve
d.
Discharge line by-pass
• Question 27
0 out of 3.3 points
What is the primary cause of smog formation?
Selected Answer: [None Given]
Answers: a.
Dioxins
b.
Oxidants
c.
Nitrogen oxides
d.
Toxins
• Question 28
0 out of 3.3 points
The expansion valve is located between the:
Selected Answer: [None Given]
Answers: a.
Compressor and condenser
b.
Condenser and evaporator
c.
Receiver and evaporator
d.
Evaporator and compressor
• Question 29
0 out of 3.3 points
Calcium chloride is sometimes used in refrigeration system as a:
Selected Answer: [None Given]
Answers: a.
Lubricant
b.
Refrigerant
c.
Secondary Coolant
d.
Primary Coolant
• Question 30
0 out of 3.3 points
How is ammonia system purged so that operator will not be overcome by
the fumes?
Selected Answer: [None Given]
Answers: a.
Into the bucket of water
b.
Back into the compressor
c.
Into the atmospheric line
d.
Into a bucket of lube oil
Tuesday, March 17, 2020 4:06:21 PM PST
• Question 1
0 out of 3.3 points
You want to change refrigerants in a Freon 12 plant using a reciprocating
single acting compressor which refrigerant requires the smallest amount of
change to the system?
Selected Answer: [None Given]
Answers: a.
Methyl chloride
b.
Aqua ammonia
c.
Ammonia
d.
Carbon dioxide
• Question 2
0 out of 3.3 points
An odorless refrigerant whose boiling point varies over a wide range of
temperatures.
Selected Answer: [None Given]
Answers: a.
Freon 22
b.
Freon11
c.
Freon 12
d.
Ammonia
• Question 3
0 out of 3.3 points
A leaky discharge valve can usually be detected by:
Selected Answer: [None Given]
Answers: a.
Any of the other choices
b.
A discharge pressure lower than normal
c.
A fluctuating high-pressure gauge
d.
A drop in icebox temperature
• Question 4
0 out of 3.3 points
Large-bulb alcohol thermometer used to measure air speed or atmospheric
conditions by means of cooling.
Selected Answer: [None Given]
Answers: a.
Kata Thermometer
b.
Wet Bulb Thermometer
c.
JJ Thompson Thermometer
d.
Kelvin Thermometer
• Question 5
0 out of 3.3 points
If the cooling water to the condenser suddenly fails:
Selected Answer: [None Given]
Answers: a.
An alarm will ring to notify the engineer
b.
The compressor will shutdown
c.
The solenoid valve will close
d.
The expansion valve will close
• Question 6
0 out of 3.3 points
The matching of scale model and full-scale results for a fluid dynamic
phenomena with a free surface requires equality of:
Selected Answer: [None Given]
Answers: a.
Cauchy number
b.
Weber number
c.
Froude number
d.
Reynolds number
• Question 7
0 out of 3.3 points
When air is saturated, the wet bulb depression is:
Selected Answer: [None Given]
Answers: a.
indefinite
b.
unity
c.
zero
d.
100%
• Question 8
0 out of 3.3 points
The combination of enthalpy and kinetic energy of fluid is termed as:
Selected Answer: [None Given]
Answers: a.
Stagnation enthalpy
b.
Latent enthalpy
c.
Heat enthalpy
d.
Throttling enthalpy
• Question 9
0 out of 3.3 points
The charging connection in a refrigerating system is located:
Selected Answer: [None Given]
Answers: a.
Between the condenser and the receiver
b.
Between the receiver the king valve
c.
Between the king valve and the solenoid valve
d.
Before the receiver
• Question 10
0 out of 3.3 points
The oil level in the compressor should be checked:
Selected Answer: [None Given]
Answers: a.
After a long period of operation
b.
While the compressor is in compressor
c.
Just before starting the compressor
d.
After an extended lay-up period
• Question 11
0 out of 3.3 points
The coefficient of velocity is approximately
Selected Answer: [None Given]
Answers: a.
0.20 to 0.40
b.
0.50 to 0.70
c.
0.40 to 0.60
d.
0.30 to 0.50
• Question 12
0 out of 3.3 points
A water conservation device which employs both air and water is condensed
a condenser and a cooling tower combined into one is called:
Selected Answer: [None Given]
Answers: a.
water-cooled condenser
b.
shell and tube condenser
c.
air-cooled condenser
d.
evaporative condenser
• Question 13
0 out of 3.3 points
The temperature in the dairy box should be approximately
Selected Answer: [None Given]
Answers: a.
30 to 40 deg. F
b.
0 to 10 deg. F
c.
20 to 30 deg. F
d.
10 to 20 deg f
• Question 14
0 out of 3.3 points
If m is the mass of dry air and H is the specific enthalpy of the water vapor in
air and r is the humidity ratio, then determine the latent heat of any dry air.
Selected Answer: [None Given]
Answers: a.
r / (H – m)
b.
m / (r – H)
c.
m (r x H)
d.
(m – r) / H
• Question 15
0 out of 3.3 points
The term used when the static pressure is equal to the atmospheric
pressure.
Selected Answer: [None Given]
Answers: a.
Induced draft
b.
Forced draft
c.
Balanced draft
d.
Natural draft
• Question 16
0 out of 3.3 points
In sensible heating cooling following parameter remains unchanged
a.
Selected Answer: [None Given]
Answers: a.
humidity ratio
b.
wet bulb temperature
c.
relative humidity
d.
dry bulb temperature
• Question 17
0 out of 3.3 points
Which of the following devices will prevent the relief valve from opening in
the event of excessive pressure?
Selected Answer: [None Given]
Answers: a.
High- pressure cutout switch
b.
Water failure switch
c.
Low –pressure cutout switch
d.
Any of the other choices
• Question 18
0 out of 3.3 points
An ammonia leak will turn litmus paper _________.
Selected Answer: [None Given]
Answers: a.
Blue
b.
Yellow
c.
Red
d.
Green
• Question 19
0 out of 3.3 points
What is the probable cause if a compressor runs continuously?
Selected Answer: [None Given]
Answers: a.
any of the other choices
b.
faulty cooling water valve
c.
insufficient refrigerant
d.
a clogged condenser
• Question 20
0 out of 3.3 points
All of the ff occur during reduction of a substance except
Selected Answer: [None Given]
Answers: a.
An oxidation state decrease
b.
Reduction of the oxidizing agent
c.
An increase in negative charge
d.
Loss of electrons
• Question 21
0 out of 3.3 points
Which do you think is very important in adjusting compressor “v” belts?
Selected Answer: [None Given]
Answers: a.
Make it as tight as possible
b.
Allow ½” slack
c.
Keep belts parallel
d.
Make belt just tight enough to turn pulley
• Question 22
0 out of 3.3 points
The removal of dissolved gas or other volatile component from liquid by exposing
the liquid to air or steam is known as:
Selected Answer: [None Given]
Answers: a.
Stripping
b.
Gas absorption
c.
Spraying
d.
Scrubbing
• Question 23
0 out of 3.3 points
Swimming pool water needs
Selected Answer: [None Given]
Answers: a.
de-chlorination
b.
dual-chlorination
c.
super-chlorination
d.
pre-chlorination
• Question 24
0 out of 3.3 points
As a good practical rule, the foundation depth may be taken as how many
times of the engine stroke?
Selected Answer: [None Given]
Answers: a.
3.2 to 4:2
b.
2.2 to 3:2
c.
3:5 to 4:5
d.
2:5 to 3:5
• Question 25
0 out of 3.3 points
Are highly concentrated liquid wasted produced in landfills
Selected Answer: [None Given]
Answers: a.
Urea
b.
Leachates
c.
Nitrogen dioxide
d.
Aerosols
• Question 26
0 out of 3.3 points
What do you call the storage tank for liquid refrigerant?
Selected Answer: [None Given]
Answers: a.
Any of the other choices
b.
Purging
c.
Receiver tank
d.
Charging tank
• Question 27
0 out of 3.3 points
When the evaporator coils are located in the icebox, the system is known as:
Selected Answer: [None Given]
Answers: a.
High-pressure system
b.
Low-pressure system
c.
Indirect system
d.
Direct system
• Question 28
0 out of 3.3 points
All of the following temperature have meaning in psychometrics except:
Selected Answer: [None Given]
Answers: a.
dew point
b.
dry-bulb temperature
c.
wet-bulb temperature
d.
adiabatic wall temperature
• Question 29
0 out of 3.3 points
One disadvantage of a co2 system is the fact that
Selected [None Given]
Answer:
Answers: a.
It is difficult to condense the refrigerant if the circulating water
temperature is too high
b.
It takes more refrigerant to keep the iceboxes cold
c.
Due to high pressure it is difficult to keep oil from mixing with
the refrigerant
d.
It is difficult to condense the refrigerant if the circulating water
temperature is too low
• Question 30
0 out of 3.3 points
If the air compressed without discarding heat, then what do you call this kind
of compression:
Selected Answer: [None Given]
Answers: a.
isobaric
b.
adiabatic
c.
isothermal
d.
isochoric
Tuesday, March 17, 2020 4:09:30 PM PST
• Question 1
0 out of 3.3 points
A Freon – 12 gage shows pressure and ______.
Selected Answer: [None Given]
Answers: a.
Superheat temperature
b.
Saturation temperature
c.
Back pressure
d.
Vacuum
• Question 2
0 out of 3.3 points
When heavy electrical currents are involved, the thermostat will be operated
by a
Selected Answer: [None Given]
Answers: a.
Relay
b.
Small circuit breaker
c.
Pressure pipe
d.
Fusetron
• Question 3
0 out of 3.3 points
A refrigeration system in which only part of the refrigerant passes over the
heat transfer surface is evaporated and the balance is separated from the
vapor and recirculated
Selected Answer: [None Given]
Answers: a.
Direct expansion system
b.
Chilled water system
c.
Flooded system
d.
Multiple system
• Question 4
0 out of 3.3 points
Low suction pressure is caused by:
Selected Answer: [None Given]
Answers: a.
Expansion valve causing flooding back
b.
Solenoid valve not functioning properly
c.
Air in the system
d.
Leaky compressor suction valves
• Question 5
0 out of 3.3 points
Where does the final removal of water vapor in an absorption refrigeration
system occur?
Selected Answer: [None Given]
Answers: a.
condenser
b.
analyzer
c.
generator
d.
rectifier
• Question 6
0 out of 3.3 points
If the pressure is disregarded in the various other components of a steam
gas power plants, the pressure rise in the pump or compressor is ______?
Selected Answer: [None Given]
Answers: a.
inversely proportional
b.
varying
c.
constant
d.
equal
• Question 7
0 out of 3.3 points
The kinetic energy of a moving fluid is used to isentropically compressed the
fluid to state of zero velocity. The temperature of a moving fluid at the state
zero velocity is called:
Selected Answer: [None Given]
Answers: a.
stagnation temperature
b.
critical temperature
c.
partial temperature
d.
absolute temperature
• Question 8
0 out of 3.3 points
Palladium Chloride may be used to measure the presence of:
Selected Answer: [None Given]
Answers: a.
ammonia
b.
refrigerant
c.
CO
d.
vapor
• Question 9
0 out of 3.3 points
All of the ff statements are characteristics of bases except
Selected Answer: [None Given]
Answers: a.
They turn red litmus paper blue
b.
They have a pH between 0 and 7
c.
They neutralize acids forming salts and water
d.
They conduct electricity in aqueous solutions
• Question 10
0 out of 3.3 points
A temperature measurement in an ordinary thermometer which has
constant specific humidity.
Selected Answer: [None Given]
Answers: a.
critical temperature
b.
dry bulb temperature
c.
wet bulb temperature
d.
dew point temperature
• Question 11
0 out of 3.3 points
The coefficient of velocity is approximately
Selected Answer: [None Given]
Answers: a.
0.40 to 0.60
b.
0.30 to 0.50
c.
0.20 to 0.40
d.
0.50 to 0.70
• Question 12
0 out of 3.3 points
What is the least number of compressors a multistage system that will use?
Selected Answer: [None Given]
Answers: a.
three
b.
one
c.
four
d.
two
• Question 13
0 out of 3.3 points
The solenoid valve is:
Selected Answer: [None Given]
Answers: a.
A temperature-controlled stop valve
b.
A pressure-controlled stop valve
c.
None of the other choices
d.
A Freon-controlled check valve
• Question 14
0 out of 3.3 points
If the solenoid valve closed by accident, the compressor would be stopped
by which of the following?
Selected Answer: [None Given]
Answers: a.
Low-pressure cutout switch
b.
Low-water cutout switch
c.
Automatic trip
d.
High-pressure cutout switch
• Question 15
0 out of 3.3 points
Calcium chloride is sometimes used in refrigeration system as a:
Selected Answer: [None Given]
Answers: a.
Primary Coolant
b.
Lubricant
c.
Secondary Coolant
d.
Refrigerant
• Question 16
0 out of 3.3 points
Which is not commonly used to cool and dehumidify equipment?
Selected Answer: [None Given]
Answers: a.
sodium zeolite
b.
calcium chloride
c.
activated alumina
d.
silica gel
• Question 17
0 out of 3.3 points
The normal cut-out setting of a window unit thermostat is between:
Selected Answer: [None Given]
Answers: a.
10C to 15C
b.
17C to 22C
c.
13C to 16 C
d.
16 C to 21 C
• Question 18
0 out of 3.3 points
Any foreign matter in the coal feed mechanism of the screw-feed stoker is
best removed by:
Selected Answer: [None Given]
Answers: a.
Emptying the coal hopper
b.
Reversing the stroker
c.
Forcing it through with a heaver shear pin
d.
Using the cutoff gate at the bottom of the hopper
• Question 19
0 out of 3.3 points
A dehumidifier is usually a small hermitic refrigerating system. It has both a
condenser and an evaporator. Many older systems use R – 12 or R – 500. The
newer units are:
Selected Answer: [None Given]
Answers: a.
R – 145a
b.
R - 134a
c.
R – 217a
d.
R – 121a
• Question 20
0 out of 3.3 points
In pumped storage plant
Selected [None Given]
Answer:
Answers: a.
Pressure accumulators are used
b.
Water is stored by pumping to high pressures
c.
Power is produced by pumps
d.
Downstream water is pumped upstream during offload
periods
• Question 21
0 out of 3.3 points
If the compressor were to run out continuously without lowering the
temperature, the trouble would probably be:
Selected Answer: [None Given]
Answers: a.
Insufficient refrigerant in the system
b.
Leaks in the system
c.
Any of the other choices
d.
Leaky discharge valve
• Question 22
0 out of 3.3 points
The ratio of the rated cooling capacity divided by the amount of electrical
power used:
Selected Answer: [None Given]
Answers: a.
Cooling efficiency (CE)
b.
Energy cooling ratio (ECR)
c.
Energy efficiency index (EEI)
d.
Energy efficiency ratio (EER)
• Question 23
0 out of 3.3 points
Which of the following is a type of evaporator?
Selected Answer: [None Given]
Answers: a.
shell-and-tube
b.
sell-and-tube-water cooler
c.
polyphase motors
d.
oil lantern rings
• Question 24
0 out of 3.3 points
What is the reason why a thermometer in vapor compression system is
installed close to the compressor?
Selected [None Given]
Answer:
Answers: a.
Because it helps the operator in adjusting compressor for
greatest efficiency
b.
Because the performance of other units of the cycle can be
controlled
c.
Because temperature helps in calculating the cop
d.
Because temperature indicates whether liquid or vapor
refrigerant is going to compressor
• Question 25
0 out of 3.3 points
The velocity of radiation (v) and the frequency of radiations (f) are related to the
wavelength of radiations by:
Selected Answer: [None Given]
Answers: a.
v2f
b.
vf
c.
f/v
d.
v/f
• Question 26
0 out of 3.3 points
Are by products of reaction between combustion products
Selected Answer: [None Given]
Answers: a.
Photochemicals
b.
Organics
c.
Oxidants
d.
Sediments
• Question 27
0 out of 3.3 points
The fraction of the radiation energy incident on a surface which is absorbed
by the surface is called:
Selected Answer: [None Given]
Answers: a.
Absorptivity
b.
Convection
c.
Radiation
d.
Emission
• Question 28
0 out of 3.3 points
In order to remove the fly ashes from the flue gas, which of the following
must a power plant be equipped with?
Selected Answer: [None Given]
Answers: a.
electrostatic precipitator
b.
desulphurization plant
c.
condenser
d.
demineralizer
• Question 29
0 out of 3.3 points
The expansion valve does not seem to be operating properly. There is high
superheat. Test by listening to the sound of Freon flooding through the
tubes and _______.
Selected Answer: [None Given]
Answers: a.
Any of the other choices
b.
Place bulb in hot water
c.
Warm bulb with hand
d.
Place bulb in cold water
• Question 30
0 out of 3.3 points
What is the combination of a wet and dry bulb thermometer is called a
Selected Answer: [None Given]
Answers: a.
Hygrometer
b.
Psychrometer
c.
Hygrometer or psychrometer
d.
Hydrometer
Tuesday, March 17, 2020 4:11:22 PM PST
• Question 1
0 out of 3.3 points
The removal of particulate matter from a gas flow by exposing the flow to a liquid
or slurry is known as:
Selected Answer: [None Given]
Answers: a.
Scrubbing
b.
Stripping
c.
Spraying
d.
Absorption
• Question 2
0 out of 3.3 points
When excess solute in a solution settles to the bottom of the container, the process
is called:
Selected Answer: [None Given]
Answers: a.
Precipitation
b.
Salvation
c.
Deemulsification
d.
Equation
• Question 3
0 out of 3.3 points
The fact that the amount of slightly soluble gas absorbed in a liquid is proportional
to the partial pressure of the gas is known as:
Selected Answer: [None Given]
Answers: a.
Dalton’s law
b.
Henry’s law
c.
Boyle’s law
d.
Raoult’s law
• Question 4
0 out of 3.3 points
An impulse turbine are used for:
Selected Answer: [None Given]
Answers: a.
High head
b.
Medium head
c.
Low head
d.
Very low head
• Question 5
0 out of 3.3 points
The most likely cause of high superheat would be:
Selected Answer: [None Given]
Answers: a.
Back-pressure valve set too high
b.
Expansion valve closed too much
c.
Too much refrigerant
d.
Expansion valve open too wide
• Question 6
0 out of 3.3 points
The expansion valve does not seem to be operating properly. There is high
superheat. Test by listening to the sound of Freon flooding through the
tubes and _______.
Selected Answer: [None Given]
Answers: a.
Place bulb in hot water
b.
Any of the other choices
c.
Place bulb in cold water
d.
Warm bulb with hand
• Question 7
0 out of 3.3 points
During sensible heating, the humidity remains constant but the relative
humidity.
Selected Answer: [None Given]
Answers: a.
remains constant
b.
increases
c.
zero
d.
decreases
• Question 8
0 out of 3.3 points
All of the following process can be found on a psychometric chart except:
Selected Answer: [None Given]
Answers: a.
black body radiation
b.
evaporative cooling
c.
cooling and dehumidification
d.
heating and humidifying
• Question 9
0 out of 3.3 points
Too low suction pressure could be caused by:
Selected Answer: [None Given]
Answers: a.
Dirty scale traps
b.
Too much oil in the system
c.
Any of the other choices
d.
Shortage of refrigerant gas
• Question 10
0 out of 3.3 points
In turbine installation, the air is removed from condenser by
Selected Answer: [None Given]
Answers: a.
Air pump
b.
Air injector
c.
Air jet
d.
Air ejector
• Question 11
0 out of 3.3 points
If there were a 15 F to 20 F temperature differential between the
temperature corresponding to the pressure at the compressor discharge
and the temperature at the condenser outlet, it would probably indicate the
need for __________.
Selected Answer: [None Given]
Answers: a.
More circulating water
b.
More refrigerant
c.
Purging the system
d.
Less circulating
• Question 12
0 out of 3.3 points
The function of a turbine is to:
Selected Answer: [None Given]
Answers: a.
Increase the total energy content of the flow
b.
Transfer heat from one fluid to another
c.
Extract energy from the flow
d.
Exchange heat to increase energy to the flow
• Question 13
0 out of 3.3 points
One foot water is equal to:
Selected Answer: [None Given]
Answers: a.
0.4138 lb/in2
b.
68.3 lb/in2
c.
62.43 lb/in2
d.
0.8673 lb/in2
• Question 14
0 out of 3.3 points
The vapor cycle in thermal eff’y as the refrigeration cycle to the
Selected Answer: [None Given]
Answers: a.
Energy efficiency ratio
b.
Carnot efficiency
c.
COP for a refrigerator
d.
COP for a heat pump
• Question 15
0 out of 3.3 points
The solenoid valve can be typed as a ___________.
Selected Answer: [None Given]
Answers: a.
Bimetallic valve
b.
Bellows valve
c.
Magnetic soap valve
d.
Thermal valve
• Question 16
0 out of 3.3 points
Aeration of water is done for all of the ff purposes except for:
Selected Answer: [None Given]
Answers: a.
Increasing the amount of oxygen in water
b.
Removal of bad taste
c.
Removal of temporary hardness
d.
Removal of CO2
• Question 17
0 out of 3.3 points
The coefficient of the velocity, Cv accounts for the:
Selected [None Given]
Answer:
Answers: a.
Changes in diameters of converging pipe
b.
Effects of compressibility
c.
Small effect of friction and turbulence of the orifice
d.
Effects on the flow area of contraction, friction, and
turbulence
• Question 18
0 out of 3.3 points
What is the cause of pressure drop in the boiler, condenser and the pipings
between different components? Because of this drop, steam leaving the
boiler at a lower pressure.
Selected Answer: [None Given]
Answers: a.
fluid friction
b.
negative slip
c.
positive slip
d.
low thermal efficiency
• Question 19
0 out of 3.3 points
The refrigerant temperature is at its maximum just before it enters the
_______
Selected Answer: [None Given]
Answers: a.
Condenser
b.
Expansion valve
c.
Evaporator
d.
Compressor
• Question 20
0 out of 3.3 points
What do you call the mixture if the solute particles of a solid suspended in a liquid
are larger than molecules?
Selected Answer: [None Given]
Answers: a.
Hydration
b.
Saturated solution
c.
Suspension
d.
Solution
• Question 21
0 out of 3.3 points
If frost forms on the cylinders, the cause would be:
Selected Answer: [None Given]
Answers: a.
Expansion valve not open wide enough
b.
Charging valve left open
c.
Expansion valve open too wide
d.
Dehydrator not working properly
• Question 22
0 out of 3.3 points
All of the following temperature have meaning in psychometrics except:
Selected Answer: [None Given]
Answers: a.
adiabatic wall temperature
b.
wet-bulb temperature
c.
dry-bulb temperature
d.
dew point
• Question 23
0 out of 3.3 points
When ordering an expansion valve which of the following information is
necessary?
Selected Answer: [None Given]
Answers: a.
Size, and pressure
b.
Pressure and temperature
c.
Size and tonnage
d.
Size ,tonnage , temperature and pressure
• Question 24
0 out of 3.3 points
The ratio of maximum load to the rated plant capacity is called
Selected Answer: [None Given]
Answers: a.
Maximum load factor
b.
Capacity
c.
Load factor
d.
Utilization factor
• Question 25
0 out of 3.3 points
When the outlet temperature of the evaporator exceeds the inlet
temperature the condition is called ______________.
Selected Answer: [None Given]
Answers: a.
Melting
b.
Boiling
c.
Superheating
d.
Freezing
• Question 26
0 out of 3.3 points
How many moisture be removed from air?
Selected Answer: [None Given]
Answers: a.
Condensation
b.
Absorption
c.
Condensation and absorption
d.
None of these
• Question 27
0 out of 3.3 points
TOR is a unit equivalent to:
Selected Answer: [None Given]
Answers: a.
2545 Btu/ hr
b.
12.66 kN-m/hr
c.
3413 KW- hr
d.
50.4 kCal/ sec
• Question 28
0 out of 3.3 points
When the air is saturated the wet-bulb depression is:
Selected Answer: [None Given]
Answers: a.
unity
b.
100%
c.
indefinite
d.
zero
• Question 29
0 out of 3.3 points
A water-tube condenser has a total of 60 tubes. If these two passes, then
compute the number of tubes per pass.
Selected Answer: [None Given]
Answers: a.
6
b.
15
c.
3
d.
2
• Question 30
0 out of 3.3 points
What is the reserve capacity of a plant?
Selected Answer: [None Given]
Answers: a.
Plant capacity x (1-load factor)
b.
Plant capacity- average demand
c.
Maximum demand-average demand
d.
Plant capacity - maximum demand
Tuesday, March 17, 2020 4:12:31 PM PST
• Question 1
0 out of 3.3 points
Subcooling of the refrigerant results in :
Selected Answer: [None Given]
Answers: a.
Less circulating water needed
b.
Liquid less likely to vaporize
c.
Effect of refrigerant increased and liquid less likely to vaporize
d.
Effect of refrigerant increased
• Question 2
0 out of 3.3 points
What tonnage of refrigerating machine is required if the refrigerating system
extracted 48,000 btu per hour?
Selected Answer: [None Given]
Answers: a.
3 tons
b.
4 tons
c.
2 tons
d.
5 tons
• Question 3
0 out of 3.3 points
Most people are comfortable with the relative humidity of:
Selected Answer: [None Given]
Answers: a.
20 % to 30%
b.
30 % to 70 %
c.
10 % to 40 %
d.
40 % to 80 %
• Question 4
0 out of 3.3 points
Which of the ff is a cold cathode lamp?
Selected Answer: [None Given]
Answers: a.
Low pressure mercury vapor lamp
b.
Neon lamp
c.
Sodium vapor lamp
d.
High pressure mercury vapor lamp
• Question 5
0 out of 3.3 points
The relief valve is located on the:
Selected Answer: [None Given]
Answers: a.
Discharge side of the compressor
b.
Receiver tank
c.
Outlet of the evaporator coils
d.
Discharge side on the condenser
• Question 6
0 out of 3.3 points
The ozone concentration of 0.10 parts per million (ppm) is generally
considered the maximum permissible for how many hours exposure?
Selected Answer: [None Given]
Answers: a.
3 hours
b.
4 hours
c.
7 hours
d.
8 hours
• Question 7
0 out of 3.3 points
Which of the following items is not important when using a halide torch?
Selected Answer: [None Given]
Answers: a.
Adjust to a clear white flame
b.
Adjust to a clear blue flame
c.
Move flame slowly around the joints
d.
Hold flame close to the joints
• Question 8
0 out of 3.3 points
A single trunk piston-type compressor is undesirable for a Freon unit
because the:
Selected Answer: [None Given]
Answers: a.
Lubricant temperature becomes excessive
b.
Refrigerant reduces the crankcase pressure
c.
Lubricant mixes with the refrigerant
d.
Refrigerant becomes superheated
• Question 9
0 out of 3.3 points
Copper tubing is used in Freon system because:
Selected Answer: [None Given]
Answers: a.
It is easier to check leaks
b.
Iron corrodes
c.
It is cheaper
d.
It has less internal resistance
• Question 10
0 out of 3.3 points
Which of the following acts as ignition accelerator for internal combustion engine
fuels?
Selected Answer: [None Given]
Answers: a.
Hydrogen peroxide
b.
Acetone peroxide
c.
Aromatic compounds
d.
N-heptane
• Question 11
0 out of 3.3 points
Combustion is the rapid burning of fuel and oxygen resulting in:
Selected Answer: [None Given]
Answers: a.
Release of heat
b.
Nitrogen and oxygen
c.
Oxidation
d.
Release of steam
• Question 12
0 out of 3.3 points
What do you call the storage tank for liquid refrigerant?
Selected Answer: [None Given]
Answers: a.
Purging
b.
Receiver tank
c.
Charging tank
d.
Any of the other choices
• Question 13
0 out of 3.3 points
The ratio of the rated cooling capacity divided by the amount of electrical
power used:
Selected Answer: [None Given]
Answers: a.
Energy cooling ratio (ECR)
b.
Energy efficiency ratio (EER)
c.
Cooling efficiency (CE)
d.
Energy efficiency index (EEI)
• Question 14
0 out of 3.3 points
Everything is in its normal operating position when charging a system except
the
Selected Answer: [None Given]
Answers: a.
Compressor discharge valve
b.
Solenoid valve
c.
King (receiver discharge) valve
d.
Purge valve
• Question 15
0 out of 3.3 points
The capacity of a centrifugal type compressor is controlled by which of the
following?
Selected Answer: [None Given]
Answers: a.
Regulating the suction pressure
b.
Regulating the discharge pressure
c.
Regulating the speed
d.
Regulating the speed or regulating the suction pressure
• Question 16
0 out of 3.3 points
In the absence of any heat and work interactions and any changes in
potential energy, the stagnation enthalpy of a fluid remains constant during
Selected Answer: [None Given]
Answers: a.
Variable flow
b.
Turbulent flow
c.
Unsteady flow
d.
Steady flow
• Question 17
0 out of 3.3 points
What usually happens if the specific gravity of the brine is too low?
Selected Answer: [None Given]
Answers: a.
It will be more heat-absorbing
b.
All of the other choices
c.
Solids will deposits
d.
The brine will freeze
• Question 18
0 out of 3.3 points
Which of the ff is not a sound absorptive material?
Selected Answer: [None Given]
Answers: a.
Carpets
b.
Rugs
c.
Mirrors
d.
Heavey drapes
• Question 19
0 out of 3.3 points
The volume flow passes through a venturimeter is:
Selected Answer: [None Given]
Answers: a.
Increasing
b.
Decreasing
c.
Constant
d.
Varying
• Question 20
0 out of 3.3 points
Closing the solenoid valve will stop the compressor through the ___________.
Selected Answer: [None Given]
Answers: a.
Low-pressure cutout switch
b.
Low water-pressure cutout switch
c.
Bypass relief valve
d.
High-pressure cutout switch
• Question 21
0 out of 3.3 points
A salimeter reads the
Selected Answer: [None Given]
Answers: a.
Rate of brine
b.
Relative humidity
c.
Dew point temperature
d.
Density of brine
• Question 22
0 out of 3.3 points
Absolute zero on the Fahrenheit Scale is equal to:
Selected Answer: [None Given]
Answers: a.
-273
b.
-460
c.
0
d.
-100
• Question 23
0 out of 3.3 points
A substance with surface area per unit of weight, and intricate pore structure, and a
hydrophobic surface
Selected Answer: [None Given]
Answers: a.
Homogeneous substance
b.
Activated substance
c.
Adsorbent substance
d.
Adsorbent substance
• Question 24
0 out of 3.3 points
Air can be prevented from getting into the system by:
Selected [None Given]
Answer:
Answers: a.
Keeping all glands and stuffing boxes on the low-pressure side
tight
b.
Running the refrigerant through an aerator
c.
Keeping all glands and stuffing boxes on the high- pressure
side tight
d.
Keeping the dehydrator clean at all times
• Question 25
0 out of 3.3 points
The matching model and full-scale prototype results a fluid dynamic
phenomena involving a fully submerged body requires equality of:
Selected Answer: [None Given]
Answers: a.
Froude number
b.
Cauchy number
c.
Reynolds number
d.
Weber number
• Question 26
0 out of 3.3 points
A vena contracta in a fluid jet issuing through a hole in a plate is located
approximately:
Selected Answer: [None Given]
Answers: a.
At the orifice minimum diameter
b.
At jet’s minimum diameter
c.
10 diameters downstream of the hole
d.
At the orifice maximum diameter
• Question 27
0 out of 3.3 points
An insulator with superior tensile strength but low heat resistance
Selected Answer: [None Given]
Answers: a.
Asbestos
b.
Fiberglass
c.
Rubber
d.
Plastic
• Question 28
0 out of 3.3 points
Flow measuring devices include all of the following except:
Selected Answer: [None Given]
Answers: a.
Flow nozzles
b.
Pitot tubes
c.
Venture area meters
d.
Precision tachometers
• Question 29
0 out of 3.3 points
Large-bulb alcohol thermometer used to measure air speed or atmospheric
conditions by means of cooling.
Selected Answer: [None Given]
Answers: a.
JJ Thompson Thermometer
b.
Kelvin Thermometer
c.
Kata Thermometer
d.
Wet Bulb Thermometer
• Question 30
0 out of 3.3 points
The ratio of the sum individual maximum demands of the system to the maximum
demand of the whole system is:
Selected Answer: [None Given]
Answers: a.
Power factor
b.
Diversity factor
c.
Utilization factor
d.
Demand factor
Tuesday, March 17, 2020 4:15:38 PM PST
• Question 1
0 out of 3.3 points
The operation that produces highest noise level is:
Selected Answer: [None Given]
Answers: a.
Machining
b.
Pressing
c.
Riveting
d.
Wielding
• Question 2
0 out of 3.3 points
The total pressure supplied by the fan at maximum operating conditions
Selected Answer: [None Given]
Answers: a.
Available draft
b.
Net rating or fan boost
c.
Stack effect
d.
Draft loss
• Question 3
0 out of 3.3 points
Venturi meters, pitot static gauges, orifice meters, flow nozzles, and
differential manometers all depend upon the relationship between:
Selected Answer: [None Given]
Answers: a.
Flow velocity and pressure
b.
Friction and pressure
c.
Pressure and mass flow
d.
Flow velocity and friction
• Question 4
0 out of 3.3 points
The elements of a thermostat switch are usually of the _________.
Selected Answer: [None Given]
Answers: a.
Valve type
b.
Diaphragm type
c.
Pilot-valve type
d.
Bimetal type
• Question 5
0 out of 3.3 points
The crossover connection in an ammonia system can be used to _________.
Selected Answer: [None Given]
Answers: a.
Pump air out of the system
b.
Add refrigerant to the system
c.
Reduce the pressure on the discharge side of the condenser
d.
Hot-gas defrost
• Question 6
0 out of 3.3 points
What do you call the device that is used as a low- pressure control and high-
pressure cutout on a compressor?
Selected Answer: [None Given]
Answers: a.
Controller switch
b.
Cutout
c.
Pressure controller
d.
Cutout switch
• Question 7
0 out of 3.3 points
Where does the final removal of water vapor in an absorption refrigeration
system occur?
Selected Answer: [None Given]
Answers: a.
analyzer
b.
condenser
c.
generator
d.
rectifier
• Question 8
0 out of 3.3 points
A popular air-conditioning system that includes a single outdoor condenser,
three independent evaporators, and individual evaporator temperature
control. The condensing unit is located outside on a slab. They are frequently
used in legal and medical offices, motels an homes with ducts.
Selected Answer: [None Given]
Answers: a.
window type air conditioner
b.
package terminal air conditioner
c.
console air conditioner
d.
multizone ductless split system
• Question 9
0 out of 3.3 points
The primary purpose of a turbine in a fluid is to:
Selected Answer: [None Given]
Answers: a.
Add mass to the flow
b.
None of the other choices
c.
Add energy to the flow
d.
Extract energy from the flow
• Question 10
0 out of 3.3 points
Which of the following materials is suitable for tubing in refrigeration
application where refrigerant ammonia is employed
Selected Answer: [None Given]
Answers: a.
Brass
b.
Plastic
c.
Steel
d.
Copper
• Question 11
0 out of 3.3 points
Saturation temperature is the same as;
Selected Answer: [None Given]
Answers: a.
vapor temperature
b.
steam temperature
c.
dew point
d.
humidity
• Question 12
0 out of 3.3 points
A bull’s eye in a full liquid line will appear _________.
Selected Answer: [None Given]
Answers: a.
Latent
b.
Cloudy or latent
c.
Clear
d.
Cloudy
• Question 13
0 out of 3.3 points
A hot suction line could be caused by:
Selected Answer: [None Given]
Answers: a.
Insufficient cooling water
b.
Excess refrigerant
c.
Excess cooling water
d.
Insufficient refrigerant
• Question 14
0 out of 3.3 points
Which of the following would cause low head pressure?
Selected Answer: [None Given]
Answers: a.
Insufficient cooling water
b.
Insufficient refrigerant gas
c.
Too much cooling water and/or insufficient refrigerant gas
d.
Too much cooling water
• Question 15
0 out of 3.3 points
Most stacks are built of height less than __________.
Selected Answer: [None Given]
Answers: a.
40 m
b.
50 m
c.
60 m
d.
30 m
• Question 16
0 out of 3.3 points
Combined process of cooling and humidifying is also known as:
Selected Answer: [None Given]
Answers: a.
evaporative cooling process
b.
heating and humidifying
c.
cooling tower
d.
moisture removal process
• Question 17
0 out of 3.3 points
External frost on inlet of expansion valve indicates:
Selected Answer: [None Given]
Answers: a.
Expansion valve plugged or dirty
b.
Air in system
c.
Head pressure too high
d.
Refrigerating compartment too cold
• Question 18
0 out of 3.3 points
The amount of heat necessary to bring up temperature of a unit mass of a
substance through unit degree is called:
Selected Answer: [None Given]
Answers: a.
specific volume
b.
total heat
c.
thermal heat
d.
specific heat
• Question 19
0 out of 3.3 points
The flow of a convergent section of a nozzle is always subsonic. If the flow is
subsonic then the mach number is:
Selected Answer: [None Given]
Answers: a.
greater than unity
b.
less than unity
c.
near than unity
d.
unity
• Question 20
0 out of 3.3 points
A bell coleman cycle is also known as
Selected Answer: [None Given]
Answers: a.
Reversed Rankine cycle
b.
Reversed Otto cycle
c.
Reversed Joule cycle
d.
Reversed Carnot cycle
• Question 21
0 out of 3.3 points
An airconditioning system which is a combinaton of heating and cooling
system. It is designed to serve an individual room or zone.
Selected Answer: [None Given]
Answers: a.
packaged terminal air-conditioner
b.
console air conditioner
c.
multizone ductless split system
d.
window type air conditioner
• Question 22
0 out of 3.3 points
What tonnage of refrigerating machine is required if the refrigerating system
extracted 48,000 btu per hour?
Selected Answer: [None Given]
Answers: a.
4 tons
b.
5 tons
c.
3 tons
d.
2 tons
• Question 23
0 out of 3.3 points
As the steam pressure increases the steam temperature:
Selected Answer: [None Given]
Answers: a.
Decreased
b.
Fluctuates
c.
Increases
d.
Remains the same
• Question 24
0 out of 3.3 points
The other name for Swamp Cooling is:
Selected Answer: [None Given]
Answers: a.
Condensate Cooling
b.
Excelsior Cooling
c.
Wet Roof Cooling
d.
Evaporative Cooling
• Question 25
0 out of 3.3 points
The greatest decrease in refrigerant temperature occurs in the _________
Selected Answer: [None Given]
Answers: a.
Condenser
b.
Evaporator or condenser
c.
Evaporator
d.
Compressor
• Question 26
0 out of 3.3 points
All of the ff are units of energy except
Selected Answer: [None Given]
Answers: a.
MeV
b.
Joules
c.
Pascals
d.
Calories
• Question 27
0 out of 3.3 points
A pressure controller is usually operated by the movement of a
Selected Answer: [None Given]
Answers: a.
Siphon
b.
Diaphragm
c.
Bellows or diaphragm
d.
Bellows
• Question 28
0 out of 3.3 points
If the average temperature of liquid of power cycle during heat addition
process is as high as possible. Then the thermal efficiency of the cycle will:
Selected Answer: [None Given]
Answers: a.
increase
b.
zero
c.
remain constant
d.
decrease
• Question 29
0 out of 3.3 points
The purpose of the scale trap is to:
Selected Answer: [None Given]
Answers: a.
Remove insoluble gases from the refrigerant
b.
Control the amount of scale going to the compressor
c.
Remove dirt, scale and metal chips from the refrigerant
d.
Dissolved scale and dirt in the system
• Question 30
0 out of 3.3 points
What is the specific humidity of dry air?
Selected Answer: [None Given]
Answers: a.
50
b.
0
c.
20
d.
100
Tuesday, March 17, 2020 4:44:07 PM PST
• Question 1
0 out of 3.3 points
Which of the following is the cause if the outlet of a thermostatic valve is
warmer than the inlet side?
Selected Answer: [None Given]
Answers: a.
Valve is working properly
b.
Valve is not working properly
c.
None of the other choices
d.
Solenoid valve is not working properly
• Question 2
0 out of 3.3 points
What consists of weak solutions of sulfuric, hydrochloric, and to a lesser extent,
nitric acids?
Selected Answer: [None Given]
Answers: a.
pollutant
b.
Stack gas
c.
Acid rain
d.
Acid compound
• Question 3
0 out of 3.3 points
A ____ boiler has heat and gases of combustion that pass through tubes
surrounded by water
Selected Answer: [None Given]
Answers: a.
Water tube
b.
Cast iron sectional
c.
Fire tube
d.
Straight-tube
• Question 4
0 out of 3.3 points
In the hydro-electric power plant having a medium head and using a Francis
turbine, the turbine speed may be regulated through:
Selected Answer: [None Given]
Answers: a.
Deflector gate
b.
Forebay
c.
Wicket gate
d.
Nozzle
• Question 5
0 out of 3.3 points
The matching of scale model and full-scale results for a fluid dynamic
phenomena with a free surface requires equality of:
Selected Answer: [None Given]
Answers: a.
Froude number
b.
Reynolds number
c.
Weber number
d.
Cauchy number
• Question 6
0 out of 3.3 points
When excess solute in a solution settles to the bottom of the container, the process
is called:
Selected Answer: [None Given]
Answers: a.
Equation
b.
Salvation
c.
Precipitation
d.
Deemulsification
• Question 7
0 out of 3.3 points
Flash chamber is installed in the refrigeration circuit so as to:
Selected [None Given]
Answer:
Answers: a.
Reduce pressure losses through the evaporator
b.
Improve overall heat transfer coefficient
c.
Reduce the size of evaporator by avoiding vapors going to
evaporator
d.
All of the other choices
• Question 8
0 out of 3.3 points
The angle between the stack and the guy wire is usually
Selected Answer: [None Given]
Answers: a.
75 degrees
b.
30 degrees
c.
60 degrees
d.
45 degrees
• Question 9
0 out of 3.3 points
The purge valve is located:
Selected Answer: [None Given]
Answers: a.
On the receiver discharge
b.
In the lowest part of the system
c.
In the highest part of the system
d.
On the evaporator coils
• Question 10
0 out of 3.3 points
Some causes of a noisy compressor are:
Selected Answer: [None Given]
Answers: a.
Slugging due to flooding back of refrigerant
b.
Too much oil in crankcase
c.
Any of the other choices
d.
Worn bearings, pins, etc
• Question 11
0 out of 3.3 points
A Freon 2 refrigeration system is fitted with thermal expansion valves. The
valves are rated in :
Selected Answer: [None Given]
Answers: a.
Cubic feet per minute
b.
Superheat setting
c.
TOR
d.
Pound per minute
• Question 12
0 out of 3.3 points
The velocity of radiation (v) and the frequency of radiations (f) are related to the
wavelength of radiations by:
Selected Answer: [None Given]
Answers: a.
f/v
b.
v/f
c.
vf
d.
v2f
• Question 13
0 out of 3.3 points
A company is interested to produce a water turbine wherein only little
energy is required or necessary because the guide vanes are to be
controlled. The turbine must be a:
Selected Answer: [None Given]
Answers: a.
Francis turbine
b.
propeller turbine
c.
Kaplan turbine
d.
gas turbine
• Question 14
0 out of 3.3 points
An air conditioning system wherein the entire systems are mounted in the
cabinet:
Selected Answer: [None Given]
Answers: a.
Package Type Air Conditioners
b.
Centralized Air Conditioners
c.
Console Air Conditioners
d.
Multi-zone Split Ductless System
• Question 15
0 out of 3.3 points
Excessive use of cold city water makeup reduces overall efficiency because
the water must be ____ before use in the boiler
Selected Answer: [None Given]
Answers: a.
Recirculated
b.
Heated
c.
Filtered
d.
Vented
• Question 16
0 out of 3.3 points
In a refrigerating system, the heat absorbed in the evaporator per kg mass
of refrigerant passing through
Selected Answer: [None Given]
Answers: a.
Is decreased if pre- cooler is used
b.
Equals the increase in enthalpy
c.
Does not depend on the refrigerant used
d.
Equals the increase in volume
• Question 17
0 out of 3.3 points
The most likely cause of high superheat would be:
Selected Answer: [None Given]
Answers: a.
Expansion valve open too wide
b.
Expansion valve closed too much
c.
Back-pressure valve set too high
d.
Too much refrigerant
• Question 18
0 out of 3.3 points
The “refrigerating effect” of a refrigerant is?
Selected [None Given]
Answer:
Answers: a.
A or B
b.
Amount of heat it can remove in a given time
c.
Quantity of heat that 1 pound of refrigerant absorbs
d.
Quantity of heat that 1 pound of refrigerant absorbs while
flowing through the evaporator under given conditions
• Question 19
0 out of 3.3 points
All of the ff pollutants are produced because of decaying organic matter except:
Selected Answer: [None Given]
Answers: a.
Ammonia
b.
Methane
c.
Sulfur dioxide
d.
Hydrogen sulfide
• Question 20
0 out of 3.3 points
The empirical coefficient e in machine foundation if not given is assumed
Selected Answer: [None Given]
Answers: a.
0.15
b.
0.32
c.
0.25
d.
0.11
• Question 21
0 out of 3.3 points
When removing reusable refrigerant from a system, the line to the storage
drum must
Selected Answer: [None Given]
Answers: a.
have no bends in it
b.
Be above the level of the compressor
c.
Contain a strainer-dryer
d.
Be made of copper
• Question 22
0 out of 3.3 points
Which of the following gasket materials should be on Freon system?
Selected Answer: [None Given]
Answers: a.
Asbestos
b.
Rubber
c.
Asbestos or Metallic
d.
Metallic
• Question 23
0 out of 3.3 points
The schedule number of a pipe, N, indicates the thickness of the pipe wall. If
the allowable stress of the pipe is S, then what is the internal pressure equal
to?
Selected Answer: [None Given]
Answers: a.
N/S
b.
S/N
c.
N–S
d.
NxS
• Question 24
0 out of 3.3 points
What does enthalpy measure in a substance?
Selected Answer: [None Given]
Answers: a.
its heat content
b.
its coldness
c.
its humidity
d.
its dew point
• Question 25
0 out of 3.3 points
Which of the following would cause the compressor to run continuously?
Selected Answer: [None Given]
Answers: a.
High pressure switch jammed
b.
Defective thermal bulb
c.
Low pressure switch jammed
d.
Clogged strainer
• Question 26
0 out of 3.3 points
The area under load curve divided by the maximum demand represents:
Selected Answer: [None Given]
Answers: a.
Load factor
b.
Average load
c.
Connected factor
d.
Diversity factor
• Question 27
0 out of 3.3 points
If the cooling water to the condenser suddenly fails:
Selected Answer: [None Given]
Answers: a.
An alarm will ring to notify the engineer
b.
The solenoid valve will close
c.
The expansion valve will close
d.
The compressor will shutdown
• Question 28
0 out of 3.3 points
The speed at which a reaction proceeds to equilibrium is the purview of
Selected Answer: [None Given]
Answers: a.
Neutralization
b.
Le chatelier’s principle
c.
Ionization
d.
Reaction kinetics
• Question 29
0 out of 3.3 points
The solenoid valve is controlled by
Selected Answer: [None Given]
Answers: a.
The temperature in the condenser
b.
The amount of gas in the system
c.
The temperature in the icebox
d.
The amount of liquid in the system
• Question 30
0 out of 3.3 points
What is the use of a hydraulic jump?
Selected Answer: [None Given]
Answers: a.
Increase the flow rate
b.
Reduce the flow rate
c.
Reduce the velocity of flow
d.
Reduce the energy of flow
Tuesday, March 17, 2020 4:45:36 PM PST
• Question 1
0 out of 3.3 points
A water conservation device which employs both air and water is condensed
a condenser and a cooling tower combined into one is called:
Selected Answer: [None Given]
Answers: a.
water-cooled condenser
b.
evaporative condenser
c.
air-cooled condenser
d.
shell and tube condenser
• Question 2
0 out of 3.3 points
In a power driven pump, each piston stroke is displaced by 360 divide the
______.
Selected Answer: [None Given]
Answers: a.
length of the stroke
b.
number of cylinders
c.
revolution per minute
d.
bore
• Question 3
0 out of 3.3 points
An odorless refrigerant whose boiling point varies over a wide range of
temperatures.
Selected Answer: [None Given]
Answers: a.
Freon11
b.
Ammonia
c.
Freon 12
d.
Freon 22
• Question 4
0 out of 3.3 points
The purpose of the expansion valve by-pass is
to
Selected [None Given]
Answer:
Answers: a.
By-pass the compressor
b.
Increase the efficiency of the plant
c.
Increase the capacity of the evaporator
d.
Controls the refrigerant to the evaporator in case the
automatic valves fail
• Question 5
0 out of 3.3 points
Air conditioning is the process of:
Selected [None Given]
Answer:
Answers: a.
removing heat from a specific area
b.
keeping a place cool
c.
maintaining the air at a required temperature and humidity
d.
keeping a place cool and maintaining the air at a required
temperature and humidity
• Question 6
0 out of 3.3 points
A type of polymer used for labels, bottles, housewares
Selected Answer: [None Given]
Answers: a.
Polypropylene(PP)
b.
Polyvinyl chloride(PVC)
c.
Low density polyethylene (LDPE)
d.
Polystyrene(PS)
• Question 7
0 out of 3.3 points
The ratio of maximum load to the rated plant capacity
Selected Answer: [None Given]
Answers: a.
Utilization factor
b.
Capacity factor
c.
Maximum factor
d.
Load factor
• Question 8
0 out of 3.3 points
Wb is shaft work of an engine and Wi is indicated work of an engine. If
mechanical is present in the engine mechanism, then.
Selected Answer: [None Given]
Answers: a.
Wb is proportional to Wi
b.
Wb is equal to Wi
c.
Wb is less than Wi
d.
Wb is greater then Wi
• Question 9
0 out of 3.3 points
Which of the following vital components of the refrigeration system where
both temperature and pressure are increased?
Selected Answer: [None Given]
Answers: a.
Compressor
b.
Evaporator
c.
Condenser
d.
Compressor and evaporator
• Question 10
0 out of 3.3 points
Which of the ff does not use ambient air for propulsion?
Selected Answer: [None Given]
Answers: a.
Turbo jet
b.
Rocket
c.
Turbo- prop
d.
Pulse jet
• Question 11
0 out of 3.3 points
Which of the following is also known as refrigerant no. R-717?
Selected Answer: [None Given]
Answers: a.
Freon 22
b.
Freon 12
c.
Methyl chloride
d.
Ammonia
• Question 12
0 out of 3.3 points
Are by products of reaction between combustion products
Selected Answer: [None Given]
Answers: a.
Sediments
b.
Oxidants
c.
Photochemicals
d.
Organics
• Question 13
0 out of 3.3 points
In order to perform efficiently, a power cycle must be communicated with
the outside temperature of its surroundings and into one is called:
Selected Answer: [None Given]
Answers: a.
Second Law of Thermodynamics
b.
First Law of Thermodynamics
c.
Kelvin-Planck Law
d.
Kirchoff’s Law
• Question 14
0 out of 3.3 points
What is the specific humidity of dry air?
Selected Answer: [None Given]
Answers: a.
150
b.
50
c.
0
d.
100
• Question 15
0 out of 3.3 points
When air is heated, what happened to its relative humidity?
Selected Answer: [None Given]
Answers: a.
May increase or Decrease depending on temperature
b.
Increases
c.
Decreases
d.
Remain constant
• Question 16
0 out of 3.3 points
The scale trap is located between the:
Selected Answer: [None Given]
Answers: a.
Expansion valve and evaporator coils
b.
King (liquid) valve and expansion valve
c.
Compressor and oil separator
d.
Evaporator coils and compressor
• Question 17
0 out of 3.3 points
The minimum vertical distance from the floor or soil level to the top edge of
the foundation must be around __________.
Selected Answer: [None Given]
Answers: a.
150 mm
b.
100 mm
c.
120 mm
d.
200 mm
• Question 18
0 out of 3.3 points
Which of the following would cause a high suction pressure?
Selected Answer: [None Given]
Answers: a.
Dirty dehydrator
b.
Expansion valve not open wide enough
c.
King valve not open wide enough
d.
Expansion valve open too wide
• Question 19
0 out of 3.3 points
A thermostat that functions as increase or decrease instead of starting and
stopping system is called:
Selected Answer: [None Given]
Answers: a.
heating-cooling the thermostat
b.
Compound thermostat
c.
Interlocked
d.
Module
• Question 20
0 out of 3.3 points
If Wt is the turbine shaft work of a gas turbine unit, Wc in duel, then
determine its thermal efficiency.
Selected Answer: [None Given]
Answers: a.
(Wt + Wc) / Q
b.
(Wt –Wc) /Q
c.
Q / (Wt + Wc)
d.
Q / (Wt – Wc)
• Question 21
0 out of 3.3 points
To eliminate transmission of the vibration, the foundation should be isolated
from the floor slabs of building footings at least how many mm around its
perimeter?
Selected Answer: [None Given]
Answers: a.
25
b.
15
c.
30
d.
20
• Question 22
0 out of 3.3 points
What is the device used to protect the compressor from overloading due to
high head pressure
Selected Answer: [None Given]
Answers: a.
Thermostatic expansion valve
b.
Overload relay
c.
Hold back suction valve
d.
Expansion valve
• Question 23
0 out of 3.3 points
The equilibrium constant for weak solution is known as
Selected Answer: [None Given]
Answers: a.
Ionization constant
b.
Arrhenius exponent
c.
Solubility product
d.
La chatelier’s constant
• Question 24
0 out of 3.3 points
Too high suction pressure could be caused by:
Selected Answer: [None Given]
Answers: a.
Leaky suction valves
b.
Expansion valve open too wide
c.
Any of the other choices
d.
Expansion valve bulb not working properly
• Question 25
0 out of 3.3 points
If a Carnot cycle is at the same maximum and minimum temperatures as a
Rankine cycle, then how would you compare the efficiency of a Rankine cycle
to that of the Carnot cycle?
Selected Answer: [None Given]
Answers: a.
higher efficiency
b.
efficiency is the same
c.
lower efficiency
d.
efficiency is higher by 30%
• Question 26
0 out of 3.3 points
The ratio absorbed by the transfer fluid to the original incident energy striking the
collector
Selected Answer: [None Given]
Answers: a.
Transmittance
b.
Betz coefficient
c.
Shading factor
d.
Collector efficiency
• Question 27
0 out of 3.3 points
When figuring compression ratio, the ratio will increase with a ______.
Selected Answer: [None Given]
Answers: a.
Lower suction pressure and higher discharge pressure
b.
Higher suction pressure
c.
Lower suction pressure
d.
Higher discharge pressure
• Question 28
0 out of 3.3 points
When air contains all of the water vapor it can hold, it is said to be
Selected Answer: [None Given]
Answers: a.
moisture
b.
simulated
c.
saturated
d.
loaded
• Question 29
0 out of 3.3 points
Flow measuring devices include all of the ff except:
Selected Answer: [None Given]
Answers: a.
Magnetic dynamometers
b.
Venturi meters
c.
Static pressure probes
d.
Turbine and propeller meters
• Question 30
0 out of 3.3 points
As Freon leaves the expansion valve:
Selected Answer: [None Given]
Answers: a.
Pressure decrease - Volume decrease
b.
Pressure increase - Volume decrease
c.
Pressure increase - Volume increase
d.
Pressure decrease - Volume increase
Tuesday, March 17, 2020 4:47:10 PM PST
• Question 1
0 out of 3.3 points
The system should be purged:
Selected Answer: [None Given]
Answers: a.
After the system has been shut down for few hours
b.
While system is operating
c.
While starting up
d.
Once a week
• Question 2
0 out of 3.3 points
Compare in refrigerating effect per unit mass of refrigerant circulated for a
superheated cycle that produces useful cooling and a saturated cycle, for the
same vaporizing and condensing temperature.
Selected Answer: [None Given]
Answers: a.
lower for a superheated cycle
b.
the same
c.
greater for a saturated cycle
d.
greater for a superheated cycle
• Question 3
0 out of 3.3 points
During the re-expansion portion of the refrigeration compressor cycle
Selected Answer: [None Given]
Answers: a.
The suction valve is closed
b.
The discharge valve is closed
c.
The suction valve is closed and the discharge valve is closed
d.
The suction valve is open
• Question 4
0 out of 3.3 points
The pump will not cavitate if the available Net Positive Suction Head (NPSH)
is:
Selected Answer: [None Given]
Answers: a.
equal or lesser than the required NPHS
b.
zero compared to the required NPHS of 1
c.
less than the required NPHS
d.
equal or greater than the required NPHS
• Question 5
0 out of 3.3 points
The boiling point of co2 at atmospheric pressure is
Selected Answer: [None Given]
Answers: a.
110 C
b.
110 F
c.
-110 F
d.
-110 C
• Question 6
0 out of 3.3 points
If frost forms on the cylinders, the cause would be:
Selected Answer: [None Given]
Answers: a.
Charging valve left open
b.
Expansion valve open too wide
c.
Expansion valve not open wide enough
d.
Dehydrator not working properly
• Question 7
0 out of 3.3 points
When excess solute in a solution settles to the bottom of the container, the process
is called:
Selected Answer: [None Given]
Answers: a.
Precipitation
b.
Equation
c.
Salvation
d.
Deemulsification
• Question 8
0 out of 3.3 points
When the bypass factor is B, the coil efficiency for sensible cooling of air is
equal to:
Selected Answer: [None Given]
Answers: a.
1+B
b.
1 + B /B
c.
1–B/B
d.
1–B
• Question 9
0 out of 3.3 points
Heat exchanger in which water flows by gravity over the outside of tubes or
plates.
Selected Answer: [None Given]
Answers: a.
Bourdon cooler
b.
Newtonian cooler
c.
free cooler
d.
Baudelot cooler
• Question 10
0 out of 3.3 points
High superheat of the vapor in the system would cause
Selected Answer: [None Given]
Answers: a.
A more efficient unit
b.
An increase in capacity
c.
An increase in capacity and a more efficient unit
d.
A decrease in capacity
• Question 11
0 out of 3.3 points
Which has an octane rating of more than 100?
Selected Answer: [None Given]
Answers: a.
Benzol
b.
Methyl alcohol
c.
Gobar gas
d.
Ethyl alcohol
• Question 12
0 out of 3.3 points
If refrigeration controls were constructed with little or no differential, the :
Selected Answer: [None Given]
Answers: a.
Refrigerant would be subcooled
b.
Icebox would get too warm
c.
Compressor would short cycle
d.
Icebox would get too cold
• Question 13
0 out of 3.3 points
Which of the following is the most common method of heat flow in
refrigeration?
Selected Answer: [None Given]
Answers: a.
Conduction
b.
Expulsion
c.
Radiation
d.
Convection
• Question 14
0 out of 3.3 points
What is the effect of superheating the refrigerant?
Selected Answer: [None Given]
Answers: a.
It lowers the boiling point of the refrigerant
b.
It decrease the coefficient of performance
c.
It increase the coefficient of performance
d.
It increase the suction pressure of the refrigerant
• Question 15
0 out of 3.3 points
The fraction of the radiation energy incident on a surface which is absorbed
by the surface is called:
Selected Answer: [None Given]
Answers: a.
Radiation
b.
Emission
c.
Convection
d.
Absorptivity
• Question 16
0 out of 3.3 points
Are by products of reaction between combustion products
Selected Answer: [None Given]
Answers: a.
Oxidants
b.
Organics
c.
Sediments
d.
Photochemicals
• Question 17
0 out of 3.3 points
Aeration of water is done for all of the ff purposes except for:
Selected Answer: [None Given]
Answers: a.
Removal of bad taste
b.
Increasing the amount of oxygen in water
c.
Removal of CO2
d.
Removal of temporary hardness
• Question 18
0 out of 3.3 points
An excessively high head pressure would be caused by:
Selected Answer: [None Given]
Answers: a.
Too much cooling water to the condenser
b.
Insufficient cooling water to the evaporator coils
c.
Solenoid valve shutoff
d.
Insufficient cooling water to the condenser
• Question 19
0 out of 3.3 points
Boiling temperature of Freon 12 is
Selected Answer: [None Given]
Answers: a.
-29.8C
b.
-33.33 C
c.
-40.7 C
d.
-78.5 C
• Question 20
0 out of 3.3 points
The water hammer phenomenon is primarily what kind of fluid
mechanisms?
Selected Answer: [None Given]
Answers: a.
Static (a phenomena independent of time)
b.
Compressible
c.
Dynamic (a time- dependent phenomena)
d.
Incompressible
• Question 21
0 out of 3.3 points
Based on the PSME code, what should be provided in each steam outlets if
two or more boilers will be connected in parallel?
Selected Answer: [None Given]
Answers: a.
expansion and shut-off valves
b.
non-return and shut off valves
c.
expansion and check valves
d.
relief and check valves
• Question 22
0 out of 3.3 points
A salimeter reads the
Selected Answer: [None Given]
Answers: a.
Rate of brine
b.
Density of brine
c.
Relative humidity
d.
Dew point temperature
• Question 23
0 out of 3.3 points
Which of the following is a type of water turbine?
Selected Answer: [None Given]
Answers: a.
Parson
b.
Bankl
c.
Hero
d.
Pelton
• Question 24
0 out of 3.3 points
The purge valve is located:
Selected Answer: [None Given]
Answers: a.
On the evaporator coils
b.
In the highest part of the system
c.
In the lowest part of the system
d.
On the receiver discharge
• Question 25
0 out of 3.3 points
A water conservation device which employs both air and water is condensed
a condenser and a cooling tower combined into one is called:
Selected Answer: [None Given]
Answers: a.
shell and tube condenser
b.
air-cooled condenser
c.
evaporative condenser
d.
water-cooled condenser
• Question 26
0 out of 3.3 points
A Freon 2 refrigeration system is fitted with thermal expansion valves. The
valves are rated in :
Selected Answer: [None Given]
Answers: a.
TOR
b.
Pound per minute
c.
Superheat setting
d.
Cubic feet per minute
• Question 27
0 out of 3.3 points
Which of the following is a characteristic of an impulse turbine?
Selected Answer: [None Given]
Answers: a.
Steam striking blades on angle
b.
Steam reversing direction
c.
No steam reaction to velocity
d.
Steam striking blades at zero angle
• Question 28
0 out of 3.3 points
In an air conditioning unit, the thermostat fails. The unit did not start. How
do you test the operation of the thermostat?
Selected [None Given]
Answer:
Answers: a.
Cover both the inlet and outlet so that the recirculating air has
the same temperature.
b.
Cover the air outlet and air inlet with a cloth. The air will
recirculate into the unit and the temperature will quickly drop to
the cut out temperature.
c.
Cover the outlet and the air will not circulate and the thermostat
functions because no air movement.
d.
Never cover any air passage so that air can freely move and
thermostat functions well.
• Question 29
0 out of 3.3 points
When purging an ammonia condenser into a bucket of water, one can tell
when the air is out and ammonia starts to come through by the:
Selected [None Given]
Answer:
Answers: a.
Smell of the ammonia being liberated from the water
b.
Color of the water turning green
c.
Color the water turning bluish
d.
Change of bubbling sound of air to the cracking sound of
ammonia
• Question 30
0 out of 3.3 points
Flows through multi-loop systems may be computed by:
Selected Answer: [None Given]
Answers: a.
All of the other choices
b.
The hardy-cross method
c.
Trial and error
d.
Any closed- form solution of simultaneous equations
Tuesday, March 17, 2020 4:48:35 PM PST
• Question 1
0 out of 3.3 points
The suction pressure control valve is actuated by which of the following?
Selected Answer: [None Given]
Answers: a.
Bellow
b.
Thermostat
c.
Pressure diaphragm
d.
Thermal element
• Question 2
0 out of 3.3 points
Most observe properties of light and other radiant energy are consistent with
waves in nature, but in interaction with matter, electromagnetic energy behaves as
though it consists of discrete pieces or
Selected Answer: [None Given]
Answers: a.
Blocks
b.
Quanta
c.
Atomic masses
d.
Balls
• Question 3
0 out of 3.3 points
Brazing is used for joining two
Selected Answer: [None Given]
Answers: a.
One ferrous and non ferrous material
b.
Two non metals
c.
Two ferrous material
d.
Two non ferrous material
• Question 4
0 out of 3.3 points
When purging an ammonia condenser into a bucket of water, one can tell
when the air is out and ammonia starts to come through by the:
Selected [None Given]
Answer:
Answers: a.
Color of the water turning green
b.
Smell of the ammonia being liberated from the water
c.
Change of bubbling sound of air to the cracking sound of
ammonia
d.
Color the water turning bluish
• Question 5
0 out of 3.3 points
If ice will form in a solution of water and salt, then it is at temperature called:
Selected Answer: [None Given]
Answers: a.
freezing point depression
b.
critical point
c.
dew point
d.
boiling point depression
• Question 6
0 out of 3.3 points
The effective size of a target atom that interacts with a moving particle is called its
_____
Selected Answer: [None Given]
Answers: a.
Cross section
b.
Width
c.
Pseudo-area
d.
Length
• Question 7
0 out of 3.3 points
A draft tube is a part of which power plants?
Selected Answer: [None Given]
Answers: a.
Diesel engine power plant
b.
Hydro-electric power plant
c.
Gas turbine power plant
d.
Steam power plant
• Question 8
0 out of 3.3 points
A ____ boiler has water in the tubes and heat gases of combustion passing
through the tubes.
Selected Answer: [None Given]
Answers: a.
Fire tube
b.
Water tube
c.
Fire box
d.
Cast iron sectional
• Question 9
0 out of 3.3 points
The effect of superheating the refrigerant is to:
Selected Answer: [None Given]
Answers: a.
Maintains the COP
b.
Ammonia
c.
Decrease the COP
d.
Increase the COP
• Question 10
0 out of 3.3 points
In what form that water exists in air?
Selected Answer: [None Given]
Answers: a.
Liquid
b.
Vapor
c.
Saturated
d.
Solid
• Question 11
0 out of 3.3 points
What is the use of back pressure regulating valve?
Selected [None Given]
Answer:
Answers: a.
Controls the temperature in the evaporator coils
b.
Maintains a fixed pressure in the evaporator coils
c.
Controls the evaporator temperature by throttling the flow of
liquid refrigerant
d.
Controls the flow of circulating water in the condenser tubes
• Question 12
0 out of 3.3 points
Which of the following is the oil used in a refrigeration system?
Selected Answer: [None Given]
Answers: a.
Lube oil sae 10
b.
Straight mineral oil
c.
Vegetable oil
d.
Lube oil sae 20
• Question 13
0 out of 3.3 points
The sum of the internal energy and the product of pressure and specific
volume is known as the:
Selected Answer: [None Given]
Answers: a.
total work
b.
enthalpy
c.
total internal energy
d.
entropy
• Question 14
0 out of 3.3 points
Are open or closed tanks containing dozens or hundreds of slowly rotating disks
covered with a biological film of microorganisms
Selected Answer: [None Given]
Answers: a.
Bioinvetor
b.
Bioreactor
c.
Biomediator
d.
Biofilter
• Question 15
0 out of 3.3 points
Which of the ff area of work requires lowest noise level?
Selected Answer: [None Given]
Answers: a.
Library
b.
Kitchens
c.
Gymnasiums
d.
Lecture halls
• Question 16
0 out of 3.3 points
Most people are comfortable with the relative humidity of:
Selected Answer: [None Given]
Answers: a.
40 % to 80 %
b.
30 % to 70 %
c.
10 % to 40 %
d.
20 % to 30%
• Question 17
0 out of 3.3 points
If refrigeration controls were constructed with little or no differential, the :
Selected Answer: [None Given]
Answers: a.
Icebox would get too warm
b.
Refrigerant would be subcooled
c.
Compressor would short cycle
d.
Icebox would get too cold
• Question 18
0 out of 3.3 points
Large leaks in a Freon system cannot always be detected with halide torch
because it changes color with slightest amount of Freon present. A large leak
can be detected easier by applying ____________.
Selected [None Given]
Answer:
Answers: a.
A thin layer of mineral oil to all joints and watch for bubbles
b.
Sheets of litmus paper to all joints and watch for color change
c.
A lighted candle at the joints and watch for leaky spots blowing
candle flame
d.
A soapsuds solution, mixed with a little glycerin to hold the
solution together, and watch for bubbles
• Question 19
0 out of 3.3 points
In pumped storage plant
Selected [None Given]
Answer:
Answers: a.
Water is stored by pumping to high pressures
b.
Power is produced by pumps
c.
Pressure accumulators are used
d.
Downstream water is pumped upstream during offload
periods
• Question 20
0 out of 3.3 points
If the Hs is the total suction head of a pump, Hp is its suction surface
pressure, and Hf is its suction friction head, then calculate the total suction
head, Ht.
Selected Answer: [None Given]
Answers: a.
Ht = Hs + Hp – Hf
b.
Ht = Hs – Hp + Hf
c.
Ht = Hs + Ht + Hf
d.
Ht = Hs – Hp – Hf
• Question 21
0 out of 3.3 points
Based on the PSME Code, what should be the humidity of air to be used for
comfort cooling?
Selected Answer: [None Given]
Answers: a.
50% - 60% relative humidity
b.
60% - 70% relative humidity
c.
70% - 75% relative humidity
d.
75% - 80% relative humidity
• Question 22
0 out of 3.3 points
Clear ice
Selected Answer: [None Given]
Answers: a.
Is pure ice
b.
Is formed by blowing air during freezing
c.
Contains dissolved gases
d.
Contains dissolved air
• Question 23
0 out of 3.3 points
If the air compressed without discarding heat, then what do you call this kind
of compression:
Selected Answer: [None Given]
Answers: a.
adiabatic
b.
isothermal
c.
isobaric
d.
isochoric
• Question 24
0 out of 3.3 points
The low-pressure control switch:
Selected Answer: [None Given]
Answers: a.
Actuates the cooling water
b.
Cuts out the compressor to maintain proper flow
c.
Is a safety device
d.
Regulates the King Valve
• Question 25
0 out of 3.3 points
Which of the following is not essential to a compression refrigeration
system?
Selected Answer: [None Given]
Answers: a.
A receiver
b.
An evaporator
c.
A condenser
d.
A dehydrator
• Question 26
0 out of 3.3 points
Critical temperature is that temperature above which.
Selected Answer: [None Given]
Answers: a.
A gas gets immediately liquefy
b.
A gas will never liquefy
c.
Water will never evaporate
d.
Water gets evaporated
• Question 27
0 out of 3.3 points
The amount of CO2 or Freon in a cylinder is measured by
Selected Answer: [None Given]
Answers: a.
Weight
b.
Psi
c.
Volume
d.
Pressure
• Question 28
0 out of 3.3 points
Obstruction of the expansion valve is usually caused by:
Selected Answer: [None Given]
Answers: a.
Congealed oil in the system
b.
Any of the other choices
c.
Scale
d.
Water in the system
• Question 29
0 out of 3.3 points
Which of the following is the most common method of heat flow in
refrigeration?
Selected Answer: [None Given]
Answers: a.
Radiation
b.
Expulsion
c.
Convection
d.
Conduction
• Question 30
0 out of 3.3 points
Which of the following measures the density of salt in water?
Selected Answer: [None Given]
Answers: a.
Calorimeter
b.
Hydrometer
c.
Salimeter
d.
Pitot tube
Tuesday, March 17, 2020 4:50:14 PM PST
• Question 1
0 out of 3.3 points
Saturation temperature is the same as;
Selected Answer: [None Given]
Answers: a.
humidity
b.
steam temperature
c.
vapor temperature
d.
dew point
• Question 2
0 out of 3.3 points
All of the ff statements about conjugate acids and bases are true except
Selected Answer: [None Given]
Answers: a.
Strong acids tend to give weak conjugate bases
b.
The bronsted- lowry theory defines bases as proton accepetors
c.
A conjugate acids results when a base accept a proton
d.
A conjugate base results when a base accepts a proton
• Question 3
0 out of 3.3 points
Is a high- temperature process that turns incinerator ash into a safe, glass –like
material
Selected Answer: [None Given]
Answers: a.
Bioventing
b.
Vitrification
c.
Biofitration
d.
Advance oxidation
• Question 4
0 out of 3.3 points
If the pressure exerted on a liquid is higher than the saturation
corresponding to its temperature, the liquid is a:
Selected Answer: [None Given]
Answers: a.
highly superheated liquid
b.
saturated liquid
c.
sub cooled liquid
d.
superheated liquid
• Question 5
0 out of 3.3 points
Select the one in which secondary refrigerant is used
Selected Answer: [None Given]
Answers: a.
Deep freezer
b.
Ice plant
c.
Room air conditioner
d.
Domestic refrigerator
• Question 6
0 out of 3.3 points
R-22 is
Selected Answer: [None Given]
Answers: a.
Methyl chloride
b.
Dichlorodifluoromethane
c.
Trichlorodifluoromethane
d.
Monochlorodifluoromethane
• Question 7
0 out of 3.3 points
If the compressor had been running satisfactorily for a long period of time
but suddenly the compartment temperature started to rise, the trouble
might be:
Selected Answer: [None Given]
Answers: a.
The solenoid valve has jammed shut
b.
The expansion valve may contain frozen water
c.
A refrigerant leak has developed
d.
Any of the other choices
• Question 8
0 out of 3.3 points
The psychrometric chart in air conditioning determines the
Selected Answer: [None Given]
Answers: a.
saturation temperature and relatives humidity
b.
wet bulb and dry bulb temperatures
c.
moist air conditions
d.
psychrometric temperature requirements
• Question 9
0 out of 3.3 points
Which of the following vital components of the refrigeration system where
both temperature and pressure are increased?
Selected Answer: [None Given]
Answers: a.
Condenser
b.
Compressor and evaporator
c.
Compressor
d.
Evaporator
• Question 10
0 out of 3.3 points
The pump will not cavitate if the available Net Positive Suction Head (NPSH)
is:
Selected Answer: [None Given]
Answers: a.
equal or greater than the required NPHS
b.
equal or lesser than the required NPHS
c.
zero compared to the required NPHS of 1
d.
less than the required NPHS
• Question 11
0 out of 3.3 points
Which of the following fans in air conditioning systems which can be
classified as centrifugal flow?
Selected Answer: [None Given]
Answers: a.
Axial Fan
b.
Propeller fan
c.
Bi-axial fan
d.
None of these
• Question 12
0 out of 3.3 points
The coefficient of discharge is the ratio of the:
Selected Answer: [None Given]
Answers: a.
Effective head to the actual head
b.
Actual velocity to the theoretical velocity
c.
Area of vena contracta to the orifice
d.
Actual discharge to the theoretical discharge
• Question 13
0 out of 3.3 points
The force when applied to a mass of one kilogram will give mass an
acceleration of one meter per second for every second called:
Selected Answer: [None Given]
Answers: a.
Newton
b.
Watt
c.
Pascal
d.
Joule
• Question 14
0 out of 3.3 points
At 0 psig, how many BTU necessary to change 1 lb of water at 212 F of
steam?
Selected Answer: [None Given]
Answers: a.
144
b.
970
c.
180
d.
1190
• Question 15
0 out of 3.3 points
Which of the following is another name for the liquid valve?
Selected Answer: [None Given]
Answers: a.
Master valve
b.
Shutoff valve
c.
King valve
d.
Freon valve
• Question 16
0 out of 3.3 points
The size of a reciprocating pump is stamped on the builder’s plate 3” x 4’ x
6”. The diameter of the liquid cylinder is:
Selected Answer: [None Given]
Answers: a.
7”
b.
4”
c.
5”
d.
6”
• Question 17
0 out of 3.3 points
Refers to the use of composting and soil beds
Selected Answer: [None Given]
Answers: a.
Bioventing
b.
Bioreactros
c.
Biomediation
d.
Biofiltration
• Question 18
0 out of 3.3 points
The temperature bulb of the solenoid valve is attached to the ___________.
Selected Answer: [None Given]
Answers: a.
Evaporator coil inlet
b.
Evaporator coil outlet
c.
Wall of the icebox
d.
Icebox coil
• Question 19
0 out of 3.3 points
What is the specific gravity of an oil which has Baume reading g of 28
degrees F?
Selected Answer: [None Given]
Answers: a.
0.774
b.
0.215
c.
0.562
d.
0.886
• Question 20
0 out of 3.3 points
Absolute zero on the Fahrenheit Scale is equal to:
Selected Answer: [None Given]
Answers: a.
0
b.
-460
c.
-100
d.
-273
• Question 21
0 out of 3.3 points
The machine foundation must have a factor of safety of:
Selected Answer: [None Given]
Answers: a.
4
b.
7
c.
5
d.
6
• Question 22
0 out of 3.3 points
A ____ pump discharges the return water back to the boiler
Selected Answer: [None Given]
Answers: a.
Vacuum
b.
Return
c.
Feed
d.
Gear
• Question 23
0 out of 3.3 points
During sensible heating, the humidity remains constant but the relative
humidity.
Selected Answer: [None Given]
Answers: a.
zero
b.
remains constant
c.
decreases
d.
increases
• Question 24
0 out of 3.3 points
Flash chamber is installed in the refrigeration circuit so as to:
Selected [None Given]
Answer:
Answers: a.
Improve overall heat transfer coefficient
b.
All of the other choices
c.
Reduce pressure losses through the evaporator
d.
Reduce the size of evaporator by avoiding vapors going to
evaporator
• Question 25
0 out of 3.3 points
A sulfur stick burning in the presence of ammonia will give off a ______.
Selected Answer: [None Given]
Answers: a.
Dense yellow smoke
b.
Dense white smoke
c.
Dense red smoke
d.
Dense green smoke
• Question 26
0 out of 3.3 points
The intake pipe to a hydraulic turbine from a dam is:
Selected Answer: [None Given]
Answers: a.
Spiral casing
b.
Surge tank
c.
Tailrace
d.
Penstock
• Question 27
0 out of 3.3 points
To increase heat transfer in the evaporator
Selected Answer: [None Given]
Answers: a.
Increase air circulation
b.
Increase suction pressure
c.
Increase air circulation and defrost coils
d.
Defrost coils
• Question 28
0 out of 3.3 points
A refrigerating unit of one (1) ton capacity can remove
Selected Answer: [None Given]
Answers: a.
100 btu’s per min.
b.
500 btu’s per min.
c.
200 btu’s per min
d.
288 btu’s per min.
• Question 29
0 out of 3.3 points
In order to keep combustion products inside the combustion chamber and
stack system, balanced draft system may actually operate with a slight
Selected Answer: [None Given]
Answers: a.
positive pressure
b.
positive and negative pressure
c.
negative pressure
d.
absolute pressure
• Question 30
0 out of 3.3 points
What is the effect on saturated temperature if pressure of the fluid is
decreased:
Selected Answer: [None Given]
Answers: a.
saturation temperature increases
b.
saturation temperature decreases
c.
there is no effect
d.
saturation temperature remains constant
Tuesday, March 17, 2020 4:51:50 PM PST
• Question 1
0 out of 3.3 points
A ____ pump discharges the return water back to the boiler
Selected Answer: [None Given]
Answers: a.
Return
b.
Gear
c.
Vacuum
d.
Feed
• Question 2
0 out of 3.3 points
What do you call a material in a dryer?
Selected Answer: [None Given]
Answers: a.
Drain
b.
Desiccant
c.
Dryer
d.
Drain or desiccant
• Question 3
0 out of 3.3 points
What is the temperature of lubricant at which it will ignite thus limits its
application?
Selected Answer: [None Given]
Answers: a.
flash point
b.
burning point
c.
cold point
d.
use point
• Question 4
0 out of 3.3 points
The temperature of the refrigerant is highest just before it enters the :
Selected Answer: [None Given]
Answers: a.
Condenser
b.
Receiver
c.
Evaporator
d.
King valve
• Question 5
0 out of 3.3 points
When the outlet temperature at the evaporator exceeds the inlet
temperature, the condition is known as :
Selected Answer: [None Given]
Answers: a.
Saturating
b.
Dehydrating
c.
Superheating
d.
Desuperheating
• Question 6
0 out of 3.3 points
In Stirling and Ericsson Cycles, the efficiency can be increased by:
Selected Answer: [None Given]
Answers: a.
compressibility
b.
regeneration
c.
superheating
d.
subcooling
• Question 7
0 out of 3.3 points
What do you call a system in which the evaporator coils are located in a
brine solution and the brine is pumped through the icebox?
Selected Answer: [None Given]
Answers: a.
A direct system
b.
A low-pressure system
c.
An indirect system
d.
A double-evaporator system
• Question 8
0 out of 3.3 points
The coefficient of velocity is the ratio of the:
Selected Answer: [None Given]
Answers: a.
Effective head to the actual head
b.
Area of vena contracta to the orifice area
c.
Actual discharge to the theoretical discharge
d.
Actual velocity to the theoretical velocity
• Question 9
0 out of 3.3 points
What is the specific gravity of an oil which has Baume reading g of 28
degrees F?
Selected Answer: [None Given]
Answers: a.
0.886
b.
0.215
c.
0.774
d.
0.562
• Question 10
0 out of 3.3 points
What usually happened if brine has a high specific gravity ?
Selected Answer: [None Given]
Answers: a.
It will solidify
b.
It will crystallize
c.
Nothing will happen
d.
It will freeze
• Question 11
0 out of 3.3 points
The crossover connection in an ammonia system can be used to _________.
Selected Answer: [None Given]
Answers: a.
Reduce the pressure on the discharge side of the condenser
b.
Add refrigerant to the system
c.
Hot-gas defrost
d.
Pump air out of the system
• Question 12
0 out of 3.3 points
Which of the following is important for evaporator coils?
Selected Answer: [None Given]
Answers: a.
It should have air completely surrounding them
b.
It must place in front of circulating fans.
c.
It must be placed in the top of the compartment
d.
It must be secured to the sides
• Question 13
0 out of 3.3 points
Which of the following is the most common method of heat flow in
refrigeration?
Selected Answer: [None Given]
Answers: a.
Radiation
b.
Conduction
c.
Convection
d.
Expulsion
• Question 14
0 out of 3.3 points
If any of the electrically controlled devices in a Freon system malfunction,
which of the following valve will also automatically shut off?
Selected Answer: [None Given]
Answers: a.
Expansion valve
b.
Condenser cooling water inlet valve
c.
Solenoid valve
d.
King valve
• Question 15
0 out of 3.3 points
When vapor compression takes place on one side of the piston and one
during one revolution of the crankshaft, then the compressor is:
Selected Answer: [None Given]
Answers: a.
four-cycle
b.
single-acting
c.
two-revolution
d.
double-acting
• Question 16
0 out of 3.3 points
The refrigerant with the lowest boiling point is
Selected Answer: [None Given]
Answers: a.
NH3
b.
F22
c.
F12
d.
CO2
• Question 17
0 out of 3.3 points
How much will be removed by one-ton refrigeration unit?
Selected Answer: [None Given]
Answers: a.
144 Btu per min
b.
2000 Btu per min
c.
400 Btu per min
d.
200 Btu per min
• Question 18
0 out of 3.3 points
When the dry bulb and the wet bulb temperature are identical, the air is said
to be:
Selected Answer: [None Given]
Answers: a.
Saturates
b.
Compressed
c.
Dehumidified
d.
Humidified
• Question 19
0 out of 3.3 points
What usually happens if the specific gravity of the brine is too low?
Selected Answer: [None Given]
Answers: a.
Solids will deposits
b.
The brine will freeze
c.
All of the other choices
d.
It will be more heat-absorbing
• Question 20
0 out of 3.3 points
A type of polymer used for detergents, milk bottles, oil containers and toys
Selected Answer: [None Given]
Answers: a.
Polypropylene(PP)
b.
Polystyrene(PS)
c.
High density polyethelene (HDPE)
d.
Polyvinyl chloride(PVC)
• Question 21
0 out of 3.3 points
Which of the following industries have the highest consumption of water for
processing?
Selected Answer: [None Given]
Answers: a.
Foundry
b.
Paper mill
c.
Automobile industry
d.
Aluminum industry
• Question 22
0 out of 3.3 points
The oil level in the compressor should be checked:
Selected Answer: [None Given]
Answers: a.
While the compressor is in compressor
b.
After an extended lay-up period
c.
Just before starting the compressor
d.
After a long period of operation
• Question 23
0 out of 3.3 points
Concrete foundation should have steel bar reinforcement placed vertically
and horizontally to avoid ___________.
Selected Answer: [None Given]
Answers: a.
Melting
b.
Vibration
c.
Breaking
d.
Thermal cracking
• Question 24
0 out of 3.3 points
Which of the following measures the density of salt in water?
Selected Answer: [None Given]
Answers: a.
Hydrometer
b.
Pitot tube
c.
Salimeter
d.
Calorimeter
• Question 25
0 out of 3.3 points
Water in the refrigerant is liable to
Selected [None Given]
Answer:
Answers: a.
Freeze on the expansion valve seat and cut the flow of
refrigerant
b.
Freeze in the king (liquid) valve
c.
Clog the oil trap
d.
Emulsify the oil in the compressor
• Question 26
0 out of 3.3 points
According to ASME code, safety valves on low pressure boilers should be
tested by hand at least
Selected Answer: [None Given]
Answers: a.
Once a month
b.
Once a shift
c.
Once a year
d.
Twice a year
• Question 27
0 out of 3.3 points
In balance system, the available draft is
Selected Answer: [None Given]
Answers: a.
Zero
b.
100
c.
Unity
d.
Infinite
• Question 28
0 out of 3.3 points
Which of the following is the usual case of slugging?
Selected Answer: [None Given]
Answers: a.
Too much cooling water to condenser
b.
Too much refrigerant in the system
c.
Expansion valve not operating properly
d.
Too much oil in the system
• Question 29
0 out of 3.3 points
Low suction pressure is caused by:
Selected Answer: [None Given]
Answers: a.
Expansion valve causing flooding back
b.
Leaky compressor suction valves
c.
Solenoid valve not functioning properly
d.
Air in the system
• Question 30
0 out of 3.3 points
The “refrigerating effect” of a refrigerant is?
Selected [None Given]
Answer:
Answers: a.
Quantity of heat that 1 pound of refrigerant absorbs
b.
Amount of heat it can remove in a given time
c.
A or B
d.
Quantity of heat that 1 pound of refrigerant absorbs while
flowing through the evaporator under given conditions
• Question 1
0 out of 3.3 points
Which of the following is considered as comfort condition in air
conditioning?
Selected Answer: [None Given]
Answers: a.
30 C DBT, 60% RH
b.
20 C DBT, 60% RH
c.
40 C DBT, 80% RH
d.
30 C DBT, 80% RH
• Question 2
0 out of 3.3 points
When the outlet temperature of the evaporator exceeds the inlet
temperature the condition is called ______________.
Selected Answer: [None Given]
Answers: a.
Superheating
b.
Melting
c.
Freezing
d.
Boiling
• Question 3
0 out of 3.3 points
Subcooling of the refrigerant results in :
Selected Answer: [None Given]
Answers: a.
Effect of refrigerant increased
b.
Liquid less likely to vaporize
c.
Less circulating water needed
d.
Effect of refrigerant increased and liquid less likely to vaporize
• Question 4
0 out of 3.3 points
A Freon – 12 gage shows pressure and ______.
Selected Answer: [None Given]
Answers: a.
Vacuum
b.
Back pressure
c.
Superheat temperature
d.
Saturation temperature
• Question 5
0 out of 3.3 points
Peak load for a period of time divided by installed capacity is
Selected Answer: [None Given]
Answers: a.
Load factor
b.
Demand factor
c.
Capacity factor
d.
Utilization factor
• Question 6
0 out of 3.3 points
If R is the delivery rate of a pump, H is the total pumping head and e is the
efficiency of the pump, then compute the power required to drive the pump.
Selected Answer: [None Given]
Answers: a.
(R x H) / e
b.
1 – (R + H) / e
c.
R/Hxe
d.
(h – R) / e
• Question 7
0 out of 3.3 points
In guyed steel packs, the angle between wires in a set of three is
Selected Answer: [None Given]
Answers: a.
135 deg
b.
120 deg
c.
110 deg
d.
145 deg
• Question 8
0 out of 3.3 points
Large-bulb alcohol thermometer used to measure air speed or atmospheric
conditions by means of cooling.
Selected Answer: [None Given]
Answers: a.
Kata Thermometer
b.
Wet Bulb Thermometer
c.
JJ Thompson Thermometer
d.
Kelvin Thermometer
• Question 9
0 out of 3.3 points
When air is heated, what happened to its relative humidity?
Selected Answer: [None Given]
Answers: a.
Remain constant
b.
Decreases
c.
May increase or Decrease depending on temperature
d.
Increases
• Question 10
0 out of 3.3 points
The ratio of the sum of individual maximum demands of the system to the overall
maximum demand of the whole system
Selected Answer: [None Given]
Answers: a.
Diversity factor
b.
Utilization factor
c.
Power factor
d.
Demand factor
• Question 11
0 out of 3.3 points
The desirable temperature inside an air conditioned auditorium is:
Selected Answer: [None Given]
Answers: a.
5C
b.
0C
c.
10 C
d.
20 C
• Question 12
0 out of 3.3 points
The ratio of maximum load to the rated plant capacity
Selected Answer: [None Given]
Answers: a.
Utilization factor
b.
Maximum factor
c.
Capacity factor
d.
Load factor
• Question 13
0 out of 3.3 points
How should the window type air conditioning unit be placed?
Selected Answer: [None Given]
Answers: a.
Level
b.
Slant toward the inside of the home
c.
Slant toward the outside of the home
d.
Slant at approximately 15 from the horizontal
• Question 14
0 out of 3.3 points
TOR in Btu/24 hrs is
Selected Answer: [None Given]
Answers: a.
288 000
b.
29000
c.
28 500
d.
290 000
• Question 15
0 out of 3.3 points
A hot crankcase and cylinder head accompanied by a low suction pressure
would be caused by
Selected Answer: [None Given]
Answers: a.
Excess refrigerant
b.
Stuck discharge valve
c.
Air in the system
d.
Insufficient refrigerant
• Question 16
0 out of 3.3 points
Evaporative condenser is used to cool
Selected Answer: [None Given]
Answers: a.
condenser vapor
b.
condenser surface
c.
condenser liquid
d.
all of these
• Question 17
0 out of 3.3 points
What do you call the mixture when a solvent has dissolve as much solute as it can?
Selected Answer: [None Given]
Answers: a.
Saturated solution
b.
Hydration
c.
Solution
d.
Mild solution
• Question 18
0 out of 3.3 points
What is the cycle where a refrigeration system generally operates?
Selected Answer: [None Given]
Answers: a.
Closed cycle
b.
Mixed cycle
c.
Open cycle
d.
Hybrid cycle
• Question 19
0 out of 3.3 points
The critical temperature of a refrigerant is
Selected Answer: [None Given]
Answers: a.
None of the other choices
b.
The temperature above which it cannot be liquefied
c.
The temperature below which it cannot be liquefied
d.
The temperature at which it will freeze
• Question 20
0 out of 3.3 points
What is the least number of compressors a multistage system that will use?
Selected Answer: [None Given]
Answers: a.
four
b.
three
c.
one
d.
two
• Question 21
0 out of 3.3 points
Which of the ff is not pollutant from a sulfuric acid plants?
Selected Answer: [None Given]
Answers: a.
Sulfur trioxide
b.
Acid moist
c.
Hydrogen sulfide
d.
Sulfur dioxide
• Question 22
0 out of 3.3 points
The ratio of the sum individual maximum demands of the system to the maximum
demand of the whole system is:
Selected Answer: [None Given]
Answers: a.
Power factor
b.
Utilization factor
c.
Diversity factor
d.
Demand factor
• Question 23
0 out of 3.3 points
What is the combination of a wet and dry bulb thermometer is called a
Selected Answer: [None Given]
Answers: a.
Hygrometer
b.
Hydrometer
c.
Psychrometer
d.
Hygrometer or psychrometer
• Question 24
0 out of 3.3 points
If refrigeration controls were constructed with little or no differential, the :
Selected Answer: [None Given]
Answers: a.
Compressor would short cycle
b.
Icebox would get too cold
c.
Icebox would get too warm
d.
Refrigerant would be subcooled
• Question 25
0 out of 3.3 points
The design of an air supply duct of an air conditioning system
Selected Answer: [None Given]
Answers: a.
adds moisture to the air
b.
does not affect the moisture of the air
c.
lowers the temperature of the air
d.
affects the distribution
• Question 26
0 out of 3.3 points
Refers to the high – temperature removal of tarry substances from the interior of
the carbon granule, leaving a highly porous structure
Selected Answer: [None Given]
Answers: a.
Activated
b.
Breakthrough
c.
Absorbent
d.
Adsorbent
• Question 27
0 out of 3.3 points
A Freon 2 refrigeration system is fitted with thermal expansion valves. The
valves are rated in :
Selected Answer: [None Given]
Answers: a.
Superheat setting
b.
TOR
c.
Pound per minute
d.
Cubic feet per minute
• Question 28
0 out of 3.3 points
You want to change refrigerants in a Freon 12 plant using a reciprocating
single acting compressor which refrigerant requires the smallest amount of
change to the system?
Selected Answer: [None Given]
Answers: a.
Carbon dioxide
b.
Aqua ammonia
c.
Methyl chloride
d.
Ammonia
• Question 29
0 out of 3.3 points
What is the instrument used to register relative humidity?
Selected Answer: [None Given]
Answers: a.
hydrometer
b.
hygrometer
c.
manometer
d.
perometer
• Question 30
0 out of 3.3 points
Which of the ff cannot be used to describe neutron kinetic energy levels?
Selected Answer: [None Given]
Answers: a.
Slow
b.
Cold
c.
Freezing
d.
Thermal
Tuesday, March 17, 2020 5:13:15 PM PST
• Question 1
0 out of 3.3 points
A type of refrigerant that will not damage the ozone layer.
Selected Answer: [None Given]
Answers: a.
R-22
b.
R-12
c.
Hydrochlorofluorocarbons (HCHF’s)
d.
Hydrofluorocarbons (HCF’s)
• Question 2
0 out of 3.3 points
A leaky discharge valve can usually be detected by:
Selected Answer: [None Given]
Answers: a.
A fluctuating high-pressure gauge
b.
Any of the other choices
c.
A discharge pressure lower than normal
d.
A drop in icebox temperature
• Question 3
0 out of 3.3 points
If Pi is the indicated horsepower and Pb is the indicated horsepower of a
compressor, then what is mechanical efficiency, Em, equal to:
Selected Answer: [None Given]
Answers: a.
Em = Pb-Pi
b.
Em = Pb/Pi
c.
Em = Pi/Pb
d.
Em = Pi - Pb
• Question 4
0 out of 3.3 points
The method of cooling which primarily used where ambient air
temperatures are high and relative humidity is used:
Selected Answer: [None Given]
Answers: a.
Condensate cooling
b.
Swamp cooling
c.
Evaporative cooling
d.
Hydroionic cooling
• Question 5
0 out of 3.3 points
Horsepower per ton of refrigeration is expressed as:
Selected Answer: [None Given]
Answers: a.
4.75 x cop
b.
4.75/ cop
c.
Cop/ 4,75
d.
4.75/ cop
• Question 6
0 out of 3.3 points
Based on the PSME code, what should be provided in each steam outlets if
two or more boilers will be connected in parallel?
Selected Answer: [None Given]
Answers: a.
non-return and shut off valves
b.
expansion and check valves
c.
expansion and shut-off valves
d.
relief and check valves
• Question 7
0 out of 3.3 points
Which of the following is not essential to a centrifugal type compressor
system?
Selected Answer: [None Given]
Answers: a.
Evaporator
b.
Distiller
c.
Condenser
d.
Expansion valve
• Question 8
0 out of 3.3 points
Which of the following is a characteristic of an impulse turbine?
Selected Answer: [None Given]
Answers: a.
Steam striking blades at zero angle
b.
Steam reversing direction
c.
Steam striking blades on angle
d.
No steam reaction to velocity
• Question 9
0 out of 3.3 points
Type of refrigerant control which maintains pressure difference between
high side and low side pressure in refrigerating mechanism:
Selected Answer: [None Given]
Answers: a.
Suction valve
b.
Expansion valve
c.
Service valve
d.
Solenoid valve
• Question 10
0 out of 3.3 points
The suction pressure control valve is actuated by which of the following?
Selected Answer: [None Given]
Answers: a.
Thermostat
b.
Pressure diaphragm
c.
Bellow
d.
Thermal element
• Question 11
0 out of 3.3 points
Where is the excess refrigerant removed?
Selected Answer: [None Given]
Answers: a.
Discharge side of the system
b.
Suction side of the system
c.
By pass
d.
Charging side of the system
• Question 12
0 out of 3.3 points
The critical temperature of a refrigerant is
Selected Answer: [None Given]
Answers: a.
The temperature at which it will freeze
b.
None of the other choices
c.
The temperature below which it cannot be liquefied
d.
The temperature above which it cannot be liquefied
• Question 13
0 out of 3.3 points
When air is heated, what happened to its relative humidity?
Selected Answer: [None Given]
Answers: a.
May increase or Decrease depending on temperature
b.
Increases
c.
Remain constant
d.
Decreases
• Question 14
0 out of 3.3 points
Which of the following would cause the compressor to run continuously?
Selected Answer: [None Given]
Answers: a.
Defective thermal bulb
b.
High pressure switch jammed
c.
Clogged strainer
d.
Low pressure switch jammed
• Question 15
0 out of 3.3 points
When the air is saturated the wet-bulb depression is:
Selected Answer: [None Given]
Answers: a.
indefinite
b.
100%
c.
unity
d.
zero
• Question 16
0 out of 3.3 points
What do you usually do to correct slugging and flooding back?
Selected Answer: [None Given]
Answers: a.
Change in the discharge pressure
b.
Clean the traps
c.
Check the expansion valve
d.
Remove some refrigerant
• Question 17
0 out of 3.3 points
Hydraulic jump is used to:
Selected Answer: [None Given]
Answers: a.
Reduce the velocity of flow
b.
Reduce the energy of flow
c.
Reduce the flow rate
d.
Increase the flow rate
• Question 18
0 out of 3.3 points
Concrete foundation should have steel bar reinforcement placed vertically
and horizontally to avoid ___________.
Selected Answer: [None Given]
Answers: a.
Melting
b.
Thermal cracking
c.
Breaking
d.
Vibration
• Question 19
0 out of 3.3 points
During compression in a vapor compression cycle when the refrigerant is
super heated, what happened to the performance?
Selected Answer: [None Given]
Answers: a.
Cop remains unchanged
b.
Work done is increased
c.
Refrigerating effect is reduced
d.
Cop is reduced
• Question 20
0 out of 3.3 points
If the compressor were to run out continuously without lowering the
temperature, the trouble would probably be:
Selected Answer: [None Given]
Answers: a.
Leaks in the system
b.
Any of the other choices
c.
Leaky discharge valve
d.
Insufficient refrigerant in the system
• Question 21
0 out of 3.3 points
A device whose primary function is to meter the flow of refrigerant to the
evaporator
Selected Answer: [None Given]
Answers: a.
Crossover valve
b.
Thermostatic expansion valve
c.
Equalizer
d.
Sniffer valve
• Question 22
0 out of 3.3 points
The absorption of water by a dessicant often demonstrates:
Selected Answer: [None Given]
Answers: a.
Exothermic heat of solution
b.
Heat of fusion
c.
Endothermic heat of solution
d.
Heat of vaporization
• Question 23
0 out of 3.3 points
What is likely to occur when sections of the impeller of a centrifugal pump
are handling vapor and other sections are handling liquid.
Selected Answer: [None Given]
Answers: a.
complete failure to operate
b.
erosion of the pump
c.
excessive vibration
d.
high head and low capacity
• Question 24
0 out of 3.3 points
Which of the following devices used to measure the discharge of outflow
from a vessel?
Selected Answer: [None Given]
Answers: a.
Pump head
b.
Pitot tube
c.
Obstruction meter
d.
Orifice
• Question 25
0 out of 3.3 points
A sulfur stick burning in the presence of ammonia will give off a ______.
Selected Answer: [None Given]
Answers: a.
Dense white smoke
b.
Dense red smoke
c.
Dense green smoke
d.
Dense yellow smoke
• Question 26
0 out of 3.3 points
What is the pressure present inside the casing of an impulse turbine?
Selected Answer: [None Given]
Answers: a.
atmospheric pressure
b.
vacuum
c.
above atmospheric
d.
zero
• Question 27
0 out of 3.3 points
Refers to organic waste produced from biological water waste treatment
processes
Selected Answer: [None Given]
Answers: a.
Process waste
b.
Biosolids
c.
Toxic waste
d.
Extrinsic waste
• Question 28
0 out of 3.3 points
What is the instrument used to register relative humidity?
Selected Answer: [None Given]
Answers: a.
perometer
b.
hydrometer
c.
hygrometer
d.
manometer
• Question 29
0 out of 3.3 points
A number used to compare energy usage for different areas. It is calculated
by dividing the energy consumption by the footage of the conditioned area.
Selected Answer: [None Given]
Answers: a.
Energy utilization Index
b.
Energy Utilization ratio
c.
Energy utilization unit
d.
Energy utilization efficiency
• Question 30
0 out of 3.3 points
Which of the ff is not pollutant from a sulfuric acid plants?
Selected Answer: [None Given]
Answers: a.
Sulfur dioxide
b.
Sulfur trioxide
c.
Hydrogen sulfide
d.
Acid moist
Tuesday, March 17, 2020 5:32:18 PM PST
• Question 1
0 out of 3.3 points
If the air compressed without discarding heat, then what do you call this kind
of compression:
Selected Answer: [None Given]
Answers: a.
isochoric
b.
isothermal
c.
adiabatic
d.
isobaric
• Question 2
0 out of 3.3 points
What is the boiling temperature of F 22?
Selected Answer: [None Given]
Answers: a.
-40C
b.
-78.5 C
c.
-29.8C
d.
-33.33 C
• Question 3
0 out of 3.3 points
Measurement of a device’s ability to remove atmospheric air from test air.
Selected Answer: [None Given]
Answers: a.
Atmospheric Dust Spot Efficiency
b.
Baudelot Air Efficiency
c.
Atmospheric Dry Air Efficiency
d.
Test Air Efficiency
• Question 4
0 out of 3.3 points
What is the specific humidity of dry air?
Selected Answer: [None Given]
Answers: a.
100
b.
0
c.
150
d.
50
• Question 5
0 out of 3.3 points
In the hydro-electric power plant having a medium head and using a Francis
turbine, the turbine speed may be regulated through:
Selected Answer: [None Given]
Answers: a.
Wicket gate
b.
Nozzle
c.
Deflector gate
d.
Forebay
• Question 6
0 out of 3.3 points
An air conditioning system wherein the entire systems are mounted in the
cabinet:
Selected Answer: [None Given]
Answers: a.
Package Type Air Conditioners
b.
Centralized Air Conditioners
c.
Console Air Conditioners
d.
Multi-zone Split Ductless System
• Question 7
0 out of 3.3 points
If refrigeration controls were constructed with little or no differential, the :
Selected Answer: [None Given]
Answers: a.
Icebox would get too warm
b.
Compressor would short cycle
c.
Refrigerant would be subcooled
d.
Icebox would get too cold
• Question 8
0 out of 3.3 points
If the thermal bulb becomes loose on the evaporator coils, it will cause _____.
Selected Answer: [None Given]
Answers: a.
Any of the other choices
b.
Flooding back of the refrigerant
c.
An electrical short
d.
Improper operation of expansion valve
• Question 9
0 out of 3.3 points
The “refrigerating effect” of a refrigerant is?
Selected [None Given]
Answer:
Answers: a.
Quantity of heat that 1 pound of refrigerant absorbs
b.
Amount of heat it can remove in a given time
c.
Quantity of heat that 1 pound of refrigerant absorbs while
flowing through the evaporator under given conditions
d.
A or B
• Question 10
0 out of 3.3 points
Is a soluble compound that reduces a liquid’s surface tension or reduces the
interfacial tension between a liquid and a solid
Selected Answer: [None Given]
Answers: a.
Surface – acting agent or surfactant
b.
Oxygenated fuel
c.
Humectant
d.
CFC
• Question 11
0 out of 3.3 points
To test a thermostatic valve, immerse the bulb in:
Selected Answer: [None Given]
Answers: a.
Hot water
b.
Oil
c.
None of the other choices
d.
Crushed ice
• Question 12
0 out of 3.3 points
Substances that have the ability to absorb moisture from the air are called:
Selected Answer: [None Given]
Answers: a.
Moisturizer
b.
Desiccants
c.
Moisture absorber
d.
Dehygroscopic substances
• Question 13
0 out of 3.3 points
Le Chatelier’s principle predicts the direction of a state of chemical equilibrium
bases on all of the ff factors except:
Selected Answer: [None Given]
Answers: a.
Concentration
b.
Temperature
c.
Specific volume
d.
Pressure
• Question 14
0 out of 3.3 points
If W is the work output of a heat engine and H is the total heat input of an
engine, then determine its thermal efficiency, e.
Selected Answer: [None Given]
Answers: a.
e = I – (W – H)
b.
e=H/W
c.
e=W/H
d.
e=WxH
• Question 15
0 out of 3.3 points
If the superheat on the suction side of the compressor is increased, what will
happen to the tonnage capacity of the unit?
Selected Answer: [None Given]
Answers: a.
No change
b.
Increases
c.
Decreases
d.
None of the other choices
• Question 16
0 out of 3.3 points
The weight of the machine foundation is how many time of the weight of the
engine?
Selected Answer: [None Given]
Answers: a.
4 to 6 times
b.
2 to 3 times
c.
4 to 5 times
d.
3 to 5 times
• Question 17
0 out of 3.3 points
The determination of properties and behavior of atmospheric air usually the
purview of:
Selected Answer: [None Given]
Answers: a.
thermodynamics
b.
pychrometrics
c.
Forced convection
d.
kirchoff’s law
• Question 18
0 out of 3.3 points
Which of the following is a type of evaporator?
Selected Answer: [None Given]
Answers: a.
shell-and-tube
b.
oil lantern rings
c.
polyphase motors
d.
sell-and-tube-water cooler
• Question 19
0 out of 3.3 points
Based on the PSME code, what should be the effective temperature of the
air to be maintained for comfort cooling at an air movement from 0.0761 to
0.127 meter per second?
Selected Answer: [None Given]
Answers: a.
35 – 39 degrees C
b.
25 – 29 degrees C
c.
30 – 34 degrees C
d.
20 – 24 degrees C
• Question 20
0 out of 3.3 points
If one of the ship’s service boxes suddenly stops cooling, the most likely
cause would be the malfunction of the
Selected Answer: [None Given]
Answers: a.
Discharge valve
b.
Concentrating collectors
c.
Expansion valve
d.
King valve
• Question 21
0 out of 3.3 points
What amount of air is required in a low bypass factor?
Selected Answer: [None Given]
Answers: a.
lesser
b.
indeterminate
c.
Increases
d.
greater
• Question 22
0 out of 3.3 points
A sulfur stick burning in the presence of ammonia will give off a ______.
Selected Answer: [None Given]
Answers: a.
Dense green smoke
b.
Dense yellow smoke
c.
Dense white smoke
d.
Dense red smoke
• Question 23
0 out of 3.3 points
At 0 psig, how many BTU necessary to change 1 lb of water at 212 F of
steam?
Selected Answer: [None Given]
Answers: a.
144
b.
1190
c.
970
d.
180
• Question 24
0 out of 3.3 points
Aside from maintaining appropriate temperature for food cold storage, how
is desiccation minimized or decreased?
Selected Answer: [None Given]
Answers: a.
maintain humidity ratio
b.
increase humidity ratio
c.
low air circulation
d.
low oxygen
• Question 25
0 out of 3.3 points
A scale trap in a Freon system will be found on the
Selected Answer: [None Given]
Answers: a.
Discharge side
b.
Receiver
c.
Condenser
d.
Suction side
• Question 26
0 out of 3.3 points
Which of the following is considered as comfort condition in air
conditioning?
Selected Answer: [None Given]
Answers: a.
30 C DBT, 80% RH
b.
40 C DBT, 80% RH
c.
20 C DBT, 60% RH
d.
30 C DBT, 60% RH
• Question 27
0 out of 3.3 points
Fan motors in air conditioning usually have:
Selected Answer: [None Given]
Answers: a.
2 or 3 speeds
b.
4 or 5 speeds
c.
1 or 2 speeds
d.
3 or 4 speeds
• Question 28
0 out of 3.3 points
What is the chemical formula of Ozone?
Selected Answer: [None Given]
Answers: a.
O3
b.
O1
c.
O2
d.
D2O
• Question 29
0 out of 3.3 points
If the compressor short-cycles on the low-pressure cutout, the trouble might
be:
Selected Answer: [None Given]
Answers: a.
Any of the other choices
b.
Lack of refrigerant
c.
Too much frost on the evaporator coils
d.
Dirty traps and strainers
• Question 30
0 out of 3.3 points
A double-seated valve allows the valve to be
Selected Answer: [None Given]
Answers: a.
Removed for replacement without shutting down
b.
Operated as a suction or discharge valve
c.
Packed in the wide open or closed position
d.
Packed only in the closed position
Tuesday, March 17, 2020 5:35:58 PM PST
• Question 1
0 out of 3.3 points
Zinc rods are found in the:
Selected Answer: [None Given]
Answers: a.
Evaporator area
b.
Salt water side of the condenser
c.
Gas side of the condenser
d.
Compressor crankcase
• Question 2
0 out of 3.3 points
Ozone filters out what type of radiation that damages crops and causes skin cancer?
Selected Answer: [None Given]
Answers: a.
X-rays
b.
Ultraviolet radiation
c.
Tropospheric radiation
d.
Atmospheric radiation
• Question 3
0 out of 3.3 points
A substance with surface area per unit of weight, and intricate pore structure, and a
hydrophobic surface
Selected Answer: [None Given]
Answers: a.
Activated substance
b.
Homogeneous substance
c.
Adsorbent substance
d.
Adsorbent substance
• Question 4
0 out of 3.3 points
What usually happens if the specific gravity of the brine is too low?
Selected Answer: [None Given]
Answers: a.
The brine will freeze
b.
Solids will deposits
c.
All of the other choices
d.
It will be more heat-absorbing
• Question 5
0 out of 3.3 points
According to ASME code, safety valves on low pressure boilers should be
tested by hand at least
Selected Answer: [None Given]
Answers: a.
Once a shift
b.
Twice a year
c.
Once a year
d.
Once a month
• Question 6
0 out of 3.3 points
What consists of weak solutions of sulfuric, hydrochloric, and to a lesser extent,
nitric acids?
Selected Answer: [None Given]
Answers: a.
pollutant
b.
Acid compound
c.
Acid rain
d.
Stack gas
• Question 7
0 out of 3.3 points
The term suction units is used with
Selected Answer: [None Given]
Answers: a.
Forced draft
b.
Induced draft
c.
Balanced draft
d.
Natural draft
• Question 8
0 out of 3.3 points
Two compressor should not be run in parallel because :
Selected Answer: [None Given]
Answers: a.
There is a possibility of losing oil or it will give over – capacity
b.
There is a possibility of losing oil
c.
It will give over – capacity
d.
It is not efficient
• Question 9
0 out of 3.3 points
Which of the ff contribute to the deterioration of the earth’s ozone layer?
Selected Answer: [None Given]
Answers: a.
Carbon monoxide
b.
Chlorofluorocarbons
c.
Refrigerants
d.
Carbon dioxide
• Question 10
0 out of 3.3 points
A liquid mixture having constant maximum and minimum boiling points.
Refrigerants comprising this mixture do not combine chemically, yet the
mixture provides constant characteristics.
Selected Answer: [None Given]
Answers: a.
Homogeneous Mixture
b.
Conzeotropic Mixture
c.
Cyrogenic Mixture
d.
Azeotropic Mixture
• Question 11
0 out of 3.3 points
When air is saturated, the wet bulb depression is:
Selected Answer: [None Given]
Answers: a.
100%
b.
unity
c.
indefinite
d.
zero
• Question 12
0 out of 3.3 points
Oxygen is approximately what percent by weight in the atmosphere?
Selected Answer: [None Given]
Answers: a.
27 %
b.
77 %
c.
73 %
d.
23 %
• Question 13
0 out of 3.3 points
If no gaskets are used in the piping joints of a Freon system, the joints must
be:
Selected Answer: [None Given]
Answers: a.
Welded joints
b.
Soldered joints
c.
Ground joints
d.
Finished joints
• Question 14
0 out of 3.3 points
The normal cut-out setting of a window unit thermostat is between:
Selected Answer: [None Given]
Answers: a.
16 C to 21 C
b.
13C to 16 C
c.
10C to 15C
d.
17C to 22C
• Question 15
0 out of 3.3 points
Which of the following power plants uses energy from uranium to produces
electric power?
Selected Answer: [None Given]
Answers: a.
Hydroelectric Plant
b.
Geothermal Plant
c.
Diesel Plant
d.
Nuclear Plant
• Question 16
0 out of 3.3 points
In order to perform efficiently, a power cycle must be communicated with
the outside temperature of its surroundings and into one is called:
Selected Answer: [None Given]
Answers: a.
Kirchoff’s Law
b.
Second Law of Thermodynamics
c.
Kelvin-Planck Law
d.
First Law of Thermodynamics
• Question 17
0 out of 3.3 points
In order to remove the fly ashes from the flue gas, which of the following
must a power plant be equipped with?
Selected Answer: [None Given]
Answers: a.
electrostatic precipitator
b.
condenser
c.
demineralizer
d.
desulphurization plant
• Question 18
0 out of 3.3 points
The relief valve is located on the:
Selected Answer: [None Given]
Answers: a.
Receiver tank
b.
Outlet of the evaporator coils
c.
Discharge side on the condenser
d.
Discharge side of the compressor
• Question 19
0 out of 3.3 points
The ratio of fugacity of actual conditions to the fugacity at some reference state is
known as:
Selected Answer: [None Given]
Answers: a.
Compressibility
b.
Gravimetric coefficient
c.
Saturation
d.
Activity
• Question 20
0 out of 3.3 points
If the compressor were to run out continuously without lowering the
temperature, the trouble would probably be:
Selected Answer: [None Given]
Answers: a.
Insufficient refrigerant in the system
b.
Leaks in the system
c.
Leaky discharge valve
d.
Any of the other choices
• Question 21
0 out of 3.3 points
Excess frost on the evaporator coils:
Selected Answer: [None Given]
Answers: a.
Keeps the icebox cooler
b.
Takes load off compressor
c.
Does not affect the system
d.
Reduces efficiency of the plant
• Question 22
0 out of 3.3 points
Horsepower per ton of refrigeration is expressed as:
Selected Answer: [None Given]
Answers: a.
Cop/ 4,75
b.
4.75/ cop
c.
4.75/ cop
d.
4.75 x cop
• Question 23
0 out of 3.3 points
What is the compression ratio of a refrigerator compressor?
Selected [None Given]
Answer:
Answers: a.
The ratio of the absolute suction pressure to the absolute
discharge pressure
b.
The ratio of the excessive liquid refrigerant in the receiver to
that in the system
c.
None of the other choices
d.
The ratio of the absolute discharge pressure to the absolute
suction pressure
• Question 24
0 out of 3.3 points
Which of the following measures the density of salt in water?
Selected Answer: [None Given]
Answers: a.
Pitot tube
b.
Salimeter
c.
Hydrometer
d.
Calorimeter
• Question 25
0 out of 3.3 points
What is the pressure at the exit of a draft tube in a turbine?
Selected Answer: [None Given]
Answers: a.
Below atmospheric
b.
Atmospheric
c.
Vacuum
d.
Gage
• Question 26
0 out of 3.3 points
When securing a freon system for repairs
Selected Answer: [None Given]
Answers: a.
Remove all refrigerant from the system
b.
Pump down to a slight vacuum
c.
Pump down to 10 to 15 pounds pressure
d.
Pump down to 1 or 2 pounds pressure
• Question 27
0 out of 3.3 points
During compression in a vapor compression cycle when the refrigerant is
super heated, what happened to the performance?
Selected Answer: [None Given]
Answers: a.
Cop is reduced
b.
Refrigerating effect is reduced
c.
Cop remains unchanged
d.
Work done is increased
• Question 28
0 out of 3.3 points
Any foreign matter in the coal feed mechanism of the screw-feed stoker is
best removed by:
Selected Answer: [None Given]
Answers: a.
Using the cutoff gate at the bottom of the hopper
b.
Emptying the coal hopper
c.
Forcing it through with a heaver shear pin
d.
Reversing the stroker
• Question 29
0 out of 3.3 points
Absolute zero is
Selected Answer: [None Given]
Answers: a.
970 degrees below zero on the Fahrenheit scale
b.
144 degrees below zero on the Fahrenheit scale
c.
460 degrees below zero on the Fahrenheit scale
d.
The same as zero on the Fahrenheit scale
• Question 30
0 out of 3.3 points
When coil surface is below the dew point temperature of the air.
Selected Answer: [None Given]
Answers: a.
air is humidified and saturated
b.
vapor will evaporate through the air
c.
moisture will condense out of the air
d.
air is dehumidified
Tuesday, March 17, 2020 5:38:23 PM PST
• Question 1
0 out of 3.3 points
The bulb for the thermal expansion valve is located:
Selected Answer: [None Given]
Answers: a.
Near the evaporator coil outlet
b.
On the bottom row of evaporator coils
c.
Near the evaporator coil inlet
d.
In the middle of evaporator coils
• Question 2
0 out of 3.3 points
A burner should always start up in ____ fire and shut down in ____ fire
Selected Answer: [None Given]
Answers: a.
High; low
b.
High; high
c.
Low;high
d.
Low;low
• Question 3
0 out of 3.3 points
All of the following are forms of drag on a body moving through a fluid
except:
Selected Answer: [None Given]
Answers: a.
D’alembert’s paradox drag
b.
Skin friction
c.
Profile drag
d.
Wake drag
• Question 4
0 out of 3.3 points
The thickness of the head gasket is important because it may cause
__________.
Selected Answer: [None Given]
Answers: a.
Re-expansion
b.
The piston to strike the head
c.
Decreased efficiency due to increased clearance
d.
All of the other choices
• Question 5
0 out of 3.3 points
If C is the capacity of the compressor in a refrigeration system and F is the
heat rejection factor, then calculate the condenser load, L.
Selected Answer: [None Given]
Answers: a.
L=F/C
b.
L=F–C
c.
L=CxF
d.
L=C/F
• Question 6
0 out of 3.3 points
Which type of compressor is used in refrigeration system?
Selected Answer: [None Given]
Answers: a.
Reciprocating
b.
Rotary sliding vane
c.
Centrifugal
d.
All of the other choices
• Question 7
0 out of 3.3 points
If the compressor had been running satisfactorily for a long period of time
but the oil level was rising slowly, one should:
Selected [None Given]
Answer:
Answers: a.
Drain out sufficient oil to bring it down to a proper running
level
b.
Shut down the compressor and check the oil level with the
machine stopped
c.
Check to see if there is too much refrigerant in the system
d.
Check the dehydrator cartridge
• Question 8
0 out of 3.3 points
In a hydro – electric plant using a Francis turbine with medium head, the speed can
be regulated using the
Selected Answer: [None Given]
Answers: a.
Nozzle
b.
Weir
c.
Deflector gate
d.
Wicket gate
• Question 9
0 out of 3.3 points
With one machine serving several compartment, when one compartment
reaches the desired temperature in that compartment is maintained by :
Selected Answer: [None Given]
Answers: a.
The back-pressure valve
b.
The expansion valve
c.
Any of the other choices
d.
The solenoid valve
• Question 10
0 out of 3.3 points
What is the least number of compressors a multistage system that will use?
Selected Answer: [None Given]
Answers: a.
one
b.
four
c.
two
d.
three
• Question 11
0 out of 3.3 points
Why should you avoid bending or twisting of fan blades in an air
conditioning unit?
Selected Answer: [None Given]
Answers: a.
It will wear out the motor bearings and cause noise
b.
It will cause ice build-up
c.
It will wear out slice suction line
d.
It decrease the volume flow of refrigerant
• Question 12
0 out of 3.3 points
How is the CO2 system is purged?
Selected [None Given]
Answer:
Answers: a.
The CO2 will come out of the purge valve in liquid form
b.
Through the king valve
c.
Pumped out with a suction pump
d.
When CO2 comes out of the purge valve, frost will form on a
piece of metal held near the outlet
• Question 13
0 out of 3.3 points
Mycotoxins are poisonous chemicals produced by:
Selected Answer: [None Given]
Answers: a.
Virus
b.
Molds
c.
Bacteria
d.
Algae
• Question 14
0 out of 3.3 points
The purpose of the dehydrator is to
Selected Answer: [None Given]
Answers: a.
Remove moisture from the crankcase oil
b.
Remove oil from the refrigerant
c.
Add more refrigerant to the system
d.
Remove moisture from the refrigerant
• Question 15
0 out of 3.3 points
Refers to organic waste produced from biological water waste treatment
processes
Selected Answer: [None Given]
Answers: a.
Biosolids
b.
Extrinsic waste
c.
Toxic waste
d.
Process waste
• Question 16
0 out of 3.3 points
The reduction of nuclear radiation intensity (called attenuation) is accomplished
by:
Selected Answer: [None Given]
Answers: a.
Matter
b.
Shielding
c.
Antimatter
d.
Neurons
• Question 17
0 out of 3.3 points
The ratio of maximum load to the rated plant capacity
Selected Answer: [None Given]
Answers: a.
Capacity factor
b.
Load factor
c.
Utilization factor
d.
Maximum factor
• Question 18
0 out of 3.3 points
How were tubing joints done?
Selected Answer: [None Given]
Answers: a.
Screwed
b.
Brazed
c.
Welded
d.
Silver soldered
• Question 19
0 out of 3.3 points
The temperature bulb of the solenoid valve is attached to the ___________.
Selected Answer: [None Given]
Answers: a.
Wall of the icebox
b.
Evaporator coil inlet
c.
Evaporator coil outlet
d.
Icebox coil
• Question 20
0 out of 3.3 points
Turbidity in water is due to:
Selected Answer: [None Given]
Answers: a.
Fungi
b.
Algae
c.
Organic salts
d.
Finally divided particles of clay, silt and organic matter
• Question 21
0 out of 3.3 points
What is the cause of pressure drop in the boiler, condenser and the pipings
between different components? Because of this drop, steam leaving the
boiler at a lower pressure.
Selected Answer: [None Given]
Answers: a.
negative slip
b.
low thermal efficiency
c.
fluid friction
d.
positive slip
• Question 22
0 out of 3.3 points
Anchor bolts in a machine foundation should be embedded in concrete of at
least how may time of the bolt diameter?
Selected Answer: [None Given]
Answers: a.
25
b.
20
c.
15
d.
30
• Question 23
0 out of 3.3 points
In order to remove the fly ashes from the flue gas, which of the following
must a power plant be equipped with?
Selected Answer: [None Given]
Answers: a.
demineralizer
b.
condenser
c.
desulphurization plant
d.
electrostatic precipitator
• Question 24
0 out of 3.3 points
Horsepower per ton of refrigeration is expressed as:
Selected Answer: [None Given]
Answers: a.
4.75/ cop
b.
4.75/ cop
c.
4.75 x cop
d.
Cop/ 4,75
• Question 25
0 out of 3.3 points
Orifice coefficients are used to determine:
Selected Answer: [None Given]
Answers: a.
Energy losses mass gains
b.
Energy losses
c.
Energy gains
d.
Mass losses
• Question 26
0 out of 3.3 points
A Freon – 12 gage shows pressure and ______.
Selected Answer: [None Given]
Answers: a.
Superheat temperature
b.
Back pressure
c.
Vacuum
d.
Saturation temperature
• Question 27
0 out of 3.3 points
The heat used to change a liquid to a gas or vapor is latent heat of
Selected Answer: [None Given]
Answers: a.
vaporization
b.
liquid
c.
fusion
d.
absorption
• Question 28
0 out of 3.3 points
The other name for Swamp Cooling is:
Selected Answer: [None Given]
Answers: a.
Evaporative Cooling
b.
Wet Roof Cooling
c.
Condensate Cooling
d.
Excelsior Cooling
• Question 29
0 out of 3.3 points
The machine foundation must have a factor of safety of:
Selected Answer: [None Given]
Answers: a.
6
b.
5
c.
4
d.
7
• Question 30
0 out of 3.3 points
Air delivered to the room from the supply duct, moving at a velocity of 150
ft/min or more is called:
Selected Answer: [None Given]
Answers: a.
Secondary Air
b.
Primary Air
c.
Air Turbulence
d.
Saturated Air
Tuesday, March 17, 2020 5:41:40 PM PST
• Question 1
0 out of 3.3 points
What usually happens if the specific gravity of the brine is too low?
Selected Answer: [None Given]
Answers: a.
It will be more heat-absorbing
b.
All of the other choices
c.
The brine will freeze
d.
Solids will deposits
• Question 2
0 out of 3.3 points
At ordinary temperature what substance will behave as inert gas and will not
react in the combustion process?
Selected Answer: [None Given]
Answers: a.
Hydrogen
b.
Nitrogen
c.
Carbon
d.
Sulfur
• Question 3
0 out of 3.3 points
When air is heated, what happened to its relative humidity?
Selected Answer: [None Given]
Answers: a.
Remain constant
b.
Increases
c.
May increase or Decrease depending on temperature
d.
Decreases
• Question 4
0 out of 3.3 points
A refrigeration control that guards the compressor forms overloads brought
about by abruptly increases loads resulting from defrosting, warm products
and others, is called:
Selected Answer: [None Given]
Answers: a.
safety valve
b.
expansion valve
c.
suction hold-back valve
d.
suction hold-back valve
• Question 5
0 out of 3.3 points
The primary purpose of a turbine in a fluid is to:
Selected Answer: [None Given]
Answers: a.
Add energy to the flow
b.
Add mass to the flow
c.
None of the other choices
d.
Extract energy from the flow
• Question 6
0 out of 3.3 points
The flow of a convergent section of a nozzle is always subsonic. If the flow is
subsonic then the mach number is:
Selected Answer: [None Given]
Answers: a.
greater than unity
b.
near than unity
c.
unity
d.
less than unity
• Question 7
0 out of 3.3 points
Air circulation in the icebox is accomplished by the use of which of the
following?
Selected Answer: [None Given]
Answers: a.
Diffuser fans
b.
Hollow sidewalls
c.
Louver doors
d.
Air vents to deck
• Question 8
0 out of 3.3 points
The cooling water regulator is automatically actuated by which of the
following?
Selected Answer: [None Given]
Answers: a.
The temperature of the refrigerant
b.
The discharge pressure of the refrigerant
c.
An electric relay
d.
A thermo relay
• Question 9
0 out of 3.3 points
If the compressor had been running satisfactorily for a long period of time
but suddenly the compartment temperature started to rise, the trouble
might be:
Selected Answer: [None Given]
Answers: a.
The expansion valve may contain frozen water
b.
The solenoid valve has jammed shut
c.
Any of the other choices
d.
A refrigerant leak has developed
• Question 10
0 out of 3.3 points
The expansion valve on a freon system controls the
Selected Answer: [None Given]
Answers: a.
Superheat of the gas leaving the evaporator
b.
Temperature of the icebox
c.
Back pressure in the evaporator
d.
Superheat of the gas leaving the compressor
• Question 11
0 out of 3.3 points
Which of the following can be the cause of low head pressure
Selected Answer: [None Given]
Answers: a.
Any of the other choices
b.
Insufficient charge of refrigerant
c.
Leaky discharge valves
d.
Too much or too cold condensing water
• Question 12
0 out of 3.3 points
The term used when the static pressure is equal to the atmospheric
pressure.
Selected Answer: [None Given]
Answers: a.
Induced draft
b.
Forced draft
c.
Balanced draft
d.
Natural draft
• Question 13
0 out of 3.3 points
In a refrigerating system, the heat absorbed in the evaporator per kg mass
of refrigerant passing through
Selected Answer: [None Given]
Answers: a.
Is decreased if pre- cooler is used
b.
Does not depend on the refrigerant used
c.
Equals the increase in enthalpy
d.
Equals the increase in volume
• Question 14
0 out of 3.3 points
In a cooling tower, the water is cooled mainly by:
Selected Answer: [None Given]
Answers: a.
conduction
b.
Evaporation
c.
convection
d.
Condensation
• Question 15
0 out of 3.3 points
There shall be no foundation bolts less than ___________.
Selected Answer: [None Given]
Answers: a.
16 mm in diameter
b.
20 mm in diameter
c.
18 mm in diameter
d.
12 mm in diameter
• Question 16
0 out of 3.3 points
An automatically controlled Freon – 12 compressor will start when the
_______.
Selected Answer: [None Given]
Answers: a.
Solenoid valve close
b.
Solenoid valve opens
c.
Expansion valve closes
d.
Expansion valve opens
• Question 17
0 out of 3.3 points
Which refrigerant has the highest critical point temperature?
Selected Answer: [None Given]
Answers: a.
Freon 12
b.
Freon 22
c.
Freon 11
d.
Ammonia
• Question 18
0 out of 3.3 points
Absolute zero is
Selected Answer: [None Given]
Answers: a.
The same as zero on the Fahrenheit scale
b.
144 degrees below zero on the Fahrenheit scale
c.
970 degrees below zero on the Fahrenheit scale
d.
460 degrees below zero on the Fahrenheit scale
• Question 19
0 out of 3.3 points
If W is the work output of a heat engine and H is the total heat input of an
engine, then determine its thermal efficiency, e.
Selected Answer: [None Given]
Answers: a.
e=WxH
b.
e=W/H
c.
e = I – (W – H)
d.
e=H/W
• Question 20
0 out of 3.3 points
The size of a reciprocating pump is stamped on the builder’s plate 3” x 4’ x
6”. The diameter of the liquid cylinder is:
Selected Answer: [None Given]
Answers: a.
6”
b.
4”
c.
7”
d.
5”
• Question 21
0 out of 3.3 points
What is the device used to protect the compressor from overloading due to
high head pressure
Selected Answer: [None Given]
Answers: a.
Expansion valve
b.
Overload relay
c.
Thermostatic expansion valve
d.
Hold back suction valve
• Question 22
0 out of 3.3 points
A “cold diffuser” is a:
Selected Answer: [None Given]
Answers: a.
None of the other choices
b.
Type of evaporator
c.
Special valve used as bypass
d.
Type of condenser
• Question 23
0 out of 3.3 points
What is the effect if the refrigerant is removed from the system too fast?
Selected Answer: [None Given]
Answers: a.
It will lower the icebox temperature
b.
It may “freeze – up” the condenser
c.
None of the other choices
d.
It may flood the evaporator
• Question 24
0 out of 3.3 points
The schedule number of a pipe, N, indicates the thickness of the pipe wall. If
the allowable stress of the pipe is S, then what is the internal pressure equal
to?
Selected Answer: [None Given]
Answers: a.
S/N
b.
N–S
c.
NxS
d.
N/S
• Question 25
0 out of 3.3 points
Hydraulic jump is used to:
Selected Answer: [None Given]
Answers: a.
Reduce the energy of flow
b.
Increase the flow rate
c.
Reduce the flow rate
d.
Reduce the velocity of flow
• Question 26
0 out of 3.3 points
When excess solute in a solution settles to the bottom of the container, the process
is called:
Selected Answer: [None Given]
Answers: a.
Salvation
b.
Equation
c.
Precipitation
d.
Deemulsification
• Question 27
0 out of 3.3 points
Which do you think is very important in adjusting compressor “v” belts?
Selected Answer: [None Given]
Answers: a.
Make belt just tight enough to turn pulley
b.
Keep belts parallel
c.
Make it as tight as possible
d.
Allow ½” slack
• Question 28
0 out of 3.3 points
External frost on inlet of expansion valve indicates:
Selected Answer: [None Given]
Answers: a.
Refrigerating compartment too cold
b.
Expansion valve plugged or dirty
c.
Air in system
d.
Head pressure too high
• Question 29
0 out of 3.3 points
What consists of weak solutions of sulfuric, hydrochloric, and to a lesser extent,
nitric acids?
Selected Answer: [None Given]
Answers: a.
Acid compound
b.
pollutant
c.
Stack gas
d.
Acid rain
• Question 30
0 out of 3.3 points
During the re-expansion portion of the refrigeration compressor cycle
Selected Answer: [None Given]
Answers: a.
The discharge valve is closed
b.
The suction valve is closed and the discharge valve is closed
c.
The suction valve is closed
d.
The suction valve is open
Tuesday, March 17, 2020 5:43:17 PM PST
• Question 1
0 out of 3.3 points
Two compressor should not be run in parallel because :
Selected Answer: [None Given]
Answers: a.
There is a possibility of losing oil
b.
There is a possibility of losing oil or it will give over – capacity
c.
It will give over – capacity
d.
It is not efficient
• Question 2
0 out of 3.3 points
Which of the following would cause the suction switch to remain open?
Selected Answer: [None Given]
Answers: a.
Insufficient spring tension
b.
Too much spring tension
c.
Bellows broken or jammed
d.
Insufficient spring tension or too much spring tension
• Question 3
0 out of 3.3 points
External frost on inlet of expansion valve indicates:
Selected Answer: [None Given]
Answers: a.
Head pressure too high
b.
Expansion valve plugged or dirty
c.
Air in system
d.
Refrigerating compartment too cold
• Question 4
0 out of 3.3 points
The minimum vertical distance from the floor or soil level to the top edge of
the foundation must be around __________.
Selected Answer: [None Given]
Answers: a.
120 mm
b.
150 mm
c.
100 mm
d.
200 mm
• Question 5
0 out of 3.3 points
A single trunk piston-type compressor is undesirable for a Freon unit
because the:
Selected Answer: [None Given]
Answers: a.
Lubricant mixes with the refrigerant
b.
Refrigerant becomes superheated
c.
Refrigerant reduces the crankcase pressure
d.
Lubricant temperature becomes excessive
• Question 6
0 out of 3.3 points
One could expect the possibility of Froude number similarity in all of the ff
cases except:
Selected Answer: [None Given]
Answers: a.
Motion of a fluid jet
b.
Surge and flood waves
c.
Subsonic airfoils
d.
Flow over spillways
• Question 7
0 out of 3.3 points
The charging connection in a refrigerating system is located:
Selected Answer: [None Given]
Answers: a.
Between the king valve and the solenoid valve
b.
Between the condenser and the receiver
c.
Between the receiver the king valve
d.
Before the receiver
• Question 8
0 out of 3.3 points
If the pressure is disregarded in the various other components of a steam
gas power plants, the pressure rise in the pump or compressor is ______?
Selected Answer: [None Given]
Answers: a.
equal
b.
inversely proportional
c.
constant
d.
varying
• Question 9
0 out of 3.3 points
The coefficient of velocity is equal to the:
Selected [None Given]
Answer:
Answers: a.
Difference of the coefficient of discharge and the coefficient of
contraction
b.
Product of the coefficient of discharge and the coefficient of
contraction
c.
Actual velocity divided by the theoretical velocity
d.
Sum of the coefficient of discharge and the coefficient of
contraction
• Question 10
0 out of 3.3 points
When the outlet temperature of the evaporator exceeds the inlet
temperature the condition is called ______________.
Selected Answer: [None Given]
Answers: a.
Boiling
b.
Superheating
c.
Melting
d.
Freezing
• Question 11
0 out of 3.3 points
All of the ff occur during reduction of a substance except
Selected Answer: [None Given]
Answers: a.
An increase in negative charge
b.
Loss of electrons
c.
Reduction of the oxidizing agent
d.
An oxidation state decrease
• Question 12
0 out of 3.3 points
What do you call the liquid reaching the compressor through the suction?
Selected Answer: [None Given]
Answers: a.
Superheating
b.
Recycling
c.
Overflowing
d.
Flooding back
• Question 13
0 out of 3.3 points
The design of an air supply duct of an air conditioning system
Selected Answer: [None Given]
Answers: a.
adds moisture to the air
b.
does not affect the moisture of the air
c.
lowers the temperature of the air
d.
affects the distribution
• Question 14
0 out of 3.3 points
In an air conditioning unit, the thermostat fails. The unit did not start. How
do you test the operation of the thermostat?
Selected [None Given]
Answer:
Answers: a.
Cover both the inlet and outlet so that the recirculating air has
the same temperature.
b.
Cover the outlet and the air will not circulate and the thermostat
functions because no air movement.
c.
Never cover any air passage so that air can freely move and
thermostat functions well.
d.
Cover the air outlet and air inlet with a cloth. The air will
recirculate into the unit and the temperature will quickly drop to
the cut out temperature.
• Question 15
0 out of 3.3 points
The area under load curve divided by the maximum demand represents:
Selected Answer: [None Given]
Answers: a.
Average load
b.
Connected factor
c.
Load factor
d.
Diversity factor
• Question 16
0 out of 3.3 points
What will happen if the expansion valve is opened too wide?
Selected [None Given]
Answer:
Answers: a.
Liquid will flood back to the compressor or the condenser coils
will overheat
b.
Liquid will flood back to the compressor
c.
The condenser coils will overheat
d.
It will make no difference, as the automatic expansion valve is
still operating
• Question 17
0 out of 3.3 points
When charging freon system, all the valves
should be
Selected Answer: [None Given]
Answers: a.
Solenoid valve
b.
Expansion valve
c.
Purge valve
d.
King (liquid) valve
• Question 18
0 out of 3.3 points
Expansion factors take into account the:
Selected Answer: [None Given]
Answers: a.
Small effect of friction and turbulence of the orifice
b.
Area of the vena contracta
c.
Effects of compressibility
d.
Changes in diameters of converging pipe
• Question 19
0 out of 3.3 points
What is the instrument used to register relative humidity?
Selected Answer: [None Given]
Answers: a.
hygrometer
b.
hydrometer
c.
manometer
d.
perometer
• Question 20
0 out of 3.3 points
High superheat of the vapor in the system would cause
Selected Answer: [None Given]
Answers: a.
A decrease in capacity
b.
An increase in capacity and a more efficient unit
c.
An increase in capacity
d.
A more efficient unit
• Question 21
0 out of 3.3 points
A type of polymer used for clear bottles.
Selected Answer: [None Given]
Answers: a.
Polyvinyl chloride(PVC)
b.
Polypropylene(PP)
c.
Low density polyethylene (LDPE)
d.
Polystyrene(PS)
• Question 22
0 out of 3.3 points
Compare the temperature of discharge vapor refrigerant leaving the
compressor for a superheated cycle and the saturated cycle, for the same
condensing temperature and pressure.
Selected Answer: [None Given]
Answers: a.
higher for the superheated cycle
b.
there is no difference
c.
higher for the saturated cycle
d.
lower for the superheated cycle
• Question 23
0 out of 3.3 points
The temperature at which water vapor in the atmosphere begins to
condense is known as the:
Selected Answer: [None Given]
Answers: a.
vapor point
b.
dew point
c.
moisture point
d.
none of the other choices
• Question 24
0 out of 3.3 points
Heating and dehumidification can be obtained simultaneously if air is passed
through:
Selected [None Given]
Answer:
Answers: a.
a liquid absorbent spray
b.
sprays of water kept at a temperature higher than the dew point
temperature of the entering air
c.
a solid absorbent surface
d.
either a solid absorbent surface or a liquid absorbent spray
• Question 25
0 out of 3.3 points
The main purpose of a subcooler in a refrigerating system especially a two
stage system is to:
Selected [None Given]
Answer:
Answers: a.
Reduce the total power requirements and return oil to the
compressor
b.
Increase the heat rejection per ton and avoid system shutdown
c.
Improve the flow of evaporator gas per ton and increase the
temperature
d.
Reduce the total power requirements and heat rejection to the
second stage
• Question 26
0 out of 3.3 points
In balance system, the available draft is
Selected Answer: [None Given]
Answers: a.
Infinite
b.
100
c.
Unity
d.
Zero
• Question 27
0 out of 3.3 points
The method of cooling which primarily used where ambient air
temperatures are high and relative humidity is used:
Selected Answer: [None Given]
Answers: a.
Evaporative cooling
b.
Hydroionic cooling
c.
Condensate cooling
d.
Swamp cooling
• Question 28
0 out of 3.3 points
The primary purpose of a turbine in a fluid is to:
Selected Answer: [None Given]
Answers: a.
Extract energy from the flow
b.
Add mass to the flow
c.
None of the other choices
d.
Add energy to the flow
• Question 29
0 out of 3.3 points
The size of a reciprocating pump is stamped on the builder’s plate 3” x 4’ x
6”. The diameter of the liquid cylinder is:
Selected Answer: [None Given]
Answers: a.
6”
b.
5”
c.
4”
d.
7”
• Question 30
0 out of 3.3 points
Thermal expansion valves are usually made of the:
Selected Answer: [None Given]
Answers: a.
Bellows type
b.
Diaphragm
c.
Diaphragm and Bellows type
d.
Magnetic type
Tuesday, March 17, 2020 5:44:38 PM PST
• Question 1
0 out of 3.3 points
The purpose of the dehydrator is to
Selected Answer: [None Given]
Answers: a.
Add more refrigerant to the system
b.
Remove moisture from the refrigerant
c.
Remove moisture from the crankcase oil
d.
Remove oil from the refrigerant
• Question 2
0 out of 3.3 points
If PV is the power required for a vapor compression refrigeration system ,
then what is the power required for an air refrigeration system, assuming
that they have the same capacity?
Selected Answer: [None Given]
Answers: a.
PV/10
b.
2V
c.
5PV
d.
1/PV
• Question 3
0 out of 3.3 points
A type of refrigerant that will not damage the ozone layer.
Selected Answer: [None Given]
Answers: a.
Hydrofluorocarbons (HCF’s)
b.
Hydrochlorofluorocarbons (HCHF’s)
c.
R-12
d.
R-22
• Question 4
0 out of 3.3 points
Which of the ff is not pollutant from a sulfuric acid plants?
Selected Answer: [None Given]
Answers: a.
Sulfur trioxide
b.
Acid moist
c.
Sulfur dioxide
d.
Hydrogen sulfide
• Question 5
0 out of 3.3 points
Before securing a compressor to do maintenance work on it, be sure to:
Selected Answer: [None Given]
Answers: a.
Purge the system
b.
Pump down the system
c.
Have spare parts ready and pump down the system
d.
Have spare parts ready
• Question 6
0 out of 3.3 points
The ice making capacity is always
Selected Answer: [None Given]
Answers: a.
Less than the refrigerating effect
b.
Greater than the refrigerating effect
c.
Equal to the refrigerating effect
d.
Directly proportional to the refrigerating effect
• Question 7
0 out of 3.3 points
When air contains all of the water vapor it can hold, it is said to be
Selected Answer: [None Given]
Answers: a.
saturated
b.
moisture
c.
loaded
d.
simulated
• Question 8
0 out of 3.3 points
In a power driven pump, each piston stroke is displaced by 360 divide the
______.
Selected Answer: [None Given]
Answers: a.
number of cylinders
b.
revolution per minute
c.
bore
d.
length of the stroke
• Question 9
0 out of 3.3 points
Substances that have the ability to absorb moisture from the air are called:
Selected Answer: [None Given]
Answers: a.
Desiccants
b.
Moisturizer
c.
Dehygroscopic substances
d.
Moisture absorber
• Question 10
0 out of 3.3 points
A scale trap in a Freon system will be found on the
Selected Answer: [None Given]
Answers: a.
Receiver
b.
Discharge side
c.
Condenser
d.
Suction side
• Question 11
0 out of 3.3 points
Which of the following is not essential to a compression refrigeration
system?
Selected Answer: [None Given]
Answers: a.
An evaporator
b.
A receiver
c.
A dehydrator
d.
A condenser
• Question 12
0 out of 3.3 points
The fact that the amount of slightly soluble gas absorbed in a liquid is proportional
to the partial pressure of the gas is known as:
Selected Answer: [None Given]
Answers: a.
Boyle’s law
b.
Henry’s law
c.
Dalton’s law
d.
Raoult’s law
• Question 13
0 out of 3.3 points
To check the lack of refrigerant or clogged refrigerant lines:
Selected Answer: [None Given]
Answers: a.
Discharge pressure and temperature must be monitored
b.
Sight glasses can be possibly used
c.
Installing king valve must be an option
d.
Installing service valves must be necessary
• Question 14
0 out of 3.3 points
Most nuclear particles can react with atoms in several different ways including
Selected Answer: [None Given]
Answers: a.
Scattering
b.
Reflection and absorption
c.
Absorption
d.
Absorption and scattering
• Question 15
0 out of 3.3 points
A Francis turbine has what flow?
Selected Answer: [None Given]
Answers: a.
Outward flow reaction
b.
Outward flow impulse
c.
Inward flow reaction
d.
Inward flow impulse
• Question 16
0 out of 3.3 points
It is used to control the air-throw distance, height, and spread, as well as the
amount of air.
Selected Answer: [None Given]
Answers: a.
Damper
b.
Register
c.
Diffuser
d.
Grille
• Question 17
0 out of 3.3 points
The faster way to remove frost from a direct expansion finned tube
evaporator is to:
Selected Answer: [None Given]
Answers: a.
Wash with warm water
b.
Send hot gas through the coil
c.
Shut down warm let frost melt
d.
Scrape off frost
• Question 18
0 out of 3.3 points
Superheat is heat added ________.
Selected Answer: [None Given]
Answers: a.
To increase pressure
b.
In changing liquid to vapor
c.
After all liquid has been changed to vapor
d.
To increase temperature
• Question 19
0 out of 3.3 points
Everything is in its normal operating position when charging a system except
the
Selected Answer: [None Given]
Answers: a.
Compressor discharge valve
b.
King (receiver discharge) valve
c.
Solenoid valve
d.
Purge valve
• Question 20
0 out of 3.3 points
Which of the following would cause expansion valve failure?
Selected Answer: [None Given]
Answers: a.
Dirt in the valve
b.
Bulb in icebox ruptured
c.
Moisture in the system
d.
Any of the other choices
• Question 21
0 out of 3.3 points
Before securing a compressor to do maintenance work on it, be sure to
Selected [None Given]
Answer:
Answers: a.
Make arrangements to have perishables taken care of
b.
Have gas mask handy
c.
Have gas mask handy and make arrangements to have
perishables taken care of
d.
Notify the engineer
• Question 22
0 out of 3.3 points
Large-bulb alcohol thermometer used to measure air speed or atmospheric
conditions by means of cooling.
Selected Answer: [None Given]
Answers: a.
Kelvin Thermometer
b.
JJ Thompson Thermometer
c.
Wet Bulb Thermometer
d.
Kata Thermometer
• Question 23
0 out of 3.3 points
The desirable temperature inside an air conditioned auditorium is:
Selected Answer: [None Given]
Answers: a.
5C
b.
0C
c.
20 C
d.
10 C
• Question 24
0 out of 3.3 points
Water is heated in a container. It expands and becomes less dense and
lighter. It rises up the container because if it’s reduced density and replaced
by cooler air. As this process continues, the heat is transferred and disturbed
all throughout. What is this mode of heat transfer called?
Selected Answer: [None Given]
Answers: a.
radiation
b.
convection
c.
condenser
d.
conduction
• Question 25
0 out of 3.3 points
If the head pressure is too high:
Selected [None Given]
Answer:
Answers: a.
The relief should open before the high-pressure cutout
b.
The high pressure cutout switch should operate before the
relief valve opens
c.
The relief valve should open and let excess refrigerant pass to
receiver
d.
Close in on the suction valve
• Question 26
0 out of 3.3 points
The ozone concentration of 0.10 parts per million (ppm) is generally
considered the maximum permissible for how many hours exposure?
Selected Answer: [None Given]
Answers: a.
7 hours
b.
8 hours
c.
4 hours
d.
3 hours
• Question 27
0 out of 3.3 points
Which of the following will cause an automatically controlled F-12
compressor to start?
Selected Answer: [None Given]
Answers: a.
Opening of the solenoid valve
b.
Closing of the expansion valve
c.
Closing of the solenoid valve
d.
Opening of the expansion valve
• Question 28
0 out of 3.3 points
The effect of superheating the refrigerant is to:
Selected Answer: [None Given]
Answers: a.
Ammonia
b.
Decrease the COP
c.
Increase the COP
d.
Maintains the COP
• Question 29
0 out of 3.3 points
Equipment leaks from plant equipments are known as
Selected Answer: [None Given]
Answers: a.
Fugitive exhausts
b.
Fugitive leaks
c.
Fugitive dusts
d.
Fugitive emissions
• Question 30
0 out of 3.3 points
The other name for Swamp Cooling is:
Selected Answer: [None Given]
Answers: a.
Excelsior Cooling
b.
Condensate Cooling
c.
Wet Roof Cooling
d.
Evaporative Cooling
Tuesday, March 17, 2020 5:46:34 PM PST
• Question 1
0 out of 3.3 points
A “cold diffuser” is a:
Selected Answer: [None Given]
Answers: a.
Special valve used as bypass
b.
Type of condenser
c.
Type of evaporator
d.
None of the other choices
• Question 2
0 out of 3.3 points
The relative humidity becomes 100 % and where the water vapor starts to
condense.
Selected Answer: [None Given]
Answers: a.
Saturated Point
b.
Critical Point
c.
Dew Point
d.
Cloud Point
• Question 3
0 out of 3.3 points
White ice is
Selected Answer: [None Given]
Answers: a.
Due to dissolved air, gasses and impurities
b.
Fast cooled water Sub cooled water
c.
Sub cooled water
d.
Formed by blowing air during freezing
• Question 4
0 out of 3.3 points
The principle of mechanical refrigeration is
Selected [None Given]
Answer:
Answers: a.
The conversion of a liquid to a gas
b.
The absorption of heat under temperature, compression,
pressure and expansion
c.
The compression of a liquid under temperature and expansion
d.
The absorption of temperature under heat, pressure,
compression and expansion
• Question 5
0 out of 3.3 points
Which of the following best described a Freon?
Selected Answer: [None Given]
Answers: a.
Non-poisonous
b.
Odorless
c.
Colorless
d.
All of the other choices
• Question 6
0 out of 3.3 points
What is the device used to protect the compressor from overloading due to
high head pressure
Selected Answer: [None Given]
Answers: a.
Overload relay
b.
Thermostatic expansion valve
c.
Hold back suction valve
d.
Expansion valve
• Question 7
0 out of 3.3 points
What tonnage of refrigerating machine is required if the refrigerating system
extracted 48,000 btu per hour?
Selected Answer: [None Given]
Answers: a.
2 tons
b.
4 tons
c.
5 tons
d.
3 tons
• Question 8
0 out of 3.3 points
Antifreeze chemicals are:
Selected Answer: [None Given]
Answers: a.
Those that are added to refrigerants for better performance
b.
Those that lower down the freezing points of liquids
c.
Same as refrigerants
d.
Those that do not freeze at all
• Question 9
0 out of 3.3 points
If Pi is the indicated horsepower and Pb is the indicated horsepower of a
compressor, then what is mechanical efficiency, Em, equal to:
Selected Answer: [None Given]
Answers: a.
Em = Pb-Pi
b.
Em = Pi - Pb
c.
Em = Pi/Pb
d.
Em = Pb/Pi
• Question 10
0 out of 3.3 points
In parallel pipe system originating and terminating in common junctions,
Selected Answer: [None Given]
Answers: a.
Lengths of each branch are equal
b.
Pressure drops through each branch are equal
c.
Mass flows through each branch are equal
d.
Flow areas of each branch are equal
• Question 11
0 out of 3.3 points
Is a soluble compound that reduces a liquid’s surface tension or reduces the
interfacial tension between a liquid and a solid
Selected Answer: [None Given]
Answers: a.
Surface – acting agent or surfactant
b.
CFC
c.
Oxygenated fuel
d.
Humectant
• Question 12
0 out of 3.3 points
The suction control switch on the compressor is a
Selected Answer: [None Given]
Answers: a.
Pressure elements
b.
Bellows
c.
Thermal elements
d.
Thermostat
• Question 13
0 out of 3.3 points
What does enthalpy measure in a substance?
Selected Answer: [None Given]
Answers: a.
its coldness
b.
its dew point
c.
its humidity
d.
its heat content
• Question 14
0 out of 3.3 points
All of the ff express units of concentration except
Selected Answer: [None Given]
Answers: a.
Molarity
b.
Formality
c.
Normality
d.
Isotropy
• Question 15
0 out of 3.3 points
The greatest decrease in refrigerant temperature occurs in the _________
Selected Answer: [None Given]
Answers: a.
Condenser
b.
Evaporator
c.
Compressor
d.
Evaporator or condenser
• Question 16
0 out of 3.3 points
In sensible cooling, moisture content.
Selected Answer: [None Given]
Answers: a.
indeterminate
b.
increases
c.
Does not change
d.
decreases
• Question 17
0 out of 3.3 points
The capacity in ice making is always
Selected Answer: [None Given]
Answers: a.
Less than the refrigerating effect
b.
Equal to the refrigerating effect
c.
Greater than the refrigerating effect
d.
Directly proportional to the refrigerating effect
• Question 18
0 out of 3.3 points
Is the subjective method in which the smoke density is visually compared to five
standardized white-black grids.
Selected Answer: [None Given]
Answers: a.
Smoke spot scale
b.
Dew point scale
c.
Dalton scale
d.
Ringelman scale
• Question 19
0 out of 3.3 points
What is the most common freon gas used in reciprocating compressors?
Selected Answer: [None Given]
Answers: a.
F-21
b.
F-12
c.
F-22
d.
F-11
• Question 20
0 out of 3.3 points
All of the ff occur during reduction of a substance except
Selected Answer: [None Given]
Answers: a.
An oxidation state decrease
b.
Loss of electrons
c.
An increase in negative charge
d.
Reduction of the oxidizing agent
• Question 21
0 out of 3.3 points
The burning of low- sulfur fuel is known as:
Selected Answer: [None Given]
Answers: a.
Fuel switching
b.
Fuel adding
c.
Fuel binding
d.
Fuel swapping
• Question 22
0 out of 3.3 points
Which of the following is the cause if the outlet of a thermostatic valve is
warmer than the inlet side?
Selected Answer: [None Given]
Answers: a.
Valve is not working properly
b.
Valve is working properly
c.
None of the other choices
d.
Solenoid valve is not working properly
• Question 23
0 out of 3.3 points
A rule which states that any waste derived from the treatment of a hazardous waste
remains a hazardous waste
Selected Answer: [None Given]
Answers: a.
The treatment rule
b.
The mixture rule
c.
The derived from rule
d.
The hazard rule
• Question 24
0 out of 3.3 points
Excess fuel oil in the fuel oil system returns to the:
Selected Answer: [None Given]
Answers: a.
Combustion chamber
b.
Suction line
c.
Fuel oil tank
d.
Burner
• Question 25
0 out of 3.3 points
What is the usual cop of a domestic refrigerator?
Selected Answer: [None Given]
Answers: a.
Less than 1
b.
Equal to 1
c.
More than 1
d.
Depends upon the make of it
• Question 26
0 out of 3.3 points
The primary purpose of a turbine in a fluid is to:
Selected Answer: [None Given]
Answers: a.
Extract energy from the flow
b.
Add mass to the flow
c.
None of the other choices
d.
Add energy to the flow
• Question 27
0 out of 3.3 points
A “CARRENE” is a type of :
Selected Answer: [None Given]
Answers: a.
Refrigerant oil
b.
Scale cleaner
c.
Refrigerant
d.
None of the other choices
• Question 28
0 out of 3.3 points
In order to perform efficiently, a power cycle must be communicated with
the outside temperature of its surroundings and into one is called:
Selected Answer: [None Given]
Answers: a.
First Law of Thermodynamics
b.
Second Law of Thermodynamics
c.
Kirchoff’s Law
d.
Kelvin-Planck Law
• Question 29
0 out of 3.3 points
Heating and dehumidification can be obtained simultaneously if air is passed
through:
Selected [None Given]
Answer:
Answers: a.
either a solid absorbent surface or a liquid absorbent spray
b.
a solid absorbent surface
c.
a liquid absorbent spray
d.
sprays of water kept at a temperature higher than the dew point
temperature of the entering air
• Question 30
0 out of 3.3 points
“Particulate matter” is defined as all particles that are emitted by a combustion
source. What is another term for particulate matter?
Selected Answer: [None Given]
Answers: a.
Aerosol
b.
Biosolids
c.
Dust
d.
Sediments
Tuesday, March 17, 2020 5:49:22 PM PST
• Question 1
0 out of 3.3 points
Ammonia will corrode
Selected Answer: [None Given]
Answers: a.
Copper
b.
Brass
c.
All of the other choices
d.
Bronze
• Question 2
0 out of 3.3 points
If the outlet of the thermostatic valve is warmer than the inlet, it indicates
Selected Answer: [None Given]
Answers: a.
Thermostatic valve working properly
b.
Thermostatic valve not working properly
c.
Flooding back
d.
Solenoid valve not working properly
• Question 3
0 out of 3.3 points
The relationship of water vapor in the air at dew point temperature to the
amount that would be in the air if the air were saturated at the dry bulb
temperature is:
Selected Answer: [None Given]
Answers: a.
partial pressure actual at dew point
b.
Percentage humidity
c.
Relative humidity
d.
Partial pressure of water
• Question 4
0 out of 3.3 points
What must be done to change the direction of rotation of a 440-volt, 3-phase
induction motor?
Selected Answer: [None Given]
Answers: a.
Reverse lines to start winding
b.
Remove any power leads
c.
Interchange any two power leads
d.
Replace any two wires to the rotor
• Question 5
0 out of 3.3 points
The ratio of maximum load to the rated plant capacity is called
Selected Answer: [None Given]
Answers: a.
Capacity
b.
Load factor
c.
Maximum load factor
d.
Utilization factor
• Question 6
0 out of 3.3 points
Moisture in a system will cause a:
Selected Answer: [None Given]
Answers: a.
High suction pressure
b.
Faulty expansion valve
c.
High suction temperature
d.
Low discharge temperature
• Question 7
0 out of 3.3 points
Reheating process is normally employed in steam plane when:
Selected Answer: [None Given]
Answers: a.
there is excess steam
b.
subcooling is required
c.
dehumidifying is required
d.
turbine undergoes excessive moisture
• Question 8
0 out of 3.3 points
Antifreeze chemicals are:
Selected Answer: [None Given]
Answers: a.
Those that lower down the freezing points of liquids
b.
Those that are added to refrigerants for better performance
c.
Those that do not freeze at all
d.
Same as refrigerants
• Question 9
0 out of 3.3 points
What is the method used to evaluate all welds performed on pressure parts
of boiler tube materials?
Selected Answer: [None Given]
Answers: a.
hydrostatic test
b.
vacuum test
c.
radiographic test
d.
orsat analysis
• Question 10
0 out of 3.3 points
Measurement of a device’s ability to remove atmospheric air from test air.
Selected Answer: [None Given]
Answers: a.
Atmospheric Dry Air Efficiency
b.
Test Air Efficiency
c.
Atmospheric Dust Spot Efficiency
d.
Baudelot Air Efficiency
• Question 11
0 out of 3.3 points
All of the ff express units of concentration except
Selected Answer: [None Given]
Answers: a.
Isotropy
b.
Molarity
c.
Normality
d.
Formality
• Question 12
0 out of 3.3 points
The relative humidity is given by the:
Selected [None Given]
Answer:
Answers: a.
ratio of the partial pressure of water vapour to the saturation
pressure
b.
ratio of the actual humidity to the saturated humidity at the
same temperature and pressure
c.
ratio of wet-bulb to dry-bulb temperature
d.
ratio of dry-bulb temperature to dew point
• Question 13
0 out of 3.3 points
The ratio of the sum of individual maximum demands of the system to the overall
maximum demand of the whole system
Selected Answer: [None Given]
Answers: a.
Demand factor
b.
Power factor
c.
Utilization factor
d.
Diversity factor
• Question 14
0 out of 3.3 points
Where is the excess refrigerant removed?
Selected Answer: [None Given]
Answers: a.
Suction side of the system
b.
By pass
c.
Charging side of the system
d.
Discharge side of the system
• Question 15
0 out of 3.3 points
The total pressure supplied by the fan at maximum operating conditions
Selected Answer: [None Given]
Answers: a.
Stack effect
b.
Net rating or fan boost
c.
Available draft
d.
Draft loss
• Question 16
0 out of 3.3 points
Is a high- temperature process that turns incinerator ash into a safe, glass –like
material
Selected Answer: [None Given]
Answers: a.
Biofitration
b.
Bioventing
c.
Advance oxidation
d.
Vitrification
• Question 17
0 out of 3.3 points
If the solenoid valve closed by accident, the compressor would be stopped
by which of the following?
Selected Answer: [None Given]
Answers: a.
Low-pressure cutout switch
b.
Automatic trip
c.
High-pressure cutout switch
d.
Low-water cutout switch
• Question 18
0 out of 3.3 points
Which of the following would cause high head pressure?
Selected Answer: [None Given]
Answers: a.
Dirty condenser
b.
Air in the condenser
c.
Any of the other choices
d.
Insufficient cooling water
• Question 19
0 out of 3.3 points
At 0 psig, how many BTU necessary to change 1 lb of water at 212 F of
steam?
Selected Answer: [None Given]
Answers: a.
180
b.
970
c.
144
d.
1190
• Question 20
0 out of 3.3 points
Which of the following is not a characteristics of Freon- 12?
Selected Answer: [None Given]
Answers: a.
Boiling point -21 F
b.
Non-inflammable
c.
Corrosive
d.
Odorless
• Question 21
0 out of 3.3 points
A ____ boiler has heat and gases of combustion that pass through tubes
surrounded by water
Selected Answer: [None Given]
Answers: a.
Fire tube
b.
Water tube
c.
Straight-tube
d.
Cast iron sectional
• Question 22
0 out of 3.3 points
When an electric motor nameplate indicate a “100-kw electric motor” then
what does a 100-kw rating refers to?
Selected Answer: [None Given]
Answers: a.
mechanical power output
b.
thermal energy output
c.
thermal energy input
d.
electrical power output
• Question 23
0 out of 3.3 points
Cooling towers are used for cooling water:
Selected Answer: [None Given]
Answers: a.
to be used for filtration of air
b.
to be used for cooling the compressor
c.
to be used for humidification
d.
to be injected in circulating air
• Question 24
0 out of 3.3 points
Which of the following is not essential to a centrifugal type compressor
system?
Selected Answer: [None Given]
Answers: a.
Expansion valve
b.
Condenser
c.
Distiller
d.
Evaporator
• Question 25
0 out of 3.3 points
What is the pressure present inside the casing of an impulse turbine?
Selected Answer: [None Given]
Answers: a.
above atmospheric
b.
zero
c.
vacuum
d.
atmospheric pressure
• Question 26
0 out of 3.3 points
The velocity of radiation (v) and the frequency of radiations (f) are related to the
wavelength of radiations by:
Selected Answer: [None Given]
Answers: a.
f/v
b.
v/f
c.
vf
d.
v2f
• Question 27
0 out of 3.3 points
As far as combustion chamber design is considered, the maximum power
output of a given engine can be increased by:
Selected Answer: [None Given]
Answers: a.
increasing combustion time
b.
increasing combustion chamber volume
c.
providing small values
d.
decreasing combustion chamber volume
• Question 28
0 out of 3.3 points
Calcium chloride is sometimes used in refrigeration system as a:
Selected Answer: [None Given]
Answers: a.
Refrigerant
b.
Primary Coolant
c.
Secondary Coolant
d.
Lubricant
• Question 29
0 out of 3.3 points
Theoretically, what is the effect of the compressor clearance on horsepower?
Selected Answer: [None Given]
Answers: a.
decrease power
b.
increase power
c.
there is no effect
d.
varies in direct proportion
• Question 30
0 out of 3.3 points
How were tubing joints done?
Selected Answer: [None Given]
Answers: a.
Brazed
b.
Screwed
c.
Welded
d.
Silver soldered
Tuesday, March 17, 2020 5:51:05 PM PST
IPE
C 1. Monochlorofluoromethane is a refrigerant B 13.A vessel permanently connected to a system
known as: by inlet and outlet pipes of storage of liquid
a. R-12 c. R-22 refrigerant
b. R-13 d. None of the above a. refrigerant container c. flash tank
b. liquid receiver d. surge tank
D 2. Which of the ff. is a proper way of leak
detection for R-12 and halocarbon B 14.Air flows through coil without making
refrigeration systems? contact with the coil surface is known as
a. stains c. toxic odor a. drawn air c. contact air
b. corrosion d. loss of cooling capacity b. bypass air d. air load

C 3. It refers to the final constant weight reached A 15.the process of lowering the temperature of
by a hygroscopic substance after being dried given space and maintaining it for the
out purpose of preserving and chilling certain
a. Bone Dry Weight c. Both a&b substances
b. Dry Weight d. NOTA a. refrigeration c. chilling
b. cooling d. air-conditioning
B 4. Is a type of dryer in w/c material to be dried
is supported on floor through w/c hot gases D 16.it will either cool or maintain a body at
pass temp below that of its surrounding is known
a. Tray dryer c. Rotary dryer as the
b. Hearth dryer d. Tower dryer a. heat engine c. heat pump
b. heat sink d. refrigerating machine
B 5. It is the temp. at w/c liquid starts to boil or
the temp. at w/c vapor begins to condense B 17.a reciprocating compressor has a piston
a. Dry blub temp c. Dew point Temp speed of 200 fpm at 400 rpm. Its stroke is
b. Sat. Temp d. Wet bulb temp approx.
a. 2.56 in c. 5.32 in
D 6. The tendency of pump to cavitate will be b. 3 in. d. 20 in
increased if:
a. impeller has smooth finish C 18.a refrigeration system that combines two
b. inlet edges are rounded refrigeration units
c. suction line velocities are lowered a. multi-evaporation system
d. fluid temp is raised b. multi-stage compression system
c. cascade system
B 7. For a single component system, the number d. air-refrigeration cycle
of properties required to define a phase
uniquely is: C 19.fire involving energized electrical equipment
a. 1 c. 3 a. class A fire c. class C fire
b. 2 d. 4 b. class B fire d. class D fire

A 8. It draws the flue gas from the system and D 20.dew point temp is temp where water starts
series them up the stack is: a. to condense
a. induced draft fan c. forced draft fan b. to boil
b. cyclone d. wind tunnel c. to condense and boil
d. to condense and boil at given pressure
D 9. The study of properties of air and its water
vapor contents A 21.glass prod process is broken down into 2
a. humidification c. ventilation gen. categories: container prod (glass
b. refrigeration d. psychrometry bottle/bottle) and sheet prod (float glass)
a. container & sheet prod
B 10.It exist when the total suction is above b. box & sheet prod
atmospheric pressure c. container & flattening prod
a. suction lift c. suction pressure d. NOTA
b. suction head d. suction line
C 22.modern detergents are comprised of three
C 11.It is the wastewater that will flow into each main components
unit of operation of the wastewater treatment a. builders, blend, extender
plant that is partially or completely treated b. binders, bleach, enzymes
a. liquid waste c. influent c. builders, bleach, enzymes
b. solid waste d. effluent d. No answer

A 12.It raises pressure of CO2 gas to 250 psig so


that it can be liquefied at -100F
a. CO2 compressor c. deodorizer
b. trim cooler d. foam trap
A 23.is a broad term applied to large number of C 30.water substance at 70 bar and 65C enters a
carbohydrates composed of carbon, boiler tube of constant inside diameter of 25
hydrogen, and oxygen present in many mm. the water leaves the boiler tube at 50
plants and characterized by a more or less bar and 700K at velocity of 100 m/s.
sweet taste Calculate the inlet volume flow in L/sec.
a. sugar c. iron At 70 bar & 65C:
b. salt d. sweet v1 = 0.001017 m3/kg
At 50 bar & 700K:
B 24.a closed vessel contains air at a pressure of v2 = 0.06081 m3/kg
160 kN/m2 gauge and temp of 30C. The air a. 0.75 c. 0.82
is heated at const. vol. to 60C w/ atm b. 0.64 d. 0.96
pressure of 759mmHg. What is the final
gauge pressure? D 31.twenty grams of oxygen gas (O2) are
a. 174 c. 167 compressed at const temp of 30C to 5% of
b. 186 d. 172 their original vol. What work is done on the
system?
C 25.what is the total reqd heating energy in `Use R of air = 0.0619 cal/gm-K
raising the temp of given amt of water when a. 824 cal c. 944 cal
the energy supplied is 1000kWh w/ heat b. 894 cal d. 1124 cal
losses of 25% ( )( )
a. 1000 c. 1333
b. 1500 d. 1250 C 32.gas is enclosed in a cylinder w/ weighted
̅̅̅̅ piston as top boundary. The gas is heated
and expands from a volume of 0.04 m3 to
0.10 m3 at const pressure of 200 kPa.
B 26.a certain gas at 101.325 kPa and 16C whose Calculate the work done by system.
vol is 2.83m3 are compressed into a storage a. 8 kJ c. 12 kJ
vessel of 0.31 m3 capacity. Before b. 10 kJ d. 14 kJ
admission, the storage vessel contained the ( )
gas at pressure and temp of 137.8kPa and
24C. After admission, the pressure has B 33.if F scale is twice C scale, what is the
increased to 1171.8 kPa. What should be the reading in the Fahrenheit scale?
final temp of gas in vessel (in Kelvin)? a. 160 c. 140
a. 298.0 c. 180 b. 320 d. 280
b. 319.0 d. 420
B 34.a cylinder & piston arrangement contains
A 27.a tank contains 80 ft3 of air at pressure of sat water vapor at 110C. The vapor is
350 psi. If the air is cooled until its pressure compressed in a reversible adiabatic process
and temp decrease to 200 psi and 70F until pressure is 1.6 MPa. Determine work
respectively, what is the decrease in internal done by the system per kg of water
energy? At 110C:
a. 4575 c. 5552 S1 = 7.2387 kJ/kg-K
b. -5507 d. 0 U1 = 2518.1 kJ/kg
At 1.6 MPa:
D 28.an electric heater is to heat 11 kg of oil per S2 = 7.2374 kJ/kg-K
min, from 4.5C to 65.5C. Specific heat of oil U2 = 2950.1 kJ/kg
is 2.1 J/gm-C. How many watts should this T2 = 400C
heater consume? a. -500 kJ/kg c. -632 kJ/kg
a. 20 448 W c. 20 400 W b. -432 kJ/kg d. -700 kJ/kg
b. 33 448 W d. 23 442 W ( )

A 35.During polytrophic process of ideal gas


C 29.find the heat transfer reqd to convert 5 kg of
state changes from 138 kPa and 5C to 827
water at pressure if 20 bar and temp of 21C
kPa and 171C. Find value of n.
into steam dryness fraction of 0.90 at same
a. 1.354 c. 1.345
pressure.
b. 1.253 d. 1.234
Properties of steam:
P = 20 bar ( ) ( )
hf = 909 kJ/kg
hfg = 1899 kJ/kg
Note: for water at 21C: C 36.calculate specific vol of air-vapor mixture
h = 88 kJ/kg in cu. m per kg of dry air when the ff.
a. 10 160 kJ c. 12 610 kJ conditions are given:
b. 11 610 kJ d. 15 610 kJ t = 30C
{[ ( )] } w = 0.015kg/kg
P = 90kPa
a. 2.12 m3/kg c. 0.99 m3/kg
3
b. 0.622 m /kg d. 1.515 m3/kg
A 37.copra enters dryer containing 40% H2O & C 44.a refrigerator having 30kW capacity
60% solid, and leaves w/ 5% H2O & 95% requires 10HP compressor. Find COP of
solid. Find wt. of H2O removed per lb. of system.
final prod. a. 2.78 c. 4.02
a. 0.583 lb/lb c. 0.475 lb/lb b. 3.78 d. 5.02
b. 1.3753 lb/lb d. 1.015 lb/lb
A 45.A VCRS is designed to have a capacity of
A 38.an air conditioning plant w/ capacity of 100 TOR. It produces chilled water from
400kW has an evaporating & condensing 22C to 2C. Its actual COP is 5.86 and 35%
temp of 3C and 37C respectively. If it uses of power supplied to compressor is lost in
R-12, what is the volume flow rate under form of friction and cylinder cooling losses.
suction conditions? Determine the size of electric motor reqd to
At 3C drive compressor in KW and vol flow rate of
hg = 352.75 kJ/kg chilled water in L/s.
v = 50.47 L/kg a. 92.31 kW, 4.199 L/s
hf = 202.78 kJ/kg b. 90.71 kW, 5.277 L/s
`Enthalpy leaving cond = 235.3503kJ/kg c. 93.75 kW, 5.724 L/s
a. 0.172 m3/s c. 3.411 m3/s d. 91.75 kW, 7.575 L/s
b. 0.281 m3/s d. 3.5 m3/s
A 46.a single acting twin cylinder ammonia
D 39.fish weighing 11,000 kg w/ temp of 20C is compressor w/ bore equal to stroke is driven
brought to cold storage & w/c shall be by engine at 250 rpm. The machine is
cooled to -10 C in 11hrs. Find the reqd plant installed in chilling plant to produce 700 kW
capacity in tons if Cp of fish above and of refrigeration at -18C evaporator temp. At
below freezing is 0.7 kCal/kg-C and 0.3 this temp, cooling effect per kg mass = 1160
kCal/kg-C respectively, w/ freezing pt at - kJ. Specific vol of vapor entering
3C. compressor = 0.592 m3/kg. Assume 85%
`The latent heat of freezing = 55.5 kCal/kg volumetric efficiency, determine the bore in
a. 25.26 c. NOTA mm.
b. 15.26 d. 24.38 a. 400 mm c. 500 mm
b.300 mm d. 450 mm
C 40.an ice plant produces 20 tons of ice per day
at -15C from water at 25C. If losses are 12% A 47.A refrigeration compressor having 10 in.
of freezing and chilling load, calculate flywheel is driven by a 4-pole AC motor. If
refrigeration capacity of plant in TOR. diameter of motor is 4 in, determine speed
a. 28.13 c. 31.5 of compressor.
b. 28.13 d. 41.5 a. 70 rpm c. 600 rpm
b. 500 rpm d. 800 rpm
A 41.calculate tons of refrigeration reqd to freeze
in 4 hrs. 3mm of 10C water on a skating risk A 48.calculate piston displacement of 2-cylinder
61m x 30m if the ground temp is 10C and compressor rotating at 1450 rpm if diameter
the air temp is 10C and the air temp above of cylinder is 2.5 in. & length of stroke is 2
rink is 16C. in.
Heat transfer coefficient a. 16.48 ft3/min c. 14.48 ft3/min
air to 0C ice = 6.8 W/m2-C b. 15.48 ft3/min d. 17.48 ft3/min
grnd to -6C = 0.3402 W/m2-C
a. 101.69 c.140.62 A 49.a duct 0.40 m high and 0.80 m wide
b. 151.68 d. 201.96 suspended from the ceiling in a corridor
makes a right angle turn in the horizontal
D 42.The power requirement of Carnot refrigerator plane. The inner radius is 0.2m and outer
in maintaining a low temp region at 300K is radius is 1.0 m measured from the same
1.5 kW per ton. Find heat rejected. center. The velocity of air in duct is 10 m/s.
a. 4.02 kW c. 6.02 kW Compute the pressure drop in this elbow.
b. 7.02 kW d. 5.02 kW Assume:
f = 0.3
D 43.4000 L/hr of distillates are to be cooled from density = 1.204 kg/m3
21C to -12C and 12% of wax by wt. is L = 10m
separated out at 15C. The cp heat of oil is 2 a. 341 Pa c. 143 Pa
kJ/kg-C and SG is 0.87. The specific heat of b. 441 Pa d. 144 Pa
wax is 2.5 and latent heat is 290 kJ/kg.
Allow 10% for losses, find the refrigerating C 50.determine the sensible heat of 5 lb of air
capacity of system. having dry bulb temp of 70F and humidity
a. 20 TOR c. 40 TOR ratio of 0.0092 lb/lb, the latter corresponding
b. 51.08 TOR d. 31.08 TOR to dew pt temp of 55F.
a. 94 BTU c. 84 BTU
b. 48 BTU d. 49 BTU
C 51.a 0.70 m vane axial fan is running at 2000 A 55.a mixture of dry air and water vapor is at
rpm delivers 7.5 m3/s against 0.08m static temp of 21C under pressure of 101kPa. The
pressure thereby consuming 12 kW. If fan dew pt temp is 15C. Calculate the relative
wheel diameter is increased from 0.70 m to humidity.
0.76 m, so the diameter ration is 1.10:1, a. 68.56% c.56.68%
what is the new static pressure? b. 65.68% d. 58.66%
a. 1.01m c. 0.097m
b. 0.314m d.31.01m A 56.a dryer is to deliver 1000 kg/hr of palay
with a final moisture content of 10%. The
B 52. Centrifugal pump has const speed of 1500 initial moisture content in the feed is 15% at
rpm and has head of 75m. What is the effect atm cond w/ 32C dry-bulb & 21C wet-bulb.
on the head of the pump if the impeller The dryer is maintained at 45C while RH of
diameter is reduced from 280mm to 230 hot humid air from dryer is 80%. If steam
mm? pressure supplied to heater is 2 MPa,
a. 25.6 mm c. 65.6 mm determine air supplied to dryer in m3/hr.
b. 50.6 mm d. 72.6 mm a. 1332.25 m3/hr c. 1223.25 m3/hr
3
b. 1233.25 m /hr d. 1523.13 m3/hr
A 53.in an air conditioning unit, 3.5 m3/s of air at
27C dry-bulb temp, 50%RH & std. atm A 57.An assembly hall was to have an air
pressure enters unit. The leaving condition conditioning unit installed w/c would be
of air is 13C dry-bulb temp and 90% RH. maintained at 26C dry-bulb & at 50%RH.
Using properties from psychro chart, The unit delivers air at 15C dry-bulb temp &
calculate refrigerating capacity in kW. the calculated sensible heat load is 150kW
a. 87.57 kW c. 57.87 kW & latent heat is 51.3kW. 20% by wt of
b. 77.57 kW d. 58.77 kW extracted air is made up of outside air at 34C
dry-bulb & 60% RH, while 80% is extracted
D 54.a stream of outdoor air is mixed with steam by air conditioner from assembly hall.
of return air in an air conditioning system Determine the air conditioner’s refrigerating
that operates at 101kPa pressure. The flow capacity in TOR & its ventilation load in
rate of outdoor system air is 2 kg/s and its kW.
condition is 35C dry-bulb temp & 25C wet- a. 83.22 TOR, 37.47 kW
bulb temp. he flow rate of return is 3 kg/s & b. 76.43 TOR, 57.34 kW
its cond is = 24C & 50%RH. Determine c. 89.56 TOR, 45.77 kW
enthalpy of mixture. d. 56.78 TOR, 47.68 kW
a. 91.56 kJ/kg c.69.15 kJ/kg
b. 51.69 kJ/kg d. 59.16 kJ/kg
71.81% 58. Sol’n:

( ) ( )

( )( ) ( ) ( )
( )

( )( )

59.13% 59. Sol’n:

HEAT BALANCE:
( ) ( )
( )( )( ) ( )

5.6 kW 60. Sol’n:


( )( )( )
[( ) ] [( ) ]

FOR TWO STAGE:


√ √( )( )

( )( )( )
[( ) ] [( ) ]
IPE Exit Exam Reviewer

1. What tonnage of refrigerating machine is required if the refrigerating system extracted


48,000 btu per hour?
Answer: 4 tons

2. An insulator with superior tensile strength but low heat resistance


Answer: Fiberglass

3. What is the combination of a wet and dry bulb thermometer is called a


Answer: Hygrometer or psychrometer

4. The weight of steel bar reinforcement show be how many time the weight of the foundation?
Answer: ½% to 1%

5. Is a high temperature process that turns incinerator ash into a safe, glass – like material
Answer: Vitrification

6. Oil is added to a freon compressor by


Answer: Pumping in with a hand pump

7. The difference between the theoretical draft and the draft loss
Answer: Available draft

8. The purpose of expansion valve by-pass is to


Answer: Controls the refrigerant to the evaporator in case the automatic valves fail

9. Load curve refers to the plot of:


Answer: Load versus time

10. What does a negative Joule-Thompson coefficient means during a throttling process?
Answer: fluid temperature rises

11. The maximum unit pressure of turbine and generator on the reinforced concrete should not
exceed
Answer: 17.62 kg/cm2

12. Water in vapor form remains a vapor as long as temperature is what relations to the dew
point temperature
Answer: below
13. The “refrigerating effect” of a refrigerant is a always:
Answer: Less than its latent heat

14. A burner should always start up in ___ fire and shut down in ___ fire
Answer: Low; low

15. A ___ boiler has water in the tubes and heat gases of combustion passing through the tubes
Answer: Water tube

16. If the compressor had been running satisfactorily for a long period of time but suddenly the
compartment temperature started to rise, the trouble might be:
Answer: Any of the other choices
(A refrigerant leak has developed, The solenoid valve has jammed shut, The expansion valve
may contain frozen water)

17. A type of polymer used for Styrofoam cups and clam shell food containers
Answer: Polystyrene (PS)

18. Mycotoxins are poisonous chemicals produced by:


Answer: Molds

19. The magnitude of the drag coefficient of a sphere in water is dependent upon all of the ff
except
Answer: Units of measure (SI or English engineering system)

20. A refrigeration system in which only part of the refrigerant passes over the heat transfer
surface is evaporated and the balance is separated from the vapor and recirculated
Answer: Flooded system

21. The boiling point of freon-12 at atmospheric pressure is


Answer: -22F

22. Which of the following vital components of the refrigeration system where both temperature
and pressure are increased?
Answer: Compressor

23. Which of the following must be done to eliminate frost on the discharge pipe of the
compressor?
Answer: None of the other choices
(Open the expansion valve, crack bypass valve, Regulates water to the condenser)

24. Orifice coefficients are used to determine:


Answer: Energy losses
25. Which of the following is used as high head turbine?
Answer: Impulse

26. A water-tube condenser has a total of 60 tubes. If these two passes, then compute the number
of tubes per pass
Answer: 15

27. Absolute zero is


Answer: 460 degrees below zero on the Fahrenheit scale

28. Which of the following can be the cause of low head pressure
Answer: Any of the other choices
(Leaky discharge valves, Too much or too cold condensing water, insufficient charge of
refrigerant)

29. What is the most common freon gas used in centrifugal compressors?
Answer: F-11

30. When purging an ammonia condenser into a bucket of water, one can tell when the air is out
and ammonia starts to come through the:
Answer: Change of bubbling sound of air to the cracking sound of ammonia

31. In the deep well installation or operation, the difference between static water level and
operating water level is known as:
Answer: Drawdown

32. White ice is


Answer: Due to dissolved air, gasses and impurities

33. Which of the following is a characteristic of an impulse turbine


Answer: Steam striking blades at zero angle

34. Heavy water is:


Answer: D2O

35. When excess solute in a solution settles to the bottom of the container, the process is called:
Answer: Precipitation

36. Calcium chloride is sometimes used in refrigeration system as a:


Answer: Secondary Coolant

37. If a compressor runs continuously, the cause might be a:


Answer: Stuck low-pressure switch
38. The thermal expansion valve is located between the:
Answer: Solenoid valve and the evaporator coils

39. The cooling water regulator is automatically actuated by which of the following?
Answer: The discharge pressure of the refrigerant

40. Based on the PSME code, what should be provided in each steam outlets if two or more
boilers will be connected in parallel?
Answer: Non-return and shut off valves

41. The amount of sensible heat for a sensible heat ratio of 0.8 and a total cooling load of 100 is
Answer: 80

42. Which refrigerant is used for the air conditioning of passenger aircraft cabin
Answer: Air

43. When a falling object reaches a speed at which the drag force equals its weight, it has
achieved:
Answer: A turbulent boundary layer

44. The minimum vertical distance from the floor or soil level to the top edge of the foundation
must be around
Answer: 120 mm

45. How many moisture be removed from air?


Answer: condensation and absorption

46. The charging valve is located between the:


Answer: King valve and the expansion valve

47. A “cold diffuser” is a:


Answer: Type of evaporator

48. Which of the following stops the compressor before the relief valve opens?
Answer: High pressure control

49. What is the excess refrigerant removed?


Answer: Charging side of the system

50. Which of the following is used in measuring the density of a brine solution?
Answer: A hydrometer
51. All of the following are common types of chemical reaction except
Answer: Fission

52. A ___ after each radiator allows ___ to pass through to the return line
Answer: Steam trap; condensate

53. A leaky solution valve can usually detected by:


Answer: Any of the other choices
(A fluctuating suction pressure gauge, a higher suction pressure, closing in on the suction
valve having no effect on the suction pressure)

54. If the cooling water to the condenser suddenly fails:


Answer: The compressor will shutdown

55. If Et is the total net energy generated by a plant in a certain period of time and Er is the rated
net energy capacity of the plant during the same period of time, then what is the plant
operating factor?
Answer: Et/Er

56. A thermometer sense which of the following?


Answer: Sensible heat

57. Is the subjective method in which the smoke density is visually compared to five
standardized white-black grids
Answer: Ringelman scale

58. Excessive head pressure is caused by


Answer: Flooded condenser tubes or cooling water temperature too high

59. What do you call a plate or vane used to direct or control movement of fluid or air within the
confined area?
Answer: Baffle

60. The total pressure supplied by the fan at maximum operating conditions
Answer: Net rating or fan boost

61. The system should be purged:


Answer: After the system has been shut down for few hours

62. What is likely to occur when sections of the impeller of a centrifugal pump are handling
vapor and other sections are handling liquid.
Answer: complete failure to operate
63. What is the chemical formula of Ozone?
Answer: O3

64. An excessively high head pressure would be caused by:


Answer: Insufficient cooling water to the condenser

65. When an electric motor nameplate indicate a “100-kW electric motor” then what does a 100-
kW rating refers to?
Answer: Mechanical power output

66. During a cooling and dehumidifying process, sensible and latent heats are removed in the
cooling coil. If Hs is the sensible heat and Ht is the total heat transferred, then determine the
coil sensible factor.
Answer: Hs/Ht

67. A single trunk piston-type compressor is undesirable for a Freon unit because the:
Answer: Lubricant mixes with the refrigerant

68. What is the first step that must be done when securing a system?
Answer: Close receiver (King) valve

69. The amount of CO can be determined by the color of Palladium Chloride. An amount of 30
ppm to 70 ppm will cause:
Answer: Slight Darkening

70. The water hammer phenomenon is primarily what kind of fluid mechanism?
Answer: Dynamic (a time-dependent phenomena)

71. The other name for Swamp Cooling is:


Answer: Wet Roof Cooling

72. The design of an air supply duct of an air condition system


Answer: affects the distribution

73. A Freon – 12 gage shows pressure and ____.


Answer: Saturation temperature

74. High superheat of the vapor in the system would cause


Answer: A decrease in capacity

75. When there is no work between the thermodynamic system and its surrounding, the quantity
of net heat transfer is equal to:
Answer: total energy of a closed system
76. The relation between the Fahrenheit absolute scale and the Celsius absolute scale is:
Answer: R = 1.8 Kelvin

77. External frost on inlet of expansion valve indicates:


Answer: Expansion valve plugged or dirty

78. The total cross section of a target atom is made up of


Answer: The absorption and scattering cross sections

79. The motor condition of an air conditioning unit can be checked with:
Answer: The continuity of light or with an ohmmeter

80. A number used to compare energy usage for different areas. It is calculated by dividing the
energy consumption by the footage of the conditioned area.
Answer: Energy utilization Index

81. What is the effect if the refrigerant is removed from the system too fast?
Answer: It may “freeze – up” the condenser

82. After the total force of the steam has lifted the safety valve off its seat, the steam enters the
____.
Answer: Huddling chamber

83. By which of the fooling processes heat mainly dissipates in cooling towers?
Answer: Evaporation

84. All of the following fluid phenomena are based on the force momentum principle of a
flowing fluid except:
Answer: Diesel automobile engines

85. All of the following are forms of drag on a body moving through a fluid except
Answer: D’alembert’s paradox drag

86. What is the use of the suction pressure regulating valve?


Answer: Maintains the back pressure in the evaporator coils

87. There shall be no foundation bolts less than _____.


Answer: 12 mm in diameter

88. The ratio of the rated cooling capacity divided by the amount of electrical power used:
Answer: Energy efficiency ratio (EER)
89. The scale trap is located between the:
Answer: King (liquid) valve and expansion valve

90. The temperature in the vegetable box should be approximately


Answer: 35 to 45 deg. F

91. A thermostat is a
Answer: Temperature-operated switch

92. The bulb for the thermal expansion valve is located:


Answer: Near the evaporator coil outlet

93. The coefficient of velocity is the ratio of the:


Answer: Actual velocity to the theoretical velocity

94. What is the specific humidity of dry air?


Answer: 0

95. What is the reason why a thermometer in vapor compression system is installed close to the
compressor?
Answer: Because it helps the operator in adjusting compressor for greatest efficiency

96. As the steam temperature increases the steam temperature:


Answer: Increases

97. Substances that have the ability to absorb moisture from the air are called:
Answer: Desiccants

98. Evaporative condenser is used to cool


Answer: condenser vapor

99. If the head pressure is too high:


Answer: The high pressure cutout switch should operate before the relief valve opens

100. A material or substance that is accidentally or intentionally introduce to the environment


in a quantity that exceeds what occurs naturally
Answer: Pollutant

101. The greatest decrease in refrigerant temperature occurs in the ____


Answer: Evaporator

102. The diesel cycle is the ideal cycle for a :


Answer: compression-ignition engine
103. The moisture in a refrigeration system can be removed with help of which of the
following?
Answer: Driers

104. What process is employed when the turbine steam power plants experience excessive
moisture?
Answer: reheating

105. The path that directs the flow of refrigerant through the compressor
Answer: Valve

106. Which of the following is another name for the liquid valve?
Answer: King valve

107. Expansion factors take into account the:


Answer: Effects of compressibility

108. The term used to express the amount moisture in a given sample of air. It is compared
with the amount of moisture in a given sample of air.
Answer: Relative Humidity

109. What are the four basic methods of determining whether the proper amount of refrigerant
being added to the system?
Answer: Bull’s eye, weight, pressure and frost line

110. Which of the following components of the window air conditioning system must be
cleaned annually?
Answer: All of these
(Evaporator and Condenser, Motor and Compressor, Fan Blades and Fan Motor)

111. At present, the number of true elementary particles, which include leptons and quarks is
thought to be
Answer: 12

112. All of the following statements about conjugate acids and bases are true except
Answer: A conjugate base results when a base accepts a proton

113. A double-seated valve allows the valve to be


Answer: Packed in the wide open or closed position

114. In order to keep combustion products inside the combustion chamber and stack system,
balanced draft system may actually operate with a slight
Answer: negative pressure
115. What is the cause of pressure drop in the boiler, condenser and the pipings between
different components? Because of this drop, steam leaving the boiler at a lower pressure.
Answer: Fluid friction

116. It is a form of oxygen photochemically produced in nature.


Answer: Ozone

117. Absolute zero on the Fahrenheit scale is equal to


Answer: -460

118. Low suction pressure is caused by:


Answer: Solenoid valve not functioning properly

119. What do you call the storage tank for liquid refrigerant?
Answer: Receiver tank

120. Why should you avoid bending or twisting of fan blades in an air conditioning unit?
Answer: It will wear out the motor bearings and cause noise

121. All of the following statements are characteristics of bases except


Answer: They have a pH between 0 and 7

122. Concrete foundation should have steel bar reinforcement placed vertically and
horizontally to avoid ____.
Answer: Thermal cracking

123. The pump will not cavitate if the available Net Positive Suction Head (NPSH) is:
Answer: equal or greater than the required NPHS

124. What will happen to the capacity if the superheat is increased on the suction side?
Answer: Decreases

125. When heavy electrical currents are involved, the thermostat will be operated by a
Answer: Relay

126. Valves in piping in an ammonia system are made of


Answer: Iron

127. Which of the following is not essential to a compression refrigeration system?


Answer: A receiver

128. Nitrogen occupied almost how much of the Earth’s atmosphere?


Answer: 3/4
129. If frost forms on the cylinders, the cause would be:
Answer: Expansion valve open too wide

130. Which of the following gasket materials should be on Freon system?


Answer: Asbestos or Metallic

131. Which of the following would cause the crankcase and head to get hot with low suction
pressure?
Answer: Insufficient refrigeration

132. Moisture in a system will cause a:


Answer: Faulty expansion valve

133. The steam turbine foundation should be designed to support the machine load plus how
many percent for impact, condenser load, floor loads and dead loads?
Answer: 25

134. When air is heated, what happened to its relative humidity?


Answer: Remain constant

135. Sweating of crankcase is caused by which of the following?


Answer: Too much oil in the system

136. What do you call a material in a dryer?


Answer: Desiccant

137. The matching model and full-scale prototype results a fluid dynamic phenomena
involving a fully submerged body requires equality of:
Answer: Reynolds number

138. Which of the following uses a secondary refrigerant?


Answer: Ice plant

139. When the outlet temperature at the evaporator exceeds the inlet temperature, the
condition is known as:
Answer: Superheating

140. In series pipe systems, all of the following parameters vary from section to section except
Answer: Mass flow

141. Which of the following is used to control refrigeration compressor capacity?


Answer: Unloader
142. TOR is a unit equivalent to:
Answer: 12.66 kN-m/hr

143. The agent used in an indirect reefer system is:


Answer: Calcium chloride or sodium chloride

144. If the W is the work output of a heat engine and H is the total heat input of an engine,
then determine its thermal efficiency, e.
Answer: e = W/H

145. Which of the following items is not important when using a halide torch?
Answer: Adjust to a clear white flame

146. A dehumidifier is usually a small hermitic refrigerating system. It has both a condenser
and an evaporator. Many older systems use R – 12 or R – 500. The newer units are:
Answer: R – 134a

147. What is the method used to evaluate all welds performed on pressure parts of boiler tube
materials?
Answer: Radiographic test

148. Faulty F – 12 compressor valves will be indicated by:


Answer: All of the other choices
(Gradual or sudden decrease in capacity, compressor running continuously, Low head
pressure – high suction pressure)

149. Heat exchanger in which water flows by gravity over the outside of tubes or plates.
Answer: Baudelot cooler

150. Which of the following is not a sound absorptive material?


Answer: Mirrors

151. The desirable temperature inside an air conditioned auditorium is:


Answer: 20 C

152. An odorless refrigerant whose boiling point varies over a wide range of temperatures.
Answer: Freon 12

153. The cooling water regulator is actuated by


Answer: Pressure of the refrigerant

154. The psychrometric chart in air conditioning determines the


Answer: Moist air conditions
155. Blowback of a safety valve is to prevent:
Answer: Chattering

156. When starting a refrigerant unit, be sure the water side of the condenser is ____.
Answer: Vented

157. A temperature measurement in an ordinary temperature which has constant specific


humidity.
Answer: Dew point temperature

158. What adding oil to a freon system, one must be sure that
Answer: All air is removed from the pump and fitting

159. The primary purpose of a turbine in a fluid is to:


Answer: Extract energy from the flow

160. Which of the following is to be checked as regular monthly maintenance schedule of a


console air conditioning units?
Answer: Duct dampers, registers and diffusers

161. A company is interested to produce a water turbine wherein only little energy is required
or necessary because the guide vanes are to be controlled. The turbine must be a:
Answer: Kaplan turbine

162. To check the lack of refrigerant or clogged refrigerant lines:


Answer: Installing service valves must be necessary

163. Condensate from the non return steam trap is pumped from the condensate return tank of
the:
Answer: Boiler

164. The locus of elevation is:


Answer: Hydraulic Gradient

165. Which of the following cannot be used to describe neutron kinetic energy levels
Answer: Freezing

166. It is important to take some moisture from the air dehumidify it if the relative humidity
reaches high levels. To do this, it requires cooling the air:
Answer: Below its dew point temperature

167. The vapor cycle in thermal efficiency as the refrigeration cycle to the
Answer: COP for a refrigerator
168. A scale trap in a Freon system will be found on the
Answer: Suction side

169. Which of the following is not used as method to measure air velocities?
Answer: Open type Barometer

170. Which of the following is considered as comfort condition in air conditioning?


Answer: 20 C DBT, 60% RH

171. Which of the following acts as ignition accelerator for internal combustion engine fuels?
Answer: Acetone peroxide

172. The purpose of the evaporator is to


Answer: Absorb latent heat of vaporization

173. The speed at which a reaction proceeds to equilibrium is the purview of


Answer: Reaction kinetics

174. What do you call the liquid reaching the compressor through the suction?
Answer: Flooding back

175. Which of the following draft rely on the stack effect to draw off combustion gases?
Answer: Natural draft

176. The relative humidity becomes 100% and where the water vapor starts to condense.
Answer: Dew point

177. Which of the following statements is correct? The relative humidity of an air water vapor
mixture.
Answer: is the ratio of partial pressure of water vapor to the saturation pressure at the
mixture temperature

178. The following are standard characteristics of Freon – 11 except:


Answer: Boiling point over 200 F

179. The bhp/ton refrigeration requirement with increase in condenser temperature. In a vapor
compression system that uses reciprocating compressor receiving refrigerant gas at constant
suction temperature, will follow:
Answer: Linearly increasing characteristic
180. In an air conditioning unit, the thermostat fails. The unit did not start. How do you test
the operation of the thermostat?
Answer: Cover the air outlet and air inlet with a cloth. The air will recirculate into the
unit and the temperature will quickly drop to the cut out temperature.

181. All of the following temperature have meaning in psychrometrics except:


Answer: Adiabatic wall temperature

182. The maximum continuous power available from a hydro – electric plant under the most
adverse hydraulic conditions is called
Answer: Firm power

183. Which of the following dehumidifier is often used to reheat the air after moisture is
removed?
Answer: Condenser

184. Which of the following is a type of evaporator?


Answer: shell-and-tube

185. All of the following factors affect rates of reaction except


Answer: Pressure

186. Which of the following would cause a high suction pressure?


Answer: Expansion valve open too wide

187. Zinc rods are found in the:


Answer: Salt water side of the condenser

188. The sum of the internal energy and the product of pressure and specific volume is known
as the:
Answer: enthalpy

189. Any foreign matter in the coal feed mechanism of the screw-feed stoker is best removed
by:
Answer: Using the cutoff gate at the bottom of the hopper

190. Based on the PSME code, what should be the humidity of air to be used for comfort
cooling?
Answer: 50% - 60% relative humidity
191. Which of the following must be checked up if an automatic Freon system will not start
up?
Answer: All of the other choices
(High pressure cutout, reset mechanism, Low-pressure cutout)

192. Refers to the use of composting soil beds


Answer: Biofiltration

193. What is the usual cop of a domestic refrigerator?


Answer: More than 1

194. In a hydro-electric plant using a Francis turbine with medium head, the speed can be
regulated using the
Answer: Wicket gate

195. The temperature bulb of the thermo-expansion valve is attached to which of the
following?
Answer: Evaporator coil outlet

196. Which of the following refrigerants would give the most trouble when operating with
warm circulating water?
Answer: CO2

197. A rule which states that any waste derived from the treatment of a hazardous waste
remains a hazardous waste
Answer: The derived from rule

198. A device for holding open the suction valve and drawing gas from the suction manifold
and returning it to the suction line without compressing it is called
Answer: Cylinder unloader

199. The intake pipe to a hydraulic turbine from a dam


Answer: Penstock

200. What consists of weak solutions of sulfuric, hydrochloric, and to a lesser extent, nitric
acids?
Answer: Acid rain

201. The expansion valve does not seem to be operating properly. There is high superheat.
Test by listening to the sound of Freon flooding through the tubes and ____.
Answer: Warm bulb with hand
202. Swimming pool water needs
Answer: de-chlorination

203. Oxygen is approximately what percent by weight in the atmosphere?


Answer: 23%

204. Which of the following could be used to check a leak in an ammonia system?
Answer: Litmus paper & Sulfur stick

205. The determination of properties and behavior of atmospheric air usually the purview of:
Answer: Psychrometrics

206. A valve the released steam from the boiler by operating at a pre-determined pressure in
order to keep the steam pressure from rising more than the pressure from which the valve is
set is called a
Answer: Safety valve

207. If the critical temperature of a refrigerant is too close to the desired condensing
temperature, the equipment must necessarily be of
Answer: Extra heavy construction

208. If a freon – 12 compressor trip out on “cut-out”, the solenoid valve closes by which of the
following?
Answer: An electrical release

209. Which of the following would cause the suction switch to remain open?
Answer: Bellows broken or jammed

210. In the upper atmosphere, ozone is made by ultraviolet light reacting with:
Answer: Oxygen

211. How is ammonia system purged so that operator will not be overcome by the fumes?
Answer: Into the bucket of water

212. A ton of refrigeration is equal to the removal of


Answer: 288,000 btu per 24 hrs.

213. As a good practical rule, the foundation depth may be taken as how many times of the
engine stroke?
Answer: 2:5 to 3:5

214. Most stacks are built of height less than _____.


Answer: 30 m
215. The phenomenon that warm air rise and cold air settle is called:
Answer: Stratification

216. Type of refrigerant control which maintains pressure difference between high side and
low side pressure in refrigerating mechanism:
Answer: Expansion valve

217. The crossover connection in an ammonia system can be used to ____.


Answer: Hot-gas defrost

218. Which of the following refrigerants has lowest freezing point temperature?
Answer: Freon 22

219. The operation that produces highest noise level is:


Answer: Riveting

220. Select the one in which secondary refrigerant is used


Answer: Ice plant

221. Which of the following is not essential to a centrifugal type compressor system?
Answer: Distiller

222. Two compressor should not be run in parallel because


Answer: There is a possibility of losing oil

223. Which of the following is also known as refrigerant no. R-717?


Answer: Ammonia

224. During sensible heating, the humidity remains constant but the relative humidity.
Answer: decreases

225. The term suction units is used with


Answer: Induced draft

226. If the compressor had been running satisfactorily for a long period of time but the oil
level was rising slowly, one should:
Answer: Shut down the compressor and check the oil level with the machine stopped

227. What amount of air is required in a low bypass factor?


Answer: Lesser

228. The reduction of nuclear radiation intensity (called attenuation) is accomplished by:
Answer: Shielding
229. The ozone concentration of 0.10 parts per million (ppm) is generally considered the
maximum permissible for how many hours exposure?
Answer: 8 Hours

230. Which do you think is very important in adjusting compressor “v” belts?
Answer: Allow ½” slack

231. An engineer inspected an air-conditioning unit. He found out that the unit does not
produce any cooling effect, however, the air-conditioning unit is running. He checked the
temperature of the condenser and evaporator and hat the unit run. He found out that there was
no change in temperature. What should he do?
Answer: Charge with new refrigerant

232. The temperature at which water vapor in the atmosphere begins to condense is known as:
Answer: dew point

233. The ice making capacity is always


Answer: Directly proportional to the refrigerating effect

234. If the compressor short-cycles on the high-pressure cutout, which of the following would
you check?
Answer: All of the other choices
(If plenty of cooling water is running through but it is not picking up heat, the condenser
tubes need cleaning, Be sure system is getting cooling water, Check for too much refrigerant
in the system)

235. What is the instrument used to register relative humidity?


Answer: Hygrometer

236. What has an octane rating of more than 100?


Answer: Benzol

237. A device used to keep moisture from passing through the system is called:
Answer: Dehydrator

238. If the freezing point of water is 0 C, which of the following is its melting point following
is its melting point
Answer: 0 C

239. What is the pressure present inside the casing of an impulse turbine?
Answer: Atmospheric pressure
240. A vena contracta in a fluid jet issuing through a hole in a plate is located approximately:
Answer: At jet’s minimum diameter

241. If the compressor short-cycles on the low-pressure cutout, the trouble might be:
Answer: Any of the other choices
(Too much frost on the evaporator coils, dirty traps and strainers, lack of refrigerant)

242. The amount of heat necessary to bring up temperature of a unit mass of a substance
through unit degree is called
Answer: Total heat

243. If the expansion valve capillary tube is pinched, which of the following be replaced?
Answer: All of the other choices (called the power element unit)
(Bulb, tube, diaphragm)

244. The coefficient of discharge is the ratio of the.


Answer: Actual discharge to the theoretical discharge

245. Pitot tube is used to measure the:


Answer: Velocity of flow

246. The principle of the centrifugal system is based on which of the following?
Answer: Kinetic energy

247. The disadvantage of a CO2 system over an ammonia system is the fact that
Answer: All of the other choices
(The pipes and fittings of a co2 system must be of the high-pressure type, The CO2 system
requires a larger prime remover, The CO2 system operates at a much higher pressure)

248. If ice will form in a solution of water and salt, then it is at temperature called:
Answer: Freezing point depression

249. The suction pressure in a Freon system should be:


Answer: The pressure which corresponds with a temperature about 20F below the
temperature of the icebox.

250. Wb is shaft work of an engine and Wi is indicated work of an engine. If mechanical is


present in the engine mechanism, then.
Answer: Wb is less than Wi

251. What is the relationship of the capacity of a centrifugal pump, Q, to impeller diameter, D,
when there are two impeller diameters in the same pump?
Answer: Q is directly proportional to the ratio of D
252. The combination of enthalpy and kinetic energy of fluid is termed as:
Answer: Stagnation enthalpy

253. Before any repair work is attempted on any gas-fired boiler the:
Answer: Main gas cock must be secured

254. An air-conditioning system in which water is chilled or cooled and which passes the
evaporator coils
Answer: Chilled water system

255. Refers to organic waste produced from biological water waste treatment processes
Answer: Biosolids

256. As far as combustion chamber design is considered, the maximum power output of a
given engine can be increased by:
Answer: Decreasing combustion chamber volume

257. A radioactive gas produce from the decay of radium within the rocks beneath a building
Answer: Radon

258. Ozone filters out what type of radiation that damages crops and causes skin cancer?
Answer: Ultraviolet radiation

259. The ratio of the sum of individual maximum demands of the system to the overall
maximum demand of the whole system
Answer: Diversity factor

260. A bell coleman cycle is also known as


Answer: Reversed Joule cycle

261. It is used deliver concentrated airstreams into a room. Many have one-way or two-way
adjustable air stream deflectors.
Answer: Register

262. The sensible heat ratio is 0.8. that is


Answer: 80% sensible heat and 20% latent heat

263. Closing the solenoid valve will stop the compressor through the ____.
Answer: Low-pressure cutout switch

264. A refrigerating unit of one (1) ton capacity can remove?


Answer: 200 btu’s per min
265. Which of the following is the usual case of slugging?
Answer: Expansion valve not operating properly

266. All of the following occur during reduction of a substance except


Answer: Loss of electrons

267. The secondary refrigerant used in milk chilling plants is generally


Answer: Glycol

268. Where is the solenoid coil installed?


Answer: Vertically over the valve

269. The thermal expansion valve responds to the


Answer: Amount of superheat in the vapor

270. To eliminate transmission of vibration, the foundation should be isolated from the floor
slabs of building footings at least how may mm around its perimeter?
Answer: 25

271. Which one is commonly used liquid absorbent?


Answer: Ethylene glycol

272. The low-pressure control switch:


Answer: Cuts out the compressor to maintain proper flow

273. If the outlet of the thermostatic valve is warmer than the inlet, it indicates
Answer: Thermostatic valve not working properly

274. Compute the wall gain load of a load storage room when A is the outside surface area of
the wall, D is the temperature differential across the wall, and U is the overall coefficient of
heat transmission.
Answer: AUD

275. A leak on the fuel oil suction line between the tank and the suction side of the fuel oil
pump would result in:
Answer: Air entering the suction line

276. How is the CO2 system is purged?


Answer: When CO2 comes out of the purge valve, frost will form on a piece of metal
held near the outlet

277. The relief valve on a CO2 machine is located:


Answer: On the discharge pipe between the compressor and discharge valve
278. The high-pressure side of the system is sometimes referred to as the:
Answer: Hot side

279. Combined pressure of cooling and humidifying is also known as:


Answer: Evaporative cooling process

280. The schedule of a pipe, N, indicates the thickness of the pipe wall. If the allowable stress
of the pipe is S, then what is the internal pressure equal to?
Answer: N x S

281. Which of the following area of work requires lowest noise level?
Answer: Library

282. Which of the following contribute to the deterioration of the earth’s ozone layer?
Answer: Chlorofluorocarbons

283. What is the least number of compressors a multistage system that will use?
Answer: Two

284. Is a soluble compound that reduces a liquid’s surface tension or reduces the interfacial
tension between a liquid and a solid
Answer: Surface – acting agent or surfactant

285. To secure the belts embedded within the foundation, the distance of the edges of the
foundation from the bedplate must be ____.
Answer: 120 mm to 300 mm

286. In Stirling and Ericsson Cycles, the efficiency can be increased by:
Answer: Regeneration

287. The thermal expansion valve


Answer: Controls the quantity of liquid refrigerant going to the evaporator coils

288. Too low suction pressure could be caused by:


Answer: Any of the other choices
(Dirty scale traps, Shortage of refrigerant gas, Too much oil in the system)

289. The static pressure drop due to friction through the boiler and stack.
Answer: Draft loss

290. In a freon-11 system there is no


Answer: Distiller
291. The coefficient of contraction is the ratio of the:
Answer: Area of vena contracta to the orifice area

292. Which of the following should not be used to clean scale traps:
Answer: Cotton waste

293. Most observe properties of light and other radiant energy are consistent with waves in
nature, but in interaction with matter, electromagnetic energy behaves as though it consists of
discrete pieces or
Answer: Quanta

294. Boilers using soft coal must have ____ furnace volume to reduce the danger of ____
Answer: Large; smoking

295. Which of the following would you apply if a person got Freon in his eyes?
Answer: Sterile mineral oil

296. The compressor will run continuously if there is:


Answer: Any of the other choices
(Too heavy a load, Insufficient refrigerant, Air in the system)

297. A hot crankcase and cylinder head accompanied by a low suction pressure would be
caused by
Answer: Insufficient refrigerant

298. One foot water is equal to:


Answer: 62.43 lb/in2

299. The purpose of the oil trap is:


Answer: To remove oil from the refrigerating gas

300. Many pressure gauges on a Freon system have two dials or graduations on one gauge.
The two dial represents:
Answer: Pressure and temperature

301. The faster way to remove frost from a direct expansion finned tube evaporator is to:
Answer: Send hot gas through the coil

302. When ordering an expansion valve which of the following information is necessary?
Answer: Size, tonnage, temperature, and pressure

303. The elements of a thermostat switch are usually of the ____.


Answer: Bimetal type
304. Which of the following characteristics that is not desirable in a refrigerant?
Answer: Low latent heat of vaporization

305. What do bodies at a temperature above absolute zero?


Answer: It emits Thermal Radiation

306. The Horizontal Scale (Abscissa) in the psychrometric Chart represents:


Answer: Dry bulb temperature

307. The thermostatic expansion valve is designed to maintain constant ____.


Answer: Superheat

308. The charging connection in a refrigerating system is located:


Answer: Between the king valve and the solenoid valve

309. The latent heat of fusion of ice is


Answer: 144 Btu

310. The ratio absorbed by the transfer fluid to the original incident energy striking the
collector
Answer: Collector efficiency

311. If m is the mass of dry air and H is the specific enthalpy of the water vapor in air and r is
the humidity ratio, then determine the latent heat of any dry air.
Answer: m(r x H)

312. The dehydrator is located between the:


Answer: Receiver and expansion valve

313. In a vapor compression cycle the lowest temperature is found in


Answer: Evaporator

314. The volume flow passes through a venturimeter is:


Answer: Constant

315. Anchor bolts in a machine foundation should be embedded in concrete of at least how
many time of the bolt diameter?
Answer: 30

316. Which do you think is the effect of “subcooling”?


Answer: It reduces the horsepower per ton of refrigeration
317. When air is saturated, the wet bulb depression is:
Answer: Zero

318. A precooler, if used is located between the:


Answer: Condenser and expansion valve

319. A Kaplan turbine is:


Answer: Low head axial flow turbine

320. The refrigerant temperature is at its maximum just before it enters the ____
Answer: Condenser

321. The machine foundation must have a factor of safety of:


Answer: 5

322. In a refrigerating system, the heat absorbed in the evaporator per kg mass of refrigerant
passing through
Answer: Equals the increase in enthalpy

323. Which of the following is the common classification of ducts?


Answer: All of these
(Conditioned Air – Duct, Recirculating, Fresh – air ducts)

324. All of the following process can be found in a psychrometric chart except:
Answer: Natural convection

325. If an electrically-operated compressor failed to start the cause might be:


Answer: Any of the other choices
(Burned out holding coils in solenoid valve, An open switch, A blown fuse)

326. Five pounds of water heated to raise the temperature 2F requires how many BTU?
Answer: 10 Btu

327. Air circulation in the icebox is accomplished by the use of which of the following?
Answer: Diffuser fans

328. In sensible heating cooling following parameter remains unchanged


Answer: Humidity ratio

329. Venturi meters, pitot static gauges, orifice meters, flow nozzles, and differential
manometers all depend upon the relationship between:
Answer: Flow velocity and pressure
330. It is used to deliver widespread, fan-shaped flows of air into the room
Answer: Diffuser

331. What turbidimeter that gives direct reading in ppm?


Answer: Jackson turbidimeter

332. How much will be removed by one-ton refrigeration unit?


Answer: 200 Btu per min

333. A double-trunk piston is used to:


Answer: All of other choices
(Prevent oil from mixing with the refrigerant, Prevent gas from getting to crankcase, Prevent
oil from mixing with the refrigerant)

334. Purpose of the receiver is to


Answer: Store the refrigerant

335. Latent heat


Answer: Cannot be measured with a thermometer

336. A type of water turbine where a jet of water is made to fall on the blades or buckets and
due to the impulse of water, the turbine starts to move.
Answer: Pelton wheel

337. When does the refrigerant gives-up heat?


Answer: When it condenses

338. A term used to mean the corrective steps taken to return the environment to its original
condition
Answer: Remediation

339. The back – pressure regulating valve:


Answer: Maintains a fixed pressure in the evaporator coils

340. What is the relationship of the horse power of a centrifugal pump, Hp, to the impeller
speed, S, if the pump is at two different rotative speeds?
Answer: Hp is directly proportional to the cube of the ratio of S

341. What must be the value of the available Net Positive Suction Head (NPHS) of a
centrifugal pump compared to its required NPHS to avoid losing priming?
Answer: Available NPHS greater than required NPHS
342. The diesel engine foundation safe soil bearing pressure is:
Answer: 4,890 kg/cm2

343. Aside from maintain appropriate temperature for food cold storage, how is desiccation
minimized or decreased?
Answer: Maintain humidity ratio

344. If C is the capacity of the compressor in a refrigeration system and F is the heat rejection
factor, then calculate the condenser load, L.
Answer: L = C x F

345. The following examples of indirect (secondary) measurements to measure flow rates
using obstruction meters except:
Answer: Weight and mass scales

346. When required, a regulator water valve in refrigerating system should be


Answer: In the water outlet

347. What is the relative humidity when the dew point and dry bulb temperature are equal?
Answer: 100%

348. Compare in refrigerating effect per unit mass of refrigerant circulated for a superheated
cycle that produces useful cooling and a saturated cycle, for the same vaporizing and
condensing temperature
Answer: Greater for a superheated cycle

349. If PV is the power required for a vapor compression refrigeration system, then what is the
power required for an air refrigeration system, assuming that they have the same capacity?
Answer: 5PV

350. What do you call the mixture if the solute particles of a solid suspended in a liquid are
larger than molecules?
Answer: Suspension

351. Lithium bromide is used in refrigeration system in:


Answer: Absorbers

352. The removal of dissolved gas or other volatile component from liquid by exposing the
liquid to air or steam is known as:
Answer: Stripping

353. Which of the following measures the density of salt in water?


Answer: Salimeter
354. In a power driven pump, each piston stroke is displaced by 360 divided the ____.
Answer: Number of cylinders

355. What is the boiling temperature of F 22?


Answer: -40 C

356. What is another name of discharge pressure?


Answer: Head pressure

357. If the compressor were to run out continuously without lowering the temperature, the
trouble would probably be:
Answer: Any of the other choices
(Insufficient refrigerant in the system, Leaky discharge valve, Leaks in the system)

358. The ratio of sum of individual maximum demands of the system to the maximum demand
of the whole system is known as
Answer: Diversity factor

359. The temperature bulb of solenoid valve is attached to the ____.


Answer: Wall of the icebox

360. What must be done to change the direction of rotation of a 440-volt, 3-phase induction
motor?
Answer: Interchange any two power leads

361. In order to remove the fly ashes from the flue gas, which of the following must a power
plant be equipped with?
Answer: Electrostatic precipitator

362. Which of the following is a type of water turbine?


Answer: Pelton

363. In sensible heating the absolute humidity remains constant but the relative humidity:
Answer: Decrease

364. All of the following process can be found on a psychrometric chart except?
Answer: Black body radiation

365. When checking zinc plates in a condenser, one should:


Answer: Clean the plates and renew worn out ones

366. Turbidity in water is due to:


Answer: Finally divided particles of clay, silt and organic matter
367. The force when applied to a mass of one kilogram will give mass an acceleration of one
meter per second for every second called:
Answer: Newton

368. A precooler is sometimes installed between the


Answer: Condenser and expansion valve

369. A type of polymer used for detergents, milk bottles, oil containers and toys
Answer: High density polyethelene (HDPE)

370. The fact that the amount of slightly soluble gas absorbed in a liquid is proportional to the
partial pressure of the gas is known as:
Answer: Henry’s Law

371. What is the pressure at the exit of a draft tube in a turbine?


Answer: Atmospheric

372. What do you call the device that is used as low-pressure control and high-pressure cutout
on a compressor?
Answer: Pressure Controller

373. Redox reactions can often be type of:


Answer: Double replacement

374. Excessive head pressure is caused by


Answer: Any of the other choices
(Dirty condenser tubes, Insufficient cooling water to condenser, Air or non condensable gas
in the system)

375. In which part of the vapor compression cycle there is abrupt change in pressure and
temperature?
Answer: Expansion valve

376. The kinetic energy of a moving fluid is used to isentropically compressed the fluid to
state of zero velocity. The temperature of a moving fluid at the state zero velocity is called:
Answer: Critical temperature

377. Which is not commonly used to cool and dehumidify equipment?


Answer: Calcium chloride

378. What is the primary cause of smog formation?


Answer: Nitrogen oxides
379. The dehydrating agent in a Freon system is usually
Answer: Activated alumina

380. The solenoid valve is actuated by which of the following?


Answer: A magnet

381. Which of the following does not use ambient air for propulsion?
Answer: Turbo-prop

382. Which of the following would not cause high suction pressure?
Answer: Insufficient refrigeration

383. What is the effect of superheating the refrigerant?


Answer: It increase the coefficient of performance

384. The color of the flame of halide torch, in case of leakage of Freon refrigerant, will change
to
Answer: Green

385. If there is too much lube oil in the system what must be done?
Answer: Remove same at once

386. If the thermal bulb becomes loose on the evaporator coils, it will cause ____.
Answer: Improper operation of expansion valve

387. The effect of superheating the refrigerant is to:


Answer: Decrease the COP

388. In power plant electrostatic precipitators is installed between:


Answer: Furnace and chimney

389. Equipment leaks from plant equipment are known as


Answer: Fugitive emissions

390. Per capita consumption of water is generally taken as:


Answer: 150-300 liters

391. What is the heat that is removed from the space to be cooled, which is the same as the
heat absorbed by the cooling coils?
Answer: Refrigerating effect

392. Horsepower per ton of refrigeration is expressed as:


Answer: Cop/4.75
393. What must be done first when opening a single packed stop valve?
Answer: Loosen the packing before opening

394. The amount of CO2 or Freon in a cylinder is measured by


Answer: Weight

395. The coefficient of velocity is approximately


Answer: 0.30 to 0.50

396. Carries needed to deliver air to the conditioned space it is made of sheet of metals like
aluminum, galvanized sheet steel and some structural materials that will not burn
Answer: Duct

397. Which of the following best described a Freon?


Answer: All of the other choices
(Colorless, Odorless, Non-poisonous)

398. Before securing a compressor to do maintenance work on it, be sure to:


Answer: Have spare parts ready and pump down the system

399. A bull’s eye in a full liquid line will appear ____.


Answer: Clear

400. What operates low-pressure cutout switch?


Answer: Bellows

401. A liquid mixture having constant maximum and minimum boiling points. Refrigerants
comprising this mixture do not combine chemically, yet the mixture provides constant
characteristics.
Answer: Azeotropic Mixture

402. Are organic compounds manufactured in oily liquid and solid forms through the late
1970s and subsequently prohibited.
Answer: Polychlorinated biphenyls (PCBs)

403. What is the temperature range of an air-conditioning application where dry air can be
considered ideal gas?
Answer: -10 to 50 C

404. The coefficient of the velocity, Cv accounts for the:


Answer: Small effect on the flow area of contraction, friction, and turbulence
405. During the re-expansion portion of the refrigeration compressor cycle
Answer: The suction valve is closed and the discharge valve is closed

406. Refers to the high-temperature removal of tarry substances from the interior of the carbon
granule, leaving a highly porous structure
Answer: Activated

407. Are by products of reaction between combustion products


Answer: Oxidants

408. Are any solid particulate matter that becomes airborne, with the exception of particulate
matter emitted from the exhaust stack of a combustion process
Answer: Dusts or fugitive dusts

409. The purpose of the dehydrator is to


Answer: Remove moisture from the refrigerant

410. The angle between the stack and guy wire is usually
Answer: 60 degrees

411. A refrigeration that deals with producing temperature of -157 C or lower.


Answer: Cryogenics

412. The purge valve is located:


Answer: In the highest part of the system

413. Thermal expansion valves are usually made of the:


Answer: Diaphragm and Bellows type

414. The purpose of the expansion valve by-pass is to


Answer: Controls the refrigerant to the evaporator in case the automatic valves fail

415. The water regulating valve is operated by the


Answer: None of the other choices
(Compressor discharge pressure, Compressor suction pressure, Compression discharge
temperature)

416. Superheat is heat added ____.


Answer: After all liquid has been changed to vapor

417. The ratio of maximum load to the rated plant capacity is called
Answer: Utilization factor
418. The highest temperature in vapor compression cycle is produced during:
Answer: Compressor discharge

419. Dry analysis is a fractional analysis of the products of combustion which does not
include:
Answer: Water vapor

420. Valves and piping in an ammonia system are made of:


Answer: Iron

421. The locus of elevation to which water will rise in the piezometer tube is termed:
Answer: Hydraulic gradient

422. A substance with surface area per unit of weight, and intricate pore structure, and a
hydrophobic surface
Answer: Adsorbent substance

423. The purpose of the expansion valve is to control the flow of the refrigerant to the
evaporator. The other function is to:
Answer: Reduce the pressure of the liquid refrigerant

424. An ammonia leak will turn litmus paper ____.


Answer: Blue

425. What is the most common freon gas used in reciprocating compressors?
Answer: F-12

426. The empirical coefficient e in machine foundation if not given is assumed


Answer: 0.11

427. The capacity of a centrifugal type compressor is controlled by which of the following?
Answer: Regulating the speed or regulating the suction pressure

428. Antifreeze chemicals are:


Answer: Those that lower down the freezing points of liquids

429. A type of refrigerant that will not damage the ozone layer
Answer: Hydrofluorocarbons (HCF’s)

430. At 0 psig, how may BTU necessary to change 1 lb of water at 212 F of steam?
Answer: 970
431. Modern stacks are seldom built higher than ____.
Answer: 60 m

432. If Pi is the indicated horsepower and Pb is the indicated horsepower of a compressor,


then what is mechanical efficiency, Em, equal to:
Answer: Em = Pi/Pb

433. A refrigeration control that guards the compressor forms overloads brought about by
abruptly increases loads resulting from defrosting, warm products, and others is called:
Answer: Suction hold-back valve

434. When a solvent has dissolve as much as it can, the mixture is called:
Answer: Saturated solution

435. Fan motors in air conditioning usually have:


Answer: 2 or 3 speeds

436. In the upper atmosphere, ozone is made by ultraviolet light reacting with:
Answer: Oxygen

437. Which of the following is the oil used in a refrigeration system?


Answer: Straight mineral oil

438. The following are all examples of indirect (secondary) miscellaneous methods to measure
flow except:
Answer: Positive displacement meters

439. Which of the following is the function of a suction pressure regulating valve?
Answer: Maintains proper back pressure

440. What is the use of back pressure regulating valve?


Answer: Maintains a fixed pressure in the evaporator coils

441. If Wt is the turbine shaft work of a gas turbine unit, Wc in duel, then determine its
thermal efficiency.
Answer: (Wt-Wc) / Q

442. The solenoid valve is controlled by


Answer: The temperature in the icebox

443. Where is the excess refrigerant removed?


Answer: Charging side of the system
444. All of the following units of energy except
Answer: Pascals

445. What usually happened if brine has a high specific gravity?


Answer: It will crystallize

446. There are three basic boiler types, namely;


Answer: Cast-iron, fire-tube and water tube boilers

447. Measurements of a device’s ability to remove atmosphere air from test air.
Answer: Atmospheric Dust Spot Efficiency

448. The normal cut-out setting of a window unit thermostat between:


Answer: 13 C to 16 C

449. The use of water to carry heat occupied spaces


Answer: Hydronic Heating System

450. Reheating process is normally employed in steam plant when:


Answer: Turbine undergoes excessive moisture

451. In the discharge line between the compressor and the condenser one would find:
Answer: High pressure, high temperature gas

452. A good refrigerant should be


Answer: All of the other choices
(Non-poisonous, non-inflammable, non-explosive)

453. What is the relationship of the horse power of a centrifugal pump, Hp, to the impeller
speed, S, if the pump is at two different rotative speeds?
Answer: Hp is directly proportional to the cube of the ratio of S

454. Which of the following is the reason when the crankcase is cooler than the suction line?
Answer: Too much refrigerant

455. Humidity is a measure of which of the following?


Answer: Water vapor content

456. High head turbine is a/an:


Answer: Impulse

457. Critical temperature is that temperature above which


Answer: A gas will never liquefy
458. The fact that a fluid’s velocity increases as the cross-sectional area of the pipe through
which it flow decrease is due to:
Answer: The continuity equation

459. Which of the following is important for evaporator coils?


Answer: It should have air completely surrounding them

460. Which of the following types of air dryers works by absorbing moisture on a solid
desiccant or drying material such as activated alumina, silicone gel, or molecular sieve?
Answer: Deliquescent dryer

461. Which of the following would cause the compressor to run continuously?
Answer: Low pressure switch jammed

462. In a window air conditioning unit which of the following is usually done by the owner?
Answer: Semi-annual cleaning or replacement of filters

463. Which of the following is not similarity between submerged culvert and a siphon?
Answer: Torricelli’s equation holds

464. Where does the final removal of water vapor in an absorption refrigeration system occur?
Answer: Rectifier

465. The constant spending of certain percentage of circulated water in a cooling tower in
order to prevent accumulation of dissolved mineral solids and other impurities in the
condenser water is called
Answer: Bleed-off

466. Which of the following refrigerant is added sometimes to other refrigerant to improve oil
circulation?
Answer: R – 170 (Ethane)

467. The suction pressure switch is operated by which of the following?


Answer: Pressure on a bellow

468. When the air is saturated the wet-bulb depression is:


Answer: Zero

469. Heating and dehumidification can be obtained simultaneously if air is passed through:
Answer: Either a solid absorbent surface or a liquid absorbent spray
470. In cooling cycle, the dry bulb temperature (db) of the air is lowered. When this happens
the relative humidity
Answer: Increase

471. What will happen when the pressure at any point inside a centrifugal pump goes below
the vapor pressure corresponding to the temperature of the liquid?
Answer: Cavitation

472. The ratio between the actual power and the apparent power in any circuit known as the
____ of that circuit
Answer: Power factor

473. Refrigerant leakage from the compressor crankcase is prevented by


Answer: Using shaft seals

474. The matching of scale and full-scale results for a fluid dynamic phenomena involving
compressible fluids requires quality of:
Answer: Mach number

475. Ammonia leaks in the condenser can be detected by:


Answer: Applying litmus paper to the circulating water discharge

476. A swinging support constructed as part of the vessel and that supports the manway cover
when it is unbolted and moved aside.
Answer: Davit

477. Which of the following type valves are not found on a Freon – 12 system?
Answer: Duplex

478. What is the main function of a receiver?


Answer: Store the refrigerant

479. The dividing point between the high pressure and low pressure sides of the refrigeration
cycle occurs at the
Answer: Expansion valve

480. If the solute particles of a solid suspended in a liquid are larger than molecules the
mixture is known as?
Answer: Suspension

481. A mechanism that removes moisture


Answer: Dehumidifiers
482. Is a water soluble organic compound prepared from ammonia. It has significant
biological and industrial usefulness
Answer: Urea or carbamide urea

483. Which of the following fans in air conditioning systems which can be classified as
centrifugal flow
Answer: Propeller fan

484. What is the use of the low – water cutout switch?


Answer: Stops the compressor when there is insufficient cooling water

485. In parallel pipe system originating and terminating in common junctions,


Answer: Pressure drops through each branch are equal

486. When the dry bulb and the wet bulb temperature are identical, the air is said to be:
Answer: Saturates

487. The temperature in which water vapour in the gas begins to condensate in a constant
pressure process.
Answer: Dew point

488. Combination gas/fuel oil burners permit the operator to switch from one fuel to ____
Answer: All of the above
(For economy, Because of a shortage of fuel being used, Because of a failure in the fuel
system being used)

489. Most nuclear particles can react with atoms in several different ways including
Answer: Absorption and scattering

490. What is the effect of superheating the refrigerant?


Answer: It increase the coefficient of performance

491. Boiling temperature of Freon 12 is


Answer: -29.8 C

492. Which of the following would cause low head pressure?


Answer: Too much cooling water and/ or insufficient refrigerant gas

493. At what temperature will water normally turns to steam?


Answer: 212 F

494. What will happen if the expansion valve is opened too wide?
Answer: Liquid will flood back to the compressor
495. All of the following are words used to describe neutron kinetic energy levels except:
Answer: Supersonic

496. If the thermal expansion valve becomes inoperative, the ice boxes will have to be
controlled by the ____.
Answer: Manual expansion valve

497. If the superheat on the suction side of the compressor is increased, what will happen to
the tonnage capacity of the unit?
Answer: Decreases

498. The solenoid valve can be typed as a ____.


Answer: Magnetic soap valve

499. The ratio of fugacity of actual conditions to the fugacity at some reference state is known
as:
Answer: Activity

500. The process that takes place in the evaporator called


Answer: Absorption of the latent heat of vaporization

501. When charging freon system, all the valves should be


Answer: King (liquid) valve

502. If the pressure exerted on a liquid is higher than the saturation corresponding to it
temperature, the liquid is a:
Answer: Sub cooled liquid

503. The most likely cause of high superheat would be:


Answer: Expansion valve closed too much

504. A good refrigerant should have a


Answer: High latent heat

505. What is the device used to protect the compressor from overloading due to high head
pressure
Answer: Overload relay

506. Which of the following it to be checked as part of weekly maintenance schedule of a


console air conditioners?
Answer: Cooling towers
507. Which of the following would not be cause for a refrigerating system to short cycle on hp
cutout?
Answer: Discharge valve leaking

508. A ____ boiler has heat and gases of combustion that pass through tubes surrounded by
water
Answer: Fire tube

509. A type of polymer used for clear bottles.


Answer: Polyvinyl Chloride (PVC)

510. In an ammonia system, the oil gauge must be kept:


Answer: Closed except when checking level oil

511. If a compartment requires the removal of 36,000 BTU per hour, how much is necessary
compressor capacity?
Answer: 3 tons

512. The oil level in the compressor should be checked:


Answer: After a long period of operation

513. Which of the following industries have the highest consumption of water for processing?
Answer: Paper mill

514. What is the lowest temperature to which water could possibly be cooled in a cooling
tower?
Answer: The temperature of adiabatic compression

515. The method of cooling which primarily used where ambient air temperatures are high and
relative humidity is used:
Answer: Swamp cooling
WASTEWATER TREATMENT PLANT

Wastewater - there is no further use, nothing can be done, hence must be disposed
- effluent of unwanted surplus substance arising from application of any process
- any substance w/c requires to be disposed of as being contaminated or otherwise spoiled
Domestic Wastewater – found in household and commercial areas
Indus trial Wastewater – found in factories
Ph - measure of alkalinity and acidity of the wastewater
COD (Chemical Oxygen Demand) – chemical approach to measure the amount of oxygen equivalent to the
portion of organic matter that is susceptible to oxidation by using a strong oxidant
BOD (Biological Oxygen Demand) – biological approach to measure amount of oxygen required by the
aerobic bacteria to oxidize (=EAT) the organic
FOG(Fats, Oil and Grease) – approximation of the amount of organic substances that are extracted for
aqueous solution of suspension by hexane to105C to the initial weight of the sample is measured
Methane Reactor – anaerobic digester where decomposition/breakdown of organic matter is taking place.
Alkalinity – capacity of water to neutralize acids due to carbonate or bicarbonates and expressed in
milligrams per liter of calcium carbonate equivalent
VFA(Volatile Fatty Acids) – biochemical decomposition of organic matter, facultative and a wide variety of
anaerobic bacteria hydrolyze and convert the complex materials to from low-molecular weight compounds
such as short-chain fatty acids bec. They can be distilled at atm. Pressure
TSS (Total Suspended Solids) – measure of the undissolved particulate matter where the dry solids are
captured by filtering a known volume of sample
VSS (Volatile Suspended Solids) – approximation of organic content of the sample where the ratio of the
weight of dry matter lost in heating a sample to 650C to the initial weight of sample measured
Toxicity – irreversible inhibition of both acidogenic and methanogenic bacteria
Influent – wastewater that flows into each unit of operation of the wastewater treatment plant that is
partially or completely treated
Biodegration – breakdown of man-made and naturally occurring compounds to their constituent elements
and compounds by microbial action through different biological renewal cycles
DO METER – used to measure amount of dissolved oxygen in aeration tank

Biological Treatment
Aerobic – treatment process where the microorganisms require oxygen for their metabolism to biodegrade
organic matter
Anaerobic – breakdown/decomposition of almost all kinds of organic matter by the action of a wide range
of microorganisms, principally methane bacteria
Chemical Treatment – use chemicals to destroy or neutralize toxicity and pollution
Physical Treatment – settling, sedimentation, filtration, flotation, evaporation and distillation
ADVANTAGES OF ANAEROBIC TREATMENT
- Smaller Area Required - Low operating cost
- Sludge Production is minimal - Less moving parts
- Biogas can be recovered - Long shutdown possible

PIPING

VISCOSITY - is termed as absolute viscosity and dynamic viscosity.

KINEMATIC VISCOSITY - is a different from absolute viscosity.

PIPE – tube with around cross section conforming to the dimensional requirements for nominal pipe size

TUBE – hollow product of round or any other cross section having continuous periphery

BELL AND SPIGOT JOINT – commonly used joint in cast iron pipe. Each piece is made w/an enlarged
diameter or bell at one end into w/c the plain or spigot end of another piece is inserted when laying. The
joint is then made tight by cement, oakum, lead, or rubber caulked in to the bell round the spigot.
BLACK PIPE – steel pipe that has not been galvanized
BONNET – part of a valve used to guide and supports the valve steam
BULL HEAD TEE – ate, the branch of w/c is larger than the run
BUTT WELD JOINT – a welded pipe joint made with the ends of the two pipes butting each other,
CARBON STEEL PIPE – steel pipe that owes its properties chiefly to the carbon w/c it contains
CHECK VALVE – valve designed to allow a fluid to pass through in one direction only
COMPRESSION JOINT – multi piece joint with cup shaped threaded nuts which when tightened, compress
tapered w so that they form a tight joint on the periphery of the tubing they connect
CROSS-OVER – small fitting with a double offset or shaped like the letter U w/ the ends turned out
EXPANSION LOOP – a large radius bend in a pipeline to absorb longitudinal expansion on the pipe line due
to heat
GALVANIZED PIPE – steel pipe coated with zinc to resist corrosion
GATE VALVE – valve employing a gate, often wedge-shaped, allowing fluid to flow when the gate is lifted
from the seat. Such valves have less resistance to flow than globe valves-stop valve or isolation valve
GLOBE VALVE – one w/a somewhat globe-shaped body w/a manually raised or lowered disc w/c when
closed rests on a seat so as to prevent passage of a fluid
-hole filing service and close regulation of flow
HEADER – a larger pipe or drum into which each of a group of boilers is connected
MANIFOLD – a fitting w/a number of branches in line connecting to smaller pipes. Used largely as an
interchangeable term with header
MEDIUM PRESSURE – when applied to valves and fittings, implies they are suitable for a working pressure
of 862 to 1207 kPa
MILL LENGTH – known as random length. Run if mill pipe is 4880mm to 6000mm in length
RELIEF VALVE – one designed to pen automatically to relieve excess pressure
RUN – a length of pipe made of more than one piece of pipe
SADDLE FLANGE – a flanged curved to fit a boiler or tank and to be attached to a threaded pipe
SCREWED FLANGE – a flange screwed on the pipe which is connected to adjoining pipe
SOCKET WELD – a joint made by use of a socket weld fitting which has a prepared female end or socket for
insertion of the pipe to which it is welded
STREET ELBOW – an elbow with male thread on one end, and female thread on the other end
WROUGHT IRON – an iron refined to a plastic state in a pudding furnace. Characterized by the presence of
about 3%of slag irregularly mixed with pure iron and about 0.50% carbon
WROUGHT PIPE – refers to both wrought steel and wrought iron
GREEN - water BROWN – oil-mineral vegetable or animal,
flammable or combustible
SILVER GRAY –steam
BLACK – other fluids, including drainage pipes
VIOLET – acid&alkalis
unless the drain is to a particular service
LIGHT BLUE- air
SAFETY RED – firefight
LIGHT ORANGE -electricity
SAFETY YELLOW – hazardous service
YELLOW OCHRE-gases

Velocity in Pipeline:

1-2m/s – liquid: pump, BFW 30-80m/s – steam: superheated

0.5-1m/s – liquid: city water 10-20m/s – air: blower

10-40m/s – steam: saturated 10-30m/s – air: compressor | gas: low&high


pressure
50-70-m/s – steam: turbine
1-5m/s – gas: smoke tube
INDUSTRIAL PUMPS
PUMPS – are industrial machine that move any fluid of high volatile chemicals from the source to the
location which the liquid is to be delivered
CENTRIFUGAL PUMP – ideal type of pump, high discharge, low head, high speed, not self priming

ROTARY PUMP – used for pumping low discharge, low head and for pumping viscous liquids like oil

RECIPROCATING PUMP – ideal type of pump, low discharge, high head, low speed and self priming

WATER HORSEPOWER – theoretical amount of energy necessary to raise a given volume of fluid from a
lower to a higher elevation
-power delivered by the pump
-power required to deliver a given developed head with no losses in the pump
Y – STRAINERS – to protect the pump placed in the line at the inlet to pumps, should not less than 40mesh
PRESSURE GAGES – to determine the pump suction and discharge pressure
FLEXIBLE CONNECTORS – used to isolate vibration made by the pump
PUMP CAPACITY – refers to the volume of fluid delivered by the pump per unit time
STATIC HEAD – height of the surface of the water above gauge point
PRESSURE HEAD – load plus gauge pressure or the water surface pressure plus the friction loss
VELOCITY HEAD – head required to produce the flow of water
DYNAMIC HEAD – pressure head plus velocity head
TOTAL SUCTION HEAD – the sum of all heads at suction of the pump
TOTAL DISCHARGED HEAD – the sum of all heads at discharge point
NET POSITIVE SUCTION HEAD – difference between the absolute dynamic pressure of the liquid measured
at the centreline of the pump and the saturation pressure corresponding to the temperature of the liquid
at the same point
TOTAL DYNAMIC HEAD – difference between total discharge head and total suction head
CAVITATION – describes a cycle of phenomena that occurs in flowing liquid because the pressure falls
below the vapour pressure of the liquid, thus releasing liquid vapors on the low pressure area forming
bubbles w/c will collapse into a region of high pressure thus causing implosion
PUMP EFFICIENCIES – sometimes called the mechanical efficiencies of the pump. Ratio of WHP and BHP
BRAKE HORSEPOWER – power required to drive the pump or power input to the pump
THERMAL EFFICIENCY – (for steam driven pump) ratio of heat equivalent of the WHP to the heat supplied
at the steam end
PUMP DUTY – foot-pounds of work done by the pump per million BTU of the steam supplied on the steam
side of the steam driven pump
VOLUMETRIC EFFICIENCY – (for reciprocating pump) the ratio of actual capacity of the pump over the
piston displacement of the pump
PISTON DISPLACEMENT – volume swept off by the piston in one stroke per unit time
SLIP – losses due to packing of the cylinders or leakage between piston and cylinder walls
OVERALL EFFICIENCY – refers to the effectiveness of the conversion of electrical power to WHP
STATIC SUCTION LIFT – the vertical distance in feet from the liquid supply level to the pump centreline, the
pump being above the supply level
STATIC SUCTION HEAD – when the pump is below the liquid supply level, it exists
STATIC DISCHARED HEAD - the vertical distance in feet from the pump centreline to the point of delivery
TOTAL STATIC HEAD – vertical distance in feet, between supply level and the discharged level of the liquid
being handled
DYNAMIC TOTAL SUCTION LIFT – sum of the static suction lift, suction function head, entrance loss in the
suction pipe and the suction velocity head

DYNAMIC TOTAL SUCTION LIFT HEAD – static suction head minus the suction friction head and suction
piping entrance loss plus any pressure in the suction line

OPEN SYSTEM – pump moves a liquid from a source located above or below the pump but open to
atmosphere

CLOSED SYSTEM – which the liquid circuit is not open to atmosphere

AXIAL FLOW – centrifugal pump in which pressure is developed by propelling of faning action of the
vanes of the impeller on the liquid

WATER FLOW METER – measure condensate, feed water, and pump discharge

AIR COMPRESSOR, BLOWERS AND INDUSTRIAL FANS

FANS – used where low pressure, from a few millimetres of 16 to 50 mmHg and comparative to large
volumes are required, lo speeds

BLOWER – is a machine to compress air or gas by centrifugal force to a final pressure not exceeding
0.24 MPa gage. It is not water cooled (35 psi or 2.5 bars and below) as the aided expense of cooling
system is not justified by the slight gain at these pressures.
- raises fermented CO2 pressure from 2” H20 to 5-7psig & necessary to overcome pipe
friction

COMPRESSOR – raises pressure of CO2 gas to 250 psig so that it can be liquefied at 100 F

COMPRESSOR – machine used to compress air or gas to a fluid pressure above 0.24Mpa, water cooled
above 35psi or 2.5bar

AIR COMPRESSOR – machine that decreases the volume and increases the pressure of a quantity of air by
mechanical means

ROTATING AIR COMPRESSOR – used for low and medium pressures, consists of bladed wheel or impeller
that spins inside a closed circular housing

RECEIVER – in an air compressor system is used to reduce the work needed during compression

ATMOSPHERIC DEW POINT (ADP) - is the temperature below which condensation occurs after the air is
expanded again to the atmosphere.

PRESSURE DEW POINT (PDP) – is the temperature below which condensation occurs at the pressure the
air is at

COOLING TOWERS

COOLING TOWER – device for reducing the temperature of a liquid, usually water by bringing it into
contact with airstream where a small portion of the liquid is evaporated and the major portion is cooled
NATURAL DRAFT COOLING TOWERS – uses the natural movement of air in cooling the water
ARTIFICIAL DRAFT COOLING TOWERS – uses fans or blowers to cause air movement in the cooling tower
FORCED DRAFT COOLING TOWER – pressure is exerted at the inlet of air to the tower by the fan
INDUCED DRAFT COOLING TOWER – pressure is exerted between the outlets of air in the tower
- draws the flue gas from the system and sends them up the stack

2-5% - water losses from cooling towers

INDUSTRIAL DRYING
TYPES OF COMMERCIAL DRYERS
- Hearth Dryer - Compartment Dryer/ Tray Dryer
- Tower Dryer - Rotary Dryer
- Tunnel / Kiln Dryer - Infrared Dryer
MATERIAL HANDLING, CLEANING AND MILLING EQUIPMENT
BASIC MATERIAL HANDLING ELEMENTS
- Lifting
- Dragging, Towing & Pushing
- Carrying & Lifting
- Conveying System
MATERIAL HANDLING TECHNOLOGY
- Containerization –covers the broad spectrum of confinement methods that are used for
storage through all phases of the manufacturing process, cycle.
- Fixed-path handling- applies to movement and storage of units loads of material with an
intermittent or a continuous flow over a fixed path from one point to another
- Mobile handling – considers the systems, equipment practices and requirements
- Warehousing- involves a wider range of planning and analysis(location of activity, sizing ,
storage equipment, packaging methods
HOPPERS – available for intermediate storage, containment and scaling of products
CYCLONES – primary function of a cyclone is to separate product from a gas steam, after separation, the
product discharges out the bottom of the cyclone and air discharges through the top exhaust
AFC BATCH –WEIGH SYSTEM – automated flexible conveyor is a natural addition to the company’s multi
purpose spiral feeder conveyor system
CONVEYOR – a mechanical system for transporting materials from one site to another
CHAIN CONVEYOR – used in horizontal transportation of materials where food nourishment chemical,
mining, saw dust and soil industry, grain silos and grain handling
ROLLER MILL – passing the malt between two closely spaced rolls
SILO DISCHARGER – used for discharging of free flowing and sluggish bulk materials from concrete and
steel silos by the help of vibro motor
AUTOMATIC METERING – often used for weighing and recording the weight of the material being used
or being carried by conveyors.

FUELS AND COMBUSTION


Fuel – an organic material composed of chemical elements w/c, in rapid chemical union w/oxygen
produces combustion (coal, baggase, wood, diesel BFO, LPG, methane)
Properties if Fuel Oils

Specific Gravity – ratio bet the weight of any volume of oil at 15C and weight of an equal volume of pure
water at 15C

Heating Value – amount of heat liberated during complete combustion per unit of volume

Viscosity – measure of the fuels resistance to flow, and oils internal friction, its thickness (cSt)

Bottom sediments and water BSW – the amount of impurities present in the fuel and given as % by vol
for water and % by weight for sediments

Ash- residue that will form after combustion due to the presence of impurities

Flash Point – temp at w/c oil will produce vapors w/c will flash in the presence of an external flame

Fire Point – temp at w/c oil vapors will no longer flash but will support continuous flame

Pour Point – temp at w/c an oil will just flow under standard conditions

Sulfur Content – the amount of sulphur in the fuel oil measure as % by weight

MECHANICAL REFRIGERATION

COMPRESSOR – compresses refrigerant vapour and causes it to flow in the system

CONDENSER – the refrigerant condenses while rejecting heat to the cooling medium, which is either air
to water

EXPANSION VALVE – reduces the pressure of the refrigerant so that low temp will be attained, regulates
the flow of the refrigerant to the evaporator

EVAPORATOR –liquid portion of the refrigerant evaporates while absorbing heat from the surrounding

COEFFICIENT OF PERFORMANCE – the ratio of the refrigerating effect to the work of compression

R-502 – azeotropic mixture of R-22 and R-115, popular refrigerant for low-temp refrigeration system

ACCUMULATOR – used to prevent or minimize entry suction line liquid refrigerant into the compressor

BACK PRESSURE – the pressure of low pressure side also known as suction pressure

FLOODED SYSTEM – only amount of refrigerant that is evaporated is fed to the heat transfer area and
liquid level is controlled by a float

LIQUID OVERFEED SYSTEM – system of feeding, mechanically or by gas pressure, low temp saturated
liquid ammonia to the evaporators at a rate which is greater than what is actually being evaporated
STEAM

STEAM – a vapour into which water is converted when heated to its boiling point

GROUPING OF BOILERS

FIRE TUBE BOILER – flue gas inside the tube and heated fluid outside the tube

WATER TUBE BOILER – flue gas outside the tube and heated fluid inside the tube

MAKE-UP WATER – treated water which is added to the system to replace steam and water lost

CONDENSATE – steam that is returned back to its liquid state normally at steam temp

FEEDWATER – water supplied to the boiler and is the sum of the condensate and make-up

BOILER WATER –water inside the boiler

BOILER STORAGE

CO2 RECOVERY INDUSTRIAL PLANT

MAIN PROCESS

-CO2 Collection -Moisture Removal

-CO2 Purification -Liquefaction

-Compression -Distribution

FOAM TRAP –equipment that is made to have CO2 gas passed against fine H20 spray

BLOWER – raises fermented CO2 pressure from 2” H20 to 5-7psig & necessary to overcome pipe friction

SCRUBBER – removes water soluble substances such as ketones, alcohol and sulphur compounds

DEODORIZER – removes non-water soluble substance from CO2 gas/ one for regeneration while other is
in service

COMPRESSOR – raises pressure of CO2 gas to 250 psig so that it can be liquefied at 10F

SURGE TANK – vertical cylindrical tank which acts as high pressure CO2 gas receiver

PRE-COOLER – cools CO2 gas with glucose as cooling medium and removes a considerable amount of
moisture by condensation

DRYER - removes moisture from CO2 gas to prevent freeze-up of CO2 condenser

CONDENSER – shell and tube heat exchanger used to liquefy CO2 gas, reduces volume if CO2 to storage

STORAGE TANK – mild steel tank located below condensers to reliquefy CO2 evaporating in the tank

VAPORIZER – converts liquid CO2 gas for use in the brewery, vaporized by warm water
1. Find the work possess for a helium gas at 20°C.
A. 609 kJ/kg B. 168 kJ/kg C. 229 kJ/kg D. 339 kJ/kg
Solution:
( )

( )

2. Two kilogram of gas is confined in a 1 m3 tank at 200 kPa and 88°C. What type of gas is in the
tank?
A. Helium B. Ethane C. Methane D. Ethene
Solution:

( ) ( )( )

( )

3. Find the enthalpy of Helium if its internal energy is 200 kJ/kg.


A. 144 kJ/kg B. 223.42 kJ/kg C. 333.42 kJ/kg D. 168 kJ/kg
Solution:

( )

4. Compute the mass of a 2 m3 propane at 280 kPa and 40°C.


A. 6.47 kg B. 5.1 kg C. 10.20 kg D. 9.47 kg
Solution:
( ) ( )

( ) ( )( )

5. Compute the air flow in ft3/min of mechanical ventilation required to exhaust an


accumulation of refrigerant due to leaks of the system capable of revolving air from the
machinery for a mass of 4 lbs refrigerant.
A. 200 B. 210 C. 220 D. 230
Solution:
1

6. Compute the free aperture cross section in m3 for the ventilation of a machinery room if the
mass of refrigerant is 9 kg.
A. 0.314 B. 0.414 C. 0.514 D. 0.614
Solution:

7. A 29.53” x 39.37” pressure vessel contains ammonia with f = 0.041. Compute the minimum
required discharge capacity of the relief device in kg/hr.
A. 106.71 kg/hr B. 108.71 kg/hr C. 110.71 kg/hr D. 112.71 kg/hr
Solution:
( )
( )( ) ( )

8. Compute the maximum length of the discharge pipe installed on an outlet of a pressure-relief
device in feet for internal pipe diameter of 0.5 inch and rated discharge capacity is 8 lb/min of
air. The rated pressure of relief valve is 10 psig.
A. 0.286 ft B. 0.386 ft C. 0.486 ft D. 0.586 ft
Solution:
( )
( ) ( )
( )

9. A thermal power plant has the heat rate of 11,363 Btu/kW-hr. Find the thermal efficiency of
the plant.
A. 28% B. 30% C. 34% D. 40%
Solution:

10. What is the hydraulic gradient of a 1 mile, 17 inches inside diameter pipe when 3300
gal/min of water flow with f = 0.03?
A. 0.00714 B. 0.00614 C. 0.00234 D. 0.00187
Solution:

( )

2
( )( )
( )( )

11. Find the loss of head in the pipe entrance if speed of flow is 10 m/s.
A. 5.10 m B. 10.2 m C. 17.4 m D. 2.55 m
Solution:
( ) [ ( )
]

12. Wet material, containing 220% moisture (dry basis), is to be dried at a rate of 1.5 kg/s in a
continuous dryer to give a product containing 10% (dry basis). Find the moisture removed in
kg/hr.
A. 3543.75 kg/hr B. 3513.75 kg/hr C. 3563.75 kg/hr D. 3593.75 kg/hr
Solution:

( ) ( )
( )

13. Copra enters a dryer containing 70% moisture and leaves 7% moisture. Find the moisture
removed on each pound of solid in the final product.
A. 6.258 lbs B. 1.258 lbs C. 4.258 lbs D. 2.258 lbs
Solution:

14. A 1 m x 1.5 m cylindrical tank is full of oil with SG = 0.92. Find the force acting at the bottom
of the tank in dynes.
A. 106.33 x 103 dynes C. 106.33 x 105 dynes
B. 106.33 x 104 dynes D. 106.33 x 106 dynes
Solution:

( )

3
15. Find the pressure at the 100 fathom depth of water in kPa.
A. 1,793.96 kPa B. 1,893.96 kPa C. 1,993.96 kPa D. 1,693.96 kPa
Solution:

( )

16. Find the depth in furlong of the ocean (SG = 1.03) if the pressure at the sea bed is 2,032.56
kPa.
A. 1 B. 2 C. 3 D. 4
Solution:

( )( )( )

17. Find the mass of 10 quartz of water.


A. 10.46 kg B. 9.46 kg C. 11.46 kg D. 8.46 kg
Solution:
-
( )( )( )
-
( )

18. Find the mass of carbon dioxide having a pressure of 20 psia at 200°F with 10 ft 3 volume.
A. 1.04 lbs B. 1.14 lbs C. 1.24 lbs D. 1.34 lbs
Solution:

( )( ) ( )( )

19. Find the heat needed to raise the temperature of water from 30°C to 100°C with 60%
quality. Consider an atmospheric pressure of 101.325 kPa. Use the approximate enthalpy
formula of liquid.
A. 293.09 kJ/kg B. 1,772.90 kJ/kg C. 1,547.90 kJ/kg D. 1,647.29 kJ/kg
Solution:

( )

4
( )
( ) ( )

20. Find the enthalpy of water at 212°F and 14.7 psi if the dryness factor is 30%. Use the
approximate enthalpy formula of liquid.
A. 461 Btu/lb B. 471 Btu/lb C. 481 Btu/lb D. 491 Btu/lb
Solution:

( )

21. An air compressor consumed 1200 kW-hr per day of energy. The electric motor driving the
compressor has an efficiency of 80%. If indicated power of compressor is 34 kW, find the
mechanical efficiency of the compressor.
A. 117.65% B. 75% C. 85% D. 90%
Solution:
( )
( )

22. A refrigeration system consumed 28,800 kW-hr per month of energy. There is 20% of
energy lost due to the cooling system of the compressor and the motor efficiency is 90%. If the
COP of the system is 6, find the mechanical efficiency of the compressor.
A. 43.15 TR B. 46.15 TR C. 49.15 TR D. 41.15 TR
Solution:
( )( )
( )
̇ ( )

̇
̇ ( )

23. A 23 ton refrigeration system has a heat rejected of 100 kW. Find the energy efficiency ratio
of the system.
A. 13.42 B. 14.42 C. 15.42 D. 16.42
Solution:
5
̇ ̇
( ) ̇
̇
( )
̇
( )

24. A 200 mm x 250 mm, 8-cylinder, 4-stroke diesel engine has a brake power of 150 kW. The
mechanical efficiency is 80%. If two of the cylinders were accidentally cut off, wha t will be the
new friction power?
A. 31.50 kW B. 33.50 kW C. 35.50 kW D. 37.50 kW
Solution:

25. If the energy efficiency ratio of the refrigeration system is 12.6, what is the COP of the
system?
A. 3.69 B. 4.23 C. 5.92 D. 6.83
Solution:

26. An air compressor has a power of 40 kW at 4% clearance. If the clearance will increase to
7%, what is the new power?
A. 70 kW B. 40 kW C. 53 kW D. 60 kW
Solution:

27. What is the approximate value of temperature of water having enthalpy of 208 Btu/lb?
A. 138.67°C B. 115.55°C C. 258.67°C D. 68.67°C
Solution:

6
28. Convert 750°R to K.
A. 390.33 K B. 395.33 K C. 410.33 K D. 416.33 K
Solution:

29. An Otto cycle has a compression ratio of 8. Find the pressure during compression.
A. 18.38 B. 16.38 C. 14.38 D. 12.28
Solution:

( ) ( )

30. A diesel cycle has a cut off ratio of 2.5 and expansion ratio of 4. Find the clearance of the
cycle.
A. 9.11% B. 5.55% C. 11.11% D. 15.15%
Solution:
( )

31. A dual cycle has an initial temperature of 30°C. The compression ratio is 6 and the heat
addition at constant volume process is 600 kJ/kg. If the cut-off ratio is 2.5, find the maximum
temperature of the cycle.
A. 3638.50°C B. 3365.50°C C. 3565.50°C D. 3965.50°C
Solution:
( ) ( ) ( )( )

( )( )( ) ( )

( ) ( )

32. A 3-stage air compressor compresses air from 100 kPa to 1000 kPa. Find the intercooler
pressure between the 1st and 2nd stage.
A. 505.44 kPa B. 108.44 kPa C. 316.23 kPa D. 215.44 kPa
Solution:
7
( ) ( )

33. A 10-stage air compressor compresses air from 100 kPa to 800 kPa. Find the intercooler
pressure between the 1st and 2nd stage.
A. 282.84 kPa B. 113.21 kPa C. 123.11 kPa D. 333.51 kPa
Solution:
√ √

34. A 3-stage air compressor compresses air from 100 kPa to 700 kPa. Find the intercooler
pressure between the 2nd and 3rd stage.
A. 365.88 kPa B. 375.88 kPa C. 385.88 kPa D. 395.88 kPa
Solution:
( ) ( )

35. Carnot cycle A, B and C are connected in series so that the heat rejected from A will be the
heat added to B. and heat rejected from B will be added to C. Each cycle operates between
30°C and 400°C. If heat added to A is 1000 kW, find the work output of C.
A. 111.44 kW B. 549.78 kW C. 247.53 kW D. 141.89 kW
Solution:

̇ ̇ ̇
̇ ̇
̇

̇ ̇
̇

̇ ̇
̇
̇
̇

36. Air compressed adiabatically from 30°C to 100°C. If the mass of air being compressed is 5
kg, find the change of entropy.
A. 1.039 kJ/K B. 0.746 kJ/K C. 0 D. 1.245 kJ/K
Solution:

8
37. Two kilogram of air in a rigid tank changes its temperature from 32°C to 150°C. Find the
work done during the process.
A. 236 B. 170 C. 195 D. 0
Solution:

38. Determine the atmospheric pressure at a location where barometric reading is 740 mm Hg
and gravitational acceleration is g = 9.7 m/s 2. Assume the temperature of mercury to be 10°C,
at which the density is 13,570 kg/m3.
A. 99.45 kPa B. 97.41 kPa C. 95.44 kPa D. 98.66 kPa
Solution:
( )( )

39. The barometer of a mountain hiker reads 930 mbars at the beginning of a hiking trip and
780 mbars at the end. Neglecting the effect of altitude on local gravitational acceleration,
determine the vertical distance climbed. Assume g = 9.7 m/s 2.
A. 1274.21 m B. 1289.00 m C. 1267.34 m D. 1583.34 m
Solution:

( )( ) ( )( )

40. The lower half of a 10 m high cylindrical container is filled with water and the upper half
with oil that has SG = 0.85. Determine the pressure difference between the top and bottom if
the cylinder.
A. 90.74 kPa B. 92.74 kPa C. 83.38 kPa D. 98.10 kPa
Solution:
( )( ) ( )

41. An ideal gas at 0.80 atmospheres and 87°C occupies 0.450 liter. How many moles are in the
sample? (R = 0.0821 liter-atm/mole-K).
A. 0.0002 mole B. 0.0378 mole C. 0.0122 mole D. 0.0091 mole
Solution:

( ) ( )( )

9
42. A certain gas at 101.325 kPa and 10°C whose volume is 2.83 m3 capacity. Before admission,
the storage vessel contained gas at a pressure and temperature of 137.8 kPa and 26°C; after
admission, the pressure increased to 1171.8 kPa. What should be the final temperature of the
gas in the vessel in Kelvin?
A. 298.0 B. 319.8 C. 180.0 D. 314.2
Solution:

( )
( )

( )
( )

( )

43. A perfect gas has a value of R = 58.8 ft-lb/lb-°R and k = 1.26. If 20 Btu are added to 10 lbs of
this gas at constant volume when initial temperature is 90°F, find the final temperature.
A. 97°F B. 104°F C. 154°F D. 185°F
Solution:

( ) -
( )( )

44. Ammonia weighing 22 kg is confirmed inside a cylinder equipped with a piston has an initial
pressure of 413 kPa at 38°C. If 3200 kJ of heat is added to the ammonia until its final pressure
and temperature is 413 kPa and 100°C, respectively. What is the amount of work done by the
fluid in kJ?
A. 667 B. 304 C. 420 D. 502
Solution:

( )( )

10
( )( )

( )

45. A tank contains 90 ft3 of air at a pressure of 350 psig; if the air is cooled until its pressure
and temperature decreases at 200 psig and 70°F respectively, what is the decrease in internal
energy?
A. 6232.09 Btu B. -5552 Btu C. 5552 Btu D. -6232.09 Btu
Solution:

( )( )
( )

( )
( )( ) - -

46. A large mining company was provided with a 3 m3 of compressed air tank. Air pressure in
the tank drops from 700 kPa to 150 kPa while the temperature remains constant at 28°C. What
percentage has the mass of air in the tank been reduced?
A. 74.09 B. 72.45 C. 76.56 D. 78.57
Solution:

47. A 4 m3/hr pump delivers water to a pressure tank. At the start, the gage read 138 kPa until
it reads 276 kPa and then the pump was shut off. The volume of the tank is 180 liters. At 276
kPa, the water occupied 2/3 of the tank volume. Determine the volume of the water that can
be taken out until the gage reads 138 kPa.
A. 31.20 liters B. 34.59 liters C. 16.87 liters D. 29.50 liters
Solution:

( )

( ) ( )( )

( ) ( )

11
48. A refrigeration plant is rated at 15 tons capacity. How many pounds of air per hour will it
cool from 70°F to 90°F at constant temperature?
A. 50,000 lb/hr B. 37,500 lb/hr C. 52,000 lb/hr D. 45,000 lb/hr
Solution:
̇ ̇

( ) ̇( )( )

49. An air standard engine has a compression ratio of 18 and a cut-off ratio of 4. If the intake air
pressure and temperature are 100 kPa and 27°C, find the work in kJ per kg.
A. 2976 B. 2166 C. 1582 D. 2751
Solution:

( )

( ) ( )

( )

-
( )( )

( )

50. Determine the air-standard efficiency of an engine operating on the diesel cycle with a
clearance of 6% when the suction pressure is 99.7 kPa and the fuel is injected for 7% of the
stroke. Assume k = 1.4.
A. 62.11% B. 51.20% C. 73.58% D. 60.02%
Solution:

12
( )( ( )
)

51. Steam at 2 MPa and 250°C in a rigid cylinder is cooled until the quality is 30%. Find the heat
rejected from the cylinder.
@ 2 Mpa and 250°C: υ = 0.11144 m3/kg u = 2679.6 kJ/kg
3
@ 2 Mpa, (saturated): υf = 0.0011767 m /kg υg = 0.09963 m3/kg
uf = 906.44kJ/kg ug = 1693.8 kJ/kg
A. -423.23 kJ/kg B. -926.26 kJ/kg C. -1265.02 kJ/kg D. 1082.34 kJ/kg
Solution:

( )
-

52. At 1.3 MPa, a mixture of steam and water has an entropy of 3 kJ/kg-K, Find the enthalpy of
the mixture.
@ 1.3MPa: s f = 2.2515 kJ/kg-K s g = 6.4952 kJ/kg-K
hf = 814.93 kJ/kg hfg = 1972.7 kJ/kg
A. 1627.71 kJ/kg B. 1533.33 kJ/kg C. 1234.45 kJ/kg D. 1162.40 kJ/kg
Solution:

( )

( )

53. A mixture with 70% quality at 500 kPa is heated isothermally until its pressure is 300 kPa.
Find the heat added during the process.
@ 600 kPa: s f = 1.8607 kJ/kg-K s fg = 4.9606 kJ/kg-K
@ 300 kPa and 151.86°C: s = 7.0888 kJ/kg-K
A. 745.92 kJ/kg B. 535.16 kJ/kg C. 982.44 kJ/kg D. 765.34 kJ/kg
Solution:
( )

( )( )

13
54. A tank contains exactly one kilogram of water consisting of liquid and vapor in equilibrium
at 1 MPa. If the liquid contains one-third and the remaining is vapor of the volume of the tank,
what is the enthalpy of the contents of the tank?
@ 1MPa: υf = 0.0011273 m3/kg υfg = 0.19444 m3/kg
hf = 762.81 kJ/kg hfg = 2015.3 kJ/kg
A. 644.40 kJ/kg B. 774.40 kJ/kg C. 785.92 kJ/kg D. 435.29 kJ/kg
Solution:

( )

55. Water substance at 70 bar and 65°C enters a boiler tube of constant inside diameter of 25
mm. The water leaves the boiler tube at 50 bar and 700 K at velocity of 150 m/s. Calculate the
inlet velocity (m/s).
From steam tables:
@ 70 bar (7Mpa) and 65°C: υ1 = 0.001017 m3/kg
@ 50 bar (5Mpa) and 700 K (427°C): υ2 = 0.06081 m3/kg
A. 1.56 B. 2.51 C. 1.672 D. 3.230
Solution:
̇ ̇

( )

56. Water substance at 70 bar and 65°C enters a boiler tube of constant inside diameter of 35
mm. The water leaves the boiler tube at 50 bar and 700 K at velocity of 150 m/s. Calculate the
inlet volume flow (liters/s).
From steam tables:
@ 70 bar (7Mpa) and 65°C: υ1 = 0.001017 m3/kg
@ 50 bar (5Mpa) and 700 K (427°C): υ2 = 0.06081 m3/kg
A. 0.821 B. 1.261 C. 0.344 D. 1.609
Solution:
̇ ̇
14
( )

( ) ( ) -

57. Steam leaves an industrial boiler at 827.4 kPa and 171.6°C. A portion of the steam is passed
through a throttling calorimeter and is exhausted to the atmosphere when the calorimeter
pressure is 101.4 kPa. How much moisture is leaving the boiler container if the temperature of
the steam at the calorimeter is 115.6°C?
@ 827.4 kPa and 171.6°C: hf = 727.25 kJ/kg hfg = 2043.2 kJ/kg
@ 101.4 kPa and 115.6°C: h2 = 2707.6 kJ/kg
A. 3.78% B. 3.08% C. 4.56% D. 2.34%
Solution:

( )

58. A throttling calorimeter is connected to a desuperheated steam line supplying steam to the
auxiliary feed pump on a ship. The line pressure measures 2.5 MPa. The calorimeter pressure is
110 kPa at 150°C. Determine the entropy of the steam line.
@ 110 kPa and 150°C: h2 = 2775.6 kJ/kg
@ 2.5 MPa: hf = 962.11 kJ/kg hfg = 1841 kJ/kg
s f = 2.5547 kJ/kg-K s fg = 3.7028 kJ/kg-K
A. 6.8 kJ/kg-K B. 6.2 kJ/kg-K C. 6.6 kJ/kg-K D. 7.5 kJ/kg-K
Solution:

( )

( ) -

59. Atmospheric pressure boils at 212°F. At the vacuum pressure at 24 in Hg, the temperature is
142°F. Find the boiling temperature when the pressure is increased by 40 psia from
atmospheric.
15
A. 449.42°F B. 536.34°F C. 479.13°F D. 263.45°F
Solution:
- ( )

60. A certain coal has the following ultimate analysis:


C = 69% N2 = 5% H = 2.5% S =7%
Determine the amount of oxygen if the heating value of fuel is 26,961.45 kJ/kg.
A. 1.5% B. 2.5% C. 3.5% D. 4.5%
Solution:
( )
( ) ( ) ( )

61. A diesel engine consumed 945 liters of fuel per day at 35°C. If the fuel was purchased at
15.6°C and 30°API at P29.00/li. Determine the cost of fuel to operate the engine per day.
A. P5,677.50 B. P4,677.50 C. P48,088.90 D. P27,127.76
Solution:

[ ( )]

( )

62. A cylindrical tank 4 m long and 3 m diameter is used for oil storage. How many days can the
tank supply the engine having 27°API with fuel consumption of 60 kg/hr?
A. 17.53 B. 5.84 C. 12.84 D. 19.84
Solution:
( ) ( )

16
̇ ̇ ( )
̇

̇
( )

63. A logging firm in Isabella operates a Diesel Electric Plant to supply its electric energy
requirements. During a 2 hour period, the plant consumed 250 gallons of fuel at 80°F and
produced 2900 kW-hrs. Industrial fuel is used at 30°API and was purchased at P30/li at 60°F.
Determine the overall thermal efficiency of the plant.
A. 26.08% B. 34.23% C. 28.00% D. 18.46%
Solution:
( ) ( )

[ ( )]
( )

̇ ( )( ) ( )( )

-
( )

̇ ( )

64. The dry exhaust gas from the oil engine has the following gravimetric analysis:
CO2 = 21.6% O2 = 4.2% N = 74.2%
Specific heats at constant pressure for each component of the exhaust gas in kCal/kg-°C are:
CO2 = 0.203 O2 = 0.219 N = 0.248
Calculate the specific gravity if the molecular weight of air is 28.97 kg/kg-mol.
A. 0.981 B. 1.244 C. 1.055 D. 0.542
Solution:

17
∑ -
-

65. A bituminous coal has the following composition:


C = 71.5% H = 5.0% O = 7.0% N = 1.3% S = 3% Ash = 7.6%
W = 3.4%
Determine the theoretical weight of Nitrogen in lb/lb of coal.
A. 2.870 B. 7.526 C. 2.274 D. 6.233
Solution:
( ) ( )

( ) ( ) ( )

( )

( )

66. A gaseous fuel mixture has a molal analysis:


H2 = 14% CH4 = 3% CO = 27% O2 = 0.6% CO2 = 4.5% N2 = 50.9%
Determine the air-fuel ratio for complete combustion on molal basis.
A. 2.130 B. 3.230 C. 1.233 D. 1.130
Solution:

( )

67. A volumetric analysis of a gas mixture is a follows:


CO2: 12% O2: 4% N2: 80% CO: 4%
What percentage of CO 2 on a mass basis?
18
A. 17.55% B. 15.55% C. 12.73% D. 19.73%
Solution:

68. The following coal has the following ultimate analysis by weight:
C = 70.5% H2 = 4.5% O2 = 6.0% N2 = 1.0% S = 3.0% Ash = 11%
Moisture = 4%
A stocker fired boiler of 195,000 kg/hr steaming capacity uses this coal as fuel. Calculate volume
of air in m3/hr with air at 60°F and 14.7 air pressure if the boiler efficiency is 70% and FE = 1.10.
A. 234,019 m3/hr B. 215,830 m3/hr C. 213,830 m3/hr D. 264,830 m3/hr
Solution:
( ) ( )

( ) ( ) ( )

( ) ( ) ( ) ( )

( )
( ) ( ) ( )

̇ ( ) ̇ ( )
̇ ̇
( )
̇ ( )

̇ ( )
̇ ̇
̇ ( )( )
̇

19
69. 23.5 kg of steam per second at 5 MPa and 400°C is produced by a steam generator. The
feedwater enters the economizer at 145°C and leaves at 205°C. The steam leaves the boiler
drum with a quality of 98%. The unit consumes 3 kg of coal per second as received having value
of 25,102 kJ/kg. What would be the overall efficiency of the unit in percent?
Steam properties:
@ 5 MPa and 400°C: h = 3195.7 kJ/kg
@ 5 MPa: hf = 1154.23 kJ/kg hfg = 1640.1 kJ/kg
@ 205°C: hf = 875.04 kJ/kg
@ At 145°C: hf = 610.63 kJ/kg
A. 65.72 B. 80.67 C. 88.28 D. 78.82
Solution:
̇ ( ) ( )
̇ ( )

70. In a Rankine cycle steam enters the turbine at 2.5 MPa (enthalpies & entropies given) and
condenser of 50 kPa (properties given), what is the thermal efficiency of the cycle?
@ 2.5 MPa: hg = 2803.1 kJ/kg s g = 6.2575 kJ/kg-K
@ 50 kPa: s f = 1.0910 kJ/kg-K s fg = 6.5029 kJ/kg-K
hf = 340.49 kJ/kg hfg = 2305.4 kJ/kg
3
υf = 0.00103 m /kg
A. 25.55% B. 45.23% C. 34.23% D. 12.34%
Solution:

( )

( )

( ) ( )
( ) ( ) ( ) ( )

71. A thermal power plant generates 5 MW and the heat generated by fuel is 13,000 kJ/s. If the
thermal efficiency is 36.15%, find the power needed for the auxiliaries.
A. 310 kW B. 300 kW C. 400 kW D. 350 kW
Solution:

20
72. A superheat steam Rankine cycle has turbine inlet conditions of 17.5 MPa and 530°C
expands in a turbine to 0.007 MPa. The turbine and pump polytropic efficiencies are 0.85 and
0.75 respectively. Pressure losses between the pump and turbine inlet are 1.5 MPa. What
should be the pump work in kJ/kg?
A. 17.34 B. 27.32 C. 25.32 D. 47.33
Solution:

( ) ( )

73. In an open feedwater heater for a steam plant, saturated steam at 7 bar is mixed with
subcooled liquid at 7 bar and 25°C. Just enough steam is supplied to ensure that the mixed
steam leaving the heater will be saturated liquid at 7 bar when heater efficiency is 95%.
Calculate the mass flow rate of the subcooled liquid if steam flow rate is 0.865 kg/s.
Steam properties:
@ 7 bar, saturated vapor: hg = 2763.5 kJ/kg
@ 7 bar and 25°C: hf = 105.5 kJ/kg
@ 7 bar, saturated liquid: hf = 697.22 kJ/kg
A. 2.725 kg/s B. 3.356 kg/s C. 2.869 kg/s D. 3.948 kg/s
Solution:
̇ ( )
̇ ( )
̇ ( )
( )
̇

74. A steam condenser receives 10 kg/s of steam with an enthalpy of 2770 kJ/kg. Steam
condenses and leaves with an enthalpy of 160 kJ/kg. Cooling water passes through the
condenser with temperature increases from 13°C to 24°C. Calculate the water flow rate in kg/s.
A. 583 B. 567 C. 523 D. 528
Solution:

̇ ( ) ̇ ( )
( ) ̇ ( )( )
21
̇

75. Steam expands adiabatically in a turbine from 2000 kPa, 400°C to 400 kPa, 250°C. What is
the effectiveness of the process in percent assuming an atmospheric pressure of 18°C? Neglect
changes in kinetic and potential energy.
Steam properties:
@ 2000 kPa and 400°C: h = 3247.6 kJ/kg s = 7.1271kJ/kg-K
@ 400 kPa and 250°C: h = 2964.2 kJ/kg s = 7.3789 kJ/kg-K
A. 82 B. 84 C. 79.60 D. 79.46
Solution:

( ) ( )( )

76. A heat exchanger was installed purposely to cool 0.50 kg of gas per second. Molecular
weight is 32 and k = 1.32. The gas is cooled from 150°C to 80°C. Water is available at the rate of
0.30 kg/s and at a temperature of 15°C. Calculate the exit temperature of the water in °C.
A. 44.86 B. 42.86 C. 46.45 D. 40.34
Solution:
-
( )
-

̇ ( ) ̇ ( )
( )( ) ( )( )

77. A 350 mm x 450 mm steam engine running at 280 rpm has a clearance steam condition of 2
MPa and 230°C and exits at 0.1 MPa. The steam consumption is 2000 kg/hr and mechanical
efficiency is 85%. If indicated mean effective pressure is 600 kPa, determine brake thermal
efficiency.
@ 2 MPa and 230°C: h1 = 2849.6 kJ/kg s 1 = 6.4423 kJ/kg-K
@ 0.1 MPa: s f = 1.3026 kJ/kg-K hf = 417.46kJ/kg s fg = 6.0568 kJ/kg-K
hfg = 2258 kJ/kg hf2 = 417.46 kJ/kg
A. 23.34% B. 15.25% C. 14.16% D. 27.34%
Solution:

22
( ) [( ) ( ) ]( )( ) ( )( )( )

̇ ( ) ( )( )

78. A steam turbine receives 5,000 kg/hr of steam at 5 MPa and 400°C and velocity of 30 m/s. It
leaves the turbine at 0.006 MPa and 85% quality and velocity of 15 m/s. Radiation loss is 10,000
kJ/hr. Find the kW developed.
@5 MPa and 400°C: h1 = 3195.7 kJ/kg s 1 = 6.6459 kJ/kg-K
@ 0.006 Mpa: hf = 151.53 kJ/kg hfg = 2415.9 kJ/kg
A. 1273.29 B. 2173.29 C. 1373.60 D. 7231.29
Solution:
( )
̇ ̇ ( )( )
̇ ̇ ( )( )

̇ ̇ ̇ ̇ ̇ ̇
̇ ( ̇ ̇ ) ̇( ) ̇
( ) ( )

79. A steam turbine with 85% stage efficiency receives steam at 7 MPa and 550°C and exhausts
at 20 kPa. Determine the turbine work.
@ 7 MPa and 550°C: h1 = 3530.9 kJ/kg s 1 = 6.9486 kJ/kg-K
@ 20 kPa (0.020 MPa): s f = 0.8320 kJ/kg-K hf = 251.40 kJ/kg
s fg = 7.0766 kJ/kg-K hfg = 2358.3 kJ/kg
A. 1,117 kJ/kg B. 1,132 kJ/kg C. 1,123.34 kJ/kg D. 1,054.95 kJ/kg
Solution:

( )

( )

23
80. A steam turbine with 80% stage efficiency receives steam at 7 MPa and 550°C and exhausts
at 20 kPa. Determine the quality at exhaust.
@ 7 MPa and 550°C: h1 = 3530.9 kJ/kg s 1 = 6.9486 kJ/kg-K
@ 20 kPa (0.020 MPa): s f = 0.8320 kJ/kg-K hf = 251.40 kJ/kg
s fg = 7.0766 kJ/kg-K hfg = 2358.3 kJ/kg
A. 96.96% B. 76.34% C. 82.34% D. 91.69%
Solution:

( )

( )

( )

81. An 18,000 kW geothermal plant has a generator efficiency of 90% and 80% respectively. If
the quality after throttling is 20% and each well discharges 400,000 kg/hr, determine the
number of wells required to produce if the change of enthalpy at entrance and exit of turbine is
500 kJ/kg.
A. 4 wells B. 2 wells C. 6 wells D. 8 wells
Solution:
̇ ̇ ( )
( )
̇ ( )
̇
̇
̇

82. A liquid dominated geothermal plant with a single flash separator receives water at 204°C.
The separator pressure is 1.04 MPa. A direct contact condenser operates at 0.034 MPa. The
turbine has a polytropic efficiency of 0.75. For cycle output of 60 MW, what is the mass flow
rate of the well water in kg/s?
@ 204°C: hf = 870.51 kJ/kg
@ 1.04 MPa: hf = 770.38 kJ/kg hfg = 2009.2 kJ/kg hg = 2779.6 kJ/kg

24
s g = 6.5729 kJ/kg-K
@ 0.034 MPa: hf = 301.40 kJ/kg hfg = 2328.8 kJ/kg s f = 0.09793 kJ/kg-K
s fg = 6.7463 kJ/kg-K
A. 2,933 B. 2,100 C. 1,860 D. 2,444
Solution:

( )

( )
̇ ̇ ( )
̇ ( )( )
̇
( )

( )

̇ ̇
̇
̇

83. An engine-generator rated 9000 kVA at 80% power factor, 3 phase, 4160 V has an efficiency
of 90%. If the overall plant efficiency is 28%, what is the heat generated by the fuel?
A. 18,800 kW B. 28,800 kW C. 7,500 kW D. 25,714 kW
Solution:

84. The indicated thermal efficiency of a two stroke diesel engine is 60%. If friction power is
15% of heat generated, determine the brake thermal efficiency of the engine.
A. 43% B. 45% C. 36% D. 37%
Solution:

̇
25
̇
̇ ̇ ̇
̇
̇ ̇

85. A 305 mm x 457 mm four stroke single acting diesel engine is rated at 150 kW at 260 rpm.
Fuel consumption at rated load is 0.56 kg/kW-hr with a heating value of 43,912 kJ/kg. Calculate
the break thermal efficiency.
A. 10.53% B. 27.45% C. 14.64% D. 18.23%
Solution:
̇
-

( )

86. A waste heat recovery boiler produces 4.8 MPa (dry saturated) steam from 104°C
feedwater. The boiler receives energy from 7 kg/s of 954°C dry air. After passing through a
waste heat boiler, the temperature of the air has been reduced to 343°C. How much steam in
kg is produced per second? Note: @ 4.80 MPa dry saturated, h = 2796 kJ/kg.
A. 1.30 B. 0.92 C. 1.81 D. 3.43
Solution:

( )

̇ ( ) ̇ ( )
( )( ) ̇ ( )
̇

87. A diesel electric power plant supplies energy for Meralco. During a 24-hour period, the plant
consumed 240 gallons of fuel at 28°C and produced 3930 kW-hr. Industrial fuel used is 28°API
and was purchased at P30 per liter at 15.6°C. What is the cost of fuel to produce one kW-hr?
A. P6.87 B. P1.10 C. P41.07 D. P5.00
Solution:

( )
[ ( )]
( )

26
( )
-

88. In a gas unit, air enters the combustion chamber at 550 kPa, 227°C and 43 m/s. The
products of combustion leave the combustor at 511 kPa, 1004°C and 180 m/s. Liquid fuel enters
with a heating value of 43,000 kJ/kg. For fuel-air ratio of 0.0229, what is the combustor
efficiency in percent?
A. 70.38% B. 79.38% C. 75.38% D. 82.38%
Solution:
( )

( ) ( ) ( )

89. The specific speed of turbine is 85 rpm and running at 450 rpm. If the head is 20 m and
generator efficiency is 90%, what is the maximum power delivered by the generator?
A. 450.51 kW B. 354.52 kW C. 650.53 kW D. 835.57 kW
Solution:


( )

( )

90. In Francis turbine, the pressure gage leading to the turbine casing reads 380 kPa. The
velocity of water entering the turbine is 8 m/s. If net head of the turbine is 45 m, find the
distance from the center of the spiral casing to the tailrace.
A. 3.0 m B. 3.5 m C. 4.0 m D. 4.5 m
Solution:

( )
27
91. A turbine has a mechanical efficiency of 93%, volumetric efficiency of 95% and total
efficiency of 82%. If the effective head is 40 m, find the total head.
A. 48.72 m B. 40.72 m C. 36.22 m D. 34.72 m
Solution:

( )

( )

92. A Pelton type turbine has 25 m head friction loss of 4.5 m. The coefficient of friction head
loss (Morse) is 0.00093 and penstock length of 80 m. What is the penstock diameter?
A. 1,355.73 mm B. 3476.12 mm C. 6771.23 mm D. 1686.73 mm
Solution:

√ √ ( )( )

( )( )( )

93. In a 9,000 kW hydro-electric plant, the overall efficiency is 88% and the actual power
received by the costumer is 110,000 kW-hrs for that day. What is the secondary power that this
plant could deliver during the entire day?
A. 58,960 kW-hrs B. 80,080 kW-hrs C. 65,960 kW-hrs D. 70,960 kW-hrs
Solution:
( )( ) -
-

94. A Pelton type turbine was installed 30 m below the head gate of the penstock. The head
loss due to friction is 12 percent of the given elevation. The length of the penstock is 100 m and
the coefficient of friction is 0.00093. Determine the power output in kW. (Use Morse equation).
A. 22,273 B. 23,234 C. 32,345 D. 34,452
Solution:
( )

28
√ √ ( )( )

( )( )( )

( ) ( )
( )( )

95. Water flows steadily with a velocity of 3.05 m/s in a horizontal pipe having a diameter of
25.24 cm. At one section of the pipe, the temperature and pressure of the water are 21°C and
689.3 kPa respectively. At a distance of 304.8 m downstream, the pressure is 516.9 kPa. What is
the friction factor?
A. 0.134 B. 0.0050 C. 0.0307 D. 0.641
Solution:

( )( )
( )( )

96. A hydro-electric plant having 30 sq. km reservoir area and 100 m head is used to generate
power. The energy utilized by the consumers whose load is connected to the power plant
during a five-hour period is 13.5x106 kWh. The overall generation efficiency is 75%. Find the fall
in the height of water in the reservoir after the 5-hour period.
A. 5.13 m B. 1.32 m C. 3.21 m D. 2.20 m
Solution:

( ) ( )( )

( )

97. The gas density of chimney is 0.75 kg /m3 and air density of 1.15 kg /m3. Find the driving
pressure if the height of the chimney is 63.71 m.
A. 0.15 kPa B. 0.25 kPa C. 0.35 kPa D. 0.45 kPa
Solution:
29
( ) ( )( )

98. The actual velocity of gas entering in a chimney is 8 m/s. The gas temperature is 25°C with a
gas constant of 0.287 kJ/kg-K. Determine the gas pressure for a mass of a gas is 50,000 kg/hr
and chimney diameter of 1.39 m.
A. 95 kPa B. 98 kPa C. 101 kPa D. 92 kPa
Solution:
̇ ( ) ( )
̇ ̇
( ) ( )( )

99. A steam generator with economizer and air heater has an overall draft loss of 25.78 cm of
water. If the stack gases are at 177°C and if the atmosphere is at 101.3 kPa and 26°C, what
theoretical height of stack in meters is needed when no draft fan is used? Assume that the gas
constant for the flue gases is the same as that for air.
A. 611.10 B. 631.10 C. 651.10 D. 671.10
Solution:

( )

( )
( )
( )
( )

100. A foundation measures 12 ft x 14 ft x 16 ft. Find the number of sacks of cement needed for
a 1:2:4 mixture.
A. 302 B. 404 C. 356 D. 598
Solution:
( )

( )

101. A rectangular foundation cross-section has a bed plate dimension of 8 ft x 10 ft. The
uniform clearance on each side is 1 ft. The height of the foundation is 4.5 ft. If the weight of the

30
steel bar reinforcements needed is ½% of weight of foundation, find the weight of the steel
bars. Use concrete density of 2400 kg/m3.
A. 173.47 kg B. 183.47 kg C. 163.47 kg D. 153.47 kg
Solution:
( )( )
( )
( )
( )

102. A steam pipe having a surface temperature of 250°C passes through a room where the
temperature is 27°C. The outside diameter of the pipe is 100 mm and emissivity factor is 0.8.
Calculate the radiated heat loss for 3 m pipe length.
A. 1434.47 W B. 3746.35 W C. 2851.82 W D. 3546.45 W
Solution:
( )( )

( )
-
( )( )( )
-

103. Brine enters a circulating brine cooler at the rate of 60 m3/hr at -8°C and leaves at -18°C.
Specific heat of brine is 1.072 kJ/kg-K and a specific gravity of 1.12. Determine the tons of
refrigeration.
A. 53.5 TR B. 65.3 TR C. 33.5 TR D. 56.9 TR
Solution:
( )
( )
̇
̇ ̇ ( )(- )

104. A turbo-charged, 16 cylinder, Vee-type diesel engine has an air consumption of 3,000 kg/hr
per cylinder at rated load and speed. This air is drawn in through a filter by a centrifugal
compressor directly connected to the exhaust gas turbine. The temperature of the air from the

31
compressor is 135°C and a counter flow air cooler reduces the air temperature to 45°C before it
goes to the engine suction header. Cooling water enters the air cooler at 30°C and leaves at
40°C. Calculate the log mean temperature difference.
A. 47.23°C B. 87.82°C C. 43.34°C D. 65.24°C
Solution:

105. Water is flowing in a pipe with radius of 30 cm at a velocity of 5 m/s at the temperature in
the pipe. The density and viscosity of the water are as follows: density = 997.9 kg/s viscosity =
1.131 Pa-s. What is the Reynolds number for this situation?
A. 2647 B. 96.2 C. 3100 D. 1140
Solution:

( )

( )

106. Compute the amount of condensate formed during a 10 minute warm-up of 180 meter
pipe conveys the saturated steam with enthalpy of vaporization, h fg = 1947.8 kJ/kg. The
minimum external temperature of pipe is 2°C. The final temperature of pipe is 195°C. The
specific heat of pipe material is 0.6 kJ/kg-°C. The specific weight is 28 kg/m.
A. 240.69 kg B. 982.45 kg C. 299.64 kg D. 423.45 kg
Solution:
( )

( ) ( )
( ) ( )( )

107. The discharge pressure of an air compressor is 5 times the suction pressure. If volume flow
at suction is 0.1 m3/s, what is the suction pressure if compressor work is 19.57 kW (use n =
1.35).

32
A. 97 kPa B. 98 kPa C. 99 kPa D. 100 kPa
Solution:

̇ [( ) ]
( )
[ ]

108. The initial condition of air in an air compressor is 98 kPa and 27 °C and discharges air at
450 kPa. The bore and stroke are 355 mm and 381 mm, respectively with percent clearance of
8% running at 300 rpm. Find the volume of air at suction.
A. 541.62 m3/hr B. 551.62 m3/hr C. 561.62 m3/hr D. 571.62 m3/hr
Solution:

( ) ( )

( ) ( )( )
( )

109. An air compressor has a suction volume of 0.35 m3/s at 97 kPa and discharges at 650 kPa.
How much power is saved by the compressor if there are two stages?
A. 18.27 kW B. 16.54 kW C. 13.86 kW D. 11.58 kW
Solution:
( )( )
̇ [( ) ] [( ) ]

√ √ ( )
( )( )( )
̇ [( ) ] [( ) ]

110. A two stage air intercooler has an intercooler pressure of 4 kg/cm2. What is the discharge
pressure if suction pressure is 1 kg/cm2?
A. 3 kg/cm2 B. 9 kg/cm2 C. 12 kg/cm2 D. 16 kg/cm2
Solution:

33
111. A two-stage air compressor at 100 kPa and 22°C discharges to 750 kPa. If the intercooler
intake is 105°C, determine the value of n.
A. 1.400 B. 1.326 C. 1.345 D. 1.288
Solution:
√ ( )

( )

( )

112. A single acting compressor has a volumetric efficiency of 89%, operates at 500 rpm. It
takes in air at 100 kPa and 30°C and discharges it at 600 kPa. The air handled is 8 m 3/min
measured at discharged condition. If compression is isentropic, find the effective mean
pressure in kPa.
A. 233.34 B. 973.17 C. 198.34 D. 204.82
Solution:

( )

( )( )
̇ [( ) ] [( ) ]

113. A water-jacketed air compressor handles 0.343 m3/s of air entering at 96.5 kPa and 21°C
and leaving at 480 kPa and 132°C; 10.9 kg/hr of cooling water enters the jacket at 15°C and
leaves at 21°C. Determine the compressor brake power.
A. 23.163 kW B. 62.65 kW C. 34.44 kW D. 19.33 kW
Solution:
34
( )

( )

( )( )
̇ [( ) ] [( ) ]

̇ ̇ ( ) ( )( )
̇ ̇

114. A double suction centrifugal pump delivers 20 ft3/s of water at a head of 12 m and running
at 650 rpm. What is the specific speed of the pump?
A. 5014.12 rpm B. 6453.12 rpm C. 2770.73 rpm D. 9968.73 rpm
Solution:

115. Determine the number of stages needed for a centrifugal pump if it is used to deliver 400
gal/min of water and pump power of 15 hp. Each impeller develops a head of 30 ft.
A. 6 B. 4 C. 5 D. 7
Solution:
̇
( )

116. The suction pressure of a pump reads 3 in. of mercury vacuum and discharge pressure
reads 140 psi is used to deliver 120 gpm of water with specific volume of 0.0163 ft 3/lb.
Determine the pump work.
A. 4.6 kW B. 5.7 kW C. 7.4 kW D. 8.4 kW
Solution:
- -

35
-

( )( )

117. A submersible pump delivers 350 gpm of water to a height of 5 ft from the ground. The
pump where installed 150 ft below the ground level and a drawdown of 8 ft during the
operation. If water level is 25 ft above the pump, determine the pump power.
A. 7.13 kW B. 4.86 kW C. 7.24 kW D. 9.27 kW
Solution:
( )

̇ ( )( )

118. A vacuum pump is used to drain a flooded mine shaft of 20°C water. The pump pressure of
water at this temperature is 2.34 kPa. The pump is incapable of lifting the water higher than 16
m. What is the atmospheric pressure?
A. 159.30 B. 132.33 C. 196.22 D. 171.9
Solution:

( )

119. A submersible, multi-stage, centrifugal deep well pump 260 gpm capacity is installed in a
well 27 ft below the static water level and running at 3000 rpm. Drawdown when pumping at
rated capacity is 10 feet. The pump delivers the water into a 25,000 gallons capacity overhead
storage tank. Total discharge head developed by pump, including friction in piping is 243 ft.
Calculate the diameter of the impeller of this pump in inches if each impeller diameter
developed a head of 38 ft.
A. 3.28 B. 5.33 C. 3.71 D. 6.34
36
Solution:


( ) √ ( )( )

120. A fan draws 1.42 m3 per second of air at a static pressure of 2.54 cm of water through a
duct 300 mm diameter and discharges it through a duct of 275 mm diameter. Determine the
static fan efficiency if total fan is 75% and air is measured at 25°C and 760 mm Hg.
A. 50.11% B. 53.69% C. 65.67% D. 45.34%
Solution:

( )
( )

( )

( )

( )

( )( )

( )( )

121. A water cooler uses 50 lb/hr of melting ice to cool running water from 80°F to 42°F. Based
on the inside coil area, Ui = 110 Btu/hr-ft2-°F. Find the gpm of water cooled.
A. 0.10 GPM B. 0.21 GPM C. 0.38 GPM D. 0.45 GPM
Solution:
̇ ̇ ̇ ( )
( ) ̇ ( )( )
̇
( )

37
122. The charge in a Diesel engine consists of 18.34 grams of fuel, with lower heating value of
42,571 kJ/kg, and 409 grams of fuel and products of combustion. At the beginning of
compression, T1 = 60°C. Let rk = 14. For constant c p = 1.11 kJ/kg-°C, what should be the cut-off
ratio in the corresponding ideal cycle?
A. 2.05 B. 2.34 C. 5.34 D. 2.97
Solution:
( )

( )

( )
( )
( )( )

123. The gain of entropy during isothermal nonflow process of 5 lb of air at 60°F is 0.462 Btu/°R.
Find the V1/V2.
A. 3.85 B. 0.259 C. 1.0 D. 0.296
Solution:

( )

124. An auditorium seating 1500 people is to be maintained at 80°F dry bulb and 60°F wet bulb
temperature when outdoor air is at 91°F dry bulb and 75°F wet bulb. Solar heat load is 110,000
Btu/hr and supply air is at 60°F, determine the amount of supply air.
A. 93,229.17 lb/hr B. 83,229.17 lb/hr C. 73,229.17 lb/hr D. 63,229.17 lb/hr
Solution:

̇ ( )
̇ ̇ ( )
̇( )( )
38
̇

125. In a Brayton cycle that operates between temperature limits of 300 K and 1773 K with k =
1.4, determine the temperature at the end of compression (isentropic) for maximum work of
the cycle.
A. 780 K B. 690.5 K C. 730 K D. 350 K
Solution:

( )

126. A 35% solution leaves the absorber and 30% solution enters the absorber. The heat
removed from the absorber by cooling water is 547.6 Btu and ammonia is superheated by 10°.
Find the pound per pound of ammonia gas from the evaporating coils.
A. 11 B. 12 C. 13 D. 14
Solution:

127. A Carnot refrigeration system operates at T max/Tmin = 1.5. Find the kW per ton of
refrigeration.
A. 1.91 B. 2.15 C. 1.76 D. 1.55
Solution:
̇
( ) ( )

( )

128. Assume 8 ft3 of air at 100 psi and 100°F are compressed isothermally to a volume of 2 ft 3.
For each end state of the process, find the bulk modulus.
A. 400 and 100 psi B. 400 and 110 psi C. 400 and 120 psi D. 400 and 130 psi
Solution:
- ( )

( ) ( )

- ( )

39
- ( )

129. Predict the pressure of nitrogen gas at T = 200 K and υ = 0.00385 L/g and b = 0.00141 L/g; a
= 0.178 L2-kPa/g2. Use Van der Waals equation.
A. 15,331 kPa B. 14,331 kPa C. 13,331 kPa D. 12,331 kPa
Solution:
( )
( )

130. A Francis turbine is to be operated at a speed of 600 rpm and with discharge of 4 m3/s. If r1
= 0.6 m, β = 110°, and the blade height B is 10 cm, what should be the guide vane angle α1 for
nonseparating flow condition at the runner entrance?
A. 14.4° B. 15.4° C. 16.4° D. 17.4°
Solution:
( )

( )( )

( )( )

( )

131. The total head of fan is 187 m and has a static pressure of 210 mm of water gage, what is
the velocity of air flowing if the density of air is 1.15 kg/m3?
A. 6.85 m/s B. 3.45 m/s C. 4.39 m/s D. 9.28 m/s
Solution:
( )

( )

132. A fan delivers 5.7 m3/s at a static pressure of 5.08 cm of water when operating at a speed
of 400 rpm. The power input required is 2.963 kW. If 7.05 m3/s are desired in the same fan and
installation, find the pressure in cm of water.
A. 7.77 B. 17.14 C. 11.43 D. 5.08
Solution:

40
( )

( )

133. A rigid container is closed at one end and measures 8 in diameter by 12 in long. The
container is held vertical and is slowly moved downward until the pressure in the container is
17 psia. What will be the depth of the top of the container from the free water surface?
A. 42.36 in B. 59.29 in C. 63.69 in D. 69.82 in
Solution:

( )

134. An empty, open can is 30 cm high with a 15 cm diameter. The can, with the open end
down, is pushed under water with a density of 1000 kg/m3. Find the water level in the can
when the top of the can is 50 cm below the surface.
A. 17.20 cm B. 2.12 cm C. 4.20 cm D. 5.87 cm
Solution:
( )

( ) [ ( )]

41
135. A cylindrical pipe with water flowing downward at 0.03 m3/s having top diameter of 0.08,
bottom diameter of 0.04 m and height of 1.5 m. Find the pressure inside the pipe.
A. 154.63 kPa B. 197.93 kPa C. 252.44 kPa D. 243.92 kPa
Solution:

( )

( )

( )

( )
(- )

136. Determine the size of pipe which will deliver 8 liters of medium oil (ν = 6.10x10-6 m2/s)
assuming laminar flow conditions.
A. 622 mm B. 754 mm C. 950 mm D. 835 mm
Solution:

( -
)

137. The type of flow occupying in a 1 cm diameter pipe which water flows at a velocity of 2.50
m/s. Use ν = 1.13x10-6 m2/s for water.
A. turbulent B. constant C. laminar D. none of these
Solution:
( ) -

138. What force is exerted by a water jet, 60 mm in diameter, if it strikes a wall at the rate of 15
m/s?
42
A. 636.17 N B. 442.62 N C. 764.23 N D. 563.34 N
Solution:

[ ( ) ]( )
( )( )( )

139. A 300 mm diameter pipe discharges water at a rate of 200 li/s. Point 1 on the pipe has a
pressure of 260 kPa and 3.4 m below point 1 is point 2 with a pressure of 300 kPa. Compute the
head loss between points 1 and 2.
A. 4.29 m B. 2.59 m C. 6.32 m D. 1.87 m
Solution:

( )

140. Water flowing at the rate of 10 m/s from an orifice at bottom of a reservoir. Find the
pressure at the bottom of reservoir.
A. 30 kPa B. 40 kPa C. 50 kPa D. 60 kPa
Solution:

( )
( )

141. Steam flows through a nozzle at 400°C and 1 MPa (h = 3263.9 kJ/kg) with velocity of 300
m/s. Find the stagnation enthalpy.
A. 3300 kJ/kg B. 3290 kJ/kg C. 3320 kJ/kg D. 3309 kJ/kg
Solution:

142. Air flows through a nozzle at a speed of 350 m/s. Find the stagnation temperature if the
entrance temperature is 200°C.
A. 241.25°C B. 251.25°C C. 261.25°C D. 271.25°C
Solution:
( ) ( )

43
143. Carbon dioxide flows through a nozzle with a speed of 400 m/s. Compute the dynamic
temperature.
A. 92.56 K B. 94.56 K C. 96.56 K D. 98.56 K
Solution:
-

( )

144. Carbon dioxide flows through a nozzle with speed of 380 m/s. The entrance condition of
nozzle is 250°C and 1200 kPa. Find the stagnation pressure.
A. 2,136.34 kPa B. 2,146.34 kPa C. 2,156.34 kPa D. 2,166.34 kPa
Solution:

( )

( )

145. Air enters a diffuser with a velocity of 200 m/s. Determine the velocity of sound if air
temperature is 30°C.
A. 349 m/s B. 359 m/s C. 369 m/s D. 379 m/s
Solution:
√ √ ( )( )( )

146. Air flows through a nozzle with temperature of entrance of 420 K stagnation temperature
of 468 K. Find the Mach number.
A. 0.744 B. 0.754 C. 0.764 D. 0.774
Solution:

√ √ ( )( )( )
44
147. Air at 300 K and 200 kPa is heated at constant pressure to 600 K. Determine the change of
internal energy.
A. 245.58 kJ/kg B. 235.58 kJ/kg C. 225.58 kJ/kg D. 215.58 kJ/kg
Solution:
( ) ( )( )

148. An insulated rigid tank initially contains 1.5 lb of helium at 80°F and 50 psia. A paddle
wheel with power rating of 0.02 hp is operated within the tank for 30 min. Determine the final
temperature.
A. 159.22°F B. 169.22°F C. 179.22°F D. 189.22°F
Solution:
( )

( )( ) ( )( )

149. A 4 m2 asphalt pavement with emissivity of 0.85 has a surface temperature of 50°C. Find
the maximum rate of radiation that can be emitted from the surface.
A. 2,068 .32 watts B. 2,078.32 watts C. 2,088.32 watts D. 2,098.32 watts
Solution:
̇ ( -
) ( )( )

150. Air at 10°C and 80 kPa enters a diffuser of a jet engine steadily with a velocity of 200 m/s.
The inlet area of diffuser is 0.40 m2. Determine the mass flow rate of air.
A. 72.79 kg/s B. 74.79 kg/s C. 76.79 kg/s D. 78.79 kg/s
Solution:

( )
̇ ( )( )

151. Consider a refrigerator whose 40 watts light bulb remains on continuously as a result of a
malfunction of the switch. If the refrigerator has a COP of 1.3 and the cost of electricity is 8
cents per kW-hr, determine the increase in the energy consumption of the refrigerator and its
cost per year if the switch is not fixed.
A. P 49.59 B. P 47.59 C. P 45.59 D. P 43.59
45
Solution:

̇
̇
̇ ̇ ̇
̇
( )( )

152. A 75 hp motor that has an efficiency of 91% is worn out and is replaced by a high-efficiency
motor that has an efficiency of 95.4%. Determine the reduction in heat gain of the room due to
higher efficiency under full-load conditions.
A. 2.24 kW B. 2.44 kW C. 2.64 kW D. 2.84 kW
Solution:
( )
( )
̇

153. A household refrigerator that has a power input of 450 watts and a COP of 2.5 is to cool
five large watermelons, 10 kg each, to 8°C. If the watermelons are initially at 20°C, determine
how long it will take for the refrigerator to cool them. The watermelons can be treated as water
whose specific heat is 4.2 kJ/kg-K.
A. 2220 seconds B. 2230 seconds C. 2240 seconds D. 2250 seconds
Solution:

( )
( )( )

154. When a man returns to his wall-sealed house on a summer day, he finds that the house is
at 32°C. He returns on the air conditioner which cools the entire house to 20°C in 15 minutes. If
COP is 2.5, determine the power drawn by the air conditioner. Assume the entire mass within
the house is 800 kg of air for which c v = 0.72 kJ/kg-K, cp = 1.0 kJ/kg-K.
A. 1.072 kW B. 2.072 kW C. 3.072 kW D. 4.072 kW
Solution:
46
̇ ( ) ( )( )( )

155. A heat source at 800 K losses 2000 kJ of heat to a sink at 500 K. Determine the entropy
generated during this process.
A. 1.5 kJ/K B. 2.5 kJ/K C. -2.5 kJ/K D. 4 kJ/K
Solution:
- -
-

156. Helium gas is compressed in an adiabatic compressor from an initial state of 14 psia and
50°F to a final temperature of 320°F in a reversible manner. Determine the exit pressure of
Helium.
A. 38.5 psia B. 40.5 psia C. 42.5 psia D. 44.5 psia
Solution:

( )

( )

157. Air passes thru a nozzle with efficiency of 90%. The velocity of air at the exit is 600 m/s.
Find the actual velocity at the exit.
A. 382 m/s B. 540 m/s C. 458 m/s D. 568 m/s
Solution:
( )

( )

158. A 50 kg block of iron casting at 500 K is thrown into a large lake that is at a temperature of
285 K. The iron block eventually reaches thermal equilibrium with the lake water. Assuming
average specific heat of 0.45 kJ/kg-K for the iron, determine the entropy generated during this
process.

47
A. -12.65 kJ/K B. 16.97 kJ/K C. 4.32 kJ/K D. 6.32 kJ/K
Solution:
( ) -
( )( )

159. A windmill with a 12 m diameter rotor is to be installed at a location where the wind is
blowing at an average velocity of 10 m/s. Using standard condition of air (1 atm, 25°C),
determine the maximum power that can be generated by the windmill.
A. 68 kW B. 70 kW C. 72 kW D. 74 kW
Solution:

( )

̇ ( )( )

̇ ( )

160. Consider a large furnace that can supply heat at a temperature of 2000°R at a steady rate
of 3000 Btu/s. Determine the exergy of this energy. Assume an environment temperature of
77°F.
A. 2305.19 kW B. 2315.19 kW C. 2325.19 kW D. 2335.19 kW
Solution:
( )

̇ ̇ ( )

161. A heat engine receives heat from a source at 1200 K at a rate of 500 kJ/s and rejects the
waste heat to a medium at 300 K. The power output of the engine is 180 kW. Determine the
irreversibility rate for this process.
A. 190 kW B. 195 kW C. 200 kW D. 205 kW
Solution:

̇ ̇
̇

48
162. A dealer advertises that he has just received a shipment of electric resistance heaters for
residential buildings that have an efficiency of 100 percent. Assuming an indoor temperature of
21°C and outdoor temperature of 10°C, determine the second law efficiency of these heaters.
A. 8.74% B. 6.74% C. 3.74% D. 4.74%
Solution:

163. A thermal power plant has a heat rate of 11,363 Btu/kW-hr. Find the thermal efficiency of
the plant.
A. 34% B. 24% C. 26% D. 30%
Solution:

164. A rigid tank contains 2 kmol of N2 and 6 kmol of CO 2 gases at 300 K and 115 MPa. Find the
tank volume using the ideal gas equation.
A. 7.33 m3 B. 5.33 m3 C. 3.33 m3 D. 1.33 m3
Solution:

( )( )( )

165. A spherical balloon with a diameter of 6 m is filled with helium at 20°C and 200 kPa.
Determine the mole number.
A. 9.28 kmol B. 10.28 kmol C. 11.28 kmol D. 13.28 kmol
Solution:

[ ( ) ] ( )( )

166. The air in an automobile tire with a volume of 0.53 ft3 is at 90°F and 20 psig. Determine the
amount of air that must be added to raise the pressure to the recommended value of 30 psig.
Assume the atmospheric pressure to be 14.7 psia and the temperature and the volume to
remain constant.
A. 0.026 lb B. 0.046 lb C. 0.066 lb D. 0.086 lb
49
Solution:

( )( )( ) ( )( )

( )( )( ) ( )( )

167. A rigid tank contains 20 lbm of air at 20 psia and 70°F. More air is added to the tank until
the pressure and temperature rise to 35 psia and 90°F, respectively. Determine the amount of
air added to the tank.
A. 11.73 lb B. 13.73 lb C. 15.73 lb D. 17.73 lb
Solution:

( )( )

( ) ( )( )

168. A rigid tank contains 5 kg of an ideal gas at 4 atm and 40°C. Now a valve is opened, and
half of mass of the gas is allowed to escape. If the final pressure in the tank is 1.5 atm, the final
temperature in the tank is?
A. -38°C B. -30°C C. 40°C D. 53°C
Solution:

( )( )

( ) ( )
- -

169. The pressure of an automobile tire is measured to be 200 kPa (gage) before the trip end
220 kPa (gage) after the trip at a location where the atmospheric pressure is 90 kPa. If the
temperature of the air in the tire before the trip is 25°C, the air temperature after the trip is?

50
A. 45.6°C B. 54.8°C C. 27.5°C D. 26.7°C
Solution:

170. Water is boiling at 1 atm pressure in a stainless steel pan on an electric range. It is
observed that 2 kg of liquid water evaporates in 30 min. The rate of heat transfer to the water
is?
A. 2.07 kW B. 0.47 kW C. 2.51 kW D. 3.12 kW
Solution:
̇ ̇ ( )
( )

171. Consider a person standing in a breezy room at 20°C. Determine the total rate of heat
transfer from this person if the exposed surface area and the average outer surface
temperature of the person are 1.6 m2 and 29°C, respectively, and the convection heat transfer
coefficient is 6 W/m2 with emissivity factor of 0.95.
A. 86.40 watts B. 81.70 watts C. 198.1 watts D. 168.1 watts
Solution:
̇ ( ) ( )( )
̇ ( -
)( )[( ) ( ) ]
̇ ̇ ̇

172. Water is boiler in a pan on a stove at sea level. During 10 minutes of boiling, it is observed
that 200 grams of water has evaporated. Then the rate of heat transfer to the water is:
A. 0.84 kJ/min B. 45.1 kJ/min C. 41.8 kJ/min D. 53.5 kJ/min
Solution:
̇ ̇ ( )

173. An aluminum pan whose thermal conductivity is 237 W/m-°C has a flat bottom whose
diameter is 20 cm and thickness of 0.4 cm. Heat is transferred steadily to boiling water in the
pan through its bottom at a rate of 500 watts. If the inner surface of the bottom of the pan is
105°C, determine the temperature of the surface of the bottom of the pan.
A. 95.27°C B. 105.27°C C. 115.27°C D. 125.27°C
Solution:

51
( )
( )
̇
( )( )

174. For heat transfer purposes, a standing man can be modeled as a 30 cm diameter, 170 cm
long vertical cylinder with both the top and bottom surfaces insulated and with the side surface
at an average temperature of 34°C. For a convection heat transfer coefficient of 15 W/m 2-°C,
determine the rate of heat loss from this man by convection in an environment at 20°C.
A. 316.46 watts B. 326.46 watts C. 336.46 watts D. 346.46 watts
Solution:
̇ ( ) ( )( )

175. A 5 cm diameter spherical ball whose surface is maintained at a temperature of 70°C is


suspended in the middle of a room at 20°C. If the convection heat transfer coefficient is 15
W/m2-°C and the emissivity of the surface is 0.8, determine the total heat transfer from the
ball.
A. 23.56 watts B. 32.77 watts C. 9.22 watts D. 43.45 watts
Solution:
( )
̇ ( ) ( )( )
̇ ( -
)( )[( ) ( ) ]
̇ ̇ ̇

176. A frictionless piston-cylinder device and a rigid tank contain 1.2 kmol of an ideal gas at the
same temperature, pressure and volume. Now heat is transferred, and the temperature of both
systems is raised by 15°C. The amount of extra heat that must be supplied to the gas in the
cylinder that is maintained at constant pressure is?
A. 0 B. 50 kJ C. 100 kJ D. 150 kJ
Solution:
( ) ( )( )

177. A supply of 50 kg of chicken at 6°C contained in a box is to be frozen to -18°C in a freezer.


Determine the amount of heat that needs to be removed. The latent heat of the chicken is 247
kJ/kg, and its specific heat is 3.32 kJ/kg-°C above freezing and 1.77 kJ/kg-°C below freezing. The

52
container box is 1.5 kg, and the specific heat of the box material is 1.4 kJ/kg -°C. Also the
freezing temperature of chicken is -2.8°C.
A. 15,206.4 kJ B. 50.4 kJ C. 15,156 kJ D. 1,863 kJ
Solution:
[ ( ) (- )]
( )( )

178. Water is being heated in a closed pan on top of a range while being stirred by a paddle
wheel. During the process, 30 kJ of heat is transferred to the water, and 5 kJ of heat is lost to
the surrounding air. The paddle wheel work amounts to 500 N-m. Determine the final energy of
the system if its initial energy is 10 kJ.
A. 35.5 kJ B. 40.5 kJ C. 25.5 kJ D. 14.5 kJ
Solution:

179. A classroom that normally contains 40 people is to be air-conditioned with window air-
conditioning units of 5 kW cooling capacity. A person at rest may be assumed to dissipate heat
at a rate about 360 kJ/hr. There are 10 light bulbs in the room, each with a rating of 100 watts.
The rate of heat transfer to the classroom through the walls and the windows is estimated to
be 15,000 kJ/hr. If the room is to be maintained at a constant temperature of 21°C, determine
the number of window air-conditioning units required.
A. 1 units B. 2 units C. 3 units D. 4 units
Solution:
̇ ( )( ) ( )

180. A 4 m x 5 m x 6 m room is to be heated by a baseboard resistance heater. It is desired that


the resistance heater be able to raise the air temperature in the room from 7 to 23°C within 15
minutes. Assuming no heat losses from the room and an atmospheric pressure of 100 kPa,
determine the required power of the resistance heater. Assume constant specific heats at room
temperature.
A. 2.34 kW B. 1.91 kW C. 4.56 kW D. 6.34 kW
Solution:

( )
( )
53
( ) ( )( )
( )

181. A student living in a 4 m x 6 m x 6 m dormitory room turns on her 150 watts fan before she
leaves the room on a summer day, hoping that the room will be cooler when she comes back in
the evening. Assuming all the doors and windows are tightly closed and disregarding any heat
transfer through the walls and the windows, determine the temperature in the room when she
comes back 10 hours later. Use specific heat values at room temperature, and assume the room
to be at 100 kPa and 15°C in the morning when she leaves.
A. 28.13°C B. 38.13°C C. 48.13°C D. 58.13°C
Solution:

( )
( )
( )
( ) ( )( )

182. A piston-cylinder device whose piston is resting on top of a set of stops initially contains
0.50 kg of helium gas at 100 kPa and 25°C. The mass of the piston is such that 500 kPa of
pressure is required to raise it. How much heat must be transferred to the helium before the
piston starts rising?
A. 1557.13 kJ B. 1657.13 kJ C. 1757.13 kJ D. 1857.13 kJ
Solution:

-
( )( )

( ) ( )( )

183. In order to cool 1 ton (1000 kg) of water at 20°C in an insulated tank, a person pours 80 kg
of ice at -5°C into the water. Determine the final equilibrium temperature in the tank. The
melting temperature and the heat of fusion of ice at atmospheric pressure are 0°C and 333.7
kJ/kg, respectively.
A. 12.43°C B. 14.43°C C. 16.43°C D. 18.43°C
Solution:

54
( )( ) ( )( ) ( ) ( )( )

184. A fan is powered by 0.5 hp motor and delivers air at a rate of 85 m3/min. Determine the
highest value for the average velocity of air mobilized by the fan. Take the density of air to be
1.18 kg/m3.
A. 18.23 m/s B. 21.12 m/s C. 25.34 m/s D. 32.23 m/s
Solution:

( ) ( )

√ ( )( )

185. An Ocean-Thermal Energy Conversion power plant generates 10,000 kW using a warm
surface water inlet temperature of 26°C and cold deep-water temperature of 15°C. On the basis
of a 3°C drop in the temperature of the warm water and a 3°C rise in the temperature of the
cold water due to removal and addition of heat, calculate the power required in kW to pump
the cold-deep water to the surface and through the system heat exchanger if the required
pumping pressure increase is 12 kPa. Assume a Carnot cycle efficiency and density of cold water
to be 100 kg/m3.
A. 108 B. 250 C. 146 D. 160
Solution:

̇
̇

̇
̇
̇ ̇ ̇
̇ ̇
̇( )( )
̇

( )( )

186. A plate-type solar energy collector with an absorbing surface covered by a glass plate is to
receive an incident radiation of 800 W/m2. The glass plate has a reflectivity of 0.12 and a
55
transmissivity of 0.85. The absorbing surface has an absorptivity of 0.90. The area of the
collector is 5 m2. How much solar energy in watts is absorbed by the collector?
A. 2500 B. 2880 C. 3510 D. 3060
Solution:
̇ ( )( )( )

187. A tank contains liquid nitrogen at -190°C is suspended in a vacuum shell by three stainless
steel rods of 0.80 cm in diameter and 3 meters long with a thermal conductivity of 16.3 W/m 2-
°C. If the ambient air outside the vacuum shell is 15°C, calculate the magnitude of the
conductive heat flow in watts along the support rods.
A. 0.168 B. 0.0587 C. 0.182 D. 0.176
Solution:
̇ ( )[ (- )]

188. An elastic sphere containing gas at 120 kPa has a diameter of 1.0 m. Heating the sphere
causes it to expand to a diameter of 1.3 m. During the process the pressure is proportional to
the sphere diameter. Calculate the work done by the gas in kJ.
A. 41.8 B. 50.6 C. 87.5 D. 35.4
Solution:

( )

( )

∫ ∫ ( ) ∫

189. An ideal gas with a molecular weight of 7.1 kg/kg mol is compressed from 600 kPa and 280
K to a final specific volume of 0.5 m3/kg. During the process the pressure varies according to P =
620 + 150υ + 95 υ2 where p is in kPa and υ in m3/kg. Calculate the work of compression in kJ/kg.
A. 32.8 B. 28.7 C. 35.6 D. 33.3
Solution:
( )

56
∫ ∫ ( )

190. A one cubic meter container contains a mixture of gases composed of 0.02 kg-mol of
oxygen and 0.04 kg-mol of helium at a pressure of 220 kPa. What is the temperature of this
ideal gas mixture in Kelvin?
A. 441 B. 350 C. 400 D. 450
Solution:
( ) ( )

191. Methyl alcohol (CH3OH) is burned with 25% excess air. How much unburned oxygen in kg-
moloxygen/kg-molfuel will there be in the products if the combustion is complete?
A. 0.35 B. 0.45 C. 0.37 D. 0.65
Solution:

( ) ( )

( ) ( )( )
( )( ) ( )
( )

192. A 12 DC electrical motor draws a current of 15 amps. How much work in kJ does this motor
produce over a 10-minute period of operation?
A. 108.0 B. 129.6 C. 216.0 D. 318.2
Solution:
( )( )

193. A 4 liter (2-liter per revolution at standard pressure and temperature) spark ignition engine
has a compression ratio of 8 and 2200 kJ/kg heat addition by the fluid combustion. Considering
a cold air-standard Otto cycle model, how much power will the engine produce when operating
at 2500 rpm?
A. 166.53 hp B. 73.12 hp C. 97.4 hp D. 148 hp
Solution:
( )
̇

57
( )
̇ ( )

194. A simple Rankine cycle produces 40 MW of power, 50 MW of process heated and rejects
50 MW of heat to the surroundings. What is the utilization factor of this cogeneration cycle
neglecting the pump work?
A. 50% B. 60% C. 64% D. 80%
Solution:
̇ ̇ ̇ ̇
̇ ̇
̇

195. The rate of heat transfer to the surroundings from a person at rest is 400 kJ/hr. Suppose
that the ventilation system fails in an auditorium containing 120 people and assuming that the
energy goes into the air of volume 1500 m3 initially at 300 K and 101 kPa, calculate the rate in
°C /min of air temperature change.
A. 0.81 B. 0.53 C. 0.63 D. 1.0
Solution:
̇ ̇
̇ ̇
( ) ̇( )( )
̇
( ) ( )

196. An insulated box containing helium gas falls from a balloon 4.5 km above the earth’s
surface. Calculate the temperature rise in °C of the helium when box hits the ground.
A. 15.2 B. 12.6 C. 25.3 D. 14.1
Solution:
-

( )( ) ( )

197. Consider two Carnot heat engines operating in series. The first engine receives heat from
the reservoir at 2400 K and rejects the waste heat to another reservoir at temperature T. The
second engine receives heat by the first one, convert some of it to work, and rejects the rest to
58
a reservoir at 300 K. If thermal efficiencies of both engines are the same, determine the
temperature T.
A. 849 K B. 578 K C. 763 K D. 978 K
Solution:

198. An ideal gas mixture consists of 2 kmol of N2 and 6 kmol of CO 2. The mass fraction of CO 2
is?
A. 0.175 B. 0.250 C. 0.825 D. 0.750
Solution:
( )
( ) ( )

199. An ideal gas mixture consists of 2 kmol of N2 and 6 kmol of CO 2. The apparent gas constant
of mixture is?
A. 0.208 B. 0.231 C. 0.531 D. 0.825
Solution:
( ) ( )
-

200. A Carnot cycle operates between the temperature limits of 300 K and 1500 K, and
produces 600 kW of net power. The rate of entropy change of the working fluid during heat
addition process is?
A. 0 B. 0.4 C. 0.5 D. 2.0
Solution:
̇ ( )
( )

201. Air in an ideal Diesel cycle is compressed from 3 L to 0.15 L and then it expands during the
constant pressure heat addition process to 0.3 L. Under cold air standard conditions, the
thermal effieciency of this cycle is?
A. 35% B. 44% C. 65% D. 70%
Solution:

59
[ ( )
] [ ( )
]

202. Helium gas in an ideal Otto cycle is compressed from 20°C and 2 L to 0.25 L and its
temperature increases by an additional 800°C during the heat addition process. The
temperature of Helium before the expansion process is?
A. 1700°C B. 1440°C C. 1240°C D. 880°C
Solution:

( )
( )( )

203. In an ideal Otto cycle, air is compressed from 1.20 kg/m3 and 2.2 L to 0.26 L and the net
work output of the cycle is 440 kJ/kg. The mean effective pressure for the cycle is?
A. 612 kPa B. 599 kPa C. 528 kPa D. 416 kPa
Solution:
- - -

-
( )

204. An ideal Brayton cycle has a net work output of 150 kJ/kg and backwork ratio of 0.4. if
both the turbine and the compressor had an isentropic efficiency of 80%, the net work output
of the cycle would be?
A. 60 kJ/kg B. 75 kJ/kg C. 98 kJ/kg D. 120 kJ/kg
Solution:

60
( )
( )

205. Air enters a turbojet engine at 200 m/s at a rate of 20 kg/s and exits at 800 m/s relative to
the aircraft. The thrust developed by the engine is?
A. 6 kN B. 12 kN C. 16 kN D. 20 kN
Solution:
( ) ( )

206. A thermal power has a net power of 10 MW. The backwork ratio of the plant is 0.005.
Determine the compressor work.
A. 50.15 kW B. 50.35 kW C. 50.25 kW D. 50.45 kW
Solution:
̇ ̇ ̇
̇
̇
̇
̇
̇ ̇
̇ ̇ ̇
̇ ̇
̇
̇ ( )

207. A heat engine receives heat from a source at 1200 K at a rate of 500 kJ/s and rejects the
wasted heat to a sink at 300 K. If the power output of the engine is 200 kW, the second law
efficiency of this heat engine is?
A. 35% B. 40% C. 53% D. 75%
Solution:

61
208. A water reservoir contains 100,000 kg of water at an average elevation of 60 m. The
maximum amount of electric power that can be generated from this water is ?
A. 8 kWh B. 16 kWh C. 1630 kWh D. 58,800 kWh
Solution:
( )

209. A house is maintained at 22°C in winter by electric resistance heaters. If the outdoor
temperature is 5°C, the second law efficiency of the resistance heaters is ?
A. 0% B. 5.8% C. 34% D. 77%
Solution:

210. A thermoelectric refrigerator that resembles a small ice chest is powered by a car battery,
and has a COP of 0.10. If the refrigerator cools a 0.350 L canned drink from 20°C to 4°C in 30
min, determine the average electric power consumed by the thermoelectric refrigerator.
A. 130 watts B. 110 watts C. 120 watts D. 140 watts
Solution:
̇ ̇ ( ) ( )( )

211. A Carnot refrigerator operates in a room in which the temperature is 25°C and consumes 2
kW of power when operating. If the food compartment of the refrigerator is to be maintained
at 3°C, determine the rate of heat removal from the food compartment.
A. 1504.8 kJ/min B. 12.86 kJ/min C. 1625 kJ/min D. 9.57 kJ/min
Solution:

( ) ( )
̇ ̇ ( )

212. A household refrigerator with EER 8.0 removes heat from the refrigerated space at a rate
of 90 kJ/min. Determine the rate of heat transfer to the kitchen air.
A. 101.25 kJ/min B. 63.05 kJ/min C. 128.46 kJ/min D. 80 kJ/min
Solution:

62
̇
̇ ̇
̇

213. An air-conditioning system is used to maintain a house at 75°F when the temperature
outside is 95°F. The house is gaining the heat through the walls and windows at a rate of 1250
Btu/min, and the heat generation rate within the house from people, lights, and appliances
amounts to 350 Btu/min. Determine the minimum power input required for this air-
conditioning system.
A. 10.06 hp B. 1.36 hp C. 1.41 hp D. 7.94 hp
Solution:
̇

( ) ( )

̇
̇

214. A refrigeration system is to cool bread loaves with an average mass of 450 g from 22°C to -
10°C at a rate of 500 loaves per hour by refrigerated air. Taking the average specific and latent
heats of bread to be 2.93 kJ/kg-°C and 109.3 kJ/kg, respectively, determine the product load.
A. 541.7 kJ/min B. 351.6 kJ/min C. 761.5 kJ/min D. 409.9 kJ/min
Solution:
̇ ( )
̇ ̇ ̇ ( ̇ ) ( ̇ )
( )[ (- )] ( )

215. A house that was heated by electric resistance heaters consumed 1200 kWh of electric
energy in a winter month. If this house were heated instead by a heat pump that has an
average performance factor, PF of 2.4, determine how much money the homeowner would be
saved that month. Assume a price of 0.085 $/kWh for electricity.
A. $42.5 B. $59.50 C. $102 D. $97.75
Solution:
̇
̇
( )( )

63
216. An ammonia simple saturation cycle operates with a suction pressure of 291.6 kPa and a
condenser pressure of 1204 kPa develops 15 tons of refrigeration. Determine the theoretical
horsepower of the compressor. The following enthalpies have been found: condenser entrance
= 1653 kJ/kg, exit = 346.6 kJ/kg; compressor entrance = 1450.2 kJ/kg, exit = 1653 kJ/kg.
A. 7.23 hp B. 13 hp C. 15 hp D. 8.23 hp
Solution:
̇
̇
( )
̇ ̇( )

217. An ammonia ice plant operates between a condenser temperature of 35°C and evaporator
of -15°C. It produces 10 metric tons of ice per day from water at 30°C to ice at -5°C. Assuming
simple saturation cycle, determine the horsepower of the motor if the adiabatic efficiency of
the compressor nc = 0.85 and mechanical efficiency nm = 0.95. The specific heat of ice is 2.094
kJ/kg-°C and the latent heat is 335 kJ/kg. From the table for ammonia the following enthalpies
are: condenser entrance = 1703 kJ/kg, exit = 366.1 kJ/kg; compressor entrance = 1443.9 kJ/kg,
exit = 1703 kJ/kg.
A. 17.68 hp B. 18.61 hp C. 15.5 hp D. 21.9 hp
Solution:
( ) ( )
( ) [ (- )]
( )
̇
̇
̇
( )
̇ ̇( )
̇
( )

218. A Freon 22 air conditioning under standard operating conditions of 35°C condensing and
5°C evaporating temperatures. The volume flow rate entering the compressor is 23.72 L/s.
Determine the refrigerating capacityif the refrigerating effect is 164 kJ/kg. From the table for
R22 the specific volume at the compressor entrance is 40.46 L/kg.
A. 339.3 TR B. 79.3 TR C. 96.4 TR D. 27.4 TR
Solution:

( )

64
219. The refrigerant volume flow rate at the entrance of compressor were obtained from a test
on a twin cylinder, single acting 15 cm x 20 cm, 320 rpm compressor ammonia refrigerating
plant to be 33 L/s. Determine the volumetric efficiency of the compressor.
A. 77.65% B. 87.6% C. 97.6% D. 65.65%
Solution:
( ) ( ) ( )( )( )

( )

220. A twin cylinder ammonia compressor with volume displacement of 14,726 cm3 operates at
300 rpm. Condenser and evaporator pressure are 1200 kPa and 227 kPa respectively. Specific
volume of refrigerant at the entrance of compressor is 528.26 L/kg. Compression process is
polytropic with n = 1.20 and clearance factor of compressor is 2 percent. Determine the
horsepower required.
A. 60 hp B. 70 hp C. 80 hp D. 90 hp
Solution:

( ) ( )

( ) ( )( )
( )
( )( )
̇ [( ) ] [( ) ]

( )

221. A reversed Carnot has a refrigerating COP of 2.5. Determine the ratio TH/TL.
A. 1.4 B. 1.5 C. 1.25 D. 1.2
Solution:

222. Three thousand cubic feet per minute of air are circulated over an air-cooled condenser. If
the load on the condenser is 64,800 Btu/hr, compute the temperature rise of the air passing
over the condenser. Specific volume of standard air (13.34 ft3/lb).
65
A. 10°F B. 15°F C. 20°F D. 25°F
Solution:

̇
( ) ( )
̇

223. Saturated vapor ammonia at -16°C (hT = 1442.60 kJ/kg) leaves the evaporator and enters
the compressor at -6°C (h1 = 1465 kJ/kg). The refrigerant leaves the condenser as saturated
liquid at 40°C (h4 = 390.6 kJ/kg) and enters the expansion valve at 35°C (h5 = 366.1 kJ/kg). Heat
rejected from the condenser amount to 50 kW. The work to compressor is 208 kJ/kg, while the
heat loss from compressor is 33 kJ/kg. If 95 kJ/kg of heat are lost in the piping between the
compressor discharge and condenser inlet, determine the refrigerating capacity of the system.
A. 49.5 TR B. 46.61 TR C. 12.88 TR D. 13.24 TR
Solution:

̇ ̇ ( )

224. In an actual refrigeration cycle using R12 as working fluid, the refrigerant flow rate is 0.05
kg/s. Vapor enters the expansion valve at 1.15 MPa, 40°C (h = 238.5 kJ/kg) and leaves the
evaporator at 175 kPa, -15°C (h = 345 kJ/kg). The electric iput to motor driving the compressor
is measured and found 3.0 kW. Motor efficiency at this load 92% and mechanical efficiency
82%. Determine the actual coefficient of performance for this cycle.
A. 1.58 B. 2.36 C. 1.78 D. 1.34
66
Solution:
̇ ( )
̇ ( )( )

225. In an ammonia refrigeration system, the temperature in the evaporator is -12°C and the
ammonia at the evaporator entry 0.1511 dry while at exit is 0.95 dry. If the rate of ammonia
circulation is 5.64 kg/min, determine the refrigerating capacity of the system. Enthalpy of
saturated liquid and vapor at -12°C is 144.929 kJ/kg and 1447.74 kJ/kg respectively.
A. 17.82 B. 34.82 C. 27.82 D. 4.82
Solution:
( ) ( )

( ) ( )

( )
̇ ̇( )

226. A two stage cascade vapor compression refrigeration system uses ammonia in the low-
temperature loop and R-12 in the high-temperature loop. The ammonia provides 15 tons of
cooling. If the high-loop temperature requires 10.12 kW compressor power and low loop 15.93
kW, determine the COP of the system.
A. 2.027 TR B. 5.22 TR C. 3.314 TR D. 9.1 TR
Solution:
̇
( )

227. When a man returns to his well-sealed house on a summer day, he finds that the house is
at 32°C. He turns on the air conditioner, which cools the entire house to 20°C in 15 min. If COP
of the air conditioner system is 2.5, determine the power drawn by the air conditioners.
Assume the entire mass within the house is equivalent to 800 kg of air for which c = 0.72 kJ/kg -
°C.
A. 7.08 kW B. 3.07 kW C. 19.2 kW D. 12.08 kW
Solution:
( ) ( )( )
̇
( )
̇
̇

67
228. It is desired to double the COP of a reversed Carnot engine for cooling from 5.0 by raising
the temperature of heat addition while keeping the temperature of heat rejection constant. By
what percentage must the temperature of heat addition be raised?
A. 10.1% B. 9.1% C. 8.1% D. 7.1%
Solution:

229. An ammonia water-cooled compressor receives the refrigerant at specific volume 62 L/kg.
It has a piston displacement rate of 5 m3/min. If a squirrel cage motor running at 1200 rpm
drives the compressor and average piston speed is 490 m/min, calculate size of cylinder bore.
A. 20.4 cm B. 26.0 cm C. 16.13 cm D. 13.6 cm
Solution:

( )

( )( )

230. If the initial volume of an ideal gas is compressed the one-half its original volume and to
twice its temperature, the pressure:
A. Doubles B. Quadruples C. Remains constant D. Halves
Solution:

68
( )

231. If the gage pressure of a medium is 30 kPa (vacuum) and the atmospheric pressure is 101.3
kPa, the absolute pressure will be?
A. 131.3 kPa B. -71.3 kPa C. 71.3 kPa D. -131.3 kPa
Solution:

232. If a particle has a velocity of 4 meters per second and a kinetic energy of 144 Joules, then
the mass, in kilograms of this particle must be?
A. 44 B. 16 C. 18 D. 24
Solution:

( )

233. A condenser vacuum gauge reads 715 mm Hg when the barometer stands at 757 mm Hg.
State the absolute pressure in the condenser in kN/m2 or kPa.
A. 5.6 kPa B. 5.9 kPa C. 6.5 kPa D. 5.2 kPa
Solution:
( )

234. Determine the force in newton in a piston of 465 mm2 area with a pressure of 0.172 MPa.
A. 65 N B. 72 N C. 80 N D. 111 N
Solution:
( )( )( )

235. One piston of a hydraulic press has an area of 1 cm2. The other piston has an area of 25
cm2. If a force of 150 N is applied on the smaller piston, what will be the total force on the
larger piston if both piston surfaces are on the same level?
A. 6 N B. 175 N C. 3750 N D. 4250 N
Solution:

69
236. The work done on air is 10.86 kJ/kg, determine the compressor power if it is receiving 272
kg/min of air.
A. 36.72 hp B. 49.23 hp C. 2954 hp D. 66 hp
Solution:
( ) ( )

237. A water tank 18 ft wide, 14 ft long, and 4 ft high, calculate the pressure at the bottom of
the tank.
A. 1.733 psi B. 1.999 psi C. 2.337 psi D. 3.773 psi
Solution:
( )( )

238. What is the pressure of 750 mm Hg in kN/m2?


A. 90 B. 100 C. 103 D. 110
Solution:
( )

239. A double purpose tank 18 ft wide, 24 ft long and 4 ft depth is filled with water. What is the
weight of water in the tank in long tons?
A. 49 tons B. 48 tons C. 54 tons D. 50 tons
Solution:
[ ( )( )] ( )

240. Oil flows through a 16 tube single cooler with a velocity of 2 m/s. The internal diameter of
the tube is 30 mm and oil density is 0.85 g m/ml. Find the volume flow in liters per sec.
A. 22.62 B. 32.22 C. 62.22 D. 42.62
Solution:
( ) ( )( )

241. A substance temperature was 620°R. What is the temperature in °C?


A. 50.7 B. 45.54 C. 71.11 D. 91.44
Solution:
70
[( ) ]( )

242. An unknown volume of container gas of 1 atmosphere is allowed to expand to another


container of 10 m3 volume at 500 mmHg at constant temperature. Find the unknown volume.
A. 6.58 m3 B. 6.75 m3 C. 5.67 m3 D. 7.65 m3
Solution:

( )

243. An iron block weighs 5 Newton and has a volume of 200 cm3. What is the density of the
block?
A. 2458 kg/m3 B. 2485 kg/m3 C. 2584 kg/m3 D. 2549 kg/m3
Solution:
( )
( )( )

244. If air is at a pressure of 22.22 psia and at a temperature of 800°R, what is the specific
volume?
A. 11.3 ft3/lbm B. 33.1 ft3/lbm C. 13.3 ft3/lbm D. 31.3 ft3/lbm
Solution:

( )
( )

245. The Specific gravity of mercury is 13.55. What is the specific weight of mercury?
A. 123.9 kN/m3 B. 139.2 kN/m3 C. 132.9 kN/m3 D. 193.2 kN/m3
Solution:
( )

246. The equivalent weight of mass 10 kg at a location where the acceleration of gravity is 9.77
m/s 2?
A. 97.7 N B. 79.7 N C. 77.9 N D. 977 N
Solution:
( )

71
247. A transportation company specializes in the shipment of pressurized gaseous materials. An
order is received from 100 liters of a particular gas at STP (32°F and 1 atm). What minimum
volume tank is necessary to transport the gas at 80°F and maximum pressure of 8 atm?
A. 16 liters B. 14 liters C. 10 liters D. 12 liters
Solution:

( ) ( )
( ) ( )

248. 100 g of water are mixed with 150 g of alcohol (density = 790 kg/m3). What is the specific
volume of the resulting mixtures, assuming that the fluids mixed completely?
A. 0.82 x 10-3 m3/kg B. 0.88 x 10-3 m3/kg C. 0.63 x 10-3 m3/kg D. 1.16 x 10-3 m3/kg
Solution:

249. How much does 30 lbm weigh on the moon? (g moon = 5.47 ft/s 2).
A. 2.0 lbf B. 3.2 lbf C. 3.4 lbf D. 5.096 lbf
Solution:
( )

250. A 10 kg block is raised vertically 3 meters. What is the change in potential energy?
A. 320 J B. 350 km-m2/s 2 C. 294 J D. 350 N-m
Solution:
( )( )

251. How many cubic meters is 100 gallons of liquid?


A. 3.7850 m3 B. 0.1638 m3 C. 0.3785 m3 D. 1.638 m3
Solution:
( )( )

252. Steam turbine is receiving 1014 lbm/hr of steam, determine the horsepower output of the
turbine if the work done by steam is 251 Btu/lbm.

72
A. 100 hp B. 462.7 hp C. 200 hp D. 600 hp
Solution:

̇ ( )( )

253. What is the resulting pressure when one pound of air at 15 psia and 200°F is heated at
constant volume to 800°F?
A. 32.1 psia B. 15 psia C. 28.6 psia D. 36.4 psia
Solution:

( )( )

254. A bicycle tire has a volume of 600 cm3. It is inflated with carbon dioxide to pressure of
551.43 kPa, 20°C. How many grams of CO 2 are contained in the tire? R CO2 = 0.18896 kJ/kg.K.
A. 5.98 g B. 6.43 g C. 4.63 g D. 3.83 g
Solution:
( )

( )

255. The absolute pressure at the bottom of a vertical column of water is 15.5 psia. What is the
height of this column?
A. 22 in B. 9.2 in C. 12 in D. 9.8 in
Solution:
( )( )

256. A water temperature rise of 18°F in the water cooled condenser is equivalent to what in
°C?
A. 7.78°C B. 10°C C. 263.56 K D. -9.44°C
Solution:
( )

257. An oil storage tank contains oil with specific gravity of 0.88 and depth of 20 meters. What
is the hydrostatic pressure at the bottom of the tank in kg/cm2?
A. 1.67 B. 1.76 C. 1.56 D. 1.87
Solution:

73
( )( ) ( )( )( )

258. A vertical column of water will be supported to what height by standard atmospheric
pressure?
A. 34 ft B. 36 ft C. 24 ft D. 26 ft
Solution:
( )

259. The specific weight of liquid is 60 lb/ft3. What is the equivalent to kN/m3?
A. 9.334 B. 9.249 C. 9.643 D. 9.420
Solution:
( )
( )( )[ ( )
]

260. A cylinder weighs 150 lbf. Its cross-sectional area is 40 square inches. When the cylinder
stands vertically on one end, what pressure does the cylinder exert on the floor?
A. 14.1 kPa B. 58.2 kPa C. 0.258 bar D. 0.141 bar
Solution:
( )( )

261. What is the absolute pressure exerted on the surface of a submarine cruising 300 ft below
the free surface of the sea? Assume specific gravity of sea water is 1.03.
A. 133.9 psia B. 148.6 psia C. 100.7 psia D. 103.7 psia
Solution:
( )( )

262. Air enters a nozzle steadily at 2.21 kg/m3 and 30 m/s. What is the mass flow rate through
the nozzle if the inlet area of the nozzle is 80 cm2?
A. 0.35 kg/s B. 3.5 kg/s C. 5.3 kg/s D. 0.53 kg/s
Solution:
( )( )( )

263. What is the work required to accelerate an 800-kg car from rest to 100 km/h on a level
road?
A. 308.6 kJ B. 806.3 kJ C. 608.3 kJ D. 386 kJ

74
Solution:
( )[ ( )( ] [ ( )]
)

264. Assuming that there are no heat effects and no friction effects, find the speed of a 3220-
lbm body after it falls 778 ft from rest.
A. 422 ft/s B. 424 ft/s C. 224 ft/s D. 424 ft/s
Solution:
[ ( )( )]

265. What is the flow rate through a pipe 4 inches in diameter carrying water at a velocity of 11
ft/s?
A. 430.84 gpm B. 7.18 gpm C. 340.28 gpm D. 39.16 gpm
Solution:
( ) ( ) ( )( )

266. If the specific weight of a liquid is 58.5 lbf per cubic foot, what is the specific volume of the
liquid in cm3/g?
A. 0.5321 cm3/g B. 0.6748 cm3/g C. 0.9504 cm3/g D. 1.0675 cm3/g
Solution:

( )
( ) ( ) [( ) ]

267. What is the resulting pressure when one pound of air at 0.3 psig and 200°F is heated at
constant volume to 800°F?
A. 0.572 psig B. 28.6 psia C. 7.857 psia D. 1.2 psig
Solution:

268. A small pump serving as model, when tested in laboratory using water at 3600 rpm,
delivered 30 cfs at a head of 125 ft. If the efficiency of this model pump is 84%, predict the
75
horsepower input to the prototype pump if it is to develop the same head as model pump and
the model pump has a scale ratio of 1:10. Assume the efficiency of the prototype pump is 90%.
A. 50.6 hp B. 4730 hp C. 3740 hp D. 60.5 hp
Solution:
( )( )
( )

( ) ( ) ( ) ( )

( ) ( )( )

( )( )
( )

269. Pump at is best efficiency point (BEP) has a capacity of 10,500 gpm while developing a
head of 60 ft at a rotative speed of 1450 rpm. What is the specific speed of the pump?
A. 2760 B. 1450 C. 2476 D. 6892
Solution:
( )( ) ( )
( )

270. A pump will be installed below the reservoir water surface with a required net positive
suction head (NPSHR) of 50 ft. The barometric pressure is 14.3 psia, and the vapor pressure is
0.5 psia. Assume friction losses in the intake piping are 5 ft. Find the maximum allowable
elevation of the pump relative to the water surface intake to avoid cavitation.
A. 45 ft B. 55 ft C. 18.2 ft D. 23.2 ft
Solution:
( )

( )

( )
( )
( )
( ) ( )
( ) ( ) ( )

271. A centrifugal pump is at best efficiency point (BEP). Assume the pump characteristic are
head, h = 7 m, flow rate, Q = 19 liters/s, and rotative speed n = 1170 rpm. Find the specific
speed in SI units.
76
A. 0.4 B. 0.71 C. 10.41 D. 3.94
Solution:
( )( )

( ) ( )
( )

( )
( ) ( )

272. The pressure of a confined gas at a constant temperature is tripled, what will happen to
the volume?
A. The volume will be tripled
B. The volume will be reduced to one-third of its original value
C. The volume will remain unchanged
D. The volume is constant
Solution:

273. A 15 in. diameter fan operates at 1600 rpm and develops a head of 6 in. of water and
delivers 120 cfm. What volumetric capacity for geometrically similar fan will develop 6 in of
water at 1300 rpm?
A. 147.7 cfm B. 181.8 cfm C. 97.5 cfm D. 79.2cfm
Solution:

( )

( ) ( )

( )

274. A radial-flow pump operating at maximum efficiency at a specific speed of 2500 is to


deliver 260 gpm against a head of 129 ft at a rotative speed of 2100 rpm. Find the required
number of stages (i.e., impellers).
A. 2 stages B. 3 stages C. 4 stages D. 5 stages
77
Solution:
( )

275. How many identical turbines, operating at 139.0 rpm and 91% efficiency (specific speed =
5.4), are needed to exploit a head of 1200 ft and a flow of 1660 ft 3/s?
A. 2 turbines B. 3 turbines C. 4 turbines D. 5 turbines
Solution:
( )

( )( )( )

( )

276. How many poles should a 60-Hz generator have, if it is connected to a turbine operating
under a design head of 3000 ft with a flow of 82 cfs? Assume turbine specific speed and
efficiency 3 and 84 percent respectively.
A.10-pole B. 12-pole C. 14-pole D. 16-pole
Solution:
( )

( )( )( )

( )

( )
-

277. It is proposed to build a dam in a river where the flow rate is 10 m3/s and a 32-m drop in
elevation can be achieved for flow through a turbine. If a turbine is 82 percent efficient, what is
the maximum power that can be achieved? Specific gravity of river is 0.998.
A. 2570 kW B. 3133 kW C. 3820 kW D. 262 kW
Solution:
( )[( )( )]( )

278. What type of turbine delivers 25,000 bhp at 500 rpm under a net head of 5350 ft?
A. Impulse turbine B. Francis turbine C. Kaplan turbine D. Propeller turbine

78
Solution:
( ) ( )

( )

279. A 26-hp pump delivers 475 gpm of gasoline (γ = 42.5 lb/ ft3) at 20 C with 78% efficiency.
What pressure rise result across the pump?
A. 30.2 psi B. 32.7 psi C. 120.3 psi D. 73.2 psi
Solution:
( )
( )

280. A model pump delivering water at 180°F (γ = 60.6 lb/ ft3; pvapor = 7.54 psia) at 900 gpm and
2500 rpm begins to cavitate when the inlet pressure and velocity are 13 psia and 22 fps. Find
the required NPSH of a prototype which is 4 times larger and runs at 1100 rpm.
A. 63.5 ft B. 20.49 ft C. 6.61 ft D. 36 ft
Solution:

[ ( ) ( )]
( )

( ) ( ) ( ) ( )

281. The diameter of the discharge pipe is 8 in. and that of the intake pipe is 10 in. The pressure
gage at discharge reads 32 psi, and vacuum gage at the intake reads 12 in. Hg. If the discharge
flow rate = 4.0 ft3/s of water and the brake horsepower is 49.0, find the efficiency. The intake
and the discharge are at the same elevation.
A. 82.2% B. 80.9% C. 55.8% D. 58.46%
Solution:

( ) ( )
( ) ( )

( ) ( ) (- )( )( )
( ) ( ) ( )

( )( )
( )

79
282. A piston positive-displacement pump (PDP) has a 6-in diameter and a 2.5-in stroke. Its
crankshaft rotates at 300 rpm. Calculate its output at 94 percent volumetric efficiency.
A. 12.27 cfm B. 13.5 cfm C. 10 cfm D. 11.53 cfm
Solution:
( )
( )( )( )

283. A centrifugal pump (efficiency 88%) lifts water through a total height of 40 m from a
reservoir to discharge. Pumping is through 300 m of 75 mm diameter pipe at the rate of 20
liter/s. If pipe friction, f = 0.025, what horsepower is required?
A. 28.4 kW B. 32.2 kW C. 25kW D. 9kW
Solution:
( )
( )

( )[ ( )
]
( )( )

284. In order to predict the behaviour of a small oil pump, tests are to be made on a model
using air. The pump is to be driven by a 1/20-hp motor at 1800 rpm and a ¼-hp motor is
available to drive the air at 600 rpm. Using specific gravity of oil at 0.912 and density of air
constant at 0.076 lb/ ft3, what size model should be built?
A. The model should be 2 times as large as the oil pump.
B. The model should be 5 times as large as the oil pump.
C. The model should be 8 times as large as the oil pump.
D. The model should be 10 times as large as the oil pump.
Solution:
( ) ( )

( )( )( ) ( )( )

285. A double-overhung impulse turbine installation is to develop 20,000 hp at 275 rpm under a
net head of 1100 ft. Determine the specific speed.
A. 4.34 B. 6.14 C. 203.61 D. 144
Solution:
80
( )

286. An impulse wheel at best produces 125 hp under a head of 210 ft. By what percent should
the speed be increased for a 290-ft head?
A. 82.25% B. 17.5% C. 72.41% D. 27.59%
Solution:
( )

( ) ( )

287. What is the power ratio of a pump and its 1/5 scale model if the ratio of heads is 4 to 1?
A. 20 B. 200 C. 12.5 D. 125
Solution:

( ) ( ) ( ) ( )

( ) ( ) ( ) ( )

288. The speed of a centrifugal pump is doubled. By what factor does the pump head change?
A. 0.125 B. 0.25 C. 4 D. 8
Solution:
( )

( ) ( )

289. Compute the specific volume of an air-vapor mixture in cubic meter per kg of dry air when
the following conditions prevail: t = 40°C, w = 0.015 kg/kg, and Pt = 90 kPa.
A. 0.99 m3/kg B. 0.89 m3/kg C. 0.79 m3/kg D. 0.69 m3/kg
Solution:

81
( )

290. A coil has an inlet temperature of 70°F and outlet of 80°F. If the mean temperature of the
coil is 130°F, find the bypass factor of the coil.
A. 0.28 B. 1.2 C. 0.82 D. 0.83
Solution:

291. Compute the pressure drop of 35°C air flowing with a mean velocity of 5m/s in a circular
sheet-metal duct 400mm in diameter and 25 m long. Use friction factor, f = 0.04, and ρair =
1.3799 kg/m3.
A. 431.22 Pa B. 221.34 Pa C. 312.24 Pa D. 422.31 Pa
Solution:
( )( ) ( )
( )

292. Pressure difference of 400 Pa is available to force 20°C air through a circular sheet-metal
duct 450 mm in diameter and 25 m long. At 20°C, ρ = 1.204 kg/m3 and take friction factor, f =
0.016. Determine the velocity.
A. 27.34 ft/s B. 43.72ft/s C. 89.68 ft/s D. 86.98 ft/s
Solution:

( ) ( )
( )

293. A rectangular duct has a dimension of 0.25 m by 2 m. Determine the equivalent diameter
of the duct.
A. 0.50 m B. 0.60 m C. 0.70 m D. 0.40 m
Solution:
( )( )

82
294. To what height will a barometer column rise if the atmospheric conditions are 13.9 psia
and 68°F and barometer fluid is mercury?
A. 3.56 ft B. 5.36 ft C. 2.36 ft D. 3.26 ft
Solution:

295. To what height will a barometer column rise if the atmospheric conditions are 13.9 psia
and 68°F and barometer fluid is ethyl alcohol? Note: @ 68°F; Pv =138.5 lbf/ft3 and specific
gravity of 0.79 for ethyl alcohol:
A. 79.37 in B. 37.79 in C. 353.54 in D. 453.53 in
Solution:
( )
( )

296. What is the pressure 7000 ft below the water surface of the ocean? Neglect
compressibility.
A. 512,000 psf B. 324,500 psf C. 447,000 psf D. 213,000 psf
Solution:
( ) ( )( ) ( )

297. If atmospheric air 14.7 psia and 60°F at sea level, what is the pressure at 14212 ft altitude
if air is incompressible? Note: @ 60°F, the density of air is 0.0763 lb m/ft3; P1 = 14.7 psia.
A. 5.4674 psia B. 7.5304 psia C. 7.1696 psia D. 7.1966 psia
Solution:

( )( )

298. Water (ρ = 62.4 lbm/ft3) is flowing through a pipe. A pitot-static gage registers 3.0 inches of
mercury. What is the velocity of the water in the pipe? Note: ρ Hg = 848.6 lbm/ft3.
A. 14.7 ft/s B. 41.7 ft/s C. 71.4 ft/s D. 74.1 ft/s
Solution:
( ) ( )( )( )
√ √

83
299. The mass of an outside air at 50°C in an air conditioning unit is 60 kg. Find the temperature
after mixing if the outside air mixed with 40 kg with recirculated air at 35°C.
A. 44°C B. 39°C C. 52°C D. 47°C
Solution:

( ) ( ) ( )

300. A creamery must cool 20,000 liters of milk received each day from an initial temperature
of 29°C to a final temperature of 2°C in 5 hours. If refrigeration losses amount to 10 percent of
the cooling load, what must be the capacity of their refrigerating machine? Note: specific heat
of milk is 3.9 kJ/kg-K and SG = 1.05.
A. 38.5 TOR B. 36.5 TOR C. 37.5 TOR D. 39.5 TOR
Solution:

̇ ̇
̇

̇ ( )
( )

̇ [ ( )( )]

301. How many tons of refrigeration is required to produce 10 metric tons of ice per day at -
10°C from raw water at 22°C if miscellaneous losses are 15% of the chilling and freezing load?
A. 17 TOR B. 20 TOR C. 15 TOR D. 24 TOR
Solution:

̇ ̇ ̇ ̇ ̇

̇ ̇
̇ ( )( )( )
( ) -

̇ ( )( )
( )

̇ ( )( )( )
( ) -

̇ ( )
̇

84
302. Five hundred kilograms of poultry enter a chiller at 8°C and are frozen and chilled to a final
temperature of 18°C for storage in 15 hours. The specific heat above and below freezing are
3.18 kJ/kg-°C and 1.55 kJ/kg-°C respectively. The latent heat is 246 kJ/kg and the freezing
temperature is -5°C. Compute the product load.
A. 2.75 kW B. 2.85 kW C. 2.95 kW D. 3.15 kW
Solution:
̇ ̇ ̇ ̇
̇ ( )( )( )
( ) -

̇ ( )( )
( )

̇ ( )( ) (- )
( ) -

303. Fish weighing 11,000 kg with a temperature of 20°C is brought to a cold storage and which
shall be cooled to -10°C in 11 hours. Find the required plant refrigerating capacity in tons of
refrigeration if the specific heat of fish is 0.7 kCal/kg-°C above freezing point and 0.30 kCal/kg-
°C below freezing point. The freezing point is -3°C. The latent heat of freezing is 55.5 kCal/kg.
A. 25.26 TOR B. 15.26 TOR C. 14.38 TOR D. 24.38 TOR
Solution:
̇ ̇ ̇ ̇
̇ ( )( )( )
( ) -

̇ ( )( )
( )

̇ ( )( ) (- )
( ) -

304. The power requirement of a Carnot refrigerator in maintaining a low temperature region
at 300 K is 1.5 kW per ton. Find the heat rejected.
A. 4.02 kW B. 7.02 kW C. 5.02 kW D. 6.02 kW
Solution:
̇

85
̇
̇

̇ ( )

305. A vapor compression refrigeration system is designed to have a capacity of 150 tons of
refrigeration. It produces chilled water from 22°C to 2°C. Its actual coefficient of performance is
5.86 and 35% of the power supplied to the compressor is lost in the form of friction and
cylinder cooling losses. Determine the condenser cooling water required for a temperature rise
of 10°C.
A. 14.75 kg/s B. 15.65 kg/s C. 18.65 kg/s D. 13.75 kg/s
Solution:

̇ ̇ ̇

( )

̇
̇ ̇ ( )( )

306. Determine the heat extracted from 2000 kg of water from 25°C to ice at -10°C.
A. 621,150 kJ B. 721,150 kJ C. 821,150 kJ D. 921,150 kJ
Solution:

( )( )
( )

86
( )( )

307. A single acting, twin cylinder, Ammonia compressor with bore equal to stroke is driven by
an engine at 250 rpm. The machine is installed in a chilling plant to produce 700 kW of
refrigeration at -18°C evaporating temperature. At this temperature the cooling effect per kg
mass is 1160 kJ. The specific volume of vapor entering the compressor is 0.592 m3 per kilogram.
Assume 85% volumetric efficiency, determine the bore in mm.
A. 400 mm B. 300 mm C. 450 mm D. 500 mm
Solution:
( )( )

( )

( ) ( )

308. An iron block weighs 7 Newtons and has a volume of 200 cubic centimeters. What is the
density the block?
A. 3465 kg/m3 B. 3565 kg/m3 C. 1255 kg/m3 D. 2550 kg/m3
Solution:
- ( )
( )[ ( )
]

309. If the density of the gas is 0.003 slugs per cubic foot, what is the specific weight of the gas?
A. 9.04 N/m3 B. 15.2 N/m3 C. 76.3 N/m3 D. 98.2 N/m3
Solution:
-
( )
( ) ( )( ) [( )
]

87
310. The specific gravity of mercury relative to water is 13.55. What is the specific weight of
mercury? (The specific weight of water is 62.4 lbf per cubic foot.)
A. 82.2 kN/m3 B. 102.3 kN/m3 C. 132.9 kN/m3 D. 150.9 kN/m3
Solution:
( )
( )( )[( )
]

311. If the specific weight of a liquid is 58.5 lbf per cubic foot, what is the specific volume of the
liquid in cm3/g?
A. 0.5321 cm3/g B. 0.6748 cm3/g C. 0.9504 cm3/g D. 1.0675 cm3/g
Solution:

( ) ( )
[ ( )
][ ( )
]( )( )

312. Which of the following is not a unit of pressure?


A. Pa B. bars C. kg/m-s 2 D. kg/m2
Solution:
-

313. A cylinder weighs 150 lbf. Its cross-sectional area is 40 square inches. When the cylinder
stands vertically on one end, what pressure does the cylinder exert on the floor?
A. 14.1 kPa B. 25.8 kPa C. 63.2 kPa D. 89.7 kPa
Solution:
( )

314. What pressure is a column of water 100 centimeters high equivalent to?
A. 9810 dyne/cm2 B. 9810 N/m2 C. 0.1 bars D. 0.1 atm
Solution:
( )( )( )( )( )

88
315. Water is flowing in a pipe with a radius of 10” at a velocity of 5 m/s. At the temperature in
the pipe, the density and viscosity of the water are as follows:
ρ = 997.9 kg/m3 and μ = 1.131 Pa-s.
What is the Reynold's number for this situation?
A. 44.1 B. 88.2 C. 1140 D. 2241
Solution:
( ) ( )[ ( )]( ) ( )( )

-
- ( )( )

316. How long must a current of 5.0 amperes pass through a 10 ohm resistor until a charge of
1200 coulombs passes through?
A. 1 min B. 2 min C. 3 min D. 4 min
Solution:
-
( )

317. A car moving at 70 km/hr has a mass of 1700 kg. What force is necessary to decelerate it
at a rate of 40 cm/s 2?
A. 0.680 N B. 42.5 N C. 680 N D. 4250 N
Solution:
( )

318. One hundred milliliters of water in a plastic bag of negligible mass is to be catapulted
upwards with an initial acceleration of 20.0 m/s 2. What force is necessary to do this? Assume
gravity is 9.81 m/s 2 and the density of the water is 1 g/cm3.
A. 2.00 N B. 2.98 N C. 15.0 N D. 2.00 kN
Solution:
( )

[ ( ) ] [( ) ] ( )( )( )
-

319. A boy pulls a sled with a mass of 20 kg horizontally over a surface with a coefficient of
friction of 0.20. It takes him 10 minutes to pull the sled 100 yards. What is his average power
output over these 10 minutes?
A. 4 W B. 6 W C. 8 W D. 10 W
Solution:

89
[ ( )( )]( )
( )( )

320. A force of 200 lbf acts on a block at an angle of 28° with respect to horizontal. The block is
pushed 2 feet horizontally. What is the work done by this force?
A. 215 J B. 320 J C. 480 J D. 540 J
Solution:
( ) ( ) ( ) ( )( )

321. Two particles collide, stick together and continue their motion together. Each particle has
a mass of 10 g and their respective velocities before the collision were 10 m/s and 100 m/s.
What is the energy of the system after the collision?
A. 21.8 J B. 30.2 J C. 42.8 J D. 77.9 J
Solution:
( )
( ) ( ) [( ) ]

( ) [( ) ]( ) ( )

322. A copper bar is 90 centimeters long at 86°F. What is the increase in its length when the bar
is heated to 95°F? The linear expansion coefficient for copper, α, is 1.7 x 10-5/°C.
A. 2.12 x 10 -5 m B. 3.22 x 10 -5 m C. 5.25 x 10 -5 m D. 7.65 x 10 -5 m
Solution:
( )[( ) ] ( )( )
-

323. Calculate the energy transfer rate across a 6" wall of firebrick with a temperature
difference across the wall of 50°C. The thermal conductivity of firebrick is 0.65 BTU/hr-ft-°F at
the temperature of interest.
A. 112 W/m2 B. 285 W/m2 C. 369 W/m2 D. 429 W/m2
Solution:
( )
( )( )( )( ){ ( }
- -
)
[ ( )
]

90
324. A house has brick walls 15 millimeters thick. On a cold winter day, the temperature of the
inner and outer layers of the walls are measured and found to be 20°C and -12°C, respectively.
If there is 120 m2 of exterior wall of race, and the thermal conductivity of bricks is 0.711 J/m-s-
°C, how much heat is lost through the walls per hour?
A. 182 J B. 12.5 kJ C. 655 kJ D. 655 MJ
Solution:
( ){ [ (- )] }
( )( )
- -

325. If a horsepower pump runs for 20 minutes, what is the energy used?
A. 0.06 ergs B. 0.25 kW C. 0.30 MJ D. 0.11 kW-h
Solution:

( ) ( )( )( )

326. A power of 6 kW is supplied to the motor of a crane. The motor has an efficiency of 90%.
With what constant speed does the crane lift the 800 lbf weight?
A. 0.09 m/s B. 0.32 m/s C. 0.98 m/s D. 1.52 m/s
Solution:

( )
( ) ( )

327. An engine has an efficiency of 26%. It uses 2 gallons of gasoline per hour. Gasoline has a
heating value of 20,500 BTU/lbm and a specific gravity of 0.8. What is the power output of the
engine?
A. 0.33 kW B. 20.8 kW C. 26.0 kW D. 41.7 kW
Solution:

̇ ̇ ( )( ) ( )( )( )

( )( )
( )

91
328. Two liters of an ideal gas at a temperature of T 1 = 25°C and a pressure of P1 = 0.101 MPa,
are in a 10 cm diameter cylinder with a piston at one end. The piston is depressed so that the
cylinder is shortened by 10 centimeters. The temperature increases by 2°C. What is the change
in pressure?
A. 0.156 MPa B. 0.167 MPa C. 0.251 MPa D. 0.327 MPa
Solution:

( )( ) { [ ( ) ( )] }
( ) ( )

329. The average power output of a cylinder in a combustion engine is given by:
̅
where:
p = average pressure on the piston during the stroke
L = length of the piston stroke
A = area of the piston head
N = number of strokes per second
An 8-ylinder engine has the following specifications
p = 283 kPa
L = 14 cm
d = diameter of piston head = 12 cm
N = 1500 strokes/min
What is the average power output of this engine?
A. 89.5 N/s B. 89.5 kW C. 89.5 x 103 J-m/s D. 89.5 kJ
Solution:
̅
( )
( )( )[ ( ) ]( ) ( )
( )

330. What is the power required to transfer 97,000 coulombs of charge through a potential rise
of 50 volts in one hour?
A. 0.5 kW B. 0.9 kW C. 1.3 kW D. 2.8 kW
Solution:

92
( - )
( )

331. A current of 7 amperes passes through a 12 ohm resistor. What is the power dissipated in
the resistor?
A. 84 w B. 0.59 hp C. 0.79 hp D. 7 hp
Solution:
( )( )

332. What is the pressure at point A in the tank if h = 2 feet? (g = 32.2 ft/s 2 and ρ = 1.94 slug/ft3)
A. 75 lbf/ft2 B. 85 lbf/ft2 C. 100 lbf/ft2 D. 125 lbf/ft2
Solution:
-

( )( ) ( )

333. Determine the average velocity through a circular section in which the velocity distribution
is given by [ ( ) ]. The distribution is symmetric with respect to the longitudinal
axis, r = 0, ro is the outer radius, vmax is the velocity along the longitudinal axis. Assume flow is
laminar.
A. B. C. D.
Solution:

334. A pipe has a diameter 4” at section AA, and a diameter of 2” at section BB. For an ideal
fluid flow, the velocity given is 1 ft/s at section AA. What is the flow velocity at section BB?
A. 4 ft/s B. 0.5 ft/s C. 1.0 ft/s D. 2.0 ft/s
Solution:

( ) ( ) ( )

335. A mixing tank mixes two inlet streams containing salt. The salt concentration in stream 1 is
5% by weight, at stream 2 it is 15% by weight. Stream 1 flows at 25 kg/s, and stream 2 flows at
10 kg/s. There is only one exit stream. Find the salt concentration in the exit stream.

93
A. 5% B. 8% C. 11% D. 13%
Solution:

( ) ( ) ( )

336. Water is pumped at 1 m3/s to an elevation 5 meters through a flexible hose using a 100%
efficient pump rated at 100 kilowatts. Using the same length of hose, what size motor is needed
to pump 1 m3/s of water to the tank, with no elevation gain? In both cases , both ends of the
hose are at the same temperature and pressure. Neglect kinetic energy effects.
A. 51 kW B. 22 kW C. 36 kW D. 43 kW
Solution:

( )

( )( )

337. A fluid with kinetic viscosity of 2.5 x 10 -5 ft2/s is flowing at 0.1 ft/s from an orifice 3” in
diameter. How can the fluid be described?
A. The fluid is completely turbulent.
B. The fluid is in transition zone.
C. The fluid is laminar.
D. Turbulent cannot be calculated, it must be measured.
Solution:
( )
-
( )

338. The Reynold’s number of a sphere falling in air is 1x106. If the sphere’s radius is 1 ft, what
is its velocity? (ρ = 0.00234 slug/ft3, μair = 3.8x10-7 lbf-s/ft2).
94
A. 2.5 ft/s B. 5.1 ft/s C. 40.6 ft/s D. 81.2 ft/s
Solution:

[ ( ) ]( )

- -

339. The flow rate of water through a cast iron is 5000 gallons per minute. The diameter of the
pipe is 1 foot, and the coefficient of friction is f = 0.0173. What is the pressure drop over a 100
foot length of pipe?
A. 21.078 lbf/ft2 B. 23.78 lbf/ft2 C. 337.26 lbf/in2 D. 337.26 lbf/ft2
Solution:

( )
( )( ) [( ]( ) ( )
)

( )
( )[ ]
( )( )

340. A cylindrical flash tank mounted with its axis horizontal is used to separate liquid ammonia
from ammonia vapor. The ammonia bubbles through the liquid with 70 m3/min leaving the
disengaging surface. The disengaging rate is limited to 60 m/min and the liquid level is to
operate with the liquid level one-third to the diameter from the top. Determine the diameter if
the tank is 1.5 m long.
A. 830 mm B. 730 mm C. 860 mm D. 760 mm
Solution:

95
( ) ( )

(√ )

(√ )( )

341. A 150 Hp motor is used to drive the compressor. If the heat loss from the compressor is 25
kW and the mass flow rate of the refrigerant entering the compressor is 0.50 kg/s, determine
the difference of the enthalpies between the inlet and outlet of the compressor.
A. 143.80 kJ/kg B. 153.80 kJ/kg C. 173.80 kJ/kg D. 183.80 kJ/kg
Solution:

( )

( )

342. To cool farm products, 300 kg of ice at -4.4°C are placed in bunker. Twenty four hours later
the ice has melted into water at 7.2°C. What is the average rate of cooling provided by the ice
in kJ/hr?
A. 2679.28 kJ/hr B. 5679.28 kJ/hr C. 3679.28 kJ/hr D. 4679.28 kJ/hr
Solution:
̇ ̇ ̇ ̇
̇ [ ( ) ( )]
̇

343. Determine the estimated condenser load for an open-type compressor having a cooling
capacity of 16,500 Btu/hr and a heat rejection factor of 1.32.
A. 22,280 Btu/hr B. 20,780 Btu/hr C. 21,780 Btu/hr D. 19,780 Btu/hr
Solution:

( )
96
344. If the load on water-cooled condenser is 150,000 Btu/hr and the temperature rise of the
water in the condenser is 10°F, what is the quantity of water circulated in gpm?
A. 30 GPM B. 40 GPM C. 20 GPM D. 50 GPM
Solution:
̇ ̇
̇ ( )( )
̇

345. The load on a water-cooled condenser is 90,000 Btu/hr. if the quantity of water circulated
through the condenser is 15 gpm, determine the temperature rise of the water in the
condenser.
A. 12°F B. 14°F C. 16°F D. 18°F
Solution:
̇ ̇
̇ ( )( )
-

̇
( )( )

( )

346. The weight of ammonia circulated in a machine is found to be 21.8 lb/hr. if the vapor
enters the compressor with a specific volume of 9.6 ft3/lb, calculate the piston displacement,
assuming 80% percent volume efficiency.
A. 261.6 ft3/hr B. 271.6 ft3/hr C. 281.8 ft3/hr D. 291.6 ft3/hr
Solution:

( )

97
347. A single-stage ammonia compressor is producing 10 tons of refrigeration and the power
consumed is 15 Hp. Suction pressure is 25 psi, condensing pressure is 180 psi. Brine
temperature is 20°F off brine cooler. Determine the actual coefficient of performance.
A. 10.14 B. 11.14 C. 12.14 D. 13.14
Solution:
( )
( )

348. In an ammonia condensing machine (compressor plus condenser) the water used for
condensing is 55°F and the evaporator is at 15°F. Calculate the ideal COP.
A. 11.875 B. 12.875 C. 10.875 D. 13.875
Solution:

349. How much refrigeration capacity is required to cool 2000 cfm of air from 85°F to 70°F?
A. 2.7 TOR B. 3.7 TOR C. 1.7 TOR D. 4.7 TOR
Solution:
̇ ̇

-
̇ ( )( )

̇ ( )( ) ( )( )

350. Determine the coil face area required to maintain a face velocity 400 ft/min if the air flow
rate over the coil is 2100 ft3/min.
A. 3.25 ft2 B. 4.25 ft2 C. 5.25 ft2 D. 6.25 ft2
Solution:

98
351. Calculate the heat transfer per hour through a solid brick wall 6 m long, 2.9 m high, and
225 mm thick, when the outer surface is at 5°C and the inner surface 17°C, the coefficient of
thermal conductivity of the brick being 0.6 W/m-K.
A. 2,004.48 kJ B. 3,004.48 kJ C. 2,400.48 kJ D. 3,400.48 kJ
Solution:
[( )( )]( )

( )( )

352. A vertical furnace wall is made up of an inner wall of firebrick 20 cm thick followed by
insulating brick 15 cm thick and an outer wall of steel 1 cm thick. The surface temperature of
the wall adjacent to the combustion chamber is 1200°C while that of the outer surface of steel
is 50°C. The thermal conductivities of the wall material in W/m-K are: firebrick, 10; insulating
brick, 0.26; and steel, 45. Neglecting the film resistances and contact resistance of joints,
determine the heat loss per sq.m. of wall area.
A. 1.93 W/m2 B. 2.93 W/m2 C. 1.55 W/m2 D. 2.55 W/m2
Solution:
̇

353. A composite wall is made up of an external thickness of brickwork 110 mm thick inside
which is a layer of fiberglass 75 mm thick. The fiberglass is faced internally by an insulating
board 25 mm thick. The coefficients of thermal conductivity are as follow:
Brickwork 1.5 W/m-K
Fiberglass 0.04 W/m-K
Insulating board 0.06 W/m-K
The surface transfer coefficients of the inside wall is 3.1 W/m2-K while that of the outside wall is
2.5 W/m2-K. Take the internal ambient temperature as 10°C and the external temperature is
27°C. Determine the heat loss through such wall 6m high and 10 m long.
A. 330.10 W B. 230.10 W C. 430.10 W D. 530.10 W
Solution:
̇

99
-

[ ( ) ]( )
̇

354. One insulated wall of a cold-storage compartment is 8 m long by 2.5 m high and consists of
an outer steel plate 18 mm thick. An inner wood wall is 22.5 m thick. The steel and wood are 90
mm apart to form a cavity which is filled with cork. If the temperature drop across the extreme
faces of the composite wall is 15°C. Calculate the heat transfer per hour through the wall and
the temperature drop across the thickness of the cork. Take the coefficients of thermal
conductivity for steel, cork and wood as 45, 0.045, and 0.18 W/m-K respectively.
A. 408.24 kJ, 12.12°C C. 608.24 kJ, 13.12°C
B. 708.24 kJ, 11.12°C D. 508.24 kJ, 14.12°C
Solution:
̇

[ ( ) ]( )
̇

( )
̇

355. A cubical tank of 2 m sides is constructed of metal plate 12 mm and contains water at
75°C. The surrounding air temperature is 16°C. Calculate the overall heat transfer coefficient
from water to air. Take the coefficient of thermal conductivity of the metal as 48 W/m-K, the
coefficient of heat transfer of water is 2.5 kW/m2-K and the coefficient of heat transfer of the
air is 16 W/m2-K.
A. 15.84 W/m2-°C B.14.84 W/m2-°C C. 16.84 W/m2-°C D. 13.84 W/m2-°C
Solution:

100
-

356. A cold storage compartment is 4.5 m long by 4 m wide by 2.5 m high. The four walls,
ceiling and floor are covered to a thickness of 150 mm with insulating material which has a
coefficient of thermal conductivity of 5.8x10 -2W/m-K. Calculate the quantity of heat leaking
through the insulation per hour when the outside and inside face temperature of the material is
15°C and -5°C respectively.
A. 2185.44 kJ B. 2285.44 kJ C. 3185.44 kJ D. 4185.44 kJ
Solution:
̇

[ ( ) ( ) ( )]

-
( )( )( )
̇

357. A furnace wall consists of 35 cm firebrick (k = 1.557 W/m-K), 12 cm insulating refractory (k


= 0.346) and 20 cm common brick (k = 0.692) covered with 7 cm steel plate (k = 45). The
temperature at the inner surface of the firebrick is 1,230°C and at the outer face of the steel
plate is 60°C. Atmosphere is 27°C. What is the value of the combined coefficient for convection
and radiation from the outside wall?
A. 31.13 W/m2-K B. 30.13 W/m2-K C. 41.3 W/m2-K D. 40.13 W/m2-K
Solution:
̇

̇ ( )

̇ ̇

101
358. Hot gases at 280°C flow on one side of the metal plate 10 mm thickness and air at 35°C
flows on the other side. The heat transfer coefficient of the gases is 31.5 W/m2-K and that of
the air is 32 W/m2-K. Calculate the over-all heat transfer coefficient.
A. 15.82 W/m2-K B. 16.82 W/m2-K C. 14.82 W/m2-K D. 17.82 W/m2-K
Solution:

359. The surface temperature of the hot side of the furnace wall is 1200°C. It is desired to
maintain the outside of the wall at 38°C. A 152 mm of refractory silica is used adjacent to the
combustion chamber and 10 mm of steel covers the outside. What thickness of insulating bricks
is necessary between refractory and steel, if the heat loss should be kept at 788 W/m 2? Use k =
13.84 W/m-K for refractory silica; 0.15 for insulating brick, and 45 for steel.
A. 220 mm B. 240 mm C. 260 mm D. 280 mm
Solution:

360. An insulated steam pipe located where the ambient temperature is 32°C, has an inside
diameter of 50 mm with 10 mm thick wall. The outside diameter of corrugated asbestos
insulation is 125 mm and the surface coefficient of still air, h o = 12 W/m2-K. Inside the pipe is
steam having a temperature of 150°C with film coefficient h i = 6000 W/m2-K. Thermal
conductivity of pipe and asbestos insulation are 45 and 0.12 W/m-K respectively. Determine the
heat loss per unit length of pipe.
A. 110 W B. 120 W C. 130 W D. 140 W
Solution:
̇

102
( ) ( )

( ) ( )
[ ( ) ]( ) ( ) ( )

[ ( ) ]( )

̇
̇

361. How many watts will be radiated from a spherical black body 15 cm in diameter at a
temperature of 800°C?
A. 5.34 kW B. 4.34 kW C. 6.34 kW D. 3.34 kW
Solution:

-
-
( )

( -
)( )( )

362. A wall with an area of 10 m2 is made of a 2 cm thickness of white pine (k = 0.133 W/m-°C)
followed by a 10 cm of brick (k = 0.649 W/m-°C). The pine is on the inside where the
temperature is 30°C while the outside temperature is 10°C. Assuming equilibrium conditions
exist, what is the temperature at the interface between the two metals?
A. 15.65 °C B. 17.64 °C C. 18.21 °C D. 19.31 °C
Solution:
̇

( )
̇

103
( ) ( )( )
̇ ̇

363. A counter-flow heat exchanger is designed to heat a fuel oil from 45°C to 100°C while the
heating fluid enters at 150°C and leaves at 115°C. Calculate the arithmetic mean temperature
difference.
A. 40°C B. 50°C C. 60°C D. 70°C
Solution:

364. With three different quantities x, y, and z of the same kind of liquid of temperatures 9, 21
and 38°C respectively, it is found that when x and y are mixed together the resultant
temperature is 17°C and when y and z are mixed together the resultant temperature is 28°C.
Find the resultant temperature if x and z were mixed.
A. 25.87°C B. 25.92°C C. 20.85°C D. 24.86°C
Solution:

( ) ( )

( ) ( )

( ) ( )
( ) ( )

104
365. The journals of a shaft are 380 mm diameter, it runs at 105 rpm and the coefficient of
friction between journals and bearings is 0.02. If the average load on the bearings is 200 kN,
find the heat generated per minute at the bearings.
A. 501.375 kJ B. 505.575 kJ C. 401.375 kJ D. 501.575 kJ
Solution:

( )
( )

[ ( )]( ) -
( )
( )( )( )
( )
̇ ( )( )( )

366. A reverse Carnot cycle requires 3 Hp and extracts energy from a lake to heat a house. If the
house is kept at 70°F and requires 2000 Btu per minute, what is the temperature of the lake?
A. 35.29°F B. 36.29°F C. 39.29°F D. 40.29°F
Solution:
̇ ( )

̇ ̇ ̇

̇
̇

̇ ( ) ( )
( )( )

̇ ( )
( )

105
367. An oxygen cylinder of volume 2.3 ft3 has a pressure of 2200 psig and is at 70°F. Determine
the mass of oxygen in the cylinder.
A. 25.66 lbs B. 26.66 lbs C. 27.66 lbs D. 28.66 lbs
Solution:

( )

( )( ) ( )( )

368. A group of 50 persons attend a secret meeting in a room which is 12 m wide by 10 m long
and a ceiling height of 3 m. The room is completely sealed off and insulated. Each person gives
off 150 kCal per hour of heat and occupies a volume of 0.20 m3. The room has an initial
pressure of 101.3 kPa and temperature of 16°C. Calculate the room temperature after 10
minutes. Use R = 0.287 kJ/kg-K and C v = 0.171 kCal/kg-K.
A. 33.1°C B. 37.7°C C. 38.7°C D. 31.7°C
Solution:

( )

( )

[ ( )( ) ( )]

( ) ( )( )

( )( )

106
369. One kilogram of wet steam at a pressure of 8 bar (υ g = 0.2404 m3/kg, υf = 0.0011148
m3/kg) and dryness of 0.94 is expanded until the pressure is 4 bar (υ g = 0.4625 m3/kg, υf =
0.0010836 m3/kg). If expansion follows the law PV n = C, where n = 1.12, find the dryness
fraction of the steam at the lower pressure.
A. 0.9072 B. 0.4197 C. 0.2260 D. 0.2404
Solution:

( ) ( )( )

( ) ( )

( ) ( )

( ) ( )( )

370. 2.5 liters of superheated steam at 25 bar and 400 C (υ = 0.1252 m3/kg) is expanded in an
engine to a pressure of 0.1 bar (υ g = 14.674 m3/kg, υf = 0.0010102 m3/kg) when its dryness
fraction is 0.9. Find the final volume of the steam.
A. 163.74 liters B. 263.74 liters C. 363.74 liters D. 463.74 liters
Solution:

( ) ( )
( )( )

371. A 1.5 kg of wet steam at a pressure of 5 bar (hf = 640 kJ/kg, hfg = 2109 kJ/kg) dryness 0.95
is blown into 70 liters of water 12°C (h = 50.4 kJ/kg). Find the final enthalpy of the mixture.
A. 74.80 kJ/kg B. 84.80 kJ/kg C. 94.80 kJ/kg D. 104.80 kJ/kg
Solution:

( ) ( )
107
[ ( )] ( )

( )

372. A 650 BHP diesel engine uses fuel oil of 28°API gravity, fuel consumption is 0.65 lb/BHP-hr.
Cost of fuel is P7.95 per liter. For continuous operation determine the minimum volume of
cubical day tank in cm3, ambient temperature is 45° C.
A. 4,372,890 cm3 B. 5,987,909 cm3 C. 5,291,880 cm3 D. 7,352,789 cm3
Solution:

( )

[ ( )]
[ ( )]
( )

373. A typical industrial fuel oil, C 16H32 with 20% excess air by weight. Assuming complete
oxidation of the fuel, calculate the actual air-fuel ratio by weight.
A. 17.56 kg air/kg fuel C. 16.75 kg air/kg fuel
B. 15.76 kg air/kg fuel D. 17.65 kg air/kg fuel
Solution:

( )

108
( ) ( )

( )( ) ( )

( ) ( )
( ) ( ) ( )

374. Fuel oil in a day tank for use of an industrial boiler is tested with hydrometer. The
hydrometer reading indicates a SG = 0.924 when the temperature of the oil in the tank is 35°C.
Calculate the higher heating value of the fuel.
A. 43,852.13 kJ/kg B. 53,852.13 kJ/kg C. 58,352.13 kJ/kg D. 48,352.13 kJ/kg
Solution:
( )

[ ( )]

( )

375. A diesel electric plant supplies energy for Meralco. During a 24 hr Period, the plant
consumed 200 gallons of fuel at 28°C and produced 3930 Kw-hr. Industrial fuel used is 28°API
and was purchased at P5.50 per liter at 15.6°C. What should the cost of fuel be produce one
kw-hr.?
A. P 1.05 B. P 1.10 C. P 1.069 D. P 1.00
Solution:

( )

[ ( )]
109
[ ( )]
( )

-
( )( )( )( ) -
-

376. A certain coal has the following ultimate analysis:


C = 70.5% H = 4.5% O2 = 6% N2 = 1.0%
S = 3.0% Ash = 11% Moisture = 4%
A stoker fired boiler of 175,000 kg/hr steaming capacity uses this coal as fuel. Calculate the
volume of air in m3/hr with air at 60°F (15.6°F) and 14.7 psia (101.325 kPa) the coal is burned
with 30% excess air. Boiler efficiency is 70% and factor of evaporation of 1.10.
A. 212,861.04 m3/hr B. 221,861.04 m3/hr C. 218,261.04 m3/hr D. 281,261.04 m3/hr
Solution:
̇( )( )
̇

( ) ( ) ( ) ( )

( )
( ) ( ) ( )

̇ ( ) ̇ ( ) ( )( )
̇ ̇ ̇ ( )
( )
̇ ( )

̇
̇ ̇ ( ) ( ) ( )( )

( )( )
̇

377. A diesel power plant consumed 1 m3 of fuel with 30°API at 27°C in 24 hrs. Calculate the
fuel rate in kg/hr.
A. 36.21 B. 26.25 C. 29.34 D. 39.42
Solution:
110
̇ ̇
( )

[ ( )]
( )

̇ ( )

378. A diesel power plant uses fuel with heating value of 43,000 kJ/kg. What is the density of
the fuel at 25°C?
A. 840 kg/m3 B. 873 kg/m3 C. 970 kg/m3 D. 940 kg/m3
Solution:

[ ( )]

( )

379. A water tube boiler has a capacity of 1000 kg/hr of steam. The factor of evaporation is 1.3,
boiler rating is 200%, boiler efficiency is 65% and heating surface area is 0.91 m 2 per bo.Hp. And
the heating value of fuel is 18,400 kCal/kg. The total coal available in the bunker is 50,000 kg.
Determine the no. of hrs. to consume the available fuel.
A. 853.36 hrs B. 706.57 hrs C. 979.46 hrs D. 100.75 hrs
Solution:

( )
111
( ) [ ( )]
[ ( )]

380. Two boilers are operating steadily on 91,000 kg of coal contained in a bunker. One boiler is
producing 1591 kg of steam per hour at 1.2 factor of evaporation and an efficiency of 65% and
another boiler produced 1364 kg of steam per hour at 1.15 factor of evaporation and an
efficiency of 60%. How many hrs will the coal in the bunker run the boilers if the heating value
of coal is 7,590 kCal/kg?
A. 230.80 hrs B. 280.54 hrs C. 350.35 hrs D. 300.54 hrs
Solution:

̇ ( )
̇
̇ [ ( )]
̇
[ ( )]
̇ [ ( )]

̇ [ ( )]
̇
[ ( )]
̇ [ ( )]

̇ ̇ ̇

381. The heating value of fuel supplied in a boiler is 40,000 kJ/kg. If the factor of evaporation is
1.10 and the actual specific evaporation is 10, what is the efficiency of the boiler?
A. 62.07% B. 53.08% C. 78.05% D. 54.97%

112
Solution:
̇ ( ) [ ( )] [ ( )]
̇

382. What is the rate of evaporation of a water tube boiler if the factor of evaporation is 1.10,
percent rating of 200% and the heating surface area is 250 m2?
A. 7,817.16 kg/hr B. 7,898.67 kg/hr C. 6,789.45 kg/hr D. 5,768.54 kg/hr
Solution:
̇ ( ) ̇ [ ( )]

̇ [ ( )]

383. Steam is admitted to the cylinder of an engine in such a manner the average pressure is
120 psi. The diameter of the piston is 10 in. and the length of a stroke is 12 in. What is the Hp of
the engine when it is making 300 rpm?
A. 171.4 Hp B. 175 Hp C. 173.2 Hp D. 174.4 Hp
Solution:

( ) ( )( )
( ) [ ]

( )( )

384. Steam enters a turbine stage with an enthalpy of 3628 kJ/kg at 70 m/s and leaves the same
stage with an enthalpy of 2846 kJ/kg and a velocity of 124 m/s. Calculate the power if there are
5 kg/s steam admitted at the turbine throttle?
A. 4597.45 kW B. 3976.55 kW C. 3883.81 kW D. 1675.42 kW
Solution:

113
̇ ̇ ( ) ̇ ( ) ( ) ( )( )
̇

385. Steam with an enthalpy of 800 kCal/kg enters a nozzle at a velocity of 80 m/s. Find the
velocity of the steam at the exit of the nozzle if its enthalpy is reduced to 750 kCal/kg, assuming
the nozzle is horizontal and disregarding heat losses. Take g = 9.81 m/s 2 and J constant = 427 kg
m/kCal.
A. 452.37 m/s B. 245.45 m/s C. 651.92 m/s D. 427.54 m/s
Solution:

( )

386. Steam is expanded through a nozzle and the enthalpy drop per kg of steam from the initial
pressure to the final pressure of 60 kJ. Neglecting friction, find the velocity of discharge and the
exit area of the nozzle to pass 0.20 kg/s if the specific volume of the steam at exit is 1.5 m 3/kg.
A. 346.4 m/s, 879 mm2 C. 765.6 m/s, 467 mm2
B. 356.7 m/s, 278 mm2 D. 346.4 m/s, 866 mm2
Solution:

√ √ ( ) √ ( )

( ) ( )

387. A 6 MW steam turbine generator power plant has a full-load steam rate of 8 kg/kW-hr.
Assuming that no-load steam consumption as 15% of full-load steam consumption, compute for
the hourly steam consumption at 75% load, in kg/hr.
A. 37,800 kg/hr B. 38,700 kg/hr C. 30,780 kg/hr D. 30,870 kg/hr
Solution:
114
-
̇ ( )
-

- ( )
̇ ( )
-

( ) ( )
( ) ( )
( ) ( )

̇
̇

( )

̇ ( )

388. A 4 kg of air enters a turbine with enthalpy of 600 kJ and velocity of 250 m/s. The enthalpy
at exit is 486 kJ and velocity of 170 m/s. What is the work developed if there is a heat loss of 10
kJ?
A. 122.83 kJ B. 171.2 kJ C. 80.2 kJ D. 28.3 kJ
Solution:
̇ ̇
[ ( )( ) ] [ ( )( )]

389. Calculate the drive horsepower for pumping 1703 L/min cold water to a tank suction at
127 mmHg vacuum, delivery at 5.3 kg/cm2 ga., both measured close to pump, e p = 0.65.
A. 31.42 Hp B. 20.42 Hp C. 35.42 Hp D. 23.02 Hp
Solution:

115
( )

- -
( ) ( )

( )( )( )
-
( -
)

390. Find the length of a suspension bunker to contain 181 tons of coal without surcharge;
width = 4.6 m, depth = 4.3 m. The level capacity of a suspension bunker is 5/8 wdL where: w =
width, d = depth and L = length. Density of coal is 800 kg/m3.
A. 18.30 m B. 15.80 m C. 17.61 m D. 12.61 m
Solution:

( )

( )( )

391. A 305 mm x 457 mm four stroke single acting diesel engine is rated at 150 kW at 260 rpm.
Fuel consumption at rated load is 0.26 kg/kW-hr with a heating value of 43,912 kJ/kg. Calculate
the brake thermal efficiency.
A. 31.63% B. 41.63% C. 21.63% D. 35.63%
Solution:

116
̇ ( )
-

( )

392. The brake thermal efficiency of a 1 MW diesel electric plant is 36%. Find the heat
generated by fuel in kW if the generator efficiency is 89%.
A. 3,121.10 kW B. 3,528.64 kW C. 4,121.10 kW D. 4,528.64 kW
Solution:

393. In an air-standard Brayton cycle, the compressor receives air at 101.325kPa, 21°C and it
leaves at 600 kPa at the rate of 4 kg/s. Determine the turbine work if the temperature of the air
entering the turbine is 1000°C.
A. 3000 kW B. 2701 kW C. 2028 kW D. 3500 kW

Solution:
̇ ̇ ( )

( )

( )

̇ ( )( )

117
394. Kerosene is the fuel of a gas turbine plant: fuel-air ratio, mf = 0.012, T3 = 972 K, pressure
ratio, rp = 4.5, exhaust to atmosphere. Find the available energy in kJ per kg air flow. Assume k =
1.34 and C p = 1.13.
A. 352.64 kJ/kg B. 452.64 kJ/kg C. 252.64 kJ/kg D. 552.64 kJ/kg

Solution:

( ) ( )

( )

( )

( )( )( )

395. An ideal gas turbine operates with a pressure ratio of 10 and the energy input in the high
temperature heat exchanger is 300 kW. Calculate the air flow for temperature limits of 30°C
and 1200°C.
A. 0.25 kg/s B. 0.34 kg/s C. 0.41 kg/s D. 0.51 kg/s
Solution:
̇ ̇ ( )

( )

̇ ( )( )
̇

396. In an air-standard Brayton cycle the inlet temperature and pressure are 20°C and 101.325
kPa. The turbine inlet conditions are 1200 kPa and 900°C. Determine the air flow if the turbine
produces 12 MW.
A. 21.41 kg/s B. 20.20 kg/s C. 19.25 kg/s D. 18.10 kg/s
Solution:

118
̇ ( )

( )

( )

̇ ( )( )
̇

397. A gas turbine power plant operating on the Brayton cycle delivers 15 MW to a standby
electric generator. What are the mass flow rate and the volume flow rate of air if the minimum
and maximum pressures are 100 kPa and 500 kPa respectively and temperature of 20°C and
1000°C?
A. 31.97 kg/s, 26.88 m3/s C. 41.97 kg/s, 26.88 m3/s
B. 36.98 kg/s, 28.99 m3/s D. 46.98kg/s, 28.99 m3/s
Solution:
̇ ̇

( )

( )

̇ ̇( )
̇ ( )( )
̇

̇ ( )( )
̇

398. In a hydraulic plant the difference in elevation between the surface of the water at intake
and the tailrace is 650 ft when the flow is 90 cfs, the friction loss in the penstock is 65 ft and the
head utilized by the turbine is 500 ft. The mechanical friction in the turbine is 110 Hp, and the
leakage loss is 4 cfs. Find the hydraulic efficiency.
A. 87.45% B. 84.57% C. 85.47% D. 78.54%
Solution:
119
399. A hydro-electric power plant consumes 60,000,000 kW-hr per year. What is the net head if
the expected flow is 1500 m3/min and over-all efficiency is 63%?
A. 34.34 m B. 43.43 m C. 44.33 m D. 33.44 m
Solution:

( )

400. A pelton type turbine has a gross head of 40 m and a friction head loss 6 m. What is the
penstock diameter if the penstock length is 90 m and the coefficient of friction head loss is
0.001 (Morse)?
A. 2040 mm B. 3120 mm C. 2440 mm D. 2320 mm
Solution:

√ √ ( )( )

( )( )( )

401. The water velocity of a 5 m x 1 m channel is 6 m/s. What is the annual energy produced if
the net head is 120 m and the over-all efficiency is 80%?
A. 494,247,258 kW-hrs C. 247,494,528 kW-hrs
B. 247,497,582 kW-hrs D. 472,497,582 kW-hrs
Solution:
( )
( )
120
( )( )
( )( )

( )( )
-

402. A hydro-electric impulse turbine is directly coupled to a 24 pole, 60 Hz alternator. It has a


specific speed of 60 rpm and develops 3000 Hp. What is the required diameter assuming a
peripheral speed ratio of 0.45?
A. 0.661 m B. 0.552 m C. 0.443 m D. 0.773 m
Solution:

( )

( )

√ ( )( )

403. In a hydroelectric power plant the tailwater elevation is at 500 m. What is the head water
elevation if the net head is 30 m and the head loss is 5% of the gross head?
A. 785.25 m B. 582.57 m C. 528.57 m D. 758.25 m
Solution:

121
404. The tailwater and headwater of a hydro-electric plant are 150 m and 200 m respectively.
What is the water power if the flow is 15 m3/s and a head loss of 10% of the gross head?
A. 6,621.75 kW B. 7,621.65 kW C. 5,621.76 kW D. 4,621.56 kW
Solution:

( )
( )( )

405. In a hydro-electric plant, water flows at 10 m/s in a penstock of 1 m3 cross-sectional area.


If the net head of the plant is 30 m and the turbine efficiency is 85%, what is the turbine
output?
A. 2,501.55 kW B. 2,100.21 kW C. 3,626.34 kW D. 3,124.65 kW
Solution:

( )
( )( )

406. A 75 MW power plant has an average load of 35,000 kW and a load factor of 65%. Find the
reserve over peak.
A. 21.15 MW B. 23.41 MW C. 25.38 MW D. 18.75 MW
Solution:

407. A power plant is said to have/had a use factor of 48.5% and a capacity factor of 42.4%.
How many hrs. did it operate during the year?
A. 6,600.32 hrs B.7,658.23 hrs C. 8,600.32 hrs D. 5,658.23 hrs
122
Solution:
-

( ) ( )

408. A 50,000 kW steam plant delivers an annual output of 238,000,000 kW-hr with a peak load
of 42,860 kW. What is the annual load factor and capacity factor?
A. 0.634, 0.534 B. 0.643, 0.534 C. 0.634, 0.543 D. 0.643, 0.534
Solution:

- -

409. Calculate the use factor of a power plant if the capacity factor is 35% and it operates 8000
hrs during the year?
A. 38.325% B. 33.825% C. 35.823% D. 32.538%
Solution:
( )
( )

410. If the air required for combustion is 20 kg per kg of coal and the boiler uses 3000 kg of coal
per hr, determine the mass of gas entering the chimney. Assume an ash loss of 15%.
A. 40,664 kg/hr B. 70,200 kg/hr C. 62,550 kg/hr D. 50,500 kg/hr
Solution:
̇ ̇ ̇ ̇
123
̇
̇
̇ ̇
̇ ̇ ̇ ̇
̇ ̇ ( )

411. A 15 kg gas enters a chimney at 10 m/s. If the temperature and pressure of a gas are 26°C
and 100 kPa respectively, what is the diameter of the chimney? Use R = 0.287 kJ/kg -K.
A. 1.57 m B. 2.65 m C. 2.22 m D. 1.28 m
Solution:

̇ ( )( )
̇

( )

412. A two-stage air compressor at 90 kPa and 20°C discharges at 700 kPa. Find the polytropic
exponent n if the intercooler intake temperature is 100°C.
A. 1.29 B. 1.33 C. 1.4 D. 1.25
Solution:

( )

√ √ ( )

( )

413. A two-stage compressor receives 0.35 kg/s of air at 100 kPa and 269 K and delivers it at
5000 kPa. Find the heat transferred in the intercooler.
A. 70.49 kW B. 80.49 kW C. 90.49 kW D. 100.49 kW
Solution:
̇ ̇ ( )

124
√ ( )

( )

( )

̇ ( )( )

414. A centrifugal pump discharged 20 L/s against a head of 17 m when the speed is 1500 rpm.
The diameter of the impeller was 30 cm and the brake horsepower was 6.0. A geometrically
similar pump 40 cm in diameter is to run at 1750 rpm. Assuming equal efficiencies, what brake
horsepower is required?
A. 51.55 HP B. 50.15 HP C. 40.14 HP D. 45.15 HP
Solution:

( ) ( )

415. A pump delivers 20 cfm of water having a density of 62 lb/ft 3. The suction and discharge
gage reads 5 in. Hg vacuum and 30 psi respectively. The discharge gage is 5 ft above the suction
gage. If pump efficiency is 70%, what is the motor power?
A. 5.31 HP B. 3.31 HP C. 4.31 HP D. 6.31 HP
Solution:

[ (- ) ( )]
( )

-
( )( ) ( -
)

416. Calculate the air power of a fan that delivers 1200 m3/min of air through a 1 m x 1.5 m
outlet. Static pressure is 120 mm WG and density of air is 1.18.

125
A. 20.45 kW B. 25.64 kW C. 30.45 kW D. 35.64 kW
Solution:

( )

[ ( )]

( )

[ ( )] ( )( )

417. Determine the temperature for which a thermometer with degrees Fahrenheit is
numerically twice the reading of the temperature in degrees Celsius.
A. -24.6 B. 320 C. 160 D. -12.3
Solution:
-
( ) -

( )

[ ( )]

418. During takeoff in a spaceship, an astronaut is subjected to acceleration equal to 5 times


the pull of the earth’s standard gravity. If the astronaut is 180 lb m and the takeoff is vertical,
what force does he exert on the seat?
A. 4810.9 N B. 4414.5 N C. 8829 N D. 9620 N
Solution:

( )
126
( ) ( )( ) ( ) ( )

419. A pressure cooker operates by cooking food at a higher pressure and temperature possible
at atmospheric conditions. Steam is contained in the sealed pot, with vent hole in the middle of
the cover, allowing steam to escape. The pressure is regulated by covering the vent hole with a
small weight, which is displaced slightly by escaping steam. Atmospheric pressure is 100 kPa,
the vent hole area is 7-mm2, and the pressure inside should be 250 kPa. What is the mass of the
weight?
A. 0.107 kg B. 1.05 kg C. 1.75 kg D. 0.1783 kg
Solution:

( )
-
( ) ( )

420. A barometer can be used to measure an airplane’s altitude by comparing the barometric
pressure at a given flying altitude to that on the ground. Determine an airplane’s altitude if the
pilot measures the barometric pressure at 700 mm-Hg, the ground reports it at 758 mm-Hg,
and the average air density is 1.19 kg/m3, g = 9.81 m/s 2.
A. 636 m B. 633 m C. 366 m D. 663 m
Solution:
( ) - ( )
-

( )
( )( )

421. A mixture of 0.4 lbm of helium and 0.2 lbm of oxygen is compressed polytropically from
14.7 psia and 60°F to 60 psia according to n = 1.4. Determine the final temperature, T 2.
A. 727.7°R B. 777.2°R C. 722.7°R D. 277.7°R
Solution:

( )

127
( ) ( )

422. A mixture of 0.4 lbm of helium and 0.2 lbm of oxygen is compressed polytropically from
14.7 psia and 60oF t0 60 psia according to n = 1.4. Determine the polytropic work.
A. 139 Btu B. 239 Btu C. 339 Btu D. 539 Btu
Solution:
- -

( )( - - )
( )( - - )
-
( ) -
- -

( ) ( - )( )

423. A pump in a municipality’s water-supply system receives water from the filtration beds
and pumps it up to the top of a water tower. The tower’s height is 35 m, and the inlet piping to
the pump is 2 m below the pump’s intake. The water temperature is 20 C, measured at both
the inlet and the discharge from the pump. The mass flow rate through the pump is 100 kg/s,
the diameter of the inlet piping is 25 cm, and the diameter of the discharge piping is 15 cm.
Determine the power required by the pump.
A. 77.3 kW B. 33.77 kW C. 34.42 kW D. 42.34 kW
Solution:
̇
̇ ( )

( ) ( )

( )
128
( )

( ) ( )
( )

̇ ( )( )

424. An adiabatic tank containing air is used to power an air turbine during times of peak power
demand. The tank has a volume of 500 m3 and contains air at 1000 kPa and 500 K. Determine
the mass remaining when the pressure reaches 100 kPa.
A. 273.37 kg B. 672.73 kg C. 772.73 k D. 227.73 kg
Solution:

( )

( )

( ) ( - )( )

425. Determine the air-fuel ratio on a molar basis for the complete combustion of octane, with
theoretical amount of air.
A. 95.5 kg air/kg fuel B. 59.9 kg air/kg fuel C.59.5 kg air/kg fuel D. 55.9 kgair/kg fuel
Solution:

( )

( )
[ ( )]
( )

426. During a steady state operation, a gearbox receives 60 kW throughout the input shaft and
delivers power through the output shaft. For the gearbox as the system, the rate of energy
transfer is by convection, h = 0.171 kW/m2-K is the heat transfer coefficient, A = 1.0 m2 is the
outer surface area of the gearbox, Tb = 300 K (27°C) is the temperature at the outer surface, T f =
293 K (20°C) is the temperature of the surroundings away from the immediate vicinity of the

129
gearbox. Determine the power delivered to the output shaft in kW if the heat transfer rate is -
1.2 kW.
A. 98.8 kW B. 78.8 kW C. 68.8 kW D. 58.8 kW
Solution:
̇ ̇ ̇ ̇
̇ ̇ ̇ ̇
̇ -
̇ -
̇ - (- )

427. A single acting air compressor with a clearance of 6% takes in air at atmospheric pressure
and temperature of 85°F and discharges it at a pressure of 85 psia. The air handled is 0.25
ft3/cycle measured at discharge pressure. If the compression is isentropic, find the piston
displacement per cycle, if the compressor is running at 750 rpm.
A. 0.0750 ft3/cycle B. 0.025 ft3/cycle C. 1.030 ft3/cycle D. 1.090 ft3/cycle
Solution:

( )

( ) ( )
( )( )
( )
( )( )
( )

( ) ( )

428. A single acting air compressor with a clearance of 6% takes in air at atmospheric pressure
and temperature of 85°F and discharges it at a pressure of 85 psia. The air handled is 0.25
ft3/cycle measured at discharge pressure. If the compression is isentropic, find the air hp of the
compressor if rpm is 750.
A. 16 hp B. 96 hp C. 69 hp D. 61 hp
Solution:
̇ ( )( )

130
̇ ( )( )( )
̇ [( ) ] [( ) ]
( )

429. A nozzle receives 0.5 kg/s of air at a pressure of 2700 kPa and a velocity of 30 m/s and with
an enthalpy of 923 kJ/kg, and the air leaves at a pressure of 700 kPa and with an enthalpy of
660 kJ/kg. Determine the exit velocity from the nozzle.
A. 923 m/s B. 726 m/s C. 700 m/s D. 660 m/s
Solution:

( ) ( )

( )

430. A two-stage, double acting compressor is to deliver 90 lb/min of air from 14.3 psia to 90°F
to a final pressure of 185 psia. The normal barometer is 29.8 in-Hg and the temperature is 80°F.
The pressure drop in the intercooler is 3 psi and the speed is 210 rpm and pV 1.34 = C during
compression and expansion. The clearance is 5% for both cylinders. Find the volume of free air
if the temperature of the cooling water increased by 18°F.
A. 1282 CFM B. 1230 CFM C. 1320 CFM D. 1822 CFM
Solution:
̇ ( )( )
̇
( )
̇
̇ ̇

̇ ( )[( ) ]
̇
( ) [ - ( )]
-

431. Consider 4800 lb of steam per hour flowing through a pipe at 100 psia pressure. Assume a
velocity of 5280 ft/min. What size of pipe is required? Specific volume of steam at 100 psia υ =
4.432 ft3/lb.
A. 3 in B. 5 in C. 4 in D. 6 in
Solution:

131
( ) ( )
( )
√ ( )

( )

432. A boiler plant generates 225,000 lb of steam and burns 13.9 tons of coal per hour. The coal
has a heating value of 11,400 Btu/lb. A test of the particulates leaving the boiler shows that
3804 lb of particulate is being discharged per hour. What is the particulate discharged per
million Btu heat input to the furnace?
A. 12 lb/106 Btu B. 14 lb/106 Btu C. 15 lb/106 Btu D. 16 lb/106 Btu
Solution:

433. A turbine receives 150 lbm/s of air at 63 psia and 2450°R and expands it polytropically to
14.7 psia. The exponent n is equal to 1.45 for the process. Determine the power.
A. 52,343.16 BTU/s C. 53.343.16 HP
B. 52,343.16 kW D. 53,343.16 ft-lb/s
Solution:

( )

( )

-
( )[( ) ]
( )
̇ -

-
( )
-

434. Find the thrust and efficiency of two 2-m diameter propellers through which flows a total
of 600 m3/s of air to 11.3 N/m3. The propellers are attached to an airplane moving at 250 kph
through still air. Neglect eddy losses.
A. 36,077 N, 73% B. 77,630 N, 37% C. 66,033 N, 33% D. 77,330 N, 77%
Solution:

132
( )
( )

( )

( )

( )( )

435. A liquid with a specific gravity of 1.26 is being pumped in a pipeline from A to B. At A, the
pipe diameter is 60 cm and the pressure is 300 kN/m2. At B, the pipe diameter is 30 cm and the
pressure is 330 kN/m2. Point B is 1.0 m lower than A. Find the flow rate if the pump puts 16 kW
into the flow. Neglect head loss.
A. 4.2 m3/s B. 0.42 m3/s C. 2.4 m3/s D. 0.24 m3/s
Solution:
( )

[ ( )
] [ ( )
]

( ) ( )
- ( ) ( )

436. A reciprocating compressor handles 1,400 cfm of air measured at intake where P1 = 18 psia
and T1 = 90°F. The discharge pressure is 92 psia. Calculate the work if the process of the
compression is isothermal.
A. -180.5 hp B. -179.5 hp C. -227.6 hp D. -228.6 hp
Solution:
( )( )
̇ ( ) -
-
( )
-

437. The fuel oil has the ff. analysis:


C = 89% N2 = 2% H2 = 8% S = 1%
With 3% excess air, what is the actual amount of air needed to burn the fuel oil?

133
A. 13.17 kg air/kg fuel C. 14.47 kg air/kg fuel
B. 13.47 kg air/kg fuel D. 14.17 kg air/kg fuel
Solution:
( )
( ) ( ) ( )
( )( )

438. A pump discharges 550 gpm of water to a height of 35 ft. With an efficiency of 80%, what
is the power input?
A. 6.09 hp B. 6.32 hp C. 4.74 hp D. 4.94 hp
Solution:
( )( )( )
-
( )
-

439. A room contains air at 20°C and 96 kPa at a relative humidity of 75%. Determine the
enthalpy of moist air.
where:
(Psat @ 20°C = 2.339 kPa)
(hg @ 20°C = 2538.1 kJ/kg da )
Given:
Unsaturated air (0% < Ø <100%)
td = 20°C
Pt = 96 kPa
Ø = 75%
A. 45.919 kJ/kg da B. 45.515 kJ/kg da C. 49.515 kJ/kg da D. 41.815 kJ/kg da
Solution:

( )

( ) ( )
( ) ( ) ( )

134
440. A piston moves inside a cylinder at a velocity of 6.0 m/s. The 160 mm diameter piston is
centrally located within the 160.2 mm inside diameter cylinder. The film of oil is separating the
piston from the cylinder has an absolute viscosity of 0.4 N-s/m2. Assuming a linear velocity
profile, find the shear stress in the oil. (T = μ (v/H))
Given:
μ = 0.40 N-s/m2
v = 5 m/s
H = thickness of oil film = 160.2 – 160/2 = 0.1 mm
A. 50,000 N/m2 B. 40,000 N/m2 C. 24,000 N/m2 D. 34,000 N/m2
Solution:
-
( )

441. A centrifugal pump with a 3 ft impeller diameter operates at 800 rpm. If the speed is to be
increased to 1200 rpm, determine the impeller diameter that should be used so that the same
shaft input power would be required.
A. 5.32 ft B. 2.35 ft C. 5.23 ft D. 2.93 ft
Solution:
( )
( )
( ) ( )

( ) ( )

442. Determine the mass of water vapor contained in a 150 m3 room at 100 kPa, 23°C and 40%
relative humidity. From Steam Tables: Psat @ 23°C = 2.810 kPa.
A. 1.6342 kg B. 1.9342 kg C. 1.2342 kg D. 2.2342 kg
Solution:

( )
( )

443. What is the power of the pump, HP, if it delivers 925 gal/min of water against a head of 15
m?
A. 15.38 HP B. 16.38 HP C. 10.5 HP D. 11.5 HP
Solution:

135
[ ( )]( )( ) ( )( )( )
-
-

444. Kerosene is pumped into an aircraft fuel tank through a hose that has an inside diameter
of 4 cm. If the velocity of the kerosene is 8 m/s through the hose, determine the mass flow rate.
Assume that the kerosene has a density of 800 kg/m³.
A. 7.06 kg/s B. 7.56 kg/s C. 8.06 kg/s D. 8.56 kg/s
Solution:
( )
̇ ̇ ( )( )

445. During the working stroke of an engine the heat transferred out of the system was 150
kJ/kg of working substance. The internal energy also decreased by 400 kJ/kg of working
substance. Determine the work done.
A. 250 kJ/kg B. 550 kJ/kg C. 600 kJ/kg D. 350 kJ/kg
Solution:
( )
- (- ) -

446. During the experiment on Charles Law, the volume of the gas trapped in the apparatus is
10000 mm³ when the temperature is 18°C. The temperature of the gas was then raised to
85°C. Determine the new volume of the gas trapped in the apparatus if the pressure exerted on
the gas remained constant.
A. 12302.41 mm³ B. 8128.49 mm³ C. 70833.33 mm³ D. 2117.64 mm³
Solution:

447. Find the density of oil with a specific gravity of 1.6 in-g/cm3.
A. 15.68 g/cm3 B. 99.84 g/cm3 C. 0.8 g/cm3 D. 1.6 g/cm3
Solution:

136
448. What is the absolute pressure if the gauge pressure reading is 9 bar and the atmospheric
pressure is 0.9 bar?
A. 6.3 bar B. 7.8 bar C. 9.9 bar D. 8.1 bar
Solution:

449. The tank of an air compressor has a volume of 3 ft3 and is filled with air at a temperature of
40°F. If a gage on the tank reads 150 psig, what is the mass of the air in the tank?
A. 1.78 lbs B. 2.00 lbs C. 2.67 lbs D. 1.98 lbs
Solution:
- -

( ) ( )( )
-
( )
-

450. What is the mass of acetylene gas, V = 0.94 cu. ft., R = 59.35 ft-lb/lb-°R, T = 90°F, P = 200
psia?
A. 0.816 lb B. 0.841 lb C. 0.829 lb D. 0.852 lb
Solution:
( )( )
( )

451. Specific volume is the number of cubic meters of mixture per kilogram of dry air. If dry air
has these following properties: R a = 287 J/kg-K, T = 303 K, Pa = 99.604 kPa. Solve for the specific
volume.
A. 0.873 m3/kg B. 0.853 m3/kg C. 0.953 m3/kg D. 0.783 m3/kg
Solution:
( )

452. A refrigerating system operates on the reversed Carnot Cycle. The higher temperature of
the refrigerant in the system is 120°F and the lower is 10°F. The capacity is 20 tons. Neglect
losses. Determine the coefficient of performance.
A. 2.732 B. 5.373 C. 7.372 D. 4.273
Solution:

137
( ) ( )

453. At a pressure of 60°F, a motorbike Ire is inflated to 33 psig. As it is driven along the C -5
road, the temperature rose to 76°F. Assuming the volume remains constant, determine the
final gauge pressure.
Given:
P1 = 33 psig + 14.7 psig = 47.7 psig
T1 = 60°F + 460 = 520°R
T2 = 76°F + 460 = 536°R
A. 34.47 psig B. 49.17 psig C. 35.00 psig D. 34.30 psig
Solution:

454. Steam enters a turbine stage with an enthalpy of 3700 kJ/kg and a velocity of 80 m/s and
leaves with an enthalpy of 2864 kJ/kg with a velocity of 12.8 m/s. If the rate of a steam flow
through the turbine is 0.44 kg/s, what is the work done in kW?
A. 365 kW B. 365.64 kW C. 366.0 kW D. 366.50 kW
Solution:

( )
-
( )
-

( )
-
( )
-

455. Aluminum has a specific heat of 0.902 J/g-°C. How much heat is lost when a piece of
aluminum with a mass of 23.984 g cools from a temperature of 415.0°C to a temperature of
22.0°C?
A. 8500 J B. 6000 J C. 80000 J D. 7500 J
Solution:
138
( ) ( )( )

456. If the temperature of an air parcel is -20.5°C, and its density is 0.690 kg/m3, what is the
pressure of the air parcel?
A. 40 kPa B. 50 kPa C. 60 kPa D. 70 kPa
Solution:

457. A 35.0 mL sample of gas is enclosed in a flask at 22 degrees Celsius. If the flask was placed
in an ice bath at 0 degrees Celsius, what would the new gas volume be if the pressure is held
constant?
A. 34.1 mL B. 32.1 mL C. 32.39 mL D. 33.1 mL
Solution:

458. A car engine with a power output of 65 hp has a thermal efficiency of 24%. Determine the
fuel consumption rate of this car if the fuel has a heating value of 19,000 Btu/lb m.
A. 36.28 lbm/hr B. 37.28 lbm/hr C. 37.28 lbm/hr D. 35.30 lbm/hr
Solution:
̇
̇

̇
̇ ( )

̇ ̇ ( )

139
459. The thermal efficiency of a Carnot cycle operating between 170°C and 620°C is closest to:
A. 44% B. 50% C. 63% D. 73%
Solution:

460. Compute the humidity ratio of air at 70% relative humidity and 25°C when the barometric
pressure is 101.325 kPa. From the steam tables: Psat @ 34 °C = 3.169 kPa.
A. 0.014 kg water vapor/kg dry air C. 1.4 kg water vapor/kg dry air
B. 0.14 kg water vapor/kgdry air D. 0.0014 kg water vapor/kg dry air
Solution:

( )
( )

461. A pressure gauge registers 50 psig in a region where the barometer reads 14.8 psia. Find
the absolute pressure in kPa.
A. 666.66 kPa B. 556.66 kPa C. 446.66 kPa D. 336.66 kPa
Solution:

462. Consider 1 kg of air at 32°C that expanded by a reversible polytropic process with n = 1.25
until the pressure is halved. Determine the heat transfer. Specific heat at constant volume for
air is 0.1786 kJ/kg-K.
A. 17.02 kJ heat rejected C. 17.02 kJ heat added
B. 7.05 kJ heat rejected D. 7.05 kJ heat added
Solution:

( ) ( )

( ) ( ) - -
( ) (- )( )
-

140
463. A Carnot cycle uses nitrogen (k = 1.399) as the working substance. The heat supplied is 54
kJ and the adiabatic expansion ratio is 10. Determine the heat rejected.
A. 10 kJ B. 32.4 kJ C. 21.6 kJ D. 54 kJ
Solution:

( )
( )

464. A tank contains 20 kg of air at 200 kPa (gage) and 23°C. During the heating process the
temperature of air rises. For safety purposes a technician installed a relief-type valve so that
pressure of air inside the tank never exceeds 260 kPa (gage). At what air temperature the relief
valve will start releasing air?
A. 112°C B. 92°C C. 82°C D. 102°C
Solution:

( )
( )
( ) ( )

465. An air compressor takes in 9 kg/min of air at 98 kPa; υ 1 = 0.125 m3/kg and discharges it at
680 kPa; υ2 = 0.03 m3/kg. The increase of internal energy is 93 kJ/kg and the work done on air is
163 kJ/kg; the change in potential and kinetic energy are neglected. How much heat is
transferred per kg of air?
A. 264.15 kJ/kg B. 61.85 kJ/kg C. 288.65 kJ/kg D. 78.15 kJ/kg
Solution:
( )
( )
( ) (- )
- ( )

466. During a reversible process, there are abstracted 317 kJ/s from 1.134 kg/s of a certain gas
while the temperature remains constant at 26.7°C. For this gas c p = 2.232 and cv = 1.713 kJ/kg-K.
The initial pressure is 586 kPa. Determine the final volume flow rate.
A. 0.301 m3/s B. 0.03 m3/s C. 0.5 m3/s D. 0.05 m3/s
Solution:

141
-

̇ ̇ - ( )
̇

467. Flow of water in a pipe has a velocity at 10 meters per second. Determine the velocity
head of the water.
A. 50.1 meters B. 5.10 meters C. 8.20 meters D. 100 meters
Solution:

( )

468. A Diesel cycle has a cut-off ratio of 2.20 and a compression ratio of 10. Find the cycle
efficiency.
A. 55.10% B. 59.735% C. 52.23% D. 62.37%
Solution:
[ ( )
] [ ( )
]

469. A diesel cycle has an initial temperature of 27°C. If the cut-off ratio is 2.50 and
compression ratio is 12, find the maximum cycle temperature.
A. 1634.4°C B. 1753.44°C C. 2010.3°C D. 1983.4°C
Solution:
( )

( )

470. A diesel cycle, which takes in air at 1 bar and 26°C, has a compression ratio of 19. Calculate
the operating clearance in percent.
A. 8.08 B. 8.56 C. 7.52 D. 5.55
Solution:

142
471. An Otto cycle has an initial pressure of 100 kPa and has a pressure of 400 kPa after
adiabatic compression. Find the cycle efficiency.
A. 32.70% B. 34.70% C. 36.70% D. 38.70%
Solution:

( ) ( )

472. An Otto cycle has a clearance of 8% and heat added of 1000 kJ. Find the heat rejected.
A. 564 kJ B. 353 kJ C. 709 kJ D. 867 kJ
Solution:

( )

473. An Otto cycle has a heat rejected of 300 kJ and work of 700 kJ. Find the cycle efficiency.
A. 56% B. 60% C. 70% D. 50%
Solution:

474. An Otto cycle has a pressure ratio of 7. What is the cycle compression ratio?
A. 5.18 B. 6.34 C. 7.34 D. 4.01
Solution:

( )

475. Find the power of a rotating shaft which develops a torque of 188 N-m at 1350 rpm.
A. 101.54 hp B. 53.63 hp C. 63.35 hp D. 35.63 hp
Solution:
( )( ) ( )

143
476. Determine the pressure exerted on a diver at 30 m below the free surface of the sea.
Assume a barometric pressure of 101 kPa and the specific gravity of sea water is 1.03.
A. 404 kPa B. 410 kPa C. 420 kPa D. 430 kPa
Solution:
( )( )

477. An air compressor has an inlet enthalpy of 35 Btu/lb and an exit enthalpy of 70 Btu/lb. The
mass flow rate of air is 3 lb/s. If the heat loss is 466.62 Btu/min, find the work input to the
compressor.
A. 139.59 hp B. 149.59 hp C. 159.59 hp D. 169.59 hp
Solution:
̇ ̇
- ( )( ) ̇

̇ - ( ) -

478. An automobile tire is inflated to 35 psig at 54°F. After being driven, the temperature rises
to 80°F. Determine the final gage pressure assuming that the tire is inflexible.
A. 36.51 psig B. 37.51 psig C. 38.51 psig D. 39.51 psig
Solution:

( ) ( )( )

479. A condenser vacuum gauge reads 600 mm Hg when the barometer reads 760 mm Hg.
What is the absolute condenser pressure in bar?
A. 0.0213 B. 0.061 C. 0.213 D. 0.610
Solution:
( ) ( )

480. Water flows in a pipe at the rate of 10 kg/s. If the velocity of flow is 10 m/s, find the pipe
diameter.
A. 30.23 mm B. 35.68 mm C. 38.39 mm D. 42.39 mm
Solution:
̇
144
( )( )

481. What is the resulting pressure when one kilogram of air at 104 kPa and 98°C is heated at
constant volume to 450°C?
A. 202.67 kPa B. 194.67 kPa C. 186.53 kPa D. 198.65 kPa
Solution:

( )

482. Determine the degrees of superheat of steam at 101.325 kPa and 170°C.
A. 50°C B. 70°C C. 60°C D. 80°C
Solution:

483. Calculate the approximate enthalpy of water at 90°C.


A. 366.83 kJ/kg B. 376.83 kJ/kg C. 386.83 kJ/kg D. 396.83 kJ/kg
Solution:
( ) ( )

484. A Carnot cycle operates between 30°C and 350°C. Find the cycle efficiency.
A. 51.36% B. 63.45% C. 45.37% D. 76.45%
Solution:

485. A Carnot cycle has a maximum temperature of 550°F and minimum temperature of 100°F.
If the heat added is 4200 Btu/min, find the horsepower output of the engine.
A. 34.53 B. 40.56 C. 44.13 D. 65.40
Solution:

̇ ̇ ( ) ( )

486. A Carnot cycle has a sink temperature of 100°F and a cycle efficiency of 70%. Find the
temperature of the heat souce.

145
A. 1306.70°F B. 1406.70°F C. 1506.70°F D. 1606.70°F
Solution:

487. The quality of steam is 20%. This means that:


A. mass of liquid is 20%, mass of vapor is 80%
B. mass of liquid is 20%, mass of vapor is 0%
C. mass of liquid is 80%, mass of vapor is 20%
D. none of the above
Solution:

488. Fifty kilograms of cooling water per second enter the condenser at 25°C and leaves at
50°C. Find the heat carried away by water.
A. 1234.45 kW B. 5233.75 kW C. 2340.53 kW D. 3140.25 kW
Solution:
̇ ̇ ( )( )

489. Ten kilograms per second of steam enter the turbine with an enthalpy of 3200 kJ/kg and
enter the condenser with an enthalpy of 2500 kJ/kg in a Rankine cycle. If the turbine efficiency
is 80% and the generator efficiency is 90%, determine the power plant output.
A. 4320 kW B. 3213 kW C. 4056 kW D. 5040 kW
Solution:
̇ ( )( )( )

490. Determine the quality of steam in a vessel containing 2 kg of saturated vapor and 8 kg of
saturated liquid.
A. 100% B. 20% C. 80% D. 60%
Solution:

491. The condenser of a reheat power plant rejects heat at the rate of 600 kW. The mass flow
rate of cooling water is 5 kg/s and the inlet cooling water temperature is 35°C. Calculate the
condenser cooling water exit temperature.
146
A. 43.45°C B. 53.45°C C. 63.66°C D. 74.34°C
Solution:
( )( )

492. A heat engine has a thermal efficiency of 50%. How much power does the engine produce
when heat is transferred at a rate of 10 9 kJ/hr?
A. 50 MW B. 75 MW C. 139 MW D. 147 MW
Solution:
̇ ( )
̇ ( )

493. One kilogram of air is compressed adiabatically and in a steady-flow manner. The
compression efficiency is 80% and the work done on the air is 265 kJ/kg. Compute the heat.
A. 212 kJ/kg B. 100 kJ/kg C. 0 kJ/kg D. 331.25 kJ/kg
Solution:
( )

494. Three-hundred kilojoules of heat flow by conduction from the outside to the inside of a
cold storage in one hour. If the temperature and all other conditions are the same, what is the
heat flowing through the cold storage room in two hours?
A. 600 kJ B. 900 kJ C. 300 kJ D. 1,200 kJ
Solution:
( )

495. Determine the density of air at 760 mmHg absolute and 22°C?
A. 1.014 kg/m3 B. 1.316 kg/m3 C. 1.197 kg/m3 D. 1.266 kg/m3
Solution:

( )( )
-

496. A refrigerating machine that is classified as a one-ton machine has the capacity to produce
a cooling effect of?
A. 3.516 kW B. 12,000 Btu/hr C. 211 kJ/min D. All of the above
Solution:

147
IPE PREVIOUS BOARD PROBLEMS

AIR COMPRESSOR

ME BOARD (April 1995)


1. A single air compressor handles 0.454 m 3 / sec of
atmospheric pressure , 27  C air, and delivers it to a receiver at
652.75 kPa . Its volumetric efficiency is 0.72, its compression
efficiency on an isothermal basis is 0.85 and its mechanical
efficiency is 0.90. If it rotates at 350 rpm, what power in kw is
required to drive it?

SOLUTION:

Isothermal power = P1V1 ln P2 / P1

= 101.3 (0.454) ln (625.75 / 101.4)

= 85.685 kw

drive power = 85.685


0.85(0.9)

=112 kw

ME Board( October 1992)


2. An air compressor takes air at 100 Kpa and discharges to
600 Kpa. If the volume flow of discharge is 1.2 m3 per
seconds, determine the capacity of air compressor.

SOLUTION:
P1V1n = P2 V2 n
N = 1.4 (for standard air)

100(V1)1.4 = (600) (1.2) 1.4


V1 = 4.315 m3 m3/sec
ME BOARD (April 1995)
3. An ammonia compressor has a clearance of 5% and a
pressure ratio of 5. Determine the volumetric efficiency if k for
ammonia is 1.304.

SOLUTION:

nv = 1 + c – c ( P2 / P1 ) 1/k

= 1 + 0.5 – 0.5 ( 5 ) 1/ 1.304

= 0.8782

= 87.82 %

ME Board (April 1987)


4. The piston displacement of a double acting air
compressor is 0.358
m3/sec, delivers gas from 101.325 Kpa and 300°K to 675 Kpa at
the rate of 0.166 m3/sec at 150 rpm. Value of n for compression
and expansion is 1.33. Find the compressor percent clearance.

Solution:

ηv = V1 / VD
ηv = 0.166 / 0.358
ηv = 0.4637

ηv = 1 + c – c ( P2 / P1 ) 1/n
0.4637 = 1 + c – c ( 675 / 101.325) 1/1.33

c = 0.1696
c = 16.96 %

ME Board (April 1995)


5. A single stage air compressor handles 0.454 m 3 /sec of
atmospheric pressure, 27°C air, and delivers it to a receiver at
652.75 Kpa. Its volumetric efficiency on an isothermal basis is
0.85 and its mechanical efficiency is 0.90. If it operates at 350
rpm, what power in KW is required to drive it?

Solution:
For isothermal process.
W = P1V1 ln ( P2 / P1 )
W = 101.3 ( 0.454 ) ln(652.75 / 101.3)
W = 85.685 KW

Power Input = __85.685__


0.85 (0.90)

Power Input = 112 KW

ME BOARD (1999)
6. A vapor compression refrigeration system is designed to
have a capacity of 100 tons. Its actual COP is 5.86 and 35% if
the power supplied to the compressor is lost in the form of
friction and cylinder cooling losses.

SOLUTION:

QA = 100 TOR = 351.6 kw

COP = Q A / WC

5.86 = 351.6 / WC

WC = 60 kw

MOTOR POWER = __60__


1-0.35

=92.3 kw

ME Board (October 1992)


7. A 10 hp motor is used to driven an air compressor. The
compressor efficiency is 75%. Determine the compressor
work.

Solution:
ηc = W / Brake Power
0.75 = W / 10 hp x O.746

W = 5.59 KW
ME BOARD (April 1997)
8. An ideal single stage air compressor without clearance
takes in air at 100 kpa with a temperature of 16  C and
delivered it at 413 kpa after isentropic compression. What is
the discharge work done by the compressor in kJ/kg?

A . –59.22 C. –72.5
B. –118.44 D. –145

SOLUTION:

W = -kmRT1 P2 k-1
k -1 k -1
P1

W = -kmRT1 P2 k-1
m k -1 k -1
P1

W = -145 kJ/kg
m

ME BOARD (April 1995)


9. A single stage air compressor handles 0.44 m3/s of
atmospheric pressure, 27 C air, and delivers it to receiver at
652.75 kpa. Its volumetric efficiency is 0.72, its compression
efficiency is 0.90. if it rotates at 350 rpm, what power in kW is
required to drive it?

A. 95 C. 120
B. 112 D.
100

SOLUTION:

Drive Power = Isothermal Power


ec em
Where:

Isothermal Power = P1 V1 In ( P2/P1)

Isothermal Power = 101.325 (0.454) In (652.725/101.325)

Isothermal Power – 85.694 kW


ec = 0.85
em = 0.90
thus;

Drive Power = 85.694 = 112.02 kW


(0.85) (0.90)

ME BOARD (April1997)
10. A single acting air compressor operates at 150 rpm with
an initial condition of air at 97.9 kpa and 27  C and discharges
the air at 379 kpa to a cylindrical tank. The bore and stroke
are 355mm and 381 mm respectively with 5% clearance. If the
surrounding air is at 100 kpa and 20  while compression and
expansion process PV1.3 = C, determine the free air capacity
in m3/s?

A . 0.0819 m3/s C.
0.0943 m3/s
B. 0.0856 m3/s D. 0.0642
m3/s

SOLUTION:

Pf Vf = P1 V1
Tf T1

Where:

Vd =  D2 LN =  (0.355 )2 (0.381)(150/60)
4 4

= 0.0943 m3/s

ev = 1 + c – c (P2/P1) 1/n = 1 + 0.05 – 0.05 (379/97.9) 1/1.3

= 0.908

V1 = 0.908 (0.0943)
= 0.0856 m3/s

then;
100 Vf = 97.9 (0.0856)
(20+273) (27 + 273)

thus;
Vf = 0.0819 m3/s
ME BOARD (Oct. 1997)
11. Compute the volume of air drawn per minute in a single-
stage compressor that takes in 0.90 kg of air at atmospheric
pressure and 24 C and compresses it to 10 atmospheres of
pressure, if the law of compression is PV1.2 = C. use the
specific volume of air at 0 C = 0.7734 m3/kg?

A. 0.7571 m3 C. 0.5551m3
B. 0.6561 m3 D. 0.8581m3

SOLUTION:

Volume of 0.90 kg of atmospheric pressure and 24C

V = 0.7734 ( 0.90 ) ( 24 + 273 )


( 1 ) ( 0 + 273 )

V = 0.7571 m3

ME BOARD (April 1996)


12. A single stage singe acting air compressor, running at 5
rev/s, has air intake of 22.8 liters at atmospheric pressure and
25 C. if it compresses 5.7 liters in accordance with the
compression law PV1.2 = C, calculate the temperature of the
air at the end of compression?

A. 120K C. 115K
B. 110K D. 125K

SOLUTION:

T2 = ( V1 ) n-1
T1 V2

T2 = ( 22.8 ) 1.2 – 1
25 + 273 5.7

T2 = 393.21 K
Thus; the temperature at the end of compression:
= T2 – T1 = 393.21 – 298
= 393.1K
t = 120.1 C

ME BOARD (April 1991)


13. An air compressor takes air at 90 kpa and discharges to
700 kpa. If the volume flow of discharge is .3 m3/s, compute
the capacity of the air compressor.

A. 5.63 m3/s C. 5.98 m3/s


B. 4.63 m3/s D. 4.98 m3/s

SOLUTION:
P1 V1 n = P2 V2 n
90V1 1.4 = 700 (1.3)1.4

V1 = 5.63 m3/s

ME BOARD (April 1995)


14. A single stage air compressor handles 0.454 m3/sec of
atmospheric pressure, 27 C air, and delivers it to a receiver at
65275 kPa. Its volumetric efficiency is 0.72, its compression
efficiency on an isothermal basis is 0.85 and its mechanical
efficiency is 0.90. If it rotates at 350 rpm, what power in kw is
required to drive it?

a. 95 c. 120
b. 112 d. 100

SOLUTION:

Wi =isothermal power = P1V1 ln (P2/P1)


= 101.3 (0.454) ln (652.75/101.3)
= 85.685 kw

Drive Power = 85.685/0.85(0.90)


= 112. kw

ME BOARD (April 1995)


15. An air compressor is to compress 8.5 m3/min from 98.56
kPa to 985.6 kPa. Assuming conditions ideal, and with n = 1.3,
what will be the saving in work due to two staging?

a. zero c. 5.6 kw
b. 4.6 kw d. 3.5 kw
SOLUTION:
n 1
 
nP1V1 '  P2  n 
Work of Single Stage = n  1 

P

  1

 1  

 8.5 
(1.30)(98.56)   1.30  1

=  60   985. 6  1.3

  1
1.30  1  98.56  
 
= 42.43 kw
For two stage

Px = P1 P2 = (98.56)(985.6) = 311.67 kPa

n 1
 
2nP1V1 '  Px  n 
Work for Two-Stage = n  1 
 
P1 
 1

   

 8.5 
2(1.3)(98.56)   1.3 1

=  60   311 .67 
1.3
 1
1.30  1  98.56  
 

= 36.83 kw

Saving = 42.43 – 36.83 = 5.6kW


Cooling tower

1. Water at 55 ⁰C is cooled in a cooling tower which has an efficiency


of 65%. The temperature of the surrounding air is 32 ⁰C dry bulb and 70%
relative humidity. The heat Dissipated from the condenser is 2,300,000
kj/hr. Find the capacity in liters per second of the pump used in the cooling
tower.

a. 8.50 L/s c. 7.60 L/s


b. 6.80 L/s d. 6.70 L/s

SOLUTION:

Pump Capacity = m vf @ t4

Solving For M;
E = t3 – t4
t3 – twb

From: Psychometric Chart


@ 32 ⁰C & 70% RH
twb = 27.40 ⁰C

0.65 = 55 – t4 = t4 = 37.06 ⁰C
55 – 27.40
Using energy Balance in the condenser
Mcp (t3-t4) = 2,300,000
3600

m (4.187)(55-37.06) = 2,300,000
3600
m = 8.51 kg/s
From steam table : @ t4 = 37.06 ⁰C
vf = 1.0068 L/kg

Pump Capacity = (8.51kg/s)(1.0068 L/kg)


Pump Capacity = 8.57 L/s

2. An Atmospheric cooling tower is to provide cooling for the jacket


water of a four stroke, 800kw Diesel generator. The cooling tower efficiency
is 60% at a temperature approach of 10 ⁰C. If the ambient air has a relative
humidity of 70% and dry bulb temperature of 32 ⁰C, determine the cooling
water supplied to the diesel engine in L/hr. η gen = 97% useful work = 30% 7
cooling loss = 25%.

a. 39,800 L/hr c. 45,700 L/hr


b. 35,700 L/hr d. 49,800 L/hr

SOLUTION:

Vol of water = m Vf @ t4;


Solving for m & vf;
@ tdb1 = 32 ⁰C & RH = 70%
twb = 27.45 ⁰C
Tapproach = t4 – 27.45
10 = t4 – 27.45
t4 = 37.45 ⁰C

Brake Power of Engine = Power Input to Generator


800/0.97 = 824.47 Kw = Power Input to Generator

Heat Supplied to Engine, Qa


Qa = 824.74/0.30 = 2749.14 Kw

Heat Absorbed by Cooling water, Qw:


Qw = 0.25 (2749.14)
Qw = 687.285
mw cpw (t3-t4) = 687.285
mw (4.187) ( 52.4 - 36.45) = 687.285
mw = 10.98 kg/s
mw = 39,527.14 kg/hr

Specific Volume of water @ 37.45 ⁰C , Vf = 1.007L/kg

Thus, Vol of Cooling water, Vw


Vw = 39,527 (1.007)
Vw = 39.803.83 L/hr

3. A 250,000 kg/hr of water at 35 ⁰C enters a cooling tower where it is to


be cooled to be cooled to 17.5 ⁰C. The energy is to exchanged with
atmospheric air entering the units at 15 ⁰C and leaving the unit of 30 ⁰C.
The air enters at 30% RH and leaves at 85% RH. If all process are assumed
to occur at atmospheric pressure , determine the percentage of total water
flow that is make up water.

c. 2.22% c. 4.44%
d. 3.33% d. 1.11%

SOLUTION:

Make-up water = Mass of make-up


250,000
Solving for Make up Water
@ 15 ⁰C & 30% RH
h1 = 23.02 Kj/Kg
w1 = 0.0033Kg/Kg
@ 30 ⁰C & 85% RH
h2 = 89.01 Kj/Kg
w2 = 0.0233 Kg/Kg

Heat Lost By Water = Heat Gain By Air


MwCpw∆T = Ma (h1 – h2)
250,000(4.187)(35-17.5) = ma(89.01-23.02)
ma = 277,589.41 kg/hr

Then, the mass of make-up water


M = ma(w2-w1)
M = 277,589.41 (0.0233 – 0.0033)
M = 5,551.79 kg/hr

Thus, Percentage make-up water


% Make up Water = 5,551,79kg/hr
250,000 kg/hr
% Make up Water = 2.22%

4. Eleven thousand three hundred kilograms per hour of water enters a


cooling tower at 45 ⁰C. Atmospheric air at 16 ⁰C and 55% relative humidity
enter the tower at the rate of 10,200 m³/hr and leaves at 32 ⁰C and
saturated. Determine the mass of water evaporated per hour during the
cooling process?

e. 2,192.53 kg/hr c. 2,291.53 kg/hr


f. 3,192.53 kg/hr d. 3,291.53 kg/hr

SOLUTION:

Mass of water evaporated, mw: mw = ma (sh2 = sh1)


From Psychometric chart:
Entering air at 16 ⁰C and 55% Rh
V1 = 0.83 m³/kg ; sh1 = 0.07 kg/kg
Leaving air at 32 ⁰C and 100% RH saturated
sh2 = 0.037 kg/kg
Mass of air entering the tower;

ma = 10,200 m³/hr
0.83m³/kg

ma = 12,289.16 kg/hr
Then:
mw = 12,289.16(0.307 – 0.07)
mw = 2,192.53 kg/hr

5. How much refrigeration capacity is required to cool 56.67m³ of air


per minute from 29 ⁰C to 21 ⁰C. Assume that the cooled air is saturated.

a. 4.10 L/s c. 2.10 L/s


b. 3.10 L/s d. 5.10 L/s

SOLUTION:

Ref. Capacity, Qa ;
Qa = ma (h1-h2)

Solving for ma;


From psychometric chart:
@ 21 ⁰C DB & 100% Rh
h1 = 70 kj/kg
v1 = 0.875 m³/kg
@ constant SH intersecting 29 ⁰C DB
h2 = 70kj/kg

ma = volume of air
specific volume
ma = 56.67 m³/min
0.875 m³/kg

ma = 64.766 kg/min = 1.079 kg/s

then:
Qa = 1.079 (70 – 61)
Qa = 9,715 Kw

6. Fifty Gallons per minute of water enters a cooling tower at 46 ⁰C.


Atmospheric air at 16 ⁰C. db and 55% RH enters the tower at 2.85 m³/s and
leaves at 32 ⁰C saturated. Determine the volume of water that leaves the
tower.

SOLUTION:

Volume water leaving the tower, V4


V4 = m4 ( vf @ t4)
Solvinf for m4 and vf:
@ tdb1 = 16 ⁰C and 55% RH
v1 = 0.828 m³/kg
@ tdb2 = 32 ⁰C and 100% RH
w2 = 0.0308 kg/kg
h2 = 110.9 kj/kg
h3 = hf @ 46 ⁰C = 192.62 kj/kg and v3 = 0.0010103
m³/kg
w1 = 0.0056 kg/kg
h1 = 32 kj/kg

ma = v1/υ1 = 2.85/0.828
ma = 3.44 kg/s
m3 = (50 gal/min)(3.785 L/gal)(1 min/gal)
(0.0010103 m/kg) ( 1000 L/m³)
m3 = 3.12 kg/s

By mass balance:
M3 – m4 = ma (W2-W1)
3.12 – m4 = 3.44 (0.0308 – 0.0056)
m4 = 3.09 kg/s

By energy Balance:
m3h3 – m4h4 = ma (h2-h1)
3.12 (192.62) – 3.09h4 = 3.44 (110.9 – 32)
h4 = 106.65 kj/kg

From Steam Table 1:


T4 = 25.42 ⁰C
υ4 = 1.0031 L/kg

Thus, volume of water leaving; Vw


Vw = 3.09 (1.0031)
Vw = 3.10 L/s

7. An air conditioned theater is to be maintained at 80 ⁰F dry bulb


temperature and 50% RH. The calculated total sensible heat load in the
theater is 620,000 BTU/hr, and the latent heat load is 210,000 Btu/hr. The air
mixture at 84 ⁰F dry bulb temperature and 72 ⁰F wet bulb temperature is
cooled to 63 ⁰C and 59 ⁰C wet bulb temperature by the cooling tower and
delivered as a supply air to the theater. Calculate the tons of refrigeration
required.

a. 100.65 TOR c. 142.67 TOR


b. 124.67 TOR d. 112.60 TOR

SOLUTION:
Conditioner Capacity, Qa:
Qa = ms (h4 – h1)

Solving for ms:


Qt = ms (h2 – h1)

From Psychrometric chart:


h4 = 35.82 Btu/hr
h1 = 25.78 Btu/hr
h2 = 31.35 Btu/hr
then,
620,000 + 210,000 = ms (31.35 – 25.78)
ms = 149, 012.57

thus;
Qa = 149,012.57 (35.82 – 25.78) btu/hr
12,000Btu/hr
TOR
Qa = 124.67 TOR

8. Determine the partial pressure of water vapor if the barometric


pressure is 101.325 kpa and the humidity ratio is 0.05.

a. 7.54 kpa c. 5.74 kpa


b. 4.75 kpa d. 5.47 kpa

SOLUTION:
W = 0.622 x Pv / (Pt – Pv)

0.05 = 0.622 x Pv / ( 101.325 – Pv)


thus,
Pv = 7.54 kPa

9. Determine the absolute humidity (vapor density) of an air sample


that
has a dew point temperature of 45 ⁰F if the value of the gas constant R for
low pressure water vapor is 85.66 ft-lb/lbm ⁰R. The vapor pressure
corresponding to a saturation temperature of 45 ⁰F is 0.1475 psia.

a. 0.000491 lb/ft³ c. 0.000149 lb/ft³


b. 0.000941 lb/ft³ d. 0.000194 lb/ft³

SOLUTION:

PV = mRT

m/v = ρ = P/RT

ρ = 0.1475 (144)
85.66(45+460)

ρ = 0.000491 lb/ft³

10. Air normal atmospheric pressure has a temperature of 70 ⁰F and a


dew point temperature of 50⁰F. Determine the saturation ratio of the air. The
humidity ratios corresponding to dew point temperatures of 50 ⁰F and 70 ⁰F
respectively are 0.00763 lb/lb and 0.01576 lb/lb respectively.

a. 58.51% c. 38.31%
b. 28.21% d. 48.41%

SOLUTION:

saturation ratio = Wactual x 100


Wsat

Saturation ratio = 0.00763 x 100


0.01576
thus, Saturation ratio = 48.41%

11. The evaporative condenser of an ammonia refrigeration plant has a


water flow rate of 266 kg/s and enters a natural draft cooling tower t 40 ⁰C.
The water is cooled to 29⁰C by air entering at 38 ⁰C db and 24 ⁰C wb. The air
leaves the tower as saturated at 40⁰C. Calculate the make-up water
required in kg/hr.

i. 8977 C. 8055
ii. 8055 D. 8388

SOLUTION:

Water properties: Air Properties:


@ 49⁰C; hf = 167.48 kj/kg @ 38 ⁰C db and 24⁰C
wb
@ 29⁰C; hg = 121.43 kj/jg h = 72.5 kj/kg
w = 0.013 kg/kg
@ 40C db saturated
h = 166 kj/kg, w = 0.0488 kg/kg

Solving for mass of air, ma: .


Heat absorbed by air = heat rejected by water
Ma (h2-h1) = mw cw T
Ma (166 - 72.5) = 126 (4.187) (40 – 29)
Ma = 62.07 kg/s

Then;
m = 62.07 (0.0488 – 0.013)
m = 2.22 kg/s
thus,
m = 2.22kg/s x 3600s/hr
m = 7999.08
12. Determine the approximate load on a cooling tower if the entering
and leaving temperatures are 96F and 88F, respectively and the flow rate of
the water over the tower is 30 gpm

a. 2500 btu/min c. 3000


btu/min
b. 2000 btu/min d. 3500
btu/min

SOLUTION:
Tower Load = 8.33 V T btu/min
Tower Load = 8.44 (30) (96 – 88)
Thus,
Tower Load = 2000 Btu/min

13. Determine the quantity of water lost by bleed-off if the water flow rate
over the tower is 30gpm and the range is 10F. Percent bleed-off required is
33%.

a. 0.077gpm c. 0.099 gpm


b. 0.088 gpm d. 0.066 gpm

SOLUTION:
The quantity of water lost by bleed-off,

Vw = 0.33 x 30
Vw = 0.099 gpm

14. Determine the equivalent standard air volume for 150 m³/s of air
having a dry bulb temperature of 15C.

a. 150 m³/s c. 170 m³/s


b. 160 m³/s d. 180 m³/s

SOLUTION:

Vs = 150 (21 + 273)


(15 + 273)

thus,
Vs = 153 m³/s
15. 50 GPM of water enters a cooling tower at 46 oC. Atmospheric air
at 16 C db and 55% RH enters the tower at 2.85 m3/s and leaves at 32oC
o

saturated. Other data: a) mass of h 2o entering – 3.122 kg/s; b) specific


humidity of air entering – 0.0056 kg/kg; c) specific humidity of air leaving =
0.0307 kg/kg; d) mass of air entering – 3.442 kg/s; e) specific volume of h 2o
leaving – 0.0010031 m3/kg. Determine the volume of h 2o that leaves the
tower in Li/s.

SOLUTION:

M3 = V3 = 50 GPM x 3.785 Li/gal x 1 min/60s = 3.12 kg/s


V3 0.0010031 m3/kg x 1000Li/m3

Mass balance:
m3 = m4 = ma (w2 – w1)
3.12 – m4 = 3.44 (0.0308 – 0.0056)
m4 = 3.09 kg/s

Energy balance:
m3h3 – m4h4 = ma (h2 – h1)
0.12 (192.62) – 3.09 (h4) = 3.44 (110.9 – 32)
h4 = 106.65 kJ/kg

V4 @ T4 (25.5oC) = 1.0031 Li/kg


Vw = 3.09 (1.0031) = 3.10 Li/s

16. The amount of water carried by air in cooling tower is 15 lb/min. the
change in humidity ratio in outlet and inlet is 0.025 lb/lb. Determine the
volume flow of air needed in specific volume is 13 ft 3/lb.

SOLUTION:

Volume flow rate of air = mv


Mw = ma (w2 – w1)
Ma = mw / (w2 – w1)
Ma = 15 / 0.025 = 600 lb/min
Volume flow rate of air = 600 lb/min x 13 ft3/lb
Volume flow rate of air = 7800 cfm

17. A certain sample of air has a temperature of 70 ⁰F (partial pressure


of 0.36 pisa) and a dew point temperature of 50 ⁰F. The partial pressure of
the water vapor corresponding to a 50⁰F dew point temperature is 0.178
psia. Determine the relative humidity RH.

a. 49.44% c. 39.44%
b. 59. 44% d. 69.44%

SOLUTION:
RH = Actual partial pressure x 100
Partial pressure at sat.

RH = 0.178 x 100
0.36

RH = 49.44%

Dryer

1. The density of air at 35 C and 101 kPa is 1.05 kg / m3.


The humidity
ratio is: (Oct 96)

SOLUTION:
w= Pv
Pt – Pv
PV = mRT
P = mRT
V
Pair = 1.08 (0.287) (35 + 273)
= 95.48 kPa
Pt = Pair + Pvapor
101 = 95.48 + Pv
Pv = 5.53 kPa

w = 0.622 Pv = 5.53
Pt - Pv 101 – 5.53
= 0.036 kg vapor / kg dry air

2. If the sensible heat ratio is 0.80 and the cooling load is


100kW, what is the amount of sensible heating? (Apr. 94)

SOLUTION:

SHR = sensible hat ratio

0.80 = QS
100

QS = 80kW
3. Calculate the specific volume of an air-vapor mixture in
cubic meters per kilogram of dry air when the following
condition prevail: t = 30 C, w = 0.015 kg, and PL = 90 kPa. (Oct
95)

SOLUTION:
V= RaT
Pt - Pv

w = 0.622 Pv
Pt - Pv
0.015 = 0.622 Pv
90 - Pv
Pv = 2.12 kPa

V = 0.287 (30 + 273)


90 – 2.12
= 0.99 m3 / kg
4. Compute the humidity ratio of air at 62% relative
humidity and 34 C when the barometric pressure is 101.325
kPa. (Apr 95)

SOLUTION:
w = 0.622 Pv
Pt - Pv
Psat @ 34 C = 5.32 kPa
Pv = (RH) PSat
= (0.62) (5.32)
= 3.30 kPa

w = 0.622 3.30
101.325 – 3.30

= 0.021 kg Vapor / kg dry air

5. Copra enters dryer containing 60% water and 40% of


solid sand leaves with 5% water and 95% solids. Finds the
weight of water removed based on each pound of original
product.

a. 0.58lb
b. 0.40lb
c. 0.47lb
d. 0.67lb

SOLUTION:

Let; m = weight of original product per lb of wet feed


Solid in wet feed = solid in dried product

0.95m = 0.40 (1)

m = 0.42lb

Thus;

Weight of water removed = 1 – 0.42


Weight of water removed = 0.58lb

6. A room being air-conditioned is being held at 25 C dry


bulb and 50% relative humidity. A flow rate of 5 m3 / s of
supply air at 15 C dry bulb and 80% relative humidity is being
delivered to the room to maintain that steady condition. What
is the sensible heat absorbed from the room air in kW? (Oct.
99)

SOLUTION:

PV = mRT
100 (5) = m (0.287) (15 + 273)
m = 6.049 kg / s

Qs = sensible heat
= mCp (t2 – t1)
= 6.049 (1.003) (25 – 15)
= 60.8 kW

7. A mechanical draft dry cooling tower cools the cooling


water from 60 C to 25 C at the rate of some 149.4 giga grams
per hour. Atmospheric air enters the tower at 20 C and leaves
at 35 C. The fan is driven by a 7460 kW motor. What is the
mass flow rate of the air into the cooling tower in kg per
second? (Apr. 96)

SOLUTION:
mw = 149.4 x 109 = 41,500 kg / s
1000 (3600)

Heat Loss by water = Heat gain by air


(mCp∆t) water = (mCp∆t) air
41,500 (4.187) (60 – 25) = ma (1.006) (35 – 20)
ma = 403,023 kg / s
8. What is the enthalpy of the air-vapor mixture at 65%
relative humidity and 34 C when the barometric pressure is
101.3 kPa? (Oct 96)

GIVEN:
Psat at 34 C = 5.318 kPa
hg at 34 C = 2563.6 kJ / kg

SOLUTION:
Pv = RH x Psat
= 0.65 x 5.318 = 3.4567 kPa

W = 0.622 Pv = 0.622 3.4567


P – Pv 101.3 – 3.4567

= 0.022 kg / kg

9. A coil has an inlet temperature of 60 F and outlet of 90 F.


If the mean temperature of the coil is 110 F, find the bypass
factor of the coil. (Oct. 99)

SOLUTION:

Bypass factor = t mean – t db of outlet air


t mean – t wb of inlet air

BF = 110 – 90
110 – 60

= 0.40
10. If the latent and sensible heat loads are 20 kW and 80
kW respectively, what is the sensible heat ratio? (Apr. 99)

SOLUTION:

Let: SHR = sensible heat ratio

SHR = Qs
Qs + Qs
80
80 + 20

= 0.80

11. Compute the pressure drop of 30 C air flowing with a


mean velocity of 8 m / s is a circular sheet-metal duct 300 mm
in diameter and 15 m long. Use a friction factor, f = 0.02, and
ρair = 1.1644 kg / m3 (Apr 96)

SOLUTION:
2
∆P = fLVρ
2D 2
= 0.02 (15) (8) (1.1644)
2 (0.30)

= 37.26 Pa

12. A pressure difference of 350 Pa is available to force 20 C


air through a circular sheet-metal duct 450 mm in diameter
and 25 m long. At 20 C, ρ = 1.204 kg / m3 and take friction
factor, f = 0.016. Determine the velocity. (Oct 96)
SOLUTION:
2
∆P = fLVρ
2D
2
350 = 0.016 (25) (V) (1.204)
2 (0.30)

V = 25.57 m / s

13. A duct 0.40 m high and 0.80 m wide suspended from the
ceiling in a corridor makes a right angle turn in the horizontal
plane. The inner radius is 0.2 m and the outer radius is 1.0 m
measure from the same center. The velocity of air in the duct
is 10 m / s. Compute the pressure drop in this elbow.
Assuming; f = 0.3, ρ = 1.204 kg / m3 and L = 10 m. (Apr 99)

SOLUTION:
2
∆P = fLVρ
2Deq
Deq = 2ab (for rectangular duct)
a+b
= 2 (0.4) (0.80) = 0.53 m
0.4 + 0.8
2
∆P = 0.3 (10) (10) (1.2041)
2 (0.53)

= 341 kPa
14. A rectangular duct has dimensions of 0.25 m by 1 m.
Determine the equivalent diameter of the duct. (Apr 96)

SOLUTION:

Deq = 2ab
a+b
Deq = 2 (0.25) (1)
0.25 + 1

= 0.40 m

15. A sudden enlargement in a circular duct measures 0.20 m


diameter upstream and 0.40 m downstream. The upstream
pressure is 150 Pa, downstream pressure is 200 Pa. What is
the flow rate of 20 C air through the fitting? Use ρ = 1.02041
kg / m3 (Oct 96)

SOLUTION:

Q = Au Vu 0.20 m
2 2 Vu
Vd
Ploss = Vu ρ 1 – Au 0.40
m
2 Ad
2
Au = 0.20 = 0.25
Ad 0.40
2 2
(200 – 150) = Vu (1.204) (1 – 0.25) ; Vu = 12.15 m /s
2 2
Q = π (0.20) (12.15) = 0.38 m3/ s
4
16. Wet material containing 215% moisture (dry basis) is to
be dried at the rate of 1.5 kg / s in a continuous dryer to give a
product containing 5% moisture (wet basis). The drying
medium consists of air heated to 373 K and containing water
vapor equivalent to a partial pressure of 1.40 kPa. The air
leaves the dryer at 310 K and 70% saturated. Calculate how
much air will be required to remove the moisture. (Apr 97)

SOLUTION:
amount of moisture removed = amount of moisture absorbed by
air
m = rate of flow of dried product.

1 (1.5) = 0.95 m ; m = 0.501 kg / s


1 + 2.15
amount of moisture removed = 1.5 – 0.501
= 0.999 kg / s

W1 = 0.622Pv = 0.622 (1.4)


Pt – Pv 101.325 – 1.4

= 0.00871 kg / kg

ma (0.0289 – 0.00871) = amount of moisture removed


ma (0.0289 – 0.00871) = 0.999

ma = 49.48 kg / s

17. In an auditorium maintained at a temperature not to


exceed 24 C and relative humidity not to exceed 60%, a
sensible heat load of 132 kW and 78 kg of moisture per hour to
be removed. Air is supplied to the auditorium at 18 C. How
many kiograms of air must be supplied per hour? (Oct 99)
SOLUTION:

AUDITORIUM
SPACE
Qs = ma Cp (t2 – t1) Fan
24 C db and 60% RH
132 = ma (1) (24 – 18) 18 C
ma = 22 kg / s Qs = 132kW
QL = 78 kg / hr

ma = 79,200 kg / hr

18. Copra enters a dryer containing 60% water and 40% of


solids and leaves with 5% water and 95% solids. Find the
weight of water removed based on each pound of original
product. (Apr 95)

SOLUTION:

m = weight of original product per lb of wet feed


solid in wet feed = solid in dried product

0.95 m = 0.40 (1)


m = 0.42 lb

weight of water removed = 1 – 0.42

= 0.58 lb

19. A dryer is to deliver 1000 kg / hour of palay with a final


moisture content of 10%. The initial moisture content in the
feed is 15% at atmospheric condition with 32% dry bulb and
21% wet bulb. The dryer is maintained at 45 C while the
relative humidity of the hot humid air from the dryer is 80%. If
the steam pressure supplied to the heater is 2 MPa, determine
the air supplied to the dryer in m3 / hr. (Oct 95)

SOLUTION:
amount of moisture = amount of moisture absorbed
m = amount of palay in wet feed
solid in wet feed = solid in dried product

0.85 m = 0.90 (1000)


m = 1,058.83 kg / hr

amount of moisture removed:


m = 1058.83 – 1000 = 58.28
From psychrometric chart:
W1 = W2 = 0.0111 kg / kg; V2 = 0.915 m3 / kg
W3 = 0.0515 kg / kg

the amount of moisture absorbed:


= ma (W3 - W2)
58.823 = ma (0.0515 – 0.111)
ma = 1456.015 kg / hr

Va = 1456.015 kg / hr (0.915 m3 / kg)

= 1332.25 m3 / hr

20. A 4 m x 4 m x 4 m room has a relative humidity ratio of


80%. The pressure in the room is 120 kPa and temperature is
35 C (Psat = 5.628). What is the mass of vapor in the room.
Use Rvapor = 0.4625 kN – m / kg – K. (Oct. 99)

SOLUTION:
PvV = mv Rv T

RH = Pv
Psat

0.80 = Pv
5.628
Pv = 4.5024 kPa
4.5024 [(4) (4) (4)] = mv = (0.4615) (35 + 273)
mv = 2.027 kg

21. A mixture of dry air and water vapor is at a temperature


of 21 C under a pressure of 101 kPa. The dew point
temperature is 15 C. Calculate the relative humidity. (Apr 97)

SOLUTION:

RH = Pv
Psat @ 21 C

Pv = Psat @ 15 C
= 1.7044 kPa
Psat @ 21 C = 2.4861 kPa

RH = 1.7044
2.4861

= 68.56%
Fans and Blowers

ME BOARD (April 1997)


1. A fan whose static efficiency is 40% has a capacity of
60,000ft3 per hour at 60F and barometer of 30in. Hg and gives
a static pressure of 2 in. of water column on full delivery. What
size electric motor shall be used to drive this fan?

a. ½ Hp c. 2 Hp

b. 1 Hp d. 1 ½ Hp

SOLUTION:

Brake (input) Power = Static air Power


Static Fan ef

Pbrale = Pair /es

Solving for Pair


Pair = γQh

Where:
Q = 60,000/3600 = 16.67 ft3/s
h = γwhw = (62.4)(2/12) = 10.4
γ γ γ

then;
Pair = γ(16.67)(10.4/ γ)
= 173.37 ft-lb/s (1Hp/550 ft-lb/s)
= 0.315 Hp

thus:
Pbrake = 0.315/0.40
= 0.788 Hp
(b) Use 1 Hp motor
ME BOARD (April 1997)
2. Air is flowing in a duct with velocity of 7.62 m/s and a
static pressure of 2.16 cm water gage. The duct diameter is
1.22, the barometer pressure 99.4 kPa and the gauge fluid
temperature and air temperature are 30C. What is the total
pressure of air against which the fan will operate in cm of
water?

a. 3.25 c. 3.75
b. 2.50 d.1.25

SOLUTION:
h = h s + hv
Solving for the velocity head, hv:
hv = V2 = (7.62) 2
g 2(9.81)

Solving for velocity in terms of cm of water:


hv (1000) = (2.959)(ρair)

Where:
ρair = 99.4 .
0.287(30+273)
ρair = 1.143 kg/m3
then;
hv(1000) = (2.959)(1.143)
hv = 0.0034 m of water
hv = 0.34 cm of water

thus;
h = 2.1 + 0.34
(b) h = 2.50 of water

ME BOARD (Oct 1996)


3. Air enters a fan through a duct at a velocity of 6.3 m/s
and an inlet static pressure of 2.5 cm of water less than
atmospheric pressure. The air leaves the fan though a duct at
a velocity of 11.25 m/s and a discharge static pressure of 7.62
cm of water above the atmospheric pressure. If the specific
weight of the air is 1.20 kg/m3 and the fan delivers 9.45 m3/s,
what is the fan efficiency when the power input to the fan is
13.75 kW at the coupling?

a. 71.81% c. 52.34%
b. 61.81% d. 72.34%

SOLUTION:
Fan efficiency = Air Power .
Input power
efan = Pair/Pln

Solving for Pair:


h = hs + hv
= (Pd – Ps) + (Vd2 - Vs2)
γ 2g
= [(0.0762 + 0.025)/1.20] (1000) + (11.25)2 – (6.3)2
2(9.81)
= 88.761 m of air

then;
Pair = γQh
= [1.2(0.00981)] (9.45)(88.761)
= 9.874 kW

thus;
efan = 9.874/13.75
(a) efan = 71.81%

ME BOARD (Oct 1994)


4. A fan described in manufactures table is rated to deliver
500m3/min at a static pressure (gage) of 254 cm of water when
running at 250 rpm and requiring 3.6 kW. If the fan speed is
changed to 305 rpm and the air handled were at 65C instead of
standard 21C, find the power in kW.

a. 3.82 c. 4.66
b. 5.07 d. 5.68

SOLUTION:

Power Rewuired at 305 rpm and 65C:


P1 = ρ1 = T2
P2 ρ2 T1
P1 = 65 + 273
P2 21 + 273

Solving for the power required at 305 rpm and 21C:


P1/P2 = (N1/N2) 3
3.6/P2 = (250/305) 3
P2 = 6.5 kW

then;
6.5 = 65 + 273
P2 21 + 273

Thus;
(d) P2 = 5.68 kW

ME BOARD (April 1995)


5. A fan draws 1.42 m3/s of air to a static pressure of 2.54
cm of water through a duct 300mm diameter and discharge it
through duct of 275 mm diameter. Determine the static fan
efficiency if total fan mechanical efficiency is 70% and air is
measured at 25C and 760 mm Hg.

a. 60% c. 30%
b. 50% d. 47%

SOLUTION:

es = em (hs/h)

Solving for hs:


ρair = P/ RT
= ___101.325___
(0.287)(25+273)
= 1.18 kg/m3
hs = hwρw/ ρa
= 0.0254(1000)
1.18

Solving for hv:


V = Q/A
Vs = __1.42__ = 20.09 m/s
π(0.3) 2/4
Vd = __1.42____ = 20.09 m/s
π(0.275) 2/4
hv = (Vd)2 – (Vs)2 = (23.9)2 – (20.09)2
2g 2(9.81)
hv = 8.54 m of air

Solving h:
h = h s + hv
= 21.52 + 8.54
= 30.06 m of air

thus;
es = (0.70)(21.52/30.06)
(b) es = 0.5011 or 51.11%

ME BOARD (Oct 1997)


6. Find the horsepower of an industrial fan that delivers 23
m3/s of air through a 1m by 1.2 m ; pressure is 127 mm of
water ; air temperature is 21C and barometric pressure is 760
mm of mercury.

a. 53.35 Hp c. 46.45 Hp
b. 43.33 Hp d. 56.45 Hp

SOLUTION:
P=γQh

Solving for h:
V = Q/A = 21/(1*1.2) = 21.667 ρwater @ 101.325 kPa =
998.6 kg/m3
ρair = ___101.325___ hv = _ρair (V2/2g)
8.314(21 + 273) ρwater
ρair = 1.2 kg/m3 hv = _1.20_ _(21.67)2
h = 0.127 + 0.029 = 0.156 m 998.6
(2*9.81)
hv = 0.029 m
hs = 127 mm = 0.127
m
then;
P = (9.81)(26)(0.156)
= (39.79 kW

thus;
(a) P = 53.34 Hp
ME BOARD PROBLEM
7. A fan is listed as having the followe\ing performance with
standard air:
Volume discharge = 120 m3/s
Speed = 7 rps
Static Pressure = 310 mm water gage
Brake Power = 620 kW

The system duct will remain the same and the fan will
discharge the same volume of 120 m3/s of air 93C and a
barometric pressure of 735 mm Hg when it its speed is 7 rps.
Find the brake power input and the static pressure required.

a. 482 kW, 241 mm WG c. 482 kW, 256


mm WG
b. 492 kW, 241 mm WG d. 492 kW, mm WG

SOLUTION:

Brake power input = 620 (ρ2/ρ1)

= 620 (ρ2/1.2)

Sovling for ρ2:


ρ2 = 735 (101.325/760)
0.287(93 +273)
= 0.9329 kg/m3

Solving for the static pressure, h2:


h2/ 310 = 0.9329/1.2
h2 = 241 mm of water gage

then;
Brakepower input = 620 (0.9329/1.2) = 482 kW

Thus;
(a) h2 = 241 mm & Pbrake =482 kW
ME Board Exam Problem
8. Local coal with higher heating value of 5,500 kCal/kg is
burned in a pulverized coal fired boiler with 25% excess air the
reate of 25.9 M.T. per hour when the steam generated is 220
M.T. per hour. This boiler is served by 2 forced-draft fans of
equal capacity delivering the air to 305 mm of water to the
furnace. Calculate the capacity of each fan in m3/hr if fan
capacity is to be 110 percent of the maximum requirement.
Ambient air is 100 kPa and 30C.

a. 31.75 m3/s c. 28.87 m3/s


b. 63.50 m3/s d. 57.73 m3/s

SOLUTION:

The theoretical weight of air to burn the fuel is given in an


appropriate formula when the heat value of the fuel is given:
Wta = (A/F) = HHV, kCal/kg
745
= 5,500/745
= 7.382 kg air/ kg fuel

Mass flow rate of air required for combustion


Ma = (7.382)(1.25)[25.9(100)]
= 238,922.25 kg/hr

Capacity of each fan at 110% of the maximum requirement:


Q1 = Q2 = 207,830.05(1.10)
2
= 114,306.53 m3/s = 31.75 m3/s

thus;
(a) Q = 31.75 m3/s

ME Board Exam Problem


9. A larged-draft fan is handling air at 1 atm, 43.4C unfer a
total head of 26.6 cm WG (at 43.3C). The power input to the
fan is 224 kW and the fan is 75% efficient. Compute the
volume of air handled per minute. Local gravity of acceleration
is g = 9.81 m/s2.

a. 3,862.87 m3/min c. 2,862.87


m3/min.
b. 4,862.87 m3/min d. 4,567.97 m3/min.

SOLUTION:

Power Input to fan = Shaft Power


P=γQh
efan
224 = 9.81 Q(0.266)
0.75
Q = 64.38 m3/s
= 3,862.87 m3/min
thus;
(a) 3,862.87 m3/min

ME Board April 1997


10. A fan whose static efficiency is 40% has a capacity of
60,000 ft3 / hr at 60°F and barometer of 30 in Hg and gives a
static pressure of 2 in of water column on full delivery. What
size of electric motor should be used to drive the fan?

Solution:

hs = hw ww
wa

hs = (2in/12) (62.4)
wa

hs = 10.4 / wa

Air Power = wa Q h

Air Power = wa (60,000 / 60) (10.4 / wa)


33,000
Air Power = 0.315 Hp

Therefore,

Use 1 Hp motor (standard)

Pumps

ME BD April 98
1. A pump lifts water at the rate of 283 li/s from a lake and
forces it into a tank 8m above the level of water at a pressure
of 137Kpa what is the power required in KW

SOLUTION:
P= γQH
Q=283li/s [1m³/1000li] = 0.283 m³/s
H= 8 + (137KN/m³ / 9.81 KN/m³) = 21.9653 m
P= 9.81 KN/m³ x 0.283m³/s x 21.9653m
P= 60.9807 KW

ME BD April 98
2. A pump discharge 150 li/s of water to a height of 75m if
the efficiency is 75% and the speed of the pump is 1800rpm,
what is the torque in N-m to which the drive shaft is subjected

SOLUTION:
Pw = γQH
= 9.81 KN/ m³ [0.150 m³/s] [75m]
= 110.3625 KW
Pb = Pw / ep = 110.3625KW / 0.75 = 147.15 KW
T = Pb / 2πN
= 147.15 KW / 2 π (1800rev/min) (1min/sec)
= 0.7807 KN.m x (1000N/1KN)
T= 780.7 N-m

ME BD April 98
3. A pump receives 8 kg/s of water at 220 KPa and 110°C
and discharges it at 1100KPa. Compute the power required in
KW
SOLUTION:
ρ = m/v
v = m/p = 8kg/s / 1000kg/ m³ = 0.008m³/s
P= v (Pd – Ps)
= 0.008m³/s [1100-220]
P= 7.04 KW

ME BD April 93
4. The power output is 30 Hp to a centrifugal pump that is
discharging 900gpm and which operates at 1800rpm against a
head, H=120ft, 220V, 3θ, 60Hz. if this pump is modified to
operate 1200rpm assuming its efficiency remains constant
determine the discharge in gpm the theoretical head it imparts
to the liquid and the power input to the pump.

SOLUTION:
Q 1 = N1 P1/ P2 = (N1/N2) ³
Q 2 N2 P2 = 30 / (1800/1200)³
Q2 = Q1 N2 / N1 P2= 8.8889Hp
= 900(1200)/1800
Q2= 600gpm
H1/H2 = (N1/N2)²
H2= 120 / (1800/1200) ²
H2= 53.3333ft

ME BD Oct93
5. A centrifugal pump requires 40 ft of read to deliver water
from low level to a higher left. If pump speed is 1600rpm
determine the impeller diameter of the centrifugal pump.

SOLUTION:
Vd²=2gh
Vd = √2gh
= √ 2(32.2ft/s) (40ft)
Vd = 50.7543 ft/s
D= Vd/ πN
= (50.7543/ft/s) / π (1600rev/min) (1min/60sec)
D= 0.6058 ft

ME BD Oct 95
6. A centrifugal pump powered by a direct drive induction
motor is needed to discharge 150gpm against a total head of
180ft. when turning a fully loaded speed of 3500rpm. What
type of pump should be selected?

SOLUTION:
Ns = (N√Q) / H3/4
= (3500√150) / (180)3/4
Ns= 872.2868rpm
 Radial Type

ME BD Oct 95
7. It is desired to deliver 5m³ at a head of 640m in a single
stage pump having specific speed not to exceed 40 if the
speed is not to exceed 1352rpm how many stages required?

SOLUTION:
Ns = (N√Q) / H3/4
h= head/stage= H/n
h=640/n
40 = (1352√5) / (640/n) 3/4
n= 2 stages

ME BD April 95
8. A double suction centrifugal pump delivers 70ft³/s of
water at head of 12m and running at 1250rpm. What is the
specific speed of the pump?
SOLUTION:
Ns = (N√Q) / H3/4
Q= (70/2) ft³/s [7.48gal] [60s]
Q= 15,710.1gpm
H= 12 [3.2808ft/1m]
H= 39.3696ft
Ns = (1250√15,710.1) / (39.3696)3/4
Ns= 9,968. 4753 rpm

ME BD April 97
9. A hydroelectric plant having 50 km² reservoir area and
100m head is used to generate power. The energy utilize by
the consumers whose load is connected to the power plant
during a 5hr period is 13.5x10 KWH the overall generation
efficiency is 75%. Find the fall of water in the reservoir after
5hr period?

SOLUTION:
Pw= γQH
Energy= γQHt
13.5x10 KWH = 9.81 Q (100m) (5hr) (0.75)
Q= 3669.7248m³/s
Volume after 5 hr period
V=Ah
V= 3669.7248m³/s[5hr][3600s/hr]
V= 66,055,045.87m³
h= v/a = (66,055,045.87m³) / 50km²[100m/1km]
h= 1.3211m

ME BD April 98
10. A hydroelectric plant discharges water at the rate of
0.75m³/s and enters the turbine at 0.35m/s with a pressure of
275KPa. Runner inside diameter is 520rpm and turbine
efficiency is 88% find coefficient turbine speed factor?

SOLUTION:
Ø = πDN / √2gh
H= P/γ + v²/2g
= (275 / 9.81) + (0.35² / 2(9.81))
H= 28.0389m
Ø= [π (0.55) (520rev/min) (1min/60sec)] / [√2 (9.81m/s²)
(28.0389m)
Ø= 0.6385

ME BD April 98
11. The available flow of water is 25m³/s at 30m elevation if
a hydroelectric plant is to be installed with turbine efficiency
of 85% and generator efficiency of 90% what maximum power
that the plant could generate?

SOLUTION:
Pw = γQH
= (9.81) (25m³/s) (30m)
Pw= 7357.5 KW
Pgen = 7357.5 KW (0.85) (0.90)
Pgen= 5628.4875 KW

ME BD Oct95
12. Water flows steadily with a velocity of 3.05m/s (in a
horizontal pipe) having a diameter of 15.24cm at one section
of the pipe the temp. And pressure of the water is 21°C and
689.3KPa respectively, pipe length is 304.8m and inlet
pressure is 516.9 Kpa. What is the friction factor?

SOLUTION:
H= (fLV²) / (2gD)
H= (Pd – Ps) / γ
H= (689.3 – 516.9) / (9.81)
f= H2gD / LV²
= [17.5739(2) (9.81m/s²) (0.1524m)] / [(304.8m) (3.05m/s) ²]
f= 0.0185

ME BD Oct 95
13. For a proposed hydroelectric plant the tail water and
head water elevation is 160m and 195m respectively. If
available flow is 10m³/s and headloss of 5% of water available
head what is the water power?

SOLUTION:
Pw= γQH
Hg= HHW - HTW
H= Hg- HL
Hg= 195-160 = 35m
H= 35 – 0.05 (35)
H= 33.25m
Pw= 9.81KN/m³ [10m³/s] [33.25m]
Pw= 3261.825 KW

ME BD Oct 96
14. A vacuum pump is required to drain a flooded area at
20°C pumping pressure of water at this temperature is 2340Pa.
the pump cannot lift the water higher than 10.16m. Determine
the atmospheric pressure?

SOLUTION:
(Ps/γ) + (Vs²/2g) + (Zs) = (Pd/γ) + (Vd²/2g) + (Zd)
(Ps/γ) = (2340 N/m²/ 9810N/m³) + [10.16-0] m
= 0.2385m +10.16m
Ps= 10.3985 γ
= 10.3985m[9.81KN/m³]
Ps=102.0096 KPa

ME BD Oct 96
15. A pump driven by an electrical motor moves 38gpm of
water from a tank to another tank with a total 10ft of 200ft. if
the pump efficiency is 68% what size of motor will be required?

SOLUTION:
ep = Pw/Pb
Pw = γQH
Q= 38gal/min [1ft / 7.481gal]
Q= 5.0795 ft³/min
Pw= 62.4lb/ft³ [5.0795ft³/min] [200ft]
Pw= 63,392.16 ft-lb/min [1hp/33,000ft-lb/min]
Pw= 1.921Hp
Pb= Pw/ep
= 1.921Hp /0.68
Pb= 2.825 Hp say 3 Hp

ME BD Oct 95
16. The specific speed of a turbine is 75rpm and running at
450 rpm if the head is 20m and generator efficiency is 90%,
what is the maximum delivered by the generator?
SOLUTION:
Pgen = PwNgen
Ns= (N√Hp) / (H) 5/4
75= 0.2623 N √KW
(20m) 5/4
KW= 722.3689 x .90
Pgen= 722.3689 x .90
Pgen= 650.0076 KW

ME BD Oct 95
17. A turbine has a mechanical efficiency of 95% volumetric
efficiency of 91% and total efficiency of 80% if the effective
head is 40m find the total head

SOLUTION:
ηH = h / Htotal = (0.8) / 0.95(0.97)
ηH = 0.8681
Htotal = 40 /0.8681
Htotal = 46.075m

ME BD April 98
18. A fuel pump is delivering 10 gpm of oil with s.g of 0.83.
The total head is 9.14m; find how much energy does the pump
consume?

SOLUTION:
Pw = γQH
Sg = Ps/Pw
Ps = sg x Pw
= 1000kg/m³ [0.83]
Ps = 830 kg/m³
γ=800kg/m³ [9.81]
= 8.1421 KN/m³
Pw = 8.1423 KN/m³ (6.309x10-4m³/s)(9.14m)
Pw = 0.047 kj/sec [3600sec/1hr]
Pw= 169.0271 KJ/hr

ME BD Oct 97
19. A pump with 400mm dia. Suction pipe and a 350mm
diameter discharge pipe is to deliver 20,000li/min of 15.6°C
water. Calculate the pump head in meter if suction gage is
7.5cm below pump centerline and reads 127mmHg vacuum and
discharge gage is 45 cm above the pump centerline and reads
75KPa

SOLUTION:
HT = Pd-Ps + Vd² -Vs² + (Zd-Zs)
γ 2g
Pressure Head
Ps = 127mmHg [101.325MPa / 760mmhg]
Ps = 16.9319 KPa
[75- (-16.9319) / (9.82 KN/m³)] KN/m²
Pressure Head = 9.3712 m
Velocity Head
Q = AV
A = πd² / 4
Vs = 20,000li/min [1m³/1000li] [1min/60sec]
π/4 (0.4) ²
Vs = 2.6526 m/s
Vd = 20,000li/min [1m³/1000li] [1min/60sec]
π/4 (0.35) ²
Vd = 3.4646
Velocity Head = = (3.4646² - 2.6526²) m²/s²
2 (9.81m/s²)
Velocity Head = 0.2532 m
Static Head = Zd – Zs
= 0.45m – (-0.075m)
= 0.525 m
HT = 9.38712 + 0.2532 + 0.525
HT = 10.1494 m

ME BD Oct 97
20. A centrifugal pump delivers 300,000li/hr of water to a
pressurized tank whose pressure is 280KPa. The source of the
water is 5m below the pump. The diameter of the suction pipe
is 250mm. Calculate the KW rating of the driving motor
assuming the pump efficiency to be 72%?

SOLUTION:
ep= Pw /Pb
Pressure Head
PH = Pd-Ps = (280-0) KN/m²
γ 9.81 KN/m³
PH = 28.5423m
Velocity Head
Hv = (Vd²-Vs²) / 2g
Q = 300,000li/hr [1m³/1000li][1hr/3600sec]
Q = 0.0833 m³/s
Q = AV
Vd = (0.0833 m³/s) / π/4 (0.259) ² = 1.697m/s
Vs = (0.0833 m³/s) / π/4 (9.300m) ² = 1.785m/s
Hv = (1.697² - 1.1785²) / 2 (9.81)
Hv = 0.076 m
Static Head
Hs = Zd-Zs = 0 – (-5) = 5m
HT = 28.5423 + 0.076 + 5
HT = 33.6183m
Pw = γQH
= 9.81KN/m³ [0.0833m³/s][33.6183m]
Pw = 27.472 KW
Pb = Pw/ep = 27.472 / 0.72
Pb = 38.1556 KW

FUELS AND COMBUSTION

ME Board April 1991

1. A 650 Bhp diesel engine uses fuel oil of 28° gravity, fuel
consumption is 0.65 lb/Bhp-hr. Cost of fuel is P7.95 per liter.
For continuous operation, determine the minimum volume of
cubical day tank in cm3, ambient temperature is 45 ° C.

Solution:
SG15.6°C = ____141.5 ____
131.5 + 28
= 0.887
SG45°C = SG15.6°C [ 1 –0.007 ( t – 15.6 ) ]
= 0.887 [ 1 – 0.007 ( 45 – 15.6 ) ]
= 0.869
Density of fuel = 0.869 ( 1 kg. / liters )
= 0.869 kg / liter
Solving for fuel consumption :
Mf = 0.65 ( 650 )
= 422.5 lb / hr
= 191.61 kg / hr
Vf = 191.61 kg/ hr / 0.869 kg/liter
= 220.495 liters / hr

Volume of Day tank = 220.495 li/hr x 24 hrs.


= 5,291.88 li x 1m3 / 1000 li
= 5.291.88 m3 x (100) 3 cm3 / 1 m3
Volume of Day Tank = 5,291,880 cm3

ME Board October
2. A circular fuel tank 45 feet long and 5.5 feet diameter is
used for oil storage. Calculate the number of days the supply
tank can hold for continuous operation at the following
conditions :
Steam flow = 2000 lbs / hr
Steam dry and saturated at 200 psia
Feedwater temperature = 230°F
Boiler Efficiency = 75 %
Fuel oil = 34°API

Solution:

From steam tables :


At 200 psi (1.380 Mpa), hs = 2789.6 KJ/kg
At 230°F (110°F), hf = 461.3 KJ/kg

Qh = 41,130 + 139.6 (34)


= 45,876 KJ/kg
m s = 2000/2.205 = 907 kg/hr
ηb = m s (hs - hf ) / mf Qh
0.75 = 907(2789.6 – 461.3)
mf (45,876)
mf = 61.376 kg/hr

SG15.6°C = 0.855
Density = 0.855(1000kg/m3)= 855 kg/ m3
Volume of tank = π/4 (5.5/3.281)2 (45/3.281)=30.927 m3
Total weight of fuel = 30.927 m3 x 855 kg/ m3
= 25,904 kg

Number of Days = 25,904 / 61.37 (24)


Number of Days = 17.58 days

ME Board April 1998


2. The dry exhaust gas from oil engine has the following
gravimetric analysis:
CO2 = 21.6 % 02 = 4.2 % N2 = 74.2 %
Specific heats at constant pressure for each component
of the exhaust gas in Kcal/kg °C are:
CO2 = 0.203 02 = 0.219 N2 = 0.248
Calculate the gas constant in J/kg-°K

Solution:
Converting the gravimetric analysis to volumetric:
CO2 0.216 0.216/44 = 0.004909
02 0.042 0.042/32 = 0.001312
N2 0.742 0.742/28 = 0.026500
0.032721 mols/kg-
mol

Molecular weight = 1 / 0.032721


Molecular weight = 30.56 kg / kg-mol

R = 8.314 / M
R = 8.314 / 30.56
R = 0.27206 KJ/kg-°K
R = 272.06 J/kg-°K

ME Board October 1996

3. There are 20 kg of flue gases formed per kg of fuel oil


burned in the combustion of a fuel oil C 12H26 . What is the
excess air in percent?
Solution:
C12H26 + 18.5 O2 + 18.5(3.76) N2 12 CO2+13H2
O+18.5(3.76)N2
Theo. A/F = 18.5 + 3.76(18.5)
1
Theo. A/F = 88.06 mol/mol

Theo. A/F in kg/kg = 88.06 (28.97)


12(120+26(1)
Theo. A/f in kg/kg = 15 kg air/ kg fuel

Mass of air = 20 kg flue gas – 1 kg fuel gas


Mass of air = 19 kg air

Actual A/f = 19 kg air/ kg fuel


Actual A/F = Theo. A/f (1 + e )
19 = 15 ( 1 + e )
e = 0.2667
e = 26.67 %

ME Board April 1984

4. A gaseous fuel mixture has a molal analysis:


H2 = 14% CH4 = 3% CO = 27%
O2 = 0.6% CO2 = 4.5% N2 = 50.9%
Determine the air-fuel ratio for complete combustion on
molal basis.

Solution:
Chemical reaction with oxygen:
0.14 H2 + 0.070 O2 = 0.14H2 O
0.03 CH4 + 0.060 O2 = 0.03 CO2 + 0.06 H2 O
0.27 CO2 + 0.135 O2 = 0.27 CO2
0.265 O2
Actual O2 in product = 0.265 O2 – 0.006 O2
Actual O2 in product = 0.259 O2
Molal A/F = 0.259 + 0.259 (3.76)
1
Molal A/F = 1.233 mols air/mol of fuel

ME Board April 1995


5. A steam generator burns fuel oil that has the following
chemical analysis by mass in percent:
C = 85.3 H2 = 14.1 S = 0.5 N2 = 0.1
Calculate partial pressure of Water.

Solution:
Converting the given mass analysis to molal analysis:

C: 85.3/12 = 7.108
H2: 14.1/2 = 7.050
S: 0.5/32 = 0.016
N2 : 0.1/28 = 0.004

Combustion reaction with 125% theoretical air:


7.108C + 7.05 H2 + 0.016S + 0.004N2 + 1.25 (10.649)O2 1.25
(10.649)(3.76)N2 = 7.108CO2 + 7.05H2O + 0.016SO2 +
50.054N2 + 0.25 (10.649)O2
mT = total mols in product
= 7.108 + 7.050 + 0.016 + 50.054 + 2.662
= 66.89 mols

Partial pressure of H2O = (7.050/66.890)(170)


Partial pressure of H2O = 17.92 Kpa

ME Board April 1996

6. A volumetric analysis of a gas mixture is as follows:


CO2 : 12%
O2 : 4%
N2 : 82%
CO : 2%
What is the percentage of CO on a mass basis?

Solution:
Converting to mass basis:
CO2 = 0.12 x 44 = 5.28
O2 = 0.04 x 32 = 1.28
N2 = 0.82 x 28 = 22.96
CO = 0.02 x 28 = 0.56

Total mass of product = 5.28 + 1.28 + 22.96 + 0.56


Total mass of product = 30.08 kg
% mass of CO = 0.56 / 30.08
% mass of CO = 1.9%

ME Board October 1996

7. Calculate the theoretical air needed for the complete


combustion of ethane C2 H6 for 20 kg fuel.

Solution:
C2 H6 + O2 + (3.76)N2 CO2 + H2O + (3.76)N2
Balancing the equation:
C2 H6 + 3.5 O2 + 3.5 (3.76)N2 2CO2 + 3H2 O + 3.5(3.76)N2
Theoretical A/F = 3.5(32) + (3.5)(3.76)(28)
(2x12) + (1x6)
Theoretical A/F = 16.016 kg air/ kg fuel
Mass of air needed = 16.016 (20)
Mass of air needed = 320.32 kg

HEAT TRANSFER

ME Board October 1993

1. A 15 cm thick wall has a thermal conductivity of 5 W/m-


°K. If inside and outside surface temperature of the wall are
200°C and 30°C, respectively. Determine the heat transmitted.

Solution:

Q = kA ( t2 – t1 )
X

Q = 5 ( 200 –30 )
A 0.15

Q = 5666.67 W / m2
A

Q / A = 5.6667 KW / m2
ME Board October 1986

2. A heat exchanger is to be designed for the following


specifications:

Hot gas temperature, 1145°C


Cold gas temperature, 45°C
Unit surface conductance on the hot side, 230 W/m2 -°K
Unit surface conductance on the cold side, 290 W/m2 -°K
Thermal conductivity of the metal wall, 115 W/m-°K
Find the maximum thickness of metal wall between the hot
gas and the cold gas, so that the maximum temperature of
the wall does not exceed 545°C.
Solution:
Q = h1 A ( t1 - ta )
Q / A = h1 ( t1 – t2 )
Q / A = 230 ( 1145 – 545 )
Q /A = 138,000

Q = A( t1 – t2 )
1+x+1
h1 k h2

Q / A = ( t1 – t2 )
1+x+1
h1 k h2

138,000 = _______( 1145 – 45 )_________


(1/230) + (x / 115) + (1 / 290)
x = 0.020115 m
x = 20.115 mm

ME Board April 1983

3. A surface condenser serving a 50,000 KW steam turbo-


generator unit receives exhaust steam at the rate of 196,000
kg/hr. Vacuum in condenser is 702 mm Hg. Sea water for
cooling enters at 29.5°C and leaves at 37.5°C. For steam
turbine condenser, manufacturers consider 950 Btu/lb of steam
turbine condensed as heat given up to cooling water. Calculate
logarithmic mean temperature difference in °F.

Solution:
Condenser pressure = 101.325 – ( 702 x 101.325/760)
Condenser pressure = 7.733 Kpa.

From steam table, at 0.007733 Mpa, tsat = 40.86°C

Δta = 40.86 – 29.5


Δta = 11.36°C
Δtb = 40.86 – 37.5
Δtb = 3.36°C

tmean = 11.36°C – 3.36°C


ln (11.36 / 3.36 )
tmean = 6.56°C x 1.8
tmean = 11.82°F

ME Board October 1994

4. Calculate the energy transfer rate across 6” wall of


firebrick with a temperature differences across the wall of
50°C. The thermal conductivity of the firebrick is 0.65 Btu/hr-ft-
°F at the temperature interest.

Solution:
( ta – tb ) = 50 (9/5)
( ta – tb ) = 90°F

Q = kA( ta – tb )
X

Q / A = 0.65 (90)
(6/12)

Q /A = 117 Btu/ hr-ft2

Q/A=117 Btu/ hr-ft2 x 1.055J/Btu x 1hr/3600sec x 10.76ft2 / m2


Q/A = 369 W/m2

ME Board October 1994


4. Water is flowing in a pipe with radius of 25.4 cm at a
velocity of 5 m/sec at the temperature in the pipe. The density
and viscosity of the water are as follows: density = 997.9
kg/sec ; viscosity = 1.131 Pa-s. What is the Reynold’s Number
for this situation?

Solution:
R = D vo
V
Where: D = diameter
D = 2 (25.4)
D = 50.8 cm
D = 0.508 m
vo = velocity
vo = 5 m/sec
v = kinematic viscosity
v = 1.131 / 997.9
v = 0.0011334 m2 / sec
NR = 0.508 (5)
0.0011334

NR = 2241
ME Board April 1999

5. Compute the amount of condensate form during 10


minutes warm-up of 150 meter pipe conveys the saturated
steam with enthalpy vaporization hfg = 1,947.8 LJ/kg. The
minimum external temperature of pipe is 2°C. The final
temperature of pipe is 195°C. The specific heat of pipe material
is 0.6 KJ/kg-°C. The specific weight is 28 kg/m.

Solution:

mp = mass of pipe
mp = 28 (150)
mp = 4200 kg

Heat loss by steam = Heat loss from pipe


Ms (hg - hf ) = mp cp ( t2 – t1 )
Ms (1947.8) = (4200) (0.6) (195 – 2)
Ms = 249.69 kg

ME Board Oct Ms = 249.69 kg

ME Board Octransfer in the glass surface area of 0.7m2 having


an inside temperature (room) of 25°C and 13°C outside
temperature (surrounding). The thickness of glass surface is
0.007 m. The thermal conductivity is 1.8 W/m-°K.

Solution:

Q = kA ( t2 – t1 )
X
Q = (1.8) (0.70) (25-13)
0.007
Q = 2160 Watts

Q = 2.16 KW
CHIMNEY

ME Board April 1991

1 A boiler uses 2500 kg of coal per hour and air required for
combustion is 16 kg per coal. If ash loss is 10%, determine the
mass of gas entering the chimney.

Solution:

mg = ma + mf - mash
A/F = ma / mf
16 = ma + mf
ma = 16 mf

mg = 16mf + mf – 0.1mf
mg= 16.9 mf
mg = 16.9 (2500 kg / hr)
mg = 42,250 kg/hr

ME Board October 1991

2. The gas density of chimney is 0.75 kg/m3 and air density


of 1.15 kg/ m3 . If the driving pressure is 0.25 Kpa, determine
the height of chimney.

Solution:

hw = H ( da - dg )
0.25 = H ( 1.15 – 0.75 ) ( 0.00981)
H = 63. 71 m
ME Board April 1991

3. The actual velocity of gas entering in a chimney is 8


m/sec. The gas temperature is 25°C and pressure drop of 98
Kpa with a gas constant of 0.287 KJ/kg-°K. Determine the
chimney diameter if mass of gas is 50,000 kg/hr.

Solution:
Pg Vg = mg Rg Tg
98 (Vg) = (50,000 / 3600) (0.278) (25 + 273)
Vg = 12.12 m3 / sec
Vg = A x V
11.74 = π/4 D2 (8)
D = 1.39 m

ME Board April 1990

4. A power plant situated at an altitude having an ambient


air of 96.53 Kpa and 23.88°C. Flue gases at a rate of 5.0 kg/sec
enter the stack at 200°C and leaves at 160°C. The flue gases
gravimetric analysis are 18% CO2, 7% O2 and 75% N2 . Calculate
the height of stack necessary for a driving pressure of 0.20
Kpa.

Solution:

Solving for the molecular weight and gas constant of the flue
gas:
CO2: 18% 0.18/44 = 0.00409
O2 : 7% 0.07/32 = 0.00219
N2 : 75% 0.75/28 = 0.02678
0.03306
Mg = 1 / 0.03306 = 30.25
R g = 8.314 / 30.25 = 0.275
T g = ( 200 + 160 ) / 2 = 180°C

d g = P / RT
= _____96.53______
(0.275) (180+273)
= 0.775 kg / m3
da = P / RT
= ______96.53______
(0.287)(23.88+273)
= 1.133 kg / m3
Draft = H (da - d g )
0.20 = H (1.133 – 0.775) (0.00981)
H = 56.95 m

ME Board April 1998

5. A steam boiler plant consumes 9,000 kg of coal per hour


and produces 20 kg of dry flue gases per kg of coal fired.
Outside air temperature is 32°C, average temperature of the
flue gas entering the chimney is 343°C and average
temperature of dry flue gas in the chimney is 260°C. The gage
fluid density is 994.78 kg / m 3 and the theoretical draft of 2.286
cm of H20 at the chimney base is needed when the barometric
pressure is 760 mm HG. Determine the height of the chimney.

Solution:
hw = h x w
hw = (0.02286) (994.78)
hw = 22.74 kg / m3
da = P / R a T a
P = 760 mm Hg
P = 101.325 Kpa
da = _____101.325______
(0.287)(32+273)
= 1.157 kg / m3
dg = P / R g T g
= _____101.325______
(0.287)(260+273)
= 0.662 kg / m3
hw = H (da - dg )
22.74 = H ( 1.157 – 0.062)
H = 46 m
REFRIGERATION

ME Board October 1991

1. An air-conditioning plant with a capacity of 400 KW of refrigeration


has an evaporating and condensing temperature of 3°C and 37°C
respectively. If it uses Refrigerant 12, what will be the mass of flash gas per
kg of refrigerant circulated?

Solution:

From R-12 Tables:

h1 = hg at 3°C
h1 = 352.755 KJ/kg
hf at 3°C = 202.780 KJ/kg
v1 = 0.05047 m3/kg
h3 = h4 = hf at 37°C
h3 = h4 = 235.503 KJ/kg

Let x= mass of flash gas or quality after expansion


h3 = h4 = hf + x hfg
hfg = hg - hf
hfg = 352.755 KJ/kg – 202.780 KJ/kg
hfg = 149.975 KJ/kg
255.503 KJ/kg = 202.78 KJ/kg + x (149.975)
x = 0.218
x = 21.80%

ME Board April 1995

2. An air-conditioning system of a high-rise building has a capacity of


350 KW of refrigeration; uses R-12. The evaporator and condenser
temperature are 0°C and 35°C, respectively. Determine the work of
compression in KW.

Solution:

From R-12 Tables and P-h chart:

h1 = hg at 0°C
h1 = 351.477 KJ/kg
v1= vg at 0°C
v1 = 0.0553892 m3/kg

At 0°C:
hf = 200 KJ/kg
hfg = 351.477 KJ/kg

From R-12 Chart:

h2 = 369 KJ/kg
h3 = h4 = 233.498 KJ/kg

Refrigerating Effect = m ( h1 – h4 )
350 = m (351.477 –233.498)
m = 2.967 kg/sec

Work of compression = m (h2 - h1)


Work of compression = 2.967(369-351.477)
Work of compression = 52 KW

ME Board April 1990

3. A vapor compression refrigeration system has a 30 KW motor driving


the compressor. The compressor inlet pressure and temperature are 64.17
Kpa and 20°C respectively and discharge pressure of 960 Kpa. Saturated
liquid enters the expansion valve. Using Freon 12 as refrigerant, determine
the capacity of the unit in tons of refrigeration.

Solution:

From R-12 Tables and Chart:

h1 = 345 KJ/kg
h2 = 398 KJ/kg
h3 = h4 = 238.5 KJ/kg

Compressor work = m (h2 - h1)


30 = m (398 – 345)
m = 0.566 kg/sec

Refrigerating Capacity = m ( h1 – h4 )
Refrigerating Capacity = 0.566 (345 – 238.5)
Refrigerating Capacity = 60.279 KW

Tons of refrigeration = 60.279 / 3.516


Tons of refrigeration = 17.144 tons ref.
ME Board October 1990

4. A simple vapor compression cycle develops 15 tons of refrigeration


using ammonia as refrigerant and operating at condensing temperature of
24°C and evaporating temperature of -18°C and assuming compression
are isentropic and that the gas leaving the condenser is saturated, find the
power per ton.

Solution:

From ammonia tables and chart:

h1 = 1439.94 KJ/kg
h2 = 1665 KJ/kg
h3 = h4 = 312.87 KJ/kg
v1 = 0.572875 m3/kg

Refrigerating Effect = m (h1 – h4 )


15 (3.516) = m (1439.94 – 312.87)
m = 0.0467 kg/sec

Power requirement = m (h2 - h1)


Power requirement = 0.0467 (1665 – 1439.94)
Power requirement = 10.531 KW
Power per ton = 10.531/15
Power per ton = 0.702 KW/ton

ME Board October 1994

5. Calculate the bore in mm of a single-acting, 6 cylinder ammonia


compressor running at 1200 rpm to compress 700 kg/hr of refrigerant
which vaporized at -15°C, given the following:
a. Bore and stroke = unity
b. Volumetric efficiency = 70%
c. Specific volume of NH3 = 8.15 ft3 / lb at 5°F

Solution:

v1 = 8.15 ft3 / lb x 1 m3 / 35.31 ft3 x 2.205lb/1kg


v1 = 0.5089 m3 /kg
V1 = m v 1
V1 = (700kg/hr / 60) (0.5089 m3 /kg)
V1 = 5.937 m3 / min
VD = 5.937 / 0.7
VD = 8.481 m3 / min
VD = ∏/4 D2 L N c

For D = L (unity)

2.481 = ∏/4 (D) 2 (D) (1200) (6)


D = 0.114 m
D = 114 mm

ME Board April 1996

6. A refrigeration system operates on an ideal vapor-compression


using Refrigerant 12 with an evaporator temperature of -30°C and a
condenser exit temperature of 49.3°C and requires a 74.6 KW motor to drive
the compressor. What is the capacity of the refrigerator in tons of
refrigeration? Enthalpy of condenser entrance = 382 KJ/kg, exit = 248.15; at
evaporator entrance = 248.15, exit = 338.14 KJ/kg.

Solution:

W = m (h2 - h1)
74.6 = m (382 –338.14)
m = 1.7 kg/sec

QA = m (h1 – h4)
QA = 1.7 (338.14 – 248.15)
QA = 153 KW

Refrigerating Capacity = 153 / 3.516


Refrigerating Capacity = 43.5 tons of ref.

ME Board April 1995

7. Liquid ammonia at a temperature of 26°C is available at the


expansion valve. The temperature of the vaporizing ammonia in the
evaporator is 2°C. Find the percentage of liquid vaporized while flowing
through expansion valve.
Temp. (°C) Pres.(Kpa) hf hfg hg
2°C 462.49 190.4 1255.2 1445.6
26°C 1033.97 303.6 1162.0 1465.6

Solution:

h3 = h4 = 303.6 KJ/kg

h4 = hf + x hfg
303.6 KJ/kg = 190.4 kj?kg + x (1255.2 KJ/kg)
x = .0902

x = 9.02%

ME Board October 1997

8. A refrigeration system using R-22 has a capacity of 320 KW of


refrigeration. The evaporating temperature is -10°C and the condensing
temperature is 40°C. Calculate the fraction of vapor in the mixture before
the evaporator.

Solution:
Properties of R-22 are :
hg = 401.60 KJ/kg
hf = 188.426 KJ/kg

At 40°C
hf = 249.686 KJ/kg

h3 = h4 = hf + x(hg - hf)
249.686 = 188.426 + x (401.6 – 188.426)
x = 0.287

ME Board October 1999

9. What is the coefficient of a vapor compression refrigeration system


having the following data: Enthalpy entering the compression is 181.79;
enthalpy after compression work is 207.3. After condensation the enthalpy
is 58.2 and the throttled from 0.19 Mpa to 0.18 Mpa.

Solution:

COP = h1 – h4
h2 – h 1
COP = 181.79 – 58.2
207.3 – 181.79

COP = 4.84

ME Board October 1999


10. A refrigeration system having a 22 KW capacity needs 7.8 hp
compressor. Find the COP of the system.

Solution:

COP = RE
Wc

COP = 22KW
(7.8x0.746)

COP = 3.78
IPE Exit Exam Reviewer

1. What tonnage of refrigerating machine is required if the refrigerating system extracted


48,000 btu per hour?
Answer: 4 tons

2. An insulator with superior tensile strength but low heat resistance


Answer: Fiberglass

3. What is the combination of a wet and dry bulb thermometer is called a


Answer: Hygrometer or psychrometer

4. The weight of steel bar reinforcement show be how many time the weight of the foundation?
Answer: ½% to 1%

5. Is a high temperature process that turns incinerator ash into a safe, glass – like material
Answer: Vitrification

6. Oil is added to a freon compressor by


Answer: Pumping in with a hand pump

7. The difference between the theoretical draft and the draft loss
Answer: Available draft

8. The purpose of expansion valve by-pass is to


Answer: Controls the refrigerant to the evaporator in case the automatic valves fail

9. Load curve refers to the plot of:


Answer: Load versus time

10. What does a negative Joule-Thompson coefficient means during a throttling process?
Answer: fluid temperature rises

11. The maximum unit pressure of turbine and generator on the reinforced concrete should not
exceed
Answer: 17.62 kg/cm2

12. Water in vapor form remains a vapor as long as temperature is what relations to the dew
point temperature
Answer: below
13. The “refrigerating effect” of a refrigerant is a always:
Answer: Less than its latent heat

14. A burner should always start up in ___ fire and shut down in ___ fire
Answer: Low; low

15. A ___ boiler has water in the tubes and heat gases of combustion passing through the tubes
Answer: Water tube

16. If the compressor had been running satisfactorily for a long period of time but suddenly the
compartment temperature started to rise, the trouble might be:
Answer: Any of the other choices
(A refrigerant leak has developed, The solenoid valve has jammed shut, The expansion valve
may contain frozen water)

17. A type of polymer used for Styrofoam cups and clam shell food containers
Answer: Polystyrene (PS)

18. Mycotoxins are poisonous chemicals produced by:


Answer: Molds

19. The magnitude of the drag coefficient of a sphere in water is dependent upon all of the ff
except
Answer: Units of measure (SI or English engineering system)

20. A refrigeration system in which only part of the refrigerant passes over the heat transfer
surface is evaporated and the balance is separated from the vapor and recirculated
Answer: Flooded system

21. The boiling point of freon-12 at atmospheric pressure is


Answer: -22F

22. Which of the following vital components of the refrigeration system where both temperature
and pressure are increased?
Answer: Compressor

23. Which of the following must be done to eliminate frost on the discharge pipe of the
compressor?
Answer: None of the other choices
(Open the expansion valve, crack bypass valve, Regulates water to the condenser)

24. Orifice coefficients are used to determine:


Answer: Energy losses
25. Which of the following is used as high head turbine?
Answer: Impulse

26. A water-tube condenser has a total of 60 tubes. If these two passes, then compute the number
of tubes per pass
Answer: 15

27. Absolute zero is


Answer: 460 degrees below zero on the Fahrenheit scale

28. Which of the following can be the cause of low head pressure
Answer: Any of the other choices
(Leaky discharge valves, Too much or too cold condensing water, insufficient charge of
refrigerant)

29. What is the most common freon gas used in centrifugal compressors?
Answer: F-11

30. When purging an ammonia condenser into a bucket of water, one can tell when the air is out
and ammonia starts to come through the:
Answer: Change of bubbling sound of air to the cracking sound of ammonia

31. In the deep well installation or operation, the difference between static water level and
operating water level is known as:
Answer: Drawdown

32. White ice is


Answer: Due to dissolved air, gasses and impurities

33. Which of the following is a characteristic of an impulse turbine


Answer: Steam striking blades at zero angle

34. Heavy water is:


Answer: D2O

35. When excess solute in a solution settles to the bottom of the container, the process is called:
Answer: Precipitation

36. Calcium chloride is sometimes used in refrigeration system as a:


Answer: Secondary Coolant

37. If a compressor runs continuously, the cause might be a:


Answer: Stuck low-pressure switch
38. The thermal expansion valve is located between the:
Answer: Solenoid valve and the evaporator coils

39. The cooling water regulator is automatically actuated by which of the following?
Answer: The discharge pressure of the refrigerant

40. Based on the PSME code, what should be provided in each steam outlets if two or more
boilers will be connected in parallel?
Answer: Non-return and shut off valves

41. The amount of sensible heat for a sensible heat ratio of 0.8 and a total cooling load of 100 is
Answer: 80

42. Which refrigerant is used for the air conditioning of passenger aircraft cabin
Answer: Air

43. When a falling object reaches a speed at which the drag force equals its weight, it has
achieved:
Answer: A turbulent boundary layer

44. The minimum vertical distance from the floor or soil level to the top edge of the foundation
must be around
Answer: 120 mm

45. How many moisture be removed from air?


Answer: condensation and absorption

46. The charging valve is located between the:


Answer: King valve and the expansion valve

47. A “cold diffuser” is a:


Answer: Type of evaporator

48. Which of the following stops the compressor before the relief valve opens?
Answer: High pressure control

49. What is the excess refrigerant removed?


Answer: Charging side of the system

50. Which of the following is used in measuring the density of a brine solution?
Answer: A hydrometer
51. All of the following are common types of chemical reaction except
Answer: Fission

52. A ___ after each radiator allows ___ to pass through to the return line
Answer: Steam trap; condensate

53. A leaky solution valve can usually detected by:


Answer: Any of the other choices
(A fluctuating suction pressure gauge, a higher suction pressure, closing in on the suction
valve having no effect on the suction pressure)

54. If the cooling water to the condenser suddenly fails:


Answer: The compressor will shutdown

55. If Et is the total net energy generated by a plant in a certain period of time and Er is the rated
net energy capacity of the plant during the same period of time, then what is the plant
operating factor?
Answer: Et/Er

56. A thermometer sense which of the following?


Answer: Sensible heat

57. Is the subjective method in which the smoke density is visually compared to five
standardized white-black grids
Answer: Ringelman scale

58. Excessive head pressure is caused by


Answer: Flooded condenser tubes or cooling water temperature too high

59. What do you call a plate or vane used to direct or control movement of fluid or air within the
confined area?
Answer: Baffle

60. The total pressure supplied by the fan at maximum operating conditions
Answer: Net rating or fan boost

61. The system should be purged:


Answer: After the system has been shut down for few hours

62. What is likely to occur when sections of the impeller of a centrifugal pump are handling
vapor and other sections are handling liquid.
Answer: complete failure to operate
63. What is the chemical formula of Ozone?
Answer: O3

64. An excessively high head pressure would be caused by:


Answer: Insufficient cooling water to the condenser

65. When an electric motor nameplate indicate a “100-kW electric motor” then what does a 100-
kW rating refers to?
Answer: Mechanical power output

66. During a cooling and dehumidifying process, sensible and latent heats are removed in the
cooling coil. If Hs is the sensible heat and Ht is the total heat transferred, then determine the
coil sensible factor.
Answer: Hs/Ht

67. A single trunk piston-type compressor is undesirable for a Freon unit because the:
Answer: Lubricant mixes with the refrigerant

68. What is the first step that must be done when securing a system?
Answer: Close receiver (King) valve

69. The amount of CO can be determined by the color of Palladium Chloride. An amount of 30
ppm to 70 ppm will cause:
Answer: Slight Darkening

70. The water hammer phenomenon is primarily what kind of fluid mechanism?
Answer: Dynamic (a time-dependent phenomena)

71. The other name for Swamp Cooling is:


Answer: Wet Roof Cooling

72. The design of an air supply duct of an air condition system


Answer: affects the distribution

73. A Freon – 12 gage shows pressure and ____.


Answer: Saturation temperature

74. High superheat of the vapor in the system would cause


Answer: A decrease in capacity

75. When there is no work between the thermodynamic system and its surrounding, the quantity
of net heat transfer is equal to:
Answer: total energy of a closed system
76. The relation between the Fahrenheit absolute scale and the Celsius absolute scale is:
Answer: R = 1.8 Kelvin

77. External frost on inlet of expansion valve indicates:


Answer: Expansion valve plugged or dirty

78. The total cross section of a target atom is made up of


Answer: The absorption and scattering cross sections

79. The motor condition of an air conditioning unit can be checked with:
Answer: The continuity of light or with an ohmmeter

80. A number used to compare energy usage for different areas. It is calculated by dividing the
energy consumption by the footage of the conditioned area.
Answer: Energy utilization Index

81. What is the effect if the refrigerant is removed from the system too fast?
Answer: It may “freeze – up” the condenser

82. After the total force of the steam has lifted the safety valve off its seat, the steam enters the
____.
Answer: Huddling chamber

83. By which of the fooling processes heat mainly dissipates in cooling towers?
Answer: Evaporation

84. All of the following fluid phenomena are based on the force momentum principle of a
flowing fluid except:
Answer: Diesel automobile engines

85. All of the following are forms of drag on a body moving through a fluid except
Answer: D’alembert’s paradox drag

86. What is the use of the suction pressure regulating valve?


Answer: Maintains the back pressure in the evaporator coils

87. There shall be no foundation bolts less than _____.


Answer: 12 mm in diameter

88. The ratio of the rated cooling capacity divided by the amount of electrical power used:
Answer: Energy efficiency ratio (EER)
89. The scale trap is located between the:
Answer: King (liquid) valve and expansion valve

90. The temperature in the vegetable box should be approximately


Answer: 35 to 45 deg. F

91. A thermostat is a
Answer: Temperature-operated switch

92. The bulb for the thermal expansion valve is located:


Answer: Near the evaporator coil outlet

93. The coefficient of velocity is the ratio of the:


Answer: Actual velocity to the theoretical velocity

94. What is the specific humidity of dry air?


Answer: 0

95. What is the reason why a thermometer in vapor compression system is installed close to the
compressor?
Answer: Because it helps the operator in adjusting compressor for greatest efficiency

96. As the steam temperature increases the steam temperature:


Answer: Increases

97. Substances that have the ability to absorb moisture from the air are called:
Answer: Desiccants

98. Evaporative condenser is used to cool


Answer: condenser vapor

99. If the head pressure is too high:


Answer: The high pressure cutout switch should operate before the relief valve opens

100. A material or substance that is accidentally or intentionally introduce to the environment


in a quantity that exceeds what occurs naturally
Answer: Pollutant

101. The greatest decrease in refrigerant temperature occurs in the ____


Answer: Evaporator

102. The diesel cycle is the ideal cycle for a :


Answer: compression-ignition engine
103. The moisture in a refrigeration system can be removed with help of which of the
following?
Answer: Driers

104. What process is employed when the turbine steam power plants experience excessive
moisture?
Answer: reheating

105. The path that directs the flow of refrigerant through the compressor
Answer: Valve

106. Which of the following is another name for the liquid valve?
Answer: King valve

107. Expansion factors take into account the:


Answer: Effects of compressibility

108. The term used to express the amount moisture in a given sample of air. It is compared
with the amount of moisture in a given sample of air.
Answer: Relative Humidity

109. What are the four basic methods of determining whether the proper amount of refrigerant
being added to the system?
Answer: Bull’s eye, weight, pressure and frost line

110. Which of the following components of the window air conditioning system must be
cleaned annually?
Answer: All of these
(Evaporator and Condenser, Motor and Compressor, Fan Blades and Fan Motor)

111. At present, the number of true elementary particles, which include leptons and quarks is
thought to be
Answer: 12

112. All of the following statements about conjugate acids and bases are true except
Answer: A conjugate base results when a base accepts a proton

113. A double-seated valve allows the valve to be


Answer: Packed in the wide open or closed position

114. In order to keep combustion products inside the combustion chamber and stack system,
balanced draft system may actually operate with a slight
Answer: negative pressure
115. What is the cause of pressure drop in the boiler, condenser and the pipings between
different components? Because of this drop, steam leaving the boiler at a lower pressure.
Answer: Fluid friction

116. It is a form of oxygen photochemically produced in nature.


Answer: Ozone

117. Absolute zero on the Fahrenheit scale is equal to


Answer: -460

118. Low suction pressure is caused by:


Answer: Solenoid valve not functioning properly

119. What do you call the storage tank for liquid refrigerant?
Answer: Receiver tank

120. Why should you avoid bending or twisting of fan blades in an air conditioning unit?
Answer: It will wear out the motor bearings and cause noise

121. All of the following statements are characteristics of bases except


Answer: They have a pH between 0 and 7

122. Concrete foundation should have steel bar reinforcement placed vertically and
horizontally to avoid ____.
Answer: Thermal cracking

123. The pump will not cavitate if the available Net Positive Suction Head (NPSH) is:
Answer: equal or greater than the required NPHS

124. What will happen to the capacity if the superheat is increased on the suction side?
Answer: Decreases

125. When heavy electrical currents are involved, the thermostat will be operated by a
Answer: Relay

126. Valves in piping in an ammonia system are made of


Answer: Iron

127. Which of the following is not essential to a compression refrigeration system?


Answer: A receiver

128. Nitrogen occupied almost how much of the Earth’s atmosphere?


Answer: 3/4
129. If frost forms on the cylinders, the cause would be:
Answer: Expansion valve open too wide

130. Which of the following gasket materials should be on Freon system?


Answer: Asbestos or Metallic

131. Which of the following would cause the crankcase and head to get hot with low suction
pressure?
Answer: Insufficient refrigeration

132. Moisture in a system will cause a:


Answer: Faulty expansion valve

133. The steam turbine foundation should be designed to support the machine load plus how
many percent for impact, condenser load, floor loads and dead loads?
Answer: 25

134. When air is heated, what happened to its relative humidity?


Answer: Remain constant

135. Sweating of crankcase is caused by which of the following?


Answer: Too much oil in the system

136. What do you call a material in a dryer?


Answer: Desiccant

137. The matching model and full-scale prototype results a fluid dynamic phenomena
involving a fully submerged body requires equality of:
Answer: Reynolds number

138. Which of the following uses a secondary refrigerant?


Answer: Ice plant

139. When the outlet temperature at the evaporator exceeds the inlet temperature, the
condition is known as:
Answer: Superheating

140. In series pipe systems, all of the following parameters vary from section to section except
Answer: Mass flow

141. Which of the following is used to control refrigeration compressor capacity?


Answer: Unloader
142. TOR is a unit equivalent to:
Answer: 12.66 kN-m/hr

143. The agent used in an indirect reefer system is:


Answer: Calcium chloride or sodium chloride

144. If the W is the work output of a heat engine and H is the total heat input of an engine,
then determine its thermal efficiency, e.
Answer: e = W/H

145. Which of the following items is not important when using a halide torch?
Answer: Adjust to a clear white flame

146. A dehumidifier is usually a small hermitic refrigerating system. It has both a condenser
and an evaporator. Many older systems use R – 12 or R – 500. The newer units are:
Answer: R – 134a

147. What is the method used to evaluate all welds performed on pressure parts of boiler tube
materials?
Answer: Radiographic test

148. Faulty F – 12 compressor valves will be indicated by:


Answer: All of the other choices
(Gradual or sudden decrease in capacity, compressor running continuously, Low head
pressure – high suction pressure)

149. Heat exchanger in which water flows by gravity over the outside of tubes or plates.
Answer: Baudelot cooler

150. Which of the following is not a sound absorptive material?


Answer: Mirrors

151. The desirable temperature inside an air conditioned auditorium is:


Answer: 20 C

152. An odorless refrigerant whose boiling point varies over a wide range of temperatures.
Answer: Freon 12

153. The cooling water regulator is actuated by


Answer: Pressure of the refrigerant

154. The psychrometric chart in air conditioning determines the


Answer: Moist air conditions
155. Blowback of a safety valve is to prevent:
Answer: Chattering

156. When starting a refrigerant unit, be sure the water side of the condenser is ____.
Answer: Vented

157. A temperature measurement in an ordinary temperature which has constant specific


humidity.
Answer: Dew point temperature

158. What adding oil to a freon system, one must be sure that
Answer: All air is removed from the pump and fitting

159. The primary purpose of a turbine in a fluid is to:


Answer: Extract energy from the flow

160. Which of the following is to be checked as regular monthly maintenance schedule of a


console air conditioning units?
Answer: Duct dampers, registers and diffusers

161. A company is interested to produce a water turbine wherein only little energy is required
or necessary because the guide vanes are to be controlled. The turbine must be a:
Answer: Kaplan turbine

162. To check the lack of refrigerant or clogged refrigerant lines:


Answer: Installing service valves must be necessary

163. Condensate from the non return steam trap is pumped from the condensate return tank of
the:
Answer: Boiler

164. The locus of elevation is:


Answer: Hydraulic Gradient

165. Which of the following cannot be used to describe neutron kinetic energy levels
Answer: Freezing

166. It is important to take some moisture from the air dehumidify it if the relative humidity
reaches high levels. To do this, it requires cooling the air:
Answer: Below its dew point temperature

167. The vapor cycle in thermal efficiency as the refrigeration cycle to the
Answer: COP for a refrigerator
168. A scale trap in a Freon system will be found on the
Answer: Suction side

169. Which of the following is not used as method to measure air velocities?
Answer: Open type Barometer

170. Which of the following is considered as comfort condition in air conditioning?


Answer: 20 C DBT, 60% RH

171. Which of the following acts as ignition accelerator for internal combustion engine fuels?
Answer: Acetone peroxide

172. The purpose of the evaporator is to


Answer: Absorb latent heat of vaporization

173. The speed at which a reaction proceeds to equilibrium is the purview of


Answer: Reaction kinetics

174. What do you call the liquid reaching the compressor through the suction?
Answer: Flooding back

175. Which of the following draft rely on the stack effect to draw off combustion gases?
Answer: Natural draft

176. The relative humidity becomes 100% and where the water vapor starts to condense.
Answer: Dew point

177. Which of the following statements is correct? The relative humidity of an air water vapor
mixture.
Answer: is the ratio of partial pressure of water vapor to the saturation pressure at the
mixture temperature

178. The following are standard characteristics of Freon – 11 except:


Answer: Boiling point over 200 F

179. The bhp/ton refrigeration requirement with increase in condenser temperature. In a vapor
compression system that uses reciprocating compressor receiving refrigerant gas at constant
suction temperature, will follow:
Answer: Linearly increasing characteristic
180. In an air conditioning unit, the thermostat fails. The unit did not start. How do you test
the operation of the thermostat?
Answer: Cover the air outlet and air inlet with a cloth. The air will recirculate into the
unit and the temperature will quickly drop to the cut out temperature.

181. All of the following temperature have meaning in psychrometrics except:


Answer: Adiabatic wall temperature

182. The maximum continuous power available from a hydro – electric plant under the most
adverse hydraulic conditions is called
Answer: Firm power

183. Which of the following dehumidifier is often used to reheat the air after moisture is
removed?
Answer: Condenser

184. Which of the following is a type of evaporator?


Answer: shell-and-tube

185. All of the following factors affect rates of reaction except


Answer: Pressure

186. Which of the following would cause a high suction pressure?


Answer: Expansion valve open too wide

187. Zinc rods are found in the:


Answer: Salt water side of the condenser

188. The sum of the internal energy and the product of pressure and specific volume is known
as the:
Answer: enthalpy

189. Any foreign matter in the coal feed mechanism of the screw-feed stoker is best removed
by:
Answer: Using the cutoff gate at the bottom of the hopper

190. Based on the PSME code, what should be the humidity of air to be used for comfort
cooling?
Answer: 50% - 60% relative humidity
191. Which of the following must be checked up if an automatic Freon system will not start
up?
Answer: All of the other choices
(High pressure cutout, reset mechanism, Low-pressure cutout)

192. Refers to the use of composting soil beds


Answer: Biofiltration

193. What is the usual cop of a domestic refrigerator?


Answer: More than 1

194. In a hydro-electric plant using a Francis turbine with medium head, the speed can be
regulated using the
Answer: Wicket gate

195. The temperature bulb of the thermo-expansion valve is attached to which of the
following?
Answer: Evaporator coil outlet

196. Which of the following refrigerants would give the most trouble when operating with
warm circulating water?
Answer: CO2

197. A rule which states that any waste derived from the treatment of a hazardous waste
remains a hazardous waste
Answer: The derived from rule

198. A device for holding open the suction valve and drawing gas from the suction manifold
and returning it to the suction line without compressing it is called
Answer: Cylinder unloader

199. The intake pipe to a hydraulic turbine from a dam


Answer: Penstock

200. What consists of weak solutions of sulfuric, hydrochloric, and to a lesser extent, nitric
acids?
Answer: Acid rain

201. The expansion valve does not seem to be operating properly. There is high superheat.
Test by listening to the sound of Freon flooding through the tubes and ____.
Answer: Warm bulb with hand
202. Swimming pool water needs
Answer: de-chlorination

203. Oxygen is approximately what percent by weight in the atmosphere?


Answer: 23%

204. Which of the following could be used to check a leak in an ammonia system?
Answer: Litmus paper & Sulfur stick

205. The determination of properties and behavior of atmospheric air usually the purview of:
Answer: Psychrometrics

206. A valve the released steam from the boiler by operating at a pre-determined pressure in
order to keep the steam pressure from rising more than the pressure from which the valve is
set is called a
Answer: Safety valve

207. If the critical temperature of a refrigerant is too close to the desired condensing
temperature, the equipment must necessarily be of
Answer: Extra heavy construction

208. If a freon – 12 compressor trip out on “cut-out”, the solenoid valve closes by which of the
following?
Answer: An electrical release

209. Which of the following would cause the suction switch to remain open?
Answer: Bellows broken or jammed

210. In the upper atmosphere, ozone is made by ultraviolet light reacting with:
Answer: Oxygen

211. How is ammonia system purged so that operator will not be overcome by the fumes?
Answer: Into the bucket of water

212. A ton of refrigeration is equal to the removal of


Answer: 288,000 btu per 24 hrs.

213. As a good practical rule, the foundation depth may be taken as how many times of the
engine stroke?
Answer: 2:5 to 3:5

214. Most stacks are built of height less than _____.


Answer: 30 m
215. The phenomenon that warm air rise and cold air settle is called:
Answer: Stratification

216. Type of refrigerant control which maintains pressure difference between high side and
low side pressure in refrigerating mechanism:
Answer: Expansion valve

217. The crossover connection in an ammonia system can be used to ____.


Answer: Hot-gas defrost

218. Which of the following refrigerants has lowest freezing point temperature?
Answer: Freon 22

219. The operation that produces highest noise level is:


Answer: Riveting

220. Select the one in which secondary refrigerant is used


Answer: Ice plant

221. Which of the following is not essential to a centrifugal type compressor system?
Answer: Distiller

222. Two compressor should not be run in parallel because


Answer: There is a possibility of losing oil

223. Which of the following is also known as refrigerant no. R-717?


Answer: Ammonia

224. During sensible heating, the humidity remains constant but the relative humidity.
Answer: decreases

225. The term suction units is used with


Answer: Induced draft

226. If the compressor had been running satisfactorily for a long period of time but the oil
level was rising slowly, one should:
Answer: Shut down the compressor and check the oil level with the machine stopped

227. What amount of air is required in a low bypass factor?


Answer: Lesser

228. The reduction of nuclear radiation intensity (called attenuation) is accomplished by:
Answer: Shielding
229. The ozone concentration of 0.10 parts per million (ppm) is generally considered the
maximum permissible for how many hours exposure?
Answer: 8 Hours

230. Which do you think is very important in adjusting compressor “v” belts?
Answer: Allow ½” slack

231. An engineer inspected an air-conditioning unit. He found out that the unit does not
produce any cooling effect, however, the air-conditioning unit is running. He checked the
temperature of the condenser and evaporator and hat the unit run. He found out that there was
no change in temperature. What should he do?
Answer: Charge with new refrigerant

232. The temperature at which water vapor in the atmosphere begins to condense is known as:
Answer: dew point

233. The ice making capacity is always


Answer: Directly proportional to the refrigerating effect

234. If the compressor short-cycles on the high-pressure cutout, which of the following would
you check?
Answer: All of the other choices
(If plenty of cooling water is running through but it is not picking up heat, the condenser
tubes need cleaning, Be sure system is getting cooling water, Check for too much refrigerant
in the system)

235. What is the instrument used to register relative humidity?


Answer: Hygrometer

236. What has an octane rating of more than 100?


Answer: Benzol

237. A device used to keep moisture from passing through the system is called:
Answer: Dehydrator

238. If the freezing point of water is 0 C, which of the following is its melting point following
is its melting point
Answer: 0 C

239. What is the pressure present inside the casing of an impulse turbine?
Answer: Atmospheric pressure
240. A vena contracta in a fluid jet issuing through a hole in a plate is located approximately:
Answer: At jet’s minimum diameter

241. If the compressor short-cycles on the low-pressure cutout, the trouble might be:
Answer: Any of the other choices
(Too much frost on the evaporator coils, dirty traps and strainers, lack of refrigerant)

242. The amount of heat necessary to bring up temperature of a unit mass of a substance
through unit degree is called
Answer: Total heat

243. If the expansion valve capillary tube is pinched, which of the following be replaced?
Answer: All of the other choices (called the power element unit)
(Bulb, tube, diaphragm)

244. The coefficient of discharge is the ratio of the.


Answer: Actual discharge to the theoretical discharge

245. Pitot tube is used to measure the:


Answer: Velocity of flow

246. The principle of the centrifugal system is based on which of the following?
Answer: Kinetic energy

247. The disadvantage of a CO2 system over an ammonia system is the fact that
Answer: All of the other choices
(The pipes and fittings of a co2 system must be of the high-pressure type, The CO2 system
requires a larger prime remover, The CO2 system operates at a much higher pressure)

248. If ice will form in a solution of water and salt, then it is at temperature called:
Answer: Freezing point depression

249. The suction pressure in a Freon system should be:


Answer: The pressure which corresponds with a temperature about 20F below the
temperature of the icebox.

250. Wb is shaft work of an engine and Wi is indicated work of an engine. If mechanical is


present in the engine mechanism, then.
Answer: Wb is less than Wi

251. What is the relationship of the capacity of a centrifugal pump, Q, to impeller diameter, D,
when there are two impeller diameters in the same pump?
Answer: Q is directly proportional to the ratio of D
252. The combination of enthalpy and kinetic energy of fluid is termed as:
Answer: Stagnation enthalpy

253. Before any repair work is attempted on any gas-fired boiler the:
Answer: Main gas cock must be secured

254. An air-conditioning system in which water is chilled or cooled and which passes the
evaporator coils
Answer: Chilled water system

255. Refers to organic waste produced from biological water waste treatment processes
Answer: Biosolids

256. As far as combustion chamber design is considered, the maximum power output of a
given engine can be increased by:
Answer: Decreasing combustion chamber volume

257. A radioactive gas produce from the decay of radium within the rocks beneath a building
Answer: Radon

258. Ozone filters out what type of radiation that damages crops and causes skin cancer?
Answer: Ultraviolet radiation

259. The ratio of the sum of individual maximum demands of the system to the overall
maximum demand of the whole system
Answer: Diversity factor

260. A bell coleman cycle is also known as


Answer: Reversed Joule cycle

261. It is used deliver concentrated airstreams into a room. Many have one-way or two-way
adjustable air stream deflectors.
Answer: Register

262. The sensible heat ratio is 0.8. that is


Answer: 80% sensible heat and 20% latent heat

263. Closing the solenoid valve will stop the compressor through the ____.
Answer: Low-pressure cutout switch

264. A refrigerating unit of one (1) ton capacity can remove?


Answer: 200 btu’s per min
265. Which of the following is the usual case of slugging?
Answer: Expansion valve not operating properly

266. All of the following occur during reduction of a substance except


Answer: Loss of electrons

267. The secondary refrigerant used in milk chilling plants is generally


Answer: Glycol

268. Where is the solenoid coil installed?


Answer: Vertically over the valve

269. The thermal expansion valve responds to the


Answer: Amount of superheat in the vapor

270. To eliminate transmission of vibration, the foundation should be isolated from the floor
slabs of building footings at least how may mm around its perimeter?
Answer: 25

271. Which one is commonly used liquid absorbent?


Answer: Ethylene glycol

272. The low-pressure control switch:


Answer: Cuts out the compressor to maintain proper flow

273. If the outlet of the thermostatic valve is warmer than the inlet, it indicates
Answer: Thermostatic valve not working properly

274. Compute the wall gain load of a load storage room when A is the outside surface area of
the wall, D is the temperature differential across the wall, and U is the overall coefficient of
heat transmission.
Answer: AUD

275. A leak on the fuel oil suction line between the tank and the suction side of the fuel oil
pump would result in:
Answer: Air entering the suction line

276. How is the CO2 system is purged?


Answer: When CO2 comes out of the purge valve, frost will form on a piece of metal
held near the outlet

277. The relief valve on a CO2 machine is located:


Answer: On the discharge pipe between the compressor and discharge valve
278. The high-pressure side of the system is sometimes referred to as the:
Answer: Hot side

279. Combined pressure of cooling and humidifying is also known as:


Answer: Evaporative cooling process

280. The schedule of a pipe, N, indicates the thickness of the pipe wall. If the allowable stress
of the pipe is S, then what is the internal pressure equal to?
Answer: N x S

281. Which of the following area of work requires lowest noise level?
Answer: Library

282. Which of the following contribute to the deterioration of the earth’s ozone layer?
Answer: Chlorofluorocarbons

283. What is the least number of compressors a multistage system that will use?
Answer: Two

284. Is a soluble compound that reduces a liquid’s surface tension or reduces the interfacial
tension between a liquid and a solid
Answer: Surface – acting agent or surfactant

285. To secure the belts embedded within the foundation, the distance of the edges of the
foundation from the bedplate must be ____.
Answer: 120 mm to 300 mm

286. In Stirling and Ericsson Cycles, the efficiency can be increased by:
Answer: Regeneration

287. The thermal expansion valve


Answer: Controls the quantity of liquid refrigerant going to the evaporator coils

288. Too low suction pressure could be caused by:


Answer: Any of the other choices
(Dirty scale traps, Shortage of refrigerant gas, Too much oil in the system)

289. The static pressure drop due to friction through the boiler and stack.
Answer: Draft loss

290. In a freon-11 system there is no


Answer: Distiller
291. The coefficient of contraction is the ratio of the:
Answer: Area of vena contracta to the orifice area

292. Which of the following should not be used to clean scale traps:
Answer: Cotton waste

293. Most observe properties of light and other radiant energy are consistent with waves in
nature, but in interaction with matter, electromagnetic energy behaves as though it consists of
discrete pieces or
Answer: Quanta

294. Boilers using soft coal must have ____ furnace volume to reduce the danger of ____
Answer: Large; smoking

295. Which of the following would you apply if a person got Freon in his eyes?
Answer: Sterile mineral oil

296. The compressor will run continuously if there is:


Answer: Any of the other choices
(Too heavy a load, Insufficient refrigerant, Air in the system)

297. A hot crankcase and cylinder head accompanied by a low suction pressure would be
caused by
Answer: Insufficient refrigerant

298. One foot water is equal to:


Answer: 62.43 lb/in2

299. The purpose of the oil trap is:


Answer: To remove oil from the refrigerating gas

300. Many pressure gauges on a Freon system have two dials or graduations on one gauge.
The two dial represents:
Answer: Pressure and temperature

301. The faster way to remove frost from a direct expansion finned tube evaporator is to:
Answer: Send hot gas through the coil

302. When ordering an expansion valve which of the following information is necessary?
Answer: Size, tonnage, temperature, and pressure

303. The elements of a thermostat switch are usually of the ____.


Answer: Bimetal type
304. Which of the following characteristics that is not desirable in a refrigerant?
Answer: Low latent heat of vaporization

305. What do bodies at a temperature above absolute zero?


Answer: It emits Thermal Radiation

306. The Horizontal Scale (Abscissa) in the psychrometric Chart represents:


Answer: Dry bulb temperature

307. The thermostatic expansion valve is designed to maintain constant ____.


Answer: Superheat

308. The charging connection in a refrigerating system is located:


Answer: Between the king valve and the solenoid valve

309. The latent heat of fusion of ice is


Answer: 144 Btu

310. The ratio absorbed by the transfer fluid to the original incident energy striking the
collector
Answer: Collector efficiency

311. If m is the mass of dry air and H is the specific enthalpy of the water vapor in air and r is
the humidity ratio, then determine the latent heat of any dry air.
Answer: m(r x H)

312. The dehydrator is located between the:


Answer: Receiver and expansion valve

313. In a vapor compression cycle the lowest temperature is found in


Answer: Evaporator

314. The volume flow passes through a venturimeter is:


Answer: Constant

315. Anchor bolts in a machine foundation should be embedded in concrete of at least how
many time of the bolt diameter?
Answer: 30

316. Which do you think is the effect of “subcooling”?


Answer: It reduces the horsepower per ton of refrigeration
317. When air is saturated, the wet bulb depression is:
Answer: Zero

318. A precooler, if used is located between the:


Answer: Condenser and expansion valve

319. A Kaplan turbine is:


Answer: Low head axial flow turbine

320. The refrigerant temperature is at its maximum just before it enters the ____
Answer: Condenser

321. The machine foundation must have a factor of safety of:


Answer: 5

322. In a refrigerating system, the heat absorbed in the evaporator per kg mass of refrigerant
passing through
Answer: Equals the increase in enthalpy

323. Which of the following is the common classification of ducts?


Answer: All of these
(Conditioned Air – Duct, Recirculating, Fresh – air ducts)

324. All of the following process can be found in a psychrometric chart except:
Answer: Natural convection

325. If an electrically-operated compressor failed to start the cause might be:


Answer: Any of the other choices
(Burned out holding coils in solenoid valve, An open switch, A blown fuse)

326. Five pounds of water heated to raise the temperature 2F requires how many BTU?
Answer: 10 Btu

327. Air circulation in the icebox is accomplished by the use of which of the following?
Answer: Diffuser fans

328. In sensible heating cooling following parameter remains unchanged


Answer: Humidity ratio

329. Venturi meters, pitot static gauges, orifice meters, flow nozzles, and differential
manometers all depend upon the relationship between:
Answer: Flow velocity and pressure
330. It is used to deliver widespread, fan-shaped flows of air into the room
Answer: Diffuser

331. What turbidimeter that gives direct reading in ppm?


Answer: Jackson turbidimeter

332. How much will be removed by one-ton refrigeration unit?


Answer: 200 Btu per min

333. A double-trunk piston is used to:


Answer: All of other choices
(Prevent oil from mixing with the refrigerant, Prevent gas from getting to crankcase, Prevent
oil from mixing with the refrigerant)

334. Purpose of the receiver is to


Answer: Store the refrigerant

335. Latent heat


Answer: Cannot be measured with a thermometer

336. A type of water turbine where a jet of water is made to fall on the blades or buckets and
due to the impulse of water, the turbine starts to move.
Answer: Pelton wheel

337. When does the refrigerant gives-up heat?


Answer: When it condenses

338. A term used to mean the corrective steps taken to return the environment to its original
condition
Answer: Remediation

339. The back – pressure regulating valve:


Answer: Maintains a fixed pressure in the evaporator coils

340. What is the relationship of the horse power of a centrifugal pump, Hp, to the impeller
speed, S, if the pump is at two different rotative speeds?
Answer: Hp is directly proportional to the cube of the ratio of S

341. What must be the value of the available Net Positive Suction Head (NPHS) of a
centrifugal pump compared to its required NPHS to avoid losing priming?
Answer: Available NPHS greater than required NPHS
342. The diesel engine foundation safe soil bearing pressure is:
Answer: 4,890 kg/cm2

343. Aside from maintain appropriate temperature for food cold storage, how is desiccation
minimized or decreased?
Answer: Maintain humidity ratio

344. If C is the capacity of the compressor in a refrigeration system and F is the heat rejection
factor, then calculate the condenser load, L.
Answer: L = C x F

345. The following examples of indirect (secondary) measurements to measure flow rates
using obstruction meters except:
Answer: Weight and mass scales

346. When required, a regulator water valve in refrigerating system should be


Answer: In the water outlet

347. What is the relative humidity when the dew point and dry bulb temperature are equal?
Answer: 100%

348. Compare in refrigerating effect per unit mass of refrigerant circulated for a superheated
cycle that produces useful cooling and a saturated cycle, for the same vaporizing and
condensing temperature
Answer: Greater for a superheated cycle

349. If PV is the power required for a vapor compression refrigeration system, then what is the
power required for an air refrigeration system, assuming that they have the same capacity?
Answer: 5PV

350. What do you call the mixture if the solute particles of a solid suspended in a liquid are
larger than molecules?
Answer: Suspension

351. Lithium bromide is used in refrigeration system in:


Answer: Absorbers

352. The removal of dissolved gas or other volatile component from liquid by exposing the
liquid to air or steam is known as:
Answer: Stripping

353. Which of the following measures the density of salt in water?


Answer: Salimeter
354. In a power driven pump, each piston stroke is displaced by 360 divided the ____.
Answer: Number of cylinders

355. What is the boiling temperature of F 22?


Answer: -40 C

356. What is another name of discharge pressure?


Answer: Head pressure

357. If the compressor were to run out continuously without lowering the temperature, the
trouble would probably be:
Answer: Any of the other choices
(Insufficient refrigerant in the system, Leaky discharge valve, Leaks in the system)

358. The ratio of sum of individual maximum demands of the system to the maximum demand
of the whole system is known as
Answer: Diversity factor

359. The temperature bulb of solenoid valve is attached to the ____.


Answer: Wall of the icebox

360. What must be done to change the direction of rotation of a 440-volt, 3-phase induction
motor?
Answer: Interchange any two power leads

361. In order to remove the fly ashes from the flue gas, which of the following must a power
plant be equipped with?
Answer: Electrostatic precipitator

362. Which of the following is a type of water turbine?


Answer: Pelton

363. In sensible heating the absolute humidity remains constant but the relative humidity:
Answer: Decrease

364. All of the following process can be found on a psychrometric chart except?
Answer: Black body radiation

365. When checking zinc plates in a condenser, one should:


Answer: Clean the plates and renew worn out ones

366. Turbidity in water is due to:


Answer: Finally divided particles of clay, silt and organic matter
367. The force when applied to a mass of one kilogram will give mass an acceleration of one
meter per second for every second called:
Answer: Newton

368. A precooler is sometimes installed between the


Answer: Condenser and expansion valve

369. A type of polymer used for detergents, milk bottles, oil containers and toys
Answer: High density polyethelene (HDPE)

370. The fact that the amount of slightly soluble gas absorbed in a liquid is proportional to the
partial pressure of the gas is known as:
Answer: Henry’s Law

371. What is the pressure at the exit of a draft tube in a turbine?


Answer: Atmospheric

372. What do you call the device that is used as low-pressure control and high-pressure cutout
on a compressor?
Answer: Pressure Controller

373. Redox reactions can often be type of:


Answer: Double replacement

374. Excessive head pressure is caused by


Answer: Any of the other choices
(Dirty condenser tubes, Insufficient cooling water to condenser, Air or non condensable gas
in the system)

375. In which part of the vapor compression cycle there is abrupt change in pressure and
temperature?
Answer: Expansion valve

376. The kinetic energy of a moving fluid is used to isentropically compressed the fluid to
state of zero velocity. The temperature of a moving fluid at the state zero velocity is called:
Answer: Critical temperature

377. Which is not commonly used to cool and dehumidify equipment?


Answer: Calcium chloride

378. What is the primary cause of smog formation?


Answer: Nitrogen oxides
379. The dehydrating agent in a Freon system is usually
Answer: Activated alumina

380. The solenoid valve is actuated by which of the following?


Answer: A magnet

381. Which of the following does not use ambient air for propulsion?
Answer: Turbo-prop

382. Which of the following would not cause high suction pressure?
Answer: Insufficient refrigeration

383. What is the effect of superheating the refrigerant?


Answer: It increase the coefficient of performance

384. The color of the flame of halide torch, in case of leakage of Freon refrigerant, will change
to
Answer: Green

385. If there is too much lube oil in the system what must be done?
Answer: Remove same at once

386. If the thermal bulb becomes loose on the evaporator coils, it will cause ____.
Answer: Improper operation of expansion valve

387. The effect of superheating the refrigerant is to:


Answer: Decrease the COP

388. In power plant electrostatic precipitators is installed between:


Answer: Furnace and chimney

389. Equipment leaks from plant equipment are known as


Answer: Fugitive emissions

390. Per capita consumption of water is generally taken as:


Answer: 150-300 liters

391. What is the heat that is removed from the space to be cooled, which is the same as the
heat absorbed by the cooling coils?
Answer: Refrigerating effect

392. Horsepower per ton of refrigeration is expressed as:


Answer: Cop/4.75
393. What must be done first when opening a single packed stop valve?
Answer: Loosen the packing before opening

394. The amount of CO2 or Freon in a cylinder is measured by


Answer: Weight

395. The coefficient of velocity is approximately


Answer: 0.30 to 0.50

396. Carries needed to deliver air to the conditioned space it is made of sheet of metals like
aluminum, galvanized sheet steel and some structural materials that will not burn
Answer: Duct

397. Which of the following best described a Freon?


Answer: All of the other choices
(Colorless, Odorless, Non-poisonous)

398. Before securing a compressor to do maintenance work on it, be sure to:


Answer: Have spare parts ready and pump down the system

399. A bull’s eye in a full liquid line will appear ____.


Answer: Clear

400. What operates low-pressure cutout switch?


Answer: Bellows

401. A liquid mixture having constant maximum and minimum boiling points. Refrigerants
comprising this mixture do not combine chemically, yet the mixture provides constant
characteristics.
Answer: Azeotropic Mixture

402. Are organic compounds manufactured in oily liquid and solid forms through the late
1970s and subsequently prohibited.
Answer: Polychlorinated biphenyls (PCBs)

403. What is the temperature range of an air-conditioning application where dry air can be
considered ideal gas?
Answer: -10 to 50 C

404. The coefficient of the velocity, Cv accounts for the:


Answer: Small effect on the flow area of contraction, friction, and turbulence
405. During the re-expansion portion of the refrigeration compressor cycle
Answer: The suction valve is closed and the discharge valve is closed

406. Refers to the high-temperature removal of tarry substances from the interior of the carbon
granule, leaving a highly porous structure
Answer: Activated

407. Are by products of reaction between combustion products


Answer: Oxidants

408. Are any solid particulate matter that becomes airborne, with the exception of particulate
matter emitted from the exhaust stack of a combustion process
Answer: Dusts or fugitive dusts

409. The purpose of the dehydrator is to


Answer: Remove moisture from the refrigerant

410. The angle between the stack and guy wire is usually
Answer: 60 degrees

411. A refrigeration that deals with producing temperature of -157 C or lower.


Answer: Cryogenics

412. The purge valve is located:


Answer: In the highest part of the system

413. Thermal expansion valves are usually made of the:


Answer: Diaphragm and Bellows type

414. The purpose of the expansion valve by-pass is to


Answer: Controls the refrigerant to the evaporator in case the automatic valves fail

415. The water regulating valve is operated by the


Answer: None of the other choices
(Compressor discharge pressure, Compressor suction pressure, Compression discharge
temperature)

416. Superheat is heat added ____.


Answer: After all liquid has been changed to vapor

417. The ratio of maximum load to the rated plant capacity is called
Answer: Utilization factor
418. The highest temperature in vapor compression cycle is produced during:
Answer: Compressor discharge

419. Dry analysis is a fractional analysis of the products of combustion which does not
include:
Answer: Water vapor

420. Valves and piping in an ammonia system are made of:


Answer: Iron

421. The locus of elevation to which water will rise in the piezometer tube is termed:
Answer: Hydraulic gradient

422. A substance with surface area per unit of weight, and intricate pore structure, and a
hydrophobic surface
Answer: Adsorbent substance

423. The purpose of the expansion valve is to control the flow of the refrigerant to the
evaporator. The other function is to:
Answer: Reduce the pressure of the liquid refrigerant

424. An ammonia leak will turn litmus paper ____.


Answer: Blue

425. What is the most common freon gas used in reciprocating compressors?
Answer: F-12

426. The empirical coefficient e in machine foundation if not given is assumed


Answer: 0.11

427. The capacity of a centrifugal type compressor is controlled by which of the following?
Answer: Regulating the speed or regulating the suction pressure

428. Antifreeze chemicals are:


Answer: Those that lower down the freezing points of liquids

429. A type of refrigerant that will not damage the ozone layer
Answer: Hydrofluorocarbons (HCF’s)

430. At 0 psig, how may BTU necessary to change 1 lb of water at 212 F of steam?
Answer: 970
431. Modern stacks are seldom built higher than ____.
Answer: 60 m

432. If Pi is the indicated horsepower and Pb is the indicated horsepower of a compressor,


then what is mechanical efficiency, Em, equal to:
Answer: Em = Pi/Pb

433. A refrigeration control that guards the compressor forms overloads brought about by
abruptly increases loads resulting from defrosting, warm products, and others is called:
Answer: Suction hold-back valve

434. When a solvent has dissolve as much as it can, the mixture is called:
Answer: Saturated solution

435. Fan motors in air conditioning usually have:


Answer: 2 or 3 speeds

436. In the upper atmosphere, ozone is made by ultraviolet light reacting with:
Answer: Oxygen

437. Which of the following is the oil used in a refrigeration system?


Answer: Straight mineral oil

438. The following are all examples of indirect (secondary) miscellaneous methods to measure
flow except:
Answer: Positive displacement meters

439. Which of the following is the function of a suction pressure regulating valve?
Answer: Maintains proper back pressure

440. What is the use of back pressure regulating valve?


Answer: Maintains a fixed pressure in the evaporator coils

441. If Wt is the turbine shaft work of a gas turbine unit, Wc in duel, then determine its
thermal efficiency.
Answer: (Wt-Wc) / Q

442. The solenoid valve is controlled by


Answer: The temperature in the icebox

443. Where is the excess refrigerant removed?


Answer: Charging side of the system
444. All of the following units of energy except
Answer: Pascals

445. What usually happened if brine has a high specific gravity?


Answer: It will crystallize

446. There are three basic boiler types, namely;


Answer: Cast-iron, fire-tube and water tube boilers

447. Measurements of a device’s ability to remove atmosphere air from test air.
Answer: Atmospheric Dust Spot Efficiency

448. The normal cut-out setting of a window unit thermostat between:


Answer: 13 C to 16 C

449. The use of water to carry heat occupied spaces


Answer: Hydronic Heating System

450. Reheating process is normally employed in steam plant when:


Answer: Turbine undergoes excessive moisture

451. In the discharge line between the compressor and the condenser one would find:
Answer: High pressure, high temperature gas

452. A good refrigerant should be


Answer: All of the other choices
(Non-poisonous, non-inflammable, non-explosive)

453. What is the relationship of the horse power of a centrifugal pump, Hp, to the impeller
speed, S, if the pump is at two different rotative speeds?
Answer: Hp is directly proportional to the cube of the ratio of S

454. Which of the following is the reason when the crankcase is cooler than the suction line?
Answer: Too much refrigerant

455. Humidity is a measure of which of the following?


Answer: Water vapor content

456. High head turbine is a/an:


Answer: Impulse

457. Critical temperature is that temperature above which


Answer: A gas will never liquefy
458. The fact that a fluid’s velocity increases as the cross-sectional area of the pipe through
which it flow decrease is due to:
Answer: The continuity equation

459. Which of the following is important for evaporator coils?


Answer: It should have air completely surrounding them

460. Which of the following types of air dryers works by absorbing moisture on a solid
desiccant or drying material such as activated alumina, silicone gel, or molecular sieve?
Answer: Deliquescent dryer

461. Which of the following would cause the compressor to run continuously?
Answer: Low pressure switch jammed

462. In a window air conditioning unit which of the following is usually done by the owner?
Answer: Semi-annual cleaning or replacement of filters

463. Which of the following is not similarity between submerged culvert and a siphon?
Answer: Torricelli’s equation holds

464. Where does the final removal of water vapor in an absorption refrigeration system occur?
Answer: Rectifier

465. The constant spending of certain percentage of circulated water in a cooling tower in
order to prevent accumulation of dissolved mineral solids and other impurities in the
condenser water is called
Answer: Bleed-off

466. Which of the following refrigerant is added sometimes to other refrigerant to improve oil
circulation?
Answer: R – 170 (Ethane)

467. The suction pressure switch is operated by which of the following?


Answer: Pressure on a bellow

468. When the air is saturated the wet-bulb depression is:


Answer: Zero

469. Heating and dehumidification can be obtained simultaneously if air is passed through:
Answer: Either a solid absorbent surface or a liquid absorbent spray
470. In cooling cycle, the dry bulb temperature (db) of the air is lowered. When this happens
the relative humidity
Answer: Increase

471. What will happen when the pressure at any point inside a centrifugal pump goes below
the vapor pressure corresponding to the temperature of the liquid?
Answer: Cavitation

472. The ratio between the actual power and the apparent power in any circuit known as the
____ of that circuit
Answer: Power factor

473. Refrigerant leakage from the compressor crankcase is prevented by


Answer: Using shaft seals

474. The matching of scale and full-scale results for a fluid dynamic phenomena involving
compressible fluids requires quality of:
Answer: Mach number

475. Ammonia leaks in the condenser can be detected by:


Answer: Applying litmus paper to the circulating water discharge

476. A swinging support constructed as part of the vessel and that supports the manway cover
when it is unbolted and moved aside.
Answer: Davit

477. Which of the following type valves are not found on a Freon – 12 system?
Answer: Duplex

478. What is the main function of a receiver?


Answer: Store the refrigerant

479. The dividing point between the high pressure and low pressure sides of the refrigeration
cycle occurs at the
Answer: Expansion valve

480. If the solute particles of a solid suspended in a liquid are larger than molecules the
mixture is known as?
Answer: Suspension

481. A mechanism that removes moisture


Answer: Dehumidifiers
482. Is a water soluble organic compound prepared from ammonia. It has significant
biological and industrial usefulness
Answer: Urea or carbamide urea

483. Which of the following fans in air conditioning systems which can be classified as
centrifugal flow
Answer: Propeller fan

484. What is the use of the low – water cutout switch?


Answer: Stops the compressor when there is insufficient cooling water

485. In parallel pipe system originating and terminating in common junctions,


Answer: Pressure drops through each branch are equal

486. When the dry bulb and the wet bulb temperature are identical, the air is said to be:
Answer: Saturates

487. The temperature in which water vapour in the gas begins to condensate in a constant
pressure process.
Answer: Dew point

488. Combination gas/fuel oil burners permit the operator to switch from one fuel to ____
Answer: All of the above
(For economy, Because of a shortage of fuel being used, Because of a failure in the fuel
system being used)

489. Most nuclear particles can react with atoms in several different ways including
Answer: Absorption and scattering

490. What is the effect of superheating the refrigerant?


Answer: It increase the coefficient of performance

491. Boiling temperature of Freon 12 is


Answer: -29.8 C

492. Which of the following would cause low head pressure?


Answer: Too much cooling water and/ or insufficient refrigerant gas

493. At what temperature will water normally turns to steam?


Answer: 212 F

494. What will happen if the expansion valve is opened too wide?
Answer: Liquid will flood back to the compressor
495. All of the following are words used to describe neutron kinetic energy levels except:
Answer: Supersonic

496. If the thermal expansion valve becomes inoperative, the ice boxes will have to be
controlled by the ____.
Answer: Manual expansion valve

497. If the superheat on the suction side of the compressor is increased, what will happen to
the tonnage capacity of the unit?
Answer: Decreases

498. The solenoid valve can be typed as a ____.


Answer: Magnetic soap valve

499. The ratio of fugacity of actual conditions to the fugacity at some reference state is known
as:
Answer: Activity

500. The process that takes place in the evaporator called


Answer: Absorption of the latent heat of vaporization

501. When charging freon system, all the valves should be


Answer: King (liquid) valve

502. If the pressure exerted on a liquid is higher than the saturation corresponding to it
temperature, the liquid is a:
Answer: Sub cooled liquid

503. The most likely cause of high superheat would be:


Answer: Expansion valve closed too much

504. A good refrigerant should have a


Answer: High latent heat

505. What is the device used to protect the compressor from overloading due to high head
pressure
Answer: Overload relay

506. Which of the following it to be checked as part of weekly maintenance schedule of a


console air conditioners?
Answer: Cooling towers
507. Which of the following would not be cause for a refrigerating system to short cycle on hp
cutout?
Answer: Discharge valve leaking

508. A ____ boiler has heat and gases of combustion that pass through tubes surrounded by
water
Answer: Fire tube

509. A type of polymer used for clear bottles.


Answer: Polyvinyl Chloride (PVC)

510. In an ammonia system, the oil gauge must be kept:


Answer: Closed except when checking level oil

511. If a compartment requires the removal of 36,000 BTU per hour, how much is necessary
compressor capacity?
Answer: 3 tons

512. The oil level in the compressor should be checked:


Answer: After a long period of operation

513. Which of the following industries have the highest consumption of water for processing?
Answer: Paper mill

514. What is the lowest temperature to which water could possibly be cooled in a cooling
tower?
Answer: The temperature of adiabatic compression

515. The method of cooling which primarily used where ambient air temperatures are high and
relative humidity is used:
Answer: Swamp cooling
A pump is delivering 100L/sec of water at a total head of 50 meters. Find the water power.
a. 49kW

b. 19kW

c. 29kW

d. 39kW

A rectangular duct has dimensions 0.3 m by 1 m. Determine the equivalent diameter of the duct.

Calculate the specific volume of an air-vapor mixture in cubic meters per kilogram of dry air when
the following conditions were given: t=27ºC, w=0.014 kg/kg, P=82 kPa

Eleven thousand three hundred kilograms per hour of water enters a cooling tower at
45ºC. Atmospheric air at 16ºC and 55 percent relative humidity enters the tower at the rate
of 10,200 m /hr and leaves at 32ºC and saturated. Determine the mass of water
evaporated per hour during the cooling process ?
a. 2,219.53
kg/hr

b. 2,192.53
kg/hr

c. 1,912.53
kg/hr

d. 1,292.53
kg/hr

Water substance at 70 bar and 65°C enters a boiler tube of constant inside diameter of 25
mm. The water leaves the boiler tube at 50 bar and 700 K at velocity of 100 m/s. Calculate

the inlet volume flow in li./sec. At 70 bar & 65°C, = 0.001017 m /kg, at 50 bar & 700 K,

= 0.06081 m /kg.
0.82

The density of air at 35C and 101 kPa is 1.05 kg/m3. The humidity ratio is:
0.036

In an isentropic process P = 200 psi, P = 300 psi and T = 700°R. Find T using k = 1.4.
a. 576°R

b. 590°R

c. 680°R

d. 786°R

A steam condenser receives 10 kg per second of steam with an enthalpy of 2,570 kJ/kg.
Steam condenses into liquid and leaves with an enthalpy of 160 kJ/kg. Cooling water passes
through the condenser with temperature increases from 130°C to 24°C. Calculate the
cooling water flowrate in kg/s.
a. 518

b. 523

c. 528

d. 533

Ammonia weighing 22 kg is confined inside a cylinder equipped with a piston has an initial
pressure of 413 kPa at 38°C. If 2900 kJ of heat is added to the ammonia until its pressure
and temperature are 413 kPa and 100°C, respectively. What is the amount of work done by
the fluid in kJ ?
a. 304

b. 502

c. 420

d. 667

A 1.5 kg of wet steam at a pressure of 5 bar ( h = 640 kJ/kg, h = 2109 kJ/kg ) dryness
0.95 is blown into 70 liters of water of 12°C ( h = 50.4 kJ/kg ). Find the final enthalpy of the
mixture.
a. 104.80 kJ/kg

b. 74.80 kJ/kg

c. 94.80 kJ/kg

d. 84.80 kJ/kg

Helium ( R = 0.4968 Btu/lb °R ) is compressed isothermally from 14.7 psia and 68°F. The
compression ratio is 4. Calculate the work done by the gas.
a. - 364 Btu/lb

b. - 46.7 Btu/lb

c. - 145 Btu/lb

d. - 187 Btu/lb

2
A wall with an area of 10 m is made of a 2 cm thickness of white pine ( k= 0.113 W

/ m C ) followed by 10 cm of brick ( k= 0.649 W / m C ). The pine is on the side where


the temperature is 30 C while the outside temperature is 10 C. Assuming
equilibrium conditions exists, what is the temperature at the interface between the
two metals.
a. 19.31C

b. 15.65C

c. 18.21C

d. 17.64C

A Carnot engine receives 130 Btu of heat from a hot reservoir at 700°F and rejects 49 Btu of
heat. Calculate the temperature of the cold reservoir.
a. - 20.8 °F

b. - 24.2 °F

c. - 22.7 °F

d. - 21.9 °F

A heat engine is operated between temperature limits of 1370°C and 260°C. Engine is
supplied with 14,142 kJ/kwh. Find the Carnot cycle efficiency in percent.
a. 67.56

b. 65.05

c. 69.32

d. 70.10

Nitrogen ( k = 1.4 ) is expanded isentropically. Its temperature changes from 620°F to 60°F.

Find the pressure ratio (P /P ).


a. 12.91

b. 0.08

c. 35.47

d. 26.2

It is desired to deliver 5 m³/s at a head of 640m in a single stage pump having specific
speed not to exceed 40. If the speed is not to exceed 1352 rpm, how many stages are
required.
a. 5

b. 2

c. 4
d. 3

A 95 TOR system has a compressor power of 90 Hp. Find the COP.


a. 4.87

b. 2.77

c. 3.85

d. 1.99

A refrigeration system operates on the reversed carnot cycle. The minimum and maximum
temperature are -25C and 72C. If the heat rejected at the condenser is 6000kJ/min, find the
power input required.
a. 1866.83
kJ/min

b. 1888.38
kJ/min

c. 1686.83
kJ/min

d. 1666.38
kJ/min

What is the resulting pressure when one pound of air at 15 psia and 200°F is heated at
constant volume to 800°F ?
a. 28.6
psia

b. 15 psia

c. 52.1
psia

d. 36.4
psia

A single acting, twin cylinder, ammonia compressor with bore equal to stroke is driven by
an engine at 250 rpm. The machine is installed in a chilling plant to produce 700 kW of
refrigeration at -18 C evaporating temperature. At this temperature, the cooling effect per
kg mass is 1160 kJ. The specific volume of vapor entering the compressor is 0.592 m3 per
kilogram. Assume 85 % volumetric efficiency, determine the bore in mm.
a. 400
mm

b. 350
mm

c. 250
mm

d. 300
mm

In refrigeration cycle, the heat rejected in the condenser is 150 kW and the coefficient of
performance is 3. Find the refrigerating effect.
112.5

A cylinder has a bore diameter of 15 cm, stroke of 45 cm, and volumetric compression =
3
2000 cm , find the volumetric efficiency of the engine.

a. 22.5 %

b. 20.5 %

c. 25.2 %

d. 30.2 %
1
𝐴𝐿 − 2000
−( )𝑥100%
𝐴𝐿
V=B^2(pi/4)*Strok
(1-((V-vc)/V)*100

Determine the approximate load on a cooling tower if the entering and leaving
temperatures are 96 F and 88 F, respectively and the flow rate of the water over the tower
is 30 gpm.
a 3000
. Btu/min

b 2000
. Btu/min

c 3500
. Btu/min

d 2500
. Btu/min

Water ( Specific heat, C = 4.2 kJ/kg-K ) is being heated by 1500-W heater. What is the rate
of change in temperature of 1 kg of water.
0.357

The evaporator and condenser pressures are 153 kPa and 10 atm respectively. The intercooler
pressure is approximately

𝑃𝑖 =/𝑠𝑞𝑟𝑡(𝑃1 𝑃2 )

A steam generator with economizer and air heater has an overall draft loss of 21.78 cm
water. If the stack gases are at 177C and if the atmosphere is at 101.3 kPa and 26C, what
theoretical stack height in meters is needed when no draft fans are used? Assume that the
gas constant for the flue gases is the same as that for air.
a. 565

b. 550

c. 535

d. 545

How much work is necessary to compress air in the air in an insulated cylinder from 0.20 m

to 0.01 m . Use T = 20°C and P = 100 kPa.


a. 110.1 kJ

b. 121.4 kJ
c. 113.4 kJ

d. 115.7 kJ

Water enters the condenser at 30°C and leaves at 60°C. What is the temperature difference
in °F ?
a. 16.67

b. 48.67

c. 54

d. 22

A certain gas at 101.325 kPa and 16°C whose volume is 2.83 m are compressed into a storage vessel
of 0.31 m capacity. Before admission, the storage vessel contained the gas at a pressure and
temperature of 137.8 kPa and 24°C. After admission, the pressure has increased to 1171.8 kPa.
What should be the final temperature of the gas in the vessel in Kelvin ?

a. 298.0

b. 420

c. 180

d. 319.0

What is the power of a steam jet 15 mm in diameter moving at 750 m/s? Steam condition,
1.4 kg/cm , dry and saturated ( = 1.256 m /kg ).
a. 29.67 kW

b. 39.67 kW

c. 49.67 kW
d. 19.67 kW

A refrigeration system operates on an ideal vapoer compression using R-12 with an


evaporator temperature of -30 C and a condenser exit temperature of 49.3 C and requires
74.6 kW motor to drive the compressor. What is the capacity of the refrigerator in tons of
refrigeration? Enthalpy at condenser entrance = 382 kJ/kg, exit = 248.15 kJ/kg, at evaporator
entrance = 248.15 kJ/kg, exit = 338.14 kJ/kg.
a. 40.1

b. 49.1

c. 43.1

d. 46.1

A centrifugal pump has a constant speed of 1500 rpm and has a head of 75m. What is the
effect on the head of the pump if the impeller diameter is reduced from 280 mm to 230
mm
a. 72.6
mm

b. 25.6
mm

c. 50.6
mm

d. 65.6
mm

Water enters the heater at 25°C and leaves at 80°C. What is the temperature change in F° ?
a. 99
b. 55

c. 65

d. 11

The maximum thermal efficiency possible for a power cycle operating between 1200°F is:
58.73 %

A refrigeration system operates on an ideal vapoer compression using R-12 with an


evaporator temperature of -30 C and a condenser exit temperature of 49.3 C and requires
74.6 kW motor to drive the compressor. What is the capacity of the refrigerator in tons of
refrigeration? Enthalpy at condenser entrance = 382 kJ/kg, exit = 248.15 kJ/kg, at evaporator
entrance = 248.15 kJ/kg, exit = 338.14 kJ/kg.
43.1
An electric heater is to heat 11 kg of oil per min. from 4.5°C to 65.5°C. Specific heat of the
oil is 2.1 J/gm-°C. How many watts should this heater consume ?
23442W

A pump discharge 50 tonsof water per hour to a height of 8 m, the overall efficiency of the
pumping system being 69%. Calculate the output and input power.
1.09 kW & 1.58
kW

Calculate the air power of fan that delivers 1200 m3/min air through a 1 m by 1.5 m outlet.
Static pressure of 120 mm H2O and density of air is 1.18.
a. 30.64 kW

b. 20.47 kW

c. 25.64 kW

d. 23.47 kW
A volume of 450 cm of air is measured at a pressure of 740 mm Hg absolute and a
temperature of 20°C. What is the volume in cm at 760 mm Hg absolute and 0°C ?
408.25

A creamery must cool 20,000 liters of milk received each day from initial temperature of 29
C to a final temperature of 2 C in 5 hours. If refrigeration losses amount to 10 percent of
the cooling load, what must be the capacity of the refrigeration machine? Use Cp = 3.9
kJ/kg-K and SG = 1.05
a. 39.5 TOR

b. 38.5 TOR

c. 37.5 TOR

d. 36.5 TOR

Calculate the tons of refrigeration required to freeze in 4 hrs. 3mm of 10C water on a
skating rink 61m x 30m if the ground temp is 10C and the air temperature above the rink is
16C. The heat transfer coefficient from the air to 0C ice is 6.8 W/m2-C, and the heat transfer
coefficient from the ground to -6C ice is 0.3402 W/m2-C
a. 201.96
b. 151.68

c. 101.69

d. 140.62

A pressure gage registers 50 psig in region where the barometer is 14.25 psia. Find the
absolute pressure in psia.
a. 151.32
5

b. 443

c. 64.25

d. 35.75

In a CO2 refrigerating plant, the specific enthalpy of the refrigerant as it leaves the
condenser is 135 kJ/kg and as it leaves the evaporator it is 320 kJ/kg. If the mass flow of the
refrigerant is 5 kg/min, calcylate the refrigerating effect per hour

d.55, 500 kJ/hr

How many TOR are required to produce 10MT of ice per day at -10C from raw water at 22C
if losses are 15% of the load.
17
A vessel of 0.058 m capacity is well insulated and is divided equally by rigid conducting
diaphragm. Initially both halves contain air at pressure of 137.8 kPa and 413.4 kPa and a
temperature of 27°C and 177°C respectively. What is the increase of entropy of the system
in kJ/°C ?
0.00173
In a two-stage air compressor, air at 100 kPa and 22C discharges to 750 kPa. Find the value
of n if the intercooler intake is 105 C.
a. 1.67

b. 1.4

c. 1.26

d. 1.33

The refrigerating capacity of R-12 system is 22 kW. Compressor power is 7.8HP. Determine
the COP of the refrigeration system.
a. 3.78

b. 2.82

c. 1.85

d. 2.22

A condenser vacuum gauge reads 715 mm Hg when the barometer stands at 757 mm Hg.
What is the absolute pressure in the condenser in kPa.
5.60 kpa

A 0.30 x 0.40 m branch duct leaves a 0.30 x 0.60 main duct at an angle of 60 degress. The
air temp is 20C. The dimension of the main duct remain constant following branch. The
flow rate upstream is 2.7 m3/s. What is the pressure downstream in the main duct. Note: at
20C, density is 1.2041 kg/m3.

346 Pa

Steam enters the superheat of a boiler at a pressure of 20 bar ( h = 909 kJ/kg, h = 1890

kJ/kg, = 0.09957 m /kg ) and dryness 0.98 and leaves at the same pressure at a
temperature of 350°C ( h = 3138 kJ/kg, = 0.01386 m /kg ). Find the percentage increase in
volume due to drying and superheating.
42.04

What is the temperature in °C of 2 liters of water at 30°C after 500 Calories of heat have
been added ?

30.2 C

A vessel of 0.058 m capacity is well insulated and is divided equally by rigid conducting
diaphragm. Initially both halves contain air at pressure of 137.8 kPa and 413.4 kPa and a
temperature of 27°C and 177°C respectively. What is the increase of entropy of the system
in kJ/°C ?

0.00173

Determine the c
il face area required to maintain a face velocity of 400ft/min if the air flow rate over the coil
is 2100 ft3/min
5.25

Three pounds mass of air are contained at 25 psia and 100°F. Given that R = 53.35 ft-lb

/lb . What is the volume of the container ?

24.9 ft3
A room being air conditioned is held at 25C DB and 50% RH. A flow rate of 5 m3/s of supply
air at 15 C DB and 80% RH is being delivered to the room to maintain that steady condition
at 100 kPa. What is the sensible heat absorbed from the room air in kW?
60.8
Steam flows into a turbine at the rate of 10 kg/s and 10 kW of heat are lost from the
turbine. Ignoring elevation and kinetic energy effects, calculate the power output from the
turbine.

Note: h = 2739.0 kJ/kg and h = 2300.5 kJ/kg


4375kW

For a certain ideal gas, R = 0.277 kJ/kg-K and k = 1.384. What are the values of C and C ?
a. 0.9984 kJ/kg-K, 0.6124 kJ/kg-K

b. 0.7124 kJ/kg-K, 0.8124 kJ/kg-K

c. 1 kJ/kg-K, 0.8124 kJ/kg-K

d. 0.9984 kJ/kg-K, 0.7213 kJ/kg-K

After series of state changes, the pressure and volume of 2.286 kg of Nitrogen are each
doubled. What is S ?
2.807
Copra enter dryer containing 40% water and 60% solid and leaves with 5% water and 95% solid. Find
the weight of water removed per pound of final product

a. 1.3753
lb/lb

b. 0.583 lb/lb

c. 0.475 lb/lb

d. 1.015 lb/lb
A sample of air has dry bulb temperature of 30 C and a wet bulb temperature of 25 C. The
barometric pressure is 101.325 kPa. Calculate the enthalpy of the air if its adiabatically
saturated. At 25 deg C, Pv = 3.17 kPa, hg = 2547.2 kJ/kg.

75.94

A 500 kW refrigeration system is used to produce cooled water from 24C to 3C calculate
the mass flow rate of water in kg/s
5.69

A 4 m x 4 m x 4 m room has a relative humidity ratio of 80%. The pressure in the room is

120 kPa and temperature is 35ºC (P = 5.628). What is the mass of vapor in the room. Use

R = 0.4615 kN-m/kg-K.
2.03

A Carnot engine requires 35 kJ/s from the hot source. The engine produces 15 kW of power
and the temperature of the sink is 26°C. What is the temperature of the hot source in °C ?

250.18
A dryer is to deliver 1000 kg/hour of palay with a final mositure content of 10%. The initial
moisture content in the feed is 15% at atmospheric condition with 32ºC dry bulb and 21ºC
wet bulb. The dryer is maintained at 45ºC while the relative humidity of the hot humid air
from the dryer is 80%. If the steam pressure supplied to the heater is 2 MPa, determine the
air supplied to the dryer in m /hr.
a. 1233.25 m
/hr

b. 1223.25 m
/hr

c. 1523.13 m
/hr

d. 1332.25 m
/hr (PACHECK
NA LANG)
One kilogram of water ( C = 4.2 kJ/kg-K ) is heated by 300 BTU of energy. What is the
change in temperature in K ?

75.36 K

Assuming compression is according to the law PV = constant. Calculate the initial volume of
gas at a pressure of 2 bar which will occupy a volume of 6 m when it is compressed to a
pressure of 42 bar.
126
A refrigerator has a COP of 1.6. How much work in kJ must be supplied to this refrigerator
for it to reject 1000 kJ of heat?
a. 485

b. 585

c. 285

d. 385

Water enters the condenser at 30°C and leaves at 89°C. What is the temperature difference in °F ?

Ans: secret

Calculate the enthalpy of air at 30C if the specific humidity of air is 0.015kg/kg and the hg =
2556.3 kJ/kg
68.3

A vessel with a volume of 1 m contains liquid water and water vapor in equilibrium at 600
kPa. The liquid water has a mass of 1 kg. Calculate the mass of the water vapor.
Properties of liquid water and water vapor at 600 kPa : V f = 0.001101 m /kg; V g = 0.3157 m /kg

3.16 kg
A cylinder and piston arrangement contains saturated water vapor at 110°C. The vapor is
compressed in a reversible adiabatic process until the pressure is 1.6 MPa. Determine the

work done by the system per kg of water. At 110°C, S = 7.2387 kJ/kg-K, U = 2518.1 kJ/kg

and at 1.6 MPa, S = 7.2374 kJ/kg-K, U = 2950.1 kJ/kg, T = 400°C.


a. - 700 kJ/kg

b. - 432 kJ/kg

c. - 632 kJ/kg

d. - 500 kJ/kg

I
A closed vessel contains air at a pressure of 140 kPag and temperature of 20°C. Find the
final gauge pressure if the air is heated at constant volume to 40°C. Take the atmospheric
pressure as 759 mm Hg.
a. 126.46

b. 146.46

c. 156.46

d. 136.46
Calculate the specific volume of an air-vapor mixture in cubic meters per kilogram of dry air when
the following conditions were given: t=28ºC, w=0.013 kg/kg, P=94 kPa

0.9381

The heat pump has a performance factor of 8.0 If the heat pump is used as a refrigerator,
what is the required horsepower per ton of refrigeration?
a. 0.933

b. 0.673

c. 0.873

d. 0.773
If the latent and sensible heat loads are 31 kW and 58 kW respectively, what is the sensible heat
ratio?
SH/(SH+LH)

Steam at a pressure of 9 bar ( h = 743 kJ/kg, h = 2031 kJ/kg ) is generated in an exhaust


gas boiler from feedwater at 80°C ( h = 334.9 kJ/kg ). If the dryness fraction of the steam is
0.96, determine the heat transfer per kilogram of steam.
a. 5357.86

b. 3357.86

c. 1357.86

d. 2357.86

Oil flows full bore at a velocity of 2 m/s through a nest of 16 tubes in a single pass cooler.
The internal diameter of the tubes is 30 mm and the density of the oil is 0.85 g/li. Find the
volume flow in liters per second.
a. 23

b. 20

c. 21

d. 22

What horsepower is supplied to air moving at 20 ft per min through a 1 by 3 ft duct under a
pressure of 5 in water gage?
Air flowing at a rate of 2.5 kg/s is heated in a heat exchanger from -10C to 30C. What is the
rate of heat transfer?
a. 100 KW

b. 71.8 KW

c. 418.7 KW

d. 28.7 KW
Calculate the specific volume of an air-vapor mixture in cubic meters per kilogram of dry air when
the following conditions were given: t=30ºC, w=0.016 kg/kg, P=85 kPa

Find the heat transfer required to convert 7 kg of water at a pressure of 20 bar and a
temperature of 21°C into steam dryness of fraction 92 at the same pressure.

Properties of steam:

P = 20 bar

hf = 909 kJ/kg
hfg = 1890 kJ/kg

Note: for water at 21°C, h = 88 kJ/kg

Uy pri peram ako next term

A mixture of dry air and water vapor is at a temperature of 21ºC under a pressure of 101 kPa. The
dew point temperature is 15ºC. Calculate the relative humidity.
68.56%
1. Find the work possess for a helium gas at 20°C.
A. 609 kJ/kg B. 168 kJ/kg C. 229 kJ/kg D. 339 kJ/kg
Solution:
( )

( )

2. Two kilogram of gas is confined in a 1 m3 tank at 200 kPa and 88°C. What type of gas is in the
tank?
A. Helium B. Ethane C. Methane D. Ethene
Solution:

( ) ( )( )

( )

3. Find the enthalpy of Helium if its internal energy is 200 kJ/kg.


A. 144 kJ/kg B. 223.42 kJ/kg C. 333.42 kJ/kg D. 168 kJ/kg
Solution:

( )

4. Compute the mass of a 2 m3 propane at 280 kPa and 40°C.


A. 6.47 kg B. 5.1 kg C. 10.20 kg D. 9.47 kg
Solution:
( ) ( )

( ) ( )( )

5. Compute the air flow in ft3/min of mechanical ventilation required to exhaust an


accumulation of refrigerant due to leaks of the system capable of revolving air from the
machinery for a mass of 4 lbs refrigerant.
A. 200 B. 210 C. 220 D. 230
Solution:
1

6. Compute the free aperture cross section in m3 for the ventilation of a machinery room if the
mass of refrigerant is 9 kg.
A. 0.314 B. 0.414 C. 0.514 D. 0.614
Solution:

7. A 29.53” x 39.37” pressure vessel contains ammonia with f = 0.041. Compute the minimum
required discharge capacity of the relief device in kg/hr.
A. 106.71 kg/hr B. 108.71 kg/hr C. 110.71 kg/hr D. 112.71 kg/hr
Solution:
( )
( )( ) ( )

8. Compute the maximum length of the discharge pipe installed on an outlet of a pressure-relief
device in feet for internal pipe diameter of 0.5 inch and rated discharge capacity is 8 lb/min of
air. The rated pressure of relief valve is 10 psig.
A. 0.286 ft B. 0.386 ft C. 0.486 ft D. 0.586 ft
Solution:
( )
( ) ( )
( )

9. A thermal power plant has the heat rate of 11,363 Btu/kW-hr. Find the thermal efficiency of
the plant.
A. 28% B. 30% C. 34% D. 40%
Solution:

10. What is the hydraulic gradient of a 1 mile, 17 inches inside diameter pipe when 3300
gal/min of water flow with f = 0.03?
A. 0.00714 B. 0.00614 C. 0.00234 D. 0.00187
Solution:

( )

2
( )( )
( )( )

11. Find the loss of head in the pipe entrance if speed of flow is 10 m/s.
A. 5.10 m B. 10.2 m C. 17.4 m D. 2.55 m
Solution:
( ) [ ( )
]

12. Wet material, containing 220% moisture (dry basis), is to be dried at a rate of 1.5 kg/s in a
continuous dryer to give a product containing 10% (dry basis). Find the moisture removed in
kg/hr.
A. 3543.75 kg/hr B. 3513.75 kg/hr C. 3563.75 kg/hr D. 3593.75 kg/hr
Solution:

( ) ( )
( )

13. Copra enters a dryer containing 70% moisture and leaves 7% moisture. Find the moisture
removed on each pound of solid in the final product.
A. 6.258 lbs B. 1.258 lbs C. 4.258 lbs D. 2.258 lbs
Solution:

14. A 1 m x 1.5 m cylindrical tank is full of oil with SG = 0.92. Find the force acting at the bottom
of the tank in dynes.
A. 106.33 x 103 dynes C. 106.33 x 105 dynes
B. 106.33 x 104 dynes D. 106.33 x 106 dynes
Solution:

( )

3
15. Find the pressure at the 100 fathom depth of water in kPa.
A. 1,793.96 kPa B. 1,893.96 kPa C. 1,993.96 kPa D. 1,693.96 kPa
Solution:

( )

16. Find the depth in furlong of the ocean (SG = 1.03) if the pressure at the sea bed is 2,032.56
kPa.
A. 1 B. 2 C. 3 D. 4
Solution:

( )( )( )

17. Find the mass of 10 quartz of water.


A. 10.46 kg B. 9.46 kg C. 11.46 kg D. 8.46 kg
Solution:
-
( )( )( )
-
( )

18. Find the mass of carbon dioxide having a pressure of 20 psia at 200°F with 10 ft 3 volume.
A. 1.04 lbs B. 1.14 lbs C. 1.24 lbs D. 1.34 lbs
Solution:

( )( ) ( )( )

19. Find the heat needed to raise the temperature of water from 30°C to 100°C with 60%
quality. Consider an atmospheric pressure of 101.325 kPa. Use the approximate enthalpy
formula of liquid.
A. 293.09 kJ/kg B. 1,772.90 kJ/kg C. 1,547.90 kJ/kg D. 1,647.29 kJ/kg
Solution:

( )

4
( )
( ) ( )

20. Find the enthalpy of water at 212°F and 14.7 psi if the dryness factor is 30%. Use the
approximate enthalpy formula of liquid.
A. 461 Btu/lb B. 471 Btu/lb C. 481 Btu/lb D. 491 Btu/lb
Solution:

( )

21. An air compressor consumed 1200 kW-hr per day of energy. The electric motor driving the
compressor has an efficiency of 80%. If indicated power of compressor is 34 kW, find the
mechanical efficiency of the compressor.
A. 117.65% B. 75% C. 85% D. 90%
Solution:
( )
( )

22. A refrigeration system consumed 28,800 kW-hr per month of energy. There is 20% of
energy lost due to the cooling system of the compressor and the motor efficiency is 90%. If the
COP of the system is 6, find the mechanical efficiency of the compressor.
A. 43.15 TR B. 46.15 TR C. 49.15 TR D. 41.15 TR
Solution:
( )( )
( )
̇ ( )

̇
̇ ( )

23. A 23 ton refrigeration system has a heat rejected of 100 kW. Find the energy efficiency ratio
of the system.
A. 13.42 B. 14.42 C. 15.42 D. 16.42
Solution:
5
̇ ̇
( ) ̇
̇
( )
̇
( )

24. A 200 mm x 250 mm, 8-cylinder, 4-stroke diesel engine has a brake power of 150 kW. The
mechanical efficiency is 80%. If two of the cylinders were accidentally cut off, wha t will be the
new friction power?
A. 31.50 kW B. 33.50 kW C. 35.50 kW D. 37.50 kW
Solution:

25. If the energy efficiency ratio of the refrigeration system is 12.6, what is the COP of the
system?
A. 3.69 B. 4.23 C. 5.92 D. 6.83
Solution:

26. An air compressor has a power of 40 kW at 4% clearance. If the clearance will increase to
7%, what is the new power?
A. 70 kW B. 40 kW C. 53 kW D. 60 kW
Solution:

27. What is the approximate value of temperature of water having enthalpy of 208 Btu/lb?
A. 138.67°C B. 115.55°C C. 258.67°C D. 68.67°C
Solution:

6
28. Convert 750°R to K.
A. 390.33 K B. 395.33 K C. 410.33 K D. 416.33 K
Solution:

29. An Otto cycle has a compression ratio of 8. Find the pressure during compression.
A. 18.38 B. 16.38 C. 14.38 D. 12.28
Solution:

( ) ( )

30. A diesel cycle has a cut off ratio of 2.5 and expansion ratio of 4. Find the clearance of the
cycle.
A. 9.11% B. 5.55% C. 11.11% D. 15.15%
Solution:
( )

31. A dual cycle has an initial temperature of 30°C. The compression ratio is 6 and the heat
addition at constant volume process is 600 kJ/kg. If the cut-off ratio is 2.5, find the maximum
temperature of the cycle.
A. 3638.50°C B. 3365.50°C C. 3565.50°C D. 3965.50°C
Solution:
( ) ( ) ( )( )

( )( )( ) ( )

( ) ( )

32. A 3-stage air compressor compresses air from 100 kPa to 1000 kPa. Find the intercooler
pressure between the 1st and 2nd stage.
A. 505.44 kPa B. 108.44 kPa C. 316.23 kPa D. 215.44 kPa
Solution:
7
( ) ( )

33. A 10-stage air compressor compresses air from 100 kPa to 800 kPa. Find the intercooler
pressure between the 1st and 2nd stage.
A. 282.84 kPa B. 113.21 kPa C. 123.11 kPa D. 333.51 kPa
Solution:
√ √

34. A 3-stage air compressor compresses air from 100 kPa to 700 kPa. Find the intercooler
pressure between the 2nd and 3rd stage.
A. 365.88 kPa B. 375.88 kPa C. 385.88 kPa D. 395.88 kPa
Solution:
( ) ( )

35. Carnot cycle A, B and C are connected in series so that the heat rejected from A will be the
heat added to B. and heat rejected from B will be added to C. Each cycle operates between
30°C and 400°C. If heat added to A is 1000 kW, find the work output of C.
A. 111.44 kW B. 549.78 kW C. 247.53 kW D. 141.89 kW
Solution:

̇ ̇ ̇
̇ ̇
̇

̇ ̇
̇

̇ ̇
̇
̇
̇

36. Air compressed adiabatically from 30°C to 100°C. If the mass of air being compressed is 5
kg, find the change of entropy.
A. 1.039 kJ/K B. 0.746 kJ/K C. 0 D. 1.245 kJ/K
Solution:

8
37. Two kilogram of air in a rigid tank changes its temperature from 32°C to 150°C. Find the
work done during the process.
A. 236 B. 170 C. 195 D. 0
Solution:

38. Determine the atmospheric pressure at a location where barometric reading is 740 mm Hg
and gravitational acceleration is g = 9.7 m/s 2. Assume the temperature of mercury to be 10°C,
at which the density is 13,570 kg/m3.
A. 99.45 kPa B. 97.41 kPa C. 95.44 kPa D. 98.66 kPa
Solution:
( )( )

39. The barometer of a mountain hiker reads 930 mbars at the beginning of a hiking trip and
780 mbars at the end. Neglecting the effect of altitude on local gravitational acceleration,
determine the vertical distance climbed. Assume g = 9.7 m/s 2.
A. 1274.21 m B. 1289.00 m C. 1267.34 m D. 1583.34 m
Solution:

( )( ) ( )( )

40. The lower half of a 10 m high cylindrical container is filled with water and the upper half
with oil that has SG = 0.85. Determine the pressure difference between the top and bottom if
the cylinder.
A. 90.74 kPa B. 92.74 kPa C. 83.38 kPa D. 98.10 kPa
Solution:
( )( ) ( )

41. An ideal gas at 0.80 atmospheres and 87°C occupies 0.450 liter. How many moles are in the
sample? (R = 0.0821 liter-atm/mole-K).
A. 0.0002 mole B. 0.0378 mole C. 0.0122 mole D. 0.0091 mole
Solution:

( ) ( )( )

9
42. A certain gas at 101.325 kPa and 10°C whose volume is 2.83 m3 capacity. Before admission,
the storage vessel contained gas at a pressure and temperature of 137.8 kPa and 26°C; after
admission, the pressure increased to 1171.8 kPa. What should be the final temperature of the
gas in the vessel in Kelvin?
A. 298.0 B. 319.8 C. 180.0 D. 314.2
Solution:

( )
( )

( )
( )

( )

43. A perfect gas has a value of R = 58.8 ft-lb/lb-°R and k = 1.26. If 20 Btu are added to 10 lbs of
this gas at constant volume when initial temperature is 90°F, find the final temperature.
A. 97°F B. 104°F C. 154°F D. 185°F
Solution:

( ) -
( )( )

44. Ammonia weighing 22 kg is confirmed inside a cylinder equipped with a piston has an initial
pressure of 413 kPa at 38°C. If 3200 kJ of heat is added to the ammonia until its final pressure
and temperature is 413 kPa and 100°C, respectively. What is the amount of work done by the
fluid in kJ?
A. 667 B. 304 C. 420 D. 502
Solution:

( )( )

10
( )( )

( )

45. A tank contains 90 ft3 of air at a pressure of 350 psig; if the air is cooled until its pressure
and temperature decreases at 200 psig and 70°F respectively, what is the decrease in internal
energy?
A. 6232.09 Btu B. -5552 Btu C. 5552 Btu D. -6232.09 Btu
Solution:

( )( )
( )

( )
( )( ) - -

46. A large mining company was provided with a 3 m3 of compressed air tank. Air pressure in
the tank drops from 700 kPa to 150 kPa while the temperature remains constant at 28°C. What
percentage has the mass of air in the tank been reduced?
A. 74.09 B. 72.45 C. 76.56 D. 78.57
Solution:

47. A 4 m3/hr pump delivers water to a pressure tank. At the start, the gage read 138 kPa until
it reads 276 kPa and then the pump was shut off. The volume of the tank is 180 liters. At 276
kPa, the water occupied 2/3 of the tank volume. Determine the volume of the water that can
be taken out until the gage reads 138 kPa.
A. 31.20 liters B. 34.59 liters C. 16.87 liters D. 29.50 liters
Solution:

( )

( ) ( )( )

( ) ( )

11
48. A refrigeration plant is rated at 15 tons capacity. How many pounds of air per hour will it
cool from 70°F to 90°F at constant temperature?
A. 50,000 lb/hr B. 37,500 lb/hr C. 52,000 lb/hr D. 45,000 lb/hr
Solution:
̇ ̇

( ) ̇( )( )

49. An air standard engine has a compression ratio of 18 and a cut-off ratio of 4. If the intake air
pressure and temperature are 100 kPa and 27°C, find the work in kJ per kg.
A. 2976 B. 2166 C. 1582 D. 2751
Solution:

( )

( ) ( )

( )

-
( )( )

( )

50. Determine the air-standard efficiency of an engine operating on the diesel cycle with a
clearance of 6% when the suction pressure is 99.7 kPa and the fuel is injected for 7% of the
stroke. Assume k = 1.4.
A. 62.11% B. 51.20% C. 73.58% D. 60.02%
Solution:

12
( )( ( )
)

51. Steam at 2 MPa and 250°C in a rigid cylinder is cooled until the quality is 30%. Find the heat
rejected from the cylinder.
@ 2 Mpa and 250°C: υ = 0.11144 m3/kg u = 2679.6 kJ/kg
3
@ 2 Mpa, (saturated): υf = 0.0011767 m /kg υg = 0.09963 m3/kg
uf = 906.44kJ/kg ug = 1693.8 kJ/kg
A. -423.23 kJ/kg B. -926.26 kJ/kg C. -1265.02 kJ/kg D. 1082.34 kJ/kg
Solution:

( )
-

52. At 1.3 MPa, a mixture of steam and water has an entropy of 3 kJ/kg-K, Find the enthalpy of
the mixture.
@ 1.3MPa: s f = 2.2515 kJ/kg-K s g = 6.4952 kJ/kg-K
hf = 814.93 kJ/kg hfg = 1972.7 kJ/kg
A. 1627.71 kJ/kg B. 1533.33 kJ/kg C. 1234.45 kJ/kg D. 1162.40 kJ/kg
Solution:

( )

( )

53. A mixture with 70% quality at 500 kPa is heated isothermally until its pressure is 300 kPa.
Find the heat added during the process.
@ 600 kPa: s f = 1.8607 kJ/kg-K s fg = 4.9606 kJ/kg-K
@ 300 kPa and 151.86°C: s = 7.0888 kJ/kg-K
A. 745.92 kJ/kg B. 535.16 kJ/kg C. 982.44 kJ/kg D. 765.34 kJ/kg
Solution:
( )

( )( )

13
54. A tank contains exactly one kilogram of water consisting of liquid and vapor in equilibrium
at 1 MPa. If the liquid contains one-third and the remaining is vapor of the volume of the tank,
what is the enthalpy of the contents of the tank?
@ 1MPa: υf = 0.0011273 m3/kg υfg = 0.19444 m3/kg
hf = 762.81 kJ/kg hfg = 2015.3 kJ/kg
A. 644.40 kJ/kg B. 774.40 kJ/kg C. 785.92 kJ/kg D. 435.29 kJ/kg
Solution:

( )

55. Water substance at 70 bar and 65°C enters a boiler tube of constant inside diameter of 25
mm. The water leaves the boiler tube at 50 bar and 700 K at velocity of 150 m/s. Calculate the
inlet velocity (m/s).
From steam tables:
@ 70 bar (7Mpa) and 65°C: υ1 = 0.001017 m3/kg
@ 50 bar (5Mpa) and 700 K (427°C): υ2 = 0.06081 m3/kg
A. 1.56 B. 2.51 C. 1.672 D. 3.230
Solution:
̇ ̇

( )

56. Water substance at 70 bar and 65°C enters a boiler tube of constant inside diameter of 35
mm. The water leaves the boiler tube at 50 bar and 700 K at velocity of 150 m/s. Calculate the
inlet volume flow (liters/s).
From steam tables:
@ 70 bar (7Mpa) and 65°C: υ1 = 0.001017 m3/kg
@ 50 bar (5Mpa) and 700 K (427°C): υ2 = 0.06081 m3/kg
A. 0.821 B. 1.261 C. 0.344 D. 1.609
Solution:
̇ ̇
14
( )

( ) ( ) -

57. Steam leaves an industrial boiler at 827.4 kPa and 171.6°C. A portion of the steam is passed
through a throttling calorimeter and is exhausted to the atmosphere when the calorimeter
pressure is 101.4 kPa. How much moisture is leaving the boiler container if the temperature of
the steam at the calorimeter is 115.6°C?
@ 827.4 kPa and 171.6°C: hf = 727.25 kJ/kg hfg = 2043.2 kJ/kg
@ 101.4 kPa and 115.6°C: h2 = 2707.6 kJ/kg
A. 3.78% B. 3.08% C. 4.56% D. 2.34%
Solution:

( )

58. A throttling calorimeter is connected to a desuperheated steam line supplying steam to the
auxiliary feed pump on a ship. The line pressure measures 2.5 MPa. The calorimeter pressure is
110 kPa at 150°C. Determine the entropy of the steam line.
@ 110 kPa and 150°C: h2 = 2775.6 kJ/kg
@ 2.5 MPa: hf = 962.11 kJ/kg hfg = 1841 kJ/kg
s f = 2.5547 kJ/kg-K s fg = 3.7028 kJ/kg-K
A. 6.8 kJ/kg-K B. 6.2 kJ/kg-K C. 6.6 kJ/kg-K D. 7.5 kJ/kg-K
Solution:

( )

( ) -

59. Atmospheric pressure boils at 212°F. At the vacuum pressure at 24 in Hg, the temperature is
142°F. Find the boiling temperature when the pressure is increased by 40 psia from
atmospheric.
15
A. 449.42°F B. 536.34°F C. 479.13°F D. 263.45°F
Solution:
- ( )

60. A certain coal has the following ultimate analysis:


C = 69% N2 = 5% H = 2.5% S =7%
Determine the amount of oxygen if the heating value of fuel is 26,961.45 kJ/kg.
A. 1.5% B. 2.5% C. 3.5% D. 4.5%
Solution:
( )
( ) ( ) ( )

61. A diesel engine consumed 945 liters of fuel per day at 35°C. If the fuel was purchased at
15.6°C and 30°API at P29.00/li. Determine the cost of fuel to operate the engine per day.
A. P5,677.50 B. P4,677.50 C. P48,088.90 D. P27,127.76
Solution:

[ ( )]

( )

62. A cylindrical tank 4 m long and 3 m diameter is used for oil storage. How many days can the
tank supply the engine having 27°API with fuel consumption of 60 kg/hr?
A. 17.53 B. 5.84 C. 12.84 D. 19.84
Solution:
( ) ( )

16
̇ ̇ ( )
̇

̇
( )

63. A logging firm in Isabella operates a Diesel Electric Plant to supply its electric energy
requirements. During a 2 hour period, the plant consumed 250 gallons of fuel at 80°F and
produced 2900 kW-hrs. Industrial fuel is used at 30°API and was purchased at P30/li at 60°F.
Determine the overall thermal efficiency of the plant.
A. 26.08% B. 34.23% C. 28.00% D. 18.46%
Solution:
( ) ( )

[ ( )]
( )

̇ ( )( ) ( )( )

-
( )

̇ ( )

64. The dry exhaust gas from the oil engine has the following gravimetric analysis:
CO2 = 21.6% O2 = 4.2% N = 74.2%
Specific heats at constant pressure for each component of the exhaust gas in kCal/kg-°C are:
CO2 = 0.203 O2 = 0.219 N = 0.248
Calculate the specific gravity if the molecular weight of air is 28.97 kg/kg-mol.
A. 0.981 B. 1.244 C. 1.055 D. 0.542
Solution:

17
∑ -
-

65. A bituminous coal has the following composition:


C = 71.5% H = 5.0% O = 7.0% N = 1.3% S = 3% Ash = 7.6%
W = 3.4%
Determine the theoretical weight of Nitrogen in lb/lb of coal.
A. 2.870 B. 7.526 C. 2.274 D. 6.233
Solution:
( ) ( )

( ) ( ) ( )

( )

( )

66. A gaseous fuel mixture has a molal analysis:


H2 = 14% CH4 = 3% CO = 27% O2 = 0.6% CO2 = 4.5% N2 = 50.9%
Determine the air-fuel ratio for complete combustion on molal basis.
A. 2.130 B. 3.230 C. 1.233 D. 1.130
Solution:

( )

67. A volumetric analysis of a gas mixture is a follows:


CO2: 12% O2: 4% N2: 80% CO: 4%
What percentage of CO 2 on a mass basis?
18
A. 17.55% B. 15.55% C. 12.73% D. 19.73%
Solution:

68. The following coal has the following ultimate analysis by weight:
C = 70.5% H2 = 4.5% O2 = 6.0% N2 = 1.0% S = 3.0% Ash = 11%
Moisture = 4%
A stocker fired boiler of 195,000 kg/hr steaming capacity uses this coal as fuel. Calculate volume
of air in m3/hr with air at 60°F and 14.7 air pressure if the boiler efficiency is 70% and FE = 1.10.
A. 234,019 m3/hr B. 215,830 m3/hr C. 213,830 m3/hr D. 264,830 m3/hr
Solution:
( ) ( )

( ) ( ) ( )

( ) ( ) ( ) ( )

( )
( ) ( ) ( )

̇ ( ) ̇ ( )
̇ ̇
( )
̇ ( )

̇ ( )
̇ ̇
̇ ( )( )
̇

19
69. 23.5 kg of steam per second at 5 MPa and 400°C is produced by a steam generator. The
feedwater enters the economizer at 145°C and leaves at 205°C. The steam leaves the boiler
drum with a quality of 98%. The unit consumes 3 kg of coal per second as received having value
of 25,102 kJ/kg. What would be the overall efficiency of the unit in percent?
Steam properties:
@ 5 MPa and 400°C: h = 3195.7 kJ/kg
@ 5 MPa: hf = 1154.23 kJ/kg hfg = 1640.1 kJ/kg
@ 205°C: hf = 875.04 kJ/kg
@ At 145°C: hf = 610.63 kJ/kg
A. 65.72 B. 80.67 C. 88.28 D. 78.82
Solution:
̇ ( ) ( )
̇ ( )

70. In a Rankine cycle steam enters the turbine at 2.5 MPa (enthalpies & entropies given) and
condenser of 50 kPa (properties given), what is the thermal efficiency of the cycle?
@ 2.5 MPa: hg = 2803.1 kJ/kg s g = 6.2575 kJ/kg-K
@ 50 kPa: s f = 1.0910 kJ/kg-K s fg = 6.5029 kJ/kg-K
hf = 340.49 kJ/kg hfg = 2305.4 kJ/kg
3
υf = 0.00103 m /kg
A. 25.55% B. 45.23% C. 34.23% D. 12.34%
Solution:

( )

( )

( ) ( )
( ) ( ) ( ) ( )

71. A thermal power plant generates 5 MW and the heat generated by fuel is 13,000 kJ/s. If the
thermal efficiency is 36.15%, find the power needed for the auxiliaries.
A. 310 kW B. 300 kW C. 400 kW D. 350 kW
Solution:

20
72. A superheat steam Rankine cycle has turbine inlet conditions of 17.5 MPa and 530°C
expands in a turbine to 0.007 MPa. The turbine and pump polytropic efficiencies are 0.85 and
0.75 respectively. Pressure losses between the pump and turbine inlet are 1.5 MPa. What
should be the pump work in kJ/kg?
A. 17.34 B. 27.32 C. 25.32 D. 47.33
Solution:

( ) ( )

73. In an open feedwater heater for a steam plant, saturated steam at 7 bar is mixed with
subcooled liquid at 7 bar and 25°C. Just enough steam is supplied to ensure that the mixed
steam leaving the heater will be saturated liquid at 7 bar when heater efficiency is 95%.
Calculate the mass flow rate of the subcooled liquid if steam flow rate is 0.865 kg/s.
Steam properties:
@ 7 bar, saturated vapor: hg = 2763.5 kJ/kg
@ 7 bar and 25°C: hf = 105.5 kJ/kg
@ 7 bar, saturated liquid: hf = 697.22 kJ/kg
A. 2.725 kg/s B. 3.356 kg/s C. 2.869 kg/s D. 3.948 kg/s
Solution:
̇ ( )
̇ ( )
̇ ( )
( )
̇

74. A steam condenser receives 10 kg/s of steam with an enthalpy of 2770 kJ/kg. Steam
condenses and leaves with an enthalpy of 160 kJ/kg. Cooling water passes through the
condenser with temperature increases from 13°C to 24°C. Calculate the water flow rate in kg/s.
A. 583 B. 567 C. 523 D. 528
Solution:

̇ ( ) ̇ ( )
( ) ̇ ( )( )
21
̇

75. Steam expands adiabatically in a turbine from 2000 kPa, 400°C to 400 kPa, 250°C. What is
the effectiveness of the process in percent assuming an atmospheric pressure of 18°C? Neglect
changes in kinetic and potential energy.
Steam properties:
@ 2000 kPa and 400°C: h = 3247.6 kJ/kg s = 7.1271kJ/kg-K
@ 400 kPa and 250°C: h = 2964.2 kJ/kg s = 7.3789 kJ/kg-K
A. 82 B. 84 C. 79.60 D. 79.46
Solution:

( ) ( )( )

76. A heat exchanger was installed purposely to cool 0.50 kg of gas per second. Molecular
weight is 32 and k = 1.32. The gas is cooled from 150°C to 80°C. Water is available at the rate of
0.30 kg/s and at a temperature of 15°C. Calculate the exit temperature of the water in °C.
A. 44.86 B. 42.86 C. 46.45 D. 40.34
Solution:
-
( )
-

̇ ( ) ̇ ( )
( )( ) ( )( )

77. A 350 mm x 450 mm steam engine running at 280 rpm has a clearance steam condition of 2
MPa and 230°C and exits at 0.1 MPa. The steam consumption is 2000 kg/hr and mechanical
efficiency is 85%. If indicated mean effective pressure is 600 kPa, determine brake thermal
efficiency.
@ 2 MPa and 230°C: h1 = 2849.6 kJ/kg s 1 = 6.4423 kJ/kg-K
@ 0.1 MPa: s f = 1.3026 kJ/kg-K hf = 417.46kJ/kg s fg = 6.0568 kJ/kg-K
hfg = 2258 kJ/kg hf2 = 417.46 kJ/kg
A. 23.34% B. 15.25% C. 14.16% D. 27.34%
Solution:

22
( ) [( ) ( ) ]( )( ) ( )( )( )

̇ ( ) ( )( )

78. A steam turbine receives 5,000 kg/hr of steam at 5 MPa and 400°C and velocity of 30 m/s. It
leaves the turbine at 0.006 MPa and 85% quality and velocity of 15 m/s. Radiation loss is 10,000
kJ/hr. Find the kW developed.
@5 MPa and 400°C: h1 = 3195.7 kJ/kg s 1 = 6.6459 kJ/kg-K
@ 0.006 Mpa: hf = 151.53 kJ/kg hfg = 2415.9 kJ/kg
A. 1273.29 B. 2173.29 C. 1373.60 D. 7231.29
Solution:
( )
̇ ̇ ( )( )
̇ ̇ ( )( )

̇ ̇ ̇ ̇ ̇ ̇
̇ ( ̇ ̇ ) ̇( ) ̇
( ) ( )

79. A steam turbine with 85% stage efficiency receives steam at 7 MPa and 550°C and exhausts
at 20 kPa. Determine the turbine work.
@ 7 MPa and 550°C: h1 = 3530.9 kJ/kg s 1 = 6.9486 kJ/kg-K
@ 20 kPa (0.020 MPa): s f = 0.8320 kJ/kg-K hf = 251.40 kJ/kg
s fg = 7.0766 kJ/kg-K hfg = 2358.3 kJ/kg
A. 1,117 kJ/kg B. 1,132 kJ/kg C. 1,123.34 kJ/kg D. 1,054.95 kJ/kg
Solution:

( )

( )

23
80. A steam turbine with 80% stage efficiency receives steam at 7 MPa and 550°C and exhausts
at 20 kPa. Determine the quality at exhaust.
@ 7 MPa and 550°C: h1 = 3530.9 kJ/kg s 1 = 6.9486 kJ/kg-K
@ 20 kPa (0.020 MPa): s f = 0.8320 kJ/kg-K hf = 251.40 kJ/kg
s fg = 7.0766 kJ/kg-K hfg = 2358.3 kJ/kg
A. 96.96% B. 76.34% C. 82.34% D. 91.69%
Solution:

( )

( )

( )

81. An 18,000 kW geothermal plant has a generator efficiency of 90% and 80% respectively. If
the quality after throttling is 20% and each well discharges 400,000 kg/hr, determine the
number of wells required to produce if the change of enthalpy at entrance and exit of turbine is
500 kJ/kg.
A. 4 wells B. 2 wells C. 6 wells D. 8 wells
Solution:
̇ ̇ ( )
( )
̇ ( )
̇
̇
̇

82. A liquid dominated geothermal plant with a single flash separator receives water at 204°C.
The separator pressure is 1.04 MPa. A direct contact condenser operates at 0.034 MPa. The
turbine has a polytropic efficiency of 0.75. For cycle output of 60 MW, what is the mass flow
rate of the well water in kg/s?
@ 204°C: hf = 870.51 kJ/kg
@ 1.04 MPa: hf = 770.38 kJ/kg hfg = 2009.2 kJ/kg hg = 2779.6 kJ/kg

24
s g = 6.5729 kJ/kg-K
@ 0.034 MPa: hf = 301.40 kJ/kg hfg = 2328.8 kJ/kg s f = 0.09793 kJ/kg-K
s fg = 6.7463 kJ/kg-K
A. 2,933 B. 2,100 C. 1,860 D. 2,444
Solution:

( )

( )
̇ ̇ ( )
̇ ( )( )
̇
( )

( )

̇ ̇
̇
̇

83. An engine-generator rated 9000 kVA at 80% power factor, 3 phase, 4160 V has an efficiency
of 90%. If the overall plant efficiency is 28%, what is the heat generated by the fuel?
A. 18,800 kW B. 28,800 kW C. 7,500 kW D. 25,714 kW
Solution:

84. The indicated thermal efficiency of a two stroke diesel engine is 60%. If friction power is
15% of heat generated, determine the brake thermal efficiency of the engine.
A. 43% B. 45% C. 36% D. 37%
Solution:

̇
25
̇
̇ ̇ ̇
̇
̇ ̇

85. A 305 mm x 457 mm four stroke single acting diesel engine is rated at 150 kW at 260 rpm.
Fuel consumption at rated load is 0.56 kg/kW-hr with a heating value of 43,912 kJ/kg. Calculate
the break thermal efficiency.
A. 10.53% B. 27.45% C. 14.64% D. 18.23%
Solution:
̇
-

( )

86. A waste heat recovery boiler produces 4.8 MPa (dry saturated) steam from 104°C
feedwater. The boiler receives energy from 7 kg/s of 954°C dry air. After passing through a
waste heat boiler, the temperature of the air has been reduced to 343°C. How much steam in
kg is produced per second? Note: @ 4.80 MPa dry saturated, h = 2796 kJ/kg.
A. 1.30 B. 0.92 C. 1.81 D. 3.43
Solution:

( )

̇ ( ) ̇ ( )
( )( ) ̇ ( )
̇

87. A diesel electric power plant supplies energy for Meralco. During a 24-hour period, the plant
consumed 240 gallons of fuel at 28°C and produced 3930 kW-hr. Industrial fuel used is 28°API
and was purchased at P30 per liter at 15.6°C. What is the cost of fuel to produce one kW-hr?
A. P6.87 B. P1.10 C. P41.07 D. P5.00
Solution:

( )
[ ( )]
( )

26
( )
-

88. In a gas unit, air enters the combustion chamber at 550 kPa, 227°C and 43 m/s. The
products of combustion leave the combustor at 511 kPa, 1004°C and 180 m/s. Liquid fuel enters
with a heating value of 43,000 kJ/kg. For fuel-air ratio of 0.0229, what is the combustor
efficiency in percent?
A. 70.38% B. 79.38% C. 75.38% D. 82.38%
Solution:
( )

( ) ( ) ( )

89. The specific speed of turbine is 85 rpm and running at 450 rpm. If the head is 20 m and
generator efficiency is 90%, what is the maximum power delivered by the generator?
A. 450.51 kW B. 354.52 kW C. 650.53 kW D. 835.57 kW
Solution:


( )

( )

90. In Francis turbine, the pressure gage leading to the turbine casing reads 380 kPa. The
velocity of water entering the turbine is 8 m/s. If net head of the turbine is 45 m, find the
distance from the center of the spiral casing to the tailrace.
A. 3.0 m B. 3.5 m C. 4.0 m D. 4.5 m
Solution:

( )
27
91. A turbine has a mechanical efficiency of 93%, volumetric efficiency of 95% and total
efficiency of 82%. If the effective head is 40 m, find the total head.
A. 48.72 m B. 40.72 m C. 36.22 m D. 34.72 m
Solution:

( )

( )

92. A Pelton type turbine has 25 m head friction loss of 4.5 m. The coefficient of friction head
loss (Morse) is 0.00093 and penstock length of 80 m. What is the penstock diameter?
A. 1,355.73 mm B. 3476.12 mm C. 6771.23 mm D. 1686.73 mm
Solution:

√ √ ( )( )

( )( )( )

93. In a 9,000 kW hydro-electric plant, the overall efficiency is 88% and the actual power
received by the costumer is 110,000 kW-hrs for that day. What is the secondary power that this
plant could deliver during the entire day?
A. 58,960 kW-hrs B. 80,080 kW-hrs C. 65,960 kW-hrs D. 70,960 kW-hrs
Solution:
( )( ) -
-

94. A Pelton type turbine was installed 30 m below the head gate of the penstock. The head
loss due to friction is 12 percent of the given elevation. The length of the penstock is 100 m and
the coefficient of friction is 0.00093. Determine the power output in kW. (Use Morse equation).
A. 22,273 B. 23,234 C. 32,345 D. 34,452
Solution:
( )

28
√ √ ( )( )

( )( )( )

( ) ( )
( )( )

95. Water flows steadily with a velocity of 3.05 m/s in a horizontal pipe having a diameter of
25.24 cm. At one section of the pipe, the temperature and pressure of the water are 21°C and
689.3 kPa respectively. At a distance of 304.8 m downstream, the pressure is 516.9 kPa. What is
the friction factor?
A. 0.134 B. 0.0050 C. 0.0307 D. 0.641
Solution:

( )( )
( )( )

96. A hydro-electric plant having 30 sq. km reservoir area and 100 m head is used to generate
power. The energy utilized by the consumers whose load is connected to the power plant
during a five-hour period is 13.5x106 kWh. The overall generation efficiency is 75%. Find the fall
in the height of water in the reservoir after the 5-hour period.
A. 5.13 m B. 1.32 m C. 3.21 m D. 2.20 m
Solution:

( ) ( )( )

( )

97. The gas density of chimney is 0.75 kg /m3 and air density of 1.15 kg /m3. Find the driving
pressure if the height of the chimney is 63.71 m.
A. 0.15 kPa B. 0.25 kPa C. 0.35 kPa D. 0.45 kPa
Solution:
29
( ) ( )( )

98. The actual velocity of gas entering in a chimney is 8 m/s. The gas temperature is 25°C with a
gas constant of 0.287 kJ/kg-K. Determine the gas pressure for a mass of a gas is 50,000 kg/hr
and chimney diameter of 1.39 m.
A. 95 kPa B. 98 kPa C. 101 kPa D. 92 kPa
Solution:
̇ ( ) ( )
̇ ̇
( ) ( )( )

99. A steam generator with economizer and air heater has an overall draft loss of 25.78 cm of
water. If the stack gases are at 177°C and if the atmosphere is at 101.3 kPa and 26°C, what
theoretical height of stack in meters is needed when no draft fan is used? Assume that the gas
constant for the flue gases is the same as that for air.
A. 611.10 B. 631.10 C. 651.10 D. 671.10
Solution:

( )

( )
( )
( )
( )

100. A foundation measures 12 ft x 14 ft x 16 ft. Find the number of sacks of cement needed for
a 1:2:4 mixture.
A. 302 B. 404 C. 356 D. 598
Solution:
( )

( )

101. A rectangular foundation cross-section has a bed plate dimension of 8 ft x 10 ft. The
uniform clearance on each side is 1 ft. The height of the foundation is 4.5 ft. If the weight of the

30
steel bar reinforcements needed is ½% of weight of foundation, find the weight of the steel
bars. Use concrete density of 2400 kg/m3.
A. 173.47 kg B. 183.47 kg C. 163.47 kg D. 153.47 kg
Solution:
( )( )
( )
( )
( )

102. A steam pipe having a surface temperature of 250°C passes through a room where the
temperature is 27°C. The outside diameter of the pipe is 100 mm and emissivity factor is 0.8.
Calculate the radiated heat loss for 3 m pipe length.
A. 1434.47 W B. 3746.35 W C. 2851.82 W D. 3546.45 W
Solution:
( )( )

( )
-
( )( )( )
-

103. Brine enters a circulating brine cooler at the rate of 60 m3/hr at -8°C and leaves at -18°C.
Specific heat of brine is 1.072 kJ/kg-K and a specific gravity of 1.12. Determine the tons of
refrigeration.
A. 53.5 TR B. 65.3 TR C. 33.5 TR D. 56.9 TR
Solution:
( )
( )
̇
̇ ̇ ( )(- )

104. A turbo-charged, 16 cylinder, Vee-type diesel engine has an air consumption of 3,000 kg/hr
per cylinder at rated load and speed. This air is drawn in through a filter by a centrifugal
compressor directly connected to the exhaust gas turbine. The temperature of the air from the

31
compressor is 135°C and a counter flow air cooler reduces the air temperature to 45°C before it
goes to the engine suction header. Cooling water enters the air cooler at 30°C and leaves at
40°C. Calculate the log mean temperature difference.
A. 47.23°C B. 87.82°C C. 43.34°C D. 65.24°C
Solution:

105. Water is flowing in a pipe with radius of 30 cm at a velocity of 5 m/s at the temperature in
the pipe. The density and viscosity of the water are as follows: density = 997.9 kg/s viscosity =
1.131 Pa-s. What is the Reynolds number for this situation?
A. 2647 B. 96.2 C. 3100 D. 1140
Solution:

( )

( )

106. Compute the amount of condensate formed during a 10 minute warm-up of 180 meter
pipe conveys the saturated steam with enthalpy of vaporization, h fg = 1947.8 kJ/kg. The
minimum external temperature of pipe is 2°C. The final temperature of pipe is 195°C. The
specific heat of pipe material is 0.6 kJ/kg-°C. The specific weight is 28 kg/m.
A. 240.69 kg B. 982.45 kg C. 299.64 kg D. 423.45 kg
Solution:
( )

( ) ( )
( ) ( )( )

107. The discharge pressure of an air compressor is 5 times the suction pressure. If volume flow
at suction is 0.1 m3/s, what is the suction pressure if compressor work is 19.57 kW (use n =
1.35).

32
A. 97 kPa B. 98 kPa C. 99 kPa D. 100 kPa
Solution:

̇ [( ) ]
( )
[ ]

108. The initial condition of air in an air compressor is 98 kPa and 27 °C and discharges air at
450 kPa. The bore and stroke are 355 mm and 381 mm, respectively with percent clearance of
8% running at 300 rpm. Find the volume of air at suction.
A. 541.62 m3/hr B. 551.62 m3/hr C. 561.62 m3/hr D. 571.62 m3/hr
Solution:

( ) ( )

( ) ( )( )
( )

109. An air compressor has a suction volume of 0.35 m3/s at 97 kPa and discharges at 650 kPa.
How much power is saved by the compressor if there are two stages?
A. 18.27 kW B. 16.54 kW C. 13.86 kW D. 11.58 kW
Solution:
( )( )
̇ [( ) ] [( ) ]

√ √ ( )
( )( )( )
̇ [( ) ] [( ) ]

110. A two stage air intercooler has an intercooler pressure of 4 kg/cm2. What is the discharge
pressure if suction pressure is 1 kg/cm2?
A. 3 kg/cm2 B. 9 kg/cm2 C. 12 kg/cm2 D. 16 kg/cm2
Solution:

33
111. A two-stage air compressor at 100 kPa and 22°C discharges to 750 kPa. If the intercooler
intake is 105°C, determine the value of n.
A. 1.400 B. 1.326 C. 1.345 D. 1.288
Solution:
√ ( )

( )

( )

112. A single acting compressor has a volumetric efficiency of 89%, operates at 500 rpm. It
takes in air at 100 kPa and 30°C and discharges it at 600 kPa. The air handled is 8 m 3/min
measured at discharged condition. If compression is isentropic, find the effective mean
pressure in kPa.
A. 233.34 B. 973.17 C. 198.34 D. 204.82
Solution:

( )

( )( )
̇ [( ) ] [( ) ]

113. A water-jacketed air compressor handles 0.343 m3/s of air entering at 96.5 kPa and 21°C
and leaving at 480 kPa and 132°C; 10.9 kg/hr of cooling water enters the jacket at 15°C and
leaves at 21°C. Determine the compressor brake power.
A. 23.163 kW B. 62.65 kW C. 34.44 kW D. 19.33 kW
Solution:
34
( )

( )

( )( )
̇ [( ) ] [( ) ]

̇ ̇ ( ) ( )( )
̇ ̇

114. A double suction centrifugal pump delivers 20 ft3/s of water at a head of 12 m and running
at 650 rpm. What is the specific speed of the pump?
A. 5014.12 rpm B. 6453.12 rpm C. 2770.73 rpm D. 9968.73 rpm
Solution:

115. Determine the number of stages needed for a centrifugal pump if it is used to deliver 400
gal/min of water and pump power of 15 hp. Each impeller develops a head of 30 ft.
A. 6 B. 4 C. 5 D. 7
Solution:
̇
( )

116. The suction pressure of a pump reads 3 in. of mercury vacuum and discharge pressure
reads 140 psi is used to deliver 120 gpm of water with specific volume of 0.0163 ft 3/lb.
Determine the pump work.
A. 4.6 kW B. 5.7 kW C. 7.4 kW D. 8.4 kW
Solution:
- -

35
-

( )( )

117. A submersible pump delivers 350 gpm of water to a height of 5 ft from the ground. The
pump where installed 150 ft below the ground level and a drawdown of 8 ft during the
operation. If water level is 25 ft above the pump, determine the pump power.
A. 7.13 kW B. 4.86 kW C. 7.24 kW D. 9.27 kW
Solution:
( )

̇ ( )( )

118. A vacuum pump is used to drain a flooded mine shaft of 20°C water. The pump pressure of
water at this temperature is 2.34 kPa. The pump is incapable of lifting the water higher than 16
m. What is the atmospheric pressure?
A. 159.30 B. 132.33 C. 196.22 D. 171.9
Solution:

( )

119. A submersible, multi-stage, centrifugal deep well pump 260 gpm capacity is installed in a
well 27 ft below the static water level and running at 3000 rpm. Drawdown when pumping at
rated capacity is 10 feet. The pump delivers the water into a 25,000 gallons capacity overhead
storage tank. Total discharge head developed by pump, including friction in piping is 243 ft.
Calculate the diameter of the impeller of this pump in inches if each impeller diameter
developed a head of 38 ft.
A. 3.28 B. 5.33 C. 3.71 D. 6.34
36
Solution:


( ) √ ( )( )

120. A fan draws 1.42 m3 per second of air at a static pressure of 2.54 cm of water through a
duct 300 mm diameter and discharges it through a duct of 275 mm diameter. Determine the
static fan efficiency if total fan is 75% and air is measured at 25°C and 760 mm Hg.
A. 50.11% B. 53.69% C. 65.67% D. 45.34%
Solution:

( )
( )

( )

( )

( )

( )( )

( )( )

121. A water cooler uses 50 lb/hr of melting ice to cool running water from 80°F to 42°F. Based
on the inside coil area, Ui = 110 Btu/hr-ft2-°F. Find the gpm of water cooled.
A. 0.10 GPM B. 0.21 GPM C. 0.38 GPM D. 0.45 GPM
Solution:
̇ ̇ ̇ ( )
( ) ̇ ( )( )
̇
( )

37
122. The charge in a Diesel engine consists of 18.34 grams of fuel, with lower heating value of
42,571 kJ/kg, and 409 grams of fuel and products of combustion. At the beginning of
compression, T1 = 60°C. Let rk = 14. For constant c p = 1.11 kJ/kg-°C, what should be the cut-off
ratio in the corresponding ideal cycle?
A. 2.05 B. 2.34 C. 5.34 D. 2.97
Solution:
( )

( )

( )
( )
( )( )

123. The gain of entropy during isothermal nonflow process of 5 lb of air at 60°F is 0.462 Btu/°R.
Find the V1/V2.
A. 3.85 B. 0.259 C. 1.0 D. 0.296
Solution:

( )

124. An auditorium seating 1500 people is to be maintained at 80°F dry bulb and 60°F wet bulb
temperature when outdoor air is at 91°F dry bulb and 75°F wet bulb. Solar heat load is 110,000
Btu/hr and supply air is at 60°F, determine the amount of supply air.
A. 93,229.17 lb/hr B. 83,229.17 lb/hr C. 73,229.17 lb/hr D. 63,229.17 lb/hr
Solution:

̇ ( )
̇ ̇ ( )
̇( )( )
38
̇

125. In a Brayton cycle that operates between temperature limits of 300 K and 1773 K with k =
1.4, determine the temperature at the end of compression (isentropic) for maximum work of
the cycle.
A. 780 K B. 690.5 K C. 730 K D. 350 K
Solution:

( )

126. A 35% solution leaves the absorber and 30% solution enters the absorber. The heat
removed from the absorber by cooling water is 547.6 Btu and ammonia is superheated by 10°.
Find the pound per pound of ammonia gas from the evaporating coils.
A. 11 B. 12 C. 13 D. 14
Solution:

127. A Carnot refrigeration system operates at T max/Tmin = 1.5. Find the kW per ton of
refrigeration.
A. 1.91 B. 2.15 C. 1.76 D. 1.55
Solution:
̇
( ) ( )

( )

128. Assume 8 ft3 of air at 100 psi and 100°F are compressed isothermally to a volume of 2 ft 3.
For each end state of the process, find the bulk modulus.
A. 400 and 100 psi B. 400 and 110 psi C. 400 and 120 psi D. 400 and 130 psi
Solution:
- ( )

( ) ( )

- ( )

39
- ( )

129. Predict the pressure of nitrogen gas at T = 200 K and υ = 0.00385 L/g and b = 0.00141 L/g; a
= 0.178 L2-kPa/g2. Use Van der Waals equation.
A. 15,331 kPa B. 14,331 kPa C. 13,331 kPa D. 12,331 kPa
Solution:
( )
( )

130. A Francis turbine is to be operated at a speed of 600 rpm and with discharge of 4 m3/s. If r1
= 0.6 m, β = 110°, and the blade height B is 10 cm, what should be the guide vane angle α1 for
nonseparating flow condition at the runner entrance?
A. 14.4° B. 15.4° C. 16.4° D. 17.4°
Solution:
( )

( )( )

( )( )

( )

131. The total head of fan is 187 m and has a static pressure of 210 mm of water gage, what is
the velocity of air flowing if the density of air is 1.15 kg/m3?
A. 6.85 m/s B. 3.45 m/s C. 4.39 m/s D. 9.28 m/s
Solution:
( )

( )

132. A fan delivers 5.7 m3/s at a static pressure of 5.08 cm of water when operating at a speed
of 400 rpm. The power input required is 2.963 kW. If 7.05 m3/s are desired in the same fan and
installation, find the pressure in cm of water.
A. 7.77 B. 17.14 C. 11.43 D. 5.08
Solution:

40
( )

( )

133. A rigid container is closed at one end and measures 8 in diameter by 12 in long. The
container is held vertical and is slowly moved downward until the pressure in the container is
17 psia. What will be the depth of the top of the container from the free water surface?
A. 42.36 in B. 59.29 in C. 63.69 in D. 69.82 in
Solution:

( )

134. An empty, open can is 30 cm high with a 15 cm diameter. The can, with the open end
down, is pushed under water with a density of 1000 kg/m3. Find the water level in the can
when the top of the can is 50 cm below the surface.
A. 17.20 cm B. 2.12 cm C. 4.20 cm D. 5.87 cm
Solution:
( )

( ) [ ( )]

41
135. A cylindrical pipe with water flowing downward at 0.03 m3/s having top diameter of 0.08,
bottom diameter of 0.04 m and height of 1.5 m. Find the pressure inside the pipe.
A. 154.63 kPa B. 197.93 kPa C. 252.44 kPa D. 243.92 kPa
Solution:

( )

( )

( )

( )
(- )

136. Determine the size of pipe which will deliver 8 liters of medium oil (ν = 6.10x10-6 m2/s)
assuming laminar flow conditions.
A. 622 mm B. 754 mm C. 950 mm D. 835 mm
Solution:

( -
)

137. The type of flow occupying in a 1 cm diameter pipe which water flows at a velocity of 2.50
m/s. Use ν = 1.13x10-6 m2/s for water.
A. turbulent B. constant C. laminar D. none of these
Solution:
( ) -

138. What force is exerted by a water jet, 60 mm in diameter, if it strikes a wall at the rate of 15
m/s?
42
A. 636.17 N B. 442.62 N C. 764.23 N D. 563.34 N
Solution:

[ ( ) ]( )
( )( )( )

139. A 300 mm diameter pipe discharges water at a rate of 200 li/s. Point 1 on the pipe has a
pressure of 260 kPa and 3.4 m below point 1 is point 2 with a pressure of 300 kPa. Compute the
head loss between points 1 and 2.
A. 4.29 m B. 2.59 m C. 6.32 m D. 1.87 m
Solution:

( )

140. Water flowing at the rate of 10 m/s from an orifice at bottom of a reservoir. Find the
pressure at the bottom of reservoir.
A. 30 kPa B. 40 kPa C. 50 kPa D. 60 kPa
Solution:

( )
( )

141. Steam flows through a nozzle at 400°C and 1 MPa (h = 3263.9 kJ/kg) with velocity of 300
m/s. Find the stagnation enthalpy.
A. 3300 kJ/kg B. 3290 kJ/kg C. 3320 kJ/kg D. 3309 kJ/kg
Solution:

142. Air flows through a nozzle at a speed of 350 m/s. Find the stagnation temperature if the
entrance temperature is 200°C.
A. 241.25°C B. 251.25°C C. 261.25°C D. 271.25°C
Solution:
( ) ( )

43
143. Carbon dioxide flows through a nozzle with a speed of 400 m/s. Compute the dynamic
temperature.
A. 92.56 K B. 94.56 K C. 96.56 K D. 98.56 K
Solution:
-

( )

144. Carbon dioxide flows through a nozzle with speed of 380 m/s. The entrance condition of
nozzle is 250°C and 1200 kPa. Find the stagnation pressure.
A. 2,136.34 kPa B. 2,146.34 kPa C. 2,156.34 kPa D. 2,166.34 kPa
Solution:

( )

( )

145. Air enters a diffuser with a velocity of 200 m/s. Determine the velocity of sound if air
temperature is 30°C.
A. 349 m/s B. 359 m/s C. 369 m/s D. 379 m/s
Solution:
√ √ ( )( )( )

146. Air flows through a nozzle with temperature of entrance of 420 K stagnation temperature
of 468 K. Find the Mach number.
A. 0.744 B. 0.754 C. 0.764 D. 0.774
Solution:

√ √ ( )( )( )
44
147. Air at 300 K and 200 kPa is heated at constant pressure to 600 K. Determine the change of
internal energy.
A. 245.58 kJ/kg B. 235.58 kJ/kg C. 225.58 kJ/kg D. 215.58 kJ/kg
Solution:
( ) ( )( )

148. An insulated rigid tank initially contains 1.5 lb of helium at 80°F and 50 psia. A paddle
wheel with power rating of 0.02 hp is operated within the tank for 30 min. Determine the final
temperature.
A. 159.22°F B. 169.22°F C. 179.22°F D. 189.22°F
Solution:
( )

( )( ) ( )( )

149. A 4 m2 asphalt pavement with emissivity of 0.85 has a surface temperature of 50°C. Find
the maximum rate of radiation that can be emitted from the surface.
A. 2,068 .32 watts B. 2,078.32 watts C. 2,088.32 watts D. 2,098.32 watts
Solution:
̇ ( -
) ( )( )

150. Air at 10°C and 80 kPa enters a diffuser of a jet engine steadily with a velocity of 200 m/s.
The inlet area of diffuser is 0.40 m2. Determine the mass flow rate of air.
A. 72.79 kg/s B. 74.79 kg/s C. 76.79 kg/s D. 78.79 kg/s
Solution:

( )
̇ ( )( )

151. Consider a refrigerator whose 40 watts light bulb remains on continuously as a result of a
malfunction of the switch. If the refrigerator has a COP of 1.3 and the cost of electricity is 8
cents per kW-hr, determine the increase in the energy consumption of the refrigerator and its
cost per year if the switch is not fixed.
A. P 49.59 B. P 47.59 C. P 45.59 D. P 43.59
45
Solution:

̇
̇
̇ ̇ ̇
̇
( )( )

152. A 75 hp motor that has an efficiency of 91% is worn out and is replaced by a high-efficiency
motor that has an efficiency of 95.4%. Determine the reduction in heat gain of the room due to
higher efficiency under full-load conditions.
A. 2.24 kW B. 2.44 kW C. 2.64 kW D. 2.84 kW
Solution:
( )
( )
̇

153. A household refrigerator that has a power input of 450 watts and a COP of 2.5 is to cool
five large watermelons, 10 kg each, to 8°C. If the watermelons are initially at 20°C, determine
how long it will take for the refrigerator to cool them. The watermelons can be treated as water
whose specific heat is 4.2 kJ/kg-K.
A. 2220 seconds B. 2230 seconds C. 2240 seconds D. 2250 seconds
Solution:

( )
( )( )

154. When a man returns to his wall-sealed house on a summer day, he finds that the house is
at 32°C. He returns on the air conditioner which cools the entire house to 20°C in 15 minutes. If
COP is 2.5, determine the power drawn by the air conditioner. Assume the entire mass within
the house is 800 kg of air for which c v = 0.72 kJ/kg-K, cp = 1.0 kJ/kg-K.
A. 1.072 kW B. 2.072 kW C. 3.072 kW D. 4.072 kW
Solution:
46
̇ ( ) ( )( )( )

155. A heat source at 800 K losses 2000 kJ of heat to a sink at 500 K. Determine the entropy
generated during this process.
A. 1.5 kJ/K B. 2.5 kJ/K C. -2.5 kJ/K D. 4 kJ/K
Solution:
- -
-

156. Helium gas is compressed in an adiabatic compressor from an initial state of 14 psia and
50°F to a final temperature of 320°F in a reversible manner. Determine the exit pressure of
Helium.
A. 38.5 psia B. 40.5 psia C. 42.5 psia D. 44.5 psia
Solution:

( )

( )

157. Air passes thru a nozzle with efficiency of 90%. The velocity of air at the exit is 600 m/s.
Find the actual velocity at the exit.
A. 382 m/s B. 540 m/s C. 458 m/s D. 568 m/s
Solution:
( )

( )

158. A 50 kg block of iron casting at 500 K is thrown into a large lake that is at a temperature of
285 K. The iron block eventually reaches thermal equilibrium with the lake water. Assuming
average specific heat of 0.45 kJ/kg-K for the iron, determine the entropy generated during this
process.

47
A. -12.65 kJ/K B. 16.97 kJ/K C. 4.32 kJ/K D. 6.32 kJ/K
Solution:
( ) -
( )( )

159. A windmill with a 12 m diameter rotor is to be installed at a location where the wind is
blowing at an average velocity of 10 m/s. Using standard condition of air (1 atm, 25°C),
determine the maximum power that can be generated by the windmill.
A. 68 kW B. 70 kW C. 72 kW D. 74 kW
Solution:

( )

̇ ( )( )

̇ ( )

160. Consider a large furnace that can supply heat at a temperature of 2000°R at a steady rate
of 3000 Btu/s. Determine the exergy of this energy. Assume an environment temperature of
77°F.
A. 2305.19 kW B. 2315.19 kW C. 2325.19 kW D. 2335.19 kW
Solution:
( )

̇ ̇ ( )

161. A heat engine receives heat from a source at 1200 K at a rate of 500 kJ/s and rejects the
waste heat to a medium at 300 K. The power output of the engine is 180 kW. Determine the
irreversibility rate for this process.
A. 190 kW B. 195 kW C. 200 kW D. 205 kW
Solution:

̇ ̇
̇

48
162. A dealer advertises that he has just received a shipment of electric resistance heaters for
residential buildings that have an efficiency of 100 percent. Assuming an indoor temperature of
21°C and outdoor temperature of 10°C, determine the second law efficiency of these heaters.
A. 8.74% B. 6.74% C. 3.74% D. 4.74%
Solution:

163. A thermal power plant has a heat rate of 11,363 Btu/kW-hr. Find the thermal efficiency of
the plant.
A. 34% B. 24% C. 26% D. 30%
Solution:

164. A rigid tank contains 2 kmol of N2 and 6 kmol of CO 2 gases at 300 K and 115 MPa. Find the
tank volume using the ideal gas equation.
A. 7.33 m3 B. 5.33 m3 C. 3.33 m3 D. 1.33 m3
Solution:

( )( )( )

165. A spherical balloon with a diameter of 6 m is filled with helium at 20°C and 200 kPa.
Determine the mole number.
A. 9.28 kmol B. 10.28 kmol C. 11.28 kmol D. 13.28 kmol
Solution:

[ ( ) ] ( )( )

166. The air in an automobile tire with a volume of 0.53 ft3 is at 90°F and 20 psig. Determine the
amount of air that must be added to raise the pressure to the recommended value of 30 psig.
Assume the atmospheric pressure to be 14.7 psia and the temperature and the volume to
remain constant.
A. 0.026 lb B. 0.046 lb C. 0.066 lb D. 0.086 lb
49
Solution:

( )( )( ) ( )( )

( )( )( ) ( )( )

167. A rigid tank contains 20 lbm of air at 20 psia and 70°F. More air is added to the tank until
the pressure and temperature rise to 35 psia and 90°F, respectively. Determine the amount of
air added to the tank.
A. 11.73 lb B. 13.73 lb C. 15.73 lb D. 17.73 lb
Solution:

( )( )

( ) ( )( )

168. A rigid tank contains 5 kg of an ideal gas at 4 atm and 40°C. Now a valve is opened, and
half of mass of the gas is allowed to escape. If the final pressure in the tank is 1.5 atm, the final
temperature in the tank is?
A. -38°C B. -30°C C. 40°C D. 53°C
Solution:

( )( )

( ) ( )
- -

169. The pressure of an automobile tire is measured to be 200 kPa (gage) before the trip end
220 kPa (gage) after the trip at a location where the atmospheric pressure is 90 kPa. If the
temperature of the air in the tire before the trip is 25°C, the air temperature after the trip is?

50
A. 45.6°C B. 54.8°C C. 27.5°C D. 26.7°C
Solution:

170. Water is boiling at 1 atm pressure in a stainless steel pan on an electric range. It is
observed that 2 kg of liquid water evaporates in 30 min. The rate of heat transfer to the water
is?
A. 2.07 kW B. 0.47 kW C. 2.51 kW D. 3.12 kW
Solution:
̇ ̇ ( )
( )

171. Consider a person standing in a breezy room at 20°C. Determine the total rate of heat
transfer from this person if the exposed surface area and the average outer surface
temperature of the person are 1.6 m2 and 29°C, respectively, and the convection heat transfer
coefficient is 6 W/m2 with emissivity factor of 0.95.
A. 86.40 watts B. 81.70 watts C. 198.1 watts D. 168.1 watts
Solution:
̇ ( ) ( )( )
̇ ( -
)( )[( ) ( ) ]
̇ ̇ ̇

172. Water is boiler in a pan on a stove at sea level. During 10 minutes of boiling, it is observed
that 200 grams of water has evaporated. Then the rate of heat transfer to the water is:
A. 0.84 kJ/min B. 45.1 kJ/min C. 41.8 kJ/min D. 53.5 kJ/min
Solution:
̇ ̇ ( )

173. An aluminum pan whose thermal conductivity is 237 W/m-°C has a flat bottom whose
diameter is 20 cm and thickness of 0.4 cm. Heat is transferred steadily to boiling water in the
pan through its bottom at a rate of 500 watts. If the inner surface of the bottom of the pan is
105°C, determine the temperature of the surface of the bottom of the pan.
A. 95.27°C B. 105.27°C C. 115.27°C D. 125.27°C
Solution:

51
( )
( )
̇
( )( )

174. For heat transfer purposes, a standing man can be modeled as a 30 cm diameter, 170 cm
long vertical cylinder with both the top and bottom surfaces insulated and with the side surface
at an average temperature of 34°C. For a convection heat transfer coefficient of 15 W/m 2-°C,
determine the rate of heat loss from this man by convection in an environment at 20°C.
A. 316.46 watts B. 326.46 watts C. 336.46 watts D. 346.46 watts
Solution:
̇ ( ) ( )( )

175. A 5 cm diameter spherical ball whose surface is maintained at a temperature of 70°C is


suspended in the middle of a room at 20°C. If the convection heat transfer coefficient is 15
W/m2-°C and the emissivity of the surface is 0.8, determine the total heat transfer from the
ball.
A. 23.56 watts B. 32.77 watts C. 9.22 watts D. 43.45 watts
Solution:
( )
̇ ( ) ( )( )
̇ ( -
)( )[( ) ( ) ]
̇ ̇ ̇

176. A frictionless piston-cylinder device and a rigid tank contain 1.2 kmol of an ideal gas at the
same temperature, pressure and volume. Now heat is transferred, and the temperature of both
systems is raised by 15°C. The amount of extra heat that must be supplied to the gas in the
cylinder that is maintained at constant pressure is?
A. 0 B. 50 kJ C. 100 kJ D. 150 kJ
Solution:
( ) ( )( )

177. A supply of 50 kg of chicken at 6°C contained in a box is to be frozen to -18°C in a freezer.


Determine the amount of heat that needs to be removed. The latent heat of the chicken is 247
kJ/kg, and its specific heat is 3.32 kJ/kg-°C above freezing and 1.77 kJ/kg-°C below freezing. The

52
container box is 1.5 kg, and the specific heat of the box material is 1.4 kJ/kg -°C. Also the
freezing temperature of chicken is -2.8°C.
A. 15,206.4 kJ B. 50.4 kJ C. 15,156 kJ D. 1,863 kJ
Solution:
[ ( ) (- )]
( )( )

178. Water is being heated in a closed pan on top of a range while being stirred by a paddle
wheel. During the process, 30 kJ of heat is transferred to the water, and 5 kJ of heat is lost to
the surrounding air. The paddle wheel work amounts to 500 N-m. Determine the final energy of
the system if its initial energy is 10 kJ.
A. 35.5 kJ B. 40.5 kJ C. 25.5 kJ D. 14.5 kJ
Solution:

179. A classroom that normally contains 40 people is to be air-conditioned with window air-
conditioning units of 5 kW cooling capacity. A person at rest may be assumed to dissipate heat
at a rate about 360 kJ/hr. There are 10 light bulbs in the room, each with a rating of 100 watts.
The rate of heat transfer to the classroom through the walls and the windows is estimated to
be 15,000 kJ/hr. If the room is to be maintained at a constant temperature of 21°C, determine
the number of window air-conditioning units required.
A. 1 units B. 2 units C. 3 units D. 4 units
Solution:
̇ ( )( ) ( )

180. A 4 m x 5 m x 6 m room is to be heated by a baseboard resistance heater. It is desired that


the resistance heater be able to raise the air temperature in the room from 7 to 23°C within 15
minutes. Assuming no heat losses from the room and an atmospheric pressure of 100 kPa,
determine the required power of the resistance heater. Assume constant specific heats at room
temperature.
A. 2.34 kW B. 1.91 kW C. 4.56 kW D. 6.34 kW
Solution:

( )
( )
53
( ) ( )( )
( )

181. A student living in a 4 m x 6 m x 6 m dormitory room turns on her 150 watts fan before she
leaves the room on a summer day, hoping that the room will be cooler when she comes back in
the evening. Assuming all the doors and windows are tightly closed and disregarding any heat
transfer through the walls and the windows, determine the temperature in the room when she
comes back 10 hours later. Use specific heat values at room temperature, and assume the room
to be at 100 kPa and 15°C in the morning when she leaves.
A. 28.13°C B. 38.13°C C. 48.13°C D. 58.13°C
Solution:

( )
( )
( )
( ) ( )( )

182. A piston-cylinder device whose piston is resting on top of a set of stops initially contains
0.50 kg of helium gas at 100 kPa and 25°C. The mass of the piston is such that 500 kPa of
pressure is required to raise it. How much heat must be transferred to the helium before the
piston starts rising?
A. 1557.13 kJ B. 1657.13 kJ C. 1757.13 kJ D. 1857.13 kJ
Solution:

-
( )( )

( ) ( )( )

183. In order to cool 1 ton (1000 kg) of water at 20°C in an insulated tank, a person pours 80 kg
of ice at -5°C into the water. Determine the final equilibrium temperature in the tank. The
melting temperature and the heat of fusion of ice at atmospheric pressure are 0°C and 333.7
kJ/kg, respectively.
A. 12.43°C B. 14.43°C C. 16.43°C D. 18.43°C
Solution:

54
( )( ) ( )( ) ( ) ( )( )

184. A fan is powered by 0.5 hp motor and delivers air at a rate of 85 m3/min. Determine the
highest value for the average velocity of air mobilized by the fan. Take the density of air to be
1.18 kg/m3.
A. 18.23 m/s B. 21.12 m/s C. 25.34 m/s D. 32.23 m/s
Solution:

( ) ( )

√ ( )( )

185. An Ocean-Thermal Energy Conversion power plant generates 10,000 kW using a warm
surface water inlet temperature of 26°C and cold deep-water temperature of 15°C. On the basis
of a 3°C drop in the temperature of the warm water and a 3°C rise in the temperature of the
cold water due to removal and addition of heat, calculate the power required in kW to pump
the cold-deep water to the surface and through the system heat exchanger if the required
pumping pressure increase is 12 kPa. Assume a Carnot cycle efficiency and density of cold water
to be 100 kg/m3.
A. 108 B. 250 C. 146 D. 160
Solution:

̇
̇

̇
̇
̇ ̇ ̇
̇ ̇
̇( )( )
̇

( )( )

186. A plate-type solar energy collector with an absorbing surface covered by a glass plate is to
receive an incident radiation of 800 W/m2. The glass plate has a reflectivity of 0.12 and a
55
transmissivity of 0.85. The absorbing surface has an absorptivity of 0.90. The area of the
collector is 5 m2. How much solar energy in watts is absorbed by the collector?
A. 2500 B. 2880 C. 3510 D. 3060
Solution:
̇ ( )( )( )

187. A tank contains liquid nitrogen at -190°C is suspended in a vacuum shell by three stainless
steel rods of 0.80 cm in diameter and 3 meters long with a thermal conductivity of 16.3 W/m 2-
°C. If the ambient air outside the vacuum shell is 15°C, calculate the magnitude of the
conductive heat flow in watts along the support rods.
A. 0.168 B. 0.0587 C. 0.182 D. 0.176
Solution:
̇ ( )[ (- )]

188. An elastic sphere containing gas at 120 kPa has a diameter of 1.0 m. Heating the sphere
causes it to expand to a diameter of 1.3 m. During the process the pressure is proportional to
the sphere diameter. Calculate the work done by the gas in kJ.
A. 41.8 B. 50.6 C. 87.5 D. 35.4
Solution:

( )

( )

∫ ∫ ( ) ∫

189. An ideal gas with a molecular weight of 7.1 kg/kg mol is compressed from 600 kPa and 280
K to a final specific volume of 0.5 m3/kg. During the process the pressure varies according to P =
620 + 150υ + 95 υ2 where p is in kPa and υ in m3/kg. Calculate the work of compression in kJ/kg.
A. 32.8 B. 28.7 C. 35.6 D. 33.3
Solution:
( )

56
∫ ∫ ( )

190. A one cubic meter container contains a mixture of gases composed of 0.02 kg-mol of
oxygen and 0.04 kg-mol of helium at a pressure of 220 kPa. What is the temperature of this
ideal gas mixture in Kelvin?
A. 441 B. 350 C. 400 D. 450
Solution:
( ) ( )

191. Methyl alcohol (CH3OH) is burned with 25% excess air. How much unburned oxygen in kg-
moloxygen/kg-molfuel will there be in the products if the combustion is complete?
A. 0.35 B. 0.45 C. 0.37 D. 0.65
Solution:

( ) ( )

( ) ( )( )
( )( ) ( )
( )

192. A 12 DC electrical motor draws a current of 15 amps. How much work in kJ does this motor
produce over a 10-minute period of operation?
A. 108.0 B. 129.6 C. 216.0 D. 318.2
Solution:
( )( )

193. A 4 liter (2-liter per revolution at standard pressure and temperature) spark ignition engine
has a compression ratio of 8 and 2200 kJ/kg heat addition by the fluid combustion. Considering
a cold air-standard Otto cycle model, how much power will the engine produce when operating
at 2500 rpm?
A. 166.53 hp B. 73.12 hp C. 97.4 hp D. 148 hp
Solution:
( )
̇

57
( )
̇ ( )

194. A simple Rankine cycle produces 40 MW of power, 50 MW of process heated and rejects
50 MW of heat to the surroundings. What is the utilization factor of this cogeneration cycle
neglecting the pump work?
A. 50% B. 60% C. 64% D. 80%
Solution:
̇ ̇ ̇ ̇
̇ ̇
̇

195. The rate of heat transfer to the surroundings from a person at rest is 400 kJ/hr. Suppose
that the ventilation system fails in an auditorium containing 120 people and assuming that the
energy goes into the air of volume 1500 m3 initially at 300 K and 101 kPa, calculate the rate in
°C /min of air temperature change.
A. 0.81 B. 0.53 C. 0.63 D. 1.0
Solution:
̇ ̇
̇ ̇
( ) ̇( )( )
̇
( ) ( )

196. An insulated box containing helium gas falls from a balloon 4.5 km above the earth’s
surface. Calculate the temperature rise in °C of the helium when box hits the ground.
A. 15.2 B. 12.6 C. 25.3 D. 14.1
Solution:
-

( )( ) ( )

197. Consider two Carnot heat engines operating in series. The first engine receives heat from
the reservoir at 2400 K and rejects the waste heat to another reservoir at temperature T. The
second engine receives heat by the first one, convert some of it to work, and rejects the rest to
58
a reservoir at 300 K. If thermal efficiencies of both engines are the same, determine the
temperature T.
A. 849 K B. 578 K C. 763 K D. 978 K
Solution:

198. An ideal gas mixture consists of 2 kmol of N2 and 6 kmol of CO 2. The mass fraction of CO 2
is?
A. 0.175 B. 0.250 C. 0.825 D. 0.750
Solution:
( )
( ) ( )

199. An ideal gas mixture consists of 2 kmol of N2 and 6 kmol of CO 2. The apparent gas constant
of mixture is?
A. 0.208 B. 0.231 C. 0.531 D. 0.825
Solution:
( ) ( )
-

200. A Carnot cycle operates between the temperature limits of 300 K and 1500 K, and
produces 600 kW of net power. The rate of entropy change of the working fluid during heat
addition process is?
A. 0 B. 0.4 C. 0.5 D. 2.0
Solution:
̇ ( )
( )

201. Air in an ideal Diesel cycle is compressed from 3 L to 0.15 L and then it expands during the
constant pressure heat addition process to 0.3 L. Under cold air standard conditions, the
thermal effieciency of this cycle is?
A. 35% B. 44% C. 65% D. 70%
Solution:

59
[ ( )
] [ ( )
]

202. Helium gas in an ideal Otto cycle is compressed from 20°C and 2 L to 0.25 L and its
temperature increases by an additional 800°C during the heat addition process. The
temperature of Helium before the expansion process is?
A. 1700°C B. 1440°C C. 1240°C D. 880°C
Solution:

( )
( )( )

203. In an ideal Otto cycle, air is compressed from 1.20 kg/m3 and 2.2 L to 0.26 L and the net
work output of the cycle is 440 kJ/kg. The mean effective pressure for the cycle is?
A. 612 kPa B. 599 kPa C. 528 kPa D. 416 kPa
Solution:
- - -

-
( )

204. An ideal Brayton cycle has a net work output of 150 kJ/kg and backwork ratio of 0.4. if
both the turbine and the compressor had an isentropic efficiency of 80%, the net work output
of the cycle would be?
A. 60 kJ/kg B. 75 kJ/kg C. 98 kJ/kg D. 120 kJ/kg
Solution:

60
( )
( )

205. Air enters a turbojet engine at 200 m/s at a rate of 20 kg/s and exits at 800 m/s relative to
the aircraft. The thrust developed by the engine is?
A. 6 kN B. 12 kN C. 16 kN D. 20 kN
Solution:
( ) ( )

206. A thermal power has a net power of 10 MW. The backwork ratio of the plant is 0.005.
Determine the compressor work.
A. 50.15 kW B. 50.35 kW C. 50.25 kW D. 50.45 kW
Solution:
̇ ̇ ̇
̇
̇
̇
̇
̇ ̇
̇ ̇ ̇
̇ ̇
̇
̇ ( )

207. A heat engine receives heat from a source at 1200 K at a rate of 500 kJ/s and rejects the
wasted heat to a sink at 300 K. If the power output of the engine is 200 kW, the second law
efficiency of this heat engine is?
A. 35% B. 40% C. 53% D. 75%
Solution:

61
208. A water reservoir contains 100,000 kg of water at an average elevation of 60 m. The
maximum amount of electric power that can be generated from this water is ?
A. 8 kWh B. 16 kWh C. 1630 kWh D. 58,800 kWh
Solution:
( )

209. A house is maintained at 22°C in winter by electric resistance heaters. If the outdoor
temperature is 5°C, the second law efficiency of the resistance heaters is ?
A. 0% B. 5.8% C. 34% D. 77%
Solution:

210. A thermoelectric refrigerator that resembles a small ice chest is powered by a car battery,
and has a COP of 0.10. If the refrigerator cools a 0.350 L canned drink from 20°C to 4°C in 30
min, determine the average electric power consumed by the thermoelectric refrigerator.
A. 130 watts B. 110 watts C. 120 watts D. 140 watts
Solution:
̇ ̇ ( ) ( )( )

211. A Carnot refrigerator operates in a room in which the temperature is 25°C and consumes 2
kW of power when operating. If the food compartment of the refrigerator is to be maintained
at 3°C, determine the rate of heat removal from the food compartment.
A. 1504.8 kJ/min B. 12.86 kJ/min C. 1625 kJ/min D. 9.57 kJ/min
Solution:

( ) ( )
̇ ̇ ( )

212. A household refrigerator with EER 8.0 removes heat from the refrigerated space at a rate
of 90 kJ/min. Determine the rate of heat transfer to the kitchen air.
A. 101.25 kJ/min B. 63.05 kJ/min C. 128.46 kJ/min D. 80 kJ/min
Solution:

62
̇
̇ ̇
̇

213. An air-conditioning system is used to maintain a house at 75°F when the temperature
outside is 95°F. The house is gaining the heat through the walls and windows at a rate of 1250
Btu/min, and the heat generation rate within the house from people, lights, and appliances
amounts to 350 Btu/min. Determine the minimum power input required for this air-
conditioning system.
A. 10.06 hp B. 1.36 hp C. 1.41 hp D. 7.94 hp
Solution:
̇

( ) ( )

̇
̇

214. A refrigeration system is to cool bread loaves with an average mass of 450 g from 22°C to -
10°C at a rate of 500 loaves per hour by refrigerated air. Taking the average specific and latent
heats of bread to be 2.93 kJ/kg-°C and 109.3 kJ/kg, respectively, determine the product load.
A. 541.7 kJ/min B. 351.6 kJ/min C. 761.5 kJ/min D. 409.9 kJ/min
Solution:
̇ ( )
̇ ̇ ̇ ( ̇ ) ( ̇ )
( )[ (- )] ( )

215. A house that was heated by electric resistance heaters consumed 1200 kWh of electric
energy in a winter month. If this house were heated instead by a heat pump that has an
average performance factor, PF of 2.4, determine how much money the homeowner would be
saved that month. Assume a price of 0.085 $/kWh for electricity.
A. $42.5 B. $59.50 C. $102 D. $97.75
Solution:
̇
̇
( )( )

63
216. An ammonia simple saturation cycle operates with a suction pressure of 291.6 kPa and a
condenser pressure of 1204 kPa develops 15 tons of refrigeration. Determine the theoretical
horsepower of the compressor. The following enthalpies have been found: condenser entrance
= 1653 kJ/kg, exit = 346.6 kJ/kg; compressor entrance = 1450.2 kJ/kg, exit = 1653 kJ/kg.
A. 7.23 hp B. 13 hp C. 15 hp D. 8.23 hp
Solution:
̇
̇
( )
̇ ̇( )

217. An ammonia ice plant operates between a condenser temperature of 35°C and evaporator
of -15°C. It produces 10 metric tons of ice per day from water at 30°C to ice at -5°C. Assuming
simple saturation cycle, determine the horsepower of the motor if the adiabatic efficiency of
the compressor nc = 0.85 and mechanical efficiency nm = 0.95. The specific heat of ice is 2.094
kJ/kg-°C and the latent heat is 335 kJ/kg. From the table for ammonia the following enthalpies
are: condenser entrance = 1703 kJ/kg, exit = 366.1 kJ/kg; compressor entrance = 1443.9 kJ/kg,
exit = 1703 kJ/kg.
A. 17.68 hp B. 18.61 hp C. 15.5 hp D. 21.9 hp
Solution:
( ) ( )
( ) [ (- )]
( )
̇
̇
̇
( )
̇ ̇( )
̇
( )

218. A Freon 22 air conditioning under standard operating conditions of 35°C condensing and
5°C evaporating temperatures. The volume flow rate entering the compressor is 23.72 L/s.
Determine the refrigerating capacityif the refrigerating effect is 164 kJ/kg. From the table for
R22 the specific volume at the compressor entrance is 40.46 L/kg.
A. 339.3 TR B. 79.3 TR C. 96.4 TR D. 27.4 TR
Solution:

( )

64
219. The refrigerant volume flow rate at the entrance of compressor were obtained from a test
on a twin cylinder, single acting 15 cm x 20 cm, 320 rpm compressor ammonia refrigerating
plant to be 33 L/s. Determine the volumetric efficiency of the compressor.
A. 77.65% B. 87.6% C. 97.6% D. 65.65%
Solution:
( ) ( ) ( )( )( )

( )

220. A twin cylinder ammonia compressor with volume displacement of 14,726 cm3 operates at
300 rpm. Condenser and evaporator pressure are 1200 kPa and 227 kPa respectively. Specific
volume of refrigerant at the entrance of compressor is 528.26 L/kg. Compression process is
polytropic with n = 1.20 and clearance factor of compressor is 2 percent. Determine the
horsepower required.
A. 60 hp B. 70 hp C. 80 hp D. 90 hp
Solution:

( ) ( )

( ) ( )( )
( )
( )( )
̇ [( ) ] [( ) ]

( )

221. A reversed Carnot has a refrigerating COP of 2.5. Determine the ratio TH/TL.
A. 1.4 B. 1.5 C. 1.25 D. 1.2
Solution:

222. Three thousand cubic feet per minute of air are circulated over an air-cooled condenser. If
the load on the condenser is 64,800 Btu/hr, compute the temperature rise of the air passing
over the condenser. Specific volume of standard air (13.34 ft3/lb).
65
A. 10°F B. 15°F C. 20°F D. 25°F
Solution:

̇
( ) ( )
̇

223. Saturated vapor ammonia at -16°C (hT = 1442.60 kJ/kg) leaves the evaporator and enters
the compressor at -6°C (h1 = 1465 kJ/kg). The refrigerant leaves the condenser as saturated
liquid at 40°C (h4 = 390.6 kJ/kg) and enters the expansion valve at 35°C (h5 = 366.1 kJ/kg). Heat
rejected from the condenser amount to 50 kW. The work to compressor is 208 kJ/kg, while the
heat loss from compressor is 33 kJ/kg. If 95 kJ/kg of heat are lost in the piping between the
compressor discharge and condenser inlet, determine the refrigerating capacity of the system.
A. 49.5 TR B. 46.61 TR C. 12.88 TR D. 13.24 TR
Solution:

̇ ̇ ( )

224. In an actual refrigeration cycle using R12 as working fluid, the refrigerant flow rate is 0.05
kg/s. Vapor enters the expansion valve at 1.15 MPa, 40°C (h = 238.5 kJ/kg) and leaves the
evaporator at 175 kPa, -15°C (h = 345 kJ/kg). The electric iput to motor driving the compressor
is measured and found 3.0 kW. Motor efficiency at this load 92% and mechanical efficiency
82%. Determine the actual coefficient of performance for this cycle.
A. 1.58 B. 2.36 C. 1.78 D. 1.34
66
Solution:
̇ ( )
̇ ( )( )

225. In an ammonia refrigeration system, the temperature in the evaporator is -12°C and the
ammonia at the evaporator entry 0.1511 dry while at exit is 0.95 dry. If the rate of ammonia
circulation is 5.64 kg/min, determine the refrigerating capacity of the system. Enthalpy of
saturated liquid and vapor at -12°C is 144.929 kJ/kg and 1447.74 kJ/kg respectively.
A. 17.82 B. 34.82 C. 27.82 D. 4.82
Solution:
( ) ( )

( ) ( )

( )
̇ ̇( )

226. A two stage cascade vapor compression refrigeration system uses ammonia in the low-
temperature loop and R-12 in the high-temperature loop. The ammonia provides 15 tons of
cooling. If the high-loop temperature requires 10.12 kW compressor power and low loop 15.93
kW, determine the COP of the system.
A. 2.027 TR B. 5.22 TR C. 3.314 TR D. 9.1 TR
Solution:
̇
( )

227. When a man returns to his well-sealed house on a summer day, he finds that the house is
at 32°C. He turns on the air conditioner, which cools the entire house to 20°C in 15 min. If COP
of the air conditioner system is 2.5, determine the power drawn by the air conditioners.
Assume the entire mass within the house is equivalent to 800 kg of air for which c = 0.72 kJ/kg -
°C.
A. 7.08 kW B. 3.07 kW C. 19.2 kW D. 12.08 kW
Solution:
( ) ( )( )
̇
( )
̇
̇

67
228. It is desired to double the COP of a reversed Carnot engine for cooling from 5.0 by raising
the temperature of heat addition while keeping the temperature of heat rejection constant. By
what percentage must the temperature of heat addition be raised?
A. 10.1% B. 9.1% C. 8.1% D. 7.1%
Solution:

229. An ammonia water-cooled compressor receives the refrigerant at specific volume 62 L/kg.
It has a piston displacement rate of 5 m3/min. If a squirrel cage motor running at 1200 rpm
drives the compressor and average piston speed is 490 m/min, calculate size of cylinder bore.
A. 20.4 cm B. 26.0 cm C. 16.13 cm D. 13.6 cm
Solution:

( )

( )( )

230. If the initial volume of an ideal gas is compressed the one-half its original volume and to
twice its temperature, the pressure:
A. Doubles B. Quadruples C. Remains constant D. Halves
Solution:

68
( )

231. If the gage pressure of a medium is 30 kPa (vacuum) and the atmospheric pressure is 101.3
kPa, the absolute pressure will be?
A. 131.3 kPa B. -71.3 kPa C. 71.3 kPa D. -131.3 kPa
Solution:

232. If a particle has a velocity of 4 meters per second and a kinetic energy of 144 Joules, then
the mass, in kilograms of this particle must be?
A. 44 B. 16 C. 18 D. 24
Solution:

( )

233. A condenser vacuum gauge reads 715 mm Hg when the barometer stands at 757 mm Hg.
State the absolute pressure in the condenser in kN/m2 or kPa.
A. 5.6 kPa B. 5.9 kPa C. 6.5 kPa D. 5.2 kPa
Solution:
( )

234. Determine the force in newton in a piston of 465 mm2 area with a pressure of 0.172 MPa.
A. 65 N B. 72 N C. 80 N D. 111 N
Solution:
( )( )( )

235. One piston of a hydraulic press has an area of 1 cm2. The other piston has an area of 25
cm2. If a force of 150 N is applied on the smaller piston, what will be the total force on the
larger piston if both piston surfaces are on the same level?
A. 6 N B. 175 N C. 3750 N D. 4250 N
Solution:

69
236. The work done on air is 10.86 kJ/kg, determine the compressor power if it is receiving 272
kg/min of air.
A. 36.72 hp B. 49.23 hp C. 2954 hp D. 66 hp
Solution:
( ) ( )

237. A water tank 18 ft wide, 14 ft long, and 4 ft high, calculate the pressure at the bottom of
the tank.
A. 1.733 psi B. 1.999 psi C. 2.337 psi D. 3.773 psi
Solution:
( )( )

238. What is the pressure of 750 mm Hg in kN/m2?


A. 90 B. 100 C. 103 D. 110
Solution:
( )

239. A double purpose tank 18 ft wide, 24 ft long and 4 ft depth is filled with water. What is the
weight of water in the tank in long tons?
A. 49 tons B. 48 tons C. 54 tons D. 50 tons
Solution:
[ ( )( )] ( )

240. Oil flows through a 16 tube single cooler with a velocity of 2 m/s. The internal diameter of
the tube is 30 mm and oil density is 0.85 g m/ml. Find the volume flow in liters per sec.
A. 22.62 B. 32.22 C. 62.22 D. 42.62
Solution:
( ) ( )( )

241. A substance temperature was 620°R. What is the temperature in °C?


A. 50.7 B. 45.54 C. 71.11 D. 91.44
Solution:
70
[( ) ]( )

242. An unknown volume of container gas of 1 atmosphere is allowed to expand to another


container of 10 m3 volume at 500 mmHg at constant temperature. Find the unknown volume.
A. 6.58 m3 B. 6.75 m3 C. 5.67 m3 D. 7.65 m3
Solution:

( )

243. An iron block weighs 5 Newton and has a volume of 200 cm3. What is the density of the
block?
A. 2458 kg/m3 B. 2485 kg/m3 C. 2584 kg/m3 D. 2549 kg/m3
Solution:
( )
( )( )

244. If air is at a pressure of 22.22 psia and at a temperature of 800°R, what is the specific
volume?
A. 11.3 ft3/lbm B. 33.1 ft3/lbm C. 13.3 ft3/lbm D. 31.3 ft3/lbm
Solution:

( )
( )

245. The Specific gravity of mercury is 13.55. What is the specific weight of mercury?
A. 123.9 kN/m3 B. 139.2 kN/m3 C. 132.9 kN/m3 D. 193.2 kN/m3
Solution:
( )

246. The equivalent weight of mass 10 kg at a location where the acceleration of gravity is 9.77
m/s 2?
A. 97.7 N B. 79.7 N C. 77.9 N D. 977 N
Solution:
( )

71
247. A transportation company specializes in the shipment of pressurized gaseous materials. An
order is received from 100 liters of a particular gas at STP (32°F and 1 atm). What minimum
volume tank is necessary to transport the gas at 80°F and maximum pressure of 8 atm?
A. 16 liters B. 14 liters C. 10 liters D. 12 liters
Solution:

( ) ( )
( ) ( )

248. 100 g of water are mixed with 150 g of alcohol (density = 790 kg/m3). What is the specific
volume of the resulting mixtures, assuming that the fluids mixed completely?
A. 0.82 x 10-3 m3/kg B. 0.88 x 10-3 m3/kg C. 0.63 x 10-3 m3/kg D. 1.16 x 10-3 m3/kg
Solution:

249. How much does 30 lbm weigh on the moon? (g moon = 5.47 ft/s 2).
A. 2.0 lbf B. 3.2 lbf C. 3.4 lbf D. 5.096 lbf
Solution:
( )

250. A 10 kg block is raised vertically 3 meters. What is the change in potential energy?
A. 320 J B. 350 km-m2/s 2 C. 294 J D. 350 N-m
Solution:
( )( )

251. How many cubic meters is 100 gallons of liquid?


A. 3.7850 m3 B. 0.1638 m3 C. 0.3785 m3 D. 1.638 m3
Solution:
( )( )

252. Steam turbine is receiving 1014 lbm/hr of steam, determine the horsepower output of the
turbine if the work done by steam is 251 Btu/lbm.

72
A. 100 hp B. 462.7 hp C. 200 hp D. 600 hp
Solution:

̇ ( )( )

253. What is the resulting pressure when one pound of air at 15 psia and 200°F is heated at
constant volume to 800°F?
A. 32.1 psia B. 15 psia C. 28.6 psia D. 36.4 psia
Solution:

( )( )

254. A bicycle tire has a volume of 600 cm3. It is inflated with carbon dioxide to pressure of
551.43 kPa, 20°C. How many grams of CO 2 are contained in the tire? R CO2 = 0.18896 kJ/kg.K.
A. 5.98 g B. 6.43 g C. 4.63 g D. 3.83 g
Solution:
( )

( )

255. The absolute pressure at the bottom of a vertical column of water is 15.5 psia. What is the
height of this column?
A. 22 in B. 9.2 in C. 12 in D. 9.8 in
Solution:
( )( )

256. A water temperature rise of 18°F in the water cooled condenser is equivalent to what in
°C?
A. 7.78°C B. 10°C C. 263.56 K D. -9.44°C
Solution:
( )

257. An oil storage tank contains oil with specific gravity of 0.88 and depth of 20 meters. What
is the hydrostatic pressure at the bottom of the tank in kg/cm2?
A. 1.67 B. 1.76 C. 1.56 D. 1.87
Solution:

73
( )( ) ( )( )( )

258. A vertical column of water will be supported to what height by standard atmospheric
pressure?
A. 34 ft B. 36 ft C. 24 ft D. 26 ft
Solution:
( )

259. The specific weight of liquid is 60 lb/ft3. What is the equivalent to kN/m3?
A. 9.334 B. 9.249 C. 9.643 D. 9.420
Solution:
( )
( )( )[ ( )
]

260. A cylinder weighs 150 lbf. Its cross-sectional area is 40 square inches. When the cylinder
stands vertically on one end, what pressure does the cylinder exert on the floor?
A. 14.1 kPa B. 58.2 kPa C. 0.258 bar D. 0.141 bar
Solution:
( )( )

261. What is the absolute pressure exerted on the surface of a submarine cruising 300 ft below
the free surface of the sea? Assume specific gravity of sea water is 1.03.
A. 133.9 psia B. 148.6 psia C. 100.7 psia D. 103.7 psia
Solution:
( )( )

262. Air enters a nozzle steadily at 2.21 kg/m3 and 30 m/s. What is the mass flow rate through
the nozzle if the inlet area of the nozzle is 80 cm2?
A. 0.35 kg/s B. 3.5 kg/s C. 5.3 kg/s D. 0.53 kg/s
Solution:
( )( )( )

263. What is the work required to accelerate an 800-kg car from rest to 100 km/h on a level
road?
A. 308.6 kJ B. 806.3 kJ C. 608.3 kJ D. 386 kJ

74
Solution:
( )[ ( )( ] [ ( )]
)

264. Assuming that there are no heat effects and no friction effects, find the speed of a 3220-
lbm body after it falls 778 ft from rest.
A. 422 ft/s B. 424 ft/s C. 224 ft/s D. 424 ft/s
Solution:
[ ( )( )]

265. What is the flow rate through a pipe 4 inches in diameter carrying water at a velocity of 11
ft/s?
A. 430.84 gpm B. 7.18 gpm C. 340.28 gpm D. 39.16 gpm
Solution:
( ) ( ) ( )( )

266. If the specific weight of a liquid is 58.5 lbf per cubic foot, what is the specific volume of the
liquid in cm3/g?
A. 0.5321 cm3/g B. 0.6748 cm3/g C. 0.9504 cm3/g D. 1.0675 cm3/g
Solution:

( )
( ) ( ) [( ) ]

267. What is the resulting pressure when one pound of air at 0.3 psig and 200°F is heated at
constant volume to 800°F?
A. 0.572 psig B. 28.6 psia C. 7.857 psia D. 1.2 psig
Solution:

268. A small pump serving as model, when tested in laboratory using water at 3600 rpm,
delivered 30 cfs at a head of 125 ft. If the efficiency of this model pump is 84%, predict the
75
horsepower input to the prototype pump if it is to develop the same head as model pump and
the model pump has a scale ratio of 1:10. Assume the efficiency of the prototype pump is 90%.
A. 50.6 hp B. 4730 hp C. 3740 hp D. 60.5 hp
Solution:
( )( )
( )

( ) ( ) ( ) ( )

( ) ( )( )

( )( )
( )

269. Pump at is best efficiency point (BEP) has a capacity of 10,500 gpm while developing a
head of 60 ft at a rotative speed of 1450 rpm. What is the specific speed of the pump?
A. 2760 B. 1450 C. 2476 D. 6892
Solution:
( )( ) ( )
( )

270. A pump will be installed below the reservoir water surface with a required net positive
suction head (NPSHR) of 50 ft. The barometric pressure is 14.3 psia, and the vapor pressure is
0.5 psia. Assume friction losses in the intake piping are 5 ft. Find the maximum allowable
elevation of the pump relative to the water surface intake to avoid cavitation.
A. 45 ft B. 55 ft C. 18.2 ft D. 23.2 ft
Solution:
( )

( )

( )
( )
( )
( ) ( )
( ) ( ) ( )

271. A centrifugal pump is at best efficiency point (BEP). Assume the pump characteristic are
head, h = 7 m, flow rate, Q = 19 liters/s, and rotative speed n = 1170 rpm. Find the specific
speed in SI units.
76
A. 0.4 B. 0.71 C. 10.41 D. 3.94
Solution:
( )( )

( ) ( )
( )

( )
( ) ( )

272. The pressure of a confined gas at a constant temperature is tripled, what will happen to
the volume?
A. The volume will be tripled
B. The volume will be reduced to one-third of its original value
C. The volume will remain unchanged
D. The volume is constant
Solution:

273. A 15 in. diameter fan operates at 1600 rpm and develops a head of 6 in. of water and
delivers 120 cfm. What volumetric capacity for geometrically similar fan will develop 6 in of
water at 1300 rpm?
A. 147.7 cfm B. 181.8 cfm C. 97.5 cfm D. 79.2cfm
Solution:

( )

( ) ( )

( )

274. A radial-flow pump operating at maximum efficiency at a specific speed of 2500 is to


deliver 260 gpm against a head of 129 ft at a rotative speed of 2100 rpm. Find the required
number of stages (i.e., impellers).
A. 2 stages B. 3 stages C. 4 stages D. 5 stages
77
Solution:
( )

275. How many identical turbines, operating at 139.0 rpm and 91% efficiency (specific speed =
5.4), are needed to exploit a head of 1200 ft and a flow of 1660 ft 3/s?
A. 2 turbines B. 3 turbines C. 4 turbines D. 5 turbines
Solution:
( )

( )( )( )

( )

276. How many poles should a 60-Hz generator have, if it is connected to a turbine operating
under a design head of 3000 ft with a flow of 82 cfs? Assume turbine specific speed and
efficiency 3 and 84 percent respectively.
A.10-pole B. 12-pole C. 14-pole D. 16-pole
Solution:
( )

( )( )( )

( )

( )
-

277. It is proposed to build a dam in a river where the flow rate is 10 m3/s and a 32-m drop in
elevation can be achieved for flow through a turbine. If a turbine is 82 percent efficient, what is
the maximum power that can be achieved? Specific gravity of river is 0.998.
A. 2570 kW B. 3133 kW C. 3820 kW D. 262 kW
Solution:
( )[( )( )]( )

278. What type of turbine delivers 25,000 bhp at 500 rpm under a net head of 5350 ft?
A. Impulse turbine B. Francis turbine C. Kaplan turbine D. Propeller turbine

78
Solution:
( ) ( )

( )

279. A 26-hp pump delivers 475 gpm of gasoline (γ = 42.5 lb/ ft3) at 20 C with 78% efficiency.
What pressure rise result across the pump?
A. 30.2 psi B. 32.7 psi C. 120.3 psi D. 73.2 psi
Solution:
( )
( )

280. A model pump delivering water at 180°F (γ = 60.6 lb/ ft3; pvapor = 7.54 psia) at 900 gpm and
2500 rpm begins to cavitate when the inlet pressure and velocity are 13 psia and 22 fps. Find
the required NPSH of a prototype which is 4 times larger and runs at 1100 rpm.
A. 63.5 ft B. 20.49 ft C. 6.61 ft D. 36 ft
Solution:

[ ( ) ( )]
( )

( ) ( ) ( ) ( )

281. The diameter of the discharge pipe is 8 in. and that of the intake pipe is 10 in. The pressure
gage at discharge reads 32 psi, and vacuum gage at the intake reads 12 in. Hg. If the discharge
flow rate = 4.0 ft3/s of water and the brake horsepower is 49.0, find the efficiency. The intake
and the discharge are at the same elevation.
A. 82.2% B. 80.9% C. 55.8% D. 58.46%
Solution:

( ) ( )
( ) ( )

( ) ( ) (- )( )( )
( ) ( ) ( )

( )( )
( )

79
282. A piston positive-displacement pump (PDP) has a 6-in diameter and a 2.5-in stroke. Its
crankshaft rotates at 300 rpm. Calculate its output at 94 percent volumetric efficiency.
A. 12.27 cfm B. 13.5 cfm C. 10 cfm D. 11.53 cfm
Solution:
( )
( )( )( )

283. A centrifugal pump (efficiency 88%) lifts water through a total height of 40 m from a
reservoir to discharge. Pumping is through 300 m of 75 mm diameter pipe at the rate of 20
liter/s. If pipe friction, f = 0.025, what horsepower is required?
A. 28.4 kW B. 32.2 kW C. 25kW D. 9kW
Solution:
( )
( )

( )[ ( )
]
( )( )

284. In order to predict the behaviour of a small oil pump, tests are to be made on a model
using air. The pump is to be driven by a 1/20-hp motor at 1800 rpm and a ¼-hp motor is
available to drive the air at 600 rpm. Using specific gravity of oil at 0.912 and density of air
constant at 0.076 lb/ ft3, what size model should be built?
A. The model should be 2 times as large as the oil pump.
B. The model should be 5 times as large as the oil pump.
C. The model should be 8 times as large as the oil pump.
D. The model should be 10 times as large as the oil pump.
Solution:
( ) ( )

( )( )( ) ( )( )

285. A double-overhung impulse turbine installation is to develop 20,000 hp at 275 rpm under a
net head of 1100 ft. Determine the specific speed.
A. 4.34 B. 6.14 C. 203.61 D. 144
Solution:
80
( )

286. An impulse wheel at best produces 125 hp under a head of 210 ft. By what percent should
the speed be increased for a 290-ft head?
A. 82.25% B. 17.5% C. 72.41% D. 27.59%
Solution:
( )

( ) ( )

287. What is the power ratio of a pump and its 1/5 scale model if the ratio of heads is 4 to 1?
A. 20 B. 200 C. 12.5 D. 125
Solution:

( ) ( ) ( ) ( )

( ) ( ) ( ) ( )

288. The speed of a centrifugal pump is doubled. By what factor does the pump head change?
A. 0.125 B. 0.25 C. 4 D. 8
Solution:
( )

( ) ( )

289. Compute the specific volume of an air-vapor mixture in cubic meter per kg of dry air when
the following conditions prevail: t = 40°C, w = 0.015 kg/kg, and Pt = 90 kPa.
A. 0.99 m3/kg B. 0.89 m3/kg C. 0.79 m3/kg D. 0.69 m3/kg
Solution:

81
( )

290. A coil has an inlet temperature of 70°F and outlet of 80°F. If the mean temperature of the
coil is 130°F, find the bypass factor of the coil.
A. 0.28 B. 1.2 C. 0.82 D. 0.83
Solution:

291. Compute the pressure drop of 35°C air flowing with a mean velocity of 5m/s in a circular
sheet-metal duct 400mm in diameter and 25 m long. Use friction factor, f = 0.04, and ρair =
1.3799 kg/m3.
A. 431.22 Pa B. 221.34 Pa C. 312.24 Pa D. 422.31 Pa
Solution:
( )( ) ( )
( )

292. Pressure difference of 400 Pa is available to force 20°C air through a circular sheet-metal
duct 450 mm in diameter and 25 m long. At 20°C, ρ = 1.204 kg/m3 and take friction factor, f =
0.016. Determine the velocity.
A. 27.34 ft/s B. 43.72ft/s C. 89.68 ft/s D. 86.98 ft/s
Solution:

( ) ( )
( )

293. A rectangular duct has a dimension of 0.25 m by 2 m. Determine the equivalent diameter
of the duct.
A. 0.50 m B. 0.60 m C. 0.70 m D. 0.40 m
Solution:
( )( )

82
294. To what height will a barometer column rise if the atmospheric conditions are 13.9 psia
and 68°F and barometer fluid is mercury?
A. 3.56 ft B. 5.36 ft C. 2.36 ft D. 3.26 ft
Solution:

295. To what height will a barometer column rise if the atmospheric conditions are 13.9 psia
and 68°F and barometer fluid is ethyl alcohol? Note: @ 68°F; Pv =138.5 lbf/ft3 and specific
gravity of 0.79 for ethyl alcohol:
A. 79.37 in B. 37.79 in C. 353.54 in D. 453.53 in
Solution:
( )
( )

296. What is the pressure 7000 ft below the water surface of the ocean? Neglect
compressibility.
A. 512,000 psf B. 324,500 psf C. 447,000 psf D. 213,000 psf
Solution:
( ) ( )( ) ( )

297. If atmospheric air 14.7 psia and 60°F at sea level, what is the pressure at 14212 ft altitude
if air is incompressible? Note: @ 60°F, the density of air is 0.0763 lb m/ft3; P1 = 14.7 psia.
A. 5.4674 psia B. 7.5304 psia C. 7.1696 psia D. 7.1966 psia
Solution:

( )( )

298. Water (ρ = 62.4 lbm/ft3) is flowing through a pipe. A pitot-static gage registers 3.0 inches of
mercury. What is the velocity of the water in the pipe? Note: ρ Hg = 848.6 lbm/ft3.
A. 14.7 ft/s B. 41.7 ft/s C. 71.4 ft/s D. 74.1 ft/s
Solution:
( ) ( )( )( )
√ √

83
299. The mass of an outside air at 50°C in an air conditioning unit is 60 kg. Find the temperature
after mixing if the outside air mixed with 40 kg with recirculated air at 35°C.
A. 44°C B. 39°C C. 52°C D. 47°C
Solution:

( ) ( ) ( )

300. A creamery must cool 20,000 liters of milk received each day from an initial temperature
of 29°C to a final temperature of 2°C in 5 hours. If refrigeration losses amount to 10 percent of
the cooling load, what must be the capacity of their refrigerating machine? Note: specific heat
of milk is 3.9 kJ/kg-K and SG = 1.05.
A. 38.5 TOR B. 36.5 TOR C. 37.5 TOR D. 39.5 TOR
Solution:

̇ ̇
̇

̇ ( )
( )

̇ [ ( )( )]

301. How many tons of refrigeration is required to produce 10 metric tons of ice per day at -
10°C from raw water at 22°C if miscellaneous losses are 15% of the chilling and freezing load?
A. 17 TOR B. 20 TOR C. 15 TOR D. 24 TOR
Solution:

̇ ̇ ̇ ̇ ̇

̇ ̇
̇ ( )( )( )
( ) -

̇ ( )( )
( )

̇ ( )( )( )
( ) -

̇ ( )
̇

84
302. Five hundred kilograms of poultry enter a chiller at 8°C and are frozen and chilled to a final
temperature of 18°C for storage in 15 hours. The specific heat above and below freezing are
3.18 kJ/kg-°C and 1.55 kJ/kg-°C respectively. The latent heat is 246 kJ/kg and the freezing
temperature is -5°C. Compute the product load.
A. 2.75 kW B. 2.85 kW C. 2.95 kW D. 3.15 kW
Solution:
̇ ̇ ̇ ̇
̇ ( )( )( )
( ) -

̇ ( )( )
( )

̇ ( )( ) (- )
( ) -

303. Fish weighing 11,000 kg with a temperature of 20°C is brought to a cold storage and which
shall be cooled to -10°C in 11 hours. Find the required plant refrigerating capacity in tons of
refrigeration if the specific heat of fish is 0.7 kCal/kg-°C above freezing point and 0.30 kCal/kg-
°C below freezing point. The freezing point is -3°C. The latent heat of freezing is 55.5 kCal/kg.
A. 25.26 TOR B. 15.26 TOR C. 14.38 TOR D. 24.38 TOR
Solution:
̇ ̇ ̇ ̇
̇ ( )( )( )
( ) -

̇ ( )( )
( )

̇ ( )( ) (- )
( ) -

304. The power requirement of a Carnot refrigerator in maintaining a low temperature region
at 300 K is 1.5 kW per ton. Find the heat rejected.
A. 4.02 kW B. 7.02 kW C. 5.02 kW D. 6.02 kW
Solution:
̇

85
̇
̇

̇ ( )

305. A vapor compression refrigeration system is designed to have a capacity of 150 tons of
refrigeration. It produces chilled water from 22°C to 2°C. Its actual coefficient of performance is
5.86 and 35% of the power supplied to the compressor is lost in the form of friction and
cylinder cooling losses. Determine the condenser cooling water required for a temperature rise
of 10°C.
A. 14.75 kg/s B. 15.65 kg/s C. 18.65 kg/s D. 13.75 kg/s
Solution:

̇ ̇ ̇

( )

̇
̇ ̇ ( )( )

306. Determine the heat extracted from 2000 kg of water from 25°C to ice at -10°C.
A. 621,150 kJ B. 721,150 kJ C. 821,150 kJ D. 921,150 kJ
Solution:

( )( )
( )

86
( )( )

307. A single acting, twin cylinder, Ammonia compressor with bore equal to stroke is driven by
an engine at 250 rpm. The machine is installed in a chilling plant to produce 700 kW of
refrigeration at -18°C evaporating temperature. At this temperature the cooling effect per kg
mass is 1160 kJ. The specific volume of vapor entering the compressor is 0.592 m3 per kilogram.
Assume 85% volumetric efficiency, determine the bore in mm.
A. 400 mm B. 300 mm C. 450 mm D. 500 mm
Solution:
( )( )

( )

( ) ( )

308. An iron block weighs 7 Newtons and has a volume of 200 cubic centimeters. What is the
density the block?
A. 3465 kg/m3 B. 3565 kg/m3 C. 1255 kg/m3 D. 2550 kg/m3
Solution:
- ( )
( )[ ( )
]

309. If the density of the gas is 0.003 slugs per cubic foot, what is the specific weight of the gas?
A. 9.04 N/m3 B. 15.2 N/m3 C. 76.3 N/m3 D. 98.2 N/m3
Solution:
-
( )
( ) ( )( ) [( )
]

87
310. The specific gravity of mercury relative to water is 13.55. What is the specific weight of
mercury? (The specific weight of water is 62.4 lbf per cubic foot.)
A. 82.2 kN/m3 B. 102.3 kN/m3 C. 132.9 kN/m3 D. 150.9 kN/m3
Solution:
( )
( )( )[( )
]

311. If the specific weight of a liquid is 58.5 lbf per cubic foot, what is the specific volume of the
liquid in cm3/g?
A. 0.5321 cm3/g B. 0.6748 cm3/g C. 0.9504 cm3/g D. 1.0675 cm3/g
Solution:

( ) ( )
[ ( )
][ ( )
]( )( )

312. Which of the following is not a unit of pressure?


A. Pa B. bars C. kg/m-s 2 D. kg/m2
Solution:
-

313. A cylinder weighs 150 lbf. Its cross-sectional area is 40 square inches. When the cylinder
stands vertically on one end, what pressure does the cylinder exert on the floor?
A. 14.1 kPa B. 25.8 kPa C. 63.2 kPa D. 89.7 kPa
Solution:
( )

314. What pressure is a column of water 100 centimeters high equivalent to?
A. 9810 dyne/cm2 B. 9810 N/m2 C. 0.1 bars D. 0.1 atm
Solution:
( )( )( )( )( )

88
315. Water is flowing in a pipe with a radius of 10” at a velocity of 5 m/s. At the temperature in
the pipe, the density and viscosity of the water are as follows:
ρ = 997.9 kg/m3 and μ = 1.131 Pa-s.
What is the Reynold's number for this situation?
A. 44.1 B. 88.2 C. 1140 D. 2241
Solution:
( ) ( )[ ( )]( ) ( )( )

-
- ( )( )

316. How long must a current of 5.0 amperes pass through a 10 ohm resistor until a charge of
1200 coulombs passes through?
A. 1 min B. 2 min C. 3 min D. 4 min
Solution:
-
( )

317. A car moving at 70 km/hr has a mass of 1700 kg. What force is necessary to decelerate it
at a rate of 40 cm/s 2?
A. 0.680 N B. 42.5 N C. 680 N D. 4250 N
Solution:
( )

318. One hundred milliliters of water in a plastic bag of negligible mass is to be catapulted
upwards with an initial acceleration of 20.0 m/s 2. What force is necessary to do this? Assume
gravity is 9.81 m/s 2 and the density of the water is 1 g/cm3.
A. 2.00 N B. 2.98 N C. 15.0 N D. 2.00 kN
Solution:
( )

[ ( ) ] [( ) ] ( )( )( )
-

319. A boy pulls a sled with a mass of 20 kg horizontally over a surface with a coefficient of
friction of 0.20. It takes him 10 minutes to pull the sled 100 yards. What is his average power
output over these 10 minutes?
A. 4 W B. 6 W C. 8 W D. 10 W
Solution:

89
[ ( )( )]( )
( )( )

320. A force of 200 lbf acts on a block at an angle of 28° with respect to horizontal. The block is
pushed 2 feet horizontally. What is the work done by this force?
A. 215 J B. 320 J C. 480 J D. 540 J
Solution:
( ) ( ) ( ) ( )( )

321. Two particles collide, stick together and continue their motion together. Each particle has
a mass of 10 g and their respective velocities before the collision were 10 m/s and 100 m/s.
What is the energy of the system after the collision?
A. 21.8 J B. 30.2 J C. 42.8 J D. 77.9 J
Solution:
( )
( ) ( ) [( ) ]

( ) [( ) ]( ) ( )

322. A copper bar is 90 centimeters long at 86°F. What is the increase in its length when the bar
is heated to 95°F? The linear expansion coefficient for copper, α, is 1.7 x 10-5/°C.
A. 2.12 x 10 -5 m B. 3.22 x 10 -5 m C. 5.25 x 10 -5 m D. 7.65 x 10 -5 m
Solution:
( )[( ) ] ( )( )
-

323. Calculate the energy transfer rate across a 6" wall of firebrick with a temperature
difference across the wall of 50°C. The thermal conductivity of firebrick is 0.65 BTU/hr-ft-°F at
the temperature of interest.
A. 112 W/m2 B. 285 W/m2 C. 369 W/m2 D. 429 W/m2
Solution:
( )
( )( )( )( ){ ( }
- -
)
[ ( )
]

90
324. A house has brick walls 15 millimeters thick. On a cold winter day, the temperature of the
inner and outer layers of the walls are measured and found to be 20°C and -12°C, respectively.
If there is 120 m2 of exterior wall of race, and the thermal conductivity of bricks is 0.711 J/m-s-
°C, how much heat is lost through the walls per hour?
A. 182 J B. 12.5 kJ C. 655 kJ D. 655 MJ
Solution:
( ){ [ (- )] }
( )( )
- -

325. If a horsepower pump runs for 20 minutes, what is the energy used?
A. 0.06 ergs B. 0.25 kW C. 0.30 MJ D. 0.11 kW-h
Solution:

( ) ( )( )( )

326. A power of 6 kW is supplied to the motor of a crane. The motor has an efficiency of 90%.
With what constant speed does the crane lift the 800 lbf weight?
A. 0.09 m/s B. 0.32 m/s C. 0.98 m/s D. 1.52 m/s
Solution:

( )
( ) ( )

327. An engine has an efficiency of 26%. It uses 2 gallons of gasoline per hour. Gasoline has a
heating value of 20,500 BTU/lbm and a specific gravity of 0.8. What is the power output of the
engine?
A. 0.33 kW B. 20.8 kW C. 26.0 kW D. 41.7 kW
Solution:

̇ ̇ ( )( ) ( )( )( )

( )( )
( )

91
328. Two liters of an ideal gas at a temperature of T 1 = 25°C and a pressure of P1 = 0.101 MPa,
are in a 10 cm diameter cylinder with a piston at one end. The piston is depressed so that the
cylinder is shortened by 10 centimeters. The temperature increases by 2°C. What is the change
in pressure?
A. 0.156 MPa B. 0.167 MPa C. 0.251 MPa D. 0.327 MPa
Solution:

( )( ) { [ ( ) ( )] }
( ) ( )

329. The average power output of a cylinder in a combustion engine is given by:
̅
where:
p = average pressure on the piston during the stroke
L = length of the piston stroke
A = area of the piston head
N = number of strokes per second
An 8-ylinder engine has the following specifications
p = 283 kPa
L = 14 cm
d = diameter of piston head = 12 cm
N = 1500 strokes/min
What is the average power output of this engine?
A. 89.5 N/s B. 89.5 kW C. 89.5 x 103 J-m/s D. 89.5 kJ
Solution:
̅
( )
( )( )[ ( ) ]( ) ( )
( )

330. What is the power required to transfer 97,000 coulombs of charge through a potential rise
of 50 volts in one hour?
A. 0.5 kW B. 0.9 kW C. 1.3 kW D. 2.8 kW
Solution:

92
( - )
( )

331. A current of 7 amperes passes through a 12 ohm resistor. What is the power dissipated in
the resistor?
A. 84 w B. 0.59 hp C. 0.79 hp D. 7 hp
Solution:
( )( )

332. What is the pressure at point A in the tank if h = 2 feet? (g = 32.2 ft/s 2 and ρ = 1.94 slug/ft3)
A. 75 lbf/ft2 B. 85 lbf/ft2 C. 100 lbf/ft2 D. 125 lbf/ft2
Solution:
-

( )( ) ( )

333. Determine the average velocity through a circular section in which the velocity distribution
is given by [ ( ) ]. The distribution is symmetric with respect to the longitudinal
axis, r = 0, ro is the outer radius, vmax is the velocity along the longitudinal axis. Assume flow is
laminar.
A. B. C. D.
Solution:

334. A pipe has a diameter 4” at section AA, and a diameter of 2” at section BB. For an ideal
fluid flow, the velocity given is 1 ft/s at section AA. What is the flow velocity at section BB?
A. 4 ft/s B. 0.5 ft/s C. 1.0 ft/s D. 2.0 ft/s
Solution:

( ) ( ) ( )

335. A mixing tank mixes two inlet streams containing salt. The salt concentration in stream 1 is
5% by weight, at stream 2 it is 15% by weight. Stream 1 flows at 25 kg/s, and stream 2 flows at
10 kg/s. There is only one exit stream. Find the salt concentration in the exit stream.

93
A. 5% B. 8% C. 11% D. 13%
Solution:

( ) ( ) ( )

336. Water is pumped at 1 m3/s to an elevation 5 meters through a flexible hose using a 100%
efficient pump rated at 100 kilowatts. Using the same length of hose, what size motor is needed
to pump 1 m3/s of water to the tank, with no elevation gain? In both cases , both ends of the
hose are at the same temperature and pressure. Neglect kinetic energy effects.
A. 51 kW B. 22 kW C. 36 kW D. 43 kW
Solution:

( )

( )( )

337. A fluid with kinetic viscosity of 2.5 x 10 -5 ft2/s is flowing at 0.1 ft/s from an orifice 3” in
diameter. How can the fluid be described?
A. The fluid is completely turbulent.
B. The fluid is in transition zone.
C. The fluid is laminar.
D. Turbulent cannot be calculated, it must be measured.
Solution:
( )
-
( )

338. The Reynold’s number of a sphere falling in air is 1x106. If the sphere’s radius is 1 ft, what
is its velocity? (ρ = 0.00234 slug/ft3, μair = 3.8x10-7 lbf-s/ft2).
94
A. 2.5 ft/s B. 5.1 ft/s C. 40.6 ft/s D. 81.2 ft/s
Solution:

[ ( ) ]( )

- -

339. The flow rate of water through a cast iron is 5000 gallons per minute. The diameter of the
pipe is 1 foot, and the coefficient of friction is f = 0.0173. What is the pressure drop over a 100
foot length of pipe?
A. 21.078 lbf/ft2 B. 23.78 lbf/ft2 C. 337.26 lbf/in2 D. 337.26 lbf/ft2
Solution:

( )
( )( ) [( ]( ) ( )
)

( )
( )[ ]
( )( )

340. A cylindrical flash tank mounted with its axis horizontal is used to separate liquid ammonia
from ammonia vapor. The ammonia bubbles through the liquid with 70 m3/min leaving the
disengaging surface. The disengaging rate is limited to 60 m/min and the liquid level is to
operate with the liquid level one-third to the diameter from the top. Determine the diameter if
the tank is 1.5 m long.
A. 830 mm B. 730 mm C. 860 mm D. 760 mm
Solution:

95
( ) ( )

(√ )

(√ )( )

341. A 150 Hp motor is used to drive the compressor. If the heat loss from the compressor is 25
kW and the mass flow rate of the refrigerant entering the compressor is 0.50 kg/s, determine
the difference of the enthalpies between the inlet and outlet of the compressor.
A. 143.80 kJ/kg B. 153.80 kJ/kg C. 173.80 kJ/kg D. 183.80 kJ/kg
Solution:

( )

( )

342. To cool farm products, 300 kg of ice at -4.4°C are placed in bunker. Twenty four hours later
the ice has melted into water at 7.2°C. What is the average rate of cooling provided by the ice
in kJ/hr?
A. 2679.28 kJ/hr B. 5679.28 kJ/hr C. 3679.28 kJ/hr D. 4679.28 kJ/hr
Solution:
̇ ̇ ̇ ̇
̇ [ ( ) ( )]
̇

343. Determine the estimated condenser load for an open-type compressor having a cooling
capacity of 16,500 Btu/hr and a heat rejection factor of 1.32.
A. 22,280 Btu/hr B. 20,780 Btu/hr C. 21,780 Btu/hr D. 19,780 Btu/hr
Solution:

( )
96
344. If the load on water-cooled condenser is 150,000 Btu/hr and the temperature rise of the
water in the condenser is 10°F, what is the quantity of water circulated in gpm?
A. 30 GPM B. 40 GPM C. 20 GPM D. 50 GPM
Solution:
̇ ̇
̇ ( )( )
̇

345. The load on a water-cooled condenser is 90,000 Btu/hr. if the quantity of water circulated
through the condenser is 15 gpm, determine the temperature rise of the water in the
condenser.
A. 12°F B. 14°F C. 16°F D. 18°F
Solution:
̇ ̇
̇ ( )( )
-

̇
( )( )

( )

346. The weight of ammonia circulated in a machine is found to be 21.8 lb/hr. if the vapor
enters the compressor with a specific volume of 9.6 ft3/lb, calculate the piston displacement,
assuming 80% percent volume efficiency.
A. 261.6 ft3/hr B. 271.6 ft3/hr C. 281.8 ft3/hr D. 291.6 ft3/hr
Solution:

( )

97
347. A single-stage ammonia compressor is producing 10 tons of refrigeration and the power
consumed is 15 Hp. Suction pressure is 25 psi, condensing pressure is 180 psi. Brine
temperature is 20°F off brine cooler. Determine the actual coefficient of performance.
A. 10.14 B. 11.14 C. 12.14 D. 13.14
Solution:
( )
( )

348. In an ammonia condensing machine (compressor plus condenser) the water used for
condensing is 55°F and the evaporator is at 15°F. Calculate the ideal COP.
A. 11.875 B. 12.875 C. 10.875 D. 13.875
Solution:

349. How much refrigeration capacity is required to cool 2000 cfm of air from 85°F to 70°F?
A. 2.7 TOR B. 3.7 TOR C. 1.7 TOR D. 4.7 TOR
Solution:
̇ ̇

-
̇ ( )( )

̇ ( )( ) ( )( )

350. Determine the coil face area required to maintain a face velocity 400 ft/min if the air flow
rate over the coil is 2100 ft3/min.
A. 3.25 ft2 B. 4.25 ft2 C. 5.25 ft2 D. 6.25 ft2
Solution:

98
351. Calculate the heat transfer per hour through a solid brick wall 6 m long, 2.9 m high, and
225 mm thick, when the outer surface is at 5°C and the inner surface 17°C, the coefficient of
thermal conductivity of the brick being 0.6 W/m-K.
A. 2,004.48 kJ B. 3,004.48 kJ C. 2,400.48 kJ D. 3,400.48 kJ
Solution:
[( )( )]( )

( )( )

352. A vertical furnace wall is made up of an inner wall of firebrick 20 cm thick followed by
insulating brick 15 cm thick and an outer wall of steel 1 cm thick. The surface temperature of
the wall adjacent to the combustion chamber is 1200°C while that of the outer surface of steel
is 50°C. The thermal conductivities of the wall material in W/m-K are: firebrick, 10; insulating
brick, 0.26; and steel, 45. Neglecting the film resistances and contact resistance of joints,
determine the heat loss per sq.m. of wall area.
A. 1.93 W/m2 B. 2.93 W/m2 C. 1.55 W/m2 D. 2.55 W/m2
Solution:
̇

353. A composite wall is made up of an external thickness of brickwork 110 mm thick inside
which is a layer of fiberglass 75 mm thick. The fiberglass is faced internally by an insulating
board 25 mm thick. The coefficients of thermal conductivity are as follow:
Brickwork 1.5 W/m-K
Fiberglass 0.04 W/m-K
Insulating board 0.06 W/m-K
The surface transfer coefficients of the inside wall is 3.1 W/m2-K while that of the outside wall is
2.5 W/m2-K. Take the internal ambient temperature as 10°C and the external temperature is
27°C. Determine the heat loss through such wall 6m high and 10 m long.
A. 330.10 W B. 230.10 W C. 430.10 W D. 530.10 W
Solution:
̇

99
-

[ ( ) ]( )
̇

354. One insulated wall of a cold-storage compartment is 8 m long by 2.5 m high and consists of
an outer steel plate 18 mm thick. An inner wood wall is 22.5 m thick. The steel and wood are 90
mm apart to form a cavity which is filled with cork. If the temperature drop across the extreme
faces of the composite wall is 15°C. Calculate the heat transfer per hour through the wall and
the temperature drop across the thickness of the cork. Take the coefficients of thermal
conductivity for steel, cork and wood as 45, 0.045, and 0.18 W/m-K respectively.
A. 408.24 kJ, 12.12°C C. 608.24 kJ, 13.12°C
B. 708.24 kJ, 11.12°C D. 508.24 kJ, 14.12°C
Solution:
̇

[ ( ) ]( )
̇

( )
̇

355. A cubical tank of 2 m sides is constructed of metal plate 12 mm and contains water at
75°C. The surrounding air temperature is 16°C. Calculate the overall heat transfer coefficient
from water to air. Take the coefficient of thermal conductivity of the metal as 48 W/m-K, the
coefficient of heat transfer of water is 2.5 kW/m2-K and the coefficient of heat transfer of the
air is 16 W/m2-K.
A. 15.84 W/m2-°C B.14.84 W/m2-°C C. 16.84 W/m2-°C D. 13.84 W/m2-°C
Solution:

100
-

356. A cold storage compartment is 4.5 m long by 4 m wide by 2.5 m high. The four walls,
ceiling and floor are covered to a thickness of 150 mm with insulating material which has a
coefficient of thermal conductivity of 5.8x10 -2W/m-K. Calculate the quantity of heat leaking
through the insulation per hour when the outside and inside face temperature of the material is
15°C and -5°C respectively.
A. 2185.44 kJ B. 2285.44 kJ C. 3185.44 kJ D. 4185.44 kJ
Solution:
̇

[ ( ) ( ) ( )]

-
( )( )( )
̇

357. A furnace wall consists of 35 cm firebrick (k = 1.557 W/m-K), 12 cm insulating refractory (k


= 0.346) and 20 cm common brick (k = 0.692) covered with 7 cm steel plate (k = 45). The
temperature at the inner surface of the firebrick is 1,230°C and at the outer face of the steel
plate is 60°C. Atmosphere is 27°C. What is the value of the combined coefficient for convection
and radiation from the outside wall?
A. 31.13 W/m2-K B. 30.13 W/m2-K C. 41.3 W/m2-K D. 40.13 W/m2-K
Solution:
̇

̇ ( )

̇ ̇

101
358. Hot gases at 280°C flow on one side of the metal plate 10 mm thickness and air at 35°C
flows on the other side. The heat transfer coefficient of the gases is 31.5 W/m2-K and that of
the air is 32 W/m2-K. Calculate the over-all heat transfer coefficient.
A. 15.82 W/m2-K B. 16.82 W/m2-K C. 14.82 W/m2-K D. 17.82 W/m2-K
Solution:

359. The surface temperature of the hot side of the furnace wall is 1200°C. It is desired to
maintain the outside of the wall at 38°C. A 152 mm of refractory silica is used adjacent to the
combustion chamber and 10 mm of steel covers the outside. What thickness of insulating bricks
is necessary between refractory and steel, if the heat loss should be kept at 788 W/m 2? Use k =
13.84 W/m-K for refractory silica; 0.15 for insulating brick, and 45 for steel.
A. 220 mm B. 240 mm C. 260 mm D. 280 mm
Solution:

360. An insulated steam pipe located where the ambient temperature is 32°C, has an inside
diameter of 50 mm with 10 mm thick wall. The outside diameter of corrugated asbestos
insulation is 125 mm and the surface coefficient of still air, h o = 12 W/m2-K. Inside the pipe is
steam having a temperature of 150°C with film coefficient h i = 6000 W/m2-K. Thermal
conductivity of pipe and asbestos insulation are 45 and 0.12 W/m-K respectively. Determine the
heat loss per unit length of pipe.
A. 110 W B. 120 W C. 130 W D. 140 W
Solution:
̇

102
( ) ( )

( ) ( )
[ ( ) ]( ) ( ) ( )

[ ( ) ]( )

̇
̇

361. How many watts will be radiated from a spherical black body 15 cm in diameter at a
temperature of 800°C?
A. 5.34 kW B. 4.34 kW C. 6.34 kW D. 3.34 kW
Solution:

-
-
( )

( -
)( )( )

362. A wall with an area of 10 m2 is made of a 2 cm thickness of white pine (k = 0.133 W/m-°C)
followed by a 10 cm of brick (k = 0.649 W/m-°C). The pine is on the inside where the
temperature is 30°C while the outside temperature is 10°C. Assuming equilibrium conditions
exist, what is the temperature at the interface between the two metals?
A. 15.65 °C B. 17.64 °C C. 18.21 °C D. 19.31 °C
Solution:
̇

( )
̇

103
( ) ( )( )
̇ ̇

363. A counter-flow heat exchanger is designed to heat a fuel oil from 45°C to 100°C while the
heating fluid enters at 150°C and leaves at 115°C. Calculate the arithmetic mean temperature
difference.
A. 40°C B. 50°C C. 60°C D. 70°C
Solution:

364. With three different quantities x, y, and z of the same kind of liquid of temperatures 9, 21
and 38°C respectively, it is found that when x and y are mixed together the resultant
temperature is 17°C and when y and z are mixed together the resultant temperature is 28°C.
Find the resultant temperature if x and z were mixed.
A. 25.87°C B. 25.92°C C. 20.85°C D. 24.86°C
Solution:

( ) ( )

( ) ( )

( ) ( )
( ) ( )

104
365. The journals of a shaft are 380 mm diameter, it runs at 105 rpm and the coefficient of
friction between journals and bearings is 0.02. If the average load on the bearings is 200 kN,
find the heat generated per minute at the bearings.
A. 501.375 kJ B. 505.575 kJ C. 401.375 kJ D. 501.575 kJ
Solution:

( )
( )

[ ( )]( ) -
( )
( )( )( )
( )
̇ ( )( )( )

366. A reverse Carnot cycle requires 3 Hp and extracts energy from a lake to heat a house. If the
house is kept at 70°F and requires 2000 Btu per minute, what is the temperature of the lake?
A. 35.29°F B. 36.29°F C. 39.29°F D. 40.29°F
Solution:
̇ ( )

̇ ̇ ̇

̇
̇

̇ ( ) ( )
( )( )

̇ ( )
( )

105
367. An oxygen cylinder of volume 2.3 ft3 has a pressure of 2200 psig and is at 70°F. Determine
the mass of oxygen in the cylinder.
A. 25.66 lbs B. 26.66 lbs C. 27.66 lbs D. 28.66 lbs
Solution:

( )

( )( ) ( )( )

368. A group of 50 persons attend a secret meeting in a room which is 12 m wide by 10 m long
and a ceiling height of 3 m. The room is completely sealed off and insulated. Each person gives
off 150 kCal per hour of heat and occupies a volume of 0.20 m3. The room has an initial
pressure of 101.3 kPa and temperature of 16°C. Calculate the room temperature after 10
minutes. Use R = 0.287 kJ/kg-K and C v = 0.171 kCal/kg-K.
A. 33.1°C B. 37.7°C C. 38.7°C D. 31.7°C
Solution:

( )

( )

[ ( )( ) ( )]

( ) ( )( )

( )( )

106
369. One kilogram of wet steam at a pressure of 8 bar (υ g = 0.2404 m3/kg, υf = 0.0011148
m3/kg) and dryness of 0.94 is expanded until the pressure is 4 bar (υ g = 0.4625 m3/kg, υf =
0.0010836 m3/kg). If expansion follows the law PV n = C, where n = 1.12, find the dryness
fraction of the steam at the lower pressure.
A. 0.9072 B. 0.4197 C. 0.2260 D. 0.2404
Solution:

( ) ( )( )

( ) ( )

( ) ( )

( ) ( )( )

370. 2.5 liters of superheated steam at 25 bar and 400 C (υ = 0.1252 m3/kg) is expanded in an
engine to a pressure of 0.1 bar (υ g = 14.674 m3/kg, υf = 0.0010102 m3/kg) when its dryness
fraction is 0.9. Find the final volume of the steam.
A. 163.74 liters B. 263.74 liters C. 363.74 liters D. 463.74 liters
Solution:

( ) ( )
( )( )

371. A 1.5 kg of wet steam at a pressure of 5 bar (hf = 640 kJ/kg, hfg = 2109 kJ/kg) dryness 0.95
is blown into 70 liters of water 12°C (h = 50.4 kJ/kg). Find the final enthalpy of the mixture.
A. 74.80 kJ/kg B. 84.80 kJ/kg C. 94.80 kJ/kg D. 104.80 kJ/kg
Solution:

( ) ( )
107
[ ( )] ( )

( )

372. A 650 BHP diesel engine uses fuel oil of 28°API gravity, fuel consumption is 0.65 lb/BHP-hr.
Cost of fuel is P7.95 per liter. For continuous operation determine the minimum volume of
cubical day tank in cm3, ambient temperature is 45° C.
A. 4,372,890 cm3 B. 5,987,909 cm3 C. 5,291,880 cm3 D. 7,352,789 cm3
Solution:

( )

[ ( )]
[ ( )]
( )

373. A typical industrial fuel oil, C 16H32 with 20% excess air by weight. Assuming complete
oxidation of the fuel, calculate the actual air-fuel ratio by weight.
A. 17.56 kg air/kg fuel C. 16.75 kg air/kg fuel
B. 15.76 kg air/kg fuel D. 17.65 kg air/kg fuel
Solution:

( )

108
( ) ( )

( )( ) ( )

( ) ( )
( ) ( ) ( )

374. Fuel oil in a day tank for use of an industrial boiler is tested with hydrometer. The
hydrometer reading indicates a SG = 0.924 when the temperature of the oil in the tank is 35°C.
Calculate the higher heating value of the fuel.
A. 43,852.13 kJ/kg B. 53,852.13 kJ/kg C. 58,352.13 kJ/kg D. 48,352.13 kJ/kg
Solution:
( )

[ ( )]

( )

375. A diesel electric plant supplies energy for Meralco. During a 24 hr Period, the plant
consumed 200 gallons of fuel at 28°C and produced 3930 Kw-hr. Industrial fuel used is 28°API
and was purchased at P5.50 per liter at 15.6°C. What should the cost of fuel be produce one
kw-hr.?
A. P 1.05 B. P 1.10 C. P 1.069 D. P 1.00
Solution:

( )

[ ( )]
109
[ ( )]
( )

-
( )( )( )( ) -
-

376. A certain coal has the following ultimate analysis:


C = 70.5% H = 4.5% O2 = 6% N2 = 1.0%
S = 3.0% Ash = 11% Moisture = 4%
A stoker fired boiler of 175,000 kg/hr steaming capacity uses this coal as fuel. Calculate the
volume of air in m3/hr with air at 60°F (15.6°F) and 14.7 psia (101.325 kPa) the coal is burned
with 30% excess air. Boiler efficiency is 70% and factor of evaporation of 1.10.
A. 212,861.04 m3/hr B. 221,861.04 m3/hr C. 218,261.04 m3/hr D. 281,261.04 m3/hr
Solution:
̇( )( )
̇

( ) ( ) ( ) ( )

( )
( ) ( ) ( )

̇ ( ) ̇ ( ) ( )( )
̇ ̇ ̇ ( )
( )
̇ ( )

̇
̇ ̇ ( ) ( ) ( )( )

( )( )
̇

377. A diesel power plant consumed 1 m3 of fuel with 30°API at 27°C in 24 hrs. Calculate the
fuel rate in kg/hr.
A. 36.21 B. 26.25 C. 29.34 D. 39.42
Solution:
110
̇ ̇
( )

[ ( )]
( )

̇ ( )

378. A diesel power plant uses fuel with heating value of 43,000 kJ/kg. What is the density of
the fuel at 25°C?
A. 840 kg/m3 B. 873 kg/m3 C. 970 kg/m3 D. 940 kg/m3
Solution:

[ ( )]

( )

379. A water tube boiler has a capacity of 1000 kg/hr of steam. The factor of evaporation is 1.3,
boiler rating is 200%, boiler efficiency is 65% and heating surface area is 0.91 m 2 per bo.Hp. And
the heating value of fuel is 18,400 kCal/kg. The total coal available in the bunker is 50,000 kg.
Determine the no. of hrs. to consume the available fuel.
A. 853.36 hrs B. 706.57 hrs C. 979.46 hrs D. 100.75 hrs
Solution:

( )
111
( ) [ ( )]
[ ( )]

380. Two boilers are operating steadily on 91,000 kg of coal contained in a bunker. One boiler is
producing 1591 kg of steam per hour at 1.2 factor of evaporation and an efficiency of 65% and
another boiler produced 1364 kg of steam per hour at 1.15 factor of evaporation and an
efficiency of 60%. How many hrs will the coal in the bunker run the boilers if the heating value
of coal is 7,590 kCal/kg?
A. 230.80 hrs B. 280.54 hrs C. 350.35 hrs D. 300.54 hrs
Solution:

̇ ( )
̇
̇ [ ( )]
̇
[ ( )]
̇ [ ( )]

̇ [ ( )]
̇
[ ( )]
̇ [ ( )]

̇ ̇ ̇

381. The heating value of fuel supplied in a boiler is 40,000 kJ/kg. If the factor of evaporation is
1.10 and the actual specific evaporation is 10, what is the efficiency of the boiler?
A. 62.07% B. 53.08% C. 78.05% D. 54.97%

112
Solution:
̇ ( ) [ ( )] [ ( )]
̇

382. What is the rate of evaporation of a water tube boiler if the factor of evaporation is 1.10,
percent rating of 200% and the heating surface area is 250 m2?
A. 7,817.16 kg/hr B. 7,898.67 kg/hr C. 6,789.45 kg/hr D. 5,768.54 kg/hr
Solution:
̇ ( ) ̇ [ ( )]

̇ [ ( )]

383. Steam is admitted to the cylinder of an engine in such a manner the average pressure is
120 psi. The diameter of the piston is 10 in. and the length of a stroke is 12 in. What is the Hp of
the engine when it is making 300 rpm?
A. 171.4 Hp B. 175 Hp C. 173.2 Hp D. 174.4 Hp
Solution:

( ) ( )( )
( ) [ ]

( )( )

384. Steam enters a turbine stage with an enthalpy of 3628 kJ/kg at 70 m/s and leaves the same
stage with an enthalpy of 2846 kJ/kg and a velocity of 124 m/s. Calculate the power if there are
5 kg/s steam admitted at the turbine throttle?
A. 4597.45 kW B. 3976.55 kW C. 3883.81 kW D. 1675.42 kW
Solution:

113
̇ ̇ ( ) ̇ ( ) ( ) ( )( )
̇

385. Steam with an enthalpy of 800 kCal/kg enters a nozzle at a velocity of 80 m/s. Find the
velocity of the steam at the exit of the nozzle if its enthalpy is reduced to 750 kCal/kg, assuming
the nozzle is horizontal and disregarding heat losses. Take g = 9.81 m/s 2 and J constant = 427 kg
m/kCal.
A. 452.37 m/s B. 245.45 m/s C. 651.92 m/s D. 427.54 m/s
Solution:

( )

386. Steam is expanded through a nozzle and the enthalpy drop per kg of steam from the initial
pressure to the final pressure of 60 kJ. Neglecting friction, find the velocity of discharge and the
exit area of the nozzle to pass 0.20 kg/s if the specific volume of the steam at exit is 1.5 m 3/kg.
A. 346.4 m/s, 879 mm2 C. 765.6 m/s, 467 mm2
B. 356.7 m/s, 278 mm2 D. 346.4 m/s, 866 mm2
Solution:

√ √ ( ) √ ( )

( ) ( )

387. A 6 MW steam turbine generator power plant has a full-load steam rate of 8 kg/kW-hr.
Assuming that no-load steam consumption as 15% of full-load steam consumption, compute for
the hourly steam consumption at 75% load, in kg/hr.
A. 37,800 kg/hr B. 38,700 kg/hr C. 30,780 kg/hr D. 30,870 kg/hr
Solution:
114
-
̇ ( )
-

- ( )
̇ ( )
-

( ) ( )
( ) ( )
( ) ( )

̇
̇

( )

̇ ( )

388. A 4 kg of air enters a turbine with enthalpy of 600 kJ and velocity of 250 m/s. The enthalpy
at exit is 486 kJ and velocity of 170 m/s. What is the work developed if there is a heat loss of 10
kJ?
A. 122.83 kJ B. 171.2 kJ C. 80.2 kJ D. 28.3 kJ
Solution:
̇ ̇
[ ( )( ) ] [ ( )( )]

389. Calculate the drive horsepower for pumping 1703 L/min cold water to a tank suction at
127 mmHg vacuum, delivery at 5.3 kg/cm2 ga., both measured close to pump, e p = 0.65.
A. 31.42 Hp B. 20.42 Hp C. 35.42 Hp D. 23.02 Hp
Solution:

115
( )

- -
( ) ( )

( )( )( )
-
( -
)

390. Find the length of a suspension bunker to contain 181 tons of coal without surcharge;
width = 4.6 m, depth = 4.3 m. The level capacity of a suspension bunker is 5/8 wdL where: w =
width, d = depth and L = length. Density of coal is 800 kg/m3.
A. 18.30 m B. 15.80 m C. 17.61 m D. 12.61 m
Solution:

( )

( )( )

391. A 305 mm x 457 mm four stroke single acting diesel engine is rated at 150 kW at 260 rpm.
Fuel consumption at rated load is 0.26 kg/kW-hr with a heating value of 43,912 kJ/kg. Calculate
the brake thermal efficiency.
A. 31.63% B. 41.63% C. 21.63% D. 35.63%
Solution:

116
̇ ( )
-

( )

392. The brake thermal efficiency of a 1 MW diesel electric plant is 36%. Find the heat
generated by fuel in kW if the generator efficiency is 89%.
A. 3,121.10 kW B. 3,528.64 kW C. 4,121.10 kW D. 4,528.64 kW
Solution:

393. In an air-standard Brayton cycle, the compressor receives air at 101.325kPa, 21°C and it
leaves at 600 kPa at the rate of 4 kg/s. Determine the turbine work if the temperature of the air
entering the turbine is 1000°C.
A. 3000 kW B. 2701 kW C. 2028 kW D. 3500 kW

Solution:
̇ ̇ ( )

( )

( )

̇ ( )( )

117
394. Kerosene is the fuel of a gas turbine plant: fuel-air ratio, mf = 0.012, T3 = 972 K, pressure
ratio, rp = 4.5, exhaust to atmosphere. Find the available energy in kJ per kg air flow. Assume k =
1.34 and C p = 1.13.
A. 352.64 kJ/kg B. 452.64 kJ/kg C. 252.64 kJ/kg D. 552.64 kJ/kg

Solution:

( ) ( )

( )

( )

( )( )( )

395. An ideal gas turbine operates with a pressure ratio of 10 and the energy input in the high
temperature heat exchanger is 300 kW. Calculate the air flow for temperature limits of 30°C
and 1200°C.
A. 0.25 kg/s B. 0.34 kg/s C. 0.41 kg/s D. 0.51 kg/s
Solution:
̇ ̇ ( )

( )

̇ ( )( )
̇

396. In an air-standard Brayton cycle the inlet temperature and pressure are 20°C and 101.325
kPa. The turbine inlet conditions are 1200 kPa and 900°C. Determine the air flow if the turbine
produces 12 MW.
A. 21.41 kg/s B. 20.20 kg/s C. 19.25 kg/s D. 18.10 kg/s
Solution:

118
̇ ( )

( )

( )

̇ ( )( )
̇

397. A gas turbine power plant operating on the Brayton cycle delivers 15 MW to a standby
electric generator. What are the mass flow rate and the volume flow rate of air if the minimum
and maximum pressures are 100 kPa and 500 kPa respectively and temperature of 20°C and
1000°C?
A. 31.97 kg/s, 26.88 m3/s C. 41.97 kg/s, 26.88 m3/s
B. 36.98 kg/s, 28.99 m3/s D. 46.98kg/s, 28.99 m3/s
Solution:
̇ ̇

( )

( )

̇ ̇( )
̇ ( )( )
̇

̇ ( )( )
̇

398. In a hydraulic plant the difference in elevation between the surface of the water at intake
and the tailrace is 650 ft when the flow is 90 cfs, the friction loss in the penstock is 65 ft and the
head utilized by the turbine is 500 ft. The mechanical friction in the turbine is 110 Hp, and the
leakage loss is 4 cfs. Find the hydraulic efficiency.
A. 87.45% B. 84.57% C. 85.47% D. 78.54%
Solution:
119
399. A hydro-electric power plant consumes 60,000,000 kW-hr per year. What is the net head if
the expected flow is 1500 m3/min and over-all efficiency is 63%?
A. 34.34 m B. 43.43 m C. 44.33 m D. 33.44 m
Solution:

( )

400. A pelton type turbine has a gross head of 40 m and a friction head loss 6 m. What is the
penstock diameter if the penstock length is 90 m and the coefficient of friction head loss is
0.001 (Morse)?
A. 2040 mm B. 3120 mm C. 2440 mm D. 2320 mm
Solution:

√ √ ( )( )

( )( )( )

401. The water velocity of a 5 m x 1 m channel is 6 m/s. What is the annual energy produced if
the net head is 120 m and the over-all efficiency is 80%?
A. 494,247,258 kW-hrs C. 247,494,528 kW-hrs
B. 247,497,582 kW-hrs D. 472,497,582 kW-hrs
Solution:
( )
( )
120
( )( )
( )( )

( )( )
-

402. A hydro-electric impulse turbine is directly coupled to a 24 pole, 60 Hz alternator. It has a


specific speed of 60 rpm and develops 3000 Hp. What is the required diameter assuming a
peripheral speed ratio of 0.45?
A. 0.661 m B. 0.552 m C. 0.443 m D. 0.773 m
Solution:

( )

( )

√ ( )( )

403. In a hydroelectric power plant the tailwater elevation is at 500 m. What is the head water
elevation if the net head is 30 m and the head loss is 5% of the gross head?
A. 785.25 m B. 582.57 m C. 528.57 m D. 758.25 m
Solution:

121
404. The tailwater and headwater of a hydro-electric plant are 150 m and 200 m respectively.
What is the water power if the flow is 15 m3/s and a head loss of 10% of the gross head?
A. 6,621.75 kW B. 7,621.65 kW C. 5,621.76 kW D. 4,621.56 kW
Solution:

( )
( )( )

405. In a hydro-electric plant, water flows at 10 m/s in a penstock of 1 m3 cross-sectional area.


If the net head of the plant is 30 m and the turbine efficiency is 85%, what is the turbine
output?
A. 2,501.55 kW B. 2,100.21 kW C. 3,626.34 kW D. 3,124.65 kW
Solution:

( )
( )( )

406. A 75 MW power plant has an average load of 35,000 kW and a load factor of 65%. Find the
reserve over peak.
A. 21.15 MW B. 23.41 MW C. 25.38 MW D. 18.75 MW
Solution:

407. A power plant is said to have/had a use factor of 48.5% and a capacity factor of 42.4%.
How many hrs. did it operate during the year?
A. 6,600.32 hrs B.7,658.23 hrs C. 8,600.32 hrs D. 5,658.23 hrs
122
Solution:
-

( ) ( )

408. A 50,000 kW steam plant delivers an annual output of 238,000,000 kW-hr with a peak load
of 42,860 kW. What is the annual load factor and capacity factor?
A. 0.634, 0.534 B. 0.643, 0.534 C. 0.634, 0.543 D. 0.643, 0.534
Solution:

- -

409. Calculate the use factor of a power plant if the capacity factor is 35% and it operates 8000
hrs during the year?
A. 38.325% B. 33.825% C. 35.823% D. 32.538%
Solution:
( )
( )

410. If the air required for combustion is 20 kg per kg of coal and the boiler uses 3000 kg of coal
per hr, determine the mass of gas entering the chimney. Assume an ash loss of 15%.
A. 40,664 kg/hr B. 70,200 kg/hr C. 62,550 kg/hr D. 50,500 kg/hr
Solution:
̇ ̇ ̇ ̇
123
̇
̇
̇ ̇
̇ ̇ ̇ ̇
̇ ̇ ( )

411. A 15 kg gas enters a chimney at 10 m/s. If the temperature and pressure of a gas are 26°C
and 100 kPa respectively, what is the diameter of the chimney? Use R = 0.287 kJ/kg -K.
A. 1.57 m B. 2.65 m C. 2.22 m D. 1.28 m
Solution:

̇ ( )( )
̇

( )

412. A two-stage air compressor at 90 kPa and 20°C discharges at 700 kPa. Find the polytropic
exponent n if the intercooler intake temperature is 100°C.
A. 1.29 B. 1.33 C. 1.4 D. 1.25
Solution:

( )

√ √ ( )

( )

413. A two-stage compressor receives 0.35 kg/s of air at 100 kPa and 269 K and delivers it at
5000 kPa. Find the heat transferred in the intercooler.
A. 70.49 kW B. 80.49 kW C. 90.49 kW D. 100.49 kW
Solution:
̇ ̇ ( )

124
√ ( )

( )

( )

̇ ( )( )

414. A centrifugal pump discharged 20 L/s against a head of 17 m when the speed is 1500 rpm.
The diameter of the impeller was 30 cm and the brake horsepower was 6.0. A geometrically
similar pump 40 cm in diameter is to run at 1750 rpm. Assuming equal efficiencies, what brake
horsepower is required?
A. 51.55 HP B. 50.15 HP C. 40.14 HP D. 45.15 HP
Solution:

( ) ( )

415. A pump delivers 20 cfm of water having a density of 62 lb/ft 3. The suction and discharge
gage reads 5 in. Hg vacuum and 30 psi respectively. The discharge gage is 5 ft above the suction
gage. If pump efficiency is 70%, what is the motor power?
A. 5.31 HP B. 3.31 HP C. 4.31 HP D. 6.31 HP
Solution:

[ (- ) ( )]
( )

-
( )( ) ( -
)

416. Calculate the air power of a fan that delivers 1200 m3/min of air through a 1 m x 1.5 m
outlet. Static pressure is 120 mm WG and density of air is 1.18.

125
A. 20.45 kW B. 25.64 kW C. 30.45 kW D. 35.64 kW
Solution:

( )

[ ( )]

( )

[ ( )] ( )( )

417. Determine the temperature for which a thermometer with degrees Fahrenheit is
numerically twice the reading of the temperature in degrees Celsius.
A. -24.6 B. 320 C. 160 D. -12.3
Solution:
-
( ) -

( )

[ ( )]

418. During takeoff in a spaceship, an astronaut is subjected to acceleration equal to 5 times


the pull of the earth’s standard gravity. If the astronaut is 180 lb m and the takeoff is vertical,
what force does he exert on the seat?
A. 4810.9 N B. 4414.5 N C. 8829 N D. 9620 N
Solution:

( )
126
( ) ( )( ) ( ) ( )

419. A pressure cooker operates by cooking food at a higher pressure and temperature possible
at atmospheric conditions. Steam is contained in the sealed pot, with vent hole in the middle of
the cover, allowing steam to escape. The pressure is regulated by covering the vent hole with a
small weight, which is displaced slightly by escaping steam. Atmospheric pressure is 100 kPa,
the vent hole area is 7-mm2, and the pressure inside should be 250 kPa. What is the mass of the
weight?
A. 0.107 kg B. 1.05 kg C. 1.75 kg D. 0.1783 kg
Solution:

( )
-
( ) ( )

420. A barometer can be used to measure an airplane’s altitude by comparing the barometric
pressure at a given flying altitude to that on the ground. Determine an airplane’s altitude if the
pilot measures the barometric pressure at 700 mm-Hg, the ground reports it at 758 mm-Hg,
and the average air density is 1.19 kg/m3, g = 9.81 m/s 2.
A. 636 m B. 633 m C. 366 m D. 663 m
Solution:
( ) - ( )
-

( )
( )( )

421. A mixture of 0.4 lbm of helium and 0.2 lbm of oxygen is compressed polytropically from
14.7 psia and 60°F to 60 psia according to n = 1.4. Determine the final temperature, T 2.
A. 727.7°R B. 777.2°R C. 722.7°R D. 277.7°R
Solution:

( )

127
( ) ( )

422. A mixture of 0.4 lbm of helium and 0.2 lbm of oxygen is compressed polytropically from
14.7 psia and 60oF t0 60 psia according to n = 1.4. Determine the polytropic work.
A. 139 Btu B. 239 Btu C. 339 Btu D. 539 Btu
Solution:
- -

( )( - - )
( )( - - )
-
( ) -
- -

( ) ( - )( )

423. A pump in a municipality’s water-supply system receives water from the filtration beds
and pumps it up to the top of a water tower. The tower’s height is 35 m, and the inlet piping to
the pump is 2 m below the pump’s intake. The water temperature is 20 C, measured at both
the inlet and the discharge from the pump. The mass flow rate through the pump is 100 kg/s,
the diameter of the inlet piping is 25 cm, and the diameter of the discharge piping is 15 cm.
Determine the power required by the pump.
A. 77.3 kW B. 33.77 kW C. 34.42 kW D. 42.34 kW
Solution:
̇
̇ ( )

( ) ( )

( )
128
( )

( ) ( )
( )

̇ ( )( )

424. An adiabatic tank containing air is used to power an air turbine during times of peak power
demand. The tank has a volume of 500 m3 and contains air at 1000 kPa and 500 K. Determine
the mass remaining when the pressure reaches 100 kPa.
A. 273.37 kg B. 672.73 kg C. 772.73 k D. 227.73 kg
Solution:

( )

( )

( ) ( - )( )

425. Determine the air-fuel ratio on a molar basis for the complete combustion of octane, with
theoretical amount of air.
A. 95.5 kg air/kg fuel B. 59.9 kg air/kg fuel C.59.5 kg air/kg fuel D. 55.9 kgair/kg fuel
Solution:

( )

( )
[ ( )]
( )

426. During a steady state operation, a gearbox receives 60 kW throughout the input shaft and
delivers power through the output shaft. For the gearbox as the system, the rate of energy
transfer is by convection, h = 0.171 kW/m2-K is the heat transfer coefficient, A = 1.0 m2 is the
outer surface area of the gearbox, Tb = 300 K (27°C) is the temperature at the outer surface, T f =
293 K (20°C) is the temperature of the surroundings away from the immediate vicinity of the

129
gearbox. Determine the power delivered to the output shaft in kW if the heat transfer rate is -
1.2 kW.
A. 98.8 kW B. 78.8 kW C. 68.8 kW D. 58.8 kW
Solution:
̇ ̇ ̇ ̇
̇ ̇ ̇ ̇
̇ -
̇ -
̇ - (- )

427. A single acting air compressor with a clearance of 6% takes in air at atmospheric pressure
and temperature of 85°F and discharges it at a pressure of 85 psia. The air handled is 0.25
ft3/cycle measured at discharge pressure. If the compression is isentropic, find the piston
displacement per cycle, if the compressor is running at 750 rpm.
A. 0.0750 ft3/cycle B. 0.025 ft3/cycle C. 1.030 ft3/cycle D. 1.090 ft3/cycle
Solution:

( )

( ) ( )
( )( )
( )
( )( )
( )

( ) ( )

428. A single acting air compressor with a clearance of 6% takes in air at atmospheric pressure
and temperature of 85°F and discharges it at a pressure of 85 psia. The air handled is 0.25
ft3/cycle measured at discharge pressure. If the compression is isentropic, find the air hp of the
compressor if rpm is 750.
A. 16 hp B. 96 hp C. 69 hp D. 61 hp
Solution:
̇ ( )( )

130
̇ ( )( )( )
̇ [( ) ] [( ) ]
( )

429. A nozzle receives 0.5 kg/s of air at a pressure of 2700 kPa and a velocity of 30 m/s and with
an enthalpy of 923 kJ/kg, and the air leaves at a pressure of 700 kPa and with an enthalpy of
660 kJ/kg. Determine the exit velocity from the nozzle.
A. 923 m/s B. 726 m/s C. 700 m/s D. 660 m/s
Solution:

( ) ( )

( )

430. A two-stage, double acting compressor is to deliver 90 lb/min of air from 14.3 psia to 90°F
to a final pressure of 185 psia. The normal barometer is 29.8 in-Hg and the temperature is 80°F.
The pressure drop in the intercooler is 3 psi and the speed is 210 rpm and pV 1.34 = C during
compression and expansion. The clearance is 5% for both cylinders. Find the volume of free air
if the temperature of the cooling water increased by 18°F.
A. 1282 CFM B. 1230 CFM C. 1320 CFM D. 1822 CFM
Solution:
̇ ( )( )
̇
( )
̇
̇ ̇

̇ ( )[( ) ]
̇
( ) [ - ( )]
-

431. Consider 4800 lb of steam per hour flowing through a pipe at 100 psia pressure. Assume a
velocity of 5280 ft/min. What size of pipe is required? Specific volume of steam at 100 psia υ =
4.432 ft3/lb.
A. 3 in B. 5 in C. 4 in D. 6 in
Solution:

131
( ) ( )
( )
√ ( )

( )

432. A boiler plant generates 225,000 lb of steam and burns 13.9 tons of coal per hour. The coal
has a heating value of 11,400 Btu/lb. A test of the particulates leaving the boiler shows that
3804 lb of particulate is being discharged per hour. What is the particulate discharged per
million Btu heat input to the furnace?
A. 12 lb/106 Btu B. 14 lb/106 Btu C. 15 lb/106 Btu D. 16 lb/106 Btu
Solution:

433. A turbine receives 150 lbm/s of air at 63 psia and 2450°R and expands it polytropically to
14.7 psia. The exponent n is equal to 1.45 for the process. Determine the power.
A. 52,343.16 BTU/s C. 53.343.16 HP
B. 52,343.16 kW D. 53,343.16 ft-lb/s
Solution:

( )

( )

-
( )[( ) ]
( )
̇ -

-
( )
-

434. Find the thrust and efficiency of two 2-m diameter propellers through which flows a total
of 600 m3/s of air to 11.3 N/m3. The propellers are attached to an airplane moving at 250 kph
through still air. Neglect eddy losses.
A. 36,077 N, 73% B. 77,630 N, 37% C. 66,033 N, 33% D. 77,330 N, 77%
Solution:

132
( )
( )

( )

( )

( )( )

435. A liquid with a specific gravity of 1.26 is being pumped in a pipeline from A to B. At A, the
pipe diameter is 60 cm and the pressure is 300 kN/m2. At B, the pipe diameter is 30 cm and the
pressure is 330 kN/m2. Point B is 1.0 m lower than A. Find the flow rate if the pump puts 16 kW
into the flow. Neglect head loss.
A. 4.2 m3/s B. 0.42 m3/s C. 2.4 m3/s D. 0.24 m3/s
Solution:
( )

[ ( )
] [ ( )
]

( ) ( )
- ( ) ( )

436. A reciprocating compressor handles 1,400 cfm of air measured at intake where P1 = 18 psia
and T1 = 90°F. The discharge pressure is 92 psia. Calculate the work if the process of the
compression is isothermal.
A. -180.5 hp B. -179.5 hp C. -227.6 hp D. -228.6 hp
Solution:
( )( )
̇ ( ) -
-
( )
-

437. The fuel oil has the ff. analysis:


C = 89% N2 = 2% H2 = 8% S = 1%
With 3% excess air, what is the actual amount of air needed to burn the fuel oil?

133
A. 13.17 kg air/kg fuel C. 14.47 kg air/kg fuel
B. 13.47 kg air/kg fuel D. 14.17 kg air/kg fuel
Solution:
( )
( ) ( ) ( )
( )( )

438. A pump discharges 550 gpm of water to a height of 35 ft. With an efficiency of 80%, what
is the power input?
A. 6.09 hp B. 6.32 hp C. 4.74 hp D. 4.94 hp
Solution:
( )( )( )
-
( )
-

439. A room contains air at 20°C and 96 kPa at a relative humidity of 75%. Determine the
enthalpy of moist air.
where:
(Psat @ 20°C = 2.339 kPa)
(hg @ 20°C = 2538.1 kJ/kg da )
Given:
Unsaturated air (0% < Ø <100%)
td = 20°C
Pt = 96 kPa
Ø = 75%
A. 45.919 kJ/kg da B. 45.515 kJ/kg da C. 49.515 kJ/kg da D. 41.815 kJ/kg da
Solution:

( )

( ) ( )
( ) ( ) ( )

134
440. A piston moves inside a cylinder at a velocity of 6.0 m/s. The 160 mm diameter piston is
centrally located within the 160.2 mm inside diameter cylinder. The film of oil is separating the
piston from the cylinder has an absolute viscosity of 0.4 N-s/m2. Assuming a linear velocity
profile, find the shear stress in the oil. (T = μ (v/H))
Given:
μ = 0.40 N-s/m2
v = 5 m/s
H = thickness of oil film = 160.2 – 160/2 = 0.1 mm
A. 50,000 N/m2 B. 40,000 N/m2 C. 24,000 N/m2 D. 34,000 N/m2
Solution:
-
( )

441. A centrifugal pump with a 3 ft impeller diameter operates at 800 rpm. If the speed is to be
increased to 1200 rpm, determine the impeller diameter that should be used so that the same
shaft input power would be required.
A. 5.32 ft B. 2.35 ft C. 5.23 ft D. 2.93 ft
Solution:
( )
( )
( ) ( )

( ) ( )

442. Determine the mass of water vapor contained in a 150 m3 room at 100 kPa, 23°C and 40%
relative humidity. From Steam Tables: Psat @ 23°C = 2.810 kPa.
A. 1.6342 kg B. 1.9342 kg C. 1.2342 kg D. 2.2342 kg
Solution:

( )
( )

443. What is the power of the pump, HP, if it delivers 925 gal/min of water against a head of 15
m?
A. 15.38 HP B. 16.38 HP C. 10.5 HP D. 11.5 HP
Solution:

135
[ ( )]( )( ) ( )( )( )
-
-

444. Kerosene is pumped into an aircraft fuel tank through a hose that has an inside diameter
of 4 cm. If the velocity of the kerosene is 8 m/s through the hose, determine the mass flow rate.
Assume that the kerosene has a density of 800 kg/m³.
A. 7.06 kg/s B. 7.56 kg/s C. 8.06 kg/s D. 8.56 kg/s
Solution:
( )
̇ ̇ ( )( )

445. During the working stroke of an engine the heat transferred out of the system was 150
kJ/kg of working substance. The internal energy also decreased by 400 kJ/kg of working
substance. Determine the work done.
A. 250 kJ/kg B. 550 kJ/kg C. 600 kJ/kg D. 350 kJ/kg
Solution:
( )
- (- ) -

446. During the experiment on Charles Law, the volume of the gas trapped in the apparatus is
10000 mm³ when the temperature is 18°C. The temperature of the gas was then raised to
85°C. Determine the new volume of the gas trapped in the apparatus if the pressure exerted on
the gas remained constant.
A. 12302.41 mm³ B. 8128.49 mm³ C. 70833.33 mm³ D. 2117.64 mm³
Solution:

447. Find the density of oil with a specific gravity of 1.6 in-g/cm3.
A. 15.68 g/cm3 B. 99.84 g/cm3 C. 0.8 g/cm3 D. 1.6 g/cm3
Solution:

136
448. What is the absolute pressure if the gauge pressure reading is 9 bar and the atmospheric
pressure is 0.9 bar?
A. 6.3 bar B. 7.8 bar C. 9.9 bar D. 8.1 bar
Solution:

449. The tank of an air compressor has a volume of 3 ft3 and is filled with air at a temperature of
40°F. If a gage on the tank reads 150 psig, what is the mass of the air in the tank?
A. 1.78 lbs B. 2.00 lbs C. 2.67 lbs D. 1.98 lbs
Solution:
- -

( ) ( )( )
-
( )
-

450. What is the mass of acetylene gas, V = 0.94 cu. ft., R = 59.35 ft-lb/lb-°R, T = 90°F, P = 200
psia?
A. 0.816 lb B. 0.841 lb C. 0.829 lb D. 0.852 lb
Solution:
( )( )
( )

451. Specific volume is the number of cubic meters of mixture per kilogram of dry air. If dry air
has these following properties: R a = 287 J/kg-K, T = 303 K, Pa = 99.604 kPa. Solve for the specific
volume.
A. 0.873 m3/kg B. 0.853 m3/kg C. 0.953 m3/kg D. 0.783 m3/kg
Solution:
( )

452. A refrigerating system operates on the reversed Carnot Cycle. The higher temperature of
the refrigerant in the system is 120°F and the lower is 10°F. The capacity is 20 tons. Neglect
losses. Determine the coefficient of performance.
A. 2.732 B. 5.373 C. 7.372 D. 4.273
Solution:

137
( ) ( )

453. At a pressure of 60°F, a motorbike Ire is inflated to 33 psig. As it is driven along the C -5
road, the temperature rose to 76°F. Assuming the volume remains constant, determine the
final gauge pressure.
Given:
P1 = 33 psig + 14.7 psig = 47.7 psig
T1 = 60°F + 460 = 520°R
T2 = 76°F + 460 = 536°R
A. 34.47 psig B. 49.17 psig C. 35.00 psig D. 34.30 psig
Solution:

454. Steam enters a turbine stage with an enthalpy of 3700 kJ/kg and a velocity of 80 m/s and
leaves with an enthalpy of 2864 kJ/kg with a velocity of 12.8 m/s. If the rate of a steam flow
through the turbine is 0.44 kg/s, what is the work done in kW?
A. 365 kW B. 365.64 kW C. 366.0 kW D. 366.50 kW
Solution:

( )
-
( )
-

( )
-
( )
-

455. Aluminum has a specific heat of 0.902 J/g-°C. How much heat is lost when a piece of
aluminum with a mass of 23.984 g cools from a temperature of 415.0°C to a temperature of
22.0°C?
A. 8500 J B. 6000 J C. 80000 J D. 7500 J
Solution:
138
( ) ( )( )

456. If the temperature of an air parcel is -20.5°C, and its density is 0.690 kg/m3, what is the
pressure of the air parcel?
A. 40 kPa B. 50 kPa C. 60 kPa D. 70 kPa
Solution:

457. A 35.0 mL sample of gas is enclosed in a flask at 22 degrees Celsius. If the flask was placed
in an ice bath at 0 degrees Celsius, what would the new gas volume be if the pressure is held
constant?
A. 34.1 mL B. 32.1 mL C. 32.39 mL D. 33.1 mL
Solution:

458. A car engine with a power output of 65 hp has a thermal efficiency of 24%. Determine the
fuel consumption rate of this car if the fuel has a heating value of 19,000 Btu/lb m.
A. 36.28 lbm/hr B. 37.28 lbm/hr C. 37.28 lbm/hr D. 35.30 lbm/hr
Solution:
̇
̇

̇
̇ ( )

̇ ̇ ( )

139
459. The thermal efficiency of a Carnot cycle operating between 170°C and 620°C is closest to:
A. 44% B. 50% C. 63% D. 73%
Solution:

460. Compute the humidity ratio of air at 70% relative humidity and 25°C when the barometric
pressure is 101.325 kPa. From the steam tables: Psat @ 34 °C = 3.169 kPa.
A. 0.014 kg water vapor/kg dry air C. 1.4 kg water vapor/kg dry air
B. 0.14 kg water vapor/kgdry air D. 0.0014 kg water vapor/kg dry air
Solution:

( )
( )

461. A pressure gauge registers 50 psig in a region where the barometer reads 14.8 psia. Find
the absolute pressure in kPa.
A. 666.66 kPa B. 556.66 kPa C. 446.66 kPa D. 336.66 kPa
Solution:

462. Consider 1 kg of air at 32°C that expanded by a reversible polytropic process with n = 1.25
until the pressure is halved. Determine the heat transfer. Specific heat at constant volume for
air is 0.1786 kJ/kg-K.
A. 17.02 kJ heat rejected C. 17.02 kJ heat added
B. 7.05 kJ heat rejected D. 7.05 kJ heat added
Solution:

( ) ( )

( ) ( ) - -
( ) (- )( )
-

140
463. A Carnot cycle uses nitrogen (k = 1.399) as the working substance. The heat supplied is 54
kJ and the adiabatic expansion ratio is 10. Determine the heat rejected.
A. 10 kJ B. 32.4 kJ C. 21.6 kJ D. 54 kJ
Solution:

( )
( )

464. A tank contains 20 kg of air at 200 kPa (gage) and 23°C. During the heating process the
temperature of air rises. For safety purposes a technician installed a relief-type valve so that
pressure of air inside the tank never exceeds 260 kPa (gage). At what air temperature the relief
valve will start releasing air?
A. 112°C B. 92°C C. 82°C D. 102°C
Solution:

( )
( )
( ) ( )

465. An air compressor takes in 9 kg/min of air at 98 kPa; υ 1 = 0.125 m3/kg and discharges it at
680 kPa; υ2 = 0.03 m3/kg. The increase of internal energy is 93 kJ/kg and the work done on air is
163 kJ/kg; the change in potential and kinetic energy are neglected. How much heat is
transferred per kg of air?
A. 264.15 kJ/kg B. 61.85 kJ/kg C. 288.65 kJ/kg D. 78.15 kJ/kg
Solution:
( )
( )
( ) (- )
- ( )

466. During a reversible process, there are abstracted 317 kJ/s from 1.134 kg/s of a certain gas
while the temperature remains constant at 26.7°C. For this gas c p = 2.232 and cv = 1.713 kJ/kg-K.
The initial pressure is 586 kPa. Determine the final volume flow rate.
A. 0.301 m3/s B. 0.03 m3/s C. 0.5 m3/s D. 0.05 m3/s
Solution:

141
-

̇ ̇ - ( )
̇

467. Flow of water in a pipe has a velocity at 10 meters per second. Determine the velocity
head of the water.
A. 50.1 meters B. 5.10 meters C. 8.20 meters D. 100 meters
Solution:

( )

468. A Diesel cycle has a cut-off ratio of 2.20 and a compression ratio of 10. Find the cycle
efficiency.
A. 55.10% B. 59.735% C. 52.23% D. 62.37%
Solution:
[ ( )
] [ ( )
]

469. A diesel cycle has an initial temperature of 27°C. If the cut-off ratio is 2.50 and
compression ratio is 12, find the maximum cycle temperature.
A. 1634.4°C B. 1753.44°C C. 2010.3°C D. 1983.4°C
Solution:
( )

( )

470. A diesel cycle, which takes in air at 1 bar and 26°C, has a compression ratio of 19. Calculate
the operating clearance in percent.
A. 8.08 B. 8.56 C. 7.52 D. 5.55
Solution:

142
471. An Otto cycle has an initial pressure of 100 kPa and has a pressure of 400 kPa after
adiabatic compression. Find the cycle efficiency.
A. 32.70% B. 34.70% C. 36.70% D. 38.70%
Solution:

( ) ( )

472. An Otto cycle has a clearance of 8% and heat added of 1000 kJ. Find the heat rejected.
A. 564 kJ B. 353 kJ C. 709 kJ D. 867 kJ
Solution:

( )

473. An Otto cycle has a heat rejected of 300 kJ and work of 700 kJ. Find the cycle efficiency.
A. 56% B. 60% C. 70% D. 50%
Solution:

474. An Otto cycle has a pressure ratio of 7. What is the cycle compression ratio?
A. 5.18 B. 6.34 C. 7.34 D. 4.01
Solution:

( )

475. Find the power of a rotating shaft which develops a torque of 188 N-m at 1350 rpm.
A. 101.54 hp B. 53.63 hp C. 63.35 hp D. 35.63 hp
Solution:
( )( ) ( )

143
476. Determine the pressure exerted on a diver at 30 m below the free surface of the sea.
Assume a barometric pressure of 101 kPa and the specific gravity of sea water is 1.03.
A. 404 kPa B. 410 kPa C. 420 kPa D. 430 kPa
Solution:
( )( )

477. An air compressor has an inlet enthalpy of 35 Btu/lb and an exit enthalpy of 70 Btu/lb. The
mass flow rate of air is 3 lb/s. If the heat loss is 466.62 Btu/min, find the work input to the
compressor.
A. 139.59 hp B. 149.59 hp C. 159.59 hp D. 169.59 hp
Solution:
̇ ̇
- ( )( ) ̇

̇ - ( ) -

478. An automobile tire is inflated to 35 psig at 54°F. After being driven, the temperature rises
to 80°F. Determine the final gage pressure assuming that the tire is inflexible.
A. 36.51 psig B. 37.51 psig C. 38.51 psig D. 39.51 psig
Solution:

( ) ( )( )

479. A condenser vacuum gauge reads 600 mm Hg when the barometer reads 760 mm Hg.
What is the absolute condenser pressure in bar?
A. 0.0213 B. 0.061 C. 0.213 D. 0.610
Solution:
( ) ( )

480. Water flows in a pipe at the rate of 10 kg/s. If the velocity of flow is 10 m/s, find the pipe
diameter.
A. 30.23 mm B. 35.68 mm C. 38.39 mm D. 42.39 mm
Solution:
̇
144
( )( )

481. What is the resulting pressure when one kilogram of air at 104 kPa and 98°C is heated at
constant volume to 450°C?
A. 202.67 kPa B. 194.67 kPa C. 186.53 kPa D. 198.65 kPa
Solution:

( )

482. Determine the degrees of superheat of steam at 101.325 kPa and 170°C.
A. 50°C B. 70°C C. 60°C D. 80°C
Solution:

483. Calculate the approximate enthalpy of water at 90°C.


A. 366.83 kJ/kg B. 376.83 kJ/kg C. 386.83 kJ/kg D. 396.83 kJ/kg
Solution:
( ) ( )

484. A Carnot cycle operates between 30°C and 350°C. Find the cycle efficiency.
A. 51.36% B. 63.45% C. 45.37% D. 76.45%
Solution:

485. A Carnot cycle has a maximum temperature of 550°F and minimum temperature of 100°F.
If the heat added is 4200 Btu/min, find the horsepower output of the engine.
A. 34.53 B. 40.56 C. 44.13 D. 65.40
Solution:

̇ ̇ ( ) ( )

486. A Carnot cycle has a sink temperature of 100°F and a cycle efficiency of 70%. Find the
temperature of the heat souce.

145
A. 1306.70°F B. 1406.70°F C. 1506.70°F D. 1606.70°F
Solution:

487. The quality of steam is 20%. This means that:


A. mass of liquid is 20%, mass of vapor is 80%
B. mass of liquid is 20%, mass of vapor is 0%
C. mass of liquid is 80%, mass of vapor is 20%
D. none of the above
Solution:

488. Fifty kilograms of cooling water per second enter the condenser at 25°C and leaves at
50°C. Find the heat carried away by water.
A. 1234.45 kW B. 5233.75 kW C. 2340.53 kW D. 3140.25 kW
Solution:
̇ ̇ ( )( )

489. Ten kilograms per second of steam enter the turbine with an enthalpy of 3200 kJ/kg and
enter the condenser with an enthalpy of 2500 kJ/kg in a Rankine cycle. If the turbine efficiency
is 80% and the generator efficiency is 90%, determine the power plant output.
A. 4320 kW B. 3213 kW C. 4056 kW D. 5040 kW
Solution:
̇ ( )( )( )

490. Determine the quality of steam in a vessel containing 2 kg of saturated vapor and 8 kg of
saturated liquid.
A. 100% B. 20% C. 80% D. 60%
Solution:

491. The condenser of a reheat power plant rejects heat at the rate of 600 kW. The mass flow
rate of cooling water is 5 kg/s and the inlet cooling water temperature is 35°C. Calculate the
condenser cooling water exit temperature.
146
A. 43.45°C B. 53.45°C C. 63.66°C D. 74.34°C
Solution:
( )( )

492. A heat engine has a thermal efficiency of 50%. How much power does the engine produce
when heat is transferred at a rate of 10 9 kJ/hr?
A. 50 MW B. 75 MW C. 139 MW D. 147 MW
Solution:
̇ ( )
̇ ( )

493. One kilogram of air is compressed adiabatically and in a steady-flow manner. The
compression efficiency is 80% and the work done on the air is 265 kJ/kg. Compute the heat.
A. 212 kJ/kg B. 100 kJ/kg C. 0 kJ/kg D. 331.25 kJ/kg
Solution:
( )

494. Three-hundred kilojoules of heat flow by conduction from the outside to the inside of a
cold storage in one hour. If the temperature and all other conditions are the same, what is the
heat flowing through the cold storage room in two hours?
A. 600 kJ B. 900 kJ C. 300 kJ D. 1,200 kJ
Solution:
( )

495. Determine the density of air at 760 mmHg absolute and 22°C?
A. 1.014 kg/m3 B. 1.316 kg/m3 C. 1.197 kg/m3 D. 1.266 kg/m3
Solution:

( )( )
-

496. A refrigerating machine that is classified as a one-ton machine has the capacity to produce
a cooling effect of?
A. 3.516 kW B. 12,000 Btu/hr C. 211 kJ/min D. All of the above
Solution:

147
IPE
C 1. Monochlorofluoromethane is a refrigerant B 13.A vessel permanently connected to a system
known as: by inlet and outlet pipes of storage of liquid
a. R-12 c. R-22 refrigerant
b. R-13 d. None of the above a. refrigerant container c. flash tank
b. liquid receiver d. surge tank
D 2. Which of the ff. is a proper way of leak
detection for R-12 and halocarbon B 14.Air flows through coil without making
refrigeration systems? contact with the coil surface is known as
a. stains c. toxic odor a. drawn air c. contact air
b. corrosion d. loss of cooling capacity b. bypass air d. air load

C 3. It refers to the final constant weight reached A 15.the process of lowering the temperature of
by a hygroscopic substance after being dried given space and maintaining it for the
out purpose of preserving and chilling certain
a. Bone Dry Weight c. Both a&b substances
b. Dry Weight d. NOTA a. refrigeration c. chilling
b. cooling d. air-conditioning
B 4. Is a type of dryer in w/c material to be dried
is supported on floor through w/c hot gases D 16.it will either cool or maintain a body at
pass temp below that of its surrounding is known
a. Tray dryer c. Rotary dryer as the
b. Hearth dryer d. Tower dryer a. heat engine c. heat pump
b. heat sink d. refrigerating machine
B 5. It is the temp. at w/c liquid starts to boil or
the temp. at w/c vapor begins to condense B 17.a reciprocating compressor has a piston
a. Dry blub temp c. Dew point Temp speed of 200 fpm at 400 rpm. Its stroke is
b. Sat. Temp d. Wet bulb temp approx.
a. 2.56 in c. 5.32 in
D 6. The tendency of pump to cavitate will be b. 3 in. d. 20 in
increased if:
a. impeller has smooth finish C 18.a refrigeration system that combines two
b. inlet edges are rounded refrigeration units
c. suction line velocities are lowered a. multi-evaporation system
d. fluid temp is raised b. multi-stage compression system
c. cascade system
B 7. For a single component system, the number d. air-refrigeration cycle
of properties required to define a phase
uniquely is: C 19.fire involving energized electrical equipment
a. 1 c. 3 a. class A fire c. class C fire
b. 2 d. 4 b. class B fire d. class D fire

A 8. It draws the flue gas from the system and D 20.dew point temp is temp where water starts
series them up the stack is: a. to condense
a. induced draft fan c. forced draft fan b. to boil
b. cyclone d. wind tunnel c. to condense and boil
d. to condense and boil at given pressure
D 9. The study of properties of air and its water
vapor contents A 21.glass prod process is broken down into 2
a. humidification c. ventilation gen. categories: container prod (glass
b. refrigeration d. psychrometry bottle/bottle) and sheet prod (float glass)
a. container & sheet prod
B 10.It exist when the total suction is above b. box & sheet prod
atmospheric pressure c. container & flattening prod
a. suction lift c. suction pressure d. NOTA
b. suction head d. suction line
C 22.modern detergents are comprised of three
C 11.It is the wastewater that will flow into each main components
unit of operation of the wastewater treatment a. builders, blend, extender
plant that is partially or completely treated b. binders, bleach, enzymes
a. liquid waste c. influent c. builders, bleach, enzymes
b. solid waste d. effluent d. No answer

A 12.It raises pressure of CO2 gas to 250 psig so


that it can be liquefied at -100F
a. CO2 compressor c. deodorizer
b. trim cooler d. foam trap
A 23.is a broad term applied to large number of C 30.water substance at 70 bar and 65C enters a
carbohydrates composed of carbon, boiler tube of constant inside diameter of 25
hydrogen, and oxygen present in many mm. the water leaves the boiler tube at 50
plants and characterized by a more or less bar and 700K at velocity of 100 m/s.
sweet taste Calculate the inlet volume flow in L/sec.
a. sugar c. iron At 70 bar & 65C:
b. salt d. sweet v1 = 0.001017 m3/kg
At 50 bar & 700K:
B 24.a closed vessel contains air at a pressure of v2 = 0.06081 m3/kg
160 kN/m2 gauge and temp of 30C. The air a. 0.75 c. 0.82
is heated at const. vol. to 60C w/ atm b. 0.64 d. 0.96
pressure of 759mmHg. What is the final
gauge pressure? D 31.twenty grams of oxygen gas (O2) are
a. 174 c. 167 compressed at const temp of 30C to 5% of
b. 186 d. 172 their original vol. What work is done on the
system?
C 25.what is the total reqd heating energy in `Use R of air = 0.0619 cal/gm-K
raising the temp of given amt of water when a. 824 cal c. 944 cal
the energy supplied is 1000kWh w/ heat b. 894 cal d. 1124 cal
losses of 25% ( )( )
a. 1000 c. 1333
b. 1500 d. 1250 C 32.gas is enclosed in a cylinder w/ weighted
̅̅̅̅ piston as top boundary. The gas is heated
and expands from a volume of 0.04 m3 to
0.10 m3 at const pressure of 200 kPa.
B 26.a certain gas at 101.325 kPa and 16C whose Calculate the work done by system.
vol is 2.83m3 are compressed into a storage a. 8 kJ c. 12 kJ
vessel of 0.31 m3 capacity. Before b. 10 kJ d. 14 kJ
admission, the storage vessel contained the ( )
gas at pressure and temp of 137.8kPa and
24C. After admission, the pressure has B 33.if F scale is twice C scale, what is the
increased to 1171.8 kPa. What should be the reading in the Fahrenheit scale?
final temp of gas in vessel (in Kelvin)? a. 160 c. 140
a. 298.0 c. 180 b. 320 d. 280
b. 319.0 d. 420
B 34.a cylinder & piston arrangement contains
A 27.a tank contains 80 ft3 of air at pressure of sat water vapor at 110C. The vapor is
350 psi. If the air is cooled until its pressure compressed in a reversible adiabatic process
and temp decrease to 200 psi and 70F until pressure is 1.6 MPa. Determine work
respectively, what is the decrease in internal done by the system per kg of water
energy? At 110C:
a. 4575 c. 5552 S1 = 7.2387 kJ/kg-K
b. -5507 d. 0 U1 = 2518.1 kJ/kg
At 1.6 MPa:
D 28.an electric heater is to heat 11 kg of oil per S2 = 7.2374 kJ/kg-K
min, from 4.5C to 65.5C. Specific heat of oil U2 = 2950.1 kJ/kg
is 2.1 J/gm-C. How many watts should this T2 = 400C
heater consume? a. -500 kJ/kg c. -632 kJ/kg
a. 20 448 W c. 20 400 W b. -432 kJ/kg d. -700 kJ/kg
b. 33 448 W d. 23 442 W ( )

A 35.During polytrophic process of ideal gas


C 29.find the heat transfer reqd to convert 5 kg of
state changes from 138 kPa and 5C to 827
water at pressure if 20 bar and temp of 21C
kPa and 171C. Find value of n.
into steam dryness fraction of 0.90 at same
a. 1.354 c. 1.345
pressure.
b. 1.253 d. 1.234
Properties of steam:
P = 20 bar ( ) ( )
hf = 909 kJ/kg
hfg = 1899 kJ/kg
Note: for water at 21C: C 36.calculate specific vol of air-vapor mixture
h = 88 kJ/kg in cu. m per kg of dry air when the ff.
a. 10 160 kJ c. 12 610 kJ conditions are given:
b. 11 610 kJ d. 15 610 kJ t = 30C
{[ ( )] } w = 0.015kg/kg
P = 90kPa
a. 2.12 m3/kg c. 0.99 m3/kg
3
b. 0.622 m /kg d. 1.515 m3/kg
A 37.copra enters dryer containing 40% H2O & C 44.a refrigerator having 30kW capacity
60% solid, and leaves w/ 5% H2O & 95% requires 10HP compressor. Find COP of
solid. Find wt. of H2O removed per lb. of system.
final prod. a. 2.78 c. 4.02
a. 0.583 lb/lb c. 0.475 lb/lb b. 3.78 d. 5.02
b. 1.3753 lb/lb d. 1.015 lb/lb
A 45.A VCRS is designed to have a capacity of
A 38.an air conditioning plant w/ capacity of 100 TOR. It produces chilled water from
400kW has an evaporating & condensing 22C to 2C. Its actual COP is 5.86 and 35%
temp of 3C and 37C respectively. If it uses of power supplied to compressor is lost in
R-12, what is the volume flow rate under form of friction and cylinder cooling losses.
suction conditions? Determine the size of electric motor reqd to
At 3C drive compressor in KW and vol flow rate of
hg = 352.75 kJ/kg chilled water in L/s.
v = 50.47 L/kg a. 92.31 kW, 4.199 L/s
hf = 202.78 kJ/kg b. 90.71 kW, 5.277 L/s
`Enthalpy leaving cond = 235.3503kJ/kg c. 93.75 kW, 5.724 L/s
a. 0.172 m3/s c. 3.411 m3/s d. 91.75 kW, 7.575 L/s
b. 0.281 m3/s d. 3.5 m3/s
A 46.a single acting twin cylinder ammonia
D 39.fish weighing 11,000 kg w/ temp of 20C is compressor w/ bore equal to stroke is driven
brought to cold storage & w/c shall be by engine at 250 rpm. The machine is
cooled to -10 C in 11hrs. Find the reqd plant installed in chilling plant to produce 700 kW
capacity in tons if Cp of fish above and of refrigeration at -18C evaporator temp. At
below freezing is 0.7 kCal/kg-C and 0.3 this temp, cooling effect per kg mass = 1160
kCal/kg-C respectively, w/ freezing pt at - kJ. Specific vol of vapor entering
3C. compressor = 0.592 m3/kg. Assume 85%
`The latent heat of freezing = 55.5 kCal/kg volumetric efficiency, determine the bore in
a. 25.26 c. NOTA mm.
b. 15.26 d. 24.38 a. 400 mm c. 500 mm
b.300 mm d. 450 mm
C 40.an ice plant produces 20 tons of ice per day
at -15C from water at 25C. If losses are 12% A 47.A refrigeration compressor having 10 in.
of freezing and chilling load, calculate flywheel is driven by a 4-pole AC motor. If
refrigeration capacity of plant in TOR. diameter of motor is 4 in, determine speed
a. 28.13 c. 31.5 of compressor.
b. 28.13 d. 41.5 a. 70 rpm c. 600 rpm
b. 500 rpm d. 800 rpm
A 41.calculate tons of refrigeration reqd to freeze
in 4 hrs. 3mm of 10C water on a skating risk A 48.calculate piston displacement of 2-cylinder
61m x 30m if the ground temp is 10C and compressor rotating at 1450 rpm if diameter
the air temp is 10C and the air temp above of cylinder is 2.5 in. & length of stroke is 2
rink is 16C. in.
Heat transfer coefficient a. 16.48 ft3/min c. 14.48 ft3/min
air to 0C ice = 6.8 W/m2-C b. 15.48 ft3/min d. 17.48 ft3/min
grnd to -6C = 0.3402 W/m2-C
a. 101.69 c.140.62 A 49.a duct 0.40 m high and 0.80 m wide
b. 151.68 d. 201.96 suspended from the ceiling in a corridor
makes a right angle turn in the horizontal
D 42.The power requirement of Carnot refrigerator plane. The inner radius is 0.2m and outer
in maintaining a low temp region at 300K is radius is 1.0 m measured from the same
1.5 kW per ton. Find heat rejected. center. The velocity of air in duct is 10 m/s.
a. 4.02 kW c. 6.02 kW Compute the pressure drop in this elbow.
b. 7.02 kW d. 5.02 kW Assume:
f = 0.3
D 43.4000 L/hr of distillates are to be cooled from density = 1.204 kg/m3
21C to -12C and 12% of wax by wt. is L = 10m
separated out at 15C. The cp heat of oil is 2 a. 341 Pa c. 143 Pa
kJ/kg-C and SG is 0.87. The specific heat of b. 441 Pa d. 144 Pa
wax is 2.5 and latent heat is 290 kJ/kg.
Allow 10% for losses, find the refrigerating C 50.determine the sensible heat of 5 lb of air
capacity of system. having dry bulb temp of 70F and humidity
a. 20 TOR c. 40 TOR ratio of 0.0092 lb/lb, the latter corresponding
b. 51.08 TOR d. 31.08 TOR to dew pt temp of 55F.
a. 94 BTU c. 84 BTU
b. 48 BTU d. 49 BTU
C 51.a 0.70 m vane axial fan is running at 2000 A 55.a mixture of dry air and water vapor is at
rpm delivers 7.5 m3/s against 0.08m static temp of 21C under pressure of 101kPa. The
pressure thereby consuming 12 kW. If fan dew pt temp is 15C. Calculate the relative
wheel diameter is increased from 0.70 m to humidity.
0.76 m, so the diameter ration is 1.10:1, a. 68.56% c.56.68%
what is the new static pressure? b. 65.68% d. 58.66%
a. 1.01m c. 0.097m
b. 0.314m d.31.01m A 56.a dryer is to deliver 1000 kg/hr of palay
with a final moisture content of 10%. The
B 52. Centrifugal pump has const speed of 1500 initial moisture content in the feed is 15% at
rpm and has head of 75m. What is the effect atm cond w/ 32C dry-bulb & 21C wet-bulb.
on the head of the pump if the impeller The dryer is maintained at 45C while RH of
diameter is reduced from 280mm to 230 hot humid air from dryer is 80%. If steam
mm? pressure supplied to heater is 2 MPa,
a. 25.6 mm c. 65.6 mm determine air supplied to dryer in m3/hr.
b. 50.6 mm d. 72.6 mm a. 1332.25 m3/hr c. 1223.25 m3/hr
3
b. 1233.25 m /hr d. 1523.13 m3/hr
A 53.in an air conditioning unit, 3.5 m3/s of air at
27C dry-bulb temp, 50%RH & std. atm A 57.An assembly hall was to have an air
pressure enters unit. The leaving condition conditioning unit installed w/c would be
of air is 13C dry-bulb temp and 90% RH. maintained at 26C dry-bulb & at 50%RH.
Using properties from psychro chart, The unit delivers air at 15C dry-bulb temp &
calculate refrigerating capacity in kW. the calculated sensible heat load is 150kW
a. 87.57 kW c. 57.87 kW & latent heat is 51.3kW. 20% by wt of
b. 77.57 kW d. 58.77 kW extracted air is made up of outside air at 34C
dry-bulb & 60% RH, while 80% is extracted
D 54.a stream of outdoor air is mixed with steam by air conditioner from assembly hall.
of return air in an air conditioning system Determine the air conditioner’s refrigerating
that operates at 101kPa pressure. The flow capacity in TOR & its ventilation load in
rate of outdoor system air is 2 kg/s and its kW.
condition is 35C dry-bulb temp & 25C wet- a. 83.22 TOR, 37.47 kW
bulb temp. he flow rate of return is 3 kg/s & b. 76.43 TOR, 57.34 kW
its cond is = 24C & 50%RH. Determine c. 89.56 TOR, 45.77 kW
enthalpy of mixture. d. 56.78 TOR, 47.68 kW
a. 91.56 kJ/kg c.69.15 kJ/kg
b. 51.69 kJ/kg d. 59.16 kJ/kg
71.81% 58. Sol’n:

( ) ( )

( )( ) ( ) ( )
( )

( )( )

59.13% 59. Sol’n:

HEAT BALANCE:
( ) ( )
( )( )( ) ( )

5.6 kW 60. Sol’n:


( )( )( )
[( ) ] [( ) ]

FOR TWO STAGE:


√ √( )( )

( )( )( )
[( ) ] [( ) ]
IPE Exit Exam Reviewer

1. What tonnage of refrigerating machine is required if the refrigerating system extracted


48,000 btu per hour?
Answer: 4 tons

2. An insulator with superior tensile strength but low heat resistance


Answer: Fiberglass

3. What is the combination of a wet and dry bulb thermometer is called a


Answer: Hygrometer or psychrometer

4. The weight of steel bar reinforcement show be how many time the weight of the foundation?
Answer: ½% to 1%

5. Is a high temperature process that turns incinerator ash into a safe, glass – like material
Answer: Vitrification

6. Oil is added to a freon compressor by


Answer: Pumping in with a hand pump

7. The difference between the theoretical draft and the draft loss
Answer: Available draft

8. The purpose of expansion valve by-pass is to


Answer: Controls the refrigerant to the evaporator in case the automatic valves fail

9. Load curve refers to the plot of:


Answer: Load versus time

10. What does a negative Joule-Thompson coefficient means during a throttling process?
Answer: fluid temperature rises

11. The maximum unit pressure of turbine and generator on the reinforced concrete should not
exceed
Answer: 17.62 kg/cm2

12. Water in vapor form remains a vapor as long as temperature is what relations to the dew
point temperature
Answer: below
13. The “refrigerating effect” of a refrigerant is a always:
Answer: Less than its latent heat

14. A burner should always start up in ___ fire and shut down in ___ fire
Answer: Low; low

15. A ___ boiler has water in the tubes and heat gases of combustion passing through the tubes
Answer: Water tube

16. If the compressor had been running satisfactorily for a long period of time but suddenly the
compartment temperature started to rise, the trouble might be:
Answer: Any of the other choices
(A refrigerant leak has developed, The solenoid valve has jammed shut, The expansion valve
may contain frozen water)

17. A type of polymer used for Styrofoam cups and clam shell food containers
Answer: Polystyrene (PS)

18. Mycotoxins are poisonous chemicals produced by:


Answer: Molds

19. The magnitude of the drag coefficient of a sphere in water is dependent upon all of the ff
except
Answer: Units of measure (SI or English engineering system)

20. A refrigeration system in which only part of the refrigerant passes over the heat transfer
surface is evaporated and the balance is separated from the vapor and recirculated
Answer: Flooded system

21. The boiling point of freon-12 at atmospheric pressure is


Answer: -22F

22. Which of the following vital components of the refrigeration system where both temperature
and pressure are increased?
Answer: Compressor

23. Which of the following must be done to eliminate frost on the discharge pipe of the
compressor?
Answer: None of the other choices
(Open the expansion valve, crack bypass valve, Regulates water to the condenser)

24. Orifice coefficients are used to determine:


Answer: Energy losses
25. Which of the following is used as high head turbine?
Answer: Impulse

26. A water-tube condenser has a total of 60 tubes. If these two passes, then compute the number
of tubes per pass
Answer: 15

27. Absolute zero is


Answer: 460 degrees below zero on the Fahrenheit scale

28. Which of the following can be the cause of low head pressure
Answer: Any of the other choices
(Leaky discharge valves, Too much or too cold condensing water, insufficient charge of
refrigerant)

29. What is the most common freon gas used in centrifugal compressors?
Answer: F-11

30. When purging an ammonia condenser into a bucket of water, one can tell when the air is out
and ammonia starts to come through the:
Answer: Change of bubbling sound of air to the cracking sound of ammonia

31. In the deep well installation or operation, the difference between static water level and
operating water level is known as:
Answer: Drawdown

32. White ice is


Answer: Due to dissolved air, gasses and impurities

33. Which of the following is a characteristic of an impulse turbine


Answer: Steam striking blades at zero angle

34. Heavy water is:


Answer: D2O

35. When excess solute in a solution settles to the bottom of the container, the process is called:
Answer: Precipitation

36. Calcium chloride is sometimes used in refrigeration system as a:


Answer: Secondary Coolant

37. If a compressor runs continuously, the cause might be a:


Answer: Stuck low-pressure switch
38. The thermal expansion valve is located between the:
Answer: Solenoid valve and the evaporator coils

39. The cooling water regulator is automatically actuated by which of the following?
Answer: The discharge pressure of the refrigerant

40. Based on the PSME code, what should be provided in each steam outlets if two or more
boilers will be connected in parallel?
Answer: Non-return and shut off valves

41. The amount of sensible heat for a sensible heat ratio of 0.8 and a total cooling load of 100 is
Answer: 80

42. Which refrigerant is used for the air conditioning of passenger aircraft cabin
Answer: Air

43. When a falling object reaches a speed at which the drag force equals its weight, it has
achieved:
Answer: A turbulent boundary layer

44. The minimum vertical distance from the floor or soil level to the top edge of the foundation
must be around
Answer: 120 mm

45. How many moisture be removed from air?


Answer: condensation and absorption

46. The charging valve is located between the:


Answer: King valve and the expansion valve

47. A “cold diffuser” is a:


Answer: Type of evaporator

48. Which of the following stops the compressor before the relief valve opens?
Answer: High pressure control

49. What is the excess refrigerant removed?


Answer: Charging side of the system

50. Which of the following is used in measuring the density of a brine solution?
Answer: A hydrometer
51. All of the following are common types of chemical reaction except
Answer: Fission

52. A ___ after each radiator allows ___ to pass through to the return line
Answer: Steam trap; condensate

53. A leaky solution valve can usually detected by:


Answer: Any of the other choices
(A fluctuating suction pressure gauge, a higher suction pressure, closing in on the suction
valve having no effect on the suction pressure)

54. If the cooling water to the condenser suddenly fails:


Answer: The compressor will shutdown

55. If Et is the total net energy generated by a plant in a certain period of time and Er is the rated
net energy capacity of the plant during the same period of time, then what is the plant
operating factor?
Answer: Et/Er

56. A thermometer sense which of the following?


Answer: Sensible heat

57. Is the subjective method in which the smoke density is visually compared to five
standardized white-black grids
Answer: Ringelman scale

58. Excessive head pressure is caused by


Answer: Flooded condenser tubes or cooling water temperature too high

59. What do you call a plate or vane used to direct or control movement of fluid or air within the
confined area?
Answer: Baffle

60. The total pressure supplied by the fan at maximum operating conditions
Answer: Net rating or fan boost

61. The system should be purged:


Answer: After the system has been shut down for few hours

62. What is likely to occur when sections of the impeller of a centrifugal pump are handling
vapor and other sections are handling liquid.
Answer: complete failure to operate
63. What is the chemical formula of Ozone?
Answer: O3

64. An excessively high head pressure would be caused by:


Answer: Insufficient cooling water to the condenser

65. When an electric motor nameplate indicate a “100-kW electric motor” then what does a 100-
kW rating refers to?
Answer: Mechanical power output

66. During a cooling and dehumidifying process, sensible and latent heats are removed in the
cooling coil. If Hs is the sensible heat and Ht is the total heat transferred, then determine the
coil sensible factor.
Answer: Hs/Ht

67. A single trunk piston-type compressor is undesirable for a Freon unit because the:
Answer: Lubricant mixes with the refrigerant

68. What is the first step that must be done when securing a system?
Answer: Close receiver (King) valve

69. The amount of CO can be determined by the color of Palladium Chloride. An amount of 30
ppm to 70 ppm will cause:
Answer: Slight Darkening

70. The water hammer phenomenon is primarily what kind of fluid mechanism?
Answer: Dynamic (a time-dependent phenomena)

71. The other name for Swamp Cooling is:


Answer: Wet Roof Cooling

72. The design of an air supply duct of an air condition system


Answer: affects the distribution

73. A Freon – 12 gage shows pressure and ____.


Answer: Saturation temperature

74. High superheat of the vapor in the system would cause


Answer: A decrease in capacity

75. When there is no work between the thermodynamic system and its surrounding, the quantity
of net heat transfer is equal to:
Answer: total energy of a closed system
76. The relation between the Fahrenheit absolute scale and the Celsius absolute scale is:
Answer: R = 1.8 Kelvin

77. External frost on inlet of expansion valve indicates:


Answer: Expansion valve plugged or dirty

78. The total cross section of a target atom is made up of


Answer: The absorption and scattering cross sections

79. The motor condition of an air conditioning unit can be checked with:
Answer: The continuity of light or with an ohmmeter

80. A number used to compare energy usage for different areas. It is calculated by dividing the
energy consumption by the footage of the conditioned area.
Answer: Energy utilization Index

81. What is the effect if the refrigerant is removed from the system too fast?
Answer: It may “freeze – up” the condenser

82. After the total force of the steam has lifted the safety valve off its seat, the steam enters the
____.
Answer: Huddling chamber

83. By which of the fooling processes heat mainly dissipates in cooling towers?
Answer: Evaporation

84. All of the following fluid phenomena are based on the force momentum principle of a
flowing fluid except:
Answer: Diesel automobile engines

85. All of the following are forms of drag on a body moving through a fluid except
Answer: D’alembert’s paradox drag

86. What is the use of the suction pressure regulating valve?


Answer: Maintains the back pressure in the evaporator coils

87. There shall be no foundation bolts less than _____.


Answer: 12 mm in diameter

88. The ratio of the rated cooling capacity divided by the amount of electrical power used:
Answer: Energy efficiency ratio (EER)
89. The scale trap is located between the:
Answer: King (liquid) valve and expansion valve

90. The temperature in the vegetable box should be approximately


Answer: 35 to 45 deg. F

91. A thermostat is a
Answer: Temperature-operated switch

92. The bulb for the thermal expansion valve is located:


Answer: Near the evaporator coil outlet

93. The coefficient of velocity is the ratio of the:


Answer: Actual velocity to the theoretical velocity

94. What is the specific humidity of dry air?


Answer: 0

95. What is the reason why a thermometer in vapor compression system is installed close to the
compressor?
Answer: Because it helps the operator in adjusting compressor for greatest efficiency

96. As the steam temperature increases the steam temperature:


Answer: Increases

97. Substances that have the ability to absorb moisture from the air are called:
Answer: Desiccants

98. Evaporative condenser is used to cool


Answer: condenser vapor

99. If the head pressure is too high:


Answer: The high pressure cutout switch should operate before the relief valve opens

100. A material or substance that is accidentally or intentionally introduce to the environment


in a quantity that exceeds what occurs naturally
Answer: Pollutant

101. The greatest decrease in refrigerant temperature occurs in the ____


Answer: Evaporator

102. The diesel cycle is the ideal cycle for a :


Answer: compression-ignition engine
103. The moisture in a refrigeration system can be removed with help of which of the
following?
Answer: Driers

104. What process is employed when the turbine steam power plants experience excessive
moisture?
Answer: reheating

105. The path that directs the flow of refrigerant through the compressor
Answer: Valve

106. Which of the following is another name for the liquid valve?
Answer: King valve

107. Expansion factors take into account the:


Answer: Effects of compressibility

108. The term used to express the amount moisture in a given sample of air. It is compared
with the amount of moisture in a given sample of air.
Answer: Relative Humidity

109. What are the four basic methods of determining whether the proper amount of refrigerant
being added to the system?
Answer: Bull’s eye, weight, pressure and frost line

110. Which of the following components of the window air conditioning system must be
cleaned annually?
Answer: All of these
(Evaporator and Condenser, Motor and Compressor, Fan Blades and Fan Motor)

111. At present, the number of true elementary particles, which include leptons and quarks is
thought to be
Answer: 12

112. All of the following statements about conjugate acids and bases are true except
Answer: A conjugate base results when a base accepts a proton

113. A double-seated valve allows the valve to be


Answer: Packed in the wide open or closed position

114. In order to keep combustion products inside the combustion chamber and stack system,
balanced draft system may actually operate with a slight
Answer: negative pressure
115. What is the cause of pressure drop in the boiler, condenser and the pipings between
different components? Because of this drop, steam leaving the boiler at a lower pressure.
Answer: Fluid friction

116. It is a form of oxygen photochemically produced in nature.


Answer: Ozone

117. Absolute zero on the Fahrenheit scale is equal to


Answer: -460

118. Low suction pressure is caused by:


Answer: Solenoid valve not functioning properly

119. What do you call the storage tank for liquid refrigerant?
Answer: Receiver tank

120. Why should you avoid bending or twisting of fan blades in an air conditioning unit?
Answer: It will wear out the motor bearings and cause noise

121. All of the following statements are characteristics of bases except


Answer: They have a pH between 0 and 7

122. Concrete foundation should have steel bar reinforcement placed vertically and
horizontally to avoid ____.
Answer: Thermal cracking

123. The pump will not cavitate if the available Net Positive Suction Head (NPSH) is:
Answer: equal or greater than the required NPHS

124. What will happen to the capacity if the superheat is increased on the suction side?
Answer: Decreases

125. When heavy electrical currents are involved, the thermostat will be operated by a
Answer: Relay

126. Valves in piping in an ammonia system are made of


Answer: Iron

127. Which of the following is not essential to a compression refrigeration system?


Answer: A receiver

128. Nitrogen occupied almost how much of the Earth’s atmosphere?


Answer: 3/4
129. If frost forms on the cylinders, the cause would be:
Answer: Expansion valve open too wide

130. Which of the following gasket materials should be on Freon system?


Answer: Asbestos or Metallic

131. Which of the following would cause the crankcase and head to get hot with low suction
pressure?
Answer: Insufficient refrigeration

132. Moisture in a system will cause a:


Answer: Faulty expansion valve

133. The steam turbine foundation should be designed to support the machine load plus how
many percent for impact, condenser load, floor loads and dead loads?
Answer: 25

134. When air is heated, what happened to its relative humidity?


Answer: Remain constant

135. Sweating of crankcase is caused by which of the following?


Answer: Too much oil in the system

136. What do you call a material in a dryer?


Answer: Desiccant

137. The matching model and full-scale prototype results a fluid dynamic phenomena
involving a fully submerged body requires equality of:
Answer: Reynolds number

138. Which of the following uses a secondary refrigerant?


Answer: Ice plant

139. When the outlet temperature at the evaporator exceeds the inlet temperature, the
condition is known as:
Answer: Superheating

140. In series pipe systems, all of the following parameters vary from section to section except
Answer: Mass flow

141. Which of the following is used to control refrigeration compressor capacity?


Answer: Unloader
142. TOR is a unit equivalent to:
Answer: 12.66 kN-m/hr

143. The agent used in an indirect reefer system is:


Answer: Calcium chloride or sodium chloride

144. If the W is the work output of a heat engine and H is the total heat input of an engine,
then determine its thermal efficiency, e.
Answer: e = W/H

145. Which of the following items is not important when using a halide torch?
Answer: Adjust to a clear white flame

146. A dehumidifier is usually a small hermitic refrigerating system. It has both a condenser
and an evaporator. Many older systems use R – 12 or R – 500. The newer units are:
Answer: R – 134a

147. What is the method used to evaluate all welds performed on pressure parts of boiler tube
materials?
Answer: Radiographic test

148. Faulty F – 12 compressor valves will be indicated by:


Answer: All of the other choices
(Gradual or sudden decrease in capacity, compressor running continuously, Low head
pressure – high suction pressure)

149. Heat exchanger in which water flows by gravity over the outside of tubes or plates.
Answer: Baudelot cooler

150. Which of the following is not a sound absorptive material?


Answer: Mirrors

151. The desirable temperature inside an air conditioned auditorium is:


Answer: 20 C

152. An odorless refrigerant whose boiling point varies over a wide range of temperatures.
Answer: Freon 12

153. The cooling water regulator is actuated by


Answer: Pressure of the refrigerant

154. The psychrometric chart in air conditioning determines the


Answer: Moist air conditions
155. Blowback of a safety valve is to prevent:
Answer: Chattering

156. When starting a refrigerant unit, be sure the water side of the condenser is ____.
Answer: Vented

157. A temperature measurement in an ordinary temperature which has constant specific


humidity.
Answer: Dew point temperature

158. What adding oil to a freon system, one must be sure that
Answer: All air is removed from the pump and fitting

159. The primary purpose of a turbine in a fluid is to:


Answer: Extract energy from the flow

160. Which of the following is to be checked as regular monthly maintenance schedule of a


console air conditioning units?
Answer: Duct dampers, registers and diffusers

161. A company is interested to produce a water turbine wherein only little energy is required
or necessary because the guide vanes are to be controlled. The turbine must be a:
Answer: Kaplan turbine

162. To check the lack of refrigerant or clogged refrigerant lines:


Answer: Installing service valves must be necessary

163. Condensate from the non return steam trap is pumped from the condensate return tank of
the:
Answer: Boiler

164. The locus of elevation is:


Answer: Hydraulic Gradient

165. Which of the following cannot be used to describe neutron kinetic energy levels
Answer: Freezing

166. It is important to take some moisture from the air dehumidify it if the relative humidity
reaches high levels. To do this, it requires cooling the air:
Answer: Below its dew point temperature

167. The vapor cycle in thermal efficiency as the refrigeration cycle to the
Answer: COP for a refrigerator
168. A scale trap in a Freon system will be found on the
Answer: Suction side

169. Which of the following is not used as method to measure air velocities?
Answer: Open type Barometer

170. Which of the following is considered as comfort condition in air conditioning?


Answer: 20 C DBT, 60% RH

171. Which of the following acts as ignition accelerator for internal combustion engine fuels?
Answer: Acetone peroxide

172. The purpose of the evaporator is to


Answer: Absorb latent heat of vaporization

173. The speed at which a reaction proceeds to equilibrium is the purview of


Answer: Reaction kinetics

174. What do you call the liquid reaching the compressor through the suction?
Answer: Flooding back

175. Which of the following draft rely on the stack effect to draw off combustion gases?
Answer: Natural draft

176. The relative humidity becomes 100% and where the water vapor starts to condense.
Answer: Dew point

177. Which of the following statements is correct? The relative humidity of an air water vapor
mixture.
Answer: is the ratio of partial pressure of water vapor to the saturation pressure at the
mixture temperature

178. The following are standard characteristics of Freon – 11 except:


Answer: Boiling point over 200 F

179. The bhp/ton refrigeration requirement with increase in condenser temperature. In a vapor
compression system that uses reciprocating compressor receiving refrigerant gas at constant
suction temperature, will follow:
Answer: Linearly increasing characteristic
180. In an air conditioning unit, the thermostat fails. The unit did not start. How do you test
the operation of the thermostat?
Answer: Cover the air outlet and air inlet with a cloth. The air will recirculate into the
unit and the temperature will quickly drop to the cut out temperature.

181. All of the following temperature have meaning in psychrometrics except:


Answer: Adiabatic wall temperature

182. The maximum continuous power available from a hydro – electric plant under the most
adverse hydraulic conditions is called
Answer: Firm power

183. Which of the following dehumidifier is often used to reheat the air after moisture is
removed?
Answer: Condenser

184. Which of the following is a type of evaporator?


Answer: shell-and-tube

185. All of the following factors affect rates of reaction except


Answer: Pressure

186. Which of the following would cause a high suction pressure?


Answer: Expansion valve open too wide

187. Zinc rods are found in the:


Answer: Salt water side of the condenser

188. The sum of the internal energy and the product of pressure and specific volume is known
as the:
Answer: enthalpy

189. Any foreign matter in the coal feed mechanism of the screw-feed stoker is best removed
by:
Answer: Using the cutoff gate at the bottom of the hopper

190. Based on the PSME code, what should be the humidity of air to be used for comfort
cooling?
Answer: 50% - 60% relative humidity
191. Which of the following must be checked up if an automatic Freon system will not start
up?
Answer: All of the other choices
(High pressure cutout, reset mechanism, Low-pressure cutout)

192. Refers to the use of composting soil beds


Answer: Biofiltration

193. What is the usual cop of a domestic refrigerator?


Answer: More than 1

194. In a hydro-electric plant using a Francis turbine with medium head, the speed can be
regulated using the
Answer: Wicket gate

195. The temperature bulb of the thermo-expansion valve is attached to which of the
following?
Answer: Evaporator coil outlet

196. Which of the following refrigerants would give the most trouble when operating with
warm circulating water?
Answer: CO2

197. A rule which states that any waste derived from the treatment of a hazardous waste
remains a hazardous waste
Answer: The derived from rule

198. A device for holding open the suction valve and drawing gas from the suction manifold
and returning it to the suction line without compressing it is called
Answer: Cylinder unloader

199. The intake pipe to a hydraulic turbine from a dam


Answer: Penstock

200. What consists of weak solutions of sulfuric, hydrochloric, and to a lesser extent, nitric
acids?
Answer: Acid rain

201. The expansion valve does not seem to be operating properly. There is high superheat.
Test by listening to the sound of Freon flooding through the tubes and ____.
Answer: Warm bulb with hand
202. Swimming pool water needs
Answer: de-chlorination

203. Oxygen is approximately what percent by weight in the atmosphere?


Answer: 23%

204. Which of the following could be used to check a leak in an ammonia system?
Answer: Litmus paper & Sulfur stick

205. The determination of properties and behavior of atmospheric air usually the purview of:
Answer: Psychrometrics

206. A valve the released steam from the boiler by operating at a pre-determined pressure in
order to keep the steam pressure from rising more than the pressure from which the valve is
set is called a
Answer: Safety valve

207. If the critical temperature of a refrigerant is too close to the desired condensing
temperature, the equipment must necessarily be of
Answer: Extra heavy construction

208. If a freon – 12 compressor trip out on “cut-out”, the solenoid valve closes by which of the
following?
Answer: An electrical release

209. Which of the following would cause the suction switch to remain open?
Answer: Bellows broken or jammed

210. In the upper atmosphere, ozone is made by ultraviolet light reacting with:
Answer: Oxygen

211. How is ammonia system purged so that operator will not be overcome by the fumes?
Answer: Into the bucket of water

212. A ton of refrigeration is equal to the removal of


Answer: 288,000 btu per 24 hrs.

213. As a good practical rule, the foundation depth may be taken as how many times of the
engine stroke?
Answer: 2:5 to 3:5

214. Most stacks are built of height less than _____.


Answer: 30 m
215. The phenomenon that warm air rise and cold air settle is called:
Answer: Stratification

216. Type of refrigerant control which maintains pressure difference between high side and
low side pressure in refrigerating mechanism:
Answer: Expansion valve

217. The crossover connection in an ammonia system can be used to ____.


Answer: Hot-gas defrost

218. Which of the following refrigerants has lowest freezing point temperature?
Answer: Freon 22

219. The operation that produces highest noise level is:


Answer: Riveting

220. Select the one in which secondary refrigerant is used


Answer: Ice plant

221. Which of the following is not essential to a centrifugal type compressor system?
Answer: Distiller

222. Two compressor should not be run in parallel because


Answer: There is a possibility of losing oil

223. Which of the following is also known as refrigerant no. R-717?


Answer: Ammonia

224. During sensible heating, the humidity remains constant but the relative humidity.
Answer: decreases

225. The term suction units is used with


Answer: Induced draft

226. If the compressor had been running satisfactorily for a long period of time but the oil
level was rising slowly, one should:
Answer: Shut down the compressor and check the oil level with the machine stopped

227. What amount of air is required in a low bypass factor?


Answer: Lesser

228. The reduction of nuclear radiation intensity (called attenuation) is accomplished by:
Answer: Shielding
229. The ozone concentration of 0.10 parts per million (ppm) is generally considered the
maximum permissible for how many hours exposure?
Answer: 8 Hours

230. Which do you think is very important in adjusting compressor “v” belts?
Answer: Allow ½” slack

231. An engineer inspected an air-conditioning unit. He found out that the unit does not
produce any cooling effect, however, the air-conditioning unit is running. He checked the
temperature of the condenser and evaporator and hat the unit run. He found out that there was
no change in temperature. What should he do?
Answer: Charge with new refrigerant

232. The temperature at which water vapor in the atmosphere begins to condense is known as:
Answer: dew point

233. The ice making capacity is always


Answer: Directly proportional to the refrigerating effect

234. If the compressor short-cycles on the high-pressure cutout, which of the following would
you check?
Answer: All of the other choices
(If plenty of cooling water is running through but it is not picking up heat, the condenser
tubes need cleaning, Be sure system is getting cooling water, Check for too much refrigerant
in the system)

235. What is the instrument used to register relative humidity?


Answer: Hygrometer

236. What has an octane rating of more than 100?


Answer: Benzol

237. A device used to keep moisture from passing through the system is called:
Answer: Dehydrator

238. If the freezing point of water is 0 C, which of the following is its melting point following
is its melting point
Answer: 0 C

239. What is the pressure present inside the casing of an impulse turbine?
Answer: Atmospheric pressure
240. A vena contracta in a fluid jet issuing through a hole in a plate is located approximately:
Answer: At jet’s minimum diameter

241. If the compressor short-cycles on the low-pressure cutout, the trouble might be:
Answer: Any of the other choices
(Too much frost on the evaporator coils, dirty traps and strainers, lack of refrigerant)

242. The amount of heat necessary to bring up temperature of a unit mass of a substance
through unit degree is called
Answer: Total heat

243. If the expansion valve capillary tube is pinched, which of the following be replaced?
Answer: All of the other choices (called the power element unit)
(Bulb, tube, diaphragm)

244. The coefficient of discharge is the ratio of the.


Answer: Actual discharge to the theoretical discharge

245. Pitot tube is used to measure the:


Answer: Velocity of flow

246. The principle of the centrifugal system is based on which of the following?
Answer: Kinetic energy

247. The disadvantage of a CO2 system over an ammonia system is the fact that
Answer: All of the other choices
(The pipes and fittings of a co2 system must be of the high-pressure type, The CO2 system
requires a larger prime remover, The CO2 system operates at a much higher pressure)

248. If ice will form in a solution of water and salt, then it is at temperature called:
Answer: Freezing point depression

249. The suction pressure in a Freon system should be:


Answer: The pressure which corresponds with a temperature about 20F below the
temperature of the icebox.

250. Wb is shaft work of an engine and Wi is indicated work of an engine. If mechanical is


present in the engine mechanism, then.
Answer: Wb is less than Wi

251. What is the relationship of the capacity of a centrifugal pump, Q, to impeller diameter, D,
when there are two impeller diameters in the same pump?
Answer: Q is directly proportional to the ratio of D
252. The combination of enthalpy and kinetic energy of fluid is termed as:
Answer: Stagnation enthalpy

253. Before any repair work is attempted on any gas-fired boiler the:
Answer: Main gas cock must be secured

254. An air-conditioning system in which water is chilled or cooled and which passes the
evaporator coils
Answer: Chilled water system

255. Refers to organic waste produced from biological water waste treatment processes
Answer: Biosolids

256. As far as combustion chamber design is considered, the maximum power output of a
given engine can be increased by:
Answer: Decreasing combustion chamber volume

257. A radioactive gas produce from the decay of radium within the rocks beneath a building
Answer: Radon

258. Ozone filters out what type of radiation that damages crops and causes skin cancer?
Answer: Ultraviolet radiation

259. The ratio of the sum of individual maximum demands of the system to the overall
maximum demand of the whole system
Answer: Diversity factor

260. A bell coleman cycle is also known as


Answer: Reversed Joule cycle

261. It is used deliver concentrated airstreams into a room. Many have one-way or two-way
adjustable air stream deflectors.
Answer: Register

262. The sensible heat ratio is 0.8. that is


Answer: 80% sensible heat and 20% latent heat

263. Closing the solenoid valve will stop the compressor through the ____.
Answer: Low-pressure cutout switch

264. A refrigerating unit of one (1) ton capacity can remove?


Answer: 200 btu’s per min
265. Which of the following is the usual case of slugging?
Answer: Expansion valve not operating properly

266. All of the following occur during reduction of a substance except


Answer: Loss of electrons

267. The secondary refrigerant used in milk chilling plants is generally


Answer: Glycol

268. Where is the solenoid coil installed?


Answer: Vertically over the valve

269. The thermal expansion valve responds to the


Answer: Amount of superheat in the vapor

270. To eliminate transmission of vibration, the foundation should be isolated from the floor
slabs of building footings at least how may mm around its perimeter?
Answer: 25

271. Which one is commonly used liquid absorbent?


Answer: Ethylene glycol

272. The low-pressure control switch:


Answer: Cuts out the compressor to maintain proper flow

273. If the outlet of the thermostatic valve is warmer than the inlet, it indicates
Answer: Thermostatic valve not working properly

274. Compute the wall gain load of a load storage room when A is the outside surface area of
the wall, D is the temperature differential across the wall, and U is the overall coefficient of
heat transmission.
Answer: AUD

275. A leak on the fuel oil suction line between the tank and the suction side of the fuel oil
pump would result in:
Answer: Air entering the suction line

276. How is the CO2 system is purged?


Answer: When CO2 comes out of the purge valve, frost will form on a piece of metal
held near the outlet

277. The relief valve on a CO2 machine is located:


Answer: On the discharge pipe between the compressor and discharge valve
278. The high-pressure side of the system is sometimes referred to as the:
Answer: Hot side

279. Combined pressure of cooling and humidifying is also known as:


Answer: Evaporative cooling process

280. The schedule of a pipe, N, indicates the thickness of the pipe wall. If the allowable stress
of the pipe is S, then what is the internal pressure equal to?
Answer: N x S

281. Which of the following area of work requires lowest noise level?
Answer: Library

282. Which of the following contribute to the deterioration of the earth’s ozone layer?
Answer: Chlorofluorocarbons

283. What is the least number of compressors a multistage system that will use?
Answer: Two

284. Is a soluble compound that reduces a liquid’s surface tension or reduces the interfacial
tension between a liquid and a solid
Answer: Surface – acting agent or surfactant

285. To secure the belts embedded within the foundation, the distance of the edges of the
foundation from the bedplate must be ____.
Answer: 120 mm to 300 mm

286. In Stirling and Ericsson Cycles, the efficiency can be increased by:
Answer: Regeneration

287. The thermal expansion valve


Answer: Controls the quantity of liquid refrigerant going to the evaporator coils

288. Too low suction pressure could be caused by:


Answer: Any of the other choices
(Dirty scale traps, Shortage of refrigerant gas, Too much oil in the system)

289. The static pressure drop due to friction through the boiler and stack.
Answer: Draft loss

290. In a freon-11 system there is no


Answer: Distiller
291. The coefficient of contraction is the ratio of the:
Answer: Area of vena contracta to the orifice area

292. Which of the following should not be used to clean scale traps:
Answer: Cotton waste

293. Most observe properties of light and other radiant energy are consistent with waves in
nature, but in interaction with matter, electromagnetic energy behaves as though it consists of
discrete pieces or
Answer: Quanta

294. Boilers using soft coal must have ____ furnace volume to reduce the danger of ____
Answer: Large; smoking

295. Which of the following would you apply if a person got Freon in his eyes?
Answer: Sterile mineral oil

296. The compressor will run continuously if there is:


Answer: Any of the other choices
(Too heavy a load, Insufficient refrigerant, Air in the system)

297. A hot crankcase and cylinder head accompanied by a low suction pressure would be
caused by
Answer: Insufficient refrigerant

298. One foot water is equal to:


Answer: 62.43 lb/in2

299. The purpose of the oil trap is:


Answer: To remove oil from the refrigerating gas

300. Many pressure gauges on a Freon system have two dials or graduations on one gauge.
The two dial represents:
Answer: Pressure and temperature

301. The faster way to remove frost from a direct expansion finned tube evaporator is to:
Answer: Send hot gas through the coil

302. When ordering an expansion valve which of the following information is necessary?
Answer: Size, tonnage, temperature, and pressure

303. The elements of a thermostat switch are usually of the ____.


Answer: Bimetal type
304. Which of the following characteristics that is not desirable in a refrigerant?
Answer: Low latent heat of vaporization

305. What do bodies at a temperature above absolute zero?


Answer: It emits Thermal Radiation

306. The Horizontal Scale (Abscissa) in the psychrometric Chart represents:


Answer: Dry bulb temperature

307. The thermostatic expansion valve is designed to maintain constant ____.


Answer: Superheat

308. The charging connection in a refrigerating system is located:


Answer: Between the king valve and the solenoid valve

309. The latent heat of fusion of ice is


Answer: 144 Btu

310. The ratio absorbed by the transfer fluid to the original incident energy striking the
collector
Answer: Collector efficiency

311. If m is the mass of dry air and H is the specific enthalpy of the water vapor in air and r is
the humidity ratio, then determine the latent heat of any dry air.
Answer: m(r x H)

312. The dehydrator is located between the:


Answer: Receiver and expansion valve

313. In a vapor compression cycle the lowest temperature is found in


Answer: Evaporator

314. The volume flow passes through a venturimeter is:


Answer: Constant

315. Anchor bolts in a machine foundation should be embedded in concrete of at least how
many time of the bolt diameter?
Answer: 30

316. Which do you think is the effect of “subcooling”?


Answer: It reduces the horsepower per ton of refrigeration
317. When air is saturated, the wet bulb depression is:
Answer: Zero

318. A precooler, if used is located between the:


Answer: Condenser and expansion valve

319. A Kaplan turbine is:


Answer: Low head axial flow turbine

320. The refrigerant temperature is at its maximum just before it enters the ____
Answer: Condenser

321. The machine foundation must have a factor of safety of:


Answer: 5

322. In a refrigerating system, the heat absorbed in the evaporator per kg mass of refrigerant
passing through
Answer: Equals the increase in enthalpy

323. Which of the following is the common classification of ducts?


Answer: All of these
(Conditioned Air – Duct, Recirculating, Fresh – air ducts)

324. All of the following process can be found in a psychrometric chart except:
Answer: Natural convection

325. If an electrically-operated compressor failed to start the cause might be:


Answer: Any of the other choices
(Burned out holding coils in solenoid valve, An open switch, A blown fuse)

326. Five pounds of water heated to raise the temperature 2F requires how many BTU?
Answer: 10 Btu

327. Air circulation in the icebox is accomplished by the use of which of the following?
Answer: Diffuser fans

328. In sensible heating cooling following parameter remains unchanged


Answer: Humidity ratio

329. Venturi meters, pitot static gauges, orifice meters, flow nozzles, and differential
manometers all depend upon the relationship between:
Answer: Flow velocity and pressure
330. It is used to deliver widespread, fan-shaped flows of air into the room
Answer: Diffuser

331. What turbidimeter that gives direct reading in ppm?


Answer: Jackson turbidimeter

332. How much will be removed by one-ton refrigeration unit?


Answer: 200 Btu per min

333. A double-trunk piston is used to:


Answer: All of other choices
(Prevent oil from mixing with the refrigerant, Prevent gas from getting to crankcase, Prevent
oil from mixing with the refrigerant)

334. Purpose of the receiver is to


Answer: Store the refrigerant

335. Latent heat


Answer: Cannot be measured with a thermometer

336. A type of water turbine where a jet of water is made to fall on the blades or buckets and
due to the impulse of water, the turbine starts to move.
Answer: Pelton wheel

337. When does the refrigerant gives-up heat?


Answer: When it condenses

338. A term used to mean the corrective steps taken to return the environment to its original
condition
Answer: Remediation

339. The back – pressure regulating valve:


Answer: Maintains a fixed pressure in the evaporator coils

340. What is the relationship of the horse power of a centrifugal pump, Hp, to the impeller
speed, S, if the pump is at two different rotative speeds?
Answer: Hp is directly proportional to the cube of the ratio of S

341. What must be the value of the available Net Positive Suction Head (NPHS) of a
centrifugal pump compared to its required NPHS to avoid losing priming?
Answer: Available NPHS greater than required NPHS
342. The diesel engine foundation safe soil bearing pressure is:
Answer: 4,890 kg/cm2

343. Aside from maintain appropriate temperature for food cold storage, how is desiccation
minimized or decreased?
Answer: Maintain humidity ratio

344. If C is the capacity of the compressor in a refrigeration system and F is the heat rejection
factor, then calculate the condenser load, L.
Answer: L = C x F

345. The following examples of indirect (secondary) measurements to measure flow rates
using obstruction meters except:
Answer: Weight and mass scales

346. When required, a regulator water valve in refrigerating system should be


Answer: In the water outlet

347. What is the relative humidity when the dew point and dry bulb temperature are equal?
Answer: 100%

348. Compare in refrigerating effect per unit mass of refrigerant circulated for a superheated
cycle that produces useful cooling and a saturated cycle, for the same vaporizing and
condensing temperature
Answer: Greater for a superheated cycle

349. If PV is the power required for a vapor compression refrigeration system, then what is the
power required for an air refrigeration system, assuming that they have the same capacity?
Answer: 5PV

350. What do you call the mixture if the solute particles of a solid suspended in a liquid are
larger than molecules?
Answer: Suspension

351. Lithium bromide is used in refrigeration system in:


Answer: Absorbers

352. The removal of dissolved gas or other volatile component from liquid by exposing the
liquid to air or steam is known as:
Answer: Stripping

353. Which of the following measures the density of salt in water?


Answer: Salimeter
354. In a power driven pump, each piston stroke is displaced by 360 divided the ____.
Answer: Number of cylinders

355. What is the boiling temperature of F 22?


Answer: -40 C

356. What is another name of discharge pressure?


Answer: Head pressure

357. If the compressor were to run out continuously without lowering the temperature, the
trouble would probably be:
Answer: Any of the other choices
(Insufficient refrigerant in the system, Leaky discharge valve, Leaks in the system)

358. The ratio of sum of individual maximum demands of the system to the maximum demand
of the whole system is known as
Answer: Diversity factor

359. The temperature bulb of solenoid valve is attached to the ____.


Answer: Wall of the icebox

360. What must be done to change the direction of rotation of a 440-volt, 3-phase induction
motor?
Answer: Interchange any two power leads

361. In order to remove the fly ashes from the flue gas, which of the following must a power
plant be equipped with?
Answer: Electrostatic precipitator

362. Which of the following is a type of water turbine?


Answer: Pelton

363. In sensible heating the absolute humidity remains constant but the relative humidity:
Answer: Decrease

364. All of the following process can be found on a psychrometric chart except?
Answer: Black body radiation

365. When checking zinc plates in a condenser, one should:


Answer: Clean the plates and renew worn out ones

366. Turbidity in water is due to:


Answer: Finally divided particles of clay, silt and organic matter
367. The force when applied to a mass of one kilogram will give mass an acceleration of one
meter per second for every second called:
Answer: Newton

368. A precooler is sometimes installed between the


Answer: Condenser and expansion valve

369. A type of polymer used for detergents, milk bottles, oil containers and toys
Answer: High density polyethelene (HDPE)

370. The fact that the amount of slightly soluble gas absorbed in a liquid is proportional to the
partial pressure of the gas is known as:
Answer: Henry’s Law

371. What is the pressure at the exit of a draft tube in a turbine?


Answer: Atmospheric

372. What do you call the device that is used as low-pressure control and high-pressure cutout
on a compressor?
Answer: Pressure Controller

373. Redox reactions can often be type of:


Answer: Double replacement

374. Excessive head pressure is caused by


Answer: Any of the other choices
(Dirty condenser tubes, Insufficient cooling water to condenser, Air or non condensable gas
in the system)

375. In which part of the vapor compression cycle there is abrupt change in pressure and
temperature?
Answer: Expansion valve

376. The kinetic energy of a moving fluid is used to isentropically compressed the fluid to
state of zero velocity. The temperature of a moving fluid at the state zero velocity is called:
Answer: Critical temperature

377. Which is not commonly used to cool and dehumidify equipment?


Answer: Calcium chloride

378. What is the primary cause of smog formation?


Answer: Nitrogen oxides
379. The dehydrating agent in a Freon system is usually
Answer: Activated alumina

380. The solenoid valve is actuated by which of the following?


Answer: A magnet

381. Which of the following does not use ambient air for propulsion?
Answer: Turbo-prop

382. Which of the following would not cause high suction pressure?
Answer: Insufficient refrigeration

383. What is the effect of superheating the refrigerant?


Answer: It increase the coefficient of performance

384. The color of the flame of halide torch, in case of leakage of Freon refrigerant, will change
to
Answer: Green

385. If there is too much lube oil in the system what must be done?
Answer: Remove same at once

386. If the thermal bulb becomes loose on the evaporator coils, it will cause ____.
Answer: Improper operation of expansion valve

387. The effect of superheating the refrigerant is to:


Answer: Decrease the COP

388. In power plant electrostatic precipitators is installed between:


Answer: Furnace and chimney

389. Equipment leaks from plant equipment are known as


Answer: Fugitive emissions

390. Per capita consumption of water is generally taken as:


Answer: 150-300 liters

391. What is the heat that is removed from the space to be cooled, which is the same as the
heat absorbed by the cooling coils?
Answer: Refrigerating effect

392. Horsepower per ton of refrigeration is expressed as:


Answer: Cop/4.75
393. What must be done first when opening a single packed stop valve?
Answer: Loosen the packing before opening

394. The amount of CO2 or Freon in a cylinder is measured by


Answer: Weight

395. The coefficient of velocity is approximately


Answer: 0.30 to 0.50

396. Carries needed to deliver air to the conditioned space it is made of sheet of metals like
aluminum, galvanized sheet steel and some structural materials that will not burn
Answer: Duct

397. Which of the following best described a Freon?


Answer: All of the other choices
(Colorless, Odorless, Non-poisonous)

398. Before securing a compressor to do maintenance work on it, be sure to:


Answer: Have spare parts ready and pump down the system

399. A bull’s eye in a full liquid line will appear ____.


Answer: Clear

400. What operates low-pressure cutout switch?


Answer: Bellows

401. A liquid mixture having constant maximum and minimum boiling points. Refrigerants
comprising this mixture do not combine chemically, yet the mixture provides constant
characteristics.
Answer: Azeotropic Mixture

402. Are organic compounds manufactured in oily liquid and solid forms through the late
1970s and subsequently prohibited.
Answer: Polychlorinated biphenyls (PCBs)

403. What is the temperature range of an air-conditioning application where dry air can be
considered ideal gas?
Answer: -10 to 50 C

404. The coefficient of the velocity, Cv accounts for the:


Answer: Small effect on the flow area of contraction, friction, and turbulence
405. During the re-expansion portion of the refrigeration compressor cycle
Answer: The suction valve is closed and the discharge valve is closed

406. Refers to the high-temperature removal of tarry substances from the interior of the carbon
granule, leaving a highly porous structure
Answer: Activated

407. Are by products of reaction between combustion products


Answer: Oxidants

408. Are any solid particulate matter that becomes airborne, with the exception of particulate
matter emitted from the exhaust stack of a combustion process
Answer: Dusts or fugitive dusts

409. The purpose of the dehydrator is to


Answer: Remove moisture from the refrigerant

410. The angle between the stack and guy wire is usually
Answer: 60 degrees

411. A refrigeration that deals with producing temperature of -157 C or lower.


Answer: Cryogenics

412. The purge valve is located:


Answer: In the highest part of the system

413. Thermal expansion valves are usually made of the:


Answer: Diaphragm and Bellows type

414. The purpose of the expansion valve by-pass is to


Answer: Controls the refrigerant to the evaporator in case the automatic valves fail

415. The water regulating valve is operated by the


Answer: None of the other choices
(Compressor discharge pressure, Compressor suction pressure, Compression discharge
temperature)

416. Superheat is heat added ____.


Answer: After all liquid has been changed to vapor

417. The ratio of maximum load to the rated plant capacity is called
Answer: Utilization factor
418. The highest temperature in vapor compression cycle is produced during:
Answer: Compressor discharge

419. Dry analysis is a fractional analysis of the products of combustion which does not
include:
Answer: Water vapor

420. Valves and piping in an ammonia system are made of:


Answer: Iron

421. The locus of elevation to which water will rise in the piezometer tube is termed:
Answer: Hydraulic gradient

422. A substance with surface area per unit of weight, and intricate pore structure, and a
hydrophobic surface
Answer: Adsorbent substance

423. The purpose of the expansion valve is to control the flow of the refrigerant to the
evaporator. The other function is to:
Answer: Reduce the pressure of the liquid refrigerant

424. An ammonia leak will turn litmus paper ____.


Answer: Blue

425. What is the most common freon gas used in reciprocating compressors?
Answer: F-12

426. The empirical coefficient e in machine foundation if not given is assumed


Answer: 0.11

427. The capacity of a centrifugal type compressor is controlled by which of the following?
Answer: Regulating the speed or regulating the suction pressure

428. Antifreeze chemicals are:


Answer: Those that lower down the freezing points of liquids

429. A type of refrigerant that will not damage the ozone layer
Answer: Hydrofluorocarbons (HCF’s)

430. At 0 psig, how may BTU necessary to change 1 lb of water at 212 F of steam?
Answer: 970
431. Modern stacks are seldom built higher than ____.
Answer: 60 m

432. If Pi is the indicated horsepower and Pb is the indicated horsepower of a compressor,


then what is mechanical efficiency, Em, equal to:
Answer: Em = Pi/Pb

433. A refrigeration control that guards the compressor forms overloads brought about by
abruptly increases loads resulting from defrosting, warm products, and others is called:
Answer: Suction hold-back valve

434. When a solvent has dissolve as much as it can, the mixture is called:
Answer: Saturated solution

435. Fan motors in air conditioning usually have:


Answer: 2 or 3 speeds

436. In the upper atmosphere, ozone is made by ultraviolet light reacting with:
Answer: Oxygen

437. Which of the following is the oil used in a refrigeration system?


Answer: Straight mineral oil

438. The following are all examples of indirect (secondary) miscellaneous methods to measure
flow except:
Answer: Positive displacement meters

439. Which of the following is the function of a suction pressure regulating valve?
Answer: Maintains proper back pressure

440. What is the use of back pressure regulating valve?


Answer: Maintains a fixed pressure in the evaporator coils

441. If Wt is the turbine shaft work of a gas turbine unit, Wc in duel, then determine its
thermal efficiency.
Answer: (Wt-Wc) / Q

442. The solenoid valve is controlled by


Answer: The temperature in the icebox

443. Where is the excess refrigerant removed?


Answer: Charging side of the system
444. All of the following units of energy except
Answer: Pascals

445. What usually happened if brine has a high specific gravity?


Answer: It will crystallize

446. There are three basic boiler types, namely;


Answer: Cast-iron, fire-tube and water tube boilers

447. Measurements of a device’s ability to remove atmosphere air from test air.
Answer: Atmospheric Dust Spot Efficiency

448. The normal cut-out setting of a window unit thermostat between:


Answer: 13 C to 16 C

449. The use of water to carry heat occupied spaces


Answer: Hydronic Heating System

450. Reheating process is normally employed in steam plant when:


Answer: Turbine undergoes excessive moisture

451. In the discharge line between the compressor and the condenser one would find:
Answer: High pressure, high temperature gas

452. A good refrigerant should be


Answer: All of the other choices
(Non-poisonous, non-inflammable, non-explosive)

453. What is the relationship of the horse power of a centrifugal pump, Hp, to the impeller
speed, S, if the pump is at two different rotative speeds?
Answer: Hp is directly proportional to the cube of the ratio of S

454. Which of the following is the reason when the crankcase is cooler than the suction line?
Answer: Too much refrigerant

455. Humidity is a measure of which of the following?


Answer: Water vapor content

456. High head turbine is a/an:


Answer: Impulse

457. Critical temperature is that temperature above which


Answer: A gas will never liquefy
458. The fact that a fluid’s velocity increases as the cross-sectional area of the pipe through
which it flow decrease is due to:
Answer: The continuity equation

459. Which of the following is important for evaporator coils?


Answer: It should have air completely surrounding them

460. Which of the following types of air dryers works by absorbing moisture on a solid
desiccant or drying material such as activated alumina, silicone gel, or molecular sieve?
Answer: Deliquescent dryer

461. Which of the following would cause the compressor to run continuously?
Answer: Low pressure switch jammed

462. In a window air conditioning unit which of the following is usually done by the owner?
Answer: Semi-annual cleaning or replacement of filters

463. Which of the following is not similarity between submerged culvert and a siphon?
Answer: Torricelli’s equation holds

464. Where does the final removal of water vapor in an absorption refrigeration system occur?
Answer: Rectifier

465. The constant spending of certain percentage of circulated water in a cooling tower in
order to prevent accumulation of dissolved mineral solids and other impurities in the
condenser water is called
Answer: Bleed-off

466. Which of the following refrigerant is added sometimes to other refrigerant to improve oil
circulation?
Answer: R – 170 (Ethane)

467. The suction pressure switch is operated by which of the following?


Answer: Pressure on a bellow

468. When the air is saturated the wet-bulb depression is:


Answer: Zero

469. Heating and dehumidification can be obtained simultaneously if air is passed through:
Answer: Either a solid absorbent surface or a liquid absorbent spray
470. In cooling cycle, the dry bulb temperature (db) of the air is lowered. When this happens
the relative humidity
Answer: Increase

471. What will happen when the pressure at any point inside a centrifugal pump goes below
the vapor pressure corresponding to the temperature of the liquid?
Answer: Cavitation

472. The ratio between the actual power and the apparent power in any circuit known as the
____ of that circuit
Answer: Power factor

473. Refrigerant leakage from the compressor crankcase is prevented by


Answer: Using shaft seals

474. The matching of scale and full-scale results for a fluid dynamic phenomena involving
compressible fluids requires quality of:
Answer: Mach number

475. Ammonia leaks in the condenser can be detected by:


Answer: Applying litmus paper to the circulating water discharge

476. A swinging support constructed as part of the vessel and that supports the manway cover
when it is unbolted and moved aside.
Answer: Davit

477. Which of the following type valves are not found on a Freon – 12 system?
Answer: Duplex

478. What is the main function of a receiver?


Answer: Store the refrigerant

479. The dividing point between the high pressure and low pressure sides of the refrigeration
cycle occurs at the
Answer: Expansion valve

480. If the solute particles of a solid suspended in a liquid are larger than molecules the
mixture is known as?
Answer: Suspension

481. A mechanism that removes moisture


Answer: Dehumidifiers
482. Is a water soluble organic compound prepared from ammonia. It has significant
biological and industrial usefulness
Answer: Urea or carbamide urea

483. Which of the following fans in air conditioning systems which can be classified as
centrifugal flow
Answer: Propeller fan

484. What is the use of the low – water cutout switch?


Answer: Stops the compressor when there is insufficient cooling water

485. In parallel pipe system originating and terminating in common junctions,


Answer: Pressure drops through each branch are equal

486. When the dry bulb and the wet bulb temperature are identical, the air is said to be:
Answer: Saturates

487. The temperature in which water vapour in the gas begins to condensate in a constant
pressure process.
Answer: Dew point

488. Combination gas/fuel oil burners permit the operator to switch from one fuel to ____
Answer: All of the above
(For economy, Because of a shortage of fuel being used, Because of a failure in the fuel
system being used)

489. Most nuclear particles can react with atoms in several different ways including
Answer: Absorption and scattering

490. What is the effect of superheating the refrigerant?


Answer: It increase the coefficient of performance

491. Boiling temperature of Freon 12 is


Answer: -29.8 C

492. Which of the following would cause low head pressure?


Answer: Too much cooling water and/ or insufficient refrigerant gas

493. At what temperature will water normally turns to steam?


Answer: 212 F

494. What will happen if the expansion valve is opened too wide?
Answer: Liquid will flood back to the compressor
495. All of the following are words used to describe neutron kinetic energy levels except:
Answer: Supersonic

496. If the thermal expansion valve becomes inoperative, the ice boxes will have to be
controlled by the ____.
Answer: Manual expansion valve

497. If the superheat on the suction side of the compressor is increased, what will happen to
the tonnage capacity of the unit?
Answer: Decreases

498. The solenoid valve can be typed as a ____.


Answer: Magnetic soap valve

499. The ratio of fugacity of actual conditions to the fugacity at some reference state is known
as:
Answer: Activity

500. The process that takes place in the evaporator called


Answer: Absorption of the latent heat of vaporization

501. When charging freon system, all the valves should be


Answer: King (liquid) valve

502. If the pressure exerted on a liquid is higher than the saturation corresponding to it
temperature, the liquid is a:
Answer: Sub cooled liquid

503. The most likely cause of high superheat would be:


Answer: Expansion valve closed too much

504. A good refrigerant should have a


Answer: High latent heat

505. What is the device used to protect the compressor from overloading due to high head
pressure
Answer: Overload relay

506. Which of the following it to be checked as part of weekly maintenance schedule of a


console air conditioners?
Answer: Cooling towers
507. Which of the following would not be cause for a refrigerating system to short cycle on hp
cutout?
Answer: Discharge valve leaking

508. A ____ boiler has heat and gases of combustion that pass through tubes surrounded by
water
Answer: Fire tube

509. A type of polymer used for clear bottles.


Answer: Polyvinyl Chloride (PVC)

510. In an ammonia system, the oil gauge must be kept:


Answer: Closed except when checking level oil

511. If a compartment requires the removal of 36,000 BTU per hour, how much is necessary
compressor capacity?
Answer: 3 tons

512. The oil level in the compressor should be checked:


Answer: After a long period of operation

513. Which of the following industries have the highest consumption of water for processing?
Answer: Paper mill

514. What is the lowest temperature to which water could possibly be cooled in a cooling
tower?
Answer: The temperature of adiabatic compression

515. The method of cooling which primarily used where ambient air temperatures are high and
relative humidity is used:
Answer: Swamp cooling
WASTEWATER TREATMENT PLANT

Wastewater - there is no further use, nothing can be done, hence must be disposed
- effluent of unwanted surplus substance arising from application of any process
- any substance w/c requires to be disposed of as being contaminated or otherwise spoiled
Domestic Wastewater – found in household and commercial areas
Indus trial Wastewater – found in factories
Ph - measure of alkalinity and acidity of the wastewater
COD (Chemical Oxygen Demand) – chemical approach to measure the amount of oxygen equivalent to the
portion of organic matter that is susceptible to oxidation by using a strong oxidant
BOD (Biological Oxygen Demand) – biological approach to measure amount of oxygen required by the
aerobic bacteria to oxidize (=EAT) the organic
FOG(Fats, Oil and Grease) – approximation of the amount of organic substances that are extracted for
aqueous solution of suspension by hexane to105C to the initial weight of the sample is measured
Methane Reactor – anaerobic digester where decomposition/breakdown of organic matter is taking place.
Alkalinity – capacity of water to neutralize acids due to carbonate or bicarbonates and expressed in
milligrams per liter of calcium carbonate equivalent
VFA(Volatile Fatty Acids) – biochemical decomposition of organic matter, facultative and a wide variety of
anaerobic bacteria hydrolyze and convert the complex materials to from low-molecular weight compounds
such as short-chain fatty acids bec. They can be distilled at atm. Pressure
TSS (Total Suspended Solids) – measure of the undissolved particulate matter where the dry solids are
captured by filtering a known volume of sample
VSS (Volatile Suspended Solids) – approximation of organic content of the sample where the ratio of the
weight of dry matter lost in heating a sample to 650C to the initial weight of sample measured
Toxicity – irreversible inhibition of both acidogenic and methanogenic bacteria
Influent – wastewater that flows into each unit of operation of the wastewater treatment plant that is
partially or completely treated
Biodegration – breakdown of man-made and naturally occurring compounds to their constituent elements
and compounds by microbial action through different biological renewal cycles
DO METER – used to measure amount of dissolved oxygen in aeration tank

Biological Treatment
Aerobic – treatment process where the microorganisms require oxygen for their metabolism to biodegrade
organic matter
Anaerobic – breakdown/decomposition of almost all kinds of organic matter by the action of a wide range
of microorganisms, principally methane bacteria
Chemical Treatment – use chemicals to destroy or neutralize toxicity and pollution
Physical Treatment – settling, sedimentation, filtration, flotation, evaporation and distillation
ADVANTAGES OF ANAEROBIC TREATMENT
- Smaller Area Required - Low operating cost
- Sludge Production is minimal - Less moving parts
- Biogas can be recovered - Long shutdown possible

PIPING

VISCOSITY - is termed as absolute viscosity and dynamic viscosity.

KINEMATIC VISCOSITY - is a different from absolute viscosity.

PIPE – tube with around cross section conforming to the dimensional requirements for nominal pipe size

TUBE – hollow product of round or any other cross section having continuous periphery

BELL AND SPIGOT JOINT – commonly used joint in cast iron pipe. Each piece is made w/an enlarged
diameter or bell at one end into w/c the plain or spigot end of another piece is inserted when laying. The
joint is then made tight by cement, oakum, lead, or rubber caulked in to the bell round the spigot.
BLACK PIPE – steel pipe that has not been galvanized
BONNET – part of a valve used to guide and supports the valve steam
BULL HEAD TEE – ate, the branch of w/c is larger than the run
BUTT WELD JOINT – a welded pipe joint made with the ends of the two pipes butting each other,
CARBON STEEL PIPE – steel pipe that owes its properties chiefly to the carbon w/c it contains
CHECK VALVE – valve designed to allow a fluid to pass through in one direction only
COMPRESSION JOINT – multi piece joint with cup shaped threaded nuts which when tightened, compress
tapered w so that they form a tight joint on the periphery of the tubing they connect
CROSS-OVER – small fitting with a double offset or shaped like the letter U w/ the ends turned out
EXPANSION LOOP – a large radius bend in a pipeline to absorb longitudinal expansion on the pipe line due
to heat
GALVANIZED PIPE – steel pipe coated with zinc to resist corrosion
GATE VALVE – valve employing a gate, often wedge-shaped, allowing fluid to flow when the gate is lifted
from the seat. Such valves have less resistance to flow than globe valves-stop valve or isolation valve
GLOBE VALVE – one w/a somewhat globe-shaped body w/a manually raised or lowered disc w/c when
closed rests on a seat so as to prevent passage of a fluid
-hole filing service and close regulation of flow
HEADER – a larger pipe or drum into which each of a group of boilers is connected
MANIFOLD – a fitting w/a number of branches in line connecting to smaller pipes. Used largely as an
interchangeable term with header
MEDIUM PRESSURE – when applied to valves and fittings, implies they are suitable for a working pressure
of 862 to 1207 kPa
MILL LENGTH – known as random length. Run if mill pipe is 4880mm to 6000mm in length
RELIEF VALVE – one designed to pen automatically to relieve excess pressure
RUN – a length of pipe made of more than one piece of pipe
SADDLE FLANGE – a flanged curved to fit a boiler or tank and to be attached to a threaded pipe
SCREWED FLANGE – a flange screwed on the pipe which is connected to adjoining pipe
SOCKET WELD – a joint made by use of a socket weld fitting which has a prepared female end or socket for
insertion of the pipe to which it is welded
STREET ELBOW – an elbow with male thread on one end, and female thread on the other end
WROUGHT IRON – an iron refined to a plastic state in a pudding furnace. Characterized by the presence of
about 3%of slag irregularly mixed with pure iron and about 0.50% carbon
WROUGHT PIPE – refers to both wrought steel and wrought iron
GREEN - water BROWN – oil-mineral vegetable or animal,
flammable or combustible
SILVER GRAY –steam
BLACK – other fluids, including drainage pipes
VIOLET – acid&alkalis
unless the drain is to a particular service
LIGHT BLUE- air
SAFETY RED – firefight
LIGHT ORANGE -electricity
SAFETY YELLOW – hazardous service
YELLOW OCHRE-gases

Velocity in Pipeline:

1-2m/s – liquid: pump, BFW 30-80m/s – steam: superheated

0.5-1m/s – liquid: city water 10-20m/s – air: blower

10-40m/s – steam: saturated 10-30m/s – air: compressor | gas: low&high


pressure
50-70-m/s – steam: turbine
1-5m/s – gas: smoke tube
INDUSTRIAL PUMPS
PUMPS – are industrial machine that move any fluid of high volatile chemicals from the source to the
location which the liquid is to be delivered
CENTRIFUGAL PUMP – ideal type of pump, high discharge, low head, high speed, not self priming

ROTARY PUMP – used for pumping low discharge, low head and for pumping viscous liquids like oil

RECIPROCATING PUMP – ideal type of pump, low discharge, high head, low speed and self priming

WATER HORSEPOWER – theoretical amount of energy necessary to raise a given volume of fluid from a
lower to a higher elevation
-power delivered by the pump
-power required to deliver a given developed head with no losses in the pump
Y – STRAINERS – to protect the pump placed in the line at the inlet to pumps, should not less than 40mesh
PRESSURE GAGES – to determine the pump suction and discharge pressure
FLEXIBLE CONNECTORS – used to isolate vibration made by the pump
PUMP CAPACITY – refers to the volume of fluid delivered by the pump per unit time
STATIC HEAD – height of the surface of the water above gauge point
PRESSURE HEAD – load plus gauge pressure or the water surface pressure plus the friction loss
VELOCITY HEAD – head required to produce the flow of water
DYNAMIC HEAD – pressure head plus velocity head
TOTAL SUCTION HEAD – the sum of all heads at suction of the pump
TOTAL DISCHARGED HEAD – the sum of all heads at discharge point
NET POSITIVE SUCTION HEAD – difference between the absolute dynamic pressure of the liquid measured
at the centreline of the pump and the saturation pressure corresponding to the temperature of the liquid
at the same point
TOTAL DYNAMIC HEAD – difference between total discharge head and total suction head
CAVITATION – describes a cycle of phenomena that occurs in flowing liquid because the pressure falls
below the vapour pressure of the liquid, thus releasing liquid vapors on the low pressure area forming
bubbles w/c will collapse into a region of high pressure thus causing implosion
PUMP EFFICIENCIES – sometimes called the mechanical efficiencies of the pump. Ratio of WHP and BHP
BRAKE HORSEPOWER – power required to drive the pump or power input to the pump
THERMAL EFFICIENCY – (for steam driven pump) ratio of heat equivalent of the WHP to the heat supplied
at the steam end
PUMP DUTY – foot-pounds of work done by the pump per million BTU of the steam supplied on the steam
side of the steam driven pump
VOLUMETRIC EFFICIENCY – (for reciprocating pump) the ratio of actual capacity of the pump over the
piston displacement of the pump
PISTON DISPLACEMENT – volume swept off by the piston in one stroke per unit time
SLIP – losses due to packing of the cylinders or leakage between piston and cylinder walls
OVERALL EFFICIENCY – refers to the effectiveness of the conversion of electrical power to WHP
STATIC SUCTION LIFT – the vertical distance in feet from the liquid supply level to the pump centreline, the
pump being above the supply level
STATIC SUCTION HEAD – when the pump is below the liquid supply level, it exists
STATIC DISCHARED HEAD - the vertical distance in feet from the pump centreline to the point of delivery
TOTAL STATIC HEAD – vertical distance in feet, between supply level and the discharged level of the liquid
being handled
DYNAMIC TOTAL SUCTION LIFT – sum of the static suction lift, suction function head, entrance loss in the
suction pipe and the suction velocity head

DYNAMIC TOTAL SUCTION LIFT HEAD – static suction head minus the suction friction head and suction
piping entrance loss plus any pressure in the suction line

OPEN SYSTEM – pump moves a liquid from a source located above or below the pump but open to
atmosphere

CLOSED SYSTEM – which the liquid circuit is not open to atmosphere

AXIAL FLOW – centrifugal pump in which pressure is developed by propelling of faning action of the
vanes of the impeller on the liquid

WATER FLOW METER – measure condensate, feed water, and pump discharge

AIR COMPRESSOR, BLOWERS AND INDUSTRIAL FANS

FANS – used where low pressure, from a few millimetres of 16 to 50 mmHg and comparative to large
volumes are required, lo speeds

BLOWER – is a machine to compress air or gas by centrifugal force to a final pressure not exceeding
0.24 MPa gage. It is not water cooled (35 psi or 2.5 bars and below) as the aided expense of cooling
system is not justified by the slight gain at these pressures.
- raises fermented CO2 pressure from 2” H20 to 5-7psig & necessary to overcome pipe
friction

COMPRESSOR – raises pressure of CO2 gas to 250 psig so that it can be liquefied at 100 F

COMPRESSOR – machine used to compress air or gas to a fluid pressure above 0.24Mpa, water cooled
above 35psi or 2.5bar

AIR COMPRESSOR – machine that decreases the volume and increases the pressure of a quantity of air by
mechanical means

ROTATING AIR COMPRESSOR – used for low and medium pressures, consists of bladed wheel or impeller
that spins inside a closed circular housing

RECEIVER – in an air compressor system is used to reduce the work needed during compression

ATMOSPHERIC DEW POINT (ADP) - is the temperature below which condensation occurs after the air is
expanded again to the atmosphere.

PRESSURE DEW POINT (PDP) – is the temperature below which condensation occurs at the pressure the
air is at

COOLING TOWERS

COOLING TOWER – device for reducing the temperature of a liquid, usually water by bringing it into
contact with airstream where a small portion of the liquid is evaporated and the major portion is cooled
NATURAL DRAFT COOLING TOWERS – uses the natural movement of air in cooling the water
ARTIFICIAL DRAFT COOLING TOWERS – uses fans or blowers to cause air movement in the cooling tower
FORCED DRAFT COOLING TOWER – pressure is exerted at the inlet of air to the tower by the fan
INDUCED DRAFT COOLING TOWER – pressure is exerted between the outlets of air in the tower
- draws the flue gas from the system and sends them up the stack

2-5% - water losses from cooling towers

INDUSTRIAL DRYING
TYPES OF COMMERCIAL DRYERS
- Hearth Dryer - Compartment Dryer/ Tray Dryer
- Tower Dryer - Rotary Dryer
- Tunnel / Kiln Dryer - Infrared Dryer
MATERIAL HANDLING, CLEANING AND MILLING EQUIPMENT
BASIC MATERIAL HANDLING ELEMENTS
- Lifting
- Dragging, Towing & Pushing
- Carrying & Lifting
- Conveying System
MATERIAL HANDLING TECHNOLOGY
- Containerization –covers the broad spectrum of confinement methods that are used for
storage through all phases of the manufacturing process, cycle.
- Fixed-path handling- applies to movement and storage of units loads of material with an
intermittent or a continuous flow over a fixed path from one point to another
- Mobile handling – considers the systems, equipment practices and requirements
- Warehousing- involves a wider range of planning and analysis(location of activity, sizing ,
storage equipment, packaging methods
HOPPERS – available for intermediate storage, containment and scaling of products
CYCLONES – primary function of a cyclone is to separate product from a gas steam, after separation, the
product discharges out the bottom of the cyclone and air discharges through the top exhaust
AFC BATCH –WEIGH SYSTEM – automated flexible conveyor is a natural addition to the company’s multi
purpose spiral feeder conveyor system
CONVEYOR – a mechanical system for transporting materials from one site to another
CHAIN CONVEYOR – used in horizontal transportation of materials where food nourishment chemical,
mining, saw dust and soil industry, grain silos and grain handling
ROLLER MILL – passing the malt between two closely spaced rolls
SILO DISCHARGER – used for discharging of free flowing and sluggish bulk materials from concrete and
steel silos by the help of vibro motor
AUTOMATIC METERING – often used for weighing and recording the weight of the material being used
or being carried by conveyors.

FUELS AND COMBUSTION


Fuel – an organic material composed of chemical elements w/c, in rapid chemical union w/oxygen
produces combustion (coal, baggase, wood, diesel BFO, LPG, methane)
Properties if Fuel Oils

Specific Gravity – ratio bet the weight of any volume of oil at 15C and weight of an equal volume of pure
water at 15C

Heating Value – amount of heat liberated during complete combustion per unit of volume

Viscosity – measure of the fuels resistance to flow, and oils internal friction, its thickness (cSt)

Bottom sediments and water BSW – the amount of impurities present in the fuel and given as % by vol
for water and % by weight for sediments

Ash- residue that will form after combustion due to the presence of impurities

Flash Point – temp at w/c oil will produce vapors w/c will flash in the presence of an external flame

Fire Point – temp at w/c oil vapors will no longer flash but will support continuous flame

Pour Point – temp at w/c an oil will just flow under standard conditions

Sulfur Content – the amount of sulphur in the fuel oil measure as % by weight

MECHANICAL REFRIGERATION

COMPRESSOR – compresses refrigerant vapour and causes it to flow in the system

CONDENSER – the refrigerant condenses while rejecting heat to the cooling medium, which is either air
to water

EXPANSION VALVE – reduces the pressure of the refrigerant so that low temp will be attained, regulates
the flow of the refrigerant to the evaporator

EVAPORATOR –liquid portion of the refrigerant evaporates while absorbing heat from the surrounding

COEFFICIENT OF PERFORMANCE – the ratio of the refrigerating effect to the work of compression

R-502 – azeotropic mixture of R-22 and R-115, popular refrigerant for low-temp refrigeration system

ACCUMULATOR – used to prevent or minimize entry suction line liquid refrigerant into the compressor

BACK PRESSURE – the pressure of low pressure side also known as suction pressure

FLOODED SYSTEM – only amount of refrigerant that is evaporated is fed to the heat transfer area and
liquid level is controlled by a float

LIQUID OVERFEED SYSTEM – system of feeding, mechanically or by gas pressure, low temp saturated
liquid ammonia to the evaporators at a rate which is greater than what is actually being evaporated
STEAM

STEAM – a vapour into which water is converted when heated to its boiling point

GROUPING OF BOILERS

FIRE TUBE BOILER – flue gas inside the tube and heated fluid outside the tube

WATER TUBE BOILER – flue gas outside the tube and heated fluid inside the tube

MAKE-UP WATER – treated water which is added to the system to replace steam and water lost

CONDENSATE – steam that is returned back to its liquid state normally at steam temp

FEEDWATER – water supplied to the boiler and is the sum of the condensate and make-up

BOILER WATER –water inside the boiler

BOILER STORAGE

CO2 RECOVERY INDUSTRIAL PLANT

MAIN PROCESS

-CO2 Collection -Moisture Removal

-CO2 Purification -Liquefaction

-Compression -Distribution

FOAM TRAP –equipment that is made to have CO2 gas passed against fine H20 spray

BLOWER – raises fermented CO2 pressure from 2” H20 to 5-7psig & necessary to overcome pipe friction

SCRUBBER – removes water soluble substances such as ketones, alcohol and sulphur compounds

DEODORIZER – removes non-water soluble substance from CO2 gas/ one for regeneration while other is
in service

COMPRESSOR – raises pressure of CO2 gas to 250 psig so that it can be liquefied at 10F

SURGE TANK – vertical cylindrical tank which acts as high pressure CO2 gas receiver

PRE-COOLER – cools CO2 gas with glucose as cooling medium and removes a considerable amount of
moisture by condensation

DRYER - removes moisture from CO2 gas to prevent freeze-up of CO2 condenser

CONDENSER – shell and tube heat exchanger used to liquefy CO2 gas, reduces volume if CO2 to storage

STORAGE TANK – mild steel tank located below condensers to reliquefy CO2 evaporating in the tank

VAPORIZER – converts liquid CO2 gas for use in the brewery, vaporized by warm water
CHAPTER 3 – DIESEL PLANT D. Fourth stroke 12. The portion of the piston which extends below the
piston pin and serves as a guide for the piston and
1. An engine in which the fuel is burned directly Answer: A connecting rod.
within the working cylinder. A. Piston skirt
A. Internal combustion engine 7. Compressing air in a four stroke cycle is B. Piston ring
B. External combustion engine A. First stroke C. Piston scoring
C. Compression ignition engine B. Second stroke D. Piston seizure
D. Spark ignition engine C. Third stroke
D. Fourth stroke Answer: A
Answer: A
Answer: B 13. Rings located in the grooves of the piston usually
2. An engine in which the fuel is burned outside of near the top and near the bottom.
the power cylinder. A. Piston skirt
A. Internal combustion engine B. Piston ring
B. External combustion engine 8. Which of the following strokes is produced by the C. Piston scoring
C. Compression ignition engine burning gases: D. Piston seizure
D. Spark ignition engine A. First stroke
B. Second stroke Answer: B
Answer: B C. third stroke
D. fourth stroke 14. Binding of the piston and the cylinder wall as a
3. An engine where the ignition is caused by heat of result of the lubrication having been destroyed by
compression. Answer: C excessive temperature and friction.
A. Internal combustion engine A. Piston skirt
B. External combustion engine 9. Which of the following strokes expels the burned B. Piston ring
C. Compression ignition engine gases? C. Piston scoring
D. Spark ignition engine A. First stroke D. Piston seizure
B. Second stroke
Answer: C C. Third stroke Answer: D
D. Fourth stroke
4. What temperature is required to ignite the fuel oil? 15. Grooves in the cylinder wall or piston or in both. It
A. 800 to 1000F Answer: D is caused by the piston scraping the cylinder wall
B. 700 to 900F in its movement without proper lubrication.
C. 900 to 1100F 10. Clearing a cylinder of exhaust gases by forcing A. Piston skirt
D. 1000 to 1200F into it a current of air which provides clean air for B. Piston ring
the next compression stroke. C. Piston scoring
Answer: A A. Scavenging D. Piston seizure
B. Supercharging
5. What air pressure is required to produce the C. Choking Answer: C
required ignition temperature? D. Knocking
A. 350 to 500 psi 16. A device which automatically governs or controls
B. 250 to 400 psi Answer: A the speed of an engine.
C. 450 to 600 psi A. Servomotor
D. 150 to 300 psi 11. Increasing the total amount of charging air in the B. Governor
working cylinder of the engine. C. Indicator
Answer: A A. Scavenging D. Speedometer
B. Supercharging
6. In a four stroke Diesel engine, drawing air into the C. Choking Answer: B
cylinder is what stroke? D. Knocking
A. First stroke 17. A combination of liquids which do not mix - or
B. Second stroke Answer: B combine chemically.
C. Third stroke A. Emulsion
B. Deposition 23. A chamber so proportioned with respect to the Answer: A
C. Evaporation clearance volume proper of the cylinder that only
D. Separation about 30% of the combustion takes place within 28. What are the exhaust gas temperatures at normal
the chamber itself. operating conditions?
Answer: A A. Pre combustion chamber A. Between 400 and 700F
B. Separate combustion chamber B. Between 500 and 800F
18. What temperature should be the maximum to C. Ante combustion chamber C. Between 300 and 600F
which lubricating oil is permitted to rise? D. Mixed combustion chamber D. Between 200 and 500F
A. Not more than 100F
B. Not more than 200F Answer: A Answer: A
C. Not more than 300F
D. Not more than 400F 24. The entire charge is ignited in the separate 29. How are high speed Diesel engines started?
combustion chamber before the initial expansion A. By an electric motor
Answer: A takes place, forcing the burning gases through the B. By compressed air
connecting passages and against the moving C. By diesel fuel
19. What air pressure is needed for air starting a piston. D. By an electric spark
Diesel engine? A. Pre combustion chamber
A. 250 psi B. Separate combustion chamber Answer: A
B. 150 psi C. Ante combustion chamber
C. 350 psi D. Mixed combustion chamber 30. What speeds are considered high speed diesel
D. 450 psi engine?
Answer: B A. 100 to 2000 rpm or 3000 rpm
Answer: A B. 100 to 1000 rpm
25. A chamber so designed that injection takes place C. 100 to 1500 rpm
20. At what temperature will self-igniting cartridges directly opposite its outlet, the chamber extending D. 100 to 1750 rpm
ignite? backward from the outlet.
A. About 200F A. Pre combustion chamber Answer: A
B. About 190F B. Separate combustion chamber
C. About 210F C. Ante combustion chamber 31. It is a device used for atomizing or cracking fuel oil
D. About 250F D. Air cell chamber and through which the fuel oil Is injected into the
working cylinders of Diesel engines.
Answer: B Answer: C A. Atomizer
B. Fuel spray nozzle
21. The fuel is supplied by one pump and switched to 26. Fuel is injected only into the main cylinder, during C. Injector
each cylinder by a multi outlet rotating valve or expansion of the burning gases in the main D. Cracker
distributor. chamber, when the pressure therein drops below
A. Distributor system that of the air in the cell. Answer: A
B. Injector system A. Pre combustion chamber
C. Non injector system B. Separate combustion chamber 32. What is the proper seat width of a spray valve?
D. Non distributor system C. Ante combustion chamber A. 1/16 in.
D. Air cell chamber B. 1/8 in.
Answer: A C. 1/32 in.
Answer: D D. ¼ in.
22. This is virtually a distributor two stage pumping
system that is, modified distributor system. 27. What is the temperature of the air in the cylinder Answer: A
A. Distributor system when the Diesel engine is operating at full load?
B. Injector system A. Between 800 and 1100F 33. A cylindrical vessel connected in the fuel system
C. Non injector system B. Between 800 and 1000F to absorb the shock of the injection pumps and to
D. Non distributor system C. Between 900 and 1200F provide a reservoir at high pressure for supplying
D. Between 900 and 1100F an even flow of fuel oil to the cylinders.
Answer: B A. Absorber
B. Accumulator 39. The ratio of the clearance volume to the swept C. 14 to 19
C. Governor volume. D. 10.5 to 14.5
D. Injector A. Fractional clearance
B. Compression ratio Answer: A
Answer: B C. Expansion ratio
D. Cut off ratio 45. In standard Diesel, thermal efficiency was not
34. How hot are the exhaust gases? much higher than about what percent?
A. 400 to 700F Answer: A A. 35%
B. 300 to 600F B. 30%
C. 200 to 500F 40. The piston is at maximum reach from the C. 40%
D. 500 to 800F crankshaft to which of the following? D. 50%
A. Top dead center
Answer: A B. Bottom dead center Answer: A
C. ¾ from top center
35. Which of the following is an automatic device used D. ¾ from bottom center 46. A closed heat exchanger that transfers heat from
for keeping constant air pressure? compressed air to cooler air.
A. Relief valve Answer: A A. Regenerator
B. Unloader B. Intercooler
C. Strainer 41. The piston is closest to the crankshaft to which of C. Aftercooler
D. Barometer the following? D. Reheater
A. Top dead center
Answer: B B. Bottom dead center Answer: C
C. ¾ from top center
36. What is the other term of the diameter of the D. ¾ from bottom center 47. Which of the following is used to improve cold
circular cylinder? weather starting?
A. Bore Answer: B A. Poppet valve
B. Stroke B. Glow plug
C. Swept volume 42. Which of the following engines where the C. Check valve
D. Clearance expanding combustion gases act on one end of D. Spark plug
the piston?
Answer: A A. Single acting engine Answer: B
B. Double acting engine
37. What do you call the maximum distance traveled C. Single expansion engine 48. The rated power that the manufacturer claims the
by the piston? D. Double expansion engine engine is able to provide on a continuous basis
A. Bore without incurring damage.
B. Stroke Answer: A A. Brakepower
C. Swept volume B. Continuous duty rating
D. Clearance 43. Which of the following types of engine where the C. Intermittent rating
expanding combustion gases act on both ends of D. Power rating
Answer: B the piston?
A. Single acting engine Answer: B
38. Which of the following is the product of the B. Double acting engine
cylinder area and stroke? C. Single expansion engine 49. What is the peak power rating that can be
A. Bore D. Double expansion engine produced on an occasional basis?
B. Stroke A. Brakepower
C. Swept volume Answer: B B. Continuous duty rating
D. Clearance C. Intermittent rating
44. The compression ratio of a Diesel engine varies D. Power rating
Answer: C from about:
A. 13.5 to 17.5 Answer. C
B. 8 to 10
50. Which of the following is the value of a property 5. A regenerator in a gas turbine: 11. Which of the following turbine has least weight per
that includes the effect of friction? A. Reduces heat loss during exhaust bhp developed?
A. Brake value B. Allows use of higher compression ratio A. Simple open cycle gas turbine
B. Friction value C. Improves thermal efficiency B. Open cycle gas turbine with inter-cooling and
C. Indicated value D. Allows use of fuels of inferior quality reheating
D. Actual value C. Open cycle gas turbine with inter-cooling,
Answer: C reheating and regenerating
Answer. A D. Closed cycle gas turbine
6. Which of the following compressors is generally
CHAPTER 4 – GAS TURBINE used for gas turbines? Answer: A
A. Lobe type
1. Brayton cycle has: B. Centrifugal type 12. Which of the following is/are advantage of closed
A. Two isentropic and two constant volume C. Axial flow type cycle gas turbine over open cycle gas turbine?
processes D. Reciprocating type A. No containing of working substance with
B. Two isentropic and two constant pressure combustion gases
processes Answer: C B. Inferior quality fuel can be used
C. One constant pressure, one constant volume C. Low maintenance costs
and two adiabatic processes 7. The constant pressure gas turbine works on the D. All of the above
D. Two isothermals, one constant volume and a principle of:
constant pressure process A. Carnot cycle Answer: D
B. Bell-Coleman cycle
Answer: B C. Rankine cycle 13. The range of compression ratio in a gas turbine is:
D. Brayton cycle A. 3 to 5
2. Brayton cycle cannot be used in reciprocating B. 5 to 8
engines even for same adiabatic compression Answer: D C. 8 to 12
ratio and work output because: D. 12 to 20
A. Otto cycle is highly efficient 8. What type of gas turbine is used in air craft?
B. Brayton cycle is less efficient A. Open cycle type Answer: B
C. Brayton cycle is for slow speed engines only B. Closed cycle type with reheating
D. Large volume of low pressure air cannot be C. Closed type with reheating and regeneration 14. A constant volume combustion gas turbine
efficiently handled in reciprocating engines D. Open cycle type with reheating, regeneration operates on:
and intercooling A. Ericson cycle
Answer: D B. Joule cycle
Answer: A C. Brayton cycle
3. Which cycle is generally used for gas turbine? D. Atkinson cycle
A. Otto cycle 9. In a gas turbine combined cycle plant, a waste
B. Dual cycle heat boiler is used to: Answer: D
C. Carnot cycle A. Heat air from intercooler
D. Brayton cycle B. Gases from regenerator 15. Past ME Board Question
C. Recover from regenerator Heat exchanger used to provide heat transfer
Answer: D D. None of the above between the exhaust gases and the air prior to its
entrance to the combustor.
4. When r is the compression ratio, the efficiency of Answer: C A. Evaporator
Brayton cycle is given by: B. Combustion chamber
A. 1-1/𝑟 (𝑘−1)/𝑘 10. Overall efficiency of a gas turbine is: C. Regenerator
B. 1-1/𝑟 𝑘−1 A. Equal to Carnot cycle efficiency D. Heater
C. 1-1/𝑟 𝑘 B. Equal to Rankine cycle efficiency
D. 1-1/r C. Less than Diesel cycle efficiency Answer: C
D. More than Otto or Diesel cycle efficiency
Answer: A 16. Past ME Board Question
Answer: C
How does the value for work per unit mass flow of 21. Large units gas turbine regularly operate: 27. The compression ratio based on pressures of
air in the compressor and turbine influenced by the A. In 100 to 200 MW range heavy duty gas turbine is in the range of ____.
addition of a regenerator? B. In 50 to 100 MW range A. 14 to 15
A. Slightly increased C. Over 150 MW B. 19 to 21
B. Unchanged D. Below 150 MW C. 11 to 16
C. Greatly decreased D. 16 to 18
D. Greatly increased Answer: A
Answer: A
Answer: B 22. Small units gas turbine typically have:
A. Double shafts 28. Aeroderivative combustion turbine have higher
17. Past ME Board Question B. Single shafts compression ratios typically:
What is the ideal cycle for gas turbine work? C. Triple shafts A. 14 to 15
A. Brayton cycle D. Quadruple shafts B. 19 to 21
B. Stag combined cycle C. 11 to 16
C. Bottom cycle Answer: B D. 16 to 18
D. Ericson cycle
23. Heavy duty gas turbines typically have: Answer: B
Answer: A A. Double shafts
B. Single shaft 29. Most heavy duty combustion turbines have how
18. Past ME Board Question C. Triple shafts many compression stages?
Brayton cycle cannot be used in reciprocating D. Quadruple shafts A. 14 to 16
engines even for same adiabatic compression B. 18 to 20
ratio and work output because: Answer: B C. 10 to 12
A. Brayton cycle is highly efficient D. 16 to 18
B. Brayton cycle is for low speed engines only 24. Which of the following is basically a jet engine that
C. Brayton cycle needs large air-fuel ratio exhausts into a turbine generator? Answer: D
D. Large volume of low pressure air cannot be A. Aeroderivative gas turbine
efficient handled in reciprocating engines B. Industrial gas turbine 30. The temperature of the gas entering the expander
C. Brayton engine section is typically:
Answer: D D. Joule turbine A. 1200 C to 1290 C
B. 1000 C to 1200 C
19. Past ME Board Question Answer: A C. 1500 C to 1490 C
In order to increase the gas velocity gas turbines D. 1300 C to 1390 C
generally have fixed nozzles. This is to allow the: 25. Most aeroderivative combustion turbine produce
A. Compression of gases less than: Answer: A
B. Condensation of gases A. 20 MW
C. Expansion of gases B. 30 MW 31. The exhaust temperature which makes the
D. Evaporation of gases C. 40 MW exhaust an ideal heat source for combined cycles
D. 50 MW is typically:
Answer: C A. 540 C to 590 C
Answer: C B. 600 C to 650 C
20. Combustion turbines or gas turbines are the C. 300 C to 350 C
preferred combustion engines in application much 26. The compression ratio based on pressures in the D. 440 C to 490 C
above ______. compression stage in a gas turbine is typically:
A. 8MW A. 11 to 16 Answer: A
B. 9mw B. 5 to 8
C. 10MW C. 12 to 18 32. Most combustion turbines have:
D. 7MW D. 8 to 14 A. 2 to 3 expander stages
B. 3 to 4 expander stages
Answer: C Answer: A C. 4 to 5 expander stages
D. 1 to 2 expander stages
Answer: B A. 34 to 36 % Answer: A
B. 30 to 32 %
33. The exhaust flow rate in modern heavy duty C. 40 to 42 % 44. A regenerator in a gas turbine has no effect in:
turbines per 100 MW is approximately: D. 26 to 28 % A. Compressor and turbine work
A. 240 to 250 kg/s B. Heat added
B. 140 to 150 kg/s Answer: A C. Thermal efficiency
C. 340 to 350 kg/s D. Combustor
D. 440 to 450 kg/s 39. New combustion turbines on the cutting edge of
technology ( Advanced turbine systems ) are able Answer: A
Answer: A to achieve
A. 38 to 38.5 % 45. In a Brayton cycle multiple stages of compression
34. The Brayton gas turbine cycle is also known as: B. 36 to 36.5 % and expansion will ________.
A. Joule cycle C. 40 to 40.5 % A. Increase thermal efficiency
B. Stirling cycle D. 34 to 34.5 % B. Decrease thermal efficiency
C. Ericsson cycle C. Limit thermal efficiency
D. Atkinson cycle Answer: A D. Control efficiency

Answer: A 40. Aeroderivative turbines commonly achieve Answer: A


efficiencies up to:
35. Approximately how many percent of the turbine A. 42 % 46. In a Brayton cycle, reheating and intercooling will
power is used to drive the high efficiency B. 38 % ___________.
compressor? C. 45 % A. Increase thermal efficiency
A. 50 to 75 % D. 35 % B. Decrease thermal efficiency
B. 60 to 85 % C. Limit thermal efficiency
C. 45 to 70 % Answer: A D. Control efficiency
D. 30 to 55 %
41. Which of the following is the typical backwork ratio Answer: A
Answer: A of gas turbines?
A. 50 to 75 % 47. In a Brayton cycle, reheating has no effect in:
36. Depending on the turbine construction details, the B. 40 to 65 % A. Heat added
temperature of the air entering the turbine will be C. 30 to 55 % B. Thermal efficiency
between: D. 35 to 60 % C. Backwork ratio
A. 650 C to 1000 C D. Network
B. 750 C to 1100 C Answer: A
C. 550 C to 950 C Answer: A
D. 850 C to 1200 C 42. Which of the following is an example of a
regenerator? 48. In a Brayton cycle, intercooling has no effect in:
Answer: A A. A counterflow heat exchanger A. Turbine work
B. A cross flow heat exchanger B. Thermal efficiency
37. Which of the following engines are typically used C. A mixed flow heat exchanger C. Backwork ratio
by Turbojet and turboprop? D. A parallel flow heat exchanger D. Network
A. Open combustors
B. Closed combustors Answer: B Answer: A
C. Turbo combustors
D. High combustors 43. Which of the following is an effect of having a 49. If Wt is the turbine power and Wc is the
regenerator? compressor power then the backwork ratio is
Answer: A A. Less heat is added A. Wt / Wc
B. Compressor work is reduced B. Wc / Wt
38. The full load thermal efficiency of existing heavy C. Turbine work is increased C. Wt - Wc
duty combustion turbines in simple cycles is D. Compressor work is increased Wt
approximately: D. Wt - Wc
Wc 4. A high pressure chamber or a device in which the Answer: A
paths of rapidly moving particles can be observed
Answer: B and photographed. 10. The lowest permissible water level of a boiler
A. Cloud chamber without internal furnace is _____the height of the
50. If Wt is the turbine power and Wc is the B. Combustion chamber shell.
compressor power then the network is: C. Fission chamber A. 1/2
A. Wt x Wc D. Air chamber B. 1/3
B. Wc / Wt C. 1/4
C. Wt - Wc Answer: A D. 1/5
D. Wt + Wc
5. The formation of gas bubbles in a liquid is called Answer: B
Answer: C A. Bubbling
B. Foaming 11. In case of steam engine the cut off ratio is the ratio
51. Physical limitations usually preclude more than C. Priming of:
how many stages of intercooling and reheating? D. Carryover A. Pressure at cut off to supply pressure
A. 2 B. Pressure at cut off to exhaust pressure
B. 3 Answer: B C. Pressure at cut off to mean effective pressure
C. 4 D. Fraction of piston stroke which the piston has
D. 5 6. How many check valves should be provided traveled when cut off occurs
between any feed pump and boiler?
Answer: A A. 1 Answer: D
B. 2
CHAPTER 5 – STEAM POWER PLANT C. 3 12. In a condensing steam engine
D. 4 A. Condensed steam is supplied
1. Which of the following factors does bursting B. Steam condenses inside cylinder
pressure of boiler doesn’t depend? Answer: B C. Steam condenses as soon as it leaves the
A. Tensile strength of the shell cylinder
B. Thickness of the shell 7. The water level inside the boiler is indicated by the D. Exhaust steam is condensed in a condenser
C. Diameter of the shell A. Baffles
D. Shear strength of shell material B. Fusible plug Answer: D
C. Water walls
Answer: D D. Water column 13. Flows through the nozzles and diffusers with
increasing fluid velocity will create an equivalent
2. Which of the following factors does working Answer: D A. Decrease in the static enthalpy of fluid
pressure of boiler doesn’t depend? B. Increase in the static enthalpy of fluid
A. Tensile strength of shell 8. What is the highest pressure under which C. Decrease in the internal energy of fluid
B. Thickness of shell distinguishable liquid and vapor phases can exist D. Decrease in the dynamic enthalpy of fluid
C. Factor of safety in equilibrium?
D. Type of fuel being fired A. Maximum pressure Answer: A
B. Atmosphere
Answer: D C. Critical pressure 14. The term V 2 /2Cp responds to the temperature rise
D. Peak pressure during such a process and is called the
3. Total solid impurities in feed water for a boiler A. Kinetic temperature
depend upon Answer: C B. High temperature
A. Boiler pressure C. Dynamic temperature
B. Quantity of steam to be generated 9. What is the average fuel – oil temperature range D. Elevation temperature
C. Type of fuel available of the oil in the discharge line to the boiler?
D. Quantity of steam A. 180 – 200 F ͦ Answer: C
B. 240 – 260 F ͦ
Answer: A C. 160 – 180 F ͦ 15. All of the following mechanism can supply heat to
D. 140 – 160 F ͦ a thermodynamic system except
A. Conduction 21. A converging – diverging nozzle is the standard A. Dalton’s line
B. Natural convection equipment in: B. Willan’s line
C. Adiabatic expansion A. Subsonic aircraft C. Jonval’s line
D. Radiation B. Supersonic aircraft D. Rankine line
C. Hypersonic aircraft
Answer: C D. Trisonic aircraft Answer: B

16. The flow through the nozzle is Answer: B 27. An inventor proposes to develop electrical power
A. Isentropic by withdrawing heat from the geyser fields of
B. Polytropic 22. For back pressure valves, abrupt changes in fluid northern California and converting it all to work in
C. Isobaric properties occur in a very thin section of power turbines. The scheme will not work
D. Isovolumic converging – diverging nozzle under supersonic because:
flow conditions, creating A. The geyser fields have only a limited lifetime
Answer: A A. Sound wave B. The salinity of the steam is too great
B. Tidal wave C. It violates the first law of thermodynamics
17. If the reservoir is sufficiently large, the nozzle inlet C. Shock wave D. It violates the second law of thermodynamics
velocity is D. None of these
A. Maximum Answer: D
B. Negative Answer: C
C. Positive 28. The isentropic efficiency of a turbine is given by
D. Zero 23. Is the locus of states which have the same value A. The ratio of actual to ideal energy extracted
of stagnation enthalpy and mass flux are called: B. The ratio of actual to ideal energy inputted
Answer: D A. Fanno line C. The ratio of ideal to actual energy extracted
B. Straight line D. None of the above
18. Which of the following is the pressure applied at C. Willan’s line
the nozzle discharge section? D. Cross cut line Answer: A
A. Stagnant pressure
B. Critical pressure Answer: A 29. Past ME Board Question
C. Back pressure Which of the following is not a main part of a
D. Atmospheric pressure 24. Combination of mass and momentum equations typical coal burner?
into a single equation and plotted in h-s plane yield A. Air registers
Answer: C a curve called: B. Nozzle
A. Fair line C. Atomizer
19. When the back pressure is reduced to lowest exit B. Freh line D. Ignitor
pressure, the mass flow reaches a maximum C. Cutting line
value and the flow is said to be: D. Rayleigh line Answer: C
A. Stacked
B. Choked Answer: D 30. Past ME Board Question
C. Stuck-up Measure of ability of a boiler to transfer the heat
D. Clog-up 25. Generally steam turbines in power station operate given by the furnace to the water and steam is:
at A. Grate efficiency
Answer: B A. 3000 rpm B. Stroke efficiency
B. 1000 rpm C. Furnace efficiency
20. An increase in stagnation pressure will increase C. 4000 rpm D. Boiler efficiency
the mass flux through the: D. 575 rpm
A. Diverging nozzle Answer: D
B. Converging nozzle Answer: A
C. Converging – diverging nozzle 31. Past ME Board Question
D. None of these 26. Which of the following shows the relationship of A goose neck is installed in the line connecting a
the steam consumption and the load of steam steam gauge to a boiler to:
Answer: B turbine generator? A. Maintain constant steam flow
B. Protect the gauge element 36. Past ME Board Question Answer: D
C. Prevent steam knocking The process in which heat energy is transferred to
D. Maintain steam pressure a thermal energy storage device is known as: 41. Past ME Board Question
A. Adiabatic Is one whose pressure is higher than the
Answer: B B. Regeneration saturation pressure corresponding to its
C. Intercooling temperature.
32. Past ME Board Question D. Isentropic A. Compressed liquid
Which of the following is a great advantage of a B. Saturated liquid
fire tube boiler? Answer: B C. Saturated vapor
A. Steam pressure is not ready D. Superheated vapor
B. Contains a large volume of water and 37. Past ME Board Question
requires a long interval of time to raise steam When the boiler pressure increases or when the Answer: A
and not so flexible as to changes in steam exhaust pressure decreases, the amount of
demand moisture 42. Past ME Board Question
C. Cannot use impure water A. Increases In a steam generator with good combustion
D. Radiation losses are higher because fire is B. Decreases control, what occurs if the load is increased?
inside the boiler and surrounded by water C. Constant A. Air temperature leaving air heater decreases
D. Zero B. Air temperature entering air heater increases
Answer: B C. Furnace pressure is approximately constant
Answer: A D. Economizer gas outlet temperature
33. Past ME Board Question decreases
One of the following tasks which is an example of 38. Past ME Board Question
preventive maintenance is When the number of reheat stages in a reheat Answer: C
A. Cleaning the cup on a rotary cup burner cycle is increased, the average temperature
B. Cleaning a completely clog oil strainer A. Increases 43. Past ME Board Question
C. Replacing a leaking valve B. Decreases Total solid impurities in feed water for a boiler
D. Replacing a blown fuse C. Is constant depend upon
D. Is zero A. Boiler pressure
Answer: A B. Type of fuel available
Answer: A C. Quantity of steam to be generated
34. Past ME Board Question D. Quantity of steam
The carbon dioxide ( CO2 ) percentage in the flue 39. Past ME Board Question
gas of an efficiency fired boiler should be A heat transfer device that reduces a Answer: A
approximately thermodynamic fluid from its vapor phase to its
A. 1 % liquid phase such as in vapor compression 44. Past ME Board Question
B. 12 % refrigeration plant or in a condensing steam power The gaseous state of water
C. 18 % plant A. Water gas
D. 20 % A. Flash vessel B. Blue gas
B. Cooling tower C. Water vapor
Answer: B C. Condenser D. Yellow gas
D. Steam separator
35. ME Board October 1993 Answer: C
When droplets of water are carried by steam in the Answer: C
boiler 45. A liquid boils when it vapor pressure equals
A. Priming 40. Past ME Board Question A. The gage pressure
B. Foaming A simultaneous generation of electricity and steam B. The critical pressure
C. Carryover (or heat) in a single power plant C. The ambient pressure
D. Embrittlement A. Gas turbine D. One standard atmosphere
B. Steam turbine
Answer: A C. Waste heat recovery Answer: C
D. Cogeneration
46. Past ME Board Question D. Ion exchange treatment D. Decrease in the temp. and pressure of fluid
What are the main components in a combined
cycle power plant? Answer: B Answer: B
A. Diesel engine and air compressor
B. Gas engine and waste heat boiler 51. Past ME Board Question 56. Past ME Board Question
C. Steam boiler and turbine The thermal efficiency of gas-vapor cycle as How can the average temperature during heat
D. Nuclear reactor and steam boiler compared to steam turbine or gas turbine is rejection process of a Rankine cycle be
A. Greater than decreased?
Answer: B B. Less than A. Increase boiler pressure
C. Lower than B. Increase turbine pressure
47. Past ME Board Question D. Equal to C. Increase condenser pressure
A change in the efficiency of combustion in a D. Reduce turbine exit pressure
boiler can usually be determined by comparing the Answer: A
previously recorded readings with the current Answer: D
readings of the _________. 52. Past ME Board Question
A. Stack temperature and CO A rapid increase in boiler pressure occurs when 57. Past ME Board Question
B. Over the fire draft and CO there is: Which of the following ascertains the effectiveness
C. Ringleman chart and CO2 A. Moderate drop in steam load and the size of a condenser?
D. Stack temperature and CO2 B. Constant drop in steam load A. Number of passes
C. Abrupt drop in steam load B. Thickness of the shell
Answer: D D. Gradual drop in steam load C. Tube sizes
D. Heat transfer
48. Past ME Board Question Answer: C
A boiler steam gauge should have a range of at Answer: D
least 53. Past ME Board Question
A. One half the working steam pressure The most economical and low maintenance cost 58. Past ME Board Question
B. 1 and ½ times the maximum allowable condenser. A boiler has a bursing pressure, BP of 600 kPa
working pressure A. Water – cooled and a factor of safety, FS of 8 is employed in
C. The working steam pressure B. Air – cooled design. As an engineer, would you advice to have
D. Twice the maximum allowable working C. Evaporative a working pressure, WP of 500 kPa?
pressure D. Sub – cooled A. No. WP must be higher than 500 kPa
B. No. WP is only 75 kPa at FS of 8
Answer: B Answer: B C. Yes. Since BP is 600 Pa
D. Yes. To attain better efficiency
49. Past ME Board Question 54. Past ME Board Question
In a water tube boiler, heat and gases of What is commonly done to system when the Answer: B
combustion passed: turbine has excessive moisture?
A. Through the combustion chamber only A. Frosting 59. Past ME Board Question
B. Through the tubes B. Diffusing What cycle is used in vapor cycle of steam power
C. Away from tubes C. Reheating plant?
D. Around the tubes D. Dehumidifying A. Brayton cycle
B. Diesel cycle
Answer: D Answer: C C. Ericsson cycle
D. Rankine cycle
50. Past ME Board Question 55. Past ME Board Question
A chemical method of feedwater treatment which What is the result when the fluid’s kinetic energy Answer: D
uses calcium hydroxide and sodium carbonate as during a stagnation process is transformed to
reagents enthalpy?
A. Thermal treatment A. Decrease in fluids volume
B. Lime soda treatment B. Rise in the temp. and pressure of the fluid 60. Past ME Board Question
C. Demineralization process C. Rise in fluid’s volume Gauge cock in the boiler is designed to determine:
A. Level of steam Answer: B C. X-rays
B. Specific heat D. Photons
C. Level of water 6. A dense, fine grained, light colored igneous rock
D. Pressure which is rich in silica. Answer: C
A. Felsite
Answer: C B. Feldspar 12. In geothermal power plants waste water is:
C. Flint A. Recirculated after cooling in cooling lowers
CHAPTER 6 – GEOTHERMAL PLANT D. Flux B. Discharged into sea
C. Evaporated in ponds
1. Refers to the internal heat from the earth. Answer: A D. Discharged back to earth
A. Geothermal
B. Thermal energy 7. An opening in lava or in volcanic area through Answer: D
C. Molten heat which steam and other hot gases are escaping
D. Tectonic heat into the air is called: 13. Past ME Board Question
A. Fumarole In a liquid-dominated geothermal plant, what
Answer: A B. Volcanic leaks process occurs when the saturated steam passes
C. Seismic outlets through the turbine?
2. A rock – forming crystalline mixed silicate which D. Seismic leaks A. Isobaric
constitute about 60% of the earth’s surface. B. Polytropic
A. Soil Answer: A C. Isometric
B. Feldspar D. Isentropic
C. Flux 8. Tidal power is the power generated from:
D. Flint A. Waves of the ocean Answer: B
B. Rise and fall tides
Answer: B C. Thermal energy of ocean water 14. Past ME Board Question
D. Raw sea water What do you call a conversion technology that
3. A compound rock, a crypto – crystalline form of yields electricity straight from sunlight without the
silica, which is dens, tough, breaking with a Answer: B aid of a working substance like gas or steam
conchoidal fracture. without the use of any mechanical cycle?
A. Gravel 9. Converts chemical energy directly into electrical A. Power conversion
B. Stalactite energy. B. Stirling cycle conversion
C. Flint A. Fuel cell C. Solar thermal conversion
D. Flux B. Magnetohydrodynamic generator D. Photovoltaic-energy conversion
C. Battery
Answer: C D. Thermoelectric generator Answer: D

4. Is the process of using injection wells to bubble air Answer: A 15. Past ME Board Question
through groundwater. Tidal power plant is attractive because it has:
A. Air stripping 10. Generates a voltage from incident light, usually A. Low head and intermittent power
B. Staged combustion light in the visible region. B. High head
C. Sparging A. Photovoltaic cell C. Cheap energy source
D. Soil washing B. Solar cell D. Expensive energy source
C. Dry cell
Answer: C D. A or B Answer: C

5. A popular term used by utilities to mean upgrading Answer: D 16. Past ME Board Question
existing plant. What do you call a conversion technology that
A. Replanting 11. All of the following terms are synonymous with yields electricity straight from sunlight without the
B. Repowering quanta of electromagnetic theory except: aid of a working substance like gas or steam
C. Recharging A. Packets without the use of any mechanical cycle?
D. Reorganizing B. Corpuscles A. Power conversion
B. Stirling cycle conversion 22. The flow process through shock waves is highly 28. Solar energy is captured in:
C. Solar thermal conversion irreversible and cannot be approximated as being: A. Sun capturer
D. Photovoltaic-energy conversion A. Polytropic B. Solar collector
B. Isometric C. Sun collector
Answer: D C. Hyperbolic D. Greenhouse capturer
D. Isentropic
17. Past ME Board Question Answer: B
Tidal power plant is attractive because it has: Answer: D
A. Low head and intermittent power 29. Which of the following solar collectors are
B. High head 23. Beaufort scale is used for measuring what? essentially wide, flat boxes with clear plastic or
C. Cheap energy source A. Beta and gamma radiations glass coverings known as the glazing.
D. Expensive energy source B. Wind speed A. Flat plate collectors
C. Insolation B. Concentrating collectors
Answer: C D. Depth of sea C. Focusing collectors
D. Evacuated tube collectors
18. Tidal power is the power generated from: Answer: B
A. Waves of the ocean Answer: A
B. Rise and fall tides 24. Betz law is widely used in:
C. Thermal energy of ocean water A. MHD systems 30. Which of the following collectors use mirrors
D. Raw sea water B. Solar cells and/or lenses to focus the suns energy on a small
C. Geothermal power plants absorber area.
Answer: B D. Wind mills A. Flat plate collectors
B. Concentrating collectors
19. Converts chemical energy directly into electrical Answer: D C. Non focusing collectors
energy. D. Evacuated tube collectors
A. Fuel cell 25. Rocks having excessive internal stresses may
B. Magnetohydrodynamic generator produce spalling. These rocks are called as ____ Answer: B
C. Battery A. Stratified rocks
D. Thermoelectric generator B. Popping rocks 31. Which of the following collectors are more
C. Crushed rocks complex but their efficiencies are higher?
Answer: A D. Swelling rocks A. Flat plate collectors
B. Concentrating collectors
20. Generates a voltage from incident light, usually Answer: A C. Focusing collectors
light in visible region. D. Evacuated tube collectors
A. Photovoltaic cell 26. Solar energy arrives at the outside of the earth’s
B. Solar cell atmosphere at an average rate of ________. Answer: D
C. Dry cell A. 1.354 kW/m2
D. A or B B. 1.543 kW/m2 32. Which of the following collectors are useful when
C. 2.354 kW/m2 extremely hot transfer fluid is needed and are
Answer: D D. 2.543 kW/m2 generally limited to commercial projects?
A. Flat plate collectors
21. The statement that the product of the error in the Answer: A B. Concentrating collectors
measured determination of a particle’s position C. Focusing collectors
and its momentum is of the order of Planck’s 27. How many percent of solar energy survives D. Evacuated tube collectors
constant h is known as: absorption and reflection?
A. Bohr’s theory A. 40 to 70 % Answer: D
B. D’Alembert’s paradox B. 30 to 60 %
C. The Heisenberg uncertainty principle C. 50 to 80 % 33. The shading factor in calculating the heat
D. Planck’s law D. 20 to 50 % absorbed by the solar collector has a value of
approximately:
Answer: C Answer: A A. 0.95 to 0.97
B. 0.85 to 0.87 Answer: A B. 150 to 250 MW
C. 0.75 to 0.77 C. 100 to 300 MW
D. 0.65 to 0.67 39. Trough electric technology is relatively mature but D. 150 to 400 MW
due to the low temperatures, average annual
Answer: A thermal efficiencies are only: Answer: C
A. 10 to 15 %
34. The ratio of energy absorbed by the transfer fluid B. 5 to 10 % 45. Thermal efficiency of a geothermal power plant is
to the original incident energy striking the collector. C. 15 to 20 % approximately:
A. Collector efficiency D. 25 to 30 % A. 30 %
B. Sun efficiency B. 22 %
C. Shading factor Answer: A C. 35 %
D. Absorptance D. 15 %
40. A field of heliostats or tracking mirrors
Answer: A concentrates solar energy onto a receiver on a Answer: B
central tower.
35. As the difference between the ambient air and A. Trough electric system 46. A flash steam cycle can be used if the hot water
average plate (or inlet) temperatures increases, B. Distributed collector system temperature is approximately:
what happens to the collector efficiency? C. Power tower system A. 165 C or higher
A. Increases D. Dish/Stirling system B. 150 C or higher
B. Decreases C. 145 C to 200 C
C. Constant Answer: C D. Below 200 C
D. No effect
41. In a power tower system typical thermal Answer: A
Answer: B efficiencies is in the range:
A. 10 to 15 % 47. A binary cycle using a separate heat transfer fluid
36. Distributed collector system is also known as: B. 5 to 10 % if the temperature of the hot water is between
A. Trough electric system C. 15 to 20 % approximately _________.
B. Central receiver system D. 25 to 30 % A. 165 C and 120 C
C. Power tower system B. 200 C and 150 C
D. Dish/Stirling system Answer: C C. 225 C and 175 C
D. 300 C and higher
Answer: A 42. A dish engine system has an efficiency of
A. 10 to 15 % Answer: A
37. Central receiver system is also known as: B. 5 to 10 %
A. Trough electric system C. 14 to 19 % 48. For every kilometer of depth, the temperature of
B. Distributed collector system D. 24 to 28 % the earth’ s crust increase by:
C. Power tower system A. 30 F
D. Dish/Stirling system Answer: D B. 40 F
C. 50 F
Answer: C 43. Practical and economic issues limit trough electric D. 20 F
systems to about:
38. Which of the following main approaches to solar A. 200 MW Answer: A
energy generating systems describes that B. 100 MW
parabolic tracking trough concentrators focus C. 300 MW 49. Multiple wells produce steam at 690 to 820 kPa
sunlight on evacuated glass tubes that run along D. 400 MW and temperature of ________.
the collectors focal lines. A. 205 C
A. Trough electric system Answer: A B. 155 C
B. Central receiver system C. 250 C
C. Power tower system 44. Practical and economic issues limit tower electric D. 175 C
D. Dish/Stirling system systems to approximately:
A. 100 to 200 MW Answer: A
50. In a hot rock systems, water is injected through 5. What kind of bonding do common gases that exist 11. An electricity charged atom or radical which
injection wells into artificially made fractured rock in free as diatomic molecules experiences? carries electricity through an electrolyte is called:
beds of how many kilometers below the surface? A. Ionic bonds A. Ion
A. 1 to 6 km B. Convalent bonds B. Isotope
B. 3 to 9 km C. Metallic bonds C. Molecule
C. 4 to 10 km D. Nuclear bonds D. Hole
D. 6 to 11 km
Answer: B Answer: A
Answer: A
6. Which of the following is NOT a unit of energy? 12. What is the smallest particle of an element that
CHAPTER 7 – NUCLEAR PLANT A. Atomic unit can enter into a chemical reaction?
B. MeV A. Molecule
1. Past ME Board Question C. Dynes B. Ion
What is the suggested maximum permissible dose D. Ergs C. Atom
(MPD) of gamma ray exposure for general D. Isotope
individuals not working in a nuclear setting, by Answer: C
choice, in rem/year? Answer: C
A. 1 7. The process in which a nucleus splits into smaller
B. 5 fragments. 13. Beryllium, magnesium, and calcium all belong to
C. ½ A. Fusion which elemental grouping?
D. 3 B. Fission A. Noble elements
C. Photoelectric effect B. Halogens
Answer: C D. Compton’s effect C. Lanthanons
D. Alkaline earth metals
2. Past ME Board Question Answer: B
The number of protons in the nucleus of an atom Answer: D
of the number of electrons in the orbit of an atom 8. Which of the following is NOT a unit of energy?
A. Atomic volume A. Calories 14. The thickness of material required to attenuate
B. Atomic number B. Joules radiation to a particular level depends on
C. Atomic weight C. Pascals A. The particle
D. Atomic mass D. MeV B. The particle energy
C. The shielding material
Answer: B Answer: C D. The particle, its energy, and the shielding
material
3. The process in which a nucleus splits into smaller 9. The ability of a substance to absorb neurons is
fragments. dependent upon which parameter? Answer: D
A. Fusion A. Absorption cross section
B. Fission B. Scattering cross section 15. Particles that are easily stopped within a few
C. Photoelectric effect C. Total cross section millimeter because their double charges generate
D. Compton’s effect D. Atomic number path ionization and because they are susceptible
to electrostatic interaction are:
Answer: B Answer: A A. Alpha neurons
B. Alpha radiations
4. The ability of a substance to absorb neurons is 10. Which of the following is NOT a unit of energy? C. Beta radiations
dependent upon which parameter? A. Atomic unit D. Gamma radiations
A. Absorption cross section B. MeV
B. Scattering cross section C. Dynes Answer: B
C. Total cross section D. Ergs
D. Atomic number 16. Radiation consisting of singly charged particles
Answer: C that generate to intermediate distances are called:
Answer: A A. Fast neurons
B. Alpha radiations Answer: A 27. The total energy of an electron in the same shell is
C. Beta radiations defined by the _________.
D. Gamma radiations 22. Radiation exposure, the measure of gamma A. Principal quantum number
radiation at the surface of an object is measured B. Azimuthal quantum number
Answer: C in: C. Magnetic quantum number
A. Rems D. Hund rule
17. Radiation with no charge, which produces no B. Rads
ionization , and which is difficult to attenuate thus C. Roentegens Answer: A
posing a major health threat is D. Roentegens per second
A. Slow neutrons 28. The direction of an electron’s angular momentum
B. Alpha radiations Answer: C vector is defined by the
C. Beta radiations A. Principal quantum number
D. Gamma radiations 23. Radiation exposure rate, the rate of gamma B. Azimuthal quantum number
radiation at the surface of an object is measured in C. Magnetic quantum number
Answer: D A. Rems D. Electron spin quantum number
B. Rads
18. The property of fluid at a location when the Mach C. Roentegens Answer: C
number is unity (at the throat) are called: D. Roentegens per second
A. Critical properties 29. The electron’s spin angular momentum vector is
B. Sonic properties Answer: D defined by the
C. Dynamic properties A. Azimuthal quantum number
D. Stagnation properties 24. Exposure is a measure of ionization surrounding a B. Magnetic quantum number
person, but biological damage is dependent on the C. Electron spin quantum number
Answer: A amount of energy. D. Hund rule
A. Striking the surface
19. Gamma attenuation is affected by: B. Passing through the body Answer: C
A. Their photoelectric effect C. Absorbed
B. Pair production D. Reflected by the surface 30. The magnitude of an electron’s angular
C. Compton scattering momentum vector is defined by the
D. The photoelectric effect, pair production and Answer: C A. Principal quantum number
Compton scattering B. Azimuthal quantum number
25. All of the following are practical applications of C. Electron spin quantum number
Answer: D Einstein’s principle of special relativity except D. Hund rule
_____.
20. The amount of a radiation shield’s dimensional A. Mass increase Answer: B
geometry that reduces radiation to half of its B. Length contraction
original value is called the _________. C. Time dilation 31. The fact that each orbital of a set of equal-energy
A. Half-value mass D. Space warping orbitals must be occupied with an electron before
B. Half-value thickness any orbitals has two electrons is specified by
C. Semi-cross section Answer: D which of the following?
D. Iogarithmic decrement A. Principal quantum number
26. The postulate that no signal or energy can be B. Azimuthal quantum number
Answer: B transmitted with a speed greater than the speed of C. Magnetic quantum number
light is consistent with _________. D. Hund rule
21. The amount of a radiation shield’s density that A. The Heisenburg uncertainty principle
reduces radiation to half of its original value is B. The Compton effect Answer: D
called the _____________. C. Einstein’s law
A. Half-value mass D. Newton’s second law 32. The statement that no two electrons can have the
B. Half-value thickness same set of four quantum numbers is known as
C. Semi-cross section Answer: C the
D. Iogarithmic decrement A. Hund rule
B. Heisenburg uncertainty principle B. Work function 4. A ________boiler has water in the tubes and heat
C. Pauli exclusion principle C. Coulomb energy gases of combustion passing around the tubes.
D. Schrodinger equation D. Fermi energy A. Fire tube
B. Cast iron sectional
Answer: C Answer: B C. Water tube
D. Firebox
33. All of the following terms describe the radiation 38. Which of the following is not a postulate of Bohr’s
lines from transitions of electrons in an atom theory of the hydrogen atom? Answer: C
except A. Electron orbits are discrete and non-radiating
A. Sharp and an electron may not remain between 5. Which of the following are used in boilers to direct
B. Principal these orbits the gases of combustion over the boiler heating
C. Obtuse B. The energy change experienced by an surface?
D. Fundamental electron changing from one orbit to another is A. Combustion control
quantized B. Baffles
Answer: C C. Light waves exist simultaneously as high C. Fire tubes
frequency electrical and magnetic waves D. Zone controls
34. When electrons are not shared equally between D. Angular momentum is quantized
two elements, the electrons spend more time with Answer: B
one element than with the other, the bonding is Answer: C
called: 6. The three basic type of low pressure heating
A. Ionic bonding CHAPTER 8 – BOILERS boilers are _______, _______ and _______.
B. Polar covalent bonding A. Firebox; vertical; locomotive
C. Non-polar covalent bonding 1. Which of the following is necessary to generate B. Scotch marine; wet-top; dry top
D. Metallic bonding steam in a boiler? C. Straight-tube; bent tube; multiple pass
A. A container D. Fire-tube; water tube; cast iron sectional
Answer: B B. Heat
C. Water Answer: D
35. When electrons are shared equally( e.g. when the D. All of the above
atoms are the same as in diatomic gases) the 7. In a low pressure gas system, the gas regulator
bonding is called: Answer: D reduces that city gas pressure to ____psi.
A. Ionic bonding A. 0
B. Polar covalent bonding 2. It is the part of the boiler with water on one side B. 2
C. Non-polar covalent bonding and heat on the other side C. 1
D. Resonance bonding A. Furnace volume D. 3
B. Fire side
Answer: C C. Heating surface Answer: A
D. Water side
36. All of the following are units of energy except: 8. In the _________air mixes with the fuel and burns.
A. Atomic units Answer: C A. Fire tubes
B. MeV B. Combustion chamber
C. Dynes 3. A _________boiler has heat and gases of C. Water tubes
D. Ergs combustion that pass through tubes surrounded D. Breeching
by water
Answer: C A. Fire tube Answer: B
B. Cast iron sectional
37. Einstein reasoned there was a discrete amount of C. Water tube 9. The function of the steam boiler is to ______.
energy needed to remove an electron from a D. Straight tube A. Produce condensate
surface , with the rest of the incident photon’s B. Create heat
energy contributing to the kinetic energy of the Answer: A C. Produce steam
photon, the amount of energy is called: D. Burn fuel
A. Binding energy
Answer: C Answer: C B. Automatic nonreturn valve
C. Safety valve
10. Which of the following is the most common type of 16. What are the four systems necessary to operate a D. Feedwater stop valve
water tube boiler? boiler?
A. Firebox A. Combustion; draft; steam; boiler water Answer: C
B. Scotch marine B. Water; steam; combustion; stoker
C. Vertical C. Boiler water; fuel; draft; condense 22. The MAWP on a low pressure steam boiler is
D. Straight-tube multiple pass D. Feedwater; fuel; draft; steam _______ psi.
A. 10
Answer: D Answer: D B. 20
C. 15
11. Excess fuel oil in the fuel oil system returns to the: 17. The _________forms a vacuum that draws out of D. 30
A. Combustion chamber the condensate return line and into the vacuum
B. Fuel oil tank tank. Answer: C
C. Burner A. Suction line
D. Suction line B. Main header 23. Which of the following best defines a total force
C. Vacuum pump acting on a boiler?
Answer: B D. Heating equipment A. Area times diameter
B. Area times pressure
12. It is the system that provides the air necessary for Answer: C C. Area times distance
combustion. D. The MAWP times pressure
A. Feedwater 18. Which of the following can be a function of the
B. Steam container in a boiler? Answer: B
C. Draft A. Holds the water
D. Fuel B. Collects the steam that is produced 24. The ASME code states that boilers with over
C. Transfers heat to the water to produce steam _______square feet heating surface must have
Answer: C D. All of the above two or more safety valve.
A. 200
13. The gases of combustion leave the boiler through Answer: D B. 400
the: C. 400
A. Chimney 19. At what temperature will water normally turns to D. 500
B. Blower steam?
C. Air vent A. 100 F ͦ Answer: D
D. Breeching B. 200 F ͦ
C. 150 F ͦ 25. The area of a safety valve 4” in diameter is how
Answer: D D. 212 F ͦ many square inches?
A. 2.3562
14. Which of the following boiler does not use tubes? Answer: D B. 6.2832
A. Cast iron sectional C. 3.1416
B. Water tube 20. Which of the following statements is true for cast D. 12.5664
C. Scotch marine iron sectional boilers?
D. Firebox A. Cast iron sectional boilers have fire tubes Answer: D
B. These boilers do not require water
Answer: A C. Can be expanded in size 26. Which of the following is the only valve permitted
D. None of the above between the safety valve and the boiler?
15. Which of the following will make a boiler work A. Os&y gate
more efficiently? Answer: D B. Os&y globe
A. More fuel is added C. Automatic nonreturn
B. Fire tubes are decrease in size 21. Which of the following is the most important valve D. No valves are permitted between the boiler
C. The heating surface is increased on a boiler? and the safety valve
D. All of the above A. Main steam stop valve
Answer: D B. Suction A. Sewer
C. Duplex B. Atmospheric tank
27. The range of the pressure gauge should be D. Vacuum C. Blowdown tank
_____times the MAWP of the boiler. D. Return tank or open sump
A. 1 to 2 Answer: A
B. 2 to 3 Answer: C
C. 1 ½ to 2 33. Which of the following best defines a vacuum?
D. 2 ½ to 3 A. A pressure above gauge 39. If the desired cut-in pressure of the boiler is 6 psi
B. A pressure below atmospheric and the desired cut-out pressure is 10 psi, the
Answer: C C. A pressure below gauge differential pressure setting must be _____psi.
D. A pressure equal to gauge A. 2
28. The ASME code only allows _____safety valve on B. 6
boiler. Answer: B C. 4
A. Deadweight D. 8
B. Spring-loaded pop-type 34. Safety valves are designed to pop open and stay
C. Lever until there is a ___psi drop in pressure. Answer: C
D. None of the above A. 0 to 1
B. 5 to 15 40. Impurities that build up on the surface of the water
Answer: B C. 2 to 4 in the boiler prevent _____from breaking through
D. Over 15 the surface of the water.
29. How much is the total force on a safety valve 2 ½ A. Air
in diameter and with a steam pressure of 15 psi? Answer: C B. CO2
A. 19.5413 C. Oxygen
B. 73.63125 35. What type of boiler that uses safety valves? D. Steam
C. 29.4525 A. Fire tube
D. 93.7512 B. Cast iron sectional Answer: D
C. Water tube
Answer: B D. All of the above 41. To prevent air pressure from building up in the
boiler when filling the boiler with water, the ____
30. The steam pressure gauge on the boiler is Answer: D must be open.
calibrated to read in: A. Safety valve
A. Inches of vacuum 36. The water column is located at the NOWL so the B. Boiler vent
B. Absolute pressure lowest visible part of the gauge glass is C. Main steam stop valve
C. Pounds per square inch _____above the highest surface. D. Manhole cover
D. Pressure below atmospheric pressure A. 2” to 3”
B. Just Answer: B
Answer: C C. 4” to 5”
D. Never 42. To prevent a vacuum from forming when taking
31. Live steam is prevented from entering the Bourdon the boiler off-line, which of the following must be
tube of the pressure gauge by which of the Answer: A opened when pressure is still on the boiler?
following? A. Safety valve
A. Automatic nenreturn valve 37. Blowback of a safety valve is to prevent: B. Boiler vent
B. Os&y valve A. Burner cycling C. Main steam stop valve
C. Inspector’s test cock B. Premature popping D. Man hole cover
D. Siphon C. Chattering
D. Feathering Answer: B
Answer: D
Answer: C 43. The operating range of the boiler is controlled by:
32. What pressure gauge that can read whether A. Squastat
vacuum pressure or not. 38. The boiler bottom blowdown line should discharge B. Pressure control
A. Compound to a(n)_________. C. Vaporstat
D. Modulating pressure control 49. A ______valve is installed before the _____ to C. 10
allow the steam pressure gauge to be changed if a D. 30
Answer: B malfunction occurs.
A. Globe; safety valve Answer: B
44. The ______ regulates the high and low fire of the B. Globe; U-tube siphon
burner. C. Gate; siphon 55. After the total force of the steam has lifted the
A. Aquastat D. Globe; pigtail siphon safety valve off its seat, the steam enters the
B. Pressure control ______.
C. Vaporstat Answer: C A. Huddling chamber
D. Modulating pressure control B. Steam holding tank
50. What pressure gauge that reads more pressure C. Combustion chamber
Answer: D than is actually in the boiler. D. Main steam line
A. Broken
45. The best time to blow down the boiler to remove B. Uncalibrated Answer: A
sludge and sediment is when the boiler is at: C. Slow
A. Its highest load D. Fast 56. What causes false water level readings in the
B. Half its load gauge glass?
C. Its lightest load Answer: D A. Priming
D. Anytime B. Foaming
51. According to the ASME code, safety valves on low C. Carryover
Answer: C pressure boilers should be tested by hand at least: D. Blowing down
A. Once a month
46. The level of the water in the _____ indicates the B. Once a year Answer: B
water level in the boiler. C. Once a shift
A. Condensate return tank D. Twice a year 57. Fusible plugs are required on ______ boilers by
B. Gauge glass the ASME code.
C. Try cocks Answer: A A. Coal-fired
D. Blowdown tank B. Gas-fired
52. The purpose of the safety valve is prevent the C. Fuel oil-fired
Answer: B pressure in the boiler from: D. All of the above
A. Exceeding its MAWP
47. When blowing down a boiler, the quick-opening B. Causing a boiler explosion Answer: D
valve should always be opened ______and closed C. Dropping below its MAWP
______. D. Relieving water pressure 58. On a pressure control, _______ pressure plus
A. First; first ______pressure equals _______ pressure.
B. Last; first Answer: A A. Differential; cut-out; cut-in
C. First; last B. Cut-in; cut-out; differential
D. Last; last 53. The term applied when a safety valve opens and C. Cut-in; differential; cut-in
closes rapidly. D. Cut-in pressures equals cut-out pressure
Answer: C A. Feathering
B. Chattering Answer: C
48. Which of the following is added to boiler so that C. Pressuring
water changes scale-forming salts into a non- D. Huddling 59. A burner should always start up in ______ fire and
adhering sludge? shut down in ______ fire.
A. Oxygen Answer: B A. Low; low
B. Slag B. High; low
C. Minerals 54. The safety valve on a low pressure boiler opens C. Low; high
D. Chemicals when pressure in the boiler exceeds how many D. High; high
psi?
Answer: D A. 5 Answer: A
B. 15
60. The water in the boiler is heated, turns to steam, Answer: C B. Boiler
and leaves the boiler through the: C. Makeup
A. Feedwater line 66. A ______ valve should be located between the D. Feed
B. Main steam line boiler and the valve on the feedwater line.
C. Main header A. Stop; check Answer: C
D. Main branch line B. Stop; regulator
C. Check; stop 72. Excessive use of cold city water makeup reduces
Answer: B D. Check; regulator overall efficiency because the water must be
______ before use in the boiler.
61. When steams gives up its heat in a heat Answer: A A. Vented
exchanger it turns to: B. Filtered
A. Low pressure steam 67. The feedwater ______ valve opens and closes C. Heated
B. Makeup water automatically. D. Recirculated
C. Condensate A. Return
D. Exhaust steam B. Check Answer: C
C. Bypass
Answer: C D. Equalizing 73. The vacuum pump selector switch is normally in
the position during the heating season.
62. A ________ pump discharges the return water Answer: B A. Float or vacuum
back to the boiler. B. Vacuum only
A. Feed 68. The vacuum pump pumps water and discharges C. Float only
B. Gear air to the: D. Continuous
C. Return A. Expansion tank
D. Vacuum B. Return tank Answer: A
C. Compression tank
Answer: D D. Atmosphere 74. The ______ shuts off the burner in the event of
low water.
63. How do you call a valve that controls the flow of Answer: D A. Low water alarm
water in one direction only? B. Low water fuel cut-off
A. Gate 69. The range of pressure on the vacuum switch is C. Feedwater regulator
B. Os&y usually: D. Automatic low water feeder
C. Globe A. 2 to 6 psi
D. Check B. 2” to 8” Answer: B
C. 6 to 12 psi
Answer: D D. 8” to 12” 75. Water is supplied to the condensate return tank by
the ______ pump.
64. The feedwater _____ valve should be located as Answer: B A. Vacuum
close to the shell of the boiler as practical. B. Feedwater
A. Check 70. Top test the vacuum pump, the operator puts the C. Condensate
B. Nonreturn elector switch in the _____ position. D. Return
C. Stop A. Float
D. Regulating B. Float or vacuum Answer: A
C. Vacuum
Answer: C D. Continuous 76. The feedwater regulator is located at the ______
and is connected to the boiler in the same manner
65. A _________ after each radiator allows _____ to Answer: D as the ______.
pass through to the return line. A. Right side; gauge glass
A. Steam tarp; steam 71. Water added to the boiler to replace water lost due B. NOWL; water column
B. Water trap; steam to leaks and blowing down is known as _____ C. Bottom; blowdown
C. Steam trap; condensate water. D. NOWL; safety valve
D. Water trap; condensate A. Extra
Answer: B Answer: C Answer: C

77. What maintains a constant water level in the 83. When open os&y valve offers ______ to the flow 89. The _______ steam trap is the most common
boiler? of steam. steam trap used.
A. Gauge glass A. No resistance A. Thermostatic
B. Automatic city water makeup feeder B. Velocity B. Inverted bucket
C. Water column C. Throttling action C. Return
D. Feedwater regulator D. Full resistance D. Nonreturn; float

Answer: D Answer: A Answer: A

78. The low water fuel cut-off should be tested: 84. Steam traps are _____ devices. 90. Condensate from the nonreturn steam trap is
A. Daily A. Manual pumped from the condensate return tank of the:
B. Semiannually B. Automatic A. Return tank
C. Monthly C. Electrical B. Feedwater heater
D. Annually D. Semiautomatic C. Vacuum tank
D. Boiler
Answer: A Answer: B
Answer: D
79. The burner should be _____ when the low water 85. What valve should be used for throttling flow of
fuel cut-off is blown down. material? 91. Steam returning to the vacuum tank could cause
A. Off A. Gate the condensate pump to become:
B. Tagged out B. Check A. Waterbound
C. Firing C. Globe B. Waterlogged
D. Tested D. Non-return C. Steambound
D. Steamlogged
Answer: C Answer: C
Answer: C
80. Which of the following valves must be used as 86. Steam traps removed ______ and _____ from the
boiler main steam stop valve? steam lines. 92. Steam strainers should be located on the line
A. Globe A. Air; water _____ the steam trap.
B. Check B. Water; oil A. In front of
C. Gate C. Air; oil B. Above
D. Os&y gate D. Air; steam C. After
D. Along sides of
Answer: D Answer: A
Answer: A
81. Gate valves should always be _____ or _____ 87. Condensate in the steam lines can result in:
closed. A. Carryover 93. As the steam pressure increases the steam
A. Partially open; completely B. Foaming temperature:
B. Completely open; completely C. Priming A. Decreased
C. Wide open; partly D. Water hammer B. Remains the same
D. Throttled; completely C. Increases
Answer: D D. Fluctuates
Answer: B
88. Two types of steam traps are the _____ steam Answer: C
82. Steam header valves should be _____ valves. trap and the _____ steam trap.
A. Globe A. Return; float 94. When the os&y valve is open the stem is in the
B. Check B. Nonreturn; thermostatic ______ position.
C. Gate C. Return; nonreturn A. Floating
D. Os&y gate D. Nonreturn; float B. Up
C. Locked 100. When burning No. 6 fuel oil strainers should be D. None of the above
D. Down cleaned at least once every ______ hours.
A. 8 Answer: C
Answer: B B. 12
C. 10 106. The rotary cup burner uses ______ and _____ to
95. The thermostatic steam trap opens and closes by D. 24 atomize the fuel oil.
a(n): A. High temperature; pressure
A. Float Answer: D B. High temperature; air
B. Electric sensor C. High pressure; steam
C. Inverted bucket 101. When cleaning the fuel oil strainers the _____ D. A spinning cup; high velocity air
D. Flexible bellows must be carefully replaced to prevent ______ from
getting into the system. Answer: D
Answer: D A. Gaskets; fuel oil
B. Cover; water 107. In a low pressure gas burner gas is mixed with air
96. In the float the thermostatic trap the float rises to C. Gaskets; air in the ________.
discharge: D. Cover; fuel oil A. Burner register
A. Condensate B. Combustion chamber
B. Feedwater chemicals Answer: C C. Mixing chamber before the burner register
C. Steam D. Boiler furnace
D. Water and steam 102. The ______ pump draws fuel oil from the fuel oil
tank. Answer: C
Answer: A A. Transfer
B. Condensate 108. In a high pressure gas mixes with the air on the
97. A steam trap that failed to open would cause the C. Fuel oil inside of the:
heating unit to become: D. Circulating A. Burner register
A. Steambound B. Combustion chamber
B. Very hot Answer: C C. Mixing chamber
C. Waterlogged D. Boiler furnace
D. All of the above 103. The ______ valve protects the fuel lines and pump
from excessive pressure. Answer: A
Answer: C A. Safety
B. Relief 109. Regardless of the type of fuel oil used the fuel oil
98. Which of the following whose main function is to C. Bypass must be _____ before it enters the boilers furnace.
remove dirt and impurities that may cause the D. Stop A. Pressurized
steam trap to malfunction? B. Treated
A. Vacuum pumps Answer: B C. Atomized
B. Steam strainers D. Measured
C. Globe valves 104. A high vacuum on the fuel oil suction gauge
D. Steam separators normally indicates either _____ or ______. Answer: C
A. Low viscosity; a clogged vent
Answer: B B. A closed discharged valve; hot fuel oil 110. On the low pressure gas system the manual reset
C. Cold fuel oil; a dirty suction strainer cannot be opened until the:
99. When a steam trap is functioning properly there is D. Water in the fuel oil; a warm pump A. Vaporstat proves pressure
a _______ difference in temperature between the B. Pilot is lighted
trap inlet and discharge. Answer: C C. Boiler is vented
A. 5 F ͦ to 10 F ͦ D. All of the above
B. 20 F ͦ to 30 F ͦ 105. Fuel oil burners deliver the fuel oil to the furnace in
C. 10 F ͦ to 20 F ͦ a: Answer: B
D. No A. Steady stream
B. Half spray, half steam 111. Before any repair work is attempted on any gas-
Answer: C C. Fine spray fired boiler the:
A. Insurance inspector must be notified C. Because of a failure in the fuel system being 122. The _____ is the temperature at which fuel oil
B. Main gas cock must be secured used gives off vapor that flashes when exposed to an
C. State inspector must be notified D. All of the above open flame.
D. Main solenoid valve should be secured A. Fire point
Answer: D B. Pour point
Answer: B C. Flash point
117. Hard coal, known as ______, has a ______ D. Viscosity
112. In a low pressure gas system when the vaporstat content.
proves gas pressure up to the zero- reducing A. Bituminous; high carbon Answer: C
governor the ______ starts up. B. Anthracite; high volatile
A. Induced draft fan C. Bituminous; high volatile 123. The _____ is the temperature at which fuel oil will
B. Pressure D. Anthracite; high carbon burn continuously when exposed to an open
C. Forced draft blower flame.
D. Gas pump Answer: D A. Fire point
B. Pour point
Answer: C 118. Soft coal; known as _____, has a _______ C. Flash point
content. D. Viscosity
113. The butterfly on the low pressure gas system is a A. Bituminous; high carbon
________ valve. B. Anthracite; high volatile Answer: A
A. Slow-opening C. Bituminous; high volatile
B. Screw D. Anthracite; high carbon 124. It is defined as the internal resistance of fuel oil
C. Quick-opening flow.
D. Globe Answer: C A. Fire point
B. Pour point
Answer: A 119. Boilers using soft coal must have ______ furnace C. Flash point
volume to reduce the danger of _____. D. Viscosity
114. The amount of gas supplied to the burner in the A. Small; overfiring
low pressure gas system is controlled by: B. Large; smoking Answer: D
A. Gas pressure C. Small; smoking
B. Steam pressure D. Large; overfiring 125. What must be done in order to lower the viscosity
C. The boiler operator of the fuel oil?
D. The amount of air passing through the Answer: B A. Lower its temperature
butterfly valve B. Increase its temperature
120. When burning soft coal, if the volatile gases reach C. Lower its pour point
Answer: D the heating surface before they have burned D. Increase its pour point
completely _____ and ______ develop.
115. If low water develops in the boiler having a low A. Smoke; soot Answer: B
pressure gas system the ______ will secure the B. Clinkers; ash
gas to the burner. C. Uneven heat; ash 126. A leak on the fuel oil suction line between the tank
A. Whistle valve D. High temperature; smoke and the suction side of the fuel oil pump would
B. Manual reset valve result in:
C. Vaporstat Answer: A A. The suction gauge pulsating
D. Main gas solenoid B. Air entering the suction line
121. The _____ of fuels oil is the lowest temperature at C. Pulsating of the fire in the boiler
Answer: D which it will flow as a liquid. D. All of the above
A. Fire point
116. Combination gas/fuel oil burners permit the B. Pour point Answer: B
operator to switch from one fuel to another C. Flash point
______. D. Viscosity 127. Stokers were developed to:
A. For economy A. Increase efficiency of burning coal
B. Because of a shortage of fuel being used Answer: B
B. Keep furnace temperature constant to protect B. Retort air A. Incomplete
brickwork C. Overfire air B. Perfect
C. Allow for development of larger coal-fired D. Grate zone air C. Complete
boiler D. Imperfect
D. All of the above Answer: C
Answer: A
Answer: D 133. The ram-feed stoker is a(n) ______ stoker.
A. Overfeed 139. Which of the following is required to achieve
128. Any foreign matter in the coal feed mechanism of B. Side feed complete combustion?
the screw-feed stoker is best removed by: C. Travelling grate A. Proper atomization
A. Emptying the coal hopper D. Underfeed B. Proper temperature of fuel
B. Reversing the stoker C. Time of complete combustion
C. Forcing it through with a heavy shear pin Answer: D D. All of the above
D. Using the cutoff gate at the bottom of the
hopper 134. The ram-feed stoker provides: Answer: D
A. Uniform feed
Answer: D B. Quick ash discharge 140. The combustion process must be complete before
C. Even coal distribution the gases combustion come in contact with the:
129. In the event of an obstruction clogging the feed D. All of the above A. Atmosphere
screw a _____ is used to prevent damage to B. Superheater
transmission. Answer: D C. Heating surface
A. Slip clutch D. Main steam outlet
B. Shear pin or key 135. Combustion is the rapid burning of fuel and
C. Fuse oxygen resulting in: Answer: C
D. None of the above A. Release of heat
B. Oxidation 141. Soot buildup on heating surface:
Answer: B C. Release of steam A. Acts as an insulation
D. Nitrogen and oxygen B. Prevents heat transfer
130. In order to bank the stoker it is necessary to C. Increases temperature of gases of
secure the _____ while the _____ continuous to Answer: A combustion
operate. D. All of the above
A. Combination fan; coal feeder 136. What combustion occurs when all the fuel is
B. Coal feeder; forced draft fan burned using the theoretical amount of air? Answer: D
C. Feedwater; induced draft fan A. Incomplete
D. Boiler stop valve; coal feeder B. Perfect 142. The purpose of automatic combustion controls is
C. Complete to control ______ and _______.
Answer: B D. Imperfect A. High fire; low fire
B. Stoker firing; fuel rate
131. In a screw-feed stoker, what draft fan supplies air Answer: B C. Overfire air; fuel mixture
for combustion? D. Proper air to fuel mixture; firing rate
A. Induced 137. It is the burning of all the fuel using the minimum
B. Forced amount of excess air. Answer: D
C. Combination A. Incomplete combustion
D. Natural B. Complete combustion 143. At 0 psig, how many BTU are necessary to
C. Perfect combustion ̊ of steam?
change 1 lb of water at 212 F
Answer: B D. Imperfect combustion A. 144
B. 970
132. To prevent smoke and to aid in complete Answer: B C. 180
combustion in the screw-feed stoker, _______ is D. 1190
provided using a separate damper control. 138. ______ combustion occurs when all the fuel is not
A. Underfire air burned resulting in formation of soot and smoke. Answer: B
144. Radiation is: B. Specific heat Answer: C
A. The movement of liquids or gases created by C. Latent heat
a temperature difference D. Superheat 5. One could expect the possibility of Reynolds
B. The transfer of heat by direct molecular number similarity in all of the following cases
contact Answer: C except:
C. The weight of the substance compared to A. Pumps
unity 150. Heat may be transferred in how many ways? B. Fans
D. A form of heat transfer A. One C. Turbines
B. Three D. Weirs
Answer: D C. Two
D. Four Answer: D
̊ is:
145. Steam at 100 psi and 400 F
A. Saturated Answer: B 6. One could expect the possibility of Froude number
B. Superheated similarity in all of the following cases except:
C. Wet CHAPTER 9 – HYDROELECTRIC PLANT A. Surface ships
D. None of the above B. Surface wave motion
1. In turbine installation, the air is removed from the C. Flow over weirs
Answer: B condenser by ________. D. Closed-pipe turbulent pipe flow
A. Air pump
146. Which of the following most closely defines B. Air injector Answer: D
sensible heat? C. Air ejector
A. An established relationship comparing any D. Air jet 7. One could expect the possibility of Froude number
substance to the heat content of water similarity in all of the following cases except:
B. Heat quantity that can be felt or measured by Answer: C A. Motion of a fluid jet
a thermometer B. Flow over spillways
C. Heat quantity above the point of saturation 2. The primary purpose of a turbine in a fluid loop is C. Surge and flood waves
D. Measure of heat intensity to: D. Subsonic airfoils
A. Add energy to the flow
Answer: B B. Add mass to the flow Answer: D
C. Extract energy from the flow
147. What is required to raise the temperature of 1 lb of D. None of the above 8. In fluid flow, linear momentum is:
water from 32 ̊ to 212 F̊ ? A. A vector quantity equal to the product of
A. 144 Btu Answer: C mass and velocity
B. 970.3 Btu B. A scalar quantity equal to the product mass
C. 180 Btu 3. A vena contracta in a fluid jet issuing through a and velocity
D. Saturated heat hole in a plate is located approximately: C. A scalar quantity equal to the product of force
A. 10 diameters downstream of the hole and length of time is applied
Answer: C B. At jet’s minimum diameter D. The change in impulse
C. At the orifice minimum diameter
148. Increasing the pressure has what effect of the D. At the orifice maximum diameter Answer: A
boiling point of water?
A. No change Answer: B 9. All of the following fluid phenomena are based on
B. Temperature will be raised the force momentum principle of a flowing fluid
C. Temperature will be lowered 4. The following are all examples of indirect except:
D. Temperature will increase 2 F ̊ for every psi (secondary) miscellaneous methods to measure A. Turbines
flow except: B. Pelton wheels
Answer: B A. Turbine and propeller meters C. Diesel automobile engines
B. Magnetic flow meters D. Jet engines
149. Heat absorbed by water when it changes from C. Positive displacement meters
liquid to steam at the boiling point is called: D. Hot-wire anemometers Answer: C
A. Sensible heat
10. The fact that a fluid’s velocity increases as the C. Sum of the coefficient of discharge and the Answer: D
cross sectional area of the pipe through which it coefficient of contraction
flow decreases is due to: D. Difference of the coefficient of discharge and 21. Flow measuring devices include all of the following
A. Bernoull’s equation the coefficient of contraction except:
B. The continuity equation A. Flow nozzles
C. The momentum equation Answer: B B. Venture area meters
D. The perfect gas law C. Pilot tubes
16. Which of the following is not a similarity between a D. Precision tachometers
Answer: B submerged culvert and a siphon?
A. They both operate full Answer: D
11. In the absence of any heat and work interactions B. Toricelli’s equation holds
and any changes in potential energy, the C. Both can experience entrance and exit losses 22. The following are examples of indirect (secondary)
stagnation enthalpy of a fluid remains constant D. In both, the water flows downhill measurements to measure flow rates using
during _______. obstruction meters except:
A. Unsteady flow Answer: B A. Variable area meters
B. Steady flow B. Venture meters
C. Turbulent flow 17. In parallel pipe system originating and terminating C. Volume tanks
D. Variable flow in common junctions, D. Flow nozzles
A. Mass flows through each branch are equal
Answer: B B. Pressure drops through each branch are Answer: C
equal
12. When a falling object reaches a speed at which C. Lengths of each branch are equal 23. The following are examples of indirect (secondary)
the drag force equals its weight, it has achieved: D. Flow areas of each branch are equal measurements to measure flow rates using
A. Mach one obstruction meters except:
B. A laminar boundary layer Answer: B A. Pilot static meters
C. A turbulent boundary layer B. Static pressure probes
D. Terminal velocity 18. Flows through multi-loop systems maybe C. Weight and mass scales
computed by: D. Direction-sensing probes
Answer: C A. Any closed-form solution of simultaneous
equations Answer: C
13. The coefficient of contraction is the ratio of the: B. The Hardy-Cross method
A. Area of vena contracta to the orifice area C. Trial and error 24. In series pipe system, all of the following
B. Actual discharge to the theoretical discharge D. All of the above parameters vary from section to section except:
C. Actual velocity to the theoretical velocity A. Pressure drop
D. Effective head to the actual head Answer: D B. Friction loss
C. Head loss
Answer: A 19. Flow measuring devices include all of the following D. Mass flow
except:
14. The coefficient of discharge is the ratio of the: A. Venturi meters Answer: D
A. Area of vena contracta to the orifice area B. Static pressure probes
B. Actual discharge to the theoretical discharge C. Turbine and propeller meters 25. Venturi meters, pitot static gauges, orifice meters,
C. Actual velocity to the theoretical velocity D. Magnetic dynamometers flow nozzles, and differential manometers all
D. Effective head to the actual head depend upon the relationship between:
Answer: D A. Flow velocity and friction
Answer: B B. Flow velocity and pressure
20. Flow measuring devices include all of the following C. Friction and pressure
15. The coefficient of velocity is equal to the: except: D. Pressure and mass flow
A. Product of the coefficient of discharge and A. Orifice plate meters
the coefficient of contraction B. Hot-wire anemometers Answer: B
B. Actual velocity divided by the theoretical C. Magnetic flow meters
velocity D. Mercury barometers
26. The combination of enthalpy and kinetic energy of A. Reynolds number B. D’Alembert’s paradox
fluid is termed as: B. Weber number C. Newton’s second law
A. Latent enthalpy C. Froude number D. The second law of thermodynamics
B. Heat enthalpy D. Cauchy number
C. Throttling enthalpy Answer: B
D. Stagnation enthalpy Answer: B
37. One could expect the possibility of Reynold’s
Answer: D 32. The matching of scale model and full-scale number similarity in all of the following cases
prototype results for a fluid dynamic phenomena except:
27. The coefficient of velocity, Cv , accounts for the: involving a fully submerged body requires equality A. Submarines
A. Effects on the flow area of contraction, friction of: B. Torpedoes
and turbulence A. Reynolds number C. Seaplane hulls
B. Small effect of friction and turbulence of the B. Weber number D. Supersonic aircraft
orifice C. Froude number
C. Changes in diameters of a converging pipe D. Mach number Answer: C
D. Effects of compressibility
Answer: A 38. The function of a turbine is to:
Answer: B A. Transfer heat from one fluid to another
33. The water hammer phenomenon is primarily what B. Increase the total energy content of the flow
28. Expansion factors take into account the: kind of fluid mechanics? C. Extract energy from the flow
A. Area of the vena contracta A. Static (a phenomena independent of time) D. Exchange heat to increase energy to the flow
B. Small effect of friction and turbulence of the B. Dynamic (a time-dependent phenomena)
orifice C. Compressible Answer: C
C. Changes in diameters of a converging pipe D. Incompressible
D. Effects of compressibility 39. Pitot tube is use to measure the:
Answer: B A. Velocity of mass
Answer: D B. Velocity of pipe
34. All of the following are forms of drag on a body C. Flow
29. The matching of scale model and full-scale results moving through a fluid except: D. Velocity of flow
for a fluid dynamic phenomena with a free surface A. Skin friction
requires equality of: B. Wake drag Answer: D
A. Reynolds number C. Profile drag
B. Weber number D. D’Alembert’s paradox drag 40. Orifice coefficients are used to determine:
C. Froude number A. Energy losses
D. Cauchy number Answer: D B. Energy gains
C. Mass losses
Answer: C 35. The magnitude of the drag coefficient of a sphere D. Energy losses and mass gains
in water is dependent upon all of the following
30. The matching of scale model and full-scale results except: Answer: A
for a fluid dynamic phenomena involving A. Fluid density
compressible fluids requires equality of: B. Fluid velocity 41. The coefficient of velocity is the ratio of the:
A. Reynolds number C. Units of measure (SI or English Engineering A. Area of vena contracta to the orifice area
B. Froude number System) B. Actual discharge to the theoretical discharge
C. Cauchy number D. Drag force C. Actual discharge velocity to the theoretical
D. Mach number discharge velocity
Answer: C D. Effective head to the actual head
Answer: D
36. The fact that there is no resistance to bodies Answer: C
31. The matching of scale model and full-scale moving through an ideal (non-viscous) fluids is
prototype results for a fluid dynamic phenomena known as: 42. The volume flow passes through a venturimeter is:
involving surface tension requires equality of: A. Reynold’s analogy A. Increasing
B. Decreasing 48. Past ME Board Question A. Pelton wheel
C. Constant What is the use of a Hydraulic jump? B. Steam turbine
D. Varying A. Increase the flow rate C. Francis turbine
B. Reduce the flow rate D. Reaction turbine
Answer: C C. Reduce the velocity of flow
D. Reduce the energy of flow Answer: A
43. What must be done to change the direction of
rotation of a 440-volt, 3-phase induction motor? Answer: D 54. Past ME Board Question
A. Reverse lines to start winding The lowest portion to storage basin from where
B. Interchange any two power leads 49. Past ME Board Question the water is not drawn is:
C. Replace any two wires to the rotor A Kaplan turbine is: A. Bottom storage
D. Remove any power leads A. A high head mixed flow turbine B. Sub-soil storage
B. An inward flow impulse turbine C. Spring reserve
Answer: B C. An outward flow reaction turbine D. Dead storage
D. Low head axial flow turbine
44. A draft tube is a part of which power plants? Answer: D
A. Diesel engine power plant Answer: D
B. Gas turbine power plant 55. Past ME Board Question
C. Steam power plant 50. Past ME Board Question In a hydro-electric plant using a Francis turbine
D. Hydro-electric power plant The locus of elevation is: with medium head, the speed can be regulated
A. Critical point using the:
Answer: D B. Hydraulic gradient A. Deflector gate
C. Energy gradient B. Nozzle
45. Past ME Board Question D. Friction gradient C. Wicket gate
In the hydro-electric plant having a medium head D. Weir
and using a Francis turbine, the turbine speed Answer: B
may be regulated through: Answer: C
A. Deflector gate 51. Past ME Board Question
B. Nozzle The locus of elevation to which water will rise in 56. Past ME Board Question
C. Wicket gate the piezometer tube is termed: In the deep well installation or operation, the
D. Forebay A. Energy gradient difference between static water level and
B. Friction head operating water level is known as _______.
Answer: C C. Hydraulic gradient A. Suction lift
D. Critical path B. Drawdown
46. Past ME Board Question C. Priming level
A Francis turbine has what flow? Answer: C D. Clogging
A. Inward flow reaction
B. Outward flow impulse 52. Past ME Board Question Answer: B
C. Outward flow reaction The intake pipe to a hydraulic turbinefrom a dam
D. Inward flow impulse is: 57. Past ME Board Question
A. Tailrace Which of the following is a characteristic of an
Answer: A B. Spiral casing impulse turbine?
C. Surge tank A. Steam striking blades on angle
47. Past ME Board Question D. Penstock B. No steam reaction to velocity
Which of the following is a type of water turbine? C. Steam striking blades at zero angle
A. Parson Answer: D D. Steam reversing direction
B. Hero
C. Pelton 53. Past ME Board Question Answer: C
D. Bankl A type of water turbine where a jet of water is
made to fall on the blades or buckets and due to 58. Past ME Board Question
Answer: C the impulse of water, the turbine starts to move.
What is the pressure at the exit of a draft tube in a C. High head C. Steam striking blades to zero angle
turbine? D. Very low head D. Steam reversing direction
A. Below atmospheric
B. Vacuum Answer: C Answer: C
C. Atmospheric
D. Gage 64. Past ME Board Question 69. Past ME Board Question
High head turbine is a/an: A type of water turbine where a jet of water is
Answer: C A. Impulse made to fall on the blades or buckets and due to
B. Francis the impulse of water, the turbine starts to move.
59. Past ME Board Question C. Reaction A. Pelton wheel
An impulse turbine are used for: D. Propeller B. Steam turbine
A. Low head C. Francis turbine
B. Medium head Answer: A D. Reaction turbine
C. High head
D. Very low head 65. Past ME Board Question Answer: A
The lowest portion to storage basin from where
Answer: C the water is not drawn is: 70. Past ME Board Question
A. Bottom storage The intake pipe to a hydraulic turbine from a dam
60. Past ME Board Question B. Sub-soil storage is:
Which of the following is used as high head C. Spring reserve A. Tailrace
turbine? D. Dead storage B. Spiral casing
A. Impulse C. Surge tank
B. Francis Answer: D D. Penstock
C. Reaction
D. Propeller 66. Past ME Board Question Answer: D
In a hydro-electric plant using a Francis turbine
Answer: A with medium head, the speed can be regulated 71. Past ME Board Question
using the In the hydro-electric plant having a medium head
61. In pumped storage plant A. Deflector gate and using a Francis turbine, the turbine speed
A. Power is produced by pumps B. Nozzle may be regulated through:
B. Water is stored by pumping to high pressures C. Wicket gate A. Deflector gate
C. Pressure accumulators are used D. Weir B. Nozzle
D. Downstream water is pumped upstream C. Wicket gate
during offload periods Answer: C D. Forebay

Answer: D 67. Past ME Board Question Answer: C


In the deep well installation or operation, the
62. Past ME Board Question difference between static water level and 72. Past ME Board Question
What is the pressure at the exit of a draft tube in a operating water level is called ________. A Francis turbine has what flow?
turbine? A. Suction lift A. Inward flow reaction
A. Below atmospheric B. Drawdown B. Outward flow impulse
B. Vacuum C. Priming level C. Outward flow reaction
C. Atmospheric D. Clogging D. Inward flow impulse
D. Gage
Answer: B Answer: A
Answer: C
68. Past ME Board Question 73. Past ME Board Question
63. Past ME Board Question Which of the following is a characteristic of an Which of the following is a type of water turbine?
An impulse turbine has: impulse turbine? A. Parson
A. Low head A. Steam striking blades of angle B. Hero
B. Medium head B. No steam reaction to velocity C. Pelton
D. Bankl 79. The primary purpose of a turbine in a fluid loop is C. Surge and flood waves
to: D. Subsonic airfoils
Answer: C A. Add energy to the flow
B. Add mass to the flow Answer: D
74. Past ME Board Question C. Extract energy from the flow
Hydraulic jump is used to: D. None of the above 85. In fluid flow, linear momentum is:
A. Increase the flow rate A. A vector quantity equal to the product of
B. Reduce the flow rate Answer: C mass and velocity
C. Reduce the velocity of flow B. A scalar quantity equal to the product mass
D. Reduce the energy of flow 80. A vena contracta in a fluid jet issuing through a and velocity
hole in a plate is located approximately: C. A scalar quantity equal to the product of force
Answer: D A. 10 diameters downstream of the hole and length of time is applied
B. At jet’s minimum diameter D. The change in impulse
75. Past ME Board Question C. At the orifice minimum diameter
A Kaplan turbine is: D. At the orifice maximum diameter Answer: A
A. A high head mixed flow turbine
B. An inward flow impulse turbine Answer: B 86. All of the following fluid phenomena are based on
C. An outward flow reaction turbine the force momentum principle of a flowing fluid
D. Low head axial flow turbine 81. The following are all examples of indirect except:
(secondary) miscellaneous methods to measure A. Turbines
Answer: D flow except: B. Pelton wheels
A. Turbine and propeller meters C. Diesel automobile engines
76. Past ME Board Question B. Magnetic flow meters D. Jet engines
The locus of elevation is: C. Positive displacement meters
A. Critical point D. Hot-wire anemometers Answer: C
B. Hydraulic gradient
C. Energy gradient Answer: C 87. The fact that a fluid’s velocity increases as the
D. Friction gradient cross sectional area of the pipe through which it
82. One could expect the possibility of Reynolds flow decreases is due to:
Answer: B number similarity in all of the following cases A. Bernoull’s equation
except: B. The continuity equation
77. Past ME Board Question A. Pumps C. The momentum equation
The locus of elevations to which water will rise in B. Fans D. The perfect gas law
the piezometer tube is termed: C. Turbines
A. Energy gradient D. Weirs Answer: B
B. Friction head
C. Hydraulic gradient Answer: D 88. In the absence of any heat and work interactions
D. Critical path and any changes in potential energy, the
83. One could expect the possibility of Froude number stagnation enthalpy of a fluid remains constant
Answer: C similarity in all of the following cases except: during:
A. Surface ships A. Unsteady flow
78. In turbine installation, the air is removed from the B. Surface wave motion B. Steady flow
condenser by: C. Flow over weirs C. Turbulent flow
A. Air pump D. Closed-pipe turbulent pipe flow D. Variable flow
B. Air injector
C. Air ejector Answer: D Answer: B
D. Air jet
84. One could expect the possibility of Froude number 89. When a falling object reaches a speed at which
Answer: C similarity in all of the following cases except: the drag force equals its weight, it has achieved:
A. Motion of a fluid jet A. Mach one
B. Flow over spillways B. A laminar boundary layer
C. A turbulent boundary layer 95. Flows through multi-loop systems may be B. Static pressure probes
D. Terminal velocity computed by: C. Weight and mass scales
A. Any closed-form solution of simultaneous D. Direction-sensing probes
Answer: C equations
B. The Hardy-Cross method Answer: C
90. The coefficient of contraction is the ratio of the: C. Trial and error
A. Area of vena contracta to the orifice area D. All of the above 101. In series pipe systems, all of the following
B. Actual discharge to the theoretical discharge parameters vary from section to section except:
C. Actual velocity to the theoretical velocity Answer: D A. Pressure drop
D. Effective head to the actual head B. Friction loss
96. Flow measuring devices include all of the following C. Head loss
Answer: A except: D. Mass flow
A. Venturi meters
91. The coefficient of discharge is the ratio of the: B. Static pressure probes Answer: D
A. Area of vena contracta to the orifice area C. Turbine and propeller meters
B. Actual discharge to the theoretical discharge D. Magnetic dynamometers 102. Venturi meters, pitot static gauges, orifice meters,
C. Actual velocity to the theoretical velocity flow nozzles, and differential manometers all
D. Effective head to the actual head Answer: D depend upon the relationship between:
A. Flow velocity and friction
Answer: B 97. Flow measuring devices include all of the following B. Flow velocity and pressure
except: C. Friction and pressure
92. The coefficient of velocity is equal to the: A. Orifice plat meters D. Pressure and mass flow
A. Product of the coefficient of discharge and B. Hot-wire anemometers
the coefficient of contraction C. Magnetic flow meters Answer: B
B. Actual velocity divided by the theoretical D. Mercury barometers
velocity 103. The combination of enthalpy and kinetic energy of
C. Sum of the coefficient of discharge and the Answer: D fluid is termed as:
coefficient of contraction A. Latent enthalpy
D. Difference of the coefficient of discharge and 98. Flow measuring devices include all of the following B. Heat enthalpy
the coefficient of contraction except: C. Throttling enthalpy
A. Flow nozzles D. Stagnation enthalpy
Answer: B B. Venture area meters
C. Pitot tubes Answer: D
93. Which of the following is not a similarity betweena D. Precision tachometers
submerged culvert and a siphon? 104. The coefficient of velocity, Cv accounts for the:
A. They both operate full Answer: D A. Effects on the flow area of contraction, friction
B. Torricelli’s equation holds and turbulence
C. Both can experience entrance and exit losses 99. The following are examples of indirect (secondary) B. Small effect of friction and turbulence of the
D. In both, the water flows downhill measurements to measure flow rates using orifice
obstruction meters except: C. Changes in diameter of a converging pipe
Answer: B A. Variable area meters D. Effects of compressibility
B. Venture meters
94. In parallel pipe system originating and terminating C. Volume tanks Answer: B
in common junctions: D. Flow nozzles
A. Mass flows through each branch are equal 105. Expansion factors take into account the:
B. Pressure drops through each branch are Answer: C A. Area of the vena contracta
equal B. Small effect of friction and turbulence of the
C. Lengths of each branch are equal 100. The following are examples of indirect (secondary) orifice
D. Flow areas of each branch are equal measurements to measure flow rates using C. Changes in diameter of a converging pipe
obstruction meters except: D. Effects of compressibility
Answer: B A. Pitot static meters
Answer: D 111. All of the following are forms of drag on a body C. Flow
moving through a fluid except: D. Velocity of flow
106. The matching of scale model and full-scale results A. Skin friction
for a fluid dynamic phenomena with a free surface B. Wake drag Answer: D
rquires equality of: C. Profile drag
A. Reynolds number D. D’Alembert’s paradox drag 117. Orifice coefficients are used to determine
B. Weber number A. Energy losses
C. Froude number Answer: D B. Energy gains
D. Cauchy number C. Mass losses
112. The magnitude of the drag coefficient of a sphere D. Energy losses and mass gains
Answer: C in water is dependent upon all of the following
except: Answer: A
107. The matching of scale model and full-scale results A. Fluid density
for a fluid dynamic phenomena involving B. Fluid velocity 118. The coefficient of velocity is the ratio of the:
compressible fluids requires equality of: C. Units of measure (SI or English Engineering A. Area of vena contracta to the orifice area
A. Reynolds number System) B. Actual discharge to the theoretical discharge
B. Froude number D. Drag force C. Actual discharge velocity to the theoretical
C. Cauchy number discharge velocity
D. Mach number Answer: C D. Effective head to the actual head

Answer: D 113. The fact that there is no resistance to bodies Answer: C


moving through an ideal (non-viscous) fluids is
108. The matching of scale model and full-scale known as: 119. The volume flow passes through a venturimeter is:
prototype results for a fluid dynamic phenomena A. Reynold’s analogy A. Increasing
involving surface tension requires equality of: B. D’Alembert’s paradox B. Decreasing
A. Reynolds number C. Newton’s second law C. Constant
B. Weber number D. The second law of thermodynamics D. Varying
C. Froude number
D. Cauchy number Answer: B Answer: C

Answer: B 114. One could expect the possibility of Reynolds 120. Which of the following devices used to measure
number similarity in all of the following cases the discharge of outflow from a vessel?
109. The matching of scale model and full-scale except: A. Pitot tube
prototype results for a fluid dynamic phenomena A. Submarines B. Orifice
involving a fully submerged body requires equality B. Torpedoes C. Pump head
of: C. Seaplane hulls D. Obstruction meter
A. Reynolds number D. Supersonic aircraft
B. Weber number Answer: B
C. Froude number Answer: C
D. Mach number CHAPTER 10 – VARIABLE LOAD & ENVIRONMENTAL
115. The function of a turbine is to: ENG’G.
Answer: A A. Transfer heat from one liquid to another
B. Increase the total energy content of the flow 1. The maximum continuous power available from a
110. The water hammer phenomenon is primarily what C. Extract energy from the flow hydro-electric power plant under the most adverse
kind of fluid mechanics? D. Exchange heat to increase energy to the flow hydraulic conditions is called _______.
A. Static (a phenomena independent of time) A. Base power
B. Dynamic (a time-dependent phenomena) Answer: C B. Firm power
C. Compressible C. Primary power
D. Incompressible 116. Pitot tube is use to measure the: D. Secondary power
A. Velocity of mass
Answer: B B. Velocity of pipe Answer: B
2. The area under load curve divided by maximum D. Demand factor Answer: C
demand represents:
A. Load factor Answer: A 13. Load curve refers to the plot of:
B. Connected load A. Load versus generating capacity
C. Average load 8. Past ME Board Question B. Load versus current
D. Diversity factor The ratio between the actual power and the C. Load versus time
apparent power in any circuit is known as the D. Load versus cost of power
Answer: D ______ of that circuit.
A. Measured power Answer: C
3. What is the reverse capacity of a plant? B. Capacity
A. Maximum demand – average demand C. Power factor 14. The ratio of maximum load to the rated plant
B. Plant capacity – maximum demand D. KVAR capacity
C. Plant capacity – average demand A. Load factor
D. Plant capacity x (1 – load factor) Answer: C B. Utilization factor
C. Maximum load factor
Answer: B 9. Past ME Board Question D. Capacity factor
The ratio of maximum load to the rated plant
4. Load curve refers to the plot of: capacity Answer: B
A. Load versus generating capacity A. Load factor
B. Load versus current B. Utilization factor 15. The ratio of the average load to the peak load over
C. Load versus time C. Maximum load factor a designated period of time is called _____.
D. Load versus cost of power D. Capacity factor A. Load factor
B. Reactive factor
Answer: C Answer: B C. Diversity factor
D. Plant use factor
5. The ratio of maximum load to the rated plant 10. Past ME Board Question
capacity is called ________. The ratio of the average load to the peak load over Answer: A
A. Load factor a designated period of time is called
B. Utilization factor A. Load factor 16. Past ME Board Question
C. Maximum load factor B. Reactive factor Peak load for a period of time divided by installed
D. Capacity C. Diversity factor capacity is _______.
D. Plant use factor A. Capacity factor
Answer: B B. Demand factor
Answer: A C. Utilization factor
6. Past ME Board Question D. Load factor
Peak load for a period of time divided by installed 11. Past ME Board Question
capacity is: The ratio of the sum of individual maximum Answer: C
A. Capacity factor demands of the system to the overall maximum
B. Demand factor demand of the whole system. 17. The area under load curve divided by maximum
C. Utilization factor A. Demand factor demand represents
D. Load factor B. Diversity factor A. Load factor
C. Power factor B. Connected load
Answer: C D. Utilization factor C. Average load
D. Diversity factor
7. Past ME Board Question Answer: B
The ratio of the sum of individual maximum Answer: D
demands of the system to the maximum demand 12. Load curve refers to the plot of:
of the whole system is: A. Load versus generating capacity 18. Past ME Board Question
A. Diversity factor B. Load versus current The ratio of the sum of individual maximum
B. Utilization factor C. Load versus time demands of the system to the overall maximum
C. Power factor D. Load versus cost of power demand of the whole system.
A. Demand factor D. D2 O A. v/f
B. Diversity factor B. f/v
C. Power factor Answer: D C. vf
D. Utilization factor D. v 2f
24. Scintillation counter are used to detect:
Answer: B A. Beta rays Answer: A
B. X-ray
19. Past ME Board Question C. Gamma rays 30. Swimming pool water needs
The ratio of the sum of individual maximum D. All of the above A. Pre-chlorination
demands of the system to the maximum demand B. Super-chlorination
of the whole system is known as ________. Answer: D C. Dual-chlorination
A. Diversity factor D. De-chlorination
B. Utilization factor 25. All of the following are common types of chemical
C. Power factor reactions except: Answer: D
D. Demand factor A. Direct combination
B. Fission 31. Aeration of water is done for all of the following
Answer: A C. Decomposition purposes except for:
D. Double replacement A. Removal of CO2
20. Past ME Board Question B. Removal of bad taste
The ratio between the actual power and the Answer: B C. Increasing the amount of oxygen in water
apparent power in any circuit is known as the D. Removal of temporary hardness
_______ of that circuit. 26. Redox reactions can often be type of:
A. Measured power A. Direct combination Answer: D
B. Capacity B. Fission
C. Power factor C. Decomposition 32. Which of the following area of work requires
D. KVAR D. Double replacement lowest noise level?
A. Gymnasiums
Answer: C Answer: D B. Library
C. Lecture halls
21. Yeast as raw material for beer making is added to 27. The fact that the amount of slightly soluble gas D. Kitchens
the equipment called: absorbed in a liquid is proportional to the partial
A. Fermenters pressure of the gas is known as: Answer: B
B. Brew kettle A. Dalton’s law
C. Cooler B. Henry’s law 33. Mycotoxins are poisonous chemicals produced by:
D. Starting tube C. Raoult’s law A. Bacteria
D. Boyle’s law B. Molds
Answer: A C. Virus
Answer: B D. Algae
22. A major cause of air pollution resulting from the
burning of fuel oil is: 28. All of the following pollutants are produced Answer: B
A. Nitrous because of decaying organic matter except:
B. Hydrogen A. Ammonia 34. Turbidity in water is due to:
C. Sulfur dioxide B. Sulfur dioxide A. Algae
D. Silicon C. Methane B. Fungi
D. Hydrogen sulfide C. Organic salts
Answer: C D. Finally divided particles of clay, silt and
Answer: B organic matter
23. Heavy water is:
A. B2 O 29. The velocity of radiations (v) and the frequency of Answer: D
B. H2 O radiations (f) are related to the wavelength of
C. W2 O radiations by:
35. Per capita consumption of water is generally taken Answer: D D. Reflection and absorption
as:
A. 50 – 100 liters 41. Which has an octane rating of more than 100? Answer: C
B. 150 – 300 liters A. Gobar gas
C. 400 – 500 liters B. Methyl alcohol 47. The operation that produces highest noise level is
D. 750 – 1000 liters C. Benzol :
D. Ethyl alcohol A. Welding
Answer: B B. Riveting
Answer: C C. Machining
36. Which of the following industries have the highest D. Pressing
consumption of water for processing? 42. The part per million is identical to:
A. Foundry A. Grains per gallon Answer: B
B. Automobile industry B. Pounds per cubic foot
C. Paper mill C. Milligrams per kg 48. The ratio absorbed by the transfer fluid to the
D. Aluminum industry D. Tones per acre foot original incident energy striking the collector.
A. Shading factor
Answer: C Answer: C B. Collector efficiency
C. Betz coefficient
37. The ratio activity of water because of strontium 43. In power plant an electrostatic precipitators is D. Transmittance
can be removed by: installed between:
A. Aeration A. Forced draft fan and induced draft fan Answer: B
B. Alum coagulation B. Furnace and chimney
C. Phosphate coagulation C. Primary air and secondary air 49. A swinging support constructed as part of the
D. Lime D. Furnace and forced draft fan vessel and that supports the manway cover when
it is unbolted and moved aside.
Answer: C Answer: B A. Davit
B. Handhole
38. Chemical used for checking the growth of algae in 44. At present, the number of true elementary C. Nozzle neck
reservoirs is particles, which include leptons and quarks, is D. Manway arm
A. Copper sulphate thought to be:
B. Alum A. 4 Answer: A
C. Bleaching powder B. 8
D. Brine C. 10 50. Is the subjective method in which the smoke
D. 12 density is visually compared to five standardized
Answer: C white-black grids.
Answer: D A. Ringelman scale
39. Which of the following is not the pollutant from a B. Dalton scale
sulfuric acid plants? 45. The effective size of a target atom that interacts C. Smoke spot scale
A. Sulfur dioxide with a moving particle is called its _______. D. Dew point scale
B. Sulfur trioxide A. Length
C. Acid moist B. Width Answer: A
D. Hydrogen sulfide C. Cross section
D. Pseudo-area 51. The ratio of fugacity at actual conditions to the
Answer: D fugacity at some reference state is known as:
Answer: C A. Compressibility
40. Which of the following is a cold cathode lamp? B. Activity
A. Sodium vapor lamp 46. Most nuclear particles can react with atoms in C. Gravimetric coefficient
B. High pressure mercury vapor lamp several different ways including ________. D. Saturation
C. Low pressure mercury vapor lamp A. Absorption
D. Neon lamp B. Scattering Answer: B
C. Absorption and scattering
52. A type of polymer used for detergents, milk Answer: A D. CFCs
bottles, oil containers and toys.
A. Polyvinyl chloride (PVC) 58. The removal of particulate matter from a gas flow Answer: A
B. Polystyrene (PS) by exposing the flow to a liquid or slurry is known
C. High density polyethelene (HDPE) as: 63. Equipment leaks from plant equipments are known
D. Polypropylene (PP) A. Stripping as ________.
B. Spraying A. Fugitive leaks
Answer: C C. Scrubbing B. Fugitive dusts
D. Absorption C. Fugitive exhausts
53. A type of polymer used for grocery bags a food D. Fugitive emissions
wraps. Answer: C
A. Polyvinyl chloride (PVC) Answer: D
B. Polystyrene (PS) 59. The drop in a solvent’s vapor pressure and the
C. Low density polyethelene (LDPE) increase in mole fraction as solute is added is 64. What is a substance that absorbs or retains
D. Polypropylene (PP) described by: moisture?
A. Dalton’s law A. Surface acting agent
Answer: C B. Henry’s law B. Dry ultra-fine coal
C. Raoult’s law C. Fossil fuel
54. A radioactive gas produced from the decay of D. Boyle’s law D. Humectant
radium within the rocks beneath a building.
A. Spills Answer: C Answer: D
B. Radon
C. Smoke 60. All of the following statements are characteristics 65. Is a soluble compound that reduces a liquid’s
D. Smog of bases except ________. surface tension or reduces the interfacial tension
A. They conduct electricity in aqueous solutions between a liquid and a solid.
Answer: B B. They turn red litmus paper blue A. Surface – acting agent or surfactant
C. They have a pH between 0 to 7 B. Humectant
55. Consists of ground – level ozone and peroxyacyl D. They neutralize acids forming salts and water C. Oxygenated fuel
nitrates (PAN) D. CFC
A. Smog Answer: C
B. Spills Answer: A
C. Sulfur oxide 61. All of the following statements about conjugate
D. Smoke acids and bases are true except _______. 66. A material of substance that is accidentally or
A. A conjugate acid results when a base intentionally introduced to the environment in a
Answer: A accept a proton quantity that exceeds what occurs naturally.
B. A conjugate base results when a base A. Waste
56. A term used to mean the corrective steps taken to accepts a proton B. Sludge
return the environment to its original condition. C. Strong acids tend to give weak conjugate C. Pollutant
A. Stabilization bases D. Biosolids
B. Remediation D. The Bronsted – Lowry theory defines bases
C. Greenhouse effect as proton accepetors Answer: C
D. Opacity
Answer: B 67. Refers to organic waste produced from biological
Answer: B waterwaste treatment processes.
62. Are any solid particulate matter that becomes A. Toxic waste
57. The burning of low – sulfur fuel is known as: airborne, with the exception of particulate matter B. Biosolids
A. Fuel switching emitted from the exhaust stack of a combustion C. Extrinsic waste
B. Fuel adding process. D. Process waste
C. Fuel binding A. Dusts or fugitive dusts
D. Fuel swapping B. Dioxins Answer: B
C. Fugitive emissions
68. A rule which states that any solid waste mixed with D. Asbestos 79. Are organic compounds manufactured in oily liquid
hazardous waste becomes hazardous. and solid forms through the late 1970s and
A. The mixture rule Answer: C subsequently prohibited.
B. The derived from rule A. Polychlorinated biphenyls (PCBs)
C. The environmental rule 74. Are highly concentrated liquid wastes produced in B. Polyvinyl chloride (PVC)
D. The hazard rule landfills. C. Trihalomethanes
A. Aerosols D. Plastic
Answer: A B. Leachates
C. Nitrogen dioxide Answer: A
69. A rule which states that any waste derived from D. Urea
the treatment of a hazardous waste remains a 80. A type of polymer used for Styrofoam cups and
hazardous waste. Answer: B “clam shell” food containers.
A. The mixture rule A. Polyvinyl chloride (PVC)
B. The treatment rule 75. What is the primary cause of smog formation? B. Polystyrene (PS)
C. The derived from rule A. Toxins C. Low density polyethelene (LDPE)
D. The hazard rule B. Dioxins D. Polypropylene (PP)
C. Oxidants
Answer: C D. Nitrogen oxides Answer: B

70. Generally refers to sulfur trioxide SO3 in the flue Answer: D 81. A type of polymer used for labels, bottles and
gas. housewares.
A. Acid rain 76. Is a water soluble organic compound prepared A. Polyvinyl chloride (PVC)
B. Acid gas from ammonia. It has significant biological and B. Polystyrene (PS)
C. Hydrochloric acids industrial usefulness. C. Low density polyethelene (LDPE)
D. Stack gas A. Urea or carbamide urea D. Polypropylene (PP)
B. Nitrite
Answer: B C. Chlorinated organics Answer: D
D. Oxidants
71. What consists of weak solutions of sulfuric, 82. A type of polymer used for clear bottles.
hydrochloric, and to a lesser extent, nitric acids? Answer: A A. Polyvinyl chloride (PVC)
A. Stack gas B. Polystyrene (PS)
B. Acid rain 77. Are by products of reaction between combustion C. Low density polyethelene (LDPE)
C. Acid compound products D. Polypropylene (PP)
D. Pollutant A. Photochemicals
B. Oxidants Answer: A
Answer: B C. Organics
D. Sediments 83. Refers to the high – temperature removal of tarry
72. A fibrous silicate mineral material that is inert, substances from the interior of the carbon granule,
strong and incombustible. Answer: B leaving a highly porous structure.
A. Fiberglass A. Absorbent
B. Plastic 78. “Particulate matter” is defined as all particles that B. Activated
C. Rubber are emitted by a combustion source. What is C. Breakthrough
D. Asbestos another term for particulate matter? D. Adsorbent
A. Dust
Answer: D B. Aerosol Answer: B
C. Biosolids
73. An insulator with superior tensile strength but low D. Sediments 84. A substance with high surface area per unit
heat resistance. weight, and intricate pore structure, and a
A. Rubber Answer: B hydrophobic surface.
B. Plastic A. Adsorbent substance
C. Fiberglass B. Adsorbent substance
C. Homogenous substance 90. All of the following occur during reduction of a D. Saturated solution
D. Activated substance substance except __________.
A. An increase in negative charge Answer: B
Answer: A B. Loss of electrons
C. An oxidation state decrease 96. What do you call the mixture when a solvent has
85. Is a high – temperature process that turns D. Reduction of the oxidizing agent dissolved as much solute as it can?
incinerator ash into a safe glass-like material. A. Solution
A. Advance oxidation Answer: B B. Mild solution
B. Biofitration C. Hydration
C. Vitrification 91. All of the following are units of energy except: D. Saturated solution
D. Bioventing A. Calories
B. Joules Answer: D
Answer: C C. Pascals
D. MeV 97. The equilibrium constant for weak solution is
86. Refers to the use of composting and soil beds. known as:
A. Biofiltration Answer: C A. Ionization constant
B. Biomediation B. Arrhenius exponent
C. Bioventing 92. What do you call the mixture if the solute particles C. Solubility product
D. Bioreactors of a solid suspended in a liquid are larger than D. La Chatelier’s constant
molecules?
Answer: A A. Solution Answer: A
B. Suspension
87. Are open or closed tanks containing dozens or C. Hydration 98. If the solute particles of a solid suspended in a
hundreds of slowly rotating disks covered with a D. Saturated solution liquid are larger than the molecules, the mixture is
biological film of microorganisms. known as:
A. Biofilter Answer: B A. Solution
B. Bioreactor B. Suspension
C. Biomediator 93. What do you call the mixture when a solvent has C. Hydration
D. Bioinventor dissolved as much solute as it can? D. Saturated solution
A. Solution
Answer: B B. Mild solution Answer: B
C. Hydration
88. Refers to the treatment of contaminated soil in a D. Saturated solution 99. When a solvent has dissolved as much solute as it
large plastic-covered tank. can, the mixture is called:
A. Bioventing Answer: D A. Solution
B. Biological recycling B. Suspension
C. Biocycle 94. The equilibrium constant for weak solution is C. Hydration
D. Bioremediation known as D. Saturated solution
A. Ionization constant
Answer: A B. Arrhenius exponent Answer: D
C. Solubility product
89. All of the following occur during oxidation of a D. La Chatelier’s constant 100. When excess solute in a solution settles to the
substance except __________. bottom of the container, the process is called:
A. Oxidation state increases Answer: A A. Salvation
B. Loss of electrons B. Deemulsification
C. The substance becomes less negative 95. What do you call the mixture if the solute particles C. Precipitation
D. Oxidation of the oxidizing agent of solid suspended in a liquid are larger than the D. Equation
molecules?
Answer: D A. Solution Answer: C
B. Suspension
C. Hydration
101. All of the following express units of concentration C. Endothermic heat solution 112. All of the following factors affect rates of reaction
except: D. Exothermic heat solution except:
A. Normality A. Exposed surface area
B. Molarity Answer: C B. Concentrations
C. Formality C. Temperature
D. Isotropy 107. The absorption of water by a dessicant often D. Pressure
demonstrates:
Answer: D A. Heat of fusion Answer: D
B. Heat of vaporization
102. The desirable temperature inside an air C. Endothermic heat of solution 113. Le Chatelier’s principle predicts the direction of a
conditioned auditorium is: D. Exothermic heat of solution state of chemical equilibrium based on all of the
A. 0 C ̊ following factors except:
B. 5 C ̊ Answer: D A. Temperature
C. 10 C ̊ B. Specific volume
D. 20 C ̊ 108. A substance that absorbs moisture from the air is: C. Pressure
A. Deliquescent D. Concentration
Answer: D B. Efflorescent
C. Effervescent Answer: B
103. Which of the following acts as ignition accelerator D. A precipitant
for internal combustion engine fuels? 114. The amount of energy necessary to cause a
A. Acetone peroxide Answer: A reaction to occur is called the _______.
B. Hydrogen peroxide A. Heat of formation
C. Aromatic compounds 109. The removal of a dissolved gas or other volatile B. Heat of solution
D. N-heptane component from liquid by exposing the liquid to air C. Activation of energy
or steam is known as: D. Heat of fusion
Answer: A A. Stripping
B. Gas absorption Answer: C
104. Which of the following does not use ambient air for C. Spraying
propulsion? D. Scrubbing 115. The equilibrium constant for weak solutions is
A. Turbo jet known as ________.
B. Turbo-prop Answer: A A. The ionization constant
C. Pulse jet B. The Arrhenius exponent
D. Rocket 110. Which of the following contribute to the C. The solubility product
deterioration of the Earth’s ozone layer? D. Le Chatelier’s principle
Answer: B A. Carbon monoxide
B. Chlorofluorocarbons Answer: A
105. Most observed properties of light and other radiant C. Carbon dioxide
energy are consistent with waves in nature, but in D. Refrigerants 116. The speed at which a reaction proceeds to
interactions with matter, electromagnetic energy equilibrium is the purview of _________.
behaves as though it consists of discrete pieces or Answer: B A. Reaction kinetics
A. Blocks B. Le Chatelier’s principle
B. Balls 111. Ozone filters out what type of radiation that C. Neutralization
C. Quanta damages crops and causes skin cancer? D. Ionization
D. Atomic masses A. Tropospheric radiation
B. Atmospheric radiation Answer: A
Answer: C C. X-rays
D. Ultraviolet radiation 117. The color of water is measured:
106. The amount of energy absorbed when a A. In terms of pH value
substance enters a solution is called the: Answer: D B. In terms of degree of hardness
A. Heat of fusion C. In terms of platinum cobalt scale
B. Heat of sublimation D. Ppm of dissolved solids
Answer: C C. Supersonic 4. Liquids and gases take the following
D. Relativistic characteristic(s) of their contents.
118. What turbidimeter that gives direct readings in A. Volume
ppm? Answer: C B. Shape
A. Jackson turbidimeter C. Shape and volume
B. Hellige turbidimeter 124. The reduction of nuclear radiation intensity (called D. Neither shape nor volume
C. Baylis turbidimeter attenuation) is accomplished by:
D. All of the above A. Matter Answer: B
B. Antimatter
Answer: A C. Shielding 5. Alcohol finds use in manometers as:
D. Neurons A. It provides a suitable meniscus for the
119. Which of the following is not a sound absorptive inclined tube
material? Answer: C B. Its density being less can provide longer
A. Rugs length for a pressure difference, thus more
B. Mirrors 125. The ability of substance to absorb neurons accuracy can be obtained
C. Carpets depends upon its _________. C. A and B above are correct
D. Heavy drapes A. Absorption cross section D. Cheap and easily available
B. Scattering cross section
Answer: B C. Total cross section Answer: C
D. Atomic number
120. The total cross section of a target atom is made up 6. Which of the following statements about a
of ___________. Answer: A Newtonian fluid is most accurate?
A. The absorption cross section A. Shear stress is proportional to strain
B. The scattering cross section CHAPTER 11 – FLUID MECHANICS B. Viscosity is zero
C. The absorption and scattering cross sections C. Shear stress is multi – valued
D. The reflection and absorption cross sections 1. If the energy of the incident photon is less than the D. Shear stress is proportional to rate of strain
work function:
Answer: C A. An electron will be ejected Answer: D
B. More than one electron will be ejected
121. Which of the following cannot be a phenomenon of C. An electron will not be ejected 7. The normal stress is the same in all directions at a
neutron interactions? D. Less than one electron will be ejected point in fluid:
A. Inelastic scattering A. Independent of the motion of one fluid layer
B. Elastic scattering Answer: C relative to an adjacent layer
C. Fission B. When there is no motion of one fluid layer
D. Fusion 2. For supersonic flow, the pressure of fluid must relative to an adjacent layer
decrease as the fluid flow area of the duct. C. Only if the fluid is frictionless
Answer: D A. Increases D. Only if fluid is frictionless and incompressible
B. Decreases
122. Which of the following cannot be used to describe C. Remain the same Answer: B
neutron kinetic energy levels? D. None of these
A. Cold 8. Which of the following is not a characteristic of
B. Thermal Answer: A fluid pressure?
C. Slow A. It is the same in all directions at a point in the
D. Freezing 3. Density in term of viscosity is: fluid
A. Kinematic viscosity / dynamic viscosity B. Its acts normal to a surface
Answer: D B. Dynamic viscosity / kinematic viscosity C. It is a shear stress
C. Kinematic viscosity x dynamic viscosity D. It is linear with depth
123. All of the following are words used to describe D. None of the above
neutron kinetic energy levels except: Answer: C
A. Slow (resonant) Answer: B
B. Fast
9. The length of mercury column at a place at an B. surface tension D. States that energy is constant everywhere in
altitude will change with respect to that at ground C. bulk modulus the fluid
in: D. hysteresis
A. A linear relation Answer: D
B. A parabolic relation Answer: D
C. Will remain constant 20. A pilot tube can be used to measure fluid velocity
D. First slowly and then steeply 15. Which of the following can be used to measure the as described by the Bernoulli’s equation and the
flow of water in a pipe of diameter 3000 mm? relationship between:
Answer: D A. Venturimeter A. Kinetic energy and static pressure
B. Rotameter B. Fluid pressure and height of the fluid
10. All of the following dimensionless parameters are C. Nozzle C. Fluid pressure and impact energy
applicable to fluid flow problems except the D. Pilot tube D. Pressure and momentum
_______.
A. Reynolds number Answer: D Answer: A
B. Froude number
C. Mach number 16. The pressure at a given depth due to several 21. In order to avoid vaporization in the pipe line, the
D. Biot number immiscible liquids is: pipe line over the ridge is laid in such a way that it
A. The average of the individual pressures is not more than:
Answer: D B. The sum of the individual pressures A. 2.4 m above the hydraulic gradient
C. Independent of the individual pressures B. 6.4 m above the hydraulic gradient
11. Mass density of liquid (ρ) is given by which of the D. Unknown C. 10.0 m above the hydraulic gradient
following? D. 5.0 m above the hydraulic gradient
A. Ρ = Mass / volume Answer: B
B. Ρ = metric slug / 𝑚2 Answer: B
C. Ρ = kg 𝑠𝑒𝑐 2 / 𝑚4 17. The equation of continuity of flow is applicable if:
D. all of the above A. The flow is one dimensional 22. The stream function is a useful parameter in
B. The flow is steady describing_____________.
Answer: D C. The velocity is uniform over the cross – A. The conservation of mass
section B. The conservation of momentum
12. the speed of sound in all fluid is most closely D. All of the above conditions are together C. The conservation of energy
related to all of the following properties D. The equation of state
except________. Answer: D
A. Compressibility Answer: A
B. Density 18. Uniform flow takes place when:
C. Bulk module A. Conditions remain unchanged with time at 23. For high speed flows, the potential energy of fluids
D. Thermal conductivity any point are:
B. Rate of change of velocity of fluid is zero A. Positive
Answer: D C. At every point the velocity vector is identical B. Negative
in magnitude and direction for any given C. Negligible
13. Under which condition, the specific weight of water instant D. None of these
is 1000 kg/𝑚3 ? D. The change in transverse direction is zero
A. At normal pressure of 760 mm Answer: C
B. At 4 C ̊ temperature Answer: C
C. At mean sea level 24. McLeod gauge used for low pressure
D. All of the above 19. The continuity equation of an ideal fluid flow. measurement operates on the principle of
A. States that the net rate in – flow into any _________.
Answer: D small volume must be zero A. Gas law
B. Applies to irrotational flow only B. Boyle’s law
14. All of the following can be characteristics of fluids C. States that the energy remains constant C. Charles law
except_________. along streamline D. Pascal’s law
A. kinematic viscosity
Answer: B Answer: A B. Upward in the direction of the flow
C. Level (no slope)
25. A Kaplan turbine is 31. For stable equilibrium of floating body its D. There is no effect of friction on the energy
A. A high head mixed flow turbine metacenter should be: grade line
B. An impulse turbine, inward flow A. Below the center of gravity
C. A reaction turbine, outward flow B. Below the center of buoyancy Answer: A
D. Low head axial flow turbine C. above the center of buoyancy
D. above the center of gravity 37. The pilot tube is a device used for measurement of
Answer: D A. Pressure
Answer: D B. Flow
26. The most common method for calculating frictional C. Velocity
energy loss for laminar flowing fluids in noncircular 32. Center of pressure on an inclined plane has ___. D. Discharge
pipe is: A. At the centroid
A. The Darcy equation B. Above the centroid Answer: C
B. The Hagan – Poisevill equation C. Below the centroid
C. The Hazen - Williams equation D. At metacenter 38. Hydrometer is used to find out
D. The Swamee – Jin equation A. Specific gravity liquids
Answer: C B. Specific gravity solids
Answer: A C. Specific gravity gases
33. The line action of the buoyant forces always acts D. Relative humidity
27. The parameter f in the expression for head-loss is through the centroid of the ______.
A. The fraction of flow that is totally turbulent A. Submerged body Answer: A
B. The Darcy friction factor B. Volume of the floating body
C. The height of roughness scale in turbulent C. Volume of the fluid vertically above the body 39. The fluid forces taken into consideration in the
flow D. Displaced volume of the fluid Navier Strokes equation are:
D. The static coefficient of friction A. Gravity, pressure and viscous
Answer: D B. Gravity, pressure and turbulent
Answer: B C. Pressure, viscous and turbulent
34. The hydraulic grade line of a pipe denotes which D. Gravity, viscous and turbulent
28. Friction factor for both laminar and turbulent flows of the following?
can be found plotted in a A. Total energy Answer: A
A. Steam table B. Pressure energy
B. Psychometric chart C. Potential energy 40. Permissible velocity of water flowing through
C. Moody diagram D. The sum of pressure energy and potential concrete tunnel, is generally
D. Mollier diagram energy A. 4-5 m/s
B. 10-12 m/s
Answer: C Answer: D C. 13-16 m/s
D. 20 m/s
29. Which of the following is relative velocity? 35. The energy grade line of a pipeline denotes which
A. The difference between two velocities of the following? Answer: A
B. Average velocity A. Total energy
C. Sum of two velocities B. Pressure energy 41. Orifice refers to an opening
D. Vector difference of two velocities C. Potential energy A. With closed perimeter and of regular form
D. The sum of pressure energy and potential through which water flows
Answer: D energy B. With prolonged sides having length of 2 to 3
diameters of opening in thick wall
30. Which of the following is the highest head? Answer: A C. With partially full flow
A. 33 inch Hg D. In hydraulic structure with regulation
B. 31.0 ft. water 36. The presence of friction in the energy grade line provision
C. 1.013 kg kg/𝑐𝑚2 will always cause the line to slope
D. 75.0 cm of Hg A. Down in the direction of the flow Answer: D
42. The value of coefficient of discharge in A. Sonic C. Kutter’s C
comparison to coefficient of velocity is found to B. Sub-sonic D. Value of k in Darcy - Weisbach formula
be_______. C. Supersonic
A. More D. Supersonic on one side and sub-sonic on the Answer: B
B. Less other side
C. Same 53. What is the coefficient of contraction?
D. More/less depending on flow Answer: D A. The ratio of the area of vena contracta to the
area of the orifice
Answer: B 48. Which of the following is the basic of Bernoulli’s B. The ration of actual discharge to the
law for fluid flow? theoretical discharge
43. Weir refers to an opening A. Continuity equation C. The ratio of the actual velocity to the
A. Having closed perimeter and of regular form B. Principle of conservation of energy theoretical velocity
through which water flows C. Fourier’s law D. The ratio of the effective head to the actua;
B. Having prolonged sides with length of 2 to 3 D. Principle of conservation of mass head
diameters of opening in thick wall
C. Having partially full flow Answer: B Answer: A
D. In hydraulic structures with regulation
provision 49. Which of the following is NOT a characteristic of 54. Where is vena contracta most likely loacated?
fluid pressure? A. At the orifice
Answer: C A. It is a shear stress B. At a distance approximately ½ the diameter
B. It is the same in all directions at a point in the of the orifice
44. Which of the following parameters determine the fluid C. At a distance approximately equal to the
friction factor of turbulent flow in a rough pipe? C. It acts normal to a surface diameter of the orifice
A. Froude number and relative roughness D. It is linear with depth D. At a distance approximately twice the
B. Froude number and Mach number diameter of the orifice
C. Reynolds number and relative roughness Answer: A
D. Mach number and relative roughness Answer: B
50. Refers to the compressibility of a fluid, the
Answer: C fractional change in fluid volume per unit change 55. A substance that is able to flow and yields to any
in fluid. force tending to change its shape without
45. Power transmitted through a pipe is maximum A. Viscosity changing its volume such as water and air.
when the loss of head due to friction is: B. Bulk modulus A. Fluid
A. One-half of the total head supplied C. Density B. Flux
B. One-third of the total head supplied D. Pressure C. Gas oil
C. One-fourth of the total head supplied D. Water gas
D. Equal to the total head supplied Answer: D
Answer: A
Answer: B 51. A pilot tube can be used to measure fluid velocity
as described by the Bernoulli’s equation and the 56. The velocity of a fluid particle at the center of the
46. In a nozzle if back pressure is same as inlet relationship between: pipe section is______.
pressure; then_______________. A. Kinetic energy and static pressure A. Maximum
A. No flow takes place B. Fluid pressure and static pressure B. Minimum
B. Maximum flow takes place C. Fluid pressure and impact energy C. Average
C. Flow becomes subsonic in diverging section D. Pressure and momentum D. Logarithmic average
D. Flow becomes supersonic in converging as
well as supersonic section Answer: A Answer: A

Answer: A 52. The ratio of the area to the wetted penmeter is 57. For supersonic flow, the pressure of fluid must
known as __________. increase as the fluid flow area of the duct:
47. The flow on two sides of a normal shock wave is A. Flow factor A. Increases
called___________. B. Hydraulic radius B. Decreases
C. Constant 63. Which of the following is not a characteristic of real B. Density
D. None of these fluids? C. Pressure
A. Finite viscosity D. Viscosity
Answer: B B. Non-uniform velocity distributions
C. Compressibility Answer: C
58. Which is incorrect statement regarding apparent D. Experience of eddy current and turbulence
shear forces. 69. Property of a fluid whereby its own molecules are
A. It can never be found in frictionless fluid Answer: D attracted is known as ________.
regardless of its motion A. Adhesion
B. It can never be found when the fluid is at rest 64. Which of the following is not the mass density of B. Cohesion
C. It depends upon cohesive forces water? C. Surface tension
D. It may occur owing to cohesion when the fluid A. 62.5 lbm/𝑓𝑡 3 D. Viscosity
is at rest B. 100 kg/𝑚3
C. 1 g/𝑐𝑚3 Answer: B
Answer: D D. 1 kg/L
70. The term subsonic flow refers to a flowing gas with
59. The time required for half a quantity of radioactive Answer: B a speed:
particles to decay (disintegrate) is called A. Less than the local speed of sound
its_____________. 65. The upper critical Reynolds number for pipe flow B. Equal to the speed of sound
A. Average life is: C. Greater than the speed of sound
B. Median life A. Of no practical importance to designers D. Much greater than the speed of sound
C. Time constant B. Always used to design pipes for strength
D. Half time C. The number at which turbulent flow changes Answer: A
over to laminar flow
Answer: D D. The number at which laminar flow changes 71. The pressure at a point in a fluid will not be same
into turbulent flow in all the directions if the fluid is:
60. SI unit of viscosity is: A. Viscous
A. 10 times poise Answer: A B. Viscous and static
B. 9.81 times poise C. Inviscous and in motion
C. 1/9.81 time poise 66. Which of the following statements about gauge D. Viscous and is in motion
D. 1/10 times poise pressure is most correct? Gauge pressure are
measured relative to _________. Answer: D
Answer: A A. Atmospheric pressure
B. A vacuum 72. The statement that “the hydrostatic pressure a
61. For computation convenience, fluids are usually C. Each other fluid exerts on an immersed object or on container
classed as: D. The surface walls is a function only of fluid depth” is
A. Rotational or irrotational A. The perfect gas law
B. Real or ideal Answer: A B. D’Alembert’s paradox
C. Laminar or turbulent C. The hydrostatic paradox
D. Newtonian or non-newtonian 67. The volumetric change of the fluid caused by a D. Boyle’s law
resistance is called ________.
Answer: B A. Volumetric strain Answer: C
B. Volumetric index
62. Which of the following is not a dimensionless C. Compressibility 73. Bernoulli’s equation is s/an ___________.
parameter? D. Adhesion A. Momentum equation
A. Kinetic viscosity B. Conservation of energy equation
B. Weber number Answer: D C. Conservation of mass equation
C. Darcy Weisbach friction factor D. Equation of state
D. Froude number 68. Compressibility of a fluid relates the fractional
change in fluid volume per unit change in fluid. Answer: B
Answer: A A. Temperature
74. An ideal fluid is one that: D. Important only in supersonic flow 85. Which of the following turbine is different from the
A. Is very viscous others?
B. Obeys Newton’s law of viscosity Answer: C A. Fourneyron turbine
C. Is assumed in problems in conduit flow B. Francis turbine
D. Is frictionless and incompressible 80. Fully turbulent flow in a pipe is characterized by all C. Kaplan turbine
of the following except: D. Pelton wheel
Answer: D A. A parabolic velocity profile
B. A momentum exchange due to fluid masses Answer: D
75. The relationship between pressure and altitude in rather than molecules
the atmosphere is given by the: C. A maximum velocity at the fluid center line 86. Running away speed of a Pelton wheel gives:
A. Perfect gas law D. A 1/7 velocity profile A. Actual operating speed
B. Conservation of mass B. No load speed
C. Barometric height relationship Answer: A C. Full load speed
D. First law of thermodynamics D. No load speed when governor mechanism
81. The laminar friction factor of fluid flowing through a fails
Answer: C pipe is a function of all of the following except:
A. Fluid velocity Answer: D
76. The fact the buoyant force on a floating object B. Pipe diameter
equal to the weight of the water displaced is: C. Pipe roughness 87. Which of the following turbine is different from the
A. Bernoulli’s law D. Reynolds number others?
B. Archimedes’ principle A. Pelton wheel
C. The law of diminishing returns Answer: C B. Banki turbine
D. The conservation of mass C. Jonval turbine
82. The continuity equation is applicable to: D. Kaplan turbine
Answer: B A. Viscous unviscous fluid
B. Compressibility of fluids Answer: D
77. Which of the following terms does not appear in C. Conservation of mass
the steady flow energy equation (the extended D. Steady unsteady flow 88. The characteristic length of the Reynold’s number
Bernoulli’s equation)? used to calculate the friction in noncircular full
A. Kinetic energy Answer: C running pipes is based on the __________.
B. Potential energy A. Run length
C. Friction losses 83. The rise or fall of head ‘h’ in a capillary tube of B. Pipe length
D. Hysteresis losses diameter ‘d ‘ and liquid surface tension ‘s’ and C. Hydraulic diameter (the equivalent diameter)
specific weight ‘w’ is given by: D. Wetted circumference
Answer: D A. 4s/wd
B. 4ds/w Answer: C
78. Neglecting the forces due to inertia, gravity and C. 4wd/s
frictional resistance, the design of a channel can D. 4ws/d 89. The hydraulic radius of noncircular pipe is:
be made by comparing A. The square root of the flow area
A. Weber number Answer: A B. The ratio of the area to the wetted perimeter
B. Reynolds number C. The radius of a pipe of equivalent area
C. Froude’s number 84. The study of the practical laws of fluid flow and the D. None of the above
D. Prant’l number resistance of open pipes and channels.
A. Fluid mechanics Answer: B
Answer: C B. Hydraulics
C. Aerodynamics 90. The Darcy equation can be used for all liquids and
79. The difference between stagnation pressure and D. Thermodynamics flows except:
total pressure is: A. Water
A. Due to height difference Answer: B B. Alcohol
B. Due to fluid kinetic energy C. Gasoline
C. None of the terms are interchangeable D. Air flowing supersonically
Answer: D D. There is no effect of friction on the energy D. Viscosity factor
grade line
91. The Hazen – Williams formula for head loss due to Answer: A
friction is based upon: Answer: A
A. Rigorous mathematical derivation 102. If the Mach number is greater than 1 but lesser
B. Empirical data 97. The presence of a minor loss in the energy grade than 5, what is the standard classification of the
C. Semi-empirical analysis line will cause the line to slope: travel?
D. Screndipity A. Down in the direction of the flow A. Transonic travel
B. Upward in the direction of the flow B. Subsonic travel
Answer: B C. Vertically downward C. Hypersonic travel
D. There is no effect of friction on the energy D. Supersonic travel
92. The extended Bernoulli equation includes all of the grade line
following terms except: Answer: D
A. Potential energy Answer: A
B. Kinetic energy 103. What is measured by a Pitot tube?
C. Nuclear energy 98. What do you call the pressure which the fluid A. Volumetric discharge
D. Friction losses exerts on an immersed object or container walls? B. Mass flow
A. Normal pressure C. Pressure
Answer: C B. Standard liquid pressure D. Velocity
C. Hydrostatic pressure
93. An equipotential line is one that: D. Gage pressure Answer: D
A. Has no velocity component tangent to it
B. Has uniformly varying dynamic pressure Answer: C 104. What is the difference between the energy grade
C. Has no velocity component normal to it line and the hydraulic grade line?
D. Exists in case of rotational flow 99. Viscosity for a fluid is defined as the constant of A. Potential energy
proportionality between shear stress and what B. Pressure energy
Answer: A other variable? C. Kinetic energy
A. The spatial derivative of velocity D. Friction losses
94. What is the use of a Hydraulic jump? B. The time derivative of pressure
A. Increase the flow rate C. The time derivative of density Answer: C
B. Reduce the flow rate D. The spatial derivative of density
C. Reduce the velocity of flow 105. Kinetic energy is not neglected in calculations of:
D. Reduce the energy of flow Answer: A A. High speed flow
B. Low speed flow
Answer: D 100. What is the classification of the fluid flow if the fluid C. Steady flow
travels parallel to the adjacent layers and the D. Equilibrium flow
95. What do you call the lowest portion to storage paths of the individual particles do not cross each
basin from where the water is not drawn? other? Answer: A
A. Bottom storage A. Steady flow
B. Sub soil storage B. Laminar flow 106. Discharge losses through orifice are due to:
C. Spring reserve C. Uniform flow A. Friction losses
D. Dead storage D. Turbulent flow B. Minor losses
C. Both friction and minor losses
Answer: D Answer: B D. Pressure losses

96. The presence of friction in the hydraulic grade line 101. Which of the following refers to the measure of a Answer: C
will always cause the line to slope: fluid’s sensitivity to changes in viscosity with
A. Down in the direction of the flow changes in temperature? 107. Which of the following is considered as an
B. Upward in the direction of the flow A. Viscosity index important parameter in the study of compressible
C. Level (no slope) B. Coefficient of viscosity flow?
C. Viscosity ratio A. Speed of fluid
B. Speed of sound C. M>1 The volumetric change of the fluid caused by a
C. Speed of light D. M=1 resistance is called:
D. Speed of fluid flow A. Volumetric change
Answer: D B. Volumetric index
Answer: B C. Compressibility
114. The pressure decreases as the temperature and D. Adhesion
108. Is the velocity at which an infinitesimal small velocity increases while the fluid velocity and
pressure wave travels through a medium. Mach number: Answer: D
A. Subsonic velocity A. Increases
B. Hypersonic velocity B. Decreases 120. Past ME Board Question
C. Sonic velocity C. Remains constant The energy of a fluid flowing at any section in a
D. Monatomic velocity D. None of these pipeline is a function of:
A. Velocity of flow only
Answer: C Answer: A B. Pressure only
C. Height above a chosen datum, density,
109. It is the ratio of the actual velocity of the fluid to the 115. The Mach number is unity or one at the location of internal energy, pressure and velocity of flow
velocity of sound. smallest flow area, called the: D. Pressure, height above a chosen datum,
A. Mach number A. Decreasing are velocity of flow, density of fluid
B. Froude number B. Throat
C. Sonic number C. Increasing area Answer: C
D. Euler number D. None of these
121. Past ME Board Question
Answer: A Answer: B If the fluid travels parallel to the adjacent layers
and the paths of individual particles do not cross,
110. The flow is called sonic when Mach number is: 116. What happens to the velocity of fluid after passing the fluid is said to be:
A. Equal to 1 the throat although the flow area A. Turbulent
B. Less than 1 A. Increases rapidly B. Critical
C. More than 1 B. Decreases rapidly C. Dynamic
D. None of these C. Remains constant D. Laminar
D. None of these
Answer: A Answer: D
Answer: A
111. The following flow is sub-sonic when Mach no. is: 122. Past ME Board Question
A. Greater than 1 117. Which of the following is an example of Newtonian Center of pressure on an inclined plane lies:
B. Less than 1 fluid? A. At the centroid
C. More than 1 A. Motor oils B. Above the centroid
D. None of these B. Gas C. Below the centroid
C. Paints D. At the metacenter
Answer: B D. Clay slurries
Answer: C
112. The flow is supersonic when Mach no. is: Answer: B
A. Greater than zero 123. Past ME Board Question
B. Less than 1 118. What is the critical pressure of water? At any instant, if the number of particles passing
C. Greater than 1 A. 150 kg/cm3 every cross-section of the stream is the same, the
D. None of these B. Less than 200 kg/cm2 flow is said to be:
C. More than 200 kg/cm2 A. Steady flow
Answer: C D. 100 kg/cm2 B. Uniform flow
C. Continuous flow
113. The flow is transonic when Answer: C D. Laminar flow
A. M = 0
B. M < 1 119. Past ME Board Question Answer: A
124. Past ME Board Question D. 1 + c + c(𝑝2 /𝑝1 )𝑛+1 7. Which of the following give the optimum intermediate
The ratio of cross-sectional area of flow to the pressure in case of two stage compression?
𝑝 +𝑝
wetted perimeter is: Answer: A A. 𝑝2 = 1 2
2
A. Hydraulic lead 𝑝1 +𝑝2
B. 𝑝2 = 𝑝1 +
B. Hydraulic section 2. In case of axial flow compressors for minimum fluid 2

C. Hydraulic mean depth friction and blade tip clearance losses, the blades of C. 𝑝2 = √𝑝1 𝑝3
D. Hydraulic gradient an axial flow compressor are designed for____ D. 𝑝2 = 𝑝1+ √𝑝1 2 + 𝑝3 2
A. 80% reaction
Answer: C B. 85% reaction Answer: C
C. 60% reaction
125. Past ME Board Question D. 53% reaction 8. In air compressor performance curve a surge line
If A is the cross-sectional area of the flow and Pw represents what?
is the wetted perimeter of a pipe, then what is the Answer: D A. Limit of compressor efficiency
hydraulic depth. Hd ? B. Limit of compressor discharge
A. Pw – A 3. Centrifugal blowers can supply C. Limit of stable operation
B. Pw / A A. Large volumes of air at low pressures D. Lower critical speed of shaft
C. A / Pw B. Small volumes of air at high pressures
D. Pw x A C. Large volumes of air at high pressures Answer: C
D. Small volumes of air at low pressures
Answer: C 9. Which of the following is a displacement
Answer: A compressor?
126. Past ME Board Question A. Reciprocating air compressor
If Q is the volume in gallon; D is height or 4. Which of the following is a safety device on a B. Vane blower
elevation in ft. and m is weight in lbs. per gallon, compressor? C. Centrifugal blower
what is the desired energy to lift the water from A. Relief valve D. Axial flow compressors
lower to higher elevation? B. Strainer
A. E = mD/Q C. Over speed shut down Answer: B
B. E = mDQ D. Over pressure shut down
C. E = mQ/D 10. Stalling of the blades of axial flow compressor is:
D. E = QD/m Answer: A A. An unsteady periodic and reversal of flow
B. The fixed mass flow rate irrespective of
Answer: B 5. In a four stage compressor system, the first and third pressure ratio
stage pressures are 1 and 9 kg/cm2 respectively. C. The reduction in lift force at higher angle of
127. Past ME Board Question What is the fourth stage delivery pressure? incidence
The flow of the convergent section of a nozzle is A. 9 kg/cm2 D. All of the above
always subsonic. If the flow is subsonic then the B. 81 kg/cm2
Mach number is: C. 27 kg/cm2 Answer: C
A. Greater than unity D. 243 kg/cm2
B. Less than unity 11. Surging is
C. Near unity Answer: C A. An unsteady, periodic and reversal of flow in
D. Unity the compressor
6. For a six compression of air set, the minimum work B. The fixed mass flow rate irrespective of
Answer: B conditions are: pressure ratio
A. Pressure rise per stage will be equal C. The reduction in lift force at higher angle of
CHAPTER 12 – FLUID MACHINERY B. Work done in successive stages will be in incidence
geometrical progression D. None of the above
1. The volumetric efficiency of a compressor having C. Cylinder volumes will be same
clearance factor is given by: D. Temperature rise in the cylinders will be the Answer: A
A. 1 + c – c(𝑝2 /𝑝1 )1/𝑛 same
B. 1 + c + c(𝑝2 /𝑝1 )1−𝑛 12. In an axial flow compressor, the pressure rise takes
C. 1 + c - c(𝑝2 /𝑝1 )𝑛 Answer: D place in:
A. Fixed blades only D. 70 to 90 psig D. All suction valves are open and all discharge
B. Moving blades only valves are open
C. Both fixed and moving blades Answer: B
D. None of the above Answer: B
18. The capacity of portable air compressors used for
Answer: C construction, mining, road building, and painting 23. Past ME Board Question
ranges from ______________. Pump used to increase air pressure above normal,
13. An axial flow compressor is suitable for: A. 1 𝑓𝑡 3 /min to 2000 𝑓𝑡 3 /min air is then used as a motive power.
A. High volume flow rates with small pressure B. 30 𝑓𝑡 3 /min to 2500 𝑓𝑡 3 /min A. Air cooled engine
rise C. 1 𝑓𝑡 3 /min to 2500 𝑓𝑡 3 /min B. Air compressor
B. Low volume flow rates with low pressure rise D. 30 𝑓𝑡 3 /min to 2000 𝑓𝑡 3 /min C. Air condenser
C. High volume flow rates with high pressure D. Air injection
rise Answer: A
D. Low volume flow rates with high pressure rise Answer: B
19. The power of portable air compressors used for
Answer: A construction, mining, road building, and painting 24. A device used for raising fluids from a lower to higher
ranges from: level.
14. Which of the following is a positive displacement A. 1 hp to 500 hp A. Compressor
rotary compressor? B. ½ hp to 500 hp B. Pump
A. Roots blower C. 1 hp to 1000 hp C. Turbine
B. Centrifugal compressor D. ½ hp to 1000 hp D. Blowers
C. Axial flow compressor
D. None of the above Answer: B Answer: B

Answer: A 20. Most permanent installations use piston compressors 25. When a pump is opening at a vacuum of 4 in Hg,
available as stock items in size ranging from which of the following is not correct?
15. Choking is A. 1 𝑓𝑡 3 /min to 2000 𝑓𝑡 3 /min A. The pressure is 25.92 in Hg
A. Change of mass flow rate in proportion to B. 30 𝑓𝑡 3 /min to 2500 𝑓𝑡 3 /min B. The pressure is 10.721 psia
pressure ratio C. 1 𝑓𝑡 3 /min to 2500 𝑓𝑡 3 /min C. The pressure is 158.4 torr
B. Change of mass flow rate in inverse D. 30 𝑓𝑡 3 /min to 2000 𝑓𝑡 3 /min D. The pressure is 0.8663 atm
proportion to pressure ratio
C. Fixed mass flow rate irrespective of pressure Answer: B Answer: C
D. All of the above
21. Most permanent installations use piston compressors 26. The amount of energy actually entering the fluid from
Answer: C available as stock items of what pressure? a pump is the
A. 150 psi A. Brake horsepower
16. Crankshaft of a reciprocating compressor is basically B. 200 psi B. Hydraulic horsepower
made of what? C. 250 psi C. Theoretical horsepower
A. Semi-steel D. 300 psi D. Hydraulic, theoretical, or water horsepower
B. Aluminum alloy
C. Steel forging Answer: A Answer: D
D. Cast iron
22. When the piston is top center of the cylinder in a 27. Primary purpose of a pump in a fluid loop is to
Answer: D double-acting reciprocating compressor: A. Add energy to the flow
A. All suction valves are open and all discharge B. Add mass to the flow
17. Most plant air systems to operate the tools and valves are closed C. Extract energy from the flow
machines which require 70 to 90 psig are maintained B. All suction valves are closed and all discharge D. None of the above
at_______. valves are closed
A. 90 to 110 psig C. All suction valves are closed and all discharge Answer: A
B. 110 to 130 psig valves are open
C. 130 to 150 psig 28. The isentropic efficiency of a pump is given by the
A. Ratio of actual to ideal energy extracted 33. Past ME Board Question Answer: A
B. Ratio of ideal to actual energy supplied An aftercooler on a reciprocating air compressor is
C. Ratio of ideal to actual energy extracted used primarily to: 38. Past ME Board Question
D. Ratio of actual to ideal energy supplied A. Cool the lubricating oil The power required to deliver a given quantity of fluid
B. Condense the moisture in the compressed air against a given head with no losses in the pump is
Answer: B C. Improve compressor efficiency called:
D. Increase compressor capacity A. Wheel power
29. The electrical efficiency of a device is the ratio of the B. Brake power
: Answer: B C. Hydraulic power
A. Electrical energy output to the electrical energy D. Indicated power
input 34. Past ME Board Question
B. Mechanical energy input to the mechanical A receiver in an air compression system is used to Answer: C
energy output of the device A. Avoid cooling air before using
C. Actual energy extracted to the ideal energy B. Increase the air discharge pressure 39. Past ME Board Question
extracted C. Collect water and grease suspended in the air Fluids that are pumped in processing work are
D. Actual to the ideal energy input D. Reduce the work needed during compression frequently more viscous than water. Which of the
following statement is correct?
Answer: C Answer: C A. Reynolds number varies directly as the viscosity
B. Efficiency of a pump increases as the viscosity
30. The adiabatic pump efficiency is the ratio of the 35. Past ME Board Question increases
A. Electrical energy output to the electrical energy Crankshaft of reciprocating compressor is basically C. Increased fluid friction between the pump parts
input made of: and the passing fluid increased useful work.
B. Mechanical energy input to the electric energy A. Semi-steel D. Working head increases as the viscosity
output of the device B. Aluminum alloy
C. Ideal energy input to the pump to the actual C. Cast iron Answer: C
energy input D. Steel forging
D. Actual energy extracted to the ideal energy 40. Past ME Board Question
extracted Answer: C A reciprocating pump is considered positive
displacement pump because
Answer: C 36. Past ME Board Question A. Displacement of the liquid is affected by the
Cooling water system consists of equipment to displacement of the piston
31. A condenser’s water – circulating pump is belt-driven dissipate heat absorbed by the engine jacket water, B. Positive pressure is given to the liquid
by an electric motor with an adjustable pitch motor lube oil and the heat to be removed from air C. Liquid is discharge with positive pressure
sheave. This sheave intercooler is measurable to keep the engine outlet D. Liquid is lifted due to the vacuum created inside
A. Permits adjustment of pump speed water temperature constant and the differential of the the cylinder
B. Prevent overload abd/or burnout of the pump and cooling water at a minimum preferably not to exceed:
motor A. 10 to 30 F ̊ Answer: A
C. Prevents excessive wear on the belt and motor B. 10 to 50 F ̊
shaft bearings C. 10 to 20 F ̊ 41. Past ME Board Question
D. Facilitates belt replacement D. 10 to 40 F ̊ To protect adequately the engine bearings, what type
and better arrangement of lubricating oil filter is most
Answer: C Answer: C practical?
A. Full-flow type filter installed between the
32. The function of a pump or compressor is to 37. Past ME Board Question lubricating oil pump and the bearings
A. Transfer heat from one fluid to another Which of the following is one of the most popular B. Duplex filter installed before the lubricating pump
B. Increase the total energy content of the flow types of compressor utilized for supercharging C. Bypass filter with cleanable and replaceable
C. Extract energy from the flow engine? elements
D. Exchange heat to increase energy to the flow A. Roots type blower D. Splash lubricating system in the crank case
B. Pulse turbo charger
Answer: B C. Constant pressure turbo charger Answer: A
D. Turbo compressor
42. Past ME Board Question 47. Past ME Board Question Answer: A
Medium pressure when applied to valves and fittings In order that cavitation will not take place in the
are suitable for a working pressure of: suction line of a pump, what should be the sum of 52. Past ME Board Question
A. 862 to 1200 kPa the velocity head and pressure head at suction The actual head, neglecting the kinetic energy in
B. 758 to 1000 kPa compared to the vapor pressure of the liquid? which the pump work against.
C. 500 to 1000 kPa A. sufficiently lower A. Delivery head
D. 658 to 1050 kPa B. constant B. Pressure head
C. adequately greater C. Velocity head
Answer: A D. equal D. Suction head

43. Past ME Board Question Answer: C Answer: B


The function of an unloader on an electric motor-
driven compressor is to: 48. Past ME Board Question 53. Past ME Board Question
A. Reduce the speed of the motor when the Which of the following components of a pump Flow of water in a pipe have a velocity at 10 meters
maximum pressure is reached converts mechanical energy to pressure energy? per second. Determine the velocity head of the water
B. Drain the condensate from the cylinder A. Impeller A. 50.1 meters
C. Release the pressure in the cylinder in order to B. Valve B. 5.1 meters
reduce the starting load C. Shaft C. 8.2 meters
D. Prevent excess pressure in the receiver D. Delivery pipe D. 100 meters

Answer: C Answer: A Answer: B

44. Past ME Board Question 49. Past ME Board Question 54. Past ME Board Question
An unloader is used on air compressor to: Mr. De La Cruz wanted to buy a pump for his farm. Find the velocity head for a velocity of 18 m/s
A. Relieve air pressure What is suitable for deepwell? A. 33.0 m
B. Start easier A. Reciprocating B. 0.92 m
C. Stop easier B. Airlift C. 1.8 m
D. Run faster C. Hand lift D. 16.5 m
D. Centrifugal
Answer: A Answer: D
Answer: D
45. Past ME Board Question 55. Past ME Board Question
The performance of a reciprocating compressor is 50. Past ME Board Question The size of a steam reciprocating pumps is generally
expressed as: A tank contains H2O. what is the intensity of designated by a three-digit number size as 646. The
A. Adiabatic work divided by adiabatic input pressure at a depth of 6 meters? first digit designates.
B. Adiabatic work divided by indicated work A. 68 kPa A. Stroke of the pump in inches
C. Isothermal work divided by indicated work B. 58.8 kPa B. Inside diameter of the steam cylinder measured
D. Isothermal work divided by adiabatic work C. 78.0 kPa in inches
D. 48.7 kPa C. Percent clearance
Answer: B D. Number of cylinders
Answer: B
46. Past ME Board Question Answer: B
For reciprocating compressor slip at positive or 51. Past ME Board Question
negative displacement: In order to avoid cavitation the NPSH of an 56. Past ME Board Question
A. 𝐶𝑑 = 1 installation should be: If 𝑄𝑎 is the actual discharge flow and 𝑄𝑏 is the
B. 𝐶𝑑 > 1 A. At least equal or greater than the NPSH of the theoretical discharge flow, what will the coefficient of
C. 𝐶𝑑 < 1 pump discharge be equal to during positive displacement
D. 𝐶𝑑 = 0 B. At least equal or less than the NPSH of the pump slip?
C. Equal to the NPSH of the pump only A. 𝑄𝑎 x 𝑄𝑏
Answer: C D. Greater than the NPSH of the pump only B. 𝑄𝑏 /𝑄𝑎
C. 𝑄𝑎 /𝑄𝑏
D. 1 − 𝑄𝑎 /𝑄𝑏 Answer: C Answer: A

Answer: C 62. A centrifugal pump characterized by a housing which 67. A timber, concrete or masonry enclosure having a
is split parallel to the shaft screened inlet kept partially filled with water by an
57. Past ME Board Question A. Horizontal split case pump open body of water such as pond, lake, or steams.
Two pumps are connected in series. If 𝑄1 is the B. End suction pump A. Aquifer
discharge of pump 1 and 𝑄2 is the discharge of pump C. In line pump B. Wet pit
2 where 𝑄1 <𝑄2 . What is the discharge? D. Vertical shaft turbine pump C. Ground water
A. 𝑄2 D. Well water
B. 𝑄1 Answer: A
C. 𝑄1 +𝑄2 Answer: B
D. 𝑄1 /𝑄2 63. A centrifugal pump with one or more impellers
discharging into one or more bowls and a vertical 68. Water which is available from a well, driven into
Answer: A eductor or column pipe used to connect the bowls to water bearing subsurface strata (aquifer)
the discharge head on which the pump driver is A. Aquifer
58. A pump in which the pressure is developed mounted. B. Wet pit
principally by the action of centrifugal force A. Horizontal split case pump C. Ground water
A. Centrifugal pump B. End suction pump D. Well water
B. End suction pump C. In line pump
C. In line pump D. Vertical shaft turbine pump Answer: C
D. Horizontal pump
Answer: D 69. The level with respect to the pump, of the body of
Answer: A water from which it takes suction when the pump is
64. A pump that takes suction from a public service main not in operation.
59. A single suction pump having its suction nozzle on or private use water system for the purpose of A. Static water level
the opposite side of the casing from the stuffing box increasing the effective water pressure. B. Pumping water level
and having the face of the suction nozzle A. Horizontal split case pump C. Suction head
perpendicular to the longitudinal axis of the shaft. B. Submersible pump D. Discharge head
A. Centrifugal pump C. Booster pump
B. End suction pump D. Vertical shaft turbine pump Answer: A
C. In line pump
D. Horizontal pump Answer: C 70. The level with respect to the pump, of the body of
water from which it takes suction when the pump is in
Answer: B 65. A vertical turbine pump with the pump and motor operation.
closed coupled and design to be installed A. Static water level
60. A pump with the shaft normally in a horizontal underground, as in the case of a deepwell pump. B. Pumping water level
position A. Horizontal split case pump C. Suction head
A. Centrifugal pump B. Submersible pump D. Drawdown
B. End suction pump C. Booster pump
C. In line pump D. Vertical shaft turbine pump Answer: B
D. Horizontal pump
Answer: A 71. The vertical difference between the pumping water
Answer: D level and the static water level.
66. An underground formation that contains sufficient A. Static water level
61. A centrifugal pump whose drive unit is supported by saturated permeable material to yield significant B. Pumping water level
the pump having its suction and discharge flanges on quantities of water. C. Suction head
approximately the same center. A. Aquifer D. Drawdown
A. Horizontal split case pump B. Wet pit
B. End suction pump C. Ground water Answer: D
C. In line pump D. Well water
D. Vertical shaft turbine pump
72. Imparts velocity to the liquid, resulting from Answer: B C. Seal gage
centrifugal force as the impeller is rotated. D. Mechanical seal
A. Impeller 78. Which of the following is used to mount unit rigidly
B. Casing and support bearings. Answer: D
C. Stuffing box A. Wearing rings
D. Shaft sleeve B. Stuffing box 84. A positive displacement pump consisting of a fixed
C. Frame casing containing gears, cams, screws, vanes,
Answer: A D. Coupling plungers or similar elements actuated by rotation of
the drive shaft.
73. Gives direction to the flow from the impeller and Answer: C A. Rotary pumps
converts this velocity energy into pressure energy. B. Reciprocating pumps
A. Impeller 79. Connects the pump to the driver. C. Deep well pumps
B. Casing A. Wearing rings D. Centrifugal pumps
C. Stuffing box B. Bearings
D. Shaft sleeve C. Frame Answer: A
D. Coupling
Answer: B 85. A positive displacement unit wherein the pumping
Answer: D action is accomplished by the forward and backward
74. A means of throttling the leakage which would movement of a piston or plunger inside a cylinder
otherwise occur at the point of entry of the shaft into 80. The most common means of throttling the leakage usually provided with valves.
the casing. between the inside and outside of the casing. A. Rotary pumps
A. Impeller A. Packing B. Reciprocating pumps
B. Casing B. Gland C. Deep well pumps
C. Stuffing box C. Seal gage D. Centrifugal pumps
D. Shaft sleeve D. Mechanical seal
Answer: B
Answer: B Answer: A
86. A type of rotary pump consists of an eccentrically
75. Protects the shaft where it passes through the 81. To position and adjust the packing pressure. bored cam rotated by a shaft concentric in a
stuffing box. A. Packing cylindrically bored casing, with an aburment or
A. Impeller B. Gland follower so arranged that with each rotation of the
B. Casing C. Seal gage drive shaft a positive quantity of liquid is displaced
C. Stuffing box D. Mechanical seal from the space between the cam and the pump
D. Shaft sleeve casing.
Answer: B A. Cam and piston pump
Answer: D B. Gear pump
82. Provides passage to distribute the sealing medium C. Screw pump
76. Keeps internal recirculation down to a minimum. uniformly around the portion of the shaft that passes D. Vane pump
A. Wearing rings through the stuffing box. Also known as water seal of
B. Bearings lantern ring. Answer: A
C. Frame A. Packing
D. Coupling B. Gland 87. A type of rotary pump consists of two or more gears,
C. Seal gage operating in closely fitted casing so arranged that
Answer: A D. Mechanical seal when the gear teeth unmesh on one side liquid fills
the space between the gear teeth and is carried
77. Accurately locate shaft and carry radial and thrust Answer: C around in the tooth space to the opposite side and
loads. displaced as the teeth mesh again.
A. Wearing rings 83. Provides a mechanical sealing arrangement that A. Cam and piston pump
B. Bearings takes the place of the packing. B. Gear pump
C. Frame A. Packing C. Screw pump
D. Coupling B. Gland D. Vane pump
Answer: B C. Power driven pump 1. One of the reasons for insulating the pipes is:
D. Piston pump A. They may not break under pressure
88. A type of rotary pump consists of two or three screw B. There is minimum corrosion
rotors so arranged that as the rotors turn liquid fills Answer: C C. Capacity to withstand pressure is increased
the shape between the screw threads and is D. Heat loss from the surface is minimized
displaced axially as the rotor threads mesh. 93. A type of deep well pump which are refinement of the
A. Cam and piston pump old hand pump that have played such an important Answer: D
B. Gear pump role in country home and small town water supply
C. Screw pump from wells. 2. The rate of radiant energy, that is emitted by a
D. Vane pump A. Plunger pump surface at any temperature and in small
B. Turbine pump wavelengths is found from the known rate of
Answer: C C. Ejector centrifugal pump energy that under the same conditions will be
D. Air lift emitted from a black surface, by multiplying with
89. A type of rotary pump consists of one rotor in a the absorptivity. The above enunciation is called:
casing machined eccentrically to the drive shaft. The Answer: A A. Lambert’s law
rotor is fitted with a series of vanes, blades or B. Kirchhoff’s law
buckets which follow the bore of the casing thereby 94. A type of deep well pump that represent the C. Planck’s law
displacing the liquid with each revolution of the drive application of vertical centrifugal pump to deep well D. Stefan Boltzmann’s law
shaft. service and are built for heads up to 305 meters and
A. Cam and piston pump for capacities up to 28,495 liters per minute. Answer: B
B. Gear pump A. Plunger pump
C. Screw pump B. Turbine pump 3. Which of the following is generally used to
D. Vane pump C. Ejector centrifugal pump measure the temperature inside the furnace?
D. Air lift A. Mercury thermometer
Answer: D B. Alcohol thermometer
Answer: B C. Ash thermometer
90. A type reciprocating pump having a steam cylinder D. Optical pyrometer
with no lap on valves, a water cylinder and a 95. A type of deep well pump that has come into wide
common piston rod. use for small capacities combines a single stage Answer: D
A. Direct acting steam pump centrifugal pump at the top of the well and an ejector
B. Crank and flywheel reciprocating pump or jet located down in the water. 4. All heat transfer processes:
C. Power driven pump A. Plunger pump A. Involve transfer of energy
D. Piston pump B. Turbine pump B. Involve temperature difference between the
C. Ejector centrifugal pump bodies
Answer: A D. Air lift C. Obey first law of thermodynamics
D. Obey second law of thermodynamics
91. A type of reciprocating pimp driven by compound, Answer: C
cross compound, or triple expansion steam engines. Answer: B
A. Direct acting steam pump 96. A type of deep well pump wherein compressed air is
B. Crank and flywheel reciprocating pump admitted to the well to lift water to the surface, for 5. What is thermal diffusivity?
C. Power driven pump successful operation of the system, the discharge A. A mathematical formula
D. Piston pump pipe must have its lower end submerged in the well B. A physical property of the material
water. C. A configuration for heat conduction
Answer: B A. Plunger pump D. A dimensionless parameter
B. Turbine pump
92. A type of reciprocating pump that receives its forward C. Ejector centrifugal pump Answer: B
and backward motion of the piston and plunger from D. Air lift
the rotary motion of a revolving crankshaft by means 6. Which of the following is a unit of thermal
of a crank and connecting rod. Answer: D diffusivity?
A. Direct acting steam pump A. m2 /hr
B. Crank and flywheel reciprocating pump CHAPTER 13 – HEAT TRANSFER B. kcal/m2 hr
C. kcal/m2 hr C
̊ 12. Thermal conductivity of wood depends on which of B. Radiation from suspended larger particles of
D. m2 /hr C
̊ the following? coal, coke, or ash contributing to flame
A. Moisture lumininossity
Answer: A B. Temperature C. Infared radiation from water vapor and
C. Density carbon dioxide
7. Non-isotropic conductivity is shown by which of D. All of the above D. All of the above
the following?
A. Brass Answer: D Answer: D
B. Copper
C. Wood 13. A fur coat on an animal will help the animal to 18. The statement that the emissivity and absorptivity
D. Steel remain: of a surface is surrounded by its own temperature
A. Warm in winter are the same for both monochromatic and total
Answer: C B. Cool in winter radiation is called:
C. Warm in summer A. Lambert’s law
8. For glass wool thermal conductivity changes from D. Cool in summer B. Kirchhoff’s law
sample to sample due to changes in: C. D’Alambart’s
A. Structure Answer: A D. Law of emissivity
B. Density
C. Composition 14. The nature of flow of a fluid inside a tube, whether Answer: B
D. All of the above it is turbulent or laminar, can be ascertained by:
A. Flow velocity 19. A reservoir that supplies energy in the form of heat
Answer: D B. Surface conditions is called:
C. Viscosity of fluid A. Source
9. Which of the following is the S.I. unit of thermal D. Reynolds number B. Sink
conductivity? C. Cold reservoir
A. W/m-hr- K ̊ Answer: D D. Heat reservoir
B. W/m K ̊
C. KJ/m-hr- C ̊ 15. By which of the following modes of heat transfer is Answer: A
D. W/m-hr- C ̊ the Stefan-Boltzmann law applicable?
A. Conduction 20. In regenerator type heat exchanger, heat transfer
Answer: B B. Radiation occurs by:
C. Conduction and radiation combined A. Direct mixing of hot and cold fluids
10. What is the value of the Prandt’l number for air? D. Convection and radiation combined B. A complete separation between hot and cold
A. 10 fluids
B. 6.7 Answer: B C. Flow of hot and cold fluids alternately over a
C. 67 surface
D. 0.7 16. At all wave lengths and temperatures the D. Generation of heat again and again
monochromatic emissivity of a white body is equal
Answer: D to: Answer: C
A. Zero
11. According to Prevost theory of heat exchange, B. 0.5 21. Least value of Prandt’l number can be expected in
A. It is impossible to transfer heat from low C. Unity case of ____________.
temperature source to high temperature D. 0.1 to 0.5 A. Liquid metals
source B. Sugar solution
B. Heat transfer by radiation needs no medium Answer: A C. Salt solution
C. All bodies above absolute zero emit radiation D. Water
D. Heat transfer in most of the cases occurs by 17. The radiation from flames is having
combination of conduction, convection and A. Continuous radiation from burning soot Answer: A
radiation particles of microscopic and submicroscopic
dimensions
Answer: C
22. “The boiling point of a solution is a linear function Answer: B A. Ion
of water at the same pressure.” The above B. Isotope
statement is called ___________. 27. In sugar mills can just is evaporation in: C. Molecule
A. Dubring’s rule A. Zigzag tube evaporators D. Hole
B. Petit and Dulong’s law B. Long vertical tube evaporators
C. Fick’s rule C. Short vertical tube evaporators Answer: A
D. Reynolds law D. Horizontal tube evaporators
33. The energy of a body that can be transmitted in
Answer: A Answer: B the form of heat.
A. Heat energy
23. Floating heads are provided in heat exchangers to 28. A 1-2 heat exchanger refers to which of the B. Thermal energy
A. Increase the pressure drop following? C. Entropy
B. Decrease the pressure drop A. Single pass on shell side and double pass on D. Internal energy
C. Facilitate maintenance tube side
D. Avoid deformation of tubes because of B. Single pass on tube side and double pass on Answer: B
thermal expansion shell side
C. Single liquid cools two liquids at different 34. In an isometric process, the heat transferred is
Answer: D temperature equal to:
D. Two tubes of cold fluid pass through one tube A. Change in enthalpy
24. What do you call the first stage of crystal of hot fluid B. Change in entropy
formation? C. Change in internal energy
A. Nucleation Answer: A D. Work nonflow
B. Foaming
C. Separation 29. A correction of LMTD is essential in case of: Answer: C
D. Vortexing A. Parallel flow heat exchanger
B. Counter current heat exchanger 35. A substance that is able to absorb liquids or gases
Answer: A C. Cross flow heat exchanger and is used for removing them from a given
D. None of the above medium or region.
25. In heat exchanger design, one transfer unit A. Absorbent
implies: Answer: C B. Cohesive
A. One fluid which is exchanging with another C. Adsorbent
fluid of the same chemical composition 30. Which of the following is used as entrainer in D. Adhesive
B. The section of heat exchanger which will acetic acid – water separation?
cause temperature drop of one degree A. Methyl alcohol Answer: A
centigrade B. Phosphorous
C. The section of heat exchanger where heat C. Butyl acetate 36. Radiant heat transfer is described by:
transfer surface area has been one square D. Hexane A. Newton’s law
meter B. Fourier’s law
D. Condition when the change in temperature of Answer: C C. The logarithmic mean temperature
one steam is numerically equal to the D. Kirchhoff’s law
average driving force 31. A type of radiation consisting of singly charged
particles that generate to intermediate distances. Answer: D
Answer: D A. Nuclear radiation
B. Alpha radiation 37. A reservoir that absorbs energy in the form of heat
26. Dritus Boelter equation can be applied in case of C. Beta radiation is called __________.
fluids flowing in: D. Gamma radiation A. Source
A. Transition region B. Sink
B. Turbulent region Answer: C C. Cold reservoir
C. Laminar region D. Heat reservoir
D. Any of the above 32. An electrically charged atom or radical which
carries electricity through an electrolyte is called: Answer: B
38. When the entire heat exchanger is selected as Answer: A A. Reversible process
control volume, heat becomes __________. B. Irreversible process
A. Unity 44. Radiation heat transfer is described by C. Cycle
B. Zero A. Newton’s law D. Isentropic process
C. Undefined B. The logarithmic mean temperature difference
D. Indeterminate C. Fourier’s law Answer: C
D. Kirchoff’s law
Answer: B 50. Past ME Board Question
Answer: D A theoretical body which when heated to
39. Heat is conducted in the direction of: incandescence would emit a continuous light-ray
A. Increasing temperature 45. The equivalent of ratio of emissive power to spectrum.
B. Decreasing temperature absorptivity for bodies in thermal equilibrium is A. Black body radiation
C. Increasing and decreasing temperature described by: B. Black body
D. Constant temperature A. Newton’s law C. Blue body
B. The logarithmic mean temperature difference D. White body
Answer: B C. Fourier’s law
D. Kirchoff’s law Answer: B
40. The heat transfer term in the first law of
thermodynamics may be due to any of the Answer: D 51. Past ME Board Question
following except: Which of the following is the reason for insulating
A. Conduction 46. The temperature potential between temperature at the pipes?
B. Convection the two ends of a heat exchanger are given by: A. They may not break under pressure
C. Radiation A. The logarithmic mean temperature difference B. There is minimum corrosion
D. Internal heat generation (e.g., chemical B. The Stefan-Boltzmann law C. Capacity to withstand pressure
reaction) C. Fourier’s law D. Heat loss from the surface is minimized
D. Kirchoff’s law
Answer: D Answer: D
Answer: A
41. All heat transfer processes require a medium of 52. Past ME Board Question
energy exchange except: 47. The function of a heat exchanger is to: Heat transfer due to density differential
A. Conduction A. Increase the water temperature entering the A. Convection
B. Natural convection boiler and decrease combustion B. Nuclear
C. Forced convection requirements C. Conduction
D. Radiation B. Transfer heat from one fluid to another D. Radiation
C. Increase the total energy content of the flow
Answer: D D. Exchange heat to increase energy to the flow Answer: A

42. Thermal conduction is described by: Answer: B 53. Past ME Board Question
A. Newton’s law The term “exposure” in radiological effects is used
B. The logarithmic mean temperature difference 48. The function of a superheater is to: as a measure of a gamma ray or an X-ray field in
C. The Stefan-Boltzmann law A. Increase the water temperature entering the the surface of an exposed object. Since this
D. Fourier’s law boiler and decrease combustion radiation produces ionization of the air surrounding
requirements the object, the exposure is obtained as
Answer: D B. Transfer heat from one fluid to another A. Number of ions produced per mass of air x
C. Increase the total energy content of the flow coulombs per kg
43. Convection is described by which of the following D. Exchange heat to increase energy to the flow B. Mass of air x surface area of an exposed
laws? object
A. Newton’s law Answer: D C. Mass of air over surface area of an exposed
B. The logarithmic mean temperature difference object
C. The Stefan-Boltzmann law 49. What is the series of processes that eventually D. Number of ions produced per mass of air +
D. Fourier’s law bring the system back to its original condition? coulombs per kg
Answer: A D. Thermal radiation C. Steam
D. Copper
54. Past ME Board Question Answer: A
The passing of heat energy from molecule to Answer: B
molecule through a substance 59. Past ME Board Question
A. Conduction A body that is hot compared to its surroundings 64. What do you call the effectiveness of a body as a
B. Radiation illuminates more energy than it receives, while its thermal radiator at a given temperature?
C. Conservation surrounding absorbs more energy than they give. A. Absorptivity
D. Convection The heat is transferred from one to another by B. Conductivity
energy wave motion. What is this mode of heat C. Emissivity
Answer: A transfer? D. Reflectivity
A. Radiation
55. Past ME Board Question B. Conduction Answer: C
The radiant heat transfer depends on: C. Convection
A. Temperature D. Condensation 65. The natural direction of the heat flow between two
B. Heat rays reservoirs is dependent on which of the following?
C. Heat flow from cold to hot Answer: A A. Their temperature difference
D. Humidity B. Their internal energy
60. What is the heat transfer due to density C. Their pressures
Answer: B differential? D. Their states, whether solid, liquid and gas
A. Convection
56. Past ME Board Question B. Conduction Answer: A
What kind of heat exchanger where water is C. Nuclear
heated to a point that dissolved gases are D. Radiation 66. Why are metals good conductors of heat?
liberated? A. Because they contain free electrons
A. Evaporator Answer: A B. Because their atoms are relatively far apart
B. Condenser C. Because their atoms collide infrequently
C. Intercooler 61. What do you call the passing of heat energy from D. Because they have reflecting surfaces
D. Deaerator molecule to molecule through a substance?
A. Conduction Answer: A
Answer: D B. Conservation
C. Radiation 67. In natural convection a heated portion of a fluids
57. Past ME Board Question D. Convection moves because:
Heat transfer processes which include a change of A. Its molecular motions become aligned
phase of a fluid are considered __________. Answer: A B. Of molecular collisions within it
A. Convection C. Its density is less than that of the surrounding
B. Thermal radiation 62. The transmission of heat from one place to fluid
C. Conduction another by fluid circulation between the spots of D. Of currents in surrounding fluid
D. Radiation different temperature is called _______.
A. Convection Answer: C
Answer: A B. Conservation
C. Radiation 68. In order to emit electromagnetic radiation, an
58. Past ME Board Question D. Conduction object must be at a temperature:
A hot block is cooled by blowing cool air over its A. Above 0 K
top surface. The heat that is first transferred to the Answer: A B. Above 0 C ̊
air layer close to the block is by conduction. It is C. Above that of its surrounding
eventually carried away from the surface by 63. Which of the following requires the greatest D. High enough for it to glow
___________. amount of heat per kilogram for a given increase
A. Convection in temperature? Answer: A
B. Radiation A. Ice
C. Conduction B. Water
69. The rate at which an object radiates Answer: A 80. What do you call a change of phase directly from
electromagnetic energy does not depend on its: vapor to solid without passing through the liquid
A. Surface area 75. At what particular condition that no more heat can state?
B. Mass that be removed from a substance and the A. Sublimation
C. Temperature temperature can no longer be lowered? B. Solidification
D. Ability to absorb radiation A. Freezing point C. Vaporization
B. Absolute zero D. Deposition
Answer: B C. Critical point
D. Ground zero Answer: D
70. Sublimation refers to:
A. The vaporization of solid without first Answer: B 81. Which of the following is the Stefan-Boltzmann
becoming liquid constant?
B. The melting of a solid 76. What refers to the heat transfer wherein the heat A. 5.77 x 10−7 W/m2 K 4
C. The vaporization of a liquid is transferred from one point to another by actual B. 7.67 x 10−9 W/m2 K 4
D. The condensation of a gas into liquid movement of substance? C. 4.78 x 10−10 W/m2 K4
A. Conduction D. 5.67 x 10−8 W/m2 K 4
Answer: A B. Radiation
C. Convection Answer: D
71. In the process of freeze drying, ice goes directly D. Absorption
into water vapor. What is the temperature at which 82. What is the usual value of transmissivity for
this process can take place? Answer: C opaque materials?
A. Below the triple point of water A. 0
B. At the triple point of water 77. The ratio of the radiation of actual body to the B. Indeterminate
C. Above the triple point of water radiation of a blackbody is known as _______. C. 1
D. Any of the above, depending on the pressure A. Emittance D. Infinity
B. Reflectance
Answer: A C. Absorptance Answer: A
D. Transmittance
72. What usually happen when a vapor condenses 83. A body whose emissivity is less than 1 is known
into a liquid? Answer: A as a real body. What is the other term for real
A. It evolves heat body?
B. It generates heat 78. Which of the following is the usual geometric view A. Gray body
C. Its temperature increases factor for a black body? B. White body
̊
D. It boils with temperature less than 100 C A. Zero C. Black body
B. Infinity D. Theoretical body
Answer: A C. One
D. Indeterminate Answer: A
73. In a cooling tower, the water is cooled mainly by:
A. Condensation Answer: C 84. What refers to an ideal body that absorbs all of the
B. Convection radiant energy that intrudes on it and also emits
C. Evaporation 79. What happens to the heat transferred radially the maximum possible energy when acting as a
D. Conduction across insulated pipe per unit area? source?
A. The heat will flow at constant rate A. White body
Answer: C B. Decreases with the increase in thermal B. Black body
conductivity C. Gray body
74. How do you classify a body that has an emissivity C. Decrease from pipe wall to insulated surface D. Red hot body
factor of 0.7? D. Partially increases from pipe wall to insulated
A. Gray body surface Answer: B
B. Black body
C. White body Answer: C 85. The thermal resistance for one-dimensional steady
D. Theoretical body conduction heat transfer through cylindrical wall in
the radial direction is expressed in which of the A. Enthalpy 95. Which of the following is the emissivity of white
following functions? B. Thermal energy body?
A. Linear C. Entropy A. Zero
B. Exponential D. Internal energy B. 0.5
C. Logarithmic C. 1
D. Trigonometric Answer: B D. 0 < e < 1

Answer: C 91. Which of the following is the equivalent heat Answer: A


transferred of a gas undergoing isometric
86. The law which states that “the ratio of the emissive process? 96. The mechanism of heat transfer in which there is
powers to absorptivities are equal when the two A. Change in enthalpy no medium ( i.e. water, air, solid concrete)
bodies are in thermal equilibrium” is known as: B. Change in entropy required for the heat energy to travel is:
A. Stefan-Boltzmann law C. Change in internal energy A. Conduction
B. Newton’s law of convection D. Work nonflow B. Radiation
C. Fourier’s law C. Convection
D. Kirchhoff’s law of radiation Answer: C D. Diffusion

Answer: D 92. What do you call a substance that is able to Answer: B


absorb liquids or gases and is usually used for
87. It refers to the ratio of the internal thermal removing liquids (or gases) from a given medium 97. The temperature potential between temperature at
resistance of a solid to the boundary layer thermal or region? the two ends of a heat exchanger are given by:
resistance (or external resistance of the body). A. Absorbent A. The logarithmic mean temperature difference
A. Biot number B. Liquifier B. The Stefan-Boltzmann law
B. Prandtl number C. Adsorbent C. Fourier’s law
C. Nusselt number D. Adhesive D. Kirchoff’s law
D. Reynolds number
Answer: A Answer: A
Answer: A
93. In which direction that heat is transferred through 98. Which of the following best describe function of
88. It refers to the ratio at the rate of heat transferred conduction? heat exchanger?
by conduction to the rate of energy stored. A. Increasing temperature A. Increase the water temperature entering the
A. Reynolds number B. Decreasing temperature system
B. Fourier number C. Increasing and decreasing temperature B. Transfer heat from one fluid to another
C. Biot number D. Constant temperature C. Evaluate the total energy of the flow
D. Prandtl number D. Exchange heat to increase energy to the flow
Answer: B
Answer: B Answer: B
94. Which of the following statements is based on
89. A hot block is cooled by blowing cool air over its Prevost theory of heat exchange? 99. What refers to a form of energy associated with
top surface. The heat that is first transferred to the A. All bodies above absolute zero emit radiation the kinetic random motion of large numbers of
air layer close to the block is by conduction. It is B. The substance moves because of the molecules?
eventually carried away from the surface by: decrease in its density which is caused by A. Heat
A. Conduction increase in temperature B. Heat of fusion
B. Radiation C. The substance moves because of the C. Entropy
C. Thermal application of mechanical power such as that D. Internal energy
D. Convection of a fan
D. Heat transfer in most of the cases occurs by Answer: A
Answer: D combination of conduction, convection and
radiation 100. How much id the part of light that is absorbed by
90. It is the term used to describe the energy of a the body that transmits and reflects 80% and 10%
body that can be transmitted in the form of heat. Answer: A respectively?
A. 10% transport by diffusion in the velocity and thermal energy of wave motion. What is this mode of heat
B. 30% boundary layer? transfer?
C. 20% A. Nusselt’s number A. Radiation
D. 5% B. Prandtl number B. Conduction
C. Reynold’s number C. Convection
Answer: A D. Dimensional measurement D. Condensation

101. In convection heat transfer, what happens to the Answer: B Answer: A


heat transfer coefficient if the viscosity of the fluid
increases? 106. Which of the following is the property of the solid 110. What do you call theoretical body where
A. The heat transfer coefficient will increase that provides the measure of the rate of heat absorptivity and emissivity are independent of the
B. The heat transfer coefficient will decrease transfer to the energy storage? wavelength over the spectral region of the
C. The heat transfer coefficient remains A. Thermal efficiency irradiation and the surface emission?
constant B. Thermal diffusivity A. White body
D. None of the above C. Thermal conductivity B. Opaque body
D. Thermal radiography C. Black body
Answer: B D. Transparent body
Answer: B
102. How do you call a phenomenon wherein the heat Answer: D
is transferred by motion of fluid under the action of 107. Two metals were kept together at room
mechanical device? temperature and it was found out that one is 111. Which of the following is the structure designed to
A. Forced convection colder than the other. Which of the following is the prevent the spread of fire having a fire resistance
B. Natural convection best reason why one metal is colder than the rating of not less than four hours?
C. Forced conduction other? A. Fire escape
D. Thermal radiation A. The heat transfer coefficient of one metal is B. Fire exit
of higher C. Fire shield
Answer: A B. One metal is of lower temperature as D. Fire wall
compared to the other
103. In conduction heat transfer, what happens to the C. One metal is of higher temperature as Answer: D
heat transfer per unit time when the thermal compared to the other
conductivity decreases? D. The thermal conductivity of one metal is high 112. Which of the following heat exchange device used
A. The heat flow will increase as compared to the other to provide heat transfer between the exhaust
B. The heat flow remains constant gases and air prior to the entrance of a
C. The heat flow will decrease Answer: D combustor?
D. The heat flow will partially increase and then A. Regenerator
will decrease 108. In convection heat transfer, what happens to the B. Economizer
heat transfer coefficient if the viscosity of the fluid C. Condenser
Answer: C decreases? D. Reheater
A. The heat transfer coefficient also increases
104. Which of the following is the driving force in heat B. The heat transfer coefficient will decrease Answer: A
transfer? C. The heat transfer coefficient remains
A. Temperature gradient constant 113. Which of the following transfer of heat is involved
B. Thickness gradient D. The heat transfer coefficient partially in the changing of boiling water (at 100 C̊ ) to
C. Viscosity gradient increases then decreases vapor at the same temperature?
D. Dielectric gradient A. Conduction
Answer: A B. Convection
Answer: A C. Radiation
109. A body that is hot compared to its surroundings D. Evaporation
105. Which of the following is the measure of the illuminates more energy than it receives, while its
relative effectiveness of momentum and energy surrounding absorbs more energy than they give. Answer: B
The heat is transferred from one to another by
114. Which of the following is the science of low B. The steam has higher specific heat Answer: A
temperatures? C. Steam contains more internal energy
A. Cryogenics D. Steam is everywhere thus it strikes greater 125. The rate at which heat flows through a slab of
B. Thermo-kinetics force some material does not depend on which of the
C. Thermodynamics following?
D. Ergonomics Answer: C A. The thickness of the slab
B. The area of the slab
Answer: A 120. What usually happens when vapor condenses into C. The temperature difference between two
liquid? faces
115. Which of the following thermal state of the body A. It absorbs heat D. The specific heat of the material
considered as reference to communicate heat to B. It rejects heat
the other bodies? C. Its temperature increases Answer: D
A. Temperature D. Its temperature decreases
B. Pressure 126. Which of the following is the primary function of a
C. Internal energy Answer: B thermal radiator?
D. Entropy A. To transferring the heat by using moving
121. Which of the following has the highest thermal fluids
Answer: A conductivity? B. To transfer heat from hot to cold body by
A. Mercury using a forced-draft fan
116. The true mean temperature difference is also B. Gasoline C. To transfer heat by allowing molecules to
known as: C. Water vibrate one to another
A. The average mean temperature difference D. Alcohol D. To transfer heat with or without a medium
B. The logarithmic mean temperature difference
C. The trigonometric mean temperature Answer: A Answer: D
difference
D. The exponential temperature difference 122. Which of the following is the requirement of the 127. What is the reason why Styrofoam is a good
temperature of a body for it to emit radiation? insulating material?
Answer: B A. Above zero Celsius A. Because it contains many tiny pockets of air
B. Above zero Kelvin B. Styrofoam is a white object
117. Which of the following can be a geometric view C. Above of the temperature of the surroundings C. The structure of Styrofoam is very unstable
factor of a gray body? D. High enough for it to glow and heat cannot flow
A. Greater than one D. Styrofoam structure is very dense
B. Less than one Answer: B
C. Equal to one Answer: A
D. Greater than zero but less than one 123. Which of the following is the color of iron when it is
heated to a highest temperature? 128. What usually happens to the surrounding when
Answer: D A. White water vapor condenses?
B. Red A. It warms the surrounding
118. The heat transfer by conduction occurs in which of C. Orange B. The surrounding temperature decreases
the following? D. Yellow C. It neither warm nor cold the surrounding
A. Only in liquids D. The surroundings will be dehumidified
B. Only in solids Answer: A
C. Only in liquids and gases Answer: A
D. In solids, liquids and gases 124. Which of the following is the reason why metals
are good conductors of heat? 129. The rate of radiation does not depend on which of
Answer: D A. Metals contain free electrons the following?
B. Metals have atoms the frequently collide A. Temperature of the radiating body
119. Which of the following reasons why one gram of another B. The emissivity of the radiation surface
steam at 100 C̊ causes more serious burn than C. Metals have reflecting surfaces C. The area of the radiating body
one gram of water at 100 C̊ ? D. Atoms in metals are very far to each other D. The thickness of the radiating body
A. Steam is less dense than boiling water
Answer: D B. Decreases except for ferrous metals Answer: A
C. Almost constant except for ferromagnetic
130. Which of the following is not a good conductor of materials 141. The heat transfer by convection occurs in which of
heat? D. Increases except for steel the following?
A. Metals A. Only in gases
B. Rocks Answer: C B. Only in liquids
C. Glass C. Only in gases and liquids
D. Asbestos 136. Which of the following liquids that has the highest D. Only in gases and solids
thermal conductivity?
Answer: D A. Gasoline Answer: C
B. Glycerin
131. Which of the following is not a unit of the rate of C. Water 142. In convection heat transfer, what mechanism heat
heat transfer? D. Alcohol transfer where the fluid moves due to the
A. Watt decrease in its density caused by increase in
B. Btu per hour Answer: C temperature?
C. Cal/s A. Forced convection
D. Btu/Hp-hr 137. Which of the following is not a heat exchanger? B. Natural convection
A. Condenser C. Density convection
Answer: D B. Boilers D. Radial convection
C. Evaporators
132. The thermal conductivity does not depend on D. Water hammer Answer: B
which of the following?
A. Chemical composition Answer: D CHAPTER 14 – REFRIGERATION
B. Physical state or texture
C. Temperature and pressure 138. Which of the following heat exchangers where 1. Which of the following could be used to check a
D. Gravitational pull fluid flow in the same direction and both are of leak in an ammonia system?
changing temperatures? A. Litmus paper
Answer: D A. Parallel flow B. Halide torch
B. Cross flow C. Sulfur stick
133. In Maxwell’s theory for thermal conductivity of C. Counter flow D. A and C
gases and vapors, which of the following is the D. Mixed flow
value of “a” for triatomic gases? Answer: D
A. 1.7 Answer: A
B. 2.4 2. Ammonia leaks in the condenser can be detected
C. 1.3 139. What happens to the thermal conductivity of by:
D. 2.4 diatomic gases if the temperature is increase? A. Smelling the discharge water
A. The thermal conductivity will also increase B. Applying litmus paper to the circulating water
Answer: A B. The thermal conductivity decreases discharge
C. The thermal conductivity remains constant C. Adding oil of peppermint to the system and
134. Which of the following conductivities where D. The thermal conductivity partially increases tracing the smell
Sutherland equation is used? then decreases D. Applying a soapy solution to the condenser
A. Thermal conductivities of solids heads and looking for bubbles
B. Thermal conductivities of gases Answer: A
C. Thermal conductivities of metal Answer: B
D. Thermal conductivities of liquids 140. What device is used to measure the amount of
infrared radiation in each portion of a person’s skin 3. A sulfur stick burning in the presence of ammonia
Answer: B that is emitted? will give off a _______.
A. Thermograph A. Dense yellow smoke
135. For pure metals, what happens to the thermal B. Thermometer B. Dense red smoke
conductivity if the temperature is extremely high? C. Pyrometer C. Dense white smoke
A. Approaches infinity D. Potentiometer D. Dense green smoke
Answer: C B. Pump air out of the system A. On the receiver discharge
C. Add refrigerant to the system B. In the highest part of the system
4. An ammonia leak will turn litmus paper _______. D. Reduce the pressure on the discharge side of C. In the lowest part of the system
A. Blue the condenser D. On the evaporators coils
B. Green
C. Red Answer: A Answer: B
D. Yellow
10. How is an ammonia system purged so that 16. The solenoid valve is:
Answer: A operator will not be overcome by the fumes? A. A pressure-controlled stop valve
A. Into a bucket of lube oil B. A temperature-controlled stop valve
5. In an ammonia system the oil gauge must be kept: B. Back into the compressor C. A Freon-controlled check valve
A. Closed except when checking oil level C. Into a bucket of water D. None of the above
B. Open at all times D. Into the atmospheric line
C. Close when machine is shut down Answer: B
D. Open when machine is shut down Answer: C
17. The thermal expansion valve is located between
Answer: A 11. Thermal expansion valves are usually of the: the:
A. Diaphragm A. Solenoid valve and the evaporator coils
6. Valves and piping in an ammonia system are B. Magnetic tape B. Charging valve and the solenoid valve
made of: C. Bellows type C. Receiver and the king valve
A. Brass D. A or C D. King valve and the solenoid valve
B. Bronze
C. Iron Answer: D Answer: A
D. Copper
12. When checking zinc plates in a condenser, one 18. The purpose of the scale trap is to:
Answer: C should: A. Dissolved scale and dirt in the system
A. Install all new plates B. Remove insoluble gases from the refrigerant
7. The relief valve on an ammonia machine is B. Clean the plates and renew worn out ones C. Remove dirt, scale and metal chips from the
located: C. Ground each plate to the shell refrigerant
A. On the discharge pipe from the condenser D. Paint the plates with red lead D. Control the amount of scale going to the
B. On the discharge pipe from the compressor compressor
C. In the compressor head Answer: B
D. A and B Answer: C
13. The scale trap is located between the:
Answer: C A. Compressor and oil separator 19. The oil separator (trap) is located between the:
B. Expansion valve and evaporators coils A. Compressor discharge valve and condenser
8. When purging an ammonia condenser into a C. King (liquid) valve and expansion valve B. Receiver and the expansion valve
bucket of water, one can tell when the air is out D. Evaporator coils and compressor C. Receiver and the king valve
and ammonia starts to come through by the: D. Condenser and the receiver
A. Smell of the ammonia being liberated from Answer: C
the water Answer: A
B. Color of the water turning green 14. When the evaporator coils are located in the
C. Color of the water turning bluish icebox, the system is known as: 20. The charging connection in a refrigerating system
D. Change of bubbling sound of air to the A. Indirect system is located:
crackling sound of ammonia B. Direct system A. Before the receiver
C. High-pressure system B. Between the condenser and the receiver
Answer: D D. Low-pressure system C. Between the receiver and the king valve
D. Between the king valve and the solenoid
9. The crossover connection in an ammonia system Answer: B valve
can be used to ___________.
A. Hot-gas defrost 15. The purge valve is located: Answer: D
21. The dehydrator is located between the 27. Which of the following is another name for the D. Valve type
A. Condenser and receiver liquid valve?
B. Receiver and the expansion valve A. Freon valve Answer: B
C. Condenser and the king valve B. King valve
D. Receiver and the king valve C. Shutoff valve 33. The expansion valve is located between the
D. Master valve A. Compressor and condenser
Answer: B B. Evaporator and compressor
Answer: B C. Receiver and evaporator
22. The charging valve is located between the: D. Condenser and receiver
A. King valve and the expansion valve 28. If no gaskets are used in the piping joints of a
B. Evaporator coils and the compressor Freon system, the joints must be: Answer: C
C. Compressor and the receiver A. Welded joints
D. Receiver and the condenser B. Finished joints 34. The oil separator is located between the:
C. Ground joints A. Condenser and dehydrator
Answer: A D. Soldered joints B. Compressor and condenser
C. Evaporator and compressor
23. The solenoid valve is located between the Answer: C D. Solenoid valve and thermal expansion valve
_______.
A. Thermal expansion valve and the evaporator 29. A device for holding open the suction valve and Answer: B
B. Scale trap and the thermal expansion valve drawing gas from the suction manifold and
C. King valve and the scale trap returning it to the suction line without compressing 35. Zinc rods are found in the:
D. Automatic and manual expansion valves it is called ___________. A. Salt-water side of condenser
A. Relief valve B. Compressor crankcase
Answer: B B. Suction line by-pass C. Evaporator coils
C. Cylinder unloader D. Refrigerant side of condenser
24. A device used to keep moisture from passing D. Discharge line by-pass
through the system is called: Answer: A
A. Humidifier Answer: C
B. Dehydrator 36. A double-trunk piston is used to
C. Aerator 30. The solenoid valve can be typed as a _______. A. Prevent oil from mixing with the refrigerant
D. Trap A. Bellows valve B. Absorb some of the side thrust
B. Bimetallic valve C. Prevent gas from getting to crankcase
Answer: B C. Thermal valve D. All of the above
D. Magnetic stop valve
25. The relief valve is located on the: Answer: D
A. Discharge side on the condenser Answer: D
B. Outlet of the evaporator coils 37. Which of the following gasket materials should be
C. Receiver tank 31. The bulb for the thermal expansion valve is used on a Freon system?
D. Discharge side of the compressor located A. Asbestos
A. Near the evaporator coil outlet B. Metallic
Answer: D B. In the middle of evaporator coils C. Rubber
C. Near the evaporator coil inlet D. A and B
26. The dehydrator is used: D. On the bottom row of evaporator coils
A. To remove moisture from the system Answer: D
B. When adding refrigerant to the system Answer: A
C. To remove air from the system 38. Many pressure gauges on a Freon system have
D. A and B 32. The elements of a thermostat switch are usually of two dials or graduations on one gauge. The two
the ___________ dials represent
Answer: D A. Pilot-valve type A. Suction and discharge pressure
B. Bimetal type B. Pressure and temperature
C. Diaphragm type C. Liquid and gas pressure
D. Cooling water inlet and outlet temperatures 43. The discharge pressure of the compressor should 48. Before securing a compressor to do maintenance
be: work on it, be sure to:
Answer: B A. The pressure which corresponds to a A. Purge the system
temperature from 5 F̊ to 15 F ̊ below that of B. Have spare parts ready
39. A double-pipe condenser has ___________. the condenser discharge C. Pump down the system
A. Two pipes for cooling water and one for B. The pressure which corresponds to a D. B and C
refrigerant temperature equal to that of the condenser
B. A small pipe inside a larger pipe, the cooling discharge Answer: D
water passing through the small pipe and the C. The pressure which corresponds to a
refrigerant through the large pipe temperature from 5 F ̊ to 15 F ̊ higher than 49. Obstruction of the expansion valve is usually
C. Two piping systems side by side, one with the condenser discharge caused by
cooling water and one with refrigerant D. None of the above A. Scale
D. None of the above B. Congealed oil in the system
Answer: C C. Water in the system
Answer: B D. Any of the above
44. The purpose of relief valves on refrigeration
40. When there is a Freon leak, the halide torch will machines is to: Answer: D
burn A. By-pass the compressor when dehydrating
A. Orange B. Prevent overloading in the iceboxes 50. Excess frost on the evaporator coils
B. Green C. Prevent excessive pressure in case of A. Does not affect the system
C. Blue stoppage on the discharge side of the system B. Takes load off compressor
D. White D. A and B C. Reduces efficiency of the plant
D. Keeps the icebox cooler
Answer: B Answer: C
Answer: C
41. Large leaks in a Freon system cannot always be 45. A leaky suction valve can usually be detected by
detected with a halide torch because it changes A. A higher suction pressure 51. Air circulation in the icebox is accomplished by the
color with the slightest amount of Freon present. A B. A fluctuating suction pressure gauge use of which of the following?
large leak can be detected easier by applying C. Closing in on the suction valve having no A. Hollow sidewalls
____________ effect on the suction pressure B. Diffuser fans
A. Sheets of litmus paper to all joints and watch D. Any of the above C. Louver doors
for color change D. Air vents to deck
B. A soapsuds solution, mixed with a little Answer: D
glycerin to hold the solution together, and Answer: B
watch for bubbles 46. Too low suction pressure could be caused by
C. A thin layer of mineral oil to all joints and A. Too much oil in the system 52. The cooling-water side of the condenser should be
watch for bubbles B. Shortage of refrigerant gas opened for inspection every
D. A lighted candle at the joints and watch for C. Dirty scale traps A. Six months
leaky spots blowing candle flame D. Any of the above B. Two years
C. Year
Answer: B Answer: D D. Three months

42. The relief valve on a CO2 machine is located 47. What do you call a system in which the evaporator Answer: D
A. Next to the king valve coils are located in a brine solution and the brine is
B. In the compressor head pumped through the icebox? 53. Some causes of a noisy compressor are
C. On the discharge pipe between the A. An indirect system A. Worn bearings, pins, etc.
compressor and the discharge valve B. A double-evaporator system B. Slugging due to flooding back of refrigerant
D. On the discharge pipe from the condenser C. A direct system C. Too much oil in crankcase
D. A low-pressure system D. Any of the above
Answer: C
Answer: A Answer: D
54. If the thermal expansion valve becomes Answer: C 65. What do you call the liquid reaching the
inoperative, the iceboxes will have to be controlled compressor through the suction?
by the __________. 60. What do you call the device that is used as a low- A. Superheating
A. King valve pressure control and high-pressure cutout on a B. Overflowing
B. Manual expansion valve compressor? C. Flooding back
C. Manual solenoid valve A. Pressure controller D. Recycling
D. Solenoid valve B. Controller switch
C. Cutout Answer: C
Answer: B D. Cutout switch
66. The suction pressure switch is operated by which
55. Sweating of the crankcase is caused by which of Answer: A of the following?
the following? A. Electric current
A. Too much oil in the system 61. If the solenoid valve closed by accident, the B. Pressure on a bellow
B. Insufficient superheat compressor would be stopped by which of the C. A relay cutout
C. Too much superheat following? D. Thermocouple
D. Expansion valve hung up A. Automatic trip
B. Low-pressure cutout switch Answer: B
Answer: A C. Low-water cutout switch
D. High-pressure cutout switch 67. If the compressor short-cycles on the high-
56. Which of the following is the usual cause of pressure cutout, which of the following would you
slugging? Answer: B check?
A. Too much refrigerant in the system A. Check for too much refrigerant in the system
B. Too much oil in the system 62. The purpose of the low-pressure cutout switch is B. If plenty of cooling water is running through
C. Expansion valve not operating properly to: but it is not picking up heat, the condenser
D. Too much cooling water to condenser A. Cut out the compressor at a set pressure tubes need cleaning
B. Maintain a preset suction pressure to the C. Be sure system is getting cooling water
Answer: C compressor D. All of the above
C. Maintain liquid refrigerant at the suction of the
57. What operates low-pressure cutout switch compressor Answer: D
A. Bellows D. Cut compressor in and out at a preset
B. Spring tension pressure 68. A Freon unit will tend to short-cycle when
C. A magnet operating under:
D. Water pressure Answer: D A. Normal conditions
B. Heavy loads
Answer: A 63. If the cooling water to the condenser suddenly C. Light loads
fails? D. All of the above
58. Which of the following must be checked up if an A. An alarm will ring to notify the engineer
automatic Freon system will not start up? B. The compressor will shut down Answer: C
A. High-pressure cutout C. The expansion valve will close
B. Reset mechanism D. The solenoid valve will close 69. Air is remove from the system by
C. Low-pressure cutout A. Opening the purge valve
D. All of the above Answer: B B. Increasing the amount of cooling water
C. Running the refrigerant through an aerator
Answer: D 64. The most likely cause of high superheat would be D. Running the refrigerant through a deaerator
A. Too much refrigerant
59. Which of the following is the probable cause of hot B. Expansion valve open too wide Answer: A
suction line? C. Expansion valve closed too much
A. Insufficient lubrication D. Back-pressure valve set too high 70. Short-cycling means that the machine
B. Too much refrigerant A. Runs to slow
C. Insufficient refrigerant Answer: C B. Stop and starts frequently
D. Expansion valve closed too much C. Runs too fast
D. Grounds out frequently D. Any of the above A. Insufficient cooling water
B. Too much cooling water
Answer: B Answer: D C. Insufficient refrigerant gas
D. B and C
71. The suction pressure in a Freon system should be 76. If an electrically operated compressor failed to
A. The pressure which corresponds with a start, the cause might be: Answer: D
temperature about 20 F ̊ above the A. A blown fuse
temperature of the icebox B. Burned out holding coils in solenoid valve 82. Too high suction pressure could be cause by
B. The pressure which corresponds with a C. An open switch A. Leaky suction valves
temperature about 20 F ̊ below the D. Any of the above B. Expansion valve bulb not working properly
temperature of the icebox C. Expansion valve open too wide
C. The pressure which corresponds with a Answer: D D. Any of the above
temperature equal to the temperature of the
icebox 77. The high-pressure side of the system is Answer: D
D. None of the above sometimes referred to as the
A. Hot side 83. Which of the following would cause high head
Answer: B B. Suction side pressure?
C. Cold side A. Air in the condenser
72. If any of the electrically controlled devices in a D. Cooling side B. Insufficient cooling water
Freon system malfunction, which of the following C. Dirty condenser
valve will also automatically shut off? Answer: A D. Any of the above
A. King valve
B. Condenser cooling-water inlet valve 78. If the compressor were to run continuously without Answer: D
C. Expansion valve lowering the temperature, the trouble would
D. Solenoid valve probably be: 84. An excessively high head pressure could be
A. Leaky discharge valves caused by
Answer: D B. Insufficient refrigerant in the system A. Insufficient cooling water to the condenser
C. Leaks in the system B. Insufficient cooling water to the evaporator
73. A leaky discharge valve can usually be detected D. Any of the above coils
by C. Solenoid valve shutoff
A. A drop in icebox temperature Answer: D D. Too much cooling water to the condenser
B. A discharge pressure lower than normal
C. A fluctuating high-pressure gauge 79. Which of the following would cause a high head Answer: A
D. Any of the above pressure?
A. Suction valve not open enough 85. Which of the following would cause a high suction
Answer: D B. Too much cooling water pressure?
C. Insufficient cooling water A. Expansion valve open too wide
74. The dehydrating agent in a Freon system is D. Icebox door left open B. Dirty dehydrator
usually C. King valve not open wide enough
A. Sodium chloride Answer: C D. Expansion valve not open wide enough
B. Calcium chloride
C. Activated alumina 80. If frost forms on the cylinders, the cause would be Answer: A
D. Slaked lime A. Charging valve left open
B. Expansion valve not open wide enough 86. If a compressor runs continuously, the cause
Answer: C C. Expansion valve open too wide might be a
D. Dehydrator not working properly A. Clogged scale trap
75. If the compressor short-cycles on the low-pressure B. Defective thermal bulb
cutout, the trouble might be: Answer: C C. Stuck high-pressure switch
A. Too much frost on the evaporator coils D. Stuck low-pressure switch
B. Dirty traps and strainers 81. Which of the following would cause low head
C. Lack of refrigerant pressure? Answer: D
87. Low suction pressure is caused by A. While starting up B. After a long period of operation
A. Expansion valve causing flooding back B. After the system has been shut down for a C. After an extend lay-up period
B. Solenoid valve not functioning properly few hours D. While the compressor is in operation
C. Leaky compressor suction valves C. While system is operating
D. Air in the system D. Once a week Answer: B

Answer: B Answer: B 98. The agent used in an indirect reefer system is


A. Calcium chloride
88. How is a Freon system purged? 93. If the compressor had been running satisfactorily B. Potassium chloride
A. With a reefer pump for a long period of time but suddenly the C. Sodium chloride
B. The same as an ammonia system compartment temperature started to rise, the D. A or C
C. Back to the extra supply tank trouble might be
D. The same as a CO2 system A. A refrigerant leak has developed Answer: D
B. The expansion valve may contain frozen
Answer: D water 99. Air can be prevented from getting into the system
C. The solenoid valve has jammed shut by
89. If the compressor discharge temperature is higher D. Any of the above A. Running the refrigerant through an aerator
than the receiver temperature: B. Keeping the dehydrator clean at all times
A. Add more refrigerant to the system Answer: D C. Keeping all glands and stuffing boxes on the
B. Decrease the amount of cooling water to the high-pressure side tight
condenser 94. If the compressor had been running satisfactorily D. Keeping all glands and stuffing boxes on the
C. Increase the amount of cooling water to the for a long period of time but the oil level was rising low-pressure side tight
condenser slowly, one should:
D. Remove some of the refrigerant from the A. Check the dehydrator cartridge Answer: D
system B. Check to see if there is too much refrigerant
in the system 100. Which of the following would not cause high
Answer: B C. Shut down the compressor and check the oil suction pressure?
level with the machine stopped A. Suction valve not adjusted properly
90. How is a CO2 system purged? D. Drain out sufficient oil to bring it down to the B. Expansion valve stuck open
A. When CO2 comes out of the purge valve, frost proper running level C. Leaky suction valves
will form on a piece of metal held near the D. Insufficient refrigeration
outlet Answer: C
B. Through the king valve Answer: D
C. Pumped out with a suction pump 95. The purpose of the oil trap is :
D. The CO2 will come out of the purge valve in A. To add oil to the compressor 101. Water in the refrigerant is liable to
liquid form B. To remove oil from the refrigerating gas A. Freeze on the expansion valve seat and cut
C. To remove oil from the charging tank the flow of refrigerant
Answer: A D. None of the above B. Clog the oil trap
C. Freeze in the king (liquid) valve
91. If the head pressure is too high: Answer: B D. Emulsify the oil in the compressor
A. The relief valve should open and let excess
refrigerant pass to receiver 96. Too much oil in the compressor would : Answer: A
B. Close in on the suction valve A. Absorb to much refrigerant from the system
C. The relief should open before the high- B. Deposit oil on the condenser tubes 102. The function of the expansion valve is to:
pressure cutout C. Damage the expansion valve A. Regulate the amount of liquid refrigerant to
D. The high-pressure cutout switch should D. Cause leakage through the shaft seals the expansion coils
operate before the relief valve opens B. Change the gas refrigerant to a liquid
Answer: A C. Shut off the flow of refrigerant to the
Answer: D condenser
97. The oil level in the compressor should be check D. Change the high-pressure liquid to a low-
92. The system should be purged A. Just before starting the compressor pressure liquid
Answer: A C. Amount of refrigerant going to the 114. The expansion valve on a Freon system controls
compressor the
103. When heavy electrical currents are involved, the D. Pressure of the refrigerant going to the A. Superheat of the gas leaving the compressor
thermostat will be operated by a: evaporator coils B. Back pressure in the evaporator
A. Pressure pipe C. Temperature of the icebox
B. Fusetron Answer: B D. Superheat of the gas leaving the evaporator
C. Relay
D. Small circuit breaker 109. The purpose of the evaporator is to Answer: D
A. Absorb latent heat of vaporization
Answer: C B. Absorb latent heat fusion 115. The purpose of the expansion valve by-pass is to:
C. Transfer latent heat of vaporization A. Increase the efficiency of the plant
104. Before securing a compressor to do maintenance D. Transfer latent heat of fusion B. Increase the capacity of the evaporator
work on it, be sure to: C. Control the refrigerant to the evaporator in
A. Have gas mask handy Answer: A case the automatic valves fail
B. Make arrangements to have perishables D. Bypass the compressor
taken care of 110. The purpose of the dehydrator is to
C. Notify the engineer A. Add more refrigerant to the system Answer: C
D. A and B B. Remove oil from the refrigerant
C. Remove moisture from the crankcase oil 116. The thermal expansion valve
Answer: D D. Remove moisture from the refrigerant A. Controls the amount of gas coming from the
dehydrator
105. When securing a Freon system for repairs Answer: D B. Controls the quantity of liquid refrigerant
A. Pump down to 1 or 2 pounds pressure going to the evaporator coils
B. Pump down to a slight vacuum 111. The principle of mechanical refrigeration is C. Controls the amount of gas going to the
C. Pump down to 10 to 15 pounds pressure A. The conversion of a liquid to gas receiver
D. Remove all refrigerant from the system B. The absorption of temperature under heat, D. Removes trapped oil from the refrigerant
pressure, compression and expansion
Answer: A C. The compression of a liquid under Answer: B
temperature and expansion
106. When charging Freon system, all the valves D. The absorption of heat under temperature, 117. The function of the compressor is to
should be in their normal position except the: compression, pressure and expansion A. Pull the refrigerant gas through the system
A. Solenoid valve B. Increase the pressure of the refrigerant
B. Purge valve Answer: D C. Discharge the refrigerant to the condenser
C. King (liquid) valve D. All of the above
D. Expansion valve 112. A thermostat is a
A. Temperature-operated switch Answer: D
Answer: C B. Pressure-operated switch
C. Superheat-operated switch 118. The solenoid valve is controlled by
107. The purpose of the receiver is to: D. Back-pressure-operated switch A. The amount of liquid in the system
A. Cool the refrigerant gas B. The amount of gas in the system
B. Separate the oil from the refrigerant Answer: A C. The temperature in the condenser
C. Store the refrigerant D. The temperature in the icebox
D. Condense the refrigerant 113. The thermal expansion valve responds to the
A. Amount of superheat in the vapor Answer: D
Answer: C B. Amount of superheat in the liquid
C. Temperature in the evaporator coils 119. Oil is added to a Freon compressor by
108. The solenoid valve controls the D. Pressure in the evaporator coils A. Shutting down the machine and pouring in
A. Amount of refrigerant entering the evaporator through the crankcase inspection plate
coils Answer: A opening
B. Amount of refrigerant going to the expansion B. Pumping in with an electric-driven pump
valve C. Pumping in with a hand pump
D. Pouring through oil hole in base D. 280,000 Btu per 24 hrs. A. Pressure
B. Weight
Answer: C Answer: A C. Volume
D. Psi
120. When adding oil to a Freon system, one must be 126. The boiling point of Freon-12 at atmospheric
sure that pressure is: Answer: B
A. The condenser is shut down A. + 22 F ̊
B. All air is removed from the pump and fitting B. - 22 F ̊ 132. The latent heat of fusion of ice is:
C. There is not too high a suction pressure C. + 22 C ̊ A. 500 Btu
D. The discharge pressure is not too high D. - 22 C ̊ B. 188 Btu
C. 144 Btu
Answer: B Answer: B D. 970 Btu

121. To help a person who had been overexposed to 127. The boiling point of 𝐶𝑂2 at atmospheric pressure Answer: C
ammonia gas, one would: is
A. Apply cold compresses A. – 110 F ̊ 133. Latent heat
B. Apply artificial respiration B. + 110 F ̊ A. Can be measured with a pyrometer
C. Douse with cold water C. + 110 C ̊ B. Cannot be measured with a thermometer
D. Wrap in warm blankets D. – 110 C ̊ C. Changes as the refrigerant cools
D. Can be measured with a thermometer
Answer: B Answer: A
Answer: B
122. Ammonia will corrode 128. The boiling point of 𝑁𝐻3 at atmospheric pressure
A. Brass is 134. Absolute zero is:
B. Copper A. + 28 C ̊ A. 144 ̊ below zero on the Fahrenheit scale
C. Bronze B. + 28 F ̊ B. The same as zero on the Fahrenheit scale
D. All of the above C. - 28 C ̊ C. 970 ̊ below zero on the Fahrenheit scale
D. - 28 F ̊ D. 460 ̊ below zero on the Fahrenheit scale
Answer: D
Answer: D Answer: D
123. A refrigerating unit of one (1) ton capacity can
remove: 129. Which of the following would you apply if a person 135. A ton of refrigeration is equal to the cooling effect
A. 100 Btu’s per min. got Freon in his eyes? of
B. 288 Btu’s per min. A. Clean water A. 2,000 lbs. of ice melting in 24 hrs.
C. 200 Btu’s per min. B. Soapy water B. 2,000 lbs. of water being converted to ice
D. 500 Btu’s per min. C. Sodium bicarbonate C. 2,000 lbs. of ice melting in 12 hrs.
D. Sterile mineral oil D. 2,240 lbs. of ice melting in 24 hrs.
Answer: C
Answer: D Answer: A
124. The refrigerant with the lowest boiling point is
A. 𝑁𝐻3 130. A double-seated valve allows the valve to be 136. Which of the following best described a Freon?
B. 𝐹12 A. Packed only in the closed position A. Odorless
C. 𝐶𝑂2 B. Packed in the wide open or closed position B. Non-poisonous
D. 𝐹22 C. Operated as a suction or discharge valve C. Colorless
D. Removed for replacement without shutting D. All of the above
Answer: C down
Answer: D
125. A ton of refrigeration is equal to the removal of Answer: B
A. 288,000 Btu per 24 hrs. 137. What is another name of discharge pressure?
B. 28,800 Btu per 24 hrs. 131. The amount of 𝐶𝑂2 or Freon in a cylinder is A. Absolute pressure
C. 28,000 Btu per 24 hrs. measured by B. Head pressure
C. Suction pressure C. 4 tons 148. Which of the following is the most common
D. Condenser pressure D. 3 tons method of heat flow in refrigeration?
A. Conduction
Answer: B Answer: C B. Expulsion
C. Radiation
138. Which of the following is the oil used in a 143. What is the combination of a wet and dry bulb D. Convection
refrigeration system? thermometer is called a
A. Vegetable oil A. Hygrometer Answer: D
B. Straight mineral oil B. Psychrometer
C. Lube oil SAE 20 C. Hydrometer 149. From which of the following processes where the
D. Lube oil SAE 10 D. A or B largest quantities of heat are available?
A. Vaporization
Answer: B Answer: D B. Melting
C. Fusion
139. One disadvantage of a CO2 system is the fact that 144. A pressure controller is usually operated by the D. Cooling
A. It is difficult to condense the refrigerant if the movement of a
circulating water temperature is too high. A. Bellows Answer: A
B. It takes more refrigerant to keep the iceboxes B. Siphon
cold C. Diaphragm 150. What is the most common Freon gas used in
C. Due to high pressure it is difficult to keep oil D. A or C centrifugal compressors?
from mixing with the refrigerant A. F – 12
D. It is difficult to condense the refrigerant if the Answer: D B. F – 11
circulating water temperature is too low C. F – 22
145. The critical temperature of a refrigerant is: D. F – 21
Answer: A A. The temperature at which it will freeze
B. The temperature above which it cannot be Answer: B
140. The disadvantage of CO2 system over an ammonia liquefied
system is the fact that __________ C. The temperature below which it cannot be 151. What is the most common Freon gas used in
A. The pipes and fittings of a CO2 system must liquefied reciprocating compressors?
be of high pressure type D. None of the above A. F – 12
B. The CO2 system operates at a much higher B. F – 11
pressure Answer: B C. F – 22
C. The CO2 system requires a larger prime D. F – 21
mover 146. If the critical temperature of a refrigerant is too
D. All of the above close to the desired condensing temperature, the Answer: A
equipment must necessarily be of:
Answer: D A. Extra heavy construction 152. What is the other name of hygrometer?
B. The direct expansion type A. Hydrometer
141. A good refrigerant should be: C. Light construction B. Manometer
A. Non-inflammable D. The indirect expansion type C. Psychometer
B. Non-poisonous D. A or C
C. Non-explosive Answer: A
D. All of the above Answer: C
147. A good refrigerant should have a
Answer: D A. High sensible heat 153. The temperature in the meat and fish box should
B. Low sensible heat be approximately
142. What tonnage of refrigerating machine is required C. High latent heat A. 10 ̊ to 20 F ̊
if the refrigerating system extracted 48,000Btu per D. Latent heat B. - 10 ̊ to 0 F ̊
hour? C. 0 ̊ to -10 F̊
A. 2 tons Answer: C D. - 10 ̊ to 10 F ̊
B. 5 tons
Answer: A C. System overcharge with refrigerant B. Too much cooling water to condenser
D. Insufficient cooling water C. Cooling water temperature too high
154. The temperature in the vegetable box should be D. A or C
approximately: Answer: A
A. 35 ̊ to 45 F̊ Answer: D
B. 10 ̊ to 20 F ̊ 160. Excessive head pressure is caused by:
C. 40 ̊ to 50 F ̊ A. Air or noncondensable gas in the system 166. Everything is in its normal operating position when
D. 15 ̊ to 30 F ̊ B. Dirty condenser tubes charging a system except the ________.
C. Insufficient cooling water to condenser A. Compressor discharge valve
Answer: A D. Any of the above B. Purge valve
C. Solenoid valve
155. The temperature in the dairy box should be Answer: D D. King (receiver discharge) valve
approximately
A. 20 ̊ to 30 ̊ F 161. The capacity of a centrifugal type compressor Answer: D
B. 0 ̊ to 10 ̊ F controlled by which of the following?
C. 10 ̊ to 20 ̊ F A. Regulating the discharge pressure 167. How often the condenser must be cleaned?
D. 30 ̊ to 40 ̊ F B. Regulating the speed A. Once a year
C. Regulating the suction pressure B. Every month
Answer: D D. B or C C. When pressure goes above normal
D. Every 6 months
156. Which of the following can be the cause of low Answer: D
head pressure? Answer: D
A. Too much or too cold condensing water 162. Where is solenoid coil installed?
B. Leaky discharge valves A. Vertically above the valve 168. Which do you think is the cause of a hot suction
C. Insufficient charge of refrigerant B. Horizontally above the valve line of a refrigerating compressor?
D. Any of the above C. Vertically below the valve A. Lack of refrigerant
D. Horizontally below the valve B. Insufficient condensing cooling water
Answer: D C. Excess refrigeration
Answer: A D. B or C
157. If thermostatic expansion valve did not appear to
be functioning properly, the cause could be: 163. What is the condition of the compressor when Answer: A
A. Foreign matter in the valve purging a refrigeration system?
B. Ruptured control bulb tubing A. The compressor is running 169. Which of the following vital components of the
C. Moisture in the system B. Compressor is running but the condenser is refrigeration system where both temperature and
D. Any of the above secured pressure are increased?
C. Shut down A. Compressor
Answer: D D. Running with the bypass open B. Condenser
C. Evaporator
158. Which of the following is used in measuring the Answer: C D. A and C
density of a brine solution?
A. Litmus paper 164. What must be done first when opening a single Answer: A
B. A chemical test packed stop valve?
C. A hydrometer A. Loosen the packing before opening 170. The refrigerant temperature is at its maximum just
D. A or B B. Tighten packing before opening before it enters the ____________.
C. Check to see that the seal is not scored A. Expansion valve
Answer: C D. None of the above B. Compressor
C. Condenser
159. Which of the following would not be cause for a Answer: A D. Evaporator
refrigerating system to short cycle on HP cutout?
A. Discharge valve leaking 165. Excessive head pressure is caused by: Answer: C
B. Pressure cutout set incorrectly A. Flooded condenser tubes
171. The greatest decrease in refrigerant temperature Answer: A A. An electrical release
occurs in the __________. B. Temperature and spring control
A. Evaporator 177. What is the cause of liquid “slugging”? C. Pressure and bellows control
B. Compressor A. Liquid in the compressor clearance space D. Bellows control
C. Condenser B. Excessive liquid refrigerant in the receiver
D. A or C C. The presence of liquid in the condenser Answer: A
causing excessive noise
Answer: A D. The pounding of liquid refrigerant in the 183. The thickness of the head gasket is important
suction line at a point of restriction because it may cause __________.
172. What do you call the storage tank for liquid A. Re-expansion
refrigerant? Answer: A B. Decreased efficiency due to increased
A. Receiver tank clearance
B. Charging tank 178. All refrigerator compressor valves are opened and C. The piston to strike the head
C. Purging close by __________. D. All of the above
D. Any of the above A. A cam shaft
B. Springs Answer: D
Answer: A C. Manual
D. B or C 184. If the expansion valve capillary tube is pinched,
173. A precooler is sometimes installed between the which of the following must be replaced?
___________. Answer: B A. Tube
A. Compressor and condenser B. Diaphragm
B. Condenser and expansion valve 179. Which of the following is also known as the C. Bulb
C. Expansion valve and evaporator “hidden heat” in refrigeration work? D. All of the above (called the power element
D. Evaporator and compressor A. Sensible heat unit)
B. Heat intensity
Answer: B C. Latent heat Answer: D
D. A or C
174. The solenoid valve controls which of the following? 185. When removing reusable refrigerant from a
A. Pressure in the evaporator coils Answer: C system, the line to the storage drum must
B. Amount of refrigerant entering the evaporator A. Be made of copper
C. Flow of refrigerant to the expansion valve 180. A bull’s eye in a full liquid line will appear B. Have no bends in it
D. Amount of circulating water to the condenser ________. C. Contain a strainer-dryer
A. Cloudy D. Be above the level of compressor
Answer: C B. Clear
C. Latent Answer: C
175. Closing the solenoid valve will stop the D. A or C
compressor through the _________. 186. Which of the following must be considered when
A. Low water-pressure cutout switch Answer: B adding or removing oil from a refrigerator unit?
B. Bypass relief valve A. Use new oil
C. Low-pressure cutout switch 181. A compressor capacity reduction device reduces B. Do not overcharge
D. High-pressure cutout switch compressor capacity ___________. C. Watch crankcase pressure
A. By reducing the compressor speed D. All of the above
Answer: C B. By bypassing hot gas
C. As the refrigerant load dictates Answer: D
176. What are the four basic methods of determining D. By reducing compressor horsepower
whether the proper amount of refrigerant is being proportionately 187. Where is the excess refrigerant removed?
added to the system? A. Suction side of the system
A. Bull’s-eye, weight, pressure and frost line Answer: C B. Discharge side of the system
B. Temperature, weight, pressure and frost line C. Bypass
C. Bull’s-eye, weight, pressure and dip stick 182. If a Freon -12 compressor trip out on “cut-out”, the D. Charging side of the system
D. Bull’s-eye, weight, litmus test and frost line solenoid valve closes by which of the following?
Answer: D D. Remove the heat required to melt one ton of 199. Which of the following is important for evaporator
ice in 24 hours coils?
188. A double trunk piston is used to _________. A. It must be placed in the top of the
A. Absorb side thrust Answer: D compartment
B. Seal off gas from crankcase B. It must be secured to the sides
C. Prevent oil from missing with gas 194. The temperature bulb of a solenoid valve is C. It should have air completely surrounding
D. All of the above attached to the ____________. them
A. Icebox coil D. It must be placed in front of circulating fans
Answer: D B. Wall of the icebox
C. Evaporator coil inlet Answer: C
189. How do you call the process of changing a solid to D. Evaporator coil outlet
a liquid? 200. If there were a 15 F̊ to 20 F
̊ temperature
A. Evaporation Answer: B differential between the temperature
B. Vaporization corresponding to the pressure at the compressor
C. Fusion 195. What usually happens if the specific gravity of the discharge and the temperature at the condenser
D. Condensation brine is too low? outlet, it would probably indicate the need for
A. The brine will freeze _____________.
Answer: C B. Solids will deposit A. More refrigerant
C. It will be more heat-absorbing B. Purging the system
190. If there is too much lube oil in the system, what D. All of the above C. More circulating water
must be done? D. Less circulating water
A. Remove same at once Answer: A
B. Wait until next overhaul to remove Answer: B
C. Wait until next recharging to remove 196. The cooling water regulator is automatically
D. Any of the above actuated by which of the following? 201. Which of the following is the most appropriate
A. The discharge pressure of the refrigerant definition of Latent heat?
Answer: A B. The temperature of the refrigerant A. Heat removed to melt ice
C. An electric relay B. Heat removed to change temperature of a
191. The temperature bulb of the thermo-expansion D. A thermo relay substance
valve is attached to which of the following? C. Heat added to change temperature of a
A. Icebox coil Answer: A substance
B. Evaporator coil outlet D. Heat added to change the state of substance
C. Evaporator coil inlet 197. What will happen to the capacity if the superheat
D. Wall of the icebox is increased on the suction side? Answer: D
A. Increases
Answer: B B. Decreases 202. If the compressor discharge becomes frosted, the
C. Remains the same probable cause would be __________.
192. What is the use of the suction pressure regulating D. Will double A. Refrigerant flooding back
valve? B. Expansion valve improperly set
A. Cuts in the compressor Answer: B C. Too much cooling water
B. Maintains the back pressure in the D. Insufficient cooling water
evaporator coils 198. Which do you think is very important in adjusting
C. Cuts out the compressor compressor “V” belts? Answer: A
D. Controls the expansion valve A. Allow about ½” slack
B. Make it as tight as possible 203. The low-pressure control switch:
Answer: B C. Make belt just tight enough to turn pulley A. Is a safety device
D. Keep belts parallel B. Actuates the cooling water
193. How is a one-ton plant described? C. Cuts out the compressor to maintain proper
A. Remove one ton of heat from the reefer box Answer: A flow
B. Melt one ton of ice in 24 hours D. Regulates the King valve
C. Make one ton of ice in 24 hours
Answer: C C. ̊
Boiling point over 200 F 215. Calcium chloride is sometimes used in
D. Very volatile refrigeration system as a:
204. Which of the following is an indications of faulty A. Refrigerant
Freon compressor valves? Answer: C B. Lubricant
A. Compressor runs continuously C. Primary coolant
B. Low head pressure – high suction pressure 210. Which of the following type valves are not found D. Secondary coolant
C. Gradual or sudden decrease in capacity on a Freon -12 system?
D. Any of the above A. Bellows Answer: D
B. Duplex
Answer: D C. Diaphragm 216. When the outlet temperature of the evaporator
D. Single packed exceeds the inlet temperature the condition is
205. What is the probable cause if a compressor runs called ___________.
continuously? Answer: B A. Boiling
A. A clogged condenser B. Superheating
B. Insufficient refrigerant 211. How much will be removed by one-ton C. Melting
C. Faulty cooling water valve refrigeration unit? D. Freezing
D. Any of the above A. 200 Btu per min.
B. 144 Btu per min. Answer: B
Answer: D C. 400 Btu per min.
D. 2000 Btu per min. 217. Which of the following refrigerants would give the
206. To test a thermostatic valve, immerse the bulb in: most trouble when operating with warm circulating
A. Crushed ice Answer: A water?
B. Hot water A. Freon -12
C. Oil 212. Which of the following is the primary purpose of B. CO2
D. None of the above the evaporator? C. Ammonia
A. Transmit latent heat of fusion D. Ethyl chloride
Answer: A B. Transmit latent heat of evaporation
C. Absorb latent heat of fusion Answer: B
207. If a compartment requires the removal of 36,000 D. Absorb latent heat of evaporation
Btu per hour, how much is the necessary 218. As Freon leaves the expansion valve:
compressor capacity? Answer: D A. Pressure increases – volume increases
A. 6 tons B. Pressure decreases – volume increases
B. 3 tons 213. An automatically controlled Freon -12 compressor C. Pressure decreases – volume decreases
C. 2 tons will start when the __________. D. Pressure increases – volume decreases
D. 1 ton A. Expansion valve opens
B. Solenoid valve opens Answer: B
Answer: B C. Expansion valve closes
D. Solenoid valve closes 219. The suction pressure switch is actuated by:
208. Which of the following must be done to eliminate A. Pressure on a bellows
frost on the discharge pipe of the compressor? Answer: B B. Temperature on a bellows
A. Open the expansion valve C. A thermo-pressure regulator
B. Regulate water to the condenser 214. A single trunk piston-type compressor is D. A thermostatic temperature device
C. Crack bypass valve undesirable for a Freon unit because the:
D. None of the above A. Lubricant mixes with the refrigerant Answer: A
B. Refrigerant reduces the crankcase pressure
Answer: D C. Lubricant temperature becomes excessive 220. A thermometer senses which of the following?
D. Refrigerant becomes superheated A. Latent heat
209. The following are standard characteristics of Freon B. Sensible heat
-11 except: Answer: A C. Heat of fusion
A. Non-toxic D. Specific heat
B. Separates from water
Answer: B D. B and C A. Using skirt type pistons
B. Maintaining a vacuum in the crankcase
221. Five pounds of water heated to raise the Answer: D C. Using shaft seals
temperature 2 F̊ requires how many Btu? D. Using lantern rings
A. 20 Btu 227. Absolute zero on the Fahrenheit scale is equal to:
B. 5 Btu A. -273 ̊ Answer: C
C. 2 Btu B. -460 ̊
D. 10 Btu C. 0 ̊ 233. The heat used to change a liquid to a gas or vapor
D. -100 ̊ is called latent heat of
Answer: D A. Absorption
Answer: B B. Vaporization
222. Superheat is heat added _________. C. Fusion
A. In changing liquid to vapor 228. A quick method of detecting a leaky condenser on D. Liquid
B. After all liquid has been changed to vapor a Freon system would be to __________.
C. To increase pressure A. Compare temperatures and pressures Answer: B
D. To increase temperature B. Test circulating water with proper chemicals
C. Open vent on head and test with halide torch 234. What is the compression ratio of a refrigerator
Answer: B D. Use soapsuds on condenser outlet fittings compressor?
A. The ratio of the absolute suction pressure to
223. The expansion valve does not seem to be Answer: C the absolute discharge pressure
operating properly. There is high superheat. Test B. The ratio of the absolute discharge pressure
by listening to the sound of Freon flooding through 229. Which of the following is not a characteristic of to the absolute suction pressure
the tubes and _____________. Freon -12? C. The ratio of the excessive liquid refrigerant in
A. Warm bulb with hand A. Corrosive the receiver to that in the system
B. Place bulb in cold water B. Non-inflammable D. None of the above
C. Place bulb in hot water C. Odorless
D. Any of the above D. Boiling point -21 F ̊ Answer: B

Answer: A Answer: A 235. The thermostatic expansion valve is designed to


maintain a constant _________.
224. What do you call a material in a dryer? 230. In the discharge line between the compressor and A. Superheat
A. Drain the condenser one would find: B. Flow
B. Dryer A. High pressure, high temperature gas C. Pressure
C. Desiccant B. High pressure, low temperature liquid D. Temperature
D. A or C C. High temperature, high pressure liquid
D. High pressure, low temperature gas Answer: A
Answer: C
Answer: A 236. A refrigerant gives up heat when it
225. What is the main function of a receiver? A. Vaporizes
A. Separate the oil from the refrigerant 231. For a low speed compressor the belt drive is B. Evaporates
B. Cool the hot gases preferred to the direct drive because it: C. Condenses
C. Store the refrigerant A. Eliminates the use of mineral lubricating oils D. Boils
D. Condense the refrigerant B. Permits the use of a smaller high-speed
motor Answer: C
Answer: C C. Absorbs torsional vibration
D. Reduces the number of automatic controls 237. The cooling component of a refrigeration cycle is
226. During the re-expansion portion of the refrigeration called _________.
compressor cycle Answer: B A. A receiver
A. The suction valve is open B. An evaporator
B. The suction valve is closed 232. Refrigerant leakage from the compressor C. A condenser
C. The discharge valve is closed crankcase is prevented by D. A desiccant
Answer: B D. Any of the above C. Defrost coils
D. B and C
238. When ordering an expansion valve which of the Answer: D
following information is necessary? Answer: D
A. Size and pressure 244. A “direct” refrigeration system is one in which:
B. Size, tonnage, temperature and pressure A. Ice is used for cooling 250. When the compressor is operated in short spurts
C. Pressure and temperature B. A secondary coolant is pumped through the after a lay-up or overhaul the purpose is to:
D. Size and tonnage evaporator coils A. Allow refrigerant time to circulate
C. The refrigerant passes through coils in the B. Determine actual compressor oil level
Answer: B evaporator C. Effect gradual cooling
D. Any of the above D. None of the above
239. A hot crankcase and cylinder head accompanied
by a low suction pressure would be caused by Answer: C Answer: D
A. Excess refrigerant
B. Insufficient refrigerant 245. A “CARRENE” is a type of 251. Which of the following types of valve are not used
C. Air in the system A. Refrigerant oil for suction or discharge?
D. Stuck discharge valve B. Refrigerant A. Gate
C. Scale cleaner B. Diaphragm
Answer: B D. None of the above C. Metallic
D. Ring
240. Which of the following characteristics that is not Answer: B
desirable in a refrigerant? Answer: A
A. Low latent heat fusion 246. What is the first step that must be done when
B. High latent heat of vaporization securing a system? 252. What usually happened if brine has a high specific
C. Low latent heat of vaporization A. Open bypass valve gravity?
D. A and C B. Close receiver (King) valve A. It will freeze
C. Open solenoid valve B. It will crystalize
Answer: C D. Cut out compressor on high pressure C. Nothing will happen
D. It will solidify
241. The principle of the centrifugal system is based on Answer: B
which of the following? Answer: B
A. Kinetic energy 247. When securing a Freon-12 system for repairs
B. Potential energy A. Open the line at 1 to 2 pounds pressure 253. When does the refrigerant gives-up heat?
C. Lenz’s law B. Open the line at 5 to 10 pounds pressure A. When it evaporates
D. Boyle’s law C. Pump down to a slight vacuum B. When it condenses
D. Pump down to 10 ̊ vacuum C. When it vaporizes
Answer: A D. When it boils
Answer: A
242. The temperature in the vegetable box should be Answer: B
approximately 248. The most common reason for the small usage of
A. -5 F ̊ CO2 system is: 254. The thermostatic expansion valve is designed to
B. 20 F ̊ A. It is too dangerous maintain constant_____________
C. 38 F ̊ B. Its upkeep is too high A. Pressure
D. 10 F ̊ C. The machinery is too heavy B. Flow
D. It is too costly C. Temperature
Answer: C D. Superheat
Answer: C
243. The compressor will run continuously if there is Answer: D
A. Insufficient refrigerant 249. To increase heat transfer in the evaporator
B. Air in the system A. Increase suction pressure 255. A reactor plate is essential to the operation of a
C. Too heavy a load B. Increase air circulation ___________.
A. Thermostatic valve 261. Which of the following would not cause high head C. It is easier to check leaks
B. Halide torch pressure? D. Iron corrodes
C. Solenoid bulb A. Air in system
D. None of the above B. Overcharge of refrigerant Answer: A
C. Circulating water too warm
Answer: B D. Liquid freezing back 267. The cooling water regulator is actuated by:
A. An electric relay
256. The greatest decrease in the temperature of the Answer: D B. Pressure of the refrigerant
refrigerant is at the _________ C. Temperature of the refrigerant
A. Condenser 262. A “cold diffuser” is a: D. None of the above
B. Expansion valve A. Type of condenser
C. Evaporator B. Special valve used as a bypass Answer: B
D. Receiver C. Type of evaporator
D. None of the above ̊ temperature drop between the
268. If there is a 20 F
Answer: C temperature corresponding to the pressure at the
Answer: C compressor discharge and the condenser outlet,
257. The temperature of the refrigerant is highest just one should:
before it enters the: 263. Which do you think is the effect of “subcooling”? A. Decrease the amount of circulating water
A. Receiver A. It causes flooding back to the compressor B. Increase the amount of circulating water
B. Evaporator B. It reduces the horsepower per ton of C. Purge the system
C. Condenser refrigeration D. None of the above
D. King valve C. It increases the compression ratio
D. It increases the horsepower per ton of Answer: C
Answer: C refrigeration
269. If refrigeration controls were constructed with little
258. A hot suction line could be caused by: Answer: B or no differential, the:
A. Insufficient refrigerant A. Compressor would short cycle
B. Excess refrigerant 264. Subcooling is to: B. Icebox would get too cold
C. Excess cooling water A. Cool the evaporator below its normal C. Icebox would get too warm
D. Insufficient cooling water temperature D. Refrigerant would be subcooled
B. Cool the liquid before it enters the evaporator
Answer: A C. Cool a liquid at constant pressure to a Answer: A
temperature lower than its considering
259. Which of the following devices will prevent the temperature 270. What is the use of back pressure regulating valve?
relief valve from opening in the event of excessive D. Cool the refrigerant gas before it enters the A. Controls the evaporator temperature by
pressure? receiver throttling the flow of liquid refrigerant
A. Water failure switch B. Maintains a fixed pressure in the evaporator
B. High-pressure cutout switch Answer: C coils
C. Low-pressure cutout switch C. Controls the flow of circulating water in the
D. Any of the above 265. When figuring compression ratio, the ratio will condenser tubes
increase with a ______________ D. Controls the temperature in the evaporator
Answer: B A. Lower suction pressure coils
B. Higher suction pressure
260. Which of the following would cause the suction C. Higher discharge pressure Answer: B
switch to remain open? D. A and C
A. Insufficient spring tension 271. If the thermal bulb becomes loose on the
B. Too much spring tension Answer: D evaporator coils, it will cause____________
C. Bellows broken or jammed A. An electrical short
D. A or B 266. Copper tubing is used in Freon system because: B. Flooding back of the refrigerant
A. It has less internal resistance C. Improper operation of expansion valve
Answer: C B. It is cheaper D. Any of the above
Answer: C Answer: B Answer: A

272. External frost on inlet of expansion valve indicates: 278. A Freon-12 gage shows pressure and _______ 284. The process that takes place in the evaporator is
A. Expansion valve plugged or dirty A. Superheat temperature called:
B. Head pressure too high B. Saturation temperature A. Transfer of the latent heat of vaporization
C. Refrigerating compartment too cold C. Back pressure B. Absorption of the latent heat of vaporization
D. Air in system D. Vacuum C. Absorption of the latent heat of fusion
D. Transfer of the latent heat of fusion
Answer: A Answer: B
Answer: B
273. Subcooling of the refrigerant results in: 279. Moisture in a system will cause a:
A. Less circulating water needed A. Faulty expansion valve 285. What is the use of the low-water cutout switch?
B. Effect of refrigerant increased B. High suction pressure A. Recirculates the cooling water when there is
C. Liquid less likely to vaporize C. High suction temperature too much refrigerant in the condenser
D. B and C D. Low discharge temperature B. Stops the compressor when there is no
refrigerant running to the evaporator
Answer: D Answer: A C. Stops the flow of refrigerant when the
condenser temperature is too low
274. Which of the following is the reason when the 280. Which of the following is not essential to a D. Stops the compressor when there is
crankcase is cooler than the suction line? centrifugal type of compressor system? insufficient cooling water
A. Too much refrigerant A. Evaporator
B. Insufficient refrigerant B. Distiller Answer: D
C. Expansion valve open too wide C. Condenser
D. Suction valve leaking D. Expansion valve 286. The high-pressure cutout switch:
A. Stops the flow of refrigerant when the
Answer: A Answer: B condenser temperature is too high
B. Recirculates the refrigerant through the
275. If the outlet of the thermostatic valve is warmer 281. A scale trap in a Freon system will be found on the compressor under emergency conditions
than the inlet, it indicates: : C. Stops flow of water to condenser when there
A. Flooding back A. Receiver is no refrigerant passing through
B. Thermostatic valve working properly B. Discharge side D. Stops compressor when head pressure is too
C. Thermostatic valve not working properly C. Suction side high
D. Solenoid valve not working properly D. Condenser
Answer: D
Answer: C Answer: C
287. Zinc rods are found in this:
276. High temperature of cylinder heads and crankcase 282. Which of the following would cause expansion A. Gas side of the condenser
is caused by: valve failure? B. Salt water side of the condenser
A. Insufficient refrigerant A. Dirt in the valve C. Evaporator area
B. Too much refrigerant B. Moisture in the system D. Compressor crankcase
C. High head pressure C. Bulb in icebox ruptured
D. Noncondensable gases D. Any of the above Answer: B

Answer: A Answer: D 288. Which of the following is not essential to a


compression refrigeration system?
277. Frost on the high- pressure side of a thermostatic 283. Two compressor should not be run in parallel A. A receiver
expansion valve would probably be caused by because: B. A condenser
A. High head pressure A. There is a possibility of losing oil C. An evaporator
B. Dirty expansion valve B. It will give over-capacity D. A dehydrator
C. Refrigerator box too cold C. It is not efficient
D. Loss of circulating water to condenser D. A or B Answer: A
289. What is the effect if the refrigerant is removed from the 295. With one machine serving several compartments, with
system too fast? one compartment reaches the desired temperature in that 301. High superheat of the vapor in the steam would cause
A. It may flood the evaporator compartment is maintained by: A. an increase in capacity
B. It may “freeze-up” the condenser A. the expansion valve B. a more efficient unit
C. It will lower the icebox temperature B. the solenoid valve C. a decrease in capacity
D. None of the above C. the back-pressure valve D. A and B
D. any of the above
Answer: B Answer: C
Answer: B
290. Closing the solenoid valve stops the compressor 302. The solenoid valve is actuated by which of the
through the: 296. The back-pressure regulating valve: following?
A. high-pressure cutout switch A. controls the water flow through the condenser A. a bellows
B. low water-pressure cutout switch B. stops the flow of liquid refrigerant when temperature B. a magnet
C. low-pressure cutout switch drops in evaporator C. a spring
D. high water-pressure cutout switch C. maintains a fixed pressure in the evaporator coils D. any of the above
D. none of the above
Answer: C Answer: B
Answer: C
291. A precooler, if used, is located between the: 303. Which of the following stops the compressor before the
A. expansion valve and evaporator 297. Faulty F-12 compressor valves will be indicated by: relief valve opens?
B. compressor and condenser A. low head pressure-high suction pressure A. low water-pressure cutout
C. evaporator and compressor B. compressor running continuously B. high-pressure cutout
D. condenser and expansion valve C. gradual or sudden decrease in capacity C. low oil-pressure cutout
D. all of the above D. low-pressure cutout
Answer: D Answer: B
Answer: D 304. What is the effect if the thermal bulb is loose on the
292. When the outlet temperature at the evaporator exceeds evaporator coils?
the inlet temperature, the condition is known as: 298. How were tubing joints done? A. improper operation of expansion valve
A. superheating A. brazed B. flooding back of refrigerant
B. desuperheating B. welded C. vibration causing leaks
C. dehydrating C. screwed D. improper operation of solenoid valve
D. saturating D. silver soldered
Answer: A
Answer: A Answer: D
305. If the superheat on the suction side of the compressor
299. When starting a refrigerating unit, be sure the water is increased, what will happen to the tonnage capacity of the
293. The purpose of providing hot-gas defrosting facilities: side of the condenser is _____. unit?
A. defrosting without raising compartment temperature A. secured A. increases
above 32 ℉ B. bypassed B. decreases
B. thawing frozen coils C. vented C. no change
C. defrosting automatically D. B and C D. none of the above
D. B or C
Answer: C Answer: B
Answer: A
294. The suction control switch on the compressor is a 300. The water regulating valve is operated by the 306. What do you usually do to correct slugging and flooding
A. thermal element A. compressor discharged pressure back?
B. thermostat B. compressor discharged temperature A. change the discharged pressure
C. pressure element C. compressor suction pressure B. clean the traps
D. bellows D. none of the above C. removes some refrigerant
D. check the expansion valve
Answer: C Answer: D
Answer: D Answer: B D. diesel oil

307. In a Freon-11 system there is no 313. Which of the following will cause an automatically Answer: B
A. receiver controlled F-12 compressor to start? 319. If one of the ship’s service boxes suddenly stops
B. distiller A. closing the expansion valve cooling, the most likely cause would be the malfunction of
C. condenser B. opening the expansion valve the:
D. evaporator C. closing the solenoid valve A. discharge valve
D. opening the solenoid valve B. king valve
Answer: B C. solenoid valve
Answer: D D. expansion valve
308. The suction pressure control valve is actuated by which
of the following? 314. Where is the solenoid coil installed? Answer: C
A. thermostat A. horizontally over the valve
B. bellows B. vertically over the valve 320. Which of the following items is not important when
C. thermal element C. vertically below the valve using a halide torch?
D. pressure diaphragm D. horizontally below the valve A. move flame slowly around joints
B. hold flame close to the joints
Answer: D Answer: B C. adjust to a clear white flame
309. Which of the following would cause the crankcase and D. adjust to a clear blue flame
head to get hot with low suction pressure? 315. The “refrigeration effect” of a refrigerant is:
A. excess refrigeration A. quantity of heat that 1 pound of refrigerant absorbs while Answer: C.
B. air in system flowing through the evaporator under given condition
C. insufficient refrigeration B. amount of heat it can remove in a given time 321. A system should be purged when the liquid outlet
D. insufficient cooling water C. quantity of heat that 1 pound of refrigerant absorbs temperature from the condenser is more than the
D. A or B temperature corresponding to the discharge pressure:
Answer: C A. 10 ℉ above
Answer: A B. 20 ℉ below
310. Which of the following is the function of a suction C. 20 ℉ above
pressure regulating valve? 316. The “refrigerant effect” of a refrigerant is always: D. 5 ℉ below
A. cuts out the compressor A. more than its latent heat
B. controls the expansion valve B. more than its sensible heat Answer: D
C. cuts in the compressor C. less than its sensible heat
D. maintains the proper back pressure D. less than its latent heat 322. Which of the following would cause the compressor to
run continuously?
Answer: D Answer: D A. low-pressure switch jammed
B. high-pressure switch jammed
311. Which of the following is the cause if the outlet of a 317. What will happen if the expansion valve is opened too C. clogged strainer
thermostat valve is warmer than the inlet inside? wide? D. defective thermal bulb
A. valve is working properly A. liquid will flow back to the compressor
B. valve is not working properly B. it will make no difference, as the automatic expansion Answer: A
C. solenoid valve is not working properly valve is still operating
D. none of the above C. the condenser coils will overheat 323. In a vapor compression cycle the lowest temperature is
D. A or C found in ____.
Answer: B A. receiver
Answer: A B. condenser
312. How is a thermostatic valve tested? C. compressor
A. immersing its bulb in warm water 318. Which of the following should not be used to clean D. evaporator
B. immersing its bulb in ice water scale tarps?
C. holding its bulb in one’s hand A. compressed air Answer: D
D. shorting out the cutout switch B. cotton waste 324. The moisture in a refrigeration system can be removed
C. kerosene with the help of which of the following?
A. driers B. more than 1
B. evaporators C. equal to 1 Answer: B
C. dehumidifies D. depends upon the make of it
D. coolers 336. White ice is
Answer: B A. Fast cooled water
Answer: A B. sub cooled water
331. Horsepower per ton of refrigeration is expressed as: C. due to dissolved air, gases and impurities
325. Which of the following refrigerants has lowest freezing A. 4.75/COP D. formed by blowing air during freezing
point temperature? B. 4.75 x COP
A. Freon 12 C. COP/4.75 Answer: C
B. Ammonia D. 4.75/COP
C. Freon 22 337. Clear ice
D. Freon 11 Answer: C A. is pure ice
B. contains dissolved gases
Answer: C 332. The highest temperature in vapor compression cycle is C. contains dissolved air
produced during: D. is formed by blowing air during freezing
326. During compression in a vapor compression cycle when A. expansion valve
the refrigerant is super-heated, what happens to the B. evaporator Answer: D
performance? C. condenser discharged
A. C.O.P. is reduced D. compressor discharged 338. Dry ice is
B. C.O.P. remains unchanged A. free from water
C. work done is increased Answer: D B. free from dissolved air and gases
D. refrigerating effect is reduced C. does not contain impurities
333. Which type of compressor is used in refrigeration D. solidified form of carbon dioxide
Answer: A system?
A. reciprocating Answer: D
327. A Bell-Coleman Cycle is also known as _____. B. centrifugal
A. reversed Otto cycle C. rotary sliding vane 339. The leaks in the refrigeration system using Freon can
B. reversed Joule cycle D. all of the above be detected by:
C. reversed Rankine cycle A: Halide torch which detention forms greenish flame lighting
D. reversed Carnot cycle Answer: D B. Sulfur sticks which on detection forms white smoke
C. using certain reagents
Answer: B 334. What is the reason why a thermometer in vapor D. smelling
compression system is installed close to the compressor?
328. Critical Temperature is that temperature above which: A. because it helps the operator in adjusting compressor for Answer: A
A. a gas will never liquefy greatest efficiency
B. a gas gets immediately liquefy B. because temperature indicates whether liquid or vapor 340. The lower horizontal line of the refrigeration cycle
C. water gets evaporated refrigerant is going to compressor plotted on pressure-enthalpy diagram represents:
D. water will never evaporate C. because temperature helps in calculating the coefficient A. compression of the refrigerant vapor
Answer: A of performance B. evaporation of the refrigerant liquid
D. because the performance of other units of the cycle can C. condensation of the refrigerant vapor
329. Rating of a domestic refrigerator is of the order of: be controlled D. metering of the refrigerant liquid
A. 0.1 tons
B. 50 tons Answer: A Answer: B
C. 100 tons
D. 40 tons 335. Which refrigerant has the highest critical point 341. Which of the following uses a secondary refrigerant?
temperature? A. domesticated refrigerator
Answer: A A. ammonia B. room air conditioner
B. Freon 11 C. deep freezer
330. What is the usual COP of a domestic refrigerator? C. Freon 12 D. ice plant
A. less than 1 D. Freon 22
Answer: D C. -78.5 ℃
342. Which of the following secondary refrigerants generally Answer: C D. -40 ℃
used milk chilling plants?
A. Brine 348. Which refrigerant is used for the air-conditioning of the Answer: D
B. Ammonia solution passenger aircraft cabin?
C. Glycol A. ammonia 354. The following gas is preferred in refrigeration system:
C. Sodium silicate B. Freon 12 A. F-11
C. Freon 11 B. F-22
Answer: C D. air C. 𝐶𝑂2
D. 𝑁𝐻3
343. What is the cycle where a refrigeration system Answer: D
generally operates? Answer: B
A. open cycle 349. Oil separator in a refrigeration cycle is installed
B. close cycle between the ___________. 355. The bhp/ton refrigeration requirement with increase in
C. mixed cycle A. compressor and condenser condenser temperature, in a vapor compression system that
D. Hybrid cycle B. condenser and evaporator uses reciprocating compressor receiving refrigerant gas at
C. metering device and evaporator constant suction temperature, will follow:
Answer: B D. none of the above A. linearly decreasing characteristic
B. linearly increasing characteristic
344. Which of the following is also known as Refrigerant Answer: A C. first increase rapidly and then decrease slowly
No.R-717? D. first increase slowly and then rapidly
A. Ammonia 350. A flash chamber is installed in the refrigeration circuit so
B. Freon 22 as to: Answer: B
C. Freon 12 A. improve overall heat transfer coefficient
D. methyl chloride B. reduce pressure losses through the evaporator 356. Antifreeze chemicals are:
C. reduce the size of the evaporator by avoiding vapors A. same as refrigerants
Answer: A going to evaporator B. those that are added to refrigerants for better
D. all of the above performance
345. In which part of the vapor compression cycle there is C. those that lower down the freezing points of liquids
abrupt change in pressure and temperature Answer: D D. those that do not freeze at all
A. solenoid valve
B. evaporator 351. The color of the flame of halide torch, in case of Answer: C
C. expansion valve leakage of Freon refrigerant, will change to:
D. drier A. yellow 357. The capacity of ice making is always
B. red A. less than the refrigerating effect
Answer: C C. green B. greater than the refrigerating effect
D. orange C. equal to the refrigerating effect
346. What do you call a plate or vane used to direct or D. directly proportional to the refrigerating effect
control movement of fluid or air within the confined area? Answer: C
A. baffle Answer: D
B. bellows 352. What is the boiling temperature of F-12?
C. regulator A. -33.33 ℃ 358. To check the lack of refrigerant or clogged refrigerant
D. diffuser B. -78.5 ℃ lines
C. - 29.8 ℃ A. installing service valves must be necessary
Answer: A D. -40 ℃ B. installing king valve must be an option
C. sight glasses can be possibly used
347. Brazing is used for joining two _____. Answer: C D. discharge pressure and temperature must be monitored
A. two ferrous material
B. one ferrous and non-ferrous material 353. What is the boiling temperature of F-22? Answer: A
C. two non-ferrous material A. -33.33 ℃
D. two non-metals B. - 29.8 ℃ 359. Past ME Board Question
Can any refrigerant be used with a booster system? D. -40.7 ℃ Answer: C
A. Yes Answer: C
B. No 370. Past ME Board Question
C. Used for some 365. Past ME Board Question The vapor cycle in thermal efficiency as the refrigeration
D. Used for all Boiling temperature of Freon 22 is: cycle is to the ______.
Answer: B A. -33.33 ℃ A. Energy efficiency ratio
B. -29.8 ℃ B. COP for a refrigerator
360. Past ME Board Question C. -78.5 ℃ C. COP for a heat pump
What is the chemical formula of methylene chloride? D. -41.04 ℃ D. Carnot efficiency
A. 𝐶𝐻𝐶𝑙2 𝐹 Answer: D Answer: B
B. 𝐶𝐻2 𝐶𝑙2
C. 𝐶𝐶𝑙𝐹3 366. Past ME Board Question 371. Past ME Board Question
D. 𝐶𝐵𝑅𝐹3 The faster way to remove frost from a direct-expansion The main purpose of a sub cooler in a refrigerating system
Answer: B finned-tube evaporator is to: especially a two-stage system is to:
A. sent hot gas through the coil A. Increase the heat rejection per ton and avoid
361. Past ME Board Question B. scrape off frost system shutdown
What is the chemical formula of butane? C. wash with warm water B. Improve the flow of evaporator gas per ton and
A. 𝐶2 𝐻5 𝐶𝑙 D. shut down warm let frost melt increase the temperature
B. 𝐶3 𝐻8 Answer: A C. Reduce the total power requirements and return
C. 𝐶2 𝐻4 𝑂2 oil to the compressor
D. 𝐶4 𝐻10 367. Past ME Board Question D. Reduce the total power requirements and heat
Answer: D You want to change refrigerants in a Freon 12 plant using a rejection to the second stage
reciprocating single-acting compressor. Which refrigerants Answer: D
requires smallest amount of change to the system?
362. Past ME Board Question A. Carbon dioxide 372. Past ME Board Question
Type of refrigerant control which maintains pressure B. Ammonia Ton of refrigeration is a unit equivalent to:
difference between high side and low side pressure in C. Methyl chloride A. 50.4 kCal/sec
refrigerating mechanism. D. Aqua ammonia B. 12,660 kN-m/hr
A. Suction valve Answer: C C. 3413 kW/hr
B. Expansion valve D. 2545 Btu/hr
C. Service valve Answer: B
D. Solenoid valve
Answer: B 373. Past ME Board Question
368. Past ME Board Question In a refrigerating system, the heat absorbed in the
363. Past ME Board Question Soft copper tubing that is used for refrigerant piping erected evaporator per kg mass of refrigerant passing through:
The dividing point between the high pressure and low on premises containing other than the Group 1 refrigerant A. Equals the increase in enthalpy
pressure sides of the refrigeration cycle occurs at the: must be protected by _____. B. Does not depend on the refrigerant used
A. expansion valve A. Supporting it by approved clamps C. Is decreased if pre-cooler is used
B. compressor B. Limiting its length between supports up to 10 feet D. Equals the increase in volume
C. condenser C. A rigid or flexible metal enclosure Answer: A
D. cooling coil D. Wrapping it with insulation tape throughout its
Answer: A length 374. Past ME Board Question
Answer: C Lithium bromide is used in refrigeration system in:
A. Condensate return lines
369. Past ME Board Question B. Absorbers
364. Past ME Board Question A Freon 12 refrigeration system is fitted with thermal C. Centrifugal compressors
Boiling temperature of Freon 12 is: expansion valves. The valves are rated in: D. Ion exchangers
A. -33.33 ℃ A. Pound per minute Answer: B
B. -78.5 ℃ B. Super heat setting
C. -29.8 ℃ C. Tons of refrigeration 375. Past ME Board Question
D. Cubic feet per minute
A refrigeration system in which only part of the refrigerant Answer: D D. 60
passes over the heat transfer surface is evaporated and the Answer: A
balance is separated from the vapor and recirculated. 381. Past ME Board Question
A. Direct expansion system The boiling point of Freon 12 is: 387. Past ME Board Question
B. Chilled water system A. -41.04 ℉ The high pressure of refrigerant system consist of the line to
C. Flooded system B. -40.60 ℉ the expansion valve, the receiver, the uppermost half of the
D. Multiple system C. -38.40 ℉ compressor and the:
Answer: C D. -31.40 ℉ A. Line after the expansion valve
Answer: A B. Lower most half of compressor
376. Past ME Board Question C. Condenser
A device whose primary function is to meter the flow of 382. Past ME Board Question D. Evaporator
refrigerant to the evaporator. The secondary refrigerant used in milk chilling plants is Answer: C
A. Sniffer valve generally:
B. Equalizer A. Brine 388. Past ME Board Question
C. Thermostatic expansion valve B. Ammonia Which of the following material is suitable for tubing in
D. Crossover valve C. Glycol refrigeration application where refrigerant ammonia is
Answer: C D. Sodium silicate employed?
Answer: C A. Plastic
377. Past ME Board Question B. Brass
Select the one in which secondary refrigerant is used. 383. Past ME Board Question C. Steel copper
A. Domestic refrigerator The sensible heat ratio is 0.8. That is D. Copper
B. Room air conditioner A. 20% latent heat and 60% sensible heat Answer: C
C. Deep freezer B. 80% sensible heat and 20 % latent heat
D. Ice plant C. 80% sensible heat and 20% sensible heat and 389. Past ME Board Question
Answer: D latent heat If PV is the power required for a vapor-compression
D. 20% latent heat and 80% sensible heat and latent refrigeration system, then what is the power required for an
378. Past ME Board Question heat air-refrigeration system, assuming that they have the same
AN odorless refrigerant whose boiling point varies over a Answer: B capacity?
wide range of temperature. A. 5PV
A. Freon 22 384. Past ME Board Question B. 2PV
B. Freon 12 The effect of superheating the refrigerant is to: C. PV/10
C. Ammonia A. Increase the COP D. 1/PV
D. Freon 11 B. Increase and decrease the COP Answer: A
Answer: A C. Decrease COP
D. Maintains COP 390. Past ME Board Question
379. Past ME Board Question Answer: A Tons of refrigeration in Btu/24 hours is
The part that directs the flow of refrigerant through the A. 288,000
compressor. 385. Past ME Board Question B. 28,800
A. Wrist pin The ice making capacity is always C. 290,000
B. Valve A. Directly proportional to the refrigerating effect D. 29,000
C. Piston B. Less than the refrigerating effect Answer: A
D. Connecting rod C. Greater than the refrigerating effect
Answer: B D. Equal to the refrigerating effect 391. Past ME Board Question
Answer: A R-22 is:
380. Past ME Board Question A. Dichlorodiflouromethane
The highest temperature in vapor compression cycle is 386. Past ME Board Question B. Monochlorodiflouromethane
produced during: The amount of sensible heat for a sensible heat ratio of 0.80 C. Methyl chloride
A. Receiver and a total cooling load of 100 is: D. Trichlorodiflouromethane
B. Evaporator A. 80 Answer: B
C. Condenser discharge B. 20
D. Compressor discharge C. 100 392. Past ME Board Question
A salimeter reads the: CHAPTER 15 – AIR CONDITIONING Answer: D
A. Density of brine
B. Rate of brine 1. How do you call the water vapor content of air? 7. The temperature at which water vapor in the atmosphere
C. Dew point temperature A. Moisture begins to condense is known as the:
D. Relative humidity B. Humidity A. vapor point
Answer: A C. Dew B. dew point
D. Vapor C. moisture point
393. Past ME Board Question D. none of the above
Which of the following measure the density of salt in water? Answer: B
A. Salimeter Answer: B
B. Hydrometer 2. When air contains all of the water vapor it can hold, it is
C. Pitot tube said to be 8. Saturation temperature is the same as;
D. Calorimeter A. Simulated A. dew point
Answer: A B. Loaded B. vapor temperature
C. Saturated C. steam temperature
D. Moistured D. humidity
394. Past ME Board Question
What is the device used to protect the compressor from Answer: C Answer: A
overloading due to high head pressure?
A. Overload relay 3. What is the instrument used to register relative 9. When the bypass factor is B, the coil efficiency for
B. Hold back suction valve humidity? sensible cooling of air is equal to:
C. Thermostatic expansion valve A. Hygrometer A. 1 + B
D. Expansion valve B. Hydrometer B. 1 – B
Answer: A C. Perometer C. 1 – B / B
D. Manometer D. 1 + B / B
395. Past ME Board Question
The purpose of the expansion valve is to control the flow of Answer: A Answer: B
the refrigerant to the evaporator. The other function is to:
A. Reduce the pressure of the liquid refrigerant 4. Humidity is a measure of which of the following? 10. The psychrometric chart in air conditioning determines
B. Increase the pressure of the vapor refrigerant A. water vapor content the
C. Reduce the pressure of the vapor refrigerant B. temperature A. wet bulb and dry bulb temperatures
D. Increase the pressure of the liquid refrigerant C. latent heat B. psychrometric temperature requirements
Answer: A D. any of the above C. saturation temperature and relative humidity
D. moist air conditions
396. Past ME Board Question Answer: A
It prevents the refrigerant from the condenser to go back to Answer: D
the compressor. 5. Air conditioning is the process of:
A. Check valve A. keeping a place cool 11. Which of the following is considered as comfort condition
B. Float switch B. removing heat from a specific area in air conditioning?
C. Expansion valve C. maintaining the air at a required temperature and humidity A. 40℃ DBT, 80% RH
D. Low side float D. A or C B. 30℃ DBT, 60% RH
Answer: A C. 30℃ DBT, 80% RH
Answer: C D. 20℃ DBT, 60% RH
397. If the freezing point of water is 0℃, which of the
following is its melting point? 6. In an air-conditioning system, before the air is circulated to Answer: D
A. slightly less than 0℃ the required area, it is:
B. slightly more than 0℃ A. cooled 12. The drift loss in cooling towers is about:
C. 0℃ B. filtered A. 1% only
D. 32℃ C. dehumidified B. 12 to 15%
Answer: C D. all of the above C. 10 to 20%
D. 30 to 40%
heat transfer rate of the layers dirt and foreign materials. D. All of these
Answer: C What is this factor?
A. Booster Factor Answer: A
13. Cooling towers are used for cooling water: B. Factor of Safety
A. to be injected in circulating air C. Fouling Factor 24. Fan motors in air conditioning units usually have:
B. to be used for humidification D. Compression Factor A. 2 or 3 speeds
C. to be used for filtration of air B. 3 or 4 speeds
D. to be used for cooling the compressor Answer: C C. 1 or 2 speeds
D. 4 or 5 speeds
Answer: D 19. The engineer was tested to design the air conditioning
system for a ball room dance hall. Considering that this Answer: A
14. By which of the following processes heat mainly involve a lot of activity from its users, the engineer would
dissipates in cooling towers? design that will require: 25. Which of the following capacitors that can usually be
A. conduction A. Maximum attainable effective temperature found in an air conditioning unit?
B. convection B. Constant Effective temperature A. Starting capacitors
C. radiation C. Higher Effective Temperature B. Running capacitors
D. evaporation D. Lower Effective Temperature C. Fan motor capacitors
D. all of these
Answer: D Answer: A
Answer: D
15. Which one is commonly used liquid absorbent? 20. What is the value of the Mach No. throat of the
A. silica gel converging-diverging nozzle? 26. Which of the following troubles commonly occurs inside
B. activated alumina A. Zero an air conditioning unit?
C. ethylene glycol B. Two A. Stuck Compressor
D. any of the above C. One B. Clogged Refrigerant circuit
D. Ten C. Short Circuit, open circuit, grounded motor winding
Answer: C D. All of these
Answer: C
16. In sensible heating cooling following parameter remains Answer: D
unchanged 21. What do bodies at a temperature above absolute zero?
A. dry bulb temperature A. It emits Energy 27. The motor condition of an air conditioning unit can be
B. wet bulb temperature B. It emits Thermal Radiation checked with:
C. relative humidity C. It emits Heat for Conduction A. the continuity of light or with an ohmmeter
D. humidity ratio D. It emits Heat for convection B. the flow of refrigerant in the receiver
C. the discharge pressure in the compressor
Answer: D Answer: B D. All of these

17. Heating and dehumidification can be obtained 22. When air is saturated, the wet bulb depression is: Answer: A
simultaneously if air is passed through: A. zero
A. sprays of water kept at a temperature higher than the dew B. indefinite 28. An air conditioning system wherein the entire systems
point temperature of the entering air C. unity are mounted in the cabinet:
B. a solid absorbent surface D. 100% A. Console Air Conditioners
C. a liquid absorbent spray B. Centralized Air Conditioners
D. any one of B and C Answer: A C. Package Type Air Conditioners
D. Multizone Split Ductless System
Answer: D 23. In a window air conditioning unit which of the following is
usually done by the owner? Answer: A
18. Dirt and foreign materials normally build-up on the side A. Semiannual cleaning or replacement of filters
of the condenser tubes. To ensure adequate condenser B. Annual cleaning of the evaporator, condenser fan blades, 29. The use of water to carry heat occupied spaces
capacity, a certain factor is used in calculating the overall fan motor, compressor and casing A. Hydronic Heating System
heat transfer through the walls of the tubes including the C. Inspection of fan motors and lubricate them B. Water Boiling System
C. Hydrokinetic Heating System 35. The amount of CO can be determined by the color of B. 3℃ to 5℃
D. Hydraulic Heating System Palladium Chloride. An amount of 30 ppm to 70 ppm will C. 4℃ to 5℃
cause: D. 1℃ to 2℃
Answer: A A. Slight Darkening
B. Grey Color Answer: A
30. The method of cooling which primarily used where C. Red Color
ambient air temperatures are high and relative humidity is D. Black Color 41. In an air conditioning unit, the thermostat fails. The unit
used: did not start. How do you test the operation of the
A. Swamp Cooling Answer: A thermostat?
B. Evaporative Cooling A. Cover the air outlet and air inlet with a cloth. The air will
C. Condensate Cooling 36. The term used to express the amount moisture in a given recirculate into the unit and the temperature will quickly drop
D. Hydroionic Cooling sample of air. It is compared with the amount of moisture in to the cut out temperature
a given sample of air. B. Cover the outlet and the air will not circulate and the
Answer: A A. Humidity thermostat functions because no air movement
B. Relative Humidity C. Cover both the inlet and outlet so that the recirculating air
31. The other name for Swamp Cooling is: C. Absolute Humidity has the same temperature
A. Evaporative Cooling D. Humidity Ratio D. Never cover any air passage so that the air can freely
B. Condensate Cooling move and thermostat functions well
C. Wet Roof Cooling Answer: B
D. Excelsior Cooling Answer: A
37. How should the window type air conditioning unit be
Answer: C placed? 42. Which of the following dehumidifier is often used to
A. Slant toward the inside of the home reheat the air after moisture is removed?
32. Evaporative Condenser is used to cool B. Slant toward the outside of the home A. Evaporator
A. condenser vapor C. Level B. Chiller
B. condenser surface D. Slant at approximately 15𝑜 from the horizontal C. Condenser
C. condensate liquid D. Compressor
D. All of these Answer: B
Answer: C
Answer: A 38. A dehumidifier is usually a small hermitic refrigerating
system. It has both a condenser and an evaporator. Many 43. Which of the following refrigerant is added sometimes to
33. A method of cooling which uses water as refrigerant. older systems use R-12 or R-500. The newer units use: other refrigerant to improve oil circulation?
Pressure on the water surface is reduced to lower its boiling A. R-134a A. R-117
temperature. B. R-145a B. R-777
A. Steam Jet Cooling C. R-217a C. R-170 (Ethane)
B. Evaporative Cooling D. R-121a D. R-270
C. Vortex Tube Cooling
D. Pressurized Cooling Answer: A Answer: C

Answer: A 39. The normal cut-out setting of a window unit thermostat is 44. A refrigeration that deals with producing temperature of -
between: 157 ℃ or lower,
34. Palladium Chloride may be used to measure the A. 13℃ to 16℃ A. Low temperature refrigeration
presence of: B. 16℃ to 21℃ B. Extremely low temperature refrigeration
A. vapor C. 17℃ to 22℃ C. Absolute zero refrigeration
B. CO D. 10℃ to 15℃ D. Cryogenics
C. ammonia
D. refrigerant Answer: A Answer: D

Answer: B 40. Thermostat are used with most window units. They have 45. Measurement of a device’s ability to remove atmospheric
differentials which vary between: air from test air.
A. 2℃ to 4℃ A. Atmospheric Dust Spot Efficiency
B. Atmospheric Dry Air Efficiency Answer: A
C. Test Air Efficiency Answer: A
D. Baudelot Air Efficiency 51. Substances that have the ability to absorb moisture from
the air are called: 57. The temperature below which water vapor in the air will
Answer: A A. Desiccants start to condense.
B. Moisturizer A. Condensing Temperature
46. A liquid mixture having constant maximum and minimum C. Dehygroscopic substance B. Dry Bulb Temperature
boiling points. Refrigerants comprising this mixture do not D. Moisture absorber C. Dew Point Temperature
combine chemically, yet the mixture provides constant D. Wet Bulb Temperature
characteristics Answer: A
A. Azeotropic Mixture Answer: C
B. Homogeneous Mixture 52. When the dry bub and the wet bulb temperatures are
C. Conzeotropic Mixture identical, the air is said to be: 58. Which of the following is not used as method to measure
D. Cryogenic Mixture A. saturated air velocities?
B. compressed A. Hot wire Anemometer
Answer: A C. humidified B. Rotating Anemometer
D. dehumidified C. Swinging Vane Velocimeter
47. Refrigerant in Group A1 (R-11). Chemical combination of D. Open type Barometer
carbon, chlorine and fluorine Answer: A
A. Carrene Answer: D
B. Cerrene 53. In what form that water exists in air?
C. CCL group A. Solid 59. It is a form of oxygen photochemically produced in
D. HCL group B. Liquid nature.
C. Vapor A. Ozone
Answer: A D. Saturated B. Oxidation
C. Oxidizing Agent
48. Which of the following refrigerants is popular in the Answer: C D. 𝐷𝑂2
industrial refrigerating system; also popularly absorption
system of refrigerant. 54. When air is heated, what happened to its relative Answer: A
A. R-717 (Ammonia) humidity?
B. R-600 (Butane) A. Increase 60. What is the chemical formula of Ozone?
C. R-611 (Methyl Formate) B. Decrease A. 𝑂3
D. R-504 C. Remain Constant B. 𝑂2
D. May increase or Decrease Depending on temperature C. 𝐷2 𝑂
Answer: A D. 𝑂1
Answer: B
49. Which of the following is a function of air conditioning? Answer: A
A. Temperature and Humidity Control 55. The Horizontal Scale (Abscissa) in the Psychrometric
B. Air, Filtering, Cleaning, and Purification Chart represents: 61. In the upper atmosphere, ozone is made by ultraviolet
C. Air Movement and Circulation A. Dry bulb temperature light reacting with:
D. All of these B. Wet bulb temperature A. Oxygen
C. Relative Humidity B. Hydrogen
Answer: D D. Humidity C. Nitrogen
D. Water Vapor
50. Oxygen is approximately what percent by weight in the Answer: A
atmosphere? Answer: A
A. 23% 56. Most people are comfortable with the relative humidity of:
B. 27% A. 30% to 70% 62. The ozone concentration of 0.10 parts per million (ppm)
C. 77% B. 10% to 40% is generally considered the maximum permissible for how
D. 73% C. 20% to 30% many hours exposure?
D. 40% to 80% A. 8 hrs
B. 4 hrs 68. Why should you avoid bending or twisting of fan blades Answer: A
C. 7 hrs in an air conditioning unit?
D. 3 hrs A. It will cause ice build-up 74. Air delivered to the room from the supply duct, moving at
B. It will wear out the motor bearings and cause noise a velocity of 150 ft/min or more is called:
Answer: A C. It may slice suction line A. Primary Air
D. It decreases the volume flow refrigerant B. Secondary Air
63. How may moisture be removed from air? C. Saturated Air
A. Condensation Answer: B D. Air Turbulence
B. Absorption
C. A and B 69. A type of refrigerant that will not damage the ozone layer Answer: A
D. none of these A. Hydroflourocarbons (HCF’s)
B. Hydrochloroflourocarbons (HCHF’s) 75. Carries needed to deliver air to the conditioned space. It
Answer: C C. R- 22 is made of sheet of metals like aluminum, galvanized sheet
D. R-12 steel and some structural materials that will not burn.
64. In cooling cycle, the dry bulb temperature (db) of the air A. Duct
is lowered. When this happens, the relative humidity Answer: A B. Air Inlet
A. increases C. Air Outlet
B. decreases 70. Which of the following is the type of refrigerant that D. Diffuser
C. remains constant damages Ozone layer?
D. increases or decreases depending on the temperature at A. Hydrochloroflourocarbons (HCHF’s) Answer: A
which it is cooled B. R-12
C. R-22 76. From what principle that air ducts operate?
Answer: A D. All of these A. Principle of Pressure Difference
B. Principle of Temperature Difference
65. What is the effect of superheating the refrigerant? Answer: D C. Forced Draft Fan Principle
A. It increases the Coefficient of Performance D. Principle of Natural Convection
B. It decreases the Coefficient of Performance 71. Large-bulb alcohol thermometer used to measure air
C. It lowers the boiling point of the refrigerant speed or atmospheric condition by means of cooling. Answer: A
D. It increases the suction pressure of the refrigeration A. Kata Thermometer
B. Kelvin Thermometer 77. Which of the following is the common classification of
Answer: A C. JJ Thompson Thermometer ducts?
D. Wet Bulb Thermometer A. Condition-Air Ducts
66. In sensible heating the absolute humidity remains B. Recirculating
constant but the relative humidity: Answer: A C .Fresh-air Ducts
A. increases D. All of these
B. decreases 72. Which of the following components of the window air
C. Remains constant conditioning system must be cleaned annually? Answer: D
D. zero A. Evaporator and Condenser
B. Motor and compressor 78. It is used deliver concentrated airstreams into a room.
Answer: B C. Fan Blades and Fan motor Many have one-way or two-way adjustable air stream
D. All of these deflectors.
67. The relative humidity becomes 100% and where the A. Register
water vapor starts to condense Answer: D B. Grille
A. Dew Point C. Diffuser
B. Cloud Point 73. The phenomenon that warm air rise and cold air settle is D. Damper
C. Saturated Point called:
D. Critical Point A. Stratification Answer: A
B. Sedimentation
Answer: A C. Setting due 79. It is used to control the air-throw distance, height and
D. Ventilation spread, as well as the amount of air.
A. Grille
B. Register 90. Which of the following is to be checked as part of the
C. Diffuser 85. The ratio of the rated cooling capacity divided by the weekly maintenance schedule of a console air conditioners?
D. Damper amount of electrical power used: A. Filters
A. Energy Efficiency Ratio (EER) B. Humidifiers
Answer: A B. Energy Efficiency Index (EE) C. Safety valves
C. Cooling Efficiency (CE) D. Cooling Tower
80. It is used to deliver widespread, fan-shaped flows of air D. Energy Cooling Ratio (ECR)
into the room. Answer: D
A. Grille Answer: A
B. Register 91. A mechanism that removes moisture.
C. Diffuser 86. Heat exchanger in which water flows by gravity over the A. Humidifiers
D. Damper outside of the tubes or plates. B. Dehumidifiers
A. Baudelot Cooler C. Moisturizers
Answer: C B. Free Cooler D. Cooling Towers
C. Newtonian Cooler
81. Which of the following fans in air conditioning systems D. Bourdon Cooler Answer: B
which can be classified as centrifugal flow?
A. Axial fan Answer: A 92. When coil surface temperature is below the dew point of
B. Propeller fan the air.
C. Bi-axial fan 87. A popular air conditioning system that includes a single A. moisture will condense out of the air
D. None of these outdoor condenser, three independent evaporators, and B. vapor will evaporate through the air
individual evaporator temperature control. The condensing C. air is dehumidified
Answer: B unit is located outside on a slab. They are frequently used in D. air is humidified and saturated
legal and medical offices, motels and homes without ducts.
82. The most common controller in the heating and cooling A. Multizone Ductless Split System Answer: A
systems. B. Package Terminal Air Conditioner
A. Thermostat C. Console Air Conditioner 93. Nitrogen occupied almost how much of the Earth’s
B. Pressure gage D. Window type Air Conditioner Atmosphere
C. Barometer A. One-fourth
D. Sling Psychrometer Answer: A B. Three-fourths
C. One-half
Answer: A 88. An air conditioning system which is a combination of D. One-third
heating and cooling system. It is designed to serve an
83. A thermostat that functions as increase or decrease individual room or zone. Answer: B
instead of starting and stopping system is called: A. Multizone Ductless Split System
A. modulate B. Package Terminal Air Conditioner 94. Water in vapor form remains a vapor as long as its
B. heating-cooling the thermostat C. Console Air Conditioner temperature is what relations to the dew point temperature.
C. interlocked D. Window type Air Conditioner A. above
D. compound thermostat B. equals
Answer: B C. below
Answer: A D. almost
89. Which of the following is to be checked as regular
84. A number used to compare energy usage for different monthly maintenance schedule of a console air conditioning Answer: C
areas. It is calculated by dividing the energy consumption by units?
the fottage of the conditioned are. A. Water Leaks 95. An instrument used in measuring air velocity by velocity-
A. Energy Utilization Unit Fan Speeds pressure method
B. Energy Utilization Efficiency C. Cooling Tower A. Pitot Tube
C. Energy Utilization Index D. Duct Dampers, Registers and Diffusers B. Anemometer
D. Energy Utilization Ratio C. Aneroid Barometer
Answer: D D. Flowmeter
Answer: C
Answer: A A. Calcium chloride C. Indeterminate
B. Activated alumina D. Increases
96. What is the specific humidity of dry air? C. Sodium zeolite
A. 100 D. Silica gel Answer: A
B. 20
C. 0 Answer: A 107. Past ME Board Question
D. 50 What amount of air is required in a low bypass factor?
Answer: C 102. All the following temperatures have meaning in A. Greater
psychrometrics excepts: B. Lesser
97. When hot soup was served in a cup during dinner, an A. dry-bulb temperature C. Indeterminate
engineer was so eager to drink it. Since it was hot, he added B. wet-bulb temperature D. Constant
ice cubes of ice to cool the soup stirred it. He noticed that C. adiabatic wall temperature
dew starts to form on the outermost surface of the cup. He D. dew point Answer: B
wanted to check the temperature of the outermost surface of
the cup. What is the temperature equal to? Answer: C 108. Past ME Board Question
A. superheated temperature The design of an air supply duct of an air conditioning
B. equal to zero 103. The relative humidity is given by the: system _______.
C. standard temperature A. ratio of the actual humidity to the saturated humidity at the A. Adds moisture to the air
D. equal to air’s dew point temperature same temperature and pressure B. Lowers the temperature of the air
B. ratio of the partial pressure of water vapor to the C. Does not affect the moisture of air
Answer: D saturation pressure D. Affects the distribution
C. ratio of wet-bulb to dry-bulb temperature
98. The temperature at which the water vapor in the gas D. ratio of dry-bulb temperature to dew point Answer: D
begins to condense in a constant pressure process.
A. Dew point Answer: B 109. Past ME Board Question
B. Vapor point What is the lowest temperature to which water could
C. Flue point 104. The determination of properties and behavior of possibly be cooled in a cooling tower?
D. Gas point atmospheric air usually the purview of: A. The effective temperature
A. thermodynamics B. The temperature of adiabatic compression
Answer: A B. psychrometrics C. The wet bulb depression
C. forced convection D. The dew point temperature of the air
99. All the following processes can be found on a D. Kirchoff’s law
psychrometric chart except: Answer: B
A. humidification Answer: B
B. sensible heating 110. Past ME Board Question
C. natural convection 105. Past ME Board Question Combined process of cooling and humidifying is also known
D. sensible cooling Which of the following statements is correct? as:
The relative humidity of an air water vapor mixture A. Heating and humidifying
Answer: C A. Is the ratio of the partial pressure of the water B. Cooling tower
vapor to the partial pressure of the air C. Evaporative cooling process
100. All of the following process can be found on a B. Indicates the mass of water vapor in the mixture D. Moisture removal process
psychrometric chart except: C. Is equal to the mole fraction of water vapor in the
A. heating and humidifying mixture at the mixture temperature Answer: C
B. cooling and dehumidification D. Is the ratio of the partial pressure of water vapor to
C. black body radiation the saturation pressure at the mixture temperature 111. Past ME Board Question
D. evaporative cooling Answer: D In a cooling tower, the water is cooled mainly by:
A. Condensation
Answer: C 106. Past ME Board Question B. Evaporation
In sensible cooling process, moisture content C. Convection
101. Which is not commonly used to cool and dehumidify A. Does not change D. Conduction
equipment? B. Decreases
Answer: C Answer: B D. Equal to air temperature

112. Past ME Board Question 117. Past ME Board Question Answer: C


Which of the following types of air dryers works by absorbing The relative humidity becomes 100% and where the water
moisture on a solid dessicant or drying material such as vapor starts to condense. CHAPTER 16-MACHINE FOUNDATION AND CHIMNEY
activated alumina, silicon gel, or molecular sieve? A. Critical point 1. Past ME Board Question
A. Regenerative dryer B. Saturated point Foundations are preferably built of concrete in the proportion
B. Deliquescent dryer C. Dew point of what measure of Portland cement; sand; crushed stones?
C. Spray dryer D. Cloud point A. 1 : 2 : 5
D. Refrigerated dryer B. 2 : 4 : 6
Answer: C C. 2 : 3 : 5
Answer: B D. 1 : 2 : 4
118. Past ME Board Question
113. Past ME Board Question What is the specific humidity of dry air? Answer: D
The relationship of water vapor in the air at the dew point A. 150
temperature to the amount that would be in the air if the air B. 100 2. Past ME Board Question
were saturated at the dry bulb temperature is: C. 50 For design stability, the center of gravity of the total
A. Partial pressure actual at dew point D. 0 combined engine, driven equipment and foundation should
B. Percentage humidity be kept _________.
C. Relative humidity Answer: D A. Anywhere
D. Partial pressure of water B. Above the foundation top
119. Past ME Board Question C. In line with surface of the foundation
Answer: C An engineer inspected an air-conditioning unit. He found out D. Below the foundation top
that the unit does not produce any cooling effect, however,
114. Past ME Board Question the air-conditioning unit is running. He checked the Answer: D
When the air is saturated, the wet bulb depression is: temperatures of the condenser and evaporator and had the
A. Zero unit run. He found out that there was no change in 3. The machine foundation must have a factor of safety of
B. Indefinite temperature. What should he do? A. 4
C. Unity A. Replace fuse B. 5
D. 100% B. Charge with new refrigerant C. 6
C. Replace relay D. 7
Answer: A D. Adjust door seal
Answer: B
115. Past ME Board Question Answer: B
A temperature measurement in an ordinary thermometer 4. The diesel engine foundation safe soil bearing pressure is:
which has constant specific humidity 120. Past ME Board Question A. 4,890 kg/𝑐𝑚2
A. Critical temperature What is the temperature range of air in air conditioning B. 4,500 kg/𝑐𝑚2
B. Dew point temperature application where the dry air can be considered ideal gas? C. 4,490 kg/𝑐𝑚2
C. Dry bulb temperature A. 100 to 125 ℃ D. 4,125 kg/𝑐𝑚2
D. Wet bulb temperature B. 50 to 75 ℃
C. 75 to 100 ℃ Answer: A
Answer: B D. -10 to 50 ℃
5. Anchor bolts in a machine foundation should be
116. Past ME Board Question Answer: D embedded in concrete of at least how many times the bolt
During sensible heating, the humidity remains constant but diameter?
the relative humidity. 121. Past ME Board Question A. 12
A. Increases What is the value of air stratification in air conditioning B. 20
B. Decreases design fit for human comfort? C. 30
C. Remains constant A. Minimum D. 25
D. Zero B. Maximum
C. Less than air temperature Answer: C
11. For stability, the total combined engine, driven C. 5
6. As a good practical rule, the foundation depth may be equipment and foundation center of gravity must be kept D. 6
taken as how many times of the engine stroke? _____.
A. 2.2 to 3.2 A. below the foundation top Answer: A
B. 3.2 to 4.2 B. above the foundation top
C. 2.5 to 3.5 C. 120 mm above the foundation top 17. For a diesel engine’s foundation, in pouring a concrete
D. 3.5 to 4.5 D. none of the above mixture, it should be poured:
A. one time
Answer: B Answer: A B. two times
C. three times
7. To secure the belts embedded within the foundation, the 12. There shall be no foundation bolts less than ____. D. four times
distance of the edges of the foundation from the bedplate A. 12 mm in diameter
must be _________. B. 16 mm in diameter Answer: A
A. 120 mm to 300 mm C. 18 mm in diameter
B. 150 mm to 330 mm D. 20 mm in diameter 18. For foundations of steam turbine, the concrete mixture
C. 100 mm to 280 mm should be _________.
D. 200 mm to 380 mm Answer: A A. 1: 2: 4
B. 1: 3: 5
Answer: A 13. The weight of steel bar reinforcements should be how C. 1: 2: 3
many times the weight of the foundation? D. 1: 3: 4
8. The weight of the machine foundation is how many times A. ½% to 1%
of the weight of the engine? B. 1% to 1.5% Answer: A
A. 4 to 5 times C. 3% to 5%
B. 3 to 5 times D. 3.2% to 4.2% 19. The steam turbine foundation should be designed to
C. 2 to 3 times support the machine load plus how many percent of the
D. 4 to 6 times Answer: A impact, condenser load, floor loads and dead loads?
A. 15
Answer: B 14. Concrete foundation should have steel bar B. 20
reinforcements placed vertically and horizontally to avoid C. 25
9. To eliminate transmission of vibration, the foundation ____. D. 30
should be isolated from floor slabs or building footings at A. breaking
least how many mm around its perimeter? B. thermal cracking Answer: C
A. 15 C. melting
B. 20 D. vibration 20. Which of the following draft rely on the stack effect to
C. 25 draw off combustion gases?
D. 30 Answer: B A. Natural draft
B. Forced draft
Answer: C 15. Foundations bolt length should be at least how many C. Induced draft
times the anchor bolt diameter? D. Balanced draft
10. The minimum vertical distance from the floor or soil level A. 16 times
to the top edge of the foundation must be around B. 18 times Answer: A
_________. C. 20 times
A. 150 mm D. 30 times 21. Fans located before the furnace that are used to supply
B. 100 mm air for burning.
C. 200 mm Answer: B A. Natural draft
D. 120 mm B. Forced draft
16. Foundation bolts of specified size should be used and C. Induced draft
Answer: D surrounded by a pipe sleeve with an inside diameter of at D. Balanced draft
least how many times the diameter of the anchor bolt?
A. 3 Answer: B
B. 4
22. Forced draft fans are run at relative high speed in the D. absolute pressure D. 30 m
range of:
A. 1200 to 1800 rpm Answer: A Answer: A
B. 1000 to 1600 rpm
C. 1500 to 2000 rpm 28. The static pressure drop due to friction through the boiler 34. Most stacks are built of height less than:
D. 900 to 1500 rpm and stack. A. 60 m
A. draft loss B. 50 m
Answer: A B. available draft C. 40 m
C. stack effect D. 30 m
23. Chimneys that rely on natural draft are sometimes D. fan boost
referred to as Answer: D
A. natural chimney Answer: A
B. gravity chimney 35. The average flue gas temperature is
C. normal chimney 29. The difference between the theoretical draft and the draft A. the temperature entering the stack
D. stack loss. B. the temperature leaving the stack
A. draft loss C. the temperature halfway up the stack
Answer: B B. available draft D. the sum of the temperatures
C. stack effect
24. Force draft fans create a positive pressure of: D. fan boost Answer: C
A. 0.5 to 2.5 kPa
B. 1.0 to 3.0 kPa Answer: B
C. 2.0 to 4.0 kPa 36. The coefficient of velocity is approximately
D. 0.2 to 2.2 kPa 30. In a balanced system, the available draft is: A. 0.30 to 0.50
A. unity B. 0.40 to 0.60
Answer: A B. zero C. 0.20 to 0.40
C. 100 D. 0.50 to 0.70
25. Fans that are used to draw combustion products through D. infinite
the furnace bed, stack, and pollution control system by Answer: A
injecting air into the stack after combustion. Answer: B
A. Natural draft 37. For realistic problems, the achievable stack effect
B. Forced draft 31. The total pressure supplied by the fan at maximum probably should be considered to be:
C. Induced draft operating conditions. A. 75% of the ideal
D. Balanced draft A. net rating or fan boost B. 80% of the ideal
B. draft loss C. equal to the ideal
Answer: C C. available draft D. half the ideal
D. stack effect
26. The term used when the static pressure is equal to the Answer: B
atmospheric pressure. Answer: A
A. Natural draft 38. Guyed stacks seldom exceed:
B. Forced draft 32. Generally the higher the chimney A. 1.83 m in diameter
C. Induced draft A. the smaller the stack effect B. 2.83 m in diameter
D. Balanced draft B. the greater the stack effect C. 1.45 m in diameter
C. stack effect is zero D. 2.45 m in diameter
Answer: D D. the stack effect approaches unity
Answer: A
27. In order to keep combustion products inside the Answer: B
combustion chamber and stack system, balanced draft 39. The angle between the stack and the guy wire is usually:
system may actually operate with a slight 33. Modern stacks are seldom built higher than A. 30 degrees
A. negative pressure A. 60 m B. 45 degrees
B. positive pressure B. 50 m C. 60 degrees
C. positive and negative pressures C. 40 m D. 75 degrees
45. Foundation bolts of specified size should be used and
Answer: C surrounded by a pipe sleeve with a length of at least how Answer: C
many times the diameter of the bolt?
40. Guyed stacks height seldom exceeds: A. 18
A. 30.48 m B. 14
B. 34.80 m C. 15 CHAPTER 17 – INSTRUMENTATION
C. 43.80 m D. 16 1. An instrument used for measuring evaporation, generally
D. 48.30 m that of water, into the atmosphere.
Answer: A A. Aerometer
Answer: A B. Atmometer
46. Foundation should be designed to support the machine C. Anemometer
41. The maximum unit pressure of turbine and generator on load plus how many percent of the impact, condenser load, D. Fadometer
the reinforced concrete should not exceed floor loads and dead loads?
A. 17.62 kg/𝑐𝑚2 A. 25 Answer: B
B. 18.62 kg/𝑐𝑚2 B. 34
C. 16.62 kg/𝑐𝑚2 C. 15 2. An instrument for testing the resistance of materials to
D. 19.62 kg/𝑐𝑚2 D. 36 fading when they are exposed to artificial sunlight or
ultraviolet light under controlled conditions.
Answer: A Answer: A A. Fadometer
B. Fulgurator
42. Foundations should be isolated from floor slabs or 47. In guyed steel stacks, the angel between wires in a set of C. Odometer
building footings by at least how many mm to eliminate three is: D. Atmometer
transmission of vibration. A. 120 deg.
A. 25 mm B. 110 deg. Answer: A
B. 15 mm C. 135 deg.
C. 35 mm D. 145 deg. 3. The type of calorimeter in which the specific heat of a
D. 45 mm specimen is measured by the quantity of ice it melts is the:
Answer: A A. ice calorimeter
Answer: A B. heat calorimeter
48. Guys are usually applied in how many sets? C. vapor calorimeter
43. Concrete foundations should have steel bar A. 1 to 3 D. hysometer
reinforcements placed both vertically and horizontally to B. 2 to 4
avoid C. 3 to 5 Answer: A
A. vibrations D. 4 to 6
B. thermal cracking 4. An instrument used for determination of the boiling point
C. breakdown Answer: A of water for estimating the altitude above sea level from its
D. rupture known variation with atmospheric pressure.
49. The empirical coefficient e in machine foundation if not A. Vapor calorimeter
Answer: B given is assumed B. Hysometer
A. 0.11 C. Boiler
44. Foundation bolts of specified size should be used and B. 0.25 D. Vapor bomb
surrounded by a pipe sleeve with an inside diameter of at C. 0.32
least how many times the diameter of the anchor bolt? D. 0.15 Answer: B
A. 3
B. 4 Answer: A 5. A unit of work or energy equal to 107 ergs. It is equivalent
C. 5 to the work done by a force of one Newton acting through a
D. 6 50. The term suction units is used with distance of one meter.
A. Natural draft A. Btu
Answer: A B. Forced draft B. Calorie
C. Induced draft C. Joule
D. Balanced draft D. Watt
C. Gas mixer
Answer: C D. Aerator 17. An instrument used for measuring various quantities at a
distance.
6. An electromagnetic device for separating isotopes by Answer: D A. Tachometer
electrical sorting of their ions. B. Odometer
A. Ion sorter 12. A device that supplies air to an air-injection system is C. DME
B. Isotopes sorter called D. Telemeter
C. Isotron A. air pump
D. Jolly balance B. air nozzle Answer: D
C. gas scrubber
Answer: C D. air injector 18. An apparatus used for determination of the transition
point by measuring the temperature at which the vapor
7. A device used for the determination of the specific gravity Answer: B pressure of the two modifications become equal.
of a substance by weighing it in air and in water. A. Tensiometer
A. Kelvin balance 13. An instrument used for measuring pressure, flow velocity B. Tensimeter
B. Spring balance and discharge rate of a fluid flowing in a conduit. C. Tachometer
C. Hysometer A. Kampometer D. Bourdon gage
D. Jolly balance B. Flowmeter
C. Pitot tube Answer: B
Answer: D D. Katharometer
19. An instrument for measuring the revolutions per minute
8. Which of the following instruments is NOT used to Answer: B of a rotating shaft.
measure flow rates? A. Tachometer
A. Anemometer 14. An instrument used for measuring radiant energy B. Odometer
B. Rotameter especially in the thermal region. C. All of the choices
C. Flowmeter A. Kampometer D. Speedometer
D. Velometer B. Luxmeter
C. Kapnometer Answer: A
Answer: D D. Katharometer
20. A device used for demonstrating the extreme force
9. An instrument used to measure the quality of steam is the: Answer: A exerted by water when if freezes is called:
A. psychometer A. Ice calorimeter
B. gas calorimeter 15. An instrument for determination of the composition of the B. Hysometer
C. pyrometer gas mixture by measuring variations in the thermal C. Ice bomb
D. steam calorimeter conductivity. D. Freezer
A. Kampometer
Answer: D B. Atmometer Answer: C
C. Kapnometer
10. An instrument used to measure the density of gases is D. Katharometer 21. A device to measure vertical distances or
known as: displacements?
A. aerovane Answer: D A. Kampometer
B. aerometer B. Kapnometer
C. hygrometer 16. An apparatus used for measuring the surface tension of C. Altimeter
D. atmometer a liquid by recording the force necessary to detach a metal D. Cathetometer
ring from the surface.
Answer: A A. Tachometer Answer: D
B. Tensiometer
11. Which of the following is the apparatus used for mixing C. Odometer 22. A precision instrument for measuring very short time
air with other fluids? D. Telemeter intervals.
A. Carburetor A. Clinometer
B. Dehydrator Answer: B B. Chronoscope
C. Stop watch What is the clockwork operated device which records
D. Decremeter continuously the humidity of the atmosphere? 33. Past ME Board Question
A. Hetrograph What is the function of a steam separator?
Answer: B B. Hygrometer A. Trapping the steam and letting water through
C. Hygrodeik B. Throttling
23. Which of the following is a laboratory method for the D. Hygrograph C. Changing the direction of the steam flow
determination of the boiling range of petroleum? D. Steam metering
A. Jolly balance Answer: D
B. Kelvin balance Answer: A
C. Engler distillation 29. Past ME Board Question
D. Fractional distillation A device whose function is to pass an information in an 34. Past ME Board Question
unchanged form or in some modified form. A salimeter reads the:
Answer: C A. Relay A. Density of brine
B. Sensor B. Dew point temperature
24. An instrument used to measure the humidity in the air. C. Transmitter C. Rate of brine
A. Hygrometer D. Transducer D. Relative humidity
B. Hydrometer
C. Pressure gauge Answer: A Answer: A
D. Wind vane
30. Past ME Board Question 35. Past ME Board Question
Answer: A A general term for a device that receives information in the Which of following measures the density of salt in the water?
form of one or more physical quantities, modifies the A. Salimeter
25. An instrument used to measure the specific gravity or information and or its form if required and produces a B. Pitot tube
relative density of liquids. resultant output signal. C. Hydrometer
A. Hygrometer A. Converter D. Odometer
B. Hydrometer B. Transducer
C. U – tube C. Sensor Answer: A
D. Manometer D. Scanner

Answer: B Answer: B CHAPTER 18 – BASIC ELECTRICAL ENGINEERING


1. Ebb current refers to:
26. An apparatus used for the determination of the 31. Past ME Board Question A. the eddy current
concentration of solid or liquid particles dispersed in a gas, An engine indicator is generally used to measure what? B. the movement of the tidal current away from shore or
such as the density of smoke in fog. A. Steam temperature down a tidal stream
A. Kampometer B. Heat loss C. the removal by screen of undesirable fine materials from
B. Atmometer C. Steam cylinder pressure broken ore
C. Kapnometer D. Gauge reading errors D. none of the above
D. Katharometer
Answer: C Answer: D
Answer: C
32. Past ME Board Question 2. In power station practice “spinning reverse” is
27. Past ME Board Question In the processing section, there is an instrument frequently A. reverse generating capacity that is in operation but not in
Hydrometer is used to find out: used to measure the flow rate of fluids. What is the service
A. Specific gravity of liquids instrument consisting of a vertical passage with variable B. reserve generating capacity that is connected to bus and
B. Specific gravity of solids cross-sectional area, a float and a calibrated scale? ready to take load
C. Specific gravity of gases A. Rotameter C. reserve generating capacity that is available for service
D. Relative humidity B. Pitot tube but not in operation
C. Rota Aire D. capacity of the part of plant remains under maintenance
Answer: A D. Manometer
Answer: B
28. Past ME Board Question Answer: A
3. Turnaround efficiency of battery energy storage system is 15. Which of the following plant has least pollution problem?
about 9. The string efficiency in an AC system is: A. Steam power plant
A. 75 percent A. 50.75% B. Gas turbine power plant
B. 40 percent B. 100% C. Nuclear power plant
C. 25 percent C. 25.50% D. Hydro-electric power plant
D. 80 percent D. 70.7%
Answer: D
Answer: A Answer: D
16. Which of the following power plant can be installed within
4. Most of the generators in thermal power plants run at: 10. Satellites are powered by the shortest time?
A. 15000 rpm A. solar cells A. Nuclear power plant
B. 1500 rpm B. thermo-electric generators B. Hydro-electric power plant
C. 750 rpm C. thermionic converters C. Gas turbine power plant
D. 3000 rpm D. photo-electric cells D. Diesel engine plant

Answer: B Answer: A Answer: D

5. The power supplied to domestic consumers is at 220 11. A ship is powered by: 17. Which of the following item consumes least power?
volts. This represents: A. steam turbine A. Toaster
A. mean value of voltage B. diesel engines B. Desert cooler
B. seak value of voltage C. steam turbines or diesel engine C. Electric shaver
C. RMS value of voltage D. steam turbines, diesel engines, hydraulic turbines or D. Electric iron
D. none of the above nuclear reactors
Answer: C
Answer: C Answer: C
18. Which of the following needs highest level of
6. The conductors that are used for transmitting bulk of 12. Which of the following plant is suitable for peak load? illumination?
power at high voltage are of which of the following type? A. Diesel engine plant A. Proof reading
A. Cadmium copper B. Steam power plant B. Hospital wards
B. Galvanized steel C. Nuclear power plat C. Railway platforms
C. Any of the above D. All of the above D. Foyer for cinema halls
D. Copper
Answer: A Answer: A
Answer: D
13. The term ‘critical’ is associated with power plant of the 19. The lamp that is used for cinema projectors is a:
7. When a conductor is suspended between two supports at type: A. frosted GLS lamp
the same level its shape becomes: A. steam power plant B. nitrogen filled GLS lamp
A. catenary B. diesel engine power plant C. mercury vapor lamp
B. cycloid C. gas turbine power plant D. carbon arc lamp
C. semicircle D. nuclear power plant
D. parabola Answer: D
Answer: D
Answer: A 20. In a fluorescent tube circuit, choke acts as
14. A turbocharger is a part of: A. a starter
8. Sag is provided in overhead transmission lines so that: A. thermal power plant B. a device for improving the power factor
A. repairs can be carried out easily B. diesel engine power plant C. a current limiting device
B. corona can be reduced C. hydro-electric power plant D. a source of heat
C. skin effect can be reduced D. nuclear power plant
D. safe tension is not exceeded Answer: B
Answer: B
Answer: D 21. Transient disturbances are due to:
A. switching operations 27. The statement that the product of the error in the D. 80 percent
B. load variations measured determination of a particle’s position and its
C. faults momentum is of the order Planck’s constant “h” is known as Answer: A
D. any of the above A. Bohr’s theory
B. D’Alembert’s paradox 33. The conductors that are used for transmitting bulk of
Answer: D C. the Heisenberg uncertainty principle power at high voltage are of which of the following type?
D. Planck’s law A. Cadmium copper
22. Bottoming cycle is quite common in: B. Galvanized steel
A. cement plants Answer: C C. Any of the above
B. sugar mills D. Copper
C. paper mills 28. The phenomenon of physical adhesion of molecules of
D. all of the above the surfaces of solids without chemical reaction is Answer: D
A. dredging
Answer: A B. adsorption 34. Corona generally results in
C. coking A. violet glow
23. Which of the following is the most highly developed D. liquation B. hissing noise
device for confining plasma with magnetic field? C. production of ozone gas
A. Tokamak Answer: B D. all of the above
B. Tomahawk
C. Breeder reactor 29. Fuses and circuit breakers do not protect electric motors Answer: D
D. Cyclotron from:
A. short circuiting 35. Which of the following affects corona?
Answer: A B. motor burnout A. Conductor size
C. overload B. Spacing between conductors
24. Fuses and circuit breakers do not protect electric motors D. overheating C. Line voltage
from: D. All of the above
A. short circuiting Answer: D
B. motor burnout Answer: D
C. overload 30. In which of the following systems where Betz law is
D. overheating widely used? 36. When a conductor is suspended between two supports
A. MHD system at the same level its shape becomes
Answer: D B. solar cells A. catenary
C. geothermal power plants B. cycloid
25. Corona generally results in: D. wind mills C. semicircle
A. violet glow D. parabola
B. hissing noise Answer: D
C. production of ozone gas Answer: A
D. all of the above 31. At what rpm where most of the generators in thermal
power plants run? 37. Past ME Board Question
Answer: D A. 15000 rpm What is the unit of electromagnetic wave frequency?
B. 1500 rpm A. Volts
26. Which of the following affects corona? C. 750 rpm B. Horsepower
A. Conductor size D. 3000 rpm C. Hertz
B. Spacing between conductors D. Knot
C. Line voltage Answer: B
D. All of the above Answer: C
32. Turnaround efficiency of battery energy storage system
Answer: D is about CHAPTER 19 – LATEST BOARD QUESTION
A. 75 percent 1. Past ME Board Question
B. 40 percent
C. 25 percent
A pump with capacity 𝑄1 and head 𝐻1 is connected in series Where does the final removal of water vapor in an 11. Past ME Board Question
with another pump with capacity 𝑄2 and head 𝐻2 . What is the absorption refrigeration system occur? A multi-stage refrigeration system which is realizable in
head developed if 𝑄2 is less than 𝑄1 ? A. Analyzer using different refrigerants
A. 2𝐻2 𝐻1 B. Generator A. Direct
B. 𝐻2 + 𝐻1 C. Condenser B. Cascade
C. 𝐻2 / 𝐻1 D. Rectifier C. Flooded
D. 𝐻2 𝐻1 D. Vapor
Answer: D
Answer: B Answer: B
7. Past ME Board Question
2. Past ME Board Question If the average temperature of liquid of power cycle during 12. Past ME Board Question
At ordinary temperature what substance will behave as inert heat addition process is as high as possible, then the Very large pipe diameter means a thicker wall and high cost.
gas and will not react in the combustion process? thermal efficiency of the cycle will: It also means
A. Nitrogen A. Increase A. Zero pressure drop
B. Sulfur B. Remain constant B. High pressure drop
C. Hydrogen C. Decrease C. Constant temperature
D. Carbon D. Zero D. Low pressure drop

Answer: A Answer: A Answer: B

3. Past ME Board Question 8. Past ME Board Question 13. Past ME Board Question
Which of the following cycles is ideal for spark-ignition What is the temperature of lubricant at which it will ignite Aside from maintaining appropriate temperature for food cold
reciprocating engine? thus limits its application? storage, how is desiccation minimized or decreased?
A. Diesel cycle A. Cold point A. Low oxygen
B. Dual cycle B. Burning point B. Maintain humidity ratio
C. Rankine cycle C. Flash point C. Low air circulation
D. Otto cycle D. Use point D. Increase humidity ratio

Answer: D Answer: C Answer: B

4. Past ME Board Question 9. Past ME Board Question 14. Past ME Board Question
What is the relative humidity when the dew point and dry What is the heat that is removed from the space to be During combustion process, only limited air is supplied and
bulb temperature are equal? cooled, which is the same as the heat absorbed by the not enough to supply two molecule of oxygen per one
A. 110% cooling coils? molecule of carbon, then the product is:
B. 0% A. Heating capacity A. Hydration
C. 100% B. Enthalpy B. Carbon monoxide
D. 50% C. Work compression C. Carbonic acid
D. Refrigerating effect D. Carbon dioxide
Answer: C
Answer: D Answer: B
5. Past ME Board Question
Reheating process is normally employed in steam plane 10. Past ME Board Question 15. Past ME Board Question
when: An air conditioning system in which water is chilled or cooled As far as combustion chamber design is considered, the
A. There is excess steam and which passes the evaporator coils. maximum power output of a given engine can be increased
B. Subcooling is required A. Chilled water system by:
C. Turbine undergoes excessive moisture B. Direct expansion A. Increasing combustion time
D. Dehumidifying is required C. Absorption B. Increasing combustion chamber volume
D. Water circulation system C. Decreasing combustion chamber volume
Answer: C D. Providing small values
Answer: A
6. Past ME Board Question Answer: C
A. Total energy of an open system If Pi is the indicated horsepower and Pb is the indicated
16. Past ME Board Question B. Temperature of an open system horsepower of a compressor, then what is mechanical
A water conservation device which employs both air and C. Total energy of a closed system efficiency, Em, equal to:
water is condensed a condenser and cooling tower D. Temperature of a closed system A. Em = Pb / Pi
combined into one is called: B. Em = Pi / Pb
A. Shell and tube condenser Answer: D C. Em = Pb – Pi
B. Air-cooled condenser D. Em = Pi – Pb
C. Water-cooled condenser 21. Past ME Board Question
D. Evaporative condenser In order to attain equilibrium in the condenser, the cooling Answer: B
tower range must be _______ with respect to temperature
Answer: D rise in the condenser. 26. Past ME Board Question
A. Greater What is the least number of compressors a multistage
17. Past ME Board Question B. Equal system that will use?
In order to perform efficiently, a power cycle must be C. Less A. Three
communicated with the outside temperature of its D. Zero B. Two
surroundings and into one is called: C. Four
A. Kirchoff’s Law Answer: B D. One
B. Second Law of Thermodynamics
C. Kelvin-Planck Law 22. Past ME Board Question Answer: B
D. Frist Law of Thermodynamics If ice will form in a solution of water and salt, then it is at a
temperature called: 27. Past ME Board Question
Answer: B A. Dew point The relation between the Fahrenheit absolute scale and the
B. Freezing point depression Celsius absolute scale is:
18. Past ME Board Question C. Boiling point depression A. R = 1.8 K
Water is heated in a container. It expands and becomes less D. Critical point B. R = 32 K
dense and lighter. It rises up the container because of its C. R = 3.2 K
reduced density and replaced by cooler air. As this process Answer: B D. R = 18 K
continues, the heat is transferred and disturbed all
throughout. What is this mode of heat transfer called? 23. Past ME Board Question Answer: A
A. Condenser The flow of the convergent section of a nozzle is always
B. Radiation subsonic. If the flow is subsonic then the mach number is: 28. Past ME Board Question
C. Conduction A. Greater than unity The sum of the internal energy and the product of pressure
D. Convection B. Less than unity and specific volume is known as the:
C. Near than unity A. Enthalpy
Answer: D D. Unity B. Entropy
C. Total work
19. Past ME Board Question Answer: B D. Total internal energy
When required, a regulator water valve in refrigerating
system should be 24. Past ME Board Question Answer: A
A. On the suction line on compressor line One hundred twenty percent theoretical air was supplied for
B. In the water inlet combustion. What is the equivalent excess equal to: 29. Past ME Board Question
C. Anywhere in the system A. 20% A refrigeration control that guards the compressor from
D. In the water outlet B. 100% overloads brought about by abruptly increases loads
C. 240% resulting from defrosting, warm products and others, is
Answer: D D. 120% called:
A. Safety valve
Answer: A B. Suction hold-back valve
20. Past ME Board Question C. Solenoid valve
When there is no work between the thermodynamic system 25. Past ME Board Question D. Expansion valve
and its surrounding, the quantity of net heat transfer is equal
to: Answer: B
Answer: C A. 5”
30. Past ME Board Question B. 6”
What does fossil-fuel fired power plants release, which in 35. Past ME Board Question C. 3”
turn produces the key ingredient in acid rain? The kinetic energy of a moving fluid is used to isentropically D. 4”
A. Nitrogen compressed the fluid to state of zero velocity. The
B. Sulfur emission temperature of a moving fluid at the state zero velocity is Answer: D
C. Carbon monoxide called:
D. Carbon dioxide A. Stagnation temperature 40. Past ME Board Question
B. Partial temperature One foot water is equal to:
Answer: B C. Critical temperature A. 0.4138 lb/𝑖𝑛2
D. Absolute temperature B. 68.3 lb/𝑓𝑡 2
31. Past ME Board Question C. 0.8673 lb/𝑖𝑛2
The fraction of the radiation energy incident on a surface Answer: A D. 62.43 lb/𝑓𝑡 2
which is absorbed by the surface is called:
A. Convection 36. Past ME Board Question Answer: D
B. Absorptivity The dew point temperature of the products of combustion in
C. Emission the saturation temperature that corresponds to the partial 41. Past ME Board Question
D. Radiation pressure of the _____ in the products. The freezing point of R – 22 is:
A. 𝑆𝑂2 A. -109 ℉
Answer: B B. 𝐻2 𝑂 B. -252 ℉
C. 𝑁2 C. -256 ℉
32. Past ME Board Question D. 𝐶𝑂2 D. -211 ℉
Which of the following power plants uses energy from
uranium to produce electric power? Answer: B Answer: C
A. Diesel Plant
B. Geothermal Plant 37. Past ME Board Question 42. Past ME Board Question
C. Nuclear Plant What do we call a device that is used to boiler operation that Which of the following is a type of evaporator?
D. Hydroelectric Plant will stop the burner or at least send a signal to the operator if A. Shell-and-tube condenser
the water drops to a low level that is no longer safe? B. Shell-and-tube water cooler
Answer: C A. Safety valve C. Oil lantern rings
B. Dead-weight valve D. Polyphase motors
33. Past ME Board Question C. High water cut off
Ice cubes added to a glass of water and stirred. Moisture D. Low water cut off Answer: A
starts to form on the outer surface of the glass. At this point,
what is the temperature at the outer surface called? Answer: D 43. Past ME Board Question
A. Critical temperature For Brayton cycle, the result of regeneration is:
B. Surface temperature 38. Past ME Board Question A. Increase in thermal efficiency
C. Dew point temperature Dry analysis is a fractional analysis of the products of B. Decrease thermal efficiency
D. Saturation temperature combustion which does not include: C. Moderate thermal efficiency
A. Water vapor D. Low thermal efficiency
Answer: C B. Carbon dioxide
C. Sulfur dioxide Answer: A
34. Past ME Board Question D. Carbon monoxide
The force when applied to a mass of one kilogram will give 44. Past ME Board Question
mass an acceleration of one meter per second for every Answer: A If the pressure exerted on a liquid is higher than the
second called: saturation corresponding to its temperature. The liquid is a:
A. Watt A. Saturated liquid
B. Joule B. Subcooled liquid
C. Newton 39. Past ME Board Question C. Superheated liquid
D. Pascal The size of a reciprocating pump is stamped on the builder’s D. Highly superheated liquid
plate 3” x 4” x 6”. The diameter of the liquid cylinder is:
Answer: B Answer: C
50. Past ME Board Question
45. Past ME Board Question What does a negative Joule-Thompson coefficient means 55. Past ME Board Question
In a power driven pump, each piston stroke is displaced by during a throttling process? What is the effect on saturated temperature if the pressure of
360𝑜 divided the ______. A. Fluid pressure is constant the fluid is decreased?
A. Revolution per minute B. Fluid temperature drops A. There is no effect
B. Bore C. Fluid temperature rises B. Saturation temperature decreases
C. Length of the stroke D. Fluid pressure zero C. Saturation temperature remains constant
D. Number of cylinders D. Saturation temperature increases
Answer: C
Answer: D Answer: B
51. Past ME Board Question
46. Past ME Board Question For steam power plants, increasing the operating pressure of 56. Past ME Board Question
If the air is compressed without discarding heat, then what the boiler will: If V3 is the cylinder volume after the combustion process of
do you call this kind of compression? A. Increase thermal efficiency a Diesel cycle and V2 is its cylinder volume before
A. Isobaric B. Decrease thermal efficiency combustion, then calculate the cut-off ratio, r.
B. Isochoric C. Bring thermal efficiency to zero A. r = V3 – V2
C. Adiabatic D. Make thermal efficiency constant B. r = V2 – V3
D. Isothermal C. r = V2 / V3
Answer: A D. r = V3 / V2
Answer: C
47. Past ME Board Question 52. Past ME Board Question Answer: D
If a gas possesses internal energy, then it is due to its: The ratio of the average load to that of the rated capacity of
A. Velocity a plant is called: 57. Past ME Board Question
B. Pressure and volume values A. Output factor Compare the temperature of discharge vapor refrigerant
C. Height from a certain datum level B. Load factor leaving the compressor for a superheated cycle and
D. Molecular motion C. Demand factor saturated cycle, for the same condensing temperature and
D. Capacity factor pressure.
Answer: D A. There is no difference
Answer: D B. Higher for the superheated cycle
48. Past ME Board Question C. Higher for the saturated cycle
The pressure of ammonia was detected in brine solution. 53. Past ME Board Question D. Lower for the superheated cycle
How can ammonia be removed? What is the method used to evaluate all welds performed on
A. Freeze the brine solution pressure parts of boiler tube materials? Answer: B
B. Heat the brine to a high temperature enough to A. Hydrostatic test
free the ammonia B. Orsat analysis 58. Past ME Board Question
C. Throw the brine solution C. Vacuum test A valve that releases steam from the boiler by opening at a
D. Low the temperature of the brine D. Radiographic test pre-determined pressure in order to keep the steam
pressure from rising more than the pressure from which the
Answer: B Answer: D valve is set is called a
A. Ammonia valve
49. Past ME Board Question 54. Past ME Board Question B. Stop valve
What will happen when the pressure at any point inside a If the pressure is disregarded in the various other C. Check valve
centrifugal pump goes below the vapor pressure components of a steam of gas power plants, the pressure D. Safety valve
corresponding to the temperature of the liquid? rise in the pump or compressor is ____ to the pressure drop
A. Turbulent flow in the turbine. Answer: D
B. Laminar flow A. Inversely proportional
C. Cavitation B. Constant 59. Past ME Board Question
D. Priming C. Equal What is the relationship of the capacity of a centrifugal
D. Varying pump, Q, to impeller diameter, D, when there are two
Answer: C impeller diameters in the same pump?
A. Q is directly proportional to the square of the ratio 64. Past ME Board Question C. Et – Er
of D If Hs is the total suction head of a pump, Hp is its suction D. Er / Et
B. Q is inversely proportional to the ratio of D surface pressure, and Hf is its suction friction head, then
C. Q. is directly proportional to the ratio of D calculate the total suction head, Ht. Answer: B
D. Q is inversely proportional to the square of the A. Ht = Hs – Hp – Hf
ratio of D B. Ht = Hs + Hp – Hf 69. Past ME Board Question
C. Ht = Hs + Hp + Hf If W is the work output of a heat engine and H is the total
Answer: C D. Ht = Hs – Hp + Hf heat input of an engine, then determine its thermal
efficiency, e.
60. Past ME Board Question Answer: C A. e = H/ W
If Wt is the turbine shaft work of a gas turbine unit, Wc is its B. e = W / H
compressor shaft work and Q is the heat supplied in fuel, 65. Past ME Board Question C. e = W x H
then determine its thermal efficiency. The constant spending of certain percentage of circulated D. e = l – ( W – H )
A. Q / (Wt – Wc) water in a cooling tower in order to prevent accumulation of
B. Q / (Wt + Wc) dissolved mineral solids and other impurities in the Answer: B
C. (Wt + Wc) / Q condenser water is called
D. (Wt – Wc) / Q A. Approach 70. Past ME Board Question
B. Drift Which of the following is converted to mechanical energy by
Answer: D C. Range a water.
D. Bleed-off A. Internal energy
61. Past ME Board Question B. Kinetic energy
The schedule number of a pipe, N, indicates the thickness of Answer: D C. Potential energy
the pipe wall. If the allowable stress of the pipe is S, then D. Hydraulic energy
what is the internal pressure equal to? 66. Past ME Board Question
A. N – S What process is employed when the turbine steam power Answer: C
B. N / S plants experience excessive moisture?
C. N x S A. Reheating 71. Past ME Board Question
D. S / N B. Supercooling What must be the value of the available Net Positive Suction
C. Subcooling Head (NPSH) of a centrifugal pump compared to its require
Answer: C D. Freezing NPSH to avoid losing priming?
A. Available NPSH lesser than required NPSH
62. Past ME Board Question Answer: A B. Available NPSH equal than required NPSH
What is the pressure present inside the casing of an impulse C. Available NPSH greater than required NPSH
turbine? 67. Past ME Board Question D. Available NPSH constant at all times
A. Vacuum A company is interested to produce a water turbine wherein
B. Atmospheric pressure only little energy is required or necessary because the guide Answer: C
C. Above atmospheric vanes are to be controlled. The turbine must be a:
D. Zero A. Gas turbine 72. Past ME Board Question
B. Propeller turbine What type of fuel is used with stationary power plants where
Answer: B C. Kaplan turbine gas is available by pipeline?
D. Francis turbine A. Gaseous fuels
63. Past ME Board Question B. Solid fuels
In order to remove the fly ashes from the flue gas, which of Answer: C C. Coal
the following must a power plant equipped with? D. Liquid fuels
A. Condenser 68. Past ME Board Question
B. Demineralizer If Et is the total net energy generated by a plant in a certain Answer: A
C. Electrostatic precipitator period of time and Er is the rated net energy capacity of the
D. Desulphurization plant plant during the same period of time, then what is the plant 73. Past ME Board Question
operating factor? Wb is shaft work of an engine and Wi is the indicated work of
Answer: C A. Er – Et an engine. If mechanical friction is present in the engine
B. Et / Er mechanism, then:
A. Wb is equal to Wi It is important to take some moisture from the air to If Ev is the volumetric efficiency of a reciprocating pump,
B. Wb is less than Wi dehumidify it if the relative humidity reaches high levels. To then determine SLIP which represents all losses of capacity
C. Wb is proportional to Wi do this, it requires cooling the air: given in percentage of the displacement.
D. Wb is greater than Wi A. At wet-bulb temperature A. 1 / Ev
B. Above its dew point temperature B. 1 – 1 / Ev
Answer: B C. At critical temperature C. 1 – Ev
D. Below its dew point temperature D. Ev + 1
74. Past ME Board Question
What is the saturation pressure at the critical temperature, or Answer: D Answer: C
the lowest pressure in which a substance can be in the liquid
state at its critical pressure? 79. Past ME Board Question 84. Past ME Board Question
A. External pressure What does enthalpy measure in a substance? What is the relationship of the horsepower of a centrifugal
B. Critical pressure A. Its coldness pump, Hp, to the impeller speed, S, if the pump is at two
C. Condensing pressure B. Its heat content different rotative speeds?
D. Evaporation pressure C. Its humidity A. Hp is inversely proportional to the cube of the ratio
D. Its dew point of S
Answer: B B. Hp is directly proportional to the ratio of S
Answer: B C. Hp is inversely proportional to the ratio of S
75. Past ME Board Question D. Hp is directly proportional to the cube of the ratio
What is the significance of a high dielectric strength of 80. Past ME Board Question of S
lubricating oil? Pneumatic tools are powered by:
A. Oil is thick A. Natural gas Answer: D
B. Oil is contaminant – free B. Air
C. Oil is thin C. Electricity 85. Past ME Board Question
D. Oil is not clean D. Steam What is likely to occur when sections of the impeller of a
centrifugal pump are handling vapor and other sections are
Answer: B Answer: B handling liquid?
A. Erosion of the pump
76. Past ME Board Question 81. Past ME Board Question B. High head and low capacity
Theoretically, what is the effect of the compressor clearance The pump will not cavitate if the available Net Positive C. Excessive vibration
on horsepower? Suction Head (NPSH) is: D. Complete failure to operate
A. Varies in direct proportion A. Less than the required NPSH
B. Increase power B. Equal or lesser than the required NPSH Answer: D
C. Decrease power C. Equal or greater than the required NPSH
D. There is no effect D. Zero compared to the required NPSH of 1 86. Past ME Board Question
When vapor compression takes place on one side of the
Answer: D Answer: C piston and one during one revolution of the crankshaft, then
the compressor is:
77. Past ME Board Question 82. Past ME Board Question A. Double-acting
Compare the refrigerating effect per unit mass of refrigerant A water-tube condenser has a total of 60 tubes. If these are B. Four-cycle
circulated for a superheated cycle that produces useful two passes, then compute the number of tubes per pass. C. Two-revolution
cooling and a saturated cycle, for the same vaporizing and A. 6 C D. Single-acting
condensing temperature. B. 15
A. Lower for a superheated cycle C. 3 C Answer: D
B. Greater for a saturated cycle D. 2
C. The same
D. Greater for a superheated cycle Answer: B
87. Past ME Board Question
Answer: D 83. Past ME Board Question In Stirling and Erickson Cycles, the efficiency can be
increased by:
78. Past ME Board Question A. Compressibility
B. Superheating compare the efficiency of a Rankine cycle to that of the Answer: C
C. Regeneration Carnot cycle?
D. Subcooling A. Efficiency is the same 97. Past ME Board Question
B. Higher efficiency The ratio of the average load during a certain time to peak
Answer: C C. Efficiency is higher by 30% the load occurring during the same period of time is called:
D. Lower efficiency A. Demand factor
88. Past ME Board Question B. Capacity factor
Which of the following is used to control refrigeration Answer: D C. Load factor
compressor capacity? D. Output factor
A. Dehumidifier 93. Past ME Board Question
B. Check valve Compute the wall gain load of a load storage room when A is Answer: C
C. Solenoid valve the outside surface area of the wall, D is the temperature
D. Unloader differential across the wall, and U is the overall coefficient of 98. Past ME Board Question
heat transmission. Based on the PSME Code, what should be the effective
Answer: D A. AU / D temperature of the air to be maintained for comfort cooling at
B. D / AU an air movement from 0.0761 to 0.127 meter per second?
89. Past ME Board Question C. A / UD A. 25 – 29 degrees C
During a cooling and dehumidifying process, sensible and D. AUD B. 35 – 39 degrees C
latent heats are removed in the cooling coil. If Hs is the C. 20 – 24 degrees C
sensible heat and Ht is the total heat transferred, then Answer: D D. 30 – 34 degrees C
determine the coil sensible heat factor.
A. Ht – Hs 94. Past ME Board Question Answer: C
B. Hs – Ht If m is the mass of dry air and H is the specific enthalpy of
C. Ht / Hs the water vapor in air and r is the humidity ratio, then 99. Past ME Board Question
D. Hs / Ht determine the latent heat of any dry air. If R is the delivery rate of a pump, H is the total pumping
A. m / (r – H) head and e is the efficiency of the pump, then compute the
Answer: D B. r / (H – m) power required to drive the pump.
C. (m – r) / H A. R / H x e
90. Past ME Board Question D. m (r x H) B. 1 – (R + H) / e
Based on the PSME Code, what should be provided in each C. (R x H) / e
stream outlets if two or more boilers will be connected in Answer: D D. (H – R) / e
parallel?
A. Relief and check valves 95. Past ME Board Question Answer: C
B. Non-return and shut-off valves When an electric motor nameplate indicates a “100-kw
C. Expansion and shut-off valves electric motor” then what does a 100-kw rating refers to? 100. Past ME Board Question
D. Expansion and check valves A. Thermal energy input There are three basic boiler types, namely:
B. Thermal energy output A. Water-tube, horizontal tube and cast-iron boilers
Answer: B C. Electrical power output B. Cast-iron, fire-tube and water tube boilers
D. Mechanical power output C. Fire-tube, cast-iron and water tube boilers
91. Past ME Board Question D. HRT, fire-tubes and Scotch Marine boilers
The diesel cycle is the ideal cycle for a : Answer: D
A. Absorption Answer: B
B. Steam-jet 96. Past ME Board Question
C. Compression-ignition engine For a hydraulic power to be transmitted at its maximum what 101. Past ME Board Question
D. Gas turbine should be the value of loss of head due to friction if H is the The amount of heat necessary to bring up the temperature of
total available head? a unit mass of a substance through a unit degree is called:
Answer: C A. 11H A. Specific volume
B. 1/3 H B. Thermal heat
92. Past ME Board Question C. 1/4 H C. Total heat
If a Carnot cycle is at the same maximum and minimum D. 5H D. Specific heat
temperatures as a Rankine cycle, then how would you
Answer: D

102. Past ME Board Question


What is the specific gravity of an oil which has a Baume
reading g of 28 degrees F?
A. 0.215
B. 0.562
C. 0.886
D. 0.774

Answer: C

103. Past ME Board Question


If C is the capacity of the compressor in a refrigeration
system and F is the heat rejection factor, then calculate the
condenser load, L.
A. L = F – C
B. L = F / C
C. L = C x F
D. L = C / F

Answer: C

104. Past ME Board Question


Based on the PSME Code, what should be the humidity ratio
of air to be used for comfort cooling?
A. 75% - 80% relative humidity
B. 50% - 60% relative humidity
C. 60% - 70% relative humidity
D. 70% - 75% relative humidity

Answer: B

105. Past ME Board Question


What is the cause of pressure drop in the boiler, condenser
and the pipings between different components? Because of
this drop, steam leaving the boiler is at lower pressure.
A. Positive slip
B. Fluid friction
C. Negative slip
D. Low thermal efficiency

Answer: B
IPE PREVIOUS BOARD PROBLEMS

AIR COMPRESSOR

ME BOARD (April 1995)


1. A single air compressor handles 0.454 m 3 / sec of
atmospheric pressure , 27  C air, and delivers it to a receiver at
652.75 kPa . Its volumetric efficiency is 0.72, its compression
efficiency on an isothermal basis is 0.85 and its mechanical
efficiency is 0.90. If it rotates at 350 rpm, what power in kw is
required to drive it?

SOLUTION:

Isothermal power = P1V1 ln P2 / P1

= 101.3 (0.454) ln (625.75 / 101.4)

= 85.685 kw

drive power = 85.685


0.85(0.9)

=112 kw

ME Board( October 1992)


2. An air compressor takes air at 100 Kpa and discharges to
600 Kpa. If the volume flow of discharge is 1.2 m3 per
seconds, determine the capacity of air compressor.

SOLUTION:
P1V1n = P2 V2 n
N = 1.4 (for standard air)

100(V1)1.4 = (600) (1.2) 1.4


V1 = 4.315 m3 m3/sec
ME BOARD (April 1995)
3. An ammonia compressor has a clearance of 5% and a
pressure ratio of 5. Determine the volumetric efficiency if k for
ammonia is 1.304.

SOLUTION:

nv = 1 + c – c ( P2 / P1 ) 1/k

= 1 + 0.5 – 0.5 ( 5 ) 1/ 1.304

= 0.8782

= 87.82 %

ME Board (April 1987)


4. The piston displacement of a double acting air
compressor is 0.358
m3/sec, delivers gas from 101.325 Kpa and 300°K to 675 Kpa at
the rate of 0.166 m3/sec at 150 rpm. Value of n for compression
and expansion is 1.33. Find the compressor percent clearance.

Solution:

ηv = V1 / VD
ηv = 0.166 / 0.358
ηv = 0.4637

ηv = 1 + c – c ( P2 / P1 ) 1/n
0.4637 = 1 + c – c ( 675 / 101.325) 1/1.33

c = 0.1696
c = 16.96 %

ME Board (April 1995)


5. A single stage air compressor handles 0.454 m 3 /sec of
atmospheric pressure, 27°C air, and delivers it to a receiver at
652.75 Kpa. Its volumetric efficiency on an isothermal basis is
0.85 and its mechanical efficiency is 0.90. If it operates at 350
rpm, what power in KW is required to drive it?

Solution:
For isothermal process.
W = P1V1 ln ( P2 / P1 )
W = 101.3 ( 0.454 ) ln(652.75 / 101.3)
W = 85.685 KW

Power Input = __85.685__


0.85 (0.90)

Power Input = 112 KW

ME BOARD (1999)
6. A vapor compression refrigeration system is designed to
have a capacity of 100 tons. Its actual COP is 5.86 and 35% if
the power supplied to the compressor is lost in the form of
friction and cylinder cooling losses.

SOLUTION:

QA = 100 TOR = 351.6 kw

COP = Q A / WC

5.86 = 351.6 / WC

WC = 60 kw

MOTOR POWER = __60__


1-0.35

=92.3 kw

ME Board (October 1992)


7. A 10 hp motor is used to driven an air compressor. The
compressor efficiency is 75%. Determine the compressor
work.

Solution:
ηc = W / Brake Power
0.75 = W / 10 hp x O.746

W = 5.59 KW
ME BOARD (April 1997)
8. An ideal single stage air compressor without clearance
takes in air at 100 kpa with a temperature of 16  C and
delivered it at 413 kpa after isentropic compression. What is
the discharge work done by the compressor in kJ/kg?

A . –59.22 C. –72.5
B. –118.44 D. –145

SOLUTION:

W = -kmRT1 P2 k-1
k -1 k -1
P1

W = -kmRT1 P2 k-1
m k -1 k -1
P1

W = -145 kJ/kg
m

ME BOARD (April 1995)


9. A single stage air compressor handles 0.44 m3/s of
atmospheric pressure, 27 C air, and delivers it to receiver at
652.75 kpa. Its volumetric efficiency is 0.72, its compression
efficiency is 0.90. if it rotates at 350 rpm, what power in kW is
required to drive it?

A. 95 C. 120
B. 112 D.
100

SOLUTION:

Drive Power = Isothermal Power


ec em
Where:

Isothermal Power = P1 V1 In ( P2/P1)

Isothermal Power = 101.325 (0.454) In (652.725/101.325)

Isothermal Power – 85.694 kW


ec = 0.85
em = 0.90
thus;

Drive Power = 85.694 = 112.02 kW


(0.85) (0.90)

ME BOARD (April1997)
10. A single acting air compressor operates at 150 rpm with
an initial condition of air at 97.9 kpa and 27  C and discharges
the air at 379 kpa to a cylindrical tank. The bore and stroke
are 355mm and 381 mm respectively with 5% clearance. If the
surrounding air is at 100 kpa and 20  while compression and
expansion process PV1.3 = C, determine the free air capacity
in m3/s?

A . 0.0819 m3/s C.
0.0943 m3/s
B. 0.0856 m3/s D. 0.0642
m3/s

SOLUTION:

Pf Vf = P1 V1
Tf T1

Where:

Vd =  D2 LN =  (0.355 )2 (0.381)(150/60)
4 4

= 0.0943 m3/s

ev = 1 + c – c (P2/P1) 1/n = 1 + 0.05 – 0.05 (379/97.9) 1/1.3

= 0.908

V1 = 0.908 (0.0943)
= 0.0856 m3/s

then;
100 Vf = 97.9 (0.0856)
(20+273) (27 + 273)

thus;
Vf = 0.0819 m3/s
ME BOARD (Oct. 1997)
11. Compute the volume of air drawn per minute in a single-
stage compressor that takes in 0.90 kg of air at atmospheric
pressure and 24 C and compresses it to 10 atmospheres of
pressure, if the law of compression is PV1.2 = C. use the
specific volume of air at 0 C = 0.7734 m3/kg?

A. 0.7571 m3 C. 0.5551m3
B. 0.6561 m3 D. 0.8581m3

SOLUTION:

Volume of 0.90 kg of atmospheric pressure and 24C

V = 0.7734 ( 0.90 ) ( 24 + 273 )


( 1 ) ( 0 + 273 )

V = 0.7571 m3

ME BOARD (April 1996)


12. A single stage singe acting air compressor, running at 5
rev/s, has air intake of 22.8 liters at atmospheric pressure and
25 C. if it compresses 5.7 liters in accordance with the
compression law PV1.2 = C, calculate the temperature of the
air at the end of compression?

A. 120K C. 115K
B. 110K D. 125K

SOLUTION:

T2 = ( V1 ) n-1
T1 V2

T2 = ( 22.8 ) 1.2 – 1
25 + 273 5.7

T2 = 393.21 K
Thus; the temperature at the end of compression:
= T2 – T1 = 393.21 – 298
= 393.1K
t = 120.1 C

ME BOARD (April 1991)


13. An air compressor takes air at 90 kpa and discharges to
700 kpa. If the volume flow of discharge is .3 m3/s, compute
the capacity of the air compressor.

A. 5.63 m3/s C. 5.98 m3/s


B. 4.63 m3/s D. 4.98 m3/s

SOLUTION:
P1 V1 n = P2 V2 n
90V1 1.4 = 700 (1.3)1.4

V1 = 5.63 m3/s

ME BOARD (April 1995)


14. A single stage air compressor handles 0.454 m3/sec of
atmospheric pressure, 27 C air, and delivers it to a receiver at
65275 kPa. Its volumetric efficiency is 0.72, its compression
efficiency on an isothermal basis is 0.85 and its mechanical
efficiency is 0.90. If it rotates at 350 rpm, what power in kw is
required to drive it?

a. 95 c. 120
b. 112 d. 100

SOLUTION:

Wi =isothermal power = P1V1 ln (P2/P1)


= 101.3 (0.454) ln (652.75/101.3)
= 85.685 kw

Drive Power = 85.685/0.85(0.90)


= 112. kw

ME BOARD (April 1995)


15. An air compressor is to compress 8.5 m3/min from 98.56
kPa to 985.6 kPa. Assuming conditions ideal, and with n = 1.3,
what will be the saving in work due to two staging?

a. zero c. 5.6 kw
b. 4.6 kw d. 3.5 kw
SOLUTION:
n 1
 
nP1V1 '  P2  n 
Work of Single Stage = n  1 

P

  1

 1  

 8.5 
(1.30)(98.56)   1.30  1

=  60   985. 6  1.3

  1
1.30  1  98.56  
 
= 42.43 kw
For two stage

Px = P1 P2 = (98.56)(985.6) = 311.67 kPa

n 1
 
2nP1V1 '  Px  n 
Work for Two-Stage = n  1 
 
P1 
 1

   

 8.5 
2(1.3)(98.56)   1.3 1

=  60   311 .67 
1.3
 1
1.30  1  98.56  
 

= 36.83 kw

Saving = 42.43 – 36.83 = 5.6kW


Cooling tower

1. Water at 55 ⁰C is cooled in a cooling tower which has an efficiency


of 65%. The temperature of the surrounding air is 32 ⁰C dry bulb and 70%
relative humidity. The heat Dissipated from the condenser is 2,300,000
kj/hr. Find the capacity in liters per second of the pump used in the cooling
tower.

a. 8.50 L/s c. 7.60 L/s


b. 6.80 L/s d. 6.70 L/s

SOLUTION:

Pump Capacity = m vf @ t4

Solving For M;
E = t3 – t4
t3 – twb

From: Psychometric Chart


@ 32 ⁰C & 70% RH
twb = 27.40 ⁰C

0.65 = 55 – t4 = t4 = 37.06 ⁰C
55 – 27.40
Using energy Balance in the condenser
Mcp (t3-t4) = 2,300,000
3600

m (4.187)(55-37.06) = 2,300,000
3600
m = 8.51 kg/s
From steam table : @ t4 = 37.06 ⁰C
vf = 1.0068 L/kg

Pump Capacity = (8.51kg/s)(1.0068 L/kg)


Pump Capacity = 8.57 L/s

2. An Atmospheric cooling tower is to provide cooling for the jacket


water of a four stroke, 800kw Diesel generator. The cooling tower efficiency
is 60% at a temperature approach of 10 ⁰C. If the ambient air has a relative
humidity of 70% and dry bulb temperature of 32 ⁰C, determine the cooling
water supplied to the diesel engine in L/hr. η gen = 97% useful work = 30% 7
cooling loss = 25%.

a. 39,800 L/hr c. 45,700 L/hr


b. 35,700 L/hr d. 49,800 L/hr

SOLUTION:

Vol of water = m Vf @ t4;


Solving for m & vf;
@ tdb1 = 32 ⁰C & RH = 70%
twb = 27.45 ⁰C
Tapproach = t4 – 27.45
10 = t4 – 27.45
t4 = 37.45 ⁰C

Brake Power of Engine = Power Input to Generator


800/0.97 = 824.47 Kw = Power Input to Generator

Heat Supplied to Engine, Qa


Qa = 824.74/0.30 = 2749.14 Kw

Heat Absorbed by Cooling water, Qw:


Qw = 0.25 (2749.14)
Qw = 687.285
mw cpw (t3-t4) = 687.285
mw (4.187) ( 52.4 - 36.45) = 687.285
mw = 10.98 kg/s
mw = 39,527.14 kg/hr

Specific Volume of water @ 37.45 ⁰C , Vf = 1.007L/kg

Thus, Vol of Cooling water, Vw


Vw = 39,527 (1.007)
Vw = 39.803.83 L/hr

3. A 250,000 kg/hr of water at 35 ⁰C enters a cooling tower where it is to


be cooled to be cooled to 17.5 ⁰C. The energy is to exchanged with
atmospheric air entering the units at 15 ⁰C and leaving the unit of 30 ⁰C.
The air enters at 30% RH and leaves at 85% RH. If all process are assumed
to occur at atmospheric pressure , determine the percentage of total water
flow that is make up water.

c. 2.22% c. 4.44%
d. 3.33% d. 1.11%

SOLUTION:

Make-up water = Mass of make-up


250,000
Solving for Make up Water
@ 15 ⁰C & 30% RH
h1 = 23.02 Kj/Kg
w1 = 0.0033Kg/Kg
@ 30 ⁰C & 85% RH
h2 = 89.01 Kj/Kg
w2 = 0.0233 Kg/Kg

Heat Lost By Water = Heat Gain By Air


MwCpw∆T = Ma (h1 – h2)
250,000(4.187)(35-17.5) = ma(89.01-23.02)
ma = 277,589.41 kg/hr

Then, the mass of make-up water


M = ma(w2-w1)
M = 277,589.41 (0.0233 – 0.0033)
M = 5,551.79 kg/hr

Thus, Percentage make-up water


% Make up Water = 5,551,79kg/hr
250,000 kg/hr
% Make up Water = 2.22%

4. Eleven thousand three hundred kilograms per hour of water enters a


cooling tower at 45 ⁰C. Atmospheric air at 16 ⁰C and 55% relative humidity
enter the tower at the rate of 10,200 m³/hr and leaves at 32 ⁰C and
saturated. Determine the mass of water evaporated per hour during the
cooling process?

e. 2,192.53 kg/hr c. 2,291.53 kg/hr


f. 3,192.53 kg/hr d. 3,291.53 kg/hr

SOLUTION:

Mass of water evaporated, mw: mw = ma (sh2 = sh1)


From Psychometric chart:
Entering air at 16 ⁰C and 55% Rh
V1 = 0.83 m³/kg ; sh1 = 0.07 kg/kg
Leaving air at 32 ⁰C and 100% RH saturated
sh2 = 0.037 kg/kg
Mass of air entering the tower;

ma = 10,200 m³/hr
0.83m³/kg

ma = 12,289.16 kg/hr
Then:
mw = 12,289.16(0.307 – 0.07)
mw = 2,192.53 kg/hr

5. How much refrigeration capacity is required to cool 56.67m³ of air


per minute from 29 ⁰C to 21 ⁰C. Assume that the cooled air is saturated.

a. 4.10 L/s c. 2.10 L/s


b. 3.10 L/s d. 5.10 L/s

SOLUTION:

Ref. Capacity, Qa ;
Qa = ma (h1-h2)

Solving for ma;


From psychometric chart:
@ 21 ⁰C DB & 100% Rh
h1 = 70 kj/kg
v1 = 0.875 m³/kg
@ constant SH intersecting 29 ⁰C DB
h2 = 70kj/kg

ma = volume of air
specific volume
ma = 56.67 m³/min
0.875 m³/kg

ma = 64.766 kg/min = 1.079 kg/s

then:
Qa = 1.079 (70 – 61)
Qa = 9,715 Kw

6. Fifty Gallons per minute of water enters a cooling tower at 46 ⁰C.


Atmospheric air at 16 ⁰C. db and 55% RH enters the tower at 2.85 m³/s and
leaves at 32 ⁰C saturated. Determine the volume of water that leaves the
tower.

SOLUTION:

Volume water leaving the tower, V4


V4 = m4 ( vf @ t4)
Solvinf for m4 and vf:
@ tdb1 = 16 ⁰C and 55% RH
v1 = 0.828 m³/kg
@ tdb2 = 32 ⁰C and 100% RH
w2 = 0.0308 kg/kg
h2 = 110.9 kj/kg
h3 = hf @ 46 ⁰C = 192.62 kj/kg and v3 = 0.0010103
m³/kg
w1 = 0.0056 kg/kg
h1 = 32 kj/kg

ma = v1/υ1 = 2.85/0.828
ma = 3.44 kg/s
m3 = (50 gal/min)(3.785 L/gal)(1 min/gal)
(0.0010103 m/kg) ( 1000 L/m³)
m3 = 3.12 kg/s

By mass balance:
M3 – m4 = ma (W2-W1)
3.12 – m4 = 3.44 (0.0308 – 0.0056)
m4 = 3.09 kg/s

By energy Balance:
m3h3 – m4h4 = ma (h2-h1)
3.12 (192.62) – 3.09h4 = 3.44 (110.9 – 32)
h4 = 106.65 kj/kg

From Steam Table 1:


T4 = 25.42 ⁰C
υ4 = 1.0031 L/kg

Thus, volume of water leaving; Vw


Vw = 3.09 (1.0031)
Vw = 3.10 L/s

7. An air conditioned theater is to be maintained at 80 ⁰F dry bulb


temperature and 50% RH. The calculated total sensible heat load in the
theater is 620,000 BTU/hr, and the latent heat load is 210,000 Btu/hr. The air
mixture at 84 ⁰F dry bulb temperature and 72 ⁰F wet bulb temperature is
cooled to 63 ⁰C and 59 ⁰C wet bulb temperature by the cooling tower and
delivered as a supply air to the theater. Calculate the tons of refrigeration
required.

a. 100.65 TOR c. 142.67 TOR


b. 124.67 TOR d. 112.60 TOR

SOLUTION:
Conditioner Capacity, Qa:
Qa = ms (h4 – h1)

Solving for ms:


Qt = ms (h2 – h1)

From Psychrometric chart:


h4 = 35.82 Btu/hr
h1 = 25.78 Btu/hr
h2 = 31.35 Btu/hr
then,
620,000 + 210,000 = ms (31.35 – 25.78)
ms = 149, 012.57

thus;
Qa = 149,012.57 (35.82 – 25.78) btu/hr
12,000Btu/hr
TOR
Qa = 124.67 TOR

8. Determine the partial pressure of water vapor if the barometric


pressure is 101.325 kpa and the humidity ratio is 0.05.

a. 7.54 kpa c. 5.74 kpa


b. 4.75 kpa d. 5.47 kpa

SOLUTION:
W = 0.622 x Pv / (Pt – Pv)

0.05 = 0.622 x Pv / ( 101.325 – Pv)


thus,
Pv = 7.54 kPa

9. Determine the absolute humidity (vapor density) of an air sample


that
has a dew point temperature of 45 ⁰F if the value of the gas constant R for
low pressure water vapor is 85.66 ft-lb/lbm ⁰R. The vapor pressure
corresponding to a saturation temperature of 45 ⁰F is 0.1475 psia.

a. 0.000491 lb/ft³ c. 0.000149 lb/ft³


b. 0.000941 lb/ft³ d. 0.000194 lb/ft³

SOLUTION:

PV = mRT

m/v = ρ = P/RT

ρ = 0.1475 (144)
85.66(45+460)

ρ = 0.000491 lb/ft³

10. Air normal atmospheric pressure has a temperature of 70 ⁰F and a


dew point temperature of 50⁰F. Determine the saturation ratio of the air. The
humidity ratios corresponding to dew point temperatures of 50 ⁰F and 70 ⁰F
respectively are 0.00763 lb/lb and 0.01576 lb/lb respectively.

a. 58.51% c. 38.31%
b. 28.21% d. 48.41%

SOLUTION:

saturation ratio = Wactual x 100


Wsat

Saturation ratio = 0.00763 x 100


0.01576
thus, Saturation ratio = 48.41%

11. The evaporative condenser of an ammonia refrigeration plant has a


water flow rate of 266 kg/s and enters a natural draft cooling tower t 40 ⁰C.
The water is cooled to 29⁰C by air entering at 38 ⁰C db and 24 ⁰C wb. The air
leaves the tower as saturated at 40⁰C. Calculate the make-up water
required in kg/hr.

i. 8977 C. 8055
ii. 8055 D. 8388

SOLUTION:

Water properties: Air Properties:


@ 49⁰C; hf = 167.48 kj/kg @ 38 ⁰C db and 24⁰C
wb
@ 29⁰C; hg = 121.43 kj/jg h = 72.5 kj/kg
w = 0.013 kg/kg
@ 40C db saturated
h = 166 kj/kg, w = 0.0488 kg/kg

Solving for mass of air, ma: .


Heat absorbed by air = heat rejected by water
Ma (h2-h1) = mw cw T
Ma (166 - 72.5) = 126 (4.187) (40 – 29)
Ma = 62.07 kg/s

Then;
m = 62.07 (0.0488 – 0.013)
m = 2.22 kg/s
thus,
m = 2.22kg/s x 3600s/hr
m = 7999.08
12. Determine the approximate load on a cooling tower if the entering
and leaving temperatures are 96F and 88F, respectively and the flow rate of
the water over the tower is 30 gpm

a. 2500 btu/min c. 3000


btu/min
b. 2000 btu/min d. 3500
btu/min

SOLUTION:
Tower Load = 8.33 V T btu/min
Tower Load = 8.44 (30) (96 – 88)
Thus,
Tower Load = 2000 Btu/min

13. Determine the quantity of water lost by bleed-off if the water flow rate
over the tower is 30gpm and the range is 10F. Percent bleed-off required is
33%.

a. 0.077gpm c. 0.099 gpm


b. 0.088 gpm d. 0.066 gpm

SOLUTION:
The quantity of water lost by bleed-off,

Vw = 0.33 x 30
Vw = 0.099 gpm

14. Determine the equivalent standard air volume for 150 m³/s of air
having a dry bulb temperature of 15C.

a. 150 m³/s c. 170 m³/s


b. 160 m³/s d. 180 m³/s

SOLUTION:

Vs = 150 (21 + 273)


(15 + 273)

thus,
Vs = 153 m³/s
15. 50 GPM of water enters a cooling tower at 46 oC. Atmospheric air
at 16 C db and 55% RH enters the tower at 2.85 m3/s and leaves at 32oC
o

saturated. Other data: a) mass of h 2o entering – 3.122 kg/s; b) specific


humidity of air entering – 0.0056 kg/kg; c) specific humidity of air leaving =
0.0307 kg/kg; d) mass of air entering – 3.442 kg/s; e) specific volume of h 2o
leaving – 0.0010031 m3/kg. Determine the volume of h 2o that leaves the
tower in Li/s.

SOLUTION:

M3 = V3 = 50 GPM x 3.785 Li/gal x 1 min/60s = 3.12 kg/s


V3 0.0010031 m3/kg x 1000Li/m3

Mass balance:
m3 = m4 = ma (w2 – w1)
3.12 – m4 = 3.44 (0.0308 – 0.0056)
m4 = 3.09 kg/s

Energy balance:
m3h3 – m4h4 = ma (h2 – h1)
0.12 (192.62) – 3.09 (h4) = 3.44 (110.9 – 32)
h4 = 106.65 kJ/kg

V4 @ T4 (25.5oC) = 1.0031 Li/kg


Vw = 3.09 (1.0031) = 3.10 Li/s

16. The amount of water carried by air in cooling tower is 15 lb/min. the
change in humidity ratio in outlet and inlet is 0.025 lb/lb. Determine the
volume flow of air needed in specific volume is 13 ft 3/lb.

SOLUTION:

Volume flow rate of air = mv


Mw = ma (w2 – w1)
Ma = mw / (w2 – w1)
Ma = 15 / 0.025 = 600 lb/min
Volume flow rate of air = 600 lb/min x 13 ft3/lb
Volume flow rate of air = 7800 cfm

17. A certain sample of air has a temperature of 70 ⁰F (partial pressure


of 0.36 pisa) and a dew point temperature of 50 ⁰F. The partial pressure of
the water vapor corresponding to a 50⁰F dew point temperature is 0.178
psia. Determine the relative humidity RH.

a. 49.44% c. 39.44%
b. 59. 44% d. 69.44%

SOLUTION:
RH = Actual partial pressure x 100
Partial pressure at sat.

RH = 0.178 x 100
0.36

RH = 49.44%

Dryer

1. The density of air at 35 C and 101 kPa is 1.05 kg / m3.


The humidity
ratio is: (Oct 96)

SOLUTION:
w= Pv
Pt – Pv
PV = mRT
P = mRT
V
Pair = 1.08 (0.287) (35 + 273)
= 95.48 kPa
Pt = Pair + Pvapor
101 = 95.48 + Pv
Pv = 5.53 kPa

w = 0.622 Pv = 5.53
Pt - Pv 101 – 5.53
= 0.036 kg vapor / kg dry air

2. If the sensible heat ratio is 0.80 and the cooling load is


100kW, what is the amount of sensible heating? (Apr. 94)

SOLUTION:

SHR = sensible hat ratio

0.80 = QS
100

QS = 80kW
3. Calculate the specific volume of an air-vapor mixture in
cubic meters per kilogram of dry air when the following
condition prevail: t = 30 C, w = 0.015 kg, and PL = 90 kPa. (Oct
95)

SOLUTION:
V= RaT
Pt - Pv

w = 0.622 Pv
Pt - Pv
0.015 = 0.622 Pv
90 - Pv
Pv = 2.12 kPa

V = 0.287 (30 + 273)


90 – 2.12
= 0.99 m3 / kg
4. Compute the humidity ratio of air at 62% relative
humidity and 34 C when the barometric pressure is 101.325
kPa. (Apr 95)

SOLUTION:
w = 0.622 Pv
Pt - Pv
Psat @ 34 C = 5.32 kPa
Pv = (RH) PSat
= (0.62) (5.32)
= 3.30 kPa

w = 0.622 3.30
101.325 – 3.30

= 0.021 kg Vapor / kg dry air

5. Copra enters dryer containing 60% water and 40% of


solid sand leaves with 5% water and 95% solids. Finds the
weight of water removed based on each pound of original
product.

a. 0.58lb
b. 0.40lb
c. 0.47lb
d. 0.67lb

SOLUTION:

Let; m = weight of original product per lb of wet feed


Solid in wet feed = solid in dried product

0.95m = 0.40 (1)

m = 0.42lb

Thus;

Weight of water removed = 1 – 0.42


Weight of water removed = 0.58lb

6. A room being air-conditioned is being held at 25 C dry


bulb and 50% relative humidity. A flow rate of 5 m3 / s of
supply air at 15 C dry bulb and 80% relative humidity is being
delivered to the room to maintain that steady condition. What
is the sensible heat absorbed from the room air in kW? (Oct.
99)

SOLUTION:

PV = mRT
100 (5) = m (0.287) (15 + 273)
m = 6.049 kg / s

Qs = sensible heat
= mCp (t2 – t1)
= 6.049 (1.003) (25 – 15)
= 60.8 kW

7. A mechanical draft dry cooling tower cools the cooling


water from 60 C to 25 C at the rate of some 149.4 giga grams
per hour. Atmospheric air enters the tower at 20 C and leaves
at 35 C. The fan is driven by a 7460 kW motor. What is the
mass flow rate of the air into the cooling tower in kg per
second? (Apr. 96)

SOLUTION:
mw = 149.4 x 109 = 41,500 kg / s
1000 (3600)

Heat Loss by water = Heat gain by air


(mCp∆t) water = (mCp∆t) air
41,500 (4.187) (60 – 25) = ma (1.006) (35 – 20)
ma = 403,023 kg / s
8. What is the enthalpy of the air-vapor mixture at 65%
relative humidity and 34 C when the barometric pressure is
101.3 kPa? (Oct 96)

GIVEN:
Psat at 34 C = 5.318 kPa
hg at 34 C = 2563.6 kJ / kg

SOLUTION:
Pv = RH x Psat
= 0.65 x 5.318 = 3.4567 kPa

W = 0.622 Pv = 0.622 3.4567


P – Pv 101.3 – 3.4567

= 0.022 kg / kg

9. A coil has an inlet temperature of 60 F and outlet of 90 F.


If the mean temperature of the coil is 110 F, find the bypass
factor of the coil. (Oct. 99)

SOLUTION:

Bypass factor = t mean – t db of outlet air


t mean – t wb of inlet air

BF = 110 – 90
110 – 60

= 0.40
10. If the latent and sensible heat loads are 20 kW and 80
kW respectively, what is the sensible heat ratio? (Apr. 99)

SOLUTION:

Let: SHR = sensible heat ratio

SHR = Qs
Qs + Qs
80
80 + 20

= 0.80

11. Compute the pressure drop of 30 C air flowing with a


mean velocity of 8 m / s is a circular sheet-metal duct 300 mm
in diameter and 15 m long. Use a friction factor, f = 0.02, and
ρair = 1.1644 kg / m3 (Apr 96)

SOLUTION:
2
∆P = fLVρ
2D 2
= 0.02 (15) (8) (1.1644)
2 (0.30)

= 37.26 Pa

12. A pressure difference of 350 Pa is available to force 20 C


air through a circular sheet-metal duct 450 mm in diameter
and 25 m long. At 20 C, ρ = 1.204 kg / m3 and take friction
factor, f = 0.016. Determine the velocity. (Oct 96)
SOLUTION:
2
∆P = fLVρ
2D
2
350 = 0.016 (25) (V) (1.204)
2 (0.30)

V = 25.57 m / s

13. A duct 0.40 m high and 0.80 m wide suspended from the
ceiling in a corridor makes a right angle turn in the horizontal
plane. The inner radius is 0.2 m and the outer radius is 1.0 m
measure from the same center. The velocity of air in the duct
is 10 m / s. Compute the pressure drop in this elbow.
Assuming; f = 0.3, ρ = 1.204 kg / m3 and L = 10 m. (Apr 99)

SOLUTION:
2
∆P = fLVρ
2Deq
Deq = 2ab (for rectangular duct)
a+b
= 2 (0.4) (0.80) = 0.53 m
0.4 + 0.8
2
∆P = 0.3 (10) (10) (1.2041)
2 (0.53)

= 341 kPa
14. A rectangular duct has dimensions of 0.25 m by 1 m.
Determine the equivalent diameter of the duct. (Apr 96)

SOLUTION:

Deq = 2ab
a+b
Deq = 2 (0.25) (1)
0.25 + 1

= 0.40 m

15. A sudden enlargement in a circular duct measures 0.20 m


diameter upstream and 0.40 m downstream. The upstream
pressure is 150 Pa, downstream pressure is 200 Pa. What is
the flow rate of 20 C air through the fitting? Use ρ = 1.02041
kg / m3 (Oct 96)

SOLUTION:

Q = Au Vu 0.20 m
2 2 Vu
Vd
Ploss = Vu ρ 1 – Au 0.40
m
2 Ad
2
Au = 0.20 = 0.25
Ad 0.40
2 2
(200 – 150) = Vu (1.204) (1 – 0.25) ; Vu = 12.15 m /s
2 2
Q = π (0.20) (12.15) = 0.38 m3/ s
4
16. Wet material containing 215% moisture (dry basis) is to
be dried at the rate of 1.5 kg / s in a continuous dryer to give a
product containing 5% moisture (wet basis). The drying
medium consists of air heated to 373 K and containing water
vapor equivalent to a partial pressure of 1.40 kPa. The air
leaves the dryer at 310 K and 70% saturated. Calculate how
much air will be required to remove the moisture. (Apr 97)

SOLUTION:
amount of moisture removed = amount of moisture absorbed by
air
m = rate of flow of dried product.

1 (1.5) = 0.95 m ; m = 0.501 kg / s


1 + 2.15
amount of moisture removed = 1.5 – 0.501
= 0.999 kg / s

W1 = 0.622Pv = 0.622 (1.4)


Pt – Pv 101.325 – 1.4

= 0.00871 kg / kg

ma (0.0289 – 0.00871) = amount of moisture removed


ma (0.0289 – 0.00871) = 0.999

ma = 49.48 kg / s

17. In an auditorium maintained at a temperature not to


exceed 24 C and relative humidity not to exceed 60%, a
sensible heat load of 132 kW and 78 kg of moisture per hour to
be removed. Air is supplied to the auditorium at 18 C. How
many kiograms of air must be supplied per hour? (Oct 99)
SOLUTION:

AUDITORIUM
SPACE
Qs = ma Cp (t2 – t1) Fan
24 C db and 60% RH
132 = ma (1) (24 – 18) 18 C
ma = 22 kg / s Qs = 132kW
QL = 78 kg / hr

ma = 79,200 kg / hr

18. Copra enters a dryer containing 60% water and 40% of


solids and leaves with 5% water and 95% solids. Find the
weight of water removed based on each pound of original
product. (Apr 95)

SOLUTION:

m = weight of original product per lb of wet feed


solid in wet feed = solid in dried product

0.95 m = 0.40 (1)


m = 0.42 lb

weight of water removed = 1 – 0.42

= 0.58 lb

19. A dryer is to deliver 1000 kg / hour of palay with a final


moisture content of 10%. The initial moisture content in the
feed is 15% at atmospheric condition with 32% dry bulb and
21% wet bulb. The dryer is maintained at 45 C while the
relative humidity of the hot humid air from the dryer is 80%. If
the steam pressure supplied to the heater is 2 MPa, determine
the air supplied to the dryer in m3 / hr. (Oct 95)

SOLUTION:
amount of moisture = amount of moisture absorbed
m = amount of palay in wet feed
solid in wet feed = solid in dried product

0.85 m = 0.90 (1000)


m = 1,058.83 kg / hr

amount of moisture removed:


m = 1058.83 – 1000 = 58.28
From psychrometric chart:
W1 = W2 = 0.0111 kg / kg; V2 = 0.915 m3 / kg
W3 = 0.0515 kg / kg

the amount of moisture absorbed:


= ma (W3 - W2)
58.823 = ma (0.0515 – 0.111)
ma = 1456.015 kg / hr

Va = 1456.015 kg / hr (0.915 m3 / kg)

= 1332.25 m3 / hr

20. A 4 m x 4 m x 4 m room has a relative humidity ratio of


80%. The pressure in the room is 120 kPa and temperature is
35 C (Psat = 5.628). What is the mass of vapor in the room.
Use Rvapor = 0.4625 kN – m / kg – K. (Oct. 99)

SOLUTION:
PvV = mv Rv T

RH = Pv
Psat

0.80 = Pv
5.628
Pv = 4.5024 kPa
4.5024 [(4) (4) (4)] = mv = (0.4615) (35 + 273)
mv = 2.027 kg

21. A mixture of dry air and water vapor is at a temperature


of 21 C under a pressure of 101 kPa. The dew point
temperature is 15 C. Calculate the relative humidity. (Apr 97)

SOLUTION:

RH = Pv
Psat @ 21 C

Pv = Psat @ 15 C
= 1.7044 kPa
Psat @ 21 C = 2.4861 kPa

RH = 1.7044
2.4861

= 68.56%
Fans and Blowers

ME BOARD (April 1997)


1. A fan whose static efficiency is 40% has a capacity of
60,000ft3 per hour at 60F and barometer of 30in. Hg and gives
a static pressure of 2 in. of water column on full delivery. What
size electric motor shall be used to drive this fan?

a. ½ Hp c. 2 Hp

b. 1 Hp d. 1 ½ Hp

SOLUTION:

Brake (input) Power = Static air Power


Static Fan ef

Pbrale = Pair /es

Solving for Pair


Pair = γQh

Where:
Q = 60,000/3600 = 16.67 ft3/s
h = γwhw = (62.4)(2/12) = 10.4
γ γ γ

then;
Pair = γ(16.67)(10.4/ γ)
= 173.37 ft-lb/s (1Hp/550 ft-lb/s)
= 0.315 Hp

thus:
Pbrake = 0.315/0.40
= 0.788 Hp
(b) Use 1 Hp motor
ME BOARD (April 1997)
2. Air is flowing in a duct with velocity of 7.62 m/s and a
static pressure of 2.16 cm water gage. The duct diameter is
1.22, the barometer pressure 99.4 kPa and the gauge fluid
temperature and air temperature are 30C. What is the total
pressure of air against which the fan will operate in cm of
water?

a. 3.25 c. 3.75
b. 2.50 d.1.25

SOLUTION:
h = h s + hv
Solving for the velocity head, hv:
hv = V2 = (7.62) 2
g 2(9.81)

Solving for velocity in terms of cm of water:


hv (1000) = (2.959)(ρair)

Where:
ρair = 99.4 .
0.287(30+273)
ρair = 1.143 kg/m3
then;
hv(1000) = (2.959)(1.143)
hv = 0.0034 m of water
hv = 0.34 cm of water

thus;
h = 2.1 + 0.34
(b) h = 2.50 of water

ME BOARD (Oct 1996)


3. Air enters a fan through a duct at a velocity of 6.3 m/s
and an inlet static pressure of 2.5 cm of water less than
atmospheric pressure. The air leaves the fan though a duct at
a velocity of 11.25 m/s and a discharge static pressure of 7.62
cm of water above the atmospheric pressure. If the specific
weight of the air is 1.20 kg/m3 and the fan delivers 9.45 m3/s,
what is the fan efficiency when the power input to the fan is
13.75 kW at the coupling?

a. 71.81% c. 52.34%
b. 61.81% d. 72.34%

SOLUTION:
Fan efficiency = Air Power .
Input power
efan = Pair/Pln

Solving for Pair:


h = hs + hv
= (Pd – Ps) + (Vd2 - Vs2)
γ 2g
= [(0.0762 + 0.025)/1.20] (1000) + (11.25)2 – (6.3)2
2(9.81)
= 88.761 m of air

then;
Pair = γQh
= [1.2(0.00981)] (9.45)(88.761)
= 9.874 kW

thus;
efan = 9.874/13.75
(a) efan = 71.81%

ME BOARD (Oct 1994)


4. A fan described in manufactures table is rated to deliver
500m3/min at a static pressure (gage) of 254 cm of water when
running at 250 rpm and requiring 3.6 kW. If the fan speed is
changed to 305 rpm and the air handled were at 65C instead of
standard 21C, find the power in kW.

a. 3.82 c. 4.66
b. 5.07 d. 5.68

SOLUTION:

Power Rewuired at 305 rpm and 65C:


P1 = ρ1 = T2
P2 ρ2 T1
P1 = 65 + 273
P2 21 + 273

Solving for the power required at 305 rpm and 21C:


P1/P2 = (N1/N2) 3
3.6/P2 = (250/305) 3
P2 = 6.5 kW

then;
6.5 = 65 + 273
P2 21 + 273

Thus;
(d) P2 = 5.68 kW

ME BOARD (April 1995)


5. A fan draws 1.42 m3/s of air to a static pressure of 2.54
cm of water through a duct 300mm diameter and discharge it
through duct of 275 mm diameter. Determine the static fan
efficiency if total fan mechanical efficiency is 70% and air is
measured at 25C and 760 mm Hg.

a. 60% c. 30%
b. 50% d. 47%

SOLUTION:

es = em (hs/h)

Solving for hs:


ρair = P/ RT
= ___101.325___
(0.287)(25+273)
= 1.18 kg/m3
hs = hwρw/ ρa
= 0.0254(1000)
1.18

Solving for hv:


V = Q/A
Vs = __1.42__ = 20.09 m/s
π(0.3) 2/4
Vd = __1.42____ = 20.09 m/s
π(0.275) 2/4
hv = (Vd)2 – (Vs)2 = (23.9)2 – (20.09)2
2g 2(9.81)
hv = 8.54 m of air

Solving h:
h = h s + hv
= 21.52 + 8.54
= 30.06 m of air

thus;
es = (0.70)(21.52/30.06)
(b) es = 0.5011 or 51.11%

ME BOARD (Oct 1997)


6. Find the horsepower of an industrial fan that delivers 23
m3/s of air through a 1m by 1.2 m ; pressure is 127 mm of
water ; air temperature is 21C and barometric pressure is 760
mm of mercury.

a. 53.35 Hp c. 46.45 Hp
b. 43.33 Hp d. 56.45 Hp

SOLUTION:
P=γQh

Solving for h:
V = Q/A = 21/(1*1.2) = 21.667 ρwater @ 101.325 kPa =
998.6 kg/m3
ρair = ___101.325___ hv = _ρair (V2/2g)
8.314(21 + 273) ρwater
ρair = 1.2 kg/m3 hv = _1.20_ _(21.67)2
h = 0.127 + 0.029 = 0.156 m 998.6
(2*9.81)
hv = 0.029 m
hs = 127 mm = 0.127
m
then;
P = (9.81)(26)(0.156)
= (39.79 kW

thus;
(a) P = 53.34 Hp
ME BOARD PROBLEM
7. A fan is listed as having the followe\ing performance with
standard air:
Volume discharge = 120 m3/s
Speed = 7 rps
Static Pressure = 310 mm water gage
Brake Power = 620 kW

The system duct will remain the same and the fan will
discharge the same volume of 120 m3/s of air 93C and a
barometric pressure of 735 mm Hg when it its speed is 7 rps.
Find the brake power input and the static pressure required.

a. 482 kW, 241 mm WG c. 482 kW, 256


mm WG
b. 492 kW, 241 mm WG d. 492 kW, mm WG

SOLUTION:

Brake power input = 620 (ρ2/ρ1)

= 620 (ρ2/1.2)

Sovling for ρ2:


ρ2 = 735 (101.325/760)
0.287(93 +273)
= 0.9329 kg/m3

Solving for the static pressure, h2:


h2/ 310 = 0.9329/1.2
h2 = 241 mm of water gage

then;
Brakepower input = 620 (0.9329/1.2) = 482 kW

Thus;
(a) h2 = 241 mm & Pbrake =482 kW
ME Board Exam Problem
8. Local coal with higher heating value of 5,500 kCal/kg is
burned in a pulverized coal fired boiler with 25% excess air the
reate of 25.9 M.T. per hour when the steam generated is 220
M.T. per hour. This boiler is served by 2 forced-draft fans of
equal capacity delivering the air to 305 mm of water to the
furnace. Calculate the capacity of each fan in m3/hr if fan
capacity is to be 110 percent of the maximum requirement.
Ambient air is 100 kPa and 30C.

a. 31.75 m3/s c. 28.87 m3/s


b. 63.50 m3/s d. 57.73 m3/s

SOLUTION:

The theoretical weight of air to burn the fuel is given in an


appropriate formula when the heat value of the fuel is given:
Wta = (A/F) = HHV, kCal/kg
745
= 5,500/745
= 7.382 kg air/ kg fuel

Mass flow rate of air required for combustion


Ma = (7.382)(1.25)[25.9(100)]
= 238,922.25 kg/hr

Capacity of each fan at 110% of the maximum requirement:


Q1 = Q2 = 207,830.05(1.10)
2
= 114,306.53 m3/s = 31.75 m3/s

thus;
(a) Q = 31.75 m3/s

ME Board Exam Problem


9. A larged-draft fan is handling air at 1 atm, 43.4C unfer a
total head of 26.6 cm WG (at 43.3C). The power input to the
fan is 224 kW and the fan is 75% efficient. Compute the
volume of air handled per minute. Local gravity of acceleration
is g = 9.81 m/s2.

a. 3,862.87 m3/min c. 2,862.87


m3/min.
b. 4,862.87 m3/min d. 4,567.97 m3/min.

SOLUTION:

Power Input to fan = Shaft Power


P=γQh
efan
224 = 9.81 Q(0.266)
0.75
Q = 64.38 m3/s
= 3,862.87 m3/min
thus;
(a) 3,862.87 m3/min

ME Board April 1997


10. A fan whose static efficiency is 40% has a capacity of
60,000 ft3 / hr at 60°F and barometer of 30 in Hg and gives a
static pressure of 2 in of water column on full delivery. What
size of electric motor should be used to drive the fan?

Solution:

hs = hw ww
wa

hs = (2in/12) (62.4)
wa

hs = 10.4 / wa

Air Power = wa Q h

Air Power = wa (60,000 / 60) (10.4 / wa)


33,000
Air Power = 0.315 Hp

Therefore,

Use 1 Hp motor (standard)

Pumps

ME BD April 98
1. A pump lifts water at the rate of 283 li/s from a lake and
forces it into a tank 8m above the level of water at a pressure
of 137Kpa what is the power required in KW

SOLUTION:
P= γQH
Q=283li/s [1m³/1000li] = 0.283 m³/s
H= 8 + (137KN/m³ / 9.81 KN/m³) = 21.9653 m
P= 9.81 KN/m³ x 0.283m³/s x 21.9653m
P= 60.9807 KW

ME BD April 98
2. A pump discharge 150 li/s of water to a height of 75m if
the efficiency is 75% and the speed of the pump is 1800rpm,
what is the torque in N-m to which the drive shaft is subjected

SOLUTION:
Pw = γQH
= 9.81 KN/ m³ [0.150 m³/s] [75m]
= 110.3625 KW
Pb = Pw / ep = 110.3625KW / 0.75 = 147.15 KW
T = Pb / 2πN
= 147.15 KW / 2 π (1800rev/min) (1min/sec)
= 0.7807 KN.m x (1000N/1KN)
T= 780.7 N-m

ME BD April 98
3. A pump receives 8 kg/s of water at 220 KPa and 110°C
and discharges it at 1100KPa. Compute the power required in
KW
SOLUTION:
ρ = m/v
v = m/p = 8kg/s / 1000kg/ m³ = 0.008m³/s
P= v (Pd – Ps)
= 0.008m³/s [1100-220]
P= 7.04 KW

ME BD April 93
4. The power output is 30 Hp to a centrifugal pump that is
discharging 900gpm and which operates at 1800rpm against a
head, H=120ft, 220V, 3θ, 60Hz. if this pump is modified to
operate 1200rpm assuming its efficiency remains constant
determine the discharge in gpm the theoretical head it imparts
to the liquid and the power input to the pump.

SOLUTION:
Q 1 = N1 P1/ P2 = (N1/N2) ³
Q 2 N2 P2 = 30 / (1800/1200)³
Q2 = Q1 N2 / N1 P2= 8.8889Hp
= 900(1200)/1800
Q2= 600gpm
H1/H2 = (N1/N2)²
H2= 120 / (1800/1200) ²
H2= 53.3333ft

ME BD Oct93
5. A centrifugal pump requires 40 ft of read to deliver water
from low level to a higher left. If pump speed is 1600rpm
determine the impeller diameter of the centrifugal pump.

SOLUTION:
Vd²=2gh
Vd = √2gh
= √ 2(32.2ft/s) (40ft)
Vd = 50.7543 ft/s
D= Vd/ πN
= (50.7543/ft/s) / π (1600rev/min) (1min/60sec)
D= 0.6058 ft

ME BD Oct 95
6. A centrifugal pump powered by a direct drive induction
motor is needed to discharge 150gpm against a total head of
180ft. when turning a fully loaded speed of 3500rpm. What
type of pump should be selected?

SOLUTION:
Ns = (N√Q) / H3/4
= (3500√150) / (180)3/4
Ns= 872.2868rpm
 Radial Type

ME BD Oct 95
7. It is desired to deliver 5m³ at a head of 640m in a single
stage pump having specific speed not to exceed 40 if the
speed is not to exceed 1352rpm how many stages required?

SOLUTION:
Ns = (N√Q) / H3/4
h= head/stage= H/n
h=640/n
40 = (1352√5) / (640/n) 3/4
n= 2 stages

ME BD April 95
8. A double suction centrifugal pump delivers 70ft³/s of
water at head of 12m and running at 1250rpm. What is the
specific speed of the pump?
SOLUTION:
Ns = (N√Q) / H3/4
Q= (70/2) ft³/s [7.48gal] [60s]
Q= 15,710.1gpm
H= 12 [3.2808ft/1m]
H= 39.3696ft
Ns = (1250√15,710.1) / (39.3696)3/4
Ns= 9,968. 4753 rpm

ME BD April 97
9. A hydroelectric plant having 50 km² reservoir area and
100m head is used to generate power. The energy utilize by
the consumers whose load is connected to the power plant
during a 5hr period is 13.5x10 KWH the overall generation
efficiency is 75%. Find the fall of water in the reservoir after
5hr period?

SOLUTION:
Pw= γQH
Energy= γQHt
13.5x10 KWH = 9.81 Q (100m) (5hr) (0.75)
Q= 3669.7248m³/s
Volume after 5 hr period
V=Ah
V= 3669.7248m³/s[5hr][3600s/hr]
V= 66,055,045.87m³
h= v/a = (66,055,045.87m³) / 50km²[100m/1km]
h= 1.3211m

ME BD April 98
10. A hydroelectric plant discharges water at the rate of
0.75m³/s and enters the turbine at 0.35m/s with a pressure of
275KPa. Runner inside diameter is 520rpm and turbine
efficiency is 88% find coefficient turbine speed factor?

SOLUTION:
Ø = πDN / √2gh
H= P/γ + v²/2g
= (275 / 9.81) + (0.35² / 2(9.81))
H= 28.0389m
Ø= [π (0.55) (520rev/min) (1min/60sec)] / [√2 (9.81m/s²)
(28.0389m)
Ø= 0.6385

ME BD April 98
11. The available flow of water is 25m³/s at 30m elevation if
a hydroelectric plant is to be installed with turbine efficiency
of 85% and generator efficiency of 90% what maximum power
that the plant could generate?

SOLUTION:
Pw = γQH
= (9.81) (25m³/s) (30m)
Pw= 7357.5 KW
Pgen = 7357.5 KW (0.85) (0.90)
Pgen= 5628.4875 KW

ME BD Oct95
12. Water flows steadily with a velocity of 3.05m/s (in a
horizontal pipe) having a diameter of 15.24cm at one section
of the pipe the temp. And pressure of the water is 21°C and
689.3KPa respectively, pipe length is 304.8m and inlet
pressure is 516.9 Kpa. What is the friction factor?

SOLUTION:
H= (fLV²) / (2gD)
H= (Pd – Ps) / γ
H= (689.3 – 516.9) / (9.81)
f= H2gD / LV²
= [17.5739(2) (9.81m/s²) (0.1524m)] / [(304.8m) (3.05m/s) ²]
f= 0.0185

ME BD Oct 95
13. For a proposed hydroelectric plant the tail water and
head water elevation is 160m and 195m respectively. If
available flow is 10m³/s and headloss of 5% of water available
head what is the water power?

SOLUTION:
Pw= γQH
Hg= HHW - HTW
H= Hg- HL
Hg= 195-160 = 35m
H= 35 – 0.05 (35)
H= 33.25m
Pw= 9.81KN/m³ [10m³/s] [33.25m]
Pw= 3261.825 KW

ME BD Oct 96
14. A vacuum pump is required to drain a flooded area at
20°C pumping pressure of water at this temperature is 2340Pa.
the pump cannot lift the water higher than 10.16m. Determine
the atmospheric pressure?

SOLUTION:
(Ps/γ) + (Vs²/2g) + (Zs) = (Pd/γ) + (Vd²/2g) + (Zd)
(Ps/γ) = (2340 N/m²/ 9810N/m³) + [10.16-0] m
= 0.2385m +10.16m
Ps= 10.3985 γ
= 10.3985m[9.81KN/m³]
Ps=102.0096 KPa

ME BD Oct 96
15. A pump driven by an electrical motor moves 38gpm of
water from a tank to another tank with a total 10ft of 200ft. if
the pump efficiency is 68% what size of motor will be required?

SOLUTION:
ep = Pw/Pb
Pw = γQH
Q= 38gal/min [1ft / 7.481gal]
Q= 5.0795 ft³/min
Pw= 62.4lb/ft³ [5.0795ft³/min] [200ft]
Pw= 63,392.16 ft-lb/min [1hp/33,000ft-lb/min]
Pw= 1.921Hp
Pb= Pw/ep
= 1.921Hp /0.68
Pb= 2.825 Hp say 3 Hp

ME BD Oct 95
16. The specific speed of a turbine is 75rpm and running at
450 rpm if the head is 20m and generator efficiency is 90%,
what is the maximum delivered by the generator?
SOLUTION:
Pgen = PwNgen
Ns= (N√Hp) / (H) 5/4
75= 0.2623 N √KW
(20m) 5/4
KW= 722.3689 x .90
Pgen= 722.3689 x .90
Pgen= 650.0076 KW

ME BD Oct 95
17. A turbine has a mechanical efficiency of 95% volumetric
efficiency of 91% and total efficiency of 80% if the effective
head is 40m find the total head

SOLUTION:
ηH = h / Htotal = (0.8) / 0.95(0.97)
ηH = 0.8681
Htotal = 40 /0.8681
Htotal = 46.075m

ME BD April 98
18. A fuel pump is delivering 10 gpm of oil with s.g of 0.83.
The total head is 9.14m; find how much energy does the pump
consume?

SOLUTION:
Pw = γQH
Sg = Ps/Pw
Ps = sg x Pw
= 1000kg/m³ [0.83]
Ps = 830 kg/m³
γ=800kg/m³ [9.81]
= 8.1421 KN/m³
Pw = 8.1423 KN/m³ (6.309x10-4m³/s)(9.14m)
Pw = 0.047 kj/sec [3600sec/1hr]
Pw= 169.0271 KJ/hr

ME BD Oct 97
19. A pump with 400mm dia. Suction pipe and a 350mm
diameter discharge pipe is to deliver 20,000li/min of 15.6°C
water. Calculate the pump head in meter if suction gage is
7.5cm below pump centerline and reads 127mmHg vacuum and
discharge gage is 45 cm above the pump centerline and reads
75KPa

SOLUTION:
HT = Pd-Ps + Vd² -Vs² + (Zd-Zs)
γ 2g
Pressure Head
Ps = 127mmHg [101.325MPa / 760mmhg]
Ps = 16.9319 KPa
[75- (-16.9319) / (9.82 KN/m³)] KN/m²
Pressure Head = 9.3712 m
Velocity Head
Q = AV
A = πd² / 4
Vs = 20,000li/min [1m³/1000li] [1min/60sec]
π/4 (0.4) ²
Vs = 2.6526 m/s
Vd = 20,000li/min [1m³/1000li] [1min/60sec]
π/4 (0.35) ²
Vd = 3.4646
Velocity Head = = (3.4646² - 2.6526²) m²/s²
2 (9.81m/s²)
Velocity Head = 0.2532 m
Static Head = Zd – Zs
= 0.45m – (-0.075m)
= 0.525 m
HT = 9.38712 + 0.2532 + 0.525
HT = 10.1494 m

ME BD Oct 97
20. A centrifugal pump delivers 300,000li/hr of water to a
pressurized tank whose pressure is 280KPa. The source of the
water is 5m below the pump. The diameter of the suction pipe
is 250mm. Calculate the KW rating of the driving motor
assuming the pump efficiency to be 72%?

SOLUTION:
ep= Pw /Pb
Pressure Head
PH = Pd-Ps = (280-0) KN/m²
γ 9.81 KN/m³
PH = 28.5423m
Velocity Head
Hv = (Vd²-Vs²) / 2g
Q = 300,000li/hr [1m³/1000li][1hr/3600sec]
Q = 0.0833 m³/s
Q = AV
Vd = (0.0833 m³/s) / π/4 (0.259) ² = 1.697m/s
Vs = (0.0833 m³/s) / π/4 (9.300m) ² = 1.785m/s
Hv = (1.697² - 1.1785²) / 2 (9.81)
Hv = 0.076 m
Static Head
Hs = Zd-Zs = 0 – (-5) = 5m
HT = 28.5423 + 0.076 + 5
HT = 33.6183m
Pw = γQH
= 9.81KN/m³ [0.0833m³/s][33.6183m]
Pw = 27.472 KW
Pb = Pw/ep = 27.472 / 0.72
Pb = 38.1556 KW

FUELS AND COMBUSTION

ME Board April 1991

1. A 650 Bhp diesel engine uses fuel oil of 28° gravity, fuel
consumption is 0.65 lb/Bhp-hr. Cost of fuel is P7.95 per liter.
For continuous operation, determine the minimum volume of
cubical day tank in cm3, ambient temperature is 45 ° C.

Solution:
SG15.6°C = ____141.5 ____
131.5 + 28
= 0.887
SG45°C = SG15.6°C [ 1 –0.007 ( t – 15.6 ) ]
= 0.887 [ 1 – 0.007 ( 45 – 15.6 ) ]
= 0.869
Density of fuel = 0.869 ( 1 kg. / liters )
= 0.869 kg / liter
Solving for fuel consumption :
Mf = 0.65 ( 650 )
= 422.5 lb / hr
= 191.61 kg / hr
Vf = 191.61 kg/ hr / 0.869 kg/liter
= 220.495 liters / hr

Volume of Day tank = 220.495 li/hr x 24 hrs.


= 5,291.88 li x 1m3 / 1000 li
= 5.291.88 m3 x (100) 3 cm3 / 1 m3
Volume of Day Tank = 5,291,880 cm3

ME Board October
2. A circular fuel tank 45 feet long and 5.5 feet diameter is
used for oil storage. Calculate the number of days the supply
tank can hold for continuous operation at the following
conditions :
Steam flow = 2000 lbs / hr
Steam dry and saturated at 200 psia
Feedwater temperature = 230°F
Boiler Efficiency = 75 %
Fuel oil = 34°API

Solution:

From steam tables :


At 200 psi (1.380 Mpa), hs = 2789.6 KJ/kg
At 230°F (110°F), hf = 461.3 KJ/kg

Qh = 41,130 + 139.6 (34)


= 45,876 KJ/kg
m s = 2000/2.205 = 907 kg/hr
ηb = m s (hs - hf ) / mf Qh
0.75 = 907(2789.6 – 461.3)
mf (45,876)
mf = 61.376 kg/hr

SG15.6°C = 0.855
Density = 0.855(1000kg/m3)= 855 kg/ m3
Volume of tank = π/4 (5.5/3.281)2 (45/3.281)=30.927 m3
Total weight of fuel = 30.927 m3 x 855 kg/ m3
= 25,904 kg

Number of Days = 25,904 / 61.37 (24)


Number of Days = 17.58 days

ME Board April 1998


2. The dry exhaust gas from oil engine has the following
gravimetric analysis:
CO2 = 21.6 % 02 = 4.2 % N2 = 74.2 %
Specific heats at constant pressure for each component
of the exhaust gas in Kcal/kg °C are:
CO2 = 0.203 02 = 0.219 N2 = 0.248
Calculate the gas constant in J/kg-°K

Solution:
Converting the gravimetric analysis to volumetric:
CO2 0.216 0.216/44 = 0.004909
02 0.042 0.042/32 = 0.001312
N2 0.742 0.742/28 = 0.026500
0.032721 mols/kg-
mol

Molecular weight = 1 / 0.032721


Molecular weight = 30.56 kg / kg-mol

R = 8.314 / M
R = 8.314 / 30.56
R = 0.27206 KJ/kg-°K
R = 272.06 J/kg-°K

ME Board October 1996

3. There are 20 kg of flue gases formed per kg of fuel oil


burned in the combustion of a fuel oil C 12H26 . What is the
excess air in percent?
Solution:
C12H26 + 18.5 O2 + 18.5(3.76) N2 12 CO2+13H2
O+18.5(3.76)N2
Theo. A/F = 18.5 + 3.76(18.5)
1
Theo. A/F = 88.06 mol/mol

Theo. A/F in kg/kg = 88.06 (28.97)


12(120+26(1)
Theo. A/f in kg/kg = 15 kg air/ kg fuel

Mass of air = 20 kg flue gas – 1 kg fuel gas


Mass of air = 19 kg air

Actual A/f = 19 kg air/ kg fuel


Actual A/F = Theo. A/f (1 + e )
19 = 15 ( 1 + e )
e = 0.2667
e = 26.67 %

ME Board April 1984

4. A gaseous fuel mixture has a molal analysis:


H2 = 14% CH4 = 3% CO = 27%
O2 = 0.6% CO2 = 4.5% N2 = 50.9%
Determine the air-fuel ratio for complete combustion on
molal basis.

Solution:
Chemical reaction with oxygen:
0.14 H2 + 0.070 O2 = 0.14H2 O
0.03 CH4 + 0.060 O2 = 0.03 CO2 + 0.06 H2 O
0.27 CO2 + 0.135 O2 = 0.27 CO2
0.265 O2
Actual O2 in product = 0.265 O2 – 0.006 O2
Actual O2 in product = 0.259 O2
Molal A/F = 0.259 + 0.259 (3.76)
1
Molal A/F = 1.233 mols air/mol of fuel

ME Board April 1995


5. A steam generator burns fuel oil that has the following
chemical analysis by mass in percent:
C = 85.3 H2 = 14.1 S = 0.5 N2 = 0.1
Calculate partial pressure of Water.

Solution:
Converting the given mass analysis to molal analysis:

C: 85.3/12 = 7.108
H2: 14.1/2 = 7.050
S: 0.5/32 = 0.016
N2 : 0.1/28 = 0.004

Combustion reaction with 125% theoretical air:


7.108C + 7.05 H2 + 0.016S + 0.004N2 + 1.25 (10.649)O2 1.25
(10.649)(3.76)N2 = 7.108CO2 + 7.05H2O + 0.016SO2 +
50.054N2 + 0.25 (10.649)O2
mT = total mols in product
= 7.108 + 7.050 + 0.016 + 50.054 + 2.662
= 66.89 mols

Partial pressure of H2O = (7.050/66.890)(170)


Partial pressure of H2O = 17.92 Kpa

ME Board April 1996

6. A volumetric analysis of a gas mixture is as follows:


CO2 : 12%
O2 : 4%
N2 : 82%
CO : 2%
What is the percentage of CO on a mass basis?

Solution:
Converting to mass basis:
CO2 = 0.12 x 44 = 5.28
O2 = 0.04 x 32 = 1.28
N2 = 0.82 x 28 = 22.96
CO = 0.02 x 28 = 0.56

Total mass of product = 5.28 + 1.28 + 22.96 + 0.56


Total mass of product = 30.08 kg
% mass of CO = 0.56 / 30.08
% mass of CO = 1.9%

ME Board October 1996

7. Calculate the theoretical air needed for the complete


combustion of ethane C2 H6 for 20 kg fuel.

Solution:
C2 H6 + O2 + (3.76)N2 CO2 + H2O + (3.76)N2
Balancing the equation:
C2 H6 + 3.5 O2 + 3.5 (3.76)N2 2CO2 + 3H2 O + 3.5(3.76)N2
Theoretical A/F = 3.5(32) + (3.5)(3.76)(28)
(2x12) + (1x6)
Theoretical A/F = 16.016 kg air/ kg fuel
Mass of air needed = 16.016 (20)
Mass of air needed = 320.32 kg

HEAT TRANSFER

ME Board October 1993

1. A 15 cm thick wall has a thermal conductivity of 5 W/m-


°K. If inside and outside surface temperature of the wall are
200°C and 30°C, respectively. Determine the heat transmitted.

Solution:

Q = kA ( t2 – t1 )
X

Q = 5 ( 200 –30 )
A 0.15

Q = 5666.67 W / m2
A

Q / A = 5.6667 KW / m2
ME Board October 1986

2. A heat exchanger is to be designed for the following


specifications:

Hot gas temperature, 1145°C


Cold gas temperature, 45°C
Unit surface conductance on the hot side, 230 W/m2 -°K
Unit surface conductance on the cold side, 290 W/m2 -°K
Thermal conductivity of the metal wall, 115 W/m-°K
Find the maximum thickness of metal wall between the hot
gas and the cold gas, so that the maximum temperature of
the wall does not exceed 545°C.
Solution:
Q = h1 A ( t1 - ta )
Q / A = h1 ( t1 – t2 )
Q / A = 230 ( 1145 – 545 )
Q /A = 138,000

Q = A( t1 – t2 )
1+x+1
h1 k h2

Q / A = ( t1 – t2 )
1+x+1
h1 k h2

138,000 = _______( 1145 – 45 )_________


(1/230) + (x / 115) + (1 / 290)
x = 0.020115 m
x = 20.115 mm

ME Board April 1983

3. A surface condenser serving a 50,000 KW steam turbo-


generator unit receives exhaust steam at the rate of 196,000
kg/hr. Vacuum in condenser is 702 mm Hg. Sea water for
cooling enters at 29.5°C and leaves at 37.5°C. For steam
turbine condenser, manufacturers consider 950 Btu/lb of steam
turbine condensed as heat given up to cooling water. Calculate
logarithmic mean temperature difference in °F.

Solution:
Condenser pressure = 101.325 – ( 702 x 101.325/760)
Condenser pressure = 7.733 Kpa.

From steam table, at 0.007733 Mpa, tsat = 40.86°C

Δta = 40.86 – 29.5


Δta = 11.36°C
Δtb = 40.86 – 37.5
Δtb = 3.36°C

tmean = 11.36°C – 3.36°C


ln (11.36 / 3.36 )
tmean = 6.56°C x 1.8
tmean = 11.82°F

ME Board October 1994

4. Calculate the energy transfer rate across 6” wall of


firebrick with a temperature differences across the wall of
50°C. The thermal conductivity of the firebrick is 0.65 Btu/hr-ft-
°F at the temperature interest.

Solution:
( ta – tb ) = 50 (9/5)
( ta – tb ) = 90°F

Q = kA( ta – tb )
X

Q / A = 0.65 (90)
(6/12)

Q /A = 117 Btu/ hr-ft2

Q/A=117 Btu/ hr-ft2 x 1.055J/Btu x 1hr/3600sec x 10.76ft2 / m2


Q/A = 369 W/m2

ME Board October 1994


4. Water is flowing in a pipe with radius of 25.4 cm at a
velocity of 5 m/sec at the temperature in the pipe. The density
and viscosity of the water are as follows: density = 997.9
kg/sec ; viscosity = 1.131 Pa-s. What is the Reynold’s Number
for this situation?

Solution:
R = D vo
V
Where: D = diameter
D = 2 (25.4)
D = 50.8 cm
D = 0.508 m
vo = velocity
vo = 5 m/sec
v = kinematic viscosity
v = 1.131 / 997.9
v = 0.0011334 m2 / sec
NR = 0.508 (5)
0.0011334

NR = 2241
ME Board April 1999

5. Compute the amount of condensate form during 10


minutes warm-up of 150 meter pipe conveys the saturated
steam with enthalpy vaporization hfg = 1,947.8 LJ/kg. The
minimum external temperature of pipe is 2°C. The final
temperature of pipe is 195°C. The specific heat of pipe material
is 0.6 KJ/kg-°C. The specific weight is 28 kg/m.

Solution:

mp = mass of pipe
mp = 28 (150)
mp = 4200 kg

Heat loss by steam = Heat loss from pipe


Ms (hg - hf ) = mp cp ( t2 – t1 )
Ms (1947.8) = (4200) (0.6) (195 – 2)
Ms = 249.69 kg

ME Board Oct Ms = 249.69 kg

ME Board Octransfer in the glass surface area of 0.7m2 having


an inside temperature (room) of 25°C and 13°C outside
temperature (surrounding). The thickness of glass surface is
0.007 m. The thermal conductivity is 1.8 W/m-°K.

Solution:

Q = kA ( t2 – t1 )
X
Q = (1.8) (0.70) (25-13)
0.007
Q = 2160 Watts

Q = 2.16 KW
CHIMNEY

ME Board April 1991

1 A boiler uses 2500 kg of coal per hour and air required for
combustion is 16 kg per coal. If ash loss is 10%, determine the
mass of gas entering the chimney.

Solution:

mg = ma + mf - mash
A/F = ma / mf
16 = ma + mf
ma = 16 mf

mg = 16mf + mf – 0.1mf
mg= 16.9 mf
mg = 16.9 (2500 kg / hr)
mg = 42,250 kg/hr

ME Board October 1991

2. The gas density of chimney is 0.75 kg/m3 and air density


of 1.15 kg/ m3 . If the driving pressure is 0.25 Kpa, determine
the height of chimney.

Solution:

hw = H ( da - dg )
0.25 = H ( 1.15 – 0.75 ) ( 0.00981)
H = 63. 71 m
ME Board April 1991

3. The actual velocity of gas entering in a chimney is 8


m/sec. The gas temperature is 25°C and pressure drop of 98
Kpa with a gas constant of 0.287 KJ/kg-°K. Determine the
chimney diameter if mass of gas is 50,000 kg/hr.

Solution:
Pg Vg = mg Rg Tg
98 (Vg) = (50,000 / 3600) (0.278) (25 + 273)
Vg = 12.12 m3 / sec
Vg = A x V
11.74 = π/4 D2 (8)
D = 1.39 m

ME Board April 1990

4. A power plant situated at an altitude having an ambient


air of 96.53 Kpa and 23.88°C. Flue gases at a rate of 5.0 kg/sec
enter the stack at 200°C and leaves at 160°C. The flue gases
gravimetric analysis are 18% CO2, 7% O2 and 75% N2 . Calculate
the height of stack necessary for a driving pressure of 0.20
Kpa.

Solution:

Solving for the molecular weight and gas constant of the flue
gas:
CO2: 18% 0.18/44 = 0.00409
O2 : 7% 0.07/32 = 0.00219
N2 : 75% 0.75/28 = 0.02678
0.03306
Mg = 1 / 0.03306 = 30.25
R g = 8.314 / 30.25 = 0.275
T g = ( 200 + 160 ) / 2 = 180°C

d g = P / RT
= _____96.53______
(0.275) (180+273)
= 0.775 kg / m3
da = P / RT
= ______96.53______
(0.287)(23.88+273)
= 1.133 kg / m3
Draft = H (da - d g )
0.20 = H (1.133 – 0.775) (0.00981)
H = 56.95 m

ME Board April 1998

5. A steam boiler plant consumes 9,000 kg of coal per hour


and produces 20 kg of dry flue gases per kg of coal fired.
Outside air temperature is 32°C, average temperature of the
flue gas entering the chimney is 343°C and average
temperature of dry flue gas in the chimney is 260°C. The gage
fluid density is 994.78 kg / m 3 and the theoretical draft of 2.286
cm of H20 at the chimney base is needed when the barometric
pressure is 760 mm HG. Determine the height of the chimney.

Solution:
hw = h x w
hw = (0.02286) (994.78)
hw = 22.74 kg / m3
da = P / R a T a
P = 760 mm Hg
P = 101.325 Kpa
da = _____101.325______
(0.287)(32+273)
= 1.157 kg / m3
dg = P / R g T g
= _____101.325______
(0.287)(260+273)
= 0.662 kg / m3
hw = H (da - dg )
22.74 = H ( 1.157 – 0.062)
H = 46 m
REFRIGERATION

ME Board October 1991

1. An air-conditioning plant with a capacity of 400 KW of refrigeration


has an evaporating and condensing temperature of 3°C and 37°C
respectively. If it uses Refrigerant 12, what will be the mass of flash gas per
kg of refrigerant circulated?

Solution:

From R-12 Tables:

h1 = hg at 3°C
h1 = 352.755 KJ/kg
hf at 3°C = 202.780 KJ/kg
v1 = 0.05047 m3/kg
h3 = h4 = hf at 37°C
h3 = h4 = 235.503 KJ/kg

Let x= mass of flash gas or quality after expansion


h3 = h4 = hf + x hfg
hfg = hg - hf
hfg = 352.755 KJ/kg – 202.780 KJ/kg
hfg = 149.975 KJ/kg
255.503 KJ/kg = 202.78 KJ/kg + x (149.975)
x = 0.218
x = 21.80%

ME Board April 1995

2. An air-conditioning system of a high-rise building has a capacity of


350 KW of refrigeration; uses R-12. The evaporator and condenser
temperature are 0°C and 35°C, respectively. Determine the work of
compression in KW.

Solution:

From R-12 Tables and P-h chart:

h1 = hg at 0°C
h1 = 351.477 KJ/kg
v1= vg at 0°C
v1 = 0.0553892 m3/kg

At 0°C:
hf = 200 KJ/kg
hfg = 351.477 KJ/kg

From R-12 Chart:

h2 = 369 KJ/kg
h3 = h4 = 233.498 KJ/kg

Refrigerating Effect = m ( h1 – h4 )
350 = m (351.477 –233.498)
m = 2.967 kg/sec

Work of compression = m (h2 - h1)


Work of compression = 2.967(369-351.477)
Work of compression = 52 KW

ME Board April 1990

3. A vapor compression refrigeration system has a 30 KW motor driving


the compressor. The compressor inlet pressure and temperature are 64.17
Kpa and 20°C respectively and discharge pressure of 960 Kpa. Saturated
liquid enters the expansion valve. Using Freon 12 as refrigerant, determine
the capacity of the unit in tons of refrigeration.

Solution:

From R-12 Tables and Chart:

h1 = 345 KJ/kg
h2 = 398 KJ/kg
h3 = h4 = 238.5 KJ/kg

Compressor work = m (h2 - h1)


30 = m (398 – 345)
m = 0.566 kg/sec

Refrigerating Capacity = m ( h1 – h4 )
Refrigerating Capacity = 0.566 (345 – 238.5)
Refrigerating Capacity = 60.279 KW

Tons of refrigeration = 60.279 / 3.516


Tons of refrigeration = 17.144 tons ref.
ME Board October 1990

4. A simple vapor compression cycle develops 15 tons of refrigeration


using ammonia as refrigerant and operating at condensing temperature of
24°C and evaporating temperature of -18°C and assuming compression
are isentropic and that the gas leaving the condenser is saturated, find the
power per ton.

Solution:

From ammonia tables and chart:

h1 = 1439.94 KJ/kg
h2 = 1665 KJ/kg
h3 = h4 = 312.87 KJ/kg
v1 = 0.572875 m3/kg

Refrigerating Effect = m (h1 – h4 )


15 (3.516) = m (1439.94 – 312.87)
m = 0.0467 kg/sec

Power requirement = m (h2 - h1)


Power requirement = 0.0467 (1665 – 1439.94)
Power requirement = 10.531 KW
Power per ton = 10.531/15
Power per ton = 0.702 KW/ton

ME Board October 1994

5. Calculate the bore in mm of a single-acting, 6 cylinder ammonia


compressor running at 1200 rpm to compress 700 kg/hr of refrigerant
which vaporized at -15°C, given the following:
a. Bore and stroke = unity
b. Volumetric efficiency = 70%
c. Specific volume of NH3 = 8.15 ft3 / lb at 5°F

Solution:

v1 = 8.15 ft3 / lb x 1 m3 / 35.31 ft3 x 2.205lb/1kg


v1 = 0.5089 m3 /kg
V1 = m v 1
V1 = (700kg/hr / 60) (0.5089 m3 /kg)
V1 = 5.937 m3 / min
VD = 5.937 / 0.7
VD = 8.481 m3 / min
VD = ∏/4 D2 L N c

For D = L (unity)

2.481 = ∏/4 (D) 2 (D) (1200) (6)


D = 0.114 m
D = 114 mm

ME Board April 1996

6. A refrigeration system operates on an ideal vapor-compression


using Refrigerant 12 with an evaporator temperature of -30°C and a
condenser exit temperature of 49.3°C and requires a 74.6 KW motor to drive
the compressor. What is the capacity of the refrigerator in tons of
refrigeration? Enthalpy of condenser entrance = 382 KJ/kg, exit = 248.15; at
evaporator entrance = 248.15, exit = 338.14 KJ/kg.

Solution:

W = m (h2 - h1)
74.6 = m (382 –338.14)
m = 1.7 kg/sec

QA = m (h1 – h4)
QA = 1.7 (338.14 – 248.15)
QA = 153 KW

Refrigerating Capacity = 153 / 3.516


Refrigerating Capacity = 43.5 tons of ref.

ME Board April 1995

7. Liquid ammonia at a temperature of 26°C is available at the


expansion valve. The temperature of the vaporizing ammonia in the
evaporator is 2°C. Find the percentage of liquid vaporized while flowing
through expansion valve.
Temp. (°C) Pres.(Kpa) hf hfg hg
2°C 462.49 190.4 1255.2 1445.6
26°C 1033.97 303.6 1162.0 1465.6

Solution:

h3 = h4 = 303.6 KJ/kg

h4 = hf + x hfg
303.6 KJ/kg = 190.4 kj?kg + x (1255.2 KJ/kg)
x = .0902

x = 9.02%

ME Board October 1997

8. A refrigeration system using R-22 has a capacity of 320 KW of


refrigeration. The evaporating temperature is -10°C and the condensing
temperature is 40°C. Calculate the fraction of vapor in the mixture before
the evaporator.

Solution:
Properties of R-22 are :
hg = 401.60 KJ/kg
hf = 188.426 KJ/kg

At 40°C
hf = 249.686 KJ/kg

h3 = h4 = hf + x(hg - hf)
249.686 = 188.426 + x (401.6 – 188.426)
x = 0.287

ME Board October 1999

9. What is the coefficient of a vapor compression refrigeration system


having the following data: Enthalpy entering the compression is 181.79;
enthalpy after compression work is 207.3. After condensation the enthalpy
is 58.2 and the throttled from 0.19 Mpa to 0.18 Mpa.

Solution:

COP = h1 – h4
h2 – h 1
COP = 181.79 – 58.2
207.3 – 181.79

COP = 4.84

ME Board October 1999


10. A refrigeration system having a 22 KW capacity needs 7.8 hp
compressor. Find the COP of the system.

Solution:

COP = RE
Wc

COP = 22KW
(7.8x0.746)

COP = 3.78
CHAPTER 3 – DIESEL PLANT D. Fourth stroke 12. The portion of the piston which extends below the
piston pin and serves as a guide for the piston and
1. An engine in which the fuel is burned directly Answer: A connecting rod.
within the working cylinder. A. Piston skirt
A. Internal combustion engine 7. Compressing air in a four stroke cycle is B. Piston ring
B. External combustion engine A. First stroke C. Piston scoring
C. Compression ignition engine B. Second stroke D. Piston seizure
D. Spark ignition engine C. Third stroke
D. Fourth stroke Answer: A
Answer: A
Answer: B 13. Rings located in the grooves of the piston usually
2. An engine in which the fuel is burned outside of near the top and near the bottom.
the power cylinder. A. Piston skirt
A. Internal combustion engine B. Piston ring
B. External combustion engine 8. Which of the following strokes is produced by the C. Piston scoring
C. Compression ignition engine burning gases: D. Piston seizure
D. Spark ignition engine A. First stroke
B. Second stroke Answer: B
Answer: B C. third stroke
D. fourth stroke 14. Binding of the piston and the cylinder wall as a
3. An engine where the ignition is caused by heat of result of the lubrication having been destroyed by
compression. Answer: C excessive temperature and friction.
A. Internal combustion engine A. Piston skirt
B. External combustion engine 9. Which of the following strokes expels the burned B. Piston ring
C. Compression ignition engine gases? C. Piston scoring
D. Spark ignition engine A. First stroke D. Piston seizure
B. Second stroke
Answer: C C. Third stroke Answer: D
D. Fourth stroke
4. What temperature is required to ignite the fuel oil? 15. Grooves in the cylinder wall or piston or in both. It
A. 800 to 1000F Answer: D is caused by the piston scraping the cylinder wall
B. 700 to 900F in its movement without proper lubrication.
C. 900 to 1100F 10. Clearing a cylinder of exhaust gases by forcing A. Piston skirt
D. 1000 to 1200F into it a current of air which provides clean air for B. Piston ring
the next compression stroke. C. Piston scoring
Answer: A A. Scavenging D. Piston seizure
B. Supercharging
5. What air pressure is required to produce the C. Choking Answer: C
required ignition temperature? D. Knocking
A. 350 to 500 psi 16. A device which automatically governs or controls
B. 250 to 400 psi Answer: A the speed of an engine.
C. 450 to 600 psi A. Servomotor
D. 150 to 300 psi 11. Increasing the total amount of charging air in the B. Governor
working cylinder of the engine. C. Indicator
Answer: A A. Scavenging D. Speedometer
B. Supercharging
6. In a four stroke Diesel engine, drawing air into the C. Choking Answer: B
cylinder is what stroke? D. Knocking
A. First stroke 17. A combination of liquids which do not mix - or
B. Second stroke Answer: B combine chemically.
C. Third stroke A. Emulsion
B. Deposition 23. A chamber so proportioned with respect to the Answer: A
C. Evaporation clearance volume proper of the cylinder that only
D. Separation about 30% of the combustion takes place within 28. What are the exhaust gas temperatures at normal
the chamber itself. operating conditions?
Answer: A A. Pre combustion chamber A. Between 400 and 700F
B. Separate combustion chamber B. Between 500 and 800F
18. What temperature should be the maximum to C. Ante combustion chamber C. Between 300 and 600F
which lubricating oil is permitted to rise? D. Mixed combustion chamber D. Between 200 and 500F
A. Not more than 100F
B. Not more than 200F Answer: A Answer: A
C. Not more than 300F
D. Not more than 400F 24. The entire charge is ignited in the separate 29. How are high speed Diesel engines started?
combustion chamber before the initial expansion A. By an electric motor
Answer: A takes place, forcing the burning gases through the B. By compressed air
connecting passages and against the moving C. By diesel fuel
19. What air pressure is needed for air starting a piston. D. By an electric spark
Diesel engine? A. Pre combustion chamber
A. 250 psi B. Separate combustion chamber Answer: A
B. 150 psi C. Ante combustion chamber
C. 350 psi D. Mixed combustion chamber 30. What speeds are considered high speed diesel
D. 450 psi engine?
Answer: B A. 100 to 2000 rpm or 3000 rpm
Answer: A B. 100 to 1000 rpm
25. A chamber so designed that injection takes place C. 100 to 1500 rpm
20. At what temperature will self-igniting cartridges directly opposite its outlet, the chamber extending D. 100 to 1750 rpm
ignite? backward from the outlet.
A. About 200F A. Pre combustion chamber Answer: A
B. About 190F B. Separate combustion chamber
C. About 210F C. Ante combustion chamber 31. It is a device used for atomizing or cracking fuel oil
D. About 250F D. Air cell chamber and through which the fuel oil Is injected into the
working cylinders of Diesel engines.
Answer: B Answer: C A. Atomizer
B. Fuel spray nozzle
21. The fuel is supplied by one pump and switched to 26. Fuel is injected only into the main cylinder, during C. Injector
each cylinder by a multi outlet rotating valve or expansion of the burning gases in the main D. Cracker
distributor. chamber, when the pressure therein drops below
A. Distributor system that of the air in the cell. Answer: A
B. Injector system A. Pre combustion chamber
C. Non injector system B. Separate combustion chamber 32. What is the proper seat width of a spray valve?
D. Non distributor system C. Ante combustion chamber A. 1/16 in.
D. Air cell chamber B. 1/8 in.
Answer: A C. 1/32 in.
Answer: D D. ¼ in.
22. This is virtually a distributor two stage pumping
system that is, modified distributor system. 27. What is the temperature of the air in the cylinder Answer: A
A. Distributor system when the Diesel engine is operating at full load?
B. Injector system A. Between 800 and 1100F 33. A cylindrical vessel connected in the fuel system
C. Non injector system B. Between 800 and 1000F to absorb the shock of the injection pumps and to
D. Non distributor system C. Between 900 and 1200F provide a reservoir at high pressure for supplying
D. Between 900 and 1100F an even flow of fuel oil to the cylinders.
Answer: B A. Absorber
B. Accumulator 39. The ratio of the clearance volume to the swept C. 14 to 19
C. Governor volume. D. 10.5 to 14.5
D. Injector A. Fractional clearance
B. Compression ratio Answer: A
Answer: B C. Expansion ratio
D. Cut off ratio 45. In standard Diesel, thermal efficiency was not
34. How hot are the exhaust gases? much higher than about what percent?
A. 400 to 700F Answer: A A. 35%
B. 300 to 600F B. 30%
C. 200 to 500F 40. The piston is at maximum reach from the C. 40%
D. 500 to 800F crankshaft to which of the following? D. 50%
A. Top dead center
Answer: A B. Bottom dead center Answer: A
C. ¾ from top center
35. Which of the following is an automatic device used D. ¾ from bottom center 46. A closed heat exchanger that transfers heat from
for keeping constant air pressure? compressed air to cooler air.
A. Relief valve Answer: A A. Regenerator
B. Unloader B. Intercooler
C. Strainer 41. The piston is closest to the crankshaft to which of C. Aftercooler
D. Barometer the following? D. Reheater
A. Top dead center
Answer: B B. Bottom dead center Answer: C
C. ¾ from top center
36. What is the other term of the diameter of the D. ¾ from bottom center 47. Which of the following is used to improve cold
circular cylinder? weather starting?
A. Bore Answer: B A. Poppet valve
B. Stroke B. Glow plug
C. Swept volume 42. Which of the following engines where the C. Check valve
D. Clearance expanding combustion gases act on one end of D. Spark plug
the piston?
Answer: A A. Single acting engine Answer: B
B. Double acting engine
37. What do you call the maximum distance traveled C. Single expansion engine 48. The rated power that the manufacturer claims the
by the piston? D. Double expansion engine engine is able to provide on a continuous basis
A. Bore without incurring damage.
B. Stroke Answer: A A. Brakepower
C. Swept volume B. Continuous duty rating
D. Clearance 43. Which of the following types of engine where the C. Intermittent rating
expanding combustion gases act on both ends of D. Power rating
Answer: B the piston?
A. Single acting engine Answer: B
38. Which of the following is the product of the B. Double acting engine
cylinder area and stroke? C. Single expansion engine 49. What is the peak power rating that can be
A. Bore D. Double expansion engine produced on an occasional basis?
B. Stroke A. Brakepower
C. Swept volume Answer: B B. Continuous duty rating
D. Clearance C. Intermittent rating
44. The compression ratio of a Diesel engine varies D. Power rating
Answer: C from about:
A. 13.5 to 17.5 Answer. C
B. 8 to 10
50. Which of the following is the value of a property 5. A regenerator in a gas turbine: 11. Which of the following turbine has least weight per
that includes the effect of friction? A. Reduces heat loss during exhaust bhp developed?
A. Brake value B. Allows use of higher compression ratio A. Simple open cycle gas turbine
B. Friction value C. Improves thermal efficiency B. Open cycle gas turbine with inter-cooling and
C. Indicated value D. Allows use of fuels of inferior quality reheating
D. Actual value C. Open cycle gas turbine with inter-cooling,
Answer: C reheating and regenerating
Answer. A D. Closed cycle gas turbine
6. Which of the following compressors is generally
CHAPTER 4 – GAS TURBINE used for gas turbines? Answer: A
A. Lobe type
1. Brayton cycle has: B. Centrifugal type 12. Which of the following is/are advantage of closed
A. Two isentropic and two constant volume C. Axial flow type cycle gas turbine over open cycle gas turbine?
processes D. Reciprocating type A. No containing of working substance with
B. Two isentropic and two constant pressure combustion gases
processes Answer: C B. Inferior quality fuel can be used
C. One constant pressure, one constant volume C. Low maintenance costs
and two adiabatic processes 7. The constant pressure gas turbine works on the D. All of the above
D. Two isothermals, one constant volume and a principle of:
constant pressure process A. Carnot cycle Answer: D
B. Bell-Coleman cycle
Answer: B C. Rankine cycle 13. The range of compression ratio in a gas turbine is:
D. Brayton cycle A. 3 to 5
2. Brayton cycle cannot be used in reciprocating B. 5 to 8
engines even for same adiabatic compression Answer: D C. 8 to 12
ratio and work output because: D. 12 to 20
A. Otto cycle is highly efficient 8. What type of gas turbine is used in air craft?
B. Brayton cycle is less efficient A. Open cycle type Answer: B
C. Brayton cycle is for slow speed engines only B. Closed cycle type with reheating
D. Large volume of low pressure air cannot be C. Closed type with reheating and regeneration 14. A constant volume combustion gas turbine
efficiently handled in reciprocating engines D. Open cycle type with reheating, regeneration operates on:
and intercooling A. Ericson cycle
Answer: D B. Joule cycle
Answer: A C. Brayton cycle
3. Which cycle is generally used for gas turbine? D. Atkinson cycle
A. Otto cycle 9. In a gas turbine combined cycle plant, a waste
B. Dual cycle heat boiler is used to: Answer: D
C. Carnot cycle A. Heat air from intercooler
D. Brayton cycle B. Gases from regenerator 15. Past ME Board Question
C. Recover from regenerator Heat exchanger used to provide heat transfer
Answer: D D. None of the above between the exhaust gases and the air prior to its
entrance to the combustor.
4. When r is the compression ratio, the efficiency of Answer: C A. Evaporator
Brayton cycle is given by: B. Combustion chamber
A. 1-1/𝑟 (𝑘−1)/𝑘 10. Overall efficiency of a gas turbine is: C. Regenerator
B. 1-1/𝑟 𝑘−1 A. Equal to Carnot cycle efficiency D. Heater
C. 1-1/𝑟 𝑘 B. Equal to Rankine cycle efficiency
D. 1-1/r C. Less than Diesel cycle efficiency Answer: C
D. More than Otto or Diesel cycle efficiency
Answer: A 16. Past ME Board Question
Answer: C
How does the value for work per unit mass flow of 21. Large units gas turbine regularly operate: 27. The compression ratio based on pressures of
air in the compressor and turbine influenced by the A. In 100 to 200 MW range heavy duty gas turbine is in the range of ____.
addition of a regenerator? B. In 50 to 100 MW range A. 14 to 15
A. Slightly increased C. Over 150 MW B. 19 to 21
B. Unchanged D. Below 150 MW C. 11 to 16
C. Greatly decreased D. 16 to 18
D. Greatly increased Answer: A
Answer: A
Answer: B 22. Small units gas turbine typically have:
A. Double shafts 28. Aeroderivative combustion turbine have higher
17. Past ME Board Question B. Single shafts compression ratios typically:
What is the ideal cycle for gas turbine work? C. Triple shafts A. 14 to 15
A. Brayton cycle D. Quadruple shafts B. 19 to 21
B. Stag combined cycle C. 11 to 16
C. Bottom cycle Answer: B D. 16 to 18
D. Ericson cycle
23. Heavy duty gas turbines typically have: Answer: B
Answer: A A. Double shafts
B. Single shaft 29. Most heavy duty combustion turbines have how
18. Past ME Board Question C. Triple shafts many compression stages?
Brayton cycle cannot be used in reciprocating D. Quadruple shafts A. 14 to 16
engines even for same adiabatic compression B. 18 to 20
ratio and work output because: Answer: B C. 10 to 12
A. Brayton cycle is highly efficient D. 16 to 18
B. Brayton cycle is for low speed engines only 24. Which of the following is basically a jet engine that
C. Brayton cycle needs large air-fuel ratio exhausts into a turbine generator? Answer: D
D. Large volume of low pressure air cannot be A. Aeroderivative gas turbine
efficient handled in reciprocating engines B. Industrial gas turbine 30. The temperature of the gas entering the expander
C. Brayton engine section is typically:
Answer: D D. Joule turbine A. 1200 C to 1290 C
B. 1000 C to 1200 C
19. Past ME Board Question Answer: A C. 1500 C to 1490 C
In order to increase the gas velocity gas turbines D. 1300 C to 1390 C
generally have fixed nozzles. This is to allow the: 25. Most aeroderivative combustion turbine produce
A. Compression of gases less than: Answer: A
B. Condensation of gases A. 20 MW
C. Expansion of gases B. 30 MW 31. The exhaust temperature which makes the
D. Evaporation of gases C. 40 MW exhaust an ideal heat source for combined cycles
D. 50 MW is typically:
Answer: C A. 540 C to 590 C
Answer: C B. 600 C to 650 C
20. Combustion turbines or gas turbines are the C. 300 C to 350 C
preferred combustion engines in application much 26. The compression ratio based on pressures in the D. 440 C to 490 C
above ______. compression stage in a gas turbine is typically:
A. 8MW A. 11 to 16 Answer: A
B. 9mw B. 5 to 8
C. 10MW C. 12 to 18 32. Most combustion turbines have:
D. 7MW D. 8 to 14 A. 2 to 3 expander stages
B. 3 to 4 expander stages
Answer: C Answer: A C. 4 to 5 expander stages
D. 1 to 2 expander stages
Answer: B A. 34 to 36 % Answer: A
B. 30 to 32 %
33. The exhaust flow rate in modern heavy duty C. 40 to 42 % 44. A regenerator in a gas turbine has no effect in:
turbines per 100 MW is approximately: D. 26 to 28 % A. Compressor and turbine work
A. 240 to 250 kg/s B. Heat added
B. 140 to 150 kg/s Answer: A C. Thermal efficiency
C. 340 to 350 kg/s D. Combustor
D. 440 to 450 kg/s 39. New combustion turbines on the cutting edge of
technology ( Advanced turbine systems ) are able Answer: A
Answer: A to achieve
A. 38 to 38.5 % 45. In a Brayton cycle multiple stages of compression
34. The Brayton gas turbine cycle is also known as: B. 36 to 36.5 % and expansion will ________.
A. Joule cycle C. 40 to 40.5 % A. Increase thermal efficiency
B. Stirling cycle D. 34 to 34.5 % B. Decrease thermal efficiency
C. Ericsson cycle C. Limit thermal efficiency
D. Atkinson cycle Answer: A D. Control efficiency

Answer: A 40. Aeroderivative turbines commonly achieve Answer: A


efficiencies up to:
35. Approximately how many percent of the turbine A. 42 % 46. In a Brayton cycle, reheating and intercooling will
power is used to drive the high efficiency B. 38 % ___________.
compressor? C. 45 % A. Increase thermal efficiency
A. 50 to 75 % D. 35 % B. Decrease thermal efficiency
B. 60 to 85 % C. Limit thermal efficiency
C. 45 to 70 % Answer: A D. Control efficiency
D. 30 to 55 %
41. Which of the following is the typical backwork ratio Answer: A
Answer: A of gas turbines?
A. 50 to 75 % 47. In a Brayton cycle, reheating has no effect in:
36. Depending on the turbine construction details, the B. 40 to 65 % A. Heat added
temperature of the air entering the turbine will be C. 30 to 55 % B. Thermal efficiency
between: D. 35 to 60 % C. Backwork ratio
A. 650 C to 1000 C D. Network
B. 750 C to 1100 C Answer: A
C. 550 C to 950 C Answer: A
D. 850 C to 1200 C 42. Which of the following is an example of a
regenerator? 48. In a Brayton cycle, intercooling has no effect in:
Answer: A A. A counterflow heat exchanger A. Turbine work
B. A cross flow heat exchanger B. Thermal efficiency
37. Which of the following engines are typically used C. A mixed flow heat exchanger C. Backwork ratio
by Turbojet and turboprop? D. A parallel flow heat exchanger D. Network
A. Open combustors
B. Closed combustors Answer: B Answer: A
C. Turbo combustors
D. High combustors 43. Which of the following is an effect of having a 49. If Wt is the turbine power and Wc is the
regenerator? compressor power then the backwork ratio is
Answer: A A. Less heat is added A. Wt / Wc
B. Compressor work is reduced B. Wc / Wt
38. The full load thermal efficiency of existing heavy C. Turbine work is increased C. Wt - Wc
duty combustion turbines in simple cycles is D. Compressor work is increased Wt
approximately: D. Wt - Wc
Wc 4. A high pressure chamber or a device in which the Answer: A
paths of rapidly moving particles can be observed
Answer: B and photographed. 10. The lowest permissible water level of a boiler
A. Cloud chamber without internal furnace is _____the height of the
50. If Wt is the turbine power and Wc is the B. Combustion chamber shell.
compressor power then the network is: C. Fission chamber A. 1/2
A. Wt x Wc D. Air chamber B. 1/3
B. Wc / Wt C. 1/4
C. Wt - Wc Answer: A D. 1/5
D. Wt + Wc
5. The formation of gas bubbles in a liquid is called Answer: B
Answer: C A. Bubbling
B. Foaming 11. In case of steam engine the cut off ratio is the ratio
51. Physical limitations usually preclude more than C. Priming of:
how many stages of intercooling and reheating? D. Carryover A. Pressure at cut off to supply pressure
A. 2 B. Pressure at cut off to exhaust pressure
B. 3 Answer: B C. Pressure at cut off to mean effective pressure
C. 4 D. Fraction of piston stroke which the piston has
D. 5 6. How many check valves should be provided traveled when cut off occurs
between any feed pump and boiler?
Answer: A A. 1 Answer: D
B. 2
CHAPTER 5 – STEAM POWER PLANT C. 3 12. In a condensing steam engine
D. 4 A. Condensed steam is supplied
1. Which of the following factors does bursting B. Steam condenses inside cylinder
pressure of boiler doesn’t depend? Answer: B C. Steam condenses as soon as it leaves the
A. Tensile strength of the shell cylinder
B. Thickness of the shell 7. The water level inside the boiler is indicated by the D. Exhaust steam is condensed in a condenser
C. Diameter of the shell A. Baffles
D. Shear strength of shell material B. Fusible plug Answer: D
C. Water walls
Answer: D D. Water column 13. Flows through the nozzles and diffusers with
increasing fluid velocity will create an equivalent
2. Which of the following factors does working Answer: D A. Decrease in the static enthalpy of fluid
pressure of boiler doesn’t depend? B. Increase in the static enthalpy of fluid
A. Tensile strength of shell 8. What is the highest pressure under which C. Decrease in the internal energy of fluid
B. Thickness of shell distinguishable liquid and vapor phases can exist D. Decrease in the dynamic enthalpy of fluid
C. Factor of safety in equilibrium?
D. Type of fuel being fired A. Maximum pressure Answer: A
B. Atmosphere
Answer: D C. Critical pressure 14. The term V 2 /2Cp responds to the temperature rise
D. Peak pressure during such a process and is called the
3. Total solid impurities in feed water for a boiler A. Kinetic temperature
depend upon Answer: C B. High temperature
A. Boiler pressure C. Dynamic temperature
B. Quantity of steam to be generated 9. What is the average fuel – oil temperature range D. Elevation temperature
C. Type of fuel available of the oil in the discharge line to the boiler?
D. Quantity of steam A. 180 – 200 F ͦ Answer: C
B. 240 – 260 F ͦ
Answer: A C. 160 – 180 F ͦ 15. All of the following mechanism can supply heat to
D. 140 – 160 F ͦ a thermodynamic system except
A. Conduction 21. A converging – diverging nozzle is the standard A. Dalton’s line
B. Natural convection equipment in: B. Willan’s line
C. Adiabatic expansion A. Subsonic aircraft C. Jonval’s line
D. Radiation B. Supersonic aircraft D. Rankine line
C. Hypersonic aircraft
Answer: C D. Trisonic aircraft Answer: B

16. The flow through the nozzle is Answer: B 27. An inventor proposes to develop electrical power
A. Isentropic by withdrawing heat from the geyser fields of
B. Polytropic 22. For back pressure valves, abrupt changes in fluid northern California and converting it all to work in
C. Isobaric properties occur in a very thin section of power turbines. The scheme will not work
D. Isovolumic converging – diverging nozzle under supersonic because:
flow conditions, creating A. The geyser fields have only a limited lifetime
Answer: A A. Sound wave B. The salinity of the steam is too great
B. Tidal wave C. It violates the first law of thermodynamics
17. If the reservoir is sufficiently large, the nozzle inlet C. Shock wave D. It violates the second law of thermodynamics
velocity is D. None of these
A. Maximum Answer: D
B. Negative Answer: C
C. Positive 28. The isentropic efficiency of a turbine is given by
D. Zero 23. Is the locus of states which have the same value A. The ratio of actual to ideal energy extracted
of stagnation enthalpy and mass flux are called: B. The ratio of actual to ideal energy inputted
Answer: D A. Fanno line C. The ratio of ideal to actual energy extracted
B. Straight line D. None of the above
18. Which of the following is the pressure applied at C. Willan’s line
the nozzle discharge section? D. Cross cut line Answer: A
A. Stagnant pressure
B. Critical pressure Answer: A 29. Past ME Board Question
C. Back pressure Which of the following is not a main part of a
D. Atmospheric pressure 24. Combination of mass and momentum equations typical coal burner?
into a single equation and plotted in h-s plane yield A. Air registers
Answer: C a curve called: B. Nozzle
A. Fair line C. Atomizer
19. When the back pressure is reduced to lowest exit B. Freh line D. Ignitor
pressure, the mass flow reaches a maximum C. Cutting line
value and the flow is said to be: D. Rayleigh line Answer: C
A. Stacked
B. Choked Answer: D 30. Past ME Board Question
C. Stuck-up Measure of ability of a boiler to transfer the heat
D. Clog-up 25. Generally steam turbines in power station operate given by the furnace to the water and steam is:
at A. Grate efficiency
Answer: B A. 3000 rpm B. Stroke efficiency
B. 1000 rpm C. Furnace efficiency
20. An increase in stagnation pressure will increase C. 4000 rpm D. Boiler efficiency
the mass flux through the: D. 575 rpm
A. Diverging nozzle Answer: D
B. Converging nozzle Answer: A
C. Converging – diverging nozzle 31. Past ME Board Question
D. None of these 26. Which of the following shows the relationship of A goose neck is installed in the line connecting a
the steam consumption and the load of steam steam gauge to a boiler to:
Answer: B turbine generator? A. Maintain constant steam flow
B. Protect the gauge element 36. Past ME Board Question Answer: D
C. Prevent steam knocking The process in which heat energy is transferred to
D. Maintain steam pressure a thermal energy storage device is known as: 41. Past ME Board Question
A. Adiabatic Is one whose pressure is higher than the
Answer: B B. Regeneration saturation pressure corresponding to its
C. Intercooling temperature.
32. Past ME Board Question D. Isentropic A. Compressed liquid
Which of the following is a great advantage of a B. Saturated liquid
fire tube boiler? Answer: B C. Saturated vapor
A. Steam pressure is not ready D. Superheated vapor
B. Contains a large volume of water and 37. Past ME Board Question
requires a long interval of time to raise steam When the boiler pressure increases or when the Answer: A
and not so flexible as to changes in steam exhaust pressure decreases, the amount of
demand moisture 42. Past ME Board Question
C. Cannot use impure water A. Increases In a steam generator with good combustion
D. Radiation losses are higher because fire is B. Decreases control, what occurs if the load is increased?
inside the boiler and surrounded by water C. Constant A. Air temperature leaving air heater decreases
D. Zero B. Air temperature entering air heater increases
Answer: B C. Furnace pressure is approximately constant
Answer: A D. Economizer gas outlet temperature
33. Past ME Board Question decreases
One of the following tasks which is an example of 38. Past ME Board Question
preventive maintenance is When the number of reheat stages in a reheat Answer: C
A. Cleaning the cup on a rotary cup burner cycle is increased, the average temperature
B. Cleaning a completely clog oil strainer A. Increases 43. Past ME Board Question
C. Replacing a leaking valve B. Decreases Total solid impurities in feed water for a boiler
D. Replacing a blown fuse C. Is constant depend upon
D. Is zero A. Boiler pressure
Answer: A B. Type of fuel available
Answer: A C. Quantity of steam to be generated
34. Past ME Board Question D. Quantity of steam
The carbon dioxide ( CO2 ) percentage in the flue 39. Past ME Board Question
gas of an efficiency fired boiler should be A heat transfer device that reduces a Answer: A
approximately thermodynamic fluid from its vapor phase to its
A. 1 % liquid phase such as in vapor compression 44. Past ME Board Question
B. 12 % refrigeration plant or in a condensing steam power The gaseous state of water
C. 18 % plant A. Water gas
D. 20 % A. Flash vessel B. Blue gas
B. Cooling tower C. Water vapor
Answer: B C. Condenser D. Yellow gas
D. Steam separator
35. ME Board October 1993 Answer: C
When droplets of water are carried by steam in the Answer: C
boiler 45. A liquid boils when it vapor pressure equals
A. Priming 40. Past ME Board Question A. The gage pressure
B. Foaming A simultaneous generation of electricity and steam B. The critical pressure
C. Carryover (or heat) in a single power plant C. The ambient pressure
D. Embrittlement A. Gas turbine D. One standard atmosphere
B. Steam turbine
Answer: A C. Waste heat recovery Answer: C
D. Cogeneration
46. Past ME Board Question D. Ion exchange treatment D. Decrease in the temp. and pressure of fluid
What are the main components in a combined
cycle power plant? Answer: B Answer: B
A. Diesel engine and air compressor
B. Gas engine and waste heat boiler 51. Past ME Board Question 56. Past ME Board Question
C. Steam boiler and turbine The thermal efficiency of gas-vapor cycle as How can the average temperature during heat
D. Nuclear reactor and steam boiler compared to steam turbine or gas turbine is rejection process of a Rankine cycle be
A. Greater than decreased?
Answer: B B. Less than A. Increase boiler pressure
C. Lower than B. Increase turbine pressure
47. Past ME Board Question D. Equal to C. Increase condenser pressure
A change in the efficiency of combustion in a D. Reduce turbine exit pressure
boiler can usually be determined by comparing the Answer: A
previously recorded readings with the current Answer: D
readings of the _________. 52. Past ME Board Question
A. Stack temperature and CO A rapid increase in boiler pressure occurs when 57. Past ME Board Question
B. Over the fire draft and CO there is: Which of the following ascertains the effectiveness
C. Ringleman chart and CO2 A. Moderate drop in steam load and the size of a condenser?
D. Stack temperature and CO2 B. Constant drop in steam load A. Number of passes
C. Abrupt drop in steam load B. Thickness of the shell
Answer: D D. Gradual drop in steam load C. Tube sizes
D. Heat transfer
48. Past ME Board Question Answer: C
A boiler steam gauge should have a range of at Answer: D
least 53. Past ME Board Question
A. One half the working steam pressure The most economical and low maintenance cost 58. Past ME Board Question
B. 1 and ½ times the maximum allowable condenser. A boiler has a bursing pressure, BP of 600 kPa
working pressure A. Water – cooled and a factor of safety, FS of 8 is employed in
C. The working steam pressure B. Air – cooled design. As an engineer, would you advice to have
D. Twice the maximum allowable working C. Evaporative a working pressure, WP of 500 kPa?
pressure D. Sub – cooled A. No. WP must be higher than 500 kPa
B. No. WP is only 75 kPa at FS of 8
Answer: B Answer: B C. Yes. Since BP is 600 Pa
D. Yes. To attain better efficiency
49. Past ME Board Question 54. Past ME Board Question
In a water tube boiler, heat and gases of What is commonly done to system when the Answer: B
combustion passed: turbine has excessive moisture?
A. Through the combustion chamber only A. Frosting 59. Past ME Board Question
B. Through the tubes B. Diffusing What cycle is used in vapor cycle of steam power
C. Away from tubes C. Reheating plant?
D. Around the tubes D. Dehumidifying A. Brayton cycle
B. Diesel cycle
Answer: D Answer: C C. Ericsson cycle
D. Rankine cycle
50. Past ME Board Question 55. Past ME Board Question
A chemical method of feedwater treatment which What is the result when the fluid’s kinetic energy Answer: D
uses calcium hydroxide and sodium carbonate as during a stagnation process is transformed to
reagents enthalpy?
A. Thermal treatment A. Decrease in fluids volume
B. Lime soda treatment B. Rise in the temp. and pressure of the fluid 60. Past ME Board Question
C. Demineralization process C. Rise in fluid’s volume Gauge cock in the boiler is designed to determine:
A. Level of steam Answer: B C. X-rays
B. Specific heat D. Photons
C. Level of water 6. A dense, fine grained, light colored igneous rock
D. Pressure which is rich in silica. Answer: C
A. Felsite
Answer: C B. Feldspar 12. In geothermal power plants waste water is:
C. Flint A. Recirculated after cooling in cooling lowers
CHAPTER 6 – GEOTHERMAL PLANT D. Flux B. Discharged into sea
C. Evaporated in ponds
1. Refers to the internal heat from the earth. Answer: A D. Discharged back to earth
A. Geothermal
B. Thermal energy 7. An opening in lava or in volcanic area through Answer: D
C. Molten heat which steam and other hot gases are escaping
D. Tectonic heat into the air is called: 13. Past ME Board Question
A. Fumarole In a liquid-dominated geothermal plant, what
Answer: A B. Volcanic leaks process occurs when the saturated steam passes
C. Seismic outlets through the turbine?
2. A rock – forming crystalline mixed silicate which D. Seismic leaks A. Isobaric
constitute about 60% of the earth’s surface. B. Polytropic
A. Soil Answer: A C. Isometric
B. Feldspar D. Isentropic
C. Flux 8. Tidal power is the power generated from:
D. Flint A. Waves of the ocean Answer: B
B. Rise and fall tides
Answer: B C. Thermal energy of ocean water 14. Past ME Board Question
D. Raw sea water What do you call a conversion technology that
3. A compound rock, a crypto – crystalline form of yields electricity straight from sunlight without the
silica, which is dens, tough, breaking with a Answer: B aid of a working substance like gas or steam
conchoidal fracture. without the use of any mechanical cycle?
A. Gravel 9. Converts chemical energy directly into electrical A. Power conversion
B. Stalactite energy. B. Stirling cycle conversion
C. Flint A. Fuel cell C. Solar thermal conversion
D. Flux B. Magnetohydrodynamic generator D. Photovoltaic-energy conversion
C. Battery
Answer: C D. Thermoelectric generator Answer: D

4. Is the process of using injection wells to bubble air Answer: A 15. Past ME Board Question
through groundwater. Tidal power plant is attractive because it has:
A. Air stripping 10. Generates a voltage from incident light, usually A. Low head and intermittent power
B. Staged combustion light in the visible region. B. High head
C. Sparging A. Photovoltaic cell C. Cheap energy source
D. Soil washing B. Solar cell D. Expensive energy source
C. Dry cell
Answer: C D. A or B Answer: C

5. A popular term used by utilities to mean upgrading Answer: D 16. Past ME Board Question
existing plant. What do you call a conversion technology that
A. Replanting 11. All of the following terms are synonymous with yields electricity straight from sunlight without the
B. Repowering quanta of electromagnetic theory except: aid of a working substance like gas or steam
C. Recharging A. Packets without the use of any mechanical cycle?
D. Reorganizing B. Corpuscles A. Power conversion
B. Stirling cycle conversion 22. The flow process through shock waves is highly 28. Solar energy is captured in:
C. Solar thermal conversion irreversible and cannot be approximated as being: A. Sun capturer
D. Photovoltaic-energy conversion A. Polytropic B. Solar collector
B. Isometric C. Sun collector
Answer: D C. Hyperbolic D. Greenhouse capturer
D. Isentropic
17. Past ME Board Question Answer: B
Tidal power plant is attractive because it has: Answer: D
A. Low head and intermittent power 29. Which of the following solar collectors are
B. High head 23. Beaufort scale is used for measuring what? essentially wide, flat boxes with clear plastic or
C. Cheap energy source A. Beta and gamma radiations glass coverings known as the glazing.
D. Expensive energy source B. Wind speed A. Flat plate collectors
C. Insolation B. Concentrating collectors
Answer: C D. Depth of sea C. Focusing collectors
D. Evacuated tube collectors
18. Tidal power is the power generated from: Answer: B
A. Waves of the ocean Answer: A
B. Rise and fall tides 24. Betz law is widely used in:
C. Thermal energy of ocean water A. MHD systems 30. Which of the following collectors use mirrors
D. Raw sea water B. Solar cells and/or lenses to focus the suns energy on a small
C. Geothermal power plants absorber area.
Answer: B D. Wind mills A. Flat plate collectors
B. Concentrating collectors
19. Converts chemical energy directly into electrical Answer: D C. Non focusing collectors
energy. D. Evacuated tube collectors
A. Fuel cell 25. Rocks having excessive internal stresses may
B. Magnetohydrodynamic generator produce spalling. These rocks are called as ____ Answer: B
C. Battery A. Stratified rocks
D. Thermoelectric generator B. Popping rocks 31. Which of the following collectors are more
C. Crushed rocks complex but their efficiencies are higher?
Answer: A D. Swelling rocks A. Flat plate collectors
B. Concentrating collectors
20. Generates a voltage from incident light, usually Answer: A C. Focusing collectors
light in visible region. D. Evacuated tube collectors
A. Photovoltaic cell 26. Solar energy arrives at the outside of the earth’s
B. Solar cell atmosphere at an average rate of ________. Answer: D
C. Dry cell A. 1.354 kW/m2
D. A or B B. 1.543 kW/m2 32. Which of the following collectors are useful when
C. 2.354 kW/m2 extremely hot transfer fluid is needed and are
Answer: D D. 2.543 kW/m2 generally limited to commercial projects?
A. Flat plate collectors
21. The statement that the product of the error in the Answer: A B. Concentrating collectors
measured determination of a particle’s position C. Focusing collectors
and its momentum is of the order of Planck’s 27. How many percent of solar energy survives D. Evacuated tube collectors
constant h is known as: absorption and reflection?
A. Bohr’s theory A. 40 to 70 % Answer: D
B. D’Alembert’s paradox B. 30 to 60 %
C. The Heisenberg uncertainty principle C. 50 to 80 % 33. The shading factor in calculating the heat
D. Planck’s law D. 20 to 50 % absorbed by the solar collector has a value of
approximately:
Answer: C Answer: A A. 0.95 to 0.97
B. 0.85 to 0.87 Answer: A B. 150 to 250 MW
C. 0.75 to 0.77 C. 100 to 300 MW
D. 0.65 to 0.67 39. Trough electric technology is relatively mature but D. 150 to 400 MW
due to the low temperatures, average annual
Answer: A thermal efficiencies are only: Answer: C
A. 10 to 15 %
34. The ratio of energy absorbed by the transfer fluid B. 5 to 10 % 45. Thermal efficiency of a geothermal power plant is
to the original incident energy striking the collector. C. 15 to 20 % approximately:
A. Collector efficiency D. 25 to 30 % A. 30 %
B. Sun efficiency B. 22 %
C. Shading factor Answer: A C. 35 %
D. Absorptance D. 15 %
40. A field of heliostats or tracking mirrors
Answer: A concentrates solar energy onto a receiver on a Answer: B
central tower.
35. As the difference between the ambient air and A. Trough electric system 46. A flash steam cycle can be used if the hot water
average plate (or inlet) temperatures increases, B. Distributed collector system temperature is approximately:
what happens to the collector efficiency? C. Power tower system A. 165 C or higher
A. Increases D. Dish/Stirling system B. 150 C or higher
B. Decreases C. 145 C to 200 C
C. Constant Answer: C D. Below 200 C
D. No effect
41. In a power tower system typical thermal Answer: A
Answer: B efficiencies is in the range:
A. 10 to 15 % 47. A binary cycle using a separate heat transfer fluid
36. Distributed collector system is also known as: B. 5 to 10 % if the temperature of the hot water is between
A. Trough electric system C. 15 to 20 % approximately _________.
B. Central receiver system D. 25 to 30 % A. 165 C and 120 C
C. Power tower system B. 200 C and 150 C
D. Dish/Stirling system Answer: C C. 225 C and 175 C
D. 300 C and higher
Answer: A 42. A dish engine system has an efficiency of
A. 10 to 15 % Answer: A
37. Central receiver system is also known as: B. 5 to 10 %
A. Trough electric system C. 14 to 19 % 48. For every kilometer of depth, the temperature of
B. Distributed collector system D. 24 to 28 % the earth’ s crust increase by:
C. Power tower system A. 30 F
D. Dish/Stirling system Answer: D B. 40 F
C. 50 F
Answer: C 43. Practical and economic issues limit trough electric D. 20 F
systems to about:
38. Which of the following main approaches to solar A. 200 MW Answer: A
energy generating systems describes that B. 100 MW
parabolic tracking trough concentrators focus C. 300 MW 49. Multiple wells produce steam at 690 to 820 kPa
sunlight on evacuated glass tubes that run along D. 400 MW and temperature of ________.
the collectors focal lines. A. 205 C
A. Trough electric system Answer: A B. 155 C
B. Central receiver system C. 250 C
C. Power tower system 44. Practical and economic issues limit tower electric D. 175 C
D. Dish/Stirling system systems to approximately:
A. 100 to 200 MW Answer: A
50. In a hot rock systems, water is injected through 5. What kind of bonding do common gases that exist 11. An electricity charged atom or radical which
injection wells into artificially made fractured rock in free as diatomic molecules experiences? carries electricity through an electrolyte is called:
beds of how many kilometers below the surface? A. Ionic bonds A. Ion
A. 1 to 6 km B. Convalent bonds B. Isotope
B. 3 to 9 km C. Metallic bonds C. Molecule
C. 4 to 10 km D. Nuclear bonds D. Hole
D. 6 to 11 km
Answer: B Answer: A
Answer: A
6. Which of the following is NOT a unit of energy? 12. What is the smallest particle of an element that
CHAPTER 7 – NUCLEAR PLANT A. Atomic unit can enter into a chemical reaction?
B. MeV A. Molecule
1. Past ME Board Question C. Dynes B. Ion
What is the suggested maximum permissible dose D. Ergs C. Atom
(MPD) of gamma ray exposure for general D. Isotope
individuals not working in a nuclear setting, by Answer: C
choice, in rem/year? Answer: C
A. 1 7. The process in which a nucleus splits into smaller
B. 5 fragments. 13. Beryllium, magnesium, and calcium all belong to
C. ½ A. Fusion which elemental grouping?
D. 3 B. Fission A. Noble elements
C. Photoelectric effect B. Halogens
Answer: C D. Compton’s effect C. Lanthanons
D. Alkaline earth metals
2. Past ME Board Question Answer: B
The number of protons in the nucleus of an atom Answer: D
of the number of electrons in the orbit of an atom 8. Which of the following is NOT a unit of energy?
A. Atomic volume A. Calories 14. The thickness of material required to attenuate
B. Atomic number B. Joules radiation to a particular level depends on
C. Atomic weight C. Pascals A. The particle
D. Atomic mass D. MeV B. The particle energy
C. The shielding material
Answer: B Answer: C D. The particle, its energy, and the shielding
material
3. The process in which a nucleus splits into smaller 9. The ability of a substance to absorb neurons is
fragments. dependent upon which parameter? Answer: D
A. Fusion A. Absorption cross section
B. Fission B. Scattering cross section 15. Particles that are easily stopped within a few
C. Photoelectric effect C. Total cross section millimeter because their double charges generate
D. Compton’s effect D. Atomic number path ionization and because they are susceptible
to electrostatic interaction are:
Answer: B Answer: A A. Alpha neurons
B. Alpha radiations
4. The ability of a substance to absorb neurons is 10. Which of the following is NOT a unit of energy? C. Beta radiations
dependent upon which parameter? A. Atomic unit D. Gamma radiations
A. Absorption cross section B. MeV
B. Scattering cross section C. Dynes Answer: B
C. Total cross section D. Ergs
D. Atomic number 16. Radiation consisting of singly charged particles
Answer: C that generate to intermediate distances are called:
Answer: A A. Fast neurons
B. Alpha radiations Answer: A 27. The total energy of an electron in the same shell is
C. Beta radiations defined by the _________.
D. Gamma radiations 22. Radiation exposure, the measure of gamma A. Principal quantum number
radiation at the surface of an object is measured B. Azimuthal quantum number
Answer: C in: C. Magnetic quantum number
A. Rems D. Hund rule
17. Radiation with no charge, which produces no B. Rads
ionization , and which is difficult to attenuate thus C. Roentegens Answer: A
posing a major health threat is D. Roentegens per second
A. Slow neutrons 28. The direction of an electron’s angular momentum
B. Alpha radiations Answer: C vector is defined by the
C. Beta radiations A. Principal quantum number
D. Gamma radiations 23. Radiation exposure rate, the rate of gamma B. Azimuthal quantum number
radiation at the surface of an object is measured in C. Magnetic quantum number
Answer: D A. Rems D. Electron spin quantum number
B. Rads
18. The property of fluid at a location when the Mach C. Roentegens Answer: C
number is unity (at the throat) are called: D. Roentegens per second
A. Critical properties 29. The electron’s spin angular momentum vector is
B. Sonic properties Answer: D defined by the
C. Dynamic properties A. Azimuthal quantum number
D. Stagnation properties 24. Exposure is a measure of ionization surrounding a B. Magnetic quantum number
person, but biological damage is dependent on the C. Electron spin quantum number
Answer: A amount of energy. D. Hund rule
A. Striking the surface
19. Gamma attenuation is affected by: B. Passing through the body Answer: C
A. Their photoelectric effect C. Absorbed
B. Pair production D. Reflected by the surface 30. The magnitude of an electron’s angular
C. Compton scattering momentum vector is defined by the
D. The photoelectric effect, pair production and Answer: C A. Principal quantum number
Compton scattering B. Azimuthal quantum number
25. All of the following are practical applications of C. Electron spin quantum number
Answer: D Einstein’s principle of special relativity except D. Hund rule
_____.
20. The amount of a radiation shield’s dimensional A. Mass increase Answer: B
geometry that reduces radiation to half of its B. Length contraction
original value is called the _________. C. Time dilation 31. The fact that each orbital of a set of equal-energy
A. Half-value mass D. Space warping orbitals must be occupied with an electron before
B. Half-value thickness any orbitals has two electrons is specified by
C. Semi-cross section Answer: D which of the following?
D. Iogarithmic decrement A. Principal quantum number
26. The postulate that no signal or energy can be B. Azimuthal quantum number
Answer: B transmitted with a speed greater than the speed of C. Magnetic quantum number
light is consistent with _________. D. Hund rule
21. The amount of a radiation shield’s density that A. The Heisenburg uncertainty principle
reduces radiation to half of its original value is B. The Compton effect Answer: D
called the _____________. C. Einstein’s law
A. Half-value mass D. Newton’s second law 32. The statement that no two electrons can have the
B. Half-value thickness same set of four quantum numbers is known as
C. Semi-cross section Answer: C the
D. Iogarithmic decrement A. Hund rule
B. Heisenburg uncertainty principle B. Work function 4. A ________boiler has water in the tubes and heat
C. Pauli exclusion principle C. Coulomb energy gases of combustion passing around the tubes.
D. Schrodinger equation D. Fermi energy A. Fire tube
B. Cast iron sectional
Answer: C Answer: B C. Water tube
D. Firebox
33. All of the following terms describe the radiation 38. Which of the following is not a postulate of Bohr’s
lines from transitions of electrons in an atom theory of the hydrogen atom? Answer: C
except A. Electron orbits are discrete and non-radiating
A. Sharp and an electron may not remain between 5. Which of the following are used in boilers to direct
B. Principal these orbits the gases of combustion over the boiler heating
C. Obtuse B. The energy change experienced by an surface?
D. Fundamental electron changing from one orbit to another is A. Combustion control
quantized B. Baffles
Answer: C C. Light waves exist simultaneously as high C. Fire tubes
frequency electrical and magnetic waves D. Zone controls
34. When electrons are not shared equally between D. Angular momentum is quantized
two elements, the electrons spend more time with Answer: B
one element than with the other, the bonding is Answer: C
called: 6. The three basic type of low pressure heating
A. Ionic bonding CHAPTER 8 – BOILERS boilers are _______, _______ and _______.
B. Polar covalent bonding A. Firebox; vertical; locomotive
C. Non-polar covalent bonding 1. Which of the following is necessary to generate B. Scotch marine; wet-top; dry top
D. Metallic bonding steam in a boiler? C. Straight-tube; bent tube; multiple pass
A. A container D. Fire-tube; water tube; cast iron sectional
Answer: B B. Heat
C. Water Answer: D
35. When electrons are shared equally( e.g. when the D. All of the above
atoms are the same as in diatomic gases) the 7. In a low pressure gas system, the gas regulator
bonding is called: Answer: D reduces that city gas pressure to ____psi.
A. Ionic bonding A. 0
B. Polar covalent bonding 2. It is the part of the boiler with water on one side B. 2
C. Non-polar covalent bonding and heat on the other side C. 1
D. Resonance bonding A. Furnace volume D. 3
B. Fire side
Answer: C C. Heating surface Answer: A
D. Water side
36. All of the following are units of energy except: 8. In the _________air mixes with the fuel and burns.
A. Atomic units Answer: C A. Fire tubes
B. MeV B. Combustion chamber
C. Dynes 3. A _________boiler has heat and gases of C. Water tubes
D. Ergs combustion that pass through tubes surrounded D. Breeching
by water
Answer: C A. Fire tube Answer: B
B. Cast iron sectional
37. Einstein reasoned there was a discrete amount of C. Water tube 9. The function of the steam boiler is to ______.
energy needed to remove an electron from a D. Straight tube A. Produce condensate
surface , with the rest of the incident photon’s B. Create heat
energy contributing to the kinetic energy of the Answer: A C. Produce steam
photon, the amount of energy is called: D. Burn fuel
A. Binding energy
Answer: C Answer: C B. Automatic nonreturn valve
C. Safety valve
10. Which of the following is the most common type of 16. What are the four systems necessary to operate a D. Feedwater stop valve
water tube boiler? boiler?
A. Firebox A. Combustion; draft; steam; boiler water Answer: C
B. Scotch marine B. Water; steam; combustion; stoker
C. Vertical C. Boiler water; fuel; draft; condense 22. The MAWP on a low pressure steam boiler is
D. Straight-tube multiple pass D. Feedwater; fuel; draft; steam _______ psi.
A. 10
Answer: D Answer: D B. 20
C. 15
11. Excess fuel oil in the fuel oil system returns to the: 17. The _________forms a vacuum that draws out of D. 30
A. Combustion chamber the condensate return line and into the vacuum
B. Fuel oil tank tank. Answer: C
C. Burner A. Suction line
D. Suction line B. Main header 23. Which of the following best defines a total force
C. Vacuum pump acting on a boiler?
Answer: B D. Heating equipment A. Area times diameter
B. Area times pressure
12. It is the system that provides the air necessary for Answer: C C. Area times distance
combustion. D. The MAWP times pressure
A. Feedwater 18. Which of the following can be a function of the
B. Steam container in a boiler? Answer: B
C. Draft A. Holds the water
D. Fuel B. Collects the steam that is produced 24. The ASME code states that boilers with over
C. Transfers heat to the water to produce steam _______square feet heating surface must have
Answer: C D. All of the above two or more safety valve.
A. 200
13. The gases of combustion leave the boiler through Answer: D B. 400
the: C. 400
A. Chimney 19. At what temperature will water normally turns to D. 500
B. Blower steam?
C. Air vent A. 100 F ͦ Answer: D
D. Breeching B. 200 F ͦ
C. 150 F ͦ 25. The area of a safety valve 4” in diameter is how
Answer: D D. 212 F ͦ many square inches?
A. 2.3562
14. Which of the following boiler does not use tubes? Answer: D B. 6.2832
A. Cast iron sectional C. 3.1416
B. Water tube 20. Which of the following statements is true for cast D. 12.5664
C. Scotch marine iron sectional boilers?
D. Firebox A. Cast iron sectional boilers have fire tubes Answer: D
B. These boilers do not require water
Answer: A C. Can be expanded in size 26. Which of the following is the only valve permitted
D. None of the above between the safety valve and the boiler?
15. Which of the following will make a boiler work A. Os&y gate
more efficiently? Answer: D B. Os&y globe
A. More fuel is added C. Automatic nonreturn
B. Fire tubes are decrease in size 21. Which of the following is the most important valve D. No valves are permitted between the boiler
C. The heating surface is increased on a boiler? and the safety valve
D. All of the above A. Main steam stop valve
Answer: D B. Suction A. Sewer
C. Duplex B. Atmospheric tank
27. The range of the pressure gauge should be D. Vacuum C. Blowdown tank
_____times the MAWP of the boiler. D. Return tank or open sump
A. 1 to 2 Answer: A
B. 2 to 3 Answer: C
C. 1 ½ to 2 33. Which of the following best defines a vacuum?
D. 2 ½ to 3 A. A pressure above gauge 39. If the desired cut-in pressure of the boiler is 6 psi
B. A pressure below atmospheric and the desired cut-out pressure is 10 psi, the
Answer: C C. A pressure below gauge differential pressure setting must be _____psi.
D. A pressure equal to gauge A. 2
28. The ASME code only allows _____safety valve on B. 6
boiler. Answer: B C. 4
A. Deadweight D. 8
B. Spring-loaded pop-type 34. Safety valves are designed to pop open and stay
C. Lever until there is a ___psi drop in pressure. Answer: C
D. None of the above A. 0 to 1
B. 5 to 15 40. Impurities that build up on the surface of the water
Answer: B C. 2 to 4 in the boiler prevent _____from breaking through
D. Over 15 the surface of the water.
29. How much is the total force on a safety valve 2 ½ A. Air
in diameter and with a steam pressure of 15 psi? Answer: C B. CO2
A. 19.5413 C. Oxygen
B. 73.63125 35. What type of boiler that uses safety valves? D. Steam
C. 29.4525 A. Fire tube
D. 93.7512 B. Cast iron sectional Answer: D
C. Water tube
Answer: B D. All of the above 41. To prevent air pressure from building up in the
boiler when filling the boiler with water, the ____
30. The steam pressure gauge on the boiler is Answer: D must be open.
calibrated to read in: A. Safety valve
A. Inches of vacuum 36. The water column is located at the NOWL so the B. Boiler vent
B. Absolute pressure lowest visible part of the gauge glass is C. Main steam stop valve
C. Pounds per square inch _____above the highest surface. D. Manhole cover
D. Pressure below atmospheric pressure A. 2” to 3”
B. Just Answer: B
Answer: C C. 4” to 5”
D. Never 42. To prevent a vacuum from forming when taking
31. Live steam is prevented from entering the Bourdon the boiler off-line, which of the following must be
tube of the pressure gauge by which of the Answer: A opened when pressure is still on the boiler?
following? A. Safety valve
A. Automatic nenreturn valve 37. Blowback of a safety valve is to prevent: B. Boiler vent
B. Os&y valve A. Burner cycling C. Main steam stop valve
C. Inspector’s test cock B. Premature popping D. Man hole cover
D. Siphon C. Chattering
D. Feathering Answer: B
Answer: D
Answer: C 43. The operating range of the boiler is controlled by:
32. What pressure gauge that can read whether A. Squastat
vacuum pressure or not. 38. The boiler bottom blowdown line should discharge B. Pressure control
A. Compound to a(n)_________. C. Vaporstat
D. Modulating pressure control 49. A ______valve is installed before the _____ to C. 10
allow the steam pressure gauge to be changed if a D. 30
Answer: B malfunction occurs.
A. Globe; safety valve Answer: B
44. The ______ regulates the high and low fire of the B. Globe; U-tube siphon
burner. C. Gate; siphon 55. After the total force of the steam has lifted the
A. Aquastat D. Globe; pigtail siphon safety valve off its seat, the steam enters the
B. Pressure control ______.
C. Vaporstat Answer: C A. Huddling chamber
D. Modulating pressure control B. Steam holding tank
50. What pressure gauge that reads more pressure C. Combustion chamber
Answer: D than is actually in the boiler. D. Main steam line
A. Broken
45. The best time to blow down the boiler to remove B. Uncalibrated Answer: A
sludge and sediment is when the boiler is at: C. Slow
A. Its highest load D. Fast 56. What causes false water level readings in the
B. Half its load gauge glass?
C. Its lightest load Answer: D A. Priming
D. Anytime B. Foaming
51. According to the ASME code, safety valves on low C. Carryover
Answer: C pressure boilers should be tested by hand at least: D. Blowing down
A. Once a month
46. The level of the water in the _____ indicates the B. Once a year Answer: B
water level in the boiler. C. Once a shift
A. Condensate return tank D. Twice a year 57. Fusible plugs are required on ______ boilers by
B. Gauge glass the ASME code.
C. Try cocks Answer: A A. Coal-fired
D. Blowdown tank B. Gas-fired
52. The purpose of the safety valve is prevent the C. Fuel oil-fired
Answer: B pressure in the boiler from: D. All of the above
A. Exceeding its MAWP
47. When blowing down a boiler, the quick-opening B. Causing a boiler explosion Answer: D
valve should always be opened ______and closed C. Dropping below its MAWP
______. D. Relieving water pressure 58. On a pressure control, _______ pressure plus
A. First; first ______pressure equals _______ pressure.
B. Last; first Answer: A A. Differential; cut-out; cut-in
C. First; last B. Cut-in; cut-out; differential
D. Last; last 53. The term applied when a safety valve opens and C. Cut-in; differential; cut-in
closes rapidly. D. Cut-in pressures equals cut-out pressure
Answer: C A. Feathering
B. Chattering Answer: C
48. Which of the following is added to boiler so that C. Pressuring
water changes scale-forming salts into a non- D. Huddling 59. A burner should always start up in ______ fire and
adhering sludge? shut down in ______ fire.
A. Oxygen Answer: B A. Low; low
B. Slag B. High; low
C. Minerals 54. The safety valve on a low pressure boiler opens C. Low; high
D. Chemicals when pressure in the boiler exceeds how many D. High; high
psi?
Answer: D A. 5 Answer: A
B. 15
60. The water in the boiler is heated, turns to steam, Answer: C B. Boiler
and leaves the boiler through the: C. Makeup
A. Feedwater line 66. A ______ valve should be located between the D. Feed
B. Main steam line boiler and the valve on the feedwater line.
C. Main header A. Stop; check Answer: C
D. Main branch line B. Stop; regulator
C. Check; stop 72. Excessive use of cold city water makeup reduces
Answer: B D. Check; regulator overall efficiency because the water must be
______ before use in the boiler.
61. When steams gives up its heat in a heat Answer: A A. Vented
exchanger it turns to: B. Filtered
A. Low pressure steam 67. The feedwater ______ valve opens and closes C. Heated
B. Makeup water automatically. D. Recirculated
C. Condensate A. Return
D. Exhaust steam B. Check Answer: C
C. Bypass
Answer: C D. Equalizing 73. The vacuum pump selector switch is normally in
the position during the heating season.
62. A ________ pump discharges the return water Answer: B A. Float or vacuum
back to the boiler. B. Vacuum only
A. Feed 68. The vacuum pump pumps water and discharges C. Float only
B. Gear air to the: D. Continuous
C. Return A. Expansion tank
D. Vacuum B. Return tank Answer: A
C. Compression tank
Answer: D D. Atmosphere 74. The ______ shuts off the burner in the event of
low water.
63. How do you call a valve that controls the flow of Answer: D A. Low water alarm
water in one direction only? B. Low water fuel cut-off
A. Gate 69. The range of pressure on the vacuum switch is C. Feedwater regulator
B. Os&y usually: D. Automatic low water feeder
C. Globe A. 2 to 6 psi
D. Check B. 2” to 8” Answer: B
C. 6 to 12 psi
Answer: D D. 8” to 12” 75. Water is supplied to the condensate return tank by
the ______ pump.
64. The feedwater _____ valve should be located as Answer: B A. Vacuum
close to the shell of the boiler as practical. B. Feedwater
A. Check 70. Top test the vacuum pump, the operator puts the C. Condensate
B. Nonreturn elector switch in the _____ position. D. Return
C. Stop A. Float
D. Regulating B. Float or vacuum Answer: A
C. Vacuum
Answer: C D. Continuous 76. The feedwater regulator is located at the ______
and is connected to the boiler in the same manner
65. A _________ after each radiator allows _____ to Answer: D as the ______.
pass through to the return line. A. Right side; gauge glass
A. Steam tarp; steam 71. Water added to the boiler to replace water lost due B. NOWL; water column
B. Water trap; steam to leaks and blowing down is known as _____ C. Bottom; blowdown
C. Steam trap; condensate water. D. NOWL; safety valve
D. Water trap; condensate A. Extra
Answer: B Answer: C Answer: C

77. What maintains a constant water level in the 83. When open os&y valve offers ______ to the flow 89. The _______ steam trap is the most common
boiler? of steam. steam trap used.
A. Gauge glass A. No resistance A. Thermostatic
B. Automatic city water makeup feeder B. Velocity B. Inverted bucket
C. Water column C. Throttling action C. Return
D. Feedwater regulator D. Full resistance D. Nonreturn; float

Answer: D Answer: A Answer: A

78. The low water fuel cut-off should be tested: 84. Steam traps are _____ devices. 90. Condensate from the nonreturn steam trap is
A. Daily A. Manual pumped from the condensate return tank of the:
B. Semiannually B. Automatic A. Return tank
C. Monthly C. Electrical B. Feedwater heater
D. Annually D. Semiautomatic C. Vacuum tank
D. Boiler
Answer: A Answer: B
Answer: D
79. The burner should be _____ when the low water 85. What valve should be used for throttling flow of
fuel cut-off is blown down. material? 91. Steam returning to the vacuum tank could cause
A. Off A. Gate the condensate pump to become:
B. Tagged out B. Check A. Waterbound
C. Firing C. Globe B. Waterlogged
D. Tested D. Non-return C. Steambound
D. Steamlogged
Answer: C Answer: C
Answer: C
80. Which of the following valves must be used as 86. Steam traps removed ______ and _____ from the
boiler main steam stop valve? steam lines. 92. Steam strainers should be located on the line
A. Globe A. Air; water _____ the steam trap.
B. Check B. Water; oil A. In front of
C. Gate C. Air; oil B. Above
D. Os&y gate D. Air; steam C. After
D. Along sides of
Answer: D Answer: A
Answer: A
81. Gate valves should always be _____ or _____ 87. Condensate in the steam lines can result in:
closed. A. Carryover 93. As the steam pressure increases the steam
A. Partially open; completely B. Foaming temperature:
B. Completely open; completely C. Priming A. Decreased
C. Wide open; partly D. Water hammer B. Remains the same
D. Throttled; completely C. Increases
Answer: D D. Fluctuates
Answer: B
88. Two types of steam traps are the _____ steam Answer: C
82. Steam header valves should be _____ valves. trap and the _____ steam trap.
A. Globe A. Return; float 94. When the os&y valve is open the stem is in the
B. Check B. Nonreturn; thermostatic ______ position.
C. Gate C. Return; nonreturn A. Floating
D. Os&y gate D. Nonreturn; float B. Up
C. Locked 100. When burning No. 6 fuel oil strainers should be D. None of the above
D. Down cleaned at least once every ______ hours.
A. 8 Answer: C
Answer: B B. 12
C. 10 106. The rotary cup burner uses ______ and _____ to
95. The thermostatic steam trap opens and closes by D. 24 atomize the fuel oil.
a(n): A. High temperature; pressure
A. Float Answer: D B. High temperature; air
B. Electric sensor C. High pressure; steam
C. Inverted bucket 101. When cleaning the fuel oil strainers the _____ D. A spinning cup; high velocity air
D. Flexible bellows must be carefully replaced to prevent ______ from
getting into the system. Answer: D
Answer: D A. Gaskets; fuel oil
B. Cover; water 107. In a low pressure gas burner gas is mixed with air
96. In the float the thermostatic trap the float rises to C. Gaskets; air in the ________.
discharge: D. Cover; fuel oil A. Burner register
A. Condensate B. Combustion chamber
B. Feedwater chemicals Answer: C C. Mixing chamber before the burner register
C. Steam D. Boiler furnace
D. Water and steam 102. The ______ pump draws fuel oil from the fuel oil
tank. Answer: C
Answer: A A. Transfer
B. Condensate 108. In a high pressure gas mixes with the air on the
97. A steam trap that failed to open would cause the C. Fuel oil inside of the:
heating unit to become: D. Circulating A. Burner register
A. Steambound B. Combustion chamber
B. Very hot Answer: C C. Mixing chamber
C. Waterlogged D. Boiler furnace
D. All of the above 103. The ______ valve protects the fuel lines and pump
from excessive pressure. Answer: A
Answer: C A. Safety
B. Relief 109. Regardless of the type of fuel oil used the fuel oil
98. Which of the following whose main function is to C. Bypass must be _____ before it enters the boilers furnace.
remove dirt and impurities that may cause the D. Stop A. Pressurized
steam trap to malfunction? B. Treated
A. Vacuum pumps Answer: B C. Atomized
B. Steam strainers D. Measured
C. Globe valves 104. A high vacuum on the fuel oil suction gauge
D. Steam separators normally indicates either _____ or ______. Answer: C
A. Low viscosity; a clogged vent
Answer: B B. A closed discharged valve; hot fuel oil 110. On the low pressure gas system the manual reset
C. Cold fuel oil; a dirty suction strainer cannot be opened until the:
99. When a steam trap is functioning properly there is D. Water in the fuel oil; a warm pump A. Vaporstat proves pressure
a _______ difference in temperature between the B. Pilot is lighted
trap inlet and discharge. Answer: C C. Boiler is vented
A. 5 F ͦ to 10 F ͦ D. All of the above
B. 20 F ͦ to 30 F ͦ 105. Fuel oil burners deliver the fuel oil to the furnace in
C. 10 F ͦ to 20 F ͦ a: Answer: B
D. No A. Steady stream
B. Half spray, half steam 111. Before any repair work is attempted on any gas-
Answer: C C. Fine spray fired boiler the:
A. Insurance inspector must be notified C. Because of a failure in the fuel system being 122. The _____ is the temperature at which fuel oil
B. Main gas cock must be secured used gives off vapor that flashes when exposed to an
C. State inspector must be notified D. All of the above open flame.
D. Main solenoid valve should be secured A. Fire point
Answer: D B. Pour point
Answer: B C. Flash point
117. Hard coal, known as ______, has a ______ D. Viscosity
112. In a low pressure gas system when the vaporstat content.
proves gas pressure up to the zero- reducing A. Bituminous; high carbon Answer: C
governor the ______ starts up. B. Anthracite; high volatile
A. Induced draft fan C. Bituminous; high volatile 123. The _____ is the temperature at which fuel oil will
B. Pressure D. Anthracite; high carbon burn continuously when exposed to an open
C. Forced draft blower flame.
D. Gas pump Answer: D A. Fire point
B. Pour point
Answer: C 118. Soft coal; known as _____, has a _______ C. Flash point
content. D. Viscosity
113. The butterfly on the low pressure gas system is a A. Bituminous; high carbon
________ valve. B. Anthracite; high volatile Answer: A
A. Slow-opening C. Bituminous; high volatile
B. Screw D. Anthracite; high carbon 124. It is defined as the internal resistance of fuel oil
C. Quick-opening flow.
D. Globe Answer: C A. Fire point
B. Pour point
Answer: A 119. Boilers using soft coal must have ______ furnace C. Flash point
volume to reduce the danger of _____. D. Viscosity
114. The amount of gas supplied to the burner in the A. Small; overfiring
low pressure gas system is controlled by: B. Large; smoking Answer: D
A. Gas pressure C. Small; smoking
B. Steam pressure D. Large; overfiring 125. What must be done in order to lower the viscosity
C. The boiler operator of the fuel oil?
D. The amount of air passing through the Answer: B A. Lower its temperature
butterfly valve B. Increase its temperature
120. When burning soft coal, if the volatile gases reach C. Lower its pour point
Answer: D the heating surface before they have burned D. Increase its pour point
completely _____ and ______ develop.
115. If low water develops in the boiler having a low A. Smoke; soot Answer: B
pressure gas system the ______ will secure the B. Clinkers; ash
gas to the burner. C. Uneven heat; ash 126. A leak on the fuel oil suction line between the tank
A. Whistle valve D. High temperature; smoke and the suction side of the fuel oil pump would
B. Manual reset valve result in:
C. Vaporstat Answer: A A. The suction gauge pulsating
D. Main gas solenoid B. Air entering the suction line
121. The _____ of fuels oil is the lowest temperature at C. Pulsating of the fire in the boiler
Answer: D which it will flow as a liquid. D. All of the above
A. Fire point
116. Combination gas/fuel oil burners permit the B. Pour point Answer: B
operator to switch from one fuel to another C. Flash point
______. D. Viscosity 127. Stokers were developed to:
A. For economy A. Increase efficiency of burning coal
B. Because of a shortage of fuel being used Answer: B
B. Keep furnace temperature constant to protect B. Retort air A. Incomplete
brickwork C. Overfire air B. Perfect
C. Allow for development of larger coal-fired D. Grate zone air C. Complete
boiler D. Imperfect
D. All of the above Answer: C
Answer: A
Answer: D 133. The ram-feed stoker is a(n) ______ stoker.
A. Overfeed 139. Which of the following is required to achieve
128. Any foreign matter in the coal feed mechanism of B. Side feed complete combustion?
the screw-feed stoker is best removed by: C. Travelling grate A. Proper atomization
A. Emptying the coal hopper D. Underfeed B. Proper temperature of fuel
B. Reversing the stoker C. Time of complete combustion
C. Forcing it through with a heavy shear pin Answer: D D. All of the above
D. Using the cutoff gate at the bottom of the
hopper 134. The ram-feed stoker provides: Answer: D
A. Uniform feed
Answer: D B. Quick ash discharge 140. The combustion process must be complete before
C. Even coal distribution the gases combustion come in contact with the:
129. In the event of an obstruction clogging the feed D. All of the above A. Atmosphere
screw a _____ is used to prevent damage to B. Superheater
transmission. Answer: D C. Heating surface
A. Slip clutch D. Main steam outlet
B. Shear pin or key 135. Combustion is the rapid burning of fuel and
C. Fuse oxygen resulting in: Answer: C
D. None of the above A. Release of heat
B. Oxidation 141. Soot buildup on heating surface:
Answer: B C. Release of steam A. Acts as an insulation
D. Nitrogen and oxygen B. Prevents heat transfer
130. In order to bank the stoker it is necessary to C. Increases temperature of gases of
secure the _____ while the _____ continuous to Answer: A combustion
operate. D. All of the above
A. Combination fan; coal feeder 136. What combustion occurs when all the fuel is
B. Coal feeder; forced draft fan burned using the theoretical amount of air? Answer: D
C. Feedwater; induced draft fan A. Incomplete
D. Boiler stop valve; coal feeder B. Perfect 142. The purpose of automatic combustion controls is
C. Complete to control ______ and _______.
Answer: B D. Imperfect A. High fire; low fire
B. Stoker firing; fuel rate
131. In a screw-feed stoker, what draft fan supplies air Answer: B C. Overfire air; fuel mixture
for combustion? D. Proper air to fuel mixture; firing rate
A. Induced 137. It is the burning of all the fuel using the minimum
B. Forced amount of excess air. Answer: D
C. Combination A. Incomplete combustion
D. Natural B. Complete combustion 143. At 0 psig, how many BTU are necessary to
C. Perfect combustion ̊ of steam?
change 1 lb of water at 212 F
Answer: B D. Imperfect combustion A. 144
B. 970
132. To prevent smoke and to aid in complete Answer: B C. 180
combustion in the screw-feed stoker, _______ is D. 1190
provided using a separate damper control. 138. ______ combustion occurs when all the fuel is not
A. Underfire air burned resulting in formation of soot and smoke. Answer: B
144. Radiation is: B. Specific heat Answer: C
A. The movement of liquids or gases created by C. Latent heat
a temperature difference D. Superheat 5. One could expect the possibility of Reynolds
B. The transfer of heat by direct molecular number similarity in all of the following cases
contact Answer: C except:
C. The weight of the substance compared to A. Pumps
unity 150. Heat may be transferred in how many ways? B. Fans
D. A form of heat transfer A. One C. Turbines
B. Three D. Weirs
Answer: D C. Two
D. Four Answer: D
̊ is:
145. Steam at 100 psi and 400 F
A. Saturated Answer: B 6. One could expect the possibility of Froude number
B. Superheated similarity in all of the following cases except:
C. Wet CHAPTER 9 – HYDROELECTRIC PLANT A. Surface ships
D. None of the above B. Surface wave motion
1. In turbine installation, the air is removed from the C. Flow over weirs
Answer: B condenser by ________. D. Closed-pipe turbulent pipe flow
A. Air pump
146. Which of the following most closely defines B. Air injector Answer: D
sensible heat? C. Air ejector
A. An established relationship comparing any D. Air jet 7. One could expect the possibility of Froude number
substance to the heat content of water similarity in all of the following cases except:
B. Heat quantity that can be felt or measured by Answer: C A. Motion of a fluid jet
a thermometer B. Flow over spillways
C. Heat quantity above the point of saturation 2. The primary purpose of a turbine in a fluid loop is C. Surge and flood waves
D. Measure of heat intensity to: D. Subsonic airfoils
A. Add energy to the flow
Answer: B B. Add mass to the flow Answer: D
C. Extract energy from the flow
147. What is required to raise the temperature of 1 lb of D. None of the above 8. In fluid flow, linear momentum is:
water from 32 ̊ to 212 F̊ ? A. A vector quantity equal to the product of
A. 144 Btu Answer: C mass and velocity
B. 970.3 Btu B. A scalar quantity equal to the product mass
C. 180 Btu 3. A vena contracta in a fluid jet issuing through a and velocity
D. Saturated heat hole in a plate is located approximately: C. A scalar quantity equal to the product of force
A. 10 diameters downstream of the hole and length of time is applied
Answer: C B. At jet’s minimum diameter D. The change in impulse
C. At the orifice minimum diameter
148. Increasing the pressure has what effect of the D. At the orifice maximum diameter Answer: A
boiling point of water?
A. No change Answer: B 9. All of the following fluid phenomena are based on
B. Temperature will be raised the force momentum principle of a flowing fluid
C. Temperature will be lowered 4. The following are all examples of indirect except:
D. Temperature will increase 2 F ̊ for every psi (secondary) miscellaneous methods to measure A. Turbines
flow except: B. Pelton wheels
Answer: B A. Turbine and propeller meters C. Diesel automobile engines
B. Magnetic flow meters D. Jet engines
149. Heat absorbed by water when it changes from C. Positive displacement meters
liquid to steam at the boiling point is called: D. Hot-wire anemometers Answer: C
A. Sensible heat
10. The fact that a fluid’s velocity increases as the C. Sum of the coefficient of discharge and the Answer: D
cross sectional area of the pipe through which it coefficient of contraction
flow decreases is due to: D. Difference of the coefficient of discharge and 21. Flow measuring devices include all of the following
A. Bernoull’s equation the coefficient of contraction except:
B. The continuity equation A. Flow nozzles
C. The momentum equation Answer: B B. Venture area meters
D. The perfect gas law C. Pilot tubes
16. Which of the following is not a similarity between a D. Precision tachometers
Answer: B submerged culvert and a siphon?
A. They both operate full Answer: D
11. In the absence of any heat and work interactions B. Toricelli’s equation holds
and any changes in potential energy, the C. Both can experience entrance and exit losses 22. The following are examples of indirect (secondary)
stagnation enthalpy of a fluid remains constant D. In both, the water flows downhill measurements to measure flow rates using
during _______. obstruction meters except:
A. Unsteady flow Answer: B A. Variable area meters
B. Steady flow B. Venture meters
C. Turbulent flow 17. In parallel pipe system originating and terminating C. Volume tanks
D. Variable flow in common junctions, D. Flow nozzles
A. Mass flows through each branch are equal
Answer: B B. Pressure drops through each branch are Answer: C
equal
12. When a falling object reaches a speed at which C. Lengths of each branch are equal 23. The following are examples of indirect (secondary)
the drag force equals its weight, it has achieved: D. Flow areas of each branch are equal measurements to measure flow rates using
A. Mach one obstruction meters except:
B. A laminar boundary layer Answer: B A. Pilot static meters
C. A turbulent boundary layer B. Static pressure probes
D. Terminal velocity 18. Flows through multi-loop systems maybe C. Weight and mass scales
computed by: D. Direction-sensing probes
Answer: C A. Any closed-form solution of simultaneous
equations Answer: C
13. The coefficient of contraction is the ratio of the: B. The Hardy-Cross method
A. Area of vena contracta to the orifice area C. Trial and error 24. In series pipe system, all of the following
B. Actual discharge to the theoretical discharge D. All of the above parameters vary from section to section except:
C. Actual velocity to the theoretical velocity A. Pressure drop
D. Effective head to the actual head Answer: D B. Friction loss
C. Head loss
Answer: A 19. Flow measuring devices include all of the following D. Mass flow
except:
14. The coefficient of discharge is the ratio of the: A. Venturi meters Answer: D
A. Area of vena contracta to the orifice area B. Static pressure probes
B. Actual discharge to the theoretical discharge C. Turbine and propeller meters 25. Venturi meters, pitot static gauges, orifice meters,
C. Actual velocity to the theoretical velocity D. Magnetic dynamometers flow nozzles, and differential manometers all
D. Effective head to the actual head depend upon the relationship between:
Answer: D A. Flow velocity and friction
Answer: B B. Flow velocity and pressure
20. Flow measuring devices include all of the following C. Friction and pressure
15. The coefficient of velocity is equal to the: except: D. Pressure and mass flow
A. Product of the coefficient of discharge and A. Orifice plate meters
the coefficient of contraction B. Hot-wire anemometers Answer: B
B. Actual velocity divided by the theoretical C. Magnetic flow meters
velocity D. Mercury barometers
26. The combination of enthalpy and kinetic energy of A. Reynolds number B. D’Alembert’s paradox
fluid is termed as: B. Weber number C. Newton’s second law
A. Latent enthalpy C. Froude number D. The second law of thermodynamics
B. Heat enthalpy D. Cauchy number
C. Throttling enthalpy Answer: B
D. Stagnation enthalpy Answer: B
37. One could expect the possibility of Reynold’s
Answer: D 32. The matching of scale model and full-scale number similarity in all of the following cases
prototype results for a fluid dynamic phenomena except:
27. The coefficient of velocity, Cv , accounts for the: involving a fully submerged body requires equality A. Submarines
A. Effects on the flow area of contraction, friction of: B. Torpedoes
and turbulence A. Reynolds number C. Seaplane hulls
B. Small effect of friction and turbulence of the B. Weber number D. Supersonic aircraft
orifice C. Froude number
C. Changes in diameters of a converging pipe D. Mach number Answer: C
D. Effects of compressibility
Answer: A 38. The function of a turbine is to:
Answer: B A. Transfer heat from one fluid to another
33. The water hammer phenomenon is primarily what B. Increase the total energy content of the flow
28. Expansion factors take into account the: kind of fluid mechanics? C. Extract energy from the flow
A. Area of the vena contracta A. Static (a phenomena independent of time) D. Exchange heat to increase energy to the flow
B. Small effect of friction and turbulence of the B. Dynamic (a time-dependent phenomena)
orifice C. Compressible Answer: C
C. Changes in diameters of a converging pipe D. Incompressible
D. Effects of compressibility 39. Pitot tube is use to measure the:
Answer: B A. Velocity of mass
Answer: D B. Velocity of pipe
34. All of the following are forms of drag on a body C. Flow
29. The matching of scale model and full-scale results moving through a fluid except: D. Velocity of flow
for a fluid dynamic phenomena with a free surface A. Skin friction
requires equality of: B. Wake drag Answer: D
A. Reynolds number C. Profile drag
B. Weber number D. D’Alembert’s paradox drag 40. Orifice coefficients are used to determine:
C. Froude number A. Energy losses
D. Cauchy number Answer: D B. Energy gains
C. Mass losses
Answer: C 35. The magnitude of the drag coefficient of a sphere D. Energy losses and mass gains
in water is dependent upon all of the following
30. The matching of scale model and full-scale results except: Answer: A
for a fluid dynamic phenomena involving A. Fluid density
compressible fluids requires equality of: B. Fluid velocity 41. The coefficient of velocity is the ratio of the:
A. Reynolds number C. Units of measure (SI or English Engineering A. Area of vena contracta to the orifice area
B. Froude number System) B. Actual discharge to the theoretical discharge
C. Cauchy number D. Drag force C. Actual discharge velocity to the theoretical
D. Mach number discharge velocity
Answer: C D. Effective head to the actual head
Answer: D
36. The fact that there is no resistance to bodies Answer: C
31. The matching of scale model and full-scale moving through an ideal (non-viscous) fluids is
prototype results for a fluid dynamic phenomena known as: 42. The volume flow passes through a venturimeter is:
involving surface tension requires equality of: A. Reynold’s analogy A. Increasing
B. Decreasing 48. Past ME Board Question A. Pelton wheel
C. Constant What is the use of a Hydraulic jump? B. Steam turbine
D. Varying A. Increase the flow rate C. Francis turbine
B. Reduce the flow rate D. Reaction turbine
Answer: C C. Reduce the velocity of flow
D. Reduce the energy of flow Answer: A
43. What must be done to change the direction of
rotation of a 440-volt, 3-phase induction motor? Answer: D 54. Past ME Board Question
A. Reverse lines to start winding The lowest portion to storage basin from where
B. Interchange any two power leads 49. Past ME Board Question the water is not drawn is:
C. Replace any two wires to the rotor A Kaplan turbine is: A. Bottom storage
D. Remove any power leads A. A high head mixed flow turbine B. Sub-soil storage
B. An inward flow impulse turbine C. Spring reserve
Answer: B C. An outward flow reaction turbine D. Dead storage
D. Low head axial flow turbine
44. A draft tube is a part of which power plants? Answer: D
A. Diesel engine power plant Answer: D
B. Gas turbine power plant 55. Past ME Board Question
C. Steam power plant 50. Past ME Board Question In a hydro-electric plant using a Francis turbine
D. Hydro-electric power plant The locus of elevation is: with medium head, the speed can be regulated
A. Critical point using the:
Answer: D B. Hydraulic gradient A. Deflector gate
C. Energy gradient B. Nozzle
45. Past ME Board Question D. Friction gradient C. Wicket gate
In the hydro-electric plant having a medium head D. Weir
and using a Francis turbine, the turbine speed Answer: B
may be regulated through: Answer: C
A. Deflector gate 51. Past ME Board Question
B. Nozzle The locus of elevation to which water will rise in 56. Past ME Board Question
C. Wicket gate the piezometer tube is termed: In the deep well installation or operation, the
D. Forebay A. Energy gradient difference between static water level and
B. Friction head operating water level is known as _______.
Answer: C C. Hydraulic gradient A. Suction lift
D. Critical path B. Drawdown
46. Past ME Board Question C. Priming level
A Francis turbine has what flow? Answer: C D. Clogging
A. Inward flow reaction
B. Outward flow impulse 52. Past ME Board Question Answer: B
C. Outward flow reaction The intake pipe to a hydraulic turbinefrom a dam
D. Inward flow impulse is: 57. Past ME Board Question
A. Tailrace Which of the following is a characteristic of an
Answer: A B. Spiral casing impulse turbine?
C. Surge tank A. Steam striking blades on angle
47. Past ME Board Question D. Penstock B. No steam reaction to velocity
Which of the following is a type of water turbine? C. Steam striking blades at zero angle
A. Parson Answer: D D. Steam reversing direction
B. Hero
C. Pelton 53. Past ME Board Question Answer: C
D. Bankl A type of water turbine where a jet of water is
made to fall on the blades or buckets and due to 58. Past ME Board Question
Answer: C the impulse of water, the turbine starts to move.
What is the pressure at the exit of a draft tube in a C. High head C. Steam striking blades to zero angle
turbine? D. Very low head D. Steam reversing direction
A. Below atmospheric
B. Vacuum Answer: C Answer: C
C. Atmospheric
D. Gage 64. Past ME Board Question 69. Past ME Board Question
High head turbine is a/an: A type of water turbine where a jet of water is
Answer: C A. Impulse made to fall on the blades or buckets and due to
B. Francis the impulse of water, the turbine starts to move.
59. Past ME Board Question C. Reaction A. Pelton wheel
An impulse turbine are used for: D. Propeller B. Steam turbine
A. Low head C. Francis turbine
B. Medium head Answer: A D. Reaction turbine
C. High head
D. Very low head 65. Past ME Board Question Answer: A
The lowest portion to storage basin from where
Answer: C the water is not drawn is: 70. Past ME Board Question
A. Bottom storage The intake pipe to a hydraulic turbine from a dam
60. Past ME Board Question B. Sub-soil storage is:
Which of the following is used as high head C. Spring reserve A. Tailrace
turbine? D. Dead storage B. Spiral casing
A. Impulse C. Surge tank
B. Francis Answer: D D. Penstock
C. Reaction
D. Propeller 66. Past ME Board Question Answer: D
In a hydro-electric plant using a Francis turbine
Answer: A with medium head, the speed can be regulated 71. Past ME Board Question
using the In the hydro-electric plant having a medium head
61. In pumped storage plant A. Deflector gate and using a Francis turbine, the turbine speed
A. Power is produced by pumps B. Nozzle may be regulated through:
B. Water is stored by pumping to high pressures C. Wicket gate A. Deflector gate
C. Pressure accumulators are used D. Weir B. Nozzle
D. Downstream water is pumped upstream C. Wicket gate
during offload periods Answer: C D. Forebay

Answer: D 67. Past ME Board Question Answer: C


In the deep well installation or operation, the
62. Past ME Board Question difference between static water level and 72. Past ME Board Question
What is the pressure at the exit of a draft tube in a operating water level is called ________. A Francis turbine has what flow?
turbine? A. Suction lift A. Inward flow reaction
A. Below atmospheric B. Drawdown B. Outward flow impulse
B. Vacuum C. Priming level C. Outward flow reaction
C. Atmospheric D. Clogging D. Inward flow impulse
D. Gage
Answer: B Answer: A
Answer: C
68. Past ME Board Question 73. Past ME Board Question
63. Past ME Board Question Which of the following is a characteristic of an Which of the following is a type of water turbine?
An impulse turbine has: impulse turbine? A. Parson
A. Low head A. Steam striking blades of angle B. Hero
B. Medium head B. No steam reaction to velocity C. Pelton
D. Bankl 79. The primary purpose of a turbine in a fluid loop is C. Surge and flood waves
to: D. Subsonic airfoils
Answer: C A. Add energy to the flow
B. Add mass to the flow Answer: D
74. Past ME Board Question C. Extract energy from the flow
Hydraulic jump is used to: D. None of the above 85. In fluid flow, linear momentum is:
A. Increase the flow rate A. A vector quantity equal to the product of
B. Reduce the flow rate Answer: C mass and velocity
C. Reduce the velocity of flow B. A scalar quantity equal to the product mass
D. Reduce the energy of flow 80. A vena contracta in a fluid jet issuing through a and velocity
hole in a plate is located approximately: C. A scalar quantity equal to the product of force
Answer: D A. 10 diameters downstream of the hole and length of time is applied
B. At jet’s minimum diameter D. The change in impulse
75. Past ME Board Question C. At the orifice minimum diameter
A Kaplan turbine is: D. At the orifice maximum diameter Answer: A
A. A high head mixed flow turbine
B. An inward flow impulse turbine Answer: B 86. All of the following fluid phenomena are based on
C. An outward flow reaction turbine the force momentum principle of a flowing fluid
D. Low head axial flow turbine 81. The following are all examples of indirect except:
(secondary) miscellaneous methods to measure A. Turbines
Answer: D flow except: B. Pelton wheels
A. Turbine and propeller meters C. Diesel automobile engines
76. Past ME Board Question B. Magnetic flow meters D. Jet engines
The locus of elevation is: C. Positive displacement meters
A. Critical point D. Hot-wire anemometers Answer: C
B. Hydraulic gradient
C. Energy gradient Answer: C 87. The fact that a fluid’s velocity increases as the
D. Friction gradient cross sectional area of the pipe through which it
82. One could expect the possibility of Reynolds flow decreases is due to:
Answer: B number similarity in all of the following cases A. Bernoull’s equation
except: B. The continuity equation
77. Past ME Board Question A. Pumps C. The momentum equation
The locus of elevations to which water will rise in B. Fans D. The perfect gas law
the piezometer tube is termed: C. Turbines
A. Energy gradient D. Weirs Answer: B
B. Friction head
C. Hydraulic gradient Answer: D 88. In the absence of any heat and work interactions
D. Critical path and any changes in potential energy, the
83. One could expect the possibility of Froude number stagnation enthalpy of a fluid remains constant
Answer: C similarity in all of the following cases except: during:
A. Surface ships A. Unsteady flow
78. In turbine installation, the air is removed from the B. Surface wave motion B. Steady flow
condenser by: C. Flow over weirs C. Turbulent flow
A. Air pump D. Closed-pipe turbulent pipe flow D. Variable flow
B. Air injector
C. Air ejector Answer: D Answer: B
D. Air jet
84. One could expect the possibility of Froude number 89. When a falling object reaches a speed at which
Answer: C similarity in all of the following cases except: the drag force equals its weight, it has achieved:
A. Motion of a fluid jet A. Mach one
B. Flow over spillways B. A laminar boundary layer
C. A turbulent boundary layer 95. Flows through multi-loop systems may be B. Static pressure probes
D. Terminal velocity computed by: C. Weight and mass scales
A. Any closed-form solution of simultaneous D. Direction-sensing probes
Answer: C equations
B. The Hardy-Cross method Answer: C
90. The coefficient of contraction is the ratio of the: C. Trial and error
A. Area of vena contracta to the orifice area D. All of the above 101. In series pipe systems, all of the following
B. Actual discharge to the theoretical discharge parameters vary from section to section except:
C. Actual velocity to the theoretical velocity Answer: D A. Pressure drop
D. Effective head to the actual head B. Friction loss
96. Flow measuring devices include all of the following C. Head loss
Answer: A except: D. Mass flow
A. Venturi meters
91. The coefficient of discharge is the ratio of the: B. Static pressure probes Answer: D
A. Area of vena contracta to the orifice area C. Turbine and propeller meters
B. Actual discharge to the theoretical discharge D. Magnetic dynamometers 102. Venturi meters, pitot static gauges, orifice meters,
C. Actual velocity to the theoretical velocity flow nozzles, and differential manometers all
D. Effective head to the actual head Answer: D depend upon the relationship between:
A. Flow velocity and friction
Answer: B 97. Flow measuring devices include all of the following B. Flow velocity and pressure
except: C. Friction and pressure
92. The coefficient of velocity is equal to the: A. Orifice plat meters D. Pressure and mass flow
A. Product of the coefficient of discharge and B. Hot-wire anemometers
the coefficient of contraction C. Magnetic flow meters Answer: B
B. Actual velocity divided by the theoretical D. Mercury barometers
velocity 103. The combination of enthalpy and kinetic energy of
C. Sum of the coefficient of discharge and the Answer: D fluid is termed as:
coefficient of contraction A. Latent enthalpy
D. Difference of the coefficient of discharge and 98. Flow measuring devices include all of the following B. Heat enthalpy
the coefficient of contraction except: C. Throttling enthalpy
A. Flow nozzles D. Stagnation enthalpy
Answer: B B. Venture area meters
C. Pitot tubes Answer: D
93. Which of the following is not a similarity betweena D. Precision tachometers
submerged culvert and a siphon? 104. The coefficient of velocity, Cv accounts for the:
A. They both operate full Answer: D A. Effects on the flow area of contraction, friction
B. Torricelli’s equation holds and turbulence
C. Both can experience entrance and exit losses 99. The following are examples of indirect (secondary) B. Small effect of friction and turbulence of the
D. In both, the water flows downhill measurements to measure flow rates using orifice
obstruction meters except: C. Changes in diameter of a converging pipe
Answer: B A. Variable area meters D. Effects of compressibility
B. Venture meters
94. In parallel pipe system originating and terminating C. Volume tanks Answer: B
in common junctions: D. Flow nozzles
A. Mass flows through each branch are equal 105. Expansion factors take into account the:
B. Pressure drops through each branch are Answer: C A. Area of the vena contracta
equal B. Small effect of friction and turbulence of the
C. Lengths of each branch are equal 100. The following are examples of indirect (secondary) orifice
D. Flow areas of each branch are equal measurements to measure flow rates using C. Changes in diameter of a converging pipe
obstruction meters except: D. Effects of compressibility
Answer: B A. Pitot static meters
Answer: D 111. All of the following are forms of drag on a body C. Flow
moving through a fluid except: D. Velocity of flow
106. The matching of scale model and full-scale results A. Skin friction
for a fluid dynamic phenomena with a free surface B. Wake drag Answer: D
rquires equality of: C. Profile drag
A. Reynolds number D. D’Alembert’s paradox drag 117. Orifice coefficients are used to determine
B. Weber number A. Energy losses
C. Froude number Answer: D B. Energy gains
D. Cauchy number C. Mass losses
112. The magnitude of the drag coefficient of a sphere D. Energy losses and mass gains
Answer: C in water is dependent upon all of the following
except: Answer: A
107. The matching of scale model and full-scale results A. Fluid density
for a fluid dynamic phenomena involving B. Fluid velocity 118. The coefficient of velocity is the ratio of the:
compressible fluids requires equality of: C. Units of measure (SI or English Engineering A. Area of vena contracta to the orifice area
A. Reynolds number System) B. Actual discharge to the theoretical discharge
B. Froude number D. Drag force C. Actual discharge velocity to the theoretical
C. Cauchy number discharge velocity
D. Mach number Answer: C D. Effective head to the actual head

Answer: D 113. The fact that there is no resistance to bodies Answer: C


moving through an ideal (non-viscous) fluids is
108. The matching of scale model and full-scale known as: 119. The volume flow passes through a venturimeter is:
prototype results for a fluid dynamic phenomena A. Reynold’s analogy A. Increasing
involving surface tension requires equality of: B. D’Alembert’s paradox B. Decreasing
A. Reynolds number C. Newton’s second law C. Constant
B. Weber number D. The second law of thermodynamics D. Varying
C. Froude number
D. Cauchy number Answer: B Answer: C

Answer: B 114. One could expect the possibility of Reynolds 120. Which of the following devices used to measure
number similarity in all of the following cases the discharge of outflow from a vessel?
109. The matching of scale model and full-scale except: A. Pitot tube
prototype results for a fluid dynamic phenomena A. Submarines B. Orifice
involving a fully submerged body requires equality B. Torpedoes C. Pump head
of: C. Seaplane hulls D. Obstruction meter
A. Reynolds number D. Supersonic aircraft
B. Weber number Answer: B
C. Froude number Answer: C
D. Mach number CHAPTER 10 – VARIABLE LOAD & ENVIRONMENTAL
115. The function of a turbine is to: ENG’G.
Answer: A A. Transfer heat from one liquid to another
B. Increase the total energy content of the flow 1. The maximum continuous power available from a
110. The water hammer phenomenon is primarily what C. Extract energy from the flow hydro-electric power plant under the most adverse
kind of fluid mechanics? D. Exchange heat to increase energy to the flow hydraulic conditions is called _______.
A. Static (a phenomena independent of time) A. Base power
B. Dynamic (a time-dependent phenomena) Answer: C B. Firm power
C. Compressible C. Primary power
D. Incompressible 116. Pitot tube is use to measure the: D. Secondary power
A. Velocity of mass
Answer: B B. Velocity of pipe Answer: B
2. The area under load curve divided by maximum D. Demand factor Answer: C
demand represents:
A. Load factor Answer: A 13. Load curve refers to the plot of:
B. Connected load A. Load versus generating capacity
C. Average load 8. Past ME Board Question B. Load versus current
D. Diversity factor The ratio between the actual power and the C. Load versus time
apparent power in any circuit is known as the D. Load versus cost of power
Answer: D ______ of that circuit.
A. Measured power Answer: C
3. What is the reverse capacity of a plant? B. Capacity
A. Maximum demand – average demand C. Power factor 14. The ratio of maximum load to the rated plant
B. Plant capacity – maximum demand D. KVAR capacity
C. Plant capacity – average demand A. Load factor
D. Plant capacity x (1 – load factor) Answer: C B. Utilization factor
C. Maximum load factor
Answer: B 9. Past ME Board Question D. Capacity factor
The ratio of maximum load to the rated plant
4. Load curve refers to the plot of: capacity Answer: B
A. Load versus generating capacity A. Load factor
B. Load versus current B. Utilization factor 15. The ratio of the average load to the peak load over
C. Load versus time C. Maximum load factor a designated period of time is called _____.
D. Load versus cost of power D. Capacity factor A. Load factor
B. Reactive factor
Answer: C Answer: B C. Diversity factor
D. Plant use factor
5. The ratio of maximum load to the rated plant 10. Past ME Board Question
capacity is called ________. The ratio of the average load to the peak load over Answer: A
A. Load factor a designated period of time is called
B. Utilization factor A. Load factor 16. Past ME Board Question
C. Maximum load factor B. Reactive factor Peak load for a period of time divided by installed
D. Capacity C. Diversity factor capacity is _______.
D. Plant use factor A. Capacity factor
Answer: B B. Demand factor
Answer: A C. Utilization factor
6. Past ME Board Question D. Load factor
Peak load for a period of time divided by installed 11. Past ME Board Question
capacity is: The ratio of the sum of individual maximum Answer: C
A. Capacity factor demands of the system to the overall maximum
B. Demand factor demand of the whole system. 17. The area under load curve divided by maximum
C. Utilization factor A. Demand factor demand represents
D. Load factor B. Diversity factor A. Load factor
C. Power factor B. Connected load
Answer: C D. Utilization factor C. Average load
D. Diversity factor
7. Past ME Board Question Answer: B
The ratio of the sum of individual maximum Answer: D
demands of the system to the maximum demand 12. Load curve refers to the plot of:
of the whole system is: A. Load versus generating capacity 18. Past ME Board Question
A. Diversity factor B. Load versus current The ratio of the sum of individual maximum
B. Utilization factor C. Load versus time demands of the system to the overall maximum
C. Power factor D. Load versus cost of power demand of the whole system.
A. Demand factor D. D2 O A. v/f
B. Diversity factor B. f/v
C. Power factor Answer: D C. vf
D. Utilization factor D. v 2f
24. Scintillation counter are used to detect:
Answer: B A. Beta rays Answer: A
B. X-ray
19. Past ME Board Question C. Gamma rays 30. Swimming pool water needs
The ratio of the sum of individual maximum D. All of the above A. Pre-chlorination
demands of the system to the maximum demand B. Super-chlorination
of the whole system is known as ________. Answer: D C. Dual-chlorination
A. Diversity factor D. De-chlorination
B. Utilization factor 25. All of the following are common types of chemical
C. Power factor reactions except: Answer: D
D. Demand factor A. Direct combination
B. Fission 31. Aeration of water is done for all of the following
Answer: A C. Decomposition purposes except for:
D. Double replacement A. Removal of CO2
20. Past ME Board Question B. Removal of bad taste
The ratio between the actual power and the Answer: B C. Increasing the amount of oxygen in water
apparent power in any circuit is known as the D. Removal of temporary hardness
_______ of that circuit. 26. Redox reactions can often be type of:
A. Measured power A. Direct combination Answer: D
B. Capacity B. Fission
C. Power factor C. Decomposition 32. Which of the following area of work requires
D. KVAR D. Double replacement lowest noise level?
A. Gymnasiums
Answer: C Answer: D B. Library
C. Lecture halls
21. Yeast as raw material for beer making is added to 27. The fact that the amount of slightly soluble gas D. Kitchens
the equipment called: absorbed in a liquid is proportional to the partial
A. Fermenters pressure of the gas is known as: Answer: B
B. Brew kettle A. Dalton’s law
C. Cooler B. Henry’s law 33. Mycotoxins are poisonous chemicals produced by:
D. Starting tube C. Raoult’s law A. Bacteria
D. Boyle’s law B. Molds
Answer: A C. Virus
Answer: B D. Algae
22. A major cause of air pollution resulting from the
burning of fuel oil is: 28. All of the following pollutants are produced Answer: B
A. Nitrous because of decaying organic matter except:
B. Hydrogen A. Ammonia 34. Turbidity in water is due to:
C. Sulfur dioxide B. Sulfur dioxide A. Algae
D. Silicon C. Methane B. Fungi
D. Hydrogen sulfide C. Organic salts
Answer: C D. Finally divided particles of clay, silt and
Answer: B organic matter
23. Heavy water is:
A. B2 O 29. The velocity of radiations (v) and the frequency of Answer: D
B. H2 O radiations (f) are related to the wavelength of
C. W2 O radiations by:
35. Per capita consumption of water is generally taken Answer: D D. Reflection and absorption
as:
A. 50 – 100 liters 41. Which has an octane rating of more than 100? Answer: C
B. 150 – 300 liters A. Gobar gas
C. 400 – 500 liters B. Methyl alcohol 47. The operation that produces highest noise level is
D. 750 – 1000 liters C. Benzol :
D. Ethyl alcohol A. Welding
Answer: B B. Riveting
Answer: C C. Machining
36. Which of the following industries have the highest D. Pressing
consumption of water for processing? 42. The part per million is identical to:
A. Foundry A. Grains per gallon Answer: B
B. Automobile industry B. Pounds per cubic foot
C. Paper mill C. Milligrams per kg 48. The ratio absorbed by the transfer fluid to the
D. Aluminum industry D. Tones per acre foot original incident energy striking the collector.
A. Shading factor
Answer: C Answer: C B. Collector efficiency
C. Betz coefficient
37. The ratio activity of water because of strontium 43. In power plant an electrostatic precipitators is D. Transmittance
can be removed by: installed between:
A. Aeration A. Forced draft fan and induced draft fan Answer: B
B. Alum coagulation B. Furnace and chimney
C. Phosphate coagulation C. Primary air and secondary air 49. A swinging support constructed as part of the
D. Lime D. Furnace and forced draft fan vessel and that supports the manway cover when
it is unbolted and moved aside.
Answer: C Answer: B A. Davit
B. Handhole
38. Chemical used for checking the growth of algae in 44. At present, the number of true elementary C. Nozzle neck
reservoirs is particles, which include leptons and quarks, is D. Manway arm
A. Copper sulphate thought to be:
B. Alum A. 4 Answer: A
C. Bleaching powder B. 8
D. Brine C. 10 50. Is the subjective method in which the smoke
D. 12 density is visually compared to five standardized
Answer: C white-black grids.
Answer: D A. Ringelman scale
39. Which of the following is not the pollutant from a B. Dalton scale
sulfuric acid plants? 45. The effective size of a target atom that interacts C. Smoke spot scale
A. Sulfur dioxide with a moving particle is called its _______. D. Dew point scale
B. Sulfur trioxide A. Length
C. Acid moist B. Width Answer: A
D. Hydrogen sulfide C. Cross section
D. Pseudo-area 51. The ratio of fugacity at actual conditions to the
Answer: D fugacity at some reference state is known as:
Answer: C A. Compressibility
40. Which of the following is a cold cathode lamp? B. Activity
A. Sodium vapor lamp 46. Most nuclear particles can react with atoms in C. Gravimetric coefficient
B. High pressure mercury vapor lamp several different ways including ________. D. Saturation
C. Low pressure mercury vapor lamp A. Absorption
D. Neon lamp B. Scattering Answer: B
C. Absorption and scattering
52. A type of polymer used for detergents, milk Answer: A D. CFCs
bottles, oil containers and toys.
A. Polyvinyl chloride (PVC) 58. The removal of particulate matter from a gas flow Answer: A
B. Polystyrene (PS) by exposing the flow to a liquid or slurry is known
C. High density polyethelene (HDPE) as: 63. Equipment leaks from plant equipments are known
D. Polypropylene (PP) A. Stripping as ________.
B. Spraying A. Fugitive leaks
Answer: C C. Scrubbing B. Fugitive dusts
D. Absorption C. Fugitive exhausts
53. A type of polymer used for grocery bags a food D. Fugitive emissions
wraps. Answer: C
A. Polyvinyl chloride (PVC) Answer: D
B. Polystyrene (PS) 59. The drop in a solvent’s vapor pressure and the
C. Low density polyethelene (LDPE) increase in mole fraction as solute is added is 64. What is a substance that absorbs or retains
D. Polypropylene (PP) described by: moisture?
A. Dalton’s law A. Surface acting agent
Answer: C B. Henry’s law B. Dry ultra-fine coal
C. Raoult’s law C. Fossil fuel
54. A radioactive gas produced from the decay of D. Boyle’s law D. Humectant
radium within the rocks beneath a building.
A. Spills Answer: C Answer: D
B. Radon
C. Smoke 60. All of the following statements are characteristics 65. Is a soluble compound that reduces a liquid’s
D. Smog of bases except ________. surface tension or reduces the interfacial tension
A. They conduct electricity in aqueous solutions between a liquid and a solid.
Answer: B B. They turn red litmus paper blue A. Surface – acting agent or surfactant
C. They have a pH between 0 to 7 B. Humectant
55. Consists of ground – level ozone and peroxyacyl D. They neutralize acids forming salts and water C. Oxygenated fuel
nitrates (PAN) D. CFC
A. Smog Answer: C
B. Spills Answer: A
C. Sulfur oxide 61. All of the following statements about conjugate
D. Smoke acids and bases are true except _______. 66. A material of substance that is accidentally or
A. A conjugate acid results when a base intentionally introduced to the environment in a
Answer: A accept a proton quantity that exceeds what occurs naturally.
B. A conjugate base results when a base A. Waste
56. A term used to mean the corrective steps taken to accepts a proton B. Sludge
return the environment to its original condition. C. Strong acids tend to give weak conjugate C. Pollutant
A. Stabilization bases D. Biosolids
B. Remediation D. The Bronsted – Lowry theory defines bases
C. Greenhouse effect as proton accepetors Answer: C
D. Opacity
Answer: B 67. Refers to organic waste produced from biological
Answer: B waterwaste treatment processes.
62. Are any solid particulate matter that becomes A. Toxic waste
57. The burning of low – sulfur fuel is known as: airborne, with the exception of particulate matter B. Biosolids
A. Fuel switching emitted from the exhaust stack of a combustion C. Extrinsic waste
B. Fuel adding process. D. Process waste
C. Fuel binding A. Dusts or fugitive dusts
D. Fuel swapping B. Dioxins Answer: B
C. Fugitive emissions
68. A rule which states that any solid waste mixed with D. Asbestos 79. Are organic compounds manufactured in oily liquid
hazardous waste becomes hazardous. and solid forms through the late 1970s and
A. The mixture rule Answer: C subsequently prohibited.
B. The derived from rule A. Polychlorinated biphenyls (PCBs)
C. The environmental rule 74. Are highly concentrated liquid wastes produced in B. Polyvinyl chloride (PVC)
D. The hazard rule landfills. C. Trihalomethanes
A. Aerosols D. Plastic
Answer: A B. Leachates
C. Nitrogen dioxide Answer: A
69. A rule which states that any waste derived from D. Urea
the treatment of a hazardous waste remains a 80. A type of polymer used for Styrofoam cups and
hazardous waste. Answer: B “clam shell” food containers.
A. The mixture rule A. Polyvinyl chloride (PVC)
B. The treatment rule 75. What is the primary cause of smog formation? B. Polystyrene (PS)
C. The derived from rule A. Toxins C. Low density polyethelene (LDPE)
D. The hazard rule B. Dioxins D. Polypropylene (PP)
C. Oxidants
Answer: C D. Nitrogen oxides Answer: B

70. Generally refers to sulfur trioxide SO3 in the flue Answer: D 81. A type of polymer used for labels, bottles and
gas. housewares.
A. Acid rain 76. Is a water soluble organic compound prepared A. Polyvinyl chloride (PVC)
B. Acid gas from ammonia. It has significant biological and B. Polystyrene (PS)
C. Hydrochloric acids industrial usefulness. C. Low density polyethelene (LDPE)
D. Stack gas A. Urea or carbamide urea D. Polypropylene (PP)
B. Nitrite
Answer: B C. Chlorinated organics Answer: D
D. Oxidants
71. What consists of weak solutions of sulfuric, 82. A type of polymer used for clear bottles.
hydrochloric, and to a lesser extent, nitric acids? Answer: A A. Polyvinyl chloride (PVC)
A. Stack gas B. Polystyrene (PS)
B. Acid rain 77. Are by products of reaction between combustion C. Low density polyethelene (LDPE)
C. Acid compound products D. Polypropylene (PP)
D. Pollutant A. Photochemicals
B. Oxidants Answer: A
Answer: B C. Organics
D. Sediments 83. Refers to the high – temperature removal of tarry
72. A fibrous silicate mineral material that is inert, substances from the interior of the carbon granule,
strong and incombustible. Answer: B leaving a highly porous structure.
A. Fiberglass A. Absorbent
B. Plastic 78. “Particulate matter” is defined as all particles that B. Activated
C. Rubber are emitted by a combustion source. What is C. Breakthrough
D. Asbestos another term for particulate matter? D. Adsorbent
A. Dust
Answer: D B. Aerosol Answer: B
C. Biosolids
73. An insulator with superior tensile strength but low D. Sediments 84. A substance with high surface area per unit
heat resistance. weight, and intricate pore structure, and a
A. Rubber Answer: B hydrophobic surface.
B. Plastic A. Adsorbent substance
C. Fiberglass B. Adsorbent substance
C. Homogenous substance 90. All of the following occur during reduction of a D. Saturated solution
D. Activated substance substance except __________.
A. An increase in negative charge Answer: B
Answer: A B. Loss of electrons
C. An oxidation state decrease 96. What do you call the mixture when a solvent has
85. Is a high – temperature process that turns D. Reduction of the oxidizing agent dissolved as much solute as it can?
incinerator ash into a safe glass-like material. A. Solution
A. Advance oxidation Answer: B B. Mild solution
B. Biofitration C. Hydration
C. Vitrification 91. All of the following are units of energy except: D. Saturated solution
D. Bioventing A. Calories
B. Joules Answer: D
Answer: C C. Pascals
D. MeV 97. The equilibrium constant for weak solution is
86. Refers to the use of composting and soil beds. known as:
A. Biofiltration Answer: C A. Ionization constant
B. Biomediation B. Arrhenius exponent
C. Bioventing 92. What do you call the mixture if the solute particles C. Solubility product
D. Bioreactors of a solid suspended in a liquid are larger than D. La Chatelier’s constant
molecules?
Answer: A A. Solution Answer: A
B. Suspension
87. Are open or closed tanks containing dozens or C. Hydration 98. If the solute particles of a solid suspended in a
hundreds of slowly rotating disks covered with a D. Saturated solution liquid are larger than the molecules, the mixture is
biological film of microorganisms. known as:
A. Biofilter Answer: B A. Solution
B. Bioreactor B. Suspension
C. Biomediator 93. What do you call the mixture when a solvent has C. Hydration
D. Bioinventor dissolved as much solute as it can? D. Saturated solution
A. Solution
Answer: B B. Mild solution Answer: B
C. Hydration
88. Refers to the treatment of contaminated soil in a D. Saturated solution 99. When a solvent has dissolved as much solute as it
large plastic-covered tank. can, the mixture is called:
A. Bioventing Answer: D A. Solution
B. Biological recycling B. Suspension
C. Biocycle 94. The equilibrium constant for weak solution is C. Hydration
D. Bioremediation known as D. Saturated solution
A. Ionization constant
Answer: A B. Arrhenius exponent Answer: D
C. Solubility product
89. All of the following occur during oxidation of a D. La Chatelier’s constant 100. When excess solute in a solution settles to the
substance except __________. bottom of the container, the process is called:
A. Oxidation state increases Answer: A A. Salvation
B. Loss of electrons B. Deemulsification
C. The substance becomes less negative 95. What do you call the mixture if the solute particles C. Precipitation
D. Oxidation of the oxidizing agent of solid suspended in a liquid are larger than the D. Equation
molecules?
Answer: D A. Solution Answer: C
B. Suspension
C. Hydration
101. All of the following express units of concentration C. Endothermic heat solution 112. All of the following factors affect rates of reaction
except: D. Exothermic heat solution except:
A. Normality A. Exposed surface area
B. Molarity Answer: C B. Concentrations
C. Formality C. Temperature
D. Isotropy 107. The absorption of water by a dessicant often D. Pressure
demonstrates:
Answer: D A. Heat of fusion Answer: D
B. Heat of vaporization
102. The desirable temperature inside an air C. Endothermic heat of solution 113. Le Chatelier’s principle predicts the direction of a
conditioned auditorium is: D. Exothermic heat of solution state of chemical equilibrium based on all of the
A. 0 C ̊ following factors except:
B. 5 C ̊ Answer: D A. Temperature
C. 10 C ̊ B. Specific volume
D. 20 C ̊ 108. A substance that absorbs moisture from the air is: C. Pressure
A. Deliquescent D. Concentration
Answer: D B. Efflorescent
C. Effervescent Answer: B
103. Which of the following acts as ignition accelerator D. A precipitant
for internal combustion engine fuels? 114. The amount of energy necessary to cause a
A. Acetone peroxide Answer: A reaction to occur is called the _______.
B. Hydrogen peroxide A. Heat of formation
C. Aromatic compounds 109. The removal of a dissolved gas or other volatile B. Heat of solution
D. N-heptane component from liquid by exposing the liquid to air C. Activation of energy
or steam is known as: D. Heat of fusion
Answer: A A. Stripping
B. Gas absorption Answer: C
104. Which of the following does not use ambient air for C. Spraying
propulsion? D. Scrubbing 115. The equilibrium constant for weak solutions is
A. Turbo jet known as ________.
B. Turbo-prop Answer: A A. The ionization constant
C. Pulse jet B. The Arrhenius exponent
D. Rocket 110. Which of the following contribute to the C. The solubility product
deterioration of the Earth’s ozone layer? D. Le Chatelier’s principle
Answer: B A. Carbon monoxide
B. Chlorofluorocarbons Answer: A
105. Most observed properties of light and other radiant C. Carbon dioxide
energy are consistent with waves in nature, but in D. Refrigerants 116. The speed at which a reaction proceeds to
interactions with matter, electromagnetic energy equilibrium is the purview of _________.
behaves as though it consists of discrete pieces or Answer: B A. Reaction kinetics
A. Blocks B. Le Chatelier’s principle
B. Balls 111. Ozone filters out what type of radiation that C. Neutralization
C. Quanta damages crops and causes skin cancer? D. Ionization
D. Atomic masses A. Tropospheric radiation
B. Atmospheric radiation Answer: A
Answer: C C. X-rays
D. Ultraviolet radiation 117. The color of water is measured:
106. The amount of energy absorbed when a A. In terms of pH value
substance enters a solution is called the: Answer: D B. In terms of degree of hardness
A. Heat of fusion C. In terms of platinum cobalt scale
B. Heat of sublimation D. Ppm of dissolved solids
Answer: C C. Supersonic 4. Liquids and gases take the following
D. Relativistic characteristic(s) of their contents.
118. What turbidimeter that gives direct readings in A. Volume
ppm? Answer: C B. Shape
A. Jackson turbidimeter C. Shape and volume
B. Hellige turbidimeter 124. The reduction of nuclear radiation intensity (called D. Neither shape nor volume
C. Baylis turbidimeter attenuation) is accomplished by:
D. All of the above A. Matter Answer: B
B. Antimatter
Answer: A C. Shielding 5. Alcohol finds use in manometers as:
D. Neurons A. It provides a suitable meniscus for the
119. Which of the following is not a sound absorptive inclined tube
material? Answer: C B. Its density being less can provide longer
A. Rugs length for a pressure difference, thus more
B. Mirrors 125. The ability of substance to absorb neurons accuracy can be obtained
C. Carpets depends upon its _________. C. A and B above are correct
D. Heavy drapes A. Absorption cross section D. Cheap and easily available
B. Scattering cross section
Answer: B C. Total cross section Answer: C
D. Atomic number
120. The total cross section of a target atom is made up 6. Which of the following statements about a
of ___________. Answer: A Newtonian fluid is most accurate?
A. The absorption cross section A. Shear stress is proportional to strain
B. The scattering cross section CHAPTER 11 – FLUID MECHANICS B. Viscosity is zero
C. The absorption and scattering cross sections C. Shear stress is multi – valued
D. The reflection and absorption cross sections 1. If the energy of the incident photon is less than the D. Shear stress is proportional to rate of strain
work function:
Answer: C A. An electron will be ejected Answer: D
B. More than one electron will be ejected
121. Which of the following cannot be a phenomenon of C. An electron will not be ejected 7. The normal stress is the same in all directions at a
neutron interactions? D. Less than one electron will be ejected point in fluid:
A. Inelastic scattering A. Independent of the motion of one fluid layer
B. Elastic scattering Answer: C relative to an adjacent layer
C. Fission B. When there is no motion of one fluid layer
D. Fusion 2. For supersonic flow, the pressure of fluid must relative to an adjacent layer
decrease as the fluid flow area of the duct. C. Only if the fluid is frictionless
Answer: D A. Increases D. Only if fluid is frictionless and incompressible
B. Decreases
122. Which of the following cannot be used to describe C. Remain the same Answer: B
neutron kinetic energy levels? D. None of these
A. Cold 8. Which of the following is not a characteristic of
B. Thermal Answer: A fluid pressure?
C. Slow A. It is the same in all directions at a point in the
D. Freezing 3. Density in term of viscosity is: fluid
A. Kinematic viscosity / dynamic viscosity B. Its acts normal to a surface
Answer: D B. Dynamic viscosity / kinematic viscosity C. It is a shear stress
C. Kinematic viscosity x dynamic viscosity D. It is linear with depth
123. All of the following are words used to describe D. None of the above
neutron kinetic energy levels except: Answer: C
A. Slow (resonant) Answer: B
B. Fast
9. The length of mercury column at a place at an B. surface tension D. States that energy is constant everywhere in
altitude will change with respect to that at ground C. bulk modulus the fluid
in: D. hysteresis
A. A linear relation Answer: D
B. A parabolic relation Answer: D
C. Will remain constant 20. A pilot tube can be used to measure fluid velocity
D. First slowly and then steeply 15. Which of the following can be used to measure the as described by the Bernoulli’s equation and the
flow of water in a pipe of diameter 3000 mm? relationship between:
Answer: D A. Venturimeter A. Kinetic energy and static pressure
B. Rotameter B. Fluid pressure and height of the fluid
10. All of the following dimensionless parameters are C. Nozzle C. Fluid pressure and impact energy
applicable to fluid flow problems except the D. Pilot tube D. Pressure and momentum
_______.
A. Reynolds number Answer: D Answer: A
B. Froude number
C. Mach number 16. The pressure at a given depth due to several 21. In order to avoid vaporization in the pipe line, the
D. Biot number immiscible liquids is: pipe line over the ridge is laid in such a way that it
A. The average of the individual pressures is not more than:
Answer: D B. The sum of the individual pressures A. 2.4 m above the hydraulic gradient
C. Independent of the individual pressures B. 6.4 m above the hydraulic gradient
11. Mass density of liquid (ρ) is given by which of the D. Unknown C. 10.0 m above the hydraulic gradient
following? D. 5.0 m above the hydraulic gradient
A. Ρ = Mass / volume Answer: B
B. Ρ = metric slug / 𝑚2 Answer: B
C. Ρ = kg 𝑠𝑒𝑐 2 / 𝑚4 17. The equation of continuity of flow is applicable if:
D. all of the above A. The flow is one dimensional 22. The stream function is a useful parameter in
B. The flow is steady describing_____________.
Answer: D C. The velocity is uniform over the cross – A. The conservation of mass
section B. The conservation of momentum
12. the speed of sound in all fluid is most closely D. All of the above conditions are together C. The conservation of energy
related to all of the following properties D. The equation of state
except________. Answer: D
A. Compressibility Answer: A
B. Density 18. Uniform flow takes place when:
C. Bulk module A. Conditions remain unchanged with time at 23. For high speed flows, the potential energy of fluids
D. Thermal conductivity any point are:
B. Rate of change of velocity of fluid is zero A. Positive
Answer: D C. At every point the velocity vector is identical B. Negative
in magnitude and direction for any given C. Negligible
13. Under which condition, the specific weight of water instant D. None of these
is 1000 kg/𝑚3 ? D. The change in transverse direction is zero
A. At normal pressure of 760 mm Answer: C
B. At 4 C ̊ temperature Answer: C
C. At mean sea level 24. McLeod gauge used for low pressure
D. All of the above 19. The continuity equation of an ideal fluid flow. measurement operates on the principle of
A. States that the net rate in – flow into any _________.
Answer: D small volume must be zero A. Gas law
B. Applies to irrotational flow only B. Boyle’s law
14. All of the following can be characteristics of fluids C. States that the energy remains constant C. Charles law
except_________. along streamline D. Pascal’s law
A. kinematic viscosity
Answer: B Answer: A B. Upward in the direction of the flow
C. Level (no slope)
25. A Kaplan turbine is 31. For stable equilibrium of floating body its D. There is no effect of friction on the energy
A. A high head mixed flow turbine metacenter should be: grade line
B. An impulse turbine, inward flow A. Below the center of gravity
C. A reaction turbine, outward flow B. Below the center of buoyancy Answer: A
D. Low head axial flow turbine C. above the center of buoyancy
D. above the center of gravity 37. The pilot tube is a device used for measurement of
Answer: D A. Pressure
Answer: D B. Flow
26. The most common method for calculating frictional C. Velocity
energy loss for laminar flowing fluids in noncircular 32. Center of pressure on an inclined plane has ___. D. Discharge
pipe is: A. At the centroid
A. The Darcy equation B. Above the centroid Answer: C
B. The Hagan – Poisevill equation C. Below the centroid
C. The Hazen - Williams equation D. At metacenter 38. Hydrometer is used to find out
D. The Swamee – Jin equation A. Specific gravity liquids
Answer: C B. Specific gravity solids
Answer: A C. Specific gravity gases
33. The line action of the buoyant forces always acts D. Relative humidity
27. The parameter f in the expression for head-loss is through the centroid of the ______.
A. The fraction of flow that is totally turbulent A. Submerged body Answer: A
B. The Darcy friction factor B. Volume of the floating body
C. The height of roughness scale in turbulent C. Volume of the fluid vertically above the body 39. The fluid forces taken into consideration in the
flow D. Displaced volume of the fluid Navier Strokes equation are:
D. The static coefficient of friction A. Gravity, pressure and viscous
Answer: D B. Gravity, pressure and turbulent
Answer: B C. Pressure, viscous and turbulent
34. The hydraulic grade line of a pipe denotes which D. Gravity, viscous and turbulent
28. Friction factor for both laminar and turbulent flows of the following?
can be found plotted in a A. Total energy Answer: A
A. Steam table B. Pressure energy
B. Psychometric chart C. Potential energy 40. Permissible velocity of water flowing through
C. Moody diagram D. The sum of pressure energy and potential concrete tunnel, is generally
D. Mollier diagram energy A. 4-5 m/s
B. 10-12 m/s
Answer: C Answer: D C. 13-16 m/s
D. 20 m/s
29. Which of the following is relative velocity? 35. The energy grade line of a pipeline denotes which
A. The difference between two velocities of the following? Answer: A
B. Average velocity A. Total energy
C. Sum of two velocities B. Pressure energy 41. Orifice refers to an opening
D. Vector difference of two velocities C. Potential energy A. With closed perimeter and of regular form
D. The sum of pressure energy and potential through which water flows
Answer: D energy B. With prolonged sides having length of 2 to 3
diameters of opening in thick wall
30. Which of the following is the highest head? Answer: A C. With partially full flow
A. 33 inch Hg D. In hydraulic structure with regulation
B. 31.0 ft. water 36. The presence of friction in the energy grade line provision
C. 1.013 kg kg/𝑐𝑚2 will always cause the line to slope
D. 75.0 cm of Hg A. Down in the direction of the flow Answer: D
42. The value of coefficient of discharge in A. Sonic C. Kutter’s C
comparison to coefficient of velocity is found to B. Sub-sonic D. Value of k in Darcy - Weisbach formula
be_______. C. Supersonic
A. More D. Supersonic on one side and sub-sonic on the Answer: B
B. Less other side
C. Same 53. What is the coefficient of contraction?
D. More/less depending on flow Answer: D A. The ratio of the area of vena contracta to the
area of the orifice
Answer: B 48. Which of the following is the basic of Bernoulli’s B. The ration of actual discharge to the
law for fluid flow? theoretical discharge
43. Weir refers to an opening A. Continuity equation C. The ratio of the actual velocity to the
A. Having closed perimeter and of regular form B. Principle of conservation of energy theoretical velocity
through which water flows C. Fourier’s law D. The ratio of the effective head to the actua;
B. Having prolonged sides with length of 2 to 3 D. Principle of conservation of mass head
diameters of opening in thick wall
C. Having partially full flow Answer: B Answer: A
D. In hydraulic structures with regulation
provision 49. Which of the following is NOT a characteristic of 54. Where is vena contracta most likely loacated?
fluid pressure? A. At the orifice
Answer: C A. It is a shear stress B. At a distance approximately ½ the diameter
B. It is the same in all directions at a point in the of the orifice
44. Which of the following parameters determine the fluid C. At a distance approximately equal to the
friction factor of turbulent flow in a rough pipe? C. It acts normal to a surface diameter of the orifice
A. Froude number and relative roughness D. It is linear with depth D. At a distance approximately twice the
B. Froude number and Mach number diameter of the orifice
C. Reynolds number and relative roughness Answer: A
D. Mach number and relative roughness Answer: B
50. Refers to the compressibility of a fluid, the
Answer: C fractional change in fluid volume per unit change 55. A substance that is able to flow and yields to any
in fluid. force tending to change its shape without
45. Power transmitted through a pipe is maximum A. Viscosity changing its volume such as water and air.
when the loss of head due to friction is: B. Bulk modulus A. Fluid
A. One-half of the total head supplied C. Density B. Flux
B. One-third of the total head supplied D. Pressure C. Gas oil
C. One-fourth of the total head supplied D. Water gas
D. Equal to the total head supplied Answer: D
Answer: A
Answer: B 51. A pilot tube can be used to measure fluid velocity
as described by the Bernoulli’s equation and the 56. The velocity of a fluid particle at the center of the
46. In a nozzle if back pressure is same as inlet relationship between: pipe section is______.
pressure; then_______________. A. Kinetic energy and static pressure A. Maximum
A. No flow takes place B. Fluid pressure and static pressure B. Minimum
B. Maximum flow takes place C. Fluid pressure and impact energy C. Average
C. Flow becomes subsonic in diverging section D. Pressure and momentum D. Logarithmic average
D. Flow becomes supersonic in converging as
well as supersonic section Answer: A Answer: A

Answer: A 52. The ratio of the area to the wetted penmeter is 57. For supersonic flow, the pressure of fluid must
known as __________. increase as the fluid flow area of the duct:
47. The flow on two sides of a normal shock wave is A. Flow factor A. Increases
called___________. B. Hydraulic radius B. Decreases
C. Constant 63. Which of the following is not a characteristic of real B. Density
D. None of these fluids? C. Pressure
A. Finite viscosity D. Viscosity
Answer: B B. Non-uniform velocity distributions
C. Compressibility Answer: C
58. Which is incorrect statement regarding apparent D. Experience of eddy current and turbulence
shear forces. 69. Property of a fluid whereby its own molecules are
A. It can never be found in frictionless fluid Answer: D attracted is known as ________.
regardless of its motion A. Adhesion
B. It can never be found when the fluid is at rest 64. Which of the following is not the mass density of B. Cohesion
C. It depends upon cohesive forces water? C. Surface tension
D. It may occur owing to cohesion when the fluid A. 62.5 lbm/𝑓𝑡 3 D. Viscosity
is at rest B. 100 kg/𝑚3
C. 1 g/𝑐𝑚3 Answer: B
Answer: D D. 1 kg/L
70. The term subsonic flow refers to a flowing gas with
59. The time required for half a quantity of radioactive Answer: B a speed:
particles to decay (disintegrate) is called A. Less than the local speed of sound
its_____________. 65. The upper critical Reynolds number for pipe flow B. Equal to the speed of sound
A. Average life is: C. Greater than the speed of sound
B. Median life A. Of no practical importance to designers D. Much greater than the speed of sound
C. Time constant B. Always used to design pipes for strength
D. Half time C. The number at which turbulent flow changes Answer: A
over to laminar flow
Answer: D D. The number at which laminar flow changes 71. The pressure at a point in a fluid will not be same
into turbulent flow in all the directions if the fluid is:
60. SI unit of viscosity is: A. Viscous
A. 10 times poise Answer: A B. Viscous and static
B. 9.81 times poise C. Inviscous and in motion
C. 1/9.81 time poise 66. Which of the following statements about gauge D. Viscous and is in motion
D. 1/10 times poise pressure is most correct? Gauge pressure are
measured relative to _________. Answer: D
Answer: A A. Atmospheric pressure
B. A vacuum 72. The statement that “the hydrostatic pressure a
61. For computation convenience, fluids are usually C. Each other fluid exerts on an immersed object or on container
classed as: D. The surface walls is a function only of fluid depth” is
A. Rotational or irrotational A. The perfect gas law
B. Real or ideal Answer: A B. D’Alembert’s paradox
C. Laminar or turbulent C. The hydrostatic paradox
D. Newtonian or non-newtonian 67. The volumetric change of the fluid caused by a D. Boyle’s law
resistance is called ________.
Answer: B A. Volumetric strain Answer: C
B. Volumetric index
62. Which of the following is not a dimensionless C. Compressibility 73. Bernoulli’s equation is s/an ___________.
parameter? D. Adhesion A. Momentum equation
A. Kinetic viscosity B. Conservation of energy equation
B. Weber number Answer: D C. Conservation of mass equation
C. Darcy Weisbach friction factor D. Equation of state
D. Froude number 68. Compressibility of a fluid relates the fractional
change in fluid volume per unit change in fluid. Answer: B
Answer: A A. Temperature
74. An ideal fluid is one that: D. Important only in supersonic flow 85. Which of the following turbine is different from the
A. Is very viscous others?
B. Obeys Newton’s law of viscosity Answer: C A. Fourneyron turbine
C. Is assumed in problems in conduit flow B. Francis turbine
D. Is frictionless and incompressible 80. Fully turbulent flow in a pipe is characterized by all C. Kaplan turbine
of the following except: D. Pelton wheel
Answer: D A. A parabolic velocity profile
B. A momentum exchange due to fluid masses Answer: D
75. The relationship between pressure and altitude in rather than molecules
the atmosphere is given by the: C. A maximum velocity at the fluid center line 86. Running away speed of a Pelton wheel gives:
A. Perfect gas law D. A 1/7 velocity profile A. Actual operating speed
B. Conservation of mass B. No load speed
C. Barometric height relationship Answer: A C. Full load speed
D. First law of thermodynamics D. No load speed when governor mechanism
81. The laminar friction factor of fluid flowing through a fails
Answer: C pipe is a function of all of the following except:
A. Fluid velocity Answer: D
76. The fact the buoyant force on a floating object B. Pipe diameter
equal to the weight of the water displaced is: C. Pipe roughness 87. Which of the following turbine is different from the
A. Bernoulli’s law D. Reynolds number others?
B. Archimedes’ principle A. Pelton wheel
C. The law of diminishing returns Answer: C B. Banki turbine
D. The conservation of mass C. Jonval turbine
82. The continuity equation is applicable to: D. Kaplan turbine
Answer: B A. Viscous unviscous fluid
B. Compressibility of fluids Answer: D
77. Which of the following terms does not appear in C. Conservation of mass
the steady flow energy equation (the extended D. Steady unsteady flow 88. The characteristic length of the Reynold’s number
Bernoulli’s equation)? used to calculate the friction in noncircular full
A. Kinetic energy Answer: C running pipes is based on the __________.
B. Potential energy A. Run length
C. Friction losses 83. The rise or fall of head ‘h’ in a capillary tube of B. Pipe length
D. Hysteresis losses diameter ‘d ‘ and liquid surface tension ‘s’ and C. Hydraulic diameter (the equivalent diameter)
specific weight ‘w’ is given by: D. Wetted circumference
Answer: D A. 4s/wd
B. 4ds/w Answer: C
78. Neglecting the forces due to inertia, gravity and C. 4wd/s
frictional resistance, the design of a channel can D. 4ws/d 89. The hydraulic radius of noncircular pipe is:
be made by comparing A. The square root of the flow area
A. Weber number Answer: A B. The ratio of the area to the wetted perimeter
B. Reynolds number C. The radius of a pipe of equivalent area
C. Froude’s number 84. The study of the practical laws of fluid flow and the D. None of the above
D. Prant’l number resistance of open pipes and channels.
A. Fluid mechanics Answer: B
Answer: C B. Hydraulics
C. Aerodynamics 90. The Darcy equation can be used for all liquids and
79. The difference between stagnation pressure and D. Thermodynamics flows except:
total pressure is: A. Water
A. Due to height difference Answer: B B. Alcohol
B. Due to fluid kinetic energy C. Gasoline
C. None of the terms are interchangeable D. Air flowing supersonically
Answer: D D. There is no effect of friction on the energy D. Viscosity factor
grade line
91. The Hazen – Williams formula for head loss due to Answer: A
friction is based upon: Answer: A
A. Rigorous mathematical derivation 102. If the Mach number is greater than 1 but lesser
B. Empirical data 97. The presence of a minor loss in the energy grade than 5, what is the standard classification of the
C. Semi-empirical analysis line will cause the line to slope: travel?
D. Screndipity A. Down in the direction of the flow A. Transonic travel
B. Upward in the direction of the flow B. Subsonic travel
Answer: B C. Vertically downward C. Hypersonic travel
D. There is no effect of friction on the energy D. Supersonic travel
92. The extended Bernoulli equation includes all of the grade line
following terms except: Answer: D
A. Potential energy Answer: A
B. Kinetic energy 103. What is measured by a Pitot tube?
C. Nuclear energy 98. What do you call the pressure which the fluid A. Volumetric discharge
D. Friction losses exerts on an immersed object or container walls? B. Mass flow
A. Normal pressure C. Pressure
Answer: C B. Standard liquid pressure D. Velocity
C. Hydrostatic pressure
93. An equipotential line is one that: D. Gage pressure Answer: D
A. Has no velocity component tangent to it
B. Has uniformly varying dynamic pressure Answer: C 104. What is the difference between the energy grade
C. Has no velocity component normal to it line and the hydraulic grade line?
D. Exists in case of rotational flow 99. Viscosity for a fluid is defined as the constant of A. Potential energy
proportionality between shear stress and what B. Pressure energy
Answer: A other variable? C. Kinetic energy
A. The spatial derivative of velocity D. Friction losses
94. What is the use of a Hydraulic jump? B. The time derivative of pressure
A. Increase the flow rate C. The time derivative of density Answer: C
B. Reduce the flow rate D. The spatial derivative of density
C. Reduce the velocity of flow 105. Kinetic energy is not neglected in calculations of:
D. Reduce the energy of flow Answer: A A. High speed flow
B. Low speed flow
Answer: D 100. What is the classification of the fluid flow if the fluid C. Steady flow
travels parallel to the adjacent layers and the D. Equilibrium flow
95. What do you call the lowest portion to storage paths of the individual particles do not cross each
basin from where the water is not drawn? other? Answer: A
A. Bottom storage A. Steady flow
B. Sub soil storage B. Laminar flow 106. Discharge losses through orifice are due to:
C. Spring reserve C. Uniform flow A. Friction losses
D. Dead storage D. Turbulent flow B. Minor losses
C. Both friction and minor losses
Answer: D Answer: B D. Pressure losses

96. The presence of friction in the hydraulic grade line 101. Which of the following refers to the measure of a Answer: C
will always cause the line to slope: fluid’s sensitivity to changes in viscosity with
A. Down in the direction of the flow changes in temperature? 107. Which of the following is considered as an
B. Upward in the direction of the flow A. Viscosity index important parameter in the study of compressible
C. Level (no slope) B. Coefficient of viscosity flow?
C. Viscosity ratio A. Speed of fluid
B. Speed of sound C. M>1 The volumetric change of the fluid caused by a
C. Speed of light D. M=1 resistance is called:
D. Speed of fluid flow A. Volumetric change
Answer: D B. Volumetric index
Answer: B C. Compressibility
114. The pressure decreases as the temperature and D. Adhesion
108. Is the velocity at which an infinitesimal small velocity increases while the fluid velocity and
pressure wave travels through a medium. Mach number: Answer: D
A. Subsonic velocity A. Increases
B. Hypersonic velocity B. Decreases 120. Past ME Board Question
C. Sonic velocity C. Remains constant The energy of a fluid flowing at any section in a
D. Monatomic velocity D. None of these pipeline is a function of:
A. Velocity of flow only
Answer: C Answer: A B. Pressure only
C. Height above a chosen datum, density,
109. It is the ratio of the actual velocity of the fluid to the 115. The Mach number is unity or one at the location of internal energy, pressure and velocity of flow
velocity of sound. smallest flow area, called the: D. Pressure, height above a chosen datum,
A. Mach number A. Decreasing are velocity of flow, density of fluid
B. Froude number B. Throat
C. Sonic number C. Increasing area Answer: C
D. Euler number D. None of these
121. Past ME Board Question
Answer: A Answer: B If the fluid travels parallel to the adjacent layers
and the paths of individual particles do not cross,
110. The flow is called sonic when Mach number is: 116. What happens to the velocity of fluid after passing the fluid is said to be:
A. Equal to 1 the throat although the flow area A. Turbulent
B. Less than 1 A. Increases rapidly B. Critical
C. More than 1 B. Decreases rapidly C. Dynamic
D. None of these C. Remains constant D. Laminar
D. None of these
Answer: A Answer: D
Answer: A
111. The following flow is sub-sonic when Mach no. is: 122. Past ME Board Question
A. Greater than 1 117. Which of the following is an example of Newtonian Center of pressure on an inclined plane lies:
B. Less than 1 fluid? A. At the centroid
C. More than 1 A. Motor oils B. Above the centroid
D. None of these B. Gas C. Below the centroid
C. Paints D. At the metacenter
Answer: B D. Clay slurries
Answer: C
112. The flow is supersonic when Mach no. is: Answer: B
A. Greater than zero 123. Past ME Board Question
B. Less than 1 118. What is the critical pressure of water? At any instant, if the number of particles passing
C. Greater than 1 A. 150 kg/cm3 every cross-section of the stream is the same, the
D. None of these B. Less than 200 kg/cm2 flow is said to be:
C. More than 200 kg/cm2 A. Steady flow
Answer: C D. 100 kg/cm2 B. Uniform flow
C. Continuous flow
113. The flow is transonic when Answer: C D. Laminar flow
A. M = 0
B. M < 1 119. Past ME Board Question Answer: A
124. Past ME Board Question D. 1 + c + c(𝑝2 /𝑝1 )𝑛+1 7. Which of the following give the optimum intermediate
The ratio of cross-sectional area of flow to the pressure in case of two stage compression?
𝑝 +𝑝
wetted perimeter is: Answer: A A. 𝑝2 = 1 2
2
A. Hydraulic lead 𝑝1 +𝑝2
B. 𝑝2 = 𝑝1 +
B. Hydraulic section 2. In case of axial flow compressors for minimum fluid 2

C. Hydraulic mean depth friction and blade tip clearance losses, the blades of C. 𝑝2 = √𝑝1 𝑝3
D. Hydraulic gradient an axial flow compressor are designed for____ D. 𝑝2 = 𝑝1+ √𝑝1 2 + 𝑝3 2
A. 80% reaction
Answer: C B. 85% reaction Answer: C
C. 60% reaction
125. Past ME Board Question D. 53% reaction 8. In air compressor performance curve a surge line
If A is the cross-sectional area of the flow and Pw represents what?
is the wetted perimeter of a pipe, then what is the Answer: D A. Limit of compressor efficiency
hydraulic depth. Hd ? B. Limit of compressor discharge
A. Pw – A 3. Centrifugal blowers can supply C. Limit of stable operation
B. Pw / A A. Large volumes of air at low pressures D. Lower critical speed of shaft
C. A / Pw B. Small volumes of air at high pressures
D. Pw x A C. Large volumes of air at high pressures Answer: C
D. Small volumes of air at low pressures
Answer: C 9. Which of the following is a displacement
Answer: A compressor?
126. Past ME Board Question A. Reciprocating air compressor
If Q is the volume in gallon; D is height or 4. Which of the following is a safety device on a B. Vane blower
elevation in ft. and m is weight in lbs. per gallon, compressor? C. Centrifugal blower
what is the desired energy to lift the water from A. Relief valve D. Axial flow compressors
lower to higher elevation? B. Strainer
A. E = mD/Q C. Over speed shut down Answer: B
B. E = mDQ D. Over pressure shut down
C. E = mQ/D 10. Stalling of the blades of axial flow compressor is:
D. E = QD/m Answer: A A. An unsteady periodic and reversal of flow
B. The fixed mass flow rate irrespective of
Answer: B 5. In a four stage compressor system, the first and third pressure ratio
stage pressures are 1 and 9 kg/cm2 respectively. C. The reduction in lift force at higher angle of
127. Past ME Board Question What is the fourth stage delivery pressure? incidence
The flow of the convergent section of a nozzle is A. 9 kg/cm2 D. All of the above
always subsonic. If the flow is subsonic then the B. 81 kg/cm2
Mach number is: C. 27 kg/cm2 Answer: C
A. Greater than unity D. 243 kg/cm2
B. Less than unity 11. Surging is
C. Near unity Answer: C A. An unsteady, periodic and reversal of flow in
D. Unity the compressor
6. For a six compression of air set, the minimum work B. The fixed mass flow rate irrespective of
Answer: B conditions are: pressure ratio
A. Pressure rise per stage will be equal C. The reduction in lift force at higher angle of
CHAPTER 12 – FLUID MACHINERY B. Work done in successive stages will be in incidence
geometrical progression D. None of the above
1. The volumetric efficiency of a compressor having C. Cylinder volumes will be same
clearance factor is given by: D. Temperature rise in the cylinders will be the Answer: A
A. 1 + c – c(𝑝2 /𝑝1 )1/𝑛 same
B. 1 + c + c(𝑝2 /𝑝1 )1−𝑛 12. In an axial flow compressor, the pressure rise takes
C. 1 + c - c(𝑝2 /𝑝1 )𝑛 Answer: D place in:
A. Fixed blades only D. 70 to 90 psig D. All suction valves are open and all discharge
B. Moving blades only valves are open
C. Both fixed and moving blades Answer: B
D. None of the above Answer: B
18. The capacity of portable air compressors used for
Answer: C construction, mining, road building, and painting 23. Past ME Board Question
ranges from ______________. Pump used to increase air pressure above normal,
13. An axial flow compressor is suitable for: A. 1 𝑓𝑡 3 /min to 2000 𝑓𝑡 3 /min air is then used as a motive power.
A. High volume flow rates with small pressure B. 30 𝑓𝑡 3 /min to 2500 𝑓𝑡 3 /min A. Air cooled engine
rise C. 1 𝑓𝑡 3 /min to 2500 𝑓𝑡 3 /min B. Air compressor
B. Low volume flow rates with low pressure rise D. 30 𝑓𝑡 3 /min to 2000 𝑓𝑡 3 /min C. Air condenser
C. High volume flow rates with high pressure D. Air injection
rise Answer: A
D. Low volume flow rates with high pressure rise Answer: B
19. The power of portable air compressors used for
Answer: A construction, mining, road building, and painting 24. A device used for raising fluids from a lower to higher
ranges from: level.
14. Which of the following is a positive displacement A. 1 hp to 500 hp A. Compressor
rotary compressor? B. ½ hp to 500 hp B. Pump
A. Roots blower C. 1 hp to 1000 hp C. Turbine
B. Centrifugal compressor D. ½ hp to 1000 hp D. Blowers
C. Axial flow compressor
D. None of the above Answer: B Answer: B

Answer: A 20. Most permanent installations use piston compressors 25. When a pump is opening at a vacuum of 4 in Hg,
available as stock items in size ranging from which of the following is not correct?
15. Choking is A. 1 𝑓𝑡 3 /min to 2000 𝑓𝑡 3 /min A. The pressure is 25.92 in Hg
A. Change of mass flow rate in proportion to B. 30 𝑓𝑡 3 /min to 2500 𝑓𝑡 3 /min B. The pressure is 10.721 psia
pressure ratio C. 1 𝑓𝑡 3 /min to 2500 𝑓𝑡 3 /min C. The pressure is 158.4 torr
B. Change of mass flow rate in inverse D. 30 𝑓𝑡 3 /min to 2000 𝑓𝑡 3 /min D. The pressure is 0.8663 atm
proportion to pressure ratio
C. Fixed mass flow rate irrespective of pressure Answer: B Answer: C
D. All of the above
21. Most permanent installations use piston compressors 26. The amount of energy actually entering the fluid from
Answer: C available as stock items of what pressure? a pump is the
A. 150 psi A. Brake horsepower
16. Crankshaft of a reciprocating compressor is basically B. 200 psi B. Hydraulic horsepower
made of what? C. 250 psi C. Theoretical horsepower
A. Semi-steel D. 300 psi D. Hydraulic, theoretical, or water horsepower
B. Aluminum alloy
C. Steel forging Answer: A Answer: D
D. Cast iron
22. When the piston is top center of the cylinder in a 27. Primary purpose of a pump in a fluid loop is to
Answer: D double-acting reciprocating compressor: A. Add energy to the flow
A. All suction valves are open and all discharge B. Add mass to the flow
17. Most plant air systems to operate the tools and valves are closed C. Extract energy from the flow
machines which require 70 to 90 psig are maintained B. All suction valves are closed and all discharge D. None of the above
at_______. valves are closed
A. 90 to 110 psig C. All suction valves are closed and all discharge Answer: A
B. 110 to 130 psig valves are open
C. 130 to 150 psig 28. The isentropic efficiency of a pump is given by the
A. Ratio of actual to ideal energy extracted 33. Past ME Board Question Answer: A
B. Ratio of ideal to actual energy supplied An aftercooler on a reciprocating air compressor is
C. Ratio of ideal to actual energy extracted used primarily to: 38. Past ME Board Question
D. Ratio of actual to ideal energy supplied A. Cool the lubricating oil The power required to deliver a given quantity of fluid
B. Condense the moisture in the compressed air against a given head with no losses in the pump is
Answer: B C. Improve compressor efficiency called:
D. Increase compressor capacity A. Wheel power
29. The electrical efficiency of a device is the ratio of the B. Brake power
: Answer: B C. Hydraulic power
A. Electrical energy output to the electrical energy D. Indicated power
input 34. Past ME Board Question
B. Mechanical energy input to the mechanical A receiver in an air compression system is used to Answer: C
energy output of the device A. Avoid cooling air before using
C. Actual energy extracted to the ideal energy B. Increase the air discharge pressure 39. Past ME Board Question
extracted C. Collect water and grease suspended in the air Fluids that are pumped in processing work are
D. Actual to the ideal energy input D. Reduce the work needed during compression frequently more viscous than water. Which of the
following statement is correct?
Answer: C Answer: C A. Reynolds number varies directly as the viscosity
B. Efficiency of a pump increases as the viscosity
30. The adiabatic pump efficiency is the ratio of the 35. Past ME Board Question increases
A. Electrical energy output to the electrical energy Crankshaft of reciprocating compressor is basically C. Increased fluid friction between the pump parts
input made of: and the passing fluid increased useful work.
B. Mechanical energy input to the electric energy A. Semi-steel D. Working head increases as the viscosity
output of the device B. Aluminum alloy
C. Ideal energy input to the pump to the actual C. Cast iron Answer: C
energy input D. Steel forging
D. Actual energy extracted to the ideal energy 40. Past ME Board Question
extracted Answer: C A reciprocating pump is considered positive
displacement pump because
Answer: C 36. Past ME Board Question A. Displacement of the liquid is affected by the
Cooling water system consists of equipment to displacement of the piston
31. A condenser’s water – circulating pump is belt-driven dissipate heat absorbed by the engine jacket water, B. Positive pressure is given to the liquid
by an electric motor with an adjustable pitch motor lube oil and the heat to be removed from air C. Liquid is discharge with positive pressure
sheave. This sheave intercooler is measurable to keep the engine outlet D. Liquid is lifted due to the vacuum created inside
A. Permits adjustment of pump speed water temperature constant and the differential of the the cylinder
B. Prevent overload abd/or burnout of the pump and cooling water at a minimum preferably not to exceed:
motor A. 10 to 30 F ̊ Answer: A
C. Prevents excessive wear on the belt and motor B. 10 to 50 F ̊
shaft bearings C. 10 to 20 F ̊ 41. Past ME Board Question
D. Facilitates belt replacement D. 10 to 40 F ̊ To protect adequately the engine bearings, what type
and better arrangement of lubricating oil filter is most
Answer: C Answer: C practical?
A. Full-flow type filter installed between the
32. The function of a pump or compressor is to 37. Past ME Board Question lubricating oil pump and the bearings
A. Transfer heat from one fluid to another Which of the following is one of the most popular B. Duplex filter installed before the lubricating pump
B. Increase the total energy content of the flow types of compressor utilized for supercharging C. Bypass filter with cleanable and replaceable
C. Extract energy from the flow engine? elements
D. Exchange heat to increase energy to the flow A. Roots type blower D. Splash lubricating system in the crank case
B. Pulse turbo charger
Answer: B C. Constant pressure turbo charger Answer: A
D. Turbo compressor
42. Past ME Board Question 47. Past ME Board Question Answer: A
Medium pressure when applied to valves and fittings In order that cavitation will not take place in the
are suitable for a working pressure of: suction line of a pump, what should be the sum of 52. Past ME Board Question
A. 862 to 1200 kPa the velocity head and pressure head at suction The actual head, neglecting the kinetic energy in
B. 758 to 1000 kPa compared to the vapor pressure of the liquid? which the pump work against.
C. 500 to 1000 kPa A. sufficiently lower A. Delivery head
D. 658 to 1050 kPa B. constant B. Pressure head
C. adequately greater C. Velocity head
Answer: A D. equal D. Suction head

43. Past ME Board Question Answer: C Answer: B


The function of an unloader on an electric motor-
driven compressor is to: 48. Past ME Board Question 53. Past ME Board Question
A. Reduce the speed of the motor when the Which of the following components of a pump Flow of water in a pipe have a velocity at 10 meters
maximum pressure is reached converts mechanical energy to pressure energy? per second. Determine the velocity head of the water
B. Drain the condensate from the cylinder A. Impeller A. 50.1 meters
C. Release the pressure in the cylinder in order to B. Valve B. 5.1 meters
reduce the starting load C. Shaft C. 8.2 meters
D. Prevent excess pressure in the receiver D. Delivery pipe D. 100 meters

Answer: C Answer: A Answer: B

44. Past ME Board Question 49. Past ME Board Question 54. Past ME Board Question
An unloader is used on air compressor to: Mr. De La Cruz wanted to buy a pump for his farm. Find the velocity head for a velocity of 18 m/s
A. Relieve air pressure What is suitable for deepwell? A. 33.0 m
B. Start easier A. Reciprocating B. 0.92 m
C. Stop easier B. Airlift C. 1.8 m
D. Run faster C. Hand lift D. 16.5 m
D. Centrifugal
Answer: A Answer: D
Answer: D
45. Past ME Board Question 55. Past ME Board Question
The performance of a reciprocating compressor is 50. Past ME Board Question The size of a steam reciprocating pumps is generally
expressed as: A tank contains H2O. what is the intensity of designated by a three-digit number size as 646. The
A. Adiabatic work divided by adiabatic input pressure at a depth of 6 meters? first digit designates.
B. Adiabatic work divided by indicated work A. 68 kPa A. Stroke of the pump in inches
C. Isothermal work divided by indicated work B. 58.8 kPa B. Inside diameter of the steam cylinder measured
D. Isothermal work divided by adiabatic work C. 78.0 kPa in inches
D. 48.7 kPa C. Percent clearance
Answer: B D. Number of cylinders
Answer: B
46. Past ME Board Question Answer: B
For reciprocating compressor slip at positive or 51. Past ME Board Question
negative displacement: In order to avoid cavitation the NPSH of an 56. Past ME Board Question
A. 𝐶𝑑 = 1 installation should be: If 𝑄𝑎 is the actual discharge flow and 𝑄𝑏 is the
B. 𝐶𝑑 > 1 A. At least equal or greater than the NPSH of the theoretical discharge flow, what will the coefficient of
C. 𝐶𝑑 < 1 pump discharge be equal to during positive displacement
D. 𝐶𝑑 = 0 B. At least equal or less than the NPSH of the pump slip?
C. Equal to the NPSH of the pump only A. 𝑄𝑎 x 𝑄𝑏
Answer: C D. Greater than the NPSH of the pump only B. 𝑄𝑏 /𝑄𝑎
C. 𝑄𝑎 /𝑄𝑏
D. 1 − 𝑄𝑎 /𝑄𝑏 Answer: C Answer: A

Answer: C 62. A centrifugal pump characterized by a housing which 67. A timber, concrete or masonry enclosure having a
is split parallel to the shaft screened inlet kept partially filled with water by an
57. Past ME Board Question A. Horizontal split case pump open body of water such as pond, lake, or steams.
Two pumps are connected in series. If 𝑄1 is the B. End suction pump A. Aquifer
discharge of pump 1 and 𝑄2 is the discharge of pump C. In line pump B. Wet pit
2 where 𝑄1 <𝑄2 . What is the discharge? D. Vertical shaft turbine pump C. Ground water
A. 𝑄2 D. Well water
B. 𝑄1 Answer: A
C. 𝑄1 +𝑄2 Answer: B
D. 𝑄1 /𝑄2 63. A centrifugal pump with one or more impellers
discharging into one or more bowls and a vertical 68. Water which is available from a well, driven into
Answer: A eductor or column pipe used to connect the bowls to water bearing subsurface strata (aquifer)
the discharge head on which the pump driver is A. Aquifer
58. A pump in which the pressure is developed mounted. B. Wet pit
principally by the action of centrifugal force A. Horizontal split case pump C. Ground water
A. Centrifugal pump B. End suction pump D. Well water
B. End suction pump C. In line pump
C. In line pump D. Vertical shaft turbine pump Answer: C
D. Horizontal pump
Answer: D 69. The level with respect to the pump, of the body of
Answer: A water from which it takes suction when the pump is
64. A pump that takes suction from a public service main not in operation.
59. A single suction pump having its suction nozzle on or private use water system for the purpose of A. Static water level
the opposite side of the casing from the stuffing box increasing the effective water pressure. B. Pumping water level
and having the face of the suction nozzle A. Horizontal split case pump C. Suction head
perpendicular to the longitudinal axis of the shaft. B. Submersible pump D. Discharge head
A. Centrifugal pump C. Booster pump
B. End suction pump D. Vertical shaft turbine pump Answer: A
C. In line pump
D. Horizontal pump Answer: C 70. The level with respect to the pump, of the body of
water from which it takes suction when the pump is in
Answer: B 65. A vertical turbine pump with the pump and motor operation.
closed coupled and design to be installed A. Static water level
60. A pump with the shaft normally in a horizontal underground, as in the case of a deepwell pump. B. Pumping water level
position A. Horizontal split case pump C. Suction head
A. Centrifugal pump B. Submersible pump D. Drawdown
B. End suction pump C. Booster pump
C. In line pump D. Vertical shaft turbine pump Answer: B
D. Horizontal pump
Answer: A 71. The vertical difference between the pumping water
Answer: D level and the static water level.
66. An underground formation that contains sufficient A. Static water level
61. A centrifugal pump whose drive unit is supported by saturated permeable material to yield significant B. Pumping water level
the pump having its suction and discharge flanges on quantities of water. C. Suction head
approximately the same center. A. Aquifer D. Drawdown
A. Horizontal split case pump B. Wet pit
B. End suction pump C. Ground water Answer: D
C. In line pump D. Well water
D. Vertical shaft turbine pump
72. Imparts velocity to the liquid, resulting from Answer: B C. Seal gage
centrifugal force as the impeller is rotated. D. Mechanical seal
A. Impeller 78. Which of the following is used to mount unit rigidly
B. Casing and support bearings. Answer: D
C. Stuffing box A. Wearing rings
D. Shaft sleeve B. Stuffing box 84. A positive displacement pump consisting of a fixed
C. Frame casing containing gears, cams, screws, vanes,
Answer: A D. Coupling plungers or similar elements actuated by rotation of
the drive shaft.
73. Gives direction to the flow from the impeller and Answer: C A. Rotary pumps
converts this velocity energy into pressure energy. B. Reciprocating pumps
A. Impeller 79. Connects the pump to the driver. C. Deep well pumps
B. Casing A. Wearing rings D. Centrifugal pumps
C. Stuffing box B. Bearings
D. Shaft sleeve C. Frame Answer: A
D. Coupling
Answer: B 85. A positive displacement unit wherein the pumping
Answer: D action is accomplished by the forward and backward
74. A means of throttling the leakage which would movement of a piston or plunger inside a cylinder
otherwise occur at the point of entry of the shaft into 80. The most common means of throttling the leakage usually provided with valves.
the casing. between the inside and outside of the casing. A. Rotary pumps
A. Impeller A. Packing B. Reciprocating pumps
B. Casing B. Gland C. Deep well pumps
C. Stuffing box C. Seal gage D. Centrifugal pumps
D. Shaft sleeve D. Mechanical seal
Answer: B
Answer: B Answer: A
86. A type of rotary pump consists of an eccentrically
75. Protects the shaft where it passes through the 81. To position and adjust the packing pressure. bored cam rotated by a shaft concentric in a
stuffing box. A. Packing cylindrically bored casing, with an aburment or
A. Impeller B. Gland follower so arranged that with each rotation of the
B. Casing C. Seal gage drive shaft a positive quantity of liquid is displaced
C. Stuffing box D. Mechanical seal from the space between the cam and the pump
D. Shaft sleeve casing.
Answer: B A. Cam and piston pump
Answer: D B. Gear pump
82. Provides passage to distribute the sealing medium C. Screw pump
76. Keeps internal recirculation down to a minimum. uniformly around the portion of the shaft that passes D. Vane pump
A. Wearing rings through the stuffing box. Also known as water seal of
B. Bearings lantern ring. Answer: A
C. Frame A. Packing
D. Coupling B. Gland 87. A type of rotary pump consists of two or more gears,
C. Seal gage operating in closely fitted casing so arranged that
Answer: A D. Mechanical seal when the gear teeth unmesh on one side liquid fills
the space between the gear teeth and is carried
77. Accurately locate shaft and carry radial and thrust Answer: C around in the tooth space to the opposite side and
loads. displaced as the teeth mesh again.
A. Wearing rings 83. Provides a mechanical sealing arrangement that A. Cam and piston pump
B. Bearings takes the place of the packing. B. Gear pump
C. Frame A. Packing C. Screw pump
D. Coupling B. Gland D. Vane pump
Answer: B C. Power driven pump 1. One of the reasons for insulating the pipes is:
D. Piston pump A. They may not break under pressure
88. A type of rotary pump consists of two or three screw B. There is minimum corrosion
rotors so arranged that as the rotors turn liquid fills Answer: C C. Capacity to withstand pressure is increased
the shape between the screw threads and is D. Heat loss from the surface is minimized
displaced axially as the rotor threads mesh. 93. A type of deep well pump which are refinement of the
A. Cam and piston pump old hand pump that have played such an important Answer: D
B. Gear pump role in country home and small town water supply
C. Screw pump from wells. 2. The rate of radiant energy, that is emitted by a
D. Vane pump A. Plunger pump surface at any temperature and in small
B. Turbine pump wavelengths is found from the known rate of
Answer: C C. Ejector centrifugal pump energy that under the same conditions will be
D. Air lift emitted from a black surface, by multiplying with
89. A type of rotary pump consists of one rotor in a the absorptivity. The above enunciation is called:
casing machined eccentrically to the drive shaft. The Answer: A A. Lambert’s law
rotor is fitted with a series of vanes, blades or B. Kirchhoff’s law
buckets which follow the bore of the casing thereby 94. A type of deep well pump that represent the C. Planck’s law
displacing the liquid with each revolution of the drive application of vertical centrifugal pump to deep well D. Stefan Boltzmann’s law
shaft. service and are built for heads up to 305 meters and
A. Cam and piston pump for capacities up to 28,495 liters per minute. Answer: B
B. Gear pump A. Plunger pump
C. Screw pump B. Turbine pump 3. Which of the following is generally used to
D. Vane pump C. Ejector centrifugal pump measure the temperature inside the furnace?
D. Air lift A. Mercury thermometer
Answer: D B. Alcohol thermometer
Answer: B C. Ash thermometer
90. A type reciprocating pump having a steam cylinder D. Optical pyrometer
with no lap on valves, a water cylinder and a 95. A type of deep well pump that has come into wide
common piston rod. use for small capacities combines a single stage Answer: D
A. Direct acting steam pump centrifugal pump at the top of the well and an ejector
B. Crank and flywheel reciprocating pump or jet located down in the water. 4. All heat transfer processes:
C. Power driven pump A. Plunger pump A. Involve transfer of energy
D. Piston pump B. Turbine pump B. Involve temperature difference between the
C. Ejector centrifugal pump bodies
Answer: A D. Air lift C. Obey first law of thermodynamics
D. Obey second law of thermodynamics
91. A type of reciprocating pimp driven by compound, Answer: C
cross compound, or triple expansion steam engines. Answer: B
A. Direct acting steam pump 96. A type of deep well pump wherein compressed air is
B. Crank and flywheel reciprocating pump admitted to the well to lift water to the surface, for 5. What is thermal diffusivity?
C. Power driven pump successful operation of the system, the discharge A. A mathematical formula
D. Piston pump pipe must have its lower end submerged in the well B. A physical property of the material
water. C. A configuration for heat conduction
Answer: B A. Plunger pump D. A dimensionless parameter
B. Turbine pump
92. A type of reciprocating pump that receives its forward C. Ejector centrifugal pump Answer: B
and backward motion of the piston and plunger from D. Air lift
the rotary motion of a revolving crankshaft by means 6. Which of the following is a unit of thermal
of a crank and connecting rod. Answer: D diffusivity?
A. Direct acting steam pump A. m2 /hr
B. Crank and flywheel reciprocating pump CHAPTER 13 – HEAT TRANSFER B. kcal/m2 hr
C. kcal/m2 hr C
̊ 12. Thermal conductivity of wood depends on which of B. Radiation from suspended larger particles of
D. m2 /hr C
̊ the following? coal, coke, or ash contributing to flame
A. Moisture lumininossity
Answer: A B. Temperature C. Infared radiation from water vapor and
C. Density carbon dioxide
7. Non-isotropic conductivity is shown by which of D. All of the above D. All of the above
the following?
A. Brass Answer: D Answer: D
B. Copper
C. Wood 13. A fur coat on an animal will help the animal to 18. The statement that the emissivity and absorptivity
D. Steel remain: of a surface is surrounded by its own temperature
A. Warm in winter are the same for both monochromatic and total
Answer: C B. Cool in winter radiation is called:
C. Warm in summer A. Lambert’s law
8. For glass wool thermal conductivity changes from D. Cool in summer B. Kirchhoff’s law
sample to sample due to changes in: C. D’Alambart’s
A. Structure Answer: A D. Law of emissivity
B. Density
C. Composition 14. The nature of flow of a fluid inside a tube, whether Answer: B
D. All of the above it is turbulent or laminar, can be ascertained by:
A. Flow velocity 19. A reservoir that supplies energy in the form of heat
Answer: D B. Surface conditions is called:
C. Viscosity of fluid A. Source
9. Which of the following is the S.I. unit of thermal D. Reynolds number B. Sink
conductivity? C. Cold reservoir
A. W/m-hr- K ̊ Answer: D D. Heat reservoir
B. W/m K ̊
C. KJ/m-hr- C ̊ 15. By which of the following modes of heat transfer is Answer: A
D. W/m-hr- C ̊ the Stefan-Boltzmann law applicable?
A. Conduction 20. In regenerator type heat exchanger, heat transfer
Answer: B B. Radiation occurs by:
C. Conduction and radiation combined A. Direct mixing of hot and cold fluids
10. What is the value of the Prandt’l number for air? D. Convection and radiation combined B. A complete separation between hot and cold
A. 10 fluids
B. 6.7 Answer: B C. Flow of hot and cold fluids alternately over a
C. 67 surface
D. 0.7 16. At all wave lengths and temperatures the D. Generation of heat again and again
monochromatic emissivity of a white body is equal
Answer: D to: Answer: C
A. Zero
11. According to Prevost theory of heat exchange, B. 0.5 21. Least value of Prandt’l number can be expected in
A. It is impossible to transfer heat from low C. Unity case of ____________.
temperature source to high temperature D. 0.1 to 0.5 A. Liquid metals
source B. Sugar solution
B. Heat transfer by radiation needs no medium Answer: A C. Salt solution
C. All bodies above absolute zero emit radiation D. Water
D. Heat transfer in most of the cases occurs by 17. The radiation from flames is having
combination of conduction, convection and A. Continuous radiation from burning soot Answer: A
radiation particles of microscopic and submicroscopic
dimensions
Answer: C
22. “The boiling point of a solution is a linear function Answer: B A. Ion
of water at the same pressure.” The above B. Isotope
statement is called ___________. 27. In sugar mills can just is evaporation in: C. Molecule
A. Dubring’s rule A. Zigzag tube evaporators D. Hole
B. Petit and Dulong’s law B. Long vertical tube evaporators
C. Fick’s rule C. Short vertical tube evaporators Answer: A
D. Reynolds law D. Horizontal tube evaporators
33. The energy of a body that can be transmitted in
Answer: A Answer: B the form of heat.
A. Heat energy
23. Floating heads are provided in heat exchangers to 28. A 1-2 heat exchanger refers to which of the B. Thermal energy
A. Increase the pressure drop following? C. Entropy
B. Decrease the pressure drop A. Single pass on shell side and double pass on D. Internal energy
C. Facilitate maintenance tube side
D. Avoid deformation of tubes because of B. Single pass on tube side and double pass on Answer: B
thermal expansion shell side
C. Single liquid cools two liquids at different 34. In an isometric process, the heat transferred is
Answer: D temperature equal to:
D. Two tubes of cold fluid pass through one tube A. Change in enthalpy
24. What do you call the first stage of crystal of hot fluid B. Change in entropy
formation? C. Change in internal energy
A. Nucleation Answer: A D. Work nonflow
B. Foaming
C. Separation 29. A correction of LMTD is essential in case of: Answer: C
D. Vortexing A. Parallel flow heat exchanger
B. Counter current heat exchanger 35. A substance that is able to absorb liquids or gases
Answer: A C. Cross flow heat exchanger and is used for removing them from a given
D. None of the above medium or region.
25. In heat exchanger design, one transfer unit A. Absorbent
implies: Answer: C B. Cohesive
A. One fluid which is exchanging with another C. Adsorbent
fluid of the same chemical composition 30. Which of the following is used as entrainer in D. Adhesive
B. The section of heat exchanger which will acetic acid – water separation?
cause temperature drop of one degree A. Methyl alcohol Answer: A
centigrade B. Phosphorous
C. The section of heat exchanger where heat C. Butyl acetate 36. Radiant heat transfer is described by:
transfer surface area has been one square D. Hexane A. Newton’s law
meter B. Fourier’s law
D. Condition when the change in temperature of Answer: C C. The logarithmic mean temperature
one steam is numerically equal to the D. Kirchhoff’s law
average driving force 31. A type of radiation consisting of singly charged
particles that generate to intermediate distances. Answer: D
Answer: D A. Nuclear radiation
B. Alpha radiation 37. A reservoir that absorbs energy in the form of heat
26. Dritus Boelter equation can be applied in case of C. Beta radiation is called __________.
fluids flowing in: D. Gamma radiation A. Source
A. Transition region B. Sink
B. Turbulent region Answer: C C. Cold reservoir
C. Laminar region D. Heat reservoir
D. Any of the above 32. An electrically charged atom or radical which
carries electricity through an electrolyte is called: Answer: B
38. When the entire heat exchanger is selected as Answer: A A. Reversible process
control volume, heat becomes __________. B. Irreversible process
A. Unity 44. Radiation heat transfer is described by C. Cycle
B. Zero A. Newton’s law D. Isentropic process
C. Undefined B. The logarithmic mean temperature difference
D. Indeterminate C. Fourier’s law Answer: C
D. Kirchoff’s law
Answer: B 50. Past ME Board Question
Answer: D A theoretical body which when heated to
39. Heat is conducted in the direction of: incandescence would emit a continuous light-ray
A. Increasing temperature 45. The equivalent of ratio of emissive power to spectrum.
B. Decreasing temperature absorptivity for bodies in thermal equilibrium is A. Black body radiation
C. Increasing and decreasing temperature described by: B. Black body
D. Constant temperature A. Newton’s law C. Blue body
B. The logarithmic mean temperature difference D. White body
Answer: B C. Fourier’s law
D. Kirchoff’s law Answer: B
40. The heat transfer term in the first law of
thermodynamics may be due to any of the Answer: D 51. Past ME Board Question
following except: Which of the following is the reason for insulating
A. Conduction 46. The temperature potential between temperature at the pipes?
B. Convection the two ends of a heat exchanger are given by: A. They may not break under pressure
C. Radiation A. The logarithmic mean temperature difference B. There is minimum corrosion
D. Internal heat generation (e.g., chemical B. The Stefan-Boltzmann law C. Capacity to withstand pressure
reaction) C. Fourier’s law D. Heat loss from the surface is minimized
D. Kirchoff’s law
Answer: D Answer: D
Answer: A
41. All heat transfer processes require a medium of 52. Past ME Board Question
energy exchange except: 47. The function of a heat exchanger is to: Heat transfer due to density differential
A. Conduction A. Increase the water temperature entering the A. Convection
B. Natural convection boiler and decrease combustion B. Nuclear
C. Forced convection requirements C. Conduction
D. Radiation B. Transfer heat from one fluid to another D. Radiation
C. Increase the total energy content of the flow
Answer: D D. Exchange heat to increase energy to the flow Answer: A

42. Thermal conduction is described by: Answer: B 53. Past ME Board Question
A. Newton’s law The term “exposure” in radiological effects is used
B. The logarithmic mean temperature difference 48. The function of a superheater is to: as a measure of a gamma ray or an X-ray field in
C. The Stefan-Boltzmann law A. Increase the water temperature entering the the surface of an exposed object. Since this
D. Fourier’s law boiler and decrease combustion radiation produces ionization of the air surrounding
requirements the object, the exposure is obtained as
Answer: D B. Transfer heat from one fluid to another A. Number of ions produced per mass of air x
C. Increase the total energy content of the flow coulombs per kg
43. Convection is described by which of the following D. Exchange heat to increase energy to the flow B. Mass of air x surface area of an exposed
laws? object
A. Newton’s law Answer: D C. Mass of air over surface area of an exposed
B. The logarithmic mean temperature difference object
C. The Stefan-Boltzmann law 49. What is the series of processes that eventually D. Number of ions produced per mass of air +
D. Fourier’s law bring the system back to its original condition? coulombs per kg
Answer: A D. Thermal radiation C. Steam
D. Copper
54. Past ME Board Question Answer: A
The passing of heat energy from molecule to Answer: B
molecule through a substance 59. Past ME Board Question
A. Conduction A body that is hot compared to its surroundings 64. What do you call the effectiveness of a body as a
B. Radiation illuminates more energy than it receives, while its thermal radiator at a given temperature?
C. Conservation surrounding absorbs more energy than they give. A. Absorptivity
D. Convection The heat is transferred from one to another by B. Conductivity
energy wave motion. What is this mode of heat C. Emissivity
Answer: A transfer? D. Reflectivity
A. Radiation
55. Past ME Board Question B. Conduction Answer: C
The radiant heat transfer depends on: C. Convection
A. Temperature D. Condensation 65. The natural direction of the heat flow between two
B. Heat rays reservoirs is dependent on which of the following?
C. Heat flow from cold to hot Answer: A A. Their temperature difference
D. Humidity B. Their internal energy
60. What is the heat transfer due to density C. Their pressures
Answer: B differential? D. Their states, whether solid, liquid and gas
A. Convection
56. Past ME Board Question B. Conduction Answer: A
What kind of heat exchanger where water is C. Nuclear
heated to a point that dissolved gases are D. Radiation 66. Why are metals good conductors of heat?
liberated? A. Because they contain free electrons
A. Evaporator Answer: A B. Because their atoms are relatively far apart
B. Condenser C. Because their atoms collide infrequently
C. Intercooler 61. What do you call the passing of heat energy from D. Because they have reflecting surfaces
D. Deaerator molecule to molecule through a substance?
A. Conduction Answer: A
Answer: D B. Conservation
C. Radiation 67. In natural convection a heated portion of a fluids
57. Past ME Board Question D. Convection moves because:
Heat transfer processes which include a change of A. Its molecular motions become aligned
phase of a fluid are considered __________. Answer: A B. Of molecular collisions within it
A. Convection C. Its density is less than that of the surrounding
B. Thermal radiation 62. The transmission of heat from one place to fluid
C. Conduction another by fluid circulation between the spots of D. Of currents in surrounding fluid
D. Radiation different temperature is called _______.
A. Convection Answer: C
Answer: A B. Conservation
C. Radiation 68. In order to emit electromagnetic radiation, an
58. Past ME Board Question D. Conduction object must be at a temperature:
A hot block is cooled by blowing cool air over its A. Above 0 K
top surface. The heat that is first transferred to the Answer: A B. Above 0 C ̊
air layer close to the block is by conduction. It is C. Above that of its surrounding
eventually carried away from the surface by 63. Which of the following requires the greatest D. High enough for it to glow
___________. amount of heat per kilogram for a given increase
A. Convection in temperature? Answer: A
B. Radiation A. Ice
C. Conduction B. Water
69. The rate at which an object radiates Answer: A 80. What do you call a change of phase directly from
electromagnetic energy does not depend on its: vapor to solid without passing through the liquid
A. Surface area 75. At what particular condition that no more heat can state?
B. Mass that be removed from a substance and the A. Sublimation
C. Temperature temperature can no longer be lowered? B. Solidification
D. Ability to absorb radiation A. Freezing point C. Vaporization
B. Absolute zero D. Deposition
Answer: B C. Critical point
D. Ground zero Answer: D
70. Sublimation refers to:
A. The vaporization of solid without first Answer: B 81. Which of the following is the Stefan-Boltzmann
becoming liquid constant?
B. The melting of a solid 76. What refers to the heat transfer wherein the heat A. 5.77 x 10−7 W/m2 K 4
C. The vaporization of a liquid is transferred from one point to another by actual B. 7.67 x 10−9 W/m2 K 4
D. The condensation of a gas into liquid movement of substance? C. 4.78 x 10−10 W/m2 K4
A. Conduction D. 5.67 x 10−8 W/m2 K 4
Answer: A B. Radiation
C. Convection Answer: D
71. In the process of freeze drying, ice goes directly D. Absorption
into water vapor. What is the temperature at which 82. What is the usual value of transmissivity for
this process can take place? Answer: C opaque materials?
A. Below the triple point of water A. 0
B. At the triple point of water 77. The ratio of the radiation of actual body to the B. Indeterminate
C. Above the triple point of water radiation of a blackbody is known as _______. C. 1
D. Any of the above, depending on the pressure A. Emittance D. Infinity
B. Reflectance
Answer: A C. Absorptance Answer: A
D. Transmittance
72. What usually happen when a vapor condenses 83. A body whose emissivity is less than 1 is known
into a liquid? Answer: A as a real body. What is the other term for real
A. It evolves heat body?
B. It generates heat 78. Which of the following is the usual geometric view A. Gray body
C. Its temperature increases factor for a black body? B. White body
̊
D. It boils with temperature less than 100 C A. Zero C. Black body
B. Infinity D. Theoretical body
Answer: A C. One
D. Indeterminate Answer: A
73. In a cooling tower, the water is cooled mainly by:
A. Condensation Answer: C 84. What refers to an ideal body that absorbs all of the
B. Convection radiant energy that intrudes on it and also emits
C. Evaporation 79. What happens to the heat transferred radially the maximum possible energy when acting as a
D. Conduction across insulated pipe per unit area? source?
A. The heat will flow at constant rate A. White body
Answer: C B. Decreases with the increase in thermal B. Black body
conductivity C. Gray body
74. How do you classify a body that has an emissivity C. Decrease from pipe wall to insulated surface D. Red hot body
factor of 0.7? D. Partially increases from pipe wall to insulated
A. Gray body surface Answer: B
B. Black body
C. White body Answer: C 85. The thermal resistance for one-dimensional steady
D. Theoretical body conduction heat transfer through cylindrical wall in
the radial direction is expressed in which of the A. Enthalpy 95. Which of the following is the emissivity of white
following functions? B. Thermal energy body?
A. Linear C. Entropy A. Zero
B. Exponential D. Internal energy B. 0.5
C. Logarithmic C. 1
D. Trigonometric Answer: B D. 0 < e < 1

Answer: C 91. Which of the following is the equivalent heat Answer: A


transferred of a gas undergoing isometric
86. The law which states that “the ratio of the emissive process? 96. The mechanism of heat transfer in which there is
powers to absorptivities are equal when the two A. Change in enthalpy no medium ( i.e. water, air, solid concrete)
bodies are in thermal equilibrium” is known as: B. Change in entropy required for the heat energy to travel is:
A. Stefan-Boltzmann law C. Change in internal energy A. Conduction
B. Newton’s law of convection D. Work nonflow B. Radiation
C. Fourier’s law C. Convection
D. Kirchhoff’s law of radiation Answer: C D. Diffusion

Answer: D 92. What do you call a substance that is able to Answer: B


absorb liquids or gases and is usually used for
87. It refers to the ratio of the internal thermal removing liquids (or gases) from a given medium 97. The temperature potential between temperature at
resistance of a solid to the boundary layer thermal or region? the two ends of a heat exchanger are given by:
resistance (or external resistance of the body). A. Absorbent A. The logarithmic mean temperature difference
A. Biot number B. Liquifier B. The Stefan-Boltzmann law
B. Prandtl number C. Adsorbent C. Fourier’s law
C. Nusselt number D. Adhesive D. Kirchoff’s law
D. Reynolds number
Answer: A Answer: A
Answer: A
93. In which direction that heat is transferred through 98. Which of the following best describe function of
88. It refers to the ratio at the rate of heat transferred conduction? heat exchanger?
by conduction to the rate of energy stored. A. Increasing temperature A. Increase the water temperature entering the
A. Reynolds number B. Decreasing temperature system
B. Fourier number C. Increasing and decreasing temperature B. Transfer heat from one fluid to another
C. Biot number D. Constant temperature C. Evaluate the total energy of the flow
D. Prandtl number D. Exchange heat to increase energy to the flow
Answer: B
Answer: B Answer: B
94. Which of the following statements is based on
89. A hot block is cooled by blowing cool air over its Prevost theory of heat exchange? 99. What refers to a form of energy associated with
top surface. The heat that is first transferred to the A. All bodies above absolute zero emit radiation the kinetic random motion of large numbers of
air layer close to the block is by conduction. It is B. The substance moves because of the molecules?
eventually carried away from the surface by: decrease in its density which is caused by A. Heat
A. Conduction increase in temperature B. Heat of fusion
B. Radiation C. The substance moves because of the C. Entropy
C. Thermal application of mechanical power such as that D. Internal energy
D. Convection of a fan
D. Heat transfer in most of the cases occurs by Answer: A
Answer: D combination of conduction, convection and
radiation 100. How much id the part of light that is absorbed by
90. It is the term used to describe the energy of a the body that transmits and reflects 80% and 10%
body that can be transmitted in the form of heat. Answer: A respectively?
A. 10% transport by diffusion in the velocity and thermal energy of wave motion. What is this mode of heat
B. 30% boundary layer? transfer?
C. 20% A. Nusselt’s number A. Radiation
D. 5% B. Prandtl number B. Conduction
C. Reynold’s number C. Convection
Answer: A D. Dimensional measurement D. Condensation

101. In convection heat transfer, what happens to the Answer: B Answer: A


heat transfer coefficient if the viscosity of the fluid
increases? 106. Which of the following is the property of the solid 110. What do you call theoretical body where
A. The heat transfer coefficient will increase that provides the measure of the rate of heat absorptivity and emissivity are independent of the
B. The heat transfer coefficient will decrease transfer to the energy storage? wavelength over the spectral region of the
C. The heat transfer coefficient remains A. Thermal efficiency irradiation and the surface emission?
constant B. Thermal diffusivity A. White body
D. None of the above C. Thermal conductivity B. Opaque body
D. Thermal radiography C. Black body
Answer: B D. Transparent body
Answer: B
102. How do you call a phenomenon wherein the heat Answer: D
is transferred by motion of fluid under the action of 107. Two metals were kept together at room
mechanical device? temperature and it was found out that one is 111. Which of the following is the structure designed to
A. Forced convection colder than the other. Which of the following is the prevent the spread of fire having a fire resistance
B. Natural convection best reason why one metal is colder than the rating of not less than four hours?
C. Forced conduction other? A. Fire escape
D. Thermal radiation A. The heat transfer coefficient of one metal is B. Fire exit
of higher C. Fire shield
Answer: A B. One metal is of lower temperature as D. Fire wall
compared to the other
103. In conduction heat transfer, what happens to the C. One metal is of higher temperature as Answer: D
heat transfer per unit time when the thermal compared to the other
conductivity decreases? D. The thermal conductivity of one metal is high 112. Which of the following heat exchange device used
A. The heat flow will increase as compared to the other to provide heat transfer between the exhaust
B. The heat flow remains constant gases and air prior to the entrance of a
C. The heat flow will decrease Answer: D combustor?
D. The heat flow will partially increase and then A. Regenerator
will decrease 108. In convection heat transfer, what happens to the B. Economizer
heat transfer coefficient if the viscosity of the fluid C. Condenser
Answer: C decreases? D. Reheater
A. The heat transfer coefficient also increases
104. Which of the following is the driving force in heat B. The heat transfer coefficient will decrease Answer: A
transfer? C. The heat transfer coefficient remains
A. Temperature gradient constant 113. Which of the following transfer of heat is involved
B. Thickness gradient D. The heat transfer coefficient partially in the changing of boiling water (at 100 C̊ ) to
C. Viscosity gradient increases then decreases vapor at the same temperature?
D. Dielectric gradient A. Conduction
Answer: A B. Convection
Answer: A C. Radiation
109. A body that is hot compared to its surroundings D. Evaporation
105. Which of the following is the measure of the illuminates more energy than it receives, while its
relative effectiveness of momentum and energy surrounding absorbs more energy than they give. Answer: B
The heat is transferred from one to another by
114. Which of the following is the science of low B. The steam has higher specific heat Answer: A
temperatures? C. Steam contains more internal energy
A. Cryogenics D. Steam is everywhere thus it strikes greater 125. The rate at which heat flows through a slab of
B. Thermo-kinetics force some material does not depend on which of the
C. Thermodynamics following?
D. Ergonomics Answer: C A. The thickness of the slab
B. The area of the slab
Answer: A 120. What usually happens when vapor condenses into C. The temperature difference between two
liquid? faces
115. Which of the following thermal state of the body A. It absorbs heat D. The specific heat of the material
considered as reference to communicate heat to B. It rejects heat
the other bodies? C. Its temperature increases Answer: D
A. Temperature D. Its temperature decreases
B. Pressure 126. Which of the following is the primary function of a
C. Internal energy Answer: B thermal radiator?
D. Entropy A. To transferring the heat by using moving
121. Which of the following has the highest thermal fluids
Answer: A conductivity? B. To transfer heat from hot to cold body by
A. Mercury using a forced-draft fan
116. The true mean temperature difference is also B. Gasoline C. To transfer heat by allowing molecules to
known as: C. Water vibrate one to another
A. The average mean temperature difference D. Alcohol D. To transfer heat with or without a medium
B. The logarithmic mean temperature difference
C. The trigonometric mean temperature Answer: A Answer: D
difference
D. The exponential temperature difference 122. Which of the following is the requirement of the 127. What is the reason why Styrofoam is a good
temperature of a body for it to emit radiation? insulating material?
Answer: B A. Above zero Celsius A. Because it contains many tiny pockets of air
B. Above zero Kelvin B. Styrofoam is a white object
117. Which of the following can be a geometric view C. Above of the temperature of the surroundings C. The structure of Styrofoam is very unstable
factor of a gray body? D. High enough for it to glow and heat cannot flow
A. Greater than one D. Styrofoam structure is very dense
B. Less than one Answer: B
C. Equal to one Answer: A
D. Greater than zero but less than one 123. Which of the following is the color of iron when it is
heated to a highest temperature? 128. What usually happens to the surrounding when
Answer: D A. White water vapor condenses?
B. Red A. It warms the surrounding
118. The heat transfer by conduction occurs in which of C. Orange B. The surrounding temperature decreases
the following? D. Yellow C. It neither warm nor cold the surrounding
A. Only in liquids D. The surroundings will be dehumidified
B. Only in solids Answer: A
C. Only in liquids and gases Answer: A
D. In solids, liquids and gases 124. Which of the following is the reason why metals
are good conductors of heat? 129. The rate of radiation does not depend on which of
Answer: D A. Metals contain free electrons the following?
B. Metals have atoms the frequently collide A. Temperature of the radiating body
119. Which of the following reasons why one gram of another B. The emissivity of the radiation surface
steam at 100 C̊ causes more serious burn than C. Metals have reflecting surfaces C. The area of the radiating body
one gram of water at 100 C̊ ? D. Atoms in metals are very far to each other D. The thickness of the radiating body
A. Steam is less dense than boiling water
Answer: D B. Decreases except for ferrous metals Answer: A
C. Almost constant except for ferromagnetic
130. Which of the following is not a good conductor of materials 141. The heat transfer by convection occurs in which of
heat? D. Increases except for steel the following?
A. Metals A. Only in gases
B. Rocks Answer: C B. Only in liquids
C. Glass C. Only in gases and liquids
D. Asbestos 136. Which of the following liquids that has the highest D. Only in gases and solids
thermal conductivity?
Answer: D A. Gasoline Answer: C
B. Glycerin
131. Which of the following is not a unit of the rate of C. Water 142. In convection heat transfer, what mechanism heat
heat transfer? D. Alcohol transfer where the fluid moves due to the
A. Watt decrease in its density caused by increase in
B. Btu per hour Answer: C temperature?
C. Cal/s A. Forced convection
D. Btu/Hp-hr 137. Which of the following is not a heat exchanger? B. Natural convection
A. Condenser C. Density convection
Answer: D B. Boilers D. Radial convection
C. Evaporators
132. The thermal conductivity does not depend on D. Water hammer Answer: B
which of the following?
A. Chemical composition Answer: D CHAPTER 14 – REFRIGERATION
B. Physical state or texture
C. Temperature and pressure 138. Which of the following heat exchangers where 1. Which of the following could be used to check a
D. Gravitational pull fluid flow in the same direction and both are of leak in an ammonia system?
changing temperatures? A. Litmus paper
Answer: D A. Parallel flow B. Halide torch
B. Cross flow C. Sulfur stick
133. In Maxwell’s theory for thermal conductivity of C. Counter flow D. A and C
gases and vapors, which of the following is the D. Mixed flow
value of “a” for triatomic gases? Answer: D
A. 1.7 Answer: A
B. 2.4 2. Ammonia leaks in the condenser can be detected
C. 1.3 139. What happens to the thermal conductivity of by:
D. 2.4 diatomic gases if the temperature is increase? A. Smelling the discharge water
A. The thermal conductivity will also increase B. Applying litmus paper to the circulating water
Answer: A B. The thermal conductivity decreases discharge
C. The thermal conductivity remains constant C. Adding oil of peppermint to the system and
134. Which of the following conductivities where D. The thermal conductivity partially increases tracing the smell
Sutherland equation is used? then decreases D. Applying a soapy solution to the condenser
A. Thermal conductivities of solids heads and looking for bubbles
B. Thermal conductivities of gases Answer: A
C. Thermal conductivities of metal Answer: B
D. Thermal conductivities of liquids 140. What device is used to measure the amount of
infrared radiation in each portion of a person’s skin 3. A sulfur stick burning in the presence of ammonia
Answer: B that is emitted? will give off a _______.
A. Thermograph A. Dense yellow smoke
135. For pure metals, what happens to the thermal B. Thermometer B. Dense red smoke
conductivity if the temperature is extremely high? C. Pyrometer C. Dense white smoke
A. Approaches infinity D. Potentiometer D. Dense green smoke
Answer: C B. Pump air out of the system A. On the receiver discharge
C. Add refrigerant to the system B. In the highest part of the system
4. An ammonia leak will turn litmus paper _______. D. Reduce the pressure on the discharge side of C. In the lowest part of the system
A. Blue the condenser D. On the evaporators coils
B. Green
C. Red Answer: A Answer: B
D. Yellow
10. How is an ammonia system purged so that 16. The solenoid valve is:
Answer: A operator will not be overcome by the fumes? A. A pressure-controlled stop valve
A. Into a bucket of lube oil B. A temperature-controlled stop valve
5. In an ammonia system the oil gauge must be kept: B. Back into the compressor C. A Freon-controlled check valve
A. Closed except when checking oil level C. Into a bucket of water D. None of the above
B. Open at all times D. Into the atmospheric line
C. Close when machine is shut down Answer: B
D. Open when machine is shut down Answer: C
17. The thermal expansion valve is located between
Answer: A 11. Thermal expansion valves are usually of the: the:
A. Diaphragm A. Solenoid valve and the evaporator coils
6. Valves and piping in an ammonia system are B. Magnetic tape B. Charging valve and the solenoid valve
made of: C. Bellows type C. Receiver and the king valve
A. Brass D. A or C D. King valve and the solenoid valve
B. Bronze
C. Iron Answer: D Answer: A
D. Copper
12. When checking zinc plates in a condenser, one 18. The purpose of the scale trap is to:
Answer: C should: A. Dissolved scale and dirt in the system
A. Install all new plates B. Remove insoluble gases from the refrigerant
7. The relief valve on an ammonia machine is B. Clean the plates and renew worn out ones C. Remove dirt, scale and metal chips from the
located: C. Ground each plate to the shell refrigerant
A. On the discharge pipe from the condenser D. Paint the plates with red lead D. Control the amount of scale going to the
B. On the discharge pipe from the compressor compressor
C. In the compressor head Answer: B
D. A and B Answer: C
13. The scale trap is located between the:
Answer: C A. Compressor and oil separator 19. The oil separator (trap) is located between the:
B. Expansion valve and evaporators coils A. Compressor discharge valve and condenser
8. When purging an ammonia condenser into a C. King (liquid) valve and expansion valve B. Receiver and the expansion valve
bucket of water, one can tell when the air is out D. Evaporator coils and compressor C. Receiver and the king valve
and ammonia starts to come through by the: D. Condenser and the receiver
A. Smell of the ammonia being liberated from Answer: C
the water Answer: A
B. Color of the water turning green 14. When the evaporator coils are located in the
C. Color of the water turning bluish icebox, the system is known as: 20. The charging connection in a refrigerating system
D. Change of bubbling sound of air to the A. Indirect system is located:
crackling sound of ammonia B. Direct system A. Before the receiver
C. High-pressure system B. Between the condenser and the receiver
Answer: D D. Low-pressure system C. Between the receiver and the king valve
D. Between the king valve and the solenoid
9. The crossover connection in an ammonia system Answer: B valve
can be used to ___________.
A. Hot-gas defrost 15. The purge valve is located: Answer: D
21. The dehydrator is located between the 27. Which of the following is another name for the D. Valve type
A. Condenser and receiver liquid valve?
B. Receiver and the expansion valve A. Freon valve Answer: B
C. Condenser and the king valve B. King valve
D. Receiver and the king valve C. Shutoff valve 33. The expansion valve is located between the
D. Master valve A. Compressor and condenser
Answer: B B. Evaporator and compressor
Answer: B C. Receiver and evaporator
22. The charging valve is located between the: D. Condenser and receiver
A. King valve and the expansion valve 28. If no gaskets are used in the piping joints of a
B. Evaporator coils and the compressor Freon system, the joints must be: Answer: C
C. Compressor and the receiver A. Welded joints
D. Receiver and the condenser B. Finished joints 34. The oil separator is located between the:
C. Ground joints A. Condenser and dehydrator
Answer: A D. Soldered joints B. Compressor and condenser
C. Evaporator and compressor
23. The solenoid valve is located between the Answer: C D. Solenoid valve and thermal expansion valve
_______.
A. Thermal expansion valve and the evaporator 29. A device for holding open the suction valve and Answer: B
B. Scale trap and the thermal expansion valve drawing gas from the suction manifold and
C. King valve and the scale trap returning it to the suction line without compressing 35. Zinc rods are found in the:
D. Automatic and manual expansion valves it is called ___________. A. Salt-water side of condenser
A. Relief valve B. Compressor crankcase
Answer: B B. Suction line by-pass C. Evaporator coils
C. Cylinder unloader D. Refrigerant side of condenser
24. A device used to keep moisture from passing D. Discharge line by-pass
through the system is called: Answer: A
A. Humidifier Answer: C
B. Dehydrator 36. A double-trunk piston is used to
C. Aerator 30. The solenoid valve can be typed as a _______. A. Prevent oil from mixing with the refrigerant
D. Trap A. Bellows valve B. Absorb some of the side thrust
B. Bimetallic valve C. Prevent gas from getting to crankcase
Answer: B C. Thermal valve D. All of the above
D. Magnetic stop valve
25. The relief valve is located on the: Answer: D
A. Discharge side on the condenser Answer: D
B. Outlet of the evaporator coils 37. Which of the following gasket materials should be
C. Receiver tank 31. The bulb for the thermal expansion valve is used on a Freon system?
D. Discharge side of the compressor located A. Asbestos
A. Near the evaporator coil outlet B. Metallic
Answer: D B. In the middle of evaporator coils C. Rubber
C. Near the evaporator coil inlet D. A and B
26. The dehydrator is used: D. On the bottom row of evaporator coils
A. To remove moisture from the system Answer: D
B. When adding refrigerant to the system Answer: A
C. To remove air from the system 38. Many pressure gauges on a Freon system have
D. A and B 32. The elements of a thermostat switch are usually of two dials or graduations on one gauge. The two
the ___________ dials represent
Answer: D A. Pilot-valve type A. Suction and discharge pressure
B. Bimetal type B. Pressure and temperature
C. Diaphragm type C. Liquid and gas pressure
D. Cooling water inlet and outlet temperatures 43. The discharge pressure of the compressor should 48. Before securing a compressor to do maintenance
be: work on it, be sure to:
Answer: B A. The pressure which corresponds to a A. Purge the system
temperature from 5 F̊ to 15 F ̊ below that of B. Have spare parts ready
39. A double-pipe condenser has ___________. the condenser discharge C. Pump down the system
A. Two pipes for cooling water and one for B. The pressure which corresponds to a D. B and C
refrigerant temperature equal to that of the condenser
B. A small pipe inside a larger pipe, the cooling discharge Answer: D
water passing through the small pipe and the C. The pressure which corresponds to a
refrigerant through the large pipe temperature from 5 F ̊ to 15 F ̊ higher than 49. Obstruction of the expansion valve is usually
C. Two piping systems side by side, one with the condenser discharge caused by
cooling water and one with refrigerant D. None of the above A. Scale
D. None of the above B. Congealed oil in the system
Answer: C C. Water in the system
Answer: B D. Any of the above
44. The purpose of relief valves on refrigeration
40. When there is a Freon leak, the halide torch will machines is to: Answer: D
burn A. By-pass the compressor when dehydrating
A. Orange B. Prevent overloading in the iceboxes 50. Excess frost on the evaporator coils
B. Green C. Prevent excessive pressure in case of A. Does not affect the system
C. Blue stoppage on the discharge side of the system B. Takes load off compressor
D. White D. A and B C. Reduces efficiency of the plant
D. Keeps the icebox cooler
Answer: B Answer: C
Answer: C
41. Large leaks in a Freon system cannot always be 45. A leaky suction valve can usually be detected by
detected with a halide torch because it changes A. A higher suction pressure 51. Air circulation in the icebox is accomplished by the
color with the slightest amount of Freon present. A B. A fluctuating suction pressure gauge use of which of the following?
large leak can be detected easier by applying C. Closing in on the suction valve having no A. Hollow sidewalls
____________ effect on the suction pressure B. Diffuser fans
A. Sheets of litmus paper to all joints and watch D. Any of the above C. Louver doors
for color change D. Air vents to deck
B. A soapsuds solution, mixed with a little Answer: D
glycerin to hold the solution together, and Answer: B
watch for bubbles 46. Too low suction pressure could be caused by
C. A thin layer of mineral oil to all joints and A. Too much oil in the system 52. The cooling-water side of the condenser should be
watch for bubbles B. Shortage of refrigerant gas opened for inspection every
D. A lighted candle at the joints and watch for C. Dirty scale traps A. Six months
leaky spots blowing candle flame D. Any of the above B. Two years
C. Year
Answer: B Answer: D D. Three months

42. The relief valve on a CO2 machine is located 47. What do you call a system in which the evaporator Answer: D
A. Next to the king valve coils are located in a brine solution and the brine is
B. In the compressor head pumped through the icebox? 53. Some causes of a noisy compressor are
C. On the discharge pipe between the A. An indirect system A. Worn bearings, pins, etc.
compressor and the discharge valve B. A double-evaporator system B. Slugging due to flooding back of refrigerant
D. On the discharge pipe from the condenser C. A direct system C. Too much oil in crankcase
D. A low-pressure system D. Any of the above
Answer: C
Answer: A Answer: D
54. If the thermal expansion valve becomes Answer: C 65. What do you call the liquid reaching the
inoperative, the iceboxes will have to be controlled compressor through the suction?
by the __________. 60. What do you call the device that is used as a low- A. Superheating
A. King valve pressure control and high-pressure cutout on a B. Overflowing
B. Manual expansion valve compressor? C. Flooding back
C. Manual solenoid valve A. Pressure controller D. Recycling
D. Solenoid valve B. Controller switch
C. Cutout Answer: C
Answer: B D. Cutout switch
66. The suction pressure switch is operated by which
55. Sweating of the crankcase is caused by which of Answer: A of the following?
the following? A. Electric current
A. Too much oil in the system 61. If the solenoid valve closed by accident, the B. Pressure on a bellow
B. Insufficient superheat compressor would be stopped by which of the C. A relay cutout
C. Too much superheat following? D. Thermocouple
D. Expansion valve hung up A. Automatic trip
B. Low-pressure cutout switch Answer: B
Answer: A C. Low-water cutout switch
D. High-pressure cutout switch 67. If the compressor short-cycles on the high-
56. Which of the following is the usual cause of pressure cutout, which of the following would you
slugging? Answer: B check?
A. Too much refrigerant in the system A. Check for too much refrigerant in the system
B. Too much oil in the system 62. The purpose of the low-pressure cutout switch is B. If plenty of cooling water is running through
C. Expansion valve not operating properly to: but it is not picking up heat, the condenser
D. Too much cooling water to condenser A. Cut out the compressor at a set pressure tubes need cleaning
B. Maintain a preset suction pressure to the C. Be sure system is getting cooling water
Answer: C compressor D. All of the above
C. Maintain liquid refrigerant at the suction of the
57. What operates low-pressure cutout switch compressor Answer: D
A. Bellows D. Cut compressor in and out at a preset
B. Spring tension pressure 68. A Freon unit will tend to short-cycle when
C. A magnet operating under:
D. Water pressure Answer: D A. Normal conditions
B. Heavy loads
Answer: A 63. If the cooling water to the condenser suddenly C. Light loads
fails? D. All of the above
58. Which of the following must be checked up if an A. An alarm will ring to notify the engineer
automatic Freon system will not start up? B. The compressor will shut down Answer: C
A. High-pressure cutout C. The expansion valve will close
B. Reset mechanism D. The solenoid valve will close 69. Air is remove from the system by
C. Low-pressure cutout A. Opening the purge valve
D. All of the above Answer: B B. Increasing the amount of cooling water
C. Running the refrigerant through an aerator
Answer: D 64. The most likely cause of high superheat would be D. Running the refrigerant through a deaerator
A. Too much refrigerant
59. Which of the following is the probable cause of hot B. Expansion valve open too wide Answer: A
suction line? C. Expansion valve closed too much
A. Insufficient lubrication D. Back-pressure valve set too high 70. Short-cycling means that the machine
B. Too much refrigerant A. Runs to slow
C. Insufficient refrigerant Answer: C B. Stop and starts frequently
D. Expansion valve closed too much C. Runs too fast
D. Grounds out frequently D. Any of the above A. Insufficient cooling water
B. Too much cooling water
Answer: B Answer: D C. Insufficient refrigerant gas
D. B and C
71. The suction pressure in a Freon system should be 76. If an electrically operated compressor failed to
A. The pressure which corresponds with a start, the cause might be: Answer: D
temperature about 20 F ̊ above the A. A blown fuse
temperature of the icebox B. Burned out holding coils in solenoid valve 82. Too high suction pressure could be cause by
B. The pressure which corresponds with a C. An open switch A. Leaky suction valves
temperature about 20 F ̊ below the D. Any of the above B. Expansion valve bulb not working properly
temperature of the icebox C. Expansion valve open too wide
C. The pressure which corresponds with a Answer: D D. Any of the above
temperature equal to the temperature of the
icebox 77. The high-pressure side of the system is Answer: D
D. None of the above sometimes referred to as the
A. Hot side 83. Which of the following would cause high head
Answer: B B. Suction side pressure?
C. Cold side A. Air in the condenser
72. If any of the electrically controlled devices in a D. Cooling side B. Insufficient cooling water
Freon system malfunction, which of the following C. Dirty condenser
valve will also automatically shut off? Answer: A D. Any of the above
A. King valve
B. Condenser cooling-water inlet valve 78. If the compressor were to run continuously without Answer: D
C. Expansion valve lowering the temperature, the trouble would
D. Solenoid valve probably be: 84. An excessively high head pressure could be
A. Leaky discharge valves caused by
Answer: D B. Insufficient refrigerant in the system A. Insufficient cooling water to the condenser
C. Leaks in the system B. Insufficient cooling water to the evaporator
73. A leaky discharge valve can usually be detected D. Any of the above coils
by C. Solenoid valve shutoff
A. A drop in icebox temperature Answer: D D. Too much cooling water to the condenser
B. A discharge pressure lower than normal
C. A fluctuating high-pressure gauge 79. Which of the following would cause a high head Answer: A
D. Any of the above pressure?
A. Suction valve not open enough 85. Which of the following would cause a high suction
Answer: D B. Too much cooling water pressure?
C. Insufficient cooling water A. Expansion valve open too wide
74. The dehydrating agent in a Freon system is D. Icebox door left open B. Dirty dehydrator
usually C. King valve not open wide enough
A. Sodium chloride Answer: C D. Expansion valve not open wide enough
B. Calcium chloride
C. Activated alumina 80. If frost forms on the cylinders, the cause would be Answer: A
D. Slaked lime A. Charging valve left open
B. Expansion valve not open wide enough 86. If a compressor runs continuously, the cause
Answer: C C. Expansion valve open too wide might be a
D. Dehydrator not working properly A. Clogged scale trap
75. If the compressor short-cycles on the low-pressure B. Defective thermal bulb
cutout, the trouble might be: Answer: C C. Stuck high-pressure switch
A. Too much frost on the evaporator coils D. Stuck low-pressure switch
B. Dirty traps and strainers 81. Which of the following would cause low head
C. Lack of refrigerant pressure? Answer: D
87. Low suction pressure is caused by A. While starting up B. After a long period of operation
A. Expansion valve causing flooding back B. After the system has been shut down for a C. After an extend lay-up period
B. Solenoid valve not functioning properly few hours D. While the compressor is in operation
C. Leaky compressor suction valves C. While system is operating
D. Air in the system D. Once a week Answer: B

Answer: B Answer: B 98. The agent used in an indirect reefer system is


A. Calcium chloride
88. How is a Freon system purged? 93. If the compressor had been running satisfactorily B. Potassium chloride
A. With a reefer pump for a long period of time but suddenly the C. Sodium chloride
B. The same as an ammonia system compartment temperature started to rise, the D. A or C
C. Back to the extra supply tank trouble might be
D. The same as a CO2 system A. A refrigerant leak has developed Answer: D
B. The expansion valve may contain frozen
Answer: D water 99. Air can be prevented from getting into the system
C. The solenoid valve has jammed shut by
89. If the compressor discharge temperature is higher D. Any of the above A. Running the refrigerant through an aerator
than the receiver temperature: B. Keeping the dehydrator clean at all times
A. Add more refrigerant to the system Answer: D C. Keeping all glands and stuffing boxes on the
B. Decrease the amount of cooling water to the high-pressure side tight
condenser 94. If the compressor had been running satisfactorily D. Keeping all glands and stuffing boxes on the
C. Increase the amount of cooling water to the for a long period of time but the oil level was rising low-pressure side tight
condenser slowly, one should:
D. Remove some of the refrigerant from the A. Check the dehydrator cartridge Answer: D
system B. Check to see if there is too much refrigerant
in the system 100. Which of the following would not cause high
Answer: B C. Shut down the compressor and check the oil suction pressure?
level with the machine stopped A. Suction valve not adjusted properly
90. How is a CO2 system purged? D. Drain out sufficient oil to bring it down to the B. Expansion valve stuck open
A. When CO2 comes out of the purge valve, frost proper running level C. Leaky suction valves
will form on a piece of metal held near the D. Insufficient refrigeration
outlet Answer: C
B. Through the king valve Answer: D
C. Pumped out with a suction pump 95. The purpose of the oil trap is :
D. The CO2 will come out of the purge valve in A. To add oil to the compressor 101. Water in the refrigerant is liable to
liquid form B. To remove oil from the refrigerating gas A. Freeze on the expansion valve seat and cut
C. To remove oil from the charging tank the flow of refrigerant
Answer: A D. None of the above B. Clog the oil trap
C. Freeze in the king (liquid) valve
91. If the head pressure is too high: Answer: B D. Emulsify the oil in the compressor
A. The relief valve should open and let excess
refrigerant pass to receiver 96. Too much oil in the compressor would : Answer: A
B. Close in on the suction valve A. Absorb to much refrigerant from the system
C. The relief should open before the high- B. Deposit oil on the condenser tubes 102. The function of the expansion valve is to:
pressure cutout C. Damage the expansion valve A. Regulate the amount of liquid refrigerant to
D. The high-pressure cutout switch should D. Cause leakage through the shaft seals the expansion coils
operate before the relief valve opens B. Change the gas refrigerant to a liquid
Answer: A C. Shut off the flow of refrigerant to the
Answer: D condenser
97. The oil level in the compressor should be check D. Change the high-pressure liquid to a low-
92. The system should be purged A. Just before starting the compressor pressure liquid
Answer: A C. Amount of refrigerant going to the 114. The expansion valve on a Freon system controls
compressor the
103. When heavy electrical currents are involved, the D. Pressure of the refrigerant going to the A. Superheat of the gas leaving the compressor
thermostat will be operated by a: evaporator coils B. Back pressure in the evaporator
A. Pressure pipe C. Temperature of the icebox
B. Fusetron Answer: B D. Superheat of the gas leaving the evaporator
C. Relay
D. Small circuit breaker 109. The purpose of the evaporator is to Answer: D
A. Absorb latent heat of vaporization
Answer: C B. Absorb latent heat fusion 115. The purpose of the expansion valve by-pass is to:
C. Transfer latent heat of vaporization A. Increase the efficiency of the plant
104. Before securing a compressor to do maintenance D. Transfer latent heat of fusion B. Increase the capacity of the evaporator
work on it, be sure to: C. Control the refrigerant to the evaporator in
A. Have gas mask handy Answer: A case the automatic valves fail
B. Make arrangements to have perishables D. Bypass the compressor
taken care of 110. The purpose of the dehydrator is to
C. Notify the engineer A. Add more refrigerant to the system Answer: C
D. A and B B. Remove oil from the refrigerant
C. Remove moisture from the crankcase oil 116. The thermal expansion valve
Answer: D D. Remove moisture from the refrigerant A. Controls the amount of gas coming from the
dehydrator
105. When securing a Freon system for repairs Answer: D B. Controls the quantity of liquid refrigerant
A. Pump down to 1 or 2 pounds pressure going to the evaporator coils
B. Pump down to a slight vacuum 111. The principle of mechanical refrigeration is C. Controls the amount of gas going to the
C. Pump down to 10 to 15 pounds pressure A. The conversion of a liquid to gas receiver
D. Remove all refrigerant from the system B. The absorption of temperature under heat, D. Removes trapped oil from the refrigerant
pressure, compression and expansion
Answer: A C. The compression of a liquid under Answer: B
temperature and expansion
106. When charging Freon system, all the valves D. The absorption of heat under temperature, 117. The function of the compressor is to
should be in their normal position except the: compression, pressure and expansion A. Pull the refrigerant gas through the system
A. Solenoid valve B. Increase the pressure of the refrigerant
B. Purge valve Answer: D C. Discharge the refrigerant to the condenser
C. King (liquid) valve D. All of the above
D. Expansion valve 112. A thermostat is a
A. Temperature-operated switch Answer: D
Answer: C B. Pressure-operated switch
C. Superheat-operated switch 118. The solenoid valve is controlled by
107. The purpose of the receiver is to: D. Back-pressure-operated switch A. The amount of liquid in the system
A. Cool the refrigerant gas B. The amount of gas in the system
B. Separate the oil from the refrigerant Answer: A C. The temperature in the condenser
C. Store the refrigerant D. The temperature in the icebox
D. Condense the refrigerant 113. The thermal expansion valve responds to the
A. Amount of superheat in the vapor Answer: D
Answer: C B. Amount of superheat in the liquid
C. Temperature in the evaporator coils 119. Oil is added to a Freon compressor by
108. The solenoid valve controls the D. Pressure in the evaporator coils A. Shutting down the machine and pouring in
A. Amount of refrigerant entering the evaporator through the crankcase inspection plate
coils Answer: A opening
B. Amount of refrigerant going to the expansion B. Pumping in with an electric-driven pump
valve C. Pumping in with a hand pump
D. Pouring through oil hole in base D. 280,000 Btu per 24 hrs. A. Pressure
B. Weight
Answer: C Answer: A C. Volume
D. Psi
120. When adding oil to a Freon system, one must be 126. The boiling point of Freon-12 at atmospheric
sure that pressure is: Answer: B
A. The condenser is shut down A. + 22 F ̊
B. All air is removed from the pump and fitting B. - 22 F ̊ 132. The latent heat of fusion of ice is:
C. There is not too high a suction pressure C. + 22 C ̊ A. 500 Btu
D. The discharge pressure is not too high D. - 22 C ̊ B. 188 Btu
C. 144 Btu
Answer: B Answer: B D. 970 Btu

121. To help a person who had been overexposed to 127. The boiling point of 𝐶𝑂2 at atmospheric pressure Answer: C
ammonia gas, one would: is
A. Apply cold compresses A. – 110 F ̊ 133. Latent heat
B. Apply artificial respiration B. + 110 F ̊ A. Can be measured with a pyrometer
C. Douse with cold water C. + 110 C ̊ B. Cannot be measured with a thermometer
D. Wrap in warm blankets D. – 110 C ̊ C. Changes as the refrigerant cools
D. Can be measured with a thermometer
Answer: B Answer: A
Answer: B
122. Ammonia will corrode 128. The boiling point of 𝑁𝐻3 at atmospheric pressure
A. Brass is 134. Absolute zero is:
B. Copper A. + 28 C ̊ A. 144 ̊ below zero on the Fahrenheit scale
C. Bronze B. + 28 F ̊ B. The same as zero on the Fahrenheit scale
D. All of the above C. - 28 C ̊ C. 970 ̊ below zero on the Fahrenheit scale
D. - 28 F ̊ D. 460 ̊ below zero on the Fahrenheit scale
Answer: D
Answer: D Answer: D
123. A refrigerating unit of one (1) ton capacity can
remove: 129. Which of the following would you apply if a person 135. A ton of refrigeration is equal to the cooling effect
A. 100 Btu’s per min. got Freon in his eyes? of
B. 288 Btu’s per min. A. Clean water A. 2,000 lbs. of ice melting in 24 hrs.
C. 200 Btu’s per min. B. Soapy water B. 2,000 lbs. of water being converted to ice
D. 500 Btu’s per min. C. Sodium bicarbonate C. 2,000 lbs. of ice melting in 12 hrs.
D. Sterile mineral oil D. 2,240 lbs. of ice melting in 24 hrs.
Answer: C
Answer: D Answer: A
124. The refrigerant with the lowest boiling point is
A. 𝑁𝐻3 130. A double-seated valve allows the valve to be 136. Which of the following best described a Freon?
B. 𝐹12 A. Packed only in the closed position A. Odorless
C. 𝐶𝑂2 B. Packed in the wide open or closed position B. Non-poisonous
D. 𝐹22 C. Operated as a suction or discharge valve C. Colorless
D. Removed for replacement without shutting D. All of the above
Answer: C down
Answer: D
125. A ton of refrigeration is equal to the removal of Answer: B
A. 288,000 Btu per 24 hrs. 137. What is another name of discharge pressure?
B. 28,800 Btu per 24 hrs. 131. The amount of 𝐶𝑂2 or Freon in a cylinder is A. Absolute pressure
C. 28,000 Btu per 24 hrs. measured by B. Head pressure
C. Suction pressure C. 4 tons 148. Which of the following is the most common
D. Condenser pressure D. 3 tons method of heat flow in refrigeration?
A. Conduction
Answer: B Answer: C B. Expulsion
C. Radiation
138. Which of the following is the oil used in a 143. What is the combination of a wet and dry bulb D. Convection
refrigeration system? thermometer is called a
A. Vegetable oil A. Hygrometer Answer: D
B. Straight mineral oil B. Psychrometer
C. Lube oil SAE 20 C. Hydrometer 149. From which of the following processes where the
D. Lube oil SAE 10 D. A or B largest quantities of heat are available?
A. Vaporization
Answer: B Answer: D B. Melting
C. Fusion
139. One disadvantage of a CO2 system is the fact that 144. A pressure controller is usually operated by the D. Cooling
A. It is difficult to condense the refrigerant if the movement of a
circulating water temperature is too high. A. Bellows Answer: A
B. It takes more refrigerant to keep the iceboxes B. Siphon
cold C. Diaphragm 150. What is the most common Freon gas used in
C. Due to high pressure it is difficult to keep oil D. A or C centrifugal compressors?
from mixing with the refrigerant A. F – 12
D. It is difficult to condense the refrigerant if the Answer: D B. F – 11
circulating water temperature is too low C. F – 22
145. The critical temperature of a refrigerant is: D. F – 21
Answer: A A. The temperature at which it will freeze
B. The temperature above which it cannot be Answer: B
140. The disadvantage of CO2 system over an ammonia liquefied
system is the fact that __________ C. The temperature below which it cannot be 151. What is the most common Freon gas used in
A. The pipes and fittings of a CO2 system must liquefied reciprocating compressors?
be of high pressure type D. None of the above A. F – 12
B. The CO2 system operates at a much higher B. F – 11
pressure Answer: B C. F – 22
C. The CO2 system requires a larger prime D. F – 21
mover 146. If the critical temperature of a refrigerant is too
D. All of the above close to the desired condensing temperature, the Answer: A
equipment must necessarily be of:
Answer: D A. Extra heavy construction 152. What is the other name of hygrometer?
B. The direct expansion type A. Hydrometer
141. A good refrigerant should be: C. Light construction B. Manometer
A. Non-inflammable D. The indirect expansion type C. Psychometer
B. Non-poisonous D. A or C
C. Non-explosive Answer: A
D. All of the above Answer: C
147. A good refrigerant should have a
Answer: D A. High sensible heat 153. The temperature in the meat and fish box should
B. Low sensible heat be approximately
142. What tonnage of refrigerating machine is required C. High latent heat A. 10 ̊ to 20 F ̊
if the refrigerating system extracted 48,000Btu per D. Latent heat B. - 10 ̊ to 0 F ̊
hour? C. 0 ̊ to -10 F̊
A. 2 tons Answer: C D. - 10 ̊ to 10 F ̊
B. 5 tons
Answer: A C. System overcharge with refrigerant B. Too much cooling water to condenser
D. Insufficient cooling water C. Cooling water temperature too high
154. The temperature in the vegetable box should be D. A or C
approximately: Answer: A
A. 35 ̊ to 45 F̊ Answer: D
B. 10 ̊ to 20 F ̊ 160. Excessive head pressure is caused by:
C. 40 ̊ to 50 F ̊ A. Air or noncondensable gas in the system 166. Everything is in its normal operating position when
D. 15 ̊ to 30 F ̊ B. Dirty condenser tubes charging a system except the ________.
C. Insufficient cooling water to condenser A. Compressor discharge valve
Answer: A D. Any of the above B. Purge valve
C. Solenoid valve
155. The temperature in the dairy box should be Answer: D D. King (receiver discharge) valve
approximately
A. 20 ̊ to 30 ̊ F 161. The capacity of a centrifugal type compressor Answer: D
B. 0 ̊ to 10 ̊ F controlled by which of the following?
C. 10 ̊ to 20 ̊ F A. Regulating the discharge pressure 167. How often the condenser must be cleaned?
D. 30 ̊ to 40 ̊ F B. Regulating the speed A. Once a year
C. Regulating the suction pressure B. Every month
Answer: D D. B or C C. When pressure goes above normal
D. Every 6 months
156. Which of the following can be the cause of low Answer: D
head pressure? Answer: D
A. Too much or too cold condensing water 162. Where is solenoid coil installed?
B. Leaky discharge valves A. Vertically above the valve 168. Which do you think is the cause of a hot suction
C. Insufficient charge of refrigerant B. Horizontally above the valve line of a refrigerating compressor?
D. Any of the above C. Vertically below the valve A. Lack of refrigerant
D. Horizontally below the valve B. Insufficient condensing cooling water
Answer: D C. Excess refrigeration
Answer: A D. B or C
157. If thermostatic expansion valve did not appear to
be functioning properly, the cause could be: 163. What is the condition of the compressor when Answer: A
A. Foreign matter in the valve purging a refrigeration system?
B. Ruptured control bulb tubing A. The compressor is running 169. Which of the following vital components of the
C. Moisture in the system B. Compressor is running but the condenser is refrigeration system where both temperature and
D. Any of the above secured pressure are increased?
C. Shut down A. Compressor
Answer: D D. Running with the bypass open B. Condenser
C. Evaporator
158. Which of the following is used in measuring the Answer: C D. A and C
density of a brine solution?
A. Litmus paper 164. What must be done first when opening a single Answer: A
B. A chemical test packed stop valve?
C. A hydrometer A. Loosen the packing before opening 170. The refrigerant temperature is at its maximum just
D. A or B B. Tighten packing before opening before it enters the ____________.
C. Check to see that the seal is not scored A. Expansion valve
Answer: C D. None of the above B. Compressor
C. Condenser
159. Which of the following would not be cause for a Answer: A D. Evaporator
refrigerating system to short cycle on HP cutout?
A. Discharge valve leaking 165. Excessive head pressure is caused by: Answer: C
B. Pressure cutout set incorrectly A. Flooded condenser tubes
171. The greatest decrease in refrigerant temperature Answer: A A. An electrical release
occurs in the __________. B. Temperature and spring control
A. Evaporator 177. What is the cause of liquid “slugging”? C. Pressure and bellows control
B. Compressor A. Liquid in the compressor clearance space D. Bellows control
C. Condenser B. Excessive liquid refrigerant in the receiver
D. A or C C. The presence of liquid in the condenser Answer: A
causing excessive noise
Answer: A D. The pounding of liquid refrigerant in the 183. The thickness of the head gasket is important
suction line at a point of restriction because it may cause __________.
172. What do you call the storage tank for liquid A. Re-expansion
refrigerant? Answer: A B. Decreased efficiency due to increased
A. Receiver tank clearance
B. Charging tank 178. All refrigerator compressor valves are opened and C. The piston to strike the head
C. Purging close by __________. D. All of the above
D. Any of the above A. A cam shaft
B. Springs Answer: D
Answer: A C. Manual
D. B or C 184. If the expansion valve capillary tube is pinched,
173. A precooler is sometimes installed between the which of the following must be replaced?
___________. Answer: B A. Tube
A. Compressor and condenser B. Diaphragm
B. Condenser and expansion valve 179. Which of the following is also known as the C. Bulb
C. Expansion valve and evaporator “hidden heat” in refrigeration work? D. All of the above (called the power element
D. Evaporator and compressor A. Sensible heat unit)
B. Heat intensity
Answer: B C. Latent heat Answer: D
D. A or C
174. The solenoid valve controls which of the following? 185. When removing reusable refrigerant from a
A. Pressure in the evaporator coils Answer: C system, the line to the storage drum must
B. Amount of refrigerant entering the evaporator A. Be made of copper
C. Flow of refrigerant to the expansion valve 180. A bull’s eye in a full liquid line will appear B. Have no bends in it
D. Amount of circulating water to the condenser ________. C. Contain a strainer-dryer
A. Cloudy D. Be above the level of compressor
Answer: C B. Clear
C. Latent Answer: C
175. Closing the solenoid valve will stop the D. A or C
compressor through the _________. 186. Which of the following must be considered when
A. Low water-pressure cutout switch Answer: B adding or removing oil from a refrigerator unit?
B. Bypass relief valve A. Use new oil
C. Low-pressure cutout switch 181. A compressor capacity reduction device reduces B. Do not overcharge
D. High-pressure cutout switch compressor capacity ___________. C. Watch crankcase pressure
A. By reducing the compressor speed D. All of the above
Answer: C B. By bypassing hot gas
C. As the refrigerant load dictates Answer: D
176. What are the four basic methods of determining D. By reducing compressor horsepower
whether the proper amount of refrigerant is being proportionately 187. Where is the excess refrigerant removed?
added to the system? A. Suction side of the system
A. Bull’s-eye, weight, pressure and frost line Answer: C B. Discharge side of the system
B. Temperature, weight, pressure and frost line C. Bypass
C. Bull’s-eye, weight, pressure and dip stick 182. If a Freon -12 compressor trip out on “cut-out”, the D. Charging side of the system
D. Bull’s-eye, weight, litmus test and frost line solenoid valve closes by which of the following?
Answer: D D. Remove the heat required to melt one ton of 199. Which of the following is important for evaporator
ice in 24 hours coils?
188. A double trunk piston is used to _________. A. It must be placed in the top of the
A. Absorb side thrust Answer: D compartment
B. Seal off gas from crankcase B. It must be secured to the sides
C. Prevent oil from missing with gas 194. The temperature bulb of a solenoid valve is C. It should have air completely surrounding
D. All of the above attached to the ____________. them
A. Icebox coil D. It must be placed in front of circulating fans
Answer: D B. Wall of the icebox
C. Evaporator coil inlet Answer: C
189. How do you call the process of changing a solid to D. Evaporator coil outlet
a liquid? 200. If there were a 15 F̊ to 20 F
̊ temperature
A. Evaporation Answer: B differential between the temperature
B. Vaporization corresponding to the pressure at the compressor
C. Fusion 195. What usually happens if the specific gravity of the discharge and the temperature at the condenser
D. Condensation brine is too low? outlet, it would probably indicate the need for
A. The brine will freeze _____________.
Answer: C B. Solids will deposit A. More refrigerant
C. It will be more heat-absorbing B. Purging the system
190. If there is too much lube oil in the system, what D. All of the above C. More circulating water
must be done? D. Less circulating water
A. Remove same at once Answer: A
B. Wait until next overhaul to remove Answer: B
C. Wait until next recharging to remove 196. The cooling water regulator is automatically
D. Any of the above actuated by which of the following? 201. Which of the following is the most appropriate
A. The discharge pressure of the refrigerant definition of Latent heat?
Answer: A B. The temperature of the refrigerant A. Heat removed to melt ice
C. An electric relay B. Heat removed to change temperature of a
191. The temperature bulb of the thermo-expansion D. A thermo relay substance
valve is attached to which of the following? C. Heat added to change temperature of a
A. Icebox coil Answer: A substance
B. Evaporator coil outlet D. Heat added to change the state of substance
C. Evaporator coil inlet 197. What will happen to the capacity if the superheat
D. Wall of the icebox is increased on the suction side? Answer: D
A. Increases
Answer: B B. Decreases 202. If the compressor discharge becomes frosted, the
C. Remains the same probable cause would be __________.
192. What is the use of the suction pressure regulating D. Will double A. Refrigerant flooding back
valve? B. Expansion valve improperly set
A. Cuts in the compressor Answer: B C. Too much cooling water
B. Maintains the back pressure in the D. Insufficient cooling water
evaporator coils 198. Which do you think is very important in adjusting
C. Cuts out the compressor compressor “V” belts? Answer: A
D. Controls the expansion valve A. Allow about ½” slack
B. Make it as tight as possible 203. The low-pressure control switch:
Answer: B C. Make belt just tight enough to turn pulley A. Is a safety device
D. Keep belts parallel B. Actuates the cooling water
193. How is a one-ton plant described? C. Cuts out the compressor to maintain proper
A. Remove one ton of heat from the reefer box Answer: A flow
B. Melt one ton of ice in 24 hours D. Regulates the King valve
C. Make one ton of ice in 24 hours
Answer: C C. ̊
Boiling point over 200 F 215. Calcium chloride is sometimes used in
D. Very volatile refrigeration system as a:
204. Which of the following is an indications of faulty A. Refrigerant
Freon compressor valves? Answer: C B. Lubricant
A. Compressor runs continuously C. Primary coolant
B. Low head pressure – high suction pressure 210. Which of the following type valves are not found D. Secondary coolant
C. Gradual or sudden decrease in capacity on a Freon -12 system?
D. Any of the above A. Bellows Answer: D
B. Duplex
Answer: D C. Diaphragm 216. When the outlet temperature of the evaporator
D. Single packed exceeds the inlet temperature the condition is
205. What is the probable cause if a compressor runs called ___________.
continuously? Answer: B A. Boiling
A. A clogged condenser B. Superheating
B. Insufficient refrigerant 211. How much will be removed by one-ton C. Melting
C. Faulty cooling water valve refrigeration unit? D. Freezing
D. Any of the above A. 200 Btu per min.
B. 144 Btu per min. Answer: B
Answer: D C. 400 Btu per min.
D. 2000 Btu per min. 217. Which of the following refrigerants would give the
206. To test a thermostatic valve, immerse the bulb in: most trouble when operating with warm circulating
A. Crushed ice Answer: A water?
B. Hot water A. Freon -12
C. Oil 212. Which of the following is the primary purpose of B. CO2
D. None of the above the evaporator? C. Ammonia
A. Transmit latent heat of fusion D. Ethyl chloride
Answer: A B. Transmit latent heat of evaporation
C. Absorb latent heat of fusion Answer: B
207. If a compartment requires the removal of 36,000 D. Absorb latent heat of evaporation
Btu per hour, how much is the necessary 218. As Freon leaves the expansion valve:
compressor capacity? Answer: D A. Pressure increases – volume increases
A. 6 tons B. Pressure decreases – volume increases
B. 3 tons 213. An automatically controlled Freon -12 compressor C. Pressure decreases – volume decreases
C. 2 tons will start when the __________. D. Pressure increases – volume decreases
D. 1 ton A. Expansion valve opens
B. Solenoid valve opens Answer: B
Answer: B C. Expansion valve closes
D. Solenoid valve closes 219. The suction pressure switch is actuated by:
208. Which of the following must be done to eliminate A. Pressure on a bellows
frost on the discharge pipe of the compressor? Answer: B B. Temperature on a bellows
A. Open the expansion valve C. A thermo-pressure regulator
B. Regulate water to the condenser 214. A single trunk piston-type compressor is D. A thermostatic temperature device
C. Crack bypass valve undesirable for a Freon unit because the:
D. None of the above A. Lubricant mixes with the refrigerant Answer: A
B. Refrigerant reduces the crankcase pressure
Answer: D C. Lubricant temperature becomes excessive 220. A thermometer senses which of the following?
D. Refrigerant becomes superheated A. Latent heat
209. The following are standard characteristics of Freon B. Sensible heat
-11 except: Answer: A C. Heat of fusion
A. Non-toxic D. Specific heat
B. Separates from water
Answer: B D. B and C A. Using skirt type pistons
B. Maintaining a vacuum in the crankcase
221. Five pounds of water heated to raise the Answer: D C. Using shaft seals
temperature 2 F̊ requires how many Btu? D. Using lantern rings
A. 20 Btu 227. Absolute zero on the Fahrenheit scale is equal to:
B. 5 Btu A. -273 ̊ Answer: C
C. 2 Btu B. -460 ̊
D. 10 Btu C. 0 ̊ 233. The heat used to change a liquid to a gas or vapor
D. -100 ̊ is called latent heat of
Answer: D A. Absorption
Answer: B B. Vaporization
222. Superheat is heat added _________. C. Fusion
A. In changing liquid to vapor 228. A quick method of detecting a leaky condenser on D. Liquid
B. After all liquid has been changed to vapor a Freon system would be to __________.
C. To increase pressure A. Compare temperatures and pressures Answer: B
D. To increase temperature B. Test circulating water with proper chemicals
C. Open vent on head and test with halide torch 234. What is the compression ratio of a refrigerator
Answer: B D. Use soapsuds on condenser outlet fittings compressor?
A. The ratio of the absolute suction pressure to
223. The expansion valve does not seem to be Answer: C the absolute discharge pressure
operating properly. There is high superheat. Test B. The ratio of the absolute discharge pressure
by listening to the sound of Freon flooding through 229. Which of the following is not a characteristic of to the absolute suction pressure
the tubes and _____________. Freon -12? C. The ratio of the excessive liquid refrigerant in
A. Warm bulb with hand A. Corrosive the receiver to that in the system
B. Place bulb in cold water B. Non-inflammable D. None of the above
C. Place bulb in hot water C. Odorless
D. Any of the above D. Boiling point -21 F ̊ Answer: B

Answer: A Answer: A 235. The thermostatic expansion valve is designed to


maintain a constant _________.
224. What do you call a material in a dryer? 230. In the discharge line between the compressor and A. Superheat
A. Drain the condenser one would find: B. Flow
B. Dryer A. High pressure, high temperature gas C. Pressure
C. Desiccant B. High pressure, low temperature liquid D. Temperature
D. A or C C. High temperature, high pressure liquid
D. High pressure, low temperature gas Answer: A
Answer: C
Answer: A 236. A refrigerant gives up heat when it
225. What is the main function of a receiver? A. Vaporizes
A. Separate the oil from the refrigerant 231. For a low speed compressor the belt drive is B. Evaporates
B. Cool the hot gases preferred to the direct drive because it: C. Condenses
C. Store the refrigerant A. Eliminates the use of mineral lubricating oils D. Boils
D. Condense the refrigerant B. Permits the use of a smaller high-speed
motor Answer: C
Answer: C C. Absorbs torsional vibration
D. Reduces the number of automatic controls 237. The cooling component of a refrigeration cycle is
226. During the re-expansion portion of the refrigeration called _________.
compressor cycle Answer: B A. A receiver
A. The suction valve is open B. An evaporator
B. The suction valve is closed 232. Refrigerant leakage from the compressor C. A condenser
C. The discharge valve is closed crankcase is prevented by D. A desiccant
Answer: B D. Any of the above C. Defrost coils
D. B and C
238. When ordering an expansion valve which of the Answer: D
following information is necessary? Answer: D
A. Size and pressure 244. A “direct” refrigeration system is one in which:
B. Size, tonnage, temperature and pressure A. Ice is used for cooling 250. When the compressor is operated in short spurts
C. Pressure and temperature B. A secondary coolant is pumped through the after a lay-up or overhaul the purpose is to:
D. Size and tonnage evaporator coils A. Allow refrigerant time to circulate
C. The refrigerant passes through coils in the B. Determine actual compressor oil level
Answer: B evaporator C. Effect gradual cooling
D. Any of the above D. None of the above
239. A hot crankcase and cylinder head accompanied
by a low suction pressure would be caused by Answer: C Answer: D
A. Excess refrigerant
B. Insufficient refrigerant 245. A “CARRENE” is a type of 251. Which of the following types of valve are not used
C. Air in the system A. Refrigerant oil for suction or discharge?
D. Stuck discharge valve B. Refrigerant A. Gate
C. Scale cleaner B. Diaphragm
Answer: B D. None of the above C. Metallic
D. Ring
240. Which of the following characteristics that is not Answer: B
desirable in a refrigerant? Answer: A
A. Low latent heat fusion 246. What is the first step that must be done when
B. High latent heat of vaporization securing a system? 252. What usually happened if brine has a high specific
C. Low latent heat of vaporization A. Open bypass valve gravity?
D. A and C B. Close receiver (King) valve A. It will freeze
C. Open solenoid valve B. It will crystalize
Answer: C D. Cut out compressor on high pressure C. Nothing will happen
D. It will solidify
241. The principle of the centrifugal system is based on Answer: B
which of the following? Answer: B
A. Kinetic energy 247. When securing a Freon-12 system for repairs
B. Potential energy A. Open the line at 1 to 2 pounds pressure 253. When does the refrigerant gives-up heat?
C. Lenz’s law B. Open the line at 5 to 10 pounds pressure A. When it evaporates
D. Boyle’s law C. Pump down to a slight vacuum B. When it condenses
D. Pump down to 10 ̊ vacuum C. When it vaporizes
Answer: A D. When it boils
Answer: A
242. The temperature in the vegetable box should be Answer: B
approximately 248. The most common reason for the small usage of
A. -5 F ̊ CO2 system is: 254. The thermostatic expansion valve is designed to
B. 20 F ̊ A. It is too dangerous maintain constant_____________
C. 38 F ̊ B. Its upkeep is too high A. Pressure
D. 10 F ̊ C. The machinery is too heavy B. Flow
D. It is too costly C. Temperature
Answer: C D. Superheat
Answer: C
243. The compressor will run continuously if there is Answer: D
A. Insufficient refrigerant 249. To increase heat transfer in the evaporator
B. Air in the system A. Increase suction pressure 255. A reactor plate is essential to the operation of a
C. Too heavy a load B. Increase air circulation ___________.
A. Thermostatic valve 261. Which of the following would not cause high head C. It is easier to check leaks
B. Halide torch pressure? D. Iron corrodes
C. Solenoid bulb A. Air in system
D. None of the above B. Overcharge of refrigerant Answer: A
C. Circulating water too warm
Answer: B D. Liquid freezing back 267. The cooling water regulator is actuated by:
A. An electric relay
256. The greatest decrease in the temperature of the Answer: D B. Pressure of the refrigerant
refrigerant is at the _________ C. Temperature of the refrigerant
A. Condenser 262. A “cold diffuser” is a: D. None of the above
B. Expansion valve A. Type of condenser
C. Evaporator B. Special valve used as a bypass Answer: B
D. Receiver C. Type of evaporator
D. None of the above ̊ temperature drop between the
268. If there is a 20 F
Answer: C temperature corresponding to the pressure at the
Answer: C compressor discharge and the condenser outlet,
257. The temperature of the refrigerant is highest just one should:
before it enters the: 263. Which do you think is the effect of “subcooling”? A. Decrease the amount of circulating water
A. Receiver A. It causes flooding back to the compressor B. Increase the amount of circulating water
B. Evaporator B. It reduces the horsepower per ton of C. Purge the system
C. Condenser refrigeration D. None of the above
D. King valve C. It increases the compression ratio
D. It increases the horsepower per ton of Answer: C
Answer: C refrigeration
269. If refrigeration controls were constructed with little
258. A hot suction line could be caused by: Answer: B or no differential, the:
A. Insufficient refrigerant A. Compressor would short cycle
B. Excess refrigerant 264. Subcooling is to: B. Icebox would get too cold
C. Excess cooling water A. Cool the evaporator below its normal C. Icebox would get too warm
D. Insufficient cooling water temperature D. Refrigerant would be subcooled
B. Cool the liquid before it enters the evaporator
Answer: A C. Cool a liquid at constant pressure to a Answer: A
temperature lower than its considering
259. Which of the following devices will prevent the temperature 270. What is the use of back pressure regulating valve?
relief valve from opening in the event of excessive D. Cool the refrigerant gas before it enters the A. Controls the evaporator temperature by
pressure? receiver throttling the flow of liquid refrigerant
A. Water failure switch B. Maintains a fixed pressure in the evaporator
B. High-pressure cutout switch Answer: C coils
C. Low-pressure cutout switch C. Controls the flow of circulating water in the
D. Any of the above 265. When figuring compression ratio, the ratio will condenser tubes
increase with a ______________ D. Controls the temperature in the evaporator
Answer: B A. Lower suction pressure coils
B. Higher suction pressure
260. Which of the following would cause the suction C. Higher discharge pressure Answer: B
switch to remain open? D. A and C
A. Insufficient spring tension 271. If the thermal bulb becomes loose on the
B. Too much spring tension Answer: D evaporator coils, it will cause____________
C. Bellows broken or jammed A. An electrical short
D. A or B 266. Copper tubing is used in Freon system because: B. Flooding back of the refrigerant
A. It has less internal resistance C. Improper operation of expansion valve
Answer: C B. It is cheaper D. Any of the above
Answer: C Answer: B Answer: A

272. External frost on inlet of expansion valve indicates: 278. A Freon-12 gage shows pressure and _______ 284. The process that takes place in the evaporator is
A. Expansion valve plugged or dirty A. Superheat temperature called:
B. Head pressure too high B. Saturation temperature A. Transfer of the latent heat of vaporization
C. Refrigerating compartment too cold C. Back pressure B. Absorption of the latent heat of vaporization
D. Air in system D. Vacuum C. Absorption of the latent heat of fusion
D. Transfer of the latent heat of fusion
Answer: A Answer: B
Answer: B
273. Subcooling of the refrigerant results in: 279. Moisture in a system will cause a:
A. Less circulating water needed A. Faulty expansion valve 285. What is the use of the low-water cutout switch?
B. Effect of refrigerant increased B. High suction pressure A. Recirculates the cooling water when there is
C. Liquid less likely to vaporize C. High suction temperature too much refrigerant in the condenser
D. B and C D. Low discharge temperature B. Stops the compressor when there is no
refrigerant running to the evaporator
Answer: D Answer: A C. Stops the flow of refrigerant when the
condenser temperature is too low
274. Which of the following is the reason when the 280. Which of the following is not essential to a D. Stops the compressor when there is
crankcase is cooler than the suction line? centrifugal type of compressor system? insufficient cooling water
A. Too much refrigerant A. Evaporator
B. Insufficient refrigerant B. Distiller Answer: D
C. Expansion valve open too wide C. Condenser
D. Suction valve leaking D. Expansion valve 286. The high-pressure cutout switch:
A. Stops the flow of refrigerant when the
Answer: A Answer: B condenser temperature is too high
B. Recirculates the refrigerant through the
275. If the outlet of the thermostatic valve is warmer 281. A scale trap in a Freon system will be found on the compressor under emergency conditions
than the inlet, it indicates: : C. Stops flow of water to condenser when there
A. Flooding back A. Receiver is no refrigerant passing through
B. Thermostatic valve working properly B. Discharge side D. Stops compressor when head pressure is too
C. Thermostatic valve not working properly C. Suction side high
D. Solenoid valve not working properly D. Condenser
Answer: D
Answer: C Answer: C
287. Zinc rods are found in this:
276. High temperature of cylinder heads and crankcase 282. Which of the following would cause expansion A. Gas side of the condenser
is caused by: valve failure? B. Salt water side of the condenser
A. Insufficient refrigerant A. Dirt in the valve C. Evaporator area
B. Too much refrigerant B. Moisture in the system D. Compressor crankcase
C. High head pressure C. Bulb in icebox ruptured
D. Noncondensable gases D. Any of the above Answer: B

Answer: A Answer: D 288. Which of the following is not essential to a


compression refrigeration system?
277. Frost on the high- pressure side of a thermostatic 283. Two compressor should not be run in parallel A. A receiver
expansion valve would probably be caused by because: B. A condenser
A. High head pressure A. There is a possibility of losing oil C. An evaporator
B. Dirty expansion valve B. It will give over-capacity D. A dehydrator
C. Refrigerator box too cold C. It is not efficient
D. Loss of circulating water to condenser D. A or B Answer: A
289. What is the effect if the refrigerant is removed from the 295. With one machine serving several compartments, with
system too fast? one compartment reaches the desired temperature in that 301. High superheat of the vapor in the steam would cause
A. It may flood the evaporator compartment is maintained by: A. an increase in capacity
B. It may “freeze-up” the condenser A. the expansion valve B. a more efficient unit
C. It will lower the icebox temperature B. the solenoid valve C. a decrease in capacity
D. None of the above C. the back-pressure valve D. A and B
D. any of the above
Answer: B Answer: C
Answer: B
290. Closing the solenoid valve stops the compressor 302. The solenoid valve is actuated by which of the
through the: 296. The back-pressure regulating valve: following?
A. high-pressure cutout switch A. controls the water flow through the condenser A. a bellows
B. low water-pressure cutout switch B. stops the flow of liquid refrigerant when temperature B. a magnet
C. low-pressure cutout switch drops in evaporator C. a spring
D. high water-pressure cutout switch C. maintains a fixed pressure in the evaporator coils D. any of the above
D. none of the above
Answer: C Answer: B
Answer: C
291. A precooler, if used, is located between the: 303. Which of the following stops the compressor before the
A. expansion valve and evaporator 297. Faulty F-12 compressor valves will be indicated by: relief valve opens?
B. compressor and condenser A. low head pressure-high suction pressure A. low water-pressure cutout
C. evaporator and compressor B. compressor running continuously B. high-pressure cutout
D. condenser and expansion valve C. gradual or sudden decrease in capacity C. low oil-pressure cutout
D. all of the above D. low-pressure cutout
Answer: D Answer: B
Answer: D 304. What is the effect if the thermal bulb is loose on the
292. When the outlet temperature at the evaporator exceeds evaporator coils?
the inlet temperature, the condition is known as: 298. How were tubing joints done? A. improper operation of expansion valve
A. superheating A. brazed B. flooding back of refrigerant
B. desuperheating B. welded C. vibration causing leaks
C. dehydrating C. screwed D. improper operation of solenoid valve
D. saturating D. silver soldered
Answer: A
Answer: A Answer: D
305. If the superheat on the suction side of the compressor
299. When starting a refrigerating unit, be sure the water is increased, what will happen to the tonnage capacity of the
293. The purpose of providing hot-gas defrosting facilities: side of the condenser is _____. unit?
A. defrosting without raising compartment temperature A. secured A. increases
above 32 ℉ B. bypassed B. decreases
B. thawing frozen coils C. vented C. no change
C. defrosting automatically D. B and C D. none of the above
D. B or C
Answer: C Answer: B
Answer: A
294. The suction control switch on the compressor is a 300. The water regulating valve is operated by the 306. What do you usually do to correct slugging and flooding
A. thermal element A. compressor discharged pressure back?
B. thermostat B. compressor discharged temperature A. change the discharged pressure
C. pressure element C. compressor suction pressure B. clean the traps
D. bellows D. none of the above C. removes some refrigerant
D. check the expansion valve
Answer: C Answer: D
Answer: D Answer: B D. diesel oil

307. In a Freon-11 system there is no 313. Which of the following will cause an automatically Answer: B
A. receiver controlled F-12 compressor to start? 319. If one of the ship’s service boxes suddenly stops
B. distiller A. closing the expansion valve cooling, the most likely cause would be the malfunction of
C. condenser B. opening the expansion valve the:
D. evaporator C. closing the solenoid valve A. discharge valve
D. opening the solenoid valve B. king valve
Answer: B C. solenoid valve
Answer: D D. expansion valve
308. The suction pressure control valve is actuated by which
of the following? 314. Where is the solenoid coil installed? Answer: C
A. thermostat A. horizontally over the valve
B. bellows B. vertically over the valve 320. Which of the following items is not important when
C. thermal element C. vertically below the valve using a halide torch?
D. pressure diaphragm D. horizontally below the valve A. move flame slowly around joints
B. hold flame close to the joints
Answer: D Answer: B C. adjust to a clear white flame
309. Which of the following would cause the crankcase and D. adjust to a clear blue flame
head to get hot with low suction pressure? 315. The “refrigeration effect” of a refrigerant is:
A. excess refrigeration A. quantity of heat that 1 pound of refrigerant absorbs while Answer: C.
B. air in system flowing through the evaporator under given condition
C. insufficient refrigeration B. amount of heat it can remove in a given time 321. A system should be purged when the liquid outlet
D. insufficient cooling water C. quantity of heat that 1 pound of refrigerant absorbs temperature from the condenser is more than the
D. A or B temperature corresponding to the discharge pressure:
Answer: C A. 10 ℉ above
Answer: A B. 20 ℉ below
310. Which of the following is the function of a suction C. 20 ℉ above
pressure regulating valve? 316. The “refrigerant effect” of a refrigerant is always: D. 5 ℉ below
A. cuts out the compressor A. more than its latent heat
B. controls the expansion valve B. more than its sensible heat Answer: D
C. cuts in the compressor C. less than its sensible heat
D. maintains the proper back pressure D. less than its latent heat 322. Which of the following would cause the compressor to
run continuously?
Answer: D Answer: D A. low-pressure switch jammed
B. high-pressure switch jammed
311. Which of the following is the cause if the outlet of a 317. What will happen if the expansion valve is opened too C. clogged strainer
thermostat valve is warmer than the inlet inside? wide? D. defective thermal bulb
A. valve is working properly A. liquid will flow back to the compressor
B. valve is not working properly B. it will make no difference, as the automatic expansion Answer: A
C. solenoid valve is not working properly valve is still operating
D. none of the above C. the condenser coils will overheat 323. In a vapor compression cycle the lowest temperature is
D. A or C found in ____.
Answer: B A. receiver
Answer: A B. condenser
312. How is a thermostatic valve tested? C. compressor
A. immersing its bulb in warm water 318. Which of the following should not be used to clean D. evaporator
B. immersing its bulb in ice water scale tarps?
C. holding its bulb in one’s hand A. compressed air Answer: D
D. shorting out the cutout switch B. cotton waste 324. The moisture in a refrigeration system can be removed
C. kerosene with the help of which of the following?
A. driers B. more than 1
B. evaporators C. equal to 1 Answer: B
C. dehumidifies D. depends upon the make of it
D. coolers 336. White ice is
Answer: B A. Fast cooled water
Answer: A B. sub cooled water
331. Horsepower per ton of refrigeration is expressed as: C. due to dissolved air, gases and impurities
325. Which of the following refrigerants has lowest freezing A. 4.75/COP D. formed by blowing air during freezing
point temperature? B. 4.75 x COP
A. Freon 12 C. COP/4.75 Answer: C
B. Ammonia D. 4.75/COP
C. Freon 22 337. Clear ice
D. Freon 11 Answer: C A. is pure ice
B. contains dissolved gases
Answer: C 332. The highest temperature in vapor compression cycle is C. contains dissolved air
produced during: D. is formed by blowing air during freezing
326. During compression in a vapor compression cycle when A. expansion valve
the refrigerant is super-heated, what happens to the B. evaporator Answer: D
performance? C. condenser discharged
A. C.O.P. is reduced D. compressor discharged 338. Dry ice is
B. C.O.P. remains unchanged A. free from water
C. work done is increased Answer: D B. free from dissolved air and gases
D. refrigerating effect is reduced C. does not contain impurities
333. Which type of compressor is used in refrigeration D. solidified form of carbon dioxide
Answer: A system?
A. reciprocating Answer: D
327. A Bell-Coleman Cycle is also known as _____. B. centrifugal
A. reversed Otto cycle C. rotary sliding vane 339. The leaks in the refrigeration system using Freon can
B. reversed Joule cycle D. all of the above be detected by:
C. reversed Rankine cycle A: Halide torch which detention forms greenish flame lighting
D. reversed Carnot cycle Answer: D B. Sulfur sticks which on detection forms white smoke
C. using certain reagents
Answer: B 334. What is the reason why a thermometer in vapor D. smelling
compression system is installed close to the compressor?
328. Critical Temperature is that temperature above which: A. because it helps the operator in adjusting compressor for Answer: A
A. a gas will never liquefy greatest efficiency
B. a gas gets immediately liquefy B. because temperature indicates whether liquid or vapor 340. The lower horizontal line of the refrigeration cycle
C. water gets evaporated refrigerant is going to compressor plotted on pressure-enthalpy diagram represents:
D. water will never evaporate C. because temperature helps in calculating the coefficient A. compression of the refrigerant vapor
Answer: A of performance B. evaporation of the refrigerant liquid
D. because the performance of other units of the cycle can C. condensation of the refrigerant vapor
329. Rating of a domestic refrigerator is of the order of: be controlled D. metering of the refrigerant liquid
A. 0.1 tons
B. 50 tons Answer: A Answer: B
C. 100 tons
D. 40 tons 335. Which refrigerant has the highest critical point 341. Which of the following uses a secondary refrigerant?
temperature? A. domesticated refrigerator
Answer: A A. ammonia B. room air conditioner
B. Freon 11 C. deep freezer
330. What is the usual COP of a domestic refrigerator? C. Freon 12 D. ice plant
A. less than 1 D. Freon 22
Answer: D C. -78.5 ℃
342. Which of the following secondary refrigerants generally Answer: C D. -40 ℃
used milk chilling plants?
A. Brine 348. Which refrigerant is used for the air-conditioning of the Answer: D
B. Ammonia solution passenger aircraft cabin?
C. Glycol A. ammonia 354. The following gas is preferred in refrigeration system:
C. Sodium silicate B. Freon 12 A. F-11
C. Freon 11 B. F-22
Answer: C D. air C. 𝐶𝑂2
D. 𝑁𝐻3
343. What is the cycle where a refrigeration system Answer: D
generally operates? Answer: B
A. open cycle 349. Oil separator in a refrigeration cycle is installed
B. close cycle between the ___________. 355. The bhp/ton refrigeration requirement with increase in
C. mixed cycle A. compressor and condenser condenser temperature, in a vapor compression system that
D. Hybrid cycle B. condenser and evaporator uses reciprocating compressor receiving refrigerant gas at
C. metering device and evaporator constant suction temperature, will follow:
Answer: B D. none of the above A. linearly decreasing characteristic
B. linearly increasing characteristic
344. Which of the following is also known as Refrigerant Answer: A C. first increase rapidly and then decrease slowly
No.R-717? D. first increase slowly and then rapidly
A. Ammonia 350. A flash chamber is installed in the refrigeration circuit so
B. Freon 22 as to: Answer: B
C. Freon 12 A. improve overall heat transfer coefficient
D. methyl chloride B. reduce pressure losses through the evaporator 356. Antifreeze chemicals are:
C. reduce the size of the evaporator by avoiding vapors A. same as refrigerants
Answer: A going to evaporator B. those that are added to refrigerants for better
D. all of the above performance
345. In which part of the vapor compression cycle there is C. those that lower down the freezing points of liquids
abrupt change in pressure and temperature Answer: D D. those that do not freeze at all
A. solenoid valve
B. evaporator 351. The color of the flame of halide torch, in case of Answer: C
C. expansion valve leakage of Freon refrigerant, will change to:
D. drier A. yellow 357. The capacity of ice making is always
B. red A. less than the refrigerating effect
Answer: C C. green B. greater than the refrigerating effect
D. orange C. equal to the refrigerating effect
346. What do you call a plate or vane used to direct or D. directly proportional to the refrigerating effect
control movement of fluid or air within the confined area? Answer: C
A. baffle Answer: D
B. bellows 352. What is the boiling temperature of F-12?
C. regulator A. -33.33 ℃ 358. To check the lack of refrigerant or clogged refrigerant
D. diffuser B. -78.5 ℃ lines
C. - 29.8 ℃ A. installing service valves must be necessary
Answer: A D. -40 ℃ B. installing king valve must be an option
C. sight glasses can be possibly used
347. Brazing is used for joining two _____. Answer: C D. discharge pressure and temperature must be monitored
A. two ferrous material
B. one ferrous and non-ferrous material 353. What is the boiling temperature of F-22? Answer: A
C. two non-ferrous material A. -33.33 ℃
D. two non-metals B. - 29.8 ℃ 359. Past ME Board Question
Can any refrigerant be used with a booster system? D. -40.7 ℃ Answer: C
A. Yes Answer: C
B. No 370. Past ME Board Question
C. Used for some 365. Past ME Board Question The vapor cycle in thermal efficiency as the refrigeration
D. Used for all Boiling temperature of Freon 22 is: cycle is to the ______.
Answer: B A. -33.33 ℃ A. Energy efficiency ratio
B. -29.8 ℃ B. COP for a refrigerator
360. Past ME Board Question C. -78.5 ℃ C. COP for a heat pump
What is the chemical formula of methylene chloride? D. -41.04 ℃ D. Carnot efficiency
A. 𝐶𝐻𝐶𝑙2 𝐹 Answer: D Answer: B
B. 𝐶𝐻2 𝐶𝑙2
C. 𝐶𝐶𝑙𝐹3 366. Past ME Board Question 371. Past ME Board Question
D. 𝐶𝐵𝑅𝐹3 The faster way to remove frost from a direct-expansion The main purpose of a sub cooler in a refrigerating system
Answer: B finned-tube evaporator is to: especially a two-stage system is to:
A. sent hot gas through the coil A. Increase the heat rejection per ton and avoid
361. Past ME Board Question B. scrape off frost system shutdown
What is the chemical formula of butane? C. wash with warm water B. Improve the flow of evaporator gas per ton and
A. 𝐶2 𝐻5 𝐶𝑙 D. shut down warm let frost melt increase the temperature
B. 𝐶3 𝐻8 Answer: A C. Reduce the total power requirements and return
C. 𝐶2 𝐻4 𝑂2 oil to the compressor
D. 𝐶4 𝐻10 367. Past ME Board Question D. Reduce the total power requirements and heat
Answer: D You want to change refrigerants in a Freon 12 plant using a rejection to the second stage
reciprocating single-acting compressor. Which refrigerants Answer: D
requires smallest amount of change to the system?
362. Past ME Board Question A. Carbon dioxide 372. Past ME Board Question
Type of refrigerant control which maintains pressure B. Ammonia Ton of refrigeration is a unit equivalent to:
difference between high side and low side pressure in C. Methyl chloride A. 50.4 kCal/sec
refrigerating mechanism. D. Aqua ammonia B. 12,660 kN-m/hr
A. Suction valve Answer: C C. 3413 kW/hr
B. Expansion valve D. 2545 Btu/hr
C. Service valve Answer: B
D. Solenoid valve
Answer: B 373. Past ME Board Question
368. Past ME Board Question In a refrigerating system, the heat absorbed in the
363. Past ME Board Question Soft copper tubing that is used for refrigerant piping erected evaporator per kg mass of refrigerant passing through:
The dividing point between the high pressure and low on premises containing other than the Group 1 refrigerant A. Equals the increase in enthalpy
pressure sides of the refrigeration cycle occurs at the: must be protected by _____. B. Does not depend on the refrigerant used
A. expansion valve A. Supporting it by approved clamps C. Is decreased if pre-cooler is used
B. compressor B. Limiting its length between supports up to 10 feet D. Equals the increase in volume
C. condenser C. A rigid or flexible metal enclosure Answer: A
D. cooling coil D. Wrapping it with insulation tape throughout its
Answer: A length 374. Past ME Board Question
Answer: C Lithium bromide is used in refrigeration system in:
A. Condensate return lines
369. Past ME Board Question B. Absorbers
364. Past ME Board Question A Freon 12 refrigeration system is fitted with thermal C. Centrifugal compressors
Boiling temperature of Freon 12 is: expansion valves. The valves are rated in: D. Ion exchangers
A. -33.33 ℃ A. Pound per minute Answer: B
B. -78.5 ℃ B. Super heat setting
C. -29.8 ℃ C. Tons of refrigeration 375. Past ME Board Question
D. Cubic feet per minute
A refrigeration system in which only part of the refrigerant Answer: D D. 60
passes over the heat transfer surface is evaporated and the Answer: A
balance is separated from the vapor and recirculated. 381. Past ME Board Question
A. Direct expansion system The boiling point of Freon 12 is: 387. Past ME Board Question
B. Chilled water system A. -41.04 ℉ The high pressure of refrigerant system consist of the line to
C. Flooded system B. -40.60 ℉ the expansion valve, the receiver, the uppermost half of the
D. Multiple system C. -38.40 ℉ compressor and the:
Answer: C D. -31.40 ℉ A. Line after the expansion valve
Answer: A B. Lower most half of compressor
376. Past ME Board Question C. Condenser
A device whose primary function is to meter the flow of 382. Past ME Board Question D. Evaporator
refrigerant to the evaporator. The secondary refrigerant used in milk chilling plants is Answer: C
A. Sniffer valve generally:
B. Equalizer A. Brine 388. Past ME Board Question
C. Thermostatic expansion valve B. Ammonia Which of the following material is suitable for tubing in
D. Crossover valve C. Glycol refrigeration application where refrigerant ammonia is
Answer: C D. Sodium silicate employed?
Answer: C A. Plastic
377. Past ME Board Question B. Brass
Select the one in which secondary refrigerant is used. 383. Past ME Board Question C. Steel copper
A. Domestic refrigerator The sensible heat ratio is 0.8. That is D. Copper
B. Room air conditioner A. 20% latent heat and 60% sensible heat Answer: C
C. Deep freezer B. 80% sensible heat and 20 % latent heat
D. Ice plant C. 80% sensible heat and 20% sensible heat and 389. Past ME Board Question
Answer: D latent heat If PV is the power required for a vapor-compression
D. 20% latent heat and 80% sensible heat and latent refrigeration system, then what is the power required for an
378. Past ME Board Question heat air-refrigeration system, assuming that they have the same
AN odorless refrigerant whose boiling point varies over a Answer: B capacity?
wide range of temperature. A. 5PV
A. Freon 22 384. Past ME Board Question B. 2PV
B. Freon 12 The effect of superheating the refrigerant is to: C. PV/10
C. Ammonia A. Increase the COP D. 1/PV
D. Freon 11 B. Increase and decrease the COP Answer: A
Answer: A C. Decrease COP
D. Maintains COP 390. Past ME Board Question
379. Past ME Board Question Answer: A Tons of refrigeration in Btu/24 hours is
The part that directs the flow of refrigerant through the A. 288,000
compressor. 385. Past ME Board Question B. 28,800
A. Wrist pin The ice making capacity is always C. 290,000
B. Valve A. Directly proportional to the refrigerating effect D. 29,000
C. Piston B. Less than the refrigerating effect Answer: A
D. Connecting rod C. Greater than the refrigerating effect
Answer: B D. Equal to the refrigerating effect 391. Past ME Board Question
Answer: A R-22 is:
380. Past ME Board Question A. Dichlorodiflouromethane
The highest temperature in vapor compression cycle is 386. Past ME Board Question B. Monochlorodiflouromethane
produced during: The amount of sensible heat for a sensible heat ratio of 0.80 C. Methyl chloride
A. Receiver and a total cooling load of 100 is: D. Trichlorodiflouromethane
B. Evaporator A. 80 Answer: B
C. Condenser discharge B. 20
D. Compressor discharge C. 100 392. Past ME Board Question
A salimeter reads the: CHAPTER 15 – AIR CONDITIONING Answer: D
A. Density of brine
B. Rate of brine 1. How do you call the water vapor content of air? 7. The temperature at which water vapor in the atmosphere
C. Dew point temperature A. Moisture begins to condense is known as the:
D. Relative humidity B. Humidity A. vapor point
Answer: A C. Dew B. dew point
D. Vapor C. moisture point
393. Past ME Board Question D. none of the above
Which of the following measure the density of salt in water? Answer: B
A. Salimeter Answer: B
B. Hydrometer 2. When air contains all of the water vapor it can hold, it is
C. Pitot tube said to be 8. Saturation temperature is the same as;
D. Calorimeter A. Simulated A. dew point
Answer: A B. Loaded B. vapor temperature
C. Saturated C. steam temperature
D. Moistured D. humidity
394. Past ME Board Question
What is the device used to protect the compressor from Answer: C Answer: A
overloading due to high head pressure?
A. Overload relay 3. What is the instrument used to register relative 9. When the bypass factor is B, the coil efficiency for
B. Hold back suction valve humidity? sensible cooling of air is equal to:
C. Thermostatic expansion valve A. Hygrometer A. 1 + B
D. Expansion valve B. Hydrometer B. 1 – B
Answer: A C. Perometer C. 1 – B / B
D. Manometer D. 1 + B / B
395. Past ME Board Question
The purpose of the expansion valve is to control the flow of Answer: A Answer: B
the refrigerant to the evaporator. The other function is to:
A. Reduce the pressure of the liquid refrigerant 4. Humidity is a measure of which of the following? 10. The psychrometric chart in air conditioning determines
B. Increase the pressure of the vapor refrigerant A. water vapor content the
C. Reduce the pressure of the vapor refrigerant B. temperature A. wet bulb and dry bulb temperatures
D. Increase the pressure of the liquid refrigerant C. latent heat B. psychrometric temperature requirements
Answer: A D. any of the above C. saturation temperature and relative humidity
D. moist air conditions
396. Past ME Board Question Answer: A
It prevents the refrigerant from the condenser to go back to Answer: D
the compressor. 5. Air conditioning is the process of:
A. Check valve A. keeping a place cool 11. Which of the following is considered as comfort condition
B. Float switch B. removing heat from a specific area in air conditioning?
C. Expansion valve C. maintaining the air at a required temperature and humidity A. 40℃ DBT, 80% RH
D. Low side float D. A or C B. 30℃ DBT, 60% RH
Answer: A C. 30℃ DBT, 80% RH
Answer: C D. 20℃ DBT, 60% RH
397. If the freezing point of water is 0℃, which of the
following is its melting point? 6. In an air-conditioning system, before the air is circulated to Answer: D
A. slightly less than 0℃ the required area, it is:
B. slightly more than 0℃ A. cooled 12. The drift loss in cooling towers is about:
C. 0℃ B. filtered A. 1% only
D. 32℃ C. dehumidified B. 12 to 15%
Answer: C D. all of the above C. 10 to 20%
D. 30 to 40%
heat transfer rate of the layers dirt and foreign materials. D. All of these
Answer: C What is this factor?
A. Booster Factor Answer: A
13. Cooling towers are used for cooling water: B. Factor of Safety
A. to be injected in circulating air C. Fouling Factor 24. Fan motors in air conditioning units usually have:
B. to be used for humidification D. Compression Factor A. 2 or 3 speeds
C. to be used for filtration of air B. 3 or 4 speeds
D. to be used for cooling the compressor Answer: C C. 1 or 2 speeds
D. 4 or 5 speeds
Answer: D 19. The engineer was tested to design the air conditioning
system for a ball room dance hall. Considering that this Answer: A
14. By which of the following processes heat mainly involve a lot of activity from its users, the engineer would
dissipates in cooling towers? design that will require: 25. Which of the following capacitors that can usually be
A. conduction A. Maximum attainable effective temperature found in an air conditioning unit?
B. convection B. Constant Effective temperature A. Starting capacitors
C. radiation C. Higher Effective Temperature B. Running capacitors
D. evaporation D. Lower Effective Temperature C. Fan motor capacitors
D. all of these
Answer: D Answer: A
Answer: D
15. Which one is commonly used liquid absorbent? 20. What is the value of the Mach No. throat of the
A. silica gel converging-diverging nozzle? 26. Which of the following troubles commonly occurs inside
B. activated alumina A. Zero an air conditioning unit?
C. ethylene glycol B. Two A. Stuck Compressor
D. any of the above C. One B. Clogged Refrigerant circuit
D. Ten C. Short Circuit, open circuit, grounded motor winding
Answer: C D. All of these
Answer: C
16. In sensible heating cooling following parameter remains Answer: D
unchanged 21. What do bodies at a temperature above absolute zero?
A. dry bulb temperature A. It emits Energy 27. The motor condition of an air conditioning unit can be
B. wet bulb temperature B. It emits Thermal Radiation checked with:
C. relative humidity C. It emits Heat for Conduction A. the continuity of light or with an ohmmeter
D. humidity ratio D. It emits Heat for convection B. the flow of refrigerant in the receiver
C. the discharge pressure in the compressor
Answer: D Answer: B D. All of these

17. Heating and dehumidification can be obtained 22. When air is saturated, the wet bulb depression is: Answer: A
simultaneously if air is passed through: A. zero
A. sprays of water kept at a temperature higher than the dew B. indefinite 28. An air conditioning system wherein the entire systems
point temperature of the entering air C. unity are mounted in the cabinet:
B. a solid absorbent surface D. 100% A. Console Air Conditioners
C. a liquid absorbent spray B. Centralized Air Conditioners
D. any one of B and C Answer: A C. Package Type Air Conditioners
D. Multizone Split Ductless System
Answer: D 23. In a window air conditioning unit which of the following is
usually done by the owner? Answer: A
18. Dirt and foreign materials normally build-up on the side A. Semiannual cleaning or replacement of filters
of the condenser tubes. To ensure adequate condenser B. Annual cleaning of the evaporator, condenser fan blades, 29. The use of water to carry heat occupied spaces
capacity, a certain factor is used in calculating the overall fan motor, compressor and casing A. Hydronic Heating System
heat transfer through the walls of the tubes including the C. Inspection of fan motors and lubricate them B. Water Boiling System
C. Hydrokinetic Heating System 35. The amount of CO can be determined by the color of B. 3℃ to 5℃
D. Hydraulic Heating System Palladium Chloride. An amount of 30 ppm to 70 ppm will C. 4℃ to 5℃
cause: D. 1℃ to 2℃
Answer: A A. Slight Darkening
B. Grey Color Answer: A
30. The method of cooling which primarily used where C. Red Color
ambient air temperatures are high and relative humidity is D. Black Color 41. In an air conditioning unit, the thermostat fails. The unit
used: did not start. How do you test the operation of the
A. Swamp Cooling Answer: A thermostat?
B. Evaporative Cooling A. Cover the air outlet and air inlet with a cloth. The air will
C. Condensate Cooling 36. The term used to express the amount moisture in a given recirculate into the unit and the temperature will quickly drop
D. Hydroionic Cooling sample of air. It is compared with the amount of moisture in to the cut out temperature
a given sample of air. B. Cover the outlet and the air will not circulate and the
Answer: A A. Humidity thermostat functions because no air movement
B. Relative Humidity C. Cover both the inlet and outlet so that the recirculating air
31. The other name for Swamp Cooling is: C. Absolute Humidity has the same temperature
A. Evaporative Cooling D. Humidity Ratio D. Never cover any air passage so that the air can freely
B. Condensate Cooling move and thermostat functions well
C. Wet Roof Cooling Answer: B
D. Excelsior Cooling Answer: A
37. How should the window type air conditioning unit be
Answer: C placed? 42. Which of the following dehumidifier is often used to
A. Slant toward the inside of the home reheat the air after moisture is removed?
32. Evaporative Condenser is used to cool B. Slant toward the outside of the home A. Evaporator
A. condenser vapor C. Level B. Chiller
B. condenser surface D. Slant at approximately 15𝑜 from the horizontal C. Condenser
C. condensate liquid D. Compressor
D. All of these Answer: B
Answer: C
Answer: A 38. A dehumidifier is usually a small hermitic refrigerating
system. It has both a condenser and an evaporator. Many 43. Which of the following refrigerant is added sometimes to
33. A method of cooling which uses water as refrigerant. older systems use R-12 or R-500. The newer units use: other refrigerant to improve oil circulation?
Pressure on the water surface is reduced to lower its boiling A. R-134a A. R-117
temperature. B. R-145a B. R-777
A. Steam Jet Cooling C. R-217a C. R-170 (Ethane)
B. Evaporative Cooling D. R-121a D. R-270
C. Vortex Tube Cooling
D. Pressurized Cooling Answer: A Answer: C

Answer: A 39. The normal cut-out setting of a window unit thermostat is 44. A refrigeration that deals with producing temperature of -
between: 157 ℃ or lower,
34. Palladium Chloride may be used to measure the A. 13℃ to 16℃ A. Low temperature refrigeration
presence of: B. 16℃ to 21℃ B. Extremely low temperature refrigeration
A. vapor C. 17℃ to 22℃ C. Absolute zero refrigeration
B. CO D. 10℃ to 15℃ D. Cryogenics
C. ammonia
D. refrigerant Answer: A Answer: D

Answer: B 40. Thermostat are used with most window units. They have 45. Measurement of a device’s ability to remove atmospheric
differentials which vary between: air from test air.
A. 2℃ to 4℃ A. Atmospheric Dust Spot Efficiency
B. Atmospheric Dry Air Efficiency Answer: A
C. Test Air Efficiency Answer: A
D. Baudelot Air Efficiency 51. Substances that have the ability to absorb moisture from
the air are called: 57. The temperature below which water vapor in the air will
Answer: A A. Desiccants start to condense.
B. Moisturizer A. Condensing Temperature
46. A liquid mixture having constant maximum and minimum C. Dehygroscopic substance B. Dry Bulb Temperature
boiling points. Refrigerants comprising this mixture do not D. Moisture absorber C. Dew Point Temperature
combine chemically, yet the mixture provides constant D. Wet Bulb Temperature
characteristics Answer: A
A. Azeotropic Mixture Answer: C
B. Homogeneous Mixture 52. When the dry bub and the wet bulb temperatures are
C. Conzeotropic Mixture identical, the air is said to be: 58. Which of the following is not used as method to measure
D. Cryogenic Mixture A. saturated air velocities?
B. compressed A. Hot wire Anemometer
Answer: A C. humidified B. Rotating Anemometer
D. dehumidified C. Swinging Vane Velocimeter
47. Refrigerant in Group A1 (R-11). Chemical combination of D. Open type Barometer
carbon, chlorine and fluorine Answer: A
A. Carrene Answer: D
B. Cerrene 53. In what form that water exists in air?
C. CCL group A. Solid 59. It is a form of oxygen photochemically produced in
D. HCL group B. Liquid nature.
C. Vapor A. Ozone
Answer: A D. Saturated B. Oxidation
C. Oxidizing Agent
48. Which of the following refrigerants is popular in the Answer: C D. 𝐷𝑂2
industrial refrigerating system; also popularly absorption
system of refrigerant. 54. When air is heated, what happened to its relative Answer: A
A. R-717 (Ammonia) humidity?
B. R-600 (Butane) A. Increase 60. What is the chemical formula of Ozone?
C. R-611 (Methyl Formate) B. Decrease A. 𝑂3
D. R-504 C. Remain Constant B. 𝑂2
D. May increase or Decrease Depending on temperature C. 𝐷2 𝑂
Answer: A D. 𝑂1
Answer: B
49. Which of the following is a function of air conditioning? Answer: A
A. Temperature and Humidity Control 55. The Horizontal Scale (Abscissa) in the Psychrometric
B. Air, Filtering, Cleaning, and Purification Chart represents: 61. In the upper atmosphere, ozone is made by ultraviolet
C. Air Movement and Circulation A. Dry bulb temperature light reacting with:
D. All of these B. Wet bulb temperature A. Oxygen
C. Relative Humidity B. Hydrogen
Answer: D D. Humidity C. Nitrogen
D. Water Vapor
50. Oxygen is approximately what percent by weight in the Answer: A
atmosphere? Answer: A
A. 23% 56. Most people are comfortable with the relative humidity of:
B. 27% A. 30% to 70% 62. The ozone concentration of 0.10 parts per million (ppm)
C. 77% B. 10% to 40% is generally considered the maximum permissible for how
D. 73% C. 20% to 30% many hours exposure?
D. 40% to 80% A. 8 hrs
B. 4 hrs 68. Why should you avoid bending or twisting of fan blades Answer: A
C. 7 hrs in an air conditioning unit?
D. 3 hrs A. It will cause ice build-up 74. Air delivered to the room from the supply duct, moving at
B. It will wear out the motor bearings and cause noise a velocity of 150 ft/min or more is called:
Answer: A C. It may slice suction line A. Primary Air
D. It decreases the volume flow refrigerant B. Secondary Air
63. How may moisture be removed from air? C. Saturated Air
A. Condensation Answer: B D. Air Turbulence
B. Absorption
C. A and B 69. A type of refrigerant that will not damage the ozone layer Answer: A
D. none of these A. Hydroflourocarbons (HCF’s)
B. Hydrochloroflourocarbons (HCHF’s) 75. Carries needed to deliver air to the conditioned space. It
Answer: C C. R- 22 is made of sheet of metals like aluminum, galvanized sheet
D. R-12 steel and some structural materials that will not burn.
64. In cooling cycle, the dry bulb temperature (db) of the air A. Duct
is lowered. When this happens, the relative humidity Answer: A B. Air Inlet
A. increases C. Air Outlet
B. decreases 70. Which of the following is the type of refrigerant that D. Diffuser
C. remains constant damages Ozone layer?
D. increases or decreases depending on the temperature at A. Hydrochloroflourocarbons (HCHF’s) Answer: A
which it is cooled B. R-12
C. R-22 76. From what principle that air ducts operate?
Answer: A D. All of these A. Principle of Pressure Difference
B. Principle of Temperature Difference
65. What is the effect of superheating the refrigerant? Answer: D C. Forced Draft Fan Principle
A. It increases the Coefficient of Performance D. Principle of Natural Convection
B. It decreases the Coefficient of Performance 71. Large-bulb alcohol thermometer used to measure air
C. It lowers the boiling point of the refrigerant speed or atmospheric condition by means of cooling. Answer: A
D. It increases the suction pressure of the refrigeration A. Kata Thermometer
B. Kelvin Thermometer 77. Which of the following is the common classification of
Answer: A C. JJ Thompson Thermometer ducts?
D. Wet Bulb Thermometer A. Condition-Air Ducts
66. In sensible heating the absolute humidity remains B. Recirculating
constant but the relative humidity: Answer: A C .Fresh-air Ducts
A. increases D. All of these
B. decreases 72. Which of the following components of the window air
C. Remains constant conditioning system must be cleaned annually? Answer: D
D. zero A. Evaporator and Condenser
B. Motor and compressor 78. It is used deliver concentrated airstreams into a room.
Answer: B C. Fan Blades and Fan motor Many have one-way or two-way adjustable air stream
D. All of these deflectors.
67. The relative humidity becomes 100% and where the A. Register
water vapor starts to condense Answer: D B. Grille
A. Dew Point C. Diffuser
B. Cloud Point 73. The phenomenon that warm air rise and cold air settle is D. Damper
C. Saturated Point called:
D. Critical Point A. Stratification Answer: A
B. Sedimentation
Answer: A C. Setting due 79. It is used to control the air-throw distance, height and
D. Ventilation spread, as well as the amount of air.
A. Grille
B. Register 90. Which of the following is to be checked as part of the
C. Diffuser 85. The ratio of the rated cooling capacity divided by the weekly maintenance schedule of a console air conditioners?
D. Damper amount of electrical power used: A. Filters
A. Energy Efficiency Ratio (EER) B. Humidifiers
Answer: A B. Energy Efficiency Index (EE) C. Safety valves
C. Cooling Efficiency (CE) D. Cooling Tower
80. It is used to deliver widespread, fan-shaped flows of air D. Energy Cooling Ratio (ECR)
into the room. Answer: D
A. Grille Answer: A
B. Register 91. A mechanism that removes moisture.
C. Diffuser 86. Heat exchanger in which water flows by gravity over the A. Humidifiers
D. Damper outside of the tubes or plates. B. Dehumidifiers
A. Baudelot Cooler C. Moisturizers
Answer: C B. Free Cooler D. Cooling Towers
C. Newtonian Cooler
81. Which of the following fans in air conditioning systems D. Bourdon Cooler Answer: B
which can be classified as centrifugal flow?
A. Axial fan Answer: A 92. When coil surface temperature is below the dew point of
B. Propeller fan the air.
C. Bi-axial fan 87. A popular air conditioning system that includes a single A. moisture will condense out of the air
D. None of these outdoor condenser, three independent evaporators, and B. vapor will evaporate through the air
individual evaporator temperature control. The condensing C. air is dehumidified
Answer: B unit is located outside on a slab. They are frequently used in D. air is humidified and saturated
legal and medical offices, motels and homes without ducts.
82. The most common controller in the heating and cooling A. Multizone Ductless Split System Answer: A
systems. B. Package Terminal Air Conditioner
A. Thermostat C. Console Air Conditioner 93. Nitrogen occupied almost how much of the Earth’s
B. Pressure gage D. Window type Air Conditioner Atmosphere
C. Barometer A. One-fourth
D. Sling Psychrometer Answer: A B. Three-fourths
C. One-half
Answer: A 88. An air conditioning system which is a combination of D. One-third
heating and cooling system. It is designed to serve an
83. A thermostat that functions as increase or decrease individual room or zone. Answer: B
instead of starting and stopping system is called: A. Multizone Ductless Split System
A. modulate B. Package Terminal Air Conditioner 94. Water in vapor form remains a vapor as long as its
B. heating-cooling the thermostat C. Console Air Conditioner temperature is what relations to the dew point temperature.
C. interlocked D. Window type Air Conditioner A. above
D. compound thermostat B. equals
Answer: B C. below
Answer: A D. almost
89. Which of the following is to be checked as regular
84. A number used to compare energy usage for different monthly maintenance schedule of a console air conditioning Answer: C
areas. It is calculated by dividing the energy consumption by units?
the fottage of the conditioned are. A. Water Leaks 95. An instrument used in measuring air velocity by velocity-
A. Energy Utilization Unit Fan Speeds pressure method
B. Energy Utilization Efficiency C. Cooling Tower A. Pitot Tube
C. Energy Utilization Index D. Duct Dampers, Registers and Diffusers B. Anemometer
D. Energy Utilization Ratio C. Aneroid Barometer
Answer: D D. Flowmeter
Answer: C
Answer: A A. Calcium chloride C. Indeterminate
B. Activated alumina D. Increases
96. What is the specific humidity of dry air? C. Sodium zeolite
A. 100 D. Silica gel Answer: A
B. 20
C. 0 Answer: A 107. Past ME Board Question
D. 50 What amount of air is required in a low bypass factor?
Answer: C 102. All the following temperatures have meaning in A. Greater
psychrometrics excepts: B. Lesser
97. When hot soup was served in a cup during dinner, an A. dry-bulb temperature C. Indeterminate
engineer was so eager to drink it. Since it was hot, he added B. wet-bulb temperature D. Constant
ice cubes of ice to cool the soup stirred it. He noticed that C. adiabatic wall temperature
dew starts to form on the outermost surface of the cup. He D. dew point Answer: B
wanted to check the temperature of the outermost surface of
the cup. What is the temperature equal to? Answer: C 108. Past ME Board Question
A. superheated temperature The design of an air supply duct of an air conditioning
B. equal to zero 103. The relative humidity is given by the: system _______.
C. standard temperature A. ratio of the actual humidity to the saturated humidity at the A. Adds moisture to the air
D. equal to air’s dew point temperature same temperature and pressure B. Lowers the temperature of the air
B. ratio of the partial pressure of water vapor to the C. Does not affect the moisture of air
Answer: D saturation pressure D. Affects the distribution
C. ratio of wet-bulb to dry-bulb temperature
98. The temperature at which the water vapor in the gas D. ratio of dry-bulb temperature to dew point Answer: D
begins to condense in a constant pressure process.
A. Dew point Answer: B 109. Past ME Board Question
B. Vapor point What is the lowest temperature to which water could
C. Flue point 104. The determination of properties and behavior of possibly be cooled in a cooling tower?
D. Gas point atmospheric air usually the purview of: A. The effective temperature
A. thermodynamics B. The temperature of adiabatic compression
Answer: A B. psychrometrics C. The wet bulb depression
C. forced convection D. The dew point temperature of the air
99. All the following processes can be found on a D. Kirchoff’s law
psychrometric chart except: Answer: B
A. humidification Answer: B
B. sensible heating 110. Past ME Board Question
C. natural convection 105. Past ME Board Question Combined process of cooling and humidifying is also known
D. sensible cooling Which of the following statements is correct? as:
The relative humidity of an air water vapor mixture A. Heating and humidifying
Answer: C A. Is the ratio of the partial pressure of the water B. Cooling tower
vapor to the partial pressure of the air C. Evaporative cooling process
100. All of the following process can be found on a B. Indicates the mass of water vapor in the mixture D. Moisture removal process
psychrometric chart except: C. Is equal to the mole fraction of water vapor in the
A. heating and humidifying mixture at the mixture temperature Answer: C
B. cooling and dehumidification D. Is the ratio of the partial pressure of water vapor to
C. black body radiation the saturation pressure at the mixture temperature 111. Past ME Board Question
D. evaporative cooling Answer: D In a cooling tower, the water is cooled mainly by:
A. Condensation
Answer: C 106. Past ME Board Question B. Evaporation
In sensible cooling process, moisture content C. Convection
101. Which is not commonly used to cool and dehumidify A. Does not change D. Conduction
equipment? B. Decreases
Answer: C Answer: B D. Equal to air temperature

112. Past ME Board Question 117. Past ME Board Question Answer: C


Which of the following types of air dryers works by absorbing The relative humidity becomes 100% and where the water
moisture on a solid dessicant or drying material such as vapor starts to condense. CHAPTER 16-MACHINE FOUNDATION AND CHIMNEY
activated alumina, silicon gel, or molecular sieve? A. Critical point 1. Past ME Board Question
A. Regenerative dryer B. Saturated point Foundations are preferably built of concrete in the proportion
B. Deliquescent dryer C. Dew point of what measure of Portland cement; sand; crushed stones?
C. Spray dryer D. Cloud point A. 1 : 2 : 5
D. Refrigerated dryer B. 2 : 4 : 6
Answer: C C. 2 : 3 : 5
Answer: B D. 1 : 2 : 4
118. Past ME Board Question
113. Past ME Board Question What is the specific humidity of dry air? Answer: D
The relationship of water vapor in the air at the dew point A. 150
temperature to the amount that would be in the air if the air B. 100 2. Past ME Board Question
were saturated at the dry bulb temperature is: C. 50 For design stability, the center of gravity of the total
A. Partial pressure actual at dew point D. 0 combined engine, driven equipment and foundation should
B. Percentage humidity be kept _________.
C. Relative humidity Answer: D A. Anywhere
D. Partial pressure of water B. Above the foundation top
119. Past ME Board Question C. In line with surface of the foundation
Answer: C An engineer inspected an air-conditioning unit. He found out D. Below the foundation top
that the unit does not produce any cooling effect, however,
114. Past ME Board Question the air-conditioning unit is running. He checked the Answer: D
When the air is saturated, the wet bulb depression is: temperatures of the condenser and evaporator and had the
A. Zero unit run. He found out that there was no change in 3. The machine foundation must have a factor of safety of
B. Indefinite temperature. What should he do? A. 4
C. Unity A. Replace fuse B. 5
D. 100% B. Charge with new refrigerant C. 6
C. Replace relay D. 7
Answer: A D. Adjust door seal
Answer: B
115. Past ME Board Question Answer: B
A temperature measurement in an ordinary thermometer 4. The diesel engine foundation safe soil bearing pressure is:
which has constant specific humidity 120. Past ME Board Question A. 4,890 kg/𝑐𝑚2
A. Critical temperature What is the temperature range of air in air conditioning B. 4,500 kg/𝑐𝑚2
B. Dew point temperature application where the dry air can be considered ideal gas? C. 4,490 kg/𝑐𝑚2
C. Dry bulb temperature A. 100 to 125 ℃ D. 4,125 kg/𝑐𝑚2
D. Wet bulb temperature B. 50 to 75 ℃
C. 75 to 100 ℃ Answer: A
Answer: B D. -10 to 50 ℃
5. Anchor bolts in a machine foundation should be
116. Past ME Board Question Answer: D embedded in concrete of at least how many times the bolt
During sensible heating, the humidity remains constant but diameter?
the relative humidity. 121. Past ME Board Question A. 12
A. Increases What is the value of air stratification in air conditioning B. 20
B. Decreases design fit for human comfort? C. 30
C. Remains constant A. Minimum D. 25
D. Zero B. Maximum
C. Less than air temperature Answer: C
11. For stability, the total combined engine, driven C. 5
6. As a good practical rule, the foundation depth may be equipment and foundation center of gravity must be kept D. 6
taken as how many times of the engine stroke? _____.
A. 2.2 to 3.2 A. below the foundation top Answer: A
B. 3.2 to 4.2 B. above the foundation top
C. 2.5 to 3.5 C. 120 mm above the foundation top 17. For a diesel engine’s foundation, in pouring a concrete
D. 3.5 to 4.5 D. none of the above mixture, it should be poured:
A. one time
Answer: B Answer: A B. two times
C. three times
7. To secure the belts embedded within the foundation, the 12. There shall be no foundation bolts less than ____. D. four times
distance of the edges of the foundation from the bedplate A. 12 mm in diameter
must be _________. B. 16 mm in diameter Answer: A
A. 120 mm to 300 mm C. 18 mm in diameter
B. 150 mm to 330 mm D. 20 mm in diameter 18. For foundations of steam turbine, the concrete mixture
C. 100 mm to 280 mm should be _________.
D. 200 mm to 380 mm Answer: A A. 1: 2: 4
B. 1: 3: 5
Answer: A 13. The weight of steel bar reinforcements should be how C. 1: 2: 3
many times the weight of the foundation? D. 1: 3: 4
8. The weight of the machine foundation is how many times A. ½% to 1%
of the weight of the engine? B. 1% to 1.5% Answer: A
A. 4 to 5 times C. 3% to 5%
B. 3 to 5 times D. 3.2% to 4.2% 19. The steam turbine foundation should be designed to
C. 2 to 3 times support the machine load plus how many percent of the
D. 4 to 6 times Answer: A impact, condenser load, floor loads and dead loads?
A. 15
Answer: B 14. Concrete foundation should have steel bar B. 20
reinforcements placed vertically and horizontally to avoid C. 25
9. To eliminate transmission of vibration, the foundation ____. D. 30
should be isolated from floor slabs or building footings at A. breaking
least how many mm around its perimeter? B. thermal cracking Answer: C
A. 15 C. melting
B. 20 D. vibration 20. Which of the following draft rely on the stack effect to
C. 25 draw off combustion gases?
D. 30 Answer: B A. Natural draft
B. Forced draft
Answer: C 15. Foundations bolt length should be at least how many C. Induced draft
times the anchor bolt diameter? D. Balanced draft
10. The minimum vertical distance from the floor or soil level A. 16 times
to the top edge of the foundation must be around B. 18 times Answer: A
_________. C. 20 times
A. 150 mm D. 30 times 21. Fans located before the furnace that are used to supply
B. 100 mm air for burning.
C. 200 mm Answer: B A. Natural draft
D. 120 mm B. Forced draft
16. Foundation bolts of specified size should be used and C. Induced draft
Answer: D surrounded by a pipe sleeve with an inside diameter of at D. Balanced draft
least how many times the diameter of the anchor bolt?
A. 3 Answer: B
B. 4
22. Forced draft fans are run at relative high speed in the D. absolute pressure D. 30 m
range of:
A. 1200 to 1800 rpm Answer: A Answer: A
B. 1000 to 1600 rpm
C. 1500 to 2000 rpm 28. The static pressure drop due to friction through the boiler 34. Most stacks are built of height less than:
D. 900 to 1500 rpm and stack. A. 60 m
A. draft loss B. 50 m
Answer: A B. available draft C. 40 m
C. stack effect D. 30 m
23. Chimneys that rely on natural draft are sometimes D. fan boost
referred to as Answer: D
A. natural chimney Answer: A
B. gravity chimney 35. The average flue gas temperature is
C. normal chimney 29. The difference between the theoretical draft and the draft A. the temperature entering the stack
D. stack loss. B. the temperature leaving the stack
A. draft loss C. the temperature halfway up the stack
Answer: B B. available draft D. the sum of the temperatures
C. stack effect
24. Force draft fans create a positive pressure of: D. fan boost Answer: C
A. 0.5 to 2.5 kPa
B. 1.0 to 3.0 kPa Answer: B
C. 2.0 to 4.0 kPa 36. The coefficient of velocity is approximately
D. 0.2 to 2.2 kPa 30. In a balanced system, the available draft is: A. 0.30 to 0.50
A. unity B. 0.40 to 0.60
Answer: A B. zero C. 0.20 to 0.40
C. 100 D. 0.50 to 0.70
25. Fans that are used to draw combustion products through D. infinite
the furnace bed, stack, and pollution control system by Answer: A
injecting air into the stack after combustion. Answer: B
A. Natural draft 37. For realistic problems, the achievable stack effect
B. Forced draft 31. The total pressure supplied by the fan at maximum probably should be considered to be:
C. Induced draft operating conditions. A. 75% of the ideal
D. Balanced draft A. net rating or fan boost B. 80% of the ideal
B. draft loss C. equal to the ideal
Answer: C C. available draft D. half the ideal
D. stack effect
26. The term used when the static pressure is equal to the Answer: B
atmospheric pressure. Answer: A
A. Natural draft 38. Guyed stacks seldom exceed:
B. Forced draft 32. Generally the higher the chimney A. 1.83 m in diameter
C. Induced draft A. the smaller the stack effect B. 2.83 m in diameter
D. Balanced draft B. the greater the stack effect C. 1.45 m in diameter
C. stack effect is zero D. 2.45 m in diameter
Answer: D D. the stack effect approaches unity
Answer: A
27. In order to keep combustion products inside the Answer: B
combustion chamber and stack system, balanced draft 39. The angle between the stack and the guy wire is usually:
system may actually operate with a slight 33. Modern stacks are seldom built higher than A. 30 degrees
A. negative pressure A. 60 m B. 45 degrees
B. positive pressure B. 50 m C. 60 degrees
C. positive and negative pressures C. 40 m D. 75 degrees
45. Foundation bolts of specified size should be used and
Answer: C surrounded by a pipe sleeve with a length of at least how Answer: C
many times the diameter of the bolt?
40. Guyed stacks height seldom exceeds: A. 18
A. 30.48 m B. 14
B. 34.80 m C. 15 CHAPTER 17 – INSTRUMENTATION
C. 43.80 m D. 16 1. An instrument used for measuring evaporation, generally
D. 48.30 m that of water, into the atmosphere.
Answer: A A. Aerometer
Answer: A B. Atmometer
46. Foundation should be designed to support the machine C. Anemometer
41. The maximum unit pressure of turbine and generator on load plus how many percent of the impact, condenser load, D. Fadometer
the reinforced concrete should not exceed floor loads and dead loads?
A. 17.62 kg/𝑐𝑚2 A. 25 Answer: B
B. 18.62 kg/𝑐𝑚2 B. 34
C. 16.62 kg/𝑐𝑚2 C. 15 2. An instrument for testing the resistance of materials to
D. 19.62 kg/𝑐𝑚2 D. 36 fading when they are exposed to artificial sunlight or
ultraviolet light under controlled conditions.
Answer: A Answer: A A. Fadometer
B. Fulgurator
42. Foundations should be isolated from floor slabs or 47. In guyed steel stacks, the angel between wires in a set of C. Odometer
building footings by at least how many mm to eliminate three is: D. Atmometer
transmission of vibration. A. 120 deg.
A. 25 mm B. 110 deg. Answer: A
B. 15 mm C. 135 deg.
C. 35 mm D. 145 deg. 3. The type of calorimeter in which the specific heat of a
D. 45 mm specimen is measured by the quantity of ice it melts is the:
Answer: A A. ice calorimeter
Answer: A B. heat calorimeter
48. Guys are usually applied in how many sets? C. vapor calorimeter
43. Concrete foundations should have steel bar A. 1 to 3 D. hysometer
reinforcements placed both vertically and horizontally to B. 2 to 4
avoid C. 3 to 5 Answer: A
A. vibrations D. 4 to 6
B. thermal cracking 4. An instrument used for determination of the boiling point
C. breakdown Answer: A of water for estimating the altitude above sea level from its
D. rupture known variation with atmospheric pressure.
49. The empirical coefficient e in machine foundation if not A. Vapor calorimeter
Answer: B given is assumed B. Hysometer
A. 0.11 C. Boiler
44. Foundation bolts of specified size should be used and B. 0.25 D. Vapor bomb
surrounded by a pipe sleeve with an inside diameter of at C. 0.32
least how many times the diameter of the anchor bolt? D. 0.15 Answer: B
A. 3
B. 4 Answer: A 5. A unit of work or energy equal to 107 ergs. It is equivalent
C. 5 to the work done by a force of one Newton acting through a
D. 6 50. The term suction units is used with distance of one meter.
A. Natural draft A. Btu
Answer: A B. Forced draft B. Calorie
C. Induced draft C. Joule
D. Balanced draft D. Watt
C. Gas mixer
Answer: C D. Aerator 17. An instrument used for measuring various quantities at a
distance.
6. An electromagnetic device for separating isotopes by Answer: D A. Tachometer
electrical sorting of their ions. B. Odometer
A. Ion sorter 12. A device that supplies air to an air-injection system is C. DME
B. Isotopes sorter called D. Telemeter
C. Isotron A. air pump
D. Jolly balance B. air nozzle Answer: D
C. gas scrubber
Answer: C D. air injector 18. An apparatus used for determination of the transition
point by measuring the temperature at which the vapor
7. A device used for the determination of the specific gravity Answer: B pressure of the two modifications become equal.
of a substance by weighing it in air and in water. A. Tensiometer
A. Kelvin balance 13. An instrument used for measuring pressure, flow velocity B. Tensimeter
B. Spring balance and discharge rate of a fluid flowing in a conduit. C. Tachometer
C. Hysometer A. Kampometer D. Bourdon gage
D. Jolly balance B. Flowmeter
C. Pitot tube Answer: B
Answer: D D. Katharometer
19. An instrument for measuring the revolutions per minute
8. Which of the following instruments is NOT used to Answer: B of a rotating shaft.
measure flow rates? A. Tachometer
A. Anemometer 14. An instrument used for measuring radiant energy B. Odometer
B. Rotameter especially in the thermal region. C. All of the choices
C. Flowmeter A. Kampometer D. Speedometer
D. Velometer B. Luxmeter
C. Kapnometer Answer: A
Answer: D D. Katharometer
20. A device used for demonstrating the extreme force
9. An instrument used to measure the quality of steam is the: Answer: A exerted by water when if freezes is called:
A. psychometer A. Ice calorimeter
B. gas calorimeter 15. An instrument for determination of the composition of the B. Hysometer
C. pyrometer gas mixture by measuring variations in the thermal C. Ice bomb
D. steam calorimeter conductivity. D. Freezer
A. Kampometer
Answer: D B. Atmometer Answer: C
C. Kapnometer
10. An instrument used to measure the density of gases is D. Katharometer 21. A device to measure vertical distances or
known as: displacements?
A. aerovane Answer: D A. Kampometer
B. aerometer B. Kapnometer
C. hygrometer 16. An apparatus used for measuring the surface tension of C. Altimeter
D. atmometer a liquid by recording the force necessary to detach a metal D. Cathetometer
ring from the surface.
Answer: A A. Tachometer Answer: D
B. Tensiometer
11. Which of the following is the apparatus used for mixing C. Odometer 22. A precision instrument for measuring very short time
air with other fluids? D. Telemeter intervals.
A. Carburetor A. Clinometer
B. Dehydrator Answer: B B. Chronoscope
C. Stop watch What is the clockwork operated device which records
D. Decremeter continuously the humidity of the atmosphere? 33. Past ME Board Question
A. Hetrograph What is the function of a steam separator?
Answer: B B. Hygrometer A. Trapping the steam and letting water through
C. Hygrodeik B. Throttling
23. Which of the following is a laboratory method for the D. Hygrograph C. Changing the direction of the steam flow
determination of the boiling range of petroleum? D. Steam metering
A. Jolly balance Answer: D
B. Kelvin balance Answer: A
C. Engler distillation 29. Past ME Board Question
D. Fractional distillation A device whose function is to pass an information in an 34. Past ME Board Question
unchanged form or in some modified form. A salimeter reads the:
Answer: C A. Relay A. Density of brine
B. Sensor B. Dew point temperature
24. An instrument used to measure the humidity in the air. C. Transmitter C. Rate of brine
A. Hygrometer D. Transducer D. Relative humidity
B. Hydrometer
C. Pressure gauge Answer: A Answer: A
D. Wind vane
30. Past ME Board Question 35. Past ME Board Question
Answer: A A general term for a device that receives information in the Which of following measures the density of salt in the water?
form of one or more physical quantities, modifies the A. Salimeter
25. An instrument used to measure the specific gravity or information and or its form if required and produces a B. Pitot tube
relative density of liquids. resultant output signal. C. Hydrometer
A. Hygrometer A. Converter D. Odometer
B. Hydrometer B. Transducer
C. U – tube C. Sensor Answer: A
D. Manometer D. Scanner

Answer: B Answer: B CHAPTER 18 – BASIC ELECTRICAL ENGINEERING


1. Ebb current refers to:
26. An apparatus used for the determination of the 31. Past ME Board Question A. the eddy current
concentration of solid or liquid particles dispersed in a gas, An engine indicator is generally used to measure what? B. the movement of the tidal current away from shore or
such as the density of smoke in fog. A. Steam temperature down a tidal stream
A. Kampometer B. Heat loss C. the removal by screen of undesirable fine materials from
B. Atmometer C. Steam cylinder pressure broken ore
C. Kapnometer D. Gauge reading errors D. none of the above
D. Katharometer
Answer: C Answer: D
Answer: C
32. Past ME Board Question 2. In power station practice “spinning reverse” is
27. Past ME Board Question In the processing section, there is an instrument frequently A. reverse generating capacity that is in operation but not in
Hydrometer is used to find out: used to measure the flow rate of fluids. What is the service
A. Specific gravity of liquids instrument consisting of a vertical passage with variable B. reserve generating capacity that is connected to bus and
B. Specific gravity of solids cross-sectional area, a float and a calibrated scale? ready to take load
C. Specific gravity of gases A. Rotameter C. reserve generating capacity that is available for service
D. Relative humidity B. Pitot tube but not in operation
C. Rota Aire D. capacity of the part of plant remains under maintenance
Answer: A D. Manometer
Answer: B
28. Past ME Board Question Answer: A
3. Turnaround efficiency of battery energy storage system is 15. Which of the following plant has least pollution problem?
about 9. The string efficiency in an AC system is: A. Steam power plant
A. 75 percent A. 50.75% B. Gas turbine power plant
B. 40 percent B. 100% C. Nuclear power plant
C. 25 percent C. 25.50% D. Hydro-electric power plant
D. 80 percent D. 70.7%
Answer: D
Answer: A Answer: D
16. Which of the following power plant can be installed within
4. Most of the generators in thermal power plants run at: 10. Satellites are powered by the shortest time?
A. 15000 rpm A. solar cells A. Nuclear power plant
B. 1500 rpm B. thermo-electric generators B. Hydro-electric power plant
C. 750 rpm C. thermionic converters C. Gas turbine power plant
D. 3000 rpm D. photo-electric cells D. Diesel engine plant

Answer: B Answer: A Answer: D

5. The power supplied to domestic consumers is at 220 11. A ship is powered by: 17. Which of the following item consumes least power?
volts. This represents: A. steam turbine A. Toaster
A. mean value of voltage B. diesel engines B. Desert cooler
B. seak value of voltage C. steam turbines or diesel engine C. Electric shaver
C. RMS value of voltage D. steam turbines, diesel engines, hydraulic turbines or D. Electric iron
D. none of the above nuclear reactors
Answer: C
Answer: C Answer: C
18. Which of the following needs highest level of
6. The conductors that are used for transmitting bulk of 12. Which of the following plant is suitable for peak load? illumination?
power at high voltage are of which of the following type? A. Diesel engine plant A. Proof reading
A. Cadmium copper B. Steam power plant B. Hospital wards
B. Galvanized steel C. Nuclear power plat C. Railway platforms
C. Any of the above D. All of the above D. Foyer for cinema halls
D. Copper
Answer: A Answer: A
Answer: D
13. The term ‘critical’ is associated with power plant of the 19. The lamp that is used for cinema projectors is a:
7. When a conductor is suspended between two supports at type: A. frosted GLS lamp
the same level its shape becomes: A. steam power plant B. nitrogen filled GLS lamp
A. catenary B. diesel engine power plant C. mercury vapor lamp
B. cycloid C. gas turbine power plant D. carbon arc lamp
C. semicircle D. nuclear power plant
D. parabola Answer: D
Answer: D
Answer: A 20. In a fluorescent tube circuit, choke acts as
14. A turbocharger is a part of: A. a starter
8. Sag is provided in overhead transmission lines so that: A. thermal power plant B. a device for improving the power factor
A. repairs can be carried out easily B. diesel engine power plant C. a current limiting device
B. corona can be reduced C. hydro-electric power plant D. a source of heat
C. skin effect can be reduced D. nuclear power plant
D. safe tension is not exceeded Answer: B
Answer: B
Answer: D 21. Transient disturbances are due to:
A. switching operations 27. The statement that the product of the error in the D. 80 percent
B. load variations measured determination of a particle’s position and its
C. faults momentum is of the order Planck’s constant “h” is known as Answer: A
D. any of the above A. Bohr’s theory
B. D’Alembert’s paradox 33. The conductors that are used for transmitting bulk of
Answer: D C. the Heisenberg uncertainty principle power at high voltage are of which of the following type?
D. Planck’s law A. Cadmium copper
22. Bottoming cycle is quite common in: B. Galvanized steel
A. cement plants Answer: C C. Any of the above
B. sugar mills D. Copper
C. paper mills 28. The phenomenon of physical adhesion of molecules of
D. all of the above the surfaces of solids without chemical reaction is Answer: D
A. dredging
Answer: A B. adsorption 34. Corona generally results in
C. coking A. violet glow
23. Which of the following is the most highly developed D. liquation B. hissing noise
device for confining plasma with magnetic field? C. production of ozone gas
A. Tokamak Answer: B D. all of the above
B. Tomahawk
C. Breeder reactor 29. Fuses and circuit breakers do not protect electric motors Answer: D
D. Cyclotron from:
A. short circuiting 35. Which of the following affects corona?
Answer: A B. motor burnout A. Conductor size
C. overload B. Spacing between conductors
24. Fuses and circuit breakers do not protect electric motors D. overheating C. Line voltage
from: D. All of the above
A. short circuiting Answer: D
B. motor burnout Answer: D
C. overload 30. In which of the following systems where Betz law is
D. overheating widely used? 36. When a conductor is suspended between two supports
A. MHD system at the same level its shape becomes
Answer: D B. solar cells A. catenary
C. geothermal power plants B. cycloid
25. Corona generally results in: D. wind mills C. semicircle
A. violet glow D. parabola
B. hissing noise Answer: D
C. production of ozone gas Answer: A
D. all of the above 31. At what rpm where most of the generators in thermal
power plants run? 37. Past ME Board Question
Answer: D A. 15000 rpm What is the unit of electromagnetic wave frequency?
B. 1500 rpm A. Volts
26. Which of the following affects corona? C. 750 rpm B. Horsepower
A. Conductor size D. 3000 rpm C. Hertz
B. Spacing between conductors D. Knot
C. Line voltage Answer: B
D. All of the above Answer: C
32. Turnaround efficiency of battery energy storage system
Answer: D is about CHAPTER 19 – LATEST BOARD QUESTION
A. 75 percent 1. Past ME Board Question
B. 40 percent
C. 25 percent
A pump with capacity 𝑄1 and head 𝐻1 is connected in series Where does the final removal of water vapor in an 11. Past ME Board Question
with another pump with capacity 𝑄2 and head 𝐻2 . What is the absorption refrigeration system occur? A multi-stage refrigeration system which is realizable in
head developed if 𝑄2 is less than 𝑄1 ? A. Analyzer using different refrigerants
A. 2𝐻2 𝐻1 B. Generator A. Direct
B. 𝐻2 + 𝐻1 C. Condenser B. Cascade
C. 𝐻2 / 𝐻1 D. Rectifier C. Flooded
D. 𝐻2 𝐻1 D. Vapor
Answer: D
Answer: B Answer: B
7. Past ME Board Question
2. Past ME Board Question If the average temperature of liquid of power cycle during 12. Past ME Board Question
At ordinary temperature what substance will behave as inert heat addition process is as high as possible, then the Very large pipe diameter means a thicker wall and high cost.
gas and will not react in the combustion process? thermal efficiency of the cycle will: It also means
A. Nitrogen A. Increase A. Zero pressure drop
B. Sulfur B. Remain constant B. High pressure drop
C. Hydrogen C. Decrease C. Constant temperature
D. Carbon D. Zero D. Low pressure drop

Answer: A Answer: A Answer: B

3. Past ME Board Question 8. Past ME Board Question 13. Past ME Board Question
Which of the following cycles is ideal for spark-ignition What is the temperature of lubricant at which it will ignite Aside from maintaining appropriate temperature for food cold
reciprocating engine? thus limits its application? storage, how is desiccation minimized or decreased?
A. Diesel cycle A. Cold point A. Low oxygen
B. Dual cycle B. Burning point B. Maintain humidity ratio
C. Rankine cycle C. Flash point C. Low air circulation
D. Otto cycle D. Use point D. Increase humidity ratio

Answer: D Answer: C Answer: B

4. Past ME Board Question 9. Past ME Board Question 14. Past ME Board Question
What is the relative humidity when the dew point and dry What is the heat that is removed from the space to be During combustion process, only limited air is supplied and
bulb temperature are equal? cooled, which is the same as the heat absorbed by the not enough to supply two molecule of oxygen per one
A. 110% cooling coils? molecule of carbon, then the product is:
B. 0% A. Heating capacity A. Hydration
C. 100% B. Enthalpy B. Carbon monoxide
D. 50% C. Work compression C. Carbonic acid
D. Refrigerating effect D. Carbon dioxide
Answer: C
Answer: D Answer: B
5. Past ME Board Question
Reheating process is normally employed in steam plane 10. Past ME Board Question 15. Past ME Board Question
when: An air conditioning system in which water is chilled or cooled As far as combustion chamber design is considered, the
A. There is excess steam and which passes the evaporator coils. maximum power output of a given engine can be increased
B. Subcooling is required A. Chilled water system by:
C. Turbine undergoes excessive moisture B. Direct expansion A. Increasing combustion time
D. Dehumidifying is required C. Absorption B. Increasing combustion chamber volume
D. Water circulation system C. Decreasing combustion chamber volume
Answer: C D. Providing small values
Answer: A
6. Past ME Board Question Answer: C
A. Total energy of an open system If Pi is the indicated horsepower and Pb is the indicated
16. Past ME Board Question B. Temperature of an open system horsepower of a compressor, then what is mechanical
A water conservation device which employs both air and C. Total energy of a closed system efficiency, Em, equal to:
water is condensed a condenser and cooling tower D. Temperature of a closed system A. Em = Pb / Pi
combined into one is called: B. Em = Pi / Pb
A. Shell and tube condenser Answer: D C. Em = Pb – Pi
B. Air-cooled condenser D. Em = Pi – Pb
C. Water-cooled condenser 21. Past ME Board Question
D. Evaporative condenser In order to attain equilibrium in the condenser, the cooling Answer: B
tower range must be _______ with respect to temperature
Answer: D rise in the condenser. 26. Past ME Board Question
A. Greater What is the least number of compressors a multistage
17. Past ME Board Question B. Equal system that will use?
In order to perform efficiently, a power cycle must be C. Less A. Three
communicated with the outside temperature of its D. Zero B. Two
surroundings and into one is called: C. Four
A. Kirchoff’s Law Answer: B D. One
B. Second Law of Thermodynamics
C. Kelvin-Planck Law 22. Past ME Board Question Answer: B
D. Frist Law of Thermodynamics If ice will form in a solution of water and salt, then it is at a
temperature called: 27. Past ME Board Question
Answer: B A. Dew point The relation between the Fahrenheit absolute scale and the
B. Freezing point depression Celsius absolute scale is:
18. Past ME Board Question C. Boiling point depression A. R = 1.8 K
Water is heated in a container. It expands and becomes less D. Critical point B. R = 32 K
dense and lighter. It rises up the container because of its C. R = 3.2 K
reduced density and replaced by cooler air. As this process Answer: B D. R = 18 K
continues, the heat is transferred and disturbed all
throughout. What is this mode of heat transfer called? 23. Past ME Board Question Answer: A
A. Condenser The flow of the convergent section of a nozzle is always
B. Radiation subsonic. If the flow is subsonic then the mach number is: 28. Past ME Board Question
C. Conduction A. Greater than unity The sum of the internal energy and the product of pressure
D. Convection B. Less than unity and specific volume is known as the:
C. Near than unity A. Enthalpy
Answer: D D. Unity B. Entropy
C. Total work
19. Past ME Board Question Answer: B D. Total internal energy
When required, a regulator water valve in refrigerating
system should be 24. Past ME Board Question Answer: A
A. On the suction line on compressor line One hundred twenty percent theoretical air was supplied for
B. In the water inlet combustion. What is the equivalent excess equal to: 29. Past ME Board Question
C. Anywhere in the system A. 20% A refrigeration control that guards the compressor from
D. In the water outlet B. 100% overloads brought about by abruptly increases loads
C. 240% resulting from defrosting, warm products and others, is
Answer: D D. 120% called:
A. Safety valve
Answer: A B. Suction hold-back valve
20. Past ME Board Question C. Solenoid valve
When there is no work between the thermodynamic system 25. Past ME Board Question D. Expansion valve
and its surrounding, the quantity of net heat transfer is equal
to: Answer: B
Answer: C A. 5”
30. Past ME Board Question B. 6”
What does fossil-fuel fired power plants release, which in 35. Past ME Board Question C. 3”
turn produces the key ingredient in acid rain? The kinetic energy of a moving fluid is used to isentropically D. 4”
A. Nitrogen compressed the fluid to state of zero velocity. The
B. Sulfur emission temperature of a moving fluid at the state zero velocity is Answer: D
C. Carbon monoxide called:
D. Carbon dioxide A. Stagnation temperature 40. Past ME Board Question
B. Partial temperature One foot water is equal to:
Answer: B C. Critical temperature A. 0.4138 lb/𝑖𝑛2
D. Absolute temperature B. 68.3 lb/𝑓𝑡 2
31. Past ME Board Question C. 0.8673 lb/𝑖𝑛2
The fraction of the radiation energy incident on a surface Answer: A D. 62.43 lb/𝑓𝑡 2
which is absorbed by the surface is called:
A. Convection 36. Past ME Board Question Answer: D
B. Absorptivity The dew point temperature of the products of combustion in
C. Emission the saturation temperature that corresponds to the partial 41. Past ME Board Question
D. Radiation pressure of the _____ in the products. The freezing point of R – 22 is:
A. 𝑆𝑂2 A. -109 ℉
Answer: B B. 𝐻2 𝑂 B. -252 ℉
C. 𝑁2 C. -256 ℉
32. Past ME Board Question D. 𝐶𝑂2 D. -211 ℉
Which of the following power plants uses energy from
uranium to produce electric power? Answer: B Answer: C
A. Diesel Plant
B. Geothermal Plant 37. Past ME Board Question 42. Past ME Board Question
C. Nuclear Plant What do we call a device that is used to boiler operation that Which of the following is a type of evaporator?
D. Hydroelectric Plant will stop the burner or at least send a signal to the operator if A. Shell-and-tube condenser
the water drops to a low level that is no longer safe? B. Shell-and-tube water cooler
Answer: C A. Safety valve C. Oil lantern rings
B. Dead-weight valve D. Polyphase motors
33. Past ME Board Question C. High water cut off
Ice cubes added to a glass of water and stirred. Moisture D. Low water cut off Answer: A
starts to form on the outer surface of the glass. At this point,
what is the temperature at the outer surface called? Answer: D 43. Past ME Board Question
A. Critical temperature For Brayton cycle, the result of regeneration is:
B. Surface temperature 38. Past ME Board Question A. Increase in thermal efficiency
C. Dew point temperature Dry analysis is a fractional analysis of the products of B. Decrease thermal efficiency
D. Saturation temperature combustion which does not include: C. Moderate thermal efficiency
A. Water vapor D. Low thermal efficiency
Answer: C B. Carbon dioxide
C. Sulfur dioxide Answer: A
34. Past ME Board Question D. Carbon monoxide
The force when applied to a mass of one kilogram will give 44. Past ME Board Question
mass an acceleration of one meter per second for every Answer: A If the pressure exerted on a liquid is higher than the
second called: saturation corresponding to its temperature. The liquid is a:
A. Watt A. Saturated liquid
B. Joule B. Subcooled liquid
C. Newton 39. Past ME Board Question C. Superheated liquid
D. Pascal The size of a reciprocating pump is stamped on the builder’s D. Highly superheated liquid
plate 3” x 4” x 6”. The diameter of the liquid cylinder is:
Answer: B Answer: C
50. Past ME Board Question
45. Past ME Board Question What does a negative Joule-Thompson coefficient means 55. Past ME Board Question
In a power driven pump, each piston stroke is displaced by during a throttling process? What is the effect on saturated temperature if the pressure of
360𝑜 divided the ______. A. Fluid pressure is constant the fluid is decreased?
A. Revolution per minute B. Fluid temperature drops A. There is no effect
B. Bore C. Fluid temperature rises B. Saturation temperature decreases
C. Length of the stroke D. Fluid pressure zero C. Saturation temperature remains constant
D. Number of cylinders D. Saturation temperature increases
Answer: C
Answer: D Answer: B
51. Past ME Board Question
46. Past ME Board Question For steam power plants, increasing the operating pressure of 56. Past ME Board Question
If the air is compressed without discarding heat, then what the boiler will: If V3 is the cylinder volume after the combustion process of
do you call this kind of compression? A. Increase thermal efficiency a Diesel cycle and V2 is its cylinder volume before
A. Isobaric B. Decrease thermal efficiency combustion, then calculate the cut-off ratio, r.
B. Isochoric C. Bring thermal efficiency to zero A. r = V3 – V2
C. Adiabatic D. Make thermal efficiency constant B. r = V2 – V3
D. Isothermal C. r = V2 / V3
Answer: A D. r = V3 / V2
Answer: C
47. Past ME Board Question 52. Past ME Board Question Answer: D
If a gas possesses internal energy, then it is due to its: The ratio of the average load to that of the rated capacity of
A. Velocity a plant is called: 57. Past ME Board Question
B. Pressure and volume values A. Output factor Compare the temperature of discharge vapor refrigerant
C. Height from a certain datum level B. Load factor leaving the compressor for a superheated cycle and
D. Molecular motion C. Demand factor saturated cycle, for the same condensing temperature and
D. Capacity factor pressure.
Answer: D A. There is no difference
Answer: D B. Higher for the superheated cycle
48. Past ME Board Question C. Higher for the saturated cycle
The pressure of ammonia was detected in brine solution. 53. Past ME Board Question D. Lower for the superheated cycle
How can ammonia be removed? What is the method used to evaluate all welds performed on
A. Freeze the brine solution pressure parts of boiler tube materials? Answer: B
B. Heat the brine to a high temperature enough to A. Hydrostatic test
free the ammonia B. Orsat analysis 58. Past ME Board Question
C. Throw the brine solution C. Vacuum test A valve that releases steam from the boiler by opening at a
D. Low the temperature of the brine D. Radiographic test pre-determined pressure in order to keep the steam
pressure from rising more than the pressure from which the
Answer: B Answer: D valve is set is called a
A. Ammonia valve
49. Past ME Board Question 54. Past ME Board Question B. Stop valve
What will happen when the pressure at any point inside a If the pressure is disregarded in the various other C. Check valve
centrifugal pump goes below the vapor pressure components of a steam of gas power plants, the pressure D. Safety valve
corresponding to the temperature of the liquid? rise in the pump or compressor is ____ to the pressure drop
A. Turbulent flow in the turbine. Answer: D
B. Laminar flow A. Inversely proportional
C. Cavitation B. Constant 59. Past ME Board Question
D. Priming C. Equal What is the relationship of the capacity of a centrifugal
D. Varying pump, Q, to impeller diameter, D, when there are two
Answer: C impeller diameters in the same pump?
A. Q is directly proportional to the square of the ratio 64. Past ME Board Question C. Et – Er
of D If Hs is the total suction head of a pump, Hp is its suction D. Er / Et
B. Q is inversely proportional to the ratio of D surface pressure, and Hf is its suction friction head, then
C. Q. is directly proportional to the ratio of D calculate the total suction head, Ht. Answer: B
D. Q is inversely proportional to the square of the A. Ht = Hs – Hp – Hf
ratio of D B. Ht = Hs + Hp – Hf 69. Past ME Board Question
C. Ht = Hs + Hp + Hf If W is the work output of a heat engine and H is the total
Answer: C D. Ht = Hs – Hp + Hf heat input of an engine, then determine its thermal
efficiency, e.
60. Past ME Board Question Answer: C A. e = H/ W
If Wt is the turbine shaft work of a gas turbine unit, Wc is its B. e = W / H
compressor shaft work and Q is the heat supplied in fuel, 65. Past ME Board Question C. e = W x H
then determine its thermal efficiency. The constant spending of certain percentage of circulated D. e = l – ( W – H )
A. Q / (Wt – Wc) water in a cooling tower in order to prevent accumulation of
B. Q / (Wt + Wc) dissolved mineral solids and other impurities in the Answer: B
C. (Wt + Wc) / Q condenser water is called
D. (Wt – Wc) / Q A. Approach 70. Past ME Board Question
B. Drift Which of the following is converted to mechanical energy by
Answer: D C. Range a water.
D. Bleed-off A. Internal energy
61. Past ME Board Question B. Kinetic energy
The schedule number of a pipe, N, indicates the thickness of Answer: D C. Potential energy
the pipe wall. If the allowable stress of the pipe is S, then D. Hydraulic energy
what is the internal pressure equal to? 66. Past ME Board Question
A. N – S What process is employed when the turbine steam power Answer: C
B. N / S plants experience excessive moisture?
C. N x S A. Reheating 71. Past ME Board Question
D. S / N B. Supercooling What must be the value of the available Net Positive Suction
C. Subcooling Head (NPSH) of a centrifugal pump compared to its require
Answer: C D. Freezing NPSH to avoid losing priming?
A. Available NPSH lesser than required NPSH
62. Past ME Board Question Answer: A B. Available NPSH equal than required NPSH
What is the pressure present inside the casing of an impulse C. Available NPSH greater than required NPSH
turbine? 67. Past ME Board Question D. Available NPSH constant at all times
A. Vacuum A company is interested to produce a water turbine wherein
B. Atmospheric pressure only little energy is required or necessary because the guide Answer: C
C. Above atmospheric vanes are to be controlled. The turbine must be a:
D. Zero A. Gas turbine 72. Past ME Board Question
B. Propeller turbine What type of fuel is used with stationary power plants where
Answer: B C. Kaplan turbine gas is available by pipeline?
D. Francis turbine A. Gaseous fuels
63. Past ME Board Question B. Solid fuels
In order to remove the fly ashes from the flue gas, which of Answer: C C. Coal
the following must a power plant equipped with? D. Liquid fuels
A. Condenser 68. Past ME Board Question
B. Demineralizer If Et is the total net energy generated by a plant in a certain Answer: A
C. Electrostatic precipitator period of time and Er is the rated net energy capacity of the
D. Desulphurization plant plant during the same period of time, then what is the plant 73. Past ME Board Question
operating factor? Wb is shaft work of an engine and Wi is the indicated work of
Answer: C A. Er – Et an engine. If mechanical friction is present in the engine
B. Et / Er mechanism, then:
A. Wb is equal to Wi It is important to take some moisture from the air to If Ev is the volumetric efficiency of a reciprocating pump,
B. Wb is less than Wi dehumidify it if the relative humidity reaches high levels. To then determine SLIP which represents all losses of capacity
C. Wb is proportional to Wi do this, it requires cooling the air: given in percentage of the displacement.
D. Wb is greater than Wi A. At wet-bulb temperature A. 1 / Ev
B. Above its dew point temperature B. 1 – 1 / Ev
Answer: B C. At critical temperature C. 1 – Ev
D. Below its dew point temperature D. Ev + 1
74. Past ME Board Question
What is the saturation pressure at the critical temperature, or Answer: D Answer: C
the lowest pressure in which a substance can be in the liquid
state at its critical pressure? 79. Past ME Board Question 84. Past ME Board Question
A. External pressure What does enthalpy measure in a substance? What is the relationship of the horsepower of a centrifugal
B. Critical pressure A. Its coldness pump, Hp, to the impeller speed, S, if the pump is at two
C. Condensing pressure B. Its heat content different rotative speeds?
D. Evaporation pressure C. Its humidity A. Hp is inversely proportional to the cube of the ratio
D. Its dew point of S
Answer: B B. Hp is directly proportional to the ratio of S
Answer: B C. Hp is inversely proportional to the ratio of S
75. Past ME Board Question D. Hp is directly proportional to the cube of the ratio
What is the significance of a high dielectric strength of 80. Past ME Board Question of S
lubricating oil? Pneumatic tools are powered by:
A. Oil is thick A. Natural gas Answer: D
B. Oil is contaminant – free B. Air
C. Oil is thin C. Electricity 85. Past ME Board Question
D. Oil is not clean D. Steam What is likely to occur when sections of the impeller of a
centrifugal pump are handling vapor and other sections are
Answer: B Answer: B handling liquid?
A. Erosion of the pump
76. Past ME Board Question 81. Past ME Board Question B. High head and low capacity
Theoretically, what is the effect of the compressor clearance The pump will not cavitate if the available Net Positive C. Excessive vibration
on horsepower? Suction Head (NPSH) is: D. Complete failure to operate
A. Varies in direct proportion A. Less than the required NPSH
B. Increase power B. Equal or lesser than the required NPSH Answer: D
C. Decrease power C. Equal or greater than the required NPSH
D. There is no effect D. Zero compared to the required NPSH of 1 86. Past ME Board Question
When vapor compression takes place on one side of the
Answer: D Answer: C piston and one during one revolution of the crankshaft, then
the compressor is:
77. Past ME Board Question 82. Past ME Board Question A. Double-acting
Compare the refrigerating effect per unit mass of refrigerant A water-tube condenser has a total of 60 tubes. If these are B. Four-cycle
circulated for a superheated cycle that produces useful two passes, then compute the number of tubes per pass. C. Two-revolution
cooling and a saturated cycle, for the same vaporizing and A. 6 C D. Single-acting
condensing temperature. B. 15
A. Lower for a superheated cycle C. 3 C Answer: D
B. Greater for a saturated cycle D. 2
C. The same
D. Greater for a superheated cycle Answer: B
87. Past ME Board Question
Answer: D 83. Past ME Board Question In Stirling and Erickson Cycles, the efficiency can be
increased by:
78. Past ME Board Question A. Compressibility
B. Superheating compare the efficiency of a Rankine cycle to that of the Answer: C
C. Regeneration Carnot cycle?
D. Subcooling A. Efficiency is the same 97. Past ME Board Question
B. Higher efficiency The ratio of the average load during a certain time to peak
Answer: C C. Efficiency is higher by 30% the load occurring during the same period of time is called:
D. Lower efficiency A. Demand factor
88. Past ME Board Question B. Capacity factor
Which of the following is used to control refrigeration Answer: D C. Load factor
compressor capacity? D. Output factor
A. Dehumidifier 93. Past ME Board Question
B. Check valve Compute the wall gain load of a load storage room when A is Answer: C
C. Solenoid valve the outside surface area of the wall, D is the temperature
D. Unloader differential across the wall, and U is the overall coefficient of 98. Past ME Board Question
heat transmission. Based on the PSME Code, what should be the effective
Answer: D A. AU / D temperature of the air to be maintained for comfort cooling at
B. D / AU an air movement from 0.0761 to 0.127 meter per second?
89. Past ME Board Question C. A / UD A. 25 – 29 degrees C
During a cooling and dehumidifying process, sensible and D. AUD B. 35 – 39 degrees C
latent heats are removed in the cooling coil. If Hs is the C. 20 – 24 degrees C
sensible heat and Ht is the total heat transferred, then Answer: D D. 30 – 34 degrees C
determine the coil sensible heat factor.
A. Ht – Hs 94. Past ME Board Question Answer: C
B. Hs – Ht If m is the mass of dry air and H is the specific enthalpy of
C. Ht / Hs the water vapor in air and r is the humidity ratio, then 99. Past ME Board Question
D. Hs / Ht determine the latent heat of any dry air. If R is the delivery rate of a pump, H is the total pumping
A. m / (r – H) head and e is the efficiency of the pump, then compute the
Answer: D B. r / (H – m) power required to drive the pump.
C. (m – r) / H A. R / H x e
90. Past ME Board Question D. m (r x H) B. 1 – (R + H) / e
Based on the PSME Code, what should be provided in each C. (R x H) / e
stream outlets if two or more boilers will be connected in Answer: D D. (H – R) / e
parallel?
A. Relief and check valves 95. Past ME Board Question Answer: C
B. Non-return and shut-off valves When an electric motor nameplate indicates a “100-kw
C. Expansion and shut-off valves electric motor” then what does a 100-kw rating refers to? 100. Past ME Board Question
D. Expansion and check valves A. Thermal energy input There are three basic boiler types, namely:
B. Thermal energy output A. Water-tube, horizontal tube and cast-iron boilers
Answer: B C. Electrical power output B. Cast-iron, fire-tube and water tube boilers
D. Mechanical power output C. Fire-tube, cast-iron and water tube boilers
91. Past ME Board Question D. HRT, fire-tubes and Scotch Marine boilers
The diesel cycle is the ideal cycle for a : Answer: D
A. Absorption Answer: B
B. Steam-jet 96. Past ME Board Question
C. Compression-ignition engine For a hydraulic power to be transmitted at its maximum what 101. Past ME Board Question
D. Gas turbine should be the value of loss of head due to friction if H is the The amount of heat necessary to bring up the temperature of
total available head? a unit mass of a substance through a unit degree is called:
Answer: C A. 11H A. Specific volume
B. 1/3 H B. Thermal heat
92. Past ME Board Question C. 1/4 H C. Total heat
If a Carnot cycle is at the same maximum and minimum D. 5H D. Specific heat
temperatures as a Rankine cycle, then how would you
Answer: D

102. Past ME Board Question


What is the specific gravity of an oil which has a Baume
reading g of 28 degrees F?
A. 0.215
B. 0.562
C. 0.886
D. 0.774

Answer: C

103. Past ME Board Question


If C is the capacity of the compressor in a refrigeration
system and F is the heat rejection factor, then calculate the
condenser load, L.
A. L = F – C
B. L = F / C
C. L = C x F
D. L = C / F

Answer: C

104. Past ME Board Question


Based on the PSME Code, what should be the humidity ratio
of air to be used for comfort cooling?
A. 75% - 80% relative humidity
B. 50% - 60% relative humidity
C. 60% - 70% relative humidity
D. 70% - 75% relative humidity

Answer: B

105. Past ME Board Question


What is the cause of pressure drop in the boiler, condenser
and the pipings between different components? Because of
this drop, steam leaving the boiler is at lower pressure.
A. Positive slip
B. Fluid friction
C. Negative slip
D. Low thermal efficiency

Answer: B

You might also like